Chemistry Objective NCERT Xtract WWW - examSAKHA.in PDF

You might also like

Download as pdf or txt
Download as pdf or txt
You are on page 1of 781

t

.me/
Exam_
Sakha_
Off
ici
al
EBD_8350
Corporate
Office DISHA PUBLICATION
45, 2nd Floor, Maharishi Dayanand Marg,
Corner Market, Malviya Nagar, New Delhi - 110017
Tel : 49842349 / 49842350

© Copyright
No part of this publication may be reproduced in
any form without prior permission of the publisher.

Disha
The author and the publisher do not take any legal
responsibility for any errors or misrepresentations that
might have crept in. We have tried and made our best
efforts to provide accurate up-to-date information in
this book.
All Right Reserved

By:
Dr. Anuj Kumar
Kalpana Bhargav

Typeset by Disha DTP Team

www.dishapublication.com www.mylearninggraph.com
Books & Etests
ebooks for
School & for
Competitive Competitive
Exams Exams

Write to us at feedback_disha@aiets.co.in
CONTENTS
S. No. Chapters Page No.
1. Some Basic Concepts of Chemistry 1-12
• Trend Buster NEET & JEE Main
• Chapter At a Glance
• Problem Solving Tips/ Tricks/ Points to Remember
• Exercise - 1 NCERT Based Topic wise MCQs
• Exercise - 2 Numeric/ Integer Answer Questions
• Exercise - 3 NCERT Exemplar & Past Year MCQs
• Exercise - 4 Problem Solving Skill Enhancer MCQs
2. Structure of Atom 13-28
• Trend Buster NEET & JEE Main
• Chapter At a Glance
• Problem Solving Tips/ Tricks/ Points to Remember
• Exercise - 1 NCERT Based Topic wise MCQs
• Exercise - 2 Numeric/ Integer Answer Questions
• Exercise - 3 NCERT Exemplar & Past Year MCQs
• Exercise - 4 Problem Solving Skill Enhancer MCQs
3. Classification of Elements and Periodicity in Properties 29-42
• Trend Buster NEET & JEE Main
• Chapter At a Glance
• Problem Solving Tips/ Tricks/ Points to Remember
• Exercise - 1 NCERT Based Topic wise MCQs
• Exercise - 2 NCERT Exemplar & Past Year MCQs
• Exercise - 3 Problem Solving Skill Enhancer MCQs
4. Chemical Bonding and Molecular Structure 43-60
• Trend Buster NEET & JEE Main
• Chapter At a Glance
• Problem Solving Tips/ Tricks/ Points to Remember
• Exercise - 1 NCERT Based Topic wise MCQs
• Exercise - 2 Numeric/ Integer Answer Questions
• Exercise - 3 NCERT Exemplar & Past Year MCQs
• Exercise - 4 Problem Solving Skill Enhancer MCQs
5. States of Matter 61-76
• Trend Buster NEET & JEE Main
• Chapter At a Glance
• Problem Solving Tips/ Tricks/ Points to Remember
• Exercise - 1 NCERT Based Topic wise MCQs
• Exercise - 2 Numeric/ Integer Answer Questions
• Exercise - 3 NCERT Exemplar & Past Year MCQs
• Exercise - 4 Problem Solving Skill Enhancer MCQs
6. Thermodynamics 77-92
• Trend Buster NEET & JEE Main
• Chapter At a Glance
• Problem Solving Tips/ Tricks/ Points to Remember
EBD_8350
• Exercise - 1 NCERT Based Topic wise MCQs
• Exercise - 2 Numeric/ Integer Answer Questions
• Exercise - 3 NCERT Exemplar & Past Year MCQs
• Exercise - 4 Problem Solving Skill Enhancer MCQs
7. Equilibrium 93-110
• Trend Buster NEET & JEE Main
• Chapter At a Glance
• Problem Solving Tips/ Tricks/ Points to Remember
• Exercise - 1 NCERT Based Topic wise MCQs
• Exercise - 2 Numeric/ Integer Answer Questions
• Exercise - 3 NCERT Exemplar & Past Year MCQs
• Exercise - 4 Problem Solving Skill Enhancer MCQs
8. Redox Reactions 111-120
• Trend Buster NEET & JEE Main
• Chapter At a Glance
• Problem Solving Tips/ Tricks/ Points to Remember
• Exercise - 1 NCERT Based Topic wise MCQs
• Exercise - 2 NCERT Exemplar & Past Year MCQs
• Exercise - 3 Problem Solving Skill Enhancer MCQs
9. Hydrogen 121-132
• Trend Buster NEET & JEE Main
• Chapter At a Glance
• Problem Solving Tips/ Tricks/ Points to Remember
• Exercise - 1 NCERT Based Topic wise MCQs
• Exercise - 2 NCERT Exemplar & Past Year MCQs
• Exercise - 3 Problem Solving Skill Enhancer MCQs
10. The s-Block Elements 133-144
• Trend Buster NEET & JEE Main
• Chapter At a Glance
• Problem Solving Tips/ Tricks/ Points to Remember
• Exercise - 1 NCERT Based Topic wise MCQs
• Exercise - 2 NCERT Exemplar & Past Year MCQs
• Exercise - 3 Problem Solving Skill Enhancer MCQs
11. The p-Block Elements (Group 13 and 14) 145-156
• Trend Buster NEET & JEE Main
• Chapter At a Glance
• Problem Solving Tips/ Tricks/ Points to Remember
• Exercise - 1 NCERT Based Topic wise MCQs
• Exercise - 2 NCERT Exemplar & Past Year MCQs
• Exercise - 3 Problem Solving Skill Enhancer MCQs
12. Organic Chemistry – Some Basic Principles and Techniques 157-176
• Trend Buster NEET & JEE Main
• Chapter At a Glance
• Problem Solving Tips/ Tricks/ Points to Remember
• Exercise - 1 NCERT Based Topic wise MCQs
• Exercise - 2 Numeric/ Integer Answer Questions
• Exercise - 3 NCERT Exemplar & Past Year MCQs
• Exercise - 4 Problem Solving Skill Enhancer MCQs
13. Hydrocarbons 177-196
• Trend Buster NEET & JEE Main
• Chapter At a Glance
• Problem Solving Tips/ Tricks/ Points to Remember
• Exercise - 1 NCERT Based Topic wise MCQs
• Exercise - 2 Numeric/ Integer Answer Questions
• Exercise - 3 NCERT Exemplar & Past Year MCQs
• Exercise - 4 Problem Solving Skill Enhancer MCQs
14. Environmental Chemistry 197-208
• Trend Buster NEET & JEE Main
• Chapter At a Glance
• Problem Solving Tips/ Tricks/ Points to Remember
• Exercise - 1 NCERT Based Topic wise MCQs
• Exercise - 2 NCERT Exemplar & Past Year MCQs
• Exercise - 3 Problem Solving Skill Enhancer MCQs
15. The Solid State 209-222
• Trend Buster NEET & JEE Main
• Chapter At a Glance
• Problem Solving Tips/ Tricks/ Points to Remember
• Exercise - 1 NCERT Based Topic wise MCQs
• Exercise - 2 Numeric/ Integer Answer Questions
• Exercise - 3 NCERT Exemplar & Past Year MCQs
• Exercise - 4 Problem Solving Skill Enhancer MCQs
16. Solutions 223-238
• Trend Buster NEET & JEE Main
• Chapter At a Glance
• Problem Solving Tips/ Tricks/ Points to Remember
• Exercise - 1 NCERT Based Topic wise MCQs
• Exercise - 2 Numeric/ Integer Answer Questions
• Exercise - 3 NCERT Exemplar & Past Year MCQs
• Exercise - 4 Problem Solving Skill Enhancer MCQs
17. Electrochemistry 239-256
• Trend Buster NEET & JEE Main
• Chapter At a Glance
• Problem Solving Tips/ Tricks/ Points to Remember
• Exercise - 1 NCERT Based Topic wise MCQs
• Exercise - 2 Numeric/ Integer Answer Questions
• Exercise - 3 NCERT Exemplar & Past Year MCQs
• Exercise - 4 Problem Solving Skill Enhancer MCQs
18. Chemical Kinetics 257-276
• Trend Buster NEET & JEE Main
• Chapter At a Glance
• Problem Solving Tips/ Tricks/ Points to Remember
• Exercise - 1 NCERT Based Topic wise MCQs
• Exercise - 2 Numeric/ Integer Answer Questions
• Exercise - 3 NCERT Exemplar & Past Year MCQs
EBD_8350
• Exercise - 4 Problem Solving Skill Enhancer MCQs
19. Surface Chemistry 277-290
• Trend Buster NEET & JEE Main
• Chapter At a Glance
• Problem Solving Tips/ Tricks/ Points to Remember
• Exercise - 1 NCERT Based Topic wise MCQs
• Exercise - 2 Numeric/ Integer Answer Questions
• Exercise - 3 NCERT Exemplar & Past Year MCQs
• Exercise - 4 Problem Solving Skill Enhancer MCQs
20. General Principles and Processes of Isolation of Elements 291-302
• Trend Buster NEET & JEE Main
• Chapter At a Glance
• Problem Solving Tips/ Tricks/ Points to Remember
• Exercise - 1 NCERT Based Topic wise MCQs
• Exercise - 2 NCERT Exemplar & Past Year MCQs
• Exercise - 3 Problem Solving Skill Enhancer MCQs
21. The p-Block Elements (Group 15, 16, 17 and 18) 303-318
• Trend Buster NEET & JEE Main
• Chapter At a Glance
• Problem Solving Tips/ Tricks/ Points to Remember
• Exercise - 1 NCERT Based Topic wise MCQs
• Exercise - 2 Numeric/ Integer Answer Questions
• Exercise - 3 NCERT Exemplar & Past Year MCQs
• Exercise - 4 Problem Solving Skill Enhancer MCQs
22. The d-and f-Block Elements 319-332
• Trend Buster NEET & JEE Main
• Chapter At a Glance
• Problem Solving Tips/ Tricks/ Points to Remember
• Exercise - 1 NCERT Based Topic wise MCQs
• Exercise - 2 Numeric/ Integer Answer Questions
• Exercise - 3 NCERT Exemplar & Past Year MCQs
• Exercise - 4 Problem Solving Skill Enhancer MCQs
23. Coordination Compounds 333-348
• Trend Buster NEET & JEE Main
• Chapter At a Glance
• Problem Solving Tips/ Tricks/ Points to Remember
• Exercise - 1 NCERT Based Topic wise MCQs
• Exercise - 2 Numeric/ Integer Answer Questions
• Exercise - 3 NCERT Exemplar & Past Year MCQs
• Exercise - 4 Problem Solving Skill Enhancer MCQs
24. Haloalkanes and Haloarenes 349-368
• Trend Buster NEET & JEE Main
• Chapter At a Glance
• Problem Solving Tips/ Tricks/ Points to Remember
• Exercise - 1 NCERT Based Topic wise MCQs
• Exercise - 2 Numeric/ Integer Answer Questions
• Exercise - 3 NCERT Exemplar & Past Year MCQs
• Exercise - 4 Problem Solving Skill Enhancer MCQs
25. Alcohols, Phenols and Ethers 369-388
• Trend Buster NEET & JEE Main
• Chapter At a Glance
• Problem Solving Tips/ Tricks/ Points to Remember
• Exercise - 1 NCERT Based Topic wise MCQs
• Exercise - 2 Numeric/ Integer Answer Questions
• Exercise - 3 NCERT Exemplar & Past Year MCQs
• Exercise - 4 Problem Solving Skill Enhancer MCQs
26. Aldehydes, Ketones & Carboxylic acids 389-412
• Trend Buster NEET & JEE Main
• Chapter At a Glance
• Problem Solving Tips/ Tricks/ Points to Remember
• Exercise - 1 NCERT Based Topic wise MCQs
• Exercise - 2 Numeric/ Integer Answer Questions
• Exercise - 3 NCERT Exemplar & Past Year MCQs
• Exercise - 4 Problem Solving Skill Enhancer MCQs
27. Amines 413-430
• Trend Buster NEET & JEE Main
• Chapter At a Glance
• Problem Solving Tips/ Tricks/ Points to Remember
• Exercise - 1 NCERT Based Topic wise MCQs
• Exercise - 2 NCERT Exemplar & Past Year MCQs
• Exercise - 3 Problem Solving Skill Enhancer MCQs
28. Biomolecules 431-448
• Trend Buster NEET & JEE Main
• Chapter At a Glance
• Problem Solving Tips/ Tricks/ Points to Remember
• Exercise - 1 NCERT Based Topic wise MCQs
• Exercise - 2 Numeric/ Integer Answer Questions
• Exercise - 3 NCERT Exemplar & Past Year MCQs
• Exercise - 4 Problem Solving Skill Enhancer MCQs
29. Polymers 449-460
• Trend Buster NEET & JEE Main
• Chapter At a Glance
• Problem Solving Tips/ Tricks/ Points to Remember
• Exercise - 1 NCERT Based Topic wise MCQs
• Exercise - 2 NCERT Exemplar & Past Year MCQs
• Exercise - 3 Problem Solving Skill Enhancer MCQs
30. Chemistry In Everyday Life 461-470
• Trend Buster NEET & JEE Main
• Chapter At a Glance
• Problem Solving Tips/ Tricks/ Points to Remember
• Exercise - 1 NCERT Based Topic wise MCQs
• Exercise - 2 NCERT Exemplar & Past Year MCQs
• Exercise - 3 Problem Solving Skill Enhancer MCQs
EBD_8350
Answer Keys & Explanations
S. No. Chapters Page No.
1. Some Basic Concepts of Chemistry 471-481
2. Structure of Atom 482-491
3. Classification of Elements and Periodicity in Properties 492-496
4. Chemical Bonding and Molecular Structure 497-510
5. States of Matter 511-519
6. Thermodynamics 520-528
7. Equilibrium 529-543
8. Redox Reactions 544-548
9. Hydrogen 549-552
10. The s-Block Elements 553-557
11. The p-Block Elements (Group 13 and 14) 558-563
12. Organic Chemistry – Some Basic Principles and Techniques 564-572
13. Hydrocarbons 573-585
14. Environmental Chemistry 586-588
15. The Solid State 589-596
16. Solutions 597-608
17. Electrochemistry 609-619
18. Chemical Kinetics 620-631
19. Surface Chemistry 632-637
20. General Principles and Processes of Isolation of Elements 638-642
21. The p-Block Elements (Group 15, 16, 17 and 18) 643-652
22. The d-and f-Block Elements 653-660
23. Coordination Compounds 661-672
24. Haloalkanes and Haloarenes 673-685
25. Alcohols, Phenols and Ethers 686-698
26. Aldehydes, Ketones & Carboxylic acids 699-712
27. Amines 713-721
28. Biomolecules 722-729
29. Polymers 730-733
30. Chemistry In Everyday Life 734-736

Mock Tests
Mock Test - 1 MT-1-4
Mock Test - 2 MT-5-7
Mock Test - 3 MT-8-10
Mock Test - 4 MT-11-13
Mock Test - 5 MT-14-16
Answer Key and Explanations (1-5) MT-17-32
Some Basic Concepts of
1 Chemistry
Trend Buster NEET & JEE Main

Number of Questions from 2020-15 9 7 One question has been asked every
Weightage 2.8% 4.6% year in NEET & JEE Main.

The most Important Concepts that Cover Maximum number of Questions asked in past 6 years.

Mole Concept 6 1
Stoichiometric calculations 3 4

Less Important Concepts that Cover 1 or 2 Questions asked in past 6 years.

Laws of Chemical Combination — 1


Empirical formula — 1

NEET JEE
Concept Used

2020 Laws of chemical Mole concept / Dalton’s Law 1 Average 1 Easy


Combination
2019 Mole concept and molar Mole concept / Stoichiometric 1 Easy 1 Average
masses / Stoichiometric calculation
calculation
2018 Percentage composition, Empirical formula / Mole 2 Average / 1 Difficult
molecular and empirical concept / Stoichiometric Difficult
formula / Mole concept and calculations
molar masses /
Stoichiometric calculations
2017 Stoichiometric Stoichiometric calculations 2 Average
calculations
2016 Stoichiometric Stoichiometric calculations 1 Difficult
calculations
2015 Mole concept and molar Mole concept / 5 Average 1 Difficult
masses / Stoichiometric Stoichiometric calculations
calculation
EBD_8350
2 CHEMISTRY
SOME BASIC CONCEPTS OF CHEMISTRY 3
EBD_8350
4 CHEMISTRY

Problem Solving Tips/ Tricks/ Points to Remember

4 Dulong and Pettit’s method 4 Vapour density method


Atomic mass ´ Specific heat = 6.4 (approx.) Molecular weight = 2 × vapour density
This law is applicable to solid elements only except Be, B, n-Factor
C and Si
4 On diluting a solution, eq, meq, mole or m mole of solute atomic weight
Equivalent weight =
do not change however N and M change (decrease). n-factor
4 Molality, % by weight, mole fraction do not depend on In case of acid/base the n-factor is basicity/acidity (i.e.
temperature since they involve weights. number of dissociable H+ ions/number of dissociable OH–
4 Normality, molarity, percent of volume; percent by ions) and in case of oxidizing agent/reducing agent, n-
strength, and strength are temperature dependent. They factor is number of moles of electrons gained/lost per
decrease with increase in temperature since the volume mole of oxidizing agent/reducing agent.
of solution increases with T. 4 Molality (m) expressed in terms of mole fraction of solute
4 Sometimes, the term formality is used in place of molarity. 1000 X1
(X1) is given by relation : m =
4 Standard solution is one whose N or M is known. (1– X1 )m2
4 Normality: It is defined as number of equivalents of a where m2 is the molar mass of the solvent.
solute present in one litre of solution. 4 Molality (m) is related to molarity (M) by relation :
1000M
Equivalent of solute m=
N= 1000d – Mm1
Volume of solution (L)
where d is the density (g mL–1) and m1 is the molar mass
Weight of solution
N= of the solute.
Equivalent weight of solute × Vsol (in L)
4 Percentage:
W W ´ 1000
N= = Weight of solute
E ´ Vsol (in L) E ´ Vsol (in mL) % by weight = ´ 100
Weight of solution
4 Molarity: It is defined as the moles of solute present in
one litre of solution. Volume of solute
% by volume = ´ 100
Moles of solute Volume of solution
M=
Volume of solution in litre Weight of solute
% by (W/V) = ´ 100
Weight of solute Volume of solution
M= For example, a solution is 85% by weight
Molecular weight of solute ´ Vsol (in L)
W2/Ws means: 100 g solution contains 85 g solute
4 Molality: Moles of solute present in 1 kg of solvent.
85% by volume: 100 mL solution contains 85 mL solute
Moles of solute 85% by strength (W/V): 100 mL solution contains 85 g
Molality =
Weight of solvent in kg solute
SOME BASIC CONCEPTS OF CHEMISTRY 5

Exercise 1 : NCERT Based Topic-wise MCQs


(a) 4.7 g (b) 4680 × 10–3 g
Topic 1: Properties of Matter and their Measurement
(c) 4.680 g (d) 46.80 g
1. Which one of the following set of units represents the 10. How many significant figures are in each of the following.
smallest and largest amount of energy respectively? (i) 4.003; (ii) 6.023 × 1023 ; (iii) 5000
(a) J and erg (b) erg and cal (a) 3, 4, 1 (b) 4, 3, 2 (c) 4, 4, 4 (d) 3, 4, 3
(c) cal and eV (d) eV and L-atm
2. Many countries use Fahrenheit scale for expressing
Topic 2: Laws of Chemical Combinations
temperature of atmosphere. If temperature in any such
country is measured 41°F then what is its value in celcius 11. The formation of CO and CO2 illustrates the law of
scale and would you expect hot or cold atmosphere in (a) reciprocal proportion (b) conservation of mass
that country? (c) multiple proportion (d) constant composition
(a) 15°C, cold (b) 25°C, normal 12. Which of the following is the best example of law of
(c) 5°C, cold (d) 41°C, hot conservation of mass?
3. Given P = 0.0030m, Q = 2.40m, R = 3000m, Significant (a) 12 g of carbon combines with 32 g of oxygen to form
figures in P, Q and R are respectively 44 g of CO2
(a) 2, 2, 1 (b) 2, 3, 4 (c) 4, 2, 1 (d) 4, 2, 3 (b) When 12 g of carbon is heated in a vacuum there is
4. The number of significant figures for the three numbers no change in mass
161 cm, 0.161 cm, 0.0161 cm are (c) A sample of air increases in volume when heated at
(a) 3,4 and 5 respectively (b) 3,4 and 4 respectively constant pressure but its mass remains unaltered
(c) 3,3 and 4 respectively (d) 3,3 and 3 respectively (d) The weight of a piece of platinum is the same before
5. In the final answer of the expression and after heating in air
13. In an experiment, 4g of H2 combine with 32g of O2 to form
(29.2 - 20.2) (1.79 ´105 )
36g of H2O. In another experiment, when 50g of H2 combine
1.37 with 400g of O2 then 450g of H2O is formed. Above two
the number of significant figures is : experiments follow
(a) 1 (b) 2 (c) 3 (d) 4 (a) The law of conservation of mass
6. Arrange the numbers in increasing no. of significant (b) The law of constant composition
figures. (c) Both (a) and (b)
0.002600, 2.6000, 2.6, 0.260 (d) Neither (a) nor (b)
(a) 2.6 < 0.260 < 0.002600 < 2.6000 14. Irrespective of the source, pure sample, of water always yields
(b) 2.6000 < 2.6 < 0.002600 < 0.260 88.89% mass of oxygen and 11.11% mass of hydrogen. This
(c) 0.260 < 2.6 < 0.002600 < 2.6000 is explained by the law of
(d) 0.002600 < 0.260 < 2.6 < 2.6000 (a) conservation of mass (b) multiple proportions
7. If the true value for an experimental result is 6.23 and the (c) constant composition (d) constant volume
results reported by three students X, Y and Z are : 15. The percentage of copper and oxygen in samples of CuO
X : 6.18 and 6.28 obtained by different methods were found to be the same.
Y : 6.20 and 6.023 The illustrate the law of
(a) constant proportions (b) conservation of mass
Z : 6.22 and 6.24
(c) multiple proportions (d) reciprocal proportions
Which of the following option is correct :
16. Assertion : Equal moles of different substances contain
(a) X precise, Y accurate, Z precise and accurate. same number of constituent particles.
(b) X precise and accurate, Y not precise, Z precise Reason : Equal weights of different substances contain
(c) Both X & Z precise & accurate, Y not precise. the same number of constituent particles.
(d) Both X & Y neither precise nor accurate, Z both precise (a) Assertion is correct, reason is correct; reason is a
and accurate. correct explanation for assertion.
8. A sample was weighted using two different balances. The (b) Assertion is correct, reason is correct; reason is not a
results were correct explanation for assertion.
(i) 3.929 g (ii) 4.0 g (c) Assertion is correct, reason is incorrect.
How would the weight of the sample be reported? (d) Assertion is incorrect, reason is correct.
(a) 3.93 g (b) 3g (c) 3.9 g (d) 3.929 g 17. Which one of the following pairs of compounds illustrate
9. If the density of a solution is 3.12 g mL–1, the mass of the law of multiple proportions ?
1.5 mL solution in significant figures is______. (a) H2O and Na2O (b) MgO and Na2O
(c) Na2O and BaO (d) SnCl2 and SnCl4
EBD_8350
6 CHEMISTRY

18. Two samples of lead oxide were separately reduced to (b) mass of one atom compared with the mass of one
metallic lead by heating in a current of hydrogen. The atom of hydrogen
weight of lead from one oxide was half the weight of lead (c) mass of one molecule of any substance compared
obtained from the other oxide. The data illustrates with the mass of one atom of C-12
(a) law of reciprocal proportions (d) None of the above
(b) law of constant proportions 26. The average atomic mass of neon based on following data
(c) law of multiple proportions is :
(d) law of equivalent proportions Isotope Relative abundance
19. In compound A, 1.00g of nitrogen unites with 0.57g of 20Ne 0.9051
oxygen. In compound B, 2.00g of nitrogen combines with 21Ne 0.0027
2.24g of oxygen. In compound C, 3.00g of nitrogen 22Ne 0.0922
combines with 5.11g of oxygen. These results obey the (a) 0.33 u (b) 20.187 u (c) 6.729 u (d) 18.058 u
following law 27. What is the average atomic mass of bromine from the
(a) law of constant proportion following data : (abundance is in %)
(b) law of multiple proportion Isotope Mass Abundance
(c) law of reciprocal proportion 79Br 78.9183361 50.69
(d) Dalton’s law of partial pressure 81Br 80.916289 49.31
20. The molecular weight of O2 and SO2 are 32 and 64
(a) 79.9 (b) 76.6 (c) 75.9 (d) 69.9
respectively. At 15°C and 150 mm Hg pressure, one litre of
28. What is the mass of an atom of oxygen (in g)?
O2 contains ‘N’ molecules. The number of molecules in
(a) 2.656 × 10–23 (b) 1.567 × 10–22
two litres of SO2 under the same conditions of temperature –22
(c) 2.0 × 10 (d) 3.5 × 10–23
and pressure will be :
(a) N/2 (b) 1 N 29. If the mass of the one atom is found to be 2.324784×10–23g,
(c) 2 N (d) 4 N then this atom can be ?
21. 10 dm3 of N2 gas and 10 dm 3 of gas X at the same (a) Oxygen (b) Carbon
temperature contain the same number of molecules, the (c) Fluorine (d) Nitrogen
gas X is 30. Match the columns
(a) CO2 (b) CO (c) H2 (d) NO Column-I Column-II
22. Equal volumes of two gases A and B are kept in a container (A) C6H5NH2 (p) 84
at the same temperature and pressure. Avogadro’s law is (B) C6H6 (q) 100
invalid if (C) C6H12 (r) 93
(a) the gases are reactive (D) CaCO3 (s) 78
(b) the gases are non-reactive (a) A – (p), B – (r), C – (q), D – (s)
(c) gas A has more number of molecules than gas B. (b) A – (r), B – (s), C – (p), D – (q)
(d) None of these (c) A – (r), B – (p), C – (q), D – (s)
23. Consider the following statements. (d) A – (r), B – (q), C – (s), D – (p)
(i) Atoms of H, O, N and C have identical properties but
different mass. Topic 4: Mole Concepts and Molar Masses
(ii) Matter is divisible into atoms which are further
31. What is the mass of 1 molecule of CO.
indivisible.
(iii) The ratio of N: H in NH3 is 1 : 3 and N : O in nitric (a) 2.325 × 10–23 (b) 4.65 × 10–23
(c) 3.732 × 10 –23 (d) 2.895 × 10–23
oxide is 2 : 1.
(iv) Dalton’s atomic theory support law of conservation 32. Calculate the volume at STP occupied by 240 g of SO2.
of mass. (a) 64 (b) 84 (c) 59 (d) 73
Which of the following pairs of statements is true 33. At S.T.P. the density of CCl4 vapours in g/L will be nearest
according to Dalton’s atomic theory ? to:
(a) (i) and (ii) (b) (ii) and (iii) (a) 6.87 (b) 3.42 (c) 10.26 (d) 4.57
(c) (ii) and (iv) (d) (i) and (iv) 34. Which has maximum number of molecules?
24. 100 mL NO2 and NH3 kept at same temperature and (a) 7 g N2 (b) 2 g H2
pressure. What is true about their number of molecules (c) 16 g NO2 (d) 16 g O2
(a) NO2 > NH3 (b) NO2 < NH3
35. Amount of oxygen in 32.2 g Na2SO4.10 H2O is
(c) NO2 = NH3 (d) NO2 + NH3 = 1 mole (a) 20.8 g (b) 2.24 g
(c) 22.4 g (d) 2.08 g
Topic 3: Atomic and Molecular Masses 36. The number of molecules in 8.96 litre of a gas at 0ºC and 1
25. Molecular mass is defined as the atm. pressure is approximately
(a) mass of one atom compared with the mass of one (a) 6.023 × 1023 (b) 12.04 × 1023
(c) 18.06 × 10 23 (d) 24.08 × 1022
molecule
SOME BASIC CONCEPTS OF CHEMISTRY 7

37. The mass of a molecule of water is (c) Assertion is correct, reason is incorrect.
(a) 3 × 10–25 kg (b) 3 × 10–26 kg (d) Assertion is incorrect, reason is correct.
–26
(c) 1.5 × 10 kg (d) 2.5 × 10–26 kg 51. How many moles of magnesium phosphate, Mg3(PO4)2
38. One mole of CO2 contains : will contain 0.25 mole of oxygen atoms?
(a) 3 g atoms of CO2 (a) 1.25 × 10–2 (b) 2.5 × 10–2
(b) 18.1 × 1023 molecules of CO2 (c) 0.02 (d) 3.125 × 10–2
(c) 6.02 × 1023 atoms of O 52. Volume occupied by one molecule of water
(d) 6.02 × 1023 atoms of C (density = 1 g cm–3) is :
39. Volume of a gas at NTP is 1.12 × 10–7cm3. The number of (a) 9.0 × 10–23 cm3 (b) 6.023 × 10– 23 cm3
–23
(c) 3.0 × 10 cm 3 (d) 5.5 × 10– 23 cm3
molecules in it is :
(a) 3.01 × 1012 (b) 3.01 × 1024 53. 1 c.c. N2O at NTP contains :
(c) 3.01 × 10 23 (d) 3.01 × 1020 1.8 6.02
40. How many moles of Al2(SO4)3 would be in 50 g of the (a) ´ 10 22 atoms (b) ´ 10 23 molecules
224 22400
substance ?
(a) 0.083 mole (b) 0.952 mole (c) 1.32 ´ 10 23 electrons (d) All of the above
(c) 0.481 mole (d) 0.140 mole 224
54. How much time (in hours) would it take to distribute one
41. The weight of one molecule of a compound C 60H122 is Avogadro number of wheat grains if 1020 grains are
(a) 1.2 × 10–20 gram (b) 1.4 × 10–21 gram distributed each second ?
(c) 5.025 × 1023 gram (d) 6.023 × 1023 gram (a) 0.1673 (b) 1.673 (c) 16.73 (d) 167.3
42. The maximum number of molecules are present in
55. 10 g of hydrogen and 64 g of oxygen were filled in a steel
(a) 15 L of H2 gas at STP (b) 5 L of N2 gas at STP
vessel and exploded. Amount of water produced in this
(c) 0.5 g of H2 gas (d) 10 g of O2 gas
reaction will be:
43. The number of atoms in 0.1 mol of a triatomic gas is : (a) 3 mol (b) 4 mol (c) 1 mol (d) 2 mol
(NA = 6.02 ×1023 mol–1)
56. Which one of the following is the lightest?
(a) 6.026 × 1022 (b) 1.806 × 1023
23 (a) 0.2 mole of hydrogen gas
(c) 3.600 × 10 (d) 1.800 × 1022
(b) 6.023 × 1022 molecules of nitrogen
44. The number of gram molecules of oxygen in 6.02 × 1024
CO molecules is (c) 0.1 g of silver
(a) 10 g molecules (b) 5 g molecules (d) 0.1 mole of oxygen gas
(c) 1 g molecules (d) 0.5 g molelcules
45. The number of oxygen atoms in 4.4 g of CO2 is Topic 5: Percentage Composition, Molecular and
(a) 1.2 × 1023 (b) 6 × 1022 Empirical Formula
(c) 6 × 1023 (d) 12 × 1023
46. Number of atoms in 558.5g Fe (at. wt. of Fe = 55.85 g mol–1) 57. The empirical formula of a compound is CH2. One mole of this
is compound has a mass of 42 grams. Its molecular formula is:
(a) twice that in 60 g carbon (b) 6.023 × 1022 (a) C3H6 (b) C3H8 (c) CH2 (d) C2H2
(c) half that in 8 g He (d) 558.5 × 6.023 × 1023 58. The empirical formula of an acid is CH2O2, the probable
47. The number of moles of oxygen in one litre of air containing molecular formula of acid may be :
21% oxygen by volume, under standard conditions are (a) C3H6O4 (b) CH2O (c) CH2O2 (d) C2H4O2
(a) 0.0093 mole (b) 0.21 mole 59. An organic compound contains carbon, hydrogen and
(c) 2.10 mole (d) 0.186 mole oxygen. Its elemental analysis gave C, 38.71% and H, 9.67%.
48. How many atoms are contained in one mole of sucrose The empirical formula of the compound would be :
(C12H22O11)? (a) CH3O (b) CH2O (c) CHO (d) CH4O
(a) 20 × 6.02 × 1023 atoms/mol 60. In a compound C, H and N atoms are present in 9 : 1 : 3.5 by
(b) 45 × 6.02 × 1023 atoms/mol weight. Molecular weight of compound is 108. Molecular
(c) 5 × 6.02 × 1023 atoms/mol formula of compound is
(d) None of these (a) C2H6N2 (b) C3H4N (c) C6H8N2 (d) C9H12N3
49. The mass of 1 mole of electrons is 61. A gaseous hydrocarbon gives upon combustion 0.72 g of
(a) 9.1 × 10–28g (b) 1.008 mg water and 3.08 g. of CO2. The empirical formula of the
(c) 0.55 mg (d) 9.1 × 10–27 g hydrocarbon is :
50. Assertion : Volume of a gas is inversely proportional to
(a) C2H4 (b) C3H4 (c) C6H5 (d) C7H8
the number of moles of gas.
Reason : The ratio by volume of gaseous reactants and 62. Which of the following is the correct empirical and
products is in agreement with their mole ratio. molecular formulae of a compound, if the molecular mass
(a) Assertion is correct, reason is correct; reason is a of a compound is 80 and compound contains 60% of C,
correct explanation for assertion. 5% of H and 35% of N ?
(b) Assertion is correct, reason is correct; reason is not a (a) C2H2N ; C4H4N2 (b) C3H4N2 ; C6H8N4
correct explanation for assertion. (c) C2H4N2 ; C4H8N4 (d) C2H2N ; C2H2N
EBD_8350
8 CHEMISTRY

63. Which of the following is the correct empirical and (a) 2.24 L (b) 4.96 L (c) 1.12 L (d) 0.84 L
molecular formulae of a compound, if the molecular mass 74. The mass of BaCO3 produced when excess CO2 is bubbled
of a compound is 93 and compound containing 77.43% through a solution of 0.205 mol Ba(OH)2 is :
of C, 7.53% of H and 15.05% of N ? (a) 81 g (b) 40.5 g (c) 20.25 g (d) 162 g
(a) C3H3.5N1.5 and C6H7N (b) C6H7N and C6H7N 75. 20.0 kg of N2(g) and 3.0 kg of H2(g) are mixed to produce
(c) C3H3N and C6H7N (d) C3H3N and C6H6N2 NH3(g). The amount of NH3(g) formed is
(a) 17 kg (b) 34 kg (c) 20 kg (d) 3 kg
64. Find which of the following compound can have mass 76. 1.25 g of a sample of Na2CO3 and Na2SO4 is dissolved in
ratios of C:H:O as 6:1:24 250 mL solution. 25 mL of this solution neutralises 20 mL
(a) HO-(C=O)-OH (b) HO-(C=O)-H of 0.1N H2SO4.The % of Na2CO3 in this sample is
(c) H-(C=O)-H (d) H3CO-(C=O)-H (a) 84.8% (b) 8.48% (c) 15.2% (d) 42.4%
77. A 5 molar solution of H2SO4 is diluted from 1 litre to a
Topic 6: Stoichiometry and Stoichiometric Calculations volume of 10 litres, the normality of the solution will be :
65. If 1.5 moles of oxygen combines with Al to form Al2O3, (a) 1N (b) 0.1N (c) 5N (d) 0.5N
78. 6.02 × 1020 molecules of urea are present in 100 mL of its
the mass of Al in g [Atomic mass of Al = 27] used in the
solution. The concentration of urea solution is
reaction is
(a) 0.02 M (b) 0.01 M (c) 0.001 M (d) 0.1 M
(a) 2.7 (b) 54 (c) 40.5 (d) 81
(Avogadro constant, NA = 6.02 × 1023 mol–1)
66. 12 g of Mg (atomic mass 24) will react completely with
79. Two solutions of a substance (non electrolyte) are mixed
hydrochloric acid to give
in the following manner. 480 mL of 1.5 M first solution +
(a) One mol of H2
520 mL of 1.2 M second solution. What is the molarity of
(b) 1/2 mol of H2
the final mixture?
(c) 2/3 mol of O2
(a) 2.70 M (b) 1.344 M (c) 1.50 M (d) 1.20 M
(d) both 1/2 mol of H2 and 1/2 mol of O2
80. For the reaction Fe2O3 + 3CO ® 2Fe + 3CO2, the volume
67. What is the weight of oxygen required for the complete
of carbon monoxide required to reduce one mole of ferric
combustion of 2.8 kg of ethylene ?
oxide is
(a) 2.8 kg (b) 6.4 kg (c) 9.6 kg (d) 96 kg
(a) 67.2 dm3 (b) 11.2 dm3
68. In the reaction
(c) 22.4 dm3 (d) 44.8 dm3
4 NH3 (g) + 5O2 (g) ® 4 NO(g) + 6 H 2 O(l)
81. Fat is an important source of energy and water. This is
When 1 mole of ammonia and 1 mole of O2 are made to
important for the desert animals like camel, which stores fat
react to completion,
in its hump and provide water and energy. How many
(a) 1.0 mole of H2O is produced
grams and moles of H2O are produced from the combustion
(b) 1.0 mole of NO will be produced
of fat C57H110O6 from 450 gram of fat stored in hump of
(c) all the oxygen will be consumed
camel ?
(d) all the ammonia will be consumed
69. What is the molarity of 0.2N Na 2 CO 3 solution? 163
(a) 0.1 M (b) 0 M (c) 0.4 M (d) 0.2 M C57 H110O6 + O2 ® 57CO 2 + 55H 2O
2
70. The molar solution of H 2SO 4 is equal to :
(a) 500.56 , 27.80 (b) 450, 26.80
(a) N/2 solution (b) N solution
(c) 580, 25.0 (d) 400, 26.6
(c) 2N solution (d) 3N solution
71. One kilogram of a sea water sample contains 6 mg of 82. Which of the following option represents correct limiting
dissolved O2. The concentration of O2 in the sample in reagents in reactions (i), (ii) and (iii) respectively.
ppm is (i) C + O2 ® CO2
(26g) (20g)
(a) 0.6 (b) 6.0
(c) 60.0 (d) 16.0 (ii) N2 + 3H2 ® 2NH3
(60g) (80g)
72. Match the columns.
Column-I Column-II (iii) P4 + 3O2 ® P4O6
(Physical quantity) (Unit) (100g) (200g)
(A) Molarity (p) mol (a) C, N2, O2 (b) C, N2, P4
(B) Mole fraction (q) Unitless (c) O2, H2, P4 (d) O2, N2, P4
(C) Mole (r) mol L–1 83. 10 mL of 2M NaOH solution is added to 200 mL of
(D) Molality (s) mol kg–1 0.5 M of NaOH solution. What is the final concentration ?
(a) A – (r), B – (q), C – (s), D – (p) (a) 0.57 M (b) 5.7 M (c) 11.4 M (d) 1.14 M
(b) A – (r), B – (p), C – (q), D – (s) 84. If maximum fluoride ion presence was set to be 4 ppm,
(c) A – (r), B – (q), C – (p), D – (s) what is the maximum number of moles of fluoride in 10
(d) A – (q), B – (r), C – (p), D – (s) mL drinking water ?
73. Assuming fully decomposed, the volume of CO2 released (a) 2.10 × 10–3 (b) 2.10 × 10–2
at STP on heating 9.85 g of BaCO3 (Atomic mass, Ba = 137) (c) 3.10 × 10 –3 (d) 3.3 × 10–2
will be
SOME BASIC CONCEPTS OF CHEMISTRY 9

85. The increasing order of molarity with 25 g each of NaOH, 89. A binary mixture of bivalent metals having mass 2 g,
LiOH, Al(OH)3, KOH, B(OH)3 in same volume of water? molecular mass of A and B are 15 and 30 respectively, is
(a) Al(OH)3 < B(OH)3 < KOH < NaOH < LiOH dissolved in HCl, it evolve 2.24 L H2 at STP, what is the
(b) LiOH < NaOH < KOH < B(OH)3 < Al(OH)3 mass of A present in mixture?
(c) LiOH < NaOH < B(OH)3 < KOH < Al(OH)3 (a) 1 g (b) 1.5 g (c) 0.5 g (d) 0.75 g
(d) NaOH < LiOH < B(OH)3 < Al(OH)3 < KOH 90. Assertion : In the titration of Na2CO3 with HCl using
86. The normality of 26% (wt/vol) solution of ammonia methyl orange indicator, the volume required at the
(density = 0.855 ) is approximately : equivalence point is twice that of the acid required using
(a) 1.5 (b) 0.4 (c) 15.3 (d) 4 phenolphthalein indicator.
87. Volume of water needed to mix with 10 mL 10N HNO3 to Reason : Two moles of HCl are required for the complete
get 0.1 N HNO3 is : neutralization of one mole of Na2CO3.
(a) 1000 mL (b) 990 mL (c) 1010 mL (d) 10 mL (a) Assertion is correct, reason is correct; reason is a
88. If a 25.0 mL sample of sulfuric acid is titrated with 50.0 mL correct explanation for assertion.
of 0.025 M sodium hydroxide to a phenolphthalein (b) Assertion is correct, reason is correct; reason is not a
endpoint, what is the molarity of the acid? correct explanation for assertion.
(a) 0.020 M (b) 0.100 M (c) Assertion is correct, reason is incorrect.
(c) 0.025 M (d) 0.050 M (d) Assertion is incorrect, reason is correct.

1. What is the mass of water (in g) produced from 445 g 7. 1.500g of hydroxide of a metal gave 1.000g of its oxide on
of C57H110O6 in the following reaction? heating. What is the equivalent mass of the metal ?
2C57H110O6(s) + 163 O2(g) ® 114CO2(g) + 110 H2O(1) 8. Limestone (CaCO3) decomposes into quicklime (CaO) on
2. What amount of sugar (C12H22O11) (in g) is required to strong heating. How much quantity of limestone will be
prepare 2 L of its 0.1 M aqueous solution? required to prepare 100 kg of quicklime?
3. 5 moles of AB2 weigh 125 × 10–3 kg and 10 moles of A2B2
9. 1 mole of mixture of CO and CO2 requires exactly 28 g KOH
weigh 300 × 10–3 kg. What is the sum of molar mass of A in solution for complete conversion of all the CO2 into
(MA) and molar mass of B (MB) in g mol–1 ? K2CO3. How much amount more of KOH (in g) will be
4. 5.0 g of a certain element X forms 10.0 g of its oxide having required for conversion into K2CO3. (If one mole of mixture
the formula X4O6. What is the atomic mass of X in amu? is completely oxidized to CO2)?
5. A 100% pure sample of a divalent metal carbonate weighing 2 10. 1 mol of N2 and 4 mol of H2 are allowed to react in a sealed
g on complete thermal decomposition releases 448 cc of carbon container and after the reaction some water is introduced
dioxide at STP. What is the equivalent mass of the metal? in it. The aqueous solution formed required 1 L of 1 M HCl
6. The specific heat of a metal is 0.16 cal g–1. The equivalent for neutralization. Calculate the mole fraction of the
mass of the metal is 20.04. What is the correct atomic mass gaseous product in the mixture after the reaction.
of the metal in grams?

Exercise 3 : NCERT Exemplar & Past Year MCQs


NCERT Exemplar MCQs (a) Results of both the students are neither accurate nor
precise.
1. Two students performed the same experiment separately (b) Results of student A are both precise and accurate.
and each one of them recorded two readings of mass which (c) Results of student B are neither precise nor accurate.
are given below. Correct reading of mass is 3.0 g. On the (d) Results of student B are both precise and accurate.
basis of given data, mark the correct option out of the 2. A measured temperature on Fahrenheit scale is 200°F.
following statements. What will this reading be on celsius scale?
(a) 40 °C (b) 94 °C (d) 93.3 °C (d) 30 °C
Students Readings
3. What will be the molarity of a solution, which contains 5.85
(i) (ii) g of NaCl(s) per 500 mL?
(a) 4 mol L–1 (b) 20 mol L–1
A 3.01 2.99
(c) 0.2 mol L–1 (d) 2 mol L–1
B 3.05 2.95
EBD_8350
10 CHEMISTRY

4. If 500 mL of a 5 M solution is diluted to 1500 mL, what will (b) Total mass of reactants = total mass of product,
be the molarity of the solution obtained? therefore, law of multiple proportions is followed.
(a) 1.5 M (b) 1.66 M (c) Amount of Fe2O3 can be increased by taking any
(c) 0.017 M (d) 1.59 M one of the reactants (iron or oxygen) in excess.
5. The number of atoms present in one mole of an element is (d) Amount of Fe2O3 produced will decrease if the
amount of any one of the reactants (iron or oxygen)
equal to Avogadro number. Which of the following element
is taken in excess.
contains the greatest number of atoms? 15. Which of the following statements indicates that law of
(a) 4 g He (b) 46 g Na multiple proportion is being followed?
(c) 0.40 g Ca (d) 12 g He (a) Sample of carbon dioxide taken from any source will
6. If the concentration of glucose (C6H12O6) in blood is always have carbon and oxygen in the ratio 1 : 2.
0.9 g L–1, what will be the molarity of glucose in blood? (b) Carbon forms two oxides namely CO2 and CO, where
(a) 5 M (b) 50 M (c) 0.005 M (d) 0.5 M masses of oxygen which combine with fixed mass of
7. What will be the molality of the solution containing 18.25 carbon are in the simple ratio 2 : 1.
g of HCl gas in 500 g of water? (c) When magnesium burns in oxygen, the amount of
(a) 0.1 m (b) 1 M (c) 0.5 m (d) 1 m magnesium taken for the reaction is equal to the
8. What is the mass percent of carbon in carbon dioxide? amount of magnesium in magnesium oxide formed.
(d) At constant temperature and pressure 200 mL of
(a) 0.034% (b) 27.27% (c) 3.4% (d) 28.7%
hydrogen will combine with 100 mL oxygen to
9. The empirical formula and molecular mass of a compound produce 200 mL of water vapour.
are CH2O and 180 g respectively.What will be the molecular
formula of the compound? Past Year MCQs
(a) C9H18O9 (b) CH2O (c) C6H12O6 (d) C2H4O2 16. When 22.4 litres of H2(g) is mixed with 11.2 litres of Cl2(g),
10. If the density of a solution is 3.12 g mL–1, the mass of each at S.T.P., the moles of HCl(g) formed is equal to:-
1.5 mL solution in significant figures is...... [AIPMT 2014, S]
(a) 4.7 g (b ) 4680 ×10–3 g (a) 1 mol of HCl(g) (b) 2 mol of HCl(g)
(c) 4.680 g (d) 46.80 g (c) 0.5 mol of HCl(g) (d) 1.5 mol of HCl(g)
11. Which of the following statements about a compound is 17. 1.0 g of magnesium is burnt with 0.56 g O2 in a closed
incorrect? vessel. Which reactant is left in excess and how much ?
(a) A molecule of a compound has atoms of different (At. wt. Mg = 24 ; O = 16) [AIPMT 2014, S]
elements. (a) Mg, 0.16 g (b) O2, 0.16 g
(b) A compound cannot be separated into its constituent (c) Mg, 0.44 g (d) O2, 0.28 g
18. The ratio of masses of oxygen and nitrogen in a particular
elements by physical methods of separation.
gaseous mixture is 1 : 4. The ratio of number of their
(c) A compound retains the physical properties of its
molecule is: [JEE M 2014, S]
constituent elements. (a) 1 : 4 (b) 7 : 32 (c) 1 : 8 (d) 3 : 16
(d) The ratio of atoms of different elements in a 19. A mixture of gases contains H2 and O2 gases in the ratio of
compound is fixed. 1 : 4 (w/w). What is the molar ratio of the two gases in the
12. Which of the following reactions is not correct according mixture ? [AIPMT 2015, S]
to the law of conservation of mass? (a) 4 : 1 (b) 16 : 1 (c) 2 : 1 (d) 1 : 4
(a) 2Mg (s) + O2 (g) ¾¾ ® 2MgO (s) 20. 20.0 g of a magnesium carbonate sample decomposes on
(b) C3H8 (g) + O2 (g) ¾¾ ® CO2 (g) + H2O (g) heating to give carbon dioxide and 8.0 g magnesium oxide.
What will be the percentage purity of magnesium carbonate
(c) P4 (s) + 5O2 (g) ¾¾ ® P4O10 (s)
in the sample ? [AIPMT 2015, S]
(d) CH4 (g) + 2O2 (g) ¾¾ ® CO2 (g) + 2H2O (g) (a) 75 (b) 96 (c) 60 (d) 84
13. One mole of any substance contains 6.022 × 10 23 21. The number of water molecules is maximum in :
atoms/molecules. Number of molecules of H2SO4 present [AIPMT 2015, S]
in 100 mL of 0.02 M H2SO4 solution is........ (a) 18 molecules of water (b) 1.8 gram of water
(a) 12.044 × 1020 molecules (b) 6.022 × 1023 molecules (c) 18 gram of water (d) 18 moles of water
(c) 1 × 1023 molecules (d) 12.044 × 1023 molecules 22. If Avogadro number NA, is changed from 6.022 × 1023 mol–1
14. Which of the following statements is correct about the to 6.022 × 1020 mol–1, this would change :
reaction given below? [AIPMT 2015, S]
(a) the definition of mass in units of grams
4Fe (s) + 3O2 (g) ¾¾ ® 2Fe2O3 (g)
(b) the mass of one mole of carbon
(a) Total mass of iron and oxygen in reactants = total (c) the ratio of chemical species to each other in a
mass of iron and oxygen in product therefore it balanced equation.
follows law of conservation of mass. (d) the ratio of elements to each other in a compound
SOME BASIC CONCEPTS OF CHEMISTRY 11

23. What is the mass of precipitate formed when 50 mL of 29. In which case is number of molecules of water maximum?
16.9% solution of AgNO3 is mixed with 50 mL of 5.8% NaCl (a) 18 mL of water [NEET 2018, S]
solution? (b) 0.18 g of water
(Ag = 107.8, N = 14, O = 16, Na = 23, Cl = 35.5) (c) 10–3 mol of water
[AIPMT 2015, S] (d) 0.00224 L of water vapours at 1 atm and 273 K
(a) 28 g (b) 3.5 g (c) 7 g (d) 14 g 30. The ratio of mass percent of C and H of an organic
24. The molecular formula of a commercial resin used for compound (CxHyOz) is 6 : 1. If one molecule of the above
exchanging ions in water softening is C8H7SO3– Na + compound (CxHyOz) contains half as much oxygen as
(Mol. wt. 206. What would be the maximum uptake of Ca2 + required to burn one molecule of compound C xHy
ions by the resin when expressed in mole per gram resin? completely to CO2 and H2O. The empirical formula of
compound CxHyOz is : [JEE M 2018, S]
[JEE M 2015, S]
(a) C3H6O3 (b) C2H4O (c) C3H4O2 (d) C2H4O3
2 1 1 1
(a) (b) (c) (d) 31. The number of moles of hydrogen molecule required to
309 412 103 206 produce 20 moles of ammonia through Haber’ s process is:
25. At 300 K and 1 atm, 15 mL of a gaseous hydrocarbon [NEET 2019, A]
requires 375 mL air containing 20% O2 by volume for (a) 10 (b) 20 (c) 30 (d) 40
complete combustion. After combustion the gases occupy 32. For a reaction, N2(g) + 3H2(g) ® 2NH3(g); identify
330 mL. Assuming that the water formed is in liquid form dihydrogen (H2) as a limiting reagent in the following
and the volumes were measured at the same temperature reaction mixtures. [JEE Main 2019, C]
and pressure, the formula of the hydrocarbon is: (a) 56 g of N2 + 10 g of H2 (b) 35 g of N2 + 8 g of H2
[JEE M 2016, S] (c) 28 g of N2 + 6 g of H2 (d) 14 g of N2 + 4 g of H2
(a) C4H8 (b) C4H10 (c) C3H6 (d) C3H8 33. Which one of the followings has maximum number of
26. The most abundant elements by mass in the body of a atoms ? [NEET 2020, A]
healthy human adult are : (a) 1 g of Mg(s) [Atomic mass of Mg = 24]
Oxygen (61.4%) ; Carbon (22.9%), Hydrogen (10.0%) ; and (b) 1 g of O2(g) [Atomic mass of O = 16]
Nitrogen (2.6%). The weight which a 75 kg person would (c) 1 g of Li(s) [Atomic mass of Li = 7]
gain if all 1H atoms are replaced by 2H atoms is (d) 1 g of Ag(s) [Atomic mass of Ag = 108]
[JEE M 2017, S] 34. Amongst the following statements, that which was not
(a) 15 kg (b) 37.5 kg (c) 7.5 kg (d) 10 kg proposed by Dalton was: [JEE Main 2020, C]
(a) Chemical reactions involve reorganization of atoms.
27. 1 gram of a carbonate (M2CO3) on treatment with excess These are neither created nor destroyed in a chemical
HCl produces 0.01186 mole of CO2. The molar mass of reaction.
M2CO3 in g mol–1 is : [JEE M 2017, S]
(b) All the atoms of a given element have identical
(a) 1186 (b) 84.3 (c) 118.6 (d) 11.86 properties including identical mass. Atoms of different
28. A mixture of 2.3 g formic acid and 4.5 g oxalic acid is treated elements differ in mass.
with conc. H2SO4. The evolved gaseous mixture is passed (c) When gases combine or reproduced in a chemical
through KOH pellets. Weight (in g) of the remaining reaction they do so in a simple ratio by volume
product at STP will be [NEET 2018, S] provided all gases are at the same T & P.
(a) 1.4 (b) 3.0 (c) 4.4 (d) 2.8 (d) Matter consists of indivisible atoms.

Exercise 4 : Problem Solving Skill Enhancer MCQs


1. If NA is Avogadro’s number, then number of valence 3. A certain public water supply contains 0.10 ppb (part per
electrons in 4.2g of nitride ions (N3–) is billion) of chloroform (CHCl3). How many molecules of
(a) 2.4 NA (b) 4.2 NA CHCl3 would be obtained in 0.478 mL drop of this water?
(c) 1.6 NA (d) 3.2 NA (a) 4 × 10–13 × NA (b) 10–3 × NA
2. Liquid benzene (C6H6) burns in oxygen according to the –10
(c) 4 × 10 × NA (d) None of these
equation
2C6 H 6 (l) + 15O2 (g) ¾¾® 12CO2 (g) + 6 H 2 O (g) 4. A 25.0 mm × 40.0 mm piece of gold foil is 0.25 mm thick.
How many litres of O2 at STP are needed to complete the The density of gold is 19.32 g/cm3. How many gold atoms
combustion of 39 g of liquid benzene?(Mol. wt. of O2 = 32, are in the sheet? (Atomic weight : Au = 197.0)
C6H6 = 78) (a) 7.7 × 1023 (b) 1.5 × 1023
(a) 74 L (b) 11.2 L (c) 4.3 × 10 21 (d) 1.47 × 1022
(c) 22.4 L (d) 84 L
EBD_8350
12 CHEMISTRY

5. The relation between molarity (M) and molality (m) is given (c) 80% Mg and 20% MgO
by: (d) 20% Mg and 80% MgO
(p = density of solution (g/mL), M1 = molecular mass of 10. 32 g of a sample of FeSO4.7H2O were dissolved in dilute
solute) sulphuric acid and water and its volume was made up to
1litre. 25 mL of this solution required 20 mL of 0.02 M
1000 M 1000 pM KMnO4 solution for complete oxidation. Calculate the
(a) m= (b) m=
1000 p - M1 1000 p - MM1 mass % of FeSO4.7H2O in the sample.
(a) 34.75 (b) 69.5
1000 MM 1000 M
(c) m= (d) m = (c) 89.5 (d) None of these
1000 p - MM1 1000 p - MM1 11. What quantity (in mL) of a 45% acid solution of a
6. A bottle of an aqueous H2O2 solution is labelled as '28 V' mono-protic strong acid must be mixed with a 20% solution
H2O2 and the density of the solution (in g/mL) is 1.25. of the same acid to produce 800 mL of a 29.875% acid
Choose the correct option: solution?
(a) Molality of H2O2 solution is 2 (a) 320 (b) 325 (c) 316 (d) 330
(b) Molarity of H2O2 solution is 5 12. 4 mole of a mixture of Mohr's salt and Fe2(SO4)3 requires
(c) Molality of H2O2 solution is 2.15 500 mL of 1 M K2Cr2O7 for complete oxidation in acidic
(d) None of these medium. The mole % of the Mohr's salt in the mixture is:
7. A gaseous compound is composed of 85.7% by mass (a) 25 (b) 50 (c) 60 (d) 75
carbon and 14.3% by mass hydrogen. Its density is 2.28 13. Consider the reaction 2 A + B + 3C ® P + 2Q . Starting
g/litre at 300 K and 1.0 atm pressure. Determine the with 3 mol of A, 2 mol of B and 6 mol of C, number of moles
molecular formula of the compound. of the products P and Q would respectively be:
(a) C2H2 (b) C2H4 (c) C4H8 (d) C4H10 (a) 2 and 4 (b) 4 and 2 (c) 3 and 1.5 (d) 1.5 and 3
8. Percent by mass of a solute (molar mass = 28 g) in its
14. A 0.9157 g mixture of CaBr2 and NaBr is dissolved in water,
aqueous solution is 28. Calculate the mole fraction (X) and
and AgNO3 is added to the solution to form AgBr
molality (m) of the solute in the solution.
precipitate. If the mass of the precipitate is 1.6930 g, what
(a) X = 0.2, m = 10 (b) X = 0.2, m = 125/9
is the percent by mass of NaBr in the original mixture?
(c) X = 0.8, m = 125/9 (d) X = 0.8, m = 10
9. 320 mg of a sample of magnesium having a coating of its (a) 56.18 (b) 45.73 (c) 58.25 (d) 54.18
oxide required 20 mL of 0.1 M hydrochloric acid for the 15. Suppose an element X and Y combine to form two
complete neutralisation of the latter. The composition of compounds XY2 and X3Y2 when 0.1 mole of former weigh
the sample is: 10 g while 0.05 mole of the latter weigh 9 g. What are the
(a) 87.5% Mg and 12.5% MgO atomic weights of X and Y.
(b) 12.5% Mg and 87.5% MgO (a) 40, 30 (b) 60, 40 (c) 20, 30 (d) 30, 20

ANSWER KEY
Exercise 1 : NCERT Based Topic-wise MCQs
1 (d) 10 (c) 19 (b) 28 (a) 37 (b) 46 (a) 55 (b) 64 (a) 73 (c) 82 (d)
2 (c) 11 (c) 20 (c) 29 (d) 38 (d) 47 (a) 56 (c) 65 (b) 74 (b) 83 (a)
3 (b) 12 (a) 21 (b) 30 (b) 39 (a) 48 (b) 57 (a) 66 (b) 75 (a) 84 (a)
4 (d) 13 (c) 22 (d) 31 (b) 40 (d) 49 (c) 58 (c) 67 (c) 76 (a) 85 (a)
5 (c) 14 (c) 23 (c) 32 (b) 41 (b) 50 (d) 59 (a) 68 (c) 77 (a) 86 (c)
6 (a) 15 (a) 24 (c) 33 (a) 42 (a) 51 (d) 60 (c) 69 (a) 78 (b) 87 (b)
7 (d) 16 (c) 25 (c) 34 (b) 43 (b) 52 (c) 61 (d) 70 (c) 79 (b) 88 (c)
8 (a) 17 (d) 26 (b) 35 (c) 44 (b) 53 (d) 62 (a) 71 (b) 80 (a) 89 (a)
9 (a) 18 (c) 27 (a) 36 (d) 45 (a) 54 (b) 63 (b) 72 (c) 81 (a) 90 (b)
Exercise 2 : Numeric/Integer Answer Questions
1 (495) 2 (68.4) 3 (15) 4 (24) 5 (20) 6 (40.08) 7 (10) 8 (178.57) 9 (84) 10 (10.25)
Exercise 3 : NCERT Exemplar & Past Year MCQs
1 (b) 5 (d) 9 (c) 13 (a) 17 (a) 21 (d) 25 (d) 29 (a) 33 (c)
2 (c) 6 (c) 10 (a) 14 (a) 18 (b) 22 (b) 26 (c) 30 (c) 34 (c)
3 (c) 7 (d) 11 (c) 15 (b) 19 (a) 23 (c) 27 (b) 31 (c)
4 (b) 8 (b) 12 (b) 16 (a) 20 (d) 24 (b) 28 (d) 32 (a)
Exercise 4 : Problem Solving Skill Enhancer MCQs
1 (a) 3 (a) 5 (d) 7 (c) 9 (a) 11 (c) 13 (d) 15 (a)
2 (d) 4 (d) 6 (c) 8 (b) 10 (b) 12 (d) 14 (a)
2 Structure of Atom

Trend Buster NEET & JEE Main

Number of Questions from 2020-15 5 6 Minimum one question has been asked
Weightage 2.2% 3.4% every year in NEET & JEE Main.

The most Important Concepts that Cover Maximum number of Questions asked in past 6 years.

Bohr’s model of H-atom 2 4

Less Important Concepts that Cover 1 or 2 Questions asked in past 6 years.

Quantum numbers 1 1
de broglie wavelength — 1
Orbital energy and stability 2

NEET JEE
Concept Used

2020 Quantum mechanical Quantum number — — 1 Easy


model of atom

2019 Bohr’s model of H-atom / Bohr’s model of H-atom / 2 Easy / 2 Average


Towards Quantum Orbital energy and stability Average
mechanical model of an atom

2018 Bohr’s model of H-atom Bohr’s model of H-atom 1 Easy — —

2017 Bohr’s model of H-atom / Bohr’s model of H-atom / 1 Average 1 Average


Towards Quantum Uncertainly principle / orbital
mechanical model of an atom stability / de-broglie wavelength

2016 Quantum mechanical model Quantum numbers / 1 Easy 1 Average


of an atom / Towards Quantumde-broglie wavelength
mechanical model of an atom

2015 Bohr’s model of H-atom Bohr’s model of H-atom — — 1 Average


EBD_8350
14 CHEMISTRY
STRUCTURE OF ATOM 15
EBD_8350
16 CHEMISTRY

ProblemCritical
SolvingPoints
Tips/ Tricks/ Points to Remember
to Remember

n2 4 (n + l) rule
4 The wavelength of limiting line = 1 for all series. So,
1 RH This rule states that electrons are filled in orbitals
for lyman series = according to their increasing values of n + l. When (n + l)
RH
is same for sub energy levels, the electrons first occupy
4 Spectral Series n1 n2 Region
the sublevels with lowest “n” value. Thus, order of filling
(i) Lyman Series 1 2, 3, 4 ... UV up of orbitals is as follows:
(ii) Balmer Series 2 3, 4 ... Visible
1s < 2s < 2p < 3s < 3p < 4s < 3d < 4p < 5s < 4d < 5p < 6s
(iii) Paschen Series 3 4, 5, 6 ... IR < 4f < 5d
(iv) Brackett Series 4 5, 6, 7 ... IR
4 Orbital & Spin Angular Momentum
(v) Pfund Series 5 6, 7 ... IR
h h
(vi) Humphrey 6 7, 8 ... Far IR l(l + 1), s(s + 1)
2p 2p
4 In terms of time and energy Heisenberg’s uncertainty
1/2 mn2 = hn – hn0(w) (work function or B.E.)
h
principle may be given as DE Dt ³ (for energy and hc
4p n0 = Thershold frequency W = hn0 =
time) l0
4 Radii of Orbits 4 Relation Between Potential energy (P.E), Kinetic energy
(K.E) & Total energy
(4p Î0 )n 2 h 2
r=
4p 2 mZe2 P.E.
T.E. = = – K.E.
By putting value of constants 2

n2 o n2 Ze2 1 Ze 2 1 Ze 2
r = 0.529 ´ A r = 0.529 ´ ´ 10–10 m P.E. = – , K.E. = , T.E. = –
Z Z r 2 r 2 r
n2 4 Rydberg Equation
r = 0.529 ´ 10–8 ´ cm
Z The wavelength (l), wave number ( n– ) for the
For H-like atoms. electromagnetic radiation can be calculated by Rydberg
Thus rn = r1 × n2 equation.
4 Speed of Electron 1 é 1 1 ù

2
n= = R ´ Z2 ê – ú
2pZe Z l 2 2
ëê n1 n 2 úû
v= = 2.188 ´ 106 ´ m / sec
(4p Î0 )nh n
Z = atomic number
4 The Energy of Electron R = 109677 cm–1 - Rydberg constant
Total energy (E) = K.E. + P.E. n2 = higher orbit
n1 = lower orbit
2p2 Z2 me 4
En = - 4 Total number of spectral lines
(4p Î0 )2 n 2 h 2
n(n –1)
where n = orbit number (i) ® when electron jumps from n th level
2
Z2 to ground level
E n = E1 ´ for H-like atoms
n2 (n 2 – n1 )(n 2 – n1 + 1)
(ii) ® when electron returns
21.79 ´ 10 –19 Z2 2
En = – 13.6
2 J/atom = – Z2 eV per atom from n2 to n1.
n n2
313.6 ´ Z 2
=– kcal/mol
n2
STRUCTURE OF ATOM 17

Exercise 1 : NCERT Based Topic-wise MCQs


Topic 1: Sub-Atomic Particles (c) A beam of neutrons
(d) A beam of neutrons and protons
1. Which of the scientist were able to prove that atom is no 10. Neutron is discovered by
longer non-divisible? (a) Chadwick (b) Rutherford
(a) Dalton (b) Michael Faraday (c) Yukawa (d) Dalton
(c) Thomson (d) Chadwick 11. What is the ratio of mass of an electron to the mass of a
2. Which of the following is never true for cathode rays ? proton?
(a) They possess kinetic energy. (a) 1 : 2 (b) 1 : 1 (c) 1 : 1837 (d) 1 : 3
(b) They are electromagnetic waves. 12. In which of the following the amount of deviation from
(c) They produce heat. their path in the presence of electric and magnetic field
(d) They produce mechanical pressure. will be maximum?
3. While performing cathode ray experiments, it was observed (a) N2– (b) N3– (c) N– (d) N
that there was no passage of electric current under normal 13. The increasing order for the values of e/m (charge/mass) is
conditions. Which of the following can account for this (a) e, p, n, a (b) n, p, e, a
observation ? (c) n, p, a, e (d) n, a, p, e
(a) Dust particles are present in air 14. The deflection of the particles from their path in presence
(b) Carbon dioxide is present in air of electric and magnetic field will be maximum in which of
(c) Air is a poor conductor of electricity under normal the following.
conditions (a) O (b) N (c) U (d) He
(d) None of the above 15. Which of the following pairs have identical values of e/m?
4. Which is not true with respect to cathode rays? (a) A proton and a neutron
(a) A stream of electrons (b) A proton and deuteron
(b) Charged particles (c) Deuteron and an a-particle
(c) Move with speed same as that of light (d) An electron and g-rays
(d) Can be deflected by magnetic fields
5. What is the optimum conditions required to study the Topic 2: Atomic Models
conduction of electricity through gases.
(a) High pressure and low voltage 16. Rutherford’s experiment on the scattering of a -particles
(b) High pressure and high voltage showed for the first time that the atom has :
(c) Low pressure and high voltage (a) electrons (b) protons
(d) Low pressure and low voltage (c) nucleus (d) neutrons'
6. In discharge tube experiment stream of negatively charged 17. When atoms are bombarded with alpha particles, only a
particles travel from few in million suffer deflection, oth ers pass out
(a) anode to cathode undeflected.
(b) cathode to anode This is because
(c) Both (a) and (b) (a) the force of repulsion on the moving alpha particle is
(d) Electrons does not travel small
7. Millikan performed an experiment method to determine (b) the force of attraction between alpha particle and
which of the following? oppositely charged electrons is very small
(a) Mass of the electron (b) Charge of the electron (c) there is only one nucleus and large number of
(c) e/m ratio of electron (d) Both (a) and (b) electrons
8. Which is correct statement about proton? (d) the nucleus occupies much smaller volume compared
(a) Proton is nucleus of deuterium to the volume of the atom
(b) Proton is a-particle 18. Rutherford’s a-particle dispersion experiment concludes
(c) Proton is ionized hydrogen molecule (a) all positive ions are deposited at small part
(d) Proton is ionized hydrogen atom (b) all negative ions are deposited at small part
9. When beryllium is bombarded with alpha particles (c) proton moves around the electron
(Chadwick’s experiment), extremely penetrating radiations, (d) neutrons are charged particles.
which cannot be deflected by electrical or magnetic field are 19. Which of the following scientists explained his model on
given out. These are : the basis of centrifugal force ?
(a) A beam of protons (a) Thomson (b) Dalton
(b) Alpha rays (c) Millikan (d) Rutherford
EBD_8350
18 CHEMISTRY

20. Which of the following does not contain number of (a) Increases by 33% (b) Decreases by 33%
40
neutrons equal to that of 18 Ar ? (c) Increases by 66% (d) Decreases by 66%
(a) 41 (b) 43 (c) 40 (d) 42 29. Which one of the following is not the characteristic of
19 K 21 Sc 21 Sc 20 Ca
Planck’s quantum theory of radiation ?
21. Number of protons, neutrons and electrons in the element
(a) The energy is not absorbed or emitted in whole number
231
89 X is or multiple of quantum
(a) 89, 89, 242 (b) 89, 142, 89 (b) Radiation is associated with energy
(c) 89, 71, 89 (d) 89, 231, 89 (c) Radiation energy is not emitted or absorbed
22. Which of the following pairs will have same chemical continuously but in the form of small packets called
properties ? quanta
(a) 14 and 15 (b) O2– and F– (d) This magnitude of energy associated with a quantum
6 C 7 N
40 40 35 37 is proportional to the frequency.
(c) 18 Ar and 19 K (d) 17 Cl and 17 Cl
23. What is the difference between two species if one has 30. The value of Planck's constant is 6.63 × 10–34 Js. The
atomic mass = 14 and atomic number = 7 whereas the velocity of light is 3.0 × 108 m s–1. Which value is closest to
other has atomic mass = 14 and atomic number = 6? the wavelength in nanometers of a quantum of light with
(a) Neutrons (b) Protons frequency of 8 × 1015 s–1 ?
(c) Electrons (d) All of these (a) 3 × 107 (b) 2 × 10–25 (c) 5 × 10–18 (d) 4 × 101
24. From the data given below A, B, C and D respectively 31. In the photo-electron emission, the energy of the emitted
are, electron is
(A) 10 e–, atomic no. 11 (B) 10 e–, atomic no. 6
(a) greater than the incident photon
(C) 10 e–, atomic no. 10 (D) 10 e–, atomic no. 9
(a) Na+, C4–, Ne, F– (b) C4–, Ne, Na–, F– (b) same as than of the incident photon
– +
(c) F , Na , Ne, C 4– (d) F–, Na+, C4–, Ne (c) smaller than the incident photon
25. Match the columns. (d) proportional to the intensity of incident photon
Column-I Column-II 32. In photoelectric effect, at which frequency, electron will
1 2
(A) 1 H, 1 H and 13 H (p) Isobars be ejected with certain kinetic energy (n0 = threshold
frequency).
(B) 14 and 14 (q) Isotopes
6C 7N (a) n > n0 (b) n0 > n (c) n0 ³ n (d) n ³ n0
(C) Na+and Mg2+ (r) Isoelectronic species 33. Which of the following statements are not correct about
(a) A – (p), B – (q), C – (r) electromagnetic radiation ?
(b) A – (q), B – (p), C – (r) (i) Electromagnetic waves require medium to travel.
(c) A – (r), B – (q), C – (p)
(ii) Different electromagnetic radiations travel at same
(d) A – (p), B – (r), C – (q)
26. The number of electrons, neutrons and protons in a species speed in vaccum.
are equal to 10, 8 and 8 respectively. The proper symbol of (iii) The oscillating electric and magnetic fields produced
the species is by oscillating charged particles are perpendicular to
(a) 16O 8 (b) 18O 8 (c) 18Ne 10 (d) 16O82– each other, but not to the direction of propagation.
27. Assertion : All isotopes of a given element show the same (iv) The oscillating electric field and magnetic field are
type of chemical behaviour. perpendicular to each other, and also to the direction
Reason : The chemical properties of an atom are controlled of propagation.
by the number of electrons in the atom. (a) (i), (ii) and (iii) (b) (ii) and (iii)
(a) Assertion is correct, reason is correct; reason is a (c) (i) and (iii) (d) (i) and (iv)
correct explanation for assertion. 34. Assertion : Black body is an ideal body that emits and
(b) Assertion is correct, reason is correct; reason is not absorbs radiations of all frequencies.
a correct explanation for assertion Reason : The frequency of radiation emitted by a body
(c) Assertion is correct, reason is incorrect goes from a lower frequency to higher frequency with an
(d) Assertion is incorrect, reason is correct.
increase in temperature.
Topic 3: Developments Leading to the Bohr’s Model of Atom (a) Assertion is correct, reason is correct; reason is a
correct explanation for assertion.
28. If the wavelength of the electromagnetic radiation is (b) Assertion is correct, reason is correct; reason is not
increased to thrice the digital value, then what will be the a correct explanation for assertion
percent chan ge in th e value of frequency of the (c) Assertion is correct, reason is incorrect
electromagnetic radiation. (d) Assertion is incorrect, reason is correct.
STRUCTURE OF ATOM 19

35. Arrange the electromagnetic radiations a, b, c, d and e 41. According to Bohr's theory, the angular momentum of an
in increasing order of energy. Frequencies of a, b and c electron in 5th orbit is
are 1015, 1014 and 1017 respectively whereas wavelength (a) 10 h/p (b) 2.5 h/p (c) 25 h/p (d) 1.0 h/p
of (d) and (e) are 350 nm and 100 nm respectively ? 42. The third line of the Balmer series, in the emission spectrum
(a) a, b, c, d, e (b) a, b, d, e, c of the hydrogen atom, is due to the transition from the
(c) a, d, b, e, c (d) b, d, a, e, c (a) fourth Bohr orbit to the first Bohr orbit
36. Which of the following statement(s) is/are incorrect
(b) fifth Bohr orbit to the second Bohr orbit
regarding photoelectric effect?
(i) The number of electrons ejected is proportional to (c) sixth Bohr orbit to the third Bohr orbit
the intensity of light. (d) seventh Bohr orbit to the third Bohr orbit
(ii) There is some time lag between the striking of light 43. Which one of the following pairs is not correctly
beam on the metal surface and the ejection of electrons. matched ?
(iii) The kinetic energy of ejected electrons depends upon (a) Rutherford-Proton (b) J.J. Thomson-Electron
the brightness of light. (c) J.H. Chadwick-Neutron (d) Bohr-Isotopes
(iv) The kinetic energy of the ejected electron is 44. If r is the radius of the first orbit, the radius of nth orbit of
proportional to the frequency of the incident radiation. H-atom is given by
(a) (i) and (ii) (b) (ii) and (iii)
(a) rn2 (b) rn (c) r/n (d) r2 n2
(c) (ii) only (d) (ii) and (iv)
45. The radius of hydrogen atom in the ground state is 0.53 Å.
37. What will be the difference between electromagnetic
The radius of Li2+ ion (atomic number = 3) in a similar state is
radiation shown in A and B respectively ?
(a) 0.17 Å (b) 0.265 Å (c) 0.53 Å (d) 1.06 Å
46. The energy of second Bohr orbit of the hydrogen atom is
-328 kJ mol-1; hence the energy of fourth Bohr orbit would
(A)
be:
(a) -41 kJ mol-1 (b) -82 kJ mol-1
(c) -164 kJ mol -1 (d) -1312 kJ mol-1
47. The energy of an electron in second Bohr orbit of hydrogen
(B) atom is :
(a) –5.44 × 10–19 eV (b) –5.44 × 10–19 cal
(c) –5.44 × 10 kJ–19 (d) –5.44 × 10–19 J
(i) Velocity (ii) Wavelength
48. The ionisation potential of a hydrogen atom is –13.6 eV.
(iii) Frequency (iv) Energy
(a) (ii) only (b) (ii) and (iv) What will be the energy of the atom corresponding to
(c) (ii), (iii) and (iv) (d) (iv) only n = 2.
38. A 600 W mercury lamp emits monochromatic radiation of (a) – 3.4 eV (b) – 6.8 eV (c) – 1.7 eV (d) –2.7 eV
wavelength 331.3 nm. How many photons are emitted from 49. The line spectrum of He+ ion will resemble that of
the lamp per second ? (h = 6.626 × 10–34 Js; velocity of light (a) hydrogen atom (b) Li+ ion
= 3 × 108 ms–1) (c) helium atom (d) lithium atom
(a) 1 × 1019 (b) 1 × 1020 (c) 1 × 1021 (d) 1 × 1023 50. The wavelength of the radiation emitted, when in a
39. Calculate the velocity of ejected electron from the metal hydrogen atom electron falls from infinity to stationary
surface when light of frequency 2 × 1015 Hz fall on the state 1, would be (Rydberg constant = 1.097×107 m–1)
metal surface and the threshold frequency is 7 × 1014 Hz
(a) 406 nm (b) 192 nm
for metal ?
(a) 1.37 × 106 (b) 1.26 × 106 (c) 91 nm (d) 9.1×10–8 nm
(c) 1.45 × 10 7 (d) 1.37 × 107 51. The frequency of radiation emitted when the electron falls
from n = 4 to n = 1 in a hydrogen atom will be (Given :
Topic 4: Bohr’s Model for Hydrogen Atom ionization energy of H = 2.18 ×10 –18J atom –1and
40. An electron from one Bohr stationary orbit can go to next h = 6.625 × 10–34 J s )
higher orbit (a) 1.54 × 1015 s–1 (b) 1.03 × 1015 s–1
(a) by emission of electromagnetic radiation (c) 3.08 × 1015 s–1 (d) 2.00 × 1015 s–1
(b) by absorption of any electromagnetic radiation 52. The wavelength (in cm) of second line in the Lyman series
(c) by absorption of electromagnetic radiation of of hydrogen atomic spectrum is (Rydberg constant
particular frequency = R cm–1)
(d) without emission or absorption of electromagnetic æ 8R ö æ 9 ö æ 4 ö æ 3R ö
radiation (a) ç ÷ (b) ç ÷ (c) ç ÷ (d) ç ÷
è 9 ø è 8R ø è 3R ø è 4 ø
EBD_8350
20 CHEMISTRY

53. Bohr model can explain : 61. Which of the following transitions of electrons in the
(a) the solar spectrum hydrogen atom will emit maximum energy ?
(b) the spectrum of hydrogen molecule (a) n5 ® n4 (b) n4 ® n3
(c) spectrum of any atom or ion containing one electron (c) n3 ® n2 (d) all will emit same energy
only 62. The shortest wavelength in hydrogen spectrum of Lyman
(d) the spectrum of hydrogen atom only series when RH = 109678 cm–1 is
54. Which of the following statements do not form a part of (a) 1002.7 Å (b) 1215.67 Å
Bohr’s model of hydrogen atom ? (c) 1127.30 Å (d) 911.7 Å
(a) Energy of the electrons in the orbits are quantized 63. What is the expression of frequency (n) associated with
(b) The electron in the orbit nearest the nucleus has the absorption spectra of the photon.
lowest energy
(c) Electrons revolve in different orbits around the nucleus RH æ 1 1ö
(a) n= ç 2 - 2 ÷ ni > nf
(d) The position and velocity of the electrons in the orbit h è ni nf ø
cannot be determined simultaneously.
55. Bohr’s theory can be applied to which of the following ions. RH æ 1 1ö
(b) n = ç 2 - 2 ÷ nf > ni
(a) Na+ (b) Be2+ (c) Li+ (d) Li2+ h è ni nf ø
56. Bohr’s model is not able to account for which of the
following. R æ 1 1ö
(c) n = – H ç 2 - 2 ÷ nf > ni
(a) Stability of atom. h è ni nf ø
(b) Spectrum of neutral helium atom. (d) All the above are correct
(c) Energy of free electron at rest. 64. An electron, e1 is moving in the fifth stationary state, and
(d) Calculation of radii of the stationary states. another electron e2 is moving in the fourth stationary state.
57. Assertion : The radius of the first orbit of hydrogen atom The radius of orbit of electron, e1 is five times the radius
is 0.529Å. of orbit of electron, e2 calculate the ratio of velocity of
Reason : Radius of each circular orbit (rn) = 0.529Å (n2/Z), electron e1 (v1) to the velocity of electron e2 (v2).
where n = 1, 2, 3 and Z = atomic number. (a) 5 : 1 (b) 4 : 1
(a) Assertion is correct, reason is correct; reason is a (c) 1 : 5 (d) 1 : 4
correct explanation for assertion. 65. The Li2+ ion is moving in the third stationary state, and its
(b) Assertion is correct, reason is correct; reason is not linear momentum is 7.3 × 10–34 kgms–1. Angular momentum
a correct explanation for assertion is.
(c) Assertion is correct, reason is incorrect (a) 1.158 × 10–45 kg m2s–1 (b) 11.58 × 10–48 kg m2s–1
(d) Assertion is incorrect, reason is correct. (c) 11.58 × 10–47 kg m2s–1 (d) 12 × 10–45 kg m2s–1
58. The Bohr orbit radius for the hydrogen atom (n = 1) is 66. According to Bohr’s theory the energy required for an
approximately 0.530 Å. The radius for the first excited state electron in the Li2+ ion to be emitted from n = 2 state is
(n = 2) orbit is (in Å) (given that the ground state ionization energy of hydrogen
(a) 0.13 (b) 1.06 atom is 13.6 eV)
(c) 4.77 (d) 2.12 (a) 61.2 eV (b) 13.6 eV
59. Among species H, Li , He , Be3+ and Al3+, Bohr’s model
2+ + (c) 30.6 eV (d) 10.2 eV
was able to explain the spectra of 67. For Balmer series in the spectrum of atomic hydrogen, the
(a) All of these æ 1 1 ö
(b) None of these wave number of each line is given by v = R H çç 2 – 2 ÷÷
(c) all other species except Be3+ è n1 n2 ø
(d) all other species except Al3+ where RH is a constant and n1 and n2 are integers. Which
of the following statement(s) is (are) correct?
60. What does negative sign in the electronic energy for
hydrogen atom convey. (i) As wavelength decreases, the lines in the series
converge.
(a) Energy of electron when n = ¥
(ii) The integer n1 is equal to 2.
(b) The energy of electron in the atom is lower than the
energy of a free electron in motion (iii) The ionization energy of hydrogen can be calculated
from the wave number of these lines.
(c) The energy of electron in the atom is lower than the
(iv) The line of longest wavelength corresponds to n2 = 3.
energy of a free electron of rest
(a) (i), (ii) and (iii) (b) (ii), (iii) and (iv)
(d) The energy of electron decreases as it moves away
from nucleus (c) (i), (ii) and (iv) (d) (ii) and (iv)
STRUCTURE OF ATOM 21

Topic 5: Towards Quantum Mechanical Model of the Atom (a) Assertion is correct, reason is correct; reason is a
correct explanation for assertion.
68. If electron, hydrogen, helium and neon nuclei are all moving (b) Assertion is correct, reason is correct; reason is not
with the velocity of light, then the wavelength associated a correct explanation for assertion
with these particles are in the order
(c) Assertion is correct, reason is incorrect
(a) Electron > hydrogen > helium > neon
(b) Electron > helium > hydrogen > neon (d) Assertion is incorrect, reason is correct.
(c) Electron < hydrogen < helium < neon 77. Arrange the following elements in the order of ease of
(d) Neon < hydrogen < helium < electron detection of wave properties, in the de Broglie experiment.
69. The de Broglie wavelength of a tennis ball of mass 60 g H, Li, Be, B, K
moving with a velocity of 10 metres per second is (a) H < Be, B < Li < K. (b) H > Li > K > Be > B
approximately (c) H > Li > Be > B > K (d) H < Li < Be < B < K
(a) 10–31 metres (b) 10–16 metres 78. The velocity of particle A is 0.1 ms–1 and that of particle B
(c) 10–25 metres (d) 10–33 metres is 0.05 ms–1. If the mass of particle B is five times that of
Planck’s constant, h = 6.63 × 10–34 Js particle A, then the ratio of de-Broglie wavelengths
70. If the energy difference between the ground state of an associated with the particles A and B is
atom and its excited state is 4.4 × 10–4 J, the wavelength of (a) 2 : 5 (b) 3 : 4
photon required to produce the transition : (c) 6 : 4 (d) 5 : 2
(a) 2.26 × 10–12 m (b) 1.13 × 10–22 m 79. Two fast moving particles X and Y are associated with
(c) 4.52 × 10–16 m (d) 4.52 × 10–22 m de Broglie wavelengths 1 nm and 4 nm respectively. If
71. The mass of a photon with a wavelength equal to mass of X is nine times the mass of Y, the ratio of kinetic
1.54 × 10–8 cm is energies of X and Y would be
(a) 0.8268 × 10–34 kg (b) 1.2876 × 10–33 kg (a) 3 : 1 (b) 9 : 1
(c) 1.4285 × 10–32 kg (d) 1.8884 × 10–32 kg (c) 5 : 12 (d) 16 : 9
72. The de Broglie wavelength of a car of mass 1000 kg and 80. Uncertainty in position of an electron (mass = 9.1 × 10–28
velocity 36 km/hr is : g) moving with a velocity of 3 × 104 cm/s accurate upto
0.001% will be (use h/4p in uncertainty expression where
(a) 6.626 × l0–34 m (b) 6.626 × 10–38 m
h = 6.626 ×10–27 erg-second).
(c) 6.626 × 10–31 m (d) 6.626 × 10– 30 m
(a) 1.93 cm (b) 3.84 cm
73. Heisenberg's uncertainity principle is applicable to
(c) 5.76 cm (d) 7.68 cm
(a) atoms only (b) electron only
81. The measurement of the electron position is associated
(c) nucleus only (d) any moving object with an uncertainty in momentum, which is equal to
74. The position of both, an electron and a helium atom is 1×10–18 g cm s–1. The uncertainty in electron velocity is,
known within 1.0 nm. Further the momentum of the electron
(mass of an electron is 9 × 10– 28 g)
is known within 5.0 × 10–26 kg ms–1. The minimum
uncertainty in the measurement of the momentum of the (a) 1 × 109 cm s–1 (b) 1 × 106 cm s–1
5
(c) 1 × 10 cm s –1 (d) 1 × 1011 cm s–1
helium atom is
(a) 50 kg ms–1 (b) 80 kg ms–1 82. In an atom, an electron is moving with a speed of 600 m/s
–26
(c) 8.0 × 10 kg ms –1 (d) 5.0 × 10–26 kg ms–1 with an accuracy of 0.005%. Certainty with which the
position of the electron can be located is ( h = 6.6 × 10–34 kg
75. The Heisenberg uncertainity principle will be most
m2s–1, mass of electron, em = 9.1 × 10–31 kg)
significant for which of the following object ?
(a) 5.10 × 10 –3 m (b) 1.92 × 10 –3 m
(a) Object A of mass 9.11 × 10–30 kg
(c) 3.84 × 10 –3 m (d) 1.52 × 10 –4 m
(b) Object B of mass 9.11 × 10–28 g
(c) Object C of mass 9.11 × 10–24 mg Topic 6: Quantum Mechanical Model of Atom
(d) Object D of mass 9.11 × 10–28 kg
83. The orientation of an atomic orbital is governed by
76. Assertion : The position of an electron can be determined
exactly with the help of an electron microscope. (a) Spin quantum number
(b) Magnetic quantum number
Reason : The product of uncertainty in the measurement
of its momentum and the uncertainty in the measurement (c) Principal quantum number
of the position cannot be less than a finite limit. (d) Azimuthal quantum number
EBD_8350
22 CHEMISTRY

84. For which one of the following sets of four quantum 94. In a given atom, no two electrons can have the same values
numbers, an electron will have the heighest energy? for all the four quantum numbers. This is called
n l m s (a) Hund’s Rule
(a) 3 2 1 1/2 (b) Aufbau principle
(b) 4 2 –1 1/2 (c) Uncertainty principle
(c) 4 1 0 –1/2 (d) Pauli’s exclusion principle
(d) 5 0 0 –1/2 95. The electronic configuration of Cu (atomic number 29) is
85. Which of the following sets of quantum numbers is correct (a) 1s 2 , 2 s 2 2 p 6 ,3s 2 3 p 6 , 4 s 2 3d 9
for an electron in 4f orbital ? (b) 1s 2 ,2 s 2 2 p 6 ,3s 2 3 p 6 3d 10 , 4 s1
(a) n = 4, l = 3, m = + 1, s = + ½
(c) 1s2 ,2s2 2 p6 ,3 p 2 3 p 6 ,4s2 4 p6 ,5s2 5 p1
(b) n = 4, l = 4, m = – 4, s = – ½
(d) 1s 2 , 2 s 2 2 p 6 ,3 p 2 3 p 6 , 4 s 2 4 p 6 3d 3
(c) n = 4, l = 3, m = + 4, s = + ½
(d) n = 3, l = 2, m = – 2, s = + ½ 96. Match the columns.
86. What is the correct orbital designation of an electron with Column-I Column-II
the quantum number, n = 4, l = 3, m = – 2, s = 1/2 ? (Quantum number) (Information provided)
(a) 3s (b) 4 f (A) Principal (p) orientation of the orbital
(c) 5p (d) 6s quantum number
87. Which of the following represents correct set of the four (B) Azimuthal (q) energy and size of
quantum numbers for an electron in a 4d subshell ? orbital quantum number
(a) 4, 2, 1, 0 (C) Magnetic (r) spin of electron
(b) 4, 2, 1, – 1/2 quantum number
(c) 4, 3, 2, + 1/2 (D) Spin quantum (s) shape of the orbital
(d) 4, 3, – 2, – 1/2 number
88. The total number of electrons that can be accommodated (a) A – (q), B – (s), C – (p), D – (r)
in all orbitals having principal quantum number 2 and (b) A – (s), B – (q), C – (p), D – (r)
azimuthal quantum number 1 is
(c) A – (q), B – (p), C – (s), D – (r)
(a) 2 (b) 4
(c) 6 (d) 8 (d) A – (q), B – (s), C – (r), D – (p)
89. Which of the following is not permissible arrangement of 97. Which combinations of quantum numbers, n, l, m and s for
electrons in an atom? the electron in an atom does not provide a permissible
(a) n = 5, l = 3, m = 0, s = + 1/2 solution of the wave equation ?
(b) n = 3, l = 2, m = – 3, s = – 1/2 1 1
(c) n = 3, l = 2, m = – 2, s = – 1/2 (a) 3, 2,1, (b) 3,1,1, -
2 2
(d) n = 4, l = 0, m = 0, s = – 1/2
1 1
90. Which set of quantum numbers are not possible? (c) 3, 3,1, - (d) 3, 2, - 2,
n l m s 2 2
(a) 3 2 0 +1/2 98. The five d-orbitals are designated as d xy , d yz , d xz , d 2 2
x -y
(b) 2 2 1 +1/2 and d 2. Choose the correct statement
z
(c) 1 0 0 –1/2 (a) The shapes of the first three orbitals are similar but
(d) 3 2 –2 +1/2 that of the fourth and fifth orbitals are different
91. What will be the sum of all possible values of l and m for
(b) The shapes of all five d-orbitals are similar
n=5?
(c) The shapes of the first four orbitals are similar but
(a) 12 (b) 13
that of the fifth orbital is different
(c) 4 (d) 9
(d) The shapes of all five d-orbitals are different
92. The orbitals are called degenerate when
(a) they have the same wave functions 99. Which of the following graph correspond to one node
(b) they have the same wave functions but different
energies
(c) they have different wave functions but same energy Y Y
(d) they have the same energy (a) (b)
93. The number of orbitals present in the fifth shell will be
(a) 25 (b) 10 a0
(c) 50 (d) 20 a0
STRUCTURE OF ATOM 23

109. Match the columns.


Column-I Column-II
Y
Y
(A) Y 2 (p) Energy can be emitted
(c) (d) or absorbed in packets
a0 (B) de Broglie (q) Significant only for
a0 motion of microscopic
100. If there are five radial nodes, then what can be the correct objects.
representation of the orbital for n = 8. (C) Heisenberg’s (r) The probability of
(a) 8 d (b) 8 s finding an electron at a
(c) 8 p (d) 8 f point within an atom
101. What can be the representation of the orbital having 3 (D) Planck’s (s) Every object in motion
angular nodes and n = 5. has a wave character.
(a) 5 d (b) 5 f (a) A – (q), B – (s), C – (r), D – (p)
(c) 5 p (d) 5 s (b) A – (r), B – (p), C – (q), D – (s)
102. The number of d-electrons retained in Fe2+ (c) A – (r), B – (s), C – (q), D – (p)
(At. no. of Fe = 26) ion is (d) A – (s), B – (p), C – (r), D – (q)
(a) 4 (b) 5 110. Match the columns.
(c) 6 (d) 3 Column-I Column-II
103. The electronic configuration of an element is 1s2, 2s2 2p6, z
3s2 3p6 3d 5, 4s1. This represents its
(a) excited state (b) ground state
(c) cationic form (d) anionic form
104. The orbital diagram in which the Aufbau principle is (A) dx2 – y2 (p)
violated is : x
y
2s 2p 2s 2p
(a) ­¯ ­¯ ­ (b) ­ ­¯ ­ ­ z
(c) ­¯ ­ ­ ­ (d) ­¯ ­¯ ­ ­
105. Maximum number of electrons in a subshell of an atom is
determined by the following: (B) dxy (q)
(a) 2 l + 1 (b) 4 l – 2
y x
(c) 2 n 2 (d) 4 l + 2
106. The correct order of increasing energy of atomic orbitals is
(a) 5 p < 4 f < 6 s < 5 d (b) 5 p < 6 s < 4 f < 5 d
(c) 5 p < 5 d < 4 f < 6 s (d) None of these
(C) dyz (r)
107. For which element, the valence electron will be present in
the highest energy orbital.
(a) 3Li (b) 16S
z
(c) 20Ca (d) 21Sc
108. Which of the following electronic configuration of d-orbital
will have highest affinity for gaining an electron?
(D) dz2 (s)
(a)
x
y
(b)
(a) A – (p), B – (s), C – (r), D – (q)
(c) (b) A – (s), B – (p), C – (r), D – (q)
(c) A – (s), B – (p), C – (q), D – (r)
(d) (d) A – (s), B – (r), C – (p), D – (q)
EBD_8350
24 CHEMISTRY

111. The following quantum numbers are possible for how many (a) 25 and 1 (b) 8 and 1
orbital(s) n = 3, l = 2, m = +2 ? (c) 2 and 4 (d) 4 and 1
(a) 1 (b) 3 (c) 2 (d) 4 114. The orbital angular momentum for an electron revolving in
112. The electronic configuration of gadolinium (Atomic number an orbit is given by l (l + 1) . h . This momentum for an
64) is
s-electron will be given by 2 p
(a) [Xe] 4f 8 5d 0 6s2 (b) [Xe] 4f 3 5d 5 6s2 h
(a) zero (b)
(c) [Xe] 4f 6 5d 2 6s2 (d) [Xe] 4f 7 5d 1 6s2 2p
113. Given h 1 h
(c) 2. (d) + .
(i) n = 5, ml = + 1 2p 2 2p
115. What are the component values (in terms of h/2p) of the
(ii) n = 2, l =1, ml = –1, ms = –l/2 orbital angular momentum along the Z-direction for a 2p
The maximum number of electron(s) in an atom that can electron?
have the quantum numbers as given in (a) and (b) are 1 1 3 1 1 3
(a) + , - (b) + , + , - , -
respectively: 2 2 2 2 2 2
(c) +2, +1, 0, –1, –2 (d) +1, 0, –1

1. The ground state energy of hydrogen atom is –13.6 eV.


Calculate the energy of second excited state of He+ ion
in eV.
2. What is the work function (in eV) of the metal if the light of
wavelength 4000Å generates photoelectrons of velocity
6×105 ms-1 from it ?
(Mass of electron=9×10–31 kg; 6. An element undergoes a reaction as shown:
X + 2e– ¾® X 2–, energy released = 30.87 eV/atom. If the
Velocity of light = 3×108 ms–1;
energy released, is used to dissociate 4 g of H2 molecules,
Planck’s constant=6.626×10–34 Js; equally into H+ and H*, where H* is excited state of H
Charge of electron =1.6×10–19 JeV–1) atoms where the electron travels in orbit whose
3. Monochromatic radiation of specific wavelength is circumference equal to four times its de Broglie’s
incident on H-atoms in ground state. H-atoms absorb wavelength. Determine the least moles of X that would be
energy and emit subsequently radiations of six different required.
wavelength. Find wavelength of incident radiations in nm. Given: I.E. of H = 13.6 eV/atom, bond energy of H2 = 4.526
4. In a measurement of quantum efficiency of photosynthesis eV/molecule.
in green plants, it was found that 10 quanta of red light of 7. Two fast moving particles X and Y are associated with de
wavelength 6850 Å were needed to release one molecule Broglie wavelengths 1 nm and 4 nm respectively. If mass
of O2. The average energy storage in this process for 1 of X is nine times the mass of Y, then calculate ratio of
mole O2 evolved is 112 kcal. kinetic energies of X and Y.
What is the energy conversion efficiency in this 8. An electron has a speed of 30,000 cm sec–1 accurate upto
experiment? 0.001%. What is the uncertainty (in cm) in locating it’s
[Given: 1 cal = 4.18 J; NA = 6 ´ 1023; h = 6.63 ´ 10–34 Js] position?
9. What is the sum of radial and angular nodes in the
5. The given diagram indicates the energy levels of certain
following orbitals of H-atom?
atoms. When the system moves from 2E level to E a photon
of wave length l is emitted. Calculate the wave-length of (I) y 2px (II) y 2 (III) y 3dx (IV) y 3d 2 2
x -y
4E 10. Determine the Bohr orbit of Li 2+ ion in which electron is
photon produced during its transition from level to E moving at speed equal to the speed of electron in the first
3
in terms of l. Bohr orbit of H-atom.
STRUCTURE OF ATOM 25

Exercise 3 : NCERT Exemplar & Past Year MCQs


NCERT Exemplar MCQs (d) sum of the number of protons and neutrons is same
but the number of protons is different
1. Which of the following conclusions could not be derived
from Rutherford's a-particle scattering experiment? 7. The number of radial nodes for 3p orbital is ......... .
(a) 3 (b) 4 (c) 2 (d) 1
(a) Most of the space in the atom is empty.
(b) The radius of the atom is about 10–10 m while that of 8. Number of angular nodes for 4d orbital is ........... .
nucleus is 10–15 m. (a) 4 (b) 3 (c) 2 (d) 1
(c) Electrons move in a circular path of fixed energy 9. Which of the following is responsible to rule out the
called orbits. existence of definite paths or trajectories of electrons?
(d) Electrons and the nucleus are held together by (a) Pauli's exclusion principle
electrostatic forces of attraction. (b) Heisenberg's uncertainty principle
2. Which of the following options does not represent ground (c) Hund's rule of maximum multiplicity
state electronic configuration of an atom? (d) Aufbau principle
(a) 1s22s22p63s23p63d 84s2 10. Total number of orbitals associated with third shell will
(b) 1s22s22p63s23p63d 94s2 be ............... .
(c) 1s22s22p63s23p63d104s1 (a) 2 (b) 4 (c) 9 (d) 3
(d) 1s22s22p63s23p63d 54s1 11. Orbital angular momentum depends on ............ .
3. Which of the following statement is not correct about the (a) l (b) n and l
characteristics of cathode rays?
(c) n and m (d) m and s
(a) They start from the cathode and move towards the
12. Chlorine exists in two isotopic forms Cl-37 and Cl-35, but
anode.
its atomic mass is 35.5. This indicates the ratio of Cl-37
(b) They travel in straight line in the absence of an
and Cl-35 is approximately
external electrical or magnetic field.
(a) 1 : 2 (b) 1 : 1 (c) 1 : 3 (d) 3 : 1
(c) Characteristics of cathode rays do not depend upon
the material of electrodes in cathode ray tube. 13. If travelling at same speeds, which of the following matter
(d) Characteristics of cathode rays depend upon the waves have the shortest wavelength?
nature of gas present in the cathode ray tube. (a) Electron (b) Alpha particle (He2+)
4. Which of the following statements about the electron is (c) Neutron (d) Proton
incorrect? 14. The probability density plots of 1s and 2s orbitals are
(a) It is a negatively charged particle. given in figure.
(b) The mass of electron is equal to the mass of neutron.
(c) It is a basic constituent of all the atoms.
(d) It is a constituent of cathode rays.
1s 2s
5. Which of the following properties of atom could be
explained correctly by Thomson model of atom? The density of dots in a region represents the
(a) Overall neutrality of atom probability density of finding electrons in the region.
(b) Spectra of hydrogen atom On the basis of above diagram which of the following
(c) Position of electrons, protons and neutrons in atom statements is incorrect?
(d) Stability of atom (a) 1s and 2s orbitals are spherical in shape.
6. Two atoms are said to be isobars if (b) The probability of finding the electron is maximum
(a) they have same atomic number but different mass near the nucleus.
number (c) The probability of finding the electron at a given
(b) they have same number of electrons but different distance is equal in all directions.
number of neutrons (d) The probability density of electrons for 2s orbital
(c) they have same number of neutrons but different decreases uniformly as distance from the nucleus
number of electrons increases.
EBD_8350
26 CHEMISTRY

15. The pair of ions having same electronic configuration [JEE M 2016, S]
is .............. . (a) meV (b) 2meV
(a) Cr3+, Fe3+ (b) Fe3+, Mn2+
(c) meV (d) 2meV
(c) Fe3+, Co3+ (d) Sc3+, Cr 3+ 23. Which one is the wrong statement ? [NEET 2017, C]
16. For the electrons of oxygen atom, which of the following
(a) The uncertainty principle is DE ´ Dt ³ h / 4 p
statements is correct?
(b) Half filled and fully filled orbitals have greater stability
(a) Zeff for an electron in a 2s orbital is the same as Zeff for
due to greater exchange energy, greater symmetry and
an electron in a 2p orbital.
more balanced arrangement.
(b) An electron in the 2s orbital has the same energy as
an electron in the 2p orbital. (c) The energy of 2s orbital is less than the energy of
2p orbital in case of Hydrogen like atoms
(c) Zeff for an electron in 1s orbital is the same as Zeff for
an electron in a 2s orbital. h
(d) de-Broglies's wavelength is given by l = , where
(d) The two electrons present in the 2s orbital have spin mn
quantum numbers ms but of opposite sign. m = mass of the particle, n = group velocity of the
particle
Past Year MCQs
24. The radius of the second Bohr orbit for hydrogen atom
17. What is the maximum number of orbitals that can be is :
identified with the following quantum numbers?
n = 3, l = 1, ml = 0 [AIPMT 2014, A] (Planck's const. h = 6.6262 × 10–34 Js ; mass of
(a) 1 (b) 2 electron = 9.1091 × 10–31 kg ; charge of electron
(c) 3 (d) 4 e = 1.60210 × 10–19 C ; permittivity of vaccum
18. Calculate the energy in joule corresponding to light of Î0 = 8.854185 × 10–12 kg–1 m–3 A2) [JEE M 2017, A]
wavelength 45 nm :
(a) 1.65Å (b) 4.76Å
(Planck’s constant h = 6.63 × 10–34 Js; speed of light
(c) 0.529Å (d) 2.12Å
c = 3 × 108 ms–1) [AIPMT 2014, A]
25. The angular speed of the electron in n th orbit of Bohr
(a) 6.67 × 1015 (b) 6.67 × 1011
hydrogen atom is [NEET 2018, C]
(c) 4.42 × 10–15 (d) 4.42 × 10–18 (a) Directly proportional to n
19. The correct set of four quantum numbers for the valence
(b) Inversely proportional of n
electrons of rubidium atom (Z = 37) is: [JEE M 2014, S]
2
(c) Inversely proportional to n
1 1
(a) 5, 0, 0, + (b) 5,1, 0, + (d) Inversely proportional to n 3
2 2
26. 4d, 5p, 5f and 6p orbitals are arranged in the order of
1 1 decreasing energy. The correct option is:[NEET 2019, C]
(c) 5,1,1, + (d) 5, 0,1, +
2 2 (a) 5f > 6p > 5p > 4d
20. Which of the following is the energy of a possible excited
state of hydrogen ? [JEE M 2015, A] (b) 6p > 5f > 5p > 4d
(a) –3.4 eV (b) +6.8 eV (c) 6p > 5f > 4d > 5p
(c) +13.6 eV (d) –6.8 eV (d) 5f > 6p > 4d > 5p
21. Two electrons occupying the same orbital are distinguished 27. Which of the following series of transitions in the.spectrum
by [NEET 2016, C] of hydrogen atom falls in visible region ?[NEET 2019, C]
(a) Principal quantum number (a) Lyman series
(b) Balmer series
(b) Magnetic quantum number
(c) Paschen series
(c) Azimuthal quantum number (d) Brackett series
(d) Spin quantum number 28. For any given series of spectral lines of atomic hydrogen,
22. A stream of electrons from a heated filaments was passed let Dv = vmax ¯ vmin be the difference in maximum and
between two charged plates kept at a potential difference V minimum frequencies in cm–1. The ratio D v Lyman / D v Balmer
volt. If e and m are charge and mass of an electron, is : [JEE M 2019, A]
respectively, then the value of h/l (where l is wavelength (a) 4 : 1 (b) 9 : 4
associated with electron wave) is given by: (c) 5 : 4 (d) 27 : 5
STRUCTURE OF ATOM 27

29. For emission line of atomic hydrogen from ni = 8 to nf = n, (c) Linear with slope RH
æ 1ö (d) Linear with slope – RH
the plot of wave number ( v ) against ç ÷ will be (The 30. The number of orbitals associated with quantum numbers
è n2 ø
1
Rydberg constant, RH is in wave number unit) n = 5, ms = + is: [JEE M 2020, C]
2
[JEE M 2019, C] (a) 11 (b) 25
(a) Linear with intercept – RH (c) 50 (d) 15
(b) Non linear

Exercise 4 : Problem Solving Skill Enhancer MCQs

1. Assume that 2 × 10–17 J of light energy is needed by the


2h 2hc
interior of the human eye to see an object. How many (a)
m
( l0 - l ) (b)
m
( l0 - l )
photons of yellow light with l = 595.2 nm are needed to
generate this minimum energy?
(a) 6 (b) 30 2hc æ l 0 - l ö 2h æ 1 1 ö
-
(c) 45 (d) 60
(c) m çè ll 0 ÷ø (d) m çè l 0 l ÷ø
2. In a multi-electron atom, which of the following orbitals
7. The wave number of electromagnetic radiation emitted
described by the three quantum members will have the during the transition in between two energy levels of Li2+
same energy in the absence of magnetic and electric fields? ion whose principal quantum number sum is 4 and
(A) n = 1, l = 0, m = 0 (B) n = 2, l = 0, m = 0 difference is 2, is
(C) n = 2, l = 1, m = 1 (D) n = 3, l = 2, m = 1 (a) 3.5 RH (b) 4 RH
(E) n = 3, l = 2, m = 0
(a) (D) and (E) (b) (C) and (D) (c) 8 RH (d) 8 RH
9
(c) (B) and (C) (d) (A) and (B)
3. If the shortest wavelength of the spectral line of H-atom in 8. If two particles are associated with same kinetic energy,
the Lyman series is x, then the longest wavelength of the then the de Broglie’s wavelength (l) of these particles is
line in Balmer series of Li2+ is (a) directly proportional to the velocity
x (b) inversely proportional to the velocity
(a) 9x (b)
9 (c) independent of mass and velocity
(d) can not be predicted
5x 4x
(c) (d) 9. Which of the following statement concerning probability
4 5
density (y2) and radial distribution function (4pr2y2) for
4. A hydrogen atom in the ground state is excited by
an s-orbital of H-like species is correct?
monochromatic radiation of wavelength l Å. The resulting
(a) y2 is minimum at nucleus but 4pr2y2 is maximum at
spectrum consists of maximum 15 different lines. What is
the wavelength l? (RH = 109737 cm–1) nucleus.
(a) 937.3 Å (b) 1025 Å (b) y2 is maximum at nucleus but 4pr2y2 is minimum at
(c) 1236 Å (d) None of these nucleus.
5. If an electron undergoes transition from n = 2 to n = 1 in (c) Both y2 and 4pr2y2 are maximum at nucleus.
Li2+ ion, the energy of photon radiated will be best given (d) Both y2 and 4pr2y2 are minimum at nucleus.
by 10. If radiation corresponding to second line of "Balmer
(a) hv (b) hv1 + hv2 series" of Li2+ ion, knocked out electron from first excited
(c) hv1 + hv2 + hv3 (d) all of these state of H-atom, then kinetic energy of ejected electron
6. If l0 and l be the threshold wavelength and the would be:
wavelength of incident light, the speed of photoelectrons (a) 2.55 eV (b) 4.25 eV
ejected from the metal surface is :
(c) 11.25 eV (d) 19.55 eV
EBD_8350
28 CHEMISTRY

11. Li3+ and a proton are accelerated by the same potential, excited state can make a transition to 2nd excited state by
then de-Broglie wavelengths lLi and lp have the ratio emitting two photons of energies 4.25 eV and 5.95 eV,
(assume m Li = 9 mp ) respectively. Determine Z and n.
(a) n = 6, Z = 3 (b) n = 3, Z = 6
(a) 1 : 3 3 (b) 1 : 1
(c) n = 2, Z = 4 (d) n = 9, Z = 9
(c) 1 : 2 (d) 1 : 4 14. 1.5 L He gas is taken at 500K and 560 mm pressure in which
12. If the de-Broglie wavelength of a particle of mass m is 100 90% He are converted into He+ ions. Spectral line analysis
times its velocity, then its value in terms of its mass (m) shows that 85% ions exist in the 3rd level, 10% in the 2nd
and Planck’s constant (h) is level and rest in the ground level. Calculate the total energy
1 m h evolved when all ions return back to ground state.
(a) (b) 10 (a) 101 kJ (b) 957 kJ
10 h m
(c) 212 kJ (d) 149 kJ
1 h m 15. The radii of maximum probability for 3s, 3p and 3d electrons
(c) (d) 10 are in the order
10 m h
(a) (rmax)3d > (rmax)3p > (rmax)3s
13. A hydrogen-like atom with atomic number Z is in the higher
excited state of quantum number n. This excited state atom (b) (rmax)3d > (rmax)3s > (rmax)3p
can make a transition to the 1st excited state by (c) (rmax)3s > (rmax)3p > (rmax)3d
successively emitting two photons of energies 10 eV and (d) none of these
17eV, respectively. Alternatively, the atom from the same

ANSWER KEY
Exercise 1 : NCERT Based Topic-wise MCQs
1 (b) 13 (d) 25 (b) 37 (c) 49 (a) 61 (c) 73 (d) 85 (a) 97 (c) 109 (c)
2 (b) 14 (d) 26 (d) 38 (c) 50 (c) 62 (d) 74 (d) 86 (b) 98 (c) 110 (b)
3 (c) 15 (c) 27 (a) 39 (a) 51 (c) 63 (b) 75 (b) 87 (b) 99 (b) 111 (c)
4 (c) 16 (c) 28 (d) 40 (c) 52 (a) 64 (d) 76 (d) 88 (c) 100 (a) 112 (d)
5 (c) 17 (d) 29 (a) 41 (b) 53 (c) 65 (b) 77 (c) 89 (b) 101 (b) 113 (b)
6 (b) 18 (a) 30 (d) 42 (b) 54 (d) 66 (c) 78 (d) 90 (b) 102 (c) 114 (a)
7 (b) 19 (d) 31 (c) 43 (d) 55 (d) 67 (c) 79 (d) 91 (b) 103 (b) 115 (d)
8 (d) 20 (c) 32 (a) 44 (a) 56 (b) 68 (a) 80 (a) 92 (d) 104 (b)
9 (c) 21 (b) 33 (c) 45 (a) 57 (a) 69 (d) 81 (a) 93 (a) 105 (d)
10 (a) 22 (d) 34 (b) 46 (b) 58 (d) 70 (d) 82 (b) 94 (d) 106 (b)
11 (c) 23 (d) 35 (d) 47 (d) 59 (d) 71 (c) 83 (b) 95 (b) 107 (d)
12 (b) 24 (a) 36 (b) 48 (a) 60 (c) 72 (b) 84 (b) 96 (a) 108 (d)
Exercise 2 : Numeric/Integer Answer Questions
1 (–6.04) 2 (2.1) 3 (97.25) 4 (26.9) 5 (3) 6 (2) 7 (1.77) 8 (2) 9 (6) 10 (3)
Exercise 3 : NCERT Exemplar & Past Year MCQs
1 (c) 4 (b) 7 (d) 10 (c) 13 (b) 16 (d) 19 (a) 22 (b) 25 (d) 28 (b)
2 (b) 5 (a) 8 (c) 11 (a) 14 (d) 17 (a) 20 (a) 23 (c) 26 (a) 29 (d)
3 (d) 6 (d) 9 (b) 12 (c) 15 (b) 18 (d) 21 (d) 24 (d) 27 (b) 30 (b)
Exercise 4 : Problem Solving Skill Enhancer MCQs
1 (d) 3 (d) 5 (a) 7 (c) 9 (b) 11 (a) 13 (a) 15 (c)
2 (a) 4 (a) 6 (c) 8 (a) 10 (d) 12 (b) 14 (a)
Classification of Elements
3 and Periodicity in Properties

Trend Buster NEET & JEE Main

Number of Questions from 2020-15 9 6 Minimum one question every year has
Weightage 2.8% 3.4% been asked in NEET & JEE Main.

The most Important Concepts that Cover Maximum number of Questions asked in past 6 years.

Ionisation Energy trend 2 1


Electron gain enthalpy 1 1
Atomic/ionic radii trend 1 1
Periodic trend 2 1

Less Important Concepts that Cover 1 or 2 Questions asked in past 6 years.

Diagonal relationship — 1
Isoelectronic species — 1
Placement in periodic table/Electronic configuration 3 1

NEET JEE

2020 Periodic trends in properties Electron gain enthalpy trend / 3 Easy / 1 Easy
of elements / Present form of Oxide trend / Electronic Average
the periodic table / Electronic configuration / IUPAC
configuration Nomenclature of unknown
elements
2019 Periodic trends in properties Periodic trend / Ionisation 2 Average 1 Easy
trend
2018 — — — — — —
2017 Periodic trends in properties / Diagonal relationship / 1 Average 2 Average
Electronic configuration-s, Isoelectronic species /
p, d, f-blocks Electronic Placement in periodic table
configuration-s, p, d, f-blocks
2016 Periodic trends in properties Ionisation energy trend / 1 Average 1 Average
of elements I.E. / E.A. / Ionic size trend
2015 Periodic trends in properties Ionic radii trend / Electron 2 Difficult 1 Easy
of elements gain enthalpy trend / Atomic
radii trend
EBD_8350
30 CHEMISTRY
CLASSIFICATION OF ELEMENTS AND PERIODICITY IN PROPERTIES 31
EBD_8350
32 CHEMISTRY

Problem Solving Tips/ Tricks/ Points to Remember

4 Period no. is equal to the valence shell present in the 4 Most stable carbonate : Cs2CO3
configuration. Also for s and p-block elements. 4 Bridge metals : Na, Mg
Period no. = valence shell, for d-block = shell + 1,
4 Heaviest solid metal : Os (highest density 22.6 g/cm3 )
for f-block = shell + 2 and so on.
Similarly, K is the alkali metal directly below Na in group
4 Lowest electronegativity : Cs
IA, so we expect I1 for K to be less than that of Na : K <
4 Highest electronegativity : F Na
4 Highest ionisation potential : He From these observations we conclude that the ionization
4 Lowest ionisation potential : Cs energies follow the order : K < Na < P < Ar < Ne.

4 Highest electron affinity : Chlorine (Cl) 4 The basic character of hydroxides increases in the order:

4 Least electropositive element : Fluorine (F) CsOH > RbOH > KOH > NaOH > LiOH
4 Most reactive solid element : Li 4 The acidic character of oxides and hydroxides decreases
in the order:
4 Most reactive liquid element : Cs
HClO4 > HBrO4 > HIO4
4 Most stable element : Te
4 The non-metallic character, oxidising character and acidic
4 Largest atomic size : Cs
nature of oxides of the elements increases from left to
4 Most electropositive element : Cs (in stable element), Fr right in a period and decrease from top to bottom in a
(In all element) group.
4 Volatile d-block elements : Zn, Cd, Hg Na2O MgO Al2O3 SiO2 P4O10 SO2 Cl2O7
Strongly Basic Amphoteric Weakly Strongly Very strongly
basic acidic acidic acidic
CLASSIFICATION OF ELEMENTS AND PERIODICITY IN PROPERTIES 33

10. In the Mendeleev’s periodic table, which of the following


Topic 1: Genesis of Periodic Classification element instead of having lower atomic weight was placed
1. Periodic classification of elements is used to examine the after the element of higher atomic weight thereby ignoring
(a) periodic trends in physical properties of elements the order of increasing atomic weights.
(b) periodic trends in chemical properties of elements (a) Iodine (b) Antimony
(c) Both (a) and (b) (c) Bromine (d) Molybdenum
(d) None of the above 11. Which of the following is correct about Eka-Aluminium
2. Cl, Br, I, if this is Dobereiner’s triad and the atomic masses and Eka-Silicon ?
of Cl and I are 35.5 and 127 respectively the atomic mass of (a) Oxides of Eka-Aluminium is Al2O3 and Eka-Silicon is
Br is Si2O3
(b) Oxides of Eka-Aluminium is Ga2O3 and Eka-Silicon is
(a) 162.5 (b) 91.5
GeO2
(c) 81.25 (d) 45.625 (c) Melting point of Eka-Aluminium is lower than the
3. If the two members of a Dobereiner triad are phosphorus melting point of Eka-Silicon
and antimony, the third member of this triad is (d) Both (a) and (c)
(a) arsenic (b) sulphur
(c) iodine (d) calcium Topic 2: Modern Periodic Law and the Present Form of
4. Lothar Meyer plotted the physical properties such as the Periodic Table
atomic volume, melting point and ...X... against atomic 12. Which of the scientists given below discovered that
weight. Here, X refers to periodic table should be based on the atomic number ?
(a) mass (b) boiling point (a) Mendeleev (b) Newlands
(c) surface tension (d) None of these (c) Moseley (d) Lothar Meyer
5. The most significant contribution towards the development 13. How many elements are there in 6th period of periodic
of periodic table was made by table?
(a) Mendeleev (b) Avogadro (a) 18 (b) 8 (c) 30 (d) 32
(c) Dalton (d) Cavendish 14. The long form of periodic table consists of
6. Who developed long form of the periodic table? (a) seven periods and eight groups
(a) Lothar Meyer (b) Neils Bohr (b) seven periods and eighteen groups
(c) Mendeleev (d) Moseley (c) eight periods and eighteen groups
7. Which fact is not valid for Dobereiner's triads? (d) eighteen periods and eight groups
(a) The atomic weight of middle element is roughly 15. Which of the following statement(s) about the modern
average of the other two elements periodic table is/are incorrect ?
(b) The properties of middle element is roughly average (i) The elements in the modern periodic table are arranged
of the other two elements on the basis of their decreasing atomic number
(c) The elements of triads belong to the same group of (ii) The elements in the modern periodic table are arranged
modern periodic table on the basis of their increasing atomic masses
(d) The elements of triads have same valency electrons. (iii) Isotopes are placed in adjoining group(s) in the
8. Select the correct chronological order for the discoveries periodic table
of the following scientists. (iv) The elements in the modern periodic table are arranged
Dobereiner, Newlands, Chancourtois, Mendeleev on the basis of their increasing atomic number
(a) Chancourtois, Dobereiner, Newlands, Mendeleev (a) (i) only (b) (i), (ii) and (iii)
(b) Dobereiner, Chancourtois, Newlands, Mendeleev (c) (i), (ii) and (iv) (d) (iv) only
(c) Dobereiner, Newlands, Chancourtois, Mendeleev 16. Modern periodic table is based on the atomic number of
(d) Chancourtois, Newlands, Dobereiner, Mendeleev the elements. The experiment which proved the significance
of the atomic number was
9. The molecular formula of chloride of Eka-Aluminium and
Eka-Silicon respectively are (a) Millikan’s oil drop experiment
(a) GaCl3 and SiO4 (b) GaCl3 and AlCl3 (b) Mosley’s work on X-ray spectra
(c) AlCl3 and SiCl4 (d) GaCl3 and GeCl4 (c) Bragg’s work on X-ray diffraction
(d) Discovery of X-rays by Rontgen
EBD_8350
34 CHEMISTRY

17. The symbol and IUPAC name for the element with atomic (a) First group (b) Third group
number 120, respectively are (c) Fifth group (d) Seventh group.
(a) Ubn and unbinilium 26. The electronic configuration of four elements are given
(b) Ubn and unbiunium below. Which elements does not belong to the same family
(c) Ubn and unnibium as others?
(d) Ubn and unnilium (a) [Xe] 4f 145d101s2
18. What is the IUPAC name of the element with atomic (b) [Kr] 4d105s2
number 114 ? (c) [Ne] 3s23p5
(a) Unununnium (b) Unnilquadium (d) [Ar] 3d10 4s2
(c) Ununquadium (d) Unnilennium. 27. The elements with atomic numbers 9, 17, 35, 53 and 85
19. Element with which of the following atomic number was belong to
named by American Society as Rutherfordium, while by
(a) alkali metals
Soviet Society it was named as Kurchatovium?
(b) alkaline earth metals
(a) 108 (b) 104
(c) halogens
(c) 114 (d) 110
(d) noble gases
20. Match Column-I (IUPAC nomenclature of element) with
Column-II (IUPAC official name). 28. Which of the following pairs has both members from the
Column-I Column-II same period of the periodic table.
(A) Unnilhexium (p) Lawrencium (a) Na – Ca (b) Na – Cl
(B) Unniltrium (q) Dubnium (c) Ca – Cl (d) Cl – Br
(C) Unnilunium (r) Seaborgium 29. The elements which are characterized by the outer
(D) Unnilpentium (s) Mendelevium electronic configuration ns1 to ns2 np6 are collectively called
(a) A – (s), B – (p), C – (r), D – (q) (a) Transition elements
(b) A – (r), B – (p), C – (s), D – (q) (b) Representative elements
(c) A – (r), B – (s), C – (p), D – (q) (c) Lanthanides
(d) A – (q), B – (r), C – (p), D – (s) (d) Inner transition elements
30. f-Block elements are called inner transition elements
Topic 3: Electronic Configurations of Elements, because
the Periodic Table and Types of Elements: (a) they have properties similar to those of transition
s, p, d, f –Blocks elements
21. Elements of which group form anions most readily? (b) they exist in between transition elements
(a) Oxygen family (b) Nitrogen family (c) the last electron enters into the f-orbital of the
(c) Halogens (d) Alkali metals penultimate shell
22. Consider the following statements: (d) the last electron enters into any orbital of penultimate
(i) The discovery of inert gases later on did not disturb shell
Mendeleev's arrangement. 31. An element, which belongs to third period and group 16 in
(ii) In the present periodic table, periodicity in the the periodic table has electronic configuration.
properties of elements is related to the periodicity in (a) 1s2, 2s2 2p6, 3s2 3p3
their electronic configurations. (b) 1s2, 2s2 2p6, 3s2 3p4
Which of these statement(s) is/are correct ? (c) 1s2, 2s2 2p6, 3s2 3p5
(a) (i) only (b) (ii) only (d) 1s2, 2s2 2p4
(c) Both (i) and (ii) (d) Neither (i) nor (ii) 32. Which of the following is non-metallic ?
23. Element with electronic configuration 1s2 2s2 2p6 3s2 3p6 (a) B (b) Be
3d10 4s2 4p6 4d10 5s2 5p3 belongs to the following group of (c) Mg (d) Al
the periodic table 33. The only non-metal which is liquid at ordinary temperature
(a) 2nd (b) 5th is
(c) 3rd (d) 7th (a) Hg (b) Br2
24. In the modern periodic table one of the following does not (c) NH3 (d) None of these
have appropriate position – 34. In the long form of the periodic table all the non-metals are
(a) transition elements (b) inert gases placed in
(c) inner transition elements (d) halogens (a) s-block (b) p-block
25. If the atomic number of an element is 33, it will be placed in (c) f-block (d) d-block
the periodic table in the
CLASSIFICATION OF ELEMENTS AND PERIODICITY IN PROPERTIES 35

35. Arrange the following elements in the order of their (a) Partially filled d-orbitals and completely filled s-
increasing non-metallic character orbitals
Li, O, C, Be, F (b) Completely filled s-orbital and completely filled
(a) F < O < C < Be < Li p-orbitals
(b) Li < Be < C < O< F (c) Completely filled s-orbital and half-filled p-orbitals
(c) F < O < C < Be < Li (d) Half-filled d-robitals and completely filled s-orbitals
(d) F < O < Be < C < Li 43. Assertion : Second period consists of 8 elements.
36. Consider the following statements : Reason : Number of elements in each period is four times
(i) Metals will be found on the right side of the the number of atomic orbitals available in the energy level
periodic table. that is being filled.
(ii) The elements P, S and O belong to the same period. (a) Assertion is correct, reason is correct; reason is a
Which of these statement(s) is/are correct? correct explanation for assertion.
(a) (i) only (b) (ii) only (b) Assertion is correct, reason is correct; reason is not a
(c) Both (i) and (ii) (d) Neither (i) nor (ii) correct explanation for assertion.
37. Match the columns. (c) Assertion is correct, reason is incorrect.
Column-I Column-II (d) Assertion is incorrect, reason is correct.
(A) ‘s’ block elements (p) Cr 44. An element has electronic configuration 1s2 2s2 2p6 3s2 3p4.
(B) ‘p’ block elements (q) Na (a) Period = 3rd, block = p, group = 16
(C) ‘d’ block elements (r) Ce (b) Period = 5th, block = s, group = 1
(D) ‘f’ block elements (s) Si
(c) Period = 3rd, block = p, group = 10
(a) A – (s), B – (q), C – (p), D – (r)
(d) Period = 4th, block = d, group = 12
(b) A – (q), B – (s), C – (r), D – (p)
45. Element having atomic no. of 56 belongs to which of the
(c) A – (q), B – (p), C – (s), D – (r)
following block of periodic table?
(d) A – (q), B – (s), C – (p), D – (r)
(a) p- block (b) d-block
38. Which of the following period contains most of the man-
made radioactive elements? (c) f- block (d) s-block
(a) Seventh (b) Fifth 46. Which is a metalloid?
(c) Sixth (d) Both (a) and (c) (a) Pb (b) Sb
39. The electronic configuration of an element is (c) Bi (d) Zn
1s22s22p63s23p3. What is the atomic number of the element, 47. Which of the following statements are correct?
which is just below the above element in the periodic table? (i) The second period (n = 2) starts with lithium and
(a) 33 (b) 34 third electron enters the 2s orbital.
(c) 36 (d) 49 (ii) Successive filling of 3s and 3p orbitals gives rise to
40. Which of the following elements show the given the third period of 8 elements from sodium to argon.
properties? (iii) The fourth period (n = 4) starts at potassium and the
(i) All elements are metals. added electron fill up first 4s and 4p orbitals then 3d
(ii) Most of the elements form coloured ions, exhibit orbital is filled.
variable valence and paramagnetism. (iv) Fifth period begins with rubidium with the filling of
(iii) Oftently used as catalysts. 5s orbital and ends at xenon with the filling up of the
(a) Chalcogens 5p orbital.
(b) Transition elements (a) (i) and (ii) (b) (i), (ii) and (iii)
(c) Inner transition elements (c) (iii) and (iv) (d) (i), (ii) and (iv)
(d) Representative elements 48. Consider the following statements:
41. Element X forms a chloride with the formula XCl2, which is (i) The elements silicon, germanium and arsenic are
a solid with a high melting point. X would most likely be in called metalloids.
the same group of the periodic table as – (ii) Metalloids have properties quite different from those
(a) Na (b) Mg of metals and non-metals.
(c) Al (d) Si Which of these statement(s) is/are correct ?
42. An element X belongs to fourth period and fifteenth group
(a) (i) only (b) (ii) only
of the periodic table. Which one of the following is true
regarding the outer electronic configuration of X ? It has (c) Both (i) and (ii) (d) Neither (i) nor (ii)
EBD_8350
36 CHEMISTRY

49. Assertion : Helium is placed in group 18 along with p- 55. Why the size of an anion is larger than the parent atom?
block elements. (a) Due to increased repulsion among the electrons.
Reason : It shows properties similar to p-block elements. (b) Due to decrease in effective nuclear charge.
(a) Assertion is correct, reason is correct; reason is a (c) Due to increased in effective nuclear charge.
correct explanation for assertion. (d) Both (a) and (b)
(b) Assertion is correct, reason is correct; reason is not a 56. Alkali metals are powerful reducing agents because
correct explanation for assertion. (a) these are metals
(c) Assertion is correct, reason is incorrect. (b) their ionic radii are large
(d) Assertion is incorrect, reason is correct. (c) these are monovalent
50. Match the columns. (d) their ionisation potential is low
Column-I Column-II 57. Halogens and chalcogens family have highly ...P.. electron
(Name of element) (Period of element) gain enthalpy. Here, P refers to
(A) Hydrogen (p) 3 (a) negative (b) positive
(B) Sodium (q) 4 (c) zero (d) infinity
(C) Calcium (r) 6 58. The electron affinity for the inert gases is –
(D) Barium (s) 1 (a) zero (b) high
(E) Iodine (t) 5 (c) negative (d) positive
(a) A – (p), B – (s), C – (q), D – (r), E – (t) 59. The element with positive electron gain enthalpy is
(b) A – (s), B – (p), C – (q), D – (r), E – (t)
(a) hydrogen (b) sodium
(c) A – (s), B – (q), C – (p), D – (r), E – (t)
(c) oxygen (d) neon
(d) A – (s), B – (p), C – (q), D – (t), E – (r)
51. Match the columns. 60. An element having electronic configuration 1s22s22p63s2
3p64s1 forms
Column-I Column-II
(a) Acidic oxide (b) Basic oxide
(A) Element with largest size (p) Boron
(c) Amphoteric oxide (d) Neutral oxide
in second period
61. Consider the following statements:
(B) Element with smallest size (q) Fluorine
(i) Atomic radii decreases across a row of the periodic
in group 13
table when we move from left to right.
(C) Element with maximum (r) Bromine
(ii) Atomic radii increases down the column as we move
non-metallic character
from top to bottom.
(D) Element with smallest size (s) Lithium
(iii) Although the order of elements is based on atomic
in fourth period numbers, vertical families share similar chemical
(E) Element with most metallic (t) Lead properties.
character in group 14 Which of the statement(s) given above is/are correct?
(a) A – (s), B – (p), C – (q), D – (t), E – (r) (a) (i) and (ii) (b) (i) and (iii)
(b) A – (p), B – (s), C – (q), D – (r), E – (t) (c) (ii) and (iii) (d) (i), (ii) and (iii)
(c) A – (s), B – (q), C – (p), D – (r), E – (t) 62. Which of the following sequence correctly represents the
(d) A – (s), B – (p), C – (q), D – (r), E – (t) decreasing acidic nature of oxides?
52. Which of the following elements are found in pitch blende? (a) Li2O > BeO > B2O3 > CO2 > N2O3
(a) Actinium and protoactinium (b) N2O3 > CO2 > B2O3 > BeO > Li2O
(b) Neptunium and plutonium (c) CO2 > N2O3 > B2O3 > BeO > Li2O
(c) Actinium only (d) B2O3 > CO2 > N2O3 > Li2O > BeO
(d) Both (a) and (b) 63. Consider the following statements
53. Which of the given elements A, B, C, D and E with atomic
(i) The radius of an anion is larger than that of the parent
number 2, 3, 7, 10 and 30 respectively belong to the same
atom.
period?
(ii) The ionization energy generally increases with
(a) A, B, C (b) B, C, D
increasing atomic number in a period.
(c) A, D, E (d) B, D, E
(iii) The electronegativity of elements increases on
Topic 4: Periodic Trends in Properties of Elements moving down a group.
Which of the above statements is/are correct?
54. In the ions P3–, S2– and Cl–, the increasing order of size is
(a) (i) alone (b) (ii) alone
(a) Cl–, S2–, P3– (b) P3–, S2–, Cl–
2– – 3– (c) (i) and (ii) (d) (ii) and (iii)
(c) S , Cl , P (d) S2–, P3–, Cl–
CLASSIFICATION OF ELEMENTS AND PERIODICITY IN PROPERTIES 37

64. Match the Column-I and Column-II and select the correct (b) Assertion is correct, reason is correct; reason is not a
answer by given codes. correct explanation for assertion.
Column-I Column-II (c) Assertion is correct, reason is incorrect.
(Elements) (Properties) (d) Assertion is incorrect, reason is correct.
(A) Li+ < Al3+ < Mg2+ < K+ (p) EA (Electron affinity) 70. The order of increasing sizes of atomic radii among the
(B) Li+ > Al3+ > Mg2+ > K+ (q) Ionic radii elements O, S, Se and As is :
(C) Cl > F > Br > I (r) EN (Electronegativity) (a) As < S < O < Se
(b) Se < S < As < O
(D) F > Cl > Br > I (s) Effective nuclear charge
(c) O < S < As < Se
(a) A – (q), B – (s), C – (r), D – (p)
(d) O < S < Se < As
(b) A – (q), B – (s), C – (p), D – (r)
71. Which ionisation potential (IP) in the following equations
(c) A – (s), B – (q), C – (r), D – (p)
involves the greatest amount of energy ?
(d) A – (s), B – (q), C – (p), D – (r)
(a) Na ® Na+ + e–
65. Arrange the following in increasing order of ionic radii? (b) K+ ® K2+ + e–
C4–,N3–,F–,O2– (c) C2– ® C3+ + e–
(a) C4– < N3– < O2– < F– (d) Ca+ ® Ca2+ + e–
(b) N3– < C4– < O2– < F– 72. Arrange S, P, As in order of increasing ionisation energy
(c) F–< O2– < N3– < C4– (a) S < P < As (b) P < S < As
(d) O2– < F– < N3– < C4– (c) As < S < P (d) As < P < S
66. Which of the following property of element is directly
73. Of the given electronic configurations for the elements,
related to electronegativity?
which electronic configuration indicates that there will be
(a) Atomic radius abnormally high difference in the second and third
(b) Ionization enthalpy ionization energy for the element?
(c) Non-metallic character (a) 1s2 2s2 2p6 3s2
(d) None of these (b) 1s2 2s2 2p6 3s1
67. B has a smaller first ionization enthalpy than Be. Consider (c) 1s2 2s2 2p6 3s2 3p1
the following statements:
(d) 1s2 2s2 2p6 3s2 3p2
(I) it is easier to remove 2p electron than 2s electron
74. Which of the following metals requires the radiation of
(II) 2p electron of B is more shielded from the nucleus by highest frequency to cause the emission of electrons ?
the inner core of electrons than the 2s electrons of Be
(a) Na (b) Mg
(III) 2s electron has more penetration power than 2p
(c) K (d) Ca
electron
75. Which of the following represents the correct order of
(IV) atomic radius of B is more than Be
increasing electron gain enthalpy with negative sign for
(atomic number B = 5, Be = 4) the elements O, S, F and Cl ?
The correct statements are:
(a) Cl < F < O < S (b) O < S < F < Cl
(a) (I), (II) and (IV)
(c) F < S < O < Cl (d) S < O < Cl < F
(b) (II), (III) and (IV)
76. Which of the following will have the least negative
(c) (I), (II) and (III) electron gain enthalpy?
(d) (I), (III) and (IV) (a) P (b) S
68. The correct sequence which shows decreasing order of (c) Cl (d) F
the ionic radii of the elements is
77. Which is the correct order of electronegativity ?
(a) Al3+ > Mg 2+ > Na + > F- > O 2 -
(a) F > N < O > C
(b) Na + > Mg 2+ > Al3+ > O2- > F-
(b) F > N > O > C
(c) Na + > F - > Mg 2 + > O 2 - > Al3+
(c) F > N > O < C
(d) O2- > F- > Na + > Mg 2+ > Al3+
(d) F < N < O =C
69. Assertion : Smaller the size of an atom greater is the
electronegativity. 78. The correct order of decreasing electronegativity values
among the elements I-beryllium, II-oxygen, III-nitrogen and
Reason : Electronegativity refers to the tendency of atom
IV-magnesium is
to share electrons with other atom.
(a) Assertion is correct, reason is correct; reason is a (a) II > III > I > IV (b) III > IV > II > I
correct explanation for assertion. (c) I > II > III > IV (d) I > II > IV > III
EBD_8350
38 CHEMISTRY

79. Match Column-I with Column-II and select the correct The most reactive metal and the least reactive non-metal
answer by the given codes. of these are respectively
Columnn-I Column-II (a) I and V (b) V and II
(Atoms) (Properties) (c) II and V (d) IV and V
(A) He (p) Highly electronegative 84. Among the following transition elements, pick out the
(B) F (q) Most electropositive element/elements with highest second ionization energy.
(C) Rb (r) Strongest reducing agent (A) V (At. no = 23) (B) Cr (At. no = 24)
(D) Li (s) Highest ionisation (C) Mn (At. no = 25) (D) Cu (At. no = 29)
energy (E) Zn (At. no = 30)
(a) A – (s), B – (q), C – (r), D – (p) (a) (A) and (C) (b) (B) and (D)
(b) A – (p), B – (s), C – (q), D – (r) (c) (B) and (E) (d) Only (D)
(c) A – (s), B – (p), C – (r), D – (q) 85. As we move across the second period from C to F
(d) A – (s), B – (p), C – (q), D – (r) ionisation enthalpy increases but the trend from C to F for
80. Which of the following statement(s) is/are incorrect? ionisation enthalpy is C < O < N < F. This is because
(i) Ionization enthalpy is expressed in units of kJmol–1. (a) atomic radii of O > atomic radii of N
(ii) Ionization enthalpy is always positive. (b) electronic configuration of N is more stable than
(iii) Second ionization enthalpy will be higher than the electronic configuration of O
third ionization enthalpy. (c) atomic radii of N > atomic radii of O
(a) Only (ii) (d) None of these
(b) Only (iii) 86. Assertion : Atomic size increases along a period.
(c) (ii) and (iii) Reason : Effective nuclear charge increases as the atomic
(d) None of these number increases resulting in the increased attraction of
81. Covalent radii of atoms varies in range of 72 pm to 133 pm electrons to the nucleus.
from F to I while that of noble gases He to Xe varies from (a) Assertion is correct, reason is correct; reason is a
120 pm to 220 pm. This is because in case of noble gases correct explanation for assertion.
(a) covalent radius is very large (b) Assertion is correct, reason is correct; reason is not a
(b) van der Waal radius is considered correct explanation for assertion.
(c) metallic radii is considered (c) Assertion is correct, reason is incorrect.
(d) None of these (d) Assertion is incorrect, reason is correct.
82. The van der Waal and covalent radii of fluorine atom 87. Which one of the following statements is incorrect ?
respectively from the following figure are. (a) Greater the nuclear charge, greater is the electron
affinity
294 pm
(b) Nitrogen has zero electron affinity
(c) Electron affinity decreases from fluorine to iodine in
17th group
F F F F (d) Chlorine has highest electron affinity
88. The elements with zero electron affinity are
144 pm (a) Boron and Carbon
144 pm
(b) Beryllium and Helium
(a) 219 pm, 72 pm (b) 75 pm, 72 pm (c) Lithium and Sodium
(c) 147 pm, 72 pm (d) 147 pm, 144 pm (d) Fluorine and Chlorine
83. The first (DiH1) and second (DiH2) ionization enthalpies 89. Which is not the correct order for the stated property.
(in kJ mol –1) and the electron gain enthalpy (D egH) (a) Ba > Sr > Mg ; atomic radius
(in kJ mol–1) of the elements I, II, III, IV and V are given (b) F > O > N ; first ionization enthalpy
below (c) Cl > F > I ; electron affinity
Element D iH 1 D iH 2 D egH (d) O > Se > Te ; electronegativity
I 520 7300 – 60 90. The compounds of the s-block elements, with the exception
II 419 3051 – 48 of lithium and ...X... are predominantly ionic. Here, X refers
III 1681 3374 – 328 to
IV 1008 1846 – 295 (a) hydrogen (b) helium
V 2372 5251 + 48 (c) magnesium (d) beryllium
CLASSIFICATION OF ELEMENTS AND PERIODICITY IN PROPERTIES 39

NCERT Exemplar MCQs 9. The first ionisation enthalpies of Na, Mg, Al and Si are in
the order
1. Consider the isoelectronic species, Na +, Mg2+, F– and O2.
The correct order of increasing length of their radiiis (a) Na < Mg > Al < Si (b) Na > Mg > Al > Si
(a) F– < O2– < Mg2+ < Na+ (c) Na < Mg < Al < Si (d) Na > Mg > Al < Si
(b) Mg2+ < Na+ < F– < O2– 10. The formation of oxide ion O2–(g), from oxygen atom
requires first an exothermic and then an endothermic step
(c) O2– < F– < Na+ < Mg2+
as shown below
(d) O2– < F– < Mg2+ < Na+ O (g) + e– ® O– (g); DH° = – 141 kJ mol–1
2. Which of the following is not an actinoid? O– (g) + e– ® O2– (g); DH° = + 780 kJ mol–1
(a) Curium (Z = 96) (b) Californium (Z = 98) Thus, process of formation of O 2– in gas phase is
(c) Uranium (Z = 92) (d) Terbium (Z = 65) unfavourable even though O2– is isoelectronic with neon.
3. The order of screening effect of electrons of s, p, d and f It is due to the fact that
orbitals of a given shell of an atom on its outer shell (a) oxygen is more electronegative
electrons is (b) addition of electron in oxygen results in larger size of
(a) s > p > d > f (b) f > d > p > s the ion
(c) p < d < s > f (d) f > p > s > d (c) electron repulsion outweighs the stability gained by
4. The statement that is not correct for periodic classification achieving noble gas configuration
of elements is (d) O– ion has comparatively smaller size than oxygen
(a) the properties of elements are periodic function of atom
their atomic numbers. Comprehension given below is followed by some multiple choice
(b) non-metallic elements are less in number than metallic questions. Each question has one correct option. Choose the
elements. correct option.
(c) for transition elements, the 3d-orbitals are filled with In the modern periodic table, elements are arranged in order of
electrons after 3p-orbitals and before 4s-orbitals. increasing atomic numbers which is related to the electronic
(d) the first ionisation enthalpies of elements generally configuration. Depending upon the type of orbitals receiving
increase with increase in atomic number as we go the last electron, the elements in the periodic table have been
along a period. divided into four blocks, viz s, p, d and f.
5. Among halogens, the correct order of amount of energy The modern periodic table consists of 7 periods and 18 groups.
released in electron gain (electron gain enthalpy) is Each period begins with the filling of a new energy shell. In
accordance with the Aufbau principle, the seven periods (1 to
(a) F > Cl > Br > I (b) F < Cl < Br < I
7) have 2, 8, 8, 18, 18, 32 and 32 elements respectively.
(c) F < Cl > Br > I (d) F < Cl < Br < I
The seventh period is still incomplete. To avoid the periodic
6. The period number in the long form of the periodic table is table being too long, the two series of f-block elements, called
equal to lanthanoids and actinoids are placed at the bottom of the main
(a) magnetic quantum number of any element of the period body of the periodic table.
(b) atomic number of any element of the period 11. The element with atomic number 57 belongs to
(c) maximum principal quantum number of any element (a) s-block (b) p-block
of the period
(c) d-block (d) f-block
(d) maximum azimuthal quantum number of any element
of the period 12. The last element of the p-block in 6th period is represented
by the outermost electronic configuration.
7. The elements in which electrons are progressively filled
in 4f-orbital are called (a) 7s2 7p6 (b) 5f 14 6d10 7s2 7p0
(a) actinoids (b) transition elements (c) 4f 14 5d10 6s2 6p6 (d) 4f 14 5d10 6s2 6p4
(c) lanthanoids (d) halogens 13. Which of the elements whose atomic numbers are given
8. Which of the following is the correct order of size of the below, cannot be accommodated in the present set up of
given species the long form of the periodic table?
(a) I > I– > I+ (b) I+ > I– > I (a) 107 (b) 118
+
(c) I > I > I – (d) I– > I > I+ (c) 126 (d) 102
EBD_8350
40 CHEMISTRY

14. The electronic configuration of the element which is just (c) 1.36, 1.40 and 1.71
above the element with atomic number 43 in the same (d) 1.36, 1.71 and 1.40
group is ............
21. In which of the following options the order of arrangement
(a) 1s2 2s2 2p6 3s2 3p6 3d5 4s2 does not agree with the variation of property indicated
(b) 1s2 2s2 2p6 3s2 3p6 3d5 4s3 4p6 against it ? [NEET 2016, S]
(c) 1s2 2s2 2p6 3s2 3p6 3d 6 4s2 3+ 2+ + –
(a) Al < Mg < Na < F (increasing ionic size)
(d) 1s2 2s2 2p6 3s2 3p6 3d7 4s2 (b) B < C < N < O (increasing first ionisation enthalpy)
15. The elements with atomic numbers 35, 53 and 85 are (c) I < Br < Cl < F (increasing electron gain enthalpy)
all ................ .
(d) Li < Na < K < Rb (increasing metallic radius)
(a) noble gases (b) halogens
(c) heavy metals (d) light metals 22. Which of the following atoms has the highest first
ionization energy? [JEE M 2016, S]
16. Electronic configuration of four elements A, B, C and D
(a) K (b) Sc
are given below
(c) Rb (d) Na
A. 1s2 2s2 2p6 B. 1s2 2s2 2p4
2 2 6 1 23. The element Z = 114 has been discovered recently.
C. 1s 2s 2p 3s D. 1s2 2s2 2p5
It will belong to which of the following family/group and
Which of the following is the correct order of increasing electronic configuration ? [NEET 2017, A]
tendency to gain electron?
(a) Carbon family, [Rn] 5f 14 6d10 7s2 7p2
(a) A < C < B < D (b) A < B < C < D
(b) Oxygen family, [Rn] 5f 14 6d10 7s2 7p4
(c) D < B < C < A (d) D < A < B < C
(c) Nitrogen family, [Rn] 5f 14 6d10 7s2 7p6
Past Year MCQs (d) Halogen family, [Rn] 5f 14 6d10 7s2 7p5
24. The group having isoelectronic species is :
17. Which of the following orders of ionic radii is correctly
[JEE M 2017, S]
represented ? [AIPMT 2014, A]
(a) O2– , F–, Na+, Mg2+
(a) H– > H+ > H (b) Na+ > F– > O2– (b) O– , F–, Na, Mg+
(c) F– > O2– > Na+ (d) Al3+> Mg2+> N3– (c) O2– , F–, Na , Mg2+
18. The species Ar, K+ and Ca2+ contain the same number of (d) O– , F–, Na+ , Mg2+
electrons. In which order do their radii increase ? 25. Both lithium and magnesium display several similar
properties due to the diagonal relationship; however, the
[AIPMT 2015, A]
one which is incorrect is : [JEE M 2017, S]
2+ +
(a) Ca < Ar < K (b) Ca < K + < Ar
2+
(a) Both form basic carbonates
(c) K + < Ar < Ca 2+ (d) Ar < K + < Ca 2+ (b) Both form soluble bicarbonates
19. The formation of the oxide ion O2–(g), from oxygen atom (c) Both form nitrides
requires first an exothermic and then an endothermic step (d) Nitrates of both Li and Mg yield NO2 and O2 on heating
as shown below : 26. For the second period elements the correct increasing order
O(g) + e– ® O–(g); = –141 kJ mol–1 of first ionisation enthalpy is : [NEET 2019, C]
(a) Li < Be < B < C < N < O < F < Ne
O– (g) + e– ® O2– (g); = +780 kJ mol –1
(b) Li < B < Be < C < O < N < F < Ne
Thus process of formation of O 2– in gas phase is
(c) Li < B < Be < C < N < O < F < Ne
unfavourable even though O2– is isoelectronic with neon.
(d) Li < Be < B < C < O < N < F < Ne
It is due to the fact that [AIPMT 2015 RS, S]
27. The element having greatest difference between its first
(a) Electron repulsion outweighs the stability gained by and second ionization energies, is: [JEE M 2019, C]
achieving noble gas configuration
(a) Ca (b) Sc
(b) O– ion has comparatively smaller size than oxygen
(c) Ba (d) K
atom
28. In general, the properties that decrease and increase down
(c) Oxygen is more electronegative
a group in the periodic table, respectively, are:
(d) Addition of electron in oxygen results in larger size
of the ion. [JEE M 2019, C]
(a) atomic radius and electronegativity.
20. The ionic radii (in Å) of N3–, O2– and F– are respectively :
(b) electron gain enthalpy and electronegativity.
[JEE M 2015, A]
(c) electronegativity and atomic radius.
(a) 1.71, 1.40 and 1.36
(d) electronegativity and electron gain enthalpy.
(b) 1.71, 1.36 and 1.40
CLASSIFICATION OF ELEMENTS AND PERIODICITY IN PROPERTIES 41

29. The number of protons, neutrons and electrons in 31. Match the following : [NEET 2020, A]
175 Oxide Nature
71 Lu, respectively, are [NEET 2020, C] (A) CO (i) Basic
(a) 104, 71 and 71 (B) BaO (ii) Neutral
(b) 71, 71 and 104 (C) Al2O3 (iii) Acidic
(c) 175, 104 and 71 (D) Cl2O7 (iv) Amphoteric
(d) 71, 104 and 71 Which of the following is correct option?
30. Identify the incorrect match. [NEET 2020, S] (A) (B) (C) (D)
Name IUPAC Official Name (a) (ii) (i) (iv) (iii)
(A) Unnilunium (i) Mendelevium (b) (iii) (iv) (i) (ii)
(B) Unniltrium (ii) Lawrencium (c) (iv) (iii) (ii) (i)
(C) Unnilhexium (iii) Seaborgium (d) (i) (ii) (iii) (iv)
(D) Unununnium (iv) Darmstadtium 32. The electron gain enthalpy (in kj/mol) of fluorine, chlorine,
(a) (B), (ii) bromine and iodine, respectively, are: [JEE M 2020, C]
(a) –296, –325, –333 and –349
(b) (C), (iii)
(b) –349, –333, –325 and –296
(c) (D), (iv) (c) –333, –349, –325 and –296
(d) (A), (i) (d) –333, –325, –349 and –296

1. Consider the following changes (c) SO2< P2O3 < SiO2 < Al2O3
+ - + 2+ - (d) Al2O3 < SiO2< P2O3 < SO2
A ® A + e : E1 and A ® A + e : E 2
The energy required to pull out the two electrons are E 1 6. Among O, O+, O2+ and O2–, the species having most
and E2 respectively. The correct relationship between two positive and most negative value of DHeg are respectively.
energies would be (a) O+ and O (b) O2+ and O2–
2–
(a) E1 < E2 (b) E1 = E2 (c) O and O (d) O2– and O2+
(c) E1 > E2 (d) E1 ³ E2 7. Which of the following is the correct order of lattice
energy?
2. Sum of first three ionization energies of Al is 53.0 eV atom–1
and the sum of first two ionization energies of Na is (a) MgO < LiF < KBr (b) KBr < LiF < MgO
52.2 eV atom–1. Out of Al(III) and Na(II) (c) LiF < KBr < MgO (d) LiF= KBr < MgO
(a) Na (II) is more stable than Al (III) 8. Which of the following transformation requires least
(b) Al (III) is more stable than Na (II) energy?
(c) Both are equally stable (a) F– (g) ® F(g) + e– (b) P– (g) ® P(g) + e–
(d) Both are equally unstable (c) S–(g) ® S(g) + e– (d) Cl– (g) ® Cl(g) + e–
3. The formation of O2– (g) starting from O (g) is endothermic 9. Incorrect order of radius is :
by 603 kJ mol–1. If electron affinity of O(g) is –141 kJ mol–1, (a) Sr2+ < Rb+ < Br– < Se2– (b) Nb5+ < Zr4+ < Y3+
the second electron affinity of oxygen would be (c) Co > Co2+ > Co3+ > Co4+ (d) Ba2+ < Cs+ < Se2– < As3–
(a) 603 kJ mol–1 (b) –603 kJ mol–1 10. Consider the following four elements, which are
(c) –744 kJ mol –1
(d) +744 kJ mol–1 represented according to long form of periodic table
4. The electron affinity of chlorine is 3.7 eV. 1 gram of chlorine Y
is completely converted to Cl– ion in a gaseous state.
W X Z
(1 eV = 23.06 kcal mol–1).
Energy released in the process is Here W, Y and Z are left, up and right elements with respect
to the element 'X' and 'X' belongs to 16th group and 3rd
(a) 4.8 kcal (b) 7.2 kcal
period. Then according to given information the incorrect
(c) 8.2 kcal (d) 2.4 kcal statement regarding given elements is:
5. Among Al2O3, SiO2, P2O3 and SO2 the correct order of (a) Maximum electronegativity : Y
acid strength is (b) Maximum catenation property : X
(a) Al2O3 < SiO2< SO2 < P2O3 (c) Maximum electron affinity : Z
(b) SiO2< SO2 < Al2O3 < P2O3 (d) Y exhibits variable covalency
EBD_8350
42 CHEMISTRY

11. Aqueous solutions of two compounds M1 – O – H and 13. The set of three elements having successive atomic
M2 – O – H are prepared in two different beakers. If, the numbers and having the ionization energies of 2372, 520
electronegativity of M1 = 3.4, M2 = 1.2, O = 3.5 and H= 2.1 and 890 kJ per mol is
then the nature of two solutions will be respectively : (a) H, He, Li (b) He, Li, Be
(a) acidic, basic (c) Li, Be, B (d) B, C, N
(b) acidic, acidic 14. The acidic, basic and amphoteric oxides, respectively, are:
(c) basic, acidic (a) Na2O, SO3, Al2O3 (b) Cl2O, CaO, P4O10
(d) basic, basic (c) N2O3, Li2O, Al2O3 (d) MgO, Cl2O, Al2O3
12. Select the correct order for the property given in brackets: 15. In comparison to boron, berylium has :
+ +
(a) F (g) < Cl (g) : (EA order) (a) lesser nuclear charge and lesser first ionisation enthalpy.
(b) F (g) < Cl (g) : (DHeg order) (b) greater nuclear charge and lesser first ionisation enthalpy.
(c) O (g) > S (g) : (DHeg order) (c) greater nulear charge and greater first ionisation enthalpy.
(d) S2– (g) > Cl– (g) : (IE order) (d) lesser nuclear charge and greater first ionisation enthalpy.

ANSWER KEY
Exercise 1 : NCERT Based Topic-wise MCQs
1 (c) 10 (a) 19 (b) 28 (b) 37 (d) 46 (b) 55 (d) 64 (b) 73 (a) 82 (c)
2 (c) 11 (d) 20 (b) 29 (b) 38 (a) 47 (d) 56 (d) 65 (c) 74 (b) 83 (c)
3 (a) 12 (c) 21 (c) 30 (b) 39 (a) 48 (c) 57 (a) 66 (c) 75 (b) 84 (b)
4 (b) 13 (d) 22 (c) 31 (b) 40 (b) 49 (c) 58 (d) 67 (c) 76 (a) 85 (b)
5 (a) 14 (b) 23 (b) 32 (a) 41 (b) 50 (b) 59 (d) 68 (d) 77 (a) 86 (d)
6 (b) 15 (b) 24 (c) 33 (b) 42 (c) 51 (d) 60 (b) 69 (c) 78 (a) 87 (c)
7 (b) 16 (b) 25 (c) 34 (b) 43 (c) 52 (d) 61 (d) 70 (d) 79 (d) 88 (b)
8 (b) 17 (a) 26 (c) 35 (b) 44 (a) 53 (b) 62 (b) 71 (c) 80 (b) 89 (b)
9 (d) 18 (c) 27 (c) 36 (d) 45 (d) 54 (a) 63 (c) 72 (c) 81 (b) 90 (d)
Exercise 2 : NCERT Exemplar & Past Year MCQs
1 (b) 5 (c) 9 (a) 13 (c) 17 (N) 21 (b, c) 25 (a) 29 (d)
2 (d) 6 (c) 10 (c) 14 (a) 18 (b) 22 (b) 26 (b) 30 (c)
3 (a) 7 (c) 11 (c) 15 (b) 19 (a) 23 (a) 27 (d) 31 (a)
4 (c) 8 (d) 12 (c) 16 (a) 20 (a) 24 (a) 28 (c) 32 (c)
Exercise 3 : Problem Solving Skill Enhancer MCQs
1 (a) 3 (d) 5 (d) 7 (b) 9 (d) 11 (a) 13 (b) 15 (d)
2 (b) 4 (d) 6 (d) 8 (b) 10 (d) 12 (b) 14 (c)
Chemical Bonding and
4 Molecular Structure
Trend Buster NEET & JEE Main

Number of Questions from 2020-15 19 9 Chapter has more weightage for NEET
examination. Minimum two questions
Weightage 6.0% 5.1% has been asked every year.
The most Important Concepts that Cover Maximum number of Questions asked in past 6 years.

Bond order/ stability 5 —


Structure of the molecule 3 1
Isoelectronic/isostructural 3 —
Molecular orbital configuration 1 3
Bond angle 3 —

Less Important Concepts that Cover 1 or 2 Questions asked in past 6 years.

Sigma & p-bond 2 —


Intemolecular forces / Dipole moment 1 2
Ionic / covalent bond — 1
Paramagnetic / dimagnetic — 1
L.P / B.P / Hybridisation 1 1

NEET JEE

2020 Bond Parameters / Molecular Intermolecular forces / 2 Easy / 2 Easy


orbital theory Dipolemoment / Bond order Average
2019 Bonding in homonuclear / Molecular orbital 4 Easy / 2 Average
Diatomic molecules / configuration / Structure of the Average
Molecular orbital theory / molecule / Sigma & p–bond
Valence bond theory
2018 V S E PR theory / Molecular Structure of the molecule / 2 Average 3 Average /
orbital theory Ionic / Covalent Bond order / Molecular orbital Easy
bond configuration / Covalent bond
2017 Molecular orbital theory / Bond order / Paramagnetic / 3 Average / 1 Average
V S E P R Theory dimagnetic / Bond angle Easy
Isoelectronic / Isostructural
2016 V S E P R Theory / L.P / B.P reulsion / Bond 2 Easy / 1 Easy
Hybridisation angle Hybridisation Average
2015 V S E P R Theory / Isoelectronic / Isostructural / 6 Average — —
Molecular orbital theory / Bond order / stability / Sigma
VBT & p–bond / Bond angle
EBD_8350
44 CHEMISTRY
CHEMICAL BONDING AND MOLECULAR STRUCTURE 45
EBD_8350
46 CHEMISTRY

Problem Solving Tips/ Tricks/ Points to Remember


4 There are two types of H-bonding (v) o-nitrophenol is more volatile (b.pt 214 ºC)
(i) Intermolecular H-bonding (association). as compared to meta (b.pt 290 ºC) and para
H–bonding involving two or more molecules. (b.pt 279 ºC). It is due to chelation
(ii) Intramolecular H-bonding (chelation). H–bonding
1
taking place within single molecule. 4 DHf = DHsub + DHd + I.E. + E.A. + U
2
4 Applications of Intermolecular where,
H-Bonding DHsub = Enthalpy of formation ü
(i) Water : Water has the lowest molecular weight DHsub= Enthalpy of sublimation ï positive
among the hydrides of group 16 elements yet it ý
DHd = Enthalpy of dissociation ï
has the highest melting and boiling points. I.E. = Ionization energy þ
(ii) Ice has less density than water E.A. = Electron affinity ü negative
ï
Amides associate and have higher melting and U = Lattice energy or Lattice enthalpy ý
ï
boiling points. þ

4 Applications of Intramolecular 4 In order to determine the number of pp – pp or pp – dp


H-Bonding bonds, the following formula can be used.
(i) Salicylic acid is stronger acid than o - methoxy (i) Calculate the number of s bonds = number of
benzoic acid surrounding atoms
(ii) Ethyl aceto acetate - It exists in two forms (ii) p bonds = number of oxygen atoms – number of
negative charge
O
ôô (iii) Lone pair = 1/2 (Valence electron – number of
CH3 - C - CH 2COOC2H5 covalent bond)
Keto form (iv) Hybridization = s bonds + number of lone pairs
(v) Now, number of pp – pp bonds = number of
O unhybridized p-orbitals left
ô
CH3 - C = CHCOOC 2 H5 (vi) pp – dp bonds will be formed when p bonds are more
Keto form than the number of unhybridized p–orbitals left.
Enolic form is more volatile due to chelation 4 The more the electronegativity of atom involved in
d- d+ d- H-bonding, the more is the bond strength eg.
O H ----- O
HLF > HL O > HL N
CH3 – C = CH – C – OC2H5 10 kcal/mole > 7 kcal/mole > 2.0 kcal/mole
(iii) Maleic acid is stronger acid than fumaric acid 4 Formation of a chemical bond is always an exothermic
Maleate ion can be stabilised by Chelation process.
because hydrogen and oxygen responsible for 4 FeCl3 is more covalent than FeCl2 becuase polarising power
forming hydrogen bond are very near to each of Fe3+ is more than than of Fe2+.
other. On the other hand fumerate ion cannot 4 Free rotation about a s-bond is possible but it is not
stablise by chelation because hydrogen and possible in p-bond.
oxygen are on opposite sides to each other. Hence
formation of fumerate ion does not take place. 4 The hydrogen bond is very strong in HF and persist even
in vapour state. Gaseous hydrogen fluoride is largely
(iv) Acid character of nitrophenols. It follows the
polymerised into the molecular species H2F2, H3F3, H4F4,
following order
.... etc.
p - nitrophenol > o - nitrophenol > m - nitrophenol
Acid character of o - nitrophenol is suppressed 4 C2 molecule is an exception which contains no s-bond but
by chelation two p-bonds.
O d+ 4 Ionic compounds consist of ions in solid state while
H
O
d– covalent compound consist of molecules.
N
4 The relative strength of various bonds is: ionic > coordinate
O
> covalent > metallic > H-bond > van der Waal forces.
CHEMICAL BONDING AND MOLECULAR STRUCTURE 47

10. Assertion : Atoms can combine either by transfer of


Topic 1: Kossel-Lewis Approach to Chemical Bonding valence of electrons from one atom to another or by
1. The evolution of various theories of valence and the sharing of valence electrons.
interpretation of the nature of chemical bonds have closely Reason : Sharing and transfer of valence electrons is
been related to the developments in the understanding of done by atoms to have an octet in their valence shell.
(a) structure of atom (a) Assertion is correct, reason is correct; reason is a
(b) electronic configuration of elements correct explanation for assertion.
(c) periodic table (b) Assertion is correct, reason is correct; reason is not a
(d) All of the above correct explanation for assertion.
2. Who provide explanation of valence based on intertness (c) Assertion is correct, reason is incorrect.
of noble gases ? (d) Assertion is incorrect, reason is correct.
(a) Lewis (b) K&o&ssel-Lewis 11. Which of the following is/are not the condition(s) for
(c) Langmuir (d) Sidgwick & Powell Lewis dot structure?
3. Who introduced the term covalent bond ? (i) Each bond is formed as a result of sharing of an
(a) Lewis (b) Langmuir electron pair between the atoms.
(c) Nyholm and Gillespie (d) Heitler and London (ii) From the two combining atoms only one atom
4. The bond formed as a result of the electrostatic contribute electron(s) to the shared pair.
attraction between the positive and negative ions is (iii) The combining atoms attain the outer shell noble
termed as ... gas configurations as a result of the sharing of
(a) Chemical bond (b) Electrovalent bond electrons.
(c) Co-ordinate bond (d) Covalent bond (a) (i) and (iii) (b) (ii) and (iii)
5. When a metal atom combines with non-metal atom, the (c) (ii) only (d) (iii) only
non-metal atom will 12. Which of the following Lewis representation of the
(a) lose electrons and decrease in size molecules NF3, O3 and HNO3 is correct ?
(b) lose electrons and increase in size
(c) gain electrons and decrease in size
(d) gain electrons and increase in size
6. The lowest energy structure is the one with the ..........
formal charges on the atoms. Choose the correct option(s).
(a) smallest (b) highest
(a) Only I (b) Only II
(c) zero (d) negative
(c) Only III (d) I, II and III
7. Assertion : The correct Lewis structure of O3 may be drawn
as 13. Which of the following molecule(s) obey the octet rule?
+ (i) [BF4]– (ii) [AlCl4]–
1
(iii) SO2, (iv) CCl4
:

O

(a) (i), (ii), (iii), (iv) (b) (ii), (iii), (iv)
2
: :
: :

O O: 3 (c) (i), (iii), (iv) (d) (i), (ii), (iv)


Reason : The formal charges on atom 1, 2 and 3 are + 1, 0 14. A pair of compound which have odd electrons in the group
and –1 respectively. NO, CO, ClO2, N2O5, SO2 and O3 are
(a) Assertion is correct, reason is correct; reason is a (a) NO and ClO2 (b) CO and SO2
correct explanation for assertion. (c) ClO2 and CO (d) SO2 and O3
(b) Assertion is correct, reason is correct; reason is not a
15. Assertion : Sulphur compounds like SF6 and H2SO4
correct explanation for assertion.
have 12 valence electrons around S atom.
(c) Assertion is correct, reason is incorrect.
Reason : All sulphur compounds do not follow octet rule.
(d) Assertion is incorrect, reason is correct.
8. Which of the following is the electron deficient (a) Assertion is correct, reason is correct; reason is a
molecule? correct explanation for assertion.
(a) C2H6 (b) B2H6 (c) SiH4 (d) PH3 (b) Assertion is correct, reason is correct; reason is not a
9. Which of the following compounds does not follow the correct explanation for assertion.
octet rule for electron distribution? (c) Assertion is correct, reason is incorrect.
(a) PCl5 (b) PCl3 (c) H2O (d) PH3 (d) Assertion is incorrect, reason is correct.
EBD_8350
48 CHEMISTRY

(b) Assertion is correct, reason is correct; reason is not a


Topic 2: Ionic or Electrovalent Bond
correct explanation for assertion.
16. Energy required to completely separate one mole of a (c) Assertion is correct, reason is incorrect.
solid ionic compound into gaseous constituent ions is (d) Assertion is incorrect, reason is correct.
called .......... .
Topic 3: Bond Parameters
(a) Ionisation enthalpy
(b) Electron gain enthalpy 24. Which of the following structure represents structure of
O3 more accurately?
(c) Bond dissociation enthalpy
O O
(d) Lattice enthalpy

12

14
pm
m

1p

8
8p

pm
1
17.

m
Which of the following combination will form an

12
14
electrovalent bond ? O O O O
(a) P and Cl (b) NH3 and BF3
I II
(c) H and Ca (d) H and S
18. Complete the following statement by choosing the
appropriate option. O

12
pm

8
Ionic bonds will be formed more easily between elements

pm
8
12
with comparatively A and elements with O O
comparatively high negative value of B . III (c) III
(a) I (b) II (d) I and II
(a) A = low electronegativity; B = ionization enthalpy
25. The molecule which has zero dipole moment is
(b) A = low ionization enthalpy; B = electron gain enthalpy
(a) CH3Cl (b) NF3 (c) BF3 (d) ClO2
(c) A = high ionization enthalpy; B = electron gain enthalpy 26. The most polar bond is
(d) A = high electronegativity; B = ionization enthalpy (a) C – F (b) C – O (c) C – Br (d) C – S
19. In ionic solids how crystal structure get stabilized 27. According to Fajan’s rule, covalent bond is favoured by
(a) By the energy released in the formation of crystal (a) Large cation and small anion
lattice. (b) Large cation and large anion
(b) By achieving octet of electrons around the ionic (c) Small cation and large anion
species in gaseous state. (d) Small cation and small anion
(c) By electron gain enthalpy and the ionization enthalpy. 28. Assertion : CH2Cl2 is non-polar and CCl4 is polar molecule.
(d) None of these Reason : Molecule with zero dipole moment is non-polar
20. Among the following which compound will show the in nature.
highest lattice energy ? (a) Assertion is correct, reason is correct; reason is a
(a) KF (b) NaF (c) CsF (d) RbF correct explanation for assertion.
21. Which of the following pairs will form the most stable ionic (b) Assertion is correct, reason is correct; reason is not a
bond ? correct explanation for assertion.
(c) Assertion is correct, reason is incorrect.
(a) Na and Cl (b) Mg and F
(d) Assertion is incorrect, reason is correct.
(c) Li and F (d) Na and F 29. The correct sequence of increasing covalent character
22. Match the columns is represented by
Column-I Column-II (a) LiCl < NaCl < BeCl2 (b) BeCl2 < LiCl < NaCl
(A) HCl (p) Covalent compound (c) NaCl < LiCl < BeCl2 (d) BeCl2 < NaCl < LiCl
with directional bond 30. Polarisibility of halide ions increases in the order
(B) CO2 (q) Ionic compound with (a) F–, I– , Br–, Cl– (b) Cl–, Br– , I–, F–
non-directional bonds – – –
(c) I , Br , Cl , F – (d) F–, Cl– , Br–, l–
(C) NaCl (r) Polar molecule 31. Hydrogen chloride molecule contains
(D) CCl4 (s) Non-polar molecule (a) polar covalent bond (b) double bond
(a) A – (p, q, r), B – (q, r), C – (p, q), D – (r) (c) co-ordinate bond (d) electrovalent bond
(b) A – (q), B – (r), C – (p), D – (s) 32. Which one of the following molecules is expected to have
(c) A – (p, r), B – (p, s), C – (q), D – (p, s) zero dipole moment?
(d) A – (q), B – (r), C – (p, q), D – (s) (a) H2O (b) CO2 (c) SO2 (d) CaF2
23. Assertion : The lesser the lattice enthalpy, more stable is 33. Which of the following methods is used for measuring
the ionic compound. bond length ?
Reason : The lattice enthalpy is greater for ions of highest (a) X-ray diffraction
charge and smaller radii. (b) Electron-diffraction
(a) Assertion is correct, reason is correct; reason is a (c) Spectroscopic techniques
correct explanation for assertion. (d) All of these
CHEMICAL BONDING AND MOLECULAR STRUCTURE 49

34. .......... is measured as the radius of an atom’s core which 42. Which of the following possess dipole moment SF6(i),
is in contact with the core of an adjacent atom in a SO2(ii), H2S(iii) , SF4(iv) ?
bonded situation. (a) (ii) and (iii) (b) (i) and (iii)
(a) van der Waal’s radius (b) Bond length (c) (ii), (iii) and (iv) (d) (i) and (ii)
(c) Covalent radius (d) Ionic radius 43. From the given figure the van der Waal radius and covalent
35. Which of the following statement is correct? radius of the hydrogen atom respectively are
(a) Amount of energy required to break one mole of 62 pm
bonds of a particular type between two atoms in a
gaseous state is called bond enthalpy.
(b) The unit of bond enthalpy is kJ mol–1. 240 pm
(c) Larger the bond dissociation enthalpy, stronger will
be the bond in the molecule. (a) 151, 31 (b) 120, 31 (c) 31, 100 (d) 30, 120
(d) All of these 44. The covalent bond length is the shortest in which one of
36. Which one of the following pairs of molecules will have the following bonds?
permanent dipole moments for both members ? (a) C — O (b) C — C (c) C º N (d) O — H
(a) NO2 and CO2 (b) NO2 and O3 45. Among the following, the molecule of high dipole moment is
(c) SiF4 and CO2 (d) SiF4 and NO2 (a) CCl4 (b) NH3 (c) H2O (d) CHCl3
37. Which of the following is correct representation of 46. Which of the following substances has the least covalent
resonance? character ?
– .. + + .. – (a) Cl2O (b) NCl3 (c) PbCl2 (d) BaCl2
: :
:

: :

A. :O::C::O: :O::C::O: :O::C:O:


I II III Topic 4: The Valence Shell Electron Pair Repulsion
– –
:

:O: :O: :O: (VSEPR) Theory


47. Which of the following is the correct increasing order of
B. C C C lone pair of electrons on the central atom?
– – : (a) IF7 < IF5 < CIF3 < XeF2
:O:– :O:–
: :

:O: :O: :O O
: (b) IF7 < XeF2 < CIF2 < IF5
:

I II III
Choose the correct option. (c) IF7 < CIF3 < XeF2 < IF5
(a) Only A (b) Only B (d) IF7 < XeF2 < IF5 < CIF3
(c) Both A and B (d) None of these 48. The number of lone pair and bond pair of electrons on the
38. Which of the following is/are misconception(s) associated sulphur atom in sulphur dioxide molecule are respectively
with resonance ? (a) 1 and 3 (b) 4 and 1
(i) The molecule exist for a certain fraction of time in (c) 3 and 1 (d) 1 and 4
one cannonical form and for other fractions of time 49. A molecule has two lone pairs and two bond pairs around
in other cannonical forms. the central atom. The molecule shape is expected to be
(ii) The cannonical forms have no real existence. (a) V-shaped (b) triangular
(iii) There is no such equilibrium between the cannonical forms. (c) linear (d) tetrahedral
(a) (i) only (b) (ii) and (iii) 50. Which of the following molecules has trigonal planar
geometry?
(c) (i) and (iii) (d) (iii) only
(a) BF3 (b) NH3
39. Resonance is due to
(c) PCl3 (d) IF3
(a) delocalization of sigma electrons
51. The shape of stannous chloride molecule is
(b) delocalization of pi electrons
(a) see-saw
(c) migration of protons
(b) square planar
(d) Both (a) and (b)
(c) trigonal pyramidal
40. Arrange the following in increasing order of covalent
(d) bent
character (i) NaCl, (ii) RbCl, (iii) MgCl2, (iv) AlCl3 ?
52. According to VSEPR theory, the geometry of a covalent
(a) (i), (ii), (iii), (iv) (b) (iv), (ii), (i), (iii)
molecules depends upon
(c) (ii), (i), (iii), (iv) (d) (iii), (i), (ii), (iv)
(a) the number of bond pairs of electrons
41. Which of the following salt shows maximum covalent
(b) the number of lone pairs of electrons
character?
(c) the number of electron pairs present in the outer shell
(a) AlCl3 (b) MgCl2 of the central atom
(c) CsCl (d) LaCl3 (d) All of the above
EBD_8350
50 CHEMISTRY

53. The geometry of ClO3– ion according to Valence Shell (a) Assertion is correct, reason is correct; reason is a
Electron Pair Repulsion (VSEPR) theory will be correct explanation for assertion.
(a) planar triangular (b) pyramidal (b) Assertion is correct, reason is correct; reason is not a
(c) tetrahedral (d) square planar correct explanation for assertion.
54. Match Column-I with Column-II and Column-III and (c) Assertion is correct, reason is incorrect.
choose the correct option from the given codes. (d) Assertion is incorrect, reason is correct.
Column-I Column-II Column-III 61. Match the columns
Molecule (No. of lone (Shape of molecule) Column-I Column-II
pairs and (A) Trigonal planar (p) PCl5
bond pairs)
(A) NH3 (i) 1, 2 (p) Bent
120°
(B) SO2 (ii) 1, 4 (q) Trigonal pyramidal
A
(C) SF4 (iii) 2, 3 (r) T-shape
(D) ClF3 (iv) 1, 3 (s) See-Saw
(a) A – (iv, q); B – (ii, p); C – (i, r); D – (iii, s) (B) Tetrahedral (q) NH4+

:
(b) A – (iv, q); B – (i, p); C – (ii, s); D – (iii, r)
(c) A – (i, p); B – (iii, s); C – (iv, r); D – (ii, q)
109.5°
(d) A – (iv, p); B – (i, r); C – (iii, q); D – (ii, s)
A
55. Using VSEPR theory, predict the species which has square
pyramidal shape
:

:
(a) SnCl2 (b) CCl4
(c) SO3 (d) BrF5
56. Assertion : Lone pair-lone pair repulsive interactions are

:
(C) Trigonal bipyramidal (r) SF6
greater than lone pair-bond pair and bond pair-bond pair
interactions. :
Reason : The space occupied by lone pair electrons is
more as compared to bond pair electrons.
(a) Assertion is correct, reason is correct; reason is a 90°
correct explanation for assertion. : :
A
(b) Assertion is correct, reason is correct; reason is not a 120°
correct explanation for assertion.
:

(c) Assertion is correct, reason is incorrect.


(d) Assertion is incorrect, reason is correct.
57. Among the following molecules : SO2, SF4, ClF3, BrF5 and
:

XeF4, which of the following shapes does not describe (D) Octahedral (s) BF3
any of the molecules mentioned?
:

90°
(a) Bent
:
:

(b) Trigonal bipyramidal


90° A
(c) See-saw
(d) T-shape
:

58. A s-bonded molecule MX3 is T-shaped. The number of


:

non-bonding pairs of electron is (a) A – (p), B – (q), C – (r), D – (s)


(a) 0 (b) A – (s), B – (r), C – (q), D – (p)
(b) 2 (c) A – (s), B – (q), C – (p), D – (r)
(c) 1 (d) A – (r), B – (p), C – (q), D – (s)
(d) can be predicted only if atomic number of M is known. 62. In BrF3 molecule, the lone pairs occupy equatorial positions
59. Which of the following molecules is planar? to minimize
(a) SF4 (b) XeF4 (a) lone pair - bond pair repulsion only
(c) NF3 (d) SiF4 (b) bond pair - bond pair repulsion only
60. Assertion : In NH3, N is sp3 hybridised, but angle is found (c) lone pair - lone pair repulsion and lone pair - bond
to be 107°. pair repulsion
Reason : The decrease in bond angle is due to repulsion (d) lone pair - lone pair repulsion only
between the lone pair – bond pair.
CHEMICAL BONDING AND MOLECULAR STRUCTURE 51

63. Which of the following structure is most stable ? 72. Look at the following potential energy curve. Which of
the following correctly represents the most stable state
F .. F of hydrogen molecule?
F
..

..
Cl F F Cl Cl F
..

.. F F ..
F
A D
I II III O C
Choose the correct option.

Energy
(a) Only I
(b) Only II
(c) Only III B Internuclear distance ¾®
(d) All three have same stability
(a) A (b) B
Topic 5: Valence Bond Theory (c) C (d) D
73. The enolic form of a acetone contains
64. Which of the following statements is false ? (a) 9 sigma bonds, 1 pi bond and 2 lone pairs
(a) H2 molecule has one sigma bond (b) 8 sigma bonds, 2 pi bonds and 2 lone pairs
(b) HCl molecule has one sigma bond (c) 10 sigma bonds, 1 pi bond and 1 lone pair
(c) Water molecule has two sigma bonds and two lone pairs (d) 9 sigma bonds, 2 pi bonds and 1 lone pair
(d) Acetylene molecule has three pi bonds and three 74. Assertion : pi bonds are weaker than s bonds.
sigma bonds Reason : pi bonds are formed by the overlapping of p-p
65. The number of sigma (s) and pi (p) bonds present in 1,3,5,7 orbitals along their axes.
octatetraene respectively are (a) Assertion is correct, reason is correct; reason is a
(a) 14 and 3 (b) 17 and 4 correct explanation for assertion.
(b) Assertion is correct, reason is correct; reason is not a
(c) 16 and 5 (d) 15 and 4
correct explanation for assertion.
66. Allyl cyanide molecule contains (c) Assertion is correct, reason is incorrect.
(a) 9 sigma bonds, 4 pi bonds and no lone pair (d) Assertion is incorrect, reason is correct.
(b) 9 sigma bonds, 3 pi bonds and one lone pair 75. Which of the following represents zero overlap of atomic
(c) 8 sigma bonds, 5 pi bonds and one lone pair orbitals.
(d) 8 sigma bonds, 3 pi bonds and two lone pairs
(a) – + + ®z
67. The molecule not having p-bond is pz
s
(a) Cl2 (b) O2
(c) N2 (d) CO2
(b) + – + ®z
68. In hexa-1, 3-diene-5-yne the number of C — C d, C — C p pz
s
and C — H s bonds, respectively are
(a) 5, 4 and 6 (b) 6, 3 and 5
+
(c) 5, 3 and 6 (d) 6, 4 and 5
69. The angle between the overlapping of one s-orbital and (c) + ®z
one p-orbital is – s
(a) 180° (b) 120° (c) 109°28' (d) 120° 60'
70. Linear combination of two hybridized orbitals belonging pz
to two atoms and each having one electron leads to a (d) All of these
(a) sigma bond 76. The strength of bonds formed by s – s and p – p, s – p
(b) double bond overlap in the order of
(c) co-ordinate covalent bond (a) s – p > s – s > p – p (b) p – p > s – s > s – p
(d) pi bond. (c) s – s > p – p > s – p (d) s – s > s – p > p – p
71. Which of the following statements is not correct ? Topic 6: Hybridisation
(a) Double bond is shorter than a single bond
77. As the s-character of hybridised orbital increases, the bond
(b) Sigma bond is weaker than a p (pi) bond angle
(c) Double bond is stronger than a single bond (a) increase (b) decrease
(d) Covalent bond is stronger than hydrogen bond (c) becomes zero (d) does not change
EBD_8350
52 CHEMISTRY

78. The nature of hybridisation in the ammonia molecule is 92. The types of hybridisation of the five carbon atoms from
(a) sp2 (b) dp 2 left to right in the molecule
(c) sp (d) sp3 CH3 — CH == C == CH — CH3 are
79. The shape of sulphate ion is (a) sp3, sp2, sp2, sp2, sp3
(a) square planar (b) triagonal (b) sp3, sp, sp2, sp2, sp3
(c) trigonal planar (d) tetrahedral (c) sp3, sp2, sp, sp2, sp3
80. Which of the following will have sp3d 3 hybridisation? (d) sp3, sp2, sp2, sp, sp3
(a) BrF5 (b) PCl5 93. Pick out the incorrect statement from the following
(c) XeF6 (d) SF6
(a) sp hybrid orbitals are equivalent and are at an angle
81. Which of the following molecule does not have a linear of 180° with each other
arrangement of atoms ?
(b) sp 2 hybrid orbitals are equivalent and bond angle
(a) H2S (b) C2H2
between any two of them is 120°
(c) BeH2 (d) CO2
(c) sp 3d 2 hybrid orbitals are equivalent and are oriented
82. In which one of the following molecules the central atom
towards corners of a regular octahedron
said to adopt sp2 hybridisation?
(a) BeF2 (b) BF3 (d) sp 3d 3 hybrid orbitals are not equivalent
(c) C2H2 (d) NH3 94. Hybridisation and structure of I3– are
83. Considering the state of hybridisation of carbon atoms, (a) sp2 and trigonal planar (b) sp3d 2 and linear
find out the molecule among the following which is linear ? (c) sp3d and linear (d) sp3 and T-shape
(a) CH3– CH = CH–CH3 95. Which one of the following is not correct in respect of
(b) CH3 – C º C – CH3 hybridisation of orbitals?
(c) CH2 = CH – CH2 – C º CH (a) The orbitals present in the valence shell only are
(d) CH3 – CH2 – CH2 – CH3 hybridized.
84. Equilateral shape has (b) The orbitals undergoing hybridization have almost
(a) sp hybridisation (b) sp2 hybridisation equal energy.
3
(c) sp hybridisation (d) None of these (c) Promotion of electron is not essential condition for
85. In which of the following species is the underlined carbon hybridisation.
having sp3 - hybridisation ? (d) Pure atomic orbitals are more effective in forming stable
(a) CH 3 – COOH (b) CH 3 CH 2 OH bonds than hybrid orbitals.
96. Match Column-I (molecule) with Column-II (type of
(c) CH 3 COCH 3 (d) CH 2 = CH - CH 3
hybridisation) and choose the correct option from the
86. An sp3-hybrid orbital contains codes given below.
(a) 25% s-character (b) 75% s-character Column-I Column-II
(c) 50% s-character (d) 25% p-character
(Molecule) (Type of hybridisation)
87. All carbon atoms are sp2 hybridised in
(A) SF6 (p) sp3d
(a) 1, 3-butadiene (b) CH2 = C = CH2
(c) cyclohexane (d) 2-butene (B) PF5 (q) sp3
88. Hybridisation present in ClF3 is (C) BCl3 (r) sp3d 2
(a) sp2 (b) sp3 (D) C2H6 (s) sp2
(c) dsp 2 (d) sp3d (a) A – (r), B – (p), C – (s), D – (q)
89. Hybridisation states of C in CH+3 and CH4 are (b) A – (r), B – (p), C – (q), D – (s)
(a) sp2 & sp3 (b) sp3 & sp2 (c) A – (p), B – (r), C – (q), D – (s)
2
(c) sp & sp 2 (d) sp3 & sp3
(d) A – (p), B – (r), C – (s), D – (q)
90. In which of the following species, all the three types of
hybrid carbons are present? 97. What is the correct mode of hybridisation of the central
atom in the following compounds?
(a) CH2 = C = CH2 (b) CH3 – CH= CH – CH+2
(c) CH3 – C º C – CH2 + (d) CH3 – CH= CH – CH–2 NO2– SF4 PF6–
91. The trigonal bipyramidal geometry is obtained from the (a) sp sp2 sp 3
(b) sp 2 3
sp d sp 3d 2
hybridisation
(a) dsp 3 or sp 3d (b) dsp 2 or sp 2d (c) sp2 sp3 d 2sp 3
(c) d 2sp3 or sp 3d 2 (d) None of these (d) sp 3 sp 3 sp 3d 2
CHEMICAL BONDING AND MOLECULAR STRUCTURE 53

98. The type of hybridisation in xenon atom and the number 106. When two atomic orbitals combine, they form
of lone pairs present on xenon atom in xenon hexafluoride (a) one molecular orbital (b) two molecular orbital
molecule are respectively (c) three molecular orbital (d) four molecular orbital
(a) sp3d 3 , one (b) sp3d 3 , two 107. Paramagnetism is exhibited by molecules
3 3
(c) sp d , two (d) sp3d 2 , zero (a) not attracted into a magnetic field
99. Match List I and List II and pick out correct matching (b) containing only paired electrons
codes from the given choices : [AIIMS 2015, S] (c) carrying a positive charge
List I List II (d) containing unpaired electrons
Compound Structure 108. Which contains atleast one e– in s2p bonding MO?
A. ClF3 1. Square planar (a) O2 (b) B2 (c) C2 (d) Li2
B: PCl5 2. Tetrahedral 109. The given increasing order of energies of various
C. IF5 3. Trigonal bipyramidal molecular orbitals is not true for which of the following
D. CCl4 4. Square pyramidal molecule?
E. XeF4 5. T-shaped s1s < s*1s < s2s < s*2s < (p2px = p2py) < s2pz < (p*2px
Codes = p*2py) < s*2pz
(a) A-5, B-4, C-3, D-2, E-1 (a) B2 (b) C2 (c) N2 (d) O2
(b) A-5, B-3, C-4, D-2, E-1 110. In an anti-bonding molecular orbital, electron density is
(c) A-5, B-3, C-4, D-1, E-2 minimum
(d) A-4, B-3, C-5, D-2, E-1 (a) around one atom of the molecule
100. In an octahedral structure, the pair of d orbitals involved (b) between the two nuclei of the molecule
in d 2sp3 hybridisation is (c) at the region away from the nuclei of the molecule
(a) d x2 - y 2 , d z 2 (b) d xz, d x 2 - y 2 (d) at no place
111. Atomic orbital is monocentric while a molecular orbital is
(c) d 2 d xz (d) d xy, d yz
z , polycentric. What is the meaning of above statement ?
101. If an organic compound contain 92.3% C and 7.7% H, (a) Electron density in atomic orbital is given by the
than number of sp3, sp2 and sp hybridised carbon atoms electron distribution around a nucleus in an atom.
in all possible structures of compound respectively are While in molecular orbital it is given by the electron
(molecular mass = 52 g/mol) distribution around group of nuclei in a molecule.
(a) 1, 2, 5 (b) 0, 4, 4 (b) While an electron in an atomic orbital is influenced
(c) 0, 8, 4 (d) None of these by one nucleus, in a molecular orbital it is influenced
by two or more nuclei depending upon the number
Topic 7: Molecular Orbital Theory of atoms in the molecule.
(c) The electron in an atomic orbital is present in one
102. Molecular orbital theory was given by nucleus while in molecular orbital electrons are
(a) Kossel (b) Mosley present on more than one nuclei depending upon
(c) Mulliken (d) Werner the number of atoms in the molecule.
103. With increasing bond order, stability of bond (d) All of these
(a) Remain unaltered (b) Decreases 112. The difference in energy between the molecular orbital
(c) Increases (d) None of these formed and the combining atomic orbitals is called
104. If Nx is the number of bonding orbitals of an atom and Ny (a) bond energy (b) activation energy
is the number of antibonding orbitals, then the molecule/ (c) stabilization energy (d) destabilization energy
atom will be stable if 113. Which of the following statements is/are not correct for
(a) Nx > Ny (b) Nx = Ny combination of atomic orbitals?
(c) Nx < Ny (d) Nx £ Ny (i) The combining atomic orbitals must have the same
105. Assertion : The bond order of helium is always zero. or nearly the same energy.
Reason : The number of electrons in bonding molecular (ii) Greater the extent of overlap, the greater will be the
orbital and antibonding molecular orbital is equal. electron density between the nuclei of a moleculer
(a) Assertion is correct, reason is correct; reason is a orbital.
correct explanation for assertion. (iii) 2pz orbital of one atom can combine with either of
(b) Assertion is correct, reason is correct; reason is not a 2px, 2py or 2pz orbital of other atom as these orbitals
correct explanation for assertion. have same energy.
(c) Assertion is correct, reason is incorrect. (a) (i) and (ii) (b) (iii) only
(d) Assertion is incorrect, reason is correct. (c) (i) only (d) (ii) and (iii)
EBD_8350
54 CHEMISTRY

114. Assertion : Bonding molecular orbital has greater stability 123. Which of the following is not correct with respect to bond
than corresponding antibonding molecular orbital. length of the species ?
Reason : The electron density in a bonding molecular (a) C2 > C2– 2 (b) B+2 > B2
orbital is located away from the space between the (c) Li+2 > Li2 (d) O2 > O–2
nuclei while in antibonding molecular orbital it is located 124. In O2– , O2 and O22– molecular species, the total number of
between the nuclei of the bonded atoms. antibonding electrons respectively are
(a) Assertion is correct, reason is correct; reason is a (a) 7, 6, 8 (b) 1, 0, 2
correct explanation for assertion.
(c) 6, 6, 6 (d) 8, 6, 8
(b) Assertion is correct, reason is correct; reason is not a
125. According to MO theory which of the following lists ranks
correct explanation for assertion.
the nitrogen species in terms of increasing bond order?
(c) Assertion is correct, reason is incorrect. – –
(d) Assertion is incorrect, reason is correct. (a) N2–
2 < N2 < N2 (b) N2 < N2–
2 < N2

115. The compound which cannot be formed is (c) N2– < N2–
2 < N2 (d) N2– < N2 < N2–
2
(a) He (b) He+ 126. The correct statement with regard to H+2 and H2– is
(c) He2 (d) He+2 (a) both H2+ and H–2 are equally stable
Topic 8: Bonding in Some Homonuclear Diatomic Molecules (b) both H2+ and H2– do not exist
(c) H2– is more stable than H+2
116. Which molecule has the highest bond order?
(d) H2+ is more stable than H2-
(a) N2 (b) Li2
(c) He2 (d) O2 127. Mark the incorrect statement in the following
(a) the bond order in the species O2, O+2 and O–2 decreases
117. Which of the following molecule exist ?
as O+2 > O2 > O–2
(a) He2 (b) Be2
(b) the bond energy in a diatomic molecule always
(c) Li2 (d) Both (a) and (b) increases when an electron is lost
118. The ground state electronic configuration of valence shell (c) electrons in antibonding M.O. contribute to repulsion
electrons in nitrogen molecule (N 2 ) is written as between two atoms.
KK s2 s 2 , s * 2s 2 , p2 p 2x , p 2 p 2y s 2 p 2z . Bond order in (d) with increase in bond order, bond length decreases
nitrogen molecule is and bond strength increases.
(a) 0 (b) 1 128. Bond order normally gives idea of stability of a molecular
(c) 2 (d) 3 species. All the molecules viz. H2, Li2 and B2 have the
119. The bond order in N2 is + same bond order yet they are not equally stable. Their
(a) 1.5 (b) 3.0 stability order is
(c) 2.5 (d) 2.0 (a) H2 > B2 > Li2 (b) H2 > Li2 > B2
120. Which one of the following molecules is expected to exhibit (c) Li2 > B2 > H2 (d) B2 > H2 > Li2
diamagnetic behaviour ? Topic 9: Hydrogen Bonding
(a) C2 (b) N2
(c) O2 (d) S2 129. In which of the following state of compound, the
magnitude of H-bonding will be maximum and in which
121. According to molecular orbital theory, which of the
case it will be minimum ?
following statement about the magnetic character and bond
(a) Maximum = Solid, Minimum = Liquid
order is correct regarding O +2
(b) Maximum = Liquid, Minimum = Gas
(a) Paramagnetic and Bond order < O2 (c) Maximum = Solid, Minimum = Gas
(b) Paramagnetic and Bond order > O2 (d) Maximum = Gas, Minimum = Solid
(c) Diamagnetic and Bond order < O2 130. Which of the following are correctly classified ?
(d) Diamagnetic and Bond order > O2 Intermolecular Intramolecular
122. Arrange the following in increasing order of bond length H-bonding H-bonding
(i) N2 (ii) N+2 (a) HF H2O
(iii) N22+ (b) CH3OH HF
(a) (ii), (i) and (iii) (b) (ii), (iii) and (i) (c) H2O o-nitrophenol
(d) HF p-nitrophenol
(c) (iii), (ii) and (i) (d) (i), (ii) and (iii)
CHEMICAL BONDING AND MOLECULAR STRUCTURE 55

131. The boiling points at atmospheric pressure of HF, H2S, 140. The boiling point of p-nitrophenol is higher than that of
NH3 can be arranged in the following order: o-nitrophenol because
(a) HF > NH3 > H2S (a) NO2 group at p-position behave in a different way
(b) HF > H2S > NH3 from that at o-position.
(c) HF < H2S < NH3 (b) intramolecular hydrogen bonding exists in
(d) HF < NH3 < H2S p-nitrophenol.
132. Methanol and ethanol are miscible in water due to (c) there is intermolecular hydrogen bonding in
p-nitrophenol.
(a) covalent character
(d) p-nitrophenol has a higher molecular weight than
(b) hydrogen bonding character
o-nitrophenol.
(c) oxygen bonding character
141. Which one of the following is the correct order of
(d) None of these interactions?
133. The hydrogen bond is shortest in (a) Covalent < hydrogen bonding < van der Waals <
(a) S — H --- S (b) N — H --- O dipole-dipole.
(c) S — H --- O (d) F — H --- F (b) van der Waals < hydrogen bonding < dipole <
134. Hydrogen bonding is maximum in covalent.
(a) C2H5OH (b) CH3OCH3 (c) van der Waals < dipole-dipole < hydrogen bonding
(c) (CH3)2 C = O (d) CH3CHO < covalent.
135. Intramolecular hydrogen bond exists in (d) Dipole-dipole < van der Waals < hydrogen bonding
(a) ortho nitrophenol (b) ethyl alcohol < covalent.
(c) water (d) diethyl ether 142. Assertion : Water is one of the best solvent.
OH Reason : H-bonding is present in water molecules.
(a) Assertion is correct, reason is correct; reason is a
136. The vapour pressure of is higher than
correct explanation for assertion.
NO 2
OH (b) Assertion is correct, reason is correct; reason is not a
due to correct explanation for assertion.
(c) Assertion is correct, reason is incorrect.
O 2N
(d) Assertion is incorrect, reason is correct.
(a) dipole moment
143. Acetic acid exists as dimer in benzene due to
(b) dipole-dipole interaction
(a) condensation reaction
(c) H-bonding
(b) hydrogen bonding
(d) lattice structure
(c) presence of carboxyl group
137. The reason for exceptionally high boiling point of water is
(d) presence of hydrogen atom at a-carbon
(a) its high specific heat
144. Maximum number of H-bonds that can be formed by a
(b) its high dielectric constant water molecule is
(c) low ionization of water molecule (a) 2 (b) 3
(d) hydrogen bonding in the molecules of water (c) 4 (d) 6
138. Hydrogen bonding is formed in compounds containing 145. Which among the following can form intermolecular
hydrogen and H – bonding ?
(a) highly electronegative atoms
O
(b) highly electropositive atoms
(c) metal atoms with d-orbitals occupied H C CHO CHO NO 2
(d) metalloids OH
139. The low density of ice compared to water is due to
(a) induced dipole-induced dipole interactions OH
(b) dipole-induced dipole interactions OH OH
(c) hydrogen bonding interactions (A) (B) (C) (D)
(d) dipole-dipole interactions (a) A (b) B and D
(c) B, C and D (d) A and C
EBD_8350
56 CHEMISTRY

1. A diatomic molecule has a dipole moment of 1.2 D, if the total number of lone pair at central atom in compound
bond distance is 1 Å, what percentage of electronic charge (i), (ii) and (iii) respectively.
exists on each atom? 7. Calculate the value of “x + y – z” here x, y and z are total
2. The dipole moments of a diatomic molecule AB is 0.41D number of non-bonded electron pair(s), pie(p) bond(s)
and bond distance is 2.82, calculate the % ionic character and sigma (s) bonds in h ydrogen phosphite ion
of AB.
respectively.
3. The bond angle between two hybrid orbitals is 105°.
8. Consider the following compounds:
Calculate the percentage of s-character of hybrid orbital.
(i) IF5 (ii) ClI–4
4. In O-2 , O2 and O 22 - molecular species. Find the total (iii) XeO2F2 (iv) NH–2
number of antibonding electrons. (v) BCl3 (vi) BeCl2
5. There are two groups of compounds A and B. Group A (vii) AsCl+4 (viii) B(OH)3
contains three compounds Px4, Qy3, Rz2. Group B also contains (ix) NO2– (x) ClO2+
three compounds Sx4, Ty3, Uz2. Hybridization of each central The value of “x + y – z” is
atom of group A compounds is same as that of iodine in IBrCl– where x, y and z are total number of compounds in given
while in group B compounds it is same as that of iodine in compounds in which central atom used their all three p-
IBrCl+. Substituents X, Y and Z exhibit covalency of one in orbitals, only two p-orbitals and only one p-orbital in
ground state. Then find the value of x/y. hybridisation respectively.
Where, x and y are total number of lone pairs present at
9. Total number of species which used all three p-orbitals in
central atoms of compounds of group A and B respectively.
hybridisation of central atoms and should be non-polar
6. Consider the following three compounds (i) AX 2nn- , also are
+
(ii) AX3n and (iii) AX 4nn+ , where central atom A is 15th group XeO2F2, SnCl2, IF5, I3+ , XeO4, SO2, XeF7 , SeF4
element and their maximum covalency is 3n. If total number 10. Consider the following orbitals 3s, 2px, 4dxy, 4 d z2 , 3d 2 2 ,
of proton in surrounding atom X is n and value of n is one, x -y
3py, 4s, 4pz and find total number of orbital(s) having even
then calculate value of "x3 + y2 + z". (Where x, y and z are number of nodal plane.

Exercise 3 : NCERT Exemplar & Past Year MCQs


NCERT Exemplar MCQs 5. In which of the following substances will hydrogen bond
1. Polarity in a molecule and hence the dipole moment be strongest?
depends primarily on electronegativity of the constituent (a) HCl (b) H2O (c) HI (d) H2S
atoms and shape of a molecule. Which of the following 6. If the electronic configuration of an element is
has the highest dipole moment? 1s2 2s2 2p6 3s2 3p6 3d 2 4s2, the four electrons involved in
(a) CO2 (b) HI (c) H2O (d) SO2 chemical bond formation will be
2. In PO34- ion the formal charge on the oxygen atom of P – (a) 3p6 (b) 3p6, 4s2
6 2
O bond is (c) 3p , 3d (d) 3d 2, 4s2
(a) + 1 (b) – 1 (c) – 0.75 (d) + 0.75 7. Which of the following angle corresponds to sp 2
3. Which of the following species has tetrahedral geometry? hybridisation?
(a) BH -4 (b) NH -2 (c) CO32 - (d) H3 O+ (a) 90° (b) 120°
4. Number of p bonds and s bonds in the following structure is (c) 180° (d) 109°
H H
Direction (Q. no. 8-11) The electronic configurations of the
H H elements A, B and C are given below. Answer the questions
from 14 to 17 on the basis of these configurations.
H
A 1s2 2s2 2p6
H
B 1s2 2s2 2p6 3s2 3p3
H H 2 2 6 2
C 1s 2s 2p 3s 3p5
(a) 6, 19 (b) 4, 20 (c) 5, 19 (d) 5, 20
CHEMICAL BONDING AND MOLECULAR STRUCTURE 57

8. Stable form of A may be represented by the formula 21. Which of the following statement is not correct from the
(a) A (b) A 2 (c) A 3 (d) A 4 view point of molecular orbital theory?
9. Stable form of C may be represented by the formula (a) Be2 is not a stable molecule.
(a) C (b) C2 (c) C3 (d) C4 (b) He2 is not stable but He+2 is expected to exist.
10. The molecular formula of the compound formed from B (c) Bond strength of N2 is maximum amongst the
and C will be homonuclear diatomic molecules belonging to the
(a) BC (b) B2C (c) BC2 (d) BC3 second period.
11. The bond between B and C will be (d) The order of energies of molecular orbitals in N2
(a) ionic (b) covalent molecule is
(c) hydrogen (d) coordinate s2s < s*2s < s2pz < (p2px ; p2py) < (p*2px ; p*2py )
12. The electronic configuration of the outer most shell of the < s*2pz
most electronegative element is
22. Which of the following options represents the correct
(a) 2s22p5 (b) 3s23p5 (c) 4s24p5 (d) 5s25p5
bond order?
13. Amongst the following elements whose electronic
configuration are given below, the one having the highest (a) O -2 > O 2 > O 2+ (b) O -2 < O 2 < O2+
ionisation enthalpy is (c) O -2 > O 2 < O 2+ (d) O -2 < O 2 > O 2+
(a) [Ne]3s23p1 (b) [Ne]3s23p3
(c) [Ne]3s23p2 (d) [Ar]3d104s24p3 Past Year MCQs
2+
14. Isostructural species are those which have the same shape 23. Be is isoelectronic with which of the following ions?
and hybridisation. Among the given species identify the [AIPMT 2014, A]
+ + +
isostructural pairs. (a) H (b) Li (c) Na (d) Mg2+

(a) [NF3 and BF3] (b) [BF4 and NH+4] 24. Which of the following molecules has the maximum dipole

(c) [BCl3 and BrCl3] (d) [NH3 and NO3 ] moment ? [AIPMT 2014, A]

15. The types of hybrid orbitals of nitrogen in NO2+, NO3 and (a) CO2 (b) CH4 (c) NH3 (d) NF3
NH+4 respectively are expected to be 25. Which one of the following species has plane triangular
(a) sp, sp3 and sp2 (b) sp, sp2 and sp3 shape ? [AIPMT 2014, A]
2 3 (a) N3– (b) NO3–
(c) sp , sp and sp (d) sp2, sp3 and sp
(c) NO2– (d) CO2
16. Hydrogen bonds are formed in many compounds e.g.,
26. Which one of the following properties is not shown by
H2O, HF, NH3. The boiling point of such compounds
NO? [JEE M 2014, A]
depends to an extent on the strength of hydrogen bond
(a) It is diamagnetic in gaseous state
and the number of hydrogen bonds. The correct decreasing
order of the boiling points of above compounds is (b) It is neutral oxide
(c) It combines with oxygen to form nitrogen dioxide
(a) HF > H2O > NH3 (b) H2O > HF > NH3
(d) It’s bond order is 2.5
(c) NH3 > HF > H2O (d) NH3 > H2O > HF

27. For which of the following molecule significant m ¹ 0?
17. In NO3 ion, the number of bond pairs and lone pairs of [JEE M 2014, A]
electrons on nitrogen atom are
(a) 2, 2 (b) 3, 1 (c) 1, 3 (d) 4, 0 Cl CN
18. Which molecule/ion out of the following does not contain
unpaired electrons? (i) (ii)
(a) N+2 (b) O2 (c) O2–
2
(d) B2
19. In which of the following molecule/ion all the bonds are Cl CN
not equal?
– OH SH
(a) XeF4 (b) BF 4 (c) C2H4 (d) SiF4
20. Which of the following order of energies of molecular
orbitals of N2 is correct? (iii) (iv)
(a) (p2py ) < (s2pz ) < (p*2px ) » (p*2py )
(b) (p2py ) > (s2pz ) > (p*2px ) » (p*2py ) OH SH
(c) (p2py ) < (s2pz) < (p*2px) » (p*2py ) (a) Only (i) (b) (i) and (ii)
(c) Only (iii) (d) (iii) and (iv)
(d) (p2py ) > (s2pz) < (p*2px) » (p*2py )
EBD_8350
58 CHEMISTRY

28. The correct bond order in the following species is: 38. Which of the following pairs of compounds is isoelectronic
[AIPMT 2015, A] and isostructural ? [NEET 2017, A]
-
(a) O 22+ < O2– < O 2+ (b) O +2 < O 2– < O 22+ (a) TeI2, XeF2 (b) IBr2 , XeF2
(c) IF3, XeF2 (d) BeCl2, XeF2
(c) O 2– < O 2+ < O 22+ (d) O 22+ < O 2+ < O 2– 39. The species, having bond angles of 120° is :-
29. Which of the following pairs of ions are isoelectronic and [NEET 2017, S]
isostructural ? [AIPMT 2015, A] (a) ClF3 (b) NCl3 (c) BCl3 (d) PH3
(a) ClO3– , CO32– (b) SO32– , NO3– 40. Which of the following species is not paramagnetic ?
[JEE M 2017, S]
(c) ClO3– , SO32– (d) CO32– , SO32– (a) NO (b) CO (c) O2 (d) B2
30. Maximum bond angle at nitrogen is present in which of the 41. In the structure of ClF3, the number of lone pair of electrons
following? [AIPMT 2015, S] on central atom ‘Cl’ is [NEET 2018, S]
(a) One (b) Two (c) Three (d) Four
(a) NO 2– (b) NO +2 (c) NO3– (d) NO 2
42. Consider the following species : [NEET 2018, A]
31. Which of the following species contains equal number of + –
CN , CN , NO and CN
s- and p-bonds : [AIPMT 2015, S]
(a) XeO4 (b) (CN)2 Which one of these will have the highest bond order?
(c) CH2(CN)2 (d) HCO–3 (a) NO (b) CN– (c) CN (d) CN+
43. Total number of lone pair of electrons in I3– ion is :
32. Decreasing order of stability of O2, O–2, O+2 and O22– is :
[AIPMT 2015, A] [JEE M 2018, A]
(a) O+2 > O2 > O–2 > O22– (b) O22– > O–2 > O2 > O+2 (a) 3 (b) 6 (c) 9 (d) 12
44. According to molecular orbital theory, which of the
(c) O2 > O+2 > O22– > O–2 (d) O–2 > O22– > O+2 > O2
following will not be a viable molecule? [JEE M 2018, A]
33. In which of the following pairs, both the species are not
isostructural ? [AIPMT 2015, A] (a) He22+ (b) He+2 (c) H2– (d) H 2–
2

(a) SiCl4 , PCl+ 4 (b) diamond, silicon carbide 45. Which of the following compounds contain(s) no covalent
bond(s)? [JEE M 2018, S]
(c) NH3, PH3 (d) XeF4, XeO4
KCl, PH3, O2, B2H6, H2SO4
34. Consider the molecules CH4, NH3 and H2O. Which of the
given statements is false? [NEET 2016, C] (a) KCl, B2H6, PH3 (b) KCl, H2SO4
(a) The H–C–H bond angle in CH4, the H–N–H bond angle (c) KCl (d) KCl, B2H6
in NH3, and the H–O–H bond angle in H2O are all 46. The number of sigma (s) and pi (p) bonds in pent-2-en-4-
greater than 90° yne is: [NEET 2019, C]
(b) The H–O–H bond angle in H2O is larger than the (a) 10 bonds and 3 bonds (b) 8 bonds and 5 bonds
H–C–H bond angle in CH4. (c) 11 bonds and 2 bonds (d) 13 bonds and no bond
(c) The H–O–H bond angle in H2O is smaller than the 47. The correct structure of tribromooctaoxide is:
H–N–H bond angle in NH3.
[NEET 2019, C]
(d) The H–C–H bond angle in CH4 is larger than the
H–N–H bond angle in NH3. O O O
35. Predict the correct order of repulsion between electrons (a) O Br Br Br O
among the following: [NEET 2016, C]
(a) lone pair- lone pair > lone pair - bond pair > bond pair O O O
- bond pair
O O O
(b) lone pair - lone pair > bond pair - bond pair > lone pair
- bond pair (b) O Br Br Br O–
(c) bond pair - bond pair > lone pair - bond pair > lone
pair - lone pair O O– O–
(d) lone pair - bond pair > bond pair - bond pair > lone O
pair - lone pair O O–
36. The species in which the N atom is in a state of sp –
O Br Br Br O
hybridization is : [JEE M 2016, S] (c)
(a) NO3– (b) NO2 (c) NO+2 (d) NO2–
–O
O O–
37. Which of the following pairs of species have the same O O– O
bond order ? [NEET 2017, S]
(a) O2, NO+ (b) CN–, CO (d) O Br Br Br O–
(c) N 2 , O 2- (d) CO, NO O O
O–
CHEMICAL BONDING AND MOLECULAR STRUCTURE 59

48. The manganate and permanganate ions are tetrahedral, 52. Which of the following set of molecules will have zero
due to: [NEET 2019, A] dipole moment? [NEET 2020, C]
(a) The p-bonding involves overlap of p-orbitals of (a) Boron trifluoride, hydrogen fluoride, carbon dioxide,
oxygen with d-orbitals of manganese 1,3-dichlorobenzene
(b) There is no p-bonding (b) Nitrogen trifluoride, beryllium difluoride, water, 1,3-
(c) The p-bonding involves overlap of p-orbital of oxygen dichlorobenzene
with p-orbitals of manganese
(c) Boron trifluoride, beryllium difluoride, carbon dioxide,
(d) The p-bonding involves overlap of d-orbital of oxygen
1,4-dichlorobenzene
with d-orbitals of manganese
49. Which of the following diatomic molecular species has (d) Ammonia, beryllium difluoride, water, 1,4-
only bonds according to Molecular Orbital Theory ? dichlorobenzene
[NEET 2019, C] 53. Identify a molecule which does not exist.[NEET 2020, A]
(a) O2 (b) N2 (c) C2 (d) Be2 (a) Li2 (b) C2
50. Among the following, the molecule expected to be stabilized (c) O2 (d) He2
by anion formation is: [JEE M 2019, A] 54. The dipole moments of CCl4, CHCl3 and CH4 are in the
order: [JEE M 2020, A]
C2, O2, NO, F2
(a) CHCl3 < CH4 = CCl4 (b) CCl4 < CH4 < CHCl3
(a) C2 (b) F2 (c) NO (d) O2 (c) CH4 < CCl4 < CHCl3 (d) CH4 = CCl4 < CHCl3
51. According to molecular orbital theory, which of the 55. The relative strength of interionic/intermolecular forces in
following is true with respect to Li+2 and Li2– ? decreasing order is: [JEE M 2020, C]
[JEE M 2019, A] (a) dipole-dipole > ion-dipole > ion-ion
(a) Li2+ is unstable and Li2– is stable (b) ion-dipole > ion-ion > dipole-dipole
(b) Li2+ is stable and Li2– is unstable (c) ion-dipole > dipole-dipole > ion-ion
(c) Both are stable (d) ion-ion > ion-dipole > dipole-dipole
(d) Both are unstable

Exercise 4 : Problem Solving Skill Enhancer MCQs


1. Which of the following hydrocarbons has the lowest dipole 6. Which of the following ions does not involve pp – dp
moment ? bonding?
H3C CH3 (a) SO 32 - (b) PO 34- (c) NO 3- (d) XeOF4
(a) C (b) CH 3C º CCH 3 7. Which of the following is most appropriate structure of
H H C3O2?
(c) CH 3CH 2 C º CH (d) CH 2 = CH – C º CH (a) O = C = O ® C = O (b) O ® O = C = C = C
2. The covalent alkaline earth metal halide (X = Cl, Br, I) is : (c) C = O® C = C =O (d) O = C = C = C = O
(a) MgX2 (b) CaX2 (c) BeX2 (d) SrX2
8. Although CN– ion and N2 molecule are isoelectronic, yet
3. In which of the following species maximum atom can lie in N2 molecule is chemically inert because of
same plane? (a) presence of more number of electrons in bonding orbitals
(b) less bond energy
(a) XeF2O2 (b) PCl5 (c) AsH +4 (d) XeF4 (c) absence of bond polarity
4. In which of the following ionization processes, the bond (d) uneven electron distribution.
energy has increased and also the magnetic behaviour 9. Read the following statements and choose the correct
has changed from paramagnetic to diamagnetic ? option.
(a) NO ® NO + (b) N 2 ® N 2 +
(i) P—O bond length order in : PO3– 2– –
4 > HPO3 > H2PO2
(c) C 2 ® C 2 + (d) O2 ® O2 +
5. In forming (i) N 2 ® N +2 and (ii) O2 ® O+2 ; the electrons q1 q2
respectively are removed from : (ii) q2 > q1
(a) (p*2py or p*2px) and (p*2py or p*2px)
(b) (p 2py or p 2px) and (p2py or p2px) (iii) Between C2H2 and C2H4, C—H is stronger in C2H2.
(c) (p2py or p2px) and (p*2py or p*2px) (a) (i) (b) (i), (ii) & (iii)
(d) (p*2py or p*2px) and (p2py or p2px) (c) (i) & (ii) (d) (ii) & (iii)
EBD_8350
60 CHEMISTRY

10. Which of the following species contains minimum number *


*
of atoms in ‘XY’ plane? (a) H 2O + C O2 (b) H3 BO3 + OH –
(a) XeF5– (b) SF6 *
(c) IF7 (d) All (c) BF3 + N H3 (d) None of these
13. CrO2Cl2 has
11. Which of the following orbital combination can not form
(a) tetragonal structure
p-bond ?
(b) distorted tetrahedral structure
(a) px + px sideways overlapping
(c) square pyramidal structure
(b) d + p y sideways overlapping (d) octahedral structure
x2 - y 2
14. The relative stabilities of CN, CN+ and CN– are in the order
(c) dxy + dxy sideways overlapping
(a) CN > CN+ > CN– (b) CN– > CN > CN+
(d) dyz + py sideways overlapping –
(c) CN > CN > CN+ (d) CN+ > CN > CN–
12. In which of the following combination hybridisation of 15. Select the most stable cation
central atom (*) does not change? (a) NH +4 (b) CH 5+ (c) H3 O + (d) FH +2

ANSW ER KEY
Exercis e 1 : NCERT Based Topic-wis e MCQs
1 (d) 16 (d) 31 (a) 51 (d) 61 (c) 76 (d) 91 (a) 106 (b) 121 (b) 136 (c)
2 (b) 17 (c) 32 (b) 47 (a) 62 (a) 77 (a) 92 (c) 107 (d) 122 (d) 137 (d)
3 (b) 18 (b) 33 (d) 48 (d) 63 (a) 78 (d) 93 (d) 108 (a) 123 (d) 138 (a)
4 (b) 19 (a) 34 (c) 49 (a) 64 (d) 79 (d) 94 (c) 109 (d) 124 (a) 139 (c)
5 (d) 20 (b) 35 (d) 50 (a) 65 (b) 80 (c) 95 (d) 110 (b) 125 (a) 140 (c)
6 (a) 21 (b) 36 (b) 51 (d) 66 (b) 81 (a) 96 (a) 111 (b) 126 (d) 141 (c)
7 (a) 22 (c) 37 (c) 52 (d) 67 (a) 82 (b) 97 (b) 112 (c) 127 (b) 142 (b)
8 (b) 23 (d) 38 (a) 53 (b) 68 (a) 83 (b) 98 (a) 113 (b) 128 (b) 143 (b)
9 (a) 24 (c) 39 (b) 54 (b) 69 (a) 84 (b) 99 (b) 114 (c) 129 (c) 144 (c)
10 (a) 25 (c) 40 (c) 55 (d) 70 (a) 85 (b) 100 (a) 115 (c) 130 (c) 145 (c)
11 (c) 26 (a) 41 (a) 56 (a) 71 (b) 86 (a) 101 (c) 116 (a) 131 (a)
12 (a) 27 (c) 42 (c) 57 (b) 72 (b) 87 (a) 102 (c) 117 (c) 132 (b)
13 (d) 28 (d) 43 (b) 58 (b) 73 (a) 88 (d) 103 (c) 118 (d) 133 (d)
14 (a) 29 (c) 44 (d) 59 (b) 74 (c) 89 (a) 104 (a) 119 (c) 134 (a)
15 (c) 30 (d) 45 (c) 60 (a) 75 (c) 90 (c) 105 (a) 120 (a,b) 135 (a)
Exercis e 2 : Numeric/Integer Answer Questions
1 (25) 2 (76.94) 3 (21.44) 4 (21) 5 (2) 6 (9) 7 (3) 8 (8) 9 (2) 10 (2)
Exercise 3 : NCERT Exemplar & Past Year MCQs
1 (c) 7 (b) 13 (b) 19 (c) 25 (b) 31 (a) 37 (b) 43 (c) 49 (c) 55 (d)
2 (c) 8 (a) 14 (b) 20 (a) 26 (a) 32 (a) 38 (b) 44 (d) 50 (a)
3 (a) 9 (b) 15 (b) 21 (d) 27 (d) 33 (d) 39 (c) 45 (c) 51 (c)
4 (c) 10 (d) 16 (b) 22 (b) 28 (c) 34 (b) 40 (b) 46 (a) 52 (c)
5 (b) 11 (b) 17 (d) 23 (b) 29 (c) 35 (a) 41 (b) 47 (a) 53 (d)
6 (d) 12 (a) 18 (c) 24 (c) 30 (b) 36 (c) 42 (b) 48 (a) 54 (d)
Exercis e 4 : Problem Solving Skill Enhancer MCQs
1 (b) 3 (d) 5 (c) 7 (d) 9 (b) 11 (b) 13 (b) 15 (a)
2 (c) 4 (a) 6 (c) 8 (c) 10 (b) 12 (c) 14 (b)
5 States of Matter

Trend Buster NEET & JEE Main

Number of Questions from 2020-15 6 5 Minimum one question has been asked
Weightage 1.9% 2.8% every year in both NEET & JEE Main.

The most Important Concepts that Cover Maximum number of Questions asked in past 6 years.

Ideal gas law 1 2


Graham's law of diffusion / Dalton's law of 2 —
partial pressure
Real gas equation 1 —
Liquefaction of gas 1 —
Compressibility factor 1 —
Critical temperature — 1
Less Important Concepts that Cover 1 or 2 Questions asked in past 6 years.

Hydrogen bonding — 1
Relation between interatomic potential and — 1
interatomic distance
NEET JEE

2020 The gaseous state and Dalton's law of partial 1 Average — —


Gas laws pressure
2019 Behavieor of real gases / Critical temperature / 1 Average 2 Average /
Ideal gas equation Compressibility factor / Difficult
Ideal gas law
2018 Behavieor of real gases Real gas equation / 2 Easy — —
Liquefaction of gases
2017 — — — — — —
2016 Ideal gas equation Ideal gas law / Graham's 1 Average 1 Difficult
law of diffusion
2015 Behavieor of real gases / Relation between interatomic 1 Easy 2 Difficult /
Intermolecular forces / potential and interatomic Easy
Ideal gas equation distance / Hydrogen bonding/
Ideal gas law
EBD_8350
62 CHEMISTRY
STATES OF MATTER 63
EBD_8350
64 CHEMISTRY

Problem Solving Tips/ Tricks/ Points to Remember

4 Standard conditions for temperature and pressure : 4 Density and molar mass of a gas: According to the ideal
Condition T P Vm gas equation PV = nRT,
S.T.P. 273.15 K 1 bar 22.71 L PV
1 1 = P2V2
N.T.P. 293.15 K 1 atm 24.055 L T1 T2
S.A.T.P. 298.15 K 1 bar 24.789 L In terms of density, we get
old S.T.P. 273.15 K 1 atm 22.4 L m RT m RT RT
P= = =d
Mw V V Mw Mw
4 A substance in the gaseous state below Tc is called
vapour and above Tc is called gas. RT
or Mw = d where d is density
4 Fraction of molecules possessing a particular velocity at P
particular temperature is constant. 4 STP (Standard temperature and pressure) or NTP (normal
4 With increase in temperature, most probable velocity temperature and pressure) conditions are T = 0 °C
increases. = 273.15 K, P = 1 atm
4 (i) Cp – Cv = R = 2 cal = 8.314 J or T = 0 °C = 273.15 K, P = 1 bar
Cp (i) Volume of a gast at STP (P = 1 atm)
3
(ii) = R = 22.413996 L mol–1 = 22.4 L mol–1
Cv 2
(ii) Volume of a gas at STP (P = 1 bar)
(iii) For monoatomic gas, Cv = 3 calories = 22.71098 L mol–1 = 22.7 L mol–1
C 3 4 1 atm = 76 cm = 760 mm = 760 torr = 101325 Pa or N m–2
(iv) For monoatomic gases, p = = 1.66
Cv 5 = 1.01325 ´ 10–5 Pa or N m–2 = 105 Pa or N m–2
C 1 atm = 1.013 bar
(v) For diatomic gases, p = 1.40
Cv 1 bar = 0.987 atm = 102 K Pa
æ 5 7 ö 1 atm = 0.06805 psi
çè Cv = R, Cp = R ÷ø 1 Nm–2 = 6894.8 psi
3 2
Cp 4 According to Graham’s Law of diffusion, under similar
(vi) For Triatomic gases = 1.33
Cv conditions of temperature and pressure, if r1 and r2 are
rates of diffusion of two gases with densities d1 and d2
4 Viscosity then
(i) Effect of temperature on viscosity: Viscosity Mw2
decreases with increase in temperature. The r1 d2
= =
relationship between coefficient of viscosity, h r2 d1 Mw1
and absolute temperature T is h = Ae Ea / RT æ P ö
4 For gases at different pressures, ç r µ ÷
where A and Ea are constants for a given liquid Mw ø
è
(ii) Effect of pressure on viscosity: Viscosity increases r1 P1 Mw1
with increase in pressure. =
r2 P2 Mw
4 Collision frequency : The total number of collisions 2
æ T ö
occurring in a unit volume of a gas per second under 4 For gases at different temperatures, ç r µ
given set of conditions. è Mw ÷ø
z µ T2/3 (at constant P) r1 T1Mw
= 1
z µ P2 (at constant T) r2 T2 Mw
1
4 Joule-Thomson effect : When a compressed gas is 2 RT
allowed to expand through a small orifice, cooling effect 4 Most probable velocity, a =
Mw
is caused and temperature falls. This is known as Joule
Thomson effect which is observed only if the gas below 8RT
certain temperature called inversion temperature. 4 Average velocity, n =
p Mw
4 Greater the value of ‘a’ more easily the gas is liquifiable 3RT 3RT 3P
PV 2 4 RMS velocity, u = = =
a= 2 Mw Mw d
n
STATES OF MATTER 65

4 A gas can be liquefied by cooling or by applying pressure 4 Graham's law of diffusion : Undersimilar conditions if r1
or by the combined effect of both. However, the effect of and r2 are the rate of diffusion, d1 and d2 are the densities
temperature is more important because for every gas there and M1 and M2 are the molecular masses of two gases
is a particular temperature above which it cannot be respectively, then,
liquefied howsoever high pressure is applied.
r1 M2 d2
8a = =
Critical temperature : Tc = r2 M1 d1
27 Rb
a 4 Units of pressure :
4 Critical pressure : Pc = Units Symbol Value
27b2
4 Critical volume (Vc) : Vc = 3b Pascal Pa S.I. unit
4 Critical compressibility factor: Bar bar 1 bar = 105 pa
Atmosphere atm 1 atm = 1.01 × 105 pa
P V 3
Zc = c c = millmetre of mercury mm Hg 760 mm Hg = 1 atm
RTc 8
torr Torr 1 Torr = 1 mm Hg
EBD_8350
66 CHEMISTRY

Exercise 1 : NCERT Based Topic-wise MCQs


(iii) Hydrogen bonds are powerful force in determining
Topic 1: Intermolecular Forces
the structure and properties of compounds like
1. Which of the following is not a type of van der Waal's proteins, nucleic acids etc.
forces? (iv) Strength of the hydrogen bond is determined by the
(a) Dipole - dipole forces coulombic interaction between the lone-pair electrons
(b) Dipole - induced dipole forces of the electronegative atom of one molecule and the
(c) Ion - dipole forces hydrogen atom of other molecule.
(d) London forces (a) (i) and (ii) (b) (i), (ii) and (iii)
2. Who proposed the concept of dispersion force ? (c) (ii), (iii) and (iv) (d) All of these
(a) Heitler and London (b) van der Waal 9. Assertion : Gases expand and occupy all the space
(c) Gay Lussac (d) Fritz London available to them
3. Dipole-dipole interaction is stronger than the London Reason : There is no force of attraction between the
forces but is weaker than ion-ion interaction because particles of a gas at ordinary temperature and pressure.
(a) only partial charges are involved (a) Assertion is correct, reason is correct; reason is a
(b) only total charges are involved correct explanation for assertion.
(c) both (a) and (b) (b) Assertion is correct, reason is correct; reason is not
(d) sometimes (a) and sometimes (b) a correct explanation for assertion
4. Which of the following exhibits the weakest intermolecular (c) Assertion is correct, reason is incorrect
forces ? (d) Assertion is incorrect, reason is correct.
(a) NH3 (b) HCl (c) He (d) H2O 10. Arrange the following in increasing order of intermolecular
5. Which of the following statements are correct regarding interaction
the characteristic of gases ? (A) HCl (B) SF6 and (C) NaCl
(i) Gases are highly compressible. (a) A, B, C (b) A, C, B (c) B, A, C (d) B, C, A
(ii) Gases exert pressure equally in all directions.
(iii) Gases have much higher density than the solids and Topic 2: Intermolecular Forces Vs Thermal Interactions
liquids. 11. Which of the following statements regarding thermal
(iv) Gases mix evenly and completely in all proportion energy is correct ?
without any mechanical aid. (a) Thermal energy is the measure of average kinetic
Choose the correct option. energy of the particles of the matter and is thus
(a) (i), (ii) and (iii) (b) (ii), (iii) and (iv) responsible for movement of particles.
(c) (i), (ii) and (iv) (d) (i), (ii), (iii) and (iv) (b) Intermolecular forces tend to keep the molecules
6. Dipole-induced dipole interactions are present in which of together but thermal energy of the molecules tends
the following pairs : to keep them apart.
(a) Cl2 and CCl4 (b) HCl and He atoms (c) Three states of matter are the result of balance between
(c) SiF4 and He atoms (d) H2O and alcohol intermolecular forces and the thermal energy of the
7. Assertion : Three states of matter are the result of balance molecules.
between intermolecular forces and thermal energy of the (d) All of the above
molecules. 12. Which of the following are arrangement in the correct
Reason : Intermolecular forces tend to keep the molecules order?
together but thermal energy of molecules tends to keep I. Gas > Liquid > Solid (Thermal energy)
them apart. II. Solid > Liquid > Gas (Intermolecular force)
(a) Assertion is correct, reason is correct; reason is a Select the correct option.
correct explanation for assertion. (a) I only (b) II only
(b) Assertion is correct, reason is correct; reason is not (c) Both I and II (d) None of these
a correct explanation for assertion 13. Which one of the following statements is not correct about
(c) Assertion is correct, reason is incorrect the three states of matter i.e., solid, liquid and gaseous ?
(d) Assertion is incorrect, reason is correct. (a) Molecules of a solid possess least energy whereas
8. Which of the following statements are correct ? those of a gas possess highest energy.
(i) Hydrogen bonding is a special case of dipole - dipole (b) The density of solid is highest whereas that of gases
interaction. is lowest
(ii) Energy of hydrogen bond varies between 10 to (c) Gases like liquids possess definite volumes
100 kJ mol–1. (d) Molecules of a solid possess vibratory motion
STATES OF MATTER 67

Column-I Column-II
Topic 3: The Gaseous State and Gas Laws
(Graphs) (Names)
14. The first reliable measurement on properties of gases was (A) Pressure vs temperature (p) Isotherms
made by ______________ graph at constant
(a) Gay Lussac (b) Jacques charles molar volume.
(c) Robert Boyle (d) Avogadro (B) Pressure vs volume (q) Isostere
15. Which of following graph(s) represents Boyle's law graph at constant
temperature
(C) Volume vs temperature (r) Isochores
I. P II. P graph at constant
pressure
V V (s) Isobars
(a) A – (p), B – (r), C – (s)
(b) A – (r), B – (p), C – (s)
III. P IV. P (c) A – (r), B – (q), C – (p)
(d) A – (s), B – (q), C – (r)
1/V 1/V 21. Pressure in well inflated tyres of automobiles is almost
(a) Only I (b) II and IV constant, but on a hot summer day this increases
(c) I and III (d) Only III considerably and tyre may burst if pressure is not adjusted
16. 600 c.c. of a gas at a pressure of 750 mm of Hg is properly. During winters, on a cold morning one may find
compressed to 500 c.c. Taking the temperature to remain the pressure in the tyres of a vehicle decreased
constant, the increase in pressure, is considerably.
(a) 150 mm of Hg Which of the following law explain the above observations?
(b) 250 mm of Hg (a) Charle's Law (b) Avogadro Law
(c) 350 mm of Hg (c) Boyle's Law (d) Gay Lussac's Law
(d) 450 mm of Hg 22. On a ship sailing in pacific ocean where temperature is
17. Match the columns 23.4°C, a balloon is filled with 2 L air. What will be the
Column-I Column-II volume of the balloon when the ship reaches Indian ocean,
(A) Boyle's law (p) V µ n at constant T where temperature is 26.1°C ?
and P (a) 2.018 L (b) 2.8 L
(B) Charle's law (q) Ptotal = P1+ P2+ P3+ .... (c) 3.5 L (d) 1.5 L
at constant T, V 23. At constant temperature, for a given mass of an ideal gas
PV (a) The ratio of pressure and volume always remains
(C) Dalton's law (r) = Constant constant.
T
(b) Volume always remains constant.
(D) Avogadro law (s) V µ T at constant n (c) Pressure always remains constant.
and P (d) The product of pressure and volume always remains
1 constant.
(t) P µ at constant n
V 24. What is the value of X in °C for given volume vs temperature
and T curve ?
(a) A – (t), B – (s), C – (q), D – (p)
P1 P 1 < P 2 < P3 < P 4
(b) A – (s), B – (q), C – (p), D – (t) P2
Volume ®

(c) A – (r), B – (t), C – (q), D – (p) P3


(d) A – (t), B – (q), C – (s), D – (r) P4
18. The lowest hypothetical or imaginary temperature at which
gases are supposed to occupy zero volume is called
X Temperature (°C) ®
_________
(a) Kelvin temperature (a) 0° C (b) 273.15° C
(b) absolute zero (c) – 273.15° C (d) 300° C
(c) Charle's temperature 25. Air at sea level is dense. This is a practical application of
(d) constant temperature (a) Boyle’s law
19. 500 mL of nitrogen at 27°C is cooled to –5°C at the same (b) Charle’s law
pressure. The new volume becomes (c) Kelvin’s law
(a) 326.32 mL (b) 446.66 mL (d) Brown’s law
(c) 546.32 mL (d) 771.56 mL 26. Which of the following volume (V) - temperature (T) plots
20. Match the graphs between the following variables (Column- represents the behaviour of one mole of an ideal gas at
I) with their names (Column-II) : one atmospheric pressure ?
EBD_8350
68 CHEMISTRY

V(L) V(L)
(a) H2 and SO2 (b) H2 and Cl2
(36.8 L (26.8 L (c) H2 and CO2 (d) CO2 and Cl2
(22.4 L 373 K) (22.4 L 373 K)
273K) 273K)
36. 16 g of oxygen and 3 g of hydrogen are mixed and kept at
(a) (b) 760 mm of Hg pressure and 0° C. The total volume occupied
by the mixture will be nearly
T(K) T(K) (a) 22.4 litres (b) 33.6 litres
(c) 448 litres (d) 44800 mL
V(L)
V(L) 37. The density of neon will be highest at
(30.6 L
373 K) (22.4 L (a) S.T.P. (b) 0ºC, 2 atm
(22.4 L 273K) (c) 273ºC, 1 atm. (d) 273ºC, 2 atm.
(c) (d) (14.2 L
273K) 373 K) 38. At STP molar volume of an ideal gas or a combination of
T(K)
ideal gases is ___________
T(K)
(a) 22.71098 L mol–1 (b) 20.71098 L mol–1
27. If 500 mL of gas A at 400 torr and 666.6 mL of B at 600 torr (c) 22.4139 L mol –1 (d) 24.78 L mol–1
are placed in a 3 litre flask, the pressure of the system will
39. Pressure of a mixture of 4 g of O2 and 2 g of H2 confined
be
in a bulb of 1 litre at 0°C is
(a) 200 torr (b) 100 torr (c) 550 torr (d) 366 torr
(a) 25.215 atm (b) 31.205 atm
28. For 1 mol of an ideal gas at a constant temperature T, the
(c) 45.215 atm (d) 15.210 atm
plot of (log P) against (log V) is a (P : Pressure, V : Volume)
40. If three unreactive gases having partial pressures PA, PB
(a) Straight line parallel to x-axis.
and PC and their moles are 1, 2 and 3 respectively then
(b) Straight line with a negative slope.
their total pressure will be
(c) Curve starting at origin.
(d) Straight line passing through origin. (a) P = PA + PB + PC (b) P = PA + PB + PC
6
Topic 4: Ideal Gas Equation
PA + PB + PC
29. 4.4 g of a gas at STP occupies a volume of 2.24 L, the gas (c) P= (d) None of these
3
can be 41. The pressure exerted by 6.0g of methane gas in a 0.03 m3
(a) O2 (b) CO (c) NO2 (d) CO2 vessel at 129°C is (Atomic masses : C = 12.01, H = 1.01 and
30. An ideal gas is one which obeys the gas laws under R = 8.314 JK–1 mol –1)
(a) a few selected experimental conditions (a) 31684 Pa (b) 215216 Pa
(b) all experimental conditions (c) 13409 Pa (d) 41648 Pa
(c) low pressure alone 42. A gaseous mixture was prepared by taking equal mole of
(d) high temperature alone CO and N 2. If the total pressure of the mixture was
31. Select one correct statement. In the gas equation, PV = nRT found 1 atmosphere, the partial pressure of the nitrogen
(a) n is the number of molecules of a gas (N2) in the mixture is
(b) V denotes volume of one mole of the gas (a) 0.5 atm (b) 0.8 atm (c) 0.9 atm (d) 1 atm
(c) n moles of the gas have a volume V 43. A bubble of air is underwater at temperature 15°C and the
(d) P is the pressure of the gas when only one mole of pressure 1.5 bar. If the bubble rises to the surface where
gas is present. the temperature is 25°C and the pressure is 1.0 bar, what
32. The correct value of the gas constant ‘R’ is close to : will happen to the volume of the bubble ?
(a) 0.082 litre-atmosphere K (a) Volume will become greater by a factor of 1.6.
(b) Volume will become greater by a factor of 1.1.
(b) 0.082 litre-atmosphere K–1 mol–1
(c ) Volume will become smaller by a factor of 0.70.
(c) 0.082 litre – atmosphere–1 K mol–1
(d) Volume will become greater by a factor of 2.5.
(d) 0.082 litre –1 atmosphere – 1 K mol
44. A mixture contains 64 g of dioxygen and 60 g of neon at a
33. If P, V, M, T and R are pressure, volume, molar mass,
total pressure of 10 bar. The partial pressures in bar of
temperature and gas constant respectively, then for an
dioxygen and neon are respectively (atomic masses O =
ideal gas, the density is given by
16, Ne = 20)
RT P M PM (a) 4 and 6 (b) 6 and 4
(a) (b) (c) (d) (c) 5 and 5 (d) 8 and 2
PM RT V RT
45. Value of universal gas constant (R) depends upon
34. At S.T.P the volume of a gas is found to be 273 ml. What
will be the volume of this gas at 600 mm of Hg and 273°C? (a) Number of moles of gas
(a) 391.8 mL (b) 380 mL (c) 691.6 mL (d) 750 mL (b) Volume of gas
35. Dalton’s law of partial pressure will not apply to which of (c) Temperature of gas
the following mixture of gases (d) None of these
STATES OF MATTER 69

46. Two vessels of volumes 16.4 L and 5 L contain two ideal 54. An evacuated glass vessel weights 50 g when empty,
gases of molecular existence at the respective temperature 144.0 g when filled with a liquid of density 0.47 g mL–1
of 27°C and 227°C and exert 1.5 and 4.1 atmospheres and 50.5 g when filled with an ideal gas at 760 mm Hg
respectively. The ratio of the number of molecules of the at 300 K. The molar mass of the ideal gas is
former to that of the later is (Given R = 0.0821 L atm K–1 mol–1)
1 1 (a) 61.575 (b) 130.98 (c) 123.75 (d) 47.87
(a) 2 (b) 1 (c) (d)
2 3 55. Two vessels containing gases A and B are interconected
47. If 10–4 dm3 of water is introduced into a 1.0 dm3 flask at 300 as shown in the figure. The stopper is opened, the gases
K, how many moles of water are in the vapour phase when are allowed to mix homogeneously. The partial pressures
equilibrium is established ? of A and B in the mixture will be, respectively
(Given : Vapour pressure of H2O at 300 K is 3170 Pa;
R = 8.314 J K–1 mol–1) Gas A Gas B
(a) 5.56× 10–3 mol (b) 1.53 × 10–2 mol
–2
(c) 4.46 × 10 mol (d) 1.27 × 10–3 mol 12 L 8L
48. Three different gases X, Y and Z of molecular masses 2, 16 8 atm 5 atm
and 64 were enclosed in a vessel at constant temperature
till equilibrium is reached. Which of the following statement (a) 8 and 5 atom (b) 9.6 and 4 atm
is correct? (c) 4.8 and 2 atm (d) 6.4 and 4 atm
(a) Gas Z will be at the top of the vessel Topic 5: Kinetic Molecular Theory of Gases
(b) Gas Y will be at the top of the vessel
56. According to the kinetic theory of gases, in an ideal gas,
(c) Gas Z will be at the bottom and X will be at the top
(d) Gases will form homogenous mixture between two successive collisions a gas molecule travels
49. When 2 gm of a gas A is introduced into an evaluated flask (a) in a wavy path
kept at 25ºC, the pressure is found to be one atmosphere. (b) in a straight line path
If 3 gm of another gas B is then added to the same flask, (c) with an accelerated velocity
the total pressure becomes 1.5 atm. Assuming ideal gas (d) in a circular path
behaviour, calculate the ratio of the molecular weights 57. Gases consist of large number of identical particles (atoms
MA : MB. or molecules) that are so small and so far apart on the
(a) 1 : 3 (b) 1 : 1 (c) 2 : 1 (d) 3 : 1 average that the actual volume of the molecules is negligible
50. A neon-dioxygen mixture contains 70.6 g O2 and 167.5 g in comparison to the empty space between them.
neon. If pressure of the mixture of gases in the cylinder is Above given statement explain which property of gases ?
25 bar. What is the partial pressure of O2 and Ne in the (a) Gases occupy all the space available to them.
mixture respectively ? (b) Gases has fixed shape.
(a) 5.25 bar, 10 bar (b) 19.75 bar, 5.25 bar (c) Compressibility of gases.
(c) 19.75 bar, 10 bar (d) 5.25 bar, 19.75 bar (d) None of these.
51. 0.5 mole of each H2, SO2 and CH\4 are kept in a container. A 58. Which of the following assumption of kinetic molecular
hole was made in the container. After 3 h, the order of theory states that gases do not have fixed shape ?
partial pressures in the container will be (a) Particles of a gas move in all possible directions in
pSO2 > pCH4 > pH2 pH2 > pSO2 > pCH4 straight line.
(a) (b)
(b) Particles of a gas are always in constant and random
(c) pH2 > pCH4 > pSO2 (d) pSO2 > pH2 > pCH4 motion.
(c) Total energy of molecules before and after the
52. What is the ratio of pressure of the 2 g of hydrogen to
collision remains same.
that of 4 g of helium at temperature of 298 K, 20 mL
(d) None of these
volume? (consider the ideal behaviour) 59. Kinetic theory of gases proves
(a) 1 : 2 (b) 2 : 1 (a) only Boyle’s law (b) only Charles’ law
(c) 1 : 1 (d) 2 : 2 (c) only Avogadro’s law (d) All of these
53. Consider the case of hot air balloon, density of air at 60. Which one of the following is the wrong assumption of
20 °C is 1.2 kg/m3, if the air was heated to 99°C, density kinetic theory of gases ?
of air becomes 0.94 kg/m3. What would be the volume (a) Momentum and energy always remain conserved.
(in m3) at 20°C if the volume at 99°C is 2800 m3 and how (b) Pressure is the result of elastic collision of molecules
much air (in kg) has been escaped at 99°C , if the air in with the container’s wall.
inflated balloon was heated to 99°C (if the inflated volume (c) Molecules are separated by great distances compared
of balloon was found to be 2800 m3) respectively are to their sizes.
(a) 2243, 728 (b) 3495.3, 596 (d) All the molecules move in straight line between
(c) 2687, 593 (d) 2956, 771 collision and with same velocity.
EBD_8350
70 CHEMISTRY

Topic 6: Behaviour of Real Gases : Deviation from Ideal Gas (iv) has the pressure that is lower than the pressure exerted
Behaviour by the same gas behaving ideally
(a) (i) and (ii) (b) (i) and (iii)
61. When is deviation more in the behaviour of a gas from the (c) (i), (ii) and (iii) (d) (ii) and (iv)
ideal gas equation PV = nRT ?
69. Which of the following statements are correct ?
(a) At high temperature and low pressure
(i) Real gases show deviations from ideal gas law because
(b) At low temperature and high pressure
molecules interact with each other.
(c) At high temperature and high pressure
(ii) Due to interaction of molecules the pressure exerted
(d) At low temperature and low pressure
by the gas is given as :
62. In van der Waal’s equation of state of the gas law, the
constant ‘b’ is a measure of an2
preal = p ideal + 2
(a) volume occupied by the molecules V
(b) intermolecular attraction (iii) Value of 'a' is measure of magnitude of intermolecular
(c) intermolecular repulsions attractive forces within the gas and depends on
(d) intermolecular collisions per unit volume temperature and pressure of gas.
63. In van der Waal’s equation of state for a non-ideal gas, the (iv) At high pressure, volume occupied by the molecules
term that accounts for intermolecular forces is also becomes significant because instead of moving
æ a ö in volume V, these are now restricted to volume
(a) (V – b) (b) R T (c) çP + 2 ÷ (d) (RT)–1 (V-nb)
è V ø (a) (i) and (iv) (b) (i), (ii) and (iii)
64. From the given figure what can be said about the gases
(c) (i), (iii) and (iv) (d) (i) and (iii)
does not deviate much from ideal gases at
70. In case of CO and CH4 curve goes to minima then
increases with increase in pressure but in case of H2
Real
and He the curve is linear because:
Ideal
CO CH4
Pressure

H2

He

Volume
PV Ideal gas
(a) Higher pressure and low volume.
(b) Low pressure and low volume.
(c) High pressure and high volume.
(d) Low pressure and high volume.
65. The units of constant ‘a’ in van der Waal’s equation is P
(a) dm6 atm mol–2 (b) dm3 atm mol–1
(c) dm atm mol –1 (d) atm mol–1 (a) Intermolecular interactions for H2 and He are very
66. The van der Waal’s constant ‘a’ for four gases P, Q, R and low.
S are 4.17, 3.59, 6.71 and 3.8 atm L2 mol–2 respectively. (b) Molecular size or atomic size for H2 and He is small.
Therefore, the ascending order of their liquefaction is (c) Both (a) and (b)
(a) R < P < S < Q (b) Q < S < R < P (d) Neither (a) nor (b)
(c) Q < S < P < R (d) R < P < Q < S
Topic 7: Liquefaction of Gases
67. The compressibility factor for a real gas at high pressure
is : 71. An ideal gas can’t be liquefied because
RT (a) its critical temperature is always above 0°C
(a) 1 + (b) 1 (b) Its molecules are relatively smaller in size
pb
pb pb (c) it solidifies before becoming a liquid
(c) 1 + (d) 1 – (d) forces between its molecules are negligible
RT RT
68. A gas described by van der Waal’s equation 72. Select the one that when used would be considered as
(i) behaves similar to an ideal gas in the limit of large best condition for liquification of a gas.
molar volume (a) Increasing the temperature.
(ii) behaves similar to an ideal gas in the limit of large (b) Decreasing the pressure.
pressure (c) Increasing the pressur e and decreasing the
(iii) is characterised by van der Waal's coefficients that temperature.
are dependent on the identity of the gas but are (d) Decreasing the pressure and increasing the
independent of the temperature temperature.
STATES OF MATTER 71

73. Above Boyle point, real gases show ______X _______ from (c) The normal boiling point of water is 100°C and its
ideality and Z values are _______Y ________ than one. standard boiling point is 99.6°C
(a) X = Negative deviation, Y = Less (d) None of the above
(b) X = Negative deviation, Y = Greater 78. A liquid can exist only
(c) X = Positive deviation, Y = Less (a) between triple point and critical temperature
(d) X = Positive deviation, Y = Greater (b) at any temperature above the melting point
74. Choose the correct statement based on the following (c) between melting point and critical temperature
isotherms of carbon dioxide at various temperature. (d) between boiling and melting temperature
G 79. Which among the following has lowest surface tension ?
H (a) Hexane (b) Water
50°C (c) CH3OH (d) CH3CH2OH
P3 80. A pin or a needle floats on the surface of water, the reason
D 31.1°C
for this is
(a) surface tension (b) less weight
73
(c) upthrust of liquid (d) None of the above
Pressure

(Pc ) E
33.98°C 81. Which of the following phenomena is caused by surface
C 21.5°C
B (Tc )
P2 tension ?
F
(a) Particles at the bottom of river remain separated but
P1
13.1°C they stick together when taken out.
A
(b) A liquid rise in a thin capillary.
(c) Small drops of mercury form spherical bead instead
V3 V2 Vc V1 of spreading on the surface.
Volume (d) All of the above
(i) We can move from point A to F vertically by increasing 82. The surface tension of which of the following liquid is
the temperature.
maximum?
(ii) We can reach the point G by compressing the gas at
(a) C2H5OH (b) CH3OH
constant temperature.
(iii) We can move down from G towards D by increasing (c) H2O (d) C6H6
the temperature. 83. The liquid which has the highest rate of evaporation is
(iv) As soon as we cross the point D on the critical (a) petrol (b) nail-polish remover
isotherm we get liquid. (c) water (d) alcohol
(a) (i) and (ii) (b) (i), (ii) and (iii) 84. The correct order of viscosity of the following liquids will
(c) (i), (ii) and (iv) (d) (i), (ii), (iii) and (iv) be
(a) Water < methyl alcohol < dimethyl ether < glycerol
Topic 8: Liquid State (b) methyl alcohol < glycerol < water < dimethyl ether
75. At 1 atm pressure, boiling temperature is called _____X
____. (c) dimethyl ether < methyl alcohol < water < glycerol
If pressure is 1 bar then the boiling point is called (d) glycerol < dimethyl ether < water < methyl alcohol
_____Y _____ of the liquid. 85. Assertion : Liquids tend to have maximum number of
(a) X = Standard boiling point, Y = Normal boiling point molecules at their surface.
(b) X = Normal boiling point, Y = Standard boiling point Reason : Small liquid drops have spherical shape.
(c) X = Critical boiling point, Y = Normal boiling point (a) Assertion is correct, reason is correct; reason is a
(d) X = Critical boiling point, Y = Standard boiling point correct explanation for assertion.
76. Assertion : The temperature at which vapour pressure of (b) Assertion is correct, reason is correct; reason is not
a liquid is equal to the external pressure is called boiling a correct explanation for assertion
temperature. (c) Assertion is correct, reason is incorrect
Reason : At high altitude atmospheric pressure is high. (d) Assertion is incorrect, reason is correct.
(a) Assertion is correct, reason is correct; reason is a 86. Choose the incorrect statement in the following.
correct explanation for assertion.
(a) Surface tension is the force acting per unit length
(b) Assertion is correct, reason is correct; reason is not
perpendicular to the line drawn on the surface of the
a correct explanation for assertion
liquid.
(c) Assertion is correct, reason is incorrect
(b) Surface tension of a liquid increases with increase in
(d) Assertion is incorrect, reason is correct.
77. Which of the following statement is incorrect ? temperature.
(a) Standard boiling point of liquid is slightly lower than (c) The SI unit of surface tension is J m –2.
the normal boiling point. (d) Viscosity is a measure of resistance for the flow of
(b) 1 atm pressure is slightly less than 1 bar pressure liquid.
EBD_8350
72 CHEMISTRY

87. A drop of oil is placed on the surface of water. Which of (c) Both (a) and (b)
the following statement is correct ? (d) Neither (a) nor (b)
(a) It will remain on it as a sphere 90. Consider the case of honey flowing over a slope for this
(b) It will spread as a thin layer situation which of the following statement(s) is/are
(c) It will be partly as spherical droplets and partly as correct ?
thin film
(d) It will float as a distorted drop on the water surface
88. When the temperature increases, the viscosity of
(a) gases decreases and viscosity of liquids increases
(b) gases increases and viscosity of liquids decreases
(c) gases and liquids increases
(d) gases and liquids decreases q (B)
89. Water droplets was not able to maintain its spherical q (A)
(i) Velocity with which honey is flowing is slower in A
shape in the presence of gravity but mercury does, why ? than (B) (q being same in both cases)
(a) Force of attraction between atoms of mercury is (ii) Velocity increases with increase in temperature.
very high than that of molecules in case of water. (a) (i) and (ii) (b) Only (i)
(b) Surface tension of mercury is very high. (c) Only (ii) (d) Neither (i) nor (ii)

1. Sulphur dioxide and oxygen were allowed to diffuse container is heated to 1200 K at which both substances
through a porous partition. 20 dm3 of SO2 diffuses decompose completely according to the equations
through the porous partition in 60 seconds. Calculate
2NH3(g) ® N2(g) + 3H2(g)
the volume of O2 in dm3 which diffuses under the similar
condition in 30 seconds (atomic mass of sulphur = 32 u). and N2H4(g) ® N2(g) + 2H2(g)
2. 4.5 g of PCl5 on vapourisation occupied a volume of After decomposition is complete, the total pressure at
1700 mL at 1 atmosphere pressure and 227°C temperature. 1200 K is found to be 4.5 atm. Find the mole % of N2H4 in
Calculate its degree of dissociation. the original mixture.
3. A spherical balloon of 21 cm diameter is to be filled up with 7. If 250 mL of N2 over water at 30°C and a total pressure of
hydrogen at N.T.P. from a cylinder containing the gas at 740 torr is mixed with 300 mL of Ne over water at 25°C
20 atmospheres at 27 ºC. If the cylinder can hold 2.82 litres and a total pressure of 780 torr, what will be the total
of water, calculate the number of balloons that can be filled pressure if the mixture is in a 500 mL vessel over water at
up. 35°C?
4. At 27ºC, hydrogen is leaked through a tiny hole into a Given : vapour pressure (Aqueous tension) of H2O at
vessel for 20 minutes. Another unknown gas at the same 25°C, 30°C and 35°C are 23.8, 31.8 and 42.2 torr
temperature and pressure as that of H2 is leaked through respectively. Assume volume of H2O(l) is negligible in
the same hole for 20 minutes. After the effusion of the final vessel)
gases the mixture exerts a pressure of 6 atmosphere. The 8. At 300 K, the density of a certain gaseous molecule at 2
hydrogen content of the mixture is 0.7 mole. If the volume bar is double to that of dinitrogen (N2) at 4 bar. Find the
of the container is 3 litres, what is the molecular weight of molar mass of gaseous molecule.
the unknown gas? 9. Calculate the total pressure (in atm) in a 10.0 L cylinder
5. A bubble of gas released at the bottom of a lake increases which contains 0.4 g helium, 1.6 g oxygen and 1.4 g nitrogen
to four times its original volume when it reaches the at 27 °C.
surface. Assuming that atmospheric pressure is equivalent 10. At STP, a container has 1 mole of He, 2 mole Ne, 3 mole O2
to the pressure exerted by a column of water 10 m high, and 4 mole N2 without changing total pressure if 2 mole of
what is the depth of the lake (in m)? O2 is removed. Calculate the percentage decrease in the
6. A mixture of NH3(g) and N2H4(g) is placed in a sealed partial pressure of O2.
container at 300 K. The total pressure is 0.5 atm. The
STATES OF MATTER 73

Exercise 3 : NCERT Exemplar & Past Year MCQs


NCERT Exemplar MCQs 9. How does the surface tension of a liquid vary with increase
in temperature?
1. A person living in shimla observed that cooking food (a) Remains same
without using pressure cooker takes more time. The reason (b) Decreases
for this observation is that at high altitude (c) Increases
(a) pressure increases (b) temperature decreases (d) No regular pattern is followed
(c) pressure decreases (d) temperature increases
10. A plot of volume (V) versus temperature (T) for a gas at
2. Which of the following property of water can be used to constant pressure is a straight line passing through the
explain the spherical shape of rain droplets? origin. The plots at different values of pressure are shown
(a) Viscosity (b) Surface tension in figure. Which of the following order of pressure is
(c) Critical phenomena (d) Pressure correct for this gas?
3. Dipole-dipole forces act between the molecules
possessing permanent dipole. Ends of dipoles possess
P1
'partial charges'. The partial charge is P2

Volume (mL) ®
(a) more than unit electronic charge P3
(b) equal to unit electronic charge P4
(c) less than unit electronic charge
(d) double the unit electronic charge
4. The pressure of a 1 : 4 mixture of dihydrogen and dioxygen
enclosed in a vessel is one atmosphere. What would be
the partial pressure of dioxygen? Temperature (K) ®
(a) 0.8 × 105 atm (b) 0.008 Nm–2 (a) P1 > P2 > P3 > P4 (b) P1 = P2 = P3 = P4
(c) 8 × 10 Nm4 –2 (d) 0.25 atm (c) P1 < P2 < P3 < P4 (d) P1 < P2 = P3 < P4
5. As the temperature increases average kinetic energy of 11. The interaction energy of London force is inversely
molecules increases. What would be the effect of increase proportional to sixth power of the distance between two
of temperature on pressure provided the volume is interacting particles but their magnitude depends upon
constant? (a) charge of interacting particles
(a) Increases (b) Decreases (b) mass of interacting particles
(c) Remains same (d) Becomes half (c) polarisability of interacting particles
6. What is SI unit of viscosity coefficient (h)? (d) strength of permanent dipoles in the particles
(a) Pascal (b) Nsm–2 12. Gases possess characteristic critical temperature which
(c) km s –2 (d) Nm–2 depends upon the magnitude of intermolecular forces
7. Atmospheric pressures recorded in different cities are as between the particles. Following are the critical
follows temperatures of some gases.
Cities p in N/m2 Gases H 2 He O 2 N2
Shimla 1.01 × 105 Critical temperature in Kelvin 33.2 5.3 154.3 126
Bengaluru 1.2 × 105 From the above data, what would be the order of
Delhi 1.02 × 105 liquefaction of these gases? Start writing the order from
Mumbai 1.21 × 105 the gas liquefying first
(a) H2, He, O2, N2 (b) He, O2, H2, N2
Consider the above data and mark the place at which liquid
(c) N2, O2, He, H2 (d) O2, N2, H2, He
will boil first.
13. Which curve in figure represents the curve of ideal gas?
(a) Shimla (b) Bengaluru
(c) Delhi (d) Mumbai F
E D
8. Increase in kinetic energy can overcome intermolecular C A
forces of attraction. How will the viscosity of liquid be
PV ®

B
affected by the increase in temperature?
(a) Increase
(b) No effect 0 P®
(c) Decrease
(a) Only B (b) C and D
(d) No regular pattern will be followed
(c) E and F (d) A and B
EBD_8350
74 CHEMISTRY

Past Year MCQs 19. Equal moles of hydrogen and oxygen gases are placed in a
container with a pin-hole through which both can escape.
14. Equal masses of H2,O2 and methane have been taken in a
What fraction of the oxygen escapes in the time required
container of volume V at temperature 27°C in identical
for one-half of the hydrogen to escape ? [NEET 2016, A]
conditions. The ratio of the volumes of gases H2 : O2 :
(a) 1/8 (b) 1/4
methane would be : [NEET 2014, S]
(c) 3/8 (d) 1/2
(a) 8 : 16 : 1 (b) 16 : 8 : 1
(c) 16 : 1 : 2 (d) 8 : 1 : 2 20. Two closed bulbs of equal volume (V) containing an ideal
15. If Z is a compressibility factor, van der Waals equation at gas initially at pressure P i and temperature T1 are
low pressure can be written as: [JEE M 2014, S] connected through a narrow tube of negligible volume as
shown in the figure below. The temperature of one of the
RT a
(a) Z = 1 + (b) Z = 1 - bulbs is then raised to T2. The final pressure Pf is :
Pb VRT
[JEE M 2016, A]
Pb Pb
(c) Z = 1 - (d) Z = 1 +
RT RT
16. A gas such as carbon monoxide would be most likely to T1 T1 T1 T2
obey the ideal gas law at : [NEET 2015 RS, C]
(a) high temperatures and low pressures.
(b) low temperatures and high pressures.
(c) high temperatures and low pressures.
(d) low temperatures and low pressures. æ T ö æ T T ö
17. The intermolecular interaction that is dependent on the (a) 2 Pi ç 2 ÷ (b) 2 Pi ç 1 2 ÷
inverse cube of distance between the molecules is : è T1 + T2 ø è T1 + T2 ø
[JEE M 2015, C]
æ T T ö æ T1 ö
(a) London force (c) Pi ç 1 2 ÷ (d) 2 Pi ç
(b) hydrogen bond è T1 + T2 ø è T1 + T2 ÷ø
(c) ion - ion interaction 21. The correction factor ‘a’ to the ideal gas equation
(d) ion - dipole interaction
corresponds to [NEET 2018, C]
18. One mole of a monoatomic real gas satisfies the equation
(a) Density of the gas molecules
P(V – b) = RT where b is a constant. The relationship of
(b) Volume of the gas molecules
interatomic potential V(r) and interatomic distance r for
(c) Forces of attraction between the gas molecules
the gas is given by [JEE M 2015, S]
(d) Electric field present between the gas molecules
22. Given van der Waals constants for NH3, H2, O2 and CO2
are respectively 4.17, 0.244, 1.36 and 3.59, which one of the
(a) following gases is most easily liquefied? [NEET 2018, C]
(a) NH3 (b) H2
(c) CO2 (d) O2
23. A gas at 350 K and 15 bar has molar volume 20 percent
smaller than that for an ideal gas under the same conditions.
The correct option about the gas and its compressibility
factor (Z) is: [NEET 2019, A]
(b)
(a) Z > 1 and attractive forces are dominant
(b) Z > 1 and repulsive forces are dominant
(c) Z < 1 and attractive forces are dominant
(d) Z < 1 and repulsive forces are dominant
24. 0.5 moles of gas A and x moles of gas B exert a pressure
of 200 Pa in a container of volume 10 m3 at 1000 K.
(c) Given R is the gas constant in JK–1 mol–1, x is:
[JEE M 2019, A]
2R 2R
(a) (b)
4+R 4-R

(d) 4+R 4-R


(c) (d)
2R 2R
STATES OF MATTER 75

25. A mixture of N2 and Ar gases in a cylinder contains 7 g of 26. Consider the van der Waals constants, a and b, for the
N2 and 8 g of Ar. If the total pressure of the mixture of the following gases,
gases in the cylinder is 27 bar, the partial pressure of N 2 is: Gas Ar Ne Kr Xe
[Use atomic masses (in g mol–1) : N = 14, Ar = 40] a/ (atm dm6 mol–2) 1.3 0.2 5.1 4.1
[NEET 2020, A] b/ (10–2 dm3 mol–1) 3.2 1.7 1.0 5.0
Which gas is expected to have the highest critical
(a) 12 bar (b) 15 bar
temperature? [JEE M 2019, A]
(c) 18 bar (d) 9 bar (a) Kr (b) Ne (c) Xe (d) Ar

Exercise 4 : Problem Solving Skill Enhancer MCQs


1. At 100°C and 1 atm, if the density of liquid water is 1.0 g cm–3
and that of water vapour is 0.0006 g cm–3, then the volume
occupied by water molecules in 1 litre of steam at that 10 cm R=10 cm
temperature is
(a) 6 cm3 (b) 60 cm3
3 Sphere
(c) 0.6 cm (d) 0.06 cm3 (hollow)
10 cm
2. I, II, and III are three isotherms, respectively, at T1, T2 and (cube)
10 cm
T3. Temperature will be in order (I) (II) (III)
If all the containers are placed at the same temperatures,
then find the incorrect options –
(a) Pressure of the gas is minimum in (III) container
P
(b) Pressure of the gas is equal in I and II container
(c) Pressure of the gas is maximum in (I)
I
II (d) The ratio of pressure in II and III container is 4 : 3
III 6. Starting out on a trip into the mountains, you inflate the
V3 V2 V1 tires on your automobile to a recommended pressure of
3.21 × 105 Pa on a day when the temperature is – 5.0°C.
(a) T1 = T2 = T3 (b) T1 < T2 < T3 You drive to the beach, where the temperature is 28.0°C.
(c) T1 > T2 > T3 (d) T1 > T2 = T3
Assume that the volume of the tire has increased by 3%.
3. A container contains certain gas of mass ‘m’ at high
pressure. Some of the gas has been allowed to escape What is the final pressure in the tyres?
from the container and after some time the pressure of the (a) 350 Pa (b) 3500 Pa
gas becomes half and its absolute temperature 2/3 rd. The (c) 3.5 × 105 Pa (d) None of these
amount of the gas escaped is 7. At STP, a container has 1 mole of He, 2 mole Ne, 3 mole O2 and
(a) 2/3 m (b) 1/2 m
4 mole N2. without changing total pressure if 2 mole of O2 is
(c) 1/4 m (d) 1/6 m
4. A volume V of a gas at temperature T1 and at pressure p is removed. The partial pressure of O2 will be decreased by:
enclosed in a sphere. It is connected to another sphere of (a) 26% (b) 40% (c) 58.33% (d) 66.66%
volume V/2 by a tube and stopcock. The second sphere is 8. A mixture of Ne and Ar kept in a closed vessel at 250 K,
initially evacuated and the stopcock is closed. If the stopcock has a total K.E. = 3kJ. The total mass of Ne and Ar is 30 g.
is opened, the temperature of the gas in the second sphere Find mass % of Ne in gaseous mixture at 250 K.
becomes T2. The first sphere is maintained at a temperature
T1. What is the final pressure p1 within the apparatus ? (R = 8 JK–1 mol–1)
(a) 61.63 (b) 38.37 (c) 50 (d) 33.33
2 pT2 2 pT2
(a) (b) 9. Which gas shows real behaviour?
2T2 + T1 T2 + 2T1
(a) 16 g O2 at 1 atm and 273 K occupies 11.2 L
pT2 2 pT2
(c) (d) (b) 1 g H2 in 0.5 L flask exerts pressure of 24.63 atm at
2T2 + T1 T1 + T2
300 K
5. There are three closed containers in which equal amount
(c) 1 mole NH3 at 300 K and 1 atm occupies volume 22.4 L
of the gas are filled.
(d) 5.6 L of CO2 at 1 atm and 273 K is equal to 11 g
EBD_8350
76 CHEMISTRY

10. The van der Waal's equation for n = 1 mol may be expressed
æ RT ö 2 aV ab AB (g) B2 (g)
as V 3 - ç b + ÷V + - =0
è p ø p p
8.21 L
Where V is the molar volume of the gas. Which of the 8.21 L
following is correct? The final pressure is :
(a) For a temperature less than Tc, V has three real roots (a) 0.156 atm (b) 0.3125 atm
(b) For a temperature more than Tc, V has one real and (c) 0.625 atm (d) 3.2 atm
two imaginary roots 13. For a real gas (mol. mass = 60) if density at critical point is
(c) For a temperature equal to Tc all three roots of V are 4 ´ 105
real and identical 0.80 g/cm3 and its Tc = K, then van der Waals’
(d) All of these 821
2
constant a (in atm L mol ) is–2
11. For real gases, van der Waals equation is written as (a) 0.3375 (b) 3.375 (c) 1.68 (d) 0.025
æ an 2 ö 14. A spherical balloon of 21 cm diameter is to be filled up with
ç p + 2 ÷ (V – nb) = nRT hydrogen at STP from a cylinder containing the gas at
è V ø 20 atm at 28 °C. If the cylinder can hold 2.82 L of water, the
where ‘a’ and ‘b’ are van der Waals constants. number of balloons that can be filled up are
Two sets of gases are : (a) 5 (b) 7 (c) 9 (d) 10
15. Points I, II and III in the following plot respectively corre-
(I) O2, CO2, H2 and He (II) CH4, O2 and H2
spond to (Vmp : most probable velocity)
The gases given in set-I in increasing order of ‘b’ and
gases given in set-II in decreasing order of ‘a’, are arranged

Distribution function, f (v ) ®
below. Select the correct order from the following :
(a) (I) He < H2 < CO2 < O2 (II) CH4 > H2 > O2
(b) (I) O2 < He < H2 < CO2 (II) H2 > O2 > CH4
(c) (I) He < H2 < O2 < CO2 (II) CH4 > O2 > H2
(d) (I) H2 < O2 < He < CO2 (II) O2 > CH4 > H2
12. At room temperature following reaction goes to
completion 2AB(g) + B2(g) ® 2AB2(g)
AB2 is solid with negligible vapour pressure below 0°C.
At 300 K, AB in the smaller flask exerts a pressure of 3 atm Speed, v ®
and in the larger flask B2 exerts a pressure of 1 atm at 400 (a) Vmp of N2 (300 K); Vmp of O2 (400 K); Vmp of H2 (300 K)
K when they are separated out by a close valve. The gases (b) Vmp of O2 (400 K); Vmp of N2 (300 K); Vmp of H2 (300 K)
are mixed by opening the stop cock and after the end of (c) Vmp of N2 (300 K); Vmp of H2 (300 K); Vmp of O2 (400 K)
the reaction the flask are cooled to 250 K. (d) Vmp of H2 (300 K); Vmp of N2 (300 K); Vmp of O2 (400 K)
ANSWER KEY
Exercise 1 : NCERT Based Topic-wise MCQs
1 (c) 10 (c) 19 (b) 28 (b) 37 (b) 46 (a) 55 (c) 64 (d) 73 (d) 82 (c)
2 (d) 11 (d) 20 (b) 29 (d) 38 (a) 47 (d) 56 (b) 65 (a) 74 (a) 83 (b)
3 (a) 12 (c) 21 (d) 30 (b) 39 (a) 48 (d) 57 (c) 66 (c) 75 (b) 84 (c)
4 (c) 13 (c) 22 (a) 31 (c) 40 (a) 49 (a) 58 (b) 67 (c) 76 (c) 85 (d)
5 (c) 14 (c) 23 (d) 32 (b) 41 (d) 50 (d) 59 (d) 68 (b) 77 (b) 86 (b)
6 (b) 15 (c) 24 (c) 33 (d) 42 (a) 51 (a) 60 (d) 69 (a) 78 (d) 87 (b)
7 (a) 16 (a) 25 (a) 34 (c) 43 (a) 52 (c) 61 (b) 70 (c) 79 (a) 88 (b)
8 (d) 17 (a) 26 (c) 35 (b) 44 (a) 53 (a) 62 (a) 71 (d) 80 (a) 89 (c)
9 (a) 18 (b) 27 (a) 36 (d) 45 (d) 54 (a) 63 (c) 72 (c) 81 (d) 90 (a)
Exercise 2 : Numeric/Integer Answer Questions
1 (14.1) 2 (0.92) 3 (10) 4 (1033) 5 (30) 6 (25) 7 (870.6) 8 (112) 9 (0.49) 10 (58.33)
Exercise 3 : NCERT Exemplar & Past Year MCQs
1 (c) 4 (c) 7 (a) 10 (c) 13 (a) 16 (a) 19 (a) 22 (a) 25 (b)
2 (b) 5 (a) 8 (c) 11 (c) 14 (c) 17 (b) 20 (a) 23 (c) 26 (a)
3 (c) 6 (b) 9 (b) 12 (d) 15 (b) 18 (c) 21 (c) 24 (d)
Exercise 4 : Problem Solving Skill Enhancer MCQs
1 (c) 3 (c) 5 (b) 7 (c) 9 (c) 11 (c) 13 (b) 15 (a)
2 (c) 4 (a) 6 (c) 8 (d) 10 (d) 12 (c) 14 (d)
6 Thermodynamics

Trend Buster NEET & JEE Main

Number of Questions from 2020-15 9 7 Minimum 1 and maximum 2


questions have been asked.
Weightage 2.8% 4%

The most Important Concepts that Cover Maximum number of Questions asked in past 6 years.

First law of thermodynamics / Second law of 4 2


thermodynamics
DH formation 1 1
Thermodynamic conditions for spontaniety 2 —
Less Important Concepts that Cover 1 or 2 Questions asked in past 6 years.

Reaction enthalpy 1 —
Internal energy — 1
Temperature dependence of equilibrium constant — 1
Entropy 1 —
Path function — 1
Reversible isothermal expansion — 1

NEET JEE

2020 Applications / Spontaneity, Gibbs First law of thermodynamics / 2 Average 1 Average


energy change and equilibrium Second law of thermodynamics
constant
2019 Applications / Thermodynamics / Reversible isothermal expansion / 2 Average 2 Difficulty /
Spontaneity, gibbs energy First law of thermodynamics / Easy
change and equilibrium constant Path function / Entropy
2018 Spontaneity, gibbs energy Temperature dependence of 1 Difficulty 2 Average /
change and equilibrium constant / equilibrium constant / First law of Difficulty
Applications / Reaction enthalpy thermodynamics / DH formation
2017 Thermodynamics / Spontaneity, Internal energy change / 2 Easy / 1 Easy
gibbs energy change and Spontaneous reactions / First law Average
equilibrium constant / Application of thermodynamics
2016 Reaction enthalpy / Heat of formation / 1 Average 1 Average
Spontaneity, gibbs energy Thermodynamic conditions for
change and equilibrium constant spontaneous reactions
2015 Reaction enthalpy Reaction enthalpy 1 Average — —
EBD_8350
78 CHEMISTRY
THERMODYNAMICS 79
EBD_8350
80 CHEMISTRY

Problem Solving Tips/ Tricks/ Points to Remember

4 Entropy If we put the value of g we get the expression for workdone


q rev W = nCv (T2 - T1 )
DS =
T 4 Relation Between DH and DE
For a spontaneous process
DSTotal = DSsystem + DSsurr > 0 DH = DE + Dng RT
or DSuniverse >0 Dng = Number of moles of products - number of moles of
4 Thermodynamic properties: reactants (in gaseous state).
(i) Extensive properties: The properties of system DE and DH for Monoatomic Gas
which depend upon the amount of substance of
3
system. For example, mass, volume, work, etc. Internal Energy = K.E. = RT per mole
2
(ii) Intensive properties: The properties of system
which do not depend upon the amount of substance 3
present. For example, temperature, pressure, surface Enthalpy = RT + PV
2
tension, density, refractive index, etc.
4 Criterion of spontaneity: 3 5
= RT + RT = RT per mole (Q PV = RT )
(dS) U,V ³ 0 (dG) T, P £ 0 or (dU)S,V £ 0 or (dH)S, P £ 0 2 2
where inequality (< or >) is irreversible process 4 For isothermal process (DT = 0),
(spontaneous) while equality (=) is reversible process at
V2 P
equilibrium. DS = R ln = R ln 1
V1 P2
(i) Exoergonic reactions: An exoergonic reaction is a
reaction which is accompanied by decrease of Gibb’s
energy, i.e., a reaction for which the DG is negative. T2
4 For isobaric process (DP = 0), DS = C P ln
All spontaneous processes are exoergenic reactions. T1
(ii) Endoergonic reactions: An endoergonic reaction 4 Enthalpy of combustion by Bomb calorimeter,
is a reaction which is accompanied by increase in
Gibb’s energy, i.e., for which DG is positive. All non- M
DE = C × Dt ×
spontaneous processes are endoergonic reactions. m
Work of Expansion/Compression where C = heat capacity of the calorimeter
4 Entropy Change for an Ideal Gas : With Change Dt = rise in temperature = (t2 – t1)
should be in P, V and T m = mass of the substance taken
T2 V M = molecular mass of the substance
(i) DS = C V ln + R ln 2 (when T and V are two variables) 4 Efficiency of heat engine,
T1 V1
T2 P W T2 – T1 q 2 – q1
(ii) DS = CP ln + R ln 1 (when T and P are two variables) h= =
T1 P2 q2 T2 = q 2
4 Work Done (Isothermal Reversible Expansion) where q2 = heat absorbed by the system at temperature
(Maximum Work) T2 of the source q1 = heat rejected by the system at
It is given by the expression temperature T1 of the sink
V2 P W = net work done.
Wmax = - 2.303nRT log = - 2.303nRT log 1
V1 P2 4 Variables like P , V, T are called state functions because
4 Work Done (Adiabatic Reversible Expansion) their value depend only on the state of the system and
nR (T2 - T1 ) not on how it is reached.
W= where g is Poisson 's ratio which is
g -1 4 The quantities which depend on the path by which the
Cp state has been achieved, are called path function e.g. W
equal to and Cp – Cv = R DQ etc.
Cv
THERMODYNAMICS 81

4 Second law of thermodynamics : For all spontaneous 4 Change in internal energy DU is defined at constant
proesses, the total entropy change is positive. volume but enthalpy H is defined at constant pressure.
4 Third law of thermodynamics : The entropy of any pure 4 During stretching of rubber band, entropy decreases
crystalline substance approaches zero as the temperature while on hard boiling of egg, entropy increases.
approaches absolute zero.
4 A process is said to be reversible, if a change is brought out 4 Standard enthalpy of formation of an element in reference
in such a way that the process could, at any moment, be state i.e., its most stable state is taken a zero. e.g. Br2 (l),
reversed by an infinitesimal change. The system and O2(g), C (graphite), etc.
surroundings are always in near equilibrium with each other. 4 Specific heat is the quantity of heat required to raise the
4 When the process cannot be reversed, it is termed as temperature of one unit mass of a substance by one
irreversible process. These are spontaneous in nature. degree. The value for one gram of water is one calorie,
All natural processes are irreversible in nature. which is one of the highest.

Exercise 1 : NCERT Based Topic-wise MCQs


Topic 1: Thermodynamics 7. Adiabatic expansions of an ideal gas is accompanied by
(a) decrease in DE
1. Which of the following statements is not true regarding (b) increase in temperature
the laws of thermodynamics ?
(c) decrease in DS
(a) It deal with energy changes of macroscopic systems.
(b) It deal with energy changes of microscopic systems. (d) no change in any one of the above properties
(c) It does not depends on the rate at which these energy 8. Assertion : Absolute value of internal energy of a
transformations are carried out. substance cannot be determined.
(d) It depends on initial and final states of a system Reason : It is impossible to determine exact values of
undergoing the change. constitutent energies of the substances.
2. Which of the following is closed system ? (a) Assertion is correct, reason is correct; reason is a
(a) Jet engine
correct explanation for assertion.
(b) Tea placed in a steel kettle
(c) Pressure cooker (b) Assertion is correct, reason is correct; reason is not
(d) Rocket engine during propulsion a correct explanation for assertion
3. The state of a thermodynamic system is described by its (c) Assertion is correct, reason is incorrect
measurable or macroscopic (bulk) properties. These are (d) Assertion is incorrect, reason is correct.
(a) Pressure and volume 9. Read the following statements carefully and choose the
(b) Pressure, volume, temperature and amount correct option
(c) Volume, temperature and amount (i) Internal energy, U, of the system is a state function.
(d) Pressure and temperature (ii) –w shows, that work is done on the system.
4. Enthalpy change (DH) of a system depends upon its (iii) +w shows, that work is done by the system
(a) Initial state (a) (i) and (ii) are correct (b) (ii) and (iii) are correct
(b) Final state (c) (i) and (iii) are correct (d) Only (i) is correct
(c) Both on initial and final state 10. Which of the following are state functions ?
(d) None of these (I) q + w (II) q
5. Which of the following factors affect the internal energy (III) w (IV) H - TS
of the system ? (a) (I) and (IV) (b) (II), (III)
(a) Heat passes into or out of the system. (c) (II), (IV) (d) (III), (IV)
(b) Work is done on or by the system. 11. Among the following, the state function(s) is (are)
(c) Matter enters or leaves the system. (i) Internal energy
(d) All of the above (ii) Irreversible expansion work
6. Assertion : T, P and V are state variables or state functions. (iii) Reversible expansion work
Reason : Their values depend on the state of the system (iv) Molar enthalpy
and how it is reached. (a) (ii) and (iii) (b) (i), (ii) and (iii)
(a) Assertion is correct, reason is correct; reason is a (c) (i) and (iv) (d) (i) only
correct explanation for assertion. 12. According to the first law of thermodynamics which of the
(b) Assertion is correct, reason is correct; reason is not following quantities represents change in a state function ?
a correct explanation for assertion (a) qrev (b) qrev – Wrev
(c) Assertion is correct, reason is incorrect (c) qrev/Wrev (d) qrev + Wrev
(d) Assertion is incorrect, reason is correct.
EBD_8350
82 CHEMISTRY

13. According to the first law of thermodynamics, DU = q + W. V2


In special cases the statement can be expressed in different (c) w = nRT ln (isothermal reversible expansion of
V1
ways. Which of the following is not a correct expression ? an ideal gas)
(a) At constant temperature q = –W (d) For a system of constant volume, heat involved
(b) When no work is done DU = q directly changes to internal energy.
(c) In gaseous system DU = q + PDV 18. An ideal gas expands in volume from 1×10–3 to 1 × 10–2 m3
(d) When work is done by the system : DU = q + W at 300 K against a constant pressure of 1×105 Nm–2. The
14. Assertion : A process is called adiabatic if the system work done is
does not exchange heat with the surroundings. (a) 270 kJ (b) – 900 kJ (c) – 900 J (d) 900 kJ
Reason : It does not involve increase or decrease in 19. The difference between DH and DU is usually significant
temperature of the system. for systems consisting of
(a) Assertion is correct, reason is correct; reason is a (a) only solids (b) only liquids
correct explanation for assertion. (c) both solids and liquids (d) only gases
(b) Assertion is correct, reason is correct; reason is not 20. Assume each reaction is carried out in an open container.
a correct explanation for assertion For which reaction will DH = DE ?
(c) Assertion is correct, reason is incorrect
(a) C(s) + 2H2O (g) ® 2H2 (g) + CO2 (g)
(d) Assertion is incorrect, reason is correct.
(b) PCl5 (g) ® PCl3 (g) + Cl2 (g)
Topic 2: Applications (c) 2CO (g) + O2 (g) ® 2CO2 (g)
15. Figure below is showing that one mole of an ideal gas is (d) H2 (g) + Br 2 (g) ® 2 HBr (g)
fitted with a frictionless piston. Total volume of the gas is 1
21. For the reaction CO (g ) + O 2 (g ) ® CO 2 (g )
Vi and pressure of the gas inside is p. If external pressure is 2
Pex which is greater than p, is applied, piston is moved Which one of the statement is correct at constant T and P?
inward till the pressure inside becomes equal to Pex. (a) DH =DE
(b) DH < DE
Pex (c) DH > DE
(d) DH is independent of physical state of the reactants
Pressure, P

22. For the reaction


Area = PexDV C3 H8 (g) + 5O2 (g) ® 3CO2 (g) + 4H 2 O(l)
at constant temperature, DH – DE is
(a) – RT (b) + RT
Vf Vi Volume, V
(c) – 3 RT (d) + 3 RT
23. Which is an extensive property of the system ?
Pex (a) Volume (b) Viscosity
(c) Temperature (d) Refractive index
24. If DH is the change in enthalpy and DE, the change in
internal energy accompanying a gaseous reaction, then
Pex
(a) DH is always greater than DE
(b) DH < DE only if the number of moles of the products
is greater than the number of moles of the reactants
(c) DH is always less than DE
l (d) DH < DE only if the number of moles of products is
What does the shaded area represents in the figure ?
less than the number of moles of the reactants
(a) Work done (b) Pressure change 25. Assertion : Internal energy is an extensive property.
(c) Volume change (d) Temperature change Reason : Internal energy depends upon the amount of the
16. When 1 mol of a gas is heated at constant volume, system.
temperature is raised from 298 to 308 K. If heat supplied to (a) Assertion is correct, reason is correct; reason is a
the gas is 500 J, then which statement is correct ? correct explanation for assertion.
(a) q = w = 500 J, DU = 0 (b) q = DU = 500 J, w = 0 (b) Assertion is correct, reason is correct; reason is not
(c) q = –w = 500 J, DU = 0 (d) DU = 0, q = w = –500 J a correct explanation for assertion
17. Which of the following statements/relationships is not (c) Assertion is correct, reason is incorrect
correct in thermodynamic changes ? (d) Assertion is incorrect, reason is correct.
(a) DU = 0 (isothermal reversible expansion of a gas) 26. Assertion : The mass and volume of a substance are the
V extensive properties and are proportional to each other.
(b) w = – nRT ln 2 (isothermal reversible expansion of Reason : The ratio of mass of a sample to its volume is an
V1
an ideal gas) intensive property.
THERMODYNAMICS 83

(a) Assertion is correct, reason is correct; reason is a 34. Match the columns
correct explanation for assertion. Column-I Column-II
(b) Assertion is correct, reason is correct; reason is not (A) pext = 0 (p) Free expansion of an
a correct explanation for assertion ideal gas
(c) Assertion is correct, reason is incorrect (B) q = pext (Vf – Vi) (q) Adiabatic change
(d) Assertion is incorrect, reason is correct. (C) q = 2.303 nRT log (Vf / Vi) (r) Isothermal reversible
27. Calorie is equivalent to : change
(D) DU = Wad (s) Isothermal irreversible
(a) 0.4184 Joule (b) 4.184 Joule
change
(c) 41.84 Joule (d) 418.4 Joule
(a) A – (p), B – (s), C – (r), D – (q)
28. Equal volumes of two monoatomic gases, A and B, at same (b) A – (p), B – (q), C – (r), D – (s)
temperature and pressure are mixed. The ratio of specific (c) A – (p), B – (r), C – (s), D – (q)
heats (Cp/Cv) of the mixture will be : (d) A – (p), B – (r), C – (q), D – (s)
(a) 0.83 (b) 1.50 35. Assertion : For an isothermal reversible process Q = –W
(c) 3.3 (d) 1.67 i.e. work done by the system equals the heat absorbed by
29. Assertion : First law of thermodynamics is applicable to the system.
an electric fan or a heater. Reason : Enthalpy change (DH) is zero for isothermal
process.
Reason : In an electric fan, the electrical energy is converted
(a) Assertion is correct, reason is correct; reason is a
into mechanical work that moves the blades. In a heater,
correct explanation for assertion.
electrical energy is converted into heat energy. (b) Assertion is correct, reason is correct; reason is not
(a) Assertion is correct, reason is correct; reason is a a correct explanation for assertion
correct explanation for assertion. (c) Assertion is correct, reason is incorrect
(b) Assertion is correct, reason is correct; reason is not (d) Assertion is incorrect, reason is correct.
a correct explanation for assertion 36. Consider the reaction : N 2 + 3H 2 ® 2 NH 3 carried out
(c) Assertion is correct, reason is incorrect at constant temperature and pressure. If DH and DU are
(d) Assertion is incorrect, reason is correct. the enthalpy and internal energy changes for the reaction,
30. Match the columns which of the following expressions is true ?
Column-I Column-II (a) DH > DU (b) DH < DU
(A) Cm (p) Cv DT (c) DH = DU (d) DH = 0
(B) q (q) C/n 37. Among the following, the intensive properties are
(C) DU (r) Cp DT (i) molar conductivity (ii) electromotive force
(D) DH (s) C DT (iii) resistance (iv) heat capacity
(a) A – (q), B – (s), C – (r), D – (p) (a) (i) and (ii) (b) (i), (ii) and (iii)
(b) A – (q), B – (s), C – (p), D – (r) (c) (i) and (iv) (d) (i) only
(c) A – (s), B – (q), C – (p), D – (r) 38. Which of the following factors do not affect heat capacity?
(d) A – (q), B – (p), C – (r), D – (s) (a) Size of system (b) Composition of system
31. If a reaction involves only solids and liquids which of the (c) Nature of system (d) Temperature of the system
following is true ? 39. For an isothermal reversible expansion process, the value
(a) DH < DE (b) DH = DE of q can be calculated by the expression
(c) DH > DE (d) DH = DE + RTDn V2 V
32. During isothermal expansion of an ideal gas, its (a) q = 2.303nRT log (b) q = -2.303nRT log 2
V1 V1
(a) internal energy increases V1
(b) enthalpy decreases (c) q = - Pexp nRT log V (d) None of these
2
(c) enthalpy remains unaffected 40. The internal energy change when a system goes from state
(d) enthalpy reduces to zero. A to B is 40 kJ/mol. If the system goes from A to B by a
33. Assertion : At constant temperature and pressure, whatever reversible path and returns to state A by an irreversible path,
heat absorbed by the system, is used in doing work. what would be the net change in internal energy ?
Reason : Internal energy change is zero. (a) > 40 kJ (b) < 40 kJ (c) Zero (d) 40 kJ
(a) Assertion is correct, reason is correct; reason is a 41. Under isothermal condition for one mole of ideal gas what
correct explanation for assertion. is the ratio of work done under reversible to irreversible
(b) Assertion is correct, reason is correct; reason is not process, initially held at 20 atm undergoes expansion from
a correct explanation for assertion 1L to 2L, at 298K, under external pressure of 10 atm?
(c) Assertion is correct, reason is incorrect (a) 1.7 (b) 2.0 (c) 1.4 (d) 1.0
(d) Assertion is incorrect, reason is correct.
EBD_8350
84 CHEMISTRY

42. An ideal gas is allowed to expand both reversibly and 49. The following two reactions are known :
irreversibly in an isolated system. If Ti is the initial Fe2O3(s) + 3CO (g) ¾¾ ® 2Fe(s) + 3CO2(g); DH = –26.8 kJ
temperature and Tf is the final temperature, which of the FeO(s) + CO(g) ¾¾ ® Fe(s) + CO2(g); DH = –16.5 kJ
following statements is correct? The value of D H for the following reaction
(a) (Tf)rev = (Tf)irrev Fe2O3(s) + CO(g) ¾¾ ® 2FeO(s) + CO2(g) is;
(b) Tf = Ti for both reversible and irreversible processes (a) + 6.2 kJ (b) + 10.3 kJ
(c) (Tf)irrev > (Tf)rev (c) – 43.3 kJ (d) – 10.3 kJ
50. Assertion : When a solid melts, decrease in enthalpy is
(d) Tf > Ti for reversible process but Tf = Ti for irreversible
observed.
process
Reason : Melting of a solid is endothermic.
43. How many molecules of ATP, undergo hydrolysis to raise
(a) Assertion is correct, reason is correct; reason is a
the temperature of 180 kg of water which was originally at
correct explanation for assertion.
room temperature by 1°C ? C{P,m} water = 75.32 J/mol/K,
(b) Assertion is correct, reason is correct; reason is not
DH{P} for ATP hydrolysis= 7 kcal/mol a correct explanation for assertion
(a) 1.5 × 1025 (b) 2.00 × 1023 (c) Assertion is correct, reason is incorrect
(c) 3.4 × 10 25 (d) 4.0 × 1024 (d) Assertion is incorrect, reason is correct.
44. What is the amount of heat (in Joules) absorbed by 18 g 51. Hess’s law is used to calculate :
of water initially at room temperature heated to 100°C ? If (a) enthalpy of reaction. (b) entropy of reaction
10 g of Cu is added to this water , than decrease in (c) work done in reaction (d) All of the above
temperature (in Kelvin) of water was found to be? C (p,m) 52. Assertion : The value of enthalpy of neutralization of weak
for water 75.32 J/mol K ; C (p,m) for Cu = 24.47 J/mol K. acid and strong base is numerically less than 57.1 kJ.
(a) 5649, 369 (b) 5544, 324 Reason : All the OH– ions furnished by 1 g equivalent of
(c) 5278, 342 (d) 3425, 425 strong base are not completely neutralized.
45. The molar heat capacity of water at constant pressure is (a) Assertion is correct, reason is correct; reason is a
75 JK–1 mol–1. When 1kJ of heat is supplied to 100 g of correct explanation for assertion.
water, which is free to expand, the increase in temperature (b) Assertion is correct, reason is correct; reason is not
of water is a correct explanation for assertion
(a) 6.6 K (b) 1.2 K (c) 2.4 K (d) 4.8 K (c) Assertion is correct, reason is incorrect
(d) Assertion is incorrect, reason is correct.
Topic 3: Enthalpy Change, DrH of a Reacion – Reaction Enthalpy 53. Given that bond energies of H – H and Cl – Cl are
46. Which of the following is not true regarding thermo- 430 kJ mol– 1 and 240 kJ mol–1 respectively and DHf for HCl
chemical equations? is – 90 kJ mol– 1, bond enthalpy of HCl is
(a) The coefficients in a balanced thermo-chemical (a) 380 kJ mol–1 (b) 425 kJ mol–1
(c) 245 kJ mol–1 (d) 290 kJ mol–1
equation refer to the number of moles of reactants
54. From the following bond energies:
and products involved in the reaction H – H bond energy: 431.37 kJ mol–1
(b) The coefficients in a balanced thermo-chemical C = C bond energy: 606.10 kJ mol–1
equation refer to the number of molecules of reactants C – C bond energy: 336.49 kJ mol–1
and products involved in the reaction C – H bond energy: 410.50 kJ mol–1
(c) The numerical value of Dr H refers to the number of Enthalpy for the reaction,
moles of substances specified by an equation. H H H H
| | | |
(d) Standard enthalpy change D rH will have units as C = C + H - H ¾¾ ® H - C- C - H
kJ mol-1. | | | |
H H H H
47. If enthalpies of formation of C 2 H 4 (g) , CO2(g) and will be:
H 2 O(l) at 25°C and 1atm pressure are 52, – 394 and (a) – 243.6 kJ mol–1 (b) –120.0 kJ mol–1
– 286 kJ/mol respectively, the change in ethalpy for (c) 553.0 kJ mol–1 (d) 1523.6 kJ mol–1
combustion of C2H4 is equal to 55. Which of the following statement is incorrect ?
(a) The standard enthalpy of reaction is the enthalpy
(a) – 141.2 kJ/mol (b) – 1412 kJ/mol
change for a reaction when all the participating
(c) + 14.2 kJ/mol (d) + 1412 kJ/mol substances are in their standard states.
48. The enthalpy change for a reaction does not depend upon (b) The standard state of a substance at a specified
(a) use of different reactants for the same product temperature is its pure form at 1 bar.
(b) the nature of intermediate reaction steps (c) The standard state of solid iron at 298 K is pure iron
(c) the differences in initial or final temperatures of at 1 atm
involved substances (d) Standard conditions are denoted by adding the
(d) the physical states of reactants and products superscript d- to the symbol DH e.g., –DH
THERMODYNAMICS 85

56. What is the internal energy (kJ) change occurs when 36 g (iii) CO2(g) + H2(g) = CO(g) + H2O(l)
of H2O(l) at 100°C converted to H2O(g)?
DH = – X3 kJ mol–1
DH° (vapourisation) = 40.79kJ/mol
(a) 75.38 (b) 80.98 (c) 70.98 (d) 45.89 5
(iv) C 2 H 2 (g) + O2 (g) = 2CO2(g) + H2O(l)’
57. Consider the reaction : 2
4NO2 (g) + O2 (g) ® 2N 2O5 (g), DH = + 4X4 kJ mol–1
DrH = – 111kJ.
Enthalpy of formation of H2O (l) is
If N2O5(s) is formed instead of N2O5(g) in the above
(a) + X3 kJ mol– 1 (b) – X4 kJ mol– 1
reaction, the DrH value will be : – 1
(given, DH of sublimation for N2O5 is –54 kJ mol–1) (c) + X1 kJ mol (d) – X2 kJ mol– 1
(a) + 54 kJ (b) + 219 kJ (c) – 219 J (d) – 165 kJ 63. Which of the following statements is true for the given
58. The standard enthalpy of formation (Df H° 298) for methane, reaction?
CH4 is – 74.9 kJ mol–1. In order to calculate the average Na ( s ) ® Na ( g ) ; DHd- = 108.4 kJ mol –1
energy given out in the formation of a C – H bond from this (a) The enthalpy of atomization is same as the enthalpy
it is necessary to know which one of the following? of vaporisation
(a) The dissociation energy of the hydrogen molecule, H2. (b) The enthalpy of atomization is same as the enthalpy
(b) The first four ionisation energies of carbon. of sublimation.
(c) The dissociation energy of H2 and enthalpy and (c) The enthalpy of atomization is same as the bond
sublimation of carbon (graphite). enthalpy
(d) The first four ionisation energies of carbon and (d) The enthalpy of atomization is same as the enthalpy
electron affinity of hydrogen. of solution
64. Diborane is a potential rocket fuel which undergoes
Topic 4: Enthalpies for Different Types of Reactions combustion according to the equation
59. Given that heat of neutralisation of strong acid and strong B2 H 6 (g) + 3O 2 (s) ¾¾ ® B2 O3 (s) + 3H 2O(g)
base is – 57.1 kJ. The heat produced when 0.25 mole of HCl is Calculate the enthalpy change for the combustion of
neutralised with 0.25 mole of NaOH in aqueous solution is : diborane. Given
(a) 14.275 kJ (b) 57.1 kJ 3
(c) 22.5 kJ (d) 28.6 kJ (i) 2B(s) + O 2 (g) ¾¾
® B2 O3 (s); DH = –1273 kJ per mol
2
60. Match the columns
Column-I Column-II 1
(ii) H 2 (g) + O 2 (g) ¾¾
® H 2O(l); DH = –286 kJ per mol
2
13
A. C4 H10 + O ® (p) Enthalpy of atomisation
2 2 ® H 2 O(g); DH = 44 kJ per mol
(iii) H 2 O(l) ¾¾
4CO2 + 5H2O; DH = – w
® B2H6 (g); DH = 36 kJ per mol
(iv) 2B (s) + 3H2 (g) ¾¾
B. CH4 ® C + 4H ; DH = x (q) Enthalpy of formation (a) + 2035 kJ per mol (b) – 2035 kJ per mol
C. H2+ Br2 ® 2HBr ; DH = y (r) Enthalpy of combustion (c) + 2167 kJ per mol (d) – 2167 kJ per mol
D. CO2 (s) ® CO2 (g) ; DH = z (s) Enthalpy of sublimation
Topic 5: Spontaneity, Gibbs Energy Change and
(a) A – (s), B – (p), C – (q), D – (r)
(b) A – (q), B – (r), C – (p), D – (s) Equilibrium Constant
(c) A – (r), B – (p), C – (q), D – (s) 65. Pick out the wrong statement
(d) A – (p), B – (q), C – (s), D – (r) (a) The standard free energy of formation of all elements
61. Standard enthalpy of vapourisation Dvap H° for water at is zero
100°C is 40.66 kJ mol–1. The change in internal energy of (b) A process accompanied by decrease in entropy is
vaporisation of water at 100°C (in kJ mol–1) is : spontaneous under certain conditions
(a) + 37.56 (b) – 43.76 (c) + 43.76 (d) + 40.66 (c) The entropy of a perfectly crystalline substance at
absolute zero is zero
(Assume water vapour to behave like an ideal gas).
(d) A process that leads to increase in free energy will be
62. Consider the following reactions:
spontaneous
(i) H +(aq) + OH–(aq) = H2O(l), 66. A chemical reaction will be spontaneous if it is accompanied
DH = – X1 kJ mol–1 by a decrease of
1 (a) entropy of the system.
(ii) H2(g) + O (g) = H2O(l), (b) enthalpy of the system.
2 2
(c) internal energy of the system.
DH = – X2 kJ mol–1
(d) free energy of the system.
EBD_8350
86 CHEMISTRY

67. In which of the following, entropy decreases? Reason : Entropy of the system increases with increase in
(a) Crystallization of sucrose solution temperature.
(b) Rusting of iron (a) Assertion is correct, reason is correct; reason is a
(c) Melting of ice correct explanation for assertion.
(d) Vaporization of camphor (b) Assertion is correct, reason is correct; reason is not
68. Identify the correct statement regarding entropy. a correct explanation for assertion
(a) At absolute zero temperature, entropy of a perfectly (c) Assertion is correct, reason is incorrect
crystalline substance is taken to be zero. (d) Assertion is incorrect, reason is correct.
(b) At absolute zero temperature, the entropy of a 76. A reaction is spontaneous at low temperature but non-
perfectly crystalline substance is positive. spontaneous at high temperature. Which of the following
(c) Absolute entropy of a substance can not be is true for the reaction?
determined. (a) DH > 0, DS > 0 (b) DH < 0, DS > 0
(d) At 0°C, the entropy of a perfectly crystalline (c) DH > 0, DS = 0 (d) DH < 0, DS < 0
substance is taken to be zero 77. At the sublimation temperature, for the process
69. Choose the reaction with negative DS value. ˆˆ† CO2(g)
CO2(s) ‡ˆˆ
(a) 2NaHCO3(s) ® Na2CO3(s) + CO2(g) + H2O(g) (a) DH, DS and DG are all positive
(b) Cl2(g) ® 2Cl(g) (b) DH > 0, DS > 0 and DG < 0
(c) DH < 0, DS > 0 and DG < 0
(c) 2SO2(g) + O2(g) ® 2SO3(g)
(d) DH > 0, DS > 0 and DG = 0
(d) 2KClO3(s) ® 2KCl(s) + 3O2(g) 78. Match the columns
70. Identify the correct statement regarding a spontaneous Column-I Column-II
process: (A) DH = –ve ; DS = –ve (p) Reaction will be
(a) Lowering of energy in the process is the only criterion DG = –ve non-spontaneous at
for spontaneity. high temperature
(b) For a spontaneous process in an isolated system, the (B) DH = –ve ; DS = –ve (q) Reaction will be
change in entropy is positive. DG = +ve non-spontaneous at
(c) Endothermic processes are never spontaneous. low temperature
(d) Exothermic processes are always spontaneous. (C) DH = +ve ; DS = +ve (r) Reaction will be
71. For the gas phase reaction, DG = +ve spontaneous at low
ˆˆ† PCl3(g) + Cl2(g)
PCl5(g) ‡ˆˆ temperature
(D) DH = +ve ; DS = +ve (s) Reaction will be
which of the following conditions are correct ? DG = –ve spontaneous at high
(a) DH = 0 and DS < 0 (b) DH > 0 and DS > 0 temperature
(c) DH < 0 and DS < 0 (d) DH > 0 and DS < 0
(a) A – (q), B – (r), C – (p), D – (s)
72. Match the columns
Column-I Column-II (b) A – (r), B – (p), C – (q), D – (s)
(A) Exothermic (p) DH = 0, DE = 0 (c) A – (r), B – (q), C – (s), D – (p)
(B) Spontaneous (q) DG = 0 (d) A – (q), B – (s), C – (p), D – (r)
(C) Cyclic process (r) DH is negative 79. Assertion : An exothermic process which is non-
(D) Equilibrium (s) DG is negative spontaneous at high temperature may become
(a) A – (q), B – (r), C – (p), D – (s) spontaneous at a low temperature.
(b) A – (s), B – (p), C – (r), D – (q) Reason : There occurs a decrease in entropy factor as the
(c) A – (p), B – (q), C – (s), D – (r) temperature is decreased.
(d) A – (r), B – (s), C – (p), D – (q) (a) Assertion is correct, reason is correct; reason is a
73. Unit of entropy is correct explanation for assertion.
(a) JK–1 mol–1 (b) J mol–1 (b) Assertion is correct, reason is correct; reason is not
–1 –1 –1 a correct explanation for assertion
(c) J K mol (d) JK mol–1
(c) Assertion is correct, reason is incorrect
74. Considering entropy (S) as a thermodynamic parameter,
(d) Assertion is incorrect, reason is correct.
the criterion for the spontaneity of any process is 80. What is the equilibrium constant if ATP hydrolysis by
(a) DSsystem + DSsurroundings > 0 water produces standard free energy of –50 kJ/mol under
(b) DSsystem - DSsurroundings > 0 normal body conditions ?
(a) 2.66 × 108 (b) 5.81 × 108
(c) DSsystem > 0 only 7
(c) 1.18 × 10 (d) 1.98 × 108
(d) DSsurroundings > 0 only 81. A reaction with DH = 0, is found to be spontaneous. This
75. Assertion : Many endothermic reactions that are not is due to
spontaneous at room temperature become spontaneous (a) DS is negative (b) DS is positive
at high temperature. (c) TDS is positive (d) Both (b) and (c)
THERMODYNAMICS 87

82. In conversion of lime-stone to lime, 84. Identify the correct statement for change of Gibbs energy
CaCO3 (s) ® CaO(s) + CO 2 (g) the values of DH° and DS° for a system (DGsystem) at constant temperature and pressure
(a) If DGsystem = 0, the system has attained equilibrium
are + 179.1 kJ mol–1 and 160.2 J/K respectively at 298 K and
(b) If DGsystem = 0, the system is still moving in a particular
1 bar. Assuming that DH° and DS° do not change with
direction
temperature, temperature above which conversion of
(c) If DGsystem < 0, the process is not spontaneous
limestone to lime will be spontaneous is
(d) If DGsystem > 0, the process is not spontaneous
(a) 1118 K (b) 1008 K
(c) 1200 K (d) 845 K. 85. In an irreversible process taking place at constant T and P
83. A reaction occurs spontaneously if and in which only pressure-volume work is being done, the
(a) TDS < DH and both DH and DS are + ve change in Gibbs free energy (dG) and change in entropy
(b) TDS > DH and DH is + ve and DS is - ve (dS), satisfy the criteria
(c) TDS > DH and both DH and DS are + ve (a) (dS)V, E > 0, (dG)T, P < 0 (b) (dS)V, E = 0, (dG)T, P = 0
(d) TDS = DH and both DH and DS are + ve (c) (dS)V, E = 0, (dG)T, P > 0 (d) (dS)V, E < 0, (dG)T, P < 0

1. A gas present in a cylinder fitted with a frictionless piston is the % ionization of the weak acid in molar solution
expands against a constant pressure of 1 atm from a volume (assume the acid to be monobasic)?
of 2 litre to a volume of 6 litre. In doing so, it absorbs 800 6. The heat of sublimation of iodine is 24 cal g -1 at 50°C. If
J heat from surroundings. Determine increase in internal specific heat of solid iodine and its vapour are 0.055 and
energy of process. 0.031cal g -1 respectively, Calculate the heat of sublima-
2. The heat of neutralisation of strong base and strong acid tion of iodine at 100°C.
is 57.0 kJ. Calculate the heat released when 0.5 mole of 7. When 0.2 mole of anhydrous CuSO4 is dissolved in water,
HNO3 is added to 0.20 mole of NaOH solution.
the heat evolved is 1.451 kcal. If 0.2 mole of CuSO 4 .5H 2O
3. Given
is dissolved in water, the heat absorbed is 0.264 kcal.
Reaction Energy Change
Calculate the molar heat of hydration of CuSO4.
(in kJ) 8. The specific heat of a monoatomic gas at constant
Li(s) ® Li(g) 161 pressure is 248.2 J kg–1 K–1 and at constant volume it is
Li(g) ® Li+(g) 520 149.0 J kg–1 K–1. Find the mean molar mass of the gas.
1 9. The standard entropies of CO 2 (g), C(s) and O2(g) are
F (g) ® F(g) 77
2 2 213.5, 5.740 and 205 JK–1 respectively. Calculate the
F(g) + e– ® F–(g) (Electron gain standard entropy of formation of CO2(g).
enthalpy) 10. Titanium metal is extensively used in aerospace industry
Li+ (g) + F–(g) ® Li F(s) – 1047 because the metal imparts strength to structures but does
1 not unduly add to their masses. The metal is produced by
Li (s) + F (g) ® Li F(s) – 617 the reduction of TiCl4(l) which in turn is produced from
2 2
mineral rutile TiO2(s). Calculate the Gibb’s free energy for
Based on data provided, find the value of electron gain the following reaction
enthalpy of fluorine. TiO2(s) + 2Cl2(g) ® TiCl4(l) + O2(g)
4. The standard enthalpy of formation of NH3 is – 46.0 kJ/mol.
If the enthalpy of formation of H2 from its atoms is – 436 kJ/ Given that: Hf° for TiO2 (s), TiCl 4 (l ), Cl 2 (g)
mol and that of N2 is – 712 kJ/mol, find the average bond
and O2 (g)are - 944.7, -804.2, 0.0, 0.0 kJ mol-1
enthalpy of N – H bond in NH3.
5. The enthalpy of neutralization of a weak acid in 1 M solution Also S° for TiO 2 (g), TiCl4 (l ), Cl2 (g)
with a strong base is –56.1kJ mol–1. If enthalpy of ionization
of the acid is 1.5 kJ mol–1 and enthalpy of neutralization of and O2 (g)are50.3, 252.3, 233.0, 205.1J mol –1 K –1
the strong acid with a strong base is –57.3 kJ equiv–1, what respectively.
EBD_8350
88 CHEMISTRY

Exercise 3 : NCERT Exemplar & Past Year MCQs


NCERT Exemplar MCQs 6. Df U° of formation of CH4(g) at certain temperature is
1. Thermodynamics is not concerned about –393 kJ mol–1. The value of Df H° is

(a) energy changes involved in a chemical reaction (a) zero (b) < DfU°
(b) the extent to which a chemical reaction proceeds (c) > Df U ° (d) equal to Df U°
(c) the rate at which a reaction proceeds 7. In an adiabatic process, no transfer of heat takes place
between system and surroundings. Choose the correct
(d) the feasibility of a chemical reaction
option for free expansion of an ideal gas under adiabatic
2. Which of the following statement is correct? condition from the following.
(a) The presence of reacting species in a covered beaker (a) q = 0, DT ¹ 0, W = 0
is an example of open system.
(b) q ¹ 0, DT = 0, W = 0
(b) There is an exchange of energy as well as matter between
the system and the surroundings in a closed system. (c) q = 0, DT = 0, W = 0
(c) The presence of reactants in a closed vessel made up (d) q = 0, DT < 0, W ¹ 0
of copper is an example of a closed system.
8. The pressure-volume work for an ideal gas can be calculated
(d) The presence of reactants in a thermos flask or any
other closed insulated vessel is an example of a closed Vf
system. by using the expression W = - ò pex dV . The work can also
Vi
3. The state of a gas can be described by quoting the
relationship between
be calculated from the pV-plot by using the area under the
(a) pressure, volume, temperature curve within the specified limits. When an ideal gas is
(b) temperature, amount, pressure compressed (a) reversibly or (b) irreversibly from volume
(c) amount, volume, temperature Vi to Vf . Choose the correct option.
(d) pressure, volume, temperature, amount (a) W (reversible) = W (irreversible)
4. The volume of gas is reduced to half from its original (b) W (reversible) < W (irreversible)
volume. The specific heat will be
(c) W (reversible) > W (irreversible)
(a) reduce to half (b) be doubled
(d) W (reversible) = W (irreversible) + pex.DV
(c) remain constant (d) increase four times
9. The entropy change can be calculated by using the
5. During complete combustion of one mole of butane,
2658 kJ of heat is released. The thermochemical reaction qrev
expression DS = . When water freezes in a glass
for above change is T
(a) 2C4 H10 (g) + 13O 2 (g) ® 8CO 2 (g) + 10H 2 O ( l) ; beaker, choose the correct statement amongst the
following.
Dc H = -2658.0 kJ mol -1
(a) DS (system) decreases but DS (surroundings) remains
13 the same
(b) C4 H10 ( g ) + O 2 ( g ) ® 4CO2 ( g ) + 5H 2 O ( l ) ;
2 (b) DS (system) increases but DS (surroundings)
-1 decreases
Dc H = -1329.0 kJ mol
(c) DS (system) decreases but DS (surroundings)
13
(c) C4 H10 ( g ) + O 2 ( g ) ® 4CO2 ( g ) + 5H 2 O ( l ) ; increases
2
(d) DS (system) decreases but DS (surroundings) also
-1 decreases
Dc H = -2658.0 kJ mol
13 10. On the basis of theromochemical equations (1), (2) and (3),
(d) C4 H10 ( g ) + O 2 ( g ) ® 4CO2 ( g ) + 5H 2 O ( l ) ; find out which of the algebraic relationships given in
2
options (a) to (d) is correct
Dc H = +2658.0 kJ mol -1
THERMODYNAMICS 89

1. C (graphite) + O2 (g) ® CO2 (g); DrH = x kJ mol–1 16. For the reaction : [AIPMT 2014, S]

1 X2O4(l) ® 2XO2(g)
2. C (graphite) + O2 (g) ® CO (g); DrH = y kJ mol–1
2 DU = 2.1 k cal, DS = 20 cal K–1 at 300 K
Hence DG is:-
1
3. CO (g) + O2 (g) ® CO2 (g); DrH = z kJ mol–1 (a) 2.7 k cal (b) – 2.7 k cal
2
(c) 9.3 k cal (d) – 9.3 k cal
(a) z=x+y (b) x = y – z
17. For complete combustion of ethanol,
(c) x=y+z (d) y = 2z – x
C 2 H 5 OH(l) + 3O 2 (g) ¾¾
® 2CO 2 (g) + 3H 2 O(l),
11. Consider the reactions given below. On the basis of these
reactions find out which of the algebraic relationship given the amount of heat produced as measured in bomb
in options (a) to (d) is correct? calorimeter, is 1364.47 kJ mol–1 at 25ºC. Assuming ideality
1. C (graphite) + 4 H (g) ® CH4 (g); DrH = x kJ mol–1 the enthalpy of combustion, DcH, for the reaction will be:

2. C (graphite) + 2H2 (g) ® CH4 (g); DrH = y kJ mol–1 (R = 8.314 kJ mol–1) [JEE M 2014, S]

(a) x = y (b) x = 2y (a) – 1366.95 kJ mol–1 (b) – 1361.95 kJ mol–1

(c) x > y (d) x < y (c) – 1460.95 kJ mol–1 (d) – 1350.50 kJ mol–1

12. The enthalpies of elements in their standard states are 18. The heat of combustion of carbon to CO2 is – 393.5 kJ/mol.
taken as zero. The enthalpy of formation of a compound The heat released upon formation of 35.2 g of CO2 from
carbon and oxygen gas is [AIPMT 2015, S]
(a) is always negative
(a) –315 kJ (b) +315 kJ
(b) is always positive
(c) –630 kJ (d) –3.15 kJ
(c) may be positive or negative
19. The correct thermodynamic conditions for the
(d) is never negative spontaneous reaction at all temperatures is
13. Enthalpy of sublimation of a substance is equal to [NEET 2016, C]
(a) enthalpy of fusion + enthalpy of vaporisation (a) DH < 0 and DS = 0 (b) DH > 0 and DS < 0
(b) enthalpy of fusion (c) DH < 0 and DS > 0 (d) DH < 0 and DS < 0
(c) enthalpy of vaporisation 20. The heats of combustion of carbon and carbon monoxide
are –393.5 and –283.5 kJ mol–1, respectively. The heat of
(d) twice the enthalpy of vaporisation
formation (in kJ) of carbon monoxide per mole is :
14. Which of the following is not correct?
[JEE M 2016, S]
(a) DG is zero for a reversible reaction. (a) 676.5 (b) – 110.5
(b) DG is positive for a spontaneous reaction. (c) 110.5 (d) 676.5
(c) DG is negative for a spontaneous reaction. 21. A gas is allowed to expand in a well insulated container
(d) DG is positive for a non-spontaneous reaction. against a constant external pressure of 2.5atm from an initial
volume of 2.50 L to a final volume of 4.50L. The change in
Past Year MCQs
internal energy DU of the gas in joules will be:-
15. Which of the following statements is correct for the
[NEET 2017, S]
spontaneous adsorption of a gas ? [AIPMT 2014, C]
(a) – 500 J (b) – 505 J
(a) DS is negative and, therefore, DH should be highly
(c) + 505 J (d) 1136.25 J
positive
22. For a given reaction, DH = 35.5 kJ mol –1 and DS = 83.6
(b) DS is negative and therefore, DH should be highly
JK –1 mol –1. The reaction is spontaneous at : (Assume that
negative
DH and DS do not vary with tempearature)
(c) DS is positive and, therefore, DH should be negative [NEET 2017, S]
(d) DS is positive and, therefore, DH should also be highly (a) T > 425 K (b) All temperatures
positive
(c) T > 298 K (d) T < 425 K
EBD_8350
90 CHEMISTRY

23. DU is equal to [JEE M 2017, C] 29. Among the following, the set of parameters that represents
path functions, is: [JEE M 2019, S]
(a) Isochoric work
A. q + w B. q
(b) Isobaric work
C. w D. H – TS
(c) Adiabatic work
(a) B and C
(d) Isothermal work
(b) B, C and D
24. The bond dissociation energies of X2, Y2 and XY are in
the ratio of 1 : 0.5 : 1. DH for the formation of XY is –200 kJ (c) A and D
mol–1. The bond dissociation energy of X2 will be (d) A, B and C
[NEET 2018, S] 30. Consider the reversible isothermal expansion of an ideal
(a) 200 kJ mol–1 (b) 100 kJ mol–1 gas in a closed system at two different temperatures T1
and T2 (T1 < T2). The correct graphical depiction of the
(c) 400 kJ mol–1 (d) 800 kJ mol–1 dependence of work done (w) on the final volume (V) is:
25. The combustion of benzene (l) gives CO2 (g) and H2O (l). [JEE M 2019, S]
Given that heat of combustion of benzene at constant
volume is –3263.9 kJ mol–1 at 25°C; heat of combustion
w T2 w T2
(in kJ mol–1) of benzene at constant pressure will be :
(R = 8.314 JK–1 mol–1) [JEE M 2018, S] T1 T1
(a) 4152.6 (b) – 452.46 (a) (b)
(c) 3260 (d) – 3267.6
O In V O InV
26. Which of the following lines correctly show the
temperature dependence of equilibrium constant, K, for an
exothermic reaction? [JEE M 2018, S] w T2 w T2

A
T1
ln K T1
1
(c) (d)
B
(0, 0) T(K)
++ O InV O InV
++
++ C
++
D 31. For the reaction, 2Cl(g) ¾® Cl2(g), the correct option is :
(a) DrH > 0 and DrS < 0 [NEET 2020, C]
(a) A and B (b) B and C
(b) DrH < 0 and DrS > 0
(c) C and D (d) A and D
(c) DrH < 0 and DrS < 0
27. In which case change in entropy is negative ?
(d) DrH > 0 and DrS > 0
[NEET 2019, C]
32. The correct option for free expansion of an ideal gas under
(a) Evaporation of water adiabatic condition is [NEET 2020, C]
(b) Expansion of a gas at constant temperature (a) q = 0, DT < 0 and w > 0
(c) Sublimation of solid to gas (b) q < 0, DT = 0 and w = 0
(d) 2H(g) ® H2(g) (c) q > 0, DT > 0 and w > 0
28. Under isothermal condition, a gas at 300 K expands front (d) q = 0, DT = 0 and w = 0
0.1 L to 0.25 L against a constant external pressure of 2 bar.
The work done by the gas is [Given that 1 L bar = 100 J] 33. For the reaction ; [NV, JEE M 2020, A]

[NEET 2019, A] A(l) ¾® 2B(g)

(a) –30 J (b) 5 kJ DU = 2.1 kcal, DS = 20 cal K–1 at 300 K.

(c) 25 J (d) 30 J Hence G in kcal is _______.


THERMODYNAMICS 91

Exercise 4 : Problem Solving Skill Enhancer MCQs

1. Enthalpy of sublimation of iodine is 24 cal g–1 at 200°C. If (a) 2.196 m3 (b) 1.196 m3
specific heat of I2(s) and I2(vap) are 0.055 and 0.031 cal (c) 4.196 m3 (d) 3.196 m3
g–1K–1 respectively, then enthalpy of sublimation of iodine
7. If bond enthalpies of N º N, H – H and N – H bonds are x1,
at 250°C in cal g–1 is :
x2 and x3 respectively, DH°f for NH3 will be
(a) 2.85 (b) 5.7 (c) 22.8 (d) 11.4
(a) x1 + 3x2 – 6x3 (b) ½x1 + 3/2x2 – 3x3
2. Which of the following expressions is true for an ideal
(c) 3x3 – ½x1 – 3/2x2 (d) 6x3 – x1 – 3x2
gas?
8. If the ratio of molar heat capacities of a gas at constant
æ dV ö æ dP ö CP
(a) çè ÷ =0 (b) çè ÷ø = 0 pressure and constant volume i.e., = g , the respective
dT ø P dT V CV

æ dU ö æ dU ö values of CP and CV are


(c) çè ÷ =0 (d) çè ÷ =0
dV ø T dT ø V R gR gR R
(a) , (b) ,
g -1 g -1 g -1 g -1
æ 5 ö
3. 0.5 mole each of two ideal gases A ç Cv , m = R ÷ and
è 2 ø g -1 g -1 g -1 g -1
B (Cv,m = 3R) are taken in a container and expanded (c) , (d) ,
gR R R gR
reversibly and adiabatically, during this process
temperature of gaseous mixture decreased from 350 K to 9. Heat of neutralization of a strong acid HA and a weaker
250 K. Find DH (in cal/mol) for the process: acid HB with KOH are – 13.7 and – 12.7 k cal mol–1. When
(a) – 100 R (b) – 137.5 R 1 mole of KOH was added to a mixture containing 1 mole
each of HA and HB, the heat change was – 13.5 kcal. In
(c) – 375 R (d) None of these
what ratio is the base distributed between HA and HB ?
4. A heating coil is immersed in a 100 g sample of H2O (l) at
(a) 3 : 1 (b) 1 : 3
1 atm and 100°C in a closed vessel. In this heating process,
60% of the liquid is converted into gaseous form at (c) 4 : 1 (d) 1 : 4
constant pressure of 1 atm. Densities of liquid and gaseous 10. The bond dissociation energies of CH4 and C 2H 6
water under these conditions are 1000 kg/m 3 and respectively are 360 and 620 k cal mol–1. The C – C bond
0.60 kg/m3 respectively. Magnitude of the work done for energy would be
the process is : (a) 260 k cal/mol (b) 180 k cal/mol
(a) 4997 J (b) 4970 J (c) 130 k cal/mol (d) 80 k cal/mol
(c) 9994 J (d) None of these 11. Given that:
5. For the reaction taking place at certain temperature
(i) Df H ° of N2 O is 82 kJ mol–1
ˆˆ† 2NH3 ( g ) + CO 2 ( g ) ,
NH 2 COONH 4 ( s ) ‡ˆˆ (ii) Bond energies of N º N, N=N, O = O and N = O are
if equilibrium pressure is 3X bar then DrG° would be 946, 418, 498 and 607 kJ mol–1 respectively,
(a) – RT ln 9 – 3RT ln X The resonance energy of N2O is :
(b) RT ln 4 – 3RT ln X (a) – 88 kJ (b) – 66 kJ
(c) – 3RT ln X (c) –62 kJ (d) –44 kJ
(d) None of these 12. Calculate enthalpy change for the change
6. The heat of combustion of ethane gas is 368 kcal/mol. 8S(g) ¾® S8(g), given that
Assuming that 60% of the heat is useful, how many m3 of ® 2H(g) + 2S(g), DH = 239.0 k cal mol -1 ;
H 2S2 (g) ¾¾
ethane measured at STP must be burnt to supply enough
heat to convert 50 kg of water at 10oC to steam at 100oC? ® 2H(g) + S(g), DH = 175.0 k cal mol - 1
H 2 S(g) ¾¾
(Specific heat of water is 1 cal/g, heat of vaporisation of (a) + 512.0 k cal (b) – 512.0 k cal
H2O is 540 cal/g)
(c) 508.0 k cal (d) – 508.0 k cal
EBD_8350
92 CHEMISTRY

13. Fixed mass of an ideal gas contained in a 24.63 L sealed 15. If H is considered as the function of P and T, then which of
rigid vessel at 1 atm is heated from – 73°C to 27°C. Calculate the following relations is /are correct?
change in Gibb's energy if entropy of gas is a function of
temperature as S = 2 + 10–2 T (J/K): (Use 1 atm L = 0.1 kJ) æ dH ö æ dH ö
(a) dH = ç ÷ dT + ç ÷ dP
(a) 1231.5 J (b) 1281.5 J è dT ø P è dP øT
(c) 781.5 J (d) 0
æ dH ö
14. The densities of graphite and diamond at 298 K are 2.25 (b) dH = C p dT + ç ÷ dP
è dP øT
and 3.31 g cm–3, respectively. If the standard free energy
difference (DGº) is equal to 1895 J mol–1, the pressure at
æ dH ö
which graphite will be transformed into diamond at 298 K is (c) ç ÷ =0
è d P øT
(a) 9.92 × 105 Pa (b) 11.094 × 108 Pa
(c) 10.952 × 107 Pa (d) 9.92 × 106 Pa (d) all

ANSWER KEY
Exercise 1 : NCERT Based Topic-wise MCQs
1 (b) 10 (a) 19 (d) 28 (d) 37 (a) 46 (b) 55 (c) 64 (b) 73 (a) 82 (a)
2 (c) 11 (c) 20 (d) 29 (a) 38 (d) 47 (b) 56 (a) 65 (d) 74 (a) 83 (c)
3 (b) 12 (d) 21 (b) 30 (b) 39 (a) 48 (b) 57 (d) 66 (d) 75 (b) 84 (a)
4 (c) 13 (d) 22 (c) 31 (b) 40 (c) 49 (a) 58 (a) 67 (a) 76 (d) 85 (a)
5 (d) 14 (c) 23 (a) 32 (c) 41 (a) 50 (d) 59 (a) 68 (a) 77 (d)
6 (c) 15 (a) 24 (d) 33 (a) 42 (c) 51 (a) 60 (c) 69 (c) 78 (b)
7 (a) 16 (b) 25 (a) 34 (a) 43 (a) 52 (c) 61 (a) 70 (b) 79 (b)
8 (a) 17 (c) 26 (b) 35 (b) 44 (a) 53 (b) 62 (d) 71 (b) 80 (a)
9 (d) 18 (c) 27 (b) 36 (b) 45 (c) 54 (b) 63 (b) 72 (d) 81 (b)
Exercise 2 : Numeric/Integer Answer Questions
1 (395) 2 (11.4) 3 (–328) 4 (–964) 5 (20) 6 (22.8) 7 (–17.15) 8 (83.8) 9 (2.76) 10 (158)
Exercise 3 : NCERT Exemplar & Past Year MCQs
1 (c) 5 (c) 9 (c) 13 (a) 17 (a) 21 (b) 25 (d) 29 (a) 33 (d)
2 (c) 6 (b) 10 (c) 14 (b) 18 (a) 22 (a) 26 (a) 30 (b)
3 (d) 7 (c) 11 (c) 15 (b) 19 (c) 23 (c) 27 (d) 31 (–2.70)
4 (c) 8 (b) 12 (c) 16 (b) 20 (b) 24 (d) 28 (a) 32 (c)
Exercise 4 : Problem Solving Skill Enhancer MCQs
1 (c) 3 (c) 5 (d) 7 (b) 9 (c) 11 (a) 13 (c) 15 (d)
2 (c) 4 (c) 6 (d) 8 (b) 10 (d) 12 (b) 14 (b)
7 Equilibrium

Trend Buster NEET & JEE Main

Number of Questions from 2020-15 21 9 Minimum three questions have been


Weightage 6.6% 5.10% asked every year in NEET & JEE M.

The most Important Concepts that Cover Maximum number of Questions asked in past 6 years.

Equilibrium constant / relation between equilibrium 7 4


constant and free energy / reaction quotient /
Le-chatelier's principle
Solubility Product/Molar Solubility 6 1
Ionisation of Acids and Bases/pH of the Solution 2 4
Buffer Solution 3 —
Less Important Concepts that Cover 1 or 2 Questions asked in past 6 years.

Heterogenous equilibrium 2 —
Conjugate acid and bases/salts 1 —

NEET JEE

2020 Acid, bases and salt/solubility of pH of the solution / Solubility / 2 Average 1 Difficult
sparingly soluble salt/ Relation Relation between free energy
between eqilibrium constant and and equilibrium constant
Gibbs energy
2019 Buffer solutions, solubility of Buffer solutions/Ksp/ionisation
sparingly solubles alt/acid, of acid and bases/acid 3 Average 2 Average
bases and salt and bases
2018 Law of chemical equilibrium/ Equilibrium constant/pH of the
acid,bases and salt/application/ solution/Le-chatelier 3 Average 2 Average
factor affecting eqilibria/buffer principle/Ksp
solution/solubility of sparingly
soluble salt
2017 Buffer solutions,solubility of buffer solution/solubility
sparingly soluble salt/law of product/equilibrium 3 Average 1 Average
chemical equilibrium/acid, constant/ionisation of acid and
bases and salt bases/acid and bases
2016 Application/factors affecting Le-chatelier’s principle /
equilibrium/buffer solution, molar solubility/ 3 Difficult 1 Average
solubility of springly soluble equilibrium constant/
salt/law of chemical equilibrium/ heterogenous equilibria
homogenous-heterogenous
equilibrium
2015 Buffer solutions,solubility of Buffer solution/solubility product/
sparingly soluble salt/acid, ionisation of acid and bases/ 6 Average 2 Average
bases and salt/application/factor reaction quotient/equilibrium
affecting equilibrium/reaction constant
quotient/ equilibrium constant
EBD_8350
94 CHEMISTRY
EQUILIBRIUM 95
EBD_8350
96 CHEMISTRY

Problem Solving Tips/ Tricks/ Points to Remember

4 Solubility Product (Ksp) 4 Calculation of pH Value of Buffers


The constant Ksp is known as solubility product. It is [salt]
equal to the product of the concentration of ions in · Acidic buffer pH = pK a + log
[acid]
saturated solution. This is known as Henderson's equation
For general electrolyte AxBy. [salt]
Ksp = [A+y]x [B–x]y · Basic buffer pOH = pK b + log
[base]
4 Factors Affecting Equilibrium Concentrations: A buffer has maximum buffer capacity when
Temperature [salt] [salt]
• An increase in temperature for exothermic or =1
[acid] [base]
reactions decreases the concentration of
products at equilibrium and vice-versa. 4 Variation of equilibrium constant with variation of the
• An increase in temperature for endothermic reaction equation (K = equilibrium constant for original
reactions increases the concentration of reaction) is given as:
products at equilibrium and vice-versa. When the equation the change in equilibrium constant
Pressure Reversed 1/K
• An increase in pressure to an equilibrium Divided by 2 K
favours the reaction where number of moles Multiplied by 2 K2
Divided into two steps K = K1 ×K2
show a decrease and vice-versa.
Effect or pressure (or volume) 4 Buffer capacity
Moles of acid or a base added to 1 litre of buffer
Effect of reaction Effect of pressure =
(or volume) Change in pH
4 Calculation of Degree of Dissociation
· I. D n = 0 Equilibrium is not affected
Let us consider the following reaction:
· II. D n = +ve Increase in P (or decrease in V) PCl5 (g) ƒ PCl3 (g) + Cl2 (g)
shifts the equilibrium to left Initial pressure and temperature are P1 and T1. At equilibrium,
pressure and temperature are P2 and T2, respectively. The
· III. D n = –ve Increase in P (or decrease in V) volume of vessel is constant, equal to ‘V’.
shifts the equilibrium to right
PCl5(g) PCl3(g) + Cl2(g) n V T P
MP t=0 a 0 0 a V T1 P1
4 The density of one mole of gas is given by D =
RT teq a – aa aa aa a + aa V T2 P2
Consider dissociation of PCl 5. Let x be degree of
dissociation Initial state P1V = aRT1 ... (i)
D 1+ x D-d æ 1 ö Equilibrium state P2V = (a + aa)RT2 ... (ii)
\ = \x = çQ D µ ÷ Dividing equations (i) and (ii), we get
d 1 d è moles ø
P1 T1 TP
where D is the theoretical vapour density and d is < (1 ∗ a) < 1 2
observed vapour density at temperature t °C. P2 (1 ∗ a)T2 T2 P1
1 T1P2 , T2 P1
D = Molecular mass \ a=
2 T2 P1
If nx moles of products are formed, then total number of
Alternate Method:
moles after dissociation
Initial pressure after change
1 – x + nx = 1 + x (n – 1) Initial moles of temperature
D 1 + x ( n - 1) D-d <
\ = \x = Moles at equilibrium Equilibrium pressure
d 1 d ( n - 1)
EQUILIBRIUM 97

Exercise 1 : NCERT Based Topic-wise MCQs


Topic 1: Equilibrium in Physical Processes, Equilibrium in (a) A – (p) ; B – (r) ; C – (q) ; D – (s)
Chemical Processes - Dynamic Equilibrium (b) A – (q) ; B – (s) ; C – (r) ; D – (p)
(c) A – (s) ; B – (q) ; C – (p) ; D – (r)
1. Which of the following is not a general characteristic of (d) A – (r) ; B – (s) ; C – (q) ; D – (p)
equilibria involving physical processes? 7. If the synthesis of ammonia from Haber's process is carried
(a) Equilibrium is possible only in a closed system at a out with exactly the same starting conditions (of partial
given temperature. pressure and temperature) but using D2 (deuterium) in place
(b) All measurable properties of the system remain of H2. Then
constant. (a) the equilibrium will be disturbed
(c) All the physical processes stop at equilibrium. (b) the composition of reaction mixture will remain same
(d) The opposing processes occur at the same rate and at equilibrium.
there is dynamic but stable condition. (c) Use of isotope in reaction will not produce ammonia.
2. A reaction is said to be in equilibrium when (d) At equilibrium rate of forward reaction will be greater
(a) the rate of transformation of reactant to products is than the rate of reverse reaction
equal to the rate of transformation of products to the 8. Consider the following graph and mark the correct
reactants. statement.
(b) 50% of the reactants are converted to products.

¾¾®
(c) the reaction is near completion and all the reactants HI
are converted to products.
HI
(d) the volume of reactants is just equal to the volume of Concentration
the products. H 2, I2 H 2, I 2
3. A small amount of acetone is taken in a watch glass and it
is kept open in atmosphere. Which statement is correct for
the given experiment?
(a) The rate of condensation from vapour to liquid state Time ¾¾® Equilibrium ¾¾® Time
is higher than the rate of evaporation. ˆˆ† 2HI
(a) Chemical equilibrium in the reaction, H2 + I2 ‡ˆˆ
(b) The rate of condensation from vapour to liquid state can be attained from either directions.
is equal to the rate of evaporation. (b) Equilibrium can be obtained when H2 and I2 are mixed
(c) The rate of condensation from vapour to liquid state in an open vessel.
is much less than the rate of evaporation. (c) The concentrations of H2 and I2 keep decreasing while
(d) The rate of condensation from vapour to liquid state concentration of HI keeps increasing with time.
is equal or less than the rate of evaporation. (d) We can find out equilibrium concentration of H2 and
4. Which of the following is not true about a reversible I2 from the given graph.
reaction? 9. You must have seen that when a soda water bottle is
(a) The reaction does not proceed to completion opened, some of the carbon dioxide gas dissolved in it
(b) It cannot be influenced by a catalyst fizzes out rapidly. There is equilibrium between the
(c) Number of moles of reactants and products is always molecules in the gaseous state and the molecules dissolved
in the liquid under pressure i.e.,
equal
(d) It can be attained only in a closed container CO2(gas) ‡ˆˆˆˆ† CO (in solution)
2
5. If a system is at equilibrium, the rate of forward to the Which of the following statements is/are correct regarding
reverse reaction is : this?
(a) less (b) equal (i) The phenomenon arises due to difference in solubility
(c) high (d) at equilibrium of carbon dioxide at different pressures.
6. Match the Column-I with Column-II and mark the (ii) This equilibrium is governed by Henry's law.
appropriate choice. (iii) The amount of CO2 gas dissolved in liquid increases
Column-I Column-II with decrease of temperature.
(A) Liquid ‡ˆˆˆˆ† Vapour (p) Saturated solution (iv) The amount of CO2 gas dissolved in liquid decreases
ˆˆ† Liquid with increase of temperature.
(B) Solid ‡ˆˆ (q) Boiling point
ˆˆ† Vapour (a) only (iii) is correct
(C) Solid ‡ˆˆ (r) Sublimation point
ˆˆ† Solute (b) (i) , (iii) and (iv) are correct
(D) Solute (s) ‡ˆˆ (s) Melting point
(c) (i), (ii) and (iii) are correct
(solution)
(d) All are correct
EBD_8350
98 CHEMISTRY

Topic 2: Law of Chemical Equilibrium and Equilibrium ˆˆ† N (g) + O (g)


(a) 2NO(g) ‡ˆˆ 2 2
Constant, Homogenous Equilibria, Heterogenous Equilibria ˆˆ† SO (g) + NO(g)
(b) SO (g) + NO (g) ‡ˆˆ
2 2 3
10. The equilibrium constant for the reversible reaction ˆˆ† 2HI(g)
(c) H2(g) + I2(g) ‡ˆˆ
ˆˆ† 2NH is K and for reaction
N2 + 3H2 ‡ˆˆ ˆˆ† 2CO (g)
(d) 2C(s) + O (g) ‡ˆˆ
3 2 2
1 3 18. ˆˆ† 2CO(g) , the partial
For the reaction C(s) + CO 2 (g) ‡ˆˆ
ˆˆ† NH , the equilibrium constant is K¢
N 2 + H 2 ‡ˆˆ 3
2 2 pressures of CO2 and CO are 2.0 and 4.0 atm respectively
The K and K¢ will be related as: at equilibrium. The Kp for the reaction is.
(a) K × K¢ = 1 (b) K = K¢ (a) 0.5 (b) 4.0 (c) 8.0 (d) 32.0
(c) K ' = K (d) K = K ' 19. The Kp/Kc ratio will be highest in case of
11. The rate constant for forward and backward reaction of 1
(a) CO (g) + O2 (g) ‡ˆˆ ˆˆ† CO2 (g)
hydrolysis of ester are 1.1 ´ 10–2 and 1.5 ´ 10–3 per minute 2
respectively. Equilibrium constant for the reaction ˆˆ† 2HI (g)
(b) H2 (g) + I2 (g) ‡ˆˆ
CH 3COOC 2 H 5 + H + ‡ˆˆ ˆˆ† CH 3COOH + C2 H5OH is ˆˆ†
(c) PCl5 (g) ‡ˆˆ PCl3(g) + Cl2 (g)
(a) 4.33 (b) 5.33 (c) 6.33 (d) 7.33 (d) 7H2 (g) + 2NO2 (g) ‡ˆˆˆˆ† 2NH3 (g) + 4H2O(g)

12. ˆˆ†
A reaction is A + B ‡ˆˆ C + D. Initially we start with 20. Steam reacts with iron at high temperature to give hydrogen
equal concentrations of A and B. At equilibrium we find gas and Fe 3 O4 (s). The correct expression for the
that the moles of C is two times of A. What is the equilibrium equilibrium constant is
constant of the reaction? ( pH 2 ) 2 ( pH 2 ) 4
1 1 (a) (b)
(a) (b) (c) 4 (d) 2 ( pH 2 O ) 2 ( pH 2 O ) 4
4 2
13. In the following equilibrium reaction ( pH 2 )4 [ Fe3O4 ] [Fe3O 4 ]
2A ‡ˆˆˆ†
ˆ B + C, (c) (d)
the equilibrium concentrations of A, B and C are 1 × 10–3 M,
4
( pH 2O ) [ Fe] [Fe]
2 × 10–3 M and 3 × 10–3 M respectively at 300 K. The value 21. Unit of equilibrium constant for the given reaction is
of Kc for this equilibrium at the same temperature is ˆˆ† Ni (CO) (g)
Ni(s) + 4CO(g) ‡ˆˆ 4
1 1
(a) (b) 6 (c) (d) 36 (a) (mol/L)–3 (b) (mol/L)3
6 36 (c) (mol/L)–4 (d) (mol/L)4
14. Given the reaction between 2 gases represented by A2 and 22. For the following three reactions (i), (ii) and (iii), equilibrium
B2 to give the compound AB(g). constants are given:
A2(g) + B2(g) ƒ 2 AB(g).
At equilibrium, the concentration (i) CO(g) + H 2O(g) ƒ CO2 (g) + H 2 (g); K1
of A2 = 3.0 × 10–3 M (ii) CH 4 (g) + H 2O(g) ƒ CO(g) + 3H 2 (g); K 2
of B2 = 4.2 × 10–3 M
of AB = 2.8 × 10–3 M (iii) CH 4 (g) + 2H 2O(g) ƒ CO2 (g) + 4H 2 (g); K3
lf the reaction takes place in a sealed vessel at 527 °C, then
the value of Kc will be : (a) K1 K 2 = K3 (b) K2K3 = K1
(a) 2.0 (b) 1.9 (c) 0.62 (d) 4.5 (c) K3 = K1 K2 (d) K3 × K 23 = K12
15. The thermal dissociation of calcium carbonate showing 23. For the reversible reaction,
heterogeneous equilibrium is
N2(g) + 3H2(g) 2NH3(g) at 500°C, the value of Kp is
ˆˆ† CaO(s) + CO 2 (g)
CaCO3 (s) ‡ˆˆ
1.44 × 10 –5 when partial pressure is measured in
For this reactions which of the following is/are true atmospheres. The corresponding value of Kc, with
(i) Kc = [CO2(g)] concentration in mol litre–1, is
(ii) Kp = pCO
2 1.44 ´10 -5 1.44 ´10 -5
(iii) [CaCO3(s)] and [CaO(s)] are both constant (a) (b)
(iv) [CO2(g)] is constant
(0.082 ´ 500)- 2 (8.314 ´ 773)- 2
(a) (i), (ii) and (iv) (b) (i), (ii) and (iii) 1.44 ´10 -5 1.44 ´10 -5
(c) (d)
(c) (ii) and (iv) (d) (i), (iii) and (iv) (0.082 ´ 773)2 (0.082 ´ 773)-2
16. ˆˆ† C. The unit of equilibrium constant is :
In A + B ‡ˆˆ 24. Assertion : Kp can be less than, greater than or equal to
(a) Litre mol–1 (b) Mol litre Kc.
(c) Mol litre–1 (d) No unit Reason : Relation between Kp and Kc depends on the
17. In which of the following equilibrium Kc and Kp are not change in number of moles of gaseous reactants and
equal? products (Dn).
EQUILIBRIUM 99

(a) Assertion is correct, reason is correct; reason is a 1 1 1 1


(a) K1 = = (b) K1 = =
correct explanation for assertion. K 2 K3 K 2 ( K )2
3
(b) Assertion is correct, reason is correct; reason is not 1
a correct explanation for assertion (c) K1 = K 2 = K3 (d) K1 = = K3
K2
(c) Assertion is correct, reason is incorrect
(d) Assertion is incorrect, reason is correct. 32. The dissociation equilibrium of a gas AB 2 can be
25. For a chemical reaction ; represented as :
A (g) + B (l) ƒ D (g) + E (g) ˆˆ† 2AB(g) + B2 (g)
2AB2 (g) ‡ˆˆ
Hypothetically at what temperature, Kp = Kc The degree of dissociation is ‘x’ and is small compared to
(when, R = 0.08 I-atm/mol-K) 1. The expression relating the degree of dissociation (x)
(a) T = 0 K (b) T = 1K with equilibrium constant Kp and total pressure P is :
(c) T = 12.5 K (d) T = 273 K
(a) (2Kp/P) (b) (2Kp/P)1/3
26. For the reaction C(s) + CO2(g) ® 2CO(g), Kp = 63 atm at 1/2
1000 K. If at equilibrium : PCO = 10 PCO , then the total (c) (2Kp/P) (d) (Kp/P)
pressure of the gases at equilibrium is
2 33. The values of Kp1 and Kp2 for the reactions
(a) 6.3 atm (b) 6.93 atm (c) 0.63 atm (d) 0.693 atm ˆˆ† Y + Z
X ‡ˆˆ ...(1)
27. Equilibrium constant (K) for the reaction ˆˆ†
and A ‡ˆˆ 2 B ...(2)
ˆˆ† Ni(CO) (g) can be written in terms
Ni(s) + 4CO(g) ‡ˆˆ are in the ratio of 9 : 1. If degree of dissociation of X and A
4
of be equal, then total pressure at equilibrium (1) and (2) are
ˆˆ† Ni(CO) (g); in the ratio :
(1) Ni(s) + 2CO2 (g) + 2C(s) ‡ˆˆ 4
(a) 3 : 1 (b) 1 : 9 (c) 36 : 1 (d) 1 : 1
equilibrium constant = K1.
ˆˆ† 2CO(g);
(2) CO (g) + C(s) ‡ˆˆ Topic 3: Applications of Equilibrium Constants,
2
equilibrium constant = K2. Relationship between Equilibrium Constant (K),
What is the relation between K, K1 and K2? Reaction Quotient (Q) and Gibbs Energy (G)
(a) K = (K1)/(K2)2 (b) K = (K1 . K2) Factors Affecting Equilibria
(c) K = (K1) (K2) 2 (d) K = K1/K2 34. In a reversible chemical reaction having two reactants, in
28. Two moles of PCl5 were heated in a closed vessel of 2L. At equilibrium if the concentration of the reactants are doubled
equilibrium 40% of PCl5 is dissociated into PCl3 and Cl2. then the equilibrium constant will
The value of equilibrium constant is (a) Also be doubled (b) Be halved
(a) 0.53 (b) 0.267 (c) 2.63 (d) 5.3 (c) Become one-fourth (d) Remain the same
29. PCl5 is dissociating 50% at 250°C at a total pressure of 35. If for the reaction
P atm. If equilibrium constant is Kp, then which of the N2 + 3H2 ‡ˆˆ ˆˆ† 2NH3, DH = –92.38 kJ/mol, then what
following relation is numerically correct ? happens if the temperature is increased?
(a) Kp = 3P (b) P = 3Kp (a) Reaction proceed forward
(b) Reaction proceed backward
2Kp 2P
(c) P = (d) Kp = (c) No effect on the formation of product
3 3 (d) None of these
30. For the decomposition of the compound, represented as 36. If Kc is in the range of ………… appreciable concentrations
of both reactants and products are present.
ˆˆ† 2NH 3 (g) + CO 2 (g)
NH 2COONH 4 (s) ‡ˆˆ (a) 10–4 to 104 (b) 10 – 3 to 103
3
(c) 10 to 10 6 (d) 10 – 5 to 103
the Kp = 2.9 × 10–5 atm3.
If the reaction is started with 1 mol of the compound, the 37. The reaction quotient (Q) for the reaction
total pressure at equilibrium would be : ˆˆ† 2NH 3 (g)
N 2 (g) + 3H 2 (g) ‡ˆˆ
(a) 1.94 × 10–2 atm (b) 5.82 × 10–2 atm [NH 3 ]2
–2
(c) 7.66 × 10 atm (d) 38.8 × 10–2 atm is given by Q = . The reaction will proceed
31. K1, K2 and K3 are the equilibrium constants of the following [N 2 ][H 2 ]3
reactions (I), (II) and (III) respectively: from right to left if
(I) N2 + 2O2 2NO2 (a) Q = 0 (b) Q = Kc (c) Q < Kc (d) Q > Kc
where Kc is the equilibrium constant
(II) 2NO2 N2 + 2O2 38. On doubling P and V with constant temperature the
1 equilibrium constant will
(III) NO2 N 2 + O2 (a) remain constant (b) become double
2
(c) become one-fourth (d) None of these
The correct relation from the following is
EBD_8350
100 CHEMISTRY

39. The reaction quotient Q is used to (a) A – (p), B – (q), C – (r)


(a) predict the extent of a reaction on the basis of its (b) A – (r), B – (q), C – (p)
magnitude (c) A – (p), B – (r), C – (q)
(b) predict the direction of the reaction (d) A – (q), B – (p), C – (r)
(c) calculate equilibrium concentrations 46. According to Le-chatelier’s principle, adding heat to a solid
(d) calculate equilibrium constant ˆˆ† liquid equilibrium will cause the
‡ˆˆ
40. The equilibrium which remains unaffected by pressure (a) temperature to increase
change is (b) temperature to decrease
ˆˆ† 2NO(g)
(a) N 2 (g) + O2 (g) ‡ˆˆ (c) amount of liquid to decrease
(b) 2SO2 (g) + O2 (g) ‡ˆˆ ˆˆ† 2SO3 (g) (d) amount of solid to decrease
(c) 2O3 (g) ‡ˆˆˆˆ† 3O2 (g) 47. Which of the following reaction will be favoured at low
pressure?
(d) 2NO2 (g) ‡ˆˆ ˆˆ† N 2 O4 (g)
ˆˆ† 2HI
(a) H 2 + I 2 ‡ˆˆ
41. Using the equation (K = e–DG°/RT), the reaction spontaneity ˆˆ† 2NH3
(b) N 2 + 3H 2 ‡ˆˆ
can be interpreted in terms of the value of DG° is/are
(a) If DG° < 0, then –DG°/RT is positive, and e–DG°/RT > 1 (c) PCl5 ‡ˆˆˆˆ† PCl3 + Cl 2
making K > 1, which implies a spontaneous reaction ˆˆ† 2NO
(d) N 2 + O 2 ‡ˆˆ
or the reaction which proceeds in the forward direction 48. For the manufacture of ammonia by the reaction
to such an extent that the products are present ˆˆ† 2NH3 + 2 kcal
N2 + 3H2 ‡ˆˆ
predominantly. the favourable conditions are
(b) If DG° > 0, then –DG°/RT is negative, and e–DG°/RT < 1 (a) Low temperature, low pressure and catalyst
making K < 1, which implies a non-spontaneous (b) Low temperature, high pressure and catalyst
reaction or a reaction which proceeds in the forward (c) High temperature, low pressure and catalyst
direction to such a small degree that only a very minute (d) High temperature, high pressure and catalyst
quantity of product is formed.
49. In a vessel N2, H2 and NH3 are at equilibrium. Some helium
(c) Both (a) and (b)
gas is introduced into the vessel so that total pressure
(d) None of the above
increases while temperature and volume remain constant.
42. The correct relationship between free energy change in a
According to Le Chatelier’s principle, the dissociation of NH3
reaction and the corresponding equilibrium constant, Kc is
(a) increases (b) decreases
(a) DG = RT ln Kc (b) –DG = RT ln Kc
(c) remains unchanged (d) equilibrium is disturbed
(c) DG° = RT ln Kc (d) –DG° = RT ln Kc
50. In a two-step exothermic reaction
43. Which one of the following information can be obtained
on the basis of Le Chatelier principle? A2(g) + B2(g) 3C (g) D (g)
Step 1 Step 2
(a) Dissociation constant of a weak acid
Steps 1 and 2 are favoured respectively by
(b) Entropy change in a reaction
(a) high pressure, high temperature and low pressure,
(c) Equilibrium constant of a chemical reaction
low temperature
(d) Shift in equilibrium position on changing value of a
(b) high pressure, low temperature and low pressure, high
constraint
temperature
44. Assertion : If a volume is kept constant and an inert gas
(c) low pressure, high temperature and high pressure,
such as argon is added which does not take part in the
high temperature
reaction, the equilibrium remains undisturbed.
(d) low pressure, low temperature and high pressure, low
Reason : It is because the addition of an inert gas at
temperature
constant volume does not change the partial pressure or
51. The exothermic formation of ClF3 is represented by the
the molar concentrations of the substance involved in the
equation :
reaction.
ˆˆ† 2ClF (g); DH = – 329 kJ
Cl2(g) + 3F2(g) ‡ˆˆ
(a) Assertion is correct, reason is correct; reason is a 3
correct explanation for assertion. Which of the following will increase the quantity of ClF3
(b) Assertion is correct, reason is correct; reason is not in an equilibrium mixture of Cl2, F2 and ClF3?
a correct explanation for assertion (a) Adding F2
(c) Assertion is correct, reason is incorrect (b) Increasing the volume of the container
(d) Assertion is incorrect, reason is correct. (c) Removing Cl2
45. Match the columns (d) Increasing the temperature
Column-I Column-II 52. Read the following statements and choose the correct option
(A) Qc < Kc, (p) Net reaction goes from
(i) The numerical value of the equilibrium constant for a
right to left.
reaction indicates the extent of the reaction.
(B) Qc > Kc, (q) Net reaction goes from
left to right. (ii) An equilibrium constant give information about the
(C) Qc = Kc, (r) No net reaction occurs. rate at which the equilibrium is reached.
EQUILIBRIUM 101

(iii) If Kc > 103, products predominate over reactants, i.e., 62. Calculate the pOH of a solution at 25°C that contains
if Kc is very large, the reaction proceeds nearly to 1× 10– 10 M of hydronium ions, i.e., H3O+.
completion. (a) 4.000 (b) 9.0000 (c) 1.000 (d) 7.000
(iv) If Kc < 10–3, reactants predominate over products, 63. A weak acid, HA, has a Ka of 1.00 × 10–5. If 0.100 mole of
i.e., if Kc is very small, the reaction proceeds rarely. this acid dissolved in one litre of water, the percentage of
(a) (i), (ii) and (iv) are correct acid dissociated at equilbrium is closest to
(b) (i) , (iii) and (iv) are correct (a) 1.00% (b) 99.9% (c) 0.100% (d) 99.0%
(c) (i), (ii) and (iii) are correct 64. A monobasic weak acid solution has a molarity of 0.005
(d) only (iii) is correct and pH of 5. What is the percentage ionization in this
53. Suitable conditions for melting of ice : solution?
(a) high temperature and high pressure (a) 2.0 (b) 0.2 (c) 0.5 (d) 0.25
(b) high temperature and low pressure 65. Equimolar solutions of HF, HCOOH and HCN at 298 K
(c) low temperature and low pressure have the values of Ka as 6.8 × 10–4, 1.8 × 10–4 and 4.8 × 10–9
(d) low temperature and high pressure respectively. What is the observed trend of dissociation
constants in successive stages ?
Topic 4: Acids, Bases and Salts, Ionisation of (a) HF > HCN > HCOOH (b) HF > HCOOH > HCN
Acids and Bases (c) HCN > HF > HCOOH (d) HCOOH > HCN > HF
54. A base, as defined by Bronsted theory, is a substance 66. Which of the following pKa value represents the strongest
which can acid ?
(a) lose a pair of electrons (b) donate protons (a) 10–4 (b) 10–8 (c) 10–5 (d) 10–2
(c) gain a pair of electrons (d) accept protons 67. The dissociation constant of two acids HA1 and HA2 are
55. Conjugate acid of NH -2 is : 3.14 × 10– 4 and 1.96 × 10– 5 respectively. The relative
(a) NH4+ (b) NH3 (c) NH2 (d) NH strength of the acids will be approximately
(a) 1 : 4 (b) 4 : 1 (c) 1 : 16 (d) 16 : 1
56. Which of the following molecules acts as a Lewis acid ?
68. At 298K a 0.1 M CH3COOH solution is 1.34% ionized. The
(a) (CH3)2 O (b) (CH3)3 P
ionization constant Ka for acetic acid will be
(c) (CH3)3 N (d) (CH3)3 B (a) 1.82 × 10– 5 (b) 18.2 × 10 –5
57. Which one of the following molecular hydrides acts as a (c) 0.182 × 10 –5 (d) None of these
Lewis acid? 69. In qualitative analysis, in III group NH4Cl is added before
(a) NH3 (b) H2O (c) B2H6 (d) CH4 NH4OH because
58. Water is well known amphoprotic solvent. In which (a) to increase the concentration of NH4+ions
chemical reaction water is behaving as a base only? (b) to increase concentration of Cl– ions
(a) H 2SO 4 + H 2O ¾¾ ® H3O + + HSO 4 – (c) to reduce the concentration of OH– ions
(b) H 2 O + H 2 O ¾¾ ® H 3 O + + OH – (d) to increase concentration of OH– ions
(c) H2O + NH2 – ¾¾ ® NH3 + OH – 70. The solubility of AgI in NaI solution is less than that in
(d) H 2 O + NH 3 ¾¾ ® NH 4+ + OH – pure water because :
59. The value of the ionic product of water (a) the temperature of the solution decreases
(a) depends on volume of water (b) solubility product to AgI is less than that of NaI
(b) depends on temperature (c) of common ion effect
(c) changes by adding acid or alkali (d) AgI forms complex with NaI
(d) always remains constant 71. Which of the following statements about pH and H+ ion
60. Match the columns concentration is incorrect?
Column-I Column-II (a) Addition of one drop of concentrated HCl in NH4OH
(A) Hydrochloric acid (p) Lemon and orange solution decreases pH of the solution.
(b) A solution of the mixture of one equivalent of each of
(B) Acetic acid (q) Tamarind paste.
CH3COOH and NaOH has a pH of 7
(C) Citric and ascorbic (r) Digestive juice
(c) pH of pure neutral water is not zero
acids
(d) A cold and concentrated H2SO4 has lower H+ ion
(D) Tartaric acid (s) Constituent of vinegar concentration than a dilute solution of H2SO4
(a) A – (q), B – (r), C – (p), D – (s) 72. Which one of the following is the correct statement ?
(b) A – (r), B – (s), C – (p), D – (q)
(a) HCO3– is the conjugate base of CO32–.
(c) A – (s), B – (p), C – (q), D – (r)
(b) NH2– is the conjugate acid of NH3.
(d) A – (r), B – (p), C – (s), D – (q)
(c) H2SO4 is the conjugate acid of HSO4–.
61. What is the approximate pH of a 1 × 10–3 M NaOH solution?
(a) 3 (b) 11 (c) 7 (d) 1 × 10–11 (d) NH3 is the conjugate base of NH2–.
EBD_8350
102 CHEMISTRY

73. Match the columns 83. Equal volumes of three acid solutions of pH 3, 4 and 5 are
Column-I Column-II mixed in a vessel. What will be the H+ ion concentration in
(A) HClO4 (p) Strong base the mixture ?
(B) HNO2 (q) Strong acid (a) 1.11 × 10–4 M (b) 3.7 × 10–4 M
(C) NH2– (r) Weak base (c) 3.7 × 10– 3 M (d) 1.11× 10–3 M
(D) HSO4– (s) Weak acid 84. The pKa of a weak acid, HA, is 4.80. The pKb of a weak
(a) A – (s), B – (q), C – (p), D – (r) base, BOH, is 4.78. The pH of an aqueous solution of the
(b) A – (q), B – (s), C – (p), D – (r) correspondng salt, BA, will be
(c) A – (r), B – (p), C – (q), D – (s)
(a) 9.58 (b) 4.79
(d) A – (s), B – (q), C – (p), D – (r)
74. Three reactions involving H2PO4– are given below: (c) 7.01 (d) 9.22
(i) H3PO4 + H2O® H3O+ + H2PO4– 85. Which of the following statements are correct ?
(ii) H2PO4– + H2O ® HPO42– + H3O+ (i) Ionic product of water (Kw) = [H+] [OH–] = 10–14 M2
(iii) H2PO4– + OH– ® H3PO4 + O2– (ii) At 298K [H+] = [OH–] = 10–7 M
In which of the above does H 2 PO4- act as an acid ? (iii) Kw does not depends upon temperature
(a) (ii) only (b) (i) and (ii) (iv) Molarity of pure water = 55.55 M
(c) (iii) only (d) (i) only (a) (i), (ii) and (iii) (b) (i), (ii) and (iv)
75. The pH of a 10–3 M HCl solution at 25°C if it is diluted 1000 (c) (i) and (iv) (d) (ii) and (iii)
times, will be – 86. H2S gas when passed through a solution of cations
(a) 3 (b) zero (c) 5.98 (d) 6.02 containing HCl, precipitates the cations of second
76. How many litres of water must be added to 1 litre of an group of qualitative analysis but not those belonging
aqueous solution of HCl with a pH of 1 to create an aqueous to the fourth group. It is because
solution with pH of 2 ?
(a) presence of HCl decreases the sulphide ion
(a) 0.1 L (b) 0.9 L (c) 2.0 L (d) 9.0 L
77. The pH value of a 10 M solution of HCl is concentration.
(a) less than 0 (b) equal to 0 (b) solubility product of group II sulphides is more
(c) equal to 1 (d) equal to 2 than that of group IV sulphides.
78. What is the H+ ion concentration of a solution prepared (c) presence of HCl in creases the sulphide ion
by dissolving 4 g of NaOH (Atomic weight of Na = 23 amu) concentration.
in 1000 ml? (d) sulphides of group IV cations are unstable in HCl.
(a) 10–10 M (b) 10–4 M (c) 10–1 M (d) 10–13 M 87. 100 mL of 0.04 N HCl aqueous solution is mixed with 100
79. Calculate the pH of a solution obtained by diluting 1 mL of mL of 0.02 N NaOH solution. The pH of the resulting
0.10 M weak monoacidic base to 100 mL at constant solution is:
temperature if Kb of the base is 1 × 10–5 ? (a) 1.0 (b) 1.7
(a) 8 (b) 9 (c) 10 (d) 11 (c) 2.0 (d) 2.3
80. At 25°C, the dissociation constant of a base, BOH, is 88. At 100°C the Kw of water is 55 times its value at 25°C.
1.0 ´ 10 -12. The concentration of hydroxyl ions in What will be the pH of neutral solution? (log 55 = 1.74)
0.01 M aqueous solution of the base would be (a) 6.13 (b) 7.00
(a) 1.0 ´ 10-5 mol L-1 (b) 1.0 ´ 10-6 mol L-1 (c) 7.87 (d) 5.13
-6
(c) 2.0 ´ 10 mol L -1 (d) 1.0 ´ 10-7 mol L-1 89. Values of dissociation constant, Ka are given as follows :
81. Which equilibrium can be described as an acid-base reaction Acid Ka
using the Lewis acid-base definition but not using the HCN 6.2 × 10–10
Bronsted-Lowry definition? HF 7.2 × 10–4
(a) 2NH3 + H2SO4 ‡ˆˆ ˆˆ† 2NH + + SO 2– HNO2 4.0 × 10–4
4 4
(b) NH3 + CH3COOH ‡ˆˆ ˆˆ† NH + + CH COO– Correct order of increasing base strength of the base
4 3
(c) H2O + CH3COOH ‡ˆˆ ˆˆ† H O+ + CH COO– CN–, F– and NO2– will be :
3 3 _
(d) [Cu(H2O)4]2+ + 4 NH3 ‡ˆˆ ˆˆ† [Cu(NH ) ]2+ + 4H O (a) F- < CN - < NO2
_
82. Given
3 4 2
(b) NO2 < CN - < F-
_
HF + H 2O ¾¾a¾
K
® H 3O+ + F - (c) F- < NO2 < CN -
_
(d) NO2 < F- < CN -
F - + H 2O ¾¾b¾ ® HF + OH -
K
Which of the following reaction is correct 90. If degree of dissociation of pure water at 100°C is
1 1.8 × 10–8, then the dissociation constant of water will be
(a) Kb = Kw (b) Kb =
Kw (density of H2O = 1 g/cc)
Ka (a) 1 × 10–12 (b) 1 × 10–14
(c) Ka × Kb = Kw (d) = Kw (c) 1.8 × 10 –12 (d) 1.8 × 10–14
Kb
EQUILIBRIUM 103

91. Among boron trifluoride, stannic chloride and stannous 99. What is [H+] in mol/L of a solution that is 0.20 M in
chloride, Lewis acid is represented by CH3COONa and 0.10 M in CH3COOH ? Ka for CH3COOH
(a) only boron trifluoride = 1.8 × 10–5.
(b) boron trifluoride and stannic chloride (a) 3.5 × 10–4 10 -4 (b) 1.1 × 10–5
(c) boron trifluoride and stannous chloride (c) 1.8 × 10 –5 (d) 9.0 × 10–6
(d) all three 100. Which of the following pairs constitutes a buffer?
92. An acid/ base dissociation equilibrium is dynamic (a) NaOH and NaCl (b) HNO3 and NH4NO3
involving a transfer of proton in forward and reverse directions. (c) HCl and KCl (d) HNO2 and NaNO2
Now, with passage of time in which direction equilibrium is 101. The Ksp for Cr(OH)3 is 1.6 × 10–30. The solubility of this
favoured? compound in water is :
(a) in the direction of stronger base and stronger acid (a) 4
1.6 ´ 10 -30 (b) 4
1.6 ´ 10 -30 / 27
(b) in the direction of formation of stronger base and
weaker acid (c) 1.6 ´ 10-30 / 27 (d) 1.6 ´ 10 -30
(c) in the direction of formation of weaker base and 102. The product of ionic concentration in a saturated solution
weaker acid of an electrolyte at a given temperature is constant and is
(d) in the direction of formation of weaker base and known as
stronger acid (a) Ionic product of the electrolyte
93. In HS–, I–, RNH2 and NH3, order of proton accepting (b) Solubility product
tendency will be (c) Ionization constant
(a) I– > NH3 > RNH2 > HS– (d) Dissociation constant
(b) HS– > RNH2 > NH3 > I– 103. If s and S are respectively solubility and solubility product
(c) RNH2 > NH3 > HS– > I– of a sparingly soluble binary electrolyte then :
(d) NH3 > RNH2 > HS– > I– 1
94. Ionisation constant of CH 3COOH is 1.7 × 10 –5 if (a) s = S (b) s = S2 (c) s = S1/2 (d) s = S
2
concentration of H+ ions is 3.4 × 10–4M, then find out 104. A buffer solution is prepared by mixing 0.1 M ammonia
initial concentration of CH3COOH molecules and 1.0 M ammonium chloride. At 298 K, the pKb of NH4OH
(a) 3.4 × 10–4 M (b) 3.4 × 10–3 M is 5.0. The pH of the buffer is
(c) 6.8 × 10 M–3 (d) 6.8 × 10–4 M (a) 10.0 (b) 9.0 (c) 6.0 (d) 8.0
95. The dissociation constants for acetic acid and HCN at
105. The pKa of a weak acid (HA) is 4.5. The pOH of an aqueous
25°C are 1.5 × 10–5 and 4.5 × 10–10 respectively. The
buffer solution of HA in which 50% of the acid is ionized is
equilibrium constant for the equilibrium
(a) 7.0 (b) 4.5 (c) 2.5 (d) 9.5
CN– + CH3COOH ƒ HCN + CH3COO– would be: 106. Buffer solutions have constant acidity and alkalinity
(a) 3.3 × 10– 5 (b) 3.3 × 10– 4 because
(c) 3.3 × 10 4 (d) 3.3 × 105 (a) these give unionised acid or base on reaction with
96. Ionisation of weak acid can be calculated by the formula added acid or alkali.
100 (b) acids and alkalies in these solutions are shielded from
Ka attack by other ions.
(a) 100 (b)
C 1 + 10(pK a - pH) (c) they have large excess of H+ or OH– ions
(c) Both (a) and (b) (d) None of these (d) they have fixed value of pH
97. Which of the following statements are correct ? 107. At 25°C, the solubility product of Mg(OH)2 is 1.0 × 10–11.
(i) The extent of dissociation of an acid depends on the At which pH, will Mg2+ ions start precipitating in the form
strength and polarity of the H –– A bond (where A is of Mg(OH)2 from a solution of 0.001 M Mg2+ ions?
an electronegative element.) (a) 9 (b) 10 (c) 11 (d) 8
(ii) As the strength of H –– A bond increases, the energy 108. When a buffer solution of sodium acetate and acetic acid,
required to break the bond decreases. is diluted with water :
(iii) As the electronegativity difference between the atoms (a) Acetate ion concentration increases
H and A increases, acidity increases (b) H+ ion concentration increases
(a) (i) and (ii) (b) (ii) and (iii) (c) OH– ion conc. increases
(c) (i) (d) (i), (ii) and (iii) (d) H+ ion concentration remains unaltered
Topic 5: Buffer Solutions, 109. pH of a saturated solution of Ba(OH)2 is 12. The value of
Solubility Equilibria of Sparingly Soluble Salts solubility product (Ksp) of Ba(OH)2 is :
(a) 3.3 × 10– 7 (b) 5.0 × 10–7
98. A buffer solution is prepared in which the concentration –6
(c) 4.0 × 10 (d) 5.0 × 10–6
of NH3 is 0.30M and the concentration of NH4+ is 0.20 M.
110. What is the molar solubility of Fe(OH)3 if Ksp = 1.0 × 10–38?
If the equilibrium constant, Kb for NH3 equals 1.8 × 10–5,
what is the pH of this solution ? (log 2.7 = 0.433). (a) 3.16 × 10–10 (b) 1.386 × 10–10
(a) 9.08 (b) 9.43 (c) 11.72 (d) 8.73 (c) 1.45 × 10–9 (d) 1.12 × 10–11
EBD_8350
104 CHEMISTRY

111. Why only As3+ gets precipitated as As2S3 and not Zn2+ as 114. Solubility product of silver bromide is 5.0 × 10–13. The
ZnS when H2S is passed through an acidic solution quantity of potassium bromide (molar mass taken as
containing As3+ and Zn2+? 120 g mol–1) to be added to 1 litre of 0.05 M solution of
(a) Solubility product of As2S3 is less than that of ZnS silver nitrate to start the precipitation of AgBr is
(b) Enough As3+ are present in acidic medium (a) 1.2 × 10–10 g (b) 1.2 × 10–9 g
(c) 6.2 × 10 g–5 (d) 5.0 × 10–8 g
(c) Zinc salt does not ionise in acidic medium
(d) Solubility product changes in presence of an acid 115. In qualitative analysis, the metals of Group I can be
112. Solid Ba(NO 3 ) 2 is gradually dissolved in a separated from other ions by precipitating them as chloride
1.0 × 10–4 M Na2CO3 solution. At which concentration of salts. A solution initially contains Ag+ and Pb2+ at a
Ba2+, precipitate of BaCO3 begins to form ? (Ksp for BaCO3 concentration of 0.10 M. Aqueous HCl is added to this
= 5.1 × 10–9) solution until the Cl– concentration is 0.10 M. What will
(a) 5.1 × 10–5 M (b) 7.1 × 10–8 M the concentrations of Ag+ and Pb2+ be at equilibrium?
(c) 4.1 × 10 M–5 (d) 8.1 × 1–7 M (Ksp for AgCl = 1.8 × 10–10, Ksp for PbCl2 = 1.7 × 10–5)
113. A buffer solution is prepared by mixing 10 ml of 1.0 M (a) [Ag+] = 1.8 × 10–7 M ; [Pb2+] = 1.7 × 10–6 M
CH3COOH and 20 ml of 0.5 M CH3COONa and then diluted (b) [Ag+] = 1.8 × 10–11 M ; [Pb2+] = 8.5 × 10–5 M
to 100 ml with distilled water. If pKa of CH3COOH is 4.76, (c) [Ag+] = 1.8 × 10–9 M ; [Pb2+] = 1.7 × 10–3 M
what is the pH of the buffer solution ? (d) [Ag+] = 1.8 × 10–11 M ; [Pb2+] = 8.5 × 10–4 M
(a) 5.8 (b) 4.34 (c) 5.21 (d) 4.76

1. Equilibrium constant Kp for the reaction 8. In the reaction AB(g) ƒ A(g) + B(g) at 30° C, Kp for the
ˆˆ† CaO(s) + CO (g) is 0.82 atm at 727 ºC.
CaCO3(s) ‡ˆˆ dissociation equilibrium is 1.6 × 10–3 atm. If the total
2
pressure at equilibrium is 1 atm, then calculate the
If 1 mole of CaCO3 is placed in a closed container of 20L percentage dissociation of AB.
and heated to this temperature, what amount of CaCO3 in 9. Calculate the pH at the equivalence point when a solution
grams would dissociate at equilibrium ? of 0.01 M CH3COOH is titrated with a solution of 0.01 M
2. An amount of solid NH4HS is placed in a flask already NaOH. pKa of CH3COOH is 4.74.
containing ammonia gas at a certain temperature and 1.0 10. The value of Kp for the equilibrium reaction
atm pressure. Ammonium hydrogen sulphide decomposes N 2O4 (g) ƒ 2NO 2 (g) is 2.
to yield NH3 and H2S gases in the flask. When the Calculate the percentage dissociation of N2O4(g) at a
decomposition reaction reaches equilibrium, the total pressure of 0.5 atm.
pressure in the flask rises to 2 atm. What will be the 11. A buffer solution is prepared by mixing 10 mL of 1.0 M
equilibrium constant for NH4HS decomposition at this CH3COOH and 20 mL of 0.5 M CH3COONa and then diluted
temperature? to 100 mL with distilled water. If pKa of CH3COOH is 4.76,
3. Calculate the pH of a solution obtained by diluting 1 mL of what is the pH of the buffer solution?
0.10 M weak monoacidic base to 100 mL at constant 12. At a certain temperature and 2 atm pressure equilibrium
temperature if Kb of the base is 1 × 10–5 . constant (KP) is 25 for the reaction
4. 28 g N2 and 6.0 g of H2 are heated over catalyst in a closed ˆˆ† SO3 (g) + NO(g)
SO 2 (g) + NO 2 (g) ‡ˆˆ
one litre flask of 450 °C. The entire equilibrium mixture Initially if we take 2 moles of each of the four gases and
required 500 mL of 1.0 M H2SO4 for neutralisation. Calculate 2 moles of inert gas, what would be the equilibrium partial
the value of Kc in L2 mol2 for the given reaction. pressure of NO2 in atm?
ˆˆ† 2NH3 (g)
N 2 (g) + 3H 2 (g) ‡ˆˆ 13. Find the pH of a 2 litre solution which is 0.1 M each with
respect to CH3COOH and (CH3OO)2Ba. (Ka = 1.8 × 10–5)
5. The equilibrium constant of the reaction of weak acid HA 14. On addition of increasing amount of AgNO3 to 0.1 M each
with strong base is 10–7. Find the pOH of the aqueous of NaCl and NaBr in a solution, what % of Br– ion gets
solution of 0.1M NaA. precipitated when Cl– ion starts precipitating?
6. For the reaction C(s) + CO2(g) ® 2CO(g), Kp = 63 atm at Given : Ksp (AgCl) = 1.0 × 10–10; Ksp (AgBr) = 1 × 10 –13
1000 K. If at equilibrium : PCO = 10 PCO . Find the total 15. What can be the maximum possible molarity of Co2+ ions
2
pressure (atm) of the gases at equilibrium. in 0.1 M HCl saturated with H2S (Ka = 4 × 10–21)? Given
7. The pKa of HCOOH is 3.8 and pKb of NH3 is 4.8, find the that Ksp for CoS is 2 × 10–21 and concentration of saturated
pH of aqueous solution of 1M HCOONH4. H2S = 0.1 M.
EQUILIBRIUM 105

Exercise 3 : NCERT Exemplar & Past Year MCQs


NCERT Exemplar MCQs 7. Acidity of BF3 can be explained on the basis of which of
the following concepts?
1. We know that the relationship between Kc and Kp is (a) Arrhenius concept
Kp = Kc (RT)Dn (b) Bronsted Lowry concept
What would be the value of Dn for the reaction? (c) Lewis concept
ˆˆ† NH 3 ( g ) + HI ( g )
NH 4 Cl ( s ) ‡ˆˆ (d) Bronsted Lowry as well as Lewis concept
8. Which of the following options will be correct for the
(a) 1 (b) 0.5 (c) 1.5 (d) 2 ˆˆ† B?
stage of half completion of the reaction A ‡ˆˆ
2. For the reaction, H 2 ( g ) + I2 ( g ) ‡ˆˆˆˆ† 2HI ( g ) , the – –
(a) DG = 0 (b) DG > 0
standard free energy is DG° > 0. The equilibrium constant –
(K) would be (c) DG < 0 (d) DG = –RT ln K
(a) K = 0 (b) K > 1 (c) K = 1 (d) K < 1 9. What will be the correct order of vapour pressure of water,
3. Which of the following is not a general characteristic of acetone and ether at 30°C? Given that among these
equilibria involving physical processes? compounds, water has maximum boiling point and ether
(a) Equilibrium is possible only in a closed system at a has minimum boiling point?
given temperature. (a) Water < ether < acetone
(b) All measurable properties of the system remain (b) Water < acetone < ether
constant. (c) Ether < acetone <water
(c) All the physical processes stop at equilibrium. (d) Acetone < ether < water
(d) The opposing processes occur at the same rate and 10. At 500 K, equilibrium constant, Kc, for the following
there is dynamic but stable condition. reaction is 5.
1 1
4. PCl5, PCl3, and Cl2 are at equilibrium at 500 K in a closed ˆˆ† HI ( g )
H 2 ( g ) + I 2 ( g ) ‡ˆˆ
container and their concentrations are 0.8 × 10–3 mol L–1, 2 2
1.2 × 10–3 mol L–1 and 1.2 × 10–3 mol L–1 respectively. The What would be the equilibrium constant K C for the
reaction ?
value of Kc for the reaction
ˆˆ† PCl3 (g) + Cl 2 (g) will be 2HI(g) ƒ H2(g) + I2(g)
PCl5 (g) ‡ˆˆ
(a) 0.04 (b) 0.4 (c) 25 (d) 2.5
(a) 1.8 × 103 mol L–1 (b) 1.8 × 10–3 11. In which of the following reactions, the equilibrium remains
–3
(c) 1.8 × 10 mol L –1 (d) 0.55 × 104
unaffected on addition of small amount of argon at constant
5. The ionisation constant of an acid, Ka is the measure of
volume?
strength of an acid. The Ka values of acetic acid, hypochlorous
acid and formic acid are 1.74 × 10–5, 3.0 × 10–8 and 1.8 × 10–4 ˆˆ† 2HI ( g )
(a) H 2 ( g ) + I2 ‡ˆˆ
respectively. Which of the following orders of pH of 0.1 (b) ˆˆ† PCl3 ( g ) + Cl2 ( g )
PCl5 ( g ) ‡ˆˆ
mol dm–3 solutions of these acids is correct?
(a) Acetic acid > hypochlorous acid > formic acid (c) ˆˆ† 2NH 3 ( g )
N 2 ( g ) + 3H 2 ( g ) ‡ˆˆ
(b) Hypochlorous acid > acetic acid > formic acid (d) The equilibrium will remain unaffected in all the three
(c) Formic acid > hypochlorous acid > acetic acid cases
(d) Formic acid > acetic acid > hypochlorous acid 12. When hydrochloric acid is added to cobalt solution at
6. K a1 , K a2 and K a3 are the respective ionisation constants room temperature, the following reaction takes place and
for the following reactions. thus the reaction mixture becomes blue. On cooling the
mixture it becomes pink. On the basis of this information
ˆˆ† H + + HS-
H 2S ‡ˆˆ mark the correct answer.
3+
ˆˆ† H + + S2- ˆˆ†
éë Co ( H 2 O )6 ùû ( aq ) + 4Cl- ( aq ) ‡ˆˆ
HS- ‡ˆˆ
ˆˆ† 2H + + S 2 -
H 2 S ‡ˆˆ
Pink
[ CoCl4 ]2- ( aq ) + 6H 2O ( l )
Blue
The correct relationship between K a1 , K a2 and K a3 is (a) DH > 0 for the reaction
(b) DH < 0 for the reaction
(a) K a3 = K a1 ´ K a2 (b) K a3 = K a1 + K a2 (c) DH = 0 for the reaction
(d) The sign of DH cannot be predicted on the basis of
(c) K a3 = K a1 - K a2 (d) K a3 = K a1 / K a2
this information
EBD_8350
106 CHEMISTRY

13. The pH of neutral water at 25°C is 7.0. As the temperature (a) By increasing the concentration of NH3(g)
increases, ionisation of water increases, however, the (b) By decreasing the pressure
concentration of H+ ions and OH– ions are equal. What (c) By decreasing concentration of N2(g) and H2(g)
will be the pH of pure water at 60°C? (d) By increasing pressure and decreasing temperature.
(a) Equal to 7.0 (b) Greater than 7.0 21. For a given exothermic reaction, Kp and K¢P are the
(c) Less than 7.0 (d) Equal to zero equilibrium constants at temperatures T 1 and T 2 ,
14. Which of the following will produce a buffer solution when respectively. Assuming that heat of reaction is constant in
mixed in equal volumes? temperature range between T1 and T2, it is readily observed
(a) 0.1 mol dm–3 NH4OH and 0.1 mol dm–3 HCl that: [AIPMT 2014, C]
(b) 0.05 mol dm–3 NH4OH and 0.1 mol dm–3 HCl (a) Kp > KP¢ (b) Kp < KP¢
(c) 0.1 mol dm–3 NH4OH and 0.05 mol dm–3 HCl 1
(d) 0.1 mol dm–3 CH3COONa and 0.1 mol dm–3 NaOH (c) Kp = KP¢ (d) Kp =
K¢p
15. In which of the following solvent silver chloride is most 22. Which of the following salts will give highest pH in water?
soluble? [AIPMT 2014, C]
(a) 0.1 mol dm–3 AgNO3 solution (a) KCl (b) NaCl
(b) 0.1 mol dm–3 HCl solution (c) Na2CO3 (d) CuSO4
(c) H2O 23. Using the Gibbs energy change, DG° = + 63.3kJ, for the
(d) Aqueous ammonia following reaction, [AIPMT 2014, A]
16. What will be the value of pH of 0.01 mol dm–3 CH3COOH Ag2CO3 2Ag+ (aq) + CO32–(aq)
(Ka = 1.74 × 10–5)? the Ksp of Ag2CO3(s) in water at 25°C is:-
(a) 3.4 (b) 3.6 (c) 3.9 (d) 3.0
(R = 8.314 J K–1 mol–1)
17. Ka for CH3COOH is 1.8 × 10–5 and Kb for NH4OH is (a) 3.2 × 10–26 (b) 8.0 × 10–12
1.8 × 10–5. The pH of ammonium acetate will be (c) 2.9 × 10–3 (d) 7.9 × 10–2
(a) 7.005 (b) 4.75
1 ˆˆ†SO3 (g) , if
(c) 7.0 (d) Between 6 and 7 24. For the reaction SO2 (g) + O2 (g) ‡ˆˆ
18. On increasing the pressure, in which direction will the gas 2
phase reaction proceed to re-establish equilibrium, is KP = KC(RT)x where the symbols have usual meaning then
predicted by applying the Le-Chatelier's principle. the value of x is (assuming ideality): [JEE M 2014, A]
Consider the reaction, 1
ˆˆ† 2NH3 ( g )
N 2 (g) + 3H 2 ‡ˆˆ (a) –1 (b) -
2
Which of the following is correct, if the total pressure at 1
which the equilibrium is established, is increased without (c) (d) 1
changing the temperature? 2
25. If the value of an equilibrium constant for a particular
(a) K will remain same
reaction is 1.6 × 1012, then at equilibrium the system will
(b) K will decrease
contain :- [AIPMT 2015, C]
(c) K will increase
(a) mostly reactants
(d) K will increase initially and decrease when pressure
is very high (b) mostly products
19. Which of the following statements is incorrect? (c) similar amounts of reactants and products
(a) In equilibrium mixture of ice and water kept in perfectly (d) all reactants
insulated flask, mass of ice and water does not 26. If the equilibrium constant for
change with time N2(g) + O2(g) 2NO(g) is K,
(b) The intensity of red colour increases when oxalic the equilibrium constant for [AIPMT 2015 RS, A]
acid is added to a solution containing iron (III) nitrate
and potassium thiocyanate 1 1
N2(g) + O2(g) NO(g) will be:
(c) On addition of catalyst the equilibrium constant value 2 2
is not affected 1
(d) Equilibrium constant for a reaction with negative DH 1
(a) K2 (b) K (c) K (d) K 2
value decreases as the temperature increases. 2
27. Which of the following statements is correct for a reversible
Past Year MCQs process in a state of equilibrium? [AIPMT 2015, A]
20. For the reversible reaction, [AIPMT 2014, C] (a) DG = 2.30 RT log K (b) DGº = –2.30 RT log K
N2(g) + 3H2(g) 2NH3(g) + heat (c) DGº = 2.30 RT log K (d) DG = –2.30 RT log K
The equilibrium shifts in forward direction
EQUILIBRIUM 107

28. What is the pH of the resulting solution when equal (a) The molar solubilities of MY and NY3 in water are
volumes of 0.1 M NaOH and 0.01 M HCl are mixed ? identical.
[AIPMT 2015 RS, S] (b) The molar solubility of MY in water is less than that of
(a) 12.65 (b) 2.0 (c) 7.0 (d) 1.04 NY3
29. The Ksp of Ag2CrO4, AgCl, AgBr and AgI are respectively, (c) The salts MY and NY3 are more soluble in 0.5 M KY
1.1 × 10–12, 1.8 × 10–10, 5.0 × 10–13, 8.3 × 10–17. Which one than in pure water.
of the following salts will precipitate last if AgNO3 solution (d) The addition of the salt of KY to solution of MY and
NY3 will have no effect on their solubilities.
is added to the solution containing equal moles of NaCl,
35. Consider the nitration of benzene using mixed conc. of
NaBr, NaI and Na2CrO4? [AIPMT 2015, S]
H2SO4 and HNO3. If a large amount of KHSO4 is added to
(a) AgCl (b) AgBr the mixture, the rate of nitration will be [NEET 2016, C]
(c) Ag2CrO4 (d) AgI (a) faster (b) slower
30. Which one of the following pairs of solution is not an (c) unchanged (d) doubled
acidic buffer ? [AIPMT 2015 RS, C]
36. The equilibrium constant at 298 K for a reaction
(a) HClO4 and NaClO4 ˆˆ† C + D is 100. If the initial concentration of all
A + B ‡ˆˆ
(b) CH3COOH and CH3 COONa the four species were 1 M each, then equilibrium
(c) H2CO3 and Na2CO3 concentration of D (in mol L–1) will be :
(d) H3PO4 and Na3PO4 [JEE M 2016, S]
31. The following reaction is performed at 298 K. (a) 1.818 (b) 1.182
ˆˆ† 2NO (g)
2NO(g) + O2(g) ‡ˆˆ (c) 0.182 (d) 0.818
2
The standard free energy of formation of NO(g) is 37. The equilibrium constant of the following are :
86.6 kJ/mol at 298 K. What is the standard free energy of N2 + 3H2 ƒ 2NH3 ; K 1 [NEET 2017, A]
formation of NO2(g) at 298 K? (Kp = 1.6 × 1012) N2 + O2 ƒ 2NO ; K2
[JEE M 2015, S] 1
ˆˆ† H 2 O ; K
H 2 + O2 ‡ˆˆ
ln (1.6 ´ 1012 ) 2 3
(a) 86600 –
R (298) The equilibrium constant (K) of the reaction :
(b) 0.5[2 × 86,600 – R(298) ln(1.6 × 1012)] 5
(c) R(298) ln(1.6 × 1012) – 86600 2NH3 + O2 ƒ 2NO + 3H2O, will be;
2
(d) 86600 + R(298) ln(1.6 × l012)
(a) K2 K33 / K1 (b) K2K 3/K 1
32. The standard Gibbs energy change at 300 K for the reaction
ˆˆ† B + C is 2494.2 J. At a given time, the composition
2A ‡ˆˆ (c) K23K3 / K1 (d) K1K33 / K2
1 1 38. A 20 litre container at 400 K contains CO2(g) at pressure
of the reaction mixture is [A] = , [B] = 2 and [C] = . The
2 2 0.4 atm and an excess of SrO (neglect the volume of solid
reaction proceeds in the : [R = 8.314 J/K/mol, e1 = 2.718]
SrO). The volume of the container is now decreased by
[JEE M 2015, C]
moving the movable piston fitted in the container. The
(a) forward direction because Q < KC
maximum volume of the container, when pressure of CO2
(b) reverse direction because Q < KC
attains its maximum value, will be :-
(c) forward direction because Q > KC
(d) reverse direction because Q > KC ˆˆ† SrO (s) + CO (g), Kp = 1.6 atm)
(Given that : SrCO (s) ‡ˆˆ
3 2
33. Consider the following liquid - vapour equilibrium. [NEET 2017, A]
ˆˆ† Vapour
Liquid ‡ˆˆ (a) 10 litre (b) 4 litre
Which of the following relations is correct? (c) 2 litre (d) 5 litre
[NEET 2016, C] 39. Concentration of the Ag+ ions in a saturated solution of
Ag2C2O4 is 2.2 × 10–4 mol L–1. Solubility product of
dlnG DH v dlnP DH v
(a) 2
= 2 (b) = Ag2C2O4 is :- [NEET 2017, A]
dT RT dT RT
(a) 2.66 × 10–12 (b) 4.5 × 10–11
dlnP -DH v dlnP -DH v (c) 5.3 × 10–12 (d) 2.42 × 10–8
(c) 2
= 2 (d) =
dT T dT RT 2 40. Which of the following can form buffer solution?
34. MY and NY3, two nearly insoluble salts, have the same Ksp [NEET 2017, C]
values of 6.2 × 10–13 at room temperature. Which statement (a) aq.NH3 + NH4OH (b) KOH + HNO3
would be true in regard to MY and NY3? [NEET 2016, C] (c) NaOH + HCl (d) KI + KOH
EBD_8350
108 CHEMISTRY

41. pKa of a weak acid (HA) and pKb of a weak base (BOH) BaSO4 just begins to precipitate. The final volume is 500
are 3.2 and 3.4, respectively. The pH of their salt (AB) mL. The solubility product of BaSO4 is 1 × 10–10. What is
solution is [JEE M 2017, A] the original concentration of Ba2+ ? [JEE M 2018, A]
(a) 7.2 (b) 6.9 (a) 5 × 10–9 M (b) 2 × 10–9 M
(c) 7.0 (d) 1.0 (c) 1.1 × 10–9 M (d) 1.0 × 10–10 M
42. Which one of the following conditions will favour maximum 48. Conjugate base for Brönsted acids H2O and HF are :
formation of the product in the reaction, (a) OH– and H2F+, respectively [NEET 2019, C]
[NEET 2018, C]
(b) H3O+ and F–, respectively
ˆˆ† X 2 (g) D r H = – x kJ :
A2 (g) + B2 (g) ‡ˆˆ (c) OH– and F–, respectively
(a) Low temperature and high pressure (d) H3O+ and H2F+, respectively
(b) Low temperature and low pressure
49. pH of a saturated solution of Ca(OH)2 is 9. The solubility
(c) High temperature and low pressure
product (Ksp) of Ca(OH)2 is: [NEET 2019, A]
(d) High temperature and high pressure
(a) 0.5 × 10–15 (b) 0.25 × 10–10
43. The solubility of BaSO4 in water is 2.42 × 10–3 gL–1 at
298 K. The value of its solubility product (Ksp) will be (c) 0.125 × 10–15 (d) 0.5 × 10–10
(Given molar mass of BaSO4 = 233 g mol–1) 50. Which will make basic buffer ? [NEET 2019, A]
[NEET 2018, A] (a) 50 mL of 0.1 M NaOH + 25 mL of 0.1 M CH3COOH
(a) 1.08 × 10–10 mol2L–2 (b) 100 mL of 0.1 M CH3COOH + 100 mL of 0.l M NaOH
(b) 1.08 × 10–12 mol2L–2 (c) 100 mL of 0.1 M HCl + 200 mL of 0.1 M NH4OH
(c) 1.08 × 10–8 mol2L–2 (d) 100 mL of 0.1 M HCl + 100 mL of 0.1 M NaOH
(d) 1.08 × 10–14 mol2L–2 51. Which amongst the following is the strongest acid?
44. Following solutions were prepared by mixing different [JEE M 2019, C]
volumes of NaOH and HCl of different concentrations : (a) CHBr3 (b) CHI3
[NEET 2018, S] (c) CH(CN)3 (d) CHCl3
M M 52. 20 mL of 0.1 M H2SO4 solution is added to 30 mL of 0.2 M
(i) 60 mL HCl + 40 mL NaOH NH4OH solution. The pH of the resultant mixture is:
10 10
[pKb of NH4OH = 4.7]. [JEE M 2019, A]
M M
(ii) 55 mL HCl + 45 mL NaOH (a) 5.2 (b) 9.0
10 10
(c) 5.0 (d) 9.4
M M 53. Find out the solubility of Ni(OH)2 in 0.1 M NaOH. Given that
(iii) 75 mL HCl + 25 mL NaOH the ionic product of Ni(OH)2 is 2 × 10–15 [NEET 2020, A]
5 5
(a) 2 × 10–8 M (b) 1 × 10–13 M
M M
(iv) 100 mL HCl + 100 mL NaOH (c) 1 × 108 M (d) 2 × 10–13 M
10 10
54. Hydrolysis of sucrose is given by the following reaction.
pH of which one of them will be equal to 1?
(a) (ii) (b) (i) ˆˆ† Glucose Fructose
Sucrose + H2O ‡ˆˆ
(c) (iii) (d) (iv)
If the equilibrium constant (KC) is 2 × 1013 at 300 K, the
45. Which of the following salts is the most basic in aqueous –
solution? [JEE M 2018, S] value of DrG at the same temperature will be :
(a) Al(CN)3 (b) CH3COOK [NEET 2020, A]
(c) FeCl3 (d) Pb(CH3COO)2 (a) 8.314 J mol–1K–1 × 300 K × ln(2 × 1013)
46. An aqueous solution contains 0.10 M H2S and 0.20 M (b) 8.314 J mol–1K–1 × 300 K × ln(3 × 1013)
HCl. If the equilibrium constants for the formation of HS– (c) –8.314 J mol–1K–1 × 300 K × ln(4 × 1013)
from H2S is 1.0 × 10–7 and that of S2– from HS– ions is (d) –8.314 J mol–1K–1 × 300 K × ln(2 × 1013)
1.2 × 10–13 then the concentration of S2– ions in aqueous 55. Two solutions, A and B, each of 100 L was made by
solution is: [JEE M 2018, S] dissolving 4g of NaOH and 9.8 g of H2SO4 in water,
(a) 5 × 10–8 (b) 3 × 10–20 respectively. The pH of the resultant solution obtained
(c) 6 × 10–21 (d) 5 × 10–19 from mixing 40 L of solution A and 10 L of solution. B is
47. An aqueous solution contains an unknown concentration _______. [NV, JEE M 2020, S]
of Ba2+. When 50 mL of a 1 M solution of Na2SO4 is added,
EQUILIBRIUM 109

Exercise 4 : Problem Solving Skill Enhancer MCQs


1. Phosphorus pentachloride dissociates as follows, in a (a) 1 (b) 2
closed reaction vessel (c) 4 (d) 8
PCl5(g) PCl3(g) + Cl2(g) 6. 8 mole of a gas AB3 are introduced into a 1.0 dm3 vessel.
If total pressure at equilibrium of the reaction mixture is P It dissociates as 2AB3 (g) ƒ A2 (g) + 3B2 (g)
and degree of dissociation of PCl5 is x, the partial pressure
At equilibrium, 2 mole of A2 is found to be present. The
of PCl3 will be
equilibrium constant for the reaction is
æ x ö æ x ö (a) 2 mol2 L–2 (b) 3 mol2 L–2
(a) ç ÷P (b) ç ÷P
è x -1 ø è 1- x ø (c) 27 mol L 2 –2 (d) 36 mol2 L–2
æ x ö æ 2x ö 7. For the equilibrium reaction
(c) ç ÷P (d) ç ÷P
è x +1 ø è 1- x ø 2NOBr (g) 2NO (g) + Br2 (g)
2. Consider the following equilibrium P
N 2O 4 ( g ) ƒ 2NO 2 ( g ) If PBr2 = , where P is total pressure at equilibrium. The
9
Then the select the correct graph, which shows the Kp
variation in concentrations of NO2 against concentrations ratio is equal to
P
of N2O4: 1 1 1 1
(a) (b) (c) (d)
3 81 9 27
[NO2] ¾®

[NO 2] ¾ ®

8. ˆˆ† 2NH3
For a reversible gaseous reaction, N2 + 3H 2 ‡ˆˆ
at equilibrium, if some moles of H2 are replaced by same
(a) (b) number of moles of T2 (T is tritium, isotope of H and
[N2O 4] ¾ ® assume isotopes do not have different chemical properties)
[N2O4] ¾®
without affecting other parameter, then :
(a) the sample of ammonia obtained after sometime will
[NO 2] ¾ ®
[N O2] ¾®

be radioactive.
(b) moles of N2 after the change will be different as
(c) (d) compared to moles of N2 present before the change
[N 2O4] ¾® [N 2O 4] ¾ ® (c) the value of KP or Kc will change
3. Equilibrium constant for the reaction, (d) the average molecular mass of new equilibrium will
be same as that of old equilibrium
CaCO3 (s) ƒ CaO (s) + CO2 (g), follows the equation 9. The preparation of SO3(g) by reaction
8400 1
ln Kp = 7 – , where T = absolute temperature. Find SO 2 ( g ) + O 2 ( g ) ƒ SO 3 ( g )
T 2
the equilibrium temperature if decomposition of CaCO3 is an exothermic reaction. If the preparation follows the
produces CO2 gas having partial pressure equal to following temperature-pressure relationship for its % yield,
atmospheric pressure. then for temperatures T1, T2 and T3. The correct option is:
(a) 1200 °C (b) 927 K (c) 927 °C (d) None
4. An amount of solid NH4HS is placed in a flask already 50
containing ammonia gas at a certain temperature and 40
0.50 atm pressure. Ammonium hydrogen sulphide T3
decomposes to yield NH3 and H2S gases in the flask. When 30
% yield

T2
the decomposition reaction reaches equilibrium, the total 20
pressure in the flask rises to 0.84 atm? The equilibrium 10 T1
constant for NH4HS decomposition at this temperature is
(a) 0.11 (b) 0.17 (c) 0.18 (d) 0.30 1 2 4
3
5. A vessel of 250 litre was filled with 0.01 mole of Sb2S3 and Pressure (atm)
0.01 mole of H2 to attain the equilibrium at 440°C as
ˆˆ† 2Sb ( s ) + 3H 2S ( g )
Sb 2 S3 ( s ) + 3H 2 ( g ) ‡ˆˆ (a) T3 > T2 > T1
After equilibrium, the H2S formed was analysed by (b) T1 > T2 > T3
dissolved it in water and treating with excess of Pb2+ to (c) T1 = T2 = T3
give 1.19 g of PbS as precipitate. What is the value of Kc (d) Nothing could be predicted about temperature
through given information
at 440°C?
EBD_8350
110 CHEMISTRY

10. For the reaction N 2 O4 (g) ƒ 2NO2 (g) , the value of Kp 13. Based upon the following hypothetical equilibrium at 273 K
ˆˆ† XCl . 2H O(s) + 4 H O(g) ;
XCl2. 6H2O(s) ‡ˆˆ
is 1.7 ×103 at 500 K and 1.7 × 104 at 600 K. Which of the 2 2 2
following is/are correct? Kp = 8.1 × 10–11 atm 4
(a) The proportions of NO2 in the equilibrium mixture is ˆˆ† Y HPO . 7H O(s) + 5H O(g) ;
Y2HPO4. 12H2O(s) ‡ˆˆ 2 4 2 2
increased by decrease in pressure.
Kp = 3. 2 × 10–9 atm 5
(b) The standard enthalpy change for the forward
Z2SO4. 10H2O(s) ‡ˆˆˆˆ† Z SO (s) + 10H O(g) ;
reaction in negative. 2 4 2

(c) Units of Kp is atm–1. Kp = 1.0 × 10–30 atm10


(d) At 500 K the degree of dissociation of N2O4 decreases Which is the most effective dehydrating agent at 273 K
by 50% by increasing the pressure by 100%. (Aqueous tension at 273 K = 6.0 × 10–3 atm)
11. For the complexation reaction (a) X Cl2. 6H2O(s) (b) Y2 HPO4. 7H2O(s)
Ag + (aq) + 2NH 3 (aq) [Ag(NH 3 ) 2+ ](aq) , (c) Z2SO4(s) (d) Z2SO4. 10H2O(s)
14. Calculate DrG for the reaction at 27°C
the rates of forward and reverse reactions are given by :
ˆˆ† 2A(s) + 2H + (aq)
H 2 (g) + 2Ag + (aq) ‡ˆˆ
(rate)f = 1.0 × 106 L2 mol –2 s–1[Ag+][NH3]2
(rate)r = 2.0 × 10–2 s–1 [Ag(NH3)2]+ Given : PH 2 = 0.5 bar; [Ag+] = 10–5 M;
The instability constant of the complex is : [H+] = 10–3 M; DfG° [Ag+(aq)] = 77.1 kJ/mol
(a) 5.0 × 109 (b) 2.0 × 10–4 (a) – 154.2 kJ/mol (b) – 178.9 kJ/mol
(c) 2.0 × 10 –4 (d) 2.0 × 10–8 (c) – 129.5 kJ/mol (d) None of these
12. The 0.001M Solution of Mg (NO3)2 is adjusted to pH 9, Ksp 15. A solution is saturated with respect to SrCO3 and SrF2.
of Mg(OH)2 is 8.9 × 10–12. At this pH The [CO 3 2– ] was found to be 1.2 × 10 –3 M. The
concentration of F– in the solution would be
(a) Mg(OH)2 will be precipitated
Given Ksp of SrCO3 = 7.0 ´ 10–10 M2,
(b) Mg(OH)2 is not precipitated Ksp of SrF2 = 7.9 ´ 10–10 M3,
(c) Mg(OH)3 will be precipitated (a) 1.3 × 10–3 M (b) 2.6 × 10–2 M
(d) Mg(OH)3 is not precipitated (c) 3.7 × 10 M–2 (d) 5.8 × 10–7 M

ANSWER KEY
Exercis e 1 : NCERT Based Topic-wis e MCQs
1 (c) 13 (b) 25 (c) 37 (d) 49 (c) 61 (b) 73 (b) 85 (b) 97 (c) 109 (b)
2 (a) 14 (c) 26 (b) 38 (a) 50 (d) 62 (a) 74 (a) 86 (a) 98 (b) 110 (b)
3 (c) 15 (b) 27 (a) 39 (b) 51 (a) 63 (a) 75 (c) 87 (c) 99 (d) 111 (a)
4 (c) 16 (a) 28 (b) 40 (a) 52 (b) 64 (b) 76 (d) 88 (a) 100 (d) 112 (a)
5 (b) 17 (d) 29 (b) 41 (c) 53 (a) 65 (b) 77 (a) 89 (c) 101 (b) 113 (d)
6 (b) 18 (c) 30 (b) 42 (d) 54 (d) 66 (b) 78 (d) 90 (d) 102 (b) 114 (b)
7 (b) 19 (c) 31 (b) 43 (d) 55 (b) 67 (b) 79 (c) 91 (d) 103 (c) 115 (c)
8 (a) 20 (b) 32 (b) 44 (a) 56 (d) 68 (a) 80 (d) 92 (c) 104 (a)
9 (d) 21 (a) 33 (c) 45 (d) 57 (c) 69 (c) 81 (d) 93 (c) 105 (d)
10 (c) 22 (c) 34 (d) 46 (d) 58 (a) 70 (c) 82 (c) 94 (c) 106 (a)
11 (d) 23 (d) 35 (b) 47 (c) 59 (b) 71 (b) 83 (b) 95 (c) 107 (b)
12 (c) 24 (a) 36 (b) 48 (b) 60 (b) 72 (c) 84 (c) 96 (c) 108 (d)
Exercis e 2 : Numeric/Integer Answer Questions
1 (20) 3 (10) 5 (4) 7 (6.5) 9 (8.22) 11 (4.76) 13 (5.0) 15 (0.05)
2 (0.75) 4 (0.592) 6 (6.93) 8 (4) 10 (71) 12 (0.134) 14 (99.9)
Exercise 3 : NCERT Exemplar & Past Year MCQs
1 (d) 7 (c) 13 (c) 19 (b) 25 (b) 31 (b) 37 (a) 43 (a) 49 (a) 55 (10.60)
2 (d) 8 (a) 14 (c) 20 (d) 26 (a) 32 (d) 38 (d) 44 (c) 50 (c)
3 (c) 9 (b) 15 (d) 21 (a) 27 (b) 33 (d) 39 (c) 45 (b) 51 (c)
4 (b) 10 (a) 16 (a) 22 (c) 28 (a) 34 (b) 40 (a) 46 (b) 52 (b)
5 (b) 11 (d) 17 (c) 23 (b) 29 (c) 35 (b) 41 (b) 47 (c) 53 (c)
6 (a) 12 (a) 18 (a) 24 (b) 30 (a) 36 (a) 42 (a) 48 (c) 54 (b)
Exercis e 4 : Problem Solving Skill Enhancer MCQs
1 (c) 3 (c) 5 (a) 7 (b) 9 (b) 11 (d) 13 (c) 15 (c)
2 (b) 4 (a) 6 (c) 8 (a) 10 (a) 12 (b) 14 (c)
8 Redox Reactions

Trend Buster NEET & JEE Main

Number of Questions from 2020-15 3 3 Chapter carry low weightage.


Weightage 0.95% 1.70%

The most Important Concepts that Cover Maximum number of Questions asked in past 6 years.

Oxidation state/oxidation number 1 2


Redox reaction/disproportionation reaction 1 1
Balancing of redox reaction 1 —

Less Important Concepts that Cover 1 or 2 Questions asked in past 6 years.

Electrode process — —

NEET JEE

2020 Oxidation number Oxidation number 1 Easy 1 Easy


2019 Types of redox breactions Disproportionation reactions 1 Average — —
2018 Oxidation number/balancing Oxidation state/balancing 1 Average 1 Average
of redox reaction
2017 Redox reactions Redox reactions — — 1 Easy
2016 — — — — — —
2015 — — — — — —
EBD_8350
112 CHEMISTRY
REDOX REACTIONS 113
EBD_8350
114 CHEMISTRY

Problem Solving Tips/ Tricks/ Points to Remember


4 Redox titration : Potassium permanganate is often used in Step 3 : Balance O by adding H2O.
redox titrations because it is a powerful oxidizing agent and Step 4 : Balance H by adding H+.
serves as its own indicator. In acidic solutions, the purple Step 5 : Balance net charge by adding e–.
MnO4– ion is reduced to the nearly colourless Mn2+ ion. Step 6 : Make e– gain equal to e– loss, then add half-reactions.
4 The oxidation number of N in NO2 is + 4. In NO, the Step 7 : Cancel anything that’s the same on both sides.
oxidation number is +2. Additional steps in the ion-electron method for basic solutions:
4 H2SO4 is a non-oxidizing acid when cold and dilute. Step 8 : Add to both sides of the equation the same
number of OH– as there are H+.
4 The strongest oxidizing agent in a solution of a “non- Step 9 : Combine H+ and OH– to form H2O.
oxidizing” acid is H+. Step 10 : Cancel any H2O that you can.
4 When we balance the equation, the ion-electron method 4 Fractional Oxidation State:
will tell us how H+ and H2O are involved in the reaction.
Fractional oxidation state is the average oxidation state of
4 Electrons must be added to whichever side of the half-
reaction is more positive (or less negative). the element. eg. S4O62- (tetrathionate ion), C3O2 (carbon
4 The charge on the polyatomic ion equals the sum of the suboxide), and Br3O8 (tribromooctaoxide)
oxidation numbers of its atoms.
4 Mostly, the medium in which a redox reaction is to be (i) S4O62- (4x – 12 = –2, x = 2.5)
balanced is given in the problem but if the problem does not Oxidation number of S = 2.5
state the medium explicitly, then the medium is decided by O O
looking at the reactants or products. If an acid or base is one 0 0
–O S+5 S S S +5 O–
of the reactants or products, then the medium is the same. * *
4 KMnO4 acts as an oxidant in every medium although with O O
different strength which follows the order (ii) C3O2 (3x – 4 = 0, x = 4/3)
acidic medium > neutral medium > alkaline medium Oxidation number of C = 4/3
4 Act as both oxidising and reducing agents : SO2, H2O2, +2 0 +2
O3, NO2, etc. O C C C O
4 Nature of oxides based on oxidation number *
(i) Lowest oxidation state ® Basic (MnO) (iii) Br3O8 (3x – 16 = 0, x = 16/3)
(ii) Intermediate oxidation state Oxidation number of Br = 16/3
® Amphoteric (Mn 3O4, MnO2) O O O
(iii) Highest oxidation state ® Acidic (Mn2O7) +6 +4 +6
O Br Br Br O
4 Ion-electron method-Acidic solution for Balancing the equation *
Step 1 : Divide the equation into two half-reactions. O O O
Step 2 : Balance atoms other than H and O.
4 Iodometric and Iodimetric titrations
Estimation of Reaction Reaction between oxidising
agent and reducing agent
Titrating solution is Na 2S2O3 .5H 2 O(Hypo) I2 º 2I - º 2Na 2S2 O3
I2 (Iodometry) I2 + 2Na 2S2O3 ® 2NaI + Na 2S4 O6 M
w (Na 2S2 O3 ) =
or I2 + 2S2O32- ® 2I- + S4 O62- I
2CuSO4 + 4KI ® Cu2 I2 + 2K 2SO4 + I2 2CuSO4 º I2 º 2I - º 2 Na 2S2 O3
CuSO4 (Iodimetry) + M
or 2Cu2 + 4I - ® Cu2 I2 + I2 (white ppt .) Ew of CuSO4 =
I
4 In mixed oxides like Fe3O4, the metal exist in two different 4 Oxidation number of S in Na2S4O6:
oxidation states. Fe3O4 is an equimolar mixture of FeO and O O
Fe2O3 and thus, Fe exist in +2 and +3 oxidation states. || (O) (O) ||
4 Oxidation number of S in Na2S2O3: Na+ O-— S — S — S — O- — O– — Na+
S(–2)
|| (x) || (x)
­ O O
Na+ O- — S(x)
— O - Na+ 2 × (+1) + 6 × (–2) + 2 × (0) + 2x = 0
|| (Na) (O) (S–S) (S)
O Þ x= +5
2 × (+1) + 3 × (–2) + 1 × (–2) + x = 0 4 If highly electronegative element is in its highest oxidation state,
(for Na) (for O) (coordinated S) (for S) then that compound can act as a powerful oxidant e.g. KClO4.
Þ x= +5 4 If an element is in intermediate oxidation state, then that
Thus, coordinated S is in –2 and other S atom is in +6 compound can act as both oxidising and reducing agent,
oxidation states. e.g. H2O2, H2SO3, HNO2, SO2 etc.
REDOX REACTIONS 115

Exercise 1 : NCERT Based Topic-wise MCQs


Topic 1: Redox Reactions in Terms of Electron 8. Assertion : In a reaction
Zn(s) + CuSO4 (aq) ¾® ZnSO4(aq) + Cu(s)
Transfer Reactions Zn is a reductant but itself get oxidized.
1. Given reaction, Reason : In a redox reaction, oxidant is reduced by
2K4[Fe(CN)6] (aq) + H2O2 (aq) ¾® accepting electrons and reductant is oxidized by losing
2K3[Fe(CN)6](aq) + 2KOH(aq) electrons.
The above given reaction is oxidation reaction due to (a) Assertion is correct, reason is correct; reason is a
(a) removal of a hydrogen from H2O2 correct explanation for assertion.
(b) addition of electropositive potassium to H2O2 (b) Assertion is correct, reason is correct; reason is not
(c) removal of electropositive element potassium from a correct explanation for assertion
potassium ferrocyanide (K4[Fe(CN)6]) (c) Assertion is correct, reason is incorrect
(d) All of the above are the correct reasons. (d) Assertion is incorrect, reason is correct.
2. In the reaction given below, identify the species undergoing 9. In which of the following reactions, there is no change in
redox reaction valency ?
2Na(s) + H2(g) ¾® 2NaH(s) (a) 4 KClO3 ¾ ¾® 3KClO4 + KCl
(a) Na is reduced and hydrogen is oxidised (b) SO2 + 2H2S ¾ ¾® 2H2O + 3S
(b) Na is oxidised and hydrogen is reduced (c) BaO2 + H2SO4 ¾ ¾® BaSO4 + H2O2
(c) Na undergoes oxidation and hydrogen undergoes
(d) 3 BaO + O2 ¾ ¾® 2 BaO2.
reduction
10. If rod of a metal is put in a metal ion solution which is blue
(d) Both (b) and (c)
in colour, solution turn colorless. The metal rod and
3. Which of the following is correct code for x and y in the solution respectively are?
following reaction. x (a) Zinc and Cu(II)
(b) Zinc and Ni(II)
2Na(s) + S(s) ¾¾
® Na2S(s) (c) Aluminium and Cu(II)
(d) Both (a) and (c)
y
(i) x = oxidation reaction, y = reduction reaction Topic 2: Oxidation Number
(ii) x = gain of two electrons, y = loss of two electrons,
(iii) x = reduction reaction, y = oxidation reaction 11. ‘Oxidation number of H in NaH, CaH2 and LiH,
(iv) x = loss of two electrons, y = gain of two electrons respectively is
(a) (i) and (ii) (b) (i) and (iv) (a) +1, +1, –1 (b) –1, +1, + 1
(c) (ii) and (iii) (d) (iii) and (iv) (c) +1, + 1, + 1 (d) –1,–1, –1
4. Match the columns 12. The oxidation number of chromium in potassium
Column-I Column-II dichromate is
(A) 2Mg + O2 ¾® 2MgO (p) Removal of hydrogen (a) + 6 (b) – 5 (c) – 2 (d) + 2
(B) Mg + Cl2 ¾® MgCl2 (q) Removal of 13. The oxidation number of sulphur in S8, S2F2, H2S
electropositive element respectively, are
(C) 2H2S + O2 ¾® (r) Addition of oxygen (a) 0, +1 and –2 (b) +2, +1 and –2
2S + 2H2O (c) 0, +1 and +2 (d) –2, +1 and –2
(D) 2KI + H2O + O3 ¾® (s) Addition of 14. Oxidation number of nitrogen in (NH4)2SO4 is
2KOH + I2 + O2 electronegative element, (a) –1/3 (b) –1 (c) +1 (d) –3
chlorine 15. Oxidation number of carbon in CH2Cl2 is
(a) A – (s), B – (q), C – (p), D – (r) (a) –4 (b) +4 (c) 0 (d) –2
(b) A – (r), B – (s), C – (p), D – (q) 16. The oxidation state of osmium (Os) in OsO4 is
(c) A – (s), B – (r), C – (q), D – (p) (a) + 7 (b) + 6 (c) + 4 (d) + 8
(d) A – (r), B – (p), C – (s), D – (q) 17. In which of the following compounds, the oxidation number
5. When Sn2+ changes to Sn4+ in a reaction of iodine is fractional ?
(a) It loses two electrons (b) It gains two electrons (a) IF7 (b) I3- (c) IF5 (d) IF3
(c) It loses two protons (d) It gains two protons 18. Oxidation numbers of chlorine atoms in CaOCl2 are
6. When a strip of metallic zinc is placed in an aqueous (a) 0, 0 (b) –1, –1
solution of copper nitrate the blue colour of the solution (c) –1, +1 (d) None of these
disappear due to formation of 19. The oxide, which cannot act as a reducing agent, is
(a) Cu2+ (b) Zn 2+ (c) ZnS (d) CuS (a) NO2 (b) SO2 (c) CO2 (d) ClO2
7. The number of electrons involved in the reduction of 20. The average oxidation state of sulphur in Na2S4O6 is
one nitrate ion to hydrazine is (a) +2.5 (b) +2
(a) 8 (b) 5 (c) 3 (d) 7 (c) +3.0 (d) +3.5
EBD_8350
116 CHEMISTRY

21. In which of the following compounds oxygen has highest (a) + 3, + 6 and + 5 (b) + 5, + 3 and + 6
oxidation state and in which it has lowest oxidation state? (c) – 3, + 6 and + 6 (d) + 5, + 6 and + 6
OF2, H2O2, KO2, O2F2 35. Assertion : HClO4 is a stronger acid than HClO3.
(a) Highest = KO2, lowest = H2O2 Reason : Oxidation state of Cl in HClO 4 is +VII and in
(b) Highest = OF2, lowest = K2O2 HClO3 +V.
(c) Highest = OF2, lowest = KO2 (a) Assertion is correct, reason is correct; reason is a
(d) Highest = KO2, lowest = H2O2 correct explanation for assertion.
22. Oxidation number of cobalt in K[Co(CO)4] is (b) Assertion is correct, reason is correct; reason is not
(a) +1 (b) +3 (c) –1 (d) –3 a correct explanation for assertion
23. In which of the following compounds, iron has lowest (c) Assertion is correct, reason is incorrect
oxidation state? (d) Assertion is incorrect, reason is correct.
(a) K 3[Fe(CN ) 6 ] Topic 3: Types of Redox Reactions, Balancing of
(b) K 4 [Fe(CN) 6 ] Redox Reactions
(c) FeSO 4 .( NH 4 ) 2 SO 4 .6H 2 O
(d) Fe(CO)5 36. Which one of the following reactions involves
24. In which of the compounds does 'manganese' exhibit disproportionation?
highest oxidation number ? (a) 2H2SO4 + Cu ¾® CuSO4 + 2H2O + SO2
(a) MnO2 (b) Mn 3O4 (b) As2O3 + 3H2S ¾® As2S3 + 3H2O
(c) K2MnO4 (d) MnSO4 (c) 2KOH + Cl2 ¾® KCl + KOCl + H2O
25. Among the following, identify the species with an atom in (d) Ca3P2 + 6H2O ¾® 3Ca(OH)2 + 2PH3
+6 oxidation state 37. In the reaction
(a) MnO–4 (b) Cr ( CN ) 36 – 3Br2 + 6CO 32 - + 3H 2 O ¾¾
® 5Br - + BrO3– + 6HCO 3-
(a) Bromine is oxidised and carbonate is reduced.
(c) NiF62 – (d) CrO 2 Cl 2 (b) Bromine is reduced and water is oxidised
26. Which reaction involves neither oxidation nor reduction? (c) Bromine is neither reduced nor oxidised
(a) CrO24 - ¾¾ ® Cr2 O 72- (b) Cr ¾¾ ® CrCl3 (d) Bromine is both reduced and oxidised
+ 2- +1 -2 0 0
(c) Na ¾¾ ® Na (d) 2S2 O3 ¾¾ ® S4 O62 - 38. The reaction, 2H 2 O(l) ¾¾
D
® 2H 2 (g) + O 2 (g) is an
27. The average oxidation state of Fe in Fe3O4 is
(a) + 3 (b) 8/3 (c) + 6 (d) + 2 example of
28. In oxygen difluoride, the oxidation number of oxygen is (a) addition reaction (b) decomposition reaction
(a) – 2 (b) – 1 (c) + 2 (d) +1, – 2 (c) displacement reaction (d) None of these
29. In the compounds KMnO4and K2Cr 2O7, the highest 39. How will you balance the total ionic charge of reactant
oxidation state is of the element and products if reaction is carried out in acidic solution?
(a) potassium (b) manganese (a) By using H+ ions
(c) chromium (d) oxygen (b) By using OH– ions
30. Atomic number of an element is 22. The highest O.S. (c) Adding H2O molecules to the reactant or product
exhibited by it in its compounds is (d) Multiplying by suitable coefficients.
(a) 1 (b) 2 (c) 3 (d) 4 40. C2H6 (g) + nO2(g) ® CO2(g) + H2O(l)
31. Match Column-I (compound) with Column-II (oxidation In this equation, the ratio of the coefficients of CO2 and
state of underlined element) and choose the correct option. H2O is
Column - I Column - II (a) 1 : 1 (b) 2 : 3 (c) 3 : 2 (d) 1 : 3
(A) CuO (p) 4 41. 2MnO -4 + 5H 2 O 2 + 6H + ® 2 Z + 5O 2 + 8H 2 O. In this
(B) MnO2 (q) 3 reaction Z is
(C) HAuCl4 (r) 2 (a) Mn 2+ (b) Mn 4+ (c) MnO2 (d) Mn
(D) Tl2O (s) 1 42. What is ‘A’ in the following reaction
(a) A – (r), B – (p), C – (q), D – (s) 2Fe3+(aq) +Sn2+(aq) ® 2Fe2+(aq) + A
(b) A – (s), B – (r), C – (p), D – (q) (a) Sn3+ (aq) (b) Sn4+ (aq)
(c) A – (r), B – (s), C – (p), D – (q) 2+
(c) Sn (aq) (d) Sn
(d) A – (s), B – (q), C – (p), D – (r) 43. Match the columns
32. Among NH3, HNO3, NaN3 and Mg3N2 the number of Column-I Column-II
molecules having nitrogen in negative oxidation state is (A) V2O5(s) + 5 Ca(s) ¾® (p) Disproportionation
(a) 1 (b) 2 (c) 3 (d) 4 2V(s) + 5 CaO(s) reaction
33. The correct decreasing order of oxidation number of oxygen D
in compounds BaO2, O3, KO2 and OF2 is (B) CH4(g) + 2O2(g) ¾¾® (q) Decomposition
(a) BaO2 > KO2 > O3 > OF2 CO2(g) + 2H2O(l) reaction
(b) OF2 > O3 > KO2 > BaO2 (C) P4(s) + 3OH–(aq) + 3H2O(l) (r) Combination
(c) KO2 > OF2 > O3 > BaO2 ¾® PH3(g) + 3H 2 PO 2– (aq) reaction
(d) BaO2 > O3 > OF2 > KO2
2– D
34. Oxidation numbers of P in PO3– 4 , of S in SO 4 and that of (D) 2 KClO3(s) ¾¾® (s) Displacement
Cr in Cr2O72– are respectively 2KCl(s) + 3O2(g) reaction
REDOX REACTIONS 117

(a) A – (s), B – (q), C – (r), D – (p) (a) (i) 6, (ii) 4, yes (b) (i) 6, (ii) 6, No
(b) A – (s), B – (r), C – (p), D – (q) (c) (i) 4, (ii) 4, No (d) (i) 4, (ii) 4, yes
(c) A – (r), B – (s), C – (q), D – (p) 54. Consider the following reaction :
+
(d) A – (r), B – (s), C – (p), D – (q) xMnO 4– + yC 2 O 2–
4 + zH ¾¾ ®
44. Assertion : The reaction : z
D xMn 2+ + 2 yCO 2 + H 2O
CaCO3 (s) ¾¾® CaO(s) + CO 2 (g) 2
is an example of decomposition reaction The value’s of x, y and z in the reaction are, respectively :
Reason : Above reaction is not a redox reaction. (a) 5, 2 and 16 (b) 2, 5 and 8
(a) Assertion is correct, reason is correct; reason is a (c) 2, 5 and 16 (d) 5, 2 and 8
correct explanation for assertion. 55. In the balanced chemical reaction
(b) Assertion is correct, reason is correct; reason is not IO3- + aI - + bH + ¾¾ ® cH 2 O + d I 2
a correct explanation for assertion a, b, c and d respectively corresponds to
(c) Assertion is correct, reason is incorrect (a) 5, 6, 3, 3 (b) 5, 3, 6, 3
(d) Assertion is incorrect, reason is correct. (c) 3, 5, 3, 6 (d) 5, 6, 5, 5
45. Which of the following do not show disproportionation 56. Acidic medium used in KMnO4 can be made from which
reaction? of the following acids?
ClO–4, F2, Cl2, ClO2–, ClO2–, P4, S8, and ClO– (a) HCl (b) H2SO4 (c) HI (d) HBr
(a) ClO2–, ClO4–, and ClO– (b) F2 only 57. Phosphorus, sulphur and chlorine undergo disproportion
(c) F2 and ClO4– (d) ClO4– only in the ...A... medium.
46. Which of the following elements does not show Here, A refers to
disproportionation tendency? (a) acidic (b) alkaline
(a) Cl (b) Br (c) F (d) I (c) neutral (d) Both (a) and (b)
47. In the redox reaction, 58. For the reaction : NH 3 + OCl - ¾ ¾® N 2 H 4 + Cl - in
xKMnO4 + NH3 ¾® yKNO3 + MnO2 + KOH + H2O basic medium, the coefficients of NH3, OCl– and N2H4 for
(a) x = 4, y = 6 (b) x = 3, y = 8 the balanced equation are respectively
(c) x = 8, y = 6 (d) x = 8, y = 3 (a) 2, 2, 2 (b) 2, 2, 1 (c) 2, 1, 1 (d) 4, 4, 2
48. Given : 59. What products are expected from the disproportionation
X Na2HAsO3 + Y NaBrO3 + Z HCl ® NaBr + H3AsO4 reaction of hypochlorous acid?
+ NaCl (a) HCl and Cl2O (b) HCl and HClO3
The values of X, Y and Z in the above redox reaction are (c) HClO3 and Cl2O (d) HClO2 and HClO4
respectively 60. In the disproportionation reaction
(a) 2, 1, 2 (b) 2, 1, 3 (c) 3, 1, 6 (d) 3, 1, 4 3 HClO3 ¾® HClO4 + Cl2 + 2O2 + H2O, the equivalent
49. The values of x and y in the following redox reaction mass of the oxidizing agent is
x Cl2 + 6OH - ¾® ClO 3- + yCl - + 3H 2O are (molar mass of HClO3 = 84.45)
(a) x = 5, y = 3 (b) x = 2, y = 4 (a) 16.89 (b) 32.22 (c) 84.45 (d) 28.15
(c) x = 3, y = 5 (d) x = 4, y = 2 61. If equal volume of reactants are used, then no. moles of
50. If aqueous solution of H2O2 is made acidic. For this which KMnO4 (moles per liter) used in acidic medium required to
of the following statement(s) is/are correct ? completely oxidises the 0.5 M FeSO3?
(i) This aqueous solution oxidizes I– (a) 0.3 (b) 0.1 (c) 0.2 (d) 0.4
(ii) This aqueous solution oxidizes F– Topic 4: Redox Reactions and Electrode Process
(a) Both statements (i) and (ii) are correct. 62. Given E°
(b) Statement (i) is correct and (ii) is incorrect. (i) Mg2+/Mg(s), E° = –2.36
(c) Statement (ii) is correct and (i) is incorrect. (ii) Ag+/Ag(s), E° = 0.80
(d) Both statements (i) and (ii) are incorrect. (iii) Al3+/Al(s), E° = –1.66
51. When Cl2 gas reacts with hot and concentrated sodium (iv) Cu2+/Cu(s), E° = 0.52
hydroxide solution, the oxidation number of chlorine Out of the above given elements which is the strongest
changes from oxidising agent and which is the weakest oxidising agent?
(a) zero to +1 and zero to –5 (a) (iv) is the strong whereas (ii) is the weakest
(b) zero to –1 and zero to +5 oxidising agent
(c) zero to –1 and zero to +3 (b) (ii) is the strongest whereas (i) is the weakest
(d) zero to +1 and zero to –3 oxidising agent
52. Which of the following act as reducing agents ? (c) (i) is the strongest whereas (ii) is the weakest
(i) PO34- (ii) SO3 (iii) PO32 - (iv) NH3 oxidising agent
(a) (i), (ii) and (iii) (b) Only (iii) (d) (ii) is the strongest whereas (iii) is the weakest
(c) (i), (iii) and (iv) (d) (iii) and (iv) oxidising agent
53. In the reaction shown below, oxidation state of the carbon 63. Stronger is oxidising agent, more is
in reactant and product are (i) and (ii) respectively? Is the (a) standard reduction potential of that species
given reaction a redox reaction? (b) the tendency to get itself oxidised
Na2CO3(aq) + HCl (aq) (c) the tendency to lose electrons by that species
¾¾ ® Na Å ( aq ) + Cl - ( aq ) + H 2 O ( l ) + CO 2 ( g ) (d) standard oxidation potential of that species
EBD_8350
118 CHEMISTRY

64. The standard reduction potentials at 298K for the following 69. Given :
half reactions are given against each Eo 1 = 1.36 V, E o = - 0.74 V,
Zn2+ (aq) + 2e– ƒ Zn(s) ; –0.762 V Cl2 / Cl- Cr3+ / Cr
2
Cr3+ (aq) + 3e– ƒ Cr (s); –0.740 V
Eo = 1.33V, E o = 1.51V
2H+ (aq) + 2e– ƒ H2 (g) ; 0.00 V Cr2O 72 - / Cr 3+ MnO-4 / Mn 2 +

Fe3+ (aq) + e– ƒ Fe2+ (aq) ; 0.770 V The correct order of reducing power of the species
Which is the strongest reducing agent? (Cr, Cr3+, Mn2+ and Cl–) will be
(a) Zn (s) (b) Cr (s) (c) H2(g) (d) Fe3+ (aq) (a) Mn2+ < Cl– < Cr3+ < Cr
65. Electrode potential data are given below : (b) Mn2+ < Cl3+ < Cl– < Cr
(c) Cr3+ < Cl– < Mn2+ < Cr
Fe +3 (aq) + e – ¾¾® Fe+2 (aq); E° = +0.77 V (d) Cr3+ < Cl– < Cr < Mn2+
Al3+ (aq) + 3e – ¾¾
® Al(s) ; E° = – 1.66 V 70. Arrange the following in the order of their decreasing
electrode potentials : Mg, K, Ba and Ca
Br2 (aq) + 2e – ¾¾ ® 2Br – (aq); E° = + 1.08V (a) K, Ca, Ba, Mg (b) Ba, Ca, K, Mg
Based on the data, the reducing power of Fe2+, Al and Br – (c) Ca, Mg, K, Ba (d) Mg, Ca, Ba, K
will increase in the order 71. The standard electrode potentials of four elements A, B, C
(a) Br– < Fe2+ < Al (b) Fe2+ < Al < Br – and D are –3.05, –1.66, –0.40 and +0.80. The highest

(c) Al < Br < Fe 2+ (d) Al < Fe2+ < Br– chemical reactivity will be exhibited by
66. The standard reduction potentials for Cu2+/Cu; Zn2+/Zn; (a) A (b) B (c) C (d) D
Li +/Li; Ag+/Ag and H+/H2 are + 0.34 V, – 0.762 V, 72. Which of the following statement(s) is/are correct ?
– 3.05 V, + 0.80 V and 0.00 V respectively. Choose the (i) A negative value of E° means that the redox couple is
strongest reducing agent among the following a weaker reducing agent than the H+/H2 couple.
(a) Zn (b) H2 (c) Ag (d) Li (ii) A positive E° means that the redox couple is weaker
67. Standard reduction potentials of the half reactions are reducing agent than the H+/H2.
given below : Which of the following code is incorrect regarding above
F2(g) + 2e– ¾® 2F– (aq); E° = + 2.85 V statements?
Cl2(g) + 2e– ¾® 2Cl–(aq); E° = + 1.36 V (a) Only (i) (b) only (ii)
Br2(l) + 2e– ¾® 2Br–(aq); E° = + 1.06 V (c) Both (i) and (ii) (d) Neither (i) nor (ii)
I2(s) + 2e– ¾® 2I–(aq); E° = + 0.53 V 73. Which of the following statements are correct concerning
The strongest oxidising and reducing agents respectively redox properties?
are : (i) A metal M for which E° for the half life reaction
(a) F2 and I– (b) Br2 and Cl– ˆˆ† M is very negative will be a good
M n+ + ne– ‡ˆˆ
(c) Cl2 and Br – (d) Cl2 and I2 reducing agent.
68. Standard electrode potentials of redox couples (ii) The oxidizing power of the halogens decreases from
A2+/A, B2+/B, C2+/C and D2+/D are 0.3V, – 0.5V, – 0.75V and chlorine to iodine.
0.9V respectively. Which of these is best oxidising agent (iii) The reducing power of hydrogen halides increases
and reducing agent respectively – from hydrogen chloride to hydrogen iodide
(a) D2+/D and B2+/B (b) B2+/B and D2+/D (a) (i), (ii) and (iii) (b) (i) and (ii)
(c) D2+/D and C2+/C (d) C2+/C and D2+/D (c) (i) only (d) (ii) and (iii)

NCERT Exemplar MCQs Cu2+/Cu = + 0.34; Ag+/Ag = 0.80 V


1. Which of the following is not an example of redox reaction? (a) Fe3+ (b) I2(s) (c) Cu2+ (d) Ag+
(a) CuO + H 2 ¾¾ ® Cu + H 2 O 3. The oxidation number of an element in a compound is
(b) Fe2 O3 + 3CO ¾¾ ® 2Fe + 3CO 2 evaluated on the basis of certain rules. Which of the
(c) 2K + F2 ¾¾ ® 2KF following is incorrect in this respect?
(d) BaCl 2 + H 2SO 4 ¾¾ ® BaSO 4 + 2HCl (a) The oxidation number of hydrogen is always + 1
Redox reactions are those reactions which involve change (b) The algebraic sum of all the oxidation numbers in a
compound is zero
in oxidation number of the reactants. (c) An element in the free or the uncombined state bears

2. The more positive the value of E , the greater is the oxidation number zero
tendency of the species to get reduced. Using the standard (d) In all its compounds, the oxidation number of fluorine
electrode potential of redox couples given below find out is –1
4. In which of the following compounds, an element exhibits
which of the following is the strongest oxidising agent. two different oxidation states?

E values : Fe3+/Fe2+ = + 0.77V ; I2(s)/I – = + 0.54; (a) NH2OH (b) NH4NO3 (c) N2H4 (d) N3H
REDOX REACTIONS 119

5. The largest oxidation number exhibited by an element 11. Which of the following elements does not show
depends on its outer electronic configuration. With which disproportionation tendency?
of the following outer electronic configurations the (a) Cl (b) Br (c) F (d) I
element will exhibit largest oxidation number ?
Past Year MCQs
(a) 3d1 4s2 (b) 3d2 4s2 (c) 3d5 4s1 (d) 3d5 4s2
– 12. Which of the following reactions is an example of a redox
6. E values of some redox couples are given below. On the
basis of these values, choose the correct option. reaction? [JEE M 2017, C]
– (a) XeF4 + O2F2 ® XeF6 + O2
E values: Br2/Br – = + 1.09V
Ag+/ Ag(s) = + 0.80V (b) XeF2 + PF5 ® [XeF]+ PF6–
Cu2+/Cu(s) = + 0.34V; I2 (s) / I– = + 0.54V (c) XeF6 + H2O ® XeOF4 + 2HF
(a) Cu will reduce Br – (b) Cu will reduce Ag (d) XeF6 + 2H2O ® XeO2F2 + 4HF
(c) Cu will reduce I – (d) Cu will reduce Br 2 13. For the redox reaction [NEET 2018, S]
7. Using the standard electrode potential, find out the pair – 2–
MnO 4 + C 2 O 4 + H ¾¾ + 2+
® Mn + CO 2 + H 2 O
between which redox reaction is not feasible.
– The correct coefficients of the reactants for the balanced
E values: Fe3+/ Fe2+ = + 0.77; I2/I– = + 0.54; equation are
Cu2+ / Cu = +0.34; Ag+ / Ag = + 0.80 V MnO4– Cr2O42– H+
(a) Fe3+ and I– (b) Ag+ and Cu (a) 16 5 2
3+
(c) Fe and Cu (d) Ag and Fe3+ (b) 2 5 16
Determine the E°cell of the four redox reactions. If the value of (c) 5 16 2
E°cell of a reaction is negative, that reaction will not take place. (d) 2 16 5
8. Thiosulphate reacts differently with iodine and bromine 14. The oxidation states of Cr in [Cr(H2O)6]Cl3, [Cr(C6H6)2],
in the reactions given below. and K2[Cr(CN)2(O)2(O2)(NH3)] respectively are :
2S2 O32 - + I 2 ® S4 O62 - + 2I - [JEE M 2018, S]
(a) +3, +4 and +6 (b) +3, +2 and +4
S2 O32 - + 2Br 2 + 5H 2 O ® 2SO 24 - + 2Br - + 10H + (c) +3, 0 and +6 (d) +3, 0 and + 4
Which of the following statements justifies the above dual 15. Which of the following reactions are disproportionation
behaviour of thiosulphate? reaction? [NEET 2019, C]
(a) Bromine is a stronger oxidant than iodine (1) 2Cu+ ® Cu2+ + Cu0
(b) Bromine is a weaker oxidant than iodine (2) 3MnO42– + H+ ® 2MnO–4 + MnO2 + 2H2O
D
(c) Thiosulphate undergoes oxidation by bromine and (3) 2KMnO–4 ¾¾® K2MnO4 + MnO2 + O2
reduction by iodine in these reactions (4) 2MnO–4 + 3Mn2+ + 2H2O ® 5MnO2 + 4HÅ
(d) Bromine undergoes oxidation and iodine undergoes
reduction in these reactions Select the correct option from the following:
9. Which of the following arrangements represent increasing (a) (1) and (2) only (b) (1), (2) and (3)
oxidation number of the central atom? (c) (1), (3) and (4) (d) (1) and (4) only
(a) CrO2- , ClO3- , CrO24 - , MnO4- 16. What is the change in oxidation number of carbon in the
following reaction? [NEET 2020, C]
(b) ClO3- , CrO 42 - , MnO -4 , CrO -2 CH4(g) + 4Cl2(g) ® CCl4(l) + 4HCl(g)
(c) CrO2- , ClO3- , MnO4- , CrO42 - (a) 0 to + 4 (b) – 4 to + 4
(d) CrO 24 - , MnO 4- , CrO 2- , ClO 3- (c) 0 to – 4 (d) + 4 to + 4
10. Identify disproportionation reaction 17. Oxidation number of potassium in K2O, K2O2 and KO2,
(a) CH 4 + 2O 2 ¾¾ ® CO 2 + 2H 2 O respectively, is: [JEE M 2020, S]
(b) CH 4 + 4Cl 2 ¾¾ ® CCl4 + 4HCl 1
(a) +2, +1 and + (b) +1, +1 and +1
(c) 2F2 + 2OH - ¾¾ ® 2F - + OF2 + H 2 O 2
- (c) +1, +4 and +2 (d) +1, +2 and +4
(d) 2NO 2 + 2OH ¾¾ ® NO 2- + NO 3- + H 2 O

1. Which of the following reactions do not involve oxidation 2. Which of the following represents redox reactions?
reduction? 2- - 2-
I. Cr2 O 7 + 2OH ¾¾ ® 2CrO 4 + H 2 O
I. 2Cs + 2H 2 O ¾¾ ® 2CsOH + H 2
II. Zn + CuSO4 ¾¾
® ZnSO 4 + Cu
II. 2CuI 2 ¾¾ ® 2Cul + I2
III. 2MnO -4 + 3Mn 2+ + 4OH – ¾¾ ® 5MnO 2 + 2H 2O
III. NH 4Br + KOH ¾¾ ® KBr + NH3 + H 2O
IV. 2Cu ¾¾+ 2 +
® Cu + Cu
IV. 4KCN + Fe ( CN )2 ¾¾ ® K 4 [Fe ( CN )6 ]
(a) I, II (b) I, III (c) III, IV (d) II, III, IV
(a) I, II (b) I, III (c) I, III, IV (d) III, IV
EBD_8350
120 CHEMISTRY

3. Which of the following reactions does not involve either 10. The oxidation states of sulphur in the anions SO32– ,
oxidation or reduction? S2O42–, and S2O62– follow the order.
(a) VO 2+ ¾¾
® V2 O3 ® Na +
(b) Na ¾¾ (a) S2O42– < SO32– < S2O62–
(b) SO32– < S2O42– < S2O62–
(c) CrO 24 - ¾¾ ® Cr2 O 72- (d) Zn 2+ ¾¾ ® Zn (c) S2O42– < S2O62– < SO32–
4. Which of the following is an intermolecular redox reaction? (d) S2O62– < S2O62– < SO32–
-
OH H+
(a) 2OCH — CHO ¾¾¾
® HOCH 2 — CH 2 OH 11. Cr2 O 2– ® Cr 3+ + H 2O + oxidised product of X,
7 + X ¾¾¾
Al( OC2 H5 ) X in the above reaction cannot be
(b) 2C6 H5CHO ¾¾¾¾¾¾
3 ® C H COOH +
6 5 (a) C2O42– (b) Fe2+ (c) SO42– (d) S2–
C6 H5CH 2 OH 12. The equivalent mass of oxidising agent in the following
reaction is
(c) ® 2Cr2 (SO 4 )3 +
4CrO 5 + 6H 2SO 4 ¾¾
SO 2 + 2H 2S ¾¾ ® 3S + 2H 2 O
6H 2O + 7O2 (a) 32 (b) 64 (c) 16 (d) 8
(d) As 2S3 + HNO3 ¾¾ 13. Which of the following is not a disproportionation
® H 3 AsO 4 + H 2SO 4 + NO
reaction?
5. To an acidic solution of an anion, a few drops of KMnO4 D
solution are added. Which of the following, if present, I. NH 4 NO3 ¾¾® N 2 O + H 2 O
will not decolourise the KMnO4 solution? D
II. ® PH 3 + HPO 2-
P4 ¾¾
(a) CO32– (b) NO -2 (c) S2– (d) Cl - D
III. PCl5 ¾¾® PCl3 + Cl 2
6. The oxidation number of phosphorus in Ba(H2PO2)2 is
(a) +3 (b) +2 (c) +1 (d) –1 IV. H 2 O 2 ¾¾ ® O 2 + 2e-
7. Zn gives H2 gas with H2SO4 and HCl but not with HNO3 (a) I, II (b) I, III, IV (c) II, IV (d) I, III
because 14. Which of the following is not a disproportionation
(a) Zn acts as an oxidising agent when it reacts with HNO3 reaction?
(b) HNO3 is weaker acid than H2SO4 and HCl Al( OEt )
(a) 2PhCHO ¾¾¾¾3¾ ® PhCOOCH 2 Ph
(c) In electrochemical series, Zn is above hydrogen
(b) CHO –
(d) NO3- is reduced in preference to hydronium ion
+ OH ¾¾ ® CH 2OH COO–
8. The oxidation state of nitrogen is correctly given for +
COOH
Compound Oxidation state
COO– COO–
(a) éë Co ( NH3 )5 Clùû Cl2 0 (c) NaH + H 2O ¾¾ ® NaOH + H 2
(b) NH2OH –2 (d) All of the above.
(c) (N2H5)2SO4 +2 15. Consider the following reaction occuring in basic medium
(d) Mg3N2 –3 2MnO 4– (aq) + Br – (aq) ¾¾
® 2MnO 2 (s) + BrO 3– (aq)
9. Which of the following statements is not correct? How the above reaction can be balanced further?
(a) The oxidation number of S in (NH4)2S2O8 is +6. (a) By adding 2 OH– ions on right side
(b) The oxidation number of Os in OsO4 is +8. (b) By adding one H2O molecule to left side
(c) The oxidation number of S in H2SO5 is +8. (c) By adding 2H+ ions on right side
(d) The oxidation number of O in KO2 is –1/2. (d) Both (a) and (b)
ANSW ER KEY
Exercise 1 : NCERT Based Topic-wis e MCQs
1 (c) 9 (c) 17 (b) 25 (d) 33 (b) 41 (a) 49 (c) 57 (b) 65 (a) 73 (a)
2 (d) 10 (d) 18 (c) 26 (a) 34 (d) 42 (b) 50 (b) 58 (c) 66 (d)
3 (b) 11 (d) 19 (c) 27 (b) 35 (b) 43 (b) 51 (b) 59 (b) 67 (a)
4 (b) 12 (a) 20 (a) 28 (c) 36 (c) 44 (b) 52 (d) 60 (a) 68 (c)
5 (a) 13 (a) 21 (c) 29 (b) 37 (d) 45 (c) 53 (c) 61 (a) 69 (a)
6 (b) 14 (d) 22 (c) 30 (d) 38 (b) 46 (c) 54 (c) 62 (b) 70 (d)
7 (d) 15 (c) 23 (d) 31 (a) 39 (a) 47 (d) 55 (a) 63 (a) 71 (a)
8 (a) 16 (d) 24 (c) 32 (c) 40 (b) 48 (c) 56 (b) 64 (a) 72 (a)
Exercis e 2 : NCERT Exemplar & Past Year MCQs
1 (d) 3 (a) 5 (d) 7 (d) 9 (a) 11 (c) 13 (b) 15 (a) 17 (b)
2 (d) 4 (b) 6 (d) 8 (a) 10 (d) 12 (a) 14 (c) 16 (b)
Exercise 3 : Problem Solving S kill Enhancer MCQs
1 (d) 3 (c) 5 (a) 7 (d) 9 (c) 11 (c) 13 (b) 15 (d)
2 (d) 4 (d) 6 (c) 8 (d) 10 (a) 12 (c) 14 (c)
9 Hydrogen

Trend Buster NEET & JEE Main

Number of Questions from 2020-15 2 6 Minimum one question has been


asked every year in JEE.
Weightage 0.80% 3.40%

The most Important Concepts that Cover Maximum number of Questions asked in past 6 years.

Hydrogen peroxide — 2
Water 1 3
Less Important Concepts that Cover 1 or 2 Questions asked in past 6 years.

Position of hydrogen in the periodic table 1 1

NEET JEE

2020 Water Hardness of water — — 1 Average

2019 Water/position of hydrogen water of hydration/hardness 1 Easy 2 Easy


of water/isotopes of hydrogen

2018 Hydrogen peroxide Hydrogen peroxide — — 1 Average

2017 — — — — — —

2016 Water / position of hydrogen properties of water/isotopes 1 Average 1 Average


of hydrogen
2015 Hydrogen peroxide Hydrogen peroxide — — 1 Easy
EBD_8350
122 CHEMISTRY
HYDROGEN 123
EBD_8350
124 CHEMISTRY

Problem Solving Tips/ Tricks/ Points to Remember

4 The presence of Ca2+ and Mg2+ salts make water hard. (b) H2O2 structure in solid phase at 110 K, dihedral angle is
The presence of Ca/Mg bicarbonate causes temporary 90.2°
hardness in water, whereas the presence of chlorides 4 Tests for Hydrogen Peroxide
and sulphates of calcium/magnesium causes permanent (i) It liberates I2 from acidified KI
hardness. (ii) Perchromic acid :
4 The strength of hydrogen peroxide (or its solution) is (acidified H 2 O 2 + amyl alcohol + K 2Cr2O 7 ) ® blue colour
expressed in terms of volume of oxygen (at NTP) that one (iii) With titanic sulphate it gives orange red pertitanic acid
unit volume H2O2 sample gives on heating. The sample (iv) Black lead sulphide turned white
of H2O2 that gives x mL of oxygen (at NTP) per mL of the 4 Water can be tested as
H2O2 sample is said to have a strength of “x volume”. (a) On addition of a drop of water to anhydrous copper
4 In H2O2, the oxidation state of O is –1. H2O2 acts both as sulphate, the colour changes from white to blue
an oxidising as well as reducing agent. It is a mild CuSO 4 +5H 2 O ® CuSO 4 .5H 2O
bleaching agent. Anhydrous Blue
4 Heavy water (D2O) was discovered by Urey (1933). (b) On reaction with CaC2, acetylene evolves which burns
4 30% (w/v) H2O2 is known as perhydrol. with bright flame
4 Removal of permanent hardness CaC 2 + 2H 2 O ® Ca(OH) 2 + HC º CH
Calgon process : Calgon is sodium water is passed Acetylene

through the bed of Mg2+ form soluble complex. 4 H2O2 acts as an oxidising and a reducing agent in the
Permutit process : Permutit is hydrated silicate acidic as well as basic medium
Na2Al2Si2O8.xH2O. It exchanges divalent ions such as (a) Oxidising agent:
Ca2+ and Mg2+. (i) In acidic medium: H 2O 2 ∗ 2H ∗ ∗ 2e, ® 2H 2 O
4 Deuterium peroxide, D2O2, is prepared by the action of (ii) In basic medium: H 2O 2 + 2e - ® 2OH –
D2SO4 (dissolved in H2O) in BaO2. (b) Reducing agent:
(i) BaO 2 + D2SO 4 ® BaSO 4 ¯ + D 2 O 2 (i) In acidic medium: H 2O 2 ® 2H + + O2 + 2e-
(ii) In basic medium: H2O2 + 2OH– ® 2H2O + O2 + 2e–
(ii) 2NaCl + D2SO 4 ® Na 2SO4 + 2DCl 4 Hydrogen forms most number of compounds than only
(iii) 2AgCl + D2 ® 2Ag + 2DCl other element (even more than carbon).
4 Since H2 is inflammable, a mixture of He and other gases
(iv) SOCl 2 + 2D2O ® D2SO3 + 2DCl are used in balloons.
4 Storage of Hydrogen Peroxide 4 Triatomic hydrogen (H+3) is called hyzone.
4 pH of H2O and D2O at 298 K are 7.0 and 7.35 respectively.
It is stored in presence of traces of alcohol, acetanilide
4 Ice is a good thermal insulator.
or sodium pyrophosphate which slow down the rate of 4 The state (solid, liquid or gas) of water cannot be defined
decomposition of hydrogen peroxide. in hydrates such as CuSO4·5H2O. This is because water
4 Structure of Hydrogen Peroxide molecules are not bonded to each other but with other
Hydrogen peroxide has a non-planar structure. The atoms.
molecular dimensions in the gas phase and solid phase 4 The dielectric constant of H2O and H2O2 at 298K are
are shown in figure 78.39 and 70.7 respectively.
4 Theoretically six different types of heavy water are
possible:
H H 16 17 18 16 17
95.0 pm 98.8 pm H – O – D, H – O – D, H – O – D, D – O –D, D – O – D,
147.5 pm 111.5° 145.8 pm 90.2° 18
O O O O
94.8° 101.9°
D– O –D
H H 4 Tritiated water or tritium oxide (T2O or 3H2O) is called
super-heavy water.
(a) Gas Phase (b) Solid phase
4 D2O is a toxic substance due to slower rate of D+ in enzyme
catalysis.
(a) H2O2 structure in gas phase, dihedral angle is 111.5°. 4 Perhydrol, which is 30% H2O2 is used as an antiseptic.
HYDROGEN 125

Exercise 1 : NCERT Based Topic-wise MCQs


Topic 1: Position of Hydrogen in the Periodic Topic 2: Preparation of Dihydrogen
Table and Dihydrogen 8. Which of the following is formed when zinc reacts with
1. The property of hydrogen which distinguishes it from alkali sodium hydroxide?
metals is (a) Hydrogen gas (b) Sodium zincate
(a) its electropositive character (c) Zinc oxide (d) Both (a) and (b)
(b) its affinity for non metal 9. Which one of the following pairs of substances on reaction
(c) its reducing character will not evolve H2 gas?
(d) its non-metallic character (a) Iron and H2SO4 (aqueous)
(b) Iron and steam
2. Which of the following statements is correct ?
(c) Copper and HCl (aqueous)
(a) Hydrogen has same IP as alkali metals.
(d) Sodium and ethyl alcohol
(b) Hydrogen has same electronegativity as halogens.
10. In Bosch’s process which gas is utilised for the production
(c) It has oxidation number of –1 and +1.
of hydrogen gas ?
(d) It will not be liberated at anode.
(a) Producer gas (b) Water gas
3. Number of neutrons in three isotopes of hydrogen,
(c) Coal gas (d) None of these
protium, deuterium and tritium respectively is
(a) 0, 1, 2 (b) 1, 1,1 11. Which of the following reaction(s) represents commercial
(c) 2, 1, 0 (d) 2, 0, 1 method for production of dihydrogen?
4. Choose the incorrect statement 673 K
(i) CO(g) + H2O(g) ¾¾¾¾
catalyst
® CO (g) + H (g)
2 2
(a) Dihydrogen can release more energy than petrol.
(b) The only pollutant in combustion of dihydrogen is electrolysis
(ii) 2H2O(l) ¾¾¾ ¾ ¾¾ ¾
traces of acid/base
® 2H2(g) + O2(g)
carbon dioxide.
(c) Hydrogen economy is based on the principle of (iii) Zn + 2H+ ¾¾
® Zn2+ + H2
transportation and storage of energy in the form of 1270K
liquid or gaseous dihydrogen. (iv) CH4(g) + H2O(g) ¾¾ ¾¾ ® CO(g) + 3H2(g)
Ni
(d) Hydrogen economy has advantage that energy is (a) (i), (ii) and (iii) (b) (iii) only
transmitted in the form of dihydrogen and not as (c) (i), (ii) and (iv) (d) (ii), (iii) and (iv)
electric power. 12. Which of the following statements is correct?
5. Which of the following fuel is used for runnning the (a) Production of syn gas from coal is called coal
automobiles first time in the history of India during October gasification.
2005?
673K
(a) D2O (b) H2O2 (b) CO(g) + H2O(g) ¾¾¾¾
catalyst
® CO (g) + H (g)
2 2
(c) D2 (d) H2
6. Assertion: H+ does not exist freely and is always represents water gas shift reaction.
associated with other atoms or molecules. (c) CO2 formed in water gas shift reaction is removed
Reason: Loss of the electron from hydrogen atom by scrubbing with sodium zincate solution.
results in nucleus (H+) of ~ 1.5 × 10–3 pm size. This is (d) Both (a) and (b)
extremely small as compared to normal atomic and ionic 13. Which of the following metal evolves hydrogen on reacting
sizes of 50 to 200 pm. with cold dilute HNO3 ?
(a) Assertion is correct, reason is correct; reason is a (a) Mg (b) Al
correct explanation for assertion. (c) Fe (d) Cu
(b) Assertion is correct, reason is correct; reason is not 14. Hydrogen is not obtained when zinc reacts with
a correct explanation for assertion (a) Cold water (b) dil. HCl
(c) Assertion is correct, reason is incorrect (c) dil. H2SO4 (d) Hot NaOH (20%)
(d) Assertion is incorrect, reason is correct. 15. Which one of the following pairs of substances will not
7. Hydrogen can behave as a metal produce hydrogen when reacted together?
(a) at very high temperature (a) Copper and conc. nitric acid
(b) at very low temperature (b) Ethanol and metallic sodium
(c) at very high pressure (c) Magnesium and steam
(d) at very low pressure (d) Phenol and metallic sodium
EBD_8350
126 CHEMISTRY

16. Very pure hydrogen (99.9%) can be made by which of the (d) Electron deficient hydride = CH4 and HF
following processes ? Electron precise = B2H6
(a) Reaction of methane with steam Electron rich hydride = NH3,
(b) Mixing natural hydrocarbons of high molecular weight 23. Which hydride is an ionic hydride ?
(c) Electrolysis of water (a) H2S (b) TiH1.73
(d) Reaction of salts like hydrides with water (c) NH3 (d) NaH
Topic 3: Properties of Dihydrogen 24. Saline hydrides react explosively with water, such fires
can be extinguished by
17. Which of the following is formed on reaction of carbon
(a) water (b) carbon dioxide
monoxide gas with dihydrogen in presence of cobalt as
(c) sand (d) None of these
a catalyst?
25. The polymeric hydride is
(a) Methanal (b) Methanol
(a) CaH2 (b) NaH
(c) Methane (d) Formic acid
(c) BaH2 (d) MgH2
18. Which of the following is not a use of dihydrogen ?
26. Ionic hydrides reacts with water to give
(a) It used in fuel cells for generating electrical energy.
(a) acidic solutions (b) hydride ions
(b) Atomic hydrogen and oxy-hydrogen torches are
(c) basic solutions (d) electorns
used for cutting and welding purposes.
27. Which of the following is incorrect statement?
(c) It used in the synthesis of hydroquinone and
(a) s-block elements, except Be and Mg, form ionic hydride
tartaric acid.
(b) BeH4, MgH2, CuH2, ZnH2, CaH2 and HgH2 are
(d) Both (b) and (c)
intermediate hydride
19. Hydrogen will not reduce
(a) heated cupric oxide (d) heated ferric oxide (c) p-block elements form covalent hydride
(c) heated stannic oxide (d) heated aluminium oxide (d) d-and f-block elements form ionic hydride
20. Which of the following statement(s) is/are incorrect? 28. The hydride ion, H–, is a stronger base than the hydroxide
(i) Dihydrogen reduces copper (II) oxide to copper ion, OH–. Which one of the following reactions will occur
(ii) Reaction of dihydrogen with sodium gives sodium if sodium hydride (NaH) is dissolved in water?
hydride. (a) H - (aq) + H 2 O(l) ® H 3O - (aq)
(iii) Hydroformylation of olefins yields aldehydes which (b) H - (aq) + H 2O(l) ® OH- (aq) + H 2 (g)
further undergo reduction to give alcohols. (c) H - (aq) + H 2 O(l) ® OH - (aq) + 2 H + (aq) + 2e -
(iv) Hydrogenation of vegetable oils using iron as (d) H - ( aq ) + H 2 O ( l) ® No reaction
catalyst gives edible fats.
29. The storage tanks used for H2 are made up of which
(a) (i), (ii) and (iii) (b) (i) and (iv)
metal alloy(s)
(c) (iv) only (d) (iii) and (iv)
(i) NaNi5 (ii) B2H6
21. Why is water gas (mixture of CO and H2) also called ‘syn
(iii) Ti–TiH2 (iv) Mg–MgH2
gas’?
(a) (iii) and (iv) (b) (i) and (ii)
(a) Because it is synthesised from sewage, saw – dust,
(c) (i), (iii) and (iv) (d) (ii), (iii) and (iv)
scrap wood etc.
(b) Because it is synthesised from methane gas. Topic 5: Water
(c) Because it is used in the synthesis of methanol and 30. The unusual properties of water in the condensed phase
a number of hydrocarbons. (liquid and solid states) are due to the
(d) None of these (a) presence of hydrogen and covalent bonding between
Topic 4: Hydrides the water molecules
22. Choose the correct option for following hydrides. (b) presence of covalent bonding between the water
B2H6, CH4, NH3 and HF molecules
(a) Electron deficient hydride = B2H6 and HF (c) presence of extensive hydrogen bonding between
Electron precise hydride = CH4 water molecules
Electron rich hydride = NH3 (d) presence of ionic bonding
(b) Electron deficient hydride = B2H6 31. At its melting point ice is lighter than water because
Electron precise hydride = CH4 (a) H2O molecules are more closely packed in solid state
Electron rich hydride = NH3 and HF (b) ice crystals have hollow hexagonal arrangement of
(c) Electron deficient hydride = CH4 H2O molecules.
Electron precise hydride = B2H6 (c) on melting of ice the H2O molecule shrinks in size
Electron rich hydride = NH3 and HF (d) ice froms mostly heavy water on first melting.
HYDROGEN 127

32. When two ice cubes are pressed over each other, they 40. Assertion : Calgon is used for removing permanent
unite to form one cube. Which of the following forces is hardness of water.
responsible to hold them together ? Reason : Calgon forms precipitates with Ca2+ and Mg2+.
(a) van der Waals forces (a) Assertion is correct, reason is correct; reason is a
(b) Hydrogen bond formation correct explanation for assertion.
(c) Covalent attraction (b) Assertion is correct, reason is correct; reason is not
(d) Ionic interaction a correct explanation for assertion.
33. Which of the following groups of ions makes the water hard? (c) Assertion is correct, reason is incorrect
(a) Sodium and bicarbonate (d) Assertion is incorrect, reason is correct.
(b) Magnesium and chloride 41. Calgon used as a water softener is
(c) Potassium and sulphate
(a) Na 2 [Na 4 (PO 3 )6 ] (b) Na 4 [ Na 2 (PO 3 ) 6 ]
(d) Ammonium and chloride
34. Assertion : Hard water is not suitable for laundary. (c) Na 4 [Na 4 (PO 4 )5 ] (d) Na 4 [ Na 2 ( PO 4 ) 6 ]
Reason : Soap containing sodium stearate reacts with hard
water to precipitate out as calcium or magnesium stearate. Topic 6: Hydrogen Peroxide (H2O2)
(a) Assertion is correct, reason is correct; reason is a
42. In lab H2O2 is prepared by
correct explanation for assertion.
(b) Assertion is correct, reason is correct; reason is not (a) Cold H2SO4 + BaO2 (b) HCl + BaO2
a correct explanation for assertion. (c) Conc. H2SO4 + Na2O2 (d) H2 + O2
(c) Assertion is correct, reason is incorrect. 43. 30 volume hydrogen peroxide means
(d) Assertion is incorrect, reason is correct. (a) 30% of H 2 O 2 solution
35. Water possesses a high dielectric constant, therefore : (b) 30 cm3 solution contains 1g of H 2 O 2
(a) it always contains ions
(b) it is a universal solvent (c) 1 cm3 of solution liberates 30 cm3 of O2 at STP
(c) can dissolve covalent compounds (d) 30 cm3 of solution contains 1 mole of H2O2
(d) can conduct electricity 44. The volume strength of 1.5 N H2O2 solution is :
36. Polyphosphates are used as water softening agents (a) 8.4 (b) 8.0 (c) 4.8 (d) 3.0
because they 45. Commercial 10 volume H2O2 is a solution with a strength
(a) form soluble complexes with anionic specise of approximately
(b) precipitate anionic species (a) 15% (b) 3% (c) 1% (d) 10%
(c) forms soluble complexes with cationic species 46. Which of the following is the true structure of H2O2 ?
(d) precipitate cationic species (a) H – O – O – H (b) H
37. Permanent hardness of water can be removed by adding O O
H
Calgon (NaPO3)n. This is an example of H + –
(a) Adsorption (b) Exchange of ion (c) O–O (d) ¬
(c) Precipitation (d) None H
38. Match the columns 47. When H2O2 is oxidised the product is
Column-I Column-II (a) OH– (b) O2 (c) O2– (d) HO2–
(A) Coordinated water (p) [Cu(H2O)4]4+SO2– 48. Which of the following is false about H2O2
4. H2O
(B) Interstitial water (q) C17H35COONa (a) Act as both oxidising and reducing agent
(b) Two OH bonds lies in the same plane
(C) Hydrogen-bonded (r) BaCl2.2H2O
(c) Pale blue liquid
water (s) [Cr(H2O)6]3+ 3Cl– (d) Can be oxidised by ozone
(a) A – (r), B – (s), C – (q) (b) A – (q), B – (r), C – (s) 49. The reaction
(c) A – (r), B – (q), C – (p) (d) A – (s), B – (r), C – (p)
39. Match the Column-I with Column-II and mark the H 2S + H 2 O 2 ¾
¾® S + 2H 2 O manifests
appropriate choice. (a) Acidic nature of H2O2
Column-I Column-II (b) Alkaline nature of H2O2
(A) Syn gas (p) Na6P6O18 (c) Oxidising action of H2O2
(B) Calgon (q) NaAlSiO4 (d) Reducing action of H2O2.
(C) Permutit (r) CO + H2 50. Which of the following statements is incorrect ?
(D) Producer gas (s) CO + N2 (a) H2O2 can act as an oxidising agent
(a) (A) – (p), (B) – (q), (C) – (r), (D) – (s) (b) H2O2 can act as a reducing agent
(b) (A) – (r), (B) – (p), (C) – (q), (D) – (s)
(c) H2O2 has acidic properties
(c) (A) – (r), (B) – (q), (C) – (s), (D) – (p)
(d) (A) – (r), (B) – (q), (C) – (p), (D) – (s) (d) H2O2 has basic properties
EBD_8350
128 CHEMISTRY

51. H2O2 is (c) x = HO3SOOSO3H (aq), y = H2SO4(aq)


(a) Poor polar solvent than water (d) x = H2SO4(aq) , y = HO3SOOSO3H(aq)
(b) Better polar solvent than H2O 58. H2O2 ® 2H+ + O2 + 2e– ; E° = – 0.68 V. This equation
(c) Both have equal polarity represents which of the following behaviour of H2O2.
(d) Better polar solvent but its strong auto-oxidising
(a) Reducing (b) Oxidising
ability limits its use as such
(c) Acidic (d) Catalytic
52. Assertion : H2O2 is not stored in glass bottles.
59. Which one of the following undergoes reduction with
Reason : Alkali oxides present in glass catalyse the
hydrogen peroxide in an alkaline medium ?
decomposition of H2O2.
(a) Assertion is correct, reason is correct; reason is a (a) Mn 2+ (b) HOCl
correct explanation for assertion. (c) PbS (d) I2
(b) Assertion is correct, reason is correct; reason is not
Topic 7: Heavy Water (D2O)
a correct explanation for assertion.
(c) Assertion is correct, reason is incorrect. 60. What is formed when calcium carbide reacts with heavy
(d) Assertion is incorrect, reason is correct. water?
53. HCl is added to following oxides. Which one would give (a) C2D2 (b) CaD2
H2O2 (c) Ca2D2O (d) CD2
(a) MnO2 (b) PbO2 61. D2O is used in
(c) BaO (d) None (a) motor vehicles (b) nuclear reactor
54. Which of the following is not true for hydrogen peroxide? (c) medicine (d) insecticide
(a) H2O2 decomposes slowly on exposure to light. 62. Complete the following reaction.
(b) It is kept away from dust because dust can induce Al4C3 + D2O ® x + y
explosive decomposition of the compound. (a) x = C2D2 and y = Al(OD)3
(c) H2O2 is used as bleaching agent for textiles, paper (b) x = CD4 and y = Al(OD)3
pulp etc. (c) x = CO2 and y = Al2D3
(d) It is used as a moderator in nuclear reactor. (d) x = CD4 and y = Al2D3
55. Which of the following is wrong about H2O2? It is used 63. Heavy water reacts respectively with CO2, SO3, P2O5 and
(a) As aerating agent in production of sponge rubber N2O5 to give the compounds :
(b) As an antichlor (a) D2CO3, D2SO4, D3PO2, DNO2
(c) For restoring white colour of blackened lead painting
(b) D2CO3, D2SO4, D3PO4, DNO2
(d) None of these
56. Which of the following statements are correct? (c) D2CO3, D2SO3, D3PO4, DNO2
(i) Hydrogen peroxide is industrially prepared by the (d) D2CO3, D2SO4, D3PO4, DNO3
auto-oxidation of 2-alkylanthraquinols. 64. D2O is preferred to H2O, as a moderator, in nuclear reactors
(ii) One millilitre of 30% H2O2 means that solution will because
give 100 V of oxygen at STP. (a) D2O slows down fast neutrons better
(iii) Dihedral angle of H2O2 in gas phase is 90.2° and in
solid phase dihedral angle is 111.5°. (b) D2O has high specific heat
(a) (i), (ii) and (iii) (c) D2O is cheaper
(b) (i) and (iii) (d) None of these
(c) (ii) and (iii) 65. Which of the following is not true?
(d) (i) and (ii) (a) D2O freezes at lower temperature than H2O
57. Identify x and y in following reaction: (b) Reaction between H2 and Cl2 is much faster than D2
2HSO-4 (aq) ¾¾¾¾¾
electrolysis hydrolysis and Cl2
® x ¾¾¾¾¾ ®
y + 2H+(aq) + H2O2(aq) (c) Ordinary water gets electrolysed more rapidly than
- D2O
(a) x = H2SO4 (aq), y = 2HSO4 (aq)
(d) Bond dissociation energy of D2 is greater than H2
(b) x = HO3SOOSO3H(aq), y = 2HSO -4 (aq)
HYDROGEN 129

NCERT Exemplar MCQs 8. Hydrogen peroxide is .......... .


(a) an oxidising agent
1. Hydrogen resembles halogens in many respects for which
several factors are responsible. Of the following factors (b) a reducing agent
which one is most important in this respect? (c) both an oxidising and a reducing agent
(a) Its tendency to lose an electron to form a cation. (d) neither oxidising nor reducing agent
(b) Its tendency to gain a single electron in its valence 9. When sodium peroxide is treated with dilute sulphuric
shell to attain stable electronic configuration. acid, we get ........ .
(c) Its low negative electron gain enthalpy value. (a) sodium sulphate and water
(d) Its small size. (b) sodium sulphate and oxygen
2. Which of the following hydrides is electron-precise (c) sodium sulphate, hydrogen and oxygen
hydride? (d) sodium sulphate and hydrogen peroxide
(a) B2H6 (b) NH3 10. Hydrogen peroxide is obtained by the electrolysis of .... .
(c) H2O (d) CH4 (a) water (b) sulphuric acid
(c) hydrochloric acid (d) fused sodium peroxide
3. Radioactive elements emit a, b and g rays and are
characterised by their half-lives. The radioactive isotope 11. Which of the following reactions is an example of use of
of hydrogen is water gas in the synthesis of other compounds?
(a) protium (b) deuterium 1270 K
(a) CH 4 (g) + H 2 O(g) ¾¾¾¾
® CO(g) + H 2 (g)
(c) tritium (d) hydronium Ni
4. Consider the reactions
673 K
(i) H2O2 + 2HI ¾¾ ® I2 + 2H2O (b) CO(g) + H 2 O(g) ¾¾¾¾ ® CO2 (g) + H 2 (g)
Catalyst
(ii) HOCl + H2O2 ¾¾
® H3O + + Cl– + O2
1270 K
Which of the following statements is correct about H2O2 (c) C n H 2n + 2 + nH 2 O(g) ¾¾¾¾ ® nCO + (2n + 1)H 2
Ni
with reference to these reactions ? Hydrogen peroxide is .....
(a) an oxidising agent in both (i) and (ii) Cobalt
(d) CO(g) + 2H 2 (g) ¾¾¾¾
® CH 3OH(l)
(b) an oxidising agent in (i) and reducing agent in (ii) Catalyst
(c) a reducing agent in (i) and oxidising agent in (ii)
12. Which of the following ions will cause hardness in water
(d) a reducing agent in both (i) and (ii)
sample?
5. The oxide that gives H2O2 on treatment with dilute H2SO4 is
(a) PbO2 (b) BaO2.8H2O (a) Ca2+ (b) Na+
(c) MnO2 (d) TiO2 (c) Cl– (d) K+
6. Which of the following equations depict the oxidising 13. Elements of which of the following group(s) of periodic
nature of H2O2? table do not form hydrides?
(a) 2MnO–2 + 6H+ + 5H2O2 ¾® 2Mn2+ + 8H2O + 5O2 (a) Groups 7, 8, 9
(b) Group 13
(b) 2Fe3+ + 2H+ + H2O2 ¾® 2Fe2+ + 2H2O + O2
(c) Groups 15, 16, 17
(c) 2I– + 2H+ + H2O2 ¾® I2 + 2H2O
(d) Group 14
(d) KIO4 + H2O2 ¾® KIO3 + H2O + O2 14. Why does H+ ion always get associated with other atoms
7. Which of the following equation depict reducing nature or molecules?
of H2O2? (a) Ionisation enthalpy of hydrogen resembles that of
(a) 2[Fe(CN) 6 ]4 - + 2H + + H 2 O 2 ¾¾
® alkali metals.
(b) Its reactivity is similar to halogens.
3-
2 [ Fe ( CN )6 ] + 2H 2 O (c) It resembles both alkali metals and halogens.
-
(b) I 2 + H 2 O 2 + 2OH ¾¾ -
® 2I + 2H 2 O + O 2 (d) Loss of an electron from hydrogen atom results in a
nucleus of very small size as compared to other atoms
(c) Mn 2 + + H 2O 2 ¾¾
® Mn 4+ + 2OH - or ions. Due to small size it cannot exist free.
(d) PbS + 4H 2 O 2 ¾¾
® PbSO 4 + 4H 2 O
EBD_8350
130 CHEMISTRY

15. Metal hydrides are ionic, covalent or molecular in nature. (c) It can act only as an oxidizing agent
Among LiH, NaH, RbH, CsH, the correct order of (d) It decomposes on exposure to light
increasing ionic character is 22. Which of the following statements about hydrogen is
(a) LiH > NaH > CsH > KH >RbH incorrect ? [NEET 2016, C]
(b) LiH < NaH < KH < RbH < CsH (a) hydrogen has three isotopes of which tritium is the
(c) RbH > CsH > NaH > KH > LiH most common.
(d) NaH > CsH > RbH > LiH > KH (b) Hydrogen never acts as cation in ionic salts
16. Which of the following reactions increases production of (c) Hydronium ion, H3O+ exists freely in solution
dihydrogen from synthesis gas? (d) Dihydrogen does not act as a reducing agent
23. Which one of the following statements about water is
(a) CH 4 ( g ) + H 2 O ( g ) ¾¾¾¾ ® CO ( g ) + 3H 2 ( g )
1270 K
Ni FALSE? [JEE M 2016, C]
(a) There is extensive intramolecular hydrogen bonding
(b) C ( s ) + H 2 O ( g ) ¾¾¾¾
® CO ( g ) + H 2 ( g )
1270 K
in the condensed phase.
(b) Ice formed by heavy water sinks in normal water.
(c) CO ( g ) + H 2 O ( g ) ¾¾¾¾ ® CO 2 ( g ) + H 2 ( g )
673K (c) Water is oxidized to oxygen during photosynthesis.
Catalyst
(d) Water can act both as an acid and as a base.
1270 K 24. Hydrogen peroxide oxidises [Fe(CN)6]4– to [Fe(CN)6]3– in
(d) C 2 H 6 + 2H 2 O ¾¾¾¾
Ni
® 2CO + 5H 2
acidic medium but reduces [Fe(CN)6]3– to [Fe(CN)6]4– in
17. Which of the following compounds is used for water alkaline medium. The other products formed are
softening?
respectively: [JEE M 2018, S]
(a) Ca3(PO4)2 (b) Na3PO4 (a) (H2O + O2) and H2O
(c) Na6P6O18 (d) Na2HPO4 (b) (H2O + O2) and (H2O + OH–)
18. Only one element of ........ forms hydride. (c) H2O and (H2O + O2)
(a) group 6 (b) group 7 (d) H2O and (H2O + OH–)
(c) group 8 (d) group 9
25. The method used to remove temporary hardness of water
Past Year MCQs is: [NEET 2019, C]
19. (i) H2O2 + O3 ® H2O + 2O2 (a) Calgon’s method
(b) Clark’s method
(ii) H2O2 + Ag2O ® 2Ag + H2O + O2
(c) Ion-exchange method
Role of hydrogen peroxide in the above reactions is
(d) Synthetic resins method
respectively - [AIPMT 2014, C]
26. The isotopes of hydrogen are: [JEE Main 2019, C]
(a) Oxidizing in (i) and reducing in (ii) (a) Tritium and protium only
(b) Reducing in (i) and oxidizing in (ii) (b) Protium and deuterium only
(c) Reducing in (i) and (ii) (c) Protium, deuterium and tritium
(d) Oxidizing in (i) and (ii)
(d) Deuterium and tritium only
20. In which of the following reactions H2O2 acts as a reducing
27. The number of water molecules(s) not coordinated to copper
agent? [JEE M 2014, C]
ion directly in CuSO4.5H2O, is: [JEE Main 2019, C]
(i) H2O2 + 2H + 2e ® 2H2O
+ –
(a) 2 (b) 3
(ii) H2O2 – 2e– ® O2 + 2H+ (c) 1 (d) 4
(iii) H2O2 + 2e– ® 2OH– 28. In comparison to the zeolite process for the removal of
(iv) H2O2 + 2OH –
– 2e– ® O2 + 2H2O permanent hardness, the synthetic resins method is:
(a) (i), (iii) (b) (iii), (iv) [JEE Main 2020, C]
(b) (i), (ii) (d) (ii), (iv) (a) less efficient as it exchanges only anions
21. From the following statements regarding H2O2, choose (b) more efficient as it can exchange both cations as well
the incorrect statement : [JEE M 2015, S] as anions
(a) It has to be stored in plastic or wax lined glass bottles (c) less efficient as the resins cannot be regenerated
in dark (d) more efficient as it can exchange only cations
(b) It has to be kept away from dust
HYDROGEN 131

1. When same amount of zinc is treated separately with excess 7. Why do calcium ions make water hard but sodium ions do
of sulphuric acid and excess of sodium hydroxide solution not?
the ratio of volumes of hydrogen evolved is (a) Calcium forms insoluble compounds with stearate
(a) 1 : 1 (b) 1 : 2 ions present in soap.
(c) 2 : 1 (d) 9 : 4 (b) Sodium forms insoluble compounds with stearate ions
2. When 50% solution of H2SO4 is electrolysed by passing present in soap.
a current of high density at low temperature the main (c) Calcium forms soluble compounds with stearate ions
products of electrolysis are: present in soap.
(a) oxygen and hydrogen (d) Both calcium and sodium form insoluble compounds
(b) H2 and peroxy disulphuric acid with stearate ions present in soap.
(c) H2 and SO2 8. The hydride ion, H–, is a stronger base than the hydroxide
(d) O2 and peroxy disulphuric acid ion, OH–. Which one of the following reactions will occur
3. Hydrogen can be fused to form helium at if sodium hydride (NaH) is dissolved in water?
(a) high temperature and high pressure (a) H - (aq) + H 2 O(l) ® H 3O - (aq)
(b) high temperature and low pressure (b) H - (aq) + H 2 O(l) ® OH - (aq) + H 2 (g)
(c) low temperature and high pressure
(c) H - (aq) + H 2 O(l) ® OH - (aq) + 2 H + (aq) + 2e –
(d) low temperature and low pressure
4. When a substance A reacts with water it produces a (d) H - (aq) + H 2 O(l) ® No reaction
combustible gas B and a solution of substance C in water. 9. 34 g of H2O2 is present in 1120 mL of solution. This
When another substance D reacts with this solution of C, solution is called
it also produces the same gas B on warming but D can (a) 10 vol solution
produce gas B on reaction with dilute sulphuric acid at
(b) 20 vol solution
room temperature. A imparts a deep golden yellow colour
to a smokeless flame of Bunsen burner. A, B, C and D (c) 34 vol solution
respectively are (d) 32 vol solution
(a) Na , H2, NaOH, Zn 10. 10 mL of H2O2 solution (volume strength = x) requires
(b) K, H2, KOH, Al 10 mL of N/0.56 MnO4– solution in acidic medium. Hence
x is
(c) Ca, H2, Ca(OH)2, Sn
(a) 0.56 (b) 5.6
(d) CaC2, C2H2, Ca(OH)2, Fe
(c) 0.1 (d) 10
5. Pick out the correct statement
11. The normality and volume strength of a solution made by
(a) By decreasing the temperature pure para-hydrogen
mixing 1.0 L each of 5.6 volume and 11.2 volume H2O2
can be obtained
solution are :
(b) By increasing the temperature pure ortho-hydrogen
(a) 1 N, 5.6 vol
can be obtained
(b) 1.5 N, 5.6 vol
(c) By decreasing the temperature pure ortho-hydrogen
can be obtained (c) 1.5 N, 8.4 vol
(d) By increasing the temperature pure para-hydrogen (d) 1 N, 8.4 vol
can be obtained 12. A 5.0 mL solution of H2O2 liberates 1.27 g of iodine from
6. When a sample of hard water is passed through the layer an acidified KI solution. The percentage strength of H2O2
of sodium zeolite resulting which of the following ions is
will not be present in the resulting sample of water (a) 11.2 (b) 5.6
obtained? (c) 1.7 (d) 3.4
(a) Mg2+ and Ca2+ 13. If 100 mL of acidified 2 N H2O2 is allowed to react with
(b) Ca2+ and Na+ KMnO4 solution till there is a light tinge of purple colour.
The volume of oxygen produced at STP is
(c) Mg2+ and CO32 -
(a) 2.24 L (b) 1.12 L
(d) CO32 - and Cl– (c) 3.36 L (d) 4.48 L
EBD_8350
132 CHEMISTRY

14. 100 mL of 0.01 M KMnO4 oxidises 100 mL H2O2 in acidic 15. The purity of H2O2 in a given sample is 85%. Calculate
medium. Volume of the same KMnO4 required in alkaline the weight of impure sample of H2O2 which requires
medium to oxidise 100 mL of the same H2O2 will be 10 mL of M/5 KMnO4 solution in titration in acidic
(MnO4– changes to Mn 2+ in acidic medium and to MnO2 medium
in alkaline medium) (a) 2 g (b) 0.2 g
100 500 (c) 0.17 g (d) 0.15 g
(a) mL (b) mL
3 3

300
(c) mL (d) None
5

ANSW ER KEY
Exercise 1 : NCERT Based Topic-wis e MCQs
1 (d) 8 (d) 15 (a) 22 (b) 29 (c) 36 (c) 43 (c) 50 (d) 57 (b) 64 (d)
2 (c) 9 (c) 16 (d) 23 (d) 30 (c) 37 (b) 44 (a) 51 (d) 58 (a) 65 (a)
3 (a) 10 (b) 17 (b) 24 (c) 31 (b) 38 (d) 45 (b) 52 (a) 59 (d)
4 (b) 11 (c) 18 (c) 25 (d) 32 (b) 39 (b) 46 (b) 53 (d) 60 (a)
5 (d) 12 (d) 19 (d) 26 (c) 33 (b) 40 (c) 47 (b) 54 (d) 61 (b)
6 (a) 13 (a) 20 (c) 27 (d) 34 (a) 41 (a) 48 (b) 55 (d) 62 (b)
7 (c) 14 (a) 21 (c) 28 (b) 35 (b) 42 (a) 49 (c) 56 (d) 63 (d)
Exercis e 2 : NCERT Exemplar & Past Year MCQs
1 (b) 4 (b) 7 (b) 10 (b) 13 (a) 16 (c) 19 (c) 22 (a, d) 25 (b) 28 (b)
2 (d) 5 (b) 8 (c) 11 (d) 14 (d) 17 (c) 20 (d) 23 (a) 26 (c)
3 (c) 6 (c) 9 (d) 12 (a) 15 (b) 18 (a) 21 (c) 24 (c) 27 (c)
Exercise 3 : Problem Solving S kill Enhancer MCQs
1 (a) 3 (a) 5 (a) 7 (a) 9 (a) 11 (c) 13 (a) 15 (b)
2 (b) 4 (a) 6 (a) 8 (b) 10 (d) 12 (d) 14 (b)
10 The s-Block Elements

Trend Buster NEET & JEE Main

Number of Questions from 2020-15 11 4 Minimum two questions have been


asked every year in NEET
Weightage 3.4% 2.20%

The most Important Concepts that Cover Maximum number of Questions asked in past 6 years.

General characteristics of compounds of alkaline 5 2


earth metal
Alkaline earth metals 1 1
Biological importance of s -block elements 4 —

Less Important Concepts that Cover 1 or 2 Questions asked in past 6 years.

Group 1 elments: alkali metals 1 1

NEET JEE
Concept Used

2020 Characteristics of Characteristics of 2 Easy / — —


compounds of Alkalimetals / compounds of s-block Average
Biological importance of elements / Biological
s-block elements importance of potassium
2019 Group 2 elements: alkaline Chemical properties of alkaline
earth metals/general earth metals/physical properties
characteristics of compounds of alkaline earth metal 2 Average 2 Average
of alkaline earth metals/ compounds/biological
biological importance of importance of alkaline earth
s-block elements metals
2018 Group 2 elements: alkaline Compound formation by alkaline
earth metals/general earth metals/ionic character of
characteristics of compounds alkaline earth metal hydrides/ 3 Average — —
of alkaline earth metals acidic character of alkaline
earth metal oxides
2017 — — — — — —
2016 Group 1 elements: alkali Chemical properties of alkali
metals/biological importance metals/biological importance 1 Easy 1 Average
of s-block elements of alkaline earth metals
2015 General characteristics of physical and chemical
compounds of alkaline earth properties of alkaline earth 3 Average 1 Average
metals/Biological importance metals/biological importance
of s-block elements of alkali metals
EBD_8350
134 CHEMISTRY
THE s-BLOCK ELEMENTS 135
EBD_8350
136 CHEMISTRY

Problem Solving Tips/ Tricks/ Points to Remember

4 Atomic radii : Li < Na < K < Rb < Cs 4 KO2 and Na2O2 are used as a source of oxygen (or for the
4 Ionic radii : Li+ < Na+ < K+ < Rb+ < Cs+ purification of air) in confined space such as submarines,
4 Electronegativity : Li > Na > K > Rb > Cs space shuttles and in emergency breathing instrument
4 First ionization potential : Li > Na > K > Rb > Cs such as oxygen masks.
4 Melting point Li > Na > K > Rb > Cs 4 Lithium is kept wrapped in paraffin wax. It can’t be stored
4 Colour of the flame : Li - Red, Na - Golden, K - Violet, in kerosene as it floats to the surface due to its low density.
Rb - Red, Cs - Blue, Ca - Brick red, Sr - Blood red, 4 Fr and Ra are radioactive elements.
Ba-Apple green 4 Due to their low ionisation, alkali metals, (except Li) show
4 Rb and Cs show photoelectric effect. photoelectric effect. Cs can emit photoelectrons even with
4 Stability of hydrides : LiH > NaH > KH > RbH > CsH red light.
4 The finely divided BaSO4 is called blanc fire and used in 4 Lithium salts are mostly hydrated e.g. LiCl·2H2O.
paints. D D
4 Pure Ca(H2PO4)2 is used as american baking powder. 4 Na2CO3 · 10H2O ¾¾¾¾
-9H O
® Na CO · H O ¾¾¾¾
2 3 2 -H O
®
2 2
4 Melting point: washing soda monohydrate
For the same alkali metal, the melting points decrease in Na2CO3 (anhydrous)
the order: soda ash
Fluoride > chloride > bromide > iodide 4 Fire extinguishers contain conc. H2SO4 and NaHCO3.
This is due to the decrease in lattice enthalpies as the 4 Lithium salts are used in the treatment of depressive
size of the halide ion increases (size of cation is constant). disorder.
4 The stability of peroxides and superoxides increases in 4 A suspension of magnesium hydroxide in water called
the order
milk of magnesia is used as antacid in medicine.
O2 < K2O2 < Rb2O2 < Cs2O2 NaO2 < KO2 < RbO2 < CsO2
4 Beryllium shows anomalous properties due to its small
4 Basic strength of oxides and hydroxides
size and high polarising power and resembles aluminium
BeO < MgO < CaO < SrO < BaO
(diagonal relationship) because of their similar sizes.
Be(OH)2 < Mg(OH)2 < Ca(OH)2 < Sr(OH)2 < Ba(OH)2
4 Most of the kidney stones consist of calcium oxalate
4 Solubility of hydroxides
CaC2O4 which dissolves in dilute strong acids but remains
Be(OH ) 2 < Mg (OH ) 2 < Ca (OH ) 2 < Sr (OH ) 2 < Ba (OH ) 2
insoluble in bases.
4 Solubility of sulphates 4 All enzymes that utilise ATP in phosphate transfer require
BeSO 4 > MgSO 4 > CaSO 4 > SrSO 4 > BaSO 4 magnesium as the cofactor.
4 Solubility of carbonates 4 BaSO4 is used for detecting obstruction in the digestive
BeCO3 > MgCO3 > CaCO3 > SrCO3 > BaCO3 system as ‘barium meal’. It helps in getting x-ray pictures
4 CuBr is more covalent than NaBr, although Cu + and Na+ because of greater scattering power of it.
have the same charge +1, and nearly the same size, i.e., 4 A mixture of calcium carbide and calcium phosphide used as
Cu+ (0.96D) and Na+ (1.02D). This is due to the fact that ‘holmes signal’. The mixture reacts with water to produce
Cu+ ion has a pseudo noble gas configuration, i.e., having gases PH3 and C2H2 which burn and thus guide the ships in
18e–’s in the outermost shell. sea.
4 Peroxide ( O22- ) of the first group are colourless and 4 Na2CO3 + K2CO3 is called fusion mixture which is used for
diamagnetic while superoxide ( O- 2 ) are coloured and organic qualitative analysis.
paramagnetic. 4 LiF is most insoluble alkali metal halide due to its high
4 Except Cs2O, all other monoxide, e.g., Li2O, Na2O, K2O lattice enthalpy whereas, BeF2 is most soluble alkaline
and Rb2O have antiflourite structures, Cs2O has anti earth metal halide due to its high hydration enthalpy.
CdCl2 layer structure.
THE s-BLOCK ELEMENTS 137

Exercise 1 : NCERT Based Topic-wise MCQs


Topic 1: Group 1 Elements: Alkali Metals (iii) Among the alkali metals only lithium forms a stable
nitride by direct combination with nitrogen
1. Which of the following alkali metal is highly radioactive? (a) (i), (ii) and (iii) (b) (i) and (ii)
(a) Rubidium (b) Caesium (c) (i) and (iii) (d) (ii) and (iii)
(c) Francium (d) Both (a) and (c) 15. Suppose an element is kept in air chamber, than air content
2. Which of the following statements is not correct for was evaluated after sometime , oxygen and nitrogen content
alkali metals? was found to be low comparitively. The given element will be
(a) Alkali metals are the most electropositive metals. (a) Li (b) Rb (c) Na (d) K
(b) Alkali metals exist in free state in nature.
(c) These metals have the largest size in a particular Topic 2: General Characteristics of the Compounds of
period of the periodic table. the Alkali Metals
(d) Both (b) and (c) 16. Which is most basic in character ?
3. Ionization potential of Na would be numerically the same as (a) CsOH (b) KOH (c) NaOH (d) LiOH
(a) electron affinity of Na+ 17. Which compound will show the highest lattice energy ?
(b) electronegativity of Na+ (a) RbF (b) CsF (c) NaF (d) KF
(c) electron affinity of He 18. In crystals which one of the following ionic compounds
(d) ionization potential of Mg would you expect maximum distance between centres of
4. The elements of group 1 provide a colour to the flame of cations and anions?
Bunsen burner due to (a) LiF (b) CsF (c) CsI (d) LiI
(a) low ionization potential 19. Among LiI, NaI, KI, the one which is more ionic and more
(b) low melting point soluble in water is
(c) softness (a) KI (b) NaI (c) LiI (d) None
(d) presence of one electron in the outermost orbit 20. Which of the following statement(s) is/are correct
5. The element which on burning in air gives peroxide is regarding Li2CO3 and Na2CO3 ?
(a) lithium (b) sodium (c) rubidium (d) caesium (i) Sodium salt evolve CO2 at higher temperature.
6. Which one of the alkali metals, forms only, the normal oxide, (ii) Polarization of Na+ is lesser than that of Li+.
M2O on heating in air ? (a) Both statements (i) and (ii) are correct
(a) Rb (b) K (c) Li (d) Na (b) Both statements (i) and (ii) are incorrect
7. Which of the following has density greater than water? (c) Statement (ii) is correct explanation for statement (i)
(a) Li (b) Na (c) K (d) Rb (d) Statement (i) is correct explanation for statement (ii)
8. The ionic mobility of alkali metal ions in aqueous solution
is maximum for 21. Which of the following is used as a source of oxygen in
(a) Li+ (b) Na+ (c) K+ (d) Rb+ space capsules, submarines and breathing masks ?
9. Which hydride is most stable (a) Li 2O (b) Na 2O 2 (c) KO2 (d) K 2O2
(a) NaH (b) KH (c) CsH (d) LiH 22. Which of the following salt of lithium is most soluble in
10. The most stable compound is organic solvent ?
(a) LiF (b) LiCl (c) LiBr (d) LiI (a) LiF (b) LiCl (c) LiBr (d) LiI
11. Which of the following represents a correct sequence of 23. Which of the following statements is incorrect?
reducing power of the following elements? (a) Alkali metal hydroxide are hygroscopic
(a) Li > Cs > Rb (b) Rb > Cs > Li (b) Dissolution of alkali metal hydroxide is endothermic
(c) Cs > Li > Rb (d) Li > Rb > Cs (c) Aqueous solution of alkali metal hydroxides are
12. Which of the following metal is used along with lithium strongly basic
to make the alloy named ‘white metal’ ? (d) Alkali metal hydroxides form ionic crystals
(a) Nickel (b) Aluminium 24. Suppose metal react with the oxygen to form oxide, than
(c) Silver (d) Lead aqueous solution of this oxide when added to a solution of
13. Which of the following metal is used as a coolant in HI, solution turn yellowish brown in colour. This compound
breeder nuclear reactors? is
(a) Potassium (b) Sodium (a) Na2O (b) Li2O (c) NaOH (d) Na2O2
(c) Caesium (d) Rubidium
14. Select the correct statements Topic 3: Anomalous Properties of Lithium
(i) Cs+ is more highly hydrated that the other alkali metal 25. The products obtained on heating LiNO2 will be
ions (a) Li 2 O + NO2 + O2 (b) Li 3 N + O 2
(ii) Among the alkali metals Li, Na, K and Rb, lithium has
the highest melting point (c) Li 2 O + NO + O 2 (d) LiNO3 + O2
EBD_8350
138 CHEMISTRY

26. Assertion : Lithium carbonate is not so stable to heat. Topic 4: Some Important Compounds of Sodium
Reason : Lithium being very small in size polarizes large
CO32 - ion leading to the formation of more stable Li2O 34. In Solvay ammonia process, sodium bicarbonate is
precipitated due to
and CO2
(a) presence of NH3
(a) Assertion is correct, reason is correct; reason is a
(b) reaction with CO2
correct explanation for assertion.
(c) reaction with brine solution
(b) Assertion is correct, reason is correct; reason is not
(d) reaction with NaOH
a correct explanation for assertion
35. How NH3 is recovered in Solvay process?
(c) Assertion is correct, reason is incorrect
(a) By reaction of NH4Cl and Ca(OH)2
(d) Assertion is incorrect, reason is correct.
(b) By reaction of NH4HCO3 and NaCl
27. Complete the following two reactions.
(c) By reaction of (NH4)2CO3 with H2O
(i) 4LiNO3 ® x + O2
(d) By any of the above
(ii) 2NaNO3 ® y + O2
36. Which is manufactured by electrolysis of fused NaCl?
(a) x = LiNO2, y = NaNO2
(a) NaOH (b) Na (c) NaClO (d) NaClO3
(b) x = Li2O + NO2, y = Na2O + NO2
37. Baking powder contains :
(c) x = Li2O + NO2, y = NaNO2
(a) NaHCO3, Ca(H2PO2)2 and starch
(d) x = LiNO2, y = Na2O + NO2
(b) NaHCO3, Ca(H2PO2)2
28. Lithium is strongest reducing agent among alkali metals
(c) NaHCO3, starch
due to which of the following factor?
(d) NaHCO3
(a) Ionization energy (b) Electron affinity
38. Sodium carbonate is manufactured by Solvay process. The
(c) Hydration energy (d) Lattice energy
products which can be recycled are
29. The melting point of lithium (181°C) is just double the
(a) CO2 and NH3 (b) CO2 and NH4Cl
melting point of sodium (98°C) because
(c) NaCl and CaO (d) CaCl2 and CaO.
(a) down the group, the hydration energy decreases
39. Why Solvay process cannot be extended to the
(b) down the group, the ionization energy decreases
manufacture of potassium carbonate?
(c) down the group the cohesive energy decreases
(a) Ammonium hydrogen carbonate does not react
(d) None of these
with potassium chloride.
30. Assertion: Lithium salts are mostly hydrated.
(b) Potassium hydrogen carbonate is too soluble to be
Reason : The hydration enthalpy of alkali metal ions
precipitated by the addition of ammonium
decreases with increase in ionic sizes.
hydrogencarbonate to a saturated solution of
(a) Assertion is correct, reason is correct; reason is a
potassium chloride.
correct explanation for assertion.
(c) Ammonium carbonate is precipitated out instead of
(b) Assertion is correct, reason is correct; reason is not
potassium hydrogen carbonate on reaction of
a correct explanation for assertion
ammonium hydrogen carbonate with potassium
(c) Assertion is correct, reason is incorrect
chloride
(d) Assertion is incorrect, reason is correct.
(d) None of the above
31. Which of the following does not illustrate the anomalous
40. Which of the following is/are present as impurity in
properties of lithium?
crude sodium chloride, obtained by crystallisation of brine
(a) The melting point and boiling point of Li are
solution?
comparatively high
(i) Sodium sulphate (ii) Calcium chloride
(b) Li is much softer than the other group I metals
(iii) Magnesium chloride (iv) Potassium chloride
(c) Li forms a nitride Li3N unlike group I metals
(a) (i), (ii) and (iv) (b) (ii) and (iii)
(d) The ion of Li and its compounds are more heavily
(c) (iii) and (iv) (d) (i), (ii) and (iii)
hydrated than those of the rest of the group
41. The raw materials in Solvay process are
32. Why lithium react less vigorously with water than other
(a) Na2CO3, CaCO3 and NH3
alkali metals?
(b) Na2SO4, CaCO3 and NH3
(a) Lithium has most negative E– value
(c) NaCl, NH3 and CaCO3
(b) Lithium has small size and very high hydration
(d) NaOH, CaO and NH3.
energy.
42. Match the columns.
(c) Lithium has least negative E– value
Column-I Column-II
(d) Both (a) and (b)
(Sodium compound) (Uses)
33. Li has the maximum value of ionisation potential among
(A) Sodium carbonate (p) In fire extinguisher
alkali metals i.e. lithium has the minimum tendency to ionise
(B) Sodium chloride (q) In manufacture of
to give LI+ ion. Thus, in aq. solution lithium is
glass, soap, borax
(a) strongest reducing agent
and caustic soda.
(b) poorest reducing agent
(C) Sodium hydroxide (r) In preparation of Na2O2,
(c) strongest oxidising agent
NaOH and Na2CO3
(d) poorest oxidising agent
THE s-BLOCK ELEMENTS 139

(D) Sodium hydrogen (s) In petroleum refining (iii) Compounds of alkaline earth metals are more
carbonate extensively hydrated than those of alkali metals
(a) A – (q), B – (r), C – (s), D – (p) (a) (i) and (ii) (b) (ii) and (iii)
(b) A – (s), B – (q), C – (r), D – (p) (c) (i) and (iii) (d) (i), (ii) and (iii)
(c) A – (p), B – (s), C – (r), D – (q) 53. The metals A and B form oxide but B also forms nitride
(d) A – (s), B – (r), C – (p), D – (q) when both burn in air. The A and B are
(a) Cs, K (b) Mg, Ca (c) Li, Na (d) K, Mg
Topic 5: Group 2 Elements: Alkaline Earth Metals
43. Which of the following atoms will have the smallest size ? Topic 6: General Characteristics of Compounds of
(a) Mg (b) Na (c) Be (d) Li the Alkaline Earth Metals
44. The first ionization energy of magnesium is lower than the
first ionization energy of 54. Which of the following relations is correct with respect to
(a) Lithium (b) Sodium first (I) and second (II) ionization potentials of sodium and
(c) Calcium (d) Beryllium magnesium?
45. The most electropositive amongst the alkaline earth metals is (a) I Mg = II Na (b) IMg < IINa
(a) beryllium (b) magnesium
(c) calcium (d) barium (c) I Na > I Mg (d) II Na > IIMg
46. Alkaline earth metals are not found free in nature because of 55. The first ionization energies of alkaline earth metals are
(a) their thermal instability higher than those of alkali metals. This is because
(b) their low melting points (a) there is no change in the nuclear charge
(c) their high boiling points (b) there is decrease in the nuclear charge of alkaline earth
(d) their greater chemical reactivity metals
47. Out of the following elements which one do you expect to (c) there is increase in the nuclear charge of alkaline earth
be most reactive, chemically ? metals
(a) Mg (b) Ca (c) Sr (d) Ba (d) none of these
48. Compared with the alkaline earth metals, the alkali metals 56. Which of the following alkaline earth metal hydroxides is
exhibit amphoteric in character
(a) smaller ionic radii (a) Be(OH)2 (b) Ca(OH)2 (c) Sr(OH)2 (d) Ba(OH)2
(b) highest boiling points 57. Of the metals Be, Mg, Ca and Sr of group 2. In the periodic
(c) greater hardness table the least ionic chloride would be formed by
(d) lower ionization energies (a) Be (b) Mg (c) Ca (d) Sr
49. Match the columns 58. The solubilities of carbonates decrease down the
Column-I Column-II magnesium group due to a decrease in
(Alkali metal) (Colour imparted to an (a) hydration energies of cations
oxidizing flame) (b) inter-ionic attraction
(A) Cs (p) Yellow (c) entropy of solution formation
(B) Rb (q) Blue (d) lattice energies of solids
(C) K (r) Violet 59. Which of the following on thermal decomposition yields a
(D) Na (s) Red violet basic as well as acidic oxide ?
(E) Li (t) Crimson red (a) NaNO3 (b) KClO3 (c) CaCO3 (d) NH4NO3
(a) A – (q), B – (s), C – (r), D – (p), E – (t) 60. Which one of the following on hydrolysis, gives the
(b) A – (s), B – (q), C – (r), D – (p), E – (t) corresponding metallic hydroxide, H2O2 and O2?
(c) A – (t), B – (r), C – (s), D – (p), E – (q) (a) Li2O (b) Na2O2 (c) NaO2 (d) Na2O
(d) A – (q), B – (p), C – (r), D – (p), E – (t) 61. Which one of the following is the most soluble in water?
50. Which one of the following does not react with water even (a) Mg(OH)2 (b) Sr(OH)2
under red hot condition? (c) Ca(OH)2 (d) Ba(OH)2
(a) Na (b) Be (c) Ca (d) K 62. The correct order of increasing thermal stability of K2CO3,
51. Aqueous solution of group 2 is precipitated by adding MgCO3, CaCO3 and BeCO3 is
Na2CO3, then this precipitate is tested on flame, no light (a) BeCO3< MgCO3 < CaCO3 < K2CO3
in visible region is observed , this element can be (b) MgCO3 < BeCO3 < CaCO3 < K2CO3
(a) Ba (b) Mg (c) Ca (d) Sr (c) K2CO3 < MgCO3 < CaCO3 < BeCO3
52. Which of the following statement(s) is/are correct? (d) BeCO3 < MgCO3 < K2CO3 < CaCO3
(i) The atomic and ionic radii of alkaline earth metals 63. Property of the alkaline earth metals that increases with
are smaller than those of the corresponding alkali their atomic number is
metals in the same periods. (a) solubility of their hydroxides in water
(ii) Second ionisation enthalpies of the alkaline earth (b) solubility of their sulphates in water
metals are smaller than those of the corresponding (c) ionization energy
alkali metals. (d) electronegativity
EBD_8350
140 CHEMISTRY

64. Which of the following will precipitate first when aqueous 72. Beryllium shows diagonal relationship with aluminium.
solution containing sulphate ions are added? Which of the following similarity is incorrect ?
(a) Mg2+ (b) Ca2+ (c) Sr2+ (d) Ba2+ (a) Be forms beryllates and Al forms aluminates
65. Which of the following is/are not characteristic (b) Be(OH)2, like Al(OH)3 is basic.
property(ies) of alkaline earth metals ? (c) Be like Al is rendered passive by HNO3.
(i) All alkaline earth metal oxides are basic in nature (d) Be2C like Al4C3 yields methane on hydrolysis.
and forms sparingly soluble hydroxides with water.
(ii) The hydrated chlorides, bromides and iodies of Ca, Topic 8: Some Important Compounds of Calcium
Sr and Ba on heating undergoes hydrolysis while 73. The substance not likely to contain CaCO3 is
corresponding hydrated halides of Be and Mg on (a) gypsum (b) sea shells
heating undergo dehydration. (c) dolomite (d) a marble statue
(iii) Nitrates of alkaline earth metals decompose on 74. Plaster of Paris on making paste with little water sets to
heating as below hard mass due to formation of
2M(NO3)2 ® 2MO + 4NO2 + O2 (a) CaSO4 (b) CaSO4.1/2H2O
(a) (i) only (b) (ii) only (c) CaSO4.H2O (d) CaSO4.2H2O
(c) (i) and (iii) (d) (i) and (ii) 75. Which gas is released when CaCO3 reacts with dilute
66. Magnesium burns in CO2 to form HCl?
(a) MgO + C (b) MgO + CO (a) H2 (b) CO2 (c) O2 (d) Cl2
(c) MgCO3 (d) MgO.
76. Setting of cement is an
67. If the fluoride salts of group 2 metals are dissolved in
(a) exothermic reaction
water, then which of the following will show high
(b) endothermic reaction
solubility?
(a) BaF2 (b) RbF2 (c) CaF2 (d) BeF2 (c) neither endothermic nor exothermic
68. Which of the following statement is false ? (d) example of neutralisation reaction
(a) Strontium decomposes water readily than beryllium 77. Match the columns
(b) Barium carbonate melts at a higher temperature than Column-I Column-II
calcium carbonate (A) Quick lime (p) Ca(OH)2
(c) Barium hydroxide is more soluble in water than (B) Slaked lime (q) CaO
magnesium hydroxide (C) Bleaching powder (r) Ca(OCl)2
(d) Beryllium hydroxide is more basic than barium (D) Plaster of Paris (s) 2CaSO4. H2O
hydroxide. (a) A – (p), B – (q), C – (r), D – (s)
69. Following are colours shown by some alkaline earth (b) A – (s), B – (r), C – (q), D – (p)
metals in flame test. Which of the following are not (c) A – (q), B – (p), C – (r), D – (s)
correctly matched? (d) A – (q), B – (p), C – (s), D – (r)
Metal Colour 78. Bleaching powder is obtained by the interaction of chlorine
(i) Calcium Apple green with
(ii) Strontium Crimson (a) dil. solution of Ca(OH)2
(iii) Barium Brick red (b) dry CaO
(a) (i) and (iii) (b) (i) only (c) conc. solution of Ca(OH)2
(c) (ii) only (d) (ii) and (iii) (d) dry slaked lime
79. The formula for calcium chlorite is
Topic 7: Anomalous Behaviour of Beryllium
(a) Ca(ClO4)2 (b) Ca(ClO3)2
70. Which of the following statement(s) is/are correct (c) CaClO2 (d) Ca(ClO2)2
regarding Al and Be ? 80. Bone ash contains
(i) Both of these react with alkali. (a) CaO (b) CaSO4
(ii) There is diagonal relationship among these elements. (c) Ca3 (PO4)2 (d) Ca(H2PO4)2
(a) Both (i) and (ii) (b) Only (i) 81. Mortar is a mixture of
(c) Only (ii) (d) Neither (i) nor (ii) (a) CaCO3, sand and water
71. Assertion : Compounds of beryllium is largely covalent (b) slaked lime and water
and get hydrolysed easily. (c) slaked lime, sand and water
Reason : This is due to high value of ionisation potential (d) CaCO3 and CaO
and small size of Be. 82. Which of the following is not present in Portland cement?
(a) Assertion is correct, reason is correct; reason is a (a) Ca2SiO4 (b) Ca3SiO5
correct explanation for assertion. (c) Ca3(PO4)2 (d) Ca3Al2O6
(b) Assertion is correct, reason is correct; reason is not a 83. Match the columns
correct explanation for assertion. (A) Quick lime (p) Setting fractured bones
(c) Assertion is correct, reason is incorrect. (B) Plaster of Paris (q) A constituent of chewing
(d) Assertion is incorrect, reason is correct. gum
THE s-BLOCK ELEMENTS 141

(C) Slaked lime (r) Manufacture of bleaching Topic 9: Biological Importance of s-Block Elements
powder
(D) Limestone (s) Manufacture of dyestuffs 86. Which of the following are found in biological fluids
(a) A – (p), B – (s), C – (q), D – (r) Na+, Mg2+, Ca2+, K+, Sr2+, Li+ and Ba2+
(b) A – (s), B – (p), C – (r), D – (q) (a) Mg2+, Ca2+, and Sr2+
(c) A – (q), B – (r), C – (p), D – (s) (b) Na2 +and K+
(d) A – (r), B – (q), C – (s), D – (p) (c) Na+, K+, Mg2+and Ca2+
84. For a good quality cement what should be the ratio of (d) Sr+, Li and Ba2+
following : 87. Which of the following metal is found in green colouring
I. Silica to alumina
pigment chlorophyll of plants?
II. CaO to the total of oxides of SiO2, Al2O3 and Fe2O3
(a) I = 2.5 to 4 (b) I = Nearly 4 (a) Fe (b) Mg (c) Na (d) Al
II = Greater than 2 II = Less than 2 88. The ions which participate in the transmission of nerve
(c) I = 2.5 (d) I = 2.5 to 4 signals
II = Closer to 2 II = Closer to 2 (a) Na+ (b) Be2+ (c) Cs+ (d) Ba2+
85. Which of the following statements are correct ? 89. Which of the following is the most abundant ion within
(i) Copper - beryllium alloys are used in the preparation cell fluids?
of high strength springs (a) Sodium ions (b) Potassium ions
(ii) Metallic beryllium is used for making window of X- (c) Calcium ions (d) None of these
ray tubes.
90. Calcitonin and parathyroid hormone regulate
(iii) Magnesium powder is used in incendiary bombs
and signals. concentration of which of the following element in
(iv) Barium is used in treatment of cancer. plasma?
(a) (i), (ii) and (iv) (b) (i) and (iii) (a) Calcium (b) Magnesium
(c) (i), (ii) and (iii) (d) (i), (ii), (iii) and (iv) (c) Sodium (d) Potassium

NCERT Exemplar MCQs (a) Be(OH)2 (b) Mg(OH)2


1. The alkali metals are low melting. Which of the following (c) Ca(OH)2 (d) Ba(OH)2
alkali metal is expected to melt if the room temperature 7. When sodium is dissolved in liquid ammonia, a solution
rises to 30°C? of deep blue colour is obtained. The colour of the solution
(a) Na (b) K (c) Rb (d) Cs is due to
2. Alkali metals react with water vigorously to form (a) ammoniated electron (b) sodium ion
hydroxides and dihydrogen. Which of the following alkali (c) sodium amide (d) ammoniated sodium ion
metals reacts with water least vigorously? 8. Dead burnt plaster is
(a) Li (b) Na (c) K (d) Cs 1
3. Metals form basic hydroxides. Which of the following (a) CaSO4 (b) CaSO4 . H 2 O
2
metal hydroxide is the least basic?
(c) CaSO4.H2O (d) CaSO4.2H2O
(a) Mg(OH)2 (b) Ca(OH)2
(c) Sr(OH)2 (d) Ba(OH)2 9. Suspension of slaked lime in water is known as
(a) lime water
4. Some of the Group 2 metal halides are covalent and soluble
(b) quick lime
in organic solvents. Among the following metal halides,
(c) milk of lime
the one which is soluble in ethanol is
(d) aqueous solution of slaked lime
(a) BeCl2 (b) MgCl2
10. The formula of soda ash is
(c) CaCl2 d) SrCl2
(a) Na2CO3.10H2O (b) Na2CO3.2H2O
5. The order of decreasing ionisation enthalpy in alkali
metals is (c) Na2CO3.H2O (d) Na2CO3
(a) Na > Li > K > Rb (b) Rb < Na < K < Li 11. Which of the following statements is true about Ca(OH)2?
(c) Li > Na > K > Rb (d) K < Li < Na < Rb (a) It is used in the preparation of bleaching powder.
6. Amphoteric hydroxides react with both alkalies and acids. (b) It is a light blue solid.
Which of the following Group 2 metal hydroxides is soluble (c) It does not possess disinfectant property.
in sodium hydroxide? (d) It is used in the manufacture of cement.
EBD_8350
142 CHEMISTRY

12. A chemical ‘A’ is used for the preparation of washing Past Year MCQs
soda to recover ammonia.When CO2 is bubbled through
22. The function of "Sodium pump" is a biological process
an aqueous solution of ‘A’, the solution turns milky. It is
operating in each and every cell of all animals. Which of
used in white washing due to disinfectant nature. What is the following biologically important ions is also a
the chemical formula of ‘A’? consituent of this pump : [AIPMT 2015,S]
(a) Ca(HCO3)2 (b) CaO (a) Mg2+ (b) K+ (c) Fe2+ (d) Ca2+
(c) Ca(OH)2 (d) CaCO3 23. Solubility of the alkaline earth's metal sulphates in water
13. The reducing power of a metal depends on various factors.
decreases in the sequence :- [AIPMT 2015,C]
Suggest the factor which makes Li, the strongest reducing
(a) Ca > Sr > Ba > Mg (b) Sr > Ca > Mg > Ba
agent in aqueous solution.
(c) Ba > Mg > Sr > Ca (d) Mg > Ca > Sr > Ba
(a) Sublimation enthalpy (b) lonisation enthalpy
(c) Hydration enthalpy (d) Electron–gain enthalpy 24. On heating which of the following releases CO2 most
14. Metal carbonates decompose on heating to give metal easily? [AIPMT 2015 RS,C]
oxide and carbon dioxide. Which of the metal carbonates (a) K2CO3 (b) Na2CO3 (c) MgCO3 (d) CaCO3
is most stable thermally? 25. Which one of the following alkaline earth metal sulphates
(a) MgCO3 (b) CaCO3 (c) SrCO3 (d) BaCO3 has its hydration enthalpy greater than its lattice enthalpy?
15. Which of the following carbonates given below is unstable [JEE M 2015, 2014,C]
in air and is kept in CO 2 atmosphere to avoid (a) BaSO4 (b) SrSO4 (c) CaSO4 (d) BeSO4
decomposition? 26. Which of the following statements is false ?
(a) BeCO3 (b) MgCO3 (c) CaCO3 (d) BaCO3 [NEET 2016,C]
16. The solubility of metal halides depends on their nature, (a) Mg2+ ions form a complex with ATP
lattice enthalpy and hydration enthalpy of the individual (b) Ca2+ ions are important in blood clotting
ions. Amongst fluorides of alkali metals, the lowest (c) Ca2+ ions are not important in maintaining the regular
solubility of Li in water is due to beating of the heart.
(a) ionic nature of lithium fluoride (d) Mg2+ ions are important in the green parts of plants.
(b) high lattice enthalpy 27. The main oxides formed on combustion of Li, Na and K in
(c) high hydration enthalpy for lithium ion excess of air are, respectively: [JEE M 2016, S]
(d) low ionisation enthapy of lithium atom (a) Li2O2, Na2O2 and KO2 (b) Li2O, Na2O2 and KO2
17. In the synthesis of sodium carbonate, the recovery of (c) Li2O, Na2O and KO2 (d) LiO2, Na2O2 and K2O
ammonia is done by treating NH4Cl with Ca(OH)2. The 28. Which of the following oxides is most acidic in nature?
by – product obtained in this process is
[NEET 2018, C]
(a) CaCl2 (b) NaCl (c) NaOH (d) NaHCO3
(a) MgO (b) BeO (c) CaO (d) BaO
18. By adding gypsum to cement
29. Magnesium reacts with an element (X) to form an ionic
(a) setting time of cement becomes less
(b) setting time of cement increases compound. If the ground state electronic configuration
(c) colour of cement becomes light of (X) is 1s2 2s2 2p3, the simplest formula for this compound
(d) shining surface is obtained is [NEET 2018, C]
19. Which of the following elements does not form hydride (a) Mg2X3 (b) MgX2 (c) Mg3X2 (d) Mg2X
by direct heating with dihydrogen? 30. Among CaH2, BeH2, BaH2, the order of ionic character is
(a) Be (b) Mg (c) Sr (d) Ba [NEET 2018, C]
20. A substance which gives brick red flame and breaks down (a) BeH2 < CaH2 < BaH2 (b) CaH2 < BeH2 < BaH2
on heating to give oxygen and a brown gas is (c) BaH2 < BeH2 < CaH2 (d) BeH2 < BaH2 < CaH2
(a) magnesium nitrate (b) calcium nitrate 31. Which of the following is an amphoteric hydroxide ?
(c) barium nitrate (d) strontium nitrate [NEET 2019, C]
21. Dehydration of hydrates of halides of calcium, barium and (a) Sr(OH)2 (b) Ca(OH)2
strontium i.e., CaCl2. 6H2O, BaCl2.2H2O, SrCl2.2H2O can (c) Mg(OH)2 (d) Be(OH)2
be achieved by heating. These become wet on keeping in
32. Enzymes that utilize ATP in phosphate transfer require an
air. Which of the following statements is correct about
alkaline earth metal (M) as the cofactor. M is:
these halides?
[NEET 2019, C]
(a) Act as dehydrating agent
(b) Can absorb moisture from air (a) Be (b) Mg (c) Ca (d) Sr
(c) Tendency to form hydrate decreases form calcium to 33. Magnesium powder burns in air to give: [JEE M 2019, C]
barium (a) Mg(NO3) 2 and Mg3N2 (b) MgO and Mg3N2
(d) All of the above (c) MgO only (d) MgO and Mg(NO3)2
THE s-BLOCK ELEMENTS 143

34. The alkaline earth metal nitrate that does not crystallise (c) NaCl, MgCl2 and CaCl2
with water molecules, is: [JEE M 2019, C] (d) Both MgCl2 and CaCl2
(a) Mg (NO3)2 (b) Sr (NO3)2 36. The following metal ion activates many enzymes,
(c) Ca (NO3)2 (d) Ba (NO3)2 participates in the oxidation of glucose to produce ATP
35. HCl was passed through a solution of CaCl2, MgCl2 and and with Na, is responsible for the transmission of nerve
NaCl. Which of the following compound(s) crystallise(s)? signals. [NEET 2020, C]
[NEET 2020, S] (a) Copper (b) Calcium
(a) Only NaCl (c) Potassium (d) Iron
(b) Only MgCl2

1. The alkali metals form salt-like hydrides by the direct SO Na CO S


Na 2CO3 ¾¾¾
2 ® (A) ¾¾¾¾®
2 3 (B) ¾¾¾
®
synthesis at elevated temperature. The thermal stability Heat
Solution
of these hydrides decreases in which of the following AgNO3
(C) ¾¾¾¾
® (D)
order? (a) Na2SO3, NaHSO3, Na2S, Ag2S
(a) NaH > LiH > KH > RbH > CsH
(b) NaHSO3, Na2SO3, Na2S2O3, Ag2S2O3
(b) LiH > NaH > KH > RbH > CsH
(c) CsH > RbH > KH > NaH > LiH (c) NaHSO3, Na2SO4, Na2S, Ag2O
(d) KH > NaH > LiH > CsH > RbH (d) Na2SO3, Na2SO4, Na2S2O3, Ag
2. A solid compound ‘X’ on heating gives CO2 gas and a 7. Heat Heat
ZnCl 2 + NaHCO3 ¾¾¾
® (A) ¾¾¾
® (B) + (C)g +
residue. The residue mixed with water forms ‘Y’. On passing
an excess of CO2 through ‘Y’ in water, a clear solution ‘Z’, H 2O ® (B) + NaOH ® (D)
is obtained. On boiling ‘Z’, compound ‘X’ is reformed. The Identify the compound (D) present in the solution.
compound ‘X’ is (a) ZnCO3 (b) Zn(OH)2
(a) Ca(HCO3)2 (b) CaCO3 (c) ZnO (d) Na2ZnO2
(c) Na2CO3 (d) K2CO3 8. To an acidified dicrhomate solution, a pinch of Na2O2 is
3. In curing cement plasters, water is sprinkled from time to
added and shaken. What is observed?
time. This helps in
(a) Blue colour on surface
(a) developing interlocking needle-like crystals of
(b) Green colour in the lower layer
hydrated silicates
(c) On adding Na2O2 to an acidified dichromate solution
(b) hydrating sand and gravel mixed with cement
a blue colour is appears on surface. An unstable
(c) converting sand into silicic acid
peroxide is formed which decompose after some time
(d) keeping it cool
with evolution of oxygen and a green aqueous
4. Incorrect statement is:
solution of chromic sulphate is formed in lower layer.
(a) NaHCO3 and KHCO3 have same crystal structure
(d) All of these
(b) On heating LiNO3 decomposes into Li2O and NO2
(c) Among alkali metals, Li metal impart red colour to 9. Na2SO4 is soluble in water whereas BaSO4 is sparingly
flame soluble because
(d) Li2SO4 does not form alum (a) The lattice enthalpy of Na 2SO4 is less than its
5. Which is not correctly matched hydration enthalpy.
(1) Basic strength Cs2O < Rb2O < K2O < Na2O < Li2O of (b) Sodium is monovalent ion whereas barium is a
oxides divialent ion.
(c) The hydration enthalpy of sodium sulphate is less
(2) Stability of Na2O2 < K2O2 < Rb2O2 < Cs2O2 peroxides
than its lattice enthalpy.
(3) Stability of LiHCO3 < NaHCO3 < KHCO3 < RbHCO3 < (d) The lattice enthalpy of barium sulphate is less than
CsHCO3 its hydration enthalpy.
(4) Melting point NaF < NaCl < NaBr < NaI more CO
(a) 1 and 4 (b) 1 and 3 10. KO2 + CO2 + H2O ¾¾¾¾¾ 2®
[X] + [Y]
(c) 1 and 2 (d) 2 and 3 Products [X] and [Y] are respectively :
6. In the following sequence of reaction, identify the (a) K2CO3, O2 (b) KHCO3, O2
compounds (A), (B) (C) and (D) : (c) KOH, K2CO3 (d) KHCO3, H2O
EBD_8350
144 CHEMISTRY

11. Which one is the correct statement with reference to Statement-2 : The magnitude of hydration enthalpy
solubility of MgSO4 in water? depends upon the square of the charge on the ion.
(a) SO42– ion mainly contributes towards hydration energy (a) Statement-1 is True, Statement-2 is True; Statement-2
(b) Sizes of Mg2+ and SO42– are similar is a correct explanation for Statement-1.
(c) Hydration energy of MgSO4 is higher in comparison (b) Statement-1 is True, Statement-2 is True; Statement-2
to its lattice energy is NOT a correct explanation for Statement-1.
(d) Ionic potential (charge/radius ratio) of Mg2+ is very low (c) Statement -1 is True, Statement-2 is False.
12. All of the following substances react with water. The pair (d) Statement -1 is False, Statement-2 is True.
that gives the same gaseous product is 15. Consider the following statements
(a) K and KO2 (b) Na and Na2O2 The ionic conductivity of Na+ ions is greater than that of
(c) Ca and CaH2 (d) Ba and BaO2 the Li+ ions in aqueous medium, because
13. 100 mL of tap water containing Ca(HCO3)2 was titrated 1. The ionic radius of Na+ ion is greater than that of Li+
with N/50 HCl with methyl orange as indicator. If 30 mL of ion.
HCl were required, calculate the temporary hardness as 2. The velocity of Na+ ion is greater than that of Li + ion.
parts of CaCO3 per 106 parts of water. 3. The Li+ ion is more solvated than Na+ ion in water.
(a) 150 ppm (b) 300 ppm 4. Li+ ion and Na+ ion have different electron densities.
(c) 450 ppm (d) 600 ppm Which of the statements given above are correct?
14. Statement-1 : The magnitude of the hydration enthalpies (a) 1 and 3 (b) 2 and 3
of alkaline earth metal ions (M2+) are much larger than (c) 1 and 4 (d) 2 and 4
those for the alkali metal ions.

ANSW ER KEY
Exercise 1 : NCERT Based Topic-wise MCQs
1 (c) 10 (a) 19 (a) 28 (c) 37 (a) 46 (d) 55 (c) 64 (d) 73 (a) 82 (c)
2 (b) 11 (a) 20 (c) 29 (c) 38 (a) 47 (d) 56 (a) 65 (d) 74 (d) 83 (b)
3 (a) 12 (d) 21 (c) 30 (a) 39 (b) 48 (d) 57 (a) 66 (a) 75 (b) 84 (d)
4 (a) 13 (b) 22 (d) 31 (b) 40 (d) 49 (a) 58 (a) 67 (d) 76 (a) 85 (c)
5 (b) 14 (d) 23 (b) 32 (b) 41 (c) 50 (b) 59 (c) 68 (d) 77 (c) 86 (c)
6 (c) 15 (a) 24 (d) 33 (a) 42 (a) 51 (b) 60 (c) 69 (a) 78 (d) 87 (b)
7 (d) 16 (a) 25 (a) 34 (c) 43 (c) 52 (d) 61 (d) 70 (a) 79 (d) 88 (a)
8 (d) 17 (c) 26 (a) 35 (a) 44 (d) 53 (d) 62 (a) 71 (a) 80 (c) 89 (b)
9 (d) 18 (c) 27 (c) 36 (b) 45 (d) 54 (d) 63 (a) 72 (b) 81 (c) 90 (a)
Exercise 2 : NCERT Exemplar & Past Year MCQs
1 (d) 5 (c) 9 (c) 13 (c) 17 (a) 21 (d) 25 (d) 29 (c) 33 (b)
2 (a) 6 (a) 10 (d) 14 (d) 18 (b) 22 (b) 26 (c) 30 (a) 34 (d)
3 (a) 7 (a) 11 (a) 15 (a) 19 (a) 23 (d) 27 (b) 31 (d) 35 (a)
4 (a) 8 (a) 12 (c) 16 (b) 20 (b) 24 (c) 28 (b) 32 (b) 36 (c)
Exercise 3 : Problem Solving Skill Enhancer MCQs
1 (b) 3 (a) 5 (a) 7 (d) 9 (a) 11 (c) 13 (b) 15 (b)
2 (b) 4 (a) 6 (b) 8 (d) 10 (b) 12 (c) 14 (a)
The p-block Elements
11 (Group 13 and 14)
Trend Buster NEET & JEE Main

Number of Questions from 2020-15 9 5 Minimum one question has been


Weightage 2.8% 2.90% asked in NEET & JEEM every year.

The most Important Concepts that Cover Maximum number of Questions asked in past 6 years.

Group 13 elements : The Boron Family 4 3


Group 14 elements : The Carbon Family 5 1

Less Important Concepts that Cover 1 or 2 Questions asked in past 6 years.

Allotropes of Carbon — 1

Important Trends and Anomalous Properties of


Boron and its compounds / Use of Boron and
Aluminium and their Compounds / Important — —
Trends and Anomalous Behaviour of Carbon and
Important compounds of C & Si

NEET JEE

2020 Group 13 elements / Group Oxide and allotropes of 3 Average — —


14 elements carbon, zeolites / properties
of oxides of carbon / structure
and formulae
2019 Group 14 elements : The Chemical Reactivity / 2 Average 2 Easy /
Carbon Family / Allotrops of Complex stability / Average
Carbon / Group 13 Fullerenes / Inert Pair Effect /
elements : The Boron Family Chemical reactivity of Silicon
2018 Group 13 elements : Atomic Radii order / 2 Easy 2 Average
The Boron Family Compound Stability /
Chemical Reactivity
2017 Group 14 elements : Oxidation State Stability 1 Average — —
The Carbon Family
2016 — — — — — —
2015 Group 13 elements : Oxidation State Stability Order 1 Easy — —
The Boron Family
EBD_8350
146 CHEMISTRY
THE p-BLOCK ELEMENTS (GROUP 13 AND 14) 147
EBD_8350
148 CHEMISTRY

Problem Solving Tips/ Tricks/ Points to Remember

4 Silicon dioxide has a giant covalent structure, three H3BO3¾273K


¾¾® HBO¾
433K red
2 ¾¾® H2B4O7¾¾®
¾ B2O3
dimensional network of tetrahedra cells, with four oxygen hot
Orthoboric Metaboric Tetraboric Boron
atoms surrounding each silicon ion. acid acid acid trioxide
4 Carbides: Binary compounds in which carbon combines (Boric anhydride)
with more electropositive elements than itself (except 4 Zeolites are alumino silicates of the general formula,
hydrogen). It is classified into three :- Mx/n[(AlO2)x.(SiO2)y].mH2O
(1) Ionic or salt-like carbides: Highly electropositive
where, M may be a simple cation like Na +, K+, or Ca2+
elements of groups 1, 2, 13 (except B), coinage metals,
Zn, Cd and some lanthanides. n is the charge on the simple cation, m is the number of
(i) Acetylides molecules of water of hydration.
(ii) Methanides: Contain C4– ion and liberate methane 4 Ores of aluminium are bauxite and cryolite. If bauxite
on hydrolysis, e.g., Al4C3, BeC2. contain silica as the major impurity, it is known as white
(iii) Allylides: Contain C34– ion and give propyne or bauxite and if it contains red oxide as an impurity it is
allylene on hydrolysis, e.g., Mg2C3 known as red bauxite.
(2) Covalent carbides: Formed by metalloids, e.g., B4C and 4 Bayer’s process and Hall’s process are used for the
SiC. purification of red bauxite and serpeck’s process is used
(3) Metallic or interstitial carbides: Formed by transition for purification of white bauxite.
elements. 4 The correct formula of borax is Na2[B4O5(OH)4] 8H2O
4 Oil gas – Mixture of lower hydrocarbons CH4, C2H4, 4 Boric acid, B (OH)3 has a planar structure:
H
C2H2 etc., O
4 Pb3O4 is called red lead or sindoor and is a combination
H B
of 2PbO.PbO2. O O
4 Oil dag: It is a colloidal graphite solution in oil. It protects H
against corrosion. H
4 Aqua dag: It is a colloidal suspension of fine particles of O O
H B H H
graphite in water, used as a lubricant. It forms a smooth O O
continuous dry film with extra long wear life. O
H B H H B
4 Borax is also known as Tincal or Suhaga. Tincal contains O O
O O
45% borax.
4 Octahedral borax – Na 2B4O7.5H2O (also known as H H
BO3 units are joined by hydrogen bonds.
Jeweller’s borax)
4 The boron-10 isotope has high ability to absorb neutrons
4 Diborane with ammonia forms [B2H2(NH3)2]+ [BH4]–.
When this product is heated B3N 3H6; borazol or and used as control rods in nuclear industry.
inorganic benzene is formed. On strong heating, borazon 4 Borax and boric acid is used in the manufacturing of heat
(BN)x is formed. Borazon is harder than diamond. resistant glasses (pyrex), glass-wool and fibreglass.
4 Lewis acid strength of boron trihalides is BF3 < BCl3 < 4 Carbon exists in three isotopes: 12C, 13C and 14C; of which
14C is radioactive and used in carbon dating.
BBr3 < BI3. This is due to the back-donation of electron
pair from npz orbital on halogen to vacant 2pz orbital on 4 The major component of sand is quartz (silica or silicon
B-atom. dioxide, SiO2).
4 The density of graphite and diamond are 2.22 and 3.51 g/cm3
4 Borax or Sodium Tetraborate Decahydrate
respectively.
Its aqueous solution is basic in nature due to hydrolysis.
4 For diamond, 1 carat = 200 mg
Na2B4O7+ 7H2O ® 2NaOH + 4H3BO3 4 Carbon monoxide is a colourless, odourless and water
Na2B4O7.10H2O ¾¾¾®D
Na2B4O7 + 10H2O insoluble gas.
–10H O 2 4 Resonance structures of CO2.
740 °C Å Å
Na2B 4O7 ¾¾¾® 2NaBO 2 + B 2O3 O — C ºº O « O == C == O « O ºº C — O
Glassy bead
4 Vaseline is obtained from silicones.
4 Boric Acid or Orthoboric Acid 4 Mica is naturally occurring aluminium silicate.
It behaves as a lewis acid. 4 B4C3 (boron carbide) is one of the hardest known artificial
H3BO3 + H2O ® [B(OH)4]– + H + substance.
THE p-BLOCK ELEMENTS (GROUP 13 AND 14) 149

Exercise 1 : NCERT Based Topic-wise MCQs


12. Which metal is protected by a layer of its own oxide?
Topic 1: Group 13 Elements: The Boron Family
(a) Al (b) Ag (c) Au (d) Fe
1. Which of the following is most abundant in the earth crust ? 13. Which of the following does not react with aqueous
(a) Boron (b) Aluminium NaOH ?
(c) Gallium (d) Thallium (a) B (b) Al (c) Ga (d) Tl
2. The relationship between first, second and third ionisation 14. Which of the following hydroxide is acidic ?
enthalpies of each group-13 element is (a) Al(OH)3 (b) Ca(OH)3
(a) DiH1 > DiH2 > DiH3 (b) DiH1 < DiH2 < DiH3 (c) Tl(OH)3 (d) B(OH)3
(c) DiH1 = DiH2 > DiH3 (d) DiH3 > DiH1 > DiH2 15. Which among the following oxides react with alkali?
B2O3, Al2O3 and Tl2O
3. Which of the following is/are true regarding gallium?
(a) B2O3 and Al2O3 (b) Al2O3 and Tl2O
(i) It has unusually low melting point (303 K).
(c) Only B2O3 (d) B2O3 and Tl2O
(ii) It exist in liquid state during summer.
16. What is x in the following reaction?
(iii) It has a high boiling point (2676 K).
The correct option is Al(s) + NaOH (aq) + H2O (l) ® x + H2(g)
(a) (i) and (ii) (b) (i) and (iii) (a) Na2[Al(OH)4]– (b) Na+[Al(OH)4]–
(c) Na2[Al(OH)6] – (d) Na+ [Al(OH)6]–
(c) (i), (ii) and (iii) (d) (ii) and (iii)
4. The element which shows least metallic character is 17. Assertion : Atomic radius of gallium is higher than that
(a) Indium (b) Boron of aluminium
(c) Aluminium (d) Gallium Reason : The presence of additional d-electron offer
5. Which of the following does not form M3+ ion? poor screening effect for the outer electrons from
(a) Boron (b) Aluminium increased nuclear charge.
(c) Indium (d) Gallium (a) Assertion is correct, reason is correct; reason is a
6. Thallium shows different oxidation states because correct explanation for assertion.
(a) it is transition element (b) Assertion is correct, reason is correct; reason is not
(b) of inert pair effect a correct explanation for assertion
(c) of its amphoteric character (c) Assertion is correct, reason is incorrect
(d) of its higher reactivity (d) Assertion is incorrect, reason is correct.
7. Which out of the following compounds does not exist?
(a) BF3 (b) TlCl3 Topic 2: Important Trends and Anomalous Properties of Boron
(c) TlCl5 (d) Both (b) and (c) 18. The strongest Lewis acid is
8. Aluminium vessels should not be washed with materials (a) BF3 (b) BCl3 (c) BBr3 (d) BI3
containing washing soda because 19. NH3 and BF3 form an adduct readily because they form
(a) washing soda is expensive (a) a coordinate bond (b) a hydrogen bond
(b) washing soda is easily decomposed (c) an ionic bond (d) a covalent bond
(c) washing soda reacts with aluminium to form soluble 20. The factor responsible for weak acidic nature of B–F bonds
aluminate in BF3 is
(d) washing soda reacts with aluminium to form insoluble (a) large electronegativity of fluorine
aluminium oxide (b) three centred two electron bonds in BF3
9. Amphoteric oxide among the following is (c) pp - dp back bonding
(a) B2O3 (b) Ga2O3 (c) In2O3 (d) Tl2O3 (d) pp - pp back bonding
10. Which of the following statement(s) is/are incorrect ? 21. What is the oxidation state and hybridisation of boron in
(i) Trichlorides on hydrolysis in water form tetrahedral compound formed when BCl3 undergoes reaction with
[M(OH)4]– species. the water?
(ii) Hybridisation state of metal in tetrahedral species is (a) 3, sp2d (b) 3, sp3 (c) 4, sp3 (d) 3, sp2d
sp3. 22. Which of the following statement(s) is/are incorrect ?
(iii) Aluminium chloride in acidified aqueous solution (i) Higher boranes are not flammable.
forms [Al(OH)4]– ion. (ii) Boranes are hydrolysed by water to give orthoboric
(a) (i) and (ii) (b) (ii) only acid.
(c) (iii) only (d) (i) and (iii) (iii) Boranes undergoes cleavage reactions with Lewis
11. AlCl3 on hydrolysis gives bases to give borane adducts.
(a) Al2O3. H2O (b) Al(OH)3
(a) (i) only (b) (ii) and (iii)
(c) Al2O3 (d) AlCl3.6H2O
(c) (iii) only (d) (i) and (ii)
EBD_8350
150 CHEMISTRY

Topic 3: Some Important Compounds of Boron 32. Which of the following structure is similar to graphite?
(a) B (b) B4C (c) B2H6 (d) BN
23. In borax bead test which compound is formed? 33. Orthoboric acid when heated to red hot gives
(a) Ortho-borate (b) Meta-borate (a) metaboric acid (b) pyroboric acid
(c) Double oxide (d) Tetra-borate (c) boron and water (d) boric anhydride
24. H3BO3 on heating up to 373 K yields: 34. Inorganic benzene is
(a) boric anhydride (b) orthoboric acid (a) B3H3N3 (b) BH3NH3 (c) B3H6N3 (d) H3B3N6
(c) metaboric acid (d) tetraboric acid 35. The structure of diborane (B2H6) contains
25. Boric acid is polymeric due to (a) four 2c-2e– bonds and four 3c-2e– bonds
(a) its acidic nature (b) two 2c-2e– bonds and two 3c-3e– bonds
(b) the presence of hydrogen bonds (c) two 2c-2e– bonds and four 3c-2e– bonds
(c) its monobasic nature (d) four 2c-2e– bonds and two 3c-2e– bonds
(d) its geometry 36. Reaction of diborane with ammonia gives initially
26. Match Column-I (Compound of boron) with Column-II (a) B2H6 .NH3 (b) Borazole
(Use) and choose the correct option. (c) B2H6 .3NH3 (d) [BH2(NH3)2]+[BH4]–
Column-I Column-II 37. Select the correct statements for diborane :
(A) Metal borides (p) Flux for soldering (i) Boron is approximately sp3 hybridised
metals (ii) B – H – B angle is 180°
(B) Boron fibres (q) Bullet-proof vest (iii) There are two terminal B – H bonds for each boron
(C) Borax (r) As a mild antiseptic atom
(D) Boric acid (s) As control rods in (iv) There are only 12 bonding electrons
nuclear industry (a) (i), (ii) and (iv) (b) (i), (ii) and (iii)
(a) A – (q), B – (s), C – (r), D – (p) (c) (ii), (iii) and (iv) (d) (i), (iii) and (iv)
(b) A – (q), B – (s), C – (p), D – (r) 38. Identify (i) to (v) in reactions (1) and (2) On the basis of
(c) A – (s), B – (q), C – (r), D – (p) your identification choose the correct code for matching
(d) A – (s), B – (q), C – (p), D – (r) Column-I with Column-II.
27. In reaction D D
BF3 + 3LiBH4 ® 3LiF + X ; X is 1. Na2B4O7.10H2O ¾¾
® (i) ¾¾
® (ii) + (iii)
(a) B4H10 (b) B2H6 (c) BH3 (d) B3H8 2. Na2B4O7+7H2O ¾¾ ® (iv) + (v)
28. The bonds present in borazole or inorganic benzene are Column-I Column-II
(a) 9 s, 6p (b) 12 s, 3p (c) 6 s, 9p (d) 15 s only (A) (i) (p) H3BO3
29. Match the columns (B) (ii) (q) B2O3
Column-I Column-II (C) (iii) (r) NaBO2
(A) Borax-bead (p) Alum (D) (iv) (s) NaOH
(B) Inorganic benzene (q) Diborane (E) (v) (t) Na2B4O7
(C) Antiseptic (r) Metaborate (a) A – (t), B – (s), C – (p), D – (q), E – (r)
(D) Bridged hydrogens (s) Borazole (b) A – (r), B – (q), C – (s), D – (p), E – (t)
(a) A – (p), B – (r), C – (q), D – (s) (c) A – (t), B – (r), C – (q), D – (p), E – (s)
(b) A – (r), B – (s), C – (p), D – (q) (d) A – (t), B – (r), C – (s), D – (q), E – (p)
(c) A – (s), B – (r), C – (p), D – (q) 39. What is the colour obtained when borax is heated in a
(d) A – (q), B – (r), C – (s), D – (p) Bunsen burner flame with CoO?
30. Which one of the following is the correct statement? (a) Blue (b) Black (c) Green (d) Violet
(a) Boric acid is a protonic acid 40. The hybridisation of boron atom in orthoboric acid is
(b) Beryllium exhibits coordination number of six (a) sp (b) sp2 (c) sp3 (d) sp3 d
(c) Chlorides of both beryllium and aluminium have
41. Which of the following reaction shows production of
bridged structures in solid phase
diborane on industrial scale ?
(d) B2H6.2NH3 is known as ‘inorganic benzene’
31. Which of the following statements about H3BO3 is not (a) 4BF3 + 3LiAlH4 ¾¾
® 2B2H6 + 3LiF + 3AlF3
correct? (b) 2NaBH4 + I2 ¾¾
® B2H6 + 2NaI + H2
(a) It is a strong tribasic acid 450K
(b) It is prepared by acidifying an aqueous solution of borax (c) 2BF3 + 6NaH ¾¾¾® B2H6 + 6NaF
(c) It has a layer structure in which planar BO3 units are (d) Both (b) and (c)
joined by hydrogen bonds 42. The product/s formed when diborane is hydrolysed is/are
(d) It does not act as proton donor but acts as a Lewis (a) B2O3 and H3BO3 (b) B2O3 only
acid by accepting a lone pair of electrons (c) H3BO3 and H2 (d) H3BO3 only
THE p-BLOCK ELEMENTS (GROUP 13 AND 14) 151

55. The reducing power of divalent species decreases in the


Topic 4: Uses of Boron and Aluminium and Their Compounds
order
43. Which of the following properties of aluminium makes it (a) Ge > Sn > Pb (b) Sn > Ge > Pb
useful for food packaging ? (c) Pb > Sn > Ge (d) None of these
(a) Good electrical conductivity 56. The elements commonly used for making transistors are
(b) Good thermal conductivity (a) C and Si (b) Ga and In
(c) Low density (c) P and As (d) Si and Ge
(d) Non toxicity 57. The element which is exclusively applied as semi-conductor
44. BF3 is used as a catalyst in several industrial processes (a) Au (b) Ge (c) Pt (d) Si
due to its 58. Which of the following is not correct?
(a) strong reducing nature (a) Ge(OH)2 is amphoteric
(b) weak reducing action (b) GeCl2 is more stable than GeCl4
(c) strong Lewis acid nature
(c) GeO2 is weakly acidic
(d) weak Lewis acid character
(d) GeCl4 in HCl forms [ GeCl6]2– ion
45. Assertion : The use of aluminium and its compounds for
59. The main reason that SiCl4 is easily hydrolysed as
domestic purposes is now reduced considerably.
compared to CCl4 is that
Reason : The highly toxic nature of aluminium is the
(a) Si-Si bond is weaker
responsible factor.
(b) SiCl4 can form hydrogen bonds
(a) Assertion is correct, reason is correct; reason is a
(c) SiCl4 is covalent
correct explanation for assertion.
(d) Si can extend its coordination number beyond four
(b) Assertion is correct, reason is correct; reason is not
60. Which halide is least stable and has doubtful existence
a correct explanation for assertion
(a) CI4 (b) GeI4 (c) SnI4 (d) PbI4
(c) Assertion is correct, reason is incorrect
(d) Assertion is incorrect, reason is correct. 61. PbF4, PbCl4 exist but PbBr 4 and PbI4 do not exist because
of
Topic 5: Group 14 Elements : The Carbon Family (a) large size of Br– and I–
(b) strong oxidising character of Pb4+
46. Mark the oxide which is amphoteric in character
(c) strong reducing character of Pb4+
(a) CO2 (b) SiO 2 (c) SnO2 (d) CaO
(d) low electronegativity of Br – and I–.
47. The inert pair effect is most prominent in
62. Lead pipes are readily corroded by
(a) C (b) Pb (c) Ge (d) Si
48. The most stable +2 oxidation state is exhibited by (a) H2SO4 (b) HCl
(a) Fe (b) Sn (c) Pb (d) Si (c) CH3COOH (d) pure water
49. The oxide of lead used in lead accumulators is 63. Lead pipes are not suitable for drinking water because
(a) PbO (b) Pb 2 O3 (c) Pb3O 4 (d) PbO 2 (a) lead forms basic lead carbonate
50. Least thermally stable is (b) lead reacts with water containing air to form Pb(OH)2
(a) CCl4 (b) SiCl4 (c) GeCl4 (d) GeBr4 (c) a layer of lead dioxide is deposited over pipes
51. Which of the following halides is the most stable? (d) lead reacts with air to form litharge
(a) CF4 (b) CI4 (c) CBr4 (d) CCl4 64. Which of the following species exists (A) [SiF6]2– ,
52. Unlike the other elements of its group, carbon and silicon (B) [GeCl6]2– and (C) [CCl6]2– ?
does not form MX2 type molecules because (a) (A) and (B) (b) (B) and (C)
(a) energetically this is not possible (c) Only (C) (d) (A) and (C)
(b) carbon undergoes catenation 65. Ge (II) compounds are powerful reducing agents
(c) it is non-metallic whereas Pb (IV) compounds are strong oxidants. It is
(d) carbon does not contain d-orbital because
53. Which of the following is/are not correctly matched ? (a) Pb is more electropositive than Ge
(i) GeO2 – Acidic (ii) PbO2– Amphoteric (b) ionization potential of lead is less than that of Ge
(iii) CO – Neutral (iv) SiO2 – Amphoteric
(c) ionic radii of Pb2+ and Pb4+ are larger than those of
(a) (i) and (iv) (b) (iv) only
Ge2+ and Ge4+
(c) (ii) only (d) (iii) only
(d) of more pronounced inert pair effect in lead than
54. The stability of dihalides of Si, Ge, Sn and Pb increases
in Ge
steadily in the sequence
66. A group 14 element is oxidised to form corresponding
(a) PbX 2 << SnX 2 << GeX 2 << SiX 2
oxide which is gaseous in nature. When dissolved in water,
(b) GeX2 << SiX2 << SnX2 << PbX2 pH of the water decreases; further addition of group 2
(c) SiX2 << GeX2 << PbX2 << SnX2 hydroxides leads to precipitation. This oxide can be
(d) SiX2 << GeX2 << SnX2 << PbX2. (a) GeO2 (b) CO (c) CO2 (d) SnO2
EBD_8350
152 CHEMISTRY

67. Which among the following can act as reducing agent 77. Match columns
(A) SnCl2, (B) CO and (C) PbCl2 ? Column-I Column-II
(a) (A) and (B) (b) (B) and (C) (A) Graphite fibres (p) Abrasive for sharpening
(c) (C) and (A) (d) Only (B) hard tools
68. Lead is not affected by dil. HCl in cold because (B) Carbon black (q) Formation of light
(a) Pb is less electronegative than H weight composites.
(b) PbO film is formed which resists chemical attack by (C) Charcoal (r) Used in water filters to
acid remove organic
(c) PbCl2 protective coating gets formed on Pb surface contaminators
(d) PbO2 film is always present on Pb surface, which resist (D) Diamond (s) As filler in automobile
chemical attack tyres
69. Assertion : PbI4 of lead does not exist. (a) A – (s), B – (q), C – (r), D – (p)
(b) A – (q), B – (s), C – (r), D – (p)
Reason : Pb–I bond initially formed during the reaction
(c) A – (q), B – (r), C – (s), D – (p)
does not release enough energy to unpair 6s2 electrons.
(d) A – (p), B – (r), C – (s), D – (q)
(a) Assertion is correct, reason is correct; reason is a
78. Graphite is a soft solid lubricant extremely difficult to melt.
correct explanation for assertion.
The reason for this anomalous behaviour is that graphite
(b) Assertion is correct, reason is correct; reason is not
(a) is an allotropic form of diamond
a correct explanation for assertion
(b) has molecules of variable molecular masses like
(c) Assertion is correct, reason is incorrect polymers
(d) Assertion is incorrect, reason is correct. (c) has carbon atoms arranged in large plates of rings of
strongly bound carbon atoms with weak inter plate
Topic 6: Important Trends and Anomalous Behaviour of Carbon bonds
70. Carbon and silicon belong to group 14. The maximum (d) is a non-crystalline substance
coordination number of carbon in commonly occurring 79. Which one of the following allotropic forms of carbon is
compounds is 4, whereas that of silicon is 6. This is due to isomorphous with crystalline silicon?
(a) large size of silicon (a) Graphite (b) Coal (c) Coke (d) Diamond
(b) more electropositive nature of silicon 80. Assertion : Graphite is thermodynamically most stable
(c) availability of d-orbitals in silicon allotrope of carbon.
(d) Both (a) and (b) Reason : D fH of graphite is taken as zero.
71. The catenation tendency of C, Si and Ge is in the order (a) Assertion is correct, reason is correct; reason is a
Ge < Si < C. The bond energies (in kJ mol –1) of C-C, Si-Si correct explanation for assertion.
and Ge-Ge bonds, respectively are (b) Assertion is correct, reason is correct; reason is not
(a) 167, 180, 348 (b) 180, 167, 348 a correct explanation for assertion
(c) 348, 167, 180 (d) 348, 180, 167 (c) Assertion is correct, reason is incorrect
72. The element that does not show catenation among the (d) Assertion is incorrect, reason is correct.
following p-block elements is
(a) carbon (b) silicon Topic 8: Some Important Compounds of Carbon and Silicon
(c) germanium (d) lead
81. Crystalline form of silica is called
(a) crystalline silicon (b) quartz
Topic 7: Allotropes of Carbon
(c) rock (d) talc
73. Buckminster fullerene is 82. Glass reacts with HF to produce
(a) pure graphite (b) C-60 (a) SiF4 (b) H2SiF6 (c) H2SiO3 (d) Na3AlF6
(c) diamond (d) C-90 83. Producer gas is the mixture of
74. The hybridisation state of carbon in fullerene is (a) CO + N2 (b) CO + H2
(a) sp (b) sp2 (c) sp3 (d) sp3d (c) CO + water vapours (d) N2 + CH4
75. In graphite, electrons are 84. In silica (SiO2), each silicon atom is bonded to
(a) localised on every third C-atom (a) two oxygen atoms
(b) present in anti-bonding orbital (b) four oxygen atoms
(c) localised on each C-atom (c) one silicon and two oxygen atoms
(d) spread out between the structure (d) one silicon and three oxygen atoms
76. The percentage of s-character of the hybrid orbitals of 85. R3SiCl on hydrolysis forms
carbon in graphite and diamond are respectively (a) R3SiOH (b) R3Si – O – SiR3
(a) 33, 25 (b) 50, 50 (c) 67, 25 (d) 33, 67 (c) R2Si = O (d) None of these
THE p-BLOCK ELEMENTS (GROUP 13 AND 14) 153

86. Which of the following statements is false? (iv) CO2 as dry ice is used as a refrigerant for ice cream
(a) Water gas is a mixture of hydrogen and carbon and frozen food.
monoxide (a) (i) and (ii) (b) Only (ii)
(b) Producer gas is a mixture of CO and nitrogen (c) (i), (ii) and (iii) (d) (ii) and (iii)
(c) Water gas is a mixture of water vapour and hydrogen 91. Which of the following is used in surgical and cosmetic
(d) Natural gas consists of methane, ethane and gaseous plants?
hydrocarbons. (a) Silicones (b) Silicates
87. Which gas is essential constituent of almost all fuel gases? (c) Silica (d) None of these
(a) CO2 (b) N2 92. Which type of zeolite is used to convert alcohols
directly into gasoline ?
(c) CO (d) H2O
(a) ZSM – 3 (b) ZSM – 5
88. CO2 is used for extinguishing fire because (c) ZSM – 2 (d) All of these
(a) it has a relatively high critical temperature 93. Which one of the following statements about the zeolites
(b) in solid state, it is called dry ice is false ?
(c) it is neither combustible nor a supporter of combustion (a) They are used as cation exchangers
(d) it is a colourless gas (b) They have open structure which enables them to take
89. The correct statement with respect to CO is up small molecules
(a) it combines with H2O to give carbonic acid (c) Zeolites are aluminosilicates having three dimensional
(b) it reacts with haemoglobin in RBC network
(c) it is powerful oxidising agent (d) None of the above
(d) it is used to prepare aerated drinks 94. Coal gas is a mixture of
90. Which of the following statement(s) is / are incorrect for (a) H2O and CO (b) H2, CO, N2 and CH4
CO2? (c) H2 and CO (d) CH4 and CO
(i) In laboratory CO2 is prepared by the action of dilute 95. _____ helps to maintain pH of blood between 7.26 to
HCl on calcium carbonate 7.42
(ii) Carbon dioxide is a poisonous gas (a) CO2 (b) H2CO3
(iii) Increase in carbon dioxide content in atmosphere (c) CO32 - (d) H2CO3/HCO3–
lead to increase in green house effect.

NCERT Exemplar MCQs atom. The hybridisation of orbitals of central atom in


[B(OH4)]– and the geometry of the complex are respectively.
1. The elements which exists in liquid state for a wide range
of temperature and can be used for measuring high (a) sp3, tetrahedral (b) sp3, square planar
3 2
(c) sp d , octahedral (d) dsp2, square planar
temperature is
8. Boric acid is an acid because its molecule
(a) B (b) Al (c) Ga (d) In
(a) contains replaceable H+ ion
2. Which of the following is a Lewis acid? (b) gives up a proton
(a) AlCl3 (b) MgCl2 (c) CaCl2 (d) BaCl2 (c) accepts OH– from water releasing proton
3. Which of the following oxides is acidic in nature? (d) combines with proton from water molecule
(a) B2O3 (b) Al2O3 (c) Ga2O3 (d) In2O3 9. Catenation i.e., linking of similar atoms depends on size
4. The exhibition of highest coordination number depends and electronic configuration of atoms. The tendency of
on the availability of vacant orbitals in the central atom. catenation in group 14 elements follows the order
Which of the following elements is not likely to act as (a) C > Si > Ge > Sn (b) C > > Si > Ge » Sn
central atom in MF63- ? (c) Si > C > Sn > Ge (d) Ge > Sn > Si > C
(a) B (b) Al (c) Ga (d) In 10. In the structure of diborane,
5. Quartz is extensively used as a piezoelectric material, it (a) All hydrogen atoms lie in one plane and boron atoms
contains .............. . lie in a plane perpendicular to this plane
(b) 2 boron atoms and 4 terminal hydrogen atoms lie in
(a) Pb (b) Si (c) Ti (d) Sn
the same plane and 2 bridging hydrogen atoms lie in
6. Dry ice is
the perpendicular plane
(a) solid NH3 (b) solid SO2
(c) 4 bridging hydrogen atoms and boron atoms lie in
(c) solid CO2 (d) solid N2 one plane and two terminal hydrogen atoms lie in a
7. The geometry of a complex species can be understood from plane perpendicular to this plane
the knowledge of type of hybridisation of orbitals of central (d) All the atoms are in the same plane
EBD_8350
154 CHEMISTRY

11. A compound X, of boron reacts with NH3 on heating to 22. Which of the following is incorrect statement?
give another compound Y which is called inorganic [NEET 2019]
benzene. The compound X can be prepared by treating (a) PbF4 is covalent in nature
BF3 with lithium aluminium hydride. The compounds X (b) SiCl4 is easily hydrolysed
and Y are represented by the formulas. (c) GeX4 (X = F, Cl, Br, I) is more stable than GeX2
(a) B2H6, B3N3H6 (b) B2O3, B3N3H6
(d) SnF4 is ionic in nature
(c) BF3, B3N3H6 (d) B3N3H6, B2H6
23. Which of the following species is not stable ?
12. The most commonly used reducing agent is
[NEET 2019]
(a) AlCl3 (b) PbCl2 (c) SnCl4 (d) SnCl2
(a) [SiF6]2– (b) [GeCl6]2–
13. Cement, the important building material is a mixture of
oxides of several elements. Besides calcium, iron and (c) [Sn(OH)6]2– (d) [SiCl6]2–
sulphur, oxides of elements of which of the group (s) are 24. C60, an allotrope of carbon cantains: [JEE M 2019]
present in the mixture? (a) 12 hexagons and 20 pentagons.
(a) Group 2 (b) Groups 2, 13 and 14 (b) 18 hexagons and 14 pentagons.
(c) Group 2 and 13 (d) Groups 2 and 14 (c) 16 hexagons and 16 pentagons.
14. Silicon has a strong tendency to form polymers like (d) 20 hexagons and 12 pentagons.
silicones. The chain length of silicone polymer can be 25. Aluminium is usually found in +3 oxidation state. In
controlled by adding. contrast, thallium exists in +1 and + 3 oxidation states.
(a) MeSiCl3 (b) Me2SiCl2 This is due to: [JEE M 2019]
(c) Me3SiCl (d) Me4Si (a) inert pair effect
15. Ionisation enthalpy (DiH kJ mol–1) for the elements of (b) diagonal relationship
group 13 follows the order (c) lattice effect
(a) B > Al > Ga > In > Tl (b) B < Al < Ga < In < Tl (d) lanthanoid contraction
26. Correct statements among 'A' to 'D' regarding silicones
(c) B < Al > Ga < In > Tl (d) B > Al < Ga > In < Tl
are: [JEE M 2019]
Past Year MCQs (A) They are polymers with hydrophobic character.
16. The stability of +1 oxidation state among Al, Ga, In and Tl (B) They are biocompatible.
increases in the sequence : [AIPMT 2015 RS, A] (C) In general, they have high thermal stability and low
(a) Ga < In < Al < Tl (b) Al < Ga < In < Tl dielectric strength.
(c) Tl < In < Ga < Al (d) In < Tl < Ga < Al (D) Usually, they are resistant to oxidation and used as
17. It is because of inability of ns2 electrons of the valence greases.
shell to participate in bonding that: [NEET 2017, A] (a) (A), (B), (C) and (D)
(b) (A), (B) and (C) only
(a) Sn2+ is oxidising while Pb4+ is reducing
(c) (A) and (B) only
(b) Sn2+ and Pb2+ are both oxidising and reducing
(d) (A), (B) and (D) only
(c) Sn4+ is reducing while Pb4+ is oxidising
27. Identify the correct statements from the following :
(d) Sn2+ is reducing while Pb4+ is oxidising [NEET 2020, S]
18. The correct order of atomic radii in group 13 elements is (A) CO2(g) is used as refrigerant for ice-cream and frozen food.
[NEET 2018, A] (B) The structure of C60 contains twelve six carbon rings
(a) B < Al < In < Ga < Tl (b) B < Al < Ga < In < Tl and twenty five carbon rings.
(c) B < Ga < Al < In < Tl (d) B < Ga < Al < Tl < In (C) ZSM-5, a type of zeolite, is used to convert alcohols
19. Which one of the following elements is unable to form into gasoline.
MF63– ion? [NEET 2018, S] (D) CO is colourless and odourless gas.
(a) Ga (b) Al (c) In (d) B (a) (A) and (C) only (b) (B) and (C) only
20. Which of the following are Lewis acids? [JEE M 2018, A] (c) (C) and (D) only (d) (A), (B) and (C) only
(a) PH3 and BCl3 (b) AlCl3 and SiCl4 28. Which of the following is not correct about carbon
(c) PH3 and SiCl4 (d) BCl3 and AlCl3 monoxide ? [NEET 2020, C]
21. When metal ‘M’ is treated with NaOH, a white gelatinous (a) It reduces oxygen carrying ability of blood.
precipitate ‘X’ is obtained, which is soluble in excess of (b) The carboxyhaemoglobin (haemoglobin bound to CO)
NaOH. Compound ‘X’ when heated strongly gives an oxide is less stable than oxyhaemoglobin.
which is used in chromatography as an adsorbent. The (c) It is produced due to incomplete combustion.
metal ‘M’ is : [JEE M 2018, A] (d) It forms carboxyhaemoglobin
(a) Zn (b) Ca (c) Al (d) Fe
THE p-BLOCK ELEMENTS (GROUP 13 AND 14) 155

29. Match the following and identify the correct option. (A) (B) (C) (D)
[NEET 2020, S] (a) (iii) (ii) (i) (iv)
(A) CO(g) + H2(g) (i) Mg(HCO3)2 + Ca(HCO3)2
(b) (iii) (iv) (ii) (i)
(B) Temporary (ii) An electron deficient hydride
hardness of water (c) (i) (iii) (ii) (iv)
(C) B2H6 (iii) Synthesis gas (d) (iii) (i) (ii) (iv)
(D) H2O2 (iv) Non-planar structure

1. Aqueous solution of borax acts as a buffer because (a) Al3+ and Cl– ions
(a) It contains tribasic acid and strong base. (b) [Al(H2O)6]3+ and Cl– ions
(b) It contains weak acid and its salt with strong base.
(c) [AlCl2 (H2O)4]+ and [AlCl4 (H2O)2]– ions
(c) It contains number of neutral water molecules.
(d) none of the above
(d) None of the above.
8. Borax is converted into crystalline boron by the following
2. The straight chain polymer is formed by : steps:
(a) Hydrolysis of CH3SiCl3 followed by condensation X D Y
polymerisation. Borax ¾¾® H3BO3 ¾¾® B2O3 ¾¾® B
D
(b) Hydrolysis of (CH3) 4 Si followed by addition X and Y are respectively:
polymerisation. (a) HCl, Mg (b) HCl, C
(c) Hydrolysis of (CH3)2SiCl2 followed by condensation (c) C, Al (d) HCl, Al
polymerisation. 9. The dissolution of Al(OH)3 by a solution of NaOH results
(d) Hydrolysis of (CH3)3SiCl followed by condensation in the formation of:
polymerisation. (a) [Al(H2O)4 (OH2)]+ (b) [Al(H2O)3 (OH)3]
3. Which of the following reaction does not take place? (c) [Al(H2O)2 (OH4)]– (d) [Al(H2O)6 (OH3)]
2+ 4+
(a) 2HgCl2 + Sn ® Hg 2Cl2 + Sn + 2Cl-
10. ( Si2O5 ) 2n–
n
anion is obtained when:
3+ 2+ 4+ 2+
(b) 2Fe + Sn ® Sn + 2Fe
(a) no oxygen of a SiO44- tetrahedron is shared with
2+ 2+ 3+
(c) 2Fe + Sn ® 2Fe + Sn
another SiO44- tetrahedron
(d) Hg 2Cl 2 + Sn 2 + ® 2Hg + Sn 4+ + 2Cl-
4. On addition of excess of sodium hydroxide solution to (b) one oxygen of a SiO44- tetrahedron is shared with
stannous chloride solution, we obtain :
another SiO44- tetrahedron
(a) Sn(OH)2 (b) SnO2 .H2O
(c) Na2SnO2 (d) None of these (c) two oxygen of a SiO44- tetrahedron are shared with
5. The water repelling characteristic of silicones is due to
another SiO44- tetrahedron
(a) the presence of alkyl group pointed towards surface
(b) strong Si–O–Si-bonds (d) three oxygen of a SiO44- tetrahedron are shared with
(c) low surface area
(d) high van der Waal’s forces another SiO44- tetrahedron
6. Which of the following carbides consists of C34 - type of 11. The gas evolved on heating CaF 2 and SiO 2 with
concentrated H2 SO 4, on hydrolysis gives a white
anionic part?
gelatinous precipitate. The precipitate is:
(a) Al4C3 (b) CaC2
(a) hydrofluorosilicic acid
(c) B4C (d) Mg2C3 (b) silica gel
7. Anhydrous aluminium chloride (Al 2Cl 6) is covalent (c) silicic acid
compound and soluble in water giving: (d) calcium fluorosilicate
EBD_8350
156 CHEMISTRY

12. In the following reactions, the Pb compounds A and B are (a) (ii) and (iii) (b) (i), (ii) and (iii)
respectively. (c) (i) and (iv) (d) (i), (ii), (iii) and (iv)
14. Organosilicon compound can be prepared by which of the
HNO3
®A following methods?
Pb3O4 (i) Mg 2Si + H 2SO 4 ®

® B (ii) SiCl4 + Li(AlH4 ) ®


HNO3 + H2O2
(iii) CH3SiCl3 + CH3MgCl ®
(a) Pb(NO3)2 + PbO2 and Pb(NO3)2 Cu catalyst
(b) Pb(NO3)2 and Pb(NO3)2 (iv) Si + CH3Cl ¾¾¾¾¾
280 -300 °C
®

(c) PbO2 and Pb(NO3)2 (a) (i) and (iii) (b) (i), (iii) and (iv)
(d) Pb(NO3)2 and PbO2 + Pb(NO3)2 (c) (iii) and (iv) (d) Only (iv)
13. Diborane can be prepared by which of the following 15. In the following sets of reactants which two sets best
methods? exhibit the amphoteric characters of Al2O3× xH2O?
heating Set 1: Al2O3 × xH2O (s) and OH– (aq)
(i) Mg 3 B2 + H 3 PO 4 ¾¾¾¾
®
Set 2: Al2O3 × xH2O (s) and H2O (l)
ether
(ii) BX3 + LiAlH 4 ¾¾¾¾¾®
X = F, Cl, Br Set 3: Al2O3 × xH2O (s) and H+ (aq)
180 °C
(iii) BF3 (g) + NaH ¾¾¾¾ ® Set 4: Al2O3 × xH2O (s) and NH3 (aq)
in diglyme (a) 1 and 2 (b) 1 and 3
(iv) NaBH 4 + I 2 ¾¾ ¾¾¾ ®
solution
(c) 2 and 4 (d) 3 and 4

ANSWER KEY
Exercise 1 : NCERT Based Topic-wis e MCQs
1 (b) 11 (b) 21 (b) 31 (a) 41 (c) 51 (a) 61 (b) 71 (d) 81 (b) 91 (a)
2 (b) 12 (a) 22 (a) 32 (d) 42 (c) 52 (a) 62 (c) 72 (d) 82 (b) 92 (b)
3 (c) 13 (a) 23 (b) 33 (d) 43 (c) 53 (b) 63 (b) 73 (b) 83 (a) 93 (d)
4 (b) 14 (d) 24 (c) 34 (c) 44 (d) 54 (d) 64 (a) 74 (b) 84 (b) 94 (b)
5 (a) 15 (a) 25 (b) 35 (d) 45 (a) 55 (a) 65 (d) 75 (d) 85 (b) 95 (d)
6 (b) 16 (b) 26 (d) 36 (d) 46 (c) 56 (d) 66 (c) 76 (a) 86 (c)
7 (c) 17 (d) 27 (b) 37 (d) 47 (b) 57 (b) 67 (a) 77 (b) 87 (c)
8 (c) 18 (d) 28 (b) 38 (c) 48 (c) 58 (b) 68 (c) 78 (c) 88 (c)
9 (b) 19 (a) 29 (b) 39 (a) 49 (d) 59 (d) 69 (a) 79 (d) 89 (b)
10 (c) 20 (d) 30 (c) 40 (b) 50 (d) 60 (d) 70 (c) 80 (a) 90 (b)
Exercis e 2 : NCERT Exemplar & Past Year MCQs
1 (c) 4 (a) 7 (a) 10 (b) 13 (b) 16 (b) 19 (d) 22 (a) 25 (a) 28 (b)
2 (a) 5 (b) 8 (c) 11 (a) 14 (c) 17 (d) 20 (d) 23 (a) 26 (d) 29 (d)
3 (a) 6 (c) 9 (b) 12 (d) 15 (d) 18 (c) 21 (c) 24 (d) 27 (c)
Exercise 3 : Problem Solving S kill Enhancer MCQs
1 (b) 3 (c) 5 (a) 7 (c) 9 (c) 11 (d) 13 (d) 15 (b)
2 (c) 4 (c) 6 (d) 8 (d) 10 (d) 12 (a) 14 (c)
Organic Chemistry–Some
12 Basic Principles & Techniques

Trend Buster NEET & JEE Main

Number of Questions from 2020-15 13 5 This chapter has more weightage


Weightage 4.1% 2.9% for NEET compared to JEE M.

The most Important Concepts that Cover Maximum number of Questions asked in past 6 years.

Fundametal Concepts in Organic Reaction Mechanism 10 —


Nomenclature of Organic Compounds 1 1
Methods of Purification of Organic Compounds — 2

Less Important Concepts that Cover 1 or 2 Questions asked in past 6 years.

Tetravalency of Carbon / Structure representation & — —


classification of Organic Compounds
Isomerism — 1

Qualitative and Quantitative analysis of Organic Compounds 2 1

NEET JEE

2020 Qualitative and Quantitative Extraction of mixture of 2 Average 1 Difficult


Analysis / Fundamental concepts Organic Compounds / Chromato
in Organic Reaction Mechanism graphy / stability of carbocation
2019 Nomenclature of Organic IUPAC Nomenclature — — 1 Easy
Compounds
2018 Fundametal Concepts in Hybridisation / Carbocation 3 Easy — —
Organic Reaction Mechanism Stability / Inductive Effect
2017 Nomenclature of Organic IUPAC nomenclature / 2 Easy 1 Easy
Compounds / Fundametal Nature of Electrophiles /
Concepts in Organic Raction Resonance
Mechanism
2016 Fundametal Concepts in Carbanion Hybridisation / 1 Easy 1 Average
Organic Reaction Mechanism / Distillation
Methods of Purification of
Organic Compounds
2015 Fundametal Concepts in Tautomerism / Nature of 5 Easy / 1 Easy
Organic Reaction Mechanism / Nucleophiles / Hyperconguation / Average
Qualitative and Quantitative Electron Movement / Nitrogen
Analysis / Isomerism Estimation / Geometrical
Isomerism
EBD_8350
158 CHEMISTRY
ORGANIC CHEMISTRY–SOME BASIC PRINCIPLES & TECHNIQUES 159
EBD_8350
160 CHEMISTRY

Problem Solving Tips/ Tricks/ Points to Remember


Seniority table for principal groups 4 In order to find the number of hyper conjugations, following
formula is used
Group Prefix name Suffix name
No. of hyper conjugation structure = No. of alpha
–SO3H Sulpho– sulphonic acid hydrogen + 1
–COOH Carboxy– –oic acid 4 If resonance, mesomeric, hyperconjugation, inductive
–COOR Alkyl-oxycarbonyl –alkyl---oates effect and aromatic effect is asked in a single problems,
then their predomination becomes important. So always
–COXHaloformyl– –oyl halide
remember the following order
–CONH2 Carbamoyl– amide Aromatic > Resonance > Mesomeric > hyper conjugation
–CHO Formyl– –al > Inductive
–CN Cyano– –nitrile 4 The retardation factor (Rf) value is defiend as the ratio of
the distance moved by the solute and the distance moved
–CO Keto- or Oxo –one
by the solvent along the chromaplate, where both distances
–OH Hydroxy– –ol are measured from the common origin or baseline. The
–SH Mercapto– –thiol diagram explains what are the distances to be taken to
–NH2 Amino– –amine calculate these Rf s. By definition the actual Rf is that of
the centre (b). Generally, Rf values are always between
> C = C < ; —C º C– – –ene, –yne
zero and one.
–X (halogen) Halo– –
–NO2 Nitro– – Solvent front a
Rf top = Rf (a) =
–NO Nitroso– – d
a(top edge) b
–O– Epoxy– – Rf bottom = Rf (b) =
d
R— Alkyl– – d b(bottom
edge) éa + bù
4 Metamerism is a rare type of isomerism and is limited to êë 2 úû
Base Rf centre = Rf (c) =
molecules having a divalent atom like O or S and alkyl line d
group around it. It is also observed in compounds
containing four and more than four carbon atoms.

4 Reactive Intermediates:
Name Carbocations or Carbanion Free radicals Singlet Triplet
Carbonium ion carbene carbene
R
C+ R R R
R sp 2 sp3 C– C R C R C R
Structure R R sp2
vacant
R R sp2 R sp
p-orbital (Pyramidal) (Planar, paramagnetic) (Bent, diamagnetic) (Linear, paramagnetic)
(Planar)

4 Stability order of carbocations: 4 Stability order of carbanions:


+ + + + + – – – –
e > Ph3C > CH > Ph 2CH > CH 2 HC º C > ( C6 H 5 )3 C > ( C6 H 5 ) 2 C H > CH 2 = C H >
+ + +
> (CH3)3C > Ph CH2> CH2CH–CH2 – – – æ– ö – æ– ö –
C H3 > C H3 C H 2 > ç C H3 ÷ C H > ç C H 3 ÷ C
+ + +
( CH3 )2 C H > CH3 C H 2 > C H3 > è ø2 è ø3
4 Stability order of free radicals:
+ + +
g g g g
CH 2 = C H > HC º C > C6 H5
( C6 H5 )3 C > ( C6 H5 )2 C H > ( CH3 )3 C > C6 H5 C H 2 >
g g g g
CH 2 = CH – C H 2 > ( CH 3 )2 C H > C H 3 > HC º C
ORGANIC CHEMISTRY–SOME BASIC PRINCIPLES & TECHNIQUES 161

4 Types of chromatography: * For N & S (both present together) :


As in test for nitrogen; instead of green or blue colour,
Stationary Phase : Solid blood red colouration (due to Na2[Fe(SCN)]) confirms
Adsorption (alumina, Silica gel, presence of N & S both.
chromatography MgO etc.) Column
If sodium fusion is carried out with excess of Na, NaSCN
Chromatography

Mobile Phase – Liquid decomposes to NaCN and Na2S.


(Liquid-solid)
chromatography Thin layer
NaSCN + 2Na ¾® NaSCN + 2Na ¾¾
® NaCN + Na 2 S
Stationary Phase – Liquid 144244 3
Partition Gives usual test
chromatography * If N & S both are present, then Lassaigne’s extract is boiled
Thin layer
Mobile Phase – Liquid with HNO3 to expel N & S which would otherwise interfere
(Liquid-liquid) Column
chromatography
in the test of halogens.
Paper * For Halogen :
White ppt.
4 Qualitative analysis: The detection of various elements Soluble in NH4OH
(Chlorine confirms)
present in a compound is called qualitative analysis. NaX + HNO3 (dil.)
Yellow ppt.
Detection of C and H : + AgNO3 Sparingly soluble
Common reaction in NH4OH
D (Bromine confirms)
C + 2CuO ¾¾ ® 2Cu + CO2 (tested with lime water) Yellow ppt.
D Insoluble in NH 4OH
2H + CuO ¾¾ ® Cu + H2O (tested with anhydrous CuSO4) (Iodine confirms)
Detection of N, S and X : Detection of Phosphorus :
Lassaigne’s test :
Na O
Na Organic Compound ¾¾¾¾2 2®
Na3PO4
Organic compound ¾¾¾¾ ® NaCN + Na 2S + NaX + NaCNS fusion
(C, N, S, X)
fusion 1444442444443
Ionic compounds boil
Na3PO4 + 3HNO3 ¾¾¾
(Covalent compounds) ® H3PO4 + 3NaNO3
NaCNS is formed only when N and S both are present H3PO4 + 12(NH4)2MoO4 + 21HNO3 ¾¾
®
together in an O.C. Ammonium
4 Confirmatory test: melybdate
* For nitrogen : (NH4)3PO4 × 12MoO3 + 21NH4NO3 + 12H2O
(NaCN + FeSO4 + NaOH) + FeCl3 + HCl (conc.) ¾¾ ® Ammonium
Fe4[Fe(CN)6]3 × xH2O phosphomolybdate
Prussion blue * Kjeldahl method is not applicable to compounds containing
* For Sulphur : nitrogen in nitro, azo groups and nitrogen present in rings
Na2S + CH3COOH + (CH3COO)2Pb ¾¾
® PbS (e.g.,) pyridine, quinoline, isoquinoline) as nitrogen of these
(Black ppt.) compounds does not convert to (NH4)3SO4 under the
Na2S + Na2[Fe(CN)5NO] ¾¾
® Na4[Fe(CN)5NOS] conditions of this method.
Sod. nitroprusside Violet colour * CHN elemental analyser is used to determine elements, C,
H and N quantitatively, if organic substance is available in
very small quantity.
EBD_8350
162 CHEMISTRY

Exercise 1 : NCERT Based Topic-wise MCQs


Topic 1: Tetravalence of Carbon : Shapes of Organic (C) Benzenoid compound (r)
Compounds S
1. Which of the following statement(s) is/are correct ? (D) Heterocyclic aromatic (s)
compound
(i) A carbon atom having an sp hybrid orbital is less
(a) A – (r), B – (p), C – (s), D – (q)
electronegative than carbon atoms possessing sp2
(b) A – (s), B – (p), C – (q), D – (r) O
or sp3 hybridised orbitals.
(c) A – (p), B – (r), C – (s), D – (q)
(ii) p-bonds provide the most reactive centres in the (d) A – (r), B – (p), C – (q), D – (s)
molecules containing multiple bonds 7. Which of the following compounds contains 1°, 2°, 3° as
(iii) The number of s and p bonds in compound well as 4° carbon atoms ?
CH2= C= CHCH3 are 7 and 2 respectively. (a) Neopentane (b) 2-methyl pentane
(a) (i) and (iii) (b) (ii) and (iii) (c) 2,3-dimethyl butane (d) 2,2,3-trimethyl pentane
(c) (ii) only (d) (i) only 8. The compound which has one isopropyl group is
2. The state of hybridization of C2, C3, C5 and C6 of the (a) 2, 2, 3, 3 - Tetramethylpentane
hydrocarbon, (b) 2, 2 - Dimethylpentane
CH3 CH3 (c) 2, 2, 3- Trimethylpentane
| | (d) 2- Methypentane
CH3 C CH = CH CH C º CH
7 6| 5 4 3 2 1
Topic 3: Nomenclature of Organic Compounds
CH3
is in the following sequence 9. IUPAC name of following compound is :
(a) sp3, sp2, sp2 and sp (b) sp, sp2, sp2 and sp3 H
2 3
(c) sp, sp , sp and sp 2 (d) sp, sp3, sp2 and sp3 CH3 C CH2 CH3
x
3. The compound in which C uses its sp3 - hybrid orbitals
for bond formation is
X X
(a) (b) (H 2 N)2CO (a) 2 - cyclohexylbutane (b) 2 - phenylbutane
HCOOH
(c) 3 - cyclohexylbutane (d) 3 - phenylbutane
X X
(c) (CH3 )3COH (d) CH 3C HO 10. What is the IUPAC name of the following compound ?
NH 2
Topic 2: Structural Representation and classifiation of
Organic Compounds (a) 2-methyl-4-hexanamine
4. 2- Pentene contains (b) 5-methyl-3-hexanamine
(a) 15 s- and one p- bond (b) 14 s-and one p- bond (c) 2-methyl-4-amino hexane
(c) 15 s- and two p- bonds (d) 14 s- and two p- bonds (d) 5-methyl-3-amino hexane
5. The correct decreasing order of priority of functional 11. Which one of the following is ethyl-4-(dimethyl amino)
groups is butanoate ?
(a) – SO3H , – OH, – COCl, C = C
(b) – COOH, – SO3H, – COOR, – OH (a) (MeNH) 2 OEt
(c) – C º C, – NH2, – OH, C= O
(d) – CN, – CONH2, C = O, – OH
6. Match the columns (b)
Column-I Column-II OEt
NH 2
(A) Non–benzenoid compound (p)
O (c) H2N
OEt
(B) Alicyclic compound (q)

(d) Me 2 N
OEt
ORGANIC CHEMISTRY–SOME BASIC PRINCIPLES & TECHNIQUES 163

12. Identify the correct IUPAC name of the compound given 19. Match Column-I (organic compound) with Column-II
below (common name of the compound) and choose the
correct option.
Column-I Column-II
H (Organic compound) (Common name of
compound)
(A) C6H5OCH3 (p) Neopentane
(B) H3CCH2CH2 OH (q) Anisole
(a) 4 – benzyl – 5 – methyl hexanal (C) (H3C)4C (r) Acetophenone
(b) 2 – methyl – 3 – phenyl hexanal (D) C6H5COCH3 (s) n–Propyl alcohol.
(c) 5 – isopropyl – 5 – phenyl butanal (a) A – (r), B – (s), C – (p), D – (q)
(d) 5 – methyl – 4 – phenyl hexanal (b) A – (r), B – (p), C – (s), D – (q)
13. IUPAC name of (CH3)3 CCl is (c) A – (q), B – (s), C – (p), D – (r)
(a) 1-butyl chloride (d) A – (q), B – (s), C – (r), D – (p)
(b) 3-chloro butane 20. Assertion : IUPAC name of the following organic
(c) 2-chloro-2-methylpropane compound is 3, 4, 7 – trimethyloctane
(d) 2-butyl chloride CH3 - CH - CH 2 - CH 2 - CH - CH - CH 2 - CH3
14. IUPAC name of the following compound | | |
CH3 CH3 CH3
O
Reason : The numbering is done in such a way that the
CH3 branched carbon atoms get the lowest possible numbers.
C N
CH3 (a) Assertion is correct, reason is correct; reason is a
correct explanation for assertion.
(a) N, N-dimethylcyclopropane carboxamide
(b) Assertion is correct, reason is correct; reason is not
(b) N-methylcyclopropanamide' a correct explanation for assertion
(c) cyclopropionamide (c) Assertion is correct, reason is incorrect
(d) None of these (d) Assertion is incorrect, reason is correct.
15. Which of the following is a 3-methylbutyl group? 21. The IUPAC name of the following compound is
(a) CH3CH2CH2CH2CH2– NH
(b) (CH3CH2)2CH–
O
(c) (CH3)3CCH2–
(d) (CH3)2CHCH2CH2– (a) N – phenyl ethanamide
16. The IUPAC name of the compound (b) N – phenyl ethanone
CH3 — CH(CH3) — CO – CH3, is (c) N – phenyl methanamide
(d) None of these
(a) 3-methyl 2-butanone
OH
(b) 2-methyl 3-butanone
(c) isopropyl methyl ketone
22. IUPAC name of is :
(d) methyl isopropyl ketone
17. The IUPAC name of neopentane is OH
(a) 2, 2-dimethylpropane (b) 2-methylpropane (a) But-2 ene-2, 3-diol
(c) 2, 2-dimethylbutane (d) 2-methylbutane (b) Pent-2-ene-2, 3-diol
18. What is the IUPAC name of the following compound ? (c) 2-methylbut-2-ene-2, 3-diol
COOH (d) Hex-2-ene-2, 3-diol
O
Br
CH2– C – OH
OH
23. The IUPAC name of compound C is:
COOH
CH 2 CH 3
CH – COOH
(a) 6 – Bromo – 4 – ethylbenzene carboxylic acid (a) 1, 2, 3-tricarboxy-2, 1-propane 2
(b) 2 – Bromo – 4 – ethylbenzene carboxylic acid (b) 3-carboxy-3 hydroxy-1, 5-pentanedioic acid
(c) Ortho – bromo – paraethyl benzoic acid (c) 3-hydroxy-3-carboxy-1, 5-pentanedioic acid
(d) 4 – Bromo – 3 – ethyl benzoic acid (d) 2-hydroxy propane-1, 2, 3-tricarboxylic acid.
EBD_8350
164 CHEMISTRY

Topic 4: Isomerism 33. Assertion : Chain isomerism is observed in compounds


containining four or more than four carbon atoms
24. Which are isomers ? Reason : Only alkanes show chain isomerism
(a) ethyl alcohol and dimethyl ether (a) Assertion is correct, reason is correct; reason is a
(b) acetone and acetaldehyde correct explanation for assertion.
(c) propionic acid and propanone (b) Assertion is correct, reason is correct; reason is not
(d) methyl alcohol and dimethyl ether
a correct explanation for assertion
25. Methoxyethane and propanol are the examples of
(c) Assertion is correct, reason is incorrect
isomerism of the type
(d) Assertion is incorrect, reason is correct.
(a) structural (b) position
(c) functional (d) tautomerism 34. Which of the following compounds will show metamerism?
26. Isomers of propionic acid are (a) CH3 – CO – C2H5 (b) C2H5 – S – C2H5
(a) HCOOC2H5 and CH3COOCH3 (c) CH3 – O – CH3 (d) CH3 – O – C2H5
(b) HCOOC2H5 and C3H7COOH 35. The compound C4H10O can show
(c) CH3COOCH3 and C3H7OH (a) metamerism
(d) C3H7OH and CH3COCH3 (b) functional isomerism
(c) position isomerism
27. C6H5C º N and C6H5N ® < C are which type of isomers?
(d) All of these
(a) Position (b) Functional 36. Which pair of isomerism is not possible together?
(c) Tautomerism (d) Linkage (a) Ring-chain and functional
28. Which organic structure among the following is not an (b) Geometrical and optical
isomer of the compound
(c) Metamerism and functional
CH3–CO–CH2CH2CH2CH3 ? (d) Metamerism and chain
(a) CH3CH2OCH =CHCH2CH3
Topic 5: Fundamental Concepts in Organic Reaction Mechanism
(b) CH3CH = CHCH2CH2CHO
(c) (CH3)2CH–CO–CH2CH3 37. Which of the following ions is most stable ?
+ +
(d) CH3CH2COCH2CH2CH3 (a) CH 3 - C - CH 3 (b) CH 3 CH 2 CH 2
29. Which of the following compounds is isomeric with 2, 2, |
4, 4- tetramethylhexane? CH 3
(a) 3-ethyl -2, 2- dimethylpentane +
(b) 4-isopropylheptane (c) CH 3CHCH 2 CH 3 (d) None of these
(c) 4-ethyl-3-methyl-4-n-propyloctane
38. The most stable carbocation among the following is
(d) 4, 4-diethyl-3-methylheptane
30. A functional isomer of 1-butyne is + +
(a) C6 H5 CHC6 H5 (b) C6 H5CH 2
(a) 2-butyne (b) 1-butene
(c) 2-butene (d) 1, 3-butadiene + +
31. The compounds CH3CH == CHCH3 and (c) CH3 CH 2 (d) C6 H5CH 2CH 2
CH3CH2CH == CH2 39. The organic reactions which proceed through heterolytic
(a) are tautomers bond cleavage called ________
(b) are position isomers (a) ionic (b) polar
(c) contain same number of sp3– sp3, sp3– sp2 and sp2– sp2 (c) non-polar (d) Both (a) and (b)
carbon-carbon bonds
40. The shape of methyl carbanion is similar to that of –
(d) exist together in dynamic equilibrium
(a) BF3 (b) NH3
32. Match the columns
(c) methyl free radical (d) methyl carbocation
Column-I Column-II
(A) CH3COOH & (p) Functional isomers 41. Homolytic fission of C–C bond in ethane gives an
intermediate in which carbon is
HCOOCH3
(a) sp3-hybridised (b) sp2-hybridised
(B) 1 butene & 2-butene (q) Metamers (c) sp-hybridised (d) sp2d-hybridised
(C) diethyl ether & (r) Position isomers 42. Which of the following statements is not correct ?
methyl propyl ether
(a) Carbocation posses sextet of electrons.
(D) dimethyl ether (s) Chain isomers
(b) The order of carbocation stability is :
and ethanol
(a) A – (p), B – (r), C – (q), D – (p) + + +
(b) A – (q), B – (r), C – (s), D – (p) CH3 > (CH3 )2 CH > (CH3 )3 C
(c) A – (q), B – (s), C – (p), D – (r) (c) Carbocations have trigonal planar shape
(d) A – (q), B – (p), C – (s), D – (r) (d) Carbocations are formed by heterolytic cleavage
ORGANIC CHEMISTRY–SOME BASIC PRINCIPLES & TECHNIQUES 165

43. The order of stability of the following carbocations : 50. Which of the following species does not act as a
Å nucleophile?
CH 2 (a) ROH (b) ROR
Å Å
(c) PCl3 (d) BF3
CH 2 = CH - C H 2 ; CH3 - CH 2 - CH 2 ; is : 51. Which of the following pairs represent electrophiles?
I II (a) AlCl3, H2O (b) SO3, NO2+
III (c) BF3, H2O (d) NH3, SO3
(a) III > II > I (b) II > III > I
(c) I > II > III (d) III > I > II 52. Which out of A, B, C and D is/are not correctly
44. Select the most stable carbocation amongst the following categorised.
+ Nucleophile Electrophile
(a)
A. HS- Cl+
Ph B. BF3 (CH3 )3N
(b) Ph
+ +
C. H 2 N - -C = O
Ph Ph D. R 3C - X C2 H 5 O -
+ (X = H alogen)
(c)
(a) B, C and D (b) C and D
(c) C only (d) B and D
(d) +
53. Polarization of electrons in acrolein may be written as
45. What is the correct order of decreasing stability of the d- d+ d- d+
(a) CH 2 = CH - CH = O (b) CH 2 = CH - CH = O
following cations?
Å d- d- d+ d-
Å (c) CH 2 = CH - CH = O (d) CH 2 = CH - CH = O
I. (CH3 )3 II. CH3 — CH— OCH 3
Å 54. Point out the incorrect statement about resonance?
III. CH3 — CH— CH 2 — CH 2 — OCH3 (a) Resonance structures should have equal energy
(b) In resonating structures, the constituent atoms must
(a) II > I > III (b) II > III > I
be in the same position
(c) III > I > II (d) I > II > III
(c) In resonating structures, there should not be same
46. Arrange the carbanions,
number of electron pairs
(CH3 )3 C , C Cl3 , (CH3 ) 2 CH , C6 H5 CH 2 (d) Resonating structures should differ only in the
in order of their decreasing stability : location of electrons around the constituent atoms
(a) (CH3 )2 C H > C Cl3 > C6 H5 C H 2 > (CH3 )3 C 55. In which of the following, resonance will be possible?
(b) C Cl3 > C6 H5 CH 2 > (CH3 )2 CH > (CH3 )3 C (a) CH3 - CH 2 - CH 2 - CHO

(c) (CH3 )3 C > (CH3 )2 CH > C6 H5 CH 2 > CCl3 (b) CH 2 = CH - CH = O


(d) C6 H5 CH 2 > CCl3 > (CH3 )3 C > (CH3 )2 CH (c) CH 3COCH 3
47. The increasing order of stability of the following free (d) CH 2 = CH - CH 2 - CH = CH 2
radicals is 56. Assertion : Aniline is better nucleophile than anilium ion.
• • • •
(a) (C6H5)2 C H < (C6H5)3 C < (CH3)3 C < (CH3)2 C H Reason : Anilium ion has +ve charge.
• • • • (a) Assertion is correct, reason is correct; reason is a
(b) (CH3)2 C H < (CH3)3 C < (C6H5)2 C H < (C6H5)3C correct explanation for assertion.
• • • •
(c) (CH3)3 C < (CH3)2 C H < (C6H5)2 C H < (C6H5)3 C (b) Assertion is correct, reason is correct; reason is not
• • • • a correct explanation for assertion
(d) (C6H5)3C < (C6H5)2 C H < (CH3)3 C < (CH3)2 C H (c) Assertion is correct, reason is incorrect
48. Which of the following is strongest nucleophile (d) Assertion is incorrect, reason is correct.
(a) Br– (b) : OH– 57. Which of the following is not correctly matched ?
-
(c) : CN (d) C2 H 5O : Group showing + R effect Group showing – R effect
49. Which of the following represents a set of nucleophiles? (a) – NHCOR – COOH
(a) BF3, H2O, NH2– (b) AlCl3, BF3, NH3 (b) C=O – OH
(c) CN–, RCH2–, ROH (d) All of these (c) – OR – CHO
(d) – OCOR – NO2
EBD_8350
166 CHEMISTRY

58. Assertion : Energy of resonance hybrid is equal to the 65. The polarity is produced in the molecule by the
average of energies of all canonical forms. interaction of two p – bonds or between a p – bond and
Reason : Resonance hybrid cannot be presented by a single lone pair of electrons present on an adjacent atom.
structure. The above statement is true for which of the following ?
(a) Assertion is correct, reason is correct; reason is a (a) Inductive effect (b) Electromeric effect
correct explanation for assertion. (c) Resonance effect (d) Hyperconjugation
(b) Assertion is correct, reason is correct; reason is not
a correct explanation for assertion 66. Which of the following represents the correct order of
(c) Assertion is correct, reason is incorrect stability of the given carbocations ?
(d) Assertion is incorrect, reason is correct. + |+ + + |+ +
59. The kind of delocalization involving sigma bond orbitals (a) F3C > F3C - C > CH3 (b) H3C > F3C - C > F3C
is called | |
(a) inductive effect (b) hyperconjugation effect
(c) electromeric effect (d) mesomeric effect |+ + + |+ + +
60. Choose the correct order of stability of carbocation using (c) F3C - C > F3C > H3C (d) F3C - C > H3C > F3C
concept of hyperconjugation. | |
CH3 CH3 67. The most stable carbanion among the following is
|Å |Å Å Å
CH3 - C CH3 - C CH3 CH 2 CH 3 CH – CH –
2 2 CH – 2
| |
CH3 H
I II III IV (a) (b)
(a) I < II < III < IV (b) IV < III < II < I
(c) III < IV < II < I (d) None of these
61. Hyperconjugation is most useful for stabilizing which of CH2– CH2–
the following carbocations ?
(a) neo-Pentyl (b) tert-Butyl
(c) iso-Propyl (d) Ethyl (c) (d)
62. Which of the following is/are correct for inductive effect ?
(i) In inductive effect polarisation of sigma bond is
OCH3 NO2
caused by the adjacent s bond.
(ii) Halogens, – NO2 , – CN, and – CH3 are electron 68. For (i) I –, (ii) Cl – , (iii) Br –, the increasing order of
withdrawing groups. nucleophilicity would be
(iii) –CH2CH3and –OC6H5 are electron donating groups.
(a) Cl– < Br– < I– (b) I– < Cl– < Br–
(a) (i) only (b) (ii) only
(c) (i) and (iii) (d) (i), (ii) and (iii) (c) Br– < Cl– < I– (d) I– < Br– < Cl–
63. Which of the following statements regarding the resonance 69. Which of the following does not represent formation of
energy of benzene is correct? reactive intermediate correctly ?
(a) Resonance energy is the energy required to break the +
C–H bond in benzene
(i) CH3 - CN ® C H3 + CN -
- +
(b) Resonance energy is the energy required to break the (ii) CH 3 - Cu ® CH3 + Cu
C–C bond in benzene +
(c) Resonance energy is a measure of stability of benzene (iii) CH 3 - Br ® CH 3 + Br -
(d) Resonance energy is the energy required to convert +
(iv) CH 3 - Cl ® CH 3 + Cl -
(a) (ii) only (b) (ii) and (iii)
¾¾®
(c) (ii) and (iv) (d) (iii) and (iv)
64. Assertion : Different number of electron pairs are present 70. Decreasing order of stability of following alkenes is
in resonance structures. (i) CH3— CH CH2 (ii) CH3— CH CH— CH3
Reason : Resonance structures differ in the location of
electrons around the constituent atoms. CH3 CH3
(a) Assertion is correct, reason is correct; reason is a (iii) CH3 C CH CH3 (iv) CH3 C C CH3
correct explanation for assertion.
(b) Assertion is correct, reason is correct; reason is not CH 3
a correct explanation for assertion (a) (i) > (ii) > (iii) > (iv) (b) (iv) > (iii) > (ii) > (i)
(c) Assertion is correct, reason is incorrect (c) (iii) > (ii) > (i) > (iv) (d) (ii) > (iii) > (iv) > (i)
(d) Assertion is incorrect, reason is correct.
ORGANIC CHEMISTRY–SOME BASIC PRINCIPLES & TECHNIQUES 167

71. Which of the following is a false statement? 77. Arrange the following carbocations in decreasing order
(a) Free radicals, carbonium ions or carbanions are of stability.
reaction intermediates. + +
(b) Reaction between methane and chlorine in presence CH2 CHCH2 CH2 CH (CH3)3C+
of sunlight proceeds via free radical. I II III
(c) The electronegative atom in the carbon chain + +
produces +I effect. Ph 3C CH2
(d) Homolytic fission of C – C bonds gives free IV V
radicals. (a) IV > III > I > V > II (b) IV > I > III > II > V
72. With respect o benzyl carbocation and benzyl carbanion,
(c) V > IV > I > III > II (d) V > IV > III > I > II
the correct statement is
+ + 78. In a conjugated system resonance effect involves
(a) C6H5CH2 ion is more stable that CH3 ion and
– – (a) delocalization of p-electrons along a conjugated
C6H5CH2 ion is also more stable than CH3 ion system
+ +
(b) C6H5CH2 ion is less stable than CH3 ion and (b) delocalization of non-bonding electrons along a
– – conjugated system
C6H5CH2 ion is more stable than CH3 ion (c) delocalization of s-electrons into an adjacent p-
+
(c) C6H5CH2 ion is more stable than CH3 ion and
+ bond
– –
(d) all of these.
C6H5CH2 ion is less stable than CH3 ion 79. Which of the following series contains atoms/groups
+ +
(d) C6H5CH2 ion is less stable than CH3 ion and having only –M (mesomeric) effect?
– – (a) COR, OR, COOR (b) Cl, CHO, NH2
C6H5CH2 ion is also less stable than CH3 ion.
(c) NO2, CN, SO3H (d) OH, NR2, SR
73. Which of the following resonating structures of
1-methoxy-1, 3-butadiene is least stable? Topic 6: Methods of Purification of Organic Compounds
– +
(a) CH3 – CH == CH – CH == O – CH3 80. The best method for the separation of naphthalene and
+
(b) CH
– 2
== CH – C–H – CH == O – CH3 benzoic acid from their mixture is:
+
(c) CH2 – CH – CH –
== CH – O – CH3 (a) distillation (b) sublimation
+
(d) CH2 == CH – CH – CH – O – CH3 (c) chromatography (d) crystallisation
74. The decreasing order of stability of the below mentioned 81. In steam distillation the vapour pressure of volatile organic
carbocations is compound is
+ (a) equal to atmospheric pressure
CH2
(i) F3CCH2
+
(ii) (b) double the atmospheric pressure
Cl (c) less than atmospheric pressure
(d) more than atmospheric pressure
+ +
(iii) Cl3CCHCH3 (iv) 82. Fractional distillation is used when
(a) there is a large difference in the boiling point of liq-
(a) (iv) > (iii) > (ii) > (i) (b) (iv) > (ii) > (iii) > (i) uids
(c) (iii) > (iv) > (ii) > (i) (d) None of these (b) there is a small difference in the boiling points of
+ liquids
75. (CH3)4N is neither an electrophile, nor a nucleophile
(c) boiling points of liquids are same
because it
(d) liquids form a constant boiling mixture
(a) does not have electron pair for donation as well as
cannot attract electron pair 83. Purification of petroleum is carried out by
(b) neither has electron pair available for donation nor (a) fractional distillation (b) steam distillation
can accommodate electron since all shells of (c) vacuum distillation (d) simple distillation
nitrogen are fully occupied 84. Distillation under reduced pressure is employed for
(c) can act as Lewis acid and base (a) C6H6
(d) None of these (b) petrol
76. Which of the following is true? (c) CH2OHCHOHCH2OH
(a) tert-Butoxide is a stronger base as well as stronger
(d) organic compounds used in medicine
nucleophile than ethoxide.
85. Glycerol can be separated from spent lye in soap
(b) tert-Butoxide is a weaker base but stronger
nucleophile than ethoxide. industry by which of the following method ?
(c) tert-Butoxide is a stronger base, but weaker (a) Steam distillation
nucleophile than ethoxide. (b) Fractional distillation
(d) tert-Butoxide and ethoxide are equally strong bases (c) Distillation under reduced pressure
as well as strong nucleophiles. (d) Differential extraction
EBD_8350
168 CHEMISTRY

86. The latest technique for the purification of organic (ii) A fractionating column provides many surfaces for
compounds is heat exchange between the ascending vapours and
(a) chromatography (b) fractional distillation the descending condensed liquid.
(c) crystallization (d) vacuum distillation (iii) Each successive condensation and vaporisation unit
87. Which of the following is used as an adsorbent in in the fractionating column is called a theoretical
adsorption chromatography ? plate.
(a) Silica gel (b) Alumina (iv) Fractional distillation method is used to separate
(c) Zeolite (d) Both (a) and (b) different fractions of crude oil in petroleum industry.
88. Which of the following acts as the stationary phase in (a) (i), (ii) and (iv)
paper chromatography ? (b) (ii), (iii) and (iv)
(a) Water (b) Alumina (c) (i), (ii) and (iii)
(c) Silica gel (d) None of these (d) (i), (ii), (iii) and (iv)
89. The most satisfactory method to separate mixture of sugars 95. Assertion : Simple distillation can help in separating a
is mixture of propan-1-ol (boiling point 97°C) and propanone
(a) fractional crystallisation (boiling point 56°C).
(b) sublimation Reason : Liquids with a difference of more thatn 20°C in
(c) chromatography their boiling points can be separated by simple distillation.
(d) benedict reagent (a) Assertion is correct, reason is correct; reason is a
90. Chromatography is a valuable method for the separation, correct explanation for assertion.
isolation, purification and identification of the constituents (b) Assertion is correct, reason is correct; reason is not
of a mixture and it is based on general principle of a correct explanation for assertion
(a) phase rule (c) Assertion is correct, reason is incorrect
(b) phase distribution (d) Assertion is incorrect, reason is correct.
96. Assertion : Components of a mixture of red and blue inks
(c) interphase separation
can be separated by distributing the components between
(d) phase operation
stationary and mobile phases in paper chromatography.
91. In paper chromatography
Reason : The coloured components of inks migrate at
(a) moving phase is liquid and stationary phase in solid
different rates because paper selectively retains different
(b) moving phase is liquid and stationary phase is liquid
components according to the difference in their partition
(c) moving phase is solid and stationary phase is solid
between the two phases.
(d) moving phase is solid and stationary phase is liquid
(a) Assertion is correct, reason is correct; reason is a
92. Given below is the developed chromatogram of a mixture correct explanation for assertion.
of pigments. (b) Assertion is correct, reason is correct; reason is not
a correct explanation for assertion
(c) Assertion is correct, reason is incorrect
Thin layer of (d) Assertion is incorrect, reason is correct.
adsorbent 97. The most suitable method for separation of a 1 : 1 mixture of
y z
Sample dot x
ortho and para nitrophenols is
Base line
(a) Sublimation (b) Chromatography
Solvent
(c) Crystallization (d) Steam distillation
Rf values for x and y can be expressed as
Topic 7: Qualitative and Quantitative Analysis of
x y x y Organic Compounds
(a) , (b) ,
z z y z
98. In sodium fusion test of organic compounds, the nitrogen
z z of the organic compound is converted into
(c) xz, yz (d) , (a) sodamide (b) sodium cyanide
x y
(c) sodium nitrite (d) sodium nitrate
93. For the separation of two immiscible liquids which method 99. Which of the following compounds does not show
is used? Lassaigne’s test for nitrogen ?
(a) Chromatography (b) Fractionating column (a) Urea (b) Hydrazine
(c) Fractional distillation (d) Separating funnel (c) Phenylhydrazine (d) Azobenzene
94. Which of the following statements are correct for 100. The compound formed in the positive test for nitrogen
fractional distillation ? with the Lassaigne solution of an organic compound is
(i) Fractional distillation method is used if the two
liquids have sufficiently large difference in their (a) Fe4[Fe(CN)6]3 (b) Na3[Fe(CN)6]
boiling points. (c) Fe(CN)3 (d) Na4[Fe(CN)5NOS]
ORGANIC CHEMISTRY–SOME BASIC PRINCIPLES & TECHNIQUES 169

101. Match the columns 110. 0.25 g of an organic compound on Kjeldahl's analysis gave
Column - I Column - II enough ammonia to just neutralize 10cm3 of 0.5 M H2SO4.
(Elements) (Colour of precipitate The percentage of nitrogen in the compound is
formed in Lassaigne’s test) (a) 28 (b) 56 (c) 14 (d) 112
(A) Nitrogen (p) Yellow 111. The percentage of sulphur in an organic compound whose
(B) Sulphur (q) Prussian blue 0.32 g produces 0.233 g of BaSO4 [At. wt. Ba = 137, S = 32]
(C) Chlorine (r) Violet is
(D) Phosphorus (s) White (a) 1.0 (b) 10.0 (c) 23.5 (d) 32.1
(a) A – (q), B – (r), C – (p), D – (s) 112. 2.79 g of an organic compound when heated in Carius tube
(b) A – (r), B – (q), C – (p), D – (s) with conc. HNO 3 ; H 3PO 4 formed converted into
(c) A – (q), B – (r), C – (s), D – (p) MgNH 4 .PO 4 ppt. The ppt. on heating gave 1.332 g of
(d) A – (r), B – (q), C – (s), D – (p) Mg 2 P2O 7 . The percentage of P in the compound is
102. In Lassaigne’s test, the organic compound is fused with a
(a) 23.33% (b) 13.33%
piece of sodium metal in order to
(c) 33.33% (d) 26.66%
(a) increase the ionisation of the compound 113. In Duma’s method 0.52 g of an organic compound on
(b) decrease the melting point of the compound combustion gave 68.6 mL N2 at 27°C and 756 mm
(c) increase the reactivity of the compound pressure.What is the percentage of nitrogen in the
(d) convert the covalent compound into a mixture of ionic compound?
compounds (a) 12.22% (b) 14.93% (c) 15.84% (d) 16.23%
103. The Lassaigne’s extract is boiled with dil. HNO3 before
testing for halogens because 114. In Carius method of estimation of halogen, 0.15 g of an
organic compound gave 0.12 g of AgBr. What is the
(a) silver halides are soluble in HNO3
percentage of bromine in the compound?
(b) Na2S and NaCN are decomposed by HNO3
(a) 68.08% (b) 34.04% (c) 42.1% (d) 50%
(c) Ag2S is soluble in HNO3
(d) AgCN is soluble is HNO3 115. Match the column I wth column II in which formula for
104. In quantitative analysis of carbon and hydrogen, the estimation of an element is given and mark the appropriate
mass of water produced is determined by passing the choice.
mixture through a weighed U – tube containing ___X___ Column I Column II
and carbon dioxide is absorbed in concentrated solution 80 w1
of ___Y___ (A) Estimation of carbon (i) ´ ´ 100
188 w
(a) X = CaCl2 , Y = NaOH
62 w1
(b) X = Ca(OH)2 , Y = CuSO4 (B) Estimation of nitrogen (ii) ´ ´ 100
222 w
(c) X = CuSO4 , Y = Ca(OH)2
32 w1
(d) X = CaCl2 , Y = KOH (C) Estimation of bromine (iii) ´ ´ 100
233 w
105. Kjeldahl method is not applicable to which of the
28 V
following ? (D) Estimation of sulphur (iv) ´ ´ 100
(a) Nitro compounds (b) Azo compounds 22400 w
(c) Pyridine (d) All of these. 12 w1
(E) Estimation of phosphorus (v) ´ ´ 100
106. Duma's method involves the determination of nitrogen 44 w
content in the organic compound in form of (a) (A) ® (v), (B) ® (ii), (C) ® (iv), (D) ® (i), (E) ® (iii)
(a) NH3 (b) N2
(b) (A) ® (v), (B) ® (iv), (C) ® (i), (D) ® (iii), (E) ® (ii)
(c) NaCN (d) (NH4)2SO4
(c) (A) ® (v), (B) ® (iv), (C) ® (ii), (D) ® (i), (E) ® (iii)
107. In Kjeldahl’s method nitrogen present is estimated as
(d) (A) ® (iv), (B) ® (iii), (C) ® (i), (D) ® (ii), (E) ® (v)
(a) N2 (b) NH3
116. In Kjeldahl’s method for the estimation of N2, potassium
(c) NO2 (d) None of these
sulphate and copper sulphate are used. On the basis of their
108. In Kjeldahl's method of estimation of nitrogen, K2SO4 acts as functions which of the following statement(s) is/are correct?
(a) oxidising agent (b) catalytic agent (i) Potassium sulphate raises the bpt. and ensures
(c) hydrolysing agent (d) boiling point elevator complete reaction.
109. 5.0 g of an organic compound was Kjeldahlished and the (ii) Copper sulphate acts as catalyst.
NH3 evolved was absorbed in 100 mL of 1.0 MH2SO4. The (iii) Potassium sulphate acts as catalyst and copper
residual acid required 120 cm3 of 1.0 M NaOH. The % of sulphate raises the bpt.
nitrogen in the organic compound is (a) Only (iii) is correct (b) (i) and (ii) are correct
(a) 25.4 (b) 22.4 (c) 2.24 (d) 2.54 (c) Only (ii) is correct (d) None is correct
EBD_8350
170 CHEMISTRY

117. Assertion : Sulphur present in an organic compound can (c) gets converted to ammonium sulphate
be estimated quantitatively by Carius method. (d) all of these.
Reason : Sulphur is separated easily from other atoms in 119. Fluorine is not tested by Beilstein’s test because
the molecule and gets precipitated as light yellow solid. (a) it does not react with copper
(a) Assertion is correct, reason is correct; reason is a (b) copper fluoride is not volatile
correct explanation for assertion. (c) F2 is evolved as a gas
(b) Assertion is correct, reason is correct; reason is not
(d) the statement is wrong. It is well tested by Beilstein’s
a correct explanation for assertion
test.
(c) Assertion is correct, reason is incorrect
(d) Assertion is incorrect, reason is correct. 120. 0.20 g of a hydrocarbon on combustion gave 0.66 g CO2.
118. In Duma’s method of nitrogen estimation, nitrogen is The percentage of hydrogen in the hydrocarbon is about
(a) absorbed in KOH (a) 33% (b) 45%
(b) collected in upper part of graduated tube
(c) 10% (d) 90%

1. The molecular formula of diphenyl methane,


CH3
CH2 , is C13H12. OH
O
How many structural isomers are possible when one of
the hydrogens is replaced by a chlorine atom? H3C O O O

O
2. O OH
OCH3 CH3
OH
C OCH3
+
8. From the given compound, how many are optically inactive?
OCH3 C
O CH3

The positive charge of carbocation can be delocalized over (i) (ii)


how many oxygen atoms in the resonating structures? O

3. How many carbocations are possible for molecular formula


C3H5+?
4. The compound (X) has molecular formula C4H7Cl. Find
out the number of its cyclic isomers (structural and O
geometrical only excluding optical isomers).
5. Find out the number of stereoisomers obtained by (iii) (iv) O
bromination of trans-2-butene.
6. From the following compounds/ions, how many are
electrophiles?
+ + O
CH3 , NH 4 , BF3, NH3, NH2–NH2, PCl3, PCl5, SbCl5, GaCl3,
+ NH
AlCl3, F–, CN , CH3– Cl (“C” atom of halide)
(v) (vi) O
7. How many stereocentres are possible for the given O
compound?
ORGANIC CHEMISTRY–SOME BASIC PRINCIPLES & TECHNIQUES 171

Cl Cl
(vii) (viii)
(xi)

Cl Cl

9. How many structural formulae are possible for isomeric


HN
alcohols having the molecular formula C4H10O?
10. How many of the following are not ambiguous?
(ix) NH (x)
Pentane, Neopentane, sec-Butanol, sec-Pentanol

Exercise 3 : NCERT Exemplar & Past Year MCQs


NCERT Exemplar MCQs 5. In which of the following functional groups, isomerism is
not possible?
1. Which of the following is the correct IUPAC name?
(a) Alcohols (b) Aldehydes
(a) 3-Ethyl-4, 4-dimethylheptane
(c) Alkyl halides (d) Cyanides
(b) 4, 4-Dimethyl-3-ethylheptane
6. The fragrance of flowers is due to the presence of some
(c) 5-Ethyl-4, 4-dimethylheptane
steam volatile organic compounds called essential oils. These
(d) 4, 4-Bis(methyl)-3-ethylheptane
are generally insoluble in water at room temperature but are
O O miscible with water vapour in vapour phase. A suitable
|| ||
2. The IUPAC name for CH3 - C - CH 2 - CH 2 - C - OH is method for the extraction of these oils from the flowers is
(a) distillation
(a) 1-hydroxypentane-1, 4-dione
(b) 1, 4-dioxopentanol (b) crystallisation
(c) 1-carboxybutan-3-one (c) distillation under reduced pressure
(d) 4-oxopentanoic acid (d) steam distillation
3. The IUPAC name for 7. During hearing of a court case, the judge suspected that
some changes in the documents had been carried out. He
Cl asked the forensic department to check the ink used at
two different places. According to you which technique
NO2
can give the best results?
(a) Column chromatography
(b) Solvent extraction
(c) Distillation
CH3 (d) Thin layer chromatography
8. The principle involved in paper chromatography is
(a) 1-chloro-2-nitro-4-methylbenzene (a) adsorption (b) partition
(b) 1-chloro-4-methyl-2-nitrobenzene (c) solubility (d) volatility
(c) 2-chloro-1-nitro-5-methylbenzene 9. What is the correct order of decreasing stability of the
(d) m-nitro-p-chlorotoluene following cations?
Å
4. Electronegativity of carbon atoms depends upon their state I. CH3 - C H - CH3
of hybridisation. In which of the following compounds, the Å
carbon marked with asterisk is most electronegative? II. CH3 - C H - OCH3
Å
(a) CH3 – CH2 – *CH2 – CH3
III. CH3 - C H - CH 2 - OCH3
(b) CH3 – *CH = CH – CH3
(a) II > I > III (b) II > III > I
(c) CH3 – CH2 – C º *CH
(c) III > I > II (d) I > II > III
(d) CH3 – CH2 – CH = *CH2
EBD_8350
172 CHEMISTRY

10. Correct IUPAC name for H3C – CH – CH – CH3 is ......... .


| | (b) H + C—
—C
C2H5 C2H5
(a) 2-ethyl-3-methylpentane (c) + —C
H C—
(b) 3, 4-dimethylhexane
(c) 2-sec-butylbutane (d) All of these are possible
(d) 2, 3-dimethylbutane
Past Year MCQs
11. In which of the following compounds the carbon marked
with asterisk is expected to have greatest positive charge? 16. In the Kjeldahl’s method for estimation of nitrogen present
(a) *CH3 — CH2 — Cl (b) *CH3 — CH2 — Mg+Cl– in a soil sample, ammonia evolved from 0.75 g of sample
(c) *CH3 — CH2 — Br (d) *CH3 — CH2 — CH3 neutralized 10 mL of 1 M H2SO4. The percentage of nitrogen
12. Ionic species are stabilised by the dispersal of charge. in the soil is : [JEE M 2014, C]
Which of the following carboxylate ion is the most stable? (a) 37.33 (b) 45.33
(c) 35.33 (d) 43.33
O O
|| || 17. For which of the following molecule significant m ¹ 0?
-
(a) CH3 - C - O (b) Cl - CH 2 - C - O -
[JEE M 2014, C]

O Cl CN
O
|| F ||
(c) F - CH 2 - C - O- (d) CH - C - O-
F (i) (ii)
13. Electrophilic addition reactions proceed in two steps. The
first step involves the addition of an electrophile. Name Cl CN
OH SH
the type of intermediate formed in the first step of the
following addition reaction. H3C – HC = CH2 + H+ ----® ?
(a) 2° carbanion (b) 1° carbocation (iii) (iv)
(c) 2° carbocation (d) 1° carbanion
OH SH
14. Covalent bond can undergo fission in two different ways.
The correct representation involving a heterolytic fission (a) Only (i) (b) (i) and (ii)
of CH3 – Br is (c) Only (iii) (d) (iii) and (iv)
18. Given
Å CH3 CH3
(a) CH3 Br ¾ ® CH3 + Br CH3
CH3 CH3
r
® CH 3 + Br s
(b) CH 3 - Br ¾ ¾ CH3
O O O
(I) (II) (III)
(c) CH3 Br CH3 + BrÅ Which of the given compounds can exhibit tautomerism?
[AIPMT 2015, A]
Å (a) I and III (b) II and III
(d) CH3 Br CH3 + Br (c) I, II and III (d) I and II
15. The addition of HCl to an alkene proceeds in two steps. 19. Which of the following statements is not correct for a
nucleophile? [AIPMT 2015 RS, C]
The first step is the attack of H+ ion to —C
C—
(a) Nucleophile is a Lewis acid
portion which can be shown as (b) Ammonia is a nucleophile
(c) Nucleophiles attack on less e– density sites
(a) +
C— (d) Nucleophiles are not electron seeking.
H —C
ORGANIC CHEMISTRY–SOME BASIC PRINCIPLES & TECHNIQUES 173

20. Consider the following compounds [AIPMT 2015, S] (c) 3-keto-2-methylhex-5-enal


CH3 Ph (d) 3-keto-2-methylhex-4-enal
CH3—C—CH— Ph—C—Ph 27. The correct statement regarding electrophile is:-
CH3 [NEET 2017, C]
CH3
(a) Electrophile is a negatively charged species and can
(I) (II) (III)
Hyperconjugation occurs in : form a bond by accepting a pair of electrons from
(a) II only (b) III only (c) I and III (d) I only another electrophile
21. Which of the following is the most correct electron (b) Electrophiles are generally neutral species and can
displacement for a nucleophilic reaction to take place? form a bond by accepting a pair of electrons from a
[AIPMT 2015, S] nucleophile
(c) Electrophile can be either neutral or positively charged
H H2 H H2 species and can form a bond by accepting a pair of
(a) H3C—C = C – C – Cl (b) H 3C—C = C – C – Cl
H H electrons from a nucleophile
(d) Electrophile is a negatively charged species and can
H H2 H H2
(c) H 3C—C = C – C – Cl (d) H 3C—C = C – C – Cl form a bond by accepting a pair of electrons from a
H H nucleophile
22. In Duma's method for estimation of nitrogen, 0.25 g of an 28. Which of the following molecules is least resonance
organic compound gave 40 mL of nitrogen collected at 300
stabilized? [JEE M 2017, S]
K temperature and 725 mm pressure. If the aqueous tension
at 300 K is 25 mm, the percentage of nitrogen in the
compound is : [AIPMT 2015, A]
(a) (b)
(a) 18.20 (b) 16.76 (c) 15.76 (d) 17.36 O
23. Which of the following compounds will exhibit geometrical
isomerism ? [JEE M 2015, S]
(a) 2-Phenyl-1-butene (c) (d)
N O
(b) 1, 1-Diphenyl-1-propene
(c) 1-Phenyl-2-butene
29. Which of the following molecules represents the order of
(d) 3-Phenyl-1-butene
hybridisation sp2, sp2, sp, sp from left to right atoms?
24. The pair of electron in the given carbanion, CH3C º C- , is
[NEET 2018, A]
present in which of the following orbitals ? (a) HC º C – C º CH
[NEET 2016, C] (b) CH2 = CH – C º CH
(a) 2p (b) sp3 (c) sp2 (d) sp (c) CH3 – CH = CH – CH3
25. The distillation technique most suited for separating
glycerol from pent-l-ye in the soap industry is : (d) CH2 = CH – CH = CH2
[JEE M 2016, A] 30. Which of the following carbocations is expected to be
(a) Steam distillation. most stable? [NEET 2018, A]
(b) Distillation under reduced pressure.
NO2 NO2
(c) Simple distillation
Å
(d) Fractional distillation
(a) Å
(c)
26. The IUPAC name of the compound is : [NEET 2017, S]
O Y H Y H
O
H C NO2
NO2
H Å
(a) 5-formylhex-2-en-3-one (c) Y (d) H
Y
(b) 5-methyl-4-oxohex-2-en-5-al Å
EBD_8350
174 CHEMISTRY

31. Which of the following is correct with respect to – I effect (c) Hyperconjugation
of the substituents? (R = alkyl) [NEET 2018, C] (d) – I effect of – CH3 groups
(a) – NH2 < – OR < – F 34. Paper chromatography is an example of [NEET 2020, C]
(b) – NR2 < – OR < – F (a) Partition chromatography
(b) Thin layer chromatography
(c) – NR2 > – OR > – F
(c) Column chromatography
(d) – NH2 > – OR > – F
(d) Adsorption chromatography
32. The correct IUPAC name of the following compound is: 35. A solution of m-chloroaniline, m-chlorophenol and m-
NO [JEE M 2019 A] chlorobenzoic acid in ethyl acetate was extracted initially
2
with a saturated solution of NaHCO3 to give fraction A.
The left over organic phase was extracted with dilute NaOH
Cl solution to give fraction B. The final organic layer was
CH3 labelled as fraction C. Fractions A, B and C, contain
(a) 5-chloro-4-methyl-1-nitrobenzene respectively: [JEE M 2020, S]
(b) 2-chloro-1-methyl-4-nitrobenzene (a) m-chlorobenzoic acid, m-chloroaniline and m-chlorophenol
(c) 3-chloro-4-methyl-1-nitrobenzene (b) m-chlorobenzoic acid, m-chlorophenol and
(d) 2-methyl-5-nitro-1-chlorobenzene m-chloroaniline
33. A tertiary butyl carbocation is more stable than a (c) m-chlorophenol, m-chlorobenzoic acid and
secondary butyl carbocation because of which of the m-chloroaniline
following ? [NEET 2020, C] (d) m-chloroaniline, m-chlorobenzoic acid and
(a) + R effect of – CH3 groups m-chlorophenol
(b) – R effect of – CH3 groups

Exercise 4 : Problem Solving Skill Enhancer MCQs


3. Total number of a-hydrogen in given compound is:
OH
1. The IUPAC name of
O

(a) 2, 6-Dimethylhepta-2, 5-dienoic acid


(b) 3, 7-Dimethylhepta-2, 5-dienoic acid (a) 4 (b) 5 (c) 6 (d) 7
(c) 1-Hydroxy-2, 6-dimethylhepta-2, 5-dienone
(d) none of these
2. How many number of compounds, which have same
4.
IUPAC name?
OH OH
OH (1) (2)

OH
OH
OH (3) (4)
Choose the statement that best describes given compounds.
OH OH (a) 1, 3, 4 represent same compound
(a) 0 (b) 1 (b) 1 and 3 are isomer of 2 and 4
(c) 2 (d) 3 (c) 1, 4 are isomer of 2 and 3
(d) All the structure represent the same compound
ORGANIC CHEMISTRY–SOME BASIC PRINCIPLES & TECHNIQUES 175

5. The structure : (b) 4-Chloromethyl-1-methylamono-1-oxohent-4-en-3one


CH 3 H (c) N-Methyl-2-(2'-chloroethyl)-3-keto-4-pentenamide
C=C H
CH 3 C shows (d) 4-Chloroethyl-3-keto-5-methylamidopentene
COOH 11. The rate of the reaction
CH 3
(a) tautomerism
(b) geometrical isomerism R CH2 Br + N ¾®
(c) optical isomerism
(d) geometrical and optical isomerism
6. Most stable carbocation among the following is:
R CH2 N Br–
CH3 +
(a) + (b)
is influenced by the hyperconjugation effect of group R. If
R sequentially is
+ +
I. CH–3 II. CH3 – CH2 –
(c) CH2 (d) CH3
CH3
+
O O III. H3C CH IV. H3C C
O +
7. CH 3 CH3
+
(I) (II) (III) the increasing order of speed of the above reaction is
The most stable canonical structure among the given (a) IV, III, II, I (b) I, II, III, IV
structure is : (c) I, IV, III, II (d) III, II, I, IV
(a) I (b) II 12. In the estimation of carbon and hydrogen by combustion
(c) III (d) All are equally stable method which of the following is/are correct?
8. The correct arrangement for leaving tendency among the (i) A spiral of copper is introduced at the right extreme of
following groups is combustion tube if the organic compound contains
nitrogen.
– (ii) A spiral of silver is introduced if the organic compound
F- NH -2 H 2 O H3C SO2O F3CSO 2 O -
I II III V contains halogens.
IV (iii) The copper oxide in the combustion tube is replaced
by lead chromate if the organic compound contains
(a) III > I > II > IV > V (b) V > IV > III > I > II
sulphur.
(c) I > III > II > IV > V (d) V > IV > I > III > II
(a) (i) and (ii) are correct (b) (i) and (iii) are correct
9. 2, 4, 6- Trinitroiodobenzene has three C–N bonds
(b) (ii) and (iii) are correct (d) All are correct
I
13. Most stable carbocation among the following is
O2N a b NO2

labelled as a, b and c. The correct bond O O


O– OH O– OH
c R R
Å
NO2 Å
length of the three bonds is (a) N OH (b) N OH
(a) a = b = c (b) a > b > c H H
(c) a = b > c (d) a = b < c

O NHCH3 O O
O– OH O–
Å
Å R R
10. IUPAC name Cl is (c) (d)
O N OH N OH
H H
(a) 4- Acetamido-6-chloropexen-3-one
EBD_8350
176 CHEMISTRY

14. Select the most stable carbocation among the following 15. Out of I, II, III, IV, equivalent resonating structures will be
NH2 NH2 obtained in :
Å Å OV V
Å O O
2– V
(I) (II) (III) (IV) CO3 R–SO3
(a) I and II V
O O
(b) I, II and IV (I) (II) (III) (IV)
(a) I, III (b) I, III, IV
(c) I and III
(c) I, II and III (d) All of these
(d) I and IV

ANSW ER KEY
Exercise 1 : NCERT Based Topic-wise MCQs
1 (c) 13 (c) 25 (c) 37 (a) 49 (c) 61 (b) 73 (c) 85 (c) 97 (d) 109 (b)
2 (d) 14 (a) 26 (a) 38 (a) 50 (d) 62 (a) 74 (b) 86 (a) 98 (b) 110 (b)
3 (c) 15 (d) 27 (b) 39 (d) 51 (b) 63 (c) 75 (b) 87 (d) 99 (b) 111 (b)
4 (b) 16 (a) 28 (b) 40 (b) 52 (d) 64 (d) 76 (c) 88 (a) 100 (a) 112 (b)
5 (b) 17 (a) 29 (b) 41 (b) 53 (d) 65 (c) 77 (c) 89 (c) 101 (c) 113 (b)
6 (b) 18 (b) 30 (d) 42 (b) 54 (c) 66 (b) 78 (d) 90 (b) 102 (d) 114 (b)
7 (d) 19 (c) 31 (b) 43 (d) 55 (b) 67 (d) 79 (c) 91 (b) 103 (b) 115 (b)
8 (d) 20 (d) 32 (a) 44 (b) 56 (a) 68 (a) 80 (b) 92 (a) 104 (d) 116 (b)
9 (b) 21 (a) 33 (c) 45 (a) 57 (b) 69 (d) 81 (d) 93 (d) 105 (d) 117 (c)
10 (b) 22 (b) 34 (b) 46 (b) 58 (d) 70 (b) 82 (b) 94 (b) 106 (b) 118 (b)
11 (d) 23 (d) 35 (d) 47 (b) 59 (b) 71 (c) 83 (a) 95 (a) 107 (b) 119 (b)
12 (d) 24 (a) 36 (c) 48 (c) 60 (b) 72 (a) 84 (c) 96 (a) 108 (d) 120 (c)
Exercise 2 : Numeric/Integer Answer Questions
1 (4) 2 (2) 3 (3) 4 (5) 5 (1) 6 (8) 7 (6) 8 (5) 9 (4) 10 (3)
Exercise 3 : NCERT Exemplar & Past Year MCQs
1 (a) 5 (c) 9 (a) 13 (c) 17 (d) 21 (b) 25 (b) 29 (b) 33 (c)
2 (d) 6 (d) 10 (b) 14 (b) 18 (c) 22 (b) 26 (d) 30 (d) 34 (a)
3 (b) 7 (d) 11 (a) 15 (b) 19 (a) 23 (c) 27 (c) 31 (a) 35 (b)
4 (c) 8 (b) 12 (d) 16 (a) 20 (b) 24 (d) 28 (d) 32 (b)
Exercise 4 : Problem Solving Skill Enhancer MCQs
1 (a) 3 (c) 5 (c) 7 (c) 9 (c) 11 (a) 13 (a) 15 (b)
2 (a) 4 (a) 6 (a) 8 (b) 10 (c) 12 (d) 14 (c)
13 Hydrocarbons

Trend Buster NEET & JEE Main

Number of Questions from 2020-15 16 7 Minimum two questions have been


Weightage 5.0% 5.6% asked every year in NEET & JEE M.

The most Important Concepts that Cover Maximum number of Questions asked in past 6 years.

Chemical properties of alkene and alkynes 8 4


Ozonolysis 3 1
Conformation of ethane 2 —

Less Important Concepts that Cover 1 or 2 Questions asked in past 6 years.

Stereochemical aspect of addition reaction — 1


Chemical properties of benzene 1 1
Physical properties of alkane/alkene/alkyne 2 —

NEET JEE

2020 Alkene Ozonolysis 1 Average — —


2019 Aromatic hydrocarbons / aromatic electrophillic 2 Average 2 Average /
alkene / alkane / alkyne substitution / addition reaction / difficult
ozonolysis / reduction of alkyne
2018 alkyne reduction of alkynes 1 Average 1 Average

2017 alkene / alkyne / alkane stereochemical aspect of addition 3 Average / 1 Average


reaction/order of acidity of alkyne / easy
addition of water to alkyne /
conformation of ethane
2016 alkane / alkene / alkyne conformation of ethane / addition 2 Difficult 2 Average
reaction/conversion : acid / base
reaction / substitution reaction
2015 alkene / aromatic addition reaction / oxidation of 7 Average 1 Average
hydrocarbon benzene / enthalpy of
hydrogenation / structure of
alkene / ozonolysis
EBD_8350
178 CHEMISTRY
HYDROCARBONS 179
EBD_8350
180 CHEMISTRY

Problem Solving Tips/ Tricks/ Points to Remember

4 The lower the DHh (heat of hydrogenation), the more 4 Terminal alkynes have acidic H-atoms, so they form metal
stable the alkene is. alkynides with Na, ammonical cuprous chloride solution
4 Alkynes add water molecule in presence of mercuric and ammonical silver nitrate solution.
sulphate and dil. H2SO4 and form carbonyl compounds. 4 All m-directing groups are ring deactivating groups. They
+
are : – CHO, – COOH, – NO2, – CN, – NR3 , etc.
4 Conformations of n-Butane C4H10 : 4 Hydroxylation: (Cis and Trans)
Order of stability : · With 1% alkaline KMnO4(Baeyer reagent) :
Anti > Gauche > Skew > Eclipsed > Fully Eclipsed CH3 – CH = CH – CH3 + H2O + (O)
4 Hydroboration: Diborane readily reacts with alkenes
1% KMnO 4
giving trialkylboranes. Terminal alkenes give primary ¾¾¾¾¾ ® CH3 – CH – CH – CH3
| |
alkylborane, which can be oxidised by alkaline hydrogen
OH OH
peroxide to primary alcohol.
cis-Glycol
3R–CH= CH2 + BH3(B2H6) · With per acid (RCOOOH) :
H3O+
3RCH2CH3 O
P
Alkane
R – CH = CH2 + H – C – O – OH

OH
(R – CH2 – CH2)3B 3RCH2CH2OH Performic acid
1° Alcohol O OH
|
ClNH2 – HCOOH H 2O
¾¾¾ ¾¾
® R CH CH2 ¾¾¾ ® R – CH – CH 2
3RCH2CH2NH2 |
1-Alkane amine OH
Epoxide trans-Glycol

4 Some important electrophiles used in aromatic substitution are :


Electrophile Source Name of reaction
+
Cl (Chloronium) Cl2 + AlCl3 or FeCl3 Chlorination
Br+ (Bromonium) Br2 + AlBr3 or FeBr3 Bromination
+
NO 2 (Nitronium) Conc. HNO3 + Conc.H2SO4 Nitration
SO3 (Sulphur trioxide) Conc. H2SO4, Sulphonation
Fuming sulphuric acid
R+ (Alkyl carbocation) RX + AlX3 (X = Cl or Br) Friedel – Craft’s alkylation
+
R - C = O (Acyl carbocation) R – COCl + AlCl3 Friedel–Craft’s acylation
HYDROCARBONS 181

Exercise 1 : NCERT Based Topic-wise MCQs


Topic 1: Alkanes 12. Liquid hydrocarbons can be converted to a mixture of
gaseous hydrocarbons by :
1. The number of chain isomers possible for the hydrocarbon
(a) oxidation
C5H12 is
(b) cracking
(a) 1 (b) 2 (c) 3 (d) 4 (c) distillation under reduced pressure
2. Name of the given compound - (d) hydrolysis
CH3 13. n-Hexane isomerises in presence of anhydrous aluminium
CH3 chloride and hydrogen chloride gas to give
CH3 (a) 2-Methyl pentane (b) 3-Methyl pentane
(c) Both (a) and (b) (d) Neither (a) nor (b)
CH3 14. How many conformations are possible for ethane ?
(a) 2, 3-diethyl heptane (a) 2 (b) 3 (c) infinite (d) one
(b) 5-ethyl-6-methyl octane 15. Spatial arrangements of atoms which can be converted
(c) 4-ethyl-3-methyl octane into one another by rotation around a C–C single bond are
(d) 3-methyl-4-ethyl octane called
3. Which of the following statements is false for (a) Stereoisomers (b) Tautomers
isopentane? (c) Optical isomers (d) Conformers
(a) It has three CH3 groups 16. Which of the following statements are correct regarding
(b) It has one CH2 group structure of methane ?
(c) It has one CH group (i) Methane has tetrahedral structure.
(d) It has a carbon which is not bonded to hydrogen (ii) The bond angle between all H – C – H bonds is 109.5°.
4. Molecular formula of which of the following alkane can (iii) The carbon atom is sp2 hybridized.
exist in more than one structure ? (iv) C – C and C – H bond lengths are 154 pm and 112 pm
(a) CH4 (b) C2H6 (c) C3H8 (d) C4H10 respectively.
5. The reaction, (a) (i), (ii) and (iii) (b) (i), (iii) and (iv)
CH3 – Br + 2Na + Br –CH3 ® the product, is called (c) (i), (ii) and (iv) (d) (i), (ii), (iii) and (iv)
(a) Wurtz reaction (b) Perkin’s reaction
17. In the preparation of alkanes from hydrogenation of alkenes
(c) Aldol condensation (d) Levit reaction
and alkynes, finely divided catalysts are used, which of
6. Pure methane can be produced by
the following statement(s) is/are correct regarding these
(a) Wurtz reaction
catalysts
(b) Kolbe’s electrolytic method
(i) Platinum and palladium catalyse the reaction at room
(c) Soda-lime decarboxylation
temperature.
(d) Reduction with H2
(ii) Nickel catalyses the reaction at relatively higher
7. Sodium salts of carboxylic acids on heating with soda
lime give alkanes containing _______ than the carboxylic temperature and pressure.
acid. (iii) Platinum and palladium catalyse the reaction at higher
(a) one carbon more (b) one carbon less temperature.
(c) two carbon less (d) Either (a) or (b) (a) (i) and (iii) (b) (i) and (ii)
8. In the free radical chlorination of methane, the chain (c) (ii) and (iii) (d) (i) only
initiating step involves the formation of 18. Which of the following statements are correct ?
(a) chlorine free radical (b) hydrogen chloride (i) The rate of reactivity of alkanes with halogens is
(c) methyl radical (d) chloromethyl radical. F2 > Cl2 > Br2 > I2.
9. Which one of the following gives only one monochloro (ii) Rate of replacement of hydrogens of alkanes is
derivative? 3° > 2º > 1°
(a) n-Hexane (b) 2-Methylpentane (iii) Fluorination of alkanes is a very slow process.
(c) 2, 3-Dimethylpentane (d) neo-Pentane (iv) Iodination of alkanes is too violent to be controlled.
(a) (i), (ii) and (iii) (b) (i) and (ii)
10. Photochemical halogenation of alkane is an example of
(c) (ii) and (iii) (d) (i) and (iv)
(a) electrophilic substitution 19. Which of the following statements are correct ?
(b) electrophilic addition (i) Decomposition reaction of higher alkanes into
(c) nucleophilic substitution smaller fragments by the application of heat is called
(d) free radical substitution pyrolysis.
11. Complete combustion of CH4 gives : (ii) Pyrolysis and cracking are different processes.
(a) CO2 + H2O (b) CO2 + H2 (iii) Dodecane on pyrolysis gives a mixture of heptane
(c) COCl2 (d) CO + CO2 + H2O and pentene.
EBD_8350
182 CHEMISTRY

(iv) Pyrolysis follows free radical mechanism. 25. Assertion : Iodination of alkanes is reversible.
(a) (i), (ii) and (iii) (b) (i), (ii) and (iv) Reason : Iodination is carried out in presence of iodic
(c) (i), (iii) and (iv) (d) (ii) and (iv) acid.
20. Which of the following statement(s) is/are correct ? (a) Assertion is correct, reason is correct; reason is a
(i) Alkanes can have infinite number of conformations correct explanation for assertion.
by rotation around a C – C single bonds. (b) Assertion is correct, reason is correct; reason is
(ii) Rotation around C – C single bond is completely free. not a correct explanation for assertion
(iii) Rotation is hindered by a small energy barrier of (c) Assertion is correct, reason is incorrect
1-20 kJ mol–1 due to torsional strain. (d) Assertion is incorrect, reason is correct.
(a) (i) and (ii) (b) (i) and (iii) 26. How many moles of O 2 are required for complete
(c) (ii) and (iii) (d) Only (iii) combustion of one mole of iso pentane?
21. Which of the following statements are correct ? (a) 5 (b) 6 (c) 7 (d) 8
(i) Stability of conformation is affected due to torsional
AlCl3 KMnO4
strain. 27. n–Butane [X] [Y], [Y] is
HCl
(ii) Magnitude of torsional strain depends upon the angle
(a) Primary alcohol (b) Secondary alcohol
of rotation about C – C bond.
(c) Tertiary alcohol (d) Di ol
(iii) Eclipsed form has least torsional strain.
(iv) Staggered form has maximum torsional strain. 28. 1, 1-Dimethyl cyclo butane can be obtained by
(a) (i) and (iii) (b) (i) and (ii)
(c) (iii) and (iv) (d) (i) and (iv) COOH NaOH
(a) Current
22. Match the columns COOH
Column-I Column-II
Cl
(A) CH2 = CH2 ® CH3 – CH3 (p) H2, Zn, H+
(B) CH3Cl ® CH4 (q) NaOH, CaO Zn
(b) D
(C) CH3Br ® CH3CH3 (r) H2, Pt/Pd
Cl
(D) CH3COONa ® CH4 (s) Na, dry ether
(a) A – (r), B – (p), C – (s), D – (q) COOH
NaOH
(b) A – (p), B – (s), C – (r), D – (q) (c) CaO
(c) A – (s), B – (q), C – (p), D – (r) COOH heat

(d) A – (q), B – (p), C – (s), D – (r) (d) Both in (b) and (c)
23. Match the columns 29. CH3CH2COOH ¾® CH4
Column-I Column-II For this conversion sequence of reagents required are
(a) NaOH / CaO / D
Cu/523K/100 atm
(A) CH4 + O2¾¾¾¾¾¾¾ (p) HCHO
® (b) NaOH / CaO / D, KMnO4, NaOH / CaO / D
Mo O (c) NaOH / CaO / D, (CH3COO)2 Mn, NaOH / CaO / D
(B) CH4 + O2 ¾¾¾¾
2 3®
(q) (CH3)3COH
D (d) NaOH / CaO / D, AlCl3 / HCl, KMnO4, NaOH / CaO / D
(CH COO) Mn 30. Which of the following compound can be best prepared
(C) C2H6 + O2 ¾¾¾¾¾¾¾
3
D
2 ® (r) CH3OH by Wurtz reaction?
KMnO
(D) (CH3)3CH ¾¾ ¾ ¾® 4
(s) CH3COOH
oxidation (a) (b)
(a) A – (s), B – (p), C – (r), D – (s)
(b) A – (q), B – (p), C – (s), D – (r)
(c) (d)
(c) A – (r), B – (p), C – (s), D – (q)
(d) A – (p), B – (q), C – (r), D – (s) 31. Few reactions of alkanes are given below. Identify the name
24. Assertion : CH4 does not react with Cl2 in dark. of the reaction which is not correctly matched with the
Reason : Chlorination of CH4 takes place in sunlight. reaction.
CH3
(a) Assertion is correct, reason is correct; reason is a
AlCl + HCl
correct explanation for assertion. (a) CH3CH2CH2CH3 ¾¾¾¾¾
3
® CH3 CH CH3
(b) Assertion is correct, reason is correct; reason is not - Isomerisation
a correct explanation for assertion 773 K
(b) C6H14 ¾¾¾® C4H8 + C2H6 - Pyrolysis
(c) Assertion is correct, reason is incorrect
D
(d) Assertion is incorrect, reason is correct. (c) CH4 + 2O2 ¾¾
® CO2 + 2H2O - Controlled oxidation
400°C
(d) CH4 + HNO3 ¾¾¾¾
® CH3NO2 - Nitration
HYDROCARBONS 183

32. Complete the following reactions: (b) are position isomers


Zn - Cu (c) contain same number of sp3 – sp3, sp3 – sp2 and sp2
(i) C2H5I + H2 ¾¾¾¾®
C H OH X – sp2 carbon-carbon bonds
2 5

Electrolysis
(d) exist together in dynamic equilibrium
(ii) CH3COONa + H2O ¾¾¾¾¾
®Y 39. Consider the following statements : A hydrocarbon of
Cr O /Al O molecular formula C5H10 is a
(iii) CH3CH2CH2CH2CH2CH3 ¾¾¾¾¾¾ ®Z
2 3 2 3
773 K I. monosubstituted alkene
(a) X = C2H6, Y = C2H6, Z = C6H6 II. disubstituted alkene
(b) X = CH4, Y = CH3COOH, Z = CH3CH3 III. trisubstituted alkene
(c) X = C2H6,Y = CH4, Z = C4H10 Which of the following statement(s) is(are) correct?
(d) X = C2H6, Y = CH4, Z = C5H10 (a) I, II and III (b) I and II
33. In the given reactions : (c) II and III (d) I and III
Na, ether
Br2 Na, ether 40. The number of possible open chain (acyclic) isomeric
(i) CH3Br ¾¾¾¾® X ¾¾¾
hu
® Y ¾¾¾¾® Z compounds for molecular formula C5H10 would be
(a) 8 (b) 7 (c) 6 (d) 5
NaOH
NaOH Br2
(ii) CH3COOH ¾¾¾¾
® X ¾¾¾¾
CaO
® Y ¾¾¾
hu
® Z 41. Geometrical isomerism is not shown by
(a) (i) CH4 CH3Br CH3CH3 CH3
(ii) CH3COONa CH3CH3 CH3CH2Br |
(b) (i) CH3CH3 CH4 CH3Br (a) CH3CH 2C= C CH 2CH 3
|
(ii) CH3COONa CH4 CH3CH2CH3
CH3
(c) (i) CH3CH2Br CH3CH3 CH3CH2CH3
(ii) CH3COONa CH4 CH3Br
(b) C 2 H 5 - C = C - CH 2 I
(d) (i) CH3CH3 CH3CH2Br CH3CH2CH2CH3 | |
(ii) CH3COONa CH4 CH3Br H H
Topic 2: Alkenes (c) CH 2 = C(Cl)CH3
34. The restricted rotation about carbon-carbon double (d) CH 3 - CH = CH - CH = CH 2
bond in 2- butene is due to
42. By which reaction ethene is obtained from ethyne?
(a) overlap of one s- and one sp2-hybridized orbitals
(a) oxidation (b) polymerisation
(b) overlap of two sp2-hybridized orbitals (c) hydrogenation (d) dehydrogenation
(c) overlap of one p-and one sp2-hybridized orbitals 43. The major product formed when 2-bromobutane is
(d) sideways overlap of two p-orbitals treated with alcoholic KOH is
35. IUPAC name of the following compound will be (a) 2-Butanol (b) 1-Butene
(c) 1-Butanol (d) trans-2-butene
CH3 - CH = C - CH 2 - CH3
| 44. Ethyl alcohol is heated with conc. H 2SO4 . The product
CH 2 - CH 2 - CH3 formed is :
(a) 3–ethyl–2–hexene (b) 3–propyl–2–hexene
(a) H3C - C - OC2 H5 (b) C2 H 6
(c) 3–propyl–3–hexene (d) 4–ethyl–4–hexene ||
36. The name of ClCH 2 - C = C - CH 2 Cl according to O
| |
Br Br (c) C2 H 4 (d) C2 H 2
IUPAC nomenclature system is 45. When hydrochloric acid gas is treated with propene in
(a) 2, 3- dibromo -1, 4- dichlorobutene-2 presence of benzoyl peroxide, it gives
(b) 1, 4- dichloro-2, 3- dibromobutene-2 (a) 2-Chloropropane (b) Allyl chloride
(c) dichlorodibromobutene (c) No reaction (d) n-Propyl chloride.
(d) dichlorodibromobutane 46. "The addition of unsymmetrical reagents to unsymmetrical
37. Which one of the following exhibits geometrical isomerism? alkenes occurs in such a way that the negative part of the
(a) 1, 2-Dibromopropene (b) 2, 3-Dimethylbut-2-ene addendum goes to that carbon atom of the double bond
which carries lesser number of hydrogen atoms" is called
(c) 2, 3-Dibromobut-2-ene (d) Both (a) and (c)
by :
38. The compounds CH3CH == CHCH3 and
(a) Saytzeff rule (b) Markovnikov's rule
CH3CH2CH == CH2
(c) Kharasch effect (d) Anti-Saytzeff rule
(a) are tautomers
EBD_8350
184 CHEMISTRY

47. When one mole of an alkene on ozonolysis produces 2 (a) 2-butene (b) 1-butene
moles of propanone, the alkene is (c) propene (d) 1-hexene
(a) 3-methyl-1-butene (b) 2, 3-dimethyl-1-butene 56. Ethylene reacts with alkaline KMnO4 to form
(c) 2, 3-dimethyl-2-pentene (d) 2, 3-dimethyl-2-butene (a) Oxalic acid (b) HCHO
48. Reaction of HBr with propene in the presence of peroxide (c) Ethyl alcohol (d) Glycol
gives 57. The reaction of HI with CH3 – CH = CH2 at 400 °C yields :
(a) isopropyl bromide (b) 3-bromo propane (a) CH2 I – CH = CH2 (b) CH3 – CHI – CH3
(c) allyl bromide (d) n-propyl bromide (c) CH3 – CH2 – CH2 I (d) CH2I – CH2 – CH2I
58. Ethene when treated with Br2 in the presence of CCl4 which
49. H 3 C - CH - CH = CH 2 + HBr ® A
| compound is formed?
CH 3 (a) 1, 2-Dibromoethane (b) 1-Bromo-2-chloroethane
A(predominantly) is : (c) Both (a) and (b) (d) 1, 1, 1-Tribromoethane
(a) CH 3 - CH - CH 2 - CH 2 Br 59. Isopropyl alcohol is obtained by reacting which of the
| following alkenes with conc. H2SO4 and H2O
CH 3 (a) Ethylene (b) Propylene
(c) 2-methylpropene (d) Isoprene
Br 60. Which of the following statements are correct ?
|
(b) CH3 - C- CH 2 - CH3 (i) The general formula of alkenes is CnH2n.
| (ii) Alkenes are also known as paraffins.
CH3 (iii) Bond length of C–C double bond in alkene is shorter
than C–C single bond in alkane.
(c) CH3 - CH- CH - CH 3 (iv) Carbon–Carbon double bond in alkene consists of
| | two sigma bonds.
Br CH3 (v) Alkenes are easily attacked by electrophilic reagent.
(d) CH 3 - CH - CH - CH 3 (a) (i) and (iv) (b) (i), (iii) and (v)
| | (c) (i) and (iii) (d) (i), (ii), (iv) and (v)
CH 3 Br 61. Which of the following statements are correct ?
50. Butene-1 may be converted to butane by reaction with (i) Cis form of alkene is polar whereas trans form is
(a) Sn – HCl (b) Zn – Hg non-polar
(c) Pd/H2 (d) Zn – HCl (ii) Cis form of alkene is non-polar whereas trans form is
51. A reagent used to test unsaturation in alkene is - polar.
(a) ammonical Cu2Cl2 (b) ammonical AgNO3 (iii) In case of solid alkenes the trans isomer has higher
(c) solution of Br2 in CCl4 (d) conc. H2SO4 melting point than the cis isomer.
52. In the given reaction (iv) Cis and trans both form have same properties.
X (a) (i) and (iii) (b) (ii) and (iii)
CH 3CH 2 CH = CHCH 3 ¾¾® (c) (i), (iii) and (iv) (d) (i) and (iv)
CH3CH 2COOH +CH3COOH 62. Match the columns
Column-I Column-II
The X is
(A) CH º CH + H2 ¾® CH2 = CH2 (p) Zn
(a) C2 H5ONa (b) Conc. HCl +Anhy.ZnCl2
(B) CH3CH2Br ¾® CH2 = CH2 (q) Conc. H2SO4
(c) Anh. AlCl3 (d) KMnO4/OH–
(C) CH2BrCH2Br ¾® CH2 = CH2 (r) Pd/C
53. In the following sequence of reactions, the alkene affords
the compound ‘B’ (D) CH3CH2OH ¾® CH2 = CH2 (s) Alc. KOH
O H O (a) A – (r), B – (s), C – (p), D – (q)
CH 3 - CH = CH - CH 3 ¾¾3¾
® A ¾¾¾
2 ® B.
(b) A – (s), B – (r), C – (q), D – (p)
Zn
The compound B is (c) A – (q), B – (p), C – (s), D – (r)
(a) CH3CH2CHO (b) CH3COCH3 (d) A – (r), B – (s), C – (q), D – (p)
(c) CH3CH2COCH3 (d) CH3CHO 63. Assertion : 1-Butene on reaction with HBr in the presence
54. One mole of a symmetrical alkene on ozonolysis gives two of a peroxide produces 1-bromobutane.
moles of an aldehyde having a molecular mass of 44 u. The Reason : It involves the free radical mechanism.
alkene is (a) Assertion is correct, reason is correct; reason is a
(a) propene (b) 1-butene correct explanation for assertion.
(c) 2-butene (d) ethene (b) Assertion is correct, reason is correct; reason is not
55. The alkene that will give the same product with HBr in a correct explanation for assertion
the absence as well as in the presence of peroxide is (c) Assertion is correct, reason is incorrect
(d) Assertion is incorrect, reason is correct.
HYDROCARBONS 185

64. Assertion : All the hydrogen atoms in CH2 = C = CH2 lie (a) During the reaction hydrogen atom is eliminated
in one plane. from the b - carbon atom.
Reason : Carbon atoms are sp2 and sp hybridised. (b) Rate of reaction for same alkyl group;
(a) Assertion is correct, reason is correct; reason is a Iodine > Bromine > Chlorine
correct explanation for assertion. (c) Rate of reaction; (CH3)3C – > (CH3)2CH – > CH3CH2 –
(b) Assertion is correct, reason is correct; reason is (d) Only nature of halogen atom determines rate of the
not a correct explanation for assertion reaction.
(c) Assertion is correct, reason is incorrect 73. Arrange in decreasing order of reactivity towards HCl
(d) Assertion is incorrect, reason is correct. (1) F—CH==CH2 (2) (CH3)2 C==CH2
65. Correct order of stability is : (3) NO2—CH==CH2 (4) (CN)2 C==C(CN)2
(a) cis -2- butene > 1-butene > trans -2-butene (a) 2 > 1 > 3 > 4 (b) 4 > 1 > 3 > 2
(b) trans-2-butene > cis-2-butene > 1-butene (c) 1 > 4 > 2 > 3 (d) 3 > 2 > 4 > 1
(c) 1-butene > cis-2-butene > trans-2- butene
(d) cis-2-butene > trans-2-butene > 1-butene
66. Which of the following is correct set of physical properties 74. + Br HBr + X
of the geometrical isomers?
Most probable structure of would be
CH3 CH3

(a) (b)
Dipole moment B.P. M.P. Stability
(a) I > II I > II II > I I > II
(b) II > I II > I II > I II > I
(c) I > II I > II I > II I > II (c) (d)
(d) II > I II > I I > II I > II
67. But-2-ene exhibits cis,trans-isomerism due to 75. In the given reaction
(a) rotation around C3 – C4 sigma bond
(b) restricted rotation around C = C bond
(c) rotation around C1 – C2 bond C7H12 + HBr Br
(d) rotation around C2 – C3 double bond [X]
[Major]
68. When 3, 3-dimethyl 2-butanol is heated with H2SO4, the
Structure of [X] can not be
major product obtained is
(a) 2,3-dimethyl 2-butene
(b) 3, 3-dimethyl 1- butene (a) –CH=CH2 (b)
(c) 2, 3-dimethyl 1- butene
(d) cis & trans isomers of 2, 3-dimethyl 2-butene
69. The only alcohol that can be prepared by the indirect (c) (d)
hydration of alkene is
(a) Ethyl alcohol (b) Propyl alcohol
76. An organic compound C6H12 (X) on reduction gives C6H14
(c) Isobutyl alcohol (d) Methyl alcohol
(Y). X on ozonolysis gives two aldehydes C2H4O (I) and
70. A hydrocarbon, A on chlorination gives B which on heating
C4H8O (II). Identify the compounds X, Y and aldehydes (I)
with alcoholic potassium hydroxide changes into another
and (II).
hydrocarbon C. The latter decolourises Baeyer's reagent
(a) X = CH3CH == CHCH2CH2CH3,
and on ozonolysis forms formaldehyde only. A is
Y = CH3(CH2)4CH3,
(a) Ethane (b) Butane
(I) = CH3CHO, (II) = CH3(CH2)2CHO
(c) Methane (d) Ethene
(b) X = CH3CH2CH == CHCH2CH3,
71. In the presence of peroxide, hydrogen chloride and
Y = CH3(CH2)4CH4, (I) = CH3CHO, (II) = CH3CHO
hydrogen iodide do not give anti-Markovnikov’s addition
(c) X = CH3CH2CH2CH2CH == CH2,
to alkenes because
Y = CH3(CH2)3CH3,
(a) Both are highly ionic
(b) One is oxidising and the other is reducing (d) X = CH3CH2 C CH CH3 ,
(c) One of the steps is endothermic in both the cases CH3
(d) All the steps are exothermic in both the cases
Y = CH3(CH2)3CH3,
72. Which of the following statements is incorrect regarding
(I) = CH3CHO, (II) = CH3CH2CHO
dehydrohalogenation of alkenes ?
EBD_8350
186 CHEMISTRY

77. The products for the following reactions are 80. Number of alkynes for formula C5H8 is –
Br (a) 2 (b) 3 (c) 4 (d) 5
(i) CH3 C CH2 CH3 + alc. KOH ¾® X 81.
Reagent
R - CH 2 - CCl 2 - R ¾¾¾¾® R - C º C - R
H The reagent is
(a) Na (b) HCl in H2O
O (c) KOH in C2H5OH (d) Zn in alcohol.
CH2 ¾® Y + Z
3
(ii) CH3 CH CH
82. Calcium carbide when treated with water gives :
CH3 (a) ethylene (b) methane
(a) X = (CH3)2C == CH2, Y = CH3CH2CHO, Z = CH3CH2CHO (c) acetylene (d) ethane
(b) X = CH2 == CH2, Y = CH3CHO, Z = CH3COOH 83. Which one of the following has the minimum boiling
(c) X = CH3 — CH == CH — CH3, point ?
(a) 1-Butene (b) 1-Butyne
CH3 (c) n- Butane (d) Isobutane
Y = CH3 CH CHO , Z = HCHO 84. Ammonical silver nitrate forms a white precipitate easily
with
(d) X = CH3 — CH == C(CH3)2, Y = HCHO, Z = CH3CHO
78. A compound X decolourises Br 2 water and reacts slowly (a) CH3C º CH (b) CH3C º CCH3
with conc. H2SO4 to give an addition product. X reacts (c) CH3CH = CH 2 (d) CH2 = CH2
with HBr to form Y. Y reacts with NaOH to form Z. On 85. When acetylene is passed through dil. H2SO4 in presence
oxidation Z gives hexan-3-one. X, Y and Z in the reactions of HgSO4, the compound formed is
are (a) ether (b) acetaldehyde
Br 2 water (c) acetic acid (d) ketone
X ¾¾¾® Decolourisation 86. Which of the following will be the final product when C2H2
conc. H2SO 4
reacts with HCl
CH 2 CH3
Addition product (a) P (b) |
CHCl CHCl 2
HBr NaOH [O]
X ¾¾¾
® Y ¾¾¾¾
® Z ¾¾¾
®
CHCl
CH3CH2CH2 C CH2CH3 (c) P (d) None of these
CHCl
O 87. The hydrocarbon which can react with sodium in liquid
Hexan-3-one
ammonia is
(a) X = CH3CH2CH == CHCH3,
Y = CH3CH2CH(Br)CH(Br)CH2CH3, (a) CH 3CH 2 CH 2 C º CCH 2CH 2 CH3
Z = CH3CH2CH3 (b) CH 3CH 2C º CH
(b) X = CH3CH == CHCH3,
Y = CH3CH(Br)CH(Br)CH3, (c) CH 3CH = CHCH3
Z = CH3CH2CH2OH (d) CH 3CH 2C º CCH 2CH3
(c) X = CH3CH2CH == CHCH2CH3,
88. Which of these will not react with acetylene?
Y = CH3 CH2 CH CH2CH2CH3 , (a) NaOH (b) Ammonical AgNO3
(c) Na (d) HCl.
Br
89. When acetylene is passed over heated iron tube, the
Z = CH3 CH 2CH2 CH CH2CH3 product obtained is –
OH (a) C2H2 (b) C4H4 (c) C6H6 (d) C8H8
(d) X = CH3CH2CH2CH == CHCH3, 90. But–2–yne on chlorination gives
Y = CH3CH2CH2CH2CH2CH2Br, (a) 1 –chlorobutane
Z = CH3CH2CH2CH2OH (b) 1, 2 –dichlorobutane
(c) 1, 1, 2, 2 –tetrachlorobutane
Topic 3: Alkynes (d) 2, 2, 3, 3 –tetrachlorobutane
79. The IUPAC name of the compound 91. When propyne reacts with aqueous H2SO4 in the presence
CH3CH = CHC º CH is of HgSO4, the major product is
(a) Pent-l-yn-3-ene (b) Pent-4-yn-2-ene (a) Propanal (b) Propyl hydrogen sulphate
(c) Pent-3-en-1-yne (d) Pent-2-en-4-yne (c) Acetone (d) Propanol
HYDROCARBONS 187

92. Propyne on polymerisation gives (c) Acetylene contains least number of hydrogens
(a) Mesitylene (b) Benzene among the possible hydrocarbons having two carbons
(c) Ethyl benzene (d) Propyl benzene (d) Acetylene belongs to the class of alkynes with
93. What happens when a mixture of acetylene and hydrogen molecular formula, CnH2n – 2.
is passed over heated Lindlar’s catalyst ? 101. The correct increasing order of acidity of the following
(a) Ethane and water are formed alkynes
(b) Ethylene is formed (1) CH3 – C º C – CH3 (2) CH3 – C º CH
(c) Acetylene and ethane are formed
(3) CH º CH
(d) None of these
(a) 1 < 2 < 3 (b) 2 < 3 < 1
94. Which of the following reaction is shown by alkynes ?
(c) 3 < 2 < 1 (d) 1 < 3 < 2
(a) Addition (b) Substitution
102. Identify the alkyne in the following sequence of reactions.
(c) Polymerization (d) All of these
95. Which of the following reactions will yield 2, H Ozonolysis
Alkyne ¾¾¾¾¾¾¾
2 ® A ¾¾¾¾¾
®
Lindlar's catalyst
2-dibromopropane ? Wacker
(a) HC º CH + 2HBr ® B ¬¾¾¾¾ C2H 4
only Process
(b) CH3C º CH + 2HBr ® (a) H3C - C º C - CH3
(c) CH3CH = CH 2 + HBr ® (b) H3C - CH 2 - C º CH
(d) CH3CH = CHBr + HBr ® (c) H 2C = CH - C º CH
96. In the given reactions (d) HC º C - CH 2 - C º CH
A
CH 3C º CH ¾¾® CH 3CBr2 CHBr2 103. Which of the following represents the correct order of
B acidic strength ?
CH 3C º CH ¾¾
® CH 3CBr2 CH 3
(i) HC º CH > H 2C = CH 2 > CH3 - CH3
Hg 2 + H + (ii) HC º CH > CH3 - CH3 > H 2C = CH 2
CH3C º CH ¾¾¾¾¾
®C
333 K
(iii) CH3C º CH > HC º CH > CH3 - C º C - CH3
Hg 2+ H +
HC º CH ¾¾¾¾¾
®D (iv) HC º CH > CH3 - C º CH > CH3 - C º C - CH3
333 K
(a) (i) and (iii) (b) (ii) and (iv)
A, B, C and D are respectively
(c) (i) and (iv) (d) (i) and (iv)
(a) HBr, Br2, CH3COCH3, CH3CHO
104. Which of the following hydrocarbon will give following
(b) Br2, HBr, CH3COCH3, CH3CHO
three products [X], [Y] and [Z] on heating with KMnO4
(c) HBr, HBr, CH3COCH3, CH3CHO
(d) Br2, HBr, CH3CH2CHO, CH3CHO
97. Which of the following polymer can be used as electrodes
in batteries ?
(a) Polypropene (b) Polyacetylene
(c) Polyethene (d) Polyisoprene
98. Which of the following statements are correct ?
(i) Alkynes on reduction with palladised charcoal form
cis alkenes.
(ii) Alkynes on reduction with palladised charcoal form (a)
trans alkenes.
(iii) Alkynes on reduction with sodium in liquid ammonia
form trans alkenes.
(iv) Propyne on reduction with palladised charcoal form a (b)
mixture of cis and trans propene.
(a) (i) and (iv) (b) (i) and (iii)
(c) (ii) and (iv) (d) (i), (iii) and (iv)
99. How many structural isomers are possible for the alkyne
C6H10 ? (c)
(a) 7 (b) 6 (c) 8 (d) 5
100. Acetylenic hydrogens are acidic because
(a) Sigma electron density of C – H bond in acetylene is
(d)
nearer to carbon, which has 50% s-character
(b) Acetylene has only one hydrogen on each carbon
EBD_8350
188 CHEMISTRY

105. In which case CO2 is not produced as a by product? Topic 4: Aromatic Hydrocarbons
(a) CH3—CH==C==CH—CH3 110. Aromatic compounds burn with sooty flame because
(a) they have a ring structure of carbon atoms
(b) (b) they have a relatively high percentage of hydrogen
(c) they have a relatively high percentage of carbon
(d) they resist reaction with oxygen of air
(c) 111. The conditions for aromaticity is :
(d) Both in (a) and (c) (a) molecule must have cyclic clouds of delocalised p
electrons
(b) molecule must contain (4n + 2)p electrons
106. (c) Both (a) and (b)
(d) None of the above
112. Select the true statement about benzene amongst the
following
(a) Because of unsaturation benzene easily undergoes
[X] and [Y] are
addition
(a) Na/Liq NH3, Br—CH2—CH2—F (b) There are two types of C – C bonds in benzene molecule
(b) Na/Liq NH3, Br—CH2—CH2—Cl (c) There is cyclic delocalisation of p-electrons in benzene
(c) Na/Liq NH3, Br—CH2—CH2—I (d) Monosubstitution of benzene gives three isomeric
(d) Na/Liq NH3, Br—CH2—CH2—CH2—Cl products.
107. Which of the following reactions does not show the acidic 113. Benzene can be directly obtained from
nature of ethyne? (a) Acetylene (b) Phenol
(a) Acetylene reacts with sodamide to form sodium (c) Chlorobenzene (d) All the above
acetylides. COONa
(b) When passed through ammoniacal cuprous chloride
solution, a red precipitate is formed. NaOH + CaO
(c) Acetylene reacts with chlorine in the dark to form di 114. ¾¾¾¾¾® A
or tetrachlorides.
(d) Acetylene when passed through ammoniacal silver The product A is
nitrate gives a white precipitate. (a) Benzene (b) Benzaldehyde
108. Which of the following does not represent polymerisation (c) Toluene (d) Benzoic acid
of alkenes and alkynes? 115. In a reaction of C6H5Y, the major product (> 60%) is
(a) nCH ºº CH ¾® —( CH == CH – CH == CH — n)
m-isomer, so the group Y is
(a) –COOH (b) –NH2
(b) 3CH ºº CH ¾® (c) –OH (d) –Cl

HNO Br
116. ¾¾¾¾ 3 ® A ¾¾¾®2 B. The compound B is
(c) nCH3 — CH == CH2 ¾® —( CH – CH 2 —
)n H SO 2 FeBr
4 2

CH3 NO2 NO2


(d) nCH2 == CH2 ¾® —( CH 2 – CH == CH – CH 2— )n Br
109. Choose the correct reagents used in the conversion. (a) (b)
(p ) (q ) Br
CH2 == CH2 ¾® CH2 – CH2 ¾® CH2 == CHBr NO2 NO2
Br Br
(c) (d)
(r ) (s)
¾® CH ºº CH ¾® Br Br
Br
p q r s 117. Chlorobenzene is o, p-directing in electrophilic substitution
(a) Br2 alc. KOH NaOH Al2O3 reaction. The directing influence is explained by
(b) HBr alc. KOH CaC2 KMnO4 (a) + M of Ph (b) + I of Cl
(c) HBr alc. KOH NaNH2 red hot (c) + M of Cl (d) + I of Ph
iron tube 118. Catalytic hydrogenation of benzene gives
(d) Br2 alc. KOH NaNH2 red hot (a) xylene (b) cyclohexane
iron tube (c) benzoic acid (d) toluene
HYDROCARBONS 189

119. The strongest ortho - para and strongest meta - directing 129. Match the following reactants in Column I with the
groups respectively are corresponding reaction products in Column II and choose
(a) –NO2 and –NH2 (b) –CONH2 and –NH2 the correct option from the codes given below.
(c) –NH2 and –CONH2 (d) –NH2 and –NO2 Column - I Column - II
120. For the formation of toluene by Friedal Crafts reaction, AlCl
reactants used in presence of anhydrous AlCl 3 are (A) Benzene + Cl2 ¾¾¾®
3
(p) Benzoic acid
(a) C2H2 and CCl4 (b) CH4 and CaCN2 (B) Benzene + CH3Cl (q) Methyl phenyl
(c) C6H6 and CH3Cl (d) C2H5 OH and Zn AlCl3
¾¾¾® ketone
121. Benzene can be obtained in the reaction
(C) Benzene + CH3COCl (r) Toluene
(a) Ethene + 1, 3-butadiene
AlCl
(b) Trimerisation of ethyne ¾¾¾®
3
(c) Reduction of PhCHO
KMnO4 / NaOH
(d) All of these (D) Toluene ¾¾¾¾¾¾ ¾® (s) Chlorobenzene
122. Benzene reacts with CH3COCl + AlCl3 to give (a) A – (s), B – (r), C – (q), D – (p)
(a) chlorobenzene (b) toluene (b) A – (s), B – (r), C – (p), D – (q)
(c) benzyl chloride (d) acetophenone (c) A – (r), B – (s), C – (p), D – (q)
123. Benzene on reaction with ozone forms __________. (d) A – (r), B – (s), C – (q), D – (p)
(a) 2 molecules of aldehyde and 1 molecule of ketone 130. Match the columns
(b) 2 molecules of ketone and 1 molecule of aldehyde Column - I Column - II
(c) triozonide (A) Alkyl halide + Alkyl halide (p) Sulphonation
(d) hexaozonide in presence of dry ether
124. AlCl3 acts as ________ in Friedel-Crafts reaction (B) Arene + Acid halide (q) Wurtz reaction
(a) nucleophile (b) electrophile in presence of AlCl3
(c) free radical (d) intermediate (C) Arene + Fuming sulphuric (r) Catalytic
125. The electrophilic substitution reactions of benzene takes in presence of AlCl3 hydrogenation
place via (D) Arene + Hydrogen (s) Friedel-Crafts
(i) generation of electrophile in presence of Ni reaction
(ii) generation of nucleophile (a) A - (p), B - (r), C - (q); D - (s)
(iii) formation of carbocation intermediate
(b) A - (s), B - (q), C - (r); D - (p)
(iv) removal of proton from the carbocation intermediate
(a) (i), (iii) and (iv) (b) (ii), (iii) and (iv) (c) A - (r), B - (p), C - (s); D - (q)
(c) (i) and (iv) (d) (ii) and (iv) (d) A - (q), B - (s), C - (p); D - (r)
126. During the nitration of benzene. In the process of 131. Match the columns
generation of nitronium ion sulphuric acid behaves as a/ Column - I Column - II
an ______ and nitric acid behave as a/an _________. (A) Aromatic (p) Planar
(a) base, acid (b) acid, base (B) Antiaromatic (q) Non-planar
(c) strong acid, weak acid (d) weak acid, strong acid (C) Huckel rule (r) 4 np localised
127. Benzene is highly unsaturated but it does not undergo electrons
addition reaction because (D) Cyclo-octatetraene (s) (4 n + 2)p delocalised
(a) p-electrons of benzene are delocalised. electrons
(b) cyclic structures do not show addition reaction (a) A - (p, s), B - (p, r), C - (s), D - (q, r)
(c) benzene is a non-reactive compound (b) A - (p, r), B - (p, s), C - (s), D - (q, r)
(d) All of the above (c) A - (p, s), B - (s), C - (p, r), D - (q, r)
128. Which of the following statements are correct ? (d) A - (q, r), B - (p, r), C - (s), D - (p, s)
(i) Polynuclear hydrocarbons contain two or more 132. Which one of these is not compatible with arenes?
benzene rings fused together.
(a) Greater stability
(ii) Polynuclear hydrocarbons have carcinogenic
(b) Delocalisation of p-electrons
property.
(iii) Polynuclear hydrocarbons are formed on incomplete (c) Electrophilic additions
combustion of organic materials like tobacco, coal (d) Resonance
and petroleum. 133. The carbon-carbon bond length in benzene is
(iv) They are also produced in human body due to various (a) Same as in C2H4
biochemical reactions. (b) In between C2H6 and C2H2
(a) (i), (ii) and (iv) (b) (i), (iii) and (iv)
(c) In between C2H4 and C2H2
(c) (ii), (iii) and (iv) (d) (i), (ii) and (iii)
(d) In between C2H6 and C2H4
EBD_8350
190 CHEMISTRY

134. (i) Chlorobenzene and (ii) benzene hexachloride are


obtained from benzene by the reaction of chlorine, in the
pesence of HNO
(a) (i) Direct sunlight and (ii) anhydrous AlCl3 140. CH3 N ¾¾¾¾
3
®?
H 2SO 4
(b) (i) Sodium hydroxide and (ii) sulphuric acid
(c) (i) Ultraviolet light and (ii) anhydrous FeCl3
(d) (i) Anhydrous AlCl3 and (ii) direct sunlight
O
135. A group which deactivates the benzene ring towards
electrophilic substitution but which directs the incoming
group principally to the o-and p-positions is (a) CH3 N
(a) –NH2 (b) –Cl
(c) –NO2 (d) –C2H5 O NO2
136. Benzene can be obtained by heating either benzoic acid
with X or phenol with Y. X and Y are respectively. O
(a) Zinc dust and soda lime
(b) Soda lime and zinc dust (b) CH3 N
(c) Zinc dust and sodium hydroxide
NO2
(d) Soda lime and copper O
137. Which of the following chemical system is non aromatic?
O
(a) (b) NO2
(c) CH3 N

(c) (d) O
S
138. Increasing order of rate of Friedel craft reaction in O
O O O
O O (d) CH3 N
O O O
NO2
O
(I) (II) 141. Reaction of SO3 is easier in

O Cl NO2
O O O
(a) (b)

(III)

(a) III < II < I (b) II < III < I COCH3 NO2
(c) I < III < II (d) I < II < III
139. Friedel craft acylation can be used to obtain (c) (d)

NO2 NO2
COCH3
142. Assertion: Benzene does not undergo addition reaction
(a) (b) easily
HO3S Reason: Benzene losses it’s aromaticity after addition
reaction and becomes less stable
COCH3
(a) Assertion is True, Reason is true: Reason is a correct
explanation for Assertion
(b) Assertion is True, Reason is true: Reason is not a
NO2 corret explanation for Assertion
(c) (d) (c) Assertion is True, Reason is False
COCH3 (d) Assertion is False, Reason is True
COCH3
HYDROCARBONS 191

143. Mark the correct decreasing order of stability. 145. Arrange the following molecules in the increasing order of
(a) Aromatic > non-aromatic > anti-aromatic s to p bond ratio.
(b) Aromatic > anti-aromatic > non-aromatic
(c) Non-aromatic > anti-aromatic > aromatic (1)
(d) Anti-aromatic > non-aromatic > aromatic

AlCl
144. +
3
¾¾® X, compound X is (2)
Cl

(3)
(a) (b)

Cl

(4)
(c) (d)
Cl Cl (a) (2) < (3) < (4) < (1) (b) (2) < (4) < (3) < (1)
(c) (3) < (2) < (1) < (4) (d) (2) < (3) < (1) < (4)

Cl
1. ¾¾¾
2 ® (A)
hn NBS, hn
¾¾¾¾®
CCl4
Find the number of monochloro derivatives formed
(excluding stereoisomers) in the above reaction. Br
8. ¾¾¾
2 ® (B)
2. Find the total number of trichloroderivatives of the D brine

compound (excluding stereoisomers). Find the number of fractions obtained after fractional
distillation of product mixture.
3. How many isomers (including geometrical and optical) are
possible for bromochlorocyclobutane?
4. How many ster eoisomers ar e possible for 9.
dichlorocyclobutane?
5. How many enantiomeric pairs are possible in
Cl /hn Monochlorination
bromochlorocyclopentane? ¾¾¾¾¾¾¾¾¾®
2 (All isomers)
(c)
6. How many alkyl bromides would yield isopentane on
reaction with Grignard reagent followed by treatment with Calculate sum of number of products formed in the reaction
water? a, b and c.
7. How many free radicals can be produced during following 10. How many alkenes, alkynes, alkadienes can be
reaction (ignoring resonating structure) ? hydrogenated to form isopentane (include all isomers)?

Exercise 3 : NCERT Exemplar & Past Year MCQs


NCERT Exemplar MCQs (a) 1 > 2 > 3 > 4 (b) 2 > 3 > 4 > 1
1. Arrange the following in decreasing order of their boiling (c) 4 > 3 > 2 > 1 (d) 3 > 2 > 4 > 1
points. 2. Arrange the halogens F2, Cl2, Br 2, I2, in order of their
1. n–butane 2. 2-methylbutane increasing reactivity with alkanes.
3. n–pentane 4. 2, 2-dimethylpropane (a) I2 < Br2 < Cl2 < F2 (b) Br2 < Cl2 < F2 < I2
(c) F2 < Cl2 < Br2 < I2 (d) Br2 < I2 < Cl2 < F2
EBD_8350
192 CHEMISTRY

3. The increasing order of reduction of alkyl halides with zinc B. CH3 — CH2 — Br
and dilute HCl is C. CH3 — CH2 — CH2 — Br
(a) R – Cl < R – I < R – Br (a) A > B > C (b) C > B > A
(b) R — Cl < R – Br < R – I (c) B > C > A (d) A > C > B
(c) R – I < R – Br < R – Cl 10. Which of the following reactions of methane is incomplete
(d) R – Br < R – I < R – Cl combustion?
4. The correct IUPAC name of the following alkane is (a) Cu / 523 K /100 atm
2CH 4 + O 2 ¾¾¾¾¾¾¾¾
® 2CH 3OH
H3 C - CH2 - CH - CH2 - CH2 - CH - CH2 - CH3 (b) Mo O
| | CH 4 + O 2 ¾¾¾¾
2 3 ® HCHO + H O
2
CH CH 2 (c) CH 4 + O 2 ¾¾® C ( s ) + 2H 2 O ( l )
| (d) CH 4 + 2O 2 ¾¾® CO 2 ( g ) + 2H 2 O ( l )
CH3 CH3 CH3
(a) 3, 6-diethyl-2-methyloctane Past Year MCQs
(b) 5-isopropyl-3-ethyloctane 11. Which of the following organic compounds has same
(c) 3-ethyl-5-isopropyloctane hybridisation as its combustion product (CO2)?
(d) 3-isopropyl-6-ethyloctane [AIPMT 2014, S]
5. The addition of HBr to 1-butene gives a mixture of (a) Ethane (b) Ethyne
products A, B and C. (c) Ethene (d) Ethanol
Br 12. The major organic compound formed by the raction of
C2H5
1, 1, 1-trichloroethane with silver powder is:
C C [JEE M 2014, S]
(A) H5C2 CH3 (B) H CH3 (a) Acetylene (b) Ethene
H Br (c) 2 - Butyne (d) 2 - Butene
(C) CH3 - CH 2 - CH 2 - CH 2 - Br 13. 2,3-Dimethyl-2-butene can be prepared by heating which
of the following compounds with a strong acid ?
The mixture consists of
[AIPMT 2015 RS, S]
(a) A and B as major and C as minor products
(b) B as major, A and C as minor products (a) (CH3)2 CH – CH – CH = CH2
|
(c) B as minor, A and C as major products CH3
(d) A and B as minor and C as major products (b) (CH3)3 C – CH = CH2
6. Which of the following will not show geometrical
(c) (CH3)2C = CH – CH2 – CH3
isomerism?
(d) (CH3)2CH – CH2 – CH = CH2
F H F F
(a) C=C (b) C=C 14. In the reaction with HCl, an alkene reacts in accordance
Cl D Cl Cl with the Markovnikoff's rule, to give a product 1-chloro-1-
C2H5 methylcyclohexane. The possible alkene is :
H3C CH3 CH3
C=C [AIPMT 2015 RS, A]
(c) (d) C=C
H5C2 CH3 C2H5 CH2 CH3
CH3
7. Arrange the following hydrogen halides in order of their
decreasing reactivity with propene. (a) (b)
(a) HCl > HBr > HI (b) HBr > HI > HCl
(c) HI > HBr > HCl (d) HCl > HI > HBr CH3
8. Arrange the following carbanions in order of their
decreasing stability.
(c) (a) and (b) (d)
A. H3C — C º C – B. H — C º C –
C. H3C — CH 2
(a) A > B > C (b) B > A > C 15. A single compound of the structure : [AIPMT 2015, A]
(c) C > B > A (d) C > A > B CH3 CH3
9. Arrange the following alkyl halides in decreasing order of OHC C C
the rate of b – elimination reaction with alcoholic KOH. C H C O
H H2 H2
| is obtainable from ozonolysis of which of the following
A. CH3 - C - CH 2 Br cyclic compounds ?
|
CH3
HYDROCARBONS 193

H3C CH3 21. In the reaction


H3C
(i) NaNH 2 / liq.NH3 (i) NaNH 2 / liq.NH 3
(a) (b) H - C º CH X Y
(ii) CH3CH 2 Br (ii) CH 3 CH 2 Br
H3C X and Y are [NEET 2016, S]
CH3 H3C (a) X = 1-Butyne ; Y = 3-Hexyne
CH3 (b) X = 2-Butyne ; Y = 3-Hexyne
(c) (d)
(c) X = 2-Butyne ; Y = 2-Hexyne
CH3
(d) X = 1-Butyne ; Y = 2-Hexyne
16. The total number of p-bond electrons in the following
22. The correct statement regarding the comparison of
structure is : [AIPMT 2015, S]
staggered and eclipsed conformation of ethane, is
H H H [NEET 2016, C]
H3C (a) The staggered conformation of ethane is less stable
H3C CH3 than eclipsed conformation, because staggered
conformation has torsional strain
(b) The eclipsed conformation of ethane is more stable
H2C H CH3 than staggered conformation, because eclipsed
(a) 8 (b) 12 (c) 16 (d) 4 conformation has no torsional strain
17. Given: (c) The eclipsed conformation of ethane is more stable
H3C CH3 H3C CH2 H2C CH2 than staggered conformation even though the
eclipsed conformation has torsional strain
(d) The staggered conformation of ethane is more stable
than eclipsed conformation, because staggered
CH3 CH3 CH2 conformation has no torsional strain.
23. The reaction of propene with HOCl (Cl2 + H2O) proceeds
(I) (II) (III)
through the intermediate: [JEE M 2016, C]
The enthalpy of the hydrogenation of these compounds
(a) CH3 – CH(OH) – CH+2 (b) CH3 – CHCl – CH2+
will be in the order as : [AIPMT 2015, C]
(c) CH3 – CH+ – CH2 – OH (d) CH3 – CH+ – CH2 – Cl
(a) III > II > I (b) II > III > I
(c) II > I > III (d) I > II > III 24. The product of the reaction given below is:
18. The oxidation of benzene by V2O5 in presence of air 1. NBS/hv
produces : [AIPMT 2015 RS, C] ¾¾¾¾¾® X [JEE M 2016, S]
2. H 2O/K2CO 3
(a) benzoic anhydride (b) maleic anhydride
(c) benzoic acid (d) benzaldehyde O CO2H
19. The reaction of C6H5CH = CHCH3 with HBr produces:
(a) (b)
[AIPMT 2015, A]
(a) C6 H5CH 2CHCH3 (b) C6 H5CH 2CH 2CH 2Br
|
Br
CH=CHCH3 OH

(c) (d)
(c) (d) C 6 H 5 C HCH 2 CH3
|
Br 25. With respect to the conformers of ethane, which of the
following statements is true ? [NEET 2017, C]
Br (a) Bond angle changes but bond length remains same
20. Which compound would give 5 - keto - 2 - methylhexanal (b) Both bond angle and bond length change
upon ozonolysis ? [JEE M 2015, A] (c) Both bond angles and bond length remains same
CH3 (d) Bond angle remains same but bond length changes
CH3 26. Which one is the correct order of acidity ?
[NEET 2017, C]
(a) (b) H3C
(a) CH º CH > CH3 – C º CH > CH2 = CH2 > CH3 – CH3
CH3 (b) CH º CH > CH2 = CH2 > CH3 – C º CH > CH3 – CH3
CH3
CH3 (c) CH3 – CH3 > CH2 = CH2 > CH3 – C º CH> CH º CH
(d) CH2 = CH2 > CH3 – CH = CH2 > CH3 – C º CH > CH º CH
(c) CH3 (d)
CH3
EBD_8350
194 CHEMISTRY

27. Predict the correct intermediate and product in the following CH3
reaction : [NEET 2017, S]


(a) Cl – CH2 – CH2 – CH
H O, H SO
H3C - C º CH ¾¾¾¾¾¾
2 2 4 ® Intermediate ® product


HgSO 4 (A) (B)
CH3
(a) A : H3C – C = CH2 B : H3C – C = CH2 CH2Cl
|


| (b) H3C – CH2 – CH – CH3
OH SO4
CH3
(b) A : H3C - C - CH3 B : H3C – C º CH


|| (c) H3C – CH2 – C – CH3
O


(c) A : H3C – C = CH2 B : H3C – C – CH3 Cl
| || CH3
OH O


(d) H3C – CH – CH
(d) A : H3C – C = CH2 B : H3C - C - CH3


| || Cl CH3
SO4 O
33. The major product of the following reaction is:
28. 3-Methyl-pent-2-ene on reaction with HBr in presence of
peroxide forms an addition product. The number of possible CH3C º CH
(i) DCl (1equiv.)
¾¾¾¾¾¾¾® [JEE M 2019, C]
(ii) DI
stereoisomers for the product is :
[JEE M 2017, C] (a) CH3CD(I)CHD(Cl) (b) CH3CD(Cl)CHD(I)
(a) Six (b) Zero (c) Two (d) Four (c) CH3CD2CH(Cl)(I) (d) CH3C(I)(Cl)CHD2
29. Hydrocarbon (A) reacts with bromine by substitution to 34. The increasing order of reactivity of the following
form an alkyl bromide which by Wurtz reaction is converted compounds towards aromatic electrophilic substitution
to gaseous hydrocarbon containing less than four carbon reaction is: [JEE M 2019, C]
atoms. (A) is [NEET 2018, S]
Cl OMe Me CN
(a) CH º CH (b) CH2 = CH2
(c) CH4 (d) CH3 – CH3
30. The trans-alkenes are formed by the reduction of alkynes
with: [JEE M 2018, C] A B C D
(a) H2-Pd/C, BaSO4 (b) NaBH4 (a) D < A < C < B (b) B < C < A < D
(c) Na/liq. NH3 (d) Sn - HCl (c) A < B < C < D (d) D < B < A < C
31. The most suitable reagent for the following conversion,
35. An alkene on ozonolysis gives methanal as one of the
is: [NEET 2019, C]
product. Its structure is [NEET 2020, C]
H3C CH3
H3C – C º C – CH3 ¾®
H H (a) (b)
cis -2-butene
(a) Na/liquid NH3 (b) H2, Pd/C, quinoline
(c) Zn/HCl (d) Hg2+/H+, H2O
32. An alkene “A” on reaction with O3 and Zn – H2O gives
propanone and ethanal in equimolar ratio. Addition of HCl (c) (d)
to alkene “A” gives “B” as the major product. The
structure of product “B” is: [NEET 2019, A]

Exercise 4 : Problem Solving Skill Enhancer MCQs


hn KOH Cl Cl
1. Hydrocarbon + Cl2 [W] [X]
(1 mole)
Et OH (a) (b)
NBS
¾¾¾ ® [Y] OH Br
hn
What would be the most probable structure of [Y] if OEt
hydrocarbon possesses six carbon atom and can not react
with Cl2 in dark? (c) (d)
Br
Br
HYDROCARBONS 195

7. Number of carbocations and transition states involved in


2. CH3 Ha
b the following reactions are respectively
H
CH==CH2
+ Br• ® + CH3OH
H2 SO4
H d OMe
OH
(a) 2, 3 (b) 3, 4 (c) 4, 5 (d) 4, 4
CH2 ¾ H c
8. Products of the following reaction:
Br• will abstract which of the hydrogen most readily? (1) O
(a) a (b) b (c) c (d) d CH3CºCCH2CH3 ¾¾¾¾¾¾ 3
(2) oxidation
® ...... are:
3. An organic compound C4H6 on ozonolysis give HCHO, (a) CH3COOH + CO2
CO2, CH3CHO. Compound will be: (b) CH3COOH + HOOC ×CH2CH3
(a) H2C == CH — CH == CH2 (b) CH3— CH == C == CH2 (c) CH3CHO + CH3CH2CHO
(c) CH3— C C — CH3 (d) (d) CH3COOH + CH3COCH3

CH3 CH3 9.
HBr (2 moles)
[X]
Zn
[Y]
4. C CH2 C CH2 (PhCO)2O2 D
CH3 CH3O
How many carbon atoms are present in product [Y].
(A) (B) (a) 14 (b) 12 (c) 8 (d) 10
CH3 (P)
,h ®
n
C CH2 Cl 2 ¾
¾
CH3OCH2 ¾
10.
(C) ¾Cl2 /Al
¾¾Cl
3
Arrange the above in the decreasing order of reactivity ®
(Q)
towards HBr:
(a) A > B > C (b) B > A > C Identify major product of both reactions P and Q respectively.
(c) B > C > A (d) A > C > B CH2 Cl
Cl
H2 SO4 (a)
5. CH2OH [X]
D
Major
H2 SO4
2 [X] [Y]
D
Major

Product [Y] is (b)


CH2—CH2 Cl
(a) Cl

Cl
CH==CH
(b) (c)

Cl
(c)
Cl
(d) (d)

6. In the presence of platinum catalyst, hydrocarbon A adds Cl Cl


hydrogen to form n-hexane. When hydrogen bromide is 11. Which of the following compounds are antiaromatic
added to A only a single bromo compound is formed. Which
of the following is A?
(a) CH 3 — CH 2 — CH = CH — CH 2 — CH 3 O
– +
(b) CH 3 — CH 2 — CH 2 — CH = CH — CH 3
(c) CH 3 — CH = CH — CH 2 — CH 2 — CH 3 (I) (II) (III) (IV) (V) (VI)
(a) (I) and (V) (b) (II) and (V)
(d) CH 2 = CH — CH 2 — CH 2 — CH 2 — CH 3
(c) (I) and (IV) (d) (III) and (VI)
EBD_8350
196 CHEMISTRY

12. How many carbocations are produced during the CH3 CH3 CH3
following transformation?
D D D
H (c) > >
CH2CH2—CH—CH ==CH2

CH3 CH3 CH3 CH3


(a) 1 (b) 2 (c) 3 (d) 4 D D D
(d) = =
H SO
13. HC º C - COOH ¾¾2 ¾¾
4® X
. The compound X is
HgSO 4
AlCl3
15. CH3—CH2—CH2—Cl + C6 H6
(a) OHC × CH2 × COOH (b) HOCH = CHCOOH
air
(c) both (a) and (b) (d) CH2 = C(OH)COOH Major [X]
14. Correct order of rate of nitration H 3O
[X] can not be
CH3 CH3 CH3
(a) Phenol
D D D (b) CH3COCH3
(a) > > O

CH3 CH3 CH3 C CH3


(c)
D D D
(b) (d) all of these will not produce
> >

ANSW ER KEY
Exercise 1 : NCERT Based Topic-wis e MCQs
1 (c) 16 (c) 31 (c) 46 (b) 61 (a) 76 (a) 91 (c) 106 (c) 121 (b) 136 (b)
2 (c) 17 (b) 32 (a) 47 (d) 62 (a) 77 (c) 92 (a) 107 (c) 122 (d) 137 (a)
3 (d) 18 (b) 33 (d) 48 (d) 63 (a) 78 (c) 93 (b) 108 (d) 123 (c) 138 (d)
4 (d) 19 (c) 34 (d) 49 (b) 64 (d) 79 (c) 94 (d) 109 (d) 124 (b) 139 (d)
5 (a) 20 (b) 35 (a) 50 (c) 65 (b) 80 (b) 95 (b) 110 (c) 125 (a) 140 (d)
6 (c) 21 (b) 36 (a) 51 (c) 66 (c) 81 (c) 96 (b) 111 (c) 126 (b) 141 (a)
7 (b) 22 (a) 37 (d) 52 (d) 67 (b) 82 (c) 97 (b) 112 (c) 127 (a) 142 (a)
8 (a) 23 (c) 38 (b) 53 (d) 68 (a) 83 (d) 98 (b) 113 (d) 128 (d) 143 (a)
9 (d) 24 (b) 39 (a) 54 (c) 69 (a) 84 (a) 99 (a) 114 (a) 129 (a) 144 (b)
10 (d) 25 (b) 40 (c) 55 (a) 70 (a) 85 (b) 100 (a) 115 (a) 130 (d) 145 (c)
11 (a) 26 (d) 41 (c) 56 (d) 71 (c) 86 (b) 101 (a) 116 (a) 131 (a)
12 (b) 27 (c) 42 (c) 57 (b) 72 (d) 87 (b) 102 (a) 117 (c) 132 (c)
13 (c) 28 (a) 43 (d) 58 (a) 73 (a) 88 (a) 103 (c) 118 (b) 133 (d)
14 (c) 29 (c) 44 (c) 59 (b) 74 (d) 89 (c) 104 (c) 119 (d) 134 (a)
15 (d) 30 (a) 45 (a) 60 (b) 75 (b) 90 (d) 105 (b) 120 (c) 135 (b)
Exercise 2 : Numeric/Integer Answer Ques tions
1 (6) 2 (4) 3 (7) 4 (5) 5 (4) 6 (4) 7 (4) 8 (6) 9 (6) 10 (6)
Exercis e 3: NCERT Exemplar & Past Year MCQs
1 (d) 5 (a) 9 (d) 13 (b) 17 (a) 21 (a) 25 (c) 29 (c) 33 (d)
2 (a) 6 (d) 10 (c) 14 (c) 18 (b) 22 (d) 26 (a) 30 (c) 34 (a)
3 (b) 7 (c) 11 (b) 15 (d) 19 (d) 23 (d) 27 (c) 31 (b) 35 (b)
4 (a) 8 (b) 12 (c) 16 (a) 20 (d) 24 (d) 28 (d) 32 (c)
Exercise 4 : Problem Solving S kill Enhancer MCQs
1 (c) 3 (b) 5 (c) 7 (d) 9 (d) 11 (d) 13 (c) 15 (c)
2 (a) 4 (b) 6 (a) 8 (b) 10 (c) 12 (c) 14 (d)
Environmental
14 Chemistry
Trend Buster NEET & JEE Main

Number of Questions from 2020-15 3 6 Minimum one Question has been


Weightage 1.3% 3.4% asked every year in JEE M

The most Important Concepts that Cover Maximum number of Questions asked in past 6 years.
.

Environmental and atmospheric pollution 2 2


Water pollution — 4

Less Important Concepts that Cover 1 or 2 Questions asked in past 6 years.

Soil pollution 1 —
Industrial waste/green chemistry — —

NEET JEE

2020 — — — — — —
2019 Pollution Green house effect / global 1 Easy 2 Average /
warming / water pollution Easy
2018 Environmental and Atmospheric pollution / 1 Easy 1 Average
atmospheric pollution / water pollution
water pollution

2017 Soil pollution / water pollution Soil pollution / water pollution 1 Easy 1 Average
2016 Water pollution Water pollution — — 1 Average
2015 Environmental and Atmospheric pollution — — 1 Easy
atmospheric pollution
EBD_8350
198 CHEMISTRY
ENVIRONMENTAL CHEMISTRY 199
EBD_8350
200 CHEMISTRY

Problem Solving Tips/ Tricks/ Points to Remember

4 Inhalation of small particles irritates the lungs and 4 The process in which nutrient enriched water bodies
exposure to such particles for long periods of time causes support a dense plant population, which kills animal life
'scarring' or 'fibrosis' of the lung lining is termed as by depriving it of oxygen and results in subsequent loss
Pneumoconiosis. of biodiversity is known as Eutrophication.
4 In acid rain 4 International Standards for Drinking Water
H2SO4 = 60-70% (i) Fluoride: Soluble fluoride is added to drinking water
HNO3 = 30-40% to bring its concentration upto 1 ppm or 1 mg dm–3.
HCl = Very small amount. The F– ions make the enamel on teeth much harder
4 Detergents and Fertilizers : These may contain by converting hydroxylapatite, [3(Ca(PO4)2.Ca(OH)2],
phosphates The addition of phosphorus to water, in the the enamel on the surface of the teeth, into much
harder fluorapatite, 3[Ca 3(PO4)2.CaF2]. F– ion
form of the phosphate anion PO43– , encourages the
concentration above 2 ppm causes brown mottling
formation of algae, which reduces the dissolved oxygen
of teeth. Excess fluoride (over 10 ppm) causes
concentration water known as eutrophication.
harmful effect to bones and teeth.
4 Acid-polluted water (pH < 3): Water downstream from a (ii) Lead: The upper limit concentration of lead in drinking
mine contaminated by acid mine drainage. Acid mine water water is about 550 ppb.
principally contains sulphuric acid produced the oxidation (iii) Sulphate: Excessive SO 42– ions (>500 ppm) in
of iron pyrites (FeS2). drinking water causes laxative effect.
4 Polychlorinated biphenyls (PCBs): Having high (iv) Nitrate: The maximum limit of NO3– in drinking water
stabilities, PCBs used as fluids in transformer capacitors. is 550 ppm. Excess nitrate in drinking water can cause
PCBs are resistant to oxidation and their release into disease such as methemoglobinemia (‘blue baby’
environment causes skin disorders in humans. syndrome).
4 Herbicides: used to kill weeds. Examples are NaClO3 4 BOD: The amount of oxygen required by bacteria to
Na3AsO3 triazines etc. break down the organic matter present in a certain volume
4 Fungicides : They are used to stop or kill fungus e.g. of a sample of water. The amount of BOD in the water is
a measure of the amount of organic material in the water.
OH Clean water would have BOD value of less than 5 ppm
Cl Cl whereas highly polluted water could have a BOD value
of 17 ppm or more.
CuSO 4 4 How to Reduce Acid Rain: (i) Reduce the emission of
(a) copper sulphate (b) SO2 and NO2 in the atmosphere. (ii) Catalytic converters
Cl must be used in cars to reduce the effect of exhaust
2,4,6-trichlorophenol fumes on the atmosphere, (iii) Reduce the acidity of the
soil by adding powdered limestone to neutralize the
4 Rodenticides : used to kill rodents e.g. acidity of the soil.
Zn 3 P2 4 Noise is unwanted annoying sound of generally 80 dB
Zincphosphide
and above.
4 SMOG: There are two types of smog:
4 In 1984, the Bhopal gas tragedy took place because
(i) Classical smog: It is also called as reducing smog. methyl isocyanate (MIC) reacted with water.
(ii) Photochemical smog: It is also called as oxidising 4 Heat islands are produced due to air pollution.
smog.
4 Faecal pollution is indicated by Escherichia coli.
4 Eichhornia plant is used for the purification of water.
ENVIRONMENTAL CHEMISTRY 201

Exercise 1 : NCERT Based Topic-wise MCQs


Topic 1: Environmental and Atmospheric Pollution 9. Ozone is an important constituent of stratosphere because
it
1. What is DDT among the following ? (a) Destroys bacteria which are harmful to human life
(a) Greenhouse gas (b) Prevents the formation of smog over large cities
(b) A fertilizer (c) Removes poisonous gases of the atmosphere by
(c) Biodegradable pollutant reacting with them
(d) Non-biodegradable pollutant (d) Absorbs ultraviolet radiation which is harmful to
2. The gas emitted by supersonic jet planes that slowly human life
depletes the concentration of ozone layer is 10. SO2 is one of the air pollutants. SO2
(a) CO (b) NO (c) SO2 (d) O2 (a) is a lung irritant
3. Carbon monoxide (CO) is harmful to man because (b) dissolves in water to form acid rain
(a) it forms carbolic acid (c) both (a) and (b)
(b) it generates excess CO2 (d) none of the above
(c) it is carcinogenic 11. Formation of London smog takes place in
(d) it competes with O2 for haemoglobin (a) winter during day time
4. The greenhouse effect is because of the (b) summer during day time
(a) presence of gases, which in general are strong infrared (c) summer during morning time
absorbers, in the atmosphere (d) winter during morning time
(b) presence of CO2 only in the atmosphere 12. Assertion : Photochemical smog is oxidising in nature.
(c) pressure of O3 and CH4 in the atmosphere Reason : Photochemical smog contains NO2 and O3, which
(d) N2O and chlorofluorohydrocarbons in the atmosphere are formed during the sequence of reactions.
5. The irritant red haze in the traffic and congested places is (a) Assertion is correct, reason is correct; reason is a
due to presence of which of the following ? correct explanation for assertion.
(i) Oxides of sulphur (b) Assertion is correct, reason is correct; reason is not
(ii) Oxides of nitrogen a correct explanation for assertion
(c) Assertion is correct, reason is incorrect
(iii) Carbon dioxide
(d) Assertion is incorrect, reason is correct.
(iv) Mists, smoke and dust
13. Assertion : Ozone is destroyed by solar radiation in upper
(v) Smog
stratosphere.
(a) (i), (iv) and (v) (b) (iii) only Reason : Thinning of the ozone layer allows excessive
(c) (ii) only (d) (ii) and (v) UV radiations to reach the surface of earth.
6. Which of the following strategy is not a correct approach (a) Assertion is correct, reason is correct; reason is a
to reduce global warming ? correct explanation for assertion.
(a) Reducing the green house gas emission by limiting (b) Assertion is correct, reason is correct; reason is not
the use of fossil fuels a correct explanation for assertion
(b) Increase the vegetation cover particularly the forest (c) Assertion is correct, reason is incorrect
for photosynthetic utilization of CO2 (d) Assertion is incorrect, reason is correct.
(c) Minimizing the use of nitrogen fertilizers in agriculture 14. The secondary precursors of photochemical smog are
for reducing N2O emission (a) SO2 and NO2
(d) Increasing the use of air conditioners, refrigeration (b) SO2 and hydrocarbons
unit and production of plastic foams and propellants (c) NO2 and hydrocarbons
in aerosal spray cans (d) O3 and PAN
7. Classical smog occurs in places of 15. High concentration of which of the following in
(a) excess SO2 (b) low temperature atmosphere leads to stiffness of flower buds which
(c) high temperature (d) excess NH3 eventually fall off from plants?
8. The smog is essentially caused by the presence of (a) NO2 (b) SO2
(a) oxides of sulphur and nitrogen (c) CFC (d) Smog
(b) O2 and N2 16. The substance which is not regarded as a pollutant?
(c) O2 and O3 (a) NO2 (b) CO2
(d) O3 and N2 (c) O3 (d) Hydrocarbons
EBD_8350
202 CHEMISTRY

17. Which of the following is/are the hazardous pollutant(s) (c) Ozone absorbs infrared radiation.
present in automobile exhaust gases? (d) Oxides of nitrogen in the atmosphere can cause the
(i) N2 (ii) CO depletion of ozone layer.
(iii) CH4 (iv) Oxides of nitrogen 27. Which of the following chemical, harmful to ozone, is
(a) (ii) and (iii) (b) (i) and (ii) released by chlorofluoro carbon?
(c) (ii) and (iv) (d) (i) and (iii) (a) Sulphur dioxide (b) Fluorine
18. Increase in global temperature increases the incidence of (c) Chlorine (d) Nitrogen dioxide
which of the following infectious disease(s) 28. In Antarctica ozone depletion is due to the formation of
(i) Sleeping sickness (ii) Yellow fever following compound
(iii) Malaria (iv) Dengue (a) acrolein (b) peroxyacetyl nitrate
(a) (ii) only (b) (i) and (ii)
(c) SO2 and SO3 (d) chlorine nitrate
(c) (iii) and (iv) (d) (i), (ii), (iii) and (iv)
29. The gas(es) not responsible for ozone depletion :
19. Green house gases
(a) NO and freons (b) SO2
(a) allow shorter wavelength to enter earth's atmosphere
(c) CO2 (d) Both (b) and (c)
while doesn't allow longer wavelength to leave the
earth's atmosphere. 30. Which of the following statement(s) is/are correct ?
(b) allow longer wavelength to enter earth atmosphere (i) Classical smog is a mixture of smoke, fog and
while doesn't allow shorter wavelength to leave the sulphur dioxide.
surface (ii) Classical smog is also called oxidising smog
(c) don't have wavelength specific character. (iii) Hydrocarbons, NO2 and PAN are components of
(d) show wavelength specific behaviour near the earth photochemical smog.
while far from earth these have wavelength (a) (i) and (iii) (b) (i) and (ii)
independent behaviour. (c) (iii) only (d) (i), (ii) and (iii)
20. The substance having the largest concentration in acid 31. In which of the following regions hydrogen and helium
rain is are found ?
(a) H2CO3 (b) HNO3 (c) HCl (d) H2SO4 (a) Stratosphere (b) Mesosphere
21. Which of the following green house gas is released in (c) Exosphere (d) Troposphere
paddy field ? 32. Assertion : Uncatalysed oxidation of sulphur dioxide is
I. CFCs II. CH4 a slow process.
III. SO2 Reason : Particulate matter in polluted air catalyses the
(a) Only I (b) Only II oxidation of sulphur dioxide.
(c) Only III (d) I and II (a) Assertion is correct, reason is correct; reason is a
22. The main element of smog is correct explanation for assertion.
(a) O3 and PAN (b) O3 (b) Assertion is correct, reason is correct; reason is not
(c) PAN (d) PPN and PBN a correct explanation for assertion
23. Photochemical smog formed in congested metropolitan (c) Assertion is correct, reason is incorrect
cities mainly consists of
(d) Assertion is incorrect, reason is correct.
(a) ozone, peroxyacetyl nitrate and NOx
33. Which of the following is the major cause of global
(b) smoke, peroxyacetyl nitrate and SO2
warming?
(c) hydrocarbons, SO2 and CO2
(a) re-radiation of U.V. rays by CO2 and H2O
(d) hydrocarbons, ozone and SOx
24. In almost all Indian metropolitan cities like Delhi, the major (b) re-radiation of I.R. rays by CO2 and H2O
atmospheric pollutant(s) is/are (c) re-radiation of I.R. rays by O2 and N2
(a) suspended particulate matter (SPM) (d) re-radiation of U.V. rays by O2 and N2
(b) oxides of sulphur 34. Assertion : Suspended particulate matter (SPM) is an
(c) carbon dioxide and carbon monoxide important pollutant released by diesel vehicles.
(d) oxides of nitrogen Reason : Catalytic converters greatly reduce pollution
25. The non-viable particulate among the following is caused by automobiles.
(a) dust (b) bacteria (a) If Assertion is correct but Reason is incorrect.
(c) moulds (d) fungi (b) If both the Assertion and Reason are incorrect.
26. Identify the incorrect statement from the following : (c) If both Assertion and Reason are correct and Reason
(a) Ozone absorbs the intense ultraviolet radiation of the is the correct explanation of Assertion.
sun. (d) If both Assertion and Reason are correct, but Reason
(b) Depletion of ozone layer is because of its chemical is not the correct explanation of Assertion.
reactions with chlorofluoro alkanes.
ENVIRONMENTAL CHEMISTRY 203

35. Which of the following is/are formed when ozone reacts 43. Match the columns
with the unburnt hydrocarbons in polluted air? Column-I Column-II
(i) Formaldehyde (ii) Acrolein (A) Oxides of sulphur (p) Global warming
(iii) Peroxyacetyl nitrate (iv) Formic acid (B) Nitrogen dioxide (q) Damage to kidney
(a) (i) and (iv) (b) (ii) only
(C) Carbon dioxide (r) ‘Blue baby’ syndrome
(c) (iii) only (d) (i), (ii) and (iii)
36. The region containing water vapour is (D) Nitrate in drinking (s) Respiratory diseases
(a) thermosphere (b) stratosphere water
(c) troposphere (d) mesosphere (E) Lead (t) Red haze in traffic and
37. Which of the following statements is incorrect ? congested areas
(a) Smoke particulates consist of solid or mixture of (a) A – (t), B – (p), C – (r), D – (s), E – (q)
solid and liquid particles formed during combustion (b) A – (s), B – (t), C – (p), D – (r), E – (q)
of organic matter. (c) A – (s), B – (q), C – (p), D – (t), E – (r)
(b) Herbicides and insecticides that miss their target (d) A – (q), B – (s), C – (t), D – (r), E – (p)
and travel through air form mists.
44. Which of the following statements about polar
(c) Organic solvents, metals and metallic oxides form
fume particles stratosphere clouds (PSCs) is not correct?
(d) None of these (a) PSCs do not react with chlorine nitrate and HCl
38. The biggest particulate matter is (b) Type I clouds are formed at about –77ºC and contain
(a) HNO3 droplets (b) Soot solid HNO3 . 3H2O
(c) H2SO4 droplets (d) Fly ash (c) Type II clouds are formed at about –85ºC and contain
39. The aromatic compounds present as particulates are some ice
(a) polycyclic aromatic hydrocarbons (d) A tight whirlpool of wind called Polar Vortex is formed
(b) benzene which surrounds Antarctica
(c) toluene 45. The false statement among the followings :
(d) nitrobenzene
(a) The average residence time of NO is one month
40. Which of the following can control the photochemical
smog ? (b) Limestone acts as a sink for SOx
(A) Use of catalytic converters in automobiles. (c) SOx can be removed from flue gases by passing
(B) Plantation of trees like pinus, pyrus, vitis etc. through a solution of citrate ions
(C) Using less sulphur containing fossil fuels. (d) Ammonia acts as a sink for NOx
(a) A and C (b) B 46. Identify the wrong statement in the following:
(c) A and B (d) A, B and C (a) Chlorofluorocarbons are responsible for ozone layer
41. Which of the following statement(s) is / are correct ? depletion
(i) Sulphuric acid, nitric acid as well as ammonium salts (b) Greenhouse effect is responsible for global warming
are components of acid rain.
(c) Acid rain is mostly because of oxides of nitrogen and
(ii) Formation of acid rain can be reduced by using less
sulphur content fossil fuels for power plants and sulphur
industries. (d) Ozone layer does not permit infrared radiation from
(iii) Catalytic converters must be used in cars to reduce the sun to reach the earth
the harmful effect of exhaust. 47. Identify the incorrect statement from the following :
(iv) Main component of catalytic converter is ceramic (a) Ozone absorbs the intense ultraviolet radiation of the
honey comb coated with metals like – Au, Ag, Pt sun.
etc. (b) Depletion of ozone layer is because of its chemical
(a) (i), (ii) and (iii) (b) (ii) and (iii) reactions with chlorofluoro alkanes.
(c) (ii), (iii) and (iv) (d) (i), (ii), (iii) and (iv) (c) Ozone absorbs infrared radiation.
42. Match the columns
(d) Oxides of nitrogen in the atmosphere can cause the
Column I Column II
(A) Acid rain (p) CHCl2 – CHF2 depletion of ozone layer.
(B) Photochemical smog (q) CO Topic 2. Water Pollution
(C) Combination with (r) CO2
haemoglobin 48. Minamata disease of Japan is due to pollution of
(D) Depletion of ozone (s) SO2 (a) Aresenic (b) Lead
layer (t) Unsaturated (c) Cynide (d) Mercury
hydrocarbons 49. The high amount of E. coli in water is the indicator of
(a) A – (r, s), B – (t), C – (q), D – (p) (a) hardness of water
(b) A – (r), B – (s), C – (q), D – (p) (b) industrial pollution
(c) A – (t,s), B – (s), C – (q), D – (r) (c) sewage pollution
(d) A – (r), B – (t), C – (q), D – (p) (d) presence of chlorine in water
EBD_8350
204 CHEMISTRY

50. Which of the following does not occur when the sewage 61. Phosphate fertilizers when added to water leads to
is discharged into water ? (a) increased growth of decomposers
(a) Increase in O2 (b) reduced algal growth
(b) Cyanophycean blooms occur (c) increased algal growth
(c) Depletion of O2 layers (d) nutrient enrichment (eutrophication)
(d) Eutrophication 62. BOD of pond is connected with
51. Which of the following metal is a water pollutant and (a) microbes & organic matter
causes sterility in human being (b) organic matter
(a) As (b) Mn (c) Mg (d) Hg (c) microbes
52. When rain is accompanied by a thunderstorm, the (d) none of these
collected rain water will have a pH value 63. Eutrophication causes reduction in
(a) Slightly lower than that of rain water without (a) dissolved oxygen (b) nutrients
thunderstorm (c) dissolved salts (d) All of the above
(b) Slightly higher than that when the thunderstorm is 64. Assertion : If BOD level of water in a reservoir is less
not there
than 5 ppm it is highly polluted.
(c) Uninfluenced by occurrence of thunderstorm
Reason : High biological oxygen demand means low
(d) Which depends upon the amount of dust in air
activity of bacteria in water.
53. Which causes death of fish in water bodies polluted by
sewage? (a) Assertion is correct, reason is correct; reason is a
(a) Foul smell (b) Pathogens correct explanation for assertion.
(c) Herbicides (d) Decrease in D.O. (b) Assertion is correct, reason is correct; reason is not
54. B.O.D. test or biochemical oxygen demand test is made for a correct explanation for assertion
measuring (c) Assertion is correct, reason is incorrect
(a) air pollution (b) water pollution (d) Assertion is incorrect, reason is correct.
(c) noise pollution (d) soil pollution 65. Sewage mostly constitutes
55. Brewery and sugar factory waste alters the quality of a (a) Non-biodegradable pollutants
water body by increasing (b) Biodegradable pollutants
(a) temperature (b) turbidity (c) Effluents
(c) pH (d) COD and BOD (d) Air pollutants
56. The maximum prescribed concentration of cadmium in 66. Assertion : The F – ions make the enamel on teeth much
drinking water in ppm is harder.
(a) 0.05 (b) 3 (c) 2 (d) 0.005 Reason : F– ions converts hydroxyapatite
57. A dental disease characterised by mottling of teeth is due [3Ca(PO4)2 Ca(OH)2] into fluorapatite [3Ca3(PO4)2. CaF2].
to presence of a certain chemical element in drinking water. (a) Assertion is correct, reason is correct; reason is a
Which is that element? correct explanation for assertion.
(a) Boron (b) Chlorine (b) Assertion is correct, reason is correct; reason is not
(c) Fluorine (d) Mercury a correct explanation for assertion
58. A lake with an inflow of domestic sewage rich in organic (c) Assertion is correct, reason is incorrect
waste may result in (d) Assertion is incorrect, reason is correct.
(a) drying of the lake very soon due to algal bloom 67. Which is known as ‘Third poison of environment’ and
(b) an increase production of fish due to lot of nutrients also creates ‘Blue baby syndrome’
(c) death of fish due to lack of oxygen
(a) Nitrate present in water
(d) increased population of aquatic food web organisms
(b) Phosphate and detergents found in water
59. Which one of the following statement is not true ?
(c) Cyanide
(a) pH of drinking water should be between 5.5 – 9.5.
(d) Pesticides
(b) Concentration of DO below 6 ppm is good for the
growth of fish. 68. Assertion : Eutrophication shows increase in productivity
(c) Clean water would have a BOD value of less than 5 in water.
ppm. Reason : With increasing eutrophication, the diversity of
(d) Oxides of sulphur, nitrogen and carbon are the most the phytoplankton increases.
widespread air pollutant. (a) Assertion is correct, reason is correct; reason is a
60. Limit of BOD prescribed by Central pollution Control Board correct explanation for assertion.
for the discharge of industrial and municipal waste waters (b) Assertion is correct, reason is correct; reason is not
into natural surface waters, is a correct explanation for assertion
(a) < 100 ppm (b) < 30 ppm (c) Assertion is correct, reason is incorrect
(c) < 3.0 ppm (d) < 10 ppm (d) Assertion is incorrect, reason is correct.
ENVIRONMENTAL CHEMISTRY 205

69. Which of the following statements are not correct? (B) Concentration of (q) 17 ppm
(i) F– ion concentration above 2 ppm causes brown dissolved oxygen
mottling in teeth. below which growth
(ii) Excessive F– (over 10 ppm) causes harmful effect to of fish gets inhibited
bones and teeth. (C) BOD value of clean (r) 5 ppm
(iii) Excessive lead in drinking water causes disease water
methemoglobinemia
(D) BOD value of (s) 10 ppm
(iv) Deficiency of sulphate in drinking water causes
polluted water.
laxative effect.
(a) A – (s), B – (s), C – (q), D – (p)
(a) (ii) and (iv) (b) (ii) and (iii)
(b) A – (p), B – (q), C – (r), D – (s)
(c) (ii), (iii) and (iv) (d) (iii) and (iv)
(c) A – (s), B – (p), C – (r), D – (q)
70. Lichens do not like to grow in cities
(d) A – (p), B – (s), C – (q), D – (r)
(a) because of absence of the right type of algae and
fungi 78. Negative soil pollution is
(b) because of lack of moisture (a) reduction in soil productivity due to erosion and over
(c) because of SO2 pollution use
(d) because natural habitat is missing (b) reduction in soil productivity due to addition of
pesticides and industrial wastes
Topic 3. Soil Pollution
(c) converting fertile land into barren land by dumping
71. Chief source of soil and water pollution is ash, sludge and garbage
(a) mining (b) agro industry (d) None of the above
(c) thermal power plant (d) All of the above 79. Which one of the following statements is correct ?
72. The effect of polluted water on soil is, that
(a) Extensive use of chemical fertilizers may lead to
(a) it decreases fertility
eutrophication of nearby water bodies
(b) it contaminates ground water
(b) Both Azotobacter and Rhizobium fix atmospheric
(c) it renders soil acidic or basic
(d) all of the above nitrogen in root nodules of plants
73. Soil is polluted by (c) Cyanobacteria such as Anabaena and Nostoc are
I. pesticides important mobilizers of phosphates and potassium
II. synthetic fertilizers for plant nutrition in soil
III. green manure (d) At present it is not possible to grow maize without
Choose the correct option. chemical fertilizers
(a) I and III (b) I and II 80. Match the columns
(c) II and III (d) I, II and III Column I Column II
74. Which of the following trophic level has least (A) Phosphate fertilisers (p) BOD level of water
concentration of toxins deposition ? in water increases
(a) Aquatic plant (b) Small fish (B) Methane in air (q) Acid rain
(c) Human being (d) Largest fish (C) Synthetic detergents (r) Global warming
75. Assertion : Excessive use of chlorinated synthetic
in water
pesticides causes soil and water pollution.
(D) Nitrogen oxides in air (s) Eutrophication
Reason : Such pesticides are non-biodegradables.
(a) A – (p,s), B – (r), C – (p), D – (q)
(a) Assertion is correct, reason is correct; reason is a
(b) A – (p), B – (s), C – (r), D – (q)
correct explanation for assertion.
(c) A – (s), B – (r), C – (q), D – (p)
(b) Assertion is correct, reason is correct; reason is not
(d) A – (p), B – (q), C – (s), D – (r)
a correct explanation for assertion
(c) Assertion is correct, reason is incorrect Topic 4. Industrial waste, strategies to control
(d) Assertion is incorrect, reason is correct.
Environmental pollution, Green chemistry
76. Thermal pollution affects mainly
(a) vegetation (b) aquatic creature 81. “Reducing potentially hazardous waste through smarter
(c) rocks (d) air production”.
77. Match the columns This represents a great step forward for
Column - I Column - II (a) green revolution
(A) Concentration of (p) 6 ppm (b) green chemistry
dissolved oxygen in (c) industrial revolution
cold water (d) green biotechnology
EBD_8350
206 CHEMISTRY

82. Which of the following statement(s) is/are true about (b) reduce the use and production of hazardous chemicals
waste recycling? (c) are related to the depletion of ozone layer
(i) Clothes can be made from recycled plastic waste. (d) study the reactions in plants
(ii) Fuel that has high octane rating and contains no 84. Use of which of the following solvent in dry cleaning
lead can be obtained from plastic waste. will result in less harm to ground water?
(iii) Technology has now been developed to produce (a) Cl2C = CCl2 (b) Liquid CO2
electricity from the garbage. (c) H2O2 (d) None of these
(a) (ii) only (b) (ii) and (iii) 85. Synthesis of ethanal commercially from which of the
(c) (iii) only (d) All of these following reagent is the part of green chemistry?
83. Green chemistry means such reactions which : (a) CH3 CH2OH (b) CH2 = CH2
(a) produce colour during reactions (c) HC º CH (d) All of these

NCERT Exemplar MCQs 6. Which of the following statements is correct?


(a) Ozone hole is a hole formed in stratosphere from which
1. Photochemical smog occurs in warm, dry and sunny
ozone oozes out
climate. One of the following is not amongst the
(b) Ozone hole is a hole formed in troposphere from which
components of photochemical smog, identify it.
ozone oozes out
(a) NO2
(c) Ozone hole is thinning of ozone layer of stratosphere
(b) O3
at some places
(c) SO2
(d) Ozone hole means vanishing of ozone layer around
(d) Unsaturated hydrocarbon
the earth completely
2. Which of the following statements is not true about 7. Which of the following practices will not come under green
classical smog? chemistry?
(a) Its main components are produced by the action of (a) If possible, making use of soap made of vegetable
sunlight on emissions of automobiles and factories oils instead of using synthetic detergents.
(b) Produced in cold and humid climate (b) Using H2O2 for bleaching purpose instead of using
(c) It contains compounds of reducing nature chlorine based bleaching agents
(d) It contains smoke, fog and sulphur dioxide (c) Using bicycles for travelling small distances instead
3. Sewage containing organic waste should not be disposed of using petrol/ diesel based vehicles
in water bodies because it causes major water pollution. (d) Using plastic cans for neatly storing substances
Fishes in such a polluted water die because of
8. Which of the following gases in not a green house gas?
(a) large number of mosquitoes
(a) CO (b) O3
(b) increase in the amount of dissolved oxygen
(c) CH4 (d) H2O vapour
(c) decrease in the amount of dissolved oxygen in water
9. Biochemical Oxygen Demand, (BOD) is a measure of
(d) clogging of gills by mud
organic material present in water. BOD value less than 5
4. Which of the following statements about photochemical
ppm indicates a water sample to be
smog is wrong?
(a) rich in dissolved oxygen
(a) It has high concentration of oxidising agents
(b) poor in dissolved oxygen
(b) It has low concentration of oxidising agent
(c) highly polluted
(c) It can be controlled by controlling the release of NO2,
(d) not suitable for aquatic life
hydrocarbons, ozone etc 10. Which of the following statement(s) is/are wrong?
(d) Plantation of some plants like pinus helps in (a) Ozone is not responsible for green house effect
controlling photochemical smog (b) Ozone can oxidise sulphur dioxide present in the
5. The gaseous envelope around the earth is known as atmosphere to sulphur trioxide
atmosphere. The lowest layer of this is extended upto 10 (c) Ozone hole is thinning of ozone layer present in
km from sea level, this layer is stratosphere
(a) stratosphere (b) troposphere (d) Ozone is produced in upper stratosphere by the action
(c) mesosphere (d) hydrosphere of UV rays on oxygen
ENVIRONMENTAL CHEMISTRY 207

11. Dinitrogen and dioxygen are main constituents of air but 16. Which of the following is a sink for CO ? [NEET 2017, C]
these do not react with each other to form oxides of nitrogen (a) Microorganism present in the soil
because ............... . (b) Oceans
(a) the reaction is endothermic and requires very high (c) Plants
temperature
(d) Haemoglobin
(b) the reaction can be initiated only in presence of a
catalyst 17. A water sample has ppm level concentration of following
(c) oxides of nitrogen are unstable anions
(d) N2 and O2 are unreactive F– = 10; SO42– = 100; NO3– = 50
12. The pollutants which come directly in the air from sources the anion/anions that make/makes the water sample
are called primary pollutants. Primary pollutants are unsuitable for drinking is/are : [JEE M 2017, S]
sometimes converted into secondary pollutants. Which (a) only NO3– (b) both SO42– and NO3–
of the following belongs to secondary air pollutants? (c) only F– (d) only SO42–
(a) CO (b) Hydrocarbon
18. Which oxide of nitrogen is not a common pollutant
(c) Peroxyacetyl nitrate (d) NO
introduced into the atmosphere both due to natural and
Past Year MCQs human activity? [NEET 2018, S]
13. Which of the following is not a common component of (a) N2O5 (b) NO2 (c) NO (d) N2O
Photochemical Smog? [AIPMT 2014, S] 19. The recommended concentration of fluoride ion in drinking
(a) Ozone (b) Acrolein water is up to 1ppm as fluoride ion is required to make
(c) Peroxyacetyl nitrate (d) Chlorofluorocarbons teeth enamel harder by converting [3Ca3(PO4)2. Ca(OH)2]
14. Assertion: Nitrogen and oxygen are the main components to : [JEE M 2018, S]
in the atmosphere but these do not react to form oxides of (a) [CaF2] (b) [3(CaF2).Ca(OH)2]
nitrogen. [JEE M 2015, S] (c) [3Ca3(PO4)2.CaF2] (d) [3{(Ca(OH)2}.CaF2]
Reason: The reaction between nitrogen and oxygen 20. A water sample has ppm level concentration of the
requires high temperature. following metals: Fe = 0.2; Mn = 5.0; Cu = 3.0; Zn = 5.0.
(a) The assertion is incorrect, but the reason is correct The metal that makes the water sample unsuitable for
(b) Both the assertion and reason are incorrect drinking is: [JEE M 2019, S]
(c) Both assertion and reason are correct, and the reason (a) Cu (b) Mn (c) Fe (d) Zn
is the correct explanation for the assertion 21. Excessive release of CO2 into the atmosphere results in:
(d) Both assertion and reason are correct, but the reason [JEE M 2019, C]
is not the correct explanation for the assertion (a) global warming (b) polar vortex
15. The concentration of fluoride, lead, nitrate and iron in a
(c) formation of smog (d) depletion of ozone
water sample from an underground lake was found to be
1000 ppb, 40 ppb, 100 ppm and 0.2 ppm, respectively. This 22. Among the following, the one that is not a green house
water is unsuitable for drinking due to high concentration gas is: [NEET 2019, C]
of : [JEE M 2016, S] (a) nitrous oxide (b) methane
(a) Nitrate (b) Iron (c) Fluoride (d) Lead (c) ozone (d) sulphur dioxide

1. Recent reports of acid rains in industrial cities are due to 2. Which one of the following substances used in dry
the effect of atmospheric pollution by: cleaning is a better strategy to control environmental
(a) Excessive release of NO2 and SO2 by burning of fossil pollution ?
fuels (a) Sulphur dioxide (b) Carbon dioxide
(b) Excessive release of CO2 by burning of fuel like wood (c) Nitrogen dioxide (d) Tetrachloroethylene
and charcoal, cutting of forests and increased animal 3. In coming years, skin related disorders will be more
population common due to
(c) Excessive release of NH3 by industrial plants and (a) pollutants in air
coal gas (b) use of detergents
(d) Excessive release of CO in atmosphere by incomplete (c) water pollution
combustion of coke, charcoal and other (d) depletion of ozone layer
carbonaceous fuels in paucity of oxygen
EBD_8350
208 CHEMISTRY

4. Water sample is reported to be highly polluted if BOD (c) H2SO4 is major contributor to acid rain, HNO3 ranks
(Biological Oxygen Demand) value of sample becomes second and HCl third in this respect
(a) more than 17 ppm (b) equal to 10 ppm (d) Fishes grow as well in warm as in cold water
(c) equal to 5 ppm (d) less than 5 ppm
7. Most hazardous metal pollutant of automobile exhaust is
5. What is the concentration of dissolved oxygen in cold
water ? (a) Mercury (b) Tin
(a) 5 ppm (b) 10 ppm (c) Cadmium (d) Lead
(c) 200, 000 ppm (d) 100 ppm 8. Green house gases can be arranged in ‘Global Warming
6. Which of the following statement is wrong? Potential’ sequence as
(a) Polar stratospheric clouds (PSCs) are clouds formed (a) N2O > CFC > CH4 > CO2
over Antarctica (b) CFC > N2O > CH4 > CO2
(b) Acid rain dissolves heavy metals such as Cu, Pb, Hg (c) CFC > CO2 > N2O > CH4
and Al from soil, rocks and sediments. (d) CO2 > CFC > N2O > CH4

ANSWER KEY
Exercise 1 : NCERT Based Topic-wise MCQs
1 (d) 10 (c) 19 (a) 28 (d) 37 (d) 46 (d) 55 (d) 64 (c) 73 (b) 82 (d)
2 (b) 11 (d) 20 (d) 29 (d) 38 (d) 47 (c) 56 (d) 65 (b) 74 (a) 83 (b)
3 (d) 12 (a) 21 (b) 30 (a) 39 (a) 48 (d) 57 (c) 66 (a) 75 (a) 84 (b)
4 (a) 13 (d) 22 (a) 31 (c) 40 (c) 49 (c) 58 (c) 67 (a) 76 (b) 85 (b)
5 (c) 14 (d) 23 (a) 32 (a) 41 (d) 50 (a) 59 (b) 68 (b) 77 (c)
6 (d) 15 (b) 24 (a) 33 (b) 42 (a) 51 (b) 60 (b) 69 (d) 78 (a)
7 (b) 16 (b) 25 (a) 34 (b) 43 (b) 52 (a) 61 (d) 70 (c) 79 (a)
8 (a) 17 (c) 26 (c) 35 (d) 44 (a) 53 (d) 62 (a) 71 (d) 80 (a)
9 (d) 18 (d) 27 (c) 36 (c) 45 (a) 54 (b) 63 (a) 72 (d) 81 (b)
Exercise 2 : NCERT Exemplar & Past Year MCQs
1 (c) 4 (b) 7 (d) 10 (a) 13 (d) 16 (a) 19 (c) 22 (d)
2 (a) 5 (b) 8 (a) 11 (a) 14 (a) 17 (c) 20 (b)
3 (c) 6 (c) 9 (a) 12 (c) 15 (a) 18 (a) 21 (a)
Exercise 3 : Problem Solving Skill Enhancer MCQs
1 (a) 2 (b) 3 (d) 4 (a) 5 (b) 6 (d) 7 (d) 8 (b)
15 The Solid State

Trend Buster NEET & JEE Main

Number of Questions from 2020-15 5 4 This chapter has equal weightage


Weightage 1.5% 2.3% for NEET & JEE M.

The most Important Concepts that Cover Maximum number of Questions asked in past 6 years.

Packing Efficiency 2 2

Less Important Concepts that Cover 1 or 2 Questions asked in past 6 years.

Closed Packed Structure 2 —


Calculation Involving Unit Cell Dimensions 1 —
Imperfection in Solids — 1
Electrical & Magnetic Properties — 1
Amorphous & Crystaline Solids and their — —
Classification / Crystal Lattice & Unit Cells / Number
of atoms in Unit Cell

NEET JEE

2020 Closed Packed Structure Relation between a and r 1 Easy — —


2019 Closed Packed Structure / Based on Tetrahedral Voids 1 Average 1 Easy
Electrical & Magnetic & Octahedral Voids /
Properties Piezoelectric Materials

2018 Calculation Involving Unit Density ratio of bcc & fcc 1 Difficult 1 Easy
Cell Dimensions / system / Frenkel Defects
Imperfection in Solids

2017 Packing Efficiency Radius of fcc system — — 1 Average

2016 — — — — — —

2015 Packing Efficiency Radius of fcc & bcc system / 2 Average 1 Average
Packing efficiency of bcc system
EBD_8350
210 CHEMISTRY
THE SOLID STATE 211
EBD_8350
212 CHEMISTRY

Problem Solving Tips/ Tricks/ Points to Remember


4 Miller Indices 4 Structure of ionic crystals
a (i) AB type: Rock salt (NaCl) type, CsCl-type zinc
d hkl = blend (ZnS) or sphalerite-type structure.
h2 + k 2 + l 2
(ii) AB2 type: Fluorite type, e.g., CaF2, BaF2
a = length of cube side and h, k and l are Miller indices. (iii) A2B type: Antifluorite type, e.g., Na2O
4 Non Stoichiometric Point Defects
4 Types of lonic Crystal :
(i) Metal excess (due to anion vacancy)
F-Centres Crystal structure type Location of particle
NaCl is yellow AB or Rock Salt (NaCl) Cl– : fcc lattice,
Na+ : octahedral voids
KCl is violet
(ii) Metal excess due to interstitial cations Zinc blende or Zns S2– : fcc lattice
Zn2+ : alternate tetrahedral voids
e.g., ZnO
AB2 or Fluorite type Ca2+ : fcc lattice,
(iii) Metal deficiency due to cation vacancy
(CaF2) F– : all tetrahedral voids
FeO
A2B or anti-fluoritetype O2– : fcc lattice,
4 All noble gases have ccp structure except He which has
(Na2O) Na+ : all tetrahedral voids
hcp structure.
Caesium chloride (CsCl) Cl– : corners of cube
4 The space occupied by hard spheres in fcc and hcp is Cs+ : body centre of cube
74%, in bcc it is 68%, in simple cubic it is 52% and in
diamond it is 34%. Thus only fcc and hcp are close 4 Effect of temperature and pressure on the crystal
packed structures. Structures:
On the application of high pressure, NaCl-type structures
4 hcp is present in Be, Mg, Ca, Cr, Mo, V, Zn. (6 : 6 coordination) transform to CsCl-type structures
4 Lead zirconate (PbZrO3) is an example of anti-ferroelectric (8 : 8 coordination), while on heating reverse occurs.
crystal.
Pressure
4 ccp is present in Fe, Cu, Ag, Au, Pt, Al and Ni. ˆˆˆˆˆ† CsCl-type structures
NaCl-type structures ‡ˆˆˆˆˆ
760 K
(6:6 coordination) (8:8 coordination)
4 TiO2, NaCl and benzene are diamagnetic, Fe, Ni, Co, CrO2
are ferromagnetic, MnO is anti ferromagnetic, ferrites are 4 Number of TVs = 2 ´ OVs = 2 ´ Number of atoms in a
close packed structure = 8/unit cell
ferrimagnetic, Barium titanate (BaTiO3), sodium potassium
4 Number of TVs = 2 ´ Number of occupied voids
tartarate (Rochelle salt) and potassium dihydrogen
4 d–spacing for scc, fcc and bcc :
phosphate (KH2PO4) are ferroelectric solids.
4 Radius of TV = 0.225r a a
For scc : d100 : d110 : d111 = a : :
Radius of OV = 0.414r 2 3
where r is the radius of ions (mostly cations) in the
packing. a a a
For fcc : d200 : d220 : d111 = : :
4 Radius ratio rule 2 2 2 3

Radius of cation r+ r a a a
Radius ratio = = or c For bcc : d200 : d110 : d222 = : :
Radius of anion r- ra 2 2 2 3
THE SOLID STATE 213

Exercise 1 : NCERT Based Topic-wise MCQs

Topic 1: Amorphous and Crystalline Solids Reason: Amorphous solids have a tendency to flow.
(a) Assertion is correct, reason is correct; reason is a
1. Crystalline solids are anisotropic in nature. What is the correct explanation for assertion.
meaning of anisotropic in the given statement?
(b) Assertion is correct, reason is correct; reason is not
(a) A regular pattern of arrangement of particles which
a correct explanation for assertion
repeats itself periodically over the entire crystal.
(c) Assertion is correct, reason is incorrect
(b) Different values of some of physical properties are
(d) Assertion is incorrect, reason is correct.
shown when measured along different directions in
the same crystals. Topic 2: Classification of Crystalline Solids
(c) An irregular arrangement of particles over the entire 9. Which of the following exists as covalent crystals in the
crystal. solid state?
(d) Same values of some of physical properties are shown (a) Iodine (b) Silicon
when measured along different directions in the same (c) Sulphur (d) Phosphorus
crystals. 10. The major binding force of diamond, silicon and quartz is
2. A crystalline solid (a) electrostatic force
(a) changes abruptly from solid to liquid when heated (b) electrical attraction
(b) has no definite melting point (c) covalent bond force
(c) undergoes deformation of its geometry easily (d) non-covalent bond force
(d) has an irregular 3-dimensional arrangements 11. In graphite electrons are :
3. Which of the following is not a crystalline solid? (a) localised on each carbon atom
(a) KCl (b) CsCl (b) spread out between the sheets
(c) Glass (d) Rhombic S (c) localised on every third carbon atom
4. Which of the following statements about amorphous solids (d) present in antibonding orbital.
is incorrect ? 12. Which one of the following forms a molecular solid when
(a) They melt over a range of temperature solidified?
(b) They are anisotropic (a) Silicon carbide (b) Calcium fluoride
(c) There is no orderly arrangement of particles (c) Rock salt (d) Methane
(d) They can be compressible 13. Match the columns
5. Which of the following amorphous solid is used as Column-I Column-II
photovoltaic material for conversion of sunlight into (Type of solid) (Example of solid)
electricity? (A) Molecular solid (p) Ag
(a) Quartz glass (b) Quartz (B) Ionic solid (q) SiC
(c) Silicon (d) Both (a) and (b) (C) Metallic solid (r) CCl4
6. Which of the following statement(s) is/are correct? (D) Covalent solid (s) MgO
(i) Crystalline solids have definite characteristic (a) A – (s), B – (r), C – (p), D – (q)
geometrical shape.
(b) A – (q), B – (s), C – (p), D – (r)
(ii) Crystalline solids have long range order.
(c) A – (r), B – (q), C – (p), D – (s)
(iii) Sodium chloride and quartz glass are examples of
(d) A – (r), B – (s), C – (p), D – (q)
crystalline solids.
14. Which of the following type of solid has high melting
(iv) Crystalline solids are isotropic in nature.
point and do not conduct electricity but its aqueous
(a) (i), (ii) and (iii) (b) (i), (ii) and (iv)
solution and melt conduct electricity ?
(c) (i) and (ii) (d) (i) only
(a) Covalent (b) Ionic
7. Which of the following is true about the value of refractive
(c) Molecular (d) Metallic
index of quartz glass ?
(a) Same in all directions Topic 3: Crystal Lattice and Unit Cells
(b) Different in different directions 15. Potassium crystallizes with a
(c) Cannot be measured (a) body-centred cubic lattice
(d) Always zero (b) face-centred cubic lattice
8. Assertion: Glass panes fixed to windows or panes of old (c) simple cubic lattice
buildings are found to be slightly thicker at the bottom. (d) orthorhombic lattice
EBD_8350
214 CHEMISTRY

16. Tetragonal crystal system has the following unit cell 24. In a cubic lattice A atom occupy all the corners. If B atom
dimensions occupy one of the opposite faces, and atom C occupy the
(a) a = b = c, a = b = g = 90° remaining faces. The simplest formulae of the compound
(b) a = b ¹ c, a = b = g = 90° is
(c) a ¹ b ¹ c, a = b = g = 90° (a) ABC3 (b) ABC2 (c) ABC (d) AB2C
(d) a = b ¹ c, a = b = 90° g = 120° 25. A compound is formed by elements A and B. The crystalline
17. Match the columns cubic structure has the A atoms at the corners of the cube
Column-I Column-II and B atoms at the body centre. The simplest formula of
(Crystal system) (Compounds) the compound is
(A) Rhombohedral (p) KNO3 (a) AB (b) A6B (c) AB6 (d) A8B 4
(B) Orthorhombic (q) Zinc blende 26. When molten zinc is converted into solid state, it acquires
(C) Cubic (r) CdS hcp structure. The number of nearest neighbours of Zn
(D) Hexagonal (s) Calcite will be
(a) A – (p), B – (q), C – (s), D – (r) (a) 6 (b) 12 (c) 8 (d) 4
(b) A – (r), B – (p), C – (q), D – (s) 27. The number of octahedral voids present in a lattice is A
(c) A – (s), B – (p), C – (q), D – (r) . The number of tetrahedral voids generated is B . The
(d) A – (q), B – (r), C – (s), D – (p) number of closed packed particles in terms of A and B are:
(a) A- equal, B- half (b) A- twice, B- equal
Topic 4: Number of Atoms in Unit Cell
(c) A- twice , B- half (d) A- equal, B- twice
18. An element occuring in the bcc structure has 12.08 × 28. The arrangement ABC ABC .......... is referred to as
1023 unit cells. The total number of atoms of the element in (a) Octahedral close packing
these cells will be
(b) Hexagonal close packing
(a) 24.16 × 1023 (b) 36.18 × 1023
23 (c) Tetrahedral close packing
(c) 6.04 × 10 (d) 12.08 × 1023
(d) Cubic close packing
19. Which of the following statements(s) is/are incorrect?
29. Match Column-I (Type of close packed structure) with
(i) Only 1/8th portion of an atom located at corner of a
cubic unit cell is in its neighbouring unit cell. Column-II (Coordination number) and choose the correct
(ii) Total number of atoms per unit cell for a face centered option.
cubic unit cell is 3 . Column-I Column-II
(iii) Atom located at the body center is shared between (A) One dimensional close (p) 12
two adjacent unit cells. packed arrangement.
(a) (iii) only (b) (ii) only (B) Square close packing in (q) 6
(c) (i) and (ii) (d) (ii) and (iii) two dimensions.
20. Na and Mg crystallize in bcc and fcc type crystals (C) Two dimensional (r) 2
respectively, then the number of atoms of Na and Mg hexagonal close packing.
present in the unit cell of their respective crystal is (D) Cubic close packed (s) 4
(a) 4 and 2 (b) 9 and 14 (c) 14 and 9 (d) 2 and 4 arrangement.
21. The number of carbon atoms per unit cell of diamond unit (a) A – (r), B – (s), C – (q), D – (p)
cell is : (b) A – (r), B – (s), C – (p), D – (q)
(a) 8 (b) 6 (c) 1 (d) 4 (c) A – (s), B – (r), C – (q), D – (p)
Topic 5: Close-Packed Structure (d) A – (s), B – (q), C – (p), D – (r)
30. In the cubic close packing, the unit cell has ______.
22. A solid has a structure in which ‘W’ atoms are located at (a) 4 tetrahedral voids each of which is shared by four
the corners of a cubic lattice ‘O’ atoms at the centre of adjacent unit cells.
edges and Na atoms at the centre of the cube. The formula (b) 4 tetrahedral voids within the unit cell.
for the compound is (c) 8 tetrahedral voids each of the which is shared by
(a) Na2WO3 (b) Na2WO2 (c) NaWO2 (d) NaWO3 four adjacent unit cells.
23. A substance AxBy crystallizes in a face centred cubic (fcc)
(d) 8 tetrahedral voids within the unit cells.
lattice, in which atoms ‘A’ occupy each corner of the cube
31. Which of the following metal(s) show(s) hexagonal close
and atoms ‘B’ occupy the centres of each face of the cube.
packed structure (hcp) and which show face centred cubic
Identify the correct composition of the substance AxBy
(fcc) structure?
(a) AB3
(b) A4B 3 hcp fcc hcp fcc
(c) A3B (a) Ag, Zn Mg, Cu (b) Mg, Zn Ag, Cu
(d) Composition can’t be specified (c) Cu, Fe Al, Sn (d) Na, Li Zn, Cu
THE SOLID STATE 215

32. The Ca2+ and F– are located in CaF2 crystal, respectively at 39. The fraction of total volume occupied by the atoms present
face centred cubic lattice points and in in a simple cube is
(a) tetrahedral voids (b) half of tetrahedral voids p p p p
(a) (b) (c) (d)
(c) octahedral voids (d) half of octahedral voids 3 2 4 2 4 6
33. If Germanium crystallises in the same way as diamond,
40. Assertion : The packing efficiency is maximum for the fcc
then which of the following statement is not correct?
structure.
(a) Every atom in the structure is tetrahedrally bonded to
Reason : The cordination number is 12 in fcc structures.
4 atoms.
(a) Assertion is correct, reason is correct; reason is a
(b) Unit cell consists of 8 Ge atoms and co-ordination
correct explanation for assertion.
number is 4.
(b) Assertion is correct, reason is correct; reason is not
(c) All the octahedral voids are occupied.
a correct explanation for assertion
(d) All the octahedral voids and 50% tetrahedral voids
(c) Assertion is correct, reason is incorrect
remain unoccupied.
(d) Assertion is incorrect, reason is correct.
34. Assertion : In crystal lattice, the size of the tetrahedral
41. Packing efficiency by arrangement of atoms in two
hole is larger than an octahedral hole.
dimensional hexagonal close packing is
Reason : The cations occupy less space than anions in
(a) 60.43 (b) 65.78 (c) 59.78 (d) 68.76
crystal packing.
(a) Assertion is correct, reason is correct; reason is a Topic 7: Calculation Involving Unit Cell Dimensions
correct explanation for assertion. 42. Pottasium has a bcc structure with nearest neighbour
(b) Assertion is correct, reason is correct; reason is not distance 4.52 Å. Its atomic weight is 39. Its density (in kg
a correct explanation for assertion m–3) will be
(c) Assertion is correct, reason is incorrect (a) 454 (b) 804 (c) 852 (d) 910
(d) Assertion is incorrect, reason is correct. 43. Iron crystallizes in a b.c.c. system with a lattice parameter
35. Match the columns of 2.861 Å. Calculate the density of iron in the b.c.c. system
Column-I Column-II (Atomic weight of Fe = 56, NA = 6.02 × 1023 mol–1)
(A) Square close packing (p) Triangular voids (a) 7.92 g ml–1 (b) 8.96 g ml–1
(c) 2.78 g ml –1 (d) 6.72 g ml–1
in two dimensions
(B) Hexagonal close (q) Pattern of spheres is 44. A metal has an fcc lattice. The edge length of the unit cell
packing in two repeated every fourth is 404 pm. The density of the metal is 2.72 g cm–3. The
dimensions layer molar mass of the metal is :
(C) Hexagonal close (r) Coordination number 4 (NA Avogadro’s constant = 6.02 × 1023 mol–1)
packing in three (a) 30 g mol–1 (b) 27 g mol–1
(c) 20 g mol –1 (d) 40 g mol–1
dimensions
(D) Cubic close packing (s) Pattern of sphere is 45. The cubic unit cell of a metal (molar mass = 63.55g mol–1)
in three dimensions repeated alternate layers has an edge length of 362 pm. Its density is 8.92g cm–3.
(a) A – (r), B – (p), C – (s), D – (q) The type of unit cell is
(b) A – (p), B – (s), C – (q), D – (r) (a) primitive (b) face centered
(c) A – (s), B – (p), C – (q), D – (s) (c) body centered (d) end centered
(d) A – (r), B – (p), C – (s), D – (q) 46. A metal crystallizes in 2 cubic phases fcc and bcc whose
unit cell lengths are 3.5 Å and 3.0Å respectively. The ratio
Topic 6: Packing Efficiency of their densities is
36. Percentages of free space in cubic close packed structure (a) 0.72 (b) 2.04 (c) 1.46 (d) 3.12
and in body centered packed structure are respectively 47. If ‘a’ stands for the edge length of the cubic systems :
simple cubic, body centred cubic and face centred cubic,
(a) 30% and 26% (b) 26% and 32%
then the ratio of radii of the spheres in these systems will
(c) 32% and 48% (d) 48% and 26% be respectively,
37. A metallic crystal crystallizes into a lattice containing a
sequence of layers AB AB AB......Any packing of spheres 1 3 1 1 1
(a) a: a: a (b) a : 3a : a
leaves out voids in the lattice. What percentage of volume 2 4 2 2 2 2
of this lattice is empty space?
1 3 3
(a) 74% (b) 26% (c) a: a: a (d) 1a : 3a : 2a
2 2 2
(c) 50% (d) none of these.
48. Edge length of unit cell is 3.608 × 10–8 cm, which crystallizes
38. AB; crystallizes in a body centred cubic lattice with edge
in fcc and is determined to have a density of 8.92 g/cm3.
length ‘a’ equal to 387 pm. The distance between two The mass of four atoms is (g)
oppositely charged ions in the lattice is : (a) 4.18 × 10–22 (b) 1.67 × 10–21
(a) 335 pm (b) 250 pm (c) 200 pm (d) 300 pm (c) 2.09 × 10–22 (d) 8.37 × 10–22
EBD_8350
216 CHEMISTRY

49. The edge length of unit cell of a metal having molecular (b) Assertion is correct, reason is correct; reason is not
weight 75 g/mol is 5Å which crystallizes in cubic lattice. If a correct explanation for assertion
the density is 2g/cc, then find the radius of metal atom. (c) Assertion is correct, reason is incorrect
(NA = 6 × 1023). Give the answer in pm. (d) Assertion is incorrect, reason is correct.
(a) 217 pm (b) 210 pm (c) 220 pm (d) 205 pm 59. Which of the following statements is not correct ?
50. The number of atoms in 100 g of an fcc crystal with density, (a) Vacancy defect results in decrease in density of
d = 10 g/cm3 and cell edge equal to 100 pm, is equal to substance.
(a) 1 × 1025 (b) 2 × 1025 (c) 3 × 1025 (d) 4 × 1025 (b) Vacancy defect can develop when a substance is
51. A metallic element exists as cubic lattice. Each edge of the heated.
(c) Interstitial defect increases the density of the
unit cell is 2.88 Å. The density of the metal is 7.20 g cm–3.
substance.
How many unit cell will be present in 100 g of the metal?
(d) Ionic solids show interstitial defects only.
(a) 6.85 × 102 (b) 5.82 × 1023
5 60. When electrons are trapped into the crystal in anion
(c) 4.37 × 10 (d) 2.12 × 106
vacancy, the defect is known as :
52. CsBr crystallises in a body centered cubic lattice. The unit (a) Schottky defect (b) Frenkel defect
cell length is 436.6 pm. Given that the atomic mass of Cs = (c) Stoichiometric defect (d) F-centre
133 and that of Br = 80 amu and Avogadro number being 61. The appearance of colour in solid alkali metal halides is
6.02 × 1023 mol–1 the density of CsBr is generally due to
(a) 0.425 g/cm3 (b) 8.5 g/cm3 (a) Schottky defect (b) Frenkel defect
(c) 4.25 g/cm 3 (d) 82.5 g/cm3 (c) Interstitial positions (d) F-centre
Topic 8: Imperfections in Solids 62. In stoichiometric defects, the types of compound exhibit
Frenkel defects have/has
53. Schottky defect in crystals is observed when (a) Low co-ordination nos.
(a) an ion leaves its normal site and occupies an interstitial (b) High co-ordination
site (c) Small difference in the size of cations and anions
(b) unequal number of cations and anions are missing (d) None of these
from the lattice 63. The crystal with metal deficiency defect is
(c) density of the crystal increases (a) NaCl (b) FeO (c) KCl (d) ZnO
(d) equal number of cations and anions are missing from 64. Which of the following has Frenkel defects?
the lattice (a) Sodium chloride (b) Graphite
54. Which defect causes decrease in the density of crystal (c) Silver bromide (d) Diamond
(a) Frenkel (b) Schottky 65. Doping of AgCl crystals with CdCl2 results in
(c) Interstitial (d) F – centre (a) Frenkel defect
55. Frenkel and Schottky defects are : (b) Schottky defect
(a) nucleus defects (b) non-crystal defects (c) Substitutional cation vacancy
(c) crystal defects (d) nuclear defects (d) Formation of F - centres
56. Crystal defect indicated in the diagram below is 66. Which of the following statements is /are correct?
+ - +
Na Cl Na Cl Na Cl- + - (i) LiCl crystals are pink due to metal excess defect due
Cl– c Cl– Na+ c Na+ to presence of extra Li+ ion at interstitial sites.
Na+Cl– c Cl– Na+ Cl– (ii) Zinc oxide on heating turns yellow because its anionic
Cl– Na+ Cl–Na+ c Na+ sites are occupied by unpaired electrons.
(a) Interstitial defect (iii) In FeO crystals some Fe2+ are missing and the loss
(b) Schottky defect of positive charge is made up by the presence of
(c) Frenkel defect required number of Fe3+ ions.
(d) Frenkel and Schottky defects (a) (i) and (ii) (b) (i), (ii) and (iii)
57. In a solid lattice, the cation has left a lattice site and is (c) (ii) and (iii) (d) (iii) only
located at an interstitial position, the lattice defect is : 67. Match the columns
(a) Interstitial defect (b) Valency defect Column-I Column-II
(c) Frenkel defect (d) Schottky defect (A) Impurity defect (p) NaCl with anionic sites
58. Assertion : In any ionic solid (MX) with Schottky defects, F-centres
the number of positive and negative ions are same. (B) Metal excess defect (q) FeO with Fe3+
(C) Metal deficiency (r) NaCl with Sr2+ and some
Reason : Equal number of cation and anion vacancies are
defect cationic sites vacant
present.
(a) Assertion is correct, reason is correct; reason is a (a) A – (r), B – (p), C – (q) (b) A – (p), B – (q), C – (r)
correct explanation for assertion. (c) A – (r), B – (q), C – (p) (d) A – (q), B – (p), C – (r)
THE SOLID STATE 217

68. A group 1 hydride crystal when heated in presence of its 77. Which of the folliowing metal oxides is anti-ferromagnetic
constituent metal vapour shows pink color. This metal can be in nature?
(a) Na (b) K (c) Rb (d) Li (a) MnO2 (b) TiO2 (c) VO2 (d) CrO2
69. Assertion : Schottky defect is produced in solids when 78. Consider oxygen and chromium dioxide, both are placed
equal number of cations and anions are lost from lattice in magnetic field:
sites. This defect is observed in ionic crystals formed by (i) Oxygen is attracted strongly in magnetic field.
cations and anions of nearly equal size. (ii) Magnetic field persist in chromium dioxide while in
Reason : We observe Schottky defect in crystals of KCl, oxygen it is not.
NaCl. (a) Both Statements are correct.
(a) If both Assertion and Reason are correct and Reason (b) Statements (i) is correct only.
is the correct explanation of Assertion. (c) Statements (ii) is correct only.
(d) Both Statements are incorrect.
(b) If both Assertion and Reason are correct, but Reason
79. Match the columns
is not the correct explanation of Assertion.
Column-I Column-II
(c) If Assertion is correct but Reason is incorrect.
(A) Mg in solid state (p) p-Type semiconductor
(d) If both the Assertion and Reason are incorrect.
(B) MgCl2 in molten state (q) n-Type semiconductor
Topic 9: Electrical Properties and Magnetic Properties (C) Silicon with phosphorus (r) Electrolytic conductors
70. What is the energy gap between valence band and (D) Germanium with boron (s) Electronic conductors
(a) A – (q), B – (p), C – (r), D – (s)
conduction band in crystal of insulators ?
(b) A – (p), B – (q), C – (s), D – (r)
(a) Both the bands are overlapped with each other
(c) A – (s), B – (r), C – (q), D – (p)
(b) Very small
(d) A – (r), B – (s), C – (p), D – (q)
(c) Infinite
80. A solid with high electrical and thermal conductivity is
(d) Very large
(a) Si (b) Li (c) NaCl (d) Ice
71. Doping of silicon (Si) with boron (B) leads to :
81. Which of the following compound is like metallic copper
(a) n-type semiconductor (b) p-type semiconductor
in its conductivity and appearance?
(c) metal (d) insulator
(a) VO3 (b) TiO3 (c) ReO3 (d) CrO2
72. The addition of arsenic to germanium makes the latter a 82. White crystal of zinc oxide is heated
(a) metallic conductor (b) intrinsic semiconductor (i) Metal excess defect is created.
(c) mixed conductor (d) extrinsic semiconductor (ii) Crystal become p-type semiconductor
73. Magnetic moment of electron is due to which of the (iii) Crystal become yellow in color.
following reason? (iv) Free electron are created.
(a) Due to its orbital motion around the nucleus. (a) (i), (iii) and (iv)
(b) Due to its spin around its own axis. (b) (i), (ii) and (iii)
(c) Due to negative charge on electron. (c) (ii) and (iv)
(d) Both (a) and (b). (d) All statement(s) are correct.
83. Which of the following is ferroelectric compound?
74. Which of the following type of substances can be
(a) BaTlO3 (b) K4[Fe(CN)6]
permanently magnetised? (c) Pb2O3 (d) None of these
(a) Diamagnetic (b) Ferromagnetic 84. Which one of the following statements is correct?
(c) Ferrimagnetic (d) Antiferromagnetic (a) NaCl is a paramagnetic salt
75. Which of the following structure represents ferrimagnetism? (b) CuSO4 is a diamagnetic salt
(a) ­¯­¯­¯ (b) ­­­­­ (c) MnO is an example of ferromagnetic substance
(d) Ferrimagnetic substance like ZnFe 2O4 becomes
(c)¯¯¯¯¯ (d) ­ ­ ¯ ­ ­ ¯ paramagentic on heating
76. Assertion : Electrical conductivity of semiconductors 85. Match the columns
increases with increasing temperature. Column-I Column-II
Reason : With increase in temperature, large number of (Compound) (Magnetic Property)
electrons from the valence band can jump to the (A) NaCl (p) Ferrimagnetic
conduction band. (B) MnO (q) Paramagnetic
(C) CrCl3 (r) Ferromagnetic
(a) Assertion is correct, reason is correct; reason is a
(D) CrO2 (s) Diamagnetic
correct explanation for assertion.
(E) MgFe2O4 (t) Antiferromagnetic
(b) Assertion is correct, reason is correct; reason is not (a) A – (p), B – (r), C – (q), D – (t), E – (s)
a correct explanation for assertion (b) A – (t), B – (q), C – (r), D – (p), E – (s)
(c) Assertion is correct, reason is incorrect (c) A – (r), B – (t), C – (q), D – (p), E – (s)
(d) Assertion is incorrect, reason is correct. (d) A – (s), B – (t), C – (q), D – (r), E – (p)
EBD_8350
218 CHEMISTRY

1. The edge length of unit cell of a metal having molecular 8. A molecule A2B (Mwt. = 166.4) occupies triclinic lattice
weight 75 g/mol is 5Å which crystallizes in cubic lattice. with a = 5 Å, b = 8 Å, and c = 4 Å. If the density of AB2 is
If the density is 2 g/cc and the radius of metal atom is 5.2 g cm– 3, find the number of molecules present in one
43.3 x pm. (NA = 6 × 1023). Find the value of x. unit cell.
2. In face centred cubic (fcc) crystal lattice, edge length is
9. Consider the bcc unit cells of the solids 1 and 2 with the
400 pm. The diameter of greatest sphere is 29.29 d pm which
position of atoms are shown below. The radius of atom B
can be fit into the interstitial void without distortion of
is twice that of atom A. The unit cell edge length is 50%
lattice. Find the value of d.
3. AB; crystallizes in a body centred cubic lattice with edge more in solid 2 than in 1. What is the approximate packing
length ‘a’ equal to 387 pm. What is the distance between efficiency (in %) in solid 2?
two oppositely charged ions in the lattice?
4. If calcium crystallizes in bcc arrangement and the radius
of Ca atom is 96 pm, and the volume of unit cell of Ca is
x × 10–30 m3. Find the value of x.
5. The radii of Na+ and Cl– ions are 100 pm and 200 pm
respectively. Calculate the edge length of NaCl unit cell.
6. CsBr has bcc structure with edge length 4.3Å. What is the
shortest interionic distance (in Å) between Cs+ and Br – .
7. A metallic crystal crystallizes into a lattice containing a Solid 1 Solid 2

sequence of layers AB AB AB......Any packing of spheres 10. Find the sum of number of atoms present in a simple cubic,
leaves out voids in the lattice. What percentage of volume body centered cubic and face centered cubic structure.
of this lattice is empty space?

Exercise 3 : NCERT Exemplar & Past Year MCQs


NCERT Exemplar MCQs
(a)
1. Which of the following conditions favours the existence
of a substance in the solid state? (b)
(a) High temperature
(c)
(b) Low temperature
(c) High thermal energy (d)
(d) Weak cohesive forces 5. Which of the following is true about the value of refractive
2. Which of the following is not a characteristic of a index of quartz glass?
crystalline solid? (a) Same in all directions
(a) Definite and characteristic heat of fusion (b) Different in different directions
(b) Isotropic nature (c) Cannot be measured
(c) A regular periodically repeated pattern of (d) Always zero
arrangement of constituent particles in the entire 6. Iodine molecules are held in the crystals lattice by......... .
crystal (a) London forces
(d) A true solid (b) dipole – dipole interactions
3. Which of the following is an amorphous solid? (c) covalent bonds
(a) Graphite (C) (b) Quartz glass (SiO2) (d) coulombic forces
(c) Chrome alum (d) Silicon carbide (SiC) 7. Which of the following is not the characteristic of ionic
4. Which of the following arrangements shows schematic solids?
alignment of magnetic moments of antiferromagnetic (a) Very low value of electrical conductivity in the molten
substances? state
THE SOLID STATE 219

(b) Brittle nature 19. Which of the following represents correct order of
(c) Very strong forces of interactions conductivity in solids?
(d) Anisotropic nature (a) kmetals >> kinsulators < ksemiconductors
8. Graphite is a good conductor of electricity due to the (b) kmetals << kinsulators < ksemiconductors
presence of ........ . (c) kmetals. > ksemmiconductors > kinsulators = zero
(a) lone pair of electrons (b) free valence electrons
(d) kmetals < ksemiconductors > kinsulators ¹ zero
(c) cations (d) anions
20. Which of the following statement is not true about
9. Graphite cannot be classified as ....... .
amorphous solids?
(a) conducting solid (b) network solid
(a) On heating they may become crystalline at certain
(c) covalent solid (d) ionic solid
temperature
10. Cations are present in the interstitial sites in ...... .
(b) They may become crystalline on keeping for long
(a) Frenkel defect
time
(b) Schottky defect
(c) Amorphous solids can be moulded by heating
(c) vacancy defect
(d) metal deficiency defect (d) They are anisotropic in nature
11. Schottky defect is observed in crystals when ....... . 21. The sharp melting point of crystalline solids is due to
(a) some cations move from their lattice site to interstitial (a) a regular arrangement of constituent particles
sites observed over a short distance in the crystal lattice
(b) equal number of cations and anions are missing from (b) a regular arrangement of constituent particles
the lattice observed over a long distance in the crystal lattice
(c) some lattice sites are occupied by electrons (c) same arrangement of constituent particles in different
(d) some impurity is present in the lattice directions
12. To get a n-type semiconductor from silicon, it should be (d) different arrangement of constituent particles in
doped with a substance with valency ............ . different directions.
(a) 2 (b) 1 (c) 3 (d) 5 22. Which of the following is a network solid?
13. In which of the following pair, most efficient packing is (a) SO2 (solid) (b) I2
present? (c) Diamond (d) H2O (ice)
(a) hcp and bcc (b) hcp and ccp 23. Which of the following solids is not an electrical
(c) bcc and ccp (d) bcc and simple cubic cell conductor?
14. The percentage of empty space in a body centred cubic 1. Mg(s) 2. TiO (s)
arrangement is ............ . 3. I2(s) 4. H2O(s)
(a) 74 (b) 68 (c) 32 (d) 26 (a) Only 1 (b) Only 2
15. What is the coordination number in a square close packed (c) 3 and 4 (d) 2, 3 and 4
structure in two dimensions? 24. Which of the following oxides shows electrical properties
(a) 2 (b) 3 (c) 4 (d) 6 like metals?
16. Silicon doped with electron rich impurity forms..... . (a) SiO2 (b) MgO (c) SO2(s) (d) CrO2
(a) p – type semiconductor 25. The lattice site in a pure crystal cannot be occupied by .....
(b) n – type semiconductor (a) molecule (b) ion
(c) intrinsic semiconductor (c) electron (d) atom
(d) insulator 26. Which of the following is true about the charge acquired
17. A ferromagnetic substance becomes a permanent magnet by p – type semiconductors?
when it is placed in a magnetic field because........ . (a) Positive
(a) all the domains get oriented in the direction of (b) Neutral
magnetic field (c) Negative
(b) all the domains get oriented in the direction opposite (d) Depends on concentrations of p impurity
to the direction of magnetic field 27. The total number of tetrahedral voids in the face-centred
(c) domains get oriented randomly unit cell is ......... .
(d) domains are not affected by magnetic field (a) 6 (b) 8 (c) 10 (d) 12
18. The correct order of the packing efficiency in different 28. Which of the following point defects are shown by
types of unit cells is ...... . AgBr(s) crystals?
(a) fcc < bcc < simple cubic 1. Schottky defect 2. Frenkel defect
(b) fcc > bcc > simple cubic 3. Metal excess defect 4. Metal deficiency defect
(c) fcc < bcc > simple cubic (a) 1 and 2 (b) 3 and 4
(d) bcc < fcc > simple cubic (c) 1 and 3 (d) 2 and 4
EBD_8350
220 CHEMISTRY

29. Which of the following statement is not true about the 36. Which of the following oxides behaves as conductor or
hexagonal close packing? insulator depending upon temperature?
(a) The coordination number is 12 (a) TiO (b) SiO2 (c) TiO3 (d) MgO
(b) It has 74% packing efficiency 37. Which of the following is not true about the ionic solids?
(c) Tetrahedral voids of the second layer are covered by (a) Bigger ions form the close packed structure
the spheres of the third layer (b) Smaller ions occupy either the tetrahedral or the
(d) In this arrangement, spheres of the fourth layer are octahedral voids depending upon their size
exactly aligned with those of the first layer (c) Occupation of all the voids is not necessary
30. In which of the following structures, coordination number (d) The fraction of octahedral or tetrahedral voids
for cations and anions in the packed structure will be occupied depends upon the radii of the ions
same? occupying the voids
(a) Cl– ions form fcc lattice and Na+ ions occupy all Past Year MCQs
octahedral voids of the unit cell
(b) Ca2+ ions form fcc lattice and F– ions occupy all the 38. If a is the length of the side of a cube, the distance between
the body centered atom and one corner atom in the cube
eight tetrahedral voids of the unit cell
will be : [AIPMT 2014, S]
(c) O2– ions form fcc lattice and Na+ ions occupy all the
eight tetrahedral voids of the unit cell 2 4 3 3
(d) S2– ions form fcc lattice and Zn2+ ions go into alternate (a) a (b) a (c) a (d) a
3 3 4 2
tetrahedral voids of the unit cell
31. Which kind of defects are introduced by doping? 39. CsCl crystallises in body centred cubic lattice. If ‘a’ is its
(a) Dislocation defect (b) Schottky defect edge length then which of the following expressions is
(c) Frenkel defect (d) Electronic defect correct? [JEE M 2014, S]
32. Which of the following statements is not true? 3a
(a) r +r = 3a (b) r +r =
(a) Paramagnetic substances are weakly attracted by Cs + Cl- Cs + Cl- 2
magnetic field.
3
(b) Ferromagnetic substances cannot be magnetised (c) r +r =a (d) rCs+ + rCl- = 3a
Cs+ Cl- 2
permanently.
40. The correct statement for the molecule, CsI3 is:
(c) The domains in antiferromagnetic substances are
[JEE M 2014, S]
oppositely oriented with respect to each other.
(a) It is a covalent molecule.
(d) Pairing of electrons cancels their magnetic moment
(b) It contains Cs+ and I3- ions.
in the diamagnetic substances.
(c) It contains Cs3+ and I– ions.
33. Which of the following defects is also known as dislocation
(d) It contains Cs+, I– and lattice I2 molecule.
defect?
(a) Frenkel defect 41. A given metal crystallizes out with a cubic structure having
edge length of 361 pm. If there are four metal atoms in one
(b) Schottky defect
unit cell, what is the radius of one atom? [AIPMT 2015,
(c) Non – stoichiometric defect
S]
(d) Simple interstitial defect
34. In the cubic close packing, the unit cell has ....... . (a) 127 pm (b) 80 pm (c) 108 pm (d) 40 pm
(a) 4 tetrahedral voids each of which is shared by four 42. The vacant space in bcc lattice cell is : [AIPMT 2015, S]
adjacent unit cells (a) 26 % (b) 48 % (c) 23 % (d) 32 %
(b) 4 tetrahedral voids within the unit cell 43. Sodium metal crystallizes in a body centred cubic lattice
(c) 8 tetrahedral voids each of which is shared by four with a unit cell edge of 4.29Å. The radius of sodium atom is
adjacent unit cells approximately : [JEE M 2015, S]
(d) 8 tetrahedral voids within the unit cells
(a) 5.72Å (b) 0.93Å (c) 1.86Å (d) 3.22Å
35. The edge lengths of the unit cells in terms of the radius of
spheres constituting fcc, bcc and simple cubic unit cells 44. A metal crystallises in a face centred cubic structure. If the
edge length of its unit cell is 'a', the closest approach
are respectively ...... .
between two atoms in metallic crystal will be :
4r 4r [JEE M 2017, S]
(a) 2 2r , , 2r (b) , 2 2r , 2 r
3 3 (a) 2a (b) 2 2a
a
4r 4r (c) 2a (d)
(c) 2r , 2 2r , (d) 2r , , 2 2r 2
3 3
THE SOLID STATE 221

45. Iron exhibits bcc structure at room temperature. Above (a) C2A3 (b) C3A2
900°C, it transforms to fcc structure. The ratio of density (c) C3A4 (d) C4A3
of iron at room temperature to that at 900°C (assuming 48. An element has a body centered cubic (bcc) structure
molar mass and atomic radii of iron remains constant with with a cell edge of 288 pm. The atomic radius is:
temperature) is [NEET 2018, A] [NEET 2020, A]
3 4 3 1 3 3 4
(a) (b) (c) (d) 2 ´ 288pm
2 3 2 2 4 2 (a) ´ 288pm (b)
4 3
46. Which type of ‘defect’ has the presence of cations in the
interstitial sites? [JEE M 2018, C] 4 3
(c) ´ 288pm (d) ´ 288pm
(a) Schottky defect (b) Vacancy defect 2 4
(c) Frenkel defect (d) Metal deficiency defect 49. The one that is extensively used as a piezoelectric material
47. A compound is formed by cation C and anion A. The is: [JEE M 2019, C]
anions form hexagonal close packed (hcp) lattice and the (a) tridymite (b) amorphous silica
cations occupy 75% of octahedral voids. The formula of (c) quartz (d) mica
the compound is: [NEET 2019, A]

Exercise 4 : Problem Solving Skill Enhancer MCQs


1. By X-ray diffraction, it is found that nickel (at mass = 59 g (a) 39.27% (b) 68.02%
mol–1), crystallizes with ccp. The edge length of the unit (c) 74.05% (d) 78.54%
cell is 3.5 Å. If density of Ni crystal is 9.0 g/cm3 , then
6. First three nearest neighbour distances for primitive cubic
value of Avogadro's number from the data is :
(a) 6.05 × 1023 (b) 6.11 × 1023 lattice are respectively (edge length of unit cell = a):
(c) 6.02 × 10 23 (d) 6.023 × 1023 (a) a , 2a , 3a (b) 3a , 2a , a
2. A substance AxBy crystallizes in a face centred cubic (fcc)
(c) a 2a , 2a (d) a 3a , 2a
lattice in which atoms ‘A’ occupy each corner of the cube
and atoms ‘B’ occupy the centres of each face of the cube. 7. The g-form of iron has fcc structure (edge length 386 pm)
Identify the correct composition of the substance AxBy and b-form has bcc structure (edge length 290 pm). The
(a) AB3 ratio of density in g-form and b-form is
(b) A4B 3 (a) 0.9788 (b) 1.02 (c) 1.57 (d) 0.6344
(c) A3B 8. What is the density of Na2O having antiflurite-type crystal
(d) Composition can’t be specified structure, if the edge length of cube is 100 pm and what is
3. For two ionic solids CaO and KI, identify the wrong the effect on density by 0.05% Frenkel defect?
statement amongst the following : (a) 823.5 g cm–3, density decreases
(a) The lattice energy of CaO is much large than that of (b) 414.16 g cm–3, density decreases
KI (c) 823.5 g cm–3, density remains same
(b) KI is more soluble in water (d) 414.16 g cm–3, density remains same
(c) KI has higher melting point 9. MgO crystallizes in a cubic type crystal system. The ionic
(d) CaO has higher melting point radii for Mg2+ and O2– are 0.066 and 0.140 nm respectively.
4. An alloy of copper, silver and gold is found to have copper One can conclude that the Mg2+ ions occupy:
constituting the ccp lattice. If silver atoms occupy the (a) a cubic hole in a simple cubic structure
edge centres and gold is present at body centre, the alloy
(b) every tetrahedral hole in a close packed structure
will have the formula:
(a) Cu4Ag2Au (b) Cu4Ag4Au (c) an octahedral hole in a cubic close packed structure
(c) Cu4Ag3Au (d) CuAgAu (d) every other tetrahedral hole in a close packed
5. The packing efficiency of the two-dimensional square structure
unit cell shown below is : 10. An fcc lattice has a lattice parameter a = 400 pm. Calculate
the molar volume of the lattice including all the empty
space.
(a) 10.8 mL (b) 96 mL (c) 8.6 mL (d) 9.6 mL
11. A metal crystallizes in bcc lattice. The percent fraction of
edge length not covered by atom is
(a) 10.4% (b) 13.4% (c) 12.4% (d) 11.4%
L
EBD_8350
222 CHEMISTRY

12. Which of the following has maximum value of cation/anion 14. Which of the following expression is correct for packing
ratio? fraction of NaCl if the ions along with one face are
(a) KCl (b) NaCl (c) CaF2 (d) MgCl2 diagonally removed?
13. A body centered cubic lattice is made up of hollow spheres
of B. Spheres of solid A are present in hollow spheres of 13 3 16 3 13 3 4 3
pr- + pr+ pr- + pr+
B. Radius of A is half of radius of B. What is the ratio of (a) 3 3 (b) 3 3
total volume of spheres of B unoccupied by A in a unit 8(r+ + r- )3 8(r+ + r- )3
cell and volume of unit cell?
16 3 13 3 4 3 13 3
7 3p 7 3 pr- + pr+ pr- + pr+
(a) (b) (c) 3 3 (d) 3 3
64 128 8(r+ + r- )3 8(r+ + r- )3
7.p 15. Coordination number of “C” in Be2C3 whose structure is
(c) (d) None of these
24 correlated with that of antifluorite is:
(a) 2 (b) 4 (c) 6 (d) 8

ANSW ER KEY
Exercise 1 : NCERT Based Topic-wise MCQs
1 (b) 10 (c) 19 (d) 28 (d) 37 (b) 46 (c) 55 (c) 64 (c) 73 (d) 82 (a)
2 (a) 11 (b) 20 (d) 29 (a) 38 (a) 47 (a) 56 (b) 65 (c) 74 (b) 83 (a)
3 (c) 12 (d) 21 (a) 30 (d) 39 (d) 48 (a) 57 (c) 66 (d) 75 (d) 84 (d)
4 (b) 13 (d) 22 (d) 31 (b) 40 (b) 49 (a) 58 (a) 67 (a) 76 (a) 85 (d)
5 (c) 14 (b) 23 (a) 32 (a) 41 (a) 50 (d) 59 (d) 68 (d) 77 (a)
6 (c) 15 (a) 24 (b) 33 (c) 42 (d) 51 (b) 60 (d) 69 (b) 78 (c)
7 (a) 16 (b) 25 (a) 34 (d) 43 (a) 52 (c) 61 (d) 70 (d) 79 (c)
8 (a) 17 (c) 26 (b) 35 (a) 44 (b) 53 (d) 62 (a) 71 (b) 80 (b)
9 (b) 18 (a) 27 (a) 36 (b) 45 (b) 54 (b) 63 (b) 72 (d) 81 (c)
Exercise 2 : Numeric/Integer Answer Questions
1 (5) 2 (4) 3 (335) 4 (10.9) 5 (600) 6 (3.72) 7 (26) 8 (3) 9 (90) 10 (7)
Exercise 3 : NCERT Exemplar & Past Year MCQs
1 (b) 6 (a) 11 (b) 16 (b) 21 (b) 26 (b) 31 (d) 36 (c) 41 (a) 46 (c)
2 (b) 7 (a) 12 (d) 17 (a) 22 (c) 27 (b) 32 (b) 37 (d) 42 (d) 47 (c)
3 (b) 8 (b) 13 (b) 18 (b) 23 (c) 28 (a) 33 (a) 38 (d) 43 (c) 48 (d)
4 (d) 9 (d) 14 (c) 19 (a) 24 (d) 29 (d) 34 (d) 39 (c) 44 (d) 49 (c)
5 (a) 10 (a) 15 (c) 20 (d) 25 (c) 30 (a) 35 (a) 40 (b) 45 (d)
Exercise 4 : Problem Solving Skill Enhancer MCQs
1 (b) 3 (c) 5 (d) 7 (a) 9 (c) 11 (b) 13 (a) 15 (d)
2 (a) 4 (c) 6 (a) 8 (d) 10 (d) 12 (c) 14 (a)
16 Solutions

Trend Buster NEET & JEE Main


NEET JEE Remark
Number of Questions from 2020-15 13 9 Minimum one questions have been
Weightage 4.1% 4.40% asked every year in NEET & JEE M.

The most Important Concepts that Cover Maximum number of Questions asked in past 6 years.
NEET JEEM
Types of Solution & Expressing Concentration of Solutions 3 1
Ideal & Non-ideal Solutions 3 1
Colligative Properties & Determination of Molar Mass 4 3
Abnormal Molar Mass 2 2

Less Important Concepts that Cover 1 or 2 Questions asked in past 6 years.


NEET JEEM
Vapour Pressure of Liquid Solutions 1 1
Solubility — 1

NEET JEE
Concept Used

2020 Ideal & Non-ideal Solutions / Non-ideal Solution / Raoult’s law / 2 Average 1 Difficult
Colligative properties depression in freezing point.
2019 Ideal & Non-ideal Solutions / Ideal Solution / Vapour
Vapour Pressure of Liquid pressure of Liquid-Liquid 1 Easy 4 Average /
Solutions /Solubility/Colligative Solutions / Henry's Law Difficult
Properties & Determination of Constant / Osmotic Pressure /
Molar Mass/Types of Solution Molality
& Expressing Concentration of
Solutions
2018 Abnormal Molar Mass Depression of Freezing point — — 1 Average
(DTf = iKf m)
2017 Colligative Properties & Molal depression Constant /
Determination of Molar Mass / Molarity / Colligative Properties / 3 Easy / 1 Average
Types of Solution & Expressing Degree of Association Average
Concentration of Solutions /
Abnormal Molar Mass
2016 Vapour Pressure of Liquid Vapour Pressure of Liquid - 2 Easy / 1 Average
Solutions /Colligative Properties Liquid Solutions/Lowering of Average
& Determination of Molar Mass Vapour Pressure
2015 Abnormal Molar Mass/ Ideal & van't Hoff factor /Elevation 5 Easy / 1 Average
Non-ideal Solutions / Types of of Boiling Point (DTb = iKbm) / Average
Solution & Expressing Ideal Solution / Mole Fraction /
Concentration of Solutions/ Lowering of Vapour Pressure /
Colligative Properties & Normality
Determination of Molar Mass
EBD_8350
224 CHEMISTRY
SOLUTIONS 225
EBD_8350
226 CHEMISTRY

Problem Solving Tips/ Tricks/ Points to Remember

4 Ideal Solution : Solutions which obey Raoult’s Law 4 The most frequently used semipermeable membrane in
(I) DHmix = 0, (II) DVmix = 0, the A–B intermolecular laboratory is that of copper ferrocyanide, Cu2[Fe(CN)6]
interactions are the same as A–A and B–B inter-molecular because it is very strong and can withstand very high
interactions. pressure. Remember that semipermeable membrane of
Examples: Cu2[Fe(CN)6] does not work in non-aqueous solutions
(a) benzene and toluene because it gets dissolved is non-aqueous solvents. Other
(b) ethyl bromide and ethyl chloride synthetic semipermeable membrane is that of calcium
Non Ideal Solution: Solutions which do not obey phosphate. Natural semipermeable membranes are
parchment paper, cell wall, pig’s bladder etc.
Raoult's law, (I) DHmix ¹ 0 (II) DVmix ¹ 0. 4 Colligative µ Number of particles
(i) Showing positive deviations : For such solutions properties µ Number of molecules (in case of non-
(a) A–B inter-molecular interactions are weaker than A–A electrolytes)
and B–B intermolecular interactions (b) DHmix is +ve µ Number of ions (in case of electrolytes)
(c) DVmix is +ve µ Number of moles of solute
Examples: (a) Carbon tetrachloride + benzene µ Mole fraction of solute
(b) Carbon tetrachloride + chloroform
Normal molar mass
(ii) Showing negative deviations : For such solutions 4 i = Observed molar mass
(a) A–B intermolecular interactions are stronger than A–
A and B–B intermolecular interactions (b) DHmix is –ve Observed colligative property
(c) DVmix is –ve. =
Examples: (a) Chloroform + Acetone (b) Chloroform + Benzene Normal colligative property
4 Azeotropic Mixture with Minimum Boiling Point Observed osmotic pressure
It is formed by liquids showing positive deviation. An i = Normal osmotic pressure
intermediate composition of liquids having highest vapour
pressure, hence lowest boiling point gives this azeotrope. Total number of particles after association/dissociation
=
Such azeotropes have boiling points lower than either of No. of particles before association/dissociation
the pure component e.g. Rectified spirit (ethanol 95.5% +
H2O 4.50%) bpt 351.5 K. Observed osmotic pressure
4 Azeotropic Mixture with Maximum Boiling Point : Normal osmotic pressure
It is formed by liquids showing negative deviation. An Total number of particles after association/dissociation
intermediate composition of liquids having minimum =
vapour pressure, hence highest boiling point gives this No. of particles before association/dissociation
n
azeotrope. Such azeotropes have boiling points higher degree of association a = (i – 1) and degree of
than either of the pure components e.g. Water and HNO3 n -1
(HNO3 68% + H2O 32%) bpt 393.5K. i -1
dissociation (a) =
4 Raoult’s law n -1
o 4 Some liquids on mixing, form azeotropes which are binary
PS = pA + pB = p oA x A + pB xB mixtures having the same composition in liquid and vapour
o
= (1 – xB) po + pB xB = ( po – po )xB + p o
A B A A
phase and boil at a constant temperature. It is not possible
to separate the components by fractional distillation.
P o - PS n A wA × M B
= = 4 The solutions which show a large positive deviation from
Po nB M A × wB Raoult's law, form minimum boiling azeotrope at a specific
Here, PS = Total pressure composition. e.g. ethanol-water mixture.
o 4 The solutions which show a large negative deviation from
p oA or pB = vapour pressure in pure state of component
A or B, xA or xB = mole fraction of component A or B Raoult's law, form maximum boiling azeotrape at a specific
composition. e.g., 68% nitric acid and 32% water by mass
nA and nB = no. of moles of solute and solvent wA and wB with a b.p. of 393.5K.
are the masses and MA and MB are the molar masses of
the solute and solvent respectively. 4 Dissolution of gases in liquid is always exothermic because
DS = –ve (movement of gas decreases in liquid) and DG to
4 Elevation in boiling point DTb = Kbm, where, be –ve (spontaneous process), DH has to be negative.
m = Molality
Kb = Boiling point elevation constant or Ebullioscopic 4 For ideal and non-ideal solutions, DSmix > 0.
constant 4 Modern R.O. (water purifiers) work on the principle of
4 Depression in freezing point, DTf = Kf m reverse osmosis (R.O.)
æ wA (g) ö 4 NaCl or CaCl2 (anhydrous) are used to clear snow on roads.
çè m = Molality = M (g / mol) ´ w (g) ´ 1000÷ø It depresses the freezing point at which ice is expected to
A B be formed.
SOLUTIONS 227

Exercise 1 : NCERT Based Topic-wise MCQs


Topic 1: Types of Solutions and Expressing n1 n
Concentration of Solutions (c) x1 = = 1
n1 + n2 + n3 + n4 Σn
1. The term homogenous mixtures signifies that (d) n1 + n2 + n3 + n4 = 1
(a) its composition is uniform throughout the mixture. 13. Which of the following statements, regarding the mole
(b) its properties are uniform throughout the mixture. fraction (x) of a component in solution, is incorrect?
(c) both composition and properties are uniform (a) 0 £ x £ 1 (b) x < 1
throughout the mixture. (c) x is always non-negative (d) None of these
(d) neither composition nor properties are uniform 14. An X molal solution of a compound in benzene has mole
throughout the mixture. fraction of solute equal to 0.2. The value of X is
2. Molarity of liquid HCl will be, if density of solution is (a) 14 (b) 3.2 (c) 1.4 (d) 2
1.17 g/cc 15. 25mL of a solution of barium hydroxide on titration with a
(a) 36.5 (b) 32.05 (c) 18.25 (d) 42.10 0.1 molar solution of hydrochloric acid gave a titre value of
3. 1 M, 2.5 litre NaOH solution is mixed with another 0.5 35 mL. The molarity of barium hydroxide solution was
M, 3 litre NaOH solution. Then find out the molarity of (a) 0.07 (b) 0.14 (c) 0.28 (d) 0.35
resultant solution 16. The volume of 4 N HCl and 10 N HCl required to make 1
(a) 0.80 M (b) 1.0 M (c) 0.73 M (d) 0.50 M litre of 6 N HCl are
4. The molarity of pure water is (a) 0.75 litre of 10 N HCl and 0.25 litre of 4 N HCl
(a) 50 M (b) 18 M (c) 55.6 M (d) 100 M (b) 0.50 litre of 4 N HCl and 0.50 litre of 10 N HCl
5. An aqueous solution of glucose is 10% in strength. The (c) 0.67 litre of 4 N HCl and 0.33 litre of 10 N HCl
volume in which 1 g mole of it is dissolved, will be (d) 0.80 litre of 4 N HCl and 0.20 litre of 10 N HCl
(a) 9 litre (b) 1.8 litre (c) 8 litre (d) 0.9 litre 17. Molarity of H2SO4 is 18 M. Its density is 1.8 g/ml. Hence
molality is
6. 10 g of NaCl is dissolved in 10 6 g of the solution. Its (a) 36 (b) 200 (c) 500 (d) 18
concentration is 18. How many grams of concentrated nitric acid solution should
(a) 100 ppm (b) 0.1 ppm (c) 1 ppm (d) 10 ppm be used to prepare 250 mL of 2.0M HNO 3 ? The
7. 2.5 litres of NaCl solution contain 5 moles of the solute. concentrated acid is 70% HNO3
What is the molarity? (a) 90.0 g conc. HNO3 (b) 70.0 g conc. HNO3
(a) 5 molar (b) 2 molar (c) 54.0 g conc. HNO3 (d) 45.0 g conc. HNO3
(c) 2.5 molar (d) 12.5 molar N N N
8. 5 mL of N HCl, 20 mL of N/2 H2SO4 and 30 mL of N/3 19. If 50 mL H2SO4, 30 mL HNO3, 10 mL HCl is
10 3 2
HNO3 are mixed together and volume made to one litre. mixed and solution is made to 1L. Then normality of
The normality of the resulting solution is resultant solution is
N N N N N N N
(a) (b) (c) (d) (a) (b) (c) (d) N
5 10 20 40 20 40 50
9. What is the normality of a 1 M solution of H3PO4 ? 20. A solution is prepared by dissolving 10 g NaOH in 1250
(a) 0.5 N (b) 1.0 N (c) 2.0 N (d) 3.0 N mL of a solvent of density 0.8 mL/g. The molality of the
10. 200 ml of water is added to 500 ml of 0.2 M solution. What solution in mol kg–1 is
is the molarity of this diluted solution ? (a) 0.25 (b) 0.2 (c) 0.008 (d) 0.0064
(a) 0.5010 M(b) 0.2897 M(c) 0.7093 M(d) 0.1428 M 21. Assertion : Molarity of a solution in liquid state changes
11. For preparing 0.1 N solution of a compound from its impure with temperature.
sample of which the percentage purity is known, the weight Reason : The volume of a solution changes with change
of the substance required will be in temperature.
(a) Less than the theoretical weight (a) Assertion is correct, reason is correct; reason is a
(b) More than the theoretical weight correct explanation for assertion.
(c) Same as the theoretical weight (b) Assertion is correct, reason is correct; reason is not
(d) None of these a correct explanation for assertion
12. For mixture containing “four” components which of the (c) Assertion is correct, reason is incorrect
following is correct in term of mole fraction? (d) Assertion is incorrect, reason is correct.
(a) x1+ x2+ x3+ x4 ¹ 1 22. The normality of orthophosphoric acid having purity of 70
n3 % by weight and specific gravity 1.54 is
(b) = x3 (a) 11 N (b) 22 N (c) 33 N (d) 44 N
n1 + n2 + n3
EBD_8350
228 CHEMISTRY

23. Study the given statements and choose the correct option. 31. Which of the following graph is a correct representation
(i) 3.62 mass percentage of sodium hypochlorite in water of Henry’s law?
is used as commercial bleaching solution. (a) (b)

Partial pressure of gas


Partial pressure of gas
(ii) 35% volume percentage of ethylene glycol is used
as an antifreeze (as coolent in car engines).
(iii) Concentration of dissolved oxygen in a litre of sea
water is 5.8 ppm.
(a) Statements (i) and (ii) are correct
(b) Statements (i) and (iii) are correct
(c) Statements (ii) and (iii) are correct Mole fraction of gas Mole fraction of gas
(d) Statements (i),(ii) and (iii) are correct in solution in solution
24. Match the columns
Column -I Column-II (c) (d)

Partial pressure of gas


Partial pressure of gas
(A) Mass percentage (p) Medicine and pharmacy
(B) Mass by volume (q) Concentration of
pollutants in water
(C) ppm (r) Industrial chemical
application
(D) Volume percentage (s) Liquid solutions
(a) A – (q), B – (p), C – (s), D – (r) Mole fraction of gas Mole fraction of gas
(b) A – (s), B – (r), C – (p), D – (q) in solution in solution
(c) A – (r), B – (q), C – (s), D – (p) 32. People living at high attitudes often reported with a
(d) A – (r), B – (p), C – (q), D – (s) problem of feeling weak and inability to think clearly. The
Topic 2: Solubility reason for this is.
(a) at high altitudes the partial pressure of oxygen is
25. Which of the following factor do not affect solubility of less than at the ground level.
solid solute in liquid? (b) at high altitudes the partial pressure of oxygen is
(a) Temperature (b) Pressure more than at the ground level.
(c) Nature of solute (d) All of these (c) at high altitudes the partial pressure of oxygen is
26. At the state of dynamic equilibrium, for equal to at the ground level.
solute + solvent solution. (d) None of these.
(a) Rate of dissolution = Rate of unsaturation. 33. The solubility of a solid in a liquid is significantly affected
(b) Rate of dissolution = Rate of unsaturation. by temperature changes.
(c) Rate of dissolution = Rate of saturation Solute + Solvent ‡ˆˆˆˆ† Solution.
(d) Rate of crystallization = Rate of saturation. The system being in a dynamic equilibrium must follow
27. Which of the following statements is incorrect? Le-chatelier’s principle. Considering the Le-chatelier’s
(a) A solution in which no more solute can be dissolved principle which of the following is correct?
at the same temperature and pressure is called a (a) DHsol > 0; solubility ­ ; temperature ¯
saturated solution.
(b) An unsaturated solution is one in which more solute (b) DHsol < 0; solubility ¯ ; temperature ­
can be dissolved at the same temperature. (c) DHsol > 0; solubility ¯ ; temperature ­
(c) The solution which is in dynamic equilibrium with
(d) DHsol < 0; solubility ­ ; temperature ­
undissolved solute is the saturated solution.
(d) The minimum amount of solute dissolved in a given 34. W1
amount of solvent is its solubility. Piston W1 W2 W3
28. The statement “If 0.003 moles of a gas are dissolved in 900 g
of water under a pressure of 1 atmosphere, 0.006 moles will
be dissolved under a pressure of 2 atmospheres”, illustrates
(a) Dalton’s law of partial pressure
(b) Graham’s law (a) (b)
(c) Raoult’s law On the basis of the figure given above, which of the
(d) Henry’s law following is not true?
29. Value of Henry's constant KH _______. (a) In figure (a), assuming the state of dynamic equilibrium,
(a) increases with increase in temperature. rate of gaseous particles entering and leaving the
(b) decreases with increase in temperature. solution phase is same.
(c) remains constant. (b) In figure (b), on compressing the gas, number of
(d) first increases then decreases. gaseous particles per unit volume over the solution
30. The value of Henry's constant KH is _______. increases.
(a) greater for gases with higher solubility. (c) Rate at which gaseous particles are striking the
(b) greater for gases with lower solubility. solution to enter it, decreases.
(c) constant for all gases. (d) Rate at which gaseous particles are striking the
(d) not related to the solubility of gases. solution to enter it, increases.
SOLUTIONS 229

35. Henry’s law constant of oxygen is 1.4 × 10–3 mol. lit–1. atm–1 (d) physical state of non-volatile solute present in the
at 298 K. How much of oxygen is dissolved in 100 ml at 298 solution
K when the partial pressure of oxygen is 0.5 atm? 45. A mixture of ethyl alcohol and propyl alcohol has a vapour
(a) 1.4 g (b) 3.2 g (c) 22.4 mg (d) 2.24 mg pressure of 290 mm Hg at 300 K. The vapour pressure of
36. Which is an application of Henry’s law? propyl alcohol is 200 mm Hg. If the mole fraction of ethyl
(a) Spray paint (b) Bottled water alcohol is 0.6, its vapour pressure (in mm Hg) at the same
(c) Filling up atire (d) Soft drinks (soda) temperature will be
37. Scuba divers may experience a condition called ______. (a) 360 (b) 350 (c) 300 (d) 700
To avoids this, the tanks used by scuba divers are filled 46. Read the following statements carefully and choose the
with air diluted with _____ . correct option
(a) Migrains, Hydrogen (b) Cramps, Nitrogen (i) The vapour pressure of a liquid decreases with
(c) Nausea, Oxygen (d) Bends, Helium increase of temperature.
38. Read the following statements carefully and choose the (ii) The liquid boils at the temperature at which its vapour
correct option. pressure is equal to the atmospheric pressure.
(i) Different gases have different KH values at th e (iii) Vapour pressure of the solvent decreases in the
same temperature. presence of non-volatile solute.
(ii) Higher the value of KH at a given temperature, lower (iv) Vapour pressure of the pure solvent and solution is a
is the solubility of the nature of gas in the liquid. function of temperature.
(iii) KH is a function of the nature of the gas. (a) (i), (ii) and (iv) are correct
(iv) Solubility of gases increases with increase of (b) (i), (iii), and (iv) are correct
temperature. (c) (ii), (iii), and (iv) are correct
(a) (i), (ii) and (iv) are correct. (d) (i), (ii) and (iii) are correct
(b) (ii) and (iv) are correct. 47. A plot of p1 and p2 vs the mole fractions x1 and x2 is given as.
(c) (i), (ii) and (iii) are correct.
(d) (i) and (iv) are correct. III
p2 p2°
39. Which one of the following gases has the lowest value of = p1 +
p Total
Henry’s law constant? II
Vapour pressure

p° p2
(a) N2 (b) He (c) H2 (d) CO2 1

40. Equal masses of methane and oxygen are mixed in an empty


container at 25°C. The fraction of the total pressure exerted p
1
by oxygen is I

1 273 x1 = 1 x1 = 0
(a) 1/2 (b) 2/3 (c) ´ (d) 1/3 Mole fraction
3 298 x2 = 0 x2 = 1
41. When a gas is bubbled through water at 298 K, a very In this figure, lines I and II intersect through the point for
dilute solution of the gas is obtained. Henry’s law constant which.
for the gas at 298 K is 100 kbar. If the gas exerts a partial (a) x1 ¹ 1; x2 = 1 (b) x1 = x2 ¹ 1
pressure of 1 bar, the number of millimoles of the gas 1
dissolved in one litre of water is (c) x1 = 1; x2 ¹ 1 (d) x1 = x2 =
2
(a) 0.555 (b) 5.55 (c) 0.0555 (d) 55.5 48. The vapour pressure of two liquids ‘P’ and ‘Q’ are 80
42. What is the ratio of no. of moles of oxygen to that of and 60 torr, respectively. The total vapour pressure of
nitrogen in a container of 5 litre at atmospheric pressure? solution obtained by mixing 3 mole of P and 2 mole of Q
[(KH)02 = 34.86 Kbar, (KH)N = 76.48K bar would be
2
(a) 1 : 1.71 (b) 1 : 2 (c) 2 : 1 (d) 1 : 24 (a) 72 torr (b) 140 torr (c) 68 torr (d) 20 torr
49. 18 g of glucose (C6H12O6) is added to 178.2 g of water. The
Topic 3: Vapour Presure of Liquid Solutions
vapour pressure of water for this aqueous solution is
43. Iodine and sulphur dissolve in (a) 76.00 torr (b) 752.40 torr
(a) water (b) benzene (c) 759.00 torr (d) 7.60 torr
(c) carbon disulphide (d) ethanol 50. P A and P B are the vapour pressure of pure liquid
44. The decrease in the vapour pressure of solvent depends components, A and B, respectively of an ideal binary
on the solution. If XA represents the mole fraction of component
(a) quantity of non-volatile solute present in the solution A, the total pressure of the solution will be.
(b) nature of non-volatile solute present in the solution (a) PA + XA (PB – PA) (b) PA + XA (PA – PB)
(c) molar mass of non-volatile solute present in the (c) PB + XA (PB – PA) (d) PB + XA (PA – PB)
solution
EBD_8350
230 CHEMISTRY

51. A 1% solution of KCl (I), NaCl (II), BaCl2 (III) and urea (IV) (c) A – B interaction is weaker than A –A and B – B
have their osmotic pressure at the same temperature in the interactions
ascending order (molar masses of NaCl, KCl, BaCl2 and (d) A–B interaction is stronger than A–A and B–B
urea are respectively 58.5, 74.5, 208.4 and 60 g mole–1). interactions.
Assume 100% ionization of the electrolytes at this 60. On the basis of information given below mark the correct
temperature. option.
(a) I < III < II < IV (b) III < I < II < IV (i) In bromoethane and chloroethane mixture
(c) I < II < III < IV (d) III < IV < I < II intermolecular interactions of A-A and B-B type are
52. Two liquids X and Y form an ideal solution. At 300 K, nearly same as A-B type interactions.
vapour pressure of the solution containing 1 mol of X and (ii) In ethanol and acetone mixture A-A or B-B type
3 mol of Y is 550 mm Hg. At the same temperature, if 1 mol intermolecular interactions are stronger than A-B type
of Y is further added to this solution, vapour pressure of interactions.
the solution increases by 10 mm Hg. Vapour pressure ( in (iii) In chloroform and acetone mixture A-A or B-B type
mm Hg) of X and Y in their pure states will be, respectively intermolecular interactions are weaker than A-B type
(a) 300 and 400 (b) 400 and 600 interactions.
(c) 500 and 600 (d) 200 and 300 (a) Solution (ii) and (iii) will follow Raoult's law.
53. The vapour pressure of pure benzene and toluene at a (b) Solution (i) will follow Raoult's law.
particular temperature are 100 mm and 50 mm respectively. (c) Solution (ii) will show negative deviation from Raoult's
Then the mole fraction of benzene in vapour phase in law.
contact with equimolar solution of benzene and toluene is (d) Solution (iii) will show positive deviation from Raoult's
(a) 0.67 (b) 0.75 (c) 0.33 (d) 0.50 law.
54. Consider a and b are two components of a liquid mixture, 61. Which one of the following is non-ideal solution
their corresponding vapour pressure (mmHg) are (a) Benzene + toluene
respectively 450 and 700 in pure states and total pressure (b) n-hexane + n-heptane
given is 600. Then corresponding composition in liquid (c) Ethyl bromide + ethyl iodide
phase will be (d) CCl4 + CHCl3
(a) 0.4, 0.6 (b) 0.5, 0.5 (a) 0.6, 0.4 (d) 0.3, 0.7 62. Mixtures of ethanol and acetone show positive deviation.
55. Which will form maximum boiling point azeotrope The reason is
(a) HNO3 + H2O solution (a) In pure ethanol, molecules are hydrogen bonded.
(b) C2H5OH + H2O solution (b) In pure acetone, molecules are hydrogen bonded
(c) C6H6 + C6H5CH3 solution (c) In both molecules are hydrogen bonded
(d) None of these (d) None of these
63. Which of the following liquid pairs shows a positive
Topic 4: Ideal and Non-ideal Solutions
deviation from Raoult’s law ?
56. A solution containing components A and B follows Raoult's (a) Water - Nitric acid
law when (b) Benzene - Methanol
(a) A – B attraction force is greater than A – A and B – B (c) Water - Hydrochloric acid
(b) A – B attraction force is less than A – A and B – B (d) Acetone - Chloroform
(c) A – B attraction force remains same as A – A and B – B 64. Negative deviation from Raoult’s law is observed in
(d) volume of solution is different from sum of volume of which one of the following binary liquid mixtures?
solute and solvent (a) Ethanol and acetone
57. Which one of the following is not correct for an ideal (b) Benzene and toluene
solution? (c) Acetone and chloroform
(a) It must obey Raoult’s law (d) Chloroethane and bromoethane
(b) DH = 0 65. A binary liquid solution is prepared by mixing n-heptane
(c) DH = DV ¹ 0 and ethanol. Which one of the following statements is
(d) All are correct correct regarding the behaviour of the solution?
58. All form ideal solution except (a) The solution is non-ideal, showing negative deviation
(a) C6H6 and C6H5CH3 (b) C2H6 and C2H5I from Raoult’s Law.
(c) C6H5Cl and C6H5Br (d) C2H5 I and C2H5OH. (b) The solution is non-ideal, showing positive deviation
59. A mixture of components A and B will show –ve deviation from Raoult’s Law.
when (c) n-heptane shows positive deviation while ethanol
(a) DVmix > 0 shows negative deviation from Raoult’s Law.
(b) DHmix > 0 (d) The solution formed is an ideal solution.
SOLUTIONS 231

66. The azeotropic mixture of water (b.p.100°C) and HCl 74. The vapour pressure at a given temperature of an ideal
(b.p.85°C) boils at 108.5°C. When this mixture is distilled it solution containing 0.2 mol of a non-volatile solute and 0.8
is possible to obtain mol of solvent is 60 mm of Hg. The vapour pressure of the
(a) pure HCl pure solvent at the same temperature is
(b) pure water (a) 150 mm of Hg (b) 60 mm of Hg
(c) pure water as well as pure HCl (c) 75 mm of Hg (d) 120 mm of Hg
(d) neither HCl nor H2O in their pure states 75. 12 g of a non-volatile solute dissolved in 108 g of water
67. The system that forms maximum boiling azeotrope is produces the relative lowering of vapour pressure of 0.1.
(a) carbondisulphide – acetone The molecular mass of the solute is
(b) benzene – toluene (a) 80 (b) 60 (c) 20 (d) 40
(c) acetone – chloroform 76. The vapour pressure of water at 20°C is 17.5 mm Hg. If
(d) n-hexane – n-heptane 18 g of glucose (C6H12O6) is added to 178.2 g of water at
68. Which one of the following binary mixtures forms an 20°C, the vapour pressure of the resulting solution will be
azeotrope with minimum boiling point type? (a) 17.325 mm Hg (b) 15.750 mm Hg
(a) acetone-ethanol (b) H2O-HNO3 (c) 16.500 mm Hg (d) 17.500 mm Hg
77. Which one of the following is a colligative property ?
(c) benzene-toluene (d) n-hexane-n-heptane
(a) Boiling point (b) Vapour pressure
69. Assertion : Azeotropic mixtures are formed only by non-ideal
(c) Osmotic pressure (d) Freezing point
solutions and they may have boiling points either greater
78. The rise in the boiling point of a solution containing 1.8 g
than both the components or less than both the components.
of glucose in 100 g of solvent is 0.1°C. The molal elevation
Reason : The composition of the vapour phase is same as
constant of the liquid is
that of the liquid phase of an azeotropic mixture.
(a) 0.01 K/m (b) 0.1 K/m (c) 1 K/m (d) 10 K/m
(a) Assertion is correct, reason is correct; reason is a
79. 1.00 g of a non-electrolyte solute (molar mass 250 g mol–1)
correct explanation for assertion.
was dissolved in 51.2 g of benzene. If the freezing point
(b) Assertion is correct, reason is correct; reason is not
depression constant, Kf of benzene is 5.12 K kg mol–1, the
a correct explanation for assertion
freezing point of benzene will be lowered by
(c) Assertion is correct, reason is incorrect (a) 0.3 K (b) 0.5 K (c) 0.4 K (d) 0.2
(d) Assertion is incorrect, reason is correct. 80. A 0.5 molal solution of ethylene glycol in water is used as
70. At 300 K the vapour pressure of an ideal solution coolant in a car. If the freezing point constant of water be
containing 1 mole of liquid A and 2 moles of liquid B is 500 1.86°C per mole, the mixture shall freeze at
mm of Hg. The vapour pressure of the solution increases
(a) 0.93°C (b) –0.93°C (c) 1.86°C (d) –1.86°C
by 25 mm of Hg, if one more mole of B is added to the
81. The freezing point of 1% solution of lead nitrate in water
above ideal solution at 300 K. Then the vapour pressure of
will be
A in its pure state is
(a) 2°C (b) 1°C
(a) 300 mm of Hg (b) 400 mm of Hg
(c) 500 mm of Hg (d) 600 mm of Hg (c) 0°C (d) below 0°C
82. A solution of sucrose (molar mass = 342 g mol–1) has been
Topic 5: Colligative properties and Determination of Molar Mass prepared by dissolving 68.5 g of sucrose in 1000 g of water.
71. The relative lowering of the vapour pressure is equal to The freezing point of the solution obtained will be
the ratio between the number of (Kf for water = 1.86 K kg mol –1).
(a) solute molecules to the solvent molecules (a) – 0.372°C (b) – 0.520°C
(b) solute molecules to the total molecules in the solution (d) + 0.372°C (d) – 0.570°C
(c) solvent molecules to the total molecules in the solution 83. Blood cells retain their normal shape in solution which are
(d) solvent molecules to the total number of ions of the (a) hypotonic to blood (b) isotonic to blood
solute. (c) hypertonic to blood (d) equinormal to blood.
72. Vapour pressure of benzene at 30°C is 121.8 mm Hg. When 84. Isotonic solutions have same
15 g of a non volatile solute is dissolved in 250 g of benzene (a) molar concentration (b) molality
its vapour pressure decreased to 120.2 mm Hg. The (c) normality (d) None of these
molecular weight of the solute (Mo. wt. of solvent = 78) 85. Which of the following pairs of solution are isotonic at the
(a) 356.2 (b) 456.8 (c) 530.1 (d) 656.7 same temperature ?
73. The value of P° for benzene is 640 mm of Hg. The vapour (a) 0.1 M Ca(NO3)2 and 0.1 M Na2SO4
pressure of solution containing 2.5g substance X in 39g. (b) 0.1 M NaCl and 0.1 M Na2SO4
benzene is 600mm of Hg. The molecular mass of X is (c) 0.1 M urea and 0.1 M MgCl2
(a) 65.25 (b) 130 (c) 40 (d) 80
(d) 0.1 M urea and 0.1 M NaCl
EBD_8350
232 CHEMISTRY

86. If 0.1 M solution of glucose and 0.1 M solution of urea are 95. Assertion : When a solution is separated from the pure
placed on two sides of the semipermeable membrane to solvent by a semi- permeable membrane, the solvent
equal heights, then it will be correct to say that molecules pass through it from pure solvent side to the
(a) there will be no net movement across the membrane solution side
(b) glucose will flow towards urea solution Reason : Diffusion of solvent occurs from a region of high
(c) urea will flow towards glucose solution concentration solution to a region of low concentration
(d) water will flow from urea solution to glucose solution.
87. Someone has added a non electrolyte solid to the pure (a) Assertion is correct, reason is correct; reason is a
liquid but forgot that among which of the two beakers he correct explanation for assertion.
has added that solid. This problem can be solved by (b) Assertion is correct, reason is correct; reason is not
checking a correct explanation for assertion
(a) relative lower in vapour pressure (c) Assertion is correct, reason is incorrect
(b) elevation in boiling point (d) Assertion is incorrect, reason is correct.
(c) depression in Freezing point
96. For a dilute solution containing 2.5 g of a non-volatile
(d) all above
non-electrolyte solute in 100 g of water, the elevation in
88. Vapour pressure of benzene at 30°C is 121.8 mm. When
boiling point at 1 atm pressure is 2°C. Assuming
15g of a non-volatile solute is dissolved in 250 g of benzene,
concentration of solute is much lower than th e
its vapour pressure is decreased to 120.2 mm. The molecular
concentration of solvent, the vapour pressure (mm of Hg)
weight of the solute is
(a) 35.67 g (b) 356.7 g (c) 432.8 g (d) 502.7 g of the solution is
89. 1 g of a non-volatile, non-electrolyte solute of molar (take Kb = 0.76 K kg mol–1)
mass 250 g/mol was dissolved in 51.2 g of benzene. If the (a) 724 (b) 740 (c) 736 (d) 718
freezing point depression constant Kf of benzene is 5.12 97. The difference between the boiling point and freezing point
kg K mol–1. The freezing point of benzene is lowered by of an aqueous solution containing sucrose (molecular wt
(a) 0.3 K (b) 0.5 K (c) 0.2 K (d) 0.4 K = 342 g mole–1) in 100 g of water is 105°C. If Kf and
90. The amount of solute (molar mass 60 g.mol–1) that must be Kb of water are 1.86 and 0.51 K kg mol–1 respectively, the
added to 180 g of water so that the vapour pressure of weight of sucrose in the solution is about
water is lowered by 10% is (a) 34.2 g (b) 342 g (c) 7.2 g (d) 72 g
(a) 30 g (b) 60 g (c) 120 g (d) 12 g 98. If the elevation in boiling point of a solution of non-volatile,
91. The molecular weight of benzoic acid in benzene as determined non-electrolytic and non-associating solute in a solvent
by depression in freezing point method corresponds to (Kb = x K kg mol–1) is y K, then the depression in freezing
(a) ionization of benzoic acid. point of solution of same concentration would be (Kf of
(b) dimerization of benzoic acid. the solvent = z K kg mol–1)
(c) trimerization of benzoic acid. 2xz xz
yz yz
(d) solvation of benzoic acid. (a) (b) (c) (d)
y x y 2x
92. Osmotic pressure of 0.4% urea solution is 1.64 atm and
that of 3.42% cane sugar is 2.46 atm. When equal volumes Topic 6: Abnormal Molar Mass
of the above two solutions are mixed, the osmotic pressure
99. The van’t Hoff factor i for a compound which undergoes
of the resulting solution is :
dissociation in one solvent and association in other solvent
(a) 2.05 atm (b) 2.46 atm (c) 1.64 atm (d) 4.10 atm
is respectively
93. At 25°C, at 5 % aqueous solution of glucose (molecular
(a) less than one and greater than one.
weight = 180 g mol–1) is isotonic with a 2% aqueous solution
containing an unknown solute. What is the molecular (b) less than one and less than one.
weight of the unknown solute? (c) greater than one and less than one.
(a) 60 (b) 80 (c) 72 (d) 63 (d) greater than one and greater than one.
94. Assertion : When NaCl is added to water a depression in 100. If the various terms in the given below expressions have
freezing point is observed. usual meanings, the van’t Hoff factor (i) cannot be
Reason : The lowering of vapour pressure of a solution calculated by which one of the following expressions
causes depression in the freezing point. (a) pV = inRT
(a) Assertion is correct, reason is correct; reason is a
(b) DT f = iK f .m
correct explanation for assertion.
(b) Assertion is correct, reason is correct; reason is not (c) DTb = iK b .m
a correct explanation for assertion °
Psolvent - Psolution æ n ö
= iç
è N + n ÷ø
(c) Assertion is correct, reason is incorrect (d)
°
Psolvent
(d) Assertion is incorrect, reason is correct.
SOLUTIONS 233

101. van’t Hoff factor is given by the expression ________. 106. Which of the following aqueous solution has minimum
Normal molar mass freezing point ?
(a) i = Abnormal molar mass (a) 0.01 m NaCl (b) 0.005 m C2H5OH
(c) 0.005 m MgI2 (d) 0.005 m MgSO4.
Abnormal molar mass
(b) i = Normal molar mass 107. In a 0.2 molal aqueous solution of a weak acid HX, the
degree of ionization is 0.3. Taking Kf for water as 1.85, the
Observed colligative property freezing point of the solution will be nearest to
(c) i=
Calculated colligative property (a) – 0.360ºC (b) – 0.260ºC
(d) Both (a) and (c) (c) + 0.481ºC (d) – 0.481ºC
102. At 25°C, the highest osmotic pressure is exhibited by 0.1 108. If a is the degree of dissociation of Na2SO4, the van’t
M solution of Hoff’s factor (i ) used for calculating the molecular mass is
(a) CaCl2 (b) KCl (c) glucose (d) urea (a) 1 + a (b) 1 – a
103. Which one of the following salts will have the same value (c) 1 + 2a (d) 1 – 2a
of van’t Hoff factor (i) as that of K4[Fe(CN)6].
109. The elevation in boiling point of a solution of 13.44 g of
(a) Al2(SO4)3 (b) NaCl
CuCl2 in 1 kg of water using the following information will
(c) Al(NO3)3 (d) Na2SO4. be (Molecular weight of CuCl2= 134.4 g and Kb= 0.52 K
104. Which one of the following aqueous solutions will exihibit kg mol-1)
highest boiling point ?
(a) 0.16 (b) 0.05 (c) 0.1 (d) 0.2
(a) 0.015 M urea (b) 0.01 M KNO3
(c) 0.01 M Na2SO4 (d) 0.015 M glucose 110. Freezing point of an aqueous solution is – 0.186°C. If
105. For an electrolyte, elevation of B.P. is directly proportional the values of Kb and Kf of water are respectively 0.52 K kg
to mol–1 and 1.86 K kg mol–1, then the elevation of boiling
(a) molarity (b) molality point of the solution in K is
(c) mole fraction (d) All of these (a) 0.52 (b) 1.04 (c) 1.34 (d) 0.052

1. What is the molar solubility of Al(OH)3 in 0.2 M NaOH 6. What weight of the non-volatile solute (in g), urea
solution? Given that, solubility product of Al(OH)3 (NH2 – CO – NH2) needs to be dissolved in 100g of water,
= 2.4×10–2 in order to decrease the vapour pressure of water by 25%?
2. At room temperature, a dilute solution of urea is 7. A bottle of commercial sulphuric acid (density 1.787 g/mL)
prepared by dissolving 0.60 g of urea in 360 g of water. is labelled as 86 percent by weight. What volume of the
If the vapour pressure of pure water at this temperature
acid has to be used to make 1 litre of 0.2 M H2SO4?
is 35 mmHg, what will be the lowering of vapour
8. 0.5 gm of fuming H2SO4 (Oleum) is diluted with water. This
pressure?
solution is completely neutralized by 26.7 ml of 0.4 N NaOH.
(molar mass of urea = 60 g mol–1)
3. Molal depression constant for a solvent is 4.0 K kg Find the percentage of free SO3 in the sample of oleum.
mol–1. Calculate the depression in the freezing point (in 9. The vapour pressure of pure benzene at a certain
K) of the solvent for 0.03 mol kg–1 solution K2SO4. temperature is 640mm Hg. A non-volatile solid weighing
(Assume complete dissociation of the electrolyte) 2.175g is added to 39.0g of benzene. The vapour pressure
4. The osmotic pressure of a dilute solution of an ionic of the solution is 600mm Hg. What is the molecular weight
compound XY in water is four times that of a solution (in g) of the solid substance ?
of 0.01 M BaCl2 in water. Assuming complete 10. 0.400 g of an acid HA (mol. mass = 80) was dissolved
dissociation of the given ionic compounds in water, find in 100 g of water. The solution showed a depression of
the concentration of XY (in mol L–1) in solution. freezing point of 0.12 K. What will be the dissociation
5. Calculate the amount of sodium chloride (in g) which must constant (in multiple of 10–3) of the acid at about 0°C?
be added to 1000 mL of water so that its freezing point is Given Kf (water) = 1.86 K Kg mol–1 (Assume molarity of
depressed by 0.744K. For water, Kf = 1.86 K/m. Assume
solution » molality)
density of water to be 1 g mL–1.
EBD_8350
234 CHEMISTRY

Exercise 3 : NCERT Exemplar & Past Year MCQs


NCERT Exemplar MCQs 11. The value of van't Hoff factors for KCl, NaCl and K2SO4
respectively are ............ .
1. Which of the following units is useful in relating (a) 2, 2 and 2 (b) 2, 2 and 3
concentration of solution with its vapour pressure? (c) 1, 1 and 2 (d) 1, 1 and 1
(a) Mole fraction (b) Parts per milion 12. Value of Henry's constant KH ............. .
(c) Mass percentage (d) Molality (a) increases with increase in temperature
2. On dissolving sugar in water at room temperature, solution (b) decreases with increase in temperature
feels cool to touch. Under which of the following cases, (c) remains constant
dissolution of sugar will be most rapid? (d) first increases then decreases
(a) Sugar crystals in cold water 13. The value of Henry's constant, KH is ............. .
(b) Sugar crystals in hot water (a) greater for gases with higher solubility
(c) Powdered sugar in cold water (b) greater for gases with lower solubility
(d) Powdered sugar in hot water (c) constant for all gases
3. At equilibrium the rate of dissolution of a solid solute in a (d) not related to the solubility of gases
volatile liquid solvent is ....... 14. Two beakers of capacity 500 mL were taken. One of these
(a) less than the rate of crystallisation beakers labelled as "A" was filled with 400 mL water
(b) greater than the rate of crystallisation whereas the beaker labelled "B" was filled with 400 mL of
(c) equal to the rate of crystallisation 2M solution of NaCl. At the same temperature both the
(d) zero beakers were placed in closed containers of same material
4. A beaker contains a solution of substance 'A'. Precipitation and same capacity as shown in figure.
of substance 'A' takes place when small amount of 'A' is
added to the solution. The solution is ........ .
(a) saturated (b) supersaturated A B
(c) unsaturated (d) concentrated
5. Low concentration of oxygen in the blood and tissues of
Water NaCl solution
people living at high altitude is due to ............ .
(a) low temperature At a given temperature, which of the following statement
(b) low atmospheric pressure is correct about the vapour pressure of pure water and
(c) high atmospheric pressure that of NaCl solution?
(d) both low temperature and high atmospheric pressure (a) Vapour pressure in container (A) is more than that in
6. Colligative properties depend on ............ . container (B)
(a) the nature of the solute particles dissolved in solution (b) Vapour pressure in container (A) is less than that in
(b) the number of solute particles in solution container (B)
(c) the physical properties of the solution particles (c) Vapour pressure is equal in both the containers
dissolved in solution (d) Vapour pressure in container (B) is twice the vapour
(d) the nature of solvent particles pressure in container (A)
7. Which of the following aqueous solutions should have 15. 4 L of 0.02 M aqueous solution of NaCl was diluted by
the highest boiling point? adding 1 L of water. The molality of the resultant solution
(a) 1.0 M NaOH (b) 1.0 M Na2SO4 is .................
(c) 1.0 M NH4NO3 (d) 1.0 M KNO3 (a) 0.004 (b) 0.008 (c) 0.012 (d) 0.016
8. The unit of ebullioscopic constant is 16. Maximum amount of a solid solute that can be dissolved
(a) K kg mol–1 or K (molality)–1 in a specified amount of a given liquid solvent does not
(b) mol kg K–1 or K–1 (molality) depends upon ......... .
(c) kg mol–1 K–1or K–1 (molality)–1 (a) temperature (b) nature of solute
(c) pressure (d) nature of solvent
(d) K mol kg–1or K (molality)
17. Considering the formation, breaking and strength of
9. In comparison to a 0.01 M solution of glucose, the depression
hydrogen bond, predict which of the following mixtures
in freezing point of a 0.01 M MgCl2 solution is ....... .
will show a positive deviation from Raoult's law?
(a) the same (b) about twice (a) Methanol and acetone (b) Chloroform and acetone
(c) about three time (d) about six times (c) Nitric acid and water (d) Phenol and aniline
10. At a given temperature, osmotic presure of a concentrated 18. An unripe mango placed in a concentrated salt solution
solution of a substance ............ . to prepare pickle, shrivels because .......... .
(a) is higher than that of a dilute solution (a) it gains water due to osmosis
(b) is lower than that of a dilute solution (b) it loses water due to reverse osmosis
(c) it gains water due to reverse osmosis (c) it gains water due to reverse osmosis
(d) it loses water due to osmosis (d) it loses water due to osmosis
SOLUTIONS 235

19. Which of the following statements is false? (ii) In ethanol and acetone mixture A — A or B — B type
(a) Two different solutions of sucrose of same molality intermolecular interactions are stronger than A — B
prepared in diffferent solvents will have the same type interactions.
depression in freezing point. (iii) In chloroform and acetone mixture A — A or B — B
(b) The osmotic pressure of a solution is given by the type intermolecular interactions are weaker than
equation p = CRT (where, C is the molarity of the A — B type interactions.
solution) (a) Solution (ii) and (iii) will follow Raoult's law
(c) Decreasing order of osmotic pressure for 0.01 M (b) Solution (i) will follow Raoult's law
aqueous solutions of barium chloride, potassium (c) Solution (ii) will show negative deviation from
chloride, acetic acid and sucrose is Raoult's law
BaCl2 > KCl > CH3COOH > sucrose (d) Solution (iii) will show positive deviation from
(d) According to Raoult's law, the vapour pressure Raoult's law
exerted by a volatile component of a solution is 24. If two liquids A and B form minimum boiling azeotrope at
directly proportional to its mole fraction in the some specific composition then
solution. (a) A — B interactions are stronger than those between
20. Which of the following statement is false? A — A or B — B
(a) Units of atmospheric pressure and osmotic pressure (b) vapour pressure of solution increases because more
are same number of molecules of liquids A and B can escape
(b) In reverse osmosis, solvent molecules move through a from the solution
semipermeable membrane from a region of lower (c) vapour pressure of solution decreases because less
concentration of solute to a region of higher number of molecules of only one of the liquids escape
concentration from the solution
(c) The value of molal depression constant depends on (d) A — B interactions are weaker than those between
nature of solvent A — A or B — B
(d) Relative lowering of vapour pressure, is a 25. On the basis of information given below mark the correct
dimensionless quantity option.
21. Consider the figure and mark the incorrect option. Information On adding acetone to methanol some of the
Piston (A) Piston (B)
hydrogen bonds between methanol molecules break.
SPM
(a) At specific composition methanol-acetone mixture
will form minimum boiling azeotrope and will show
positive deviation from Raoult's law
Concentrated (b) At specific composition methanol-acetone mixture
Fresh water
(A)
sodium chloride will form maximum boiling azeotrope and will show
solution in
positive deviation from Raoult's law
water (B)
(c) At specific composition methanol-acetone mixture
(a) Water will move form side (A) to side (B) if a pressure will form maximum boiling azeotrope and will show
lower than osmotic pressure is applied on piston (B) negative deviation from Raoult's law
(b) Water will move from side (B) to side (A) if a pressure (d) At specific composition methanol-acetone mixture
greater than osmotic pressure is applied on piston (B) will form minimum boiling azeotrope and will show
(c) Water will move from side (B) to side (A) if a pressure negative deviation from Raoult's law
equal to osmotic pressure is applied on piston (B) 26. KH value for Ar (g), CO2 (g), HCHO (g) and CH4 (g) are
(d) Water will move from side (A) to side (B) if pressure 40.39, 1.67, 1.83 × 10–5 and 0.413 respectively.
equal to osmotic pressure is applied on piston (A) Arrange these gases in the order of their increasing
22. We have three, aqueous solutions of NaCl labelled as 'A', solubility.
'B' and 'C' with concentrations 0.1 M,0.01M and 0.001 M, (a) HCHO < CH4 < CO2 < Ar
respectively. The value of van't Hoff factor for these (b) HCHO < CO2 < CH4 < Ar
solutions will be in the order ........... . (c) Ar < CO2 < CH4 < HCHO
(a) iA < iB < iC (b) iA > iB > iC (d) Ar < CH4 < CO2 < HCHO
(c) iA = iB = iC (d) iA < iB > iC
23. On the basis of information given below mark the correct Past Year MCQs
option. 27. Of the following 0.10 m aqueous solutions, which one will
Information exhibit the largest freezing point depression?
(i) In bromoethane and chloroethane mixture [AIPMT 2014, A]
intermolecular interactions of A — A and B — B type (a) KCl (b) C6H12O6
are nearly same as A — B type interactions. (c) Al2(SO4)3 (d) K2SO4
EBD_8350
236 CHEMISTRY

28. Consider separate solutions of 0.500 M C2H5OH(aq), 36. At 100°C the vapour pressure of a solution of 6.5g of a
0.100 M Mg3(PO4)2 (aq), 0.250 M KBr(aq) and 0.125 M solute in 100 g water is 732 mm. If Kb = 0.52, the boiling
Na3PO4(aq) at 25°C. Which statement is true about these point of this solution will be [NEET 2016, S]
solutions, assuming all salts to be strong electrolytes? (a) 101°C (b) 100 °C (c) 102 °C (d) 103 °C
[JEE M 2014, A] 37. 18 g glucose (C6H12O6) is added to 178.2 g water. The
(a) They all have the same osmotic pressure. vapour pressure of water (in torr) for this aqueous solution
(b) 0.100 M Mg3(PO4)2(aq) has the highest osmotic pressure. is: [JEE M 2016, S]
(c) 0.125 M Na3PO4(aq) has the highest osmotic pressure. (a) 752.4 (b) 759.0 (c) 7.6 (d) 76.0
(d) 0.500 M C2H5OH(aq) has the highest osmotic 38. If molality of the dilute solutions is doubled, the value of
pressure. molal depression constant (Kf ) will be:- [NEET 2017, S]
29. Which one of the following electrolytes has the same value (a) halved (b) tripled
of van't Hoff factor (i) as that of the Al2(SO4)3 (if all are (c) unchanged (d) doubled
100% ionised)? [AIPMT 2015, A] 39. Which of the following is dependent on temperature?
(a) K3[Fe(CN)6] (b) Al(NO3)3 [NEET 2017, C]
(c) K4[Fe(CN)6] (d) K2SO4 (a) Molarity (b) Mole fraction
30. The boiling point of 0.2 mol kg–1 solution of X in water is (c) Weight percentage (d) Molality
greater than equimolal solution of Y in water. Which one of 40. Which observation(s) reflect(s) colligative properties?
the following statements is true in this case ? [NEET 2017, A]
[AIPMT 2015, A] (i) A 0.5 m NaBr solution has a higher vapour pressure
(a) Molecular mass of X is greater than the molecular than a 0.5 m BaCl2 solution at the same temperature
mass of Y. (ii) Pure water freezes at the higher temperature than pure
(b) Molecular mass of X is less than the molecular mass methanol
of Y. (iii) a 0.1 m NaOH solution freezes at a lower temperature
(c) Y is undergoing dissociation in water while X than pure water
undergoes no change. Choose the correct answer from the codes given below
(d) X is undergoing dissociation in water. (a) (i), (ii) and (iii) (b) (i) and (ii)
31. Which one is not equal to zero for an ideal solution: (c) (ii) and (iii) (d) (i) and (iii)
[AIPMT 2015, C] 41. The freezing point of benzene decreases by 0.45°C when
(a) DSmix 0.2g of acetic acid is added to 20 g of benzene. If acetic
(b) DVmix acid associates to form a dimer in benzene, percentage
(c) DP = Pobserved – PRaoult association of acetic acid in benzene will be
(d) DHmix (Kf for benzene = 5.12 K kg mol–1) [JEE M 2017, S]
32. What is the mole fraction of the solute in a 1.00 m aqueous (a) 64.6% (b) 80.4% (c) 74.6% (d) 94.6%
solution ? [AIPMT 2015, A] 42. For 1 molal aqueous solution of the following compounds,
(a) 0.177 (b) 1.770 (c) 0.0354 (d) 0.0177 which one will show the highest freezing point?
33. 3 g of activated charcoal was added to 50 mL of acetic acid [JEE M 2018, A, C]
solution (0.06N) in a flask. After an hour it was filtered and (a) [Co(H2O)6]Cl3 (b) [Co(H2O)5Cl]Cl2.H2O
the strength of the filtrate was found to be 0.042 N. The
(c) [Co(H2O)4Cl2]Cl. 2H2O (d) [Co(H2O)3Cl3].3H2O
amount of acetic acid adsorbed (per gram of charcoal) is :
43. For an ideal solution, the correct option is:
[JEE M 2015, S]
[NEET 2019, C]
(a) 42 mg (b) 54 mg (c) 18 mg (d) 36 mg
(a) Dmix S = 0 at constant T and P
34. The vapour pressure of acetone at 20°C is 185 torr. When
1.2 g of a non-volatile substance was dissolved in 100 g of (b) Dmix V ¹ 0 at constant T and P
acetone at 20°C, its vapour pressure was 183 torr. The (c) Dmix H = 0 at constant T and P
molar mass (g mol–1) of the substance is :[JEE M 2015, S] (d) Dmix G = 0 at constant T and P
(a) 128 (b) 488 (c) 32 (d) 64 44. Liquid ‘M’ and liquid ‘N’ form an ideal solution. The vapour
35. Which of the following statement about the composition pressures of pure liquids ‘M’ and ‘N’ are 450 and 700
of the vapour over an ideal 1 : 1 molar mixture of benzene mmHg, respectively, at the same temperature. Then correct
and toluene is correct? Assume that the temperature is statement is: [JEE M 2019, S]
constant at 25°C. (Given : Vapour Pressure Data at 25°C, (xM = Mole fraction of ‘M’ in solution;
benzene = 12.8 kPa, toluene = 3.85 kPa) [NEET 2016, S] xN = Mole fraction of ‘N’ in solution;
(a) The vapour will contain a higher percentage of yM = Mole fraction of ‘M’ in vapour phase;
benzene yN = Mole fraction of ‘N’ in vapour phase)
(b) The vapour will contain a higher percentage of xM yM
toluene (a) x = y (b) (xM – yM) < (xN – yN)
N N
(c) The vapour will contain equal amounts of benezene
and toluene xM yM xM yM
(d) Not enough information is given to make a predication (c) x < y (d) x > y
N N N N
SOLUTIONS 237

45. Which one of the following statements regarding Henry’s 48. The freezing point depression constant (Kf) of benzene is
law is not correct? [JEE M 2019, S] 5.12 K kg mol–1. The freezing point depression for the
(a) Higher the value of KH at a given pressure, higher is solution of molality 0.078 m containing a non-electrolyte
the solubility of the gas in liquids. solute in benzene is (rounded off upto two decimal places):
(b) Different gases have different KH (Henry’s law [NEET 2020, A]
constant) values at the same temperature. (a) 0.80K (b) 0.40 K (c) 0.60 K (d) 0.20 K
(c) The partial pressure of the gas in vapour phase is 49. The mixture which shows positive deviation from Raoult's
proportional to the mole fraction of the gas in the law is [NEET 2020, S]
solution. (a) Benzene + Toluene
(d) The value of K H increases with increase of (b) Acetone + Chloroform
temperature and KH is function of the nature of the (c) Chloroethane + Bromoethane
gas. (d) Ethanol + Acetone
46. The osmotic pressure of a dilute solution of an ionic 50. At 35 °C, the vapour pressure of CS2 is 512 mm Hg and that
compound XY in water is four times that of a solution of of acetone is 344 mm Hg. A solution of CS2 in acetone has
0.01 M BaCl2 in water. Assuming complete dissociation of a total vapour pressure of 600 mm Hg. The false statement
the given ionic compounds in water, the concentration of amongst the following is: [JEE M 2020, S]
XY (in mol L–1) in solution is: [JEE M 2019, S] (a) Raoult’s law is not obeyed by this system
(a) 4 × 10–2 (b) 6 × 10–2 (c) 4 × 10–4 (d) 16 × 10–4 (b) a mixture of 100 mL CS2 and 100 mL acetone has a
47. A solution of sodium sulfate contains 92 g of Na + ions per volume < 200 mL
kilogram of water. The molality of Na+ ions in that solution (c) CS2 and acetone are less attracted to each other than
in mol kg–1 is: [JEE M 2019, S] to themselves
(a) 12 (b) 4 (c) 8 (d) 16 (d) heat must be absorbed in order to produce the solution
at 35 °C

Exercise 4 : Problem Solving Skill Enhancer MCQs


1. Formation of a solution from two components can be fractions of A in the solution and vapour in equilibrium,
considered as then
(i) Pure solvent ® separated solvent molecules, DH1
(ii) Pure solute ® separated solute molecules, DH2 X 'A X 'A
(a) =1 (b) >1
(iii) Separated solvent & solute molecules ® solution, XA XA
DH3
Solution so formed will be ideal if X 'A
(c) <1 (d) X 'A + X A =1
(a) DH soln = DH 3 -DH1 -DH 2 XA
(b) DH soln = DH1 + DH 2 + DH 3 5. Equal mass of a solute are dissolved in equal mass of two
solvents A and B and formed very dilute solution. The
(c) DH soln = DH1 + DH 2 -DH 3
relative lowering of vapour pressure for the solution B
(d) DH soln = DH1 -DH 2 -DH 3 has twice the relative lowering of vapour pressure for the
2. The solubility of a specific non-volatile salt is 4 g in 100 g solution A. If MA and MB are the molecular mass of solvents
of water at 25°C. If 2.0 g., 4.0 g and 6.0 g of the salt added A and B respectively, then:
to100 g of water at 25°C, in system X, Y and Z. The vapour (a) MA = MB (b) MB = 2 × MA
pressure would be in the order: (c) MA = 4MB (d) MA = 2MB
(a) X < Y < Z (b) X > Y > Z 6. If M normal is the normal molecular mass and a is the degree
(c) Z > X = Y (d) X > Y = Z of ionization of K3[Fe(CN)6], then the abnormal molecular
3. Two liquids A and B have vapour pressure in the ratio mass of the complex in the solution will be:
(a) M normal (1 + 2a)–1 (b) M normal (1 + 3a)–1
PA° : PB° = 1 : 3 at a certain temperature. Assume A and B
(c) M normal (1 + a) –1 (d) equal to Mnormal
form an ideal solution and the ratio of mole fractions of A 7. p1, p2, p3 and p4 atm are the osmotic pressures of 5% (mass/
to B in the vapour phase is 4 : 3, then the mole fraction of volume) solutions of urea, fructose, sucrose and KCl
B in the solution at the same temperature is : respectively at certain temperature. The correct order of
1 2 4 1 their magnitudes is :
(a) (b) (c) (d)
5 3 5 4 (a) p1 > p 4 > p 2 > p 3 (b) p1 < p 4 < p 2 < p 3
4. Liquids A and B form an ideal solution and the former has (c) p 4 > p1 > p 2 > p 3 (d) p 4 > p1 > p 3 > p 2
stronger intermolecular forces. If XA and X'A are the mole
EBD_8350
238 CHEMISTRY

8. Two moles of a liquid A ( PºA = 100 torr ) and 3 moles of (molar mass of S = 32 g mol–1 and that of Na = 23 g mol–1)
(a) 15 g (b) 25 g (c) 45 g (d) 65 g
liquid B ( PºB = 150 torr ) form a solution having vapour 11. CNS– ions give red colour with Fe3+ ions in aqueous
pressure of 120 torr. Based upon this observation one solution as :
can conclude :
(a) Interactions between like molecules > those between Fe3+ ( aq ) + 3CNS- ( aq ) ® Fe(CNS)3 ( aq )
red
unlike molecules If 0.1M KCNS solution is separated from 0.1M FeCl3
(b) Interaction between like molecules < those between solution by means of semipermeable membrane, red colour
unlike molecules will appear on:
(c) Interaction between like molecules = those between (a) FeCl3 solution side (b) KCNS solution side
unlike molecules (c) both sides (d) neither side
(d) DSmixing = 0 12. 0.010M solution of an acid HA freezes at –0.0205ºC. If Kf
9. X3Y2(i = 5) when reacted with A2B3(i = 5) in aqueous for water is 1.860 K kg mol–1, the ionization constant of the
solution, gives brown colour. These are separated by a conjugate base of the acid will be (assume 0.010
semipermeable membrane PQ as shown. Due to osmosis M = 0.010 m)
there is: (a) 1.1 × 10–4 (b) 1.1 × 10–3
0.5 M 0.01 M (c) 9.0 × 10–11 (d) 9.0 × 10–12
X3Y2 A 2B 3 13. When the super cooled liquid starts freezing, the
P
temperature
(a) falls to freezing point (b) rises to freezing point
Side R Side S (c) fluctuates (d) remains constant
14. The correct relationship between the boiling points of very
Q
dilute solutions of AlCl3 (T1) and CaCl2 (T2), having the
SPM same molar concentration is
(a) brown colour formation in side R (a) T1 = T2 (b) T1 > T2 (c) T2 > T1 (d) T2 ³ T1
(b) brown colour formation in side S
(c) formation in both of the sides R and S 15. Which of the following terms has units of temperature?
(d) no brown colour formation mK f
10. 5 g of Na2SO4 was dissolved in x g of H2O. The change in I: Kf II : mK f III : iK f IV :
freezing point was found to be 3.82°C. If Na2SO4 is 81.5% i
ionised, the value of x (Kf for water = 1.86°C kg mol –1) is (a) I, II (b) I, III
approximately : (c) II, III (d) II, IV

ANSWER KEY
Exercise 1 : NCERT Based Topic-wise MCQs
1 (c) 12 (c) 23 (d) 34 (c) 45 (b) 56 (c) 67 (c) 78 (c) 89 (d) 100 (a)
2 (b) 13 (a) 24 (d) 35 (d) 46 (c) 57 (c) 68 (a) 79 (c) 90 (b) 101 (d)
3 (c) 14 (b) 25 (b) 36 (d) 47 (d) 58 (d) 69 (b) 80 (b) 91 (b) 102 (a)
4 (c) 15 (a) 26 (b) 37 (d) 48 (a) 59 (d) 70 (a) 81 (d) 92 (a) 103 (a)
5 (b) 16 (c) 27 (d) 38 (c) 49 (b) 60 (b) 71 (b) 82 (a) 93 (c) 104 (c)
6 (d) 17 (c) 28 (d) 39 (d) 50 (d) 61 (d) 72 (a) 83 (b) 94 (a) 105 (b)
7 (b) 18 (d) 29 (a) 40 (d) 51 (d) 62 (a) 73 (d) 84 (a) 95 (c) 106 (a)
8 (d) 19 (c) 30 (b) 41 (a) 52 (b) 63 (b) 74 (c) 85 (a) 96 (a) 107 (d)
9 (d) 20 (a) 31 (c) 42 (a) 53 (a) 64 (c) 75 (c) 86 (a) 97 (d) 108 (c)
10 (d) 21 (a) 32 (a) 43 (c) 54 (a) 65 (b) 76 (a) 87 (d) 98 (b) 109 (a)
11 (b) 22 (c) 33 (b) 44 (a) 55 (a) 66 (d) 77 (c) 88 (b) 99 (c) 110 (d)
Exercise 2 : Numeric/Integer Answer Questions
1 (3) 2 (0.017) 3 (0.36) 4 (0.06) 5 (11.7) 6 (111) 7 (12.65) 8 (3.84) 9 (65.25) 10 (5.92)
Exercise 3 : NCERT Exemplar & Past Year MCQs
1 (a) 6 (b) 11 (b) 16 (c) 21 (b) 26 (c) 31 (a) 36 (a) 41 (d) 46 (b)
2 (d) 7 (b) 12 (a) 17 (a) 22 (c) 27 (c) 32 (d) 37 (a) 42 (d) 47 (b)
3 (c) 8 (a) 13 (b) 18 (d) 23 (b) 28 (a) 33 (c) 38 (c) 43 (c) 48 (b)
4 (b) 9 (c) 14 (a) 19 (a) 24 (d) 29 (c) 34 (d) 39 (a) 44 (d) 49 (d)
5 (b) 10 (a) 15 (d) 20 (b) 25 (a) 30 (d) 35 (a) 40 (d) 45 (a) 50 (b)
Exercise 4 : Problem Solving Skill Enhancer MCQs
1 (b) 3 (a) 5 (b) 7 (c) 9 (d) 11 (d) 13 (b) 15 (d)
2 (d) 4 (c) 6 (b) 8 (b) 10 (c) 12 (c) 14 (b)
17 Electrochemistry

Trend Buster NEET & JEE Main

Number of Questions from 2020-15 10 7 Minimum one question has been


Weightage 3.1% 4% asked every year in NEET and JEE M.

The most Important Concepts that Cover Maximum number of Questions asked in past 6 years.

Nernst equation 5 3
Faraday's law / electrolysis 2 3

Less Important Concepts that Cover 1 or 2 Questions asked in past 6 years.

Batteries /fuel cells/corrosion 1 1


Conductance of electrolytic solutions 2 —

NEET JEE

2020 Nernst’s equation/ electrode potential of the cell/ 2 Average 1 Average


Electrolytic cells and Faraday’s law of electrolysis/
Electrolysis Electrolysis

2019 Nernst’s equation / Nernst's equation / standard 2 Average 2 Average


Electrolytic cells and Gibb's energy / equilibrium
electrolysis constant of cell reaction /
Faraday's Law of electrolysis /
Nernst 's equation
2018 Electrolytic cell and Electrolysis / Nernst 's equation 1 Average 1 Average
electrolysis / Nernst
equation
2017 Nernst's equation / Ionic mobility / Nernst's equation / 2 Average 1 Average
electrolytic cells and oxidising and reducing agents
electrolysis
2016 Nernst's equation / Nernst's equation/corrosion 1 Average 1 Average
batteries, fuel cells and
corrosion
2015 Conductance of Conductance / fuel cell / 2 Average 1 Average
electrolytic solutions / Faraday's law
batteries / fuel cells and
corrosion / Electrolytic
cells and electrolysis
EBD_8350
240 CHEMISTRY
ELECTROCHEMISTRY 241
EBD_8350
242 CHEMISTRY

Problem Solving Tips/ Tricks/ Points to Remember

4 Name Formula SI Unit


Potential difference (E)
Resistance (R) R= ohm (Volts/Amperes)
Current (I)
1 –1
Conductance (G) ohm–1 (W ) or siemens (S)
R
a
Specific resistance (r) r = R´ ohm metre (W m)
l
–1
Specific conductance (k) k = r–1 ohm–1 metre–1(W m–1) or S/m
k ´ 1000
Equivalent conductivity (Leq) L eq = sm2 eq–1
N
k ´1000
Molar conductivity (Lm) Lm = sm2 mol–1
M
4 The least active ion (ions having lower discharge (a) Lead storage battery :
potential) is discharged first at the respective electrode. Anode - Spongy lead
The decreasing order of activity or increasing order Cathode - grid of lead packed with PbO2
of deposition of few ions is as below. Electrolyte - 38% H2SO4 by mass
· For cations : Li+, K+, Na+, Ca2+, Mg2+, Al3+, Zn2+, Fe2+, (b) Nickel-cadmium storage cell
Ni2+, H+, Cu2+, Hg2+, Ag+, Au3+. Anode - Cadmium
Thus in aqueous solution containing any of the cation Cathode - metal grid containing NiO2
Li+, Na+, K+, Ba2+, Ca2+, Mg2+ or Al3+, it is water which Electrolyte - KOH solution
is reduced at the cathode and not the metal cations. 4 According to convention, a half-cell called standard
· For anions : SO 24- , NO3- , OH - , Cl - , Br - , I - . hydrogen electrode represented by Pt(s) / H2(g) / H+ (aq)
is assigned a zero potential at all temperatures
Thus in aqueous solution of the SO42– and NO3–, it is
corresponding to the reaction.
the water which is oxidised at the anode and not the
anions. 1
H+ (aq) + e– ¾® H2 ( g)
4 Degree of Dissociation (a) 2
For weak electrolytes This consist of a platinum electrode coated with platinum
black. The electrode is dipped in an acidic solution of
L cm Molar conductivity at a given concentration concentration of 1.0 M H+ ion and pure hydrogen gas at 1
a= =
L ¥m Molar conductivity at infinite dilution bar is bubbled through it.
4 To measure the standard reduction electrode potential
4 KOHLRAUSCH'S LAW
value of E°cu2+/Cu, we will make standard hydrogen
At infinite dilution the molar conductivity of an electrolyte electrode as anode (reference half-cell) and the other half-
is the sum of the ionic conductivities of the cations and cell as cathode (Cu2+/Cu) :
anions. e.g. for AxBy. Pt (s) | H2(g, 1bar)| H+ (aq, 1M) || Cu2+ (aq, 1M) | Cu
( ) ( )
L ¥m (A x B y ) = xl ¥ A + + yl ¥ B - The measured emf of this cell is + 0.34 V which is equal to
the value of EC° u2+ /Cu .
4 (i) Primary cells : They cannot be recharged and used
again. Examples are : 4 At infinite dilutions, the molar conductivity of cation and
(a) Dry cell or Leclanche cell : anion are l ¥ ¥
+ m and l - m respectively and¥the equivalent
Anode - Zinc Container conductivity of cation and anion are l +eq and l¥ -eq
Cathode - graphite rod surrounded by MnO2 powder
Electrolyte - paste of NH4Cl + ZnCl2
¥
( )
respectively. The values of molar conductivity Ù m and
(b) Mercury Cell :
¥
( )
equivalent conductivity Ùeq for the electrolyte AxBy :
Anode - Zn-Hg amalgam ˆˆ† xA y + + yB x -
Ax B y ‡ˆˆ
Cathode - paste of (HgO + C)
Electrolyte - moist paste of KOH-ZnO Ù¥ ¥ ¥
m = xl + m + yl - m and
(ii) Secondary cells : It can be recharged and can be
1 ¥ 1
used again and again. Examples are : Ù¥
eq = l+m + l¥ ¥ ¥ ¥
- m or Ù eq = l + eq + l –eq
y x
ELECTROCHEMISTRY 243

Exercise 1 : NCERT Based Topic-wise MCQs


Topic 1: Electrochemical Cells and Galvanic Cells (i) Cu is reducing agent.
(ii) Overall cell reaction is
1. Which of the following statements about galvanic cell is
incorrect Cu ( s ) + 2Ag + ( aq ) ¾¾® Cu 2+ ( aq ) + 2Ag ( s )
(a) anode is positive (iii) Cu is cathode
(b) oxidation occurs at the electrode with lower reduction (iv) Ag is anode
potential (a) (ii), (iii) and (iv)
(c) cathode is positive (b) (ii), (iii) and (iv)
(d) reduction occurs at cathode (c) (iii) and (iv)
2. Reaction that takes place at anode in dry cell is (d) (i) and (ii)
(a) Zn 2+ + 2e- ¾¾
® Zn(s) 9. The reference electrode is made by using
(b) ® Zn 2+ + 2e -
Zn(s) ¾¾ (a) ZnCl2 (b) CuSO4
(c) HgCl2 (d) Hg2Cl2
(c) Mn 2 + + 2e - ¾¾
® Mn(s)
10. Standard electrode potential for Sn 4+ / Sn2+ couple is
(d) Mn(s) ¾¾ ® Mn + + e - + 1.5V
+ 0.15 V and that for the Cr 3+ / Cr couple is – 0.74 V. These
3. In which of the following conditions salt bridge is not
two couples in their standard state are connected to make
required in a galvanic cell?
a cell. The cell potential will be
(a) When galvanic cell is used in geyser. (a) + 1.19 V (b) + 0.89 V
(b) When distance between oxidation half cell and (c) + 0.18 V (d) + 1.83 V
reduction half cell is negligible. 11. A button cell used in watches functions as following
(c) Electrolytic solutions used in both the half cells are Zn(s) + Ag2O(s) + H2O(l)
of same concentration.
2Ag(s) + Zn2+(aq) + 2OH–(aq)
(d) When both the electrodes are dipped in the same
electrolytic solution. If half cell potentials are :
4. Which device converts chemical energy of a spontaneous Zn2+(aq) + 2e– ¾® Zn(s); Eo = – 0.76 V
redox reaction into electrical energy? Ag2O(s) + H2O (l) + 2e– ¾® 2Ag(s) + 2OH–(aq);
(a) Galvanic cell (b) Electrolytic cell Eo = 0.34 V
(c) Daniell cell (d) Both (a) and (c) o
The E cell will be :
(a) 0.42 V (b) 0.84 V
5. Zn (s) | Zn 2 + (aq) | | Cu 2 + (aq) |Cu(s) is
( anode ) ( cathode ) (c) 1.34 V (d) 1.10 V
(a) Weston cell (b) Daniell cell 12. Which of the following statements regarding given cell
(c) Calomel cell (d) Faraday cell representation is/are correct?
6. The tendency of an electrode to lose electrons is known Cd(s)|Cd2+(aq)||Ag+(aq)|Ag(s)
as (i) In the given cell ,Cd electrode act as an anode whereas
(a) electrode potential (b) reduction potential Ag electrode acts as a cathode.
(c) oxidation potential (d) e.m.f. (ii) In the given cell, Cd electrode acts as a cathode
7. The chemical reaction, whereas Ag electrode acts as a anode.
2AgCl(s) + H 2 (g) ¾¾
® 2HCl(aq) + 2Ag(s) (iii) Ecell = E -E
Ag + / Ag Cd 2+ / Cd
taking place in a galvanic cell is represented by the notation
(a) (i) and (ii) (b) Only (ii)
(a) Pt(s) | H 2 (g),1 bar |1M KCl(aq) | AgCl(s) | Ag(s) (c) Only (i) (d) (i) and (iii)
13. If salt bridge is removed from two half-cells the voltage
(b) Pt(s) | H 2 (g),1 bar |1M HCl(aq) |1MAg + (aq) | Ag(s)
(a) drops to zero (b) does not change
(c) Pt(s) | H 2 (g),1 bar |1M HCl(aq) | AgCl(s) | Ag(s) (c) increases gradually (d) increases rapidly
(d) Pt(s) | H 2 (g),1 bar |1M HCl(aq) | Ag(s) | AgCl(s) 14. In the electrolytic cell, flow of electrons is from
(a) cathode to anode in solution
8. For cell representation: (b) cathode to anode through external supply
Cu(s)|Cu2+(aq)||Ag+(aq)|Ag(s) (c) cathode to anode through internal supply
Which of the following is correct? (d) anode to cathode through internal supply
EBD_8350
244 CHEMISTRY

23. A smuggler could not carry gold by depositing iron on the


Topic 2: Nernst Equation
gold surface since
15. For the given Nernst equation (a) gold is denser
é 2+ ù
RT ë Mg û (b) iron rusts
E cell = E°cell - ln (c) gold has higher reduction potential than iron
2F é + ù 2
Ag (d) gold has lower reduction potential than iron
ë û
Which of the following representation is correct? 24. The standard e.m.f. of a galvanic cell involving cell reaction
(a) Ag+|Ag||Mg2+|Mg (b) Mg2+|Mg||Ag|Ag+ with n = 2 is found to be 0.295 V at 25°C. The equilibrium
constant of the reaction would be
(c) Mg|Mg2+||Ag+|Ag (d) Mg|Mg2+||Ag|Ag+
(Given F = 96500 C mol–1; R = 8.314JK–1mol–1)
16. Standard electrode potential of three metals X, Y and Z are (a) 2.0 × 1011 (b) 4.0 × 1012
– 1.2 V, + 0.5 V and – 3.0 V, respectively. The reducing (c) 1.0 × 10 2 (d) 1.0 × 1010
power of these metals will be : 25. According to Nernst equation, which is not correct if
(a) Y > Z > X (b) X > Y > Z Q = Kc :
(c) Z > X > Y (d) X > Y > Z RT o
(a) Ecell = 0 (b) ln Q = Ecell
17. Standard reduction potentials of the half reactions are nF
o
nFEcell
given below : o
(c) Kc =e RT (d) Ecell = Ecell
F2(g) + 2e– ® 2F– (aq); E° = + 2.85 V
Cl2(g) + 2e– ® 2Cl–(aq); E° = + 1.36 V 26. The standard emf of a cell, involving one electron change
is found to be 0.591 V at 25°C. The equilibrium constant of
Br2(l) + 2e– ® 2Br–(aq); E° = + 1.06 V
the reaction is (F = 96500 C mol–1)
I2(s) + 2e– ® 2I–(aq); E° = + 0.53 V
(a) 1.0 × 101 (b) 1.0 × 105
The strongest oxidising and reducing agents (c) 1.0 × 10 10 (d) 1.0 ×1030
respectively are 27. For the galvanic cell Zn | Zn (0.1M) || Cu2+ (1.0M)|Cu the
2+
(a) F2 and I– (b) Br2 and Cl– cell potential increase if:
(c) Cl2 and Br – (d) Cl2 and I2 (a) [Zn2+] is increased
18. The value of electrode potential (10–4 M) H+ | H2(1 atm) | Pt (b) [Cu2+] is increased
at 298 K would be (c) [Cu2+] is decreased
(a) – 0.236 V (b) + 0.404 V (d) surface area of anode is increased
(c) + 0.236 V (d) – 0.476 V 28. What is the standard cell potential E° for an electrochemical
cell in which the following reaction takes place
19. The electrode potential E of a zinc electrode at
(Zn 2 + Zn) spontaneously ?
25°C with an aqueous solution of 0.1 M ZnSO4 is Cl 2 (g) + 2Br - ¾¾
® Br2 (aq) + 2Cl - ; D G ° = -50.6 kJ
2.303RT (a) 1.2 V (b) 0.53 V
[ E° 2+ = –0.76 V. Assume = 0.06 at 298 K].
(Zn Zn) F (c) 0.26 V (d) –0.53 V
(a) + 0.73 (b) – 0.79 29. Which of the following is/are correct statement(s) for the
(c) – 0.82 (d) – 0.70 addition of Li, K, Rb to the aqueous solution of Na +.
20. For a cell reaction involving two electron change, the (i) The correct order of metals in which they reduce the
standard EMF of the cell is 0.295 V at 2°C. The equilibrium Na+ ion is Rb < K < Li.
constant of the reaction at 25°C will be: (ii) Reduction of metal ions would not take place.
(a) 29.5 × 10–2 (b) 10 (a) Statement (i) and (ii) are correct.
(c) 1 × 1010 (d) 2.95 × 10–10
(b) Statement (i) is correct only.
21. Without losing its concentration; ZnCl2 solution cannot
(c) Statement (ii) is correct only.
be kept in contact with
(a) Au (b) Al (d) Neither (i) nor (ii) is correct.
(c) Pb (d) Ag 30. Standard potentials (Eº) for some half-reactions are given
22. The oxidation potentials of A and B are +2.37 and +1.66 V below :
respectively. In chemical reactions (i) Sn 4 + + 2e - ® Sn 2 + ; E º = +0.15 V
(a) A will be replaced by B (ii) 2 Hg 2+ + 2e - ® Hg 2 2+ ; E º = +0.92 V
(b) A will replace B (iii) PbO2 + 4H+ + 2e- ® Pb 2+ + 2H 2O ; E º = +1.45 V
(c) A will not replace B Based on the above, which one of the following statements
(d) A and B will not replace each other is correct ?
ELECTROCHEMISTRY 245

(a) Sn4+ is a stronger oxidising agent than Pb4+ 38. Consider the following cell reaction:
(b) Sn2+ is a stronger reducing agent than Hg22+ 2Fe(s) + O 2 (g) + 4H + (aq) ®
(c) Hg2+ is a stronger oxidising agent than Pb4+ 2Fe 2+ (aq) + 2H 2 O(l) ; E ° = 1.67V
(d) Pb2+ is a stronger reducing agent than Sn 2+ At [Fe2+] =10–3 M, p(O2) = 0.1 atm and pH = 3, the cell
31. Consider the following four electrodes: potential at 25ºC is
P = Cu2+ (0.0001 M)/Cu(s)
(a) 1.47 V (b) 1.77 V
Q = Cu2+ (0.1 M)/Cu(s)
R = Cu2+ (0.01 M)/Cu(s) (c) 1.87 V (d) 1.57 V
S = Cu2+ (0.001 M)/Cu(s) 39. At 298K, the standard free energy of formation of H2O (l)
If the standard reduction potential of Cu2+/Cu is +0.34 V, is –237.20 kJ/mole while that of its ionisation into H+ ion
the reduction potentials in volts of the above electrodes and hydroxyl ions is 80 kJ/mole, then the emf of the
follow the order. following cell at 298 K will be
(a) P > S > R > Q (b) S > R > Q > P [Take Faraday constant F = 96500C]
(c) R > S > Q > P (d) Q > R > S > P H2 (g , 1 bar) | H+ (1M) | | OH– (1M) | O2(g, 1 bar)
32. If the following half cells have E° values as (a) 0.40 V (b) 0.81V
A3+ + e– ––® A2+, E° = y2V (c) 1.23 V (d) – 0.40 V
A2+ + 2e– –––® A, E° = –y1V 40. For a relation
The E° of the half cell, A3+ + 3e– ––® A will be D r G = - nFEcell
2 y1 - y2 y2 - 2 y1 Ecell = E°cell in which of the following condition?
(a) (b)
3 3 (a) Concentration of any one of the reacting species
(c) 2y1 – 3y2 (d) y2 – 2y1 should be unity
33. In a cell that utilises the reaction (b) Concentration of all the product species should be
Zn(s) + 2H + (aq) ® Zn 2 + (aq) + H 2 (g) ; addition of unity.
H2SO4 to cathode compartment, will (c) Concentration of all the reacting species should be
(a) increase the E and shift equilibrium to the right unity.
(b) lower the E and shift equilibrium to the right (d) Concentration of all reacting and product species
(c) lower the E and shift equlibrium to the left should be unity.
(d) increase the E and shift equilibrium to the left
Topic 3: Conductance of Electrolytic Solutions
34. Standard cell voltage for the cell Pb | Pb2+ || Sn2+ | Sn is
– 0.01 V. If the cell is to exhibit Ecell = 0, the value of 41. If 0.01 M solution of an electrolyte has a resistance of 40
[Sn 2+] / [Pb2+] should be antilog of – ohms in a cell having a cell constant of 0.4 cm–1, then its
(a) + 0.3 (b) 0.5 molar conductance in ohm–1 cm2 mol–1 is
(c) 1.5 (d) – 0.5 (a) 102 (b) 104
(c) 10 (d) 103
35. The cell, Zn | Zn 2+ (1 M) || Cu 2+ (1 M) | Cu ( E °cell = 1.10 V) 42. Specific conductance of a 0.1 N KCl solution at 23ºC is
was allowed to be completely discharged at 298 K. The 0.012 ohm–1 cm–1. Resistance of cell containing the solution
at same temperature was found to be 55 ohm. The cell
æ [Zn 2+ ] ö constant is
relative concentration of Zn2+ to Cu2+ çç 2+ ÷
÷ is
è [Cu ] ø (a) 0.0616 cm–1 (b) 0.66 cm–1
(a) 9.65 × 104 (b) antilog (24.08) (c) 6.60 cm–1 (d) 660 cm–1
(c) 37.3 (d) 1037.3. 43. The unit of equivalent conductivity is
36. What is the potential of half-cell consisting of (a) ohm cm
zinc electrode in 0.01 M ZnSO 4 solution at 25°C (b) ohm–1 cm2 (g equivalent)–1

(E )
o (c) ohm cm2 (g equivalent)
ox = 0.763 V
(d) S cm–2
(a) 0.8221 V (b) 8.221 V 44. Specific conductance of 0.1 M HNO3 is 6.3×10–2 ohm–1 cm–1.
(c) 0.5282 V (d) 9.282 V The molar conductance of the solution is
37. The oxidation potential of 0.05 M H2SO4 is (a) 100 ohm–1 cm2 (b) 515 ohm–1 cm2
(c) 630 ohm cm–1 2 (d) 6300 ohm–1 cm2
(a) –2 × 0.0591 (b) –0.01 × 0.0591
(c) –2.321 × 0.0591 (d) +1 × 0.0591
EBD_8350
246 CHEMISTRY

45. The unit of specific conductivity is 53. Which of the following solutions of KCl will have the
(a) ohm cm–1 (b) ohm cm–2 highest value of specific conductance?
(c) ohm cm –1 (d) ohm–1 cm–1 (a) 1.0 N (b) 0.1 N
46. Which of the following pair(s) is/are incorrectly matched? (c) 1.0 ×10–2N (d) 1.0 ×10–3N
(i) R (resistance) – ohm (W) 54. Which of the following expression correctly represents
(ii) r (resistivity) – ohm metre (Wm) molar conductivity?
(iii) G (conductance) – seimens or ohm (S)
K KA
(iv) k (conductivity) – seimens metre–1 (Sm–1) (a) Ùm = (b) Ùm =
C l
(a) (i), (ii) and (iii) (b) (ii) and (iii)
(c) Ù m = KV (d) All of these
(c) (i), (ii) and (iv) (d) (iii) only
47. On which of the following, magnitude of conductivity does 55. Which of the following represents variation of molar
not depends? conductance of electrolyte with (concentration) ½
respectively for weak and strong electrolyte ?
(a) Nature of material (b) Temperature
(c) Pressure (d) Mass of the material (i) (ii)

m/(S cm mol )
m/(S cm mol )
–1

–1
48. Molar conductivities ( L° m ) at infinite dilution of NaCl,

2
HCl and CH3COONa are 126.4, 425.9 and 91.0 S cm2 mol–1

2
respectively. L°m for CH3COOH will be
(a) 425.5 S cm2 mol–1 (b) 180.5 S cm2 mol–1
(c) 290.8 S cm2 mol–1 (d) 390.5 S cm2 mol–1 1 1
c / (mol / L) 2 c / (mol / L) 2
49. At 25°C, molar conductance of 0.1 molar aqueous solution
of ammonium hydroxide is 9.54 ohm–1 cm2mol–1 and at
infinite dilution its molar conductance is 238 ohm–1 cm2 (iii) m/(S cm mol ) (iv)
–1

m/(S cm mol )
mol–1. The degree or ionisation of ammonium hydroxide at

–1
the same concentration and temperature is:
2

2
(a) 20.800% (b) 4.008%
(c) 40.800% (d) 2.080%
50. The electr ical properties and their respective 1 1
SI units are given below. Identify the wrongly matched c / (mol / L) 2 c / (mol / L) 2
pair.
Electrical property SI unit (v)
m/(S cm mol )
–1

(a) Specific conductance S m–1


(b) Conductance S
2

(c) Equivalent conductance S m2 geq–1


(d) Cell constant m
51. Assertion : The resistivity for a substance is its resistance 1
when it is one meter long and its area of cross section is c / (mol / L) 2
one square meter.
Weak acid Strong acid
Reason : The SI units of resistivity is ohm metre (Wm).
(a) (iv) (v)
(a) If both assertion and reason are true and reason is
the correct explanation of assertion. (b) (ii) (iv)
(b) If both assertion and reason are true but reason is not (c) (i) (ii)
the correct explanation of assertion. (d) (iii) (ii)
(c) If assertion is true but reason is false. Electrolyte: KCl KNO 3 HCl NaOAc NaCl
56.
(d) If both assertion and reason are false. ¥ 2 –1
L (S cm mol ) : 149.9 145 426.2 91 126.5
52. The resistance of 0.01 N solution of an electrolyte was
found to be 220 ohm at 298 K using a conductivity cell Calculate L¥
using appropriate molar conductances
HOAc
with a cell constant of 0.88cm–1. The value of equivalent of the electrolytes listed above at infinite dilution in H2O
conductance of solution is – at 25°C
(a) 400 mho cm2 g eq–1 (b) 295 mho cm2 g eq–1 (a) 217.5 (b) 390.7
2
(c) 419 mho cm g eq –1 (d) 425 mho cm2 g eq–1
(c) 552.7 (d) 517.2
ELECTROCHEMISTRY 247

57. Kohlrausch’s law states that at 64. The equivalent conductances of two strong electrolytes
(a) finite dilution, each ion makes definite contribution at infinite dilution in H2O (where ions move freely through
to equivalent conductance of an electrolyte, whatever a solution) at 25°C are given below :
be the nature of the other ion of the electrolyte.
Lo CH3COONa = 91.0 S cm 2 / equiv.
(b) infinite dilution each ion makes definite contribution
to equivalent conductance of an electrolyte
Lo HCl = 426.2 S cm 2 / equiv.
depending on the nature of the other ion of the
electrolyte. What additional information/ quantity one needs to calcu-
(c) infinite dilution, each ion makes definite contribution late Lo of an aqueous solution of acetic acid?
to conductance of an electrolyte whatever be the (a) L o of chloroacetic acid (ClCH2COOH)
nature of the other ion of the electrolyte. (b) L o of NaCl
(d) infinite dilution, each ion makes definite contribution (c) L o of CH3COOK
to equivalent conductance of an electrolyte, whatever
(d) the limiting equivalent coductance of H + (l° ).
be the nature of the other ion of the electrolyte. H+
58. Which of the following expressions correctly represents 65. Resistance of 0.2 M solution of an electrolyte is 50 W. The
the equivalent conductance at infinite dilution of Al2(SO4)3, specific conductance of the solution is 1.3 S m –1. If
resistance of the 0.4 M solution of the same electrolyte is
Given that L °Al3+ an d L°SO2- are the equivalent
4 260 W, its molar conductivity is :
conductances at infinite dilution of the respective ions?
(a) 6.25 × 10–4 S m2 mol–1 (b) 625 × 10–4 S m2 mol–1
1 ° 1 2L° + 3L°
(a) L 3+ + L° 2- (b) Al3+ SO24-
(c) 62.5 S m2 mol–1 (d) 6250 S m2 mol–1
3 Al 2 SO4 66. The limiting molar conductivities of HCl, CH3COONa and
L° + L° æ ° 2- ö ´ 6 NaCl are respectively 425, 90 and 125 mho cm2 mol–1 at
(c) Al3+ SO42 - (d) èL°Al3+ + LSO
4 ø 25°C. The molar conductivity of 0.1 M CH3COOH solutions
59. At 25°C, the molar conductance at infinite dilution for the is 7.8 mho cm2 mol–1 at the same temperature. The degree
strong electrolytes NaOH, NaCl and BaCl2 are 248 × 10–4, of dissociation of 0.1 M acetic acid solution at the same
126 × 10 –4 and 280 × 10 –4 Sm 2mol –1 respectively. temperature is
L 0m Ba(OH)2 in S m2 mol –1 is (a) 0.10 (b) 0.02 (c) 0.15 (d) 0.03
67. A weak electrolyte having the limiting equivalent
(a) 52.4 × 10–4 (b) 524 × 10–4
–4
conductance of 400 S cm 2. equivalent–1 at 298 K is 2%
(c) 402 × 10 (d) 262 × 10–4 ionized in its 0.1 N solution. The resistance of this solution
60. The ion of least limiting molar conductivity among the (in ohms) in an electrolytic cell of cell constant 0.4 cm–1 at
following is this temperature is
(a) SO42– (b) H+ (a) 200 (b) 300
(c) Ca 2+ (d) CH3COO– (c) 400 (d) 500
61. Molar ionic conductivities of a two-bivalent electrolytes 68. Arrange the following in increasing order of their
x 2+ and y 2– are 57 and 73 respectively. The molar conductivity Na+ (A), K+ (B), Ca2+ (C), Mg2+(D)
conductivity of the solution formed by them will be (a) A, B, C, D (b) B, A, C, D
(a) 130 S cm2 mol–1 (b) 65 S cm2 mol–1 (c) C, A, D, B (d) A, B, D, C
69. The conductivity of electrolytic solutions depends upon
(c) 260 S cm2 mol–1 (d) 187 S cm2 mol–1
which of the following?
62. 0.1 mole, per litre solution is present in a conductivity cell (i) Size of ions produced
where electrode of 100 cm2 area are placed at 1 cm apart (ii) Viscosity of the solvent
and resistance observed is 5 × 103 Ohm, what is molar (iii) Concentration of electrolyte
conductivity of solution?
(iv) Solvation of ions produced
(a) 5 × 102 S cm2 mole–1 (b) 2 × 104 S cm2 mole–1 (a) (i) and (iii) (b) (i), (ii) and (iii)
(c) 200 S cm2 mole–1 (d) 0.02 S cm2 mole–1 (c) (i), (iii) and (iv) (d) All of these
63. A 0.5 M NaOH solution offers a resistance of 31.6 ohm in a
conductivity cell at room temperature. What shall be the Topic 4: Electrolytic Cells and Electrolysis
approximate molar conductance of this NaOH solution if
cell constant of the cell is 0.367 cm–1. 70. If 0.5 amp current is passed through acidified silver nitrate
solution for 100 minutes. The mass of silver deposited on
(a) 234 S cm2 mole–1 (b) 23.2 S cm2 mole–1 cathode, is (eq.wt.of silver nitrate = 108)
(c) 4645 S cm2 mole–1 (d) 5464 S cm2 mole–1 (a) 2.3523 g (b) 3.3575 g
(c) 5.3578 g (d) 6.3575 g
EBD_8350
248 CHEMISTRY

71. Which of the following is the use of electrolysis? 81. How many moles of Pt may be deposited on the cathode
(a) Electrorefining (b) Electroplating when 0.80 F of electricity is passed through a 1.0 M
(c) Both (a) & (b) (d) None of these solution of Pt4+?
72. An electrolytic cell contains a solution of Ag2SO4 and has (a) 1.0 mol (b) 0.20 mol
platinum electrodes. A current is passed until 1.6 g of O2 (c) 0.40 mol (d) 0.80 mol
has been liberated at anode. The amount of silver deposited 82. On electrolysis of dilute sulphuric acid using platinum
at cathode would be electrodes, the product obtained at the anode will be
(a) 107.88 g (b) 1.6 g (a) hydrogen (b) oxygen
(c) 0.8 g (d) 21.60 g (c) hydrogen sulphide (d) Sulphur dioxide
73. When 9650 coulombs of electricity is passed through a 83. Aluminium oxide may be electrolysed at 1000°C to furnish
solution of copper sulphate, the amount of copper aluminium metal (At. Mass = 27 amu; 1 Faraday = 96,500
deposited is (given at. wt. of Cu = 63.6) Coulombs). The cathode reaction is – Al3+ + 3e– ® Al
(a) 0318g (b) 3.18 g To prepare 5.12 kg of aluminium metal by this method, we
(c) 31.8g (d) 63.6g require electricity of
74. A silver cup is plated with silver by passing 965 coulombs (a) 5.49 × 101 C (b) 5.49 × 104 C
of electricity. The amount of Ag deposited is : (c) 1.83 × 107 C (d) 5.49 × 107 C
(a) 107.89 g (b) 9.89 g 84. Find the charge in coulombs required to convert 0.2 mole
(c) 1.0002 g (d) 1.08 g VO3–2 into VO4–3 –
75. The amount of electricity that can deposit 108 g of Ag from (a) 1.93 × 104 (b) 9.65 × 104
AgNO3 solution is: 5
(c) 1.93 × 10 (d) 9.65 × 105
(a) 1 F (b) 2 A
85. The number of coulombs required to reduce 12.3 g of
(c) 1 C (d) 1 A
nitrobenzene to aniline is :
76. On passing C ampere of electricity through a electrolyte
solution for t second. m gram metal deposits on cathode. (a) 115800 C (b) 5790 C
The equivalent weight E of the metal is (c) 28950 C (d) 57900 C
86. The volume of oxygen gas liberated at NTP by passing a
C ´t C´m
(a) E= (b) E= current of 9650 coulombs through acidified water is
m ´ 96500 t ´ 96500
(a) 1.12 litre (b) 2.24 litre
96500 ´ m C ´ t ´ 96500 (c) 11.2 litre (d) 0.56 litre
(c) E= (d) E =
C ´t m 87. Three faradays electricity was passed through an aqueous
77. The number of electrons passing per second through a solution of iron (II) bromide. The weight of iron metal
cross-section of copper wire carrying 10–6 amperes of (at. wt = 65) deposited at the cathode (in g) is
current per second is found to be (a) 56 (b) 84
(a) 1.6 × 10–19 (b) 6 × 10–35 (c) 112 (d) 168
(c) 6 × 10–16 (d) 6 × 1012 88. Faraday’s laws of electrolysis will fail when
78. The electric charge for electrode decomposition of one (a) temperature is increased
gram equivalent of a substance is (b) inert electrodes are used
(a) one ampere per second (c) a mixture of electrolytes is used
(b) 96500 coulombs per second (d) None of these cases
(c) one ampere for one hour 89. A current strength of 9.65 amperes is passed through excess
(d) charge on one mole of electrons fused AlCl3 for 5 hours. How many litres of chlorine will be
79. In electrolysis of dilute H2SO4 using platinum electrodes liberated at STP? (F = 96500 C)
(a) H2 is evolved at cathode (a) 2.016 (b) 1.008
(b) NH2 is produced at anode (c) 11.2 (d) 20.16
(c) Cl2 is obtained at cathode 90. A solution of copper sulphate (CuSO4) is electrolysed for
(d) O2 is produced 10 minutes with a current of 1.5 amperes. The mass of
80. Electrolysis of fused NaCl will give copper deposited at the cathode (at. mass of Cu = 63u) is
(a) Na (b) NaOH (a) 0.3892 g (b) 0.2938 g
(c) NaClO (d) None of these (c) 0.2398 g (d) 0.3928 g
ELECTROCHEMISTRY 249

91. Electrolysis of a salt solution was carried out, after some 99. One Faraday of electricity is passed through molten Al2O3,
time solution turned yellow. The salt can be aqueous solution of CuSO4 and molten NaCl taken in three
(i) NaCl (ii) KCl different electrolytic cells connected in series. The mole
(iii) RbCl (iv) KBr ratio of Al, Cu and Na deposited at the respective cathode
is
(a) (i), (ii) and (iii) (b) (ii), (ii) and (iv)
(c) (i), (ii) and (iv) (d) (i), (iii) and (iv) (a) 2 : 3 : 6 (b) 6 : 2 : 3
92. Which of the following statements is incorrect? (c) 6 : 3 : 2 (d) 1 : 2 : 3
(a) Both electronic and electrolytic conductance depends 100. On passing a current of 1.0 ampere for 16 min and 5 sec
on the nature of conducting material. through one litre solution of CuCl2, all copper of the
solution was deposited at cathode. The strength of CuCl2
(b) Both electronic and electrolytic conductance varies
solution was (Molar mass of Cu= 63.5; Faraday constant
similarly with temperature.
= 96,500 C mol–1)
(c) Electronic conductance is independent but
electrolytic conductance depends on the amount of (a) 0.01 N (b) 0.01 M
the conducting substance. (c) 0.02 M (d) 0.2 N
(d) All the above statements are incorrect. 101. 0.2964 g of copper was deposited on passage of a current
93. Which of the following statements is incorrect? of 0.5 amp for 30 mins through a solution of copper sulphate.
(a) Electrodes made up of gold participates in the Calculate the oxidation state of Cu (At. mass 63.56).
chemical reaction. (a) +1 (b) +2
(b) Electrolytic products of NaCl are Na and Cl2 whereas (c) +3 (d) +4
of aqueous NaCl are NaOH, Cl2 and H2. 102. How much charge is required, when 1 mole of Cr2O 72 -
(c) During electrolysis at cathode, reaction with higher reduce to form 1 mole of Cr 3+ ?
value of E Å is preferred. (a) 6F (b) 3F
(d) All of the above statements are incorrect. (c) 1F (d) 2F
94. When electric current is passed through acidified water,
112 mL of hydrogen gas at STP collected at the cathode in Topic 5: Batteries, Fuel Cells and Corrosion
965 seconds. The current passed in amperes is 103. Which colourless gas evolves, when NH4Cl reacts with
(a) 1.0 (b) 0.5 zinc in a dry cell battery
(c) 0.1 (d) 2.0 (a) NH4 (b) N2
95. On passing current through two cells, connected in series, (c) H2 (d) Cl2
containing solution of AgNO3 and CuSO4, 0.18 g of Ag is 104. In a hydrogen-oxygen fuel cell, combustion of hydrogen
deposited. The amount of the Cu deposited is: occurs to
(a) 0.529 g (b) 10.623 g
(a) produce high purity water
(c) 0.0529 g (d) 1.2708 g
(b) create potential difference between two electrodes
96. In the electrolysis of water, one faraday of electrical energy
would liberate (c) generate heat
(a) one mole of oxygen (d) remove adsorbed oxygen from elctrode surfaces
(b) one gram atom of oxygen 105. Among the following cells:
(c) 8 g oxygen Leclanche cell (i)
(d) 22.4 lit. of oxygen Nickel-Cadmium cell (ii)
97. Electrolysis of dilute aqueous NaCl solution was carried Lead storage battery (iii)
out by passing 10 milli ampere current. The time required Mercury cell (iv)
to liberate 0.01 mol of H2 gas at the cathode is primary cells are
(1 Faraday = 96500 C mol–1) (a) (i) and (ii) (b) (i) and (iii)
(a) 9.65 × 104 sec (b) 19.3 × 104 sec
4
(c) (ii) and (iii) (d) (i) and (iv)
(c) 28.95 × 10 sec (d) 38.6 × 104 sec
106. The electrolyte used in Leclanche cell is
98. What is the amount of chlorine evolved when 2 amperes
(a) paste of KOH and ZnO
of current is passed for 30 minutes in an aqueous solution
of NaCl ? (b) 38% solution of H2SO4
(a) 66 g (b) 1.32 g (c) moist paste of NH4Cl and ZnCl2
(c) 33 g (d) 99 g (d) moist sodium hydroxide
EBD_8350
250 CHEMISTRY

107. Which one of the following cells can convert chemical 115. Which of the following is a merit of Ni–Cd cell over lead
energy of H2 and O2 directly into electrical energy? storage battery?
(a) Mercury cell (b) Daniell cell (a) Ni–Cd cell can be re-used.
(c) Fuel cell (d) Lead storage cell (b) Ni–Cd cell is comparatively economical to manufacture
108. Prevention of corrosion of iron by zinc coating is called (c) Ni–Cd cell has comparatively longer life
(a) electrolysis (d) All the above are the merits of Ni–Cd cell over lead
storage battery.
(b) photoelectrolysis
116. Which of the following statement(s) is/ are incorrect for
(c) cathodic protection corrosion of iron?
(d) galvanization (i) Reaction occurring at anode is
109. Which of the following batteries cannot be reused?
O 2 (g) + 4H + (aq) + 4e - ¾¾
® 2H 2 O (l)
(a) Lead storage battery
(ii) Reaction occurring at cathode is
(b) Ni-Cd cell
(c) Mercury cell ® 2Fe 2 + + 4e -
2Fe (s) ¾¾
(d) Both (b) and (c) (iii) Rust is Fe2O3.xH2O
110. Which of the following statements regarding fuel cell is (iv) H+ involved in corrosion reaction is provided from
incorrect? H2CO3 which is formed due to dissolution of carbon
(a) These cells are eco-friendly. dioxide from air in to water.
(b) These cells convert energy of combustion of fuels (a) (iv) only (b) (i) only
like H2, CH4, CH3OH etc., directly into electrical (c) (i) and (ii) (d) (i), (ii) and (iv)
energy. 117. When a lead storage battery is discharged
(c) H2 – O2 fuel cell is used in Apollo space programme. (a) SO2 is evolved
(d) Fuel cells produce electricity with an efficiency of (b) Lead sulphate is consumed
about 100%. (c) Lead is formed
111. During the charging of lead storage battery, the reaction at (d) Sulphuric acid is consumed
anode is represented by
118. The most durable metal plating on iron to protect against
(a) Pb2+ + SO24 - ¾¾
® PbSO4 corrosion is
(a) nickel plating
(b) ® PbO2 + SO42 - + 4H + + 2e -
PbSO 4 + 2H 2O ¾¾
(b) copper plating
(c) ® Pb 2+ + 2e -
Pb ¾¾ (c) tin plating
(d) Pb2 + + 2e - ¾¾
® Pb (d) zinc plating
112. Hydrogen-Oxygen fuel cells are used in space craft to 119. Which of the following statements is incorrect regarding
supply dry (Leclanche) cell?
(a) power for heat and light (a) Cathode used in the cell is coated by powdered
manganese dioxide and carbon.
(b) power for pressure
(b) Most common application of this cell is in our
(c) oxygen transistors and clocks.
(d) water (c) At cathode, Mn is oxidised from + 3 to + 4.
113. The best way to prevent rusting of iron is (d) At anode Zn is oxidised from 0 to + 2.
(a) making it cathode (b) putting in saline water 120. Identify the correct statement :
(c) Both of these (d) None of these (a) Corrosion of iron can be minimized by forming a
114. Several blocks of magnesium are fixed to the bottom of a contact with another metal with a higher reduction
ship to potential
(a) make the ship lighter (b) Iron corrodes in oxygen free water
(b) prevent action of water and salt (c) Corrosion of iron can be minimized by forming an
impermeable barrier at its surface
(c) prevent puncturing by under-sea rocks
(d) Iron corrodes more rapidly in salt water because its
(d) keep away the sharks
electrochemical potential is higher
ELECTROCHEMISTRY 251

1. The standard reduction potential of a silver chloride ¥ ¥


Given : L M ( NaA) = 100 Scm2 mol -1; L M ( HCl)
electrode (metal-sparingly soluble salt electrode) is 0.209
V and for silver electrode is 0.80 V. If the moles of AgCl that = 425 Scm 2 mol -1 ;
can dissolve in 10 L of a 0.01 M NaCl solution is represented ¥
L M ( NaCl) = 125 Scm 2mol-1
as 10–Z then find the value of Z.
10. A solution containing 1M XSO4 (aq) and 1M YSO4 (aq) is
2. 2F of electricity is passed through 20 L of a solution of electrolysed. If conc. of X2+ is 10–z M when deposition of
aquous solution of KCl. Calculate the pH of the solution. Y2+ and X2+ starts simultaneously, calculate the value of Z.
3. The specific conductivity of a solution containing 1.0g of
anhydrous BaCl2 in 200 cm3 of the solution has been found 2.303RT
Given : = 0.06
to be 0.0058 S cm–1. Calculate the molar conductivity of the F
solution. (Molecular wt. of BaCl2 = 208).
E° = -0.12V; E° = -0.24V
4. The equivalent conductivities of acetic acid at 298 K at the X 2+ /X Y 2+ /Y

concentrations of 0.1 M and 0.001M are 5.20 and 11. A solution of Ni (NO3)2 is electrolysed between platinum
49.2 S cm2 eq–1 respectively. Calculate the degree of electrodes using 0.1 Faraday electricity. How many mole
dissociation of acetic acid at 0.001 M concentration. of Ni will be deposited at the cathode?
12. Find the standard Gibbs energy for the given cell
Given that : L¥ (H+ ) and L ¥ (CH 3COO - )
reaction in kJ mol–1 at 298 K
are 349.8 and 40.9 ohm–1 cm2 eq–1 respectively. Zn(s) + Cu2+ (aq) ¾® Zn2+ (aq) + Cu(s),
5. The amount of electricity which releases 2.0 g of gold from E° = 2 V at 298 K
a gold salt is same as that which dissolves 0.967g of copper (Faraday’s constant, F = 96000 C mol–1)
anode during the electrolysis of copper sulphate solution. 13. All the energy released from the reaction X ® Y, DrG° = –
What is the oxidation number of gold in the gold ion ? (At. 193 kJ mol–1 is used for oxidizing M+ as M+ ® M3+ + 2e–,
mass of Cu = 63.5; Au = 197) E° = –0.25 V
6. If Kc for the reaction Under standard conditions, find the number of moles of M+
oxidized when one mole of X is converted to Y
Cu 2+ ( aq ) + Sn 2+ ( aq ) ¾¾
® Sn 4+ ( aq ) + Cu ( s )
[F = 96500 C mol–1]
at 25°C is represented as 2.6 × 10y then find the value of y. 14. Consider the following half-cell reactions and associated
E ° 2+ = 0.34V; E ° 4+ 2 + = 0.15V ) standard half-cell potentials, and determine the maximum
(Given : Cu |Cu Sn |Sn voltage that can be obtained by combination resulting in
spontaneous processes :
7. If DG° for the half cell MnO-4 | MnO2 in an acid solution is
xF then find the value of x. AuBr4- ( aq ) + 3e- ¾¾
® Au ( s ) + 4Br - ( aq ) ; E° = – 0.86 V

° °
(Given : E MnO- |Mn 2 + = 1.5V; E MnO 2+ = 1.25V ) Eu 3+ ( aq ) + e - ¾¾
® Eu 2+ ( aq ) ; E° = – 0.43 V
4 2 |Mn

8. The equivalent conductances of sodium chloride ,


Sn 2+ ( aq ) + 2e - ¾¾
® Sn ( s ) ; E° = – 0.14 V
hydrochloric acid and sodium acetate at infinite dilution
are 126.45 , 426.16 and 91.0 ohm–1 cm2 equiv–1, respectively
at 25 ºC. Calculate the equivalent conductance of acetic IO- ( aq ) + H 2 O ( l ) + 2e- ¾¾
® I- ( aq ) + 2OH - ( aq ) ;
acid at infinite dilution. E° = + 0.49 V
9. Molar conductivity of aqueous solution of HA is 200 S 15. The e.m.f. of the cell Zn | Zn2+ (0.01M) | | Fe2+ (0.001M) | Fe
cm2 mol–1, pH of this solution is 4. Calculate the value of at 298 K is 0.2905 then the value of equilibrium constant
pKa(HA) at 25°C. for the cell reaction 10x. Find x.
EBD_8350
252 CHEMISTRY

Exercise 3 : NCERT Exemplar & Past Year MCQs

NCERT Exemplar MCQs (a) 1 F (b) 6 F


1. Which cell measure standard electrode potential of copper (c) 3 F (d) 2 F
electrode? 9. The cell constant of a conductivity cell ........ .
(a) changes with change of electrolyte
(a) Pt ( s ) |H 2 ( g, 0.1 bar ) | |H + ( aq, 1 M ) | |Cu 2+ ( aq, 1 M )| Cu
(b) changes with change of concentration of electrolyte
(b) Pt ( s ) |H 2 ( g,1 bar ) | |H + ( aq, 1 M )| |Cu 2+ ( aq, 2 M )| Cu (c) changes with temperature of electrolyte
(d) remains constant for a cell
(c) Pt ( s ) |H 2 ( g,1 bar ) | |H + ( aq, 1 M ) | |Cu 2+ ( aq, 1 M )| Cu
°
10. L m( NH4OH ) is equal to ....... .
(d) Pt ( s ) |H 2 ( g, 0.1 bar ) | |H + ( aq, 0.1 M ) | |Cu 2+ ( aq, 1 M ) | Cu
(a) L°m( NH + L°( NH - L°m ( HCl )
2. The difference between the electrode potentials of two 4OH ) 4Cl )
electrodes when no current is drawn through the cell is (b) L°m( NH + L °m( NaOH ) - L °m( NaCl )
4 Cl )
called......
(c) L °m( NH + L °m( NaCl) - L °m ( NaOH )
(a) cell potential (b) cell emf 4Cl )
(c) potential difference (d) cell voltage (d) L °m( NaOH ) + L °m( NaCl) - L °m( NH
4Cl )
3. Using the data given below, find out the strongest reducing 11. Electrode potential for Mg electrode varies according to
agent. the equation
E° = 1.33 V; E° = 1.36 V E ° 2+
= EMg 0.059 1
/ Mg - log .
3+
Cr2O2–
7 /Cr Cl2 /Cl – Mg 2 + /Mg
2 [Mg 2+ ]
E° = 1.51 V; E ° = – 0.74 V The graph of E
Mg 2 + / Mg
vs log [Mg2+] is
MnO4– /Mn 2 + Cr 3+ /Cr

(a) Cl– (b) Cr


(c) Cr3+ (d) Mn2+
®

4. Use the data given in Q. 3 and find out which of the (a) E Mg2+/ Mg

following is the strongest oxidising agent?


(a) Cl– (b) Mn2+ log [Mg 2+] ®
(c) MnO4- (d) Cr3+
5. Using the data given in Q. 3 find out in which option the
®

order of reducing power is correct. (b) E Mg2+/ Mg


3+ - 2+
(a) Cr < Cl < Mn < Cr
2+ - 3+
(b) Mn < Cl < Cr < Cr log [Mg2+] ®

(c) Cr 3+ < Cl - < Cr2O72 - < MnO4–


2+ 3+ -
(d) Mn < Cr < Cl < Cr
®

6. Use the data given in Q.3 and find out the most stable ion (c) E Mg2+/ Mg

in its reduced form.


(a) Cl– (b) Cr3+
log [Mg2+] ®
(c) Cr (d) Mn2+
7. Use the data of Q. 3 and find out the most stable oxidised
species.
®

(a) Cr3+ (b) MnO4- (d) E Mg2+/ Mg

(c) Cr2 O72 - (d) Mn2+


8. The quantity of charge required to obtain one mole of log [Mg2+] ®
aluminium from Al2O3 is ....... .
ELECTROCHEMISTRY 253

12. Which of the following statement is correct? 19. The weight of silver (at wt. = 108) displaced by a quantity
(a) Ecell and D rG of cell reaction both are extensive of electricity which displaces 5600 mL of O2 at STP will be:-
properties. [AIPMT 2014, A]
(b) Ecell and D rG of cell reaction both are intensive (a) 5.4 g (b) 10.8 g
properties. (c) 54.9 g (d) 108.0 g
(c) Ecell is an intensive property while DrG of cell reaction 20. Resistance of 0.2 M solution of an electrolyte is 50 W.
is an extensive property. The specific conductance of the solution is 1.4 S m –1.
(d) Ecell is an extensive property while DrG of cell reaction The resistance of 0.5 M solution of the same electrolyte is
is an intensive property. 280 W. The molar conductivity of 0.5 M solution of the
13. Which of the following statement is not correct about an electrolyte in S m2 mol–1 is: [JEE M 2014, A]
inert electrode in a cell? (a) 5 × 10–4 (b) 5 × 10–3
(a) It does not participate in the cell reaction (c) 5 × 103 (d) 5 × 102
(b) It provides surface either for oxidation or for reduction 21. The equivalent conductance of NaCl at concentration C
reaction and at infinite dilution are lC and l¥, respectively. The
(c) It provides surface for conduction of electrons correct relationship between lC and l¥ is given as:
(d) It provides surface for redox reaction (Where the constant B is positive) [JEE M 2014, C]
14. An electrochemical cell can behave like an electrolytic cell (a) lC = l¥ + ( B ) C
when ......
(b) lC = l¥ - ( B ) C
(a) Ecell = 0 (b) Ecell > Eext
(c) Eext > Ecell (d) Ecell = Eext (c) lC = l ¥ - ( B ) C
15. Which of the statements about solutions of electrolytes is
not correct? (d) lC = l¥ + ( B ) C
(a) Conductivity of solution depends upon size of ions 22. Given below are the half-cell reactions: [JEE M 2014, A]
(b) Conductivity depends upon viscosity of solution
Mn 2 + + 2e- ® Mn ; E 0 = -1.18 V
(c) Conductivity does not depend upon solvation of ions
present in solution ( )
2 Mn 3+ + e - ® Mn 2+ ; E 0 = +1.51V
(d) Conductivity of solution increases with temperature
16. While charging the lead storage battery ......... . The E0 for 3Mn 2 + ® Mn + 2Mn 3+ will be:
(a) PbSO4 anode is reduced to Pb (a) –2.69 V; the reaction will not occur
(b) PbSO4 cathode is reduced to Pb (b) –2.69 V; the reaction will occur
(c) PbSO4 cathode is oxidised to Pb (c) –0.33 V; the reaction will not occur
(d) PbSO4 anode is oxidised to PbO2 (d) –0.33 V; the reaction will occur
17. In the electrolysis of aqueous sodium chloride solution 23. The metal that cannot be obtained by electrolysis of an
which of the half cell reaction will occur at anode? aqueous solution of its salts is: [JEE M 2014, C]
(a) Ag (b) Ca
Na + ( aq ) + e - ¾¾
°
(a) ® Na ( s) ; Ecell = - 2.71V
(c) Cu (d) Cr
(b) 2H 2 O ( l) ® O 2 ( g ) + 4H + ( aq ) + 4e - ; Ecell
°
= 1.23V 24. Aqueous solution of which of the following compounds is
1 the best conductor of electric current?
(c) H ( aq ) + e ¾¾
+ -
® H 2 ( g ) ; Ecell
°
= 0.00 V
2 [AIPMT 2015 RS, C]
1
Cl – ( aq ) ¾¾
® Cl2 ( g ) + e - ; Ecell = 1.36 V (a) Acetic acid, C2H4O2
°
(d)
2 (b) Hydrochloric acid, HCl
Past Year MCQs (c) Ammonia, NH3
18. When 0.1 mole MnO42– is oxidised the quantity of electricity (d) Fructose, C6H12O6
required to completely oxidise MnO42– to MnO4– is: 25. A device that converts energy of combustion of fuels like
[AIPMT 2014, A] hydrogen and methane, directly into electrical energy is
(a) 96500 C (b) 2 × 96500 C known as : [AIPMT 2015, C]
(c) 9650 C (d) 96.50 C (a) Electrolytic cell (b) Dynamo
(c) Ni-Cd cell (d) Fuel Cell
EBD_8350
254 CHEMISTRY

26. Two Faraday of electricity is passed through a solution of (Atomic weight of B = 10.8 u) [JEE M 2018, A]
CuSO4. The mass of copper deposited at the cathode is (a) 6.4 hours (b) 0.8 hours
(at. mass of Cu = 63.5 amu) [JEE M 2015, A] (c) 3.2 hours (d) 1.6 hours
(a) 2g (b) 127 g 34. For a cell involving one electron E ! = 0.59 V at 298 K,
cell
(c) 0 g (d) 63.5 g
the equilibrium constant for the cell reaction is :
27. The pressure of H2 required to make the potential of H2-
electrode zero in pure water at 298 K is : [NEET 2016, A] é 2.303RT ù
ê Given that = 0.059 V at T = 298 K ú
(a) 10–14 atm (b) 10–12 atm ë F û
–10
(c) 10 atm (d) 10–4 atm [NEET 2019, A]
28. Galvanization is applying a coating of: [JEE M 2016, C] (a) 1.0 × 102 (b) 1.0 × l05
(a) Cu (b) Zn (c) 1.0 × l010 (d) 1.0 × l030
(c) Pb (d) Cr 35. For the cell reaction
29. Ionic mobility of which of the following alkali metal ions is 2Fe3+(aq) + 2I – (aq) ¾® 2Fe2+ (aq) + I2(aq)
lowest when aqueous solution of their salts are put under E °cells 0.24 V at 298 K. The standard Gibbs energy (D, G°)
an electric field ? [NEET 2017, C]
of the cell reaction is: [NEET 2019, A]
(a) K (b) Rb
[Given that Faraday constant F = 96500 C mol–1]
(c) Li (d) Na
(a) –46.32 kJ mol–1 (b) –23.16 kJ mol–1
30. In the electrochemical cell :- [NEET 2017, A]
(c) 46.32 kJ mol–1 (d) 23.16 kJ mol–1
Zn | ZnSO4 (0.01M) | | CuSO4 (1.0 M) | Cu, the emf of this
36. The anodic half-cell of lead-acid battery is recharged using
Daniel cell is E1. When the concentration of ZnSO4 is
electricity of 0.05 Faraday. The amount of PbSO 4
changed to 1.0M and that of CuSO4 changed to 0.01M, the
electrolyzed in g during the process is : (Molar mass of
emf changes to E2. From the followings, which one is the
PbSO4 = 303 g mol–1) [JEE M 2019, A]
RT
relationship between E1 and E2? (Given, = 0.059) (a) 22.8 (b) 15.2
F
(a) E1 < E2 (b) E1 > E2 (c) 7.6 (d) 11.4
(c) E2 = 0 ¹ E1 (d) E1 = E2 37. The standard Gibbs energy for the given cell reaction in
31. Given [JEE M 2017, A] kJ mol–1 at 298 K is: [JEE M 2019, A]
Zn(s) + Cu2+ (aq) ¾® Zn2+ (aq) + Cu(s),
Eo = 1.36V, E o = –0.74V,
Cl /Cl – Cr3+ /Cr E° = 2 V at 298 K
2
(Faraday’s constant, F = 96000 C mol–1)
Eo = 1.33V, E o = 1.51V. (a) – 384 (b) 384
Cr2O 27- /Cr 3+ MnO 4– /Mn 2+
(c) 192 (d) – 192
Among the following, the strongest reducing agent is 38. The number of Faradays(F) required to produce 20 g
(a) Cr (b) Mn 2+ (c) Cr3+ (d) Cl– of calcium from molten CaCl2 (Atomic mass of Ca = 40 g
32. Consider the change in oxidation state of bromine mol-1) is: [NEET 2020, A]
corresponding to different emf values as shown in the (a) 2 (b) 3
diagram below : [NEET 2018, S] (c) 4 (d) 1
39. On electrolysis of dil. sulphuric acid using Platinum (Pt)
1.82 V
BrO4– ¾¾¾® 1.5 V
BrO3– ¾¾¾® HBrO electrode, the product obtained at anode will be
[NEET 2020, S]
¾¾

Br –¬¾¾¾ Br2 ¬¾¾¾ (a) Oxygen gas (b) H2S gas


1.0652 V 1.595 V
(c) SO2 gas (d) Hydrogen gas
Then the species undergoing disproportionation is
40. Given that the standard potentials (E0) of Cu2+ / Cu and
(a) BrO3– (b) BrO 4– Cu+ / Cu are 0.34 V and 0.522 V respectively, the E0 of
(c) HBrO (d) Br2 Cu2+ / Cu + is: [JEE M 2020, A]
33. How long (approximate) should water be electrolysed by (a) + 0.182 V (b) + 0.158 V
passing through 100 amperes current so that the oxygen (c) – 0.182 V (d) – 0.158 V
released can completely burn 27.66 g of diborane?
ELECTROCHEMISTRY 255

Exercise 4 : Problem Solving Skill Enhancer MCQs


1. From the following half - cell reactions and their standard COONa
potentials, what is the smallest possible standard e.m.f. conductance at infinite dilution of the salt | is
for spontaneous reactions? COOK
– (a) 197 cm2 ohm– 1 eq– 1
PO34- ( aq ) + 2H 2 O(l) + 2e - ¾¾
® HPO 32 + 3OH - ( aq ) ;
(b) 172 cm2 ohm– 1 eq– 1
E° = – 1.05 V (c) 135.5 cm2 ohm– 1 eq– 1
PbO2 (s) + H2O (l) + 2e– ¾¾
® PbO (s) + 2OH– (aq); (d) 160.5 cm2 ohm– 1 eq– 1
E° = +0.28 V 7. For the galvanic cell :
IO 3- (aq ) + 2H 2 O(l) + 4e -
¾¾
® IO -
(aq ) + 4OH (aq ) ;
-
Al(s) | Al3+ (aq)(0.1M) || Ag + (aq)(0.1M) | Ag(s) ,
E° = + 0.56 V which of the following is/are correct ?
(a) + 0.00 V (b) + 0.74 V (a) Reaction quotient Q = 1.0 × 10–2
(c) + 0.56 V (d) + 0.28 V (b) log10Q = 3
2. The e.m.f. of the cell Zn | Zn2+ (0.01M) | | Fe2+ (0.001M) | Fe (c) Q = 1.0 × 103
at 298 K is 0.2905 then the value of equilibrium for the cell (d) Q = 1.0 × 102
reaction is
8. If E1° is standard electrode potential for Fe/Fe2+ and E 2° is
0.32 0.32
(a) (b)
e 0.0295 10 0.0295 for Fe2+/Fe3+ and E3° for Fe/Fe3+. A relation between E1° .
0.26 0.32
(c) (d) E 2° and E3° is:
10 0.0295 10 0.0591
3. In electrolysis of NaCl when Pt electrode is taken, then H2
(a) 3E3° = 2E1° + E2° (b) E3° = 2 E1° + 3E2°
is liberated at cathode while with Hg cathode it forms
sodium amalgam. This is because (c) E3° = E1° + E2° (d) E3° = 3E1° - E2°
(a) Hg is more inert than Pt
(b) more voltage is required to reduce H+ at Hg than at Pt 9. The equilibrium constant for the reaction is :
(c) Na is dissolved in Hg while it does not dissolve in Pt In 2 + + Cu 2+ ¾¾
® In 3+ + Cu + at 298 K
(d) conc. of H+ ions is larger when Pt electrode is taken
4. Co | Co2+ (C2) || Co2+ (C1) | Co; for this cell, DG is negative °
Given, ECu 2 + /Cu + = 0.15 V,
if:
(a) C2 > C1 (b) C1 > C2
E ° 3+ = -0.42 V, E ° 2 + + = - 0.40 V
(c) C1 = C2 (d) unpredictable In /In + In /In
5. Consider the following standard electrode potentials and
(a) 105 (b) 103
calculate the equilibrium constant at 25° C for the indicated
disproportionation reaction : (c) 1010 (d) 106
10. A solution containing one mole per litre of each Cu(NO 3)2,
2+ 3+
3Mn (aq) ¾¾
® Mn(s) + 2Mn ( aq ) AgNO3, Hg2(NO3)2 and Mg(NO3)2, is being electrolysed
3+ - 2+ by using inert electrodes. The values of standard electrode
Mn (aq) + e ¾¾
® Mn (aq); E ° = 1.51 V
potentials in volts (reduction potentials) are Ag+ / Ag
Mn 2 + (aq) + 2e - ¾¾
® Mn(s); E ° = -1.185 V = +0.80, Hg2+ 2+
2 / 2Hg = +0.79, Cu / Cu= +0.34, and Mg /
2+

Mg = –2.37. With increasing voltage, the sequence of


(a) 1.2 ´ 10–43 (b) 2.4 ´ 10–73
deposition of metals on the cathode will be
(c) 6.3 ´ 10–92 (d) 1.5 ´ 10–62
(a) Ag, Hg, Cu, Mg
COO – + (b) Mg, Cu, Hg, Ag
6. Given the ionic conductance of | , K , and Na+ are
COO – (c) Ag, Hg, Cu
(d) Cu, Hg, Ag
74, 50, and 73 cm2 ohm– 1 eq– 1, respectively. The equivalent
EBD_8350
256 CHEMISTRY

11. The temperature coefficient of a cell whose operation is


k ´ 1000 k
based on the reaction (a) x- y (b) x + y ´ 188
Pb(s) + HgCl2 (aq) ¾¾
® PbCl2 (aq) + Hg (l) is :
k ´ 1000 ´ 188 x + y 1000
æ dE ö -4 -1 (c) (d) ´
ç ÷ = 1.5 ´ 10 VK at 298 K x+ y k 188
è dT ø p
14. Given the ionic equivalent conductivities for the following
The change in entropy (in J/K mol) during the operation
ions:
is :
l°eq K+ = 73.5 cm2 ohm– 1 eq– 1
(a) 8627 (b) 57.9
(c) 28.95 (d) 14.475 l°eq Al3+ = 149 cm2 ohm– 1 eq– 1
12. Given the following cell at 25° C l°eq SO42– = 85.8 cm2 ohm– 1 eq– 1
The L°eq for potash alum (K2 SO4. Al2(SO4)3. 24H2O) is
H 2 CH3COOH NaOH H 2 (a) 215.92 (b) 348.3 (c) 368.2 (d) 394.1
Pt (1 atm) Pt. What will
(10 – 3 M) (10 – 3M) (1 atm) 15. During conductometric titration of 0.1 M HCl with 1.0 M
KOH, which is the correct observation.
be the potential of the cell? Given pKa of CH3COOH = 4.74 (a) Resistance of the solution decreases upto
(a) – 0.42 V (b) 0.42 V equivalence point and then increases
(c) – 0.19 V (d) 0.19 V (b) Resistance increases upto equivalence point and
13. The specific conductance of a saturated solution of silver then decreases
bromide is k S cm–1. The limiting ionic conductivity of (c) Conductance increases upto equivalence point and
Ag+ and Br – ions are x and y, respectively. The solubility then decreases
of silver bromide in g L–1 is : (molar mass of AgBr = 188) (d) Conductance decreases upto equivalence point and
then becomes almost constant.

ANSWER KEY
Exercise 1 : NCERT Based Topic-wise MCQs
1 (a) 13 (a) 25 (d) 37 (d) 49 (b) 61 (a) 73 (b) 85 (d) 97 (b) 109 (c)
2 (b) 14 (d) 26 (c) 38 (d) 50 (d) 62 (d) 74 (d) 86 (d) 98 (b) 110 (d)
3 (d) 15 (c) 27 (b) 39 (a) 51 (b) 63 (b) 75 (a) 87 (b) 99 (a) 111 (b)
4 (d) 16 (c) 28 (c) 40 (c) 52 (a) 64 (b) 76 (c) 88 (d) 100 (a) 112 (b)
5 (b) 17 (a) 29 (c) 41 (d) 53 (a) 65 (a) 77 (d) 89 (d) 101 (b) 113 (a)
6 (c) 18 (a) 30 (b) 42 (b) 54 (d) 66 (b) 78 (d) 90 (b) 102 (b) 114 (b)
7 (b) 19 (b) 31 (d) 43 (b) 55 (c) 67 (d) 79 (a) 91 (a) 103 (c) 115 (c)
8 (d) 20 (c) 32 (b) 44 (c) 56 (b) 68 (d) 80 (a) 92 (b) 104 (b) 116 (c)
9 (d) 21 (b) 33 (a) 45 (d) 57 (d) 69 (d) 81 (b) 93 (a) 105 (d) 117 (d)
10 (b) 22 (b) 34 (a) 46 (d) 58 (c) 70 (b) 82 (b) 94 (a) 106 (c) 118 (d)
11 (d) 23 (c) 35 (d) 47 (d) 59 (b) 71 (c) 83 (d) 95 (c) 107 (c) 119 (c)
12 (d) 24 (d) 36 (a) 48 (d) 60 (d) 72 (d) 84 (a) 96 (c) 108 (d) 120 (c)
Exercise 2 : Numeric/Integer Answer Questions
1. (7) 3. (241.67) 5. (3) 7. (5) 9. (4) 11. (0.05) 13. (4) 15. (10.85)
2. (13) 4. (12.5) 6. (6) 8. (390.7) 10. (4) 12. (–384) 14. (1.35)
Exercise 3 : NCERT Exemplar & Past Year MCQs
1 (c) 5 (b) 9 (d) 13 (d) 17 (d) 21 (c) 25 (d) 29 (c) 33 (c) 37 (a)
2 (b) 6 (d) 10 (b) 14 (c) 18 (c) 22 (a) 26 (d) 30 (b) 34 (c) 38 (d)
3 (b) 7 (a) 11 (b) 15 (c) 19 (d) 23 (b) 27 (a) 31 (a) 35 (a) 39 (a)
4 (c) 8 (c) 12 (c) 16 (a) 20 (a) 24 (b) 28 (b) 32 (c) 36 (c) 40 (b)
Exercise 4 : Problem Solving Skill Enhancer MCQs
1 (d) 3 (b) 5 (c) 7 (d) 9 (c) 11 (c) 13 (c) 15 (b)
2 (b) 4 (b) 6 (a) 8 (a) 10 (c) 12 (a) 14 (d)
18 Chemical Kinetics

Trend Buster NEET & JEE Main

Number of Questions from 2020-15 12 7 Chapter has equal weightage in


Weightage 3.8% 4% NEET and JEE M.

The most Important Concepts that Cover Maximum number of Questions asked in past 6 years.

Integrated rate equation 6 3


Temperature dependence of rate of the reaction 2 1
Rate of a chemical reactions 1 2
Less Important Concepts that Cover 1 or 2 Questions asked in past 6 years.

Collision theory of chemical reactions — 1


Factors influencing rate of a reaction 3 —

NEET JEE
Concept Used

2020 Integrated rate equation / 1st order reaction / 2 Average 1 Difficult


Factors influencing Factors influencing
rate of a reaction rate of a reaction
2019 Integrated rate equation / half life of a reaction / rate of 2 Average 2 Difficult
rate of reaction reaction / order of reaction / first
order reaction
2018 Integrated rate equation / first and second order of 1 Average 1 Average
factors influencing rate of reaction / order of reaction
a reaction
2017 Integrated rate equation / rate of reaction / half life / 2 Average 1 Difficult
Temperature dependence Arrhenius equation
of rate of reaction /
rate of reaction
2016 integrated rate of reaction / half life period / catalyst / 2 Average 1 Difficult
temperature dependence rate of a reaction
of rate of reaction/rate of a
rection
2015 Integrated rate equation / Zero order reaction / order of a 3 Average 1 Easy
Temperature dependence reaction / activation energy /
of rate of reaction / rate of collision theory
reaction/collision theory of
chemical reaction
EBD_8350
258 CHEMISTRY
CHEMICAL KINETICS 259
EBD_8350
260 CHEMISTRY

Problem Solving Tips/ Tricks/ Points to Remember

4 Unit of rate constant


For nth order reaction Þ (mol/L)1–n s–1 or mol1–n Ln–1s–1 (solution reactions)
Þ (atm)1–n s–1 (gaseous reaction)

– E a /RT -E a k at (t + 100 C) K 35°C


4 Arrhenius equation : k = Ae and slope = and Temperature Coefficient = 0 = K
2.303R k at t C 25°C

æ k2 ö Ea æ T2 - T1 ö
4 log ç k ÷ = ç ÷
è 1 ø 2.303 è T1T2 ø
4 Expressions for rate constant for reactions of different orders and their half life:
Reaction Order Rate law eq. Expression for rate constant Half life time
1 [A]0
A ® Products 0 Rate = k [A]0 k = {[A]0 - [A]t } t1/2 =
t 2k
2.303 [A]0 0.693
A ® Products 1 Rate = k [A] k= log t1/2 =
t [A]t k
1ì 1 1 ü
2A ® Products 2 Rate = k [A]2 k= í - ý
t î [A] [A]0 þ 1
t1/2 =
2.303 [B]0 [A] k[A]0
A + B ® Products 2 Rate = k [A] [B] k= log
t([A]0 - [B]0 ) t[A]0 [B]
1 ìï 1 1 üï 3
A ® Products 3 Rate = k [A]3 k= í 2- ý t1/2 =
2t îï [A] [A]02 þï 2k[A]20

4 Negative Order Reactions 4 Acid catalysed hydrolysis of ester (In terms of volume of
Conversion of ozone into oxygen. reagent NaOH required in titration)
Rate = K [O3]2 [O2]–1
Order with respect to oxygen is –1. H+
CH 3COOC 2 H 5 + H 2 O ¾¾¾
® CH 3COOH + C 2 H 5 OH
4 Collision Frequency (z)
NaOH react with H+ and CH3COOH
Total number of collisions which occur among the reacting
2.303 é V - V0 ù
molecules per second per unit volume is called collision k= log ê ¥ ú
frequency. Its value is given by t ë V¥ - Vt û
where
z= 2 p n s2 n 2
V0 = Volume of NaOH required in titration when reacted
n = average velocity, s = molecular diameter in cm., with H+ (at start)
n = number of molecules per cc. Vt = Volume of NaOH required in titration when reacted
4 Rate of Reaction from Collision Theory with H+ and CH3COOH at time ‘t’
It is given by V¥ = Volume of NaOH required in titration when reaction
Rate of reaction = f × z is completed.
z = collision frequency, f = fraction of effective collisions 4 Inversion of cane sugar (In terms of angle of rotation)
Dn
= = e - E a / RT H + catalysed hydrolysis
N C12 H 22 O11 + H 2 O ¾¾¾¾¾¾¾¾¾
®
Inversion
\ Rate (k) = Ze- E a /RT
4 First order rate constant in terms of pressure C6 H12 O6 + C6 H12O 6
(glucose) (fructose)
2.303 P0
A(g) ¾¾ ® B(g) + C(g) ; k = log 2.303 ær -r ö
t 2P0 - Pt k= log ç 0 ¥ ÷
where P0 = Initial partial pressure of reactant A
t è rt - r¥ ø
where,
Pt = Total pressure of gaseous system at time ‘t’
CHEMICAL KINETICS 261

r0 = initial angle of rotation of the sample t1/2 0.3


=
rt = angle of rotation after time t t x% (left) æ 100 ö
log ç ÷
r¥ = angle of rotation after completion of reaction è x ø
4 Relation between t1/2 and tx% (amount decomposed and 4 Amount of substance left after n half lives:
amount left) æ1ö
n
A0
A0 ç ÷ or
t1/2 0.3 è2ø 2n
= ;
t x% (decomposed) æ 100 ö
log ç ÷
è 100 - x ø

4 Plot of concentration vs. time.

-k
¾®

¾®
¾®

¾®
slope = k
slope = –k slope = slope = k
2.303
[A] log [A] 1 1
[A] [A]2

t ¾® t ¾® t ¾® t ¾®
zero order first order second order third order
4 Plot of rate vs. concentration : [rate µ (conc.)n]

¾®
¾®
¾®

¾®

2
[A] ¾® [A] ¾® [A] ¾® 3
[A]
zero order first order second order third order
4 Plot of half-life vs. concentration : [t1/2 µ (conc.)1–n]
¾®
¾®

¾®

¾®

t1/2 t1/2 t1/2 t1/2

2
[A] ¾® [A] ¾® 1/[A] ¾® 1/[A] ¾®
zero order first order second order third order
EBD_8350
262 CHEMISTRY

Exercise 1 : NCERT Based Topic-wise MCQs


Topic 1: Rate of a Chemical Reaction 10. Study the following graphs and choose the correct option
[R] 0
1. The rate law for the single- step reaction 2A + B ¾¾ ® 2C, (P)

Concentration
is given by:

Concentration
(a) rate = k [A].[B] (b) rate = k [A]2.[B] c1 A c2
C
(c) rate = k [2A].[B] (d) rate = k [A]2.[B]o c2 D[P]
B c1
2. The rate of a chemical reaction tells us about, Dt D
(a) the reactants taking part in reaction
(b) the products formed in the reaction t4 t 2 t Time t4 t2 t Time
(c) how slow or fast the reaction is taking place (a) (b)
(d) None of the above (i) in fig. a, A represents average rate and B represents
instantaneous rate
3. Consider the reaction
(ii) in fig. b, D represents average rate and C represents
N2 (g) + 3H2(g) ¾® 2 NH3(g) instantaneous rate
d[NH 3 ] d[H 2 ] (iii) fig. a, A represents instantaneous rate and B
The equality relationship between and -
dt dt represents average rate
is (iv) fig. b, C represents average rate and D represents
instantaneous rate
d[NH3 ] 2 d[H 2 ] d[NH3 ] 3 d[H 2 ] (a) (i) and (ii) are correct (b) (ii) and (iv) are correct
(a) + =- (b) + =-
dt 3 dt dt 2 dt (c) (i) and (iv) are correct (d) (ii) and (iii) are correct
11. The instantaneous rate of disappearance of MnO4– ion in
d[NH3 ] d[H 2 ] d[NH 3 ] 1 d[H 2 ] the following reaction is 4.56 × 10–3 Ms–1
(c) =- (d) =-
dt dt dt 3 dt 2MnO4– + 10I– + 16H+ ¾® 2Mn2+ + 5I2 + 8H2O
4. The rate of reaction The rate of appearance I2 is :
(a) increases as the reaction proceeds (a) 4.56 × 10–4 Ms–1 (b) 1.14 × 10–2 Ms–1
(b) decreases as the reaction proceeds (c) 1.14 × 10 Ms –3 –1 (d) 5.7 × 10–3 Ms–1
(c) remains the same as the reaction proceeds 12. The rate of the reaction 2N2O5 ® 4NO2 + O2 can be written
(d) may decrease or increase as the reaction proceeds in three ways :
5. The unit of rate of reaction is -d[N 2O5 ] d[NO2 ]
= k [N 2 O5 ] ; = k ¢ [N 2O5 ]
(a) mol/dm3 (b) mol/pound dt dt
(c) mol/dm3 sec (d) mol/cm3
d[O2 ]
6. In the rate equation, when the conc. of reactants is unity = k ¢¢ [N 2 O5 ]
dt
then rate is equal to
The relationship between k and k' and between k and k¢¢
(a) specific rate constant
are:
(b) average rate constant (a) k¢ = 2k ; k¢ = k (b) k¢ = 2k ; k¢¢ = k / 2
(c) instantaneous rate constant (c) k¢ = 2k ; k¢¢ = 2k (d) k¢ = k ; k¢¢ = k
(d) None of above 13. The reaction of hydrogen and iodine monochloride is given
7. The rate of reaction between two specific time intervals is as:
called H 2 (g) + 2ICl(g) ¾¾
® 2HCl(g) + I 2 (g)
(a) instantaneous rate (b) average rate The reaction is of first order with respect to both H2(g) and
(c) specific rate (d) ordinary rate ICl(g). Following mechanisms were proposed.
8. At the beginning, the decrease in the conc. of reactants is Mechanism A:
(a) slow (b) moderate H 2 (g) + 2ICl(g) ¾¾
® 2HCl(g) + I 2 (g)
(c) rapid (d) None of above Mechanism B:
9. The average rate and instantaneous rate of a reaction are H 2 (g) + ICl(g) ¾¾
® HI(g);slow
equal HI(g) + ICl(g) ¾¾® HCl(g) + I 2 (g);fast
(a) at the start Which of the above mechanism(s) can be consistent with
(b) at the end the given information about the reaction?
(c) in the middle (a) A and B both (b) Neither A nor B
(d) when two rate have time interval Dt ® 0 (c) A only (d) B only
CHEMICAL KINETICS 263

Topic 2: Factors Influencing Rate of a Reaction 22. In the reaction 2A + B ® A 2 B, if the concentration of A is
doubled and that of B is halved, then the rate of the reaction
14. Assertion: The rate of the reaction is the rate of change of
will:
concentration of a reactant or a product. (a) increase 2 times (b) increase 4 times
Reason: Rate of reaction remains constant during the (c) decrease 2 times (d) remain the same
course of reaction. 23. The order of a reaction, with respect to one of the reacting
(a) Assertion is correct, reason is correct; reason is a component Y, is zero. It implies that:
correct explanation for assertion. (a) the reaction is going on at a constant rate
(b) Assertion is correct, reason is correct; reason is not a (b) the rate of reaction does not vary with temperature
correct explanation for assertion. (c) the reaction rate is independent of the concentration
(c) Assertion is correct, reason is incorrect. of Y
(d) Assertion is incorrect, reason is correct. (d) the rate of formation of the activated complex is zero
15. Which one of the following statements for the order of a 24. The unit of rate constant for a zero order reaction is
reaction is incorrect ? (a) mol L–1 s–1 (b) L mol–1 s–1
2
(c) L mol s –2 –1 (d) s –1
(a) Order can be determined only experimentally.
25. The order of a reaction with rate equal to k[A]3/2 [B]–1/2 is:
(b) Order is not influenced by stoichiometric coefficient
1 3
of the reactants. (a) 1 (b) - (c) - (d) 2
(c) Order of reaction is sum of power to the concentration 2 2
terms of reactants to express the rate of reaction. 26. For the reaction,
H+
(d) Order of reaction is always whole number. CH3COCH3 + I2 ¾¾¾ ® products
16. The rate of the reaction 2NO + Cl2 ¾¾ The rate is governed by expression
® 2NOCl is
dx
given by the rate equation rate = k [NO]2 [Cl2] = k [acetone][H + ]
dt
The value of the rate constant can be increased by: The order w.r.t. I2 is:
(a) increasing the concentration of NO. (a) 1 (b) 0 (c) 3 (d) 2
(b) increasing the temperature. 27. Which of the following statement(s) is/are correct?
(c) increasing the concentration of the Cl 2 (i) Rate of reaction decreases with passage of time as
(d) doing all of the above the concentration of reactants decrease.
17. Units of rate constant of first and zero order reactions in (ii) For a reaction
terms of molarity M unit are respectively pP + qQ ¾¾ ® rR + sS
(a) sec–1, Msec–1 (b) sec–1, M Rate = k[ P ]x [Q] y where x = p and y = q
(c) Msec–1, sec–1 (d) M, sec–1.
(iii) Rate law is the expression in which reaction rate is
18. A reaction involving two different reactants can never given in terms of molar concentration of reactants
be with each term raised to some power, which may or
(a) bimolecular reaction (b) second order reaction may not be same as the stoichiometric coefficient of
(c) first order reaction (d) unimolecular reaction the reacting species in a balanced chemical equation.
19. 3A ® B + C , it would be a zero order reaction when (a) (i) and (iii) (b) (i) and (ii)
(a) the rate of reaction is proportional to square of (c) (ii) and (iii) (d) (i) only
concentration of A 28. The rate constant for the reaction 2 N 2 O 5 ¾¾® 4 NO 2 + O 2
(b) the rate of reaction remains same at any concentration is 3.10 × 10–5 sec–1. If the rate is 2.4 × 10–5 mol litre–1 sec–1
of A then the concentration of N 2 O 5 (in mol litre–1) is :
(c) the rate remains unchanged at any concentration of (a) 0.04 (b) 0.8 (c) 0.07 (d) 1.4
B and C 29. The rate law for a reaction between the substances A and
(d) the rate of reaction doubles if concentration of B is B is given by Rate = k [A]n [B]m
increased to double On doubling the concentration of A and halving the
20. For the following homogeneous reaction, concentration of B, the ratio of the new rate to the earlier
k
rate of the reaction will be as
A + B ¾¾ ®C
1
the unit of rate constant is (a) (m + n) (b) (n – m) (c) 2(n – m) (d) (m + n )
(a) sec –1 (b) sec–1 mol L–1 2
–1
(c) sec mol L –1 (d) sec–1 mol–2 L2 30. Which one of the following reactions is a true first order
21. Velocity constant k of a reaction is affected by reaction?
(a) Alkaline hydrolysis of ethyl acetate
(a) change in the concentration of the reactant
(b) Acid catalyst hydrolysis of ethyl acetate
(b) change of temperature
(c) Decomposition of N2O
(c) change in the concentration of the product (d) Decomposition of gaseous ammonia on a hot platinum
(d) None of the above surface
EBD_8350
264 CHEMISTRY

31. What is order with respect to A, B, C, respectively 1.0 × 10–2 0.010 0.010 0.010
[A] [B] [C] rate (M/sec.) 1.25 × 10–3 0.005 0.005 0.010
0.2 0.1 0.02 8.08 × 10–3 The order with respect to the reactants, A, B and C are
0.1 0.2 0.02 2.01 × 10–3 respectively
0.1 1.8 0.18 6.03 × 10–3 (a) 3, 2, 0 (b) 3, 2, 1 (c) 2, 2, 0 (d) 2, 1, 0
0.2 0.1 0.08 6.464 × 10–2 37. The chemical reaction 2O 3 ¾ ¾® 3O 2 proceeds as
(a) –1, 1, 3/2 (b) –1, 1, 1/2 (c) 1, 3/2, –1 (d) 1, –1, 3/2
follows:
32. The rate constant of a reaction is 3.00 × 10 3 L mol–1 sec–1.
Fast
The order of this reaction will be: O 3 ¾¾¾® O 2 + O ; O + O 3 ¾Slow
¾¾® 2O 2 the rate law
(a) 0 (b) 1 (c) 2 (d) 3 expression should be
33. During the kinetic study of the reaction, 2A + B ® C + D, (a) r = k[O3]2 (b) r = k [O3]2[O2]–1
following results were obtained: (c) r = k3 [O3][O2]2 (d) r = [O3][O2]2
Run
–1
[A](mol L ) [B](mol L )
–1 Initial rate of formation of 38. Consider a reaction aG + bH ® Products. When
–1 –1
D (mol L min ) concentration of both the reactants G and H is doubled,
–3
I 0.1 0.1 6.0 × 10 the rate increases by eight times. However, when
II 0.3 0.2 7.2 × 10
–2 concentration of G is doubled keeping the concentration
III 0.3 0.4 2.88 × 10
–1 of H fixed, the rate is doubled. The overall order of the
–2 reaction is
IV 0.4 0.1 2.40 × 10
(a) 0 (b) 1 (c) 2 (d) 3
Based on the above data which one of the following is
39. The following data pertains to reaction between A and B :
correct?
S. [A] mol L–1 [B] mol L–1 Rate (mol L–1 time–1)
(a) rate = k [A]2 [B] (b) rate = k[A] [B] No.
2
(c) rate = k [A] [B] 2
(d) rate = k [A] [B]2 1 1.0 × 10–2 2.0 × 10–2 2.0 × 10–4
2 2.0 × 10 –2 2.0 × 10–2 4.0 × 10–4
34. Nitrogen monoxide, NO, reacts with hydrogen, H2,
according to the following equation: 3 2.0 × 10–2 4.0 × 10–2 8.0 × 10–4
2NO(g) + 2H2(g) ® N2(g) + 2H2O(g) Which of the following inference(s) can be drawn from the
If the mechanism for this reaction were, above data ?
2NO(g) + H2(g) ® N2(g) + H2O2(g) ; slow (i) Rate constant of the reaction is 1.0 × 10–4.
H2O2(g) + H2(g) ® 2H2O(g) ; fast (ii) Rate law of the reaction is : rate = k[A][B]
Which of the following rate laws would we expect to obtain (iii) Rate of reaction increases four times on doubling the
experimentally? concentration of both the reactants.
(a) Rate = k[H2O2][H2] (b) Rate = k[NO]2[H2] Select the correct answer using the codes given below :
2
(c) Rate = k[NO] [H2] 2 (d) Rate = k[NO][H2] (a) (i), (ii) and (iii) (b) (i) and (ii)
(c) (ii) and (iii) (d) (iii) only
35. At high pressure, the following reaction is of zero order.

2NH3 ( g ) ¾¾¾¾¾¾¾
® N 2 ( g ) + 3H 2 ( g )
1130 K Topic 3: Integrated Rate Equations
Platinum catalyst
40. Assertion: The rate of reaction is always negative.
Which of the following statements are correct for above
Reason: Minus sign used in expressing the rate shows
reaction?
that concentration of reactant is decreasing.
(i) Rate of reaction = Rate constant
(a) Assertion is correct, reason is correct; reason is a
(ii) Rate of reaction depends on concentration of
correct explanation for assertion.
ammonia.
(b) Assertion is correct, reason is correct; reason is not a
(iii) Rate of decomposition of ammonia will remain constant
correct explanation for assertion.
until ammonia disappears completely.
(c) Assertion is correct, reason is incorrect.
(iv) Further increase in pressure will change the rate of
(d) Assertion is incorrect, reason is correct.
reaction.
41. The plot that represents the zero order reaction is :
(a) (i), (iii) and (iv) (b) (i), (ii) and (iii)
(c) (ii) and (iv) (d) (i), (ii) and (iv)
36. The initial rates of reaction [R] [R]
(a) (b)
3A + 2B + C ¾¾ ® Products, at different initial
t t
concentrations are given below:
Initial rate, [A]0, M [B]0, M [C]0, M
¾®

Ms–1 [R]
5.0 × 10–3 0.010 0.005 0.010 (c) (d) ln[R]
5.0 × 10–3 0.010 0.005 0.015 t t ¾®
CHEMICAL KINETICS 265

42. The plot of concentration of the reactant vs time for a 52. The reaction A ¾® B follows first order kinetics. The time
reaction is a straight line with a negative slope. The reaction taken for 0.8 mole of A to produce 0.6 mole of B is 1 hour.
follows a rate equation What is the time taken for conversion of 0.9 mole of A to
(a) zero order (b) first order produce 0.675 mole of B?
(c) second order (d) third order (a) 2 hours (b) 1 hour (c) 0.5 hour (d) 0.25 hour
43. The half-life of a reaction is inversely proportional to the 53. The rate of a first order reaction is 1.5 × 10–2 mol L–1 min–1 at
square of the initial concentration of the reactant. Then 0.5 M concentration of the reactant. The half life of the reaction is
the order of the reaction is (a) 0.383 min(b) 23.1 min (c) 8.73 min (d) 7.53 min
(a) 0 (b) 1 (c) 2 (d) 3 54. A reaction proceeds by first order, 75% of this reaction
44. The rate equation for a reaction, was completed in 32 min. The time required for 50%
N2O ¾® N2 + 1/2O2 completion is
is Rate = k[N2O]0 = k. If the initial concentration of the (a) 8 min (b) 16 min (c) 20 min (d) 24 min
reactant is a mol L–1, the half-life period of the reaction is
55. t 1 can be taken as the time taken for the concentration of
a a k
(a) t 1 = (b) - t 1 = ka (c) t 1 = (d) t 1 = 4
3
2
2k 2 2
k 2
a a reactant to drop to of its initial value. If the rate
4 t
45. Half life of a first order reaction is 4 s and the initial constant for a first order reaction is k, the 1 can be written
concentration of the reactant is 0.12 M. The concentration 4
of the reactant left after 16 s is as
(a) 0.0075 M(b) 0.06 M (c) 0.03 M (d) 0.015 M (a) 0.75/k (b) 0.69/k (c) 0.29/k (d) 0.10/k
46. The rate constant for a first order reaction whose half-life, 56. Consider the reaction, 2A + B ® products. When
is 480 seconds is : concentration of B alone was doubled, the half-life did not
(a) 2.88 × 10–3 sec–1 (b) 2.72 × 10–3 sec–1 change. When the concentration of A alone was doubled,
(c) 1.44 × 10 sec–3 –1 (d) 1.44 sec–1 the rate increased by two times. The unit of rate constant
for this reaction is
47. The rate constant of a first order reaction is 6.9 ´10-3 s -1 .
(a) s –1 (b) L mol–1 s–1
How much time will it take to reduce the initial
(c) no unit (d) mol L–1 s–1
concentration to its 1/8th value?
(a) 100 s (b) 200 s (c) 300 s (d) 400 s 57. For a first order reaction, a plot of log (a – x) against time is
48. Point out the wrong statement: a straight line with a negative slope equal to
For a first order reaction -k
(a) (b) – 2.303 k
(a) time for half-change (t1/2) is independent of initial 2.303
concentration 2.303 Ea
(b) change in the concentration unit does not change (c) – (d) - 2.303 R
k
the rate constant (k) 58. In a reaction A ® Products, when start is made from
(c) time for half-change × rate constant = 0.693 8.0 × 10–2 M of A, half-life is found to be 120 minute. For
(d) the unit of k is mol–1 min–1 the initial concentration 4.0 × 10–2 M, the half-life of the
49. Diazonium salt decomposes as
reaction becomes 240 minute. The order of the reaction is:
C6 H 5 N +2 Cl - ® C6 H5Cl + N 2 (a) zero (b) one (c) two (d) 0.5
At 0°C, the evolution of N2 becomes two times faster when 59. The rate law for the reaction 2X + Y ® Z is Rate = k[X][Y].
the initial concentration of the salt is doubled. Therefore, The correct statement with regard to this relation is
it is (a) the rate of the reaction is independent of [X] and [Y]
(a) a first order reaction (b) for this reaction t1/2 is independent of initial
(b) a second order reaction concentrations of reactant
(c) independent of the initial concentration of the salt (c) the rate of formation of Z is twice the rate of
(d) a zero order reaction disappearance of X
50. If half-life of a substance is 5 yrs, then the total amount of (d) the rate of disappearance of X is not equal to rate
substance left after 15 years, when initial amount is 64 of disappearance of Y
grams is 60. The decomposition of ammonia on tungsten surface at 500
(a) 16 grams (b) 2 grams (c) 32 grams (d) 8 grams. K follows zero order kinetics. The half-life period of this
51. In a 1st order reaction, reactant concentration C varies reaction is 45 minutes when the initial pressure is 4 bar.
with time t as : The half-life period (minutes) of the reaction when the initial
1 pressure is 16 bar at the same temperature is
(a) increases linearly with t
C (a) 120 (b) 60 (c) 240 (d) 180
(b) log C decreases linearly with t 61. A substance 'A' decomposes by a first order reaction
1 starting initially with [A] = 2.00 M and after 200 min, [A]
(c) C decreases with becomes 0.15 M. For this reaction t1/2 is
t
(a) 53.72 min (b) 50.49 min
1 (c) 48.45 min (d) 46.45 min
(d) log C decreases with
t
EBD_8350
266 CHEMISTRY

62. For the first order reaction


Topic 5: Temperature Dependence of the Rate of a Reaction
C 2 H 4 O(g) ® CH 4 (g) + CO(g) , the initial pressure of
67. In the Haber process for the manufacture of ammonia, the
C2H4O(g) is 80 torr and total pressure at the end of 20
following catalyst is used
minutes is 120 torr. The time needed for 75% decomposition
(a) Platinized asbestos
of C2H4O would be :
(a) 20 minutes (b) 40 minutes (b) Iron with molybdenum as promoter
(c) 80 minutes (d) 120 minutes (c) Copper oxide
(d) Alumina
Topic 4: Pseudo First Order Reaction 68. If the activation energy for the forward reaction is 150 kJ mol–1
63. Assertion: The order of a reaction may be negative. and that of the reverse reaction is 260 kJ mol –1, what is the
Reason: In some cases, the rate of reaction decreases as enthalpy change for the reaction ?
the concentration of the reactant increases. (a) 410 kJ mol–1 (b) –110 kJ mol–1
(a) Assertion is correct, reason is correct; reason is a (c) 110 kJ mol–1 (d) – 410 kJ mol–1
correct explanation for assertion. 69. In respect of the equation k = Ae - Ea / RT in chemical
(b) Assertion is correct, reason is correct; reason is not a kinetics, which one of the following statements is correct ?
correct explanation for assertion.
(a) A is adsorption factor
(c) Assertion is correct, reason is incorrect.
(d) Assertion is incorrect, reason is correct. (b) Ea is energy of activation
64. Which of the following statement(s) is/are correct ? (c) R is Rydberg’s constant
(i) For a zero order reaction concentration [R] vs time (t) (d) k is equilibrium constant
gives a straight line plot 70. In a reversible reaction the energy of activation of the
[R]0 forward reaction is 50 kcal. The energy of activation for
(ii) For a first order reaction log does not vary
linearly with time. [R] the reverse reaction will be
(iii) Inversion of cane sugar is a pseudo first order (a) < 50 kcal
reaction. (b) either greater than or less than 50 kcal
(a) (i) and (iii) (b) (i) only (c) 50 kcal
(c) (ii) and (iii) (d) (iii) only (d) > 50 kcal
65. Assertion: Hydrolysis of cane sugar is a first order reaction. 71. Which of the following statements best describes how a
Reason: Water is present in large excesss during catalyst works?
hydrolysis. (a) A catalyst changes the potential energies of the
(a) Assertion is correct, reason is correct; reason is a reactants and products.
correct explanation for assertion. (b) A catalyst decreases the temperature of the reaction
(b) Assertion is correct, reason is correct; reason is not a which leads to a faster rate.
correct explanation for assertion. (c) A catalyst lowers the activation energy for the reaction
(c) Assertion is correct, reason is incorrect. by providing a different reaction mechanism.
(d) Assertion is incorrect, reason is correct.
(d) A catalyst destroys some of the reactants, which
66. The bromination of acetone that occurs in acid solution is
lowers the concentration of the reactants.
represented by this equation.
72. For a first order reaction, the plot of log k against 1/T is a
CH3COCH3 (aq) + Br2 (aq) ® CH3COCH2Br (aq) + H+ (aq)
+ Br– (aq) straight line. The slope of the line is equal to
These kinetic data were obtained for given reaction Ea 2.303 Ea - Ea
concentrations. (a) - (b) - (c) - (d) 2.303 R
R E a R 2.303
Initial Initial rate, 73. In a zero-order reaction for every 10° rise of temperature,
Concentrations, M disappearance the rate is doubled. If the temperature is increased from
of Br2, Ms–1 10°C to 100°C, the rate of the reaction will become :
[CH3 COCH3] [Br2] [H+] (a) 256 times (b) 512 times (c) 64 times (d) 128 times
0.30 0.05 0.05 5.7×10–5 74. What is the activation energy for a reaction if its rate
0.30 0.10 0.05 5.7 × 10–5 doubles when the temperature is raised from 20°C to 35°C?
0.30 0.10 0.10 1.2 × 10–4
(R = 8.314 J mol–1 K–1)
0.40 0.05 0.20 3.1 × 10–4
(a) 269 kJ mol–1 (b) 34.7 kJ mol–1
Based on given data, the rate equations is: (c) 15.1 kJ mol –1 (d) 342 kJ mol–1
(a) Rate = k[CH3COCH3][H+] 75. A reaction having equal energies of activation for forward
(b) Rate = k [CH3COCH3][Br2] and reverse reaction has :
(c) Rate = k [CH3COCH3] [Br2] [H+]2 (a) DG = 0 (b) DH = 0
(d) Rate = k [CH3COCH3][Br2] [H+] (c) DH = DG = DS = 0 (d) DS = 0
CHEMICAL KINETICS 267

76. In an exothermic reaction if DH is the enthalpy then (iii) Rate constant k is equal to collision frequency A if
activation energy is Ea = ¥.
(a) more than DH (b) less than DH (iv) ln k vs T is a straight line.
(c) equal to DH (d) none of the above (v) ln k vs 1/T is a straight line.
1 Correct statements are
77. In the Arrhenius plot of ln k vs , a linear plot is obtained (a) (i) and (iv) (b) (ii) and (v)
T
(c) (iii) and (iv) (d) (ii) and (iii)
with a slope of –2 × 104 K. The energy of activation of the
85. The following statement(s) is (are) correct :
reaction (in kJ mole–1) is (R value is 8.3 J K–1 mol–1)
(i) A plot of log Kp versus 1/T is linear
(a) 83 (b) 166 (c) 249 (d) 332 (ii) A plot of log [X] versus time is linear for a first order
78. Plots showing the variation of the rate constant (k) with reaction, X ® P
temperature (T) are given below. The plot that follows (iii) A plot of log P versus 1/T is linear at constant volume
Arrhenius equation is (iv) A plot of P versus 1/V is linear at constant temperature
(a) (i) only (b) (ii) only
(c) (i) and (iv) (d) (i), (ii) and (iv)
86. Match the columns.
(a) (b) Column-I Column-II
(A) Catalyst alters the rate (p) cannot be fraction or
zero of reaction
(B) Molecularity (q) proper orientation is
not there always.
(C) Second half life of first (r) by lowering the
k
(c) (d) order reaction activation energy
(D) Energetically favourable (s) is same as the first
T
reactions are sometimes
79. The activation energy for a reaction is 9.0 kcal/mol. The slow
increase in the rate constant when its temperature is (a) A – (q), B – (r), C – (s), D – (p)
increased from 298 K to 308 K is (b) A – (r), B – (s), C – (p), D – (q)
(a) 63% (b) 50% (c) 100% (d) 10% (c) A – (r), B – (p), C – (s), D – (q)
80. A chemical reaction was carried out at 300 K and 280 K. (d) A – (p), B – (r), C – (s), D – (q)
The rate constants were found to be k1 and k2 respectively. 87. For a first order reaction A®P, the temperature (T)
then dependent rate constant (k) was found to follow the
(a) k2 = 4k1 (b) k2 = 2k1 1
equation log k = – (2000) + 6.0 . The pre-exponential
(c) k2 = 0.25 k1 (d) k2 = 0.5 k1 T
81. Activation energy of the reaction is factor A and the activation energy Ea, respectively, are
(a) the energy released during the reaction (a) 1.0 × 106 s–1 and 9.2 kJ mol–1
(b) the energy evolved when activated complex is formed (b) 6.0 s–1 and 16.6 kJ mol –1
(c) minimum amount of energy needed to overcome the (c) 1.0 × 106 s–1 and 16.6 kJ mol–1
potential barrier (d) 1.0 × 106 s–1 and 38.3 kJ mol–1
(d) the energy needed to form one mole of the product 88. A graph plotted between log k vs 1/T for calculating
82. In a reaction, the threshold energy is equal to activation energy is shown by
(a) activation energy + normal energy of reactants
(b) activation energy – normal energy of reactants
(c) normal energy of reactants – activation energy (a) log k (b) log k
(d) average kinetic energy of molecules of reactants
83. For the exothermic reaction A + B ® C + D, DH is the 1/T 1/T

heat of reaction and Ea is the energy of activation. The


energy of activation for the formation of A + B will be
(a) Ea (b) DH (c) Ea + DH (d) DH – Ea (c) log k (d) log k
84. Consider the following statements:
(i) Increase in concentration of reactant increases the 1/T 1/T
rate of a zero order reaction. 89. The activation energy for a hypothetical reaction,
(ii) Rate constant k is equal to collision frequency A if A ¾® Product, is 12.49 kcal/mol. If temperature is raised
Ea = 0. from 295 K to 305 K, the rate of reaction increased by
(a) 60% (b) 100% (c) 50% (d) 20%
EBD_8350
268 CHEMISTRY

90. A reactant (A) froms two products : Topic 6: Collision Theory of Chemical Reactions
k1
A ¾¾® B, Activation Energy Ea 96. In terms of the ‘Collision Theory of Chemical Kinetics’,
1
k2 the rate of a chemical reaction is proportional to
A ¾¾ ® C, Activation Energy Ea (a) the change in free energy per second
2
If Ea = 2 Ea , then k1 and k2 are related as : (b) the change in temperature per second
2 1
Ea /RT (c) the number of collisions per second
Ea /RT
(a) k2 = k1e 1 (b) k2 =k1e 2
(d) the number of products molecules
Ea /RT Ea /RT 97. According to collision theory, which of the following is
(c) k1 = Ak2e 1 (d) k1 = 2k 2e 2 NOT a true statement concerning a catalyst?
91. During decomposition of an activated complex. (a) A catalyst changes the temperature of reaction.
(i) energy is always released (b) The mechanism of a reaction will change when a
(ii) energy is always absorbed catalyst is added.
(iii) energy does not change (c) A catalyst provides a different activation energy for a
(iv) reactants may be formed reaction.
(a) (i), (ii) and (iii) (b) (i) and (iv) (d) A catalyst changes the speed of a reaction, but not
(c) (ii) and (iii) (d) (ii), (iii) and (iv) the equilibrium constant.
98. According to the collision theory of reaction rates, the
92. Rate of two reactions whose rate constants are k1 and k2
rate of reaction increases with temperature due to
are equal at 300 K such that :
(a) greater number of collision
Ea2 - Ea1 = 2 RT , (b) higher velocity of reacting molecules
(c) greater number of molecules having the activation
A2 energy
So calculate ln
A1 (d) decrease in the activation energy
(a) ln 4 (b) 2 (c) log 2 (d) 2- ln 2 99. According to which theory activation energy and proper
93. The activation energies of two reactions are E 1 and orientation of the molecules together determine the criteria
E2 (E1 > E2). If the temperature of the system is increased for an effective collision ?
from T1 to T2, the rate constant of the reactions changes (a) Arrhenius theory
from k1 to k1' in the first reaction and k2 to k2' in the second (b) Activated complex theory
reaction. Predict which of the following expression is (c) Collision theory
correct? (d) Both (a) and (c)
100. Collision theory is applicable to
k1' k 2' k1' k 2' (a) first order reactions (b) zero order reactions
(a) = (b) >
k1 k 2 k1 k 2 (c) bimolecular reactions (d) intra-molecular reactions
101. According to collision theory, not all collisions between
k1' k 2' k1' k 2' molecules lead to reaction. Which of the following
(c) < (d) = =1 statements provide reasons for the same ?
k1 k 2 k1 k 2
(i) The total energy of the two colliding molecules is
94. The rate constant, the activation energy and the arrhenius less than some minimum amount of energy.
parameter of a chemical reaction at 25°C are 3.0 × 10–4s–1, (ii) Molecules cannot react with each other unless a
104.4 kJ mol–1 and 6.0 × 1014 s–1 respectively. The value of catalyst is present.
the rate constant as T ® ¥ is (iii) Molecules that are improperly oriented during
(a) 2.0 × 1018 s–1 (b) 6.0 × 1014 s–1 collision will not react.
(c) Infinity (d) 3.6 × 1030 s–1 (iv) Molecules in different states of matter cannot react
95. Which of the following statements is incorrect ? with each other.
(a) Energy is always released when activated complex (a) (i) and (ii) (b) (i) and (iii)
decomposes to form products. (c) (ii) and (iii) (d) (i) and (iv)
(b) Peak of the energy distribution curve corresponds to 102. Match the columns
the most probable potential energy. Column - I Column - II
(c) Peak of the energy distribution curve corresponds to (A) Number of collisions per (p) Effective
the most probable kinectic energy. second per unit volume collisions.
(d) When the temperature is raised maximum of energy of the reaction mixture.
distribution curve moves to higher energy value and (B) Fraction of molecules (q) Collision
with energies equal to frequency
broadens out.
or greater than Ea
CHEMICAL KINETICS 269

(C) Molecules for which (r) e - Ea / RT 105. Consider the energy diagram of a reaction : B ® A. On the
Rate = Z AB e- Ea / RT basis of given diagram, select the correct code for matching
shows significant deviations Column-I and Column-II.
(D) Collision in which, molecules (s) Complex X
collide with sufficient K.E. molecules
and proper orientation.

Energy
(a) A – (q), B – (r), C – (s), D – (p)
B
(b) A – (r), B – (q), C – (s), D – (p)
(c) A – (q), B – (s), C – (r), D – (p) A
(d) A – (q), B – (r), C – (p), D – (s)
103. The reason for almost doubling the rate of reaction on
increasing the temperature of the reaction system by 10°C is Progress of reaction
(a) the value of threshold energy increases Column-I Column-II
(b) collision frequency increases (A) X – A (p) Enthalpy of reaction
(c) the fraction of the molecule having energy equal to
(B) X – B (q) Energy of transition state
threshold energy or more increases
(d) activation energy decreases (C) A – B (r) Activation energy of
104. In most cases, for a rise of 10 K temperature, the rate forward reaction
constant is doubled to tripled. This is due to the reason (D) X (s) Activation energy of
that backward reaction
(a) collision frequency increases by a factor of 2 to 3. (a) A – (s), B – (r), C – (q), D – (p)
(b) fraction of molecules possessing threshold energy (b) A – (q), B – (r), C – (p), D – (s)
increases by a factor of 2 to 3 (c) A – (r), B – (s), C – (p), D – (q)
(c) activation energy is lowered by a factor of 2 to 3.
(d) A – (s), B – (r), C – (p), D – (q)
(d) none of these

1. For a chemical reaction, 2X + Y ¾® Z, the rate of 7. A reaction takes place in various steps. The rate constant
appearance of Z is 0.05 mol L–1 per min. Find the rate of for first, second, third and fifth steps are
disappearance of x in mol L–1 min–1. k1, k2, k3 and k5 respectively. The overall rate constant is
2. For the reaction at 273 K 1/ 2
NO(g) + O3(g) ¾® NO2(g) + O2(g) k 2 æ k1 ö
given by k = ç ÷ .
It is observed that the pressure of NO(g) falls from 700 mm k3 è k5 ø
Hg to 500 mm Hg in 250 s. Calculate the average rate of
reaction in 10–3 atm s–1 If activation energy are 40, 60, 50 and 10 kJ/mol
3. For a first order reaction, calculate the ratio between the respectively, find the overall energy of activation (kJ/mol).
time taken to complete 3/4th of the reaction and time taken 8. The half life decomposition of gaseous CH3CHO at
to complete half of the reaction. constant temperature but at initial pressure of 364 mm and
4. The half-life period and initial concentration for a reaction 170 mm of Hg were 410 sec, and 880 sec respectively. Find
are as follows. the order of the reaction.
9. A gaseous reaction
Initial concentration (M) 350 540 158 1
t1/2 (hr) 425 275 941 X 2 (g) ¾® Y + Z (g)
2
There is increase in pressure from 100 mm to 120 mm
What is order of reaction? in 5 minutes. Calculate the rate of disappearance of
5. The rate of a reaction triples when temperature changes X2 in mm min–1.
from 20ºC to 50ºC. Calculate energy of activation for the 10. The reaction X ® Y is an exothermic reaction. Activation
reaction in kJ mol–1 (R = 8.314 JK–1 mol–1). energy of the reaction for X into Y is 150 kJ mol–1. Enthalpy
6. Sucrose decomposes in acid solution into glucose and of reaction is 135 kJ mol –1. Calculate the activation energy
fructose according to the first order rate law with t 1/2 = for the reverse reaction, Y ® X in kJ mol–1.
3×00 hours. What fraction of the sample of sucrose re- 11. 2 g of a radioactive sample having half-life of 15 days was
mains after 8 hours ? synthesised on Ist Jan. 2009. What is the amount of the
EBD_8350
270 CHEMISTRY

sample left behind on Ist March, 2009 (including both the Find the half-life period (in minutes) of the reaction when
days) in g? the initial pressure is 16 bar at the same temperature.
12. Half-lives of decomposition of NH3 on the surface of a 14. In a chemical reaction A is converted into B. The rates of
catalyst for different initial pressures are given as: reaction, starting with initial concentrations of A as
P(torr) 200 300 500 2 × 10–3 M and 1 × 10–3 M, are equal to 2.40 × 10–4 Ms–1
and 0.60 × 10–4 Ms–1 respectively. Find the order of
t 1/2(min) 10 15 25
reaction with respect to reactant A.
Find the order of the reaction.
15. The instantaneous rate of disappearance of MnO4– ion in
13. The decomposition of ammonia on tungsten surface at 500 the following reaction is 4.56 × 10–3 Ms–1.
K follows zero order kinetics. The half-life period of this
2MnO4– + 10I– + 16H+ ® 2Mn2+ + 5I2 + 8H2O
reaction is 45 minutes when the initial pressure is 4 bar.
Find the rate of appearance I2 in Ms–1.

Exercise 3 : NCERT Exemplar & Past Year MCQs


NCERT Exemplar MCQs 6. Which of the following statements is correct?
(a) The rate of a reaction decreases with passage of time
1. The role of a catalyst is to change ........... .
(a) Gibbs energy of reaction as the concentration of reactants decreases.
(b) enthalpy of reaction (b) The rate of a reaction is same at any time during the
(c) activation energy of reaction reaction.
(d) equilibrium constant (c) The rate of a reaction is independent of temperature
2. In the presence of a catalyst, the heat evolved or absorbed change.
during the reaction ........ . (d) The rate of a reaction decreases with increase in
(a) increases concentration of reactant(s).
(b) decreases 7. Which of the following expressions is correct for the rate
(c) remains unchanged of reaction given below?
(d) may increase or decrease
5Br - ( aq ) + BrO3- ( aq ) + 6H + ( aq ) ® 3Br2 ( aq ) + 3H 2 O ( l )
3. Activation energy of a chemical reaction can be determined
D [ Br - ] D [H+ ] D [ Br - ] 6 D [ H + ]
by ...... .
(a) determining the rate constant at standard temperature (a) =5 (b) =
(b) determining the rate constant at two temperatures Dt Dt Dt 5 Dt
(c) determining probability of collision
(d) using catalyst D [ Br - ] 5 D [ H + ] D [ Br - ] D [H+ ]
(c) = (d) =6
4. Consider the Arrhenius equation given below and mark Dt 6 Dt Dt Dt
the correct option. Ea 8. Rate law for the reaction A + 2B ¾® C is found to be
-
k = Ae RT Rate = k[A] [B]
(a) Rate constant increases exponentially with increasing Concentration of reactant 'B' is doubled, keeping the
activation energy and decreasing temperature concentration of 'A' constant, the value of rate constant
(b) Rate constant decreases exponentially with increasing will be ...... .
activation energy and decreasing temperature
(a) the same (b) doubled
(c) Rate constant increases exponentially with decreasing
activation energy and decreasing temperature (c) quadrupled (d) halved
(d) Rate constant increases exponentially with decreasing 9. Which of the following statements is incorrect about the
activation energy and increasing temperature collision theory of chemical reaction?
5. Which of the following statements is not correct about (a) It considers reacting molecules or atoms to be hard
order of a reaction? spheres and ignores their structural features
(a) The order of a reaction can be a fractional number (b) Number of effective collisions determines the rate of
(b) Order of a reaction is experimentally determined reaction
quantity (c) Collision of atoms or molecules possessing sufficient
(c) The order of a reaction is always equal to the sum of threshold energy results into the product formation
the stoichiometric coefficients of reactants in the (d) Molecules should collide with sufficient threshold
balanced chemical equation for a reaction energy and proper orientation for the collision to be
(d) The order of a reaction is the sum of the powers of effective
molar concentration of the reactants in the rate law 10. A first order reaction is 50% completed in 1.26 × 1014 s.
expression How much time would it take for 100% completion?
CHEMICAL KINETICS 271

(a) 1.26 × 1015 s (b) 2.52 × 1014 s


(c) 2.52 × 10 s28 (d) Infinite (a)
11. Compounds 'A' and 'B' react according to the following
chemical equation.

¾®
A(g) + 2B(g) ¾® 2C(g)
Concentration of either 'A' or 'B' were changed keeping the ln k
concentrations of one of the reactants constant and rates
were measured as a function of initial concentration.
Following results were obtained. Choose the correct option 1/T ¾®
for the rate equations for this reaction.

Experiment Initial Initial Initial (b)


concentration concentration concentration of
–1 –1 –1 –1
of [A]/mol L of [B]/mol L [C]/mol L s

¾®
1. 0.30 0.30 0.10
2. 0.30 0.60 0.40 ln k
3. 0.60 0.30 0.20

(a) Rate = k [A]2[B] (b) Rate = k [A] [B]2


(c) Rate = k [A] [B] (d) Rate = k [A]2[B]0 1/T ¾®
12. Consider figure and mark the correct option.
(c)
Activated complex

¾®
E1
¾®

Products ln k
Energy

E2

Reactants
1/T ¾®
Reaction coordinate ¾®
(a) Activation energy of forward reaction is E1 + E2 and (d)
product is less stable than reactant
(b) Activation energy of forward reaction is E1 + E2 and
¾®

product is more stable than reactant


(c) Activation energy of both forward and backward ln k
reaction is E1 + E2 and reactant is more stable than
product
(d) Activation energy of backward reaction is E1 and
product is more stable than reactant 1/T ¾®
13. Consider a first order gas phase decomposition reaction
15. A graph of volume of hydrogen released vs time for the
given below
reaction between zinc and dil. HCl is given in figure. On
A(g) ® B(g) + C(g)
the basis of this, mark the correct option.
The initial pressure of the system before decomposition of
A was pi. After lapse of time 't' total pressure of the system V5
increased by x units and became 'pt'. The rate constant k
V4
for the reaction is given as ........... .
2.303 pi 2.303 pi
(a) k = log (b) k= log V3
t pi - x t 2 pi - pt
2.303 pi 2.303 pi
(c) k= log (d) k = log
t 2 pi + pt t pi + x V2
V1
14. According to Arrhenius equation, rate constant k is equal
to A e - E a/RTRT
T. Which of the following options represents

1 0
20 30 40 50
the graph of ln k vs ?
T
EBD_8350
272 CHEMISTRY

Activated complex
V3 - V 2
(a) Average rate upto 40s is
40
V3 - V 2
(b) Average rate upto 40s is

Energy
40 - 30 (III)

V3 Reactants Products
(c) Average rate upto 40s is
40
V3 - V1 Reaction coordinate
(d) Average rate upto 40s is
40 - 20 (a) Only (I) (b) Only (II)
16. Consider the graph given in figure. Which of the following (c) Only (III) (d) Both (I) and (II)
options does not show instantaneous rate of reaction at
18. Which of the following statement is not correct for the
40th second?
catalyst?
V5 (a) It catalyses the forward and backward reactions to
V4
the same extent
(b) It alters DG of the reaction
V3
(c) It is a substance that does not changes the equilibrium
constant of a reaction
V2 (d) It provides an alternate mechanism by reducing
V1
activation energy between reactants and products.
19. Consider the reaction A ® B. The concentration of both
the reactants and the products varies exponentially with
0 time. Which of the following figures correctly describes the
20 30 40 50
change in concentration of reactants and products with
V5 - V2 V4 - V2
(a) (b) time?
50 - 3 0 50 - 30
[B]
V3 - V 2 V3 - V1
(c) (d)
40 - 30 40 - 20
Concentration

17. Which of the following graphs represents exothermic


reaction? (a)

[A]
Activated complex
Time

[B]
Energy

(I) Reactants
Concentration

Products (b)

Reaction coordinate [A]

Time
Activated complex

[B]
Energy

Concentration

(II)
(c)
Products
Reactants [A]
Reaction coordinate Time
CHEMICAL KINETICS 273

24. The activation energy of a reaction can be determined from


the slope of which of the following graphs ?
[A] [AIPMT 2015, C]
ln K l
Concentration

(a) vs.T (b) ln K vs.


(d) T T
T l
[B] (c) ln K vs. T (d) ln K vs. T
Time 25. Higher order (>3) reactions are rare due to : [JEE M 2015, C]
(a) shifting of equilibrium towards reactants due to elastic
20. The value of rate constant of a pseudo first order collisions
reaction.............
(b) loss of active species on collision
(a) depends on the concentration of reactants present in
(c) low probability of simultaneous collision of all the
small amount
reacting species
(b) depends on the concentration of reactants present in
(d) increase in entropy and activation energy as more
excess
molecules are involved
(c) is independent of the concentration of reactants
26. The rate of a first-order reaction is 0.04 mol L–1s–1 at 10
(d) depends only on temperature
seconds and 0.03 mol L–1s–1 at 20 seconds after initiation
Past Year MCQs of the reaction. The half-life period of the reaction is
21. For the non - stoichiometric reaction 2A + B ® C + D, [NEET 2016, S]
the following kinetic data were obtained in three separate (a) 24.1 s (b) 34.1 s
experiments, all at 298 K. [JEE M 2014, S] (c) 44.1 s (d) 54.1 s
27. The addition of a catalyst during a chemical reaction alters
which of the following quantities? [NEET 2016, C]
Initial Initial Initial rate of (a) Entropy (b) Internal energy
Concentration Concentration formation of C (c) Enthalpy (d) Activation energy
–1 –1
(A ) (B ) (mol L s ) 28. Decomposition of H2O2 follows a first order reaction. In
0.1 M 0.1 M 1.2 × 10
–3 fifty minutes the concentration of H2O2 decreases from
0.1 M 0.2 M 1.2 × 10
–3 0.5 to 0.125 M in one such decomposition. When the
–3 concentration of H2O2 reaches 0.05 M, the rate of
0.2 M 0.1 M 2.4 × 10
formation of O2 will be: [JEE M 2016, S]
–1
(a) 2.66 L min at STP
The rate law for the formation of C is: (b) 1.34 × 10–2 mol min–1
dC dC (c) 6.96 × 10–2 mol min–1
= k [ A][ B] = k [ A] [ B]
2
(a) (b) (d) 6.93 × 10–4 mol min–1
dt dt
29. Mechanism of a hypothetical reaction [NEET 2017, S]
dC dC
= k [ A][ B ] = k [ A]
2
(c) (d) X2 + Y2 ® 2XY is given below :
dt dt
(i) X2 ® X + X(fast)
22. The rate constant of the reaction A ® B is 0.6 × 10–3 mole per (ii) X + Y2 ƒ XY + Y (slow)
second. If the concentration of A is 5 M then concentration (iii) X + Y ® XY (fast)
of B after 20 minutes is : [AIPMT 2015 RS, S]
The overall order of the reaction will be :
(a) 1.08 M (b) 3.60 M (c) 0.36 M (d) 0.72 M (a) 2 (b) 0
23. When initial concentration of a reactant is doubled in a (c) 1.5 (d) 1
reaction, its half-life period is not affected. The order of 30. A first order reaction has a specific reaction rate of
the reaction is : [AIPMT 2015, A] 10–2 sec–1. How much time will it take for 20g of the reactant
(a) First to reduce to 5 g ? [NEET 2017, S]
(b) Second (a) 138.6 sec (b) 346.5 sec
(c) More than zero but less than first (c) 693.0 sec (d) 238.6 sec
(d) Zero
EBD_8350
274 CHEMISTRY

31. Two reactions R1 and R2 have identical pre-exponential 36. The given plots represents the variation of the
factors. Activation energy of R1 exceeds that of R2 by concentration of a reactant R with time for two different
10 kJ mol–1. If k1 and k2 are rate constants for reactions R1 reactions (i) and (ii). The respective orders of the reactions
and R2 respectively at 300 K, then ln(k2/k1) is equal to : are: [JEE M 2019, S]
(R = 8.314 J mol–1K–1) [JEE M 2017, S]
(a) 8 (b) 12
(c) 6 (d) 4
32. The correct difference between first and second order
reactions is that [NEET 2018, C]
(a) The rate of a first-order reaction does not depend on
reactant concentrations, the rate of a second-order (a) 1, 0 (b) 1, 1
reaction does depend on reactant concentrations (c) 0, 1 (d) 0, 2
(b) The half-life of a first-order reaction does not depend 37. The following results were obtained during kinetic studies
on [A]0, the half-life of a second-order reaction does of the reaction; [JEE M 2019, S]
depend on [A]0 2A + B ® Products
(c) The rate of a first-order reaction does depend on
Experiment [A] [B] Initial Rate of
reactant concentrations, the rate of a second-order
(in mol L–1) (in mol L–1) reaction
reaction does not depend on reactant concentrations
(in mol L–1 min–1)
(d) A first-order reaction can be catalyzed, a second- I 0.10 0.20 6.93 × 10–3
order reaction cannot be catalyzed
II 0.10 0.25 6.93 × 10–3
33. At 518 °C, the rate of decomposition of a sample of gaseous
III 0.20 0.30 1.386 × 10–2
acetaldehyde, initially at a pressure of 363 Torr, was 1.00
Torr s–1 when 5% had reacted and 0.5 Torr s–1 when 33% The time (in minutes) required to consume half of A is:
had reacted. The order of the reaction is : [JEE M 2018, S] (a) 5 (b) 10
(a) 2 (b) 3 (c) 1 (d) 100
38. The rate constant for a first order reaction is 4.606 × 10–3
(c) 1 (d) 0
s–1. The time required to reduce 2.0 g of the reactant to 0.2
34. If the rate constant for a first order reaction is k, the time
g is : [NEET 2020, A]
(t) required for the completion of 99% of the reaction is
(a) 200 s (b) 500 s
given by : [NEET 2019, A]
(a) t = 0.693/k (b) t = 6.909/k (c) 1000 s (d) 100 s
(c) t = 4.606/k (d) t = 2.303/k 39. An increase in the concentration of the reactants of a
35. For the chemical reaction reaction leads to change in [NEET 2020, C]
N2(g) + 3H2(g) ƒ 2NH3(g) (a) heat of reaction
the gas is correct option is: [NEET 2019, C] (b) threshold energy
1 d [H2 ] 1 d [ NH 3 ] (c) collision frequency
(a) - =-
3 dt 2 dt (d) activation energy
40. During the nuclear explosion, one of the products is 90Sr
d [ N2 ] d [ NH3 ] with half life of 6.93 years. If 1 mg of 90Sr was absorbed in
(b) - =2
dt dt the bones of a newly born baby in place of Ca, how much
time, in years, is required to reduce it by 90% if it is not
d [ N 2 ] 1 d [ NH3 ]
(c) - = lost metabolically________. [NV, JEE M 2020, S]
dt 2 dt

d [H2 ] d [ NH3 ]
(d) 3 =2
dt dt
CHEMICAL KINETICS 275

Exercise 4 : Problem Solving Skill Enhancer MCQs


1. Decomposition of NH4NO2 (aq) into N2 (g) and 2H2O(l) is (a) 2.0 × 105 s–1 (b) 1.5 × 1011 s–1
first order reaction. Which of the following graph is correct? (c) 3.0 × 1015 s–1 (d) 2 × 102 s–1
8. For the reaction:
[NH4NO2 ]

[NH4NO2 ]
[Cr(H2O)6]3+ + [SCN –] ® [Cr(H2O)5NCS]2+ + H2O
(a) (b) The rate law is: r = k[Cr(H2O)6]3+ [SCN –].
time time The value of k is 2.0 × 10– 6 L mol– 1 s– 1 at 14°C and
2.2 × 10– 5 L mol– 1 s– 1 at 30°C. What is the value of Ea?
(a) 26 kcal mol– 1 (b) 2.6 kcal mol– 1
[NH4NO2 ]

[NH4NO2 ]

(c) (d) (c) 2600 kcal mol– 1 (d) 260 kcal mol– 1
9. In the first order reaction 2 N 2 O 5 ¾
¾® 4 NO 2 + O 2 , if a
time time -1
mol L is the initial concentration of N 2 O5 , the
2. For a particular reaction with initial conc. of the reactants concentration of NO2 at time t will be
as a1 and a2, the half-life period are t1 and t2 respectively.
The order of the reaction (n) is given by : (a) a e -kt (b) a (1 - e - kt )

log ( t2 / t1 ) log ( t1 / t2 ) (c) 2a ( e - kt -1) (d) 2a (1 - e - kt )


(a) n = 1+ (b) n =
log ( a2 / a1 ) log ( a2 / a1 ) 10. The decomposition of H2O2 can be followed by titration
with KMnO4 and is found to be a first order reaction. The
log ( t1 / t2 ) log ( t2 /t1 ) rate constant is 4.5 × 10– 2. In an experiment, the initial titre
(c) n = 1+ (d) n = 1 -
log ( a2 / a1 ) log ( a2 /a1 ) value was 25 mL. The titre value will be 5 mL after a lapse of
3. A first order reaction is half completed in 45 minutes. How log e 5
long does it need 99.9% of the reaction to be completed (a) 4.5 × 10– 2 × 5 min (b) min
4.5×10 – 2
(a) 5 hours (b) 7.5 hours (c) 10 hours (d) 20 hours
4. Cyclopropane rearranges to form propene
log e 5/4
¾¾
® CH3 - CH = CH 2 (c) min (d) None of these
4.5×10 – 2
This follows first order kinetics. The rate constant is
2.714 × 10 –3 sec –1 . The initial concentration of k = 0.6 M min -1
11. For a reaction A ¾¾¾¾¾¾¾ ® 2B starting with 1 M
cyclopropane is 0.29 M. What will be the concentration of
cyclopropane after 100 sec of 'A' only, concentration of B (in M) after 100 sec. and
(a) 0.035 M (b) 0.22 M (c) 0.145 M (d) 0.0018 M 200 sec. is respectively?
5. The rate constants k1 and k2 for two different reactions are (a) 2 and 4 (b) 1 and 2
1016 . e–2000/T and 1015 . e–1000/T, respectively. The temperature (c) 2 and 3 (d) none of these
at which k1 = k2 is : 12. For an exothermic chemical process occurring in two steps
2000 as follows
(a) 1000 K (b) K (c) 2000 K (d) 1000 K (i) A + B ® X (slow) (ii) X ® AB (fast)
2.303 2.303
6. For the equilibrium, A(g) ƒ B(g), DH is –40 kJ/mol. If The progress of reaction can be best described by :
the ratio of the activation energies of the forward (Ef)
2 X
and reverse (Eb) reactions is then : P.E. A+B P.E.
3
(a) Ef = 80 kJ/mol; Eb = 120 kJ/mol (a) AB (b)
A+B AB
(b) Ef = 60 kJ/mol; Eb = 100 kJ/mol (reaction coordinate) R.C.
(c) Ef = 30 kJ/mol; Eb = 70 kJ/mol R.C.
(d) Ef = 70 kJ/mol; Eb = 30 kJ/mol
7. Activation energy and pre-exponential factor of a chemical P.E. P.E.
X
reaction are 2 × 102 kJ mol–1 and 3.0 × 1015 s–1 respectively. (c)
A+B X
(d) AB
A+B
If the rate constant of the reaction at 298 is 5.0 × 10 –1, its AB

value when T® ¥ is : R.C. R.C.


EBD_8350
276 CHEMISTRY

13. The decomposition of A follows two parallel first order


B. H 2S ƒ H + + HS- (fast equilibrium)
reactions
k1 = 1.26 × 10 –4 sec–1 Cl 2 + HS- ® 2Cl - + H + + S (slow )
B
A k2 = 3.8 × 10–5 sec–1 (a) B only (b) Both A and B
C
(c) Neither A nor B (d) A only
The percentage distribution of B and C are
(a) 90 % B and 10 % C 15. The rate constant of a reaction is 1.5 × 10– 3 at 25°C and
2.1 × 10– 2 at 60°C. The activation energy is
(b) 80 % B and 20 % C
(c) 60 % B abd 40 % C 2.1 ´ 10 – 2
35
(d) 76.83% B and 23.17 % C (a) R log e
333 1.5 ´ 10 – 2
14. Consider the reaction :
Cl 2 (aq) + H 2S(aq) ® S(s) + 2H + (aq) + 2Cl- (aq) 298 ´ 333 21
(b) R log e
The rate equation for this reaction is 35 1.5
rate = k[Cl2 ][H 2S]
298 ´ 333
Which of these mechanisms is/are consistent with this (c) R log e 2.1
35
rate equation?
A. Cl 2 + H 2S ® H + + Cl - + Cl+ + HS- (slow) 298 ´ 333 2.1
(d) R log e
35 1.5
Cl + + HS- ® H + + Cl- + S (fast)

ANSW ER KEY
Exercise 1 : NCERT Based Topic-wise MCQs
1 (b) 12 (b) 23 (c) 34 (b) 45 (a) 56 (b) 67 (b) 78 (a) 89 (b) 100 (c)
2 (c) 13 (d) 24 (a) 35 (a) 46 (c) 57 (c) 68 (b) 79 (a) 90 (c) 101 (b)
3 (a) 14 (c) 25 (a) 36 (d) 47 (c) 58 (c) 69 (b) 80 (c) 91 (b) 102 (a)
4 (d) 15 (d) 26 (b) 37 (b) 48 (d) 59 (d) 70 (b) 81 (c) 92 (b) 103 (b)
5 (c) 16 (b) 27 (a) 38 (d) 49 (a) 60 (d) 71 (c) 82 (a) 93 (b) 104 (b)
6 (a) 17 (a) 28 (b) 39 (c) 50 (d) 61 (a) 72 (d) 83 (c) 94 (b) 105 (d)
7 (b) 18 (d) 29 (c) 40 (d) 51 (b) 62 (b) 73 (b) 84 (b) 95 (b)
8 (c) 19 (b) 30 (c) 41 (b) 52 (b) 63 (a) 74 (b) 85 (d) 96 (c)
9 (d) 20 (c) 31 (d) 42 (a) 53 (b) 64 (a) 75 (b) 86 (c) 97 (a)
10 (a) 21 (b) 32 (c) 43 (d) 54 (b) 65 (a) 76 (d) 87 (d) 98 (a)
11 (b) 22 (a) 33 (d) 44 (a) 55 (c) 66 (a) 77 (b) 88 (b) 99 (c)
Exercise 2 : Numeric/Integer Answer Questions
1 (0.1) 3 (2) 5 (28.81) 7 (25) 9 (8) 11 (0.125) 13 (180) 15 (0.0114)
2 (1.053) 4 (2) 6 (0.158) 8 (2) 10 (285) 12 (0) 14 (2)
Exercise 3 : NCERT Exemplar & Past Year MCQs
1 (c) 5 (c) 9 (c) 13 (b) 17 (a) 21 (d) 25 (c) 29 (c) 33 (a) 37 (a)
2 (c) 6 (a) 10 (d) 14 (a) 18 (b) 22 (d) 26 (a) 30 (a) 34 (c) 38 (b)
3 (b) 7 (c) 11 (b) 15 (c) 19 (b) 23 (a) 27 (d) 31 (d) 35 (c) 39 (c)
4 (d) 8 (b) 12 (a) 16 (b) 20 (b) 24 (b) 28 (d) 32 (b) 36 (a) 40 (23.03)
Exercise 4 : Problem Solving Skill Enhancer MCQs
1 (d) 3 (b) 5 (d) 7 (c) 9 (d) 11 (d) 13 (d) 15 (b)
2 (c) 4 (b) 6 (a) 8 (a) 10 (b) 12 (c) 14 (d)
19 Surface Chemistry

Trend Buster NEET & JEE Main

Number of Questions from 2020-15 7 4 Minimum one question has


Weightage 2.2% 2.00% been asked in NEET every year.

The most Important Concepts that Cover Maximum number of Questions asked in past 6 years.

Adsorption 1 2
Colloids & their Classificaton 5 2

Less Important Concepts that Cover 1 or 2 Questions asked in past 6 years.

Catalysis 1 —
Emulsion & Colloids Around Us — —

NEET JEE

2020 Colloids & their Zeta potential 1 Easy — —


classification
2019 Colloids & their Formation of Negatively 1 Average 2 Easy
Classificaton / Adsorption Charged Colloidal Sol /
Adsorption Isotherm /
Classifiction of Colloids
2018 Colloids & their Hardy Schulze Rule 1 Easy — —
Classificaton
2017 Catalysis / Colloids & their Properties of Catalyst / 1 Easy 1 Average
Classificaton Tyndal Effect
2016 Adsorption / Colloids & Mechanism of Adsorption / 2 Easy 1 Easy
their Classificaton Classification of Colloids /
Adsorption Isotherm
2015 Colloids & their Properties of Colloidal Solution 1 Easy — —
Classificaton
EBD_8350
278 CHEMISTRY
SURFACE CHEMISTRY 279
EBD_8350
280 CHEMISTRY

Problem Solving Tips/ Tricks/ Points to Remember

4 If one of the products acts as a catalyst, it is called 4 Associated Colloids : The molecules of certain
autocatalysis, e.g., in the reaction of oxalic acid with substances are smaller than particles but aggregate
acidified KMnO4 solution, Mn2+ ions produced act as spontaneously in a given solvent particles of colloidal
autocatalyst. The reaction speeds up with time. size. The aggregates are known as micelles.
4 The greater the valence of the flocculating ion added, the 4 According to freundlich adsorption isotherm:
greater is its power to cause precipitation. This is known x/m= k. p1/ n ; where (n > 1)
as Hardy-Schulze rule. In the coagulation of a negative Taking log of the above equation, the following equation
sol, the flocculating power is in the order: will be observed
Al3+ >Ba2+ > Na+ log x/m = log k +1/n log p
Similarly, in the coagulation of a positive sol, the x/m is plotted on y axis and log p is on x axis. If straight
flocculating power is in the order: line is observed then only freundlich isotherm is verified.
[Fe(CN)6]4– > PO43– > SO2– 4 > Cl

Slope gives 1/n and intercept gives log k. The value of
4 Higher is the valency of active ion, the greater is its 1/n varies from 0 to 1.
coagulating power. If 1/n is 0, adsorption is independent of pressure.
4 Induced catalyst : A substance which induces the similar If 1/n is 1, adsorption changes with pressure.
reaction in another which is otherwise not possible act 4 When adsorption of gases occurs on the surface of a
as induced catalyst, e.g., sodium sulphite solution oxidises
metal, this is called occlusion.
in air to sodium sulphate but sodium arsenite is not
4 Solidified butter is gel (water is dispersed in fat), whereas
oxidised by air. In presence of sodium sulphite it is also
liquid butter is an emulsion (water is dispersed in melted
oxidised.
fat or oil).
[O]
Na 2SO3 + Na 3 AsO3 ¾¾¾
® Na 2SO 4 + Na 3AsO 4 4 To increase the process of purification, the dialysis is
air
carried out by applying electric field. This process is called
Therefore sodium sulphite is induced catalyst.
electrodialysis. The important application of
4 Peptization : The dispersal of freshly precipitated
electrodialysis process in the artificial kidney machine
substance into colloidal solution by the addition of some
(dialyser) used for the purification of blood of the patients
electrolyte having one ion common is known as
peptization. The electrolyte used is called peptizing agent whose kidneys have failed to work.
e.g. AgCl is peptized by HCl giving negative sol of 4 Examples of charged sols:
[AgCl]Cl– Positively charged sols Negatively charged sols
4 Flocculation Value : The minimum concentration in Hydrated metallic oxides, Metals, e.g. , copper, silver,
millimoles per litre required to cause the precipitation of a e.g ., Fe2 O3 ·x H2 O, gold sols
sol in 2 hours. The smaller the flocculation value, the
Al2 O3·x H2 O
higher is the precipitating power.
Basic dye stuffs, e.g ., Acidic dye stuff, e.g .,
4 Gold Number : The number of milligrams of a hydrophilic
methylene blue sol eosin, congo red sol
colloid that will prevent the precipitation of 10 mL of a
Metal oxides, e.g ., TiO2 Metal sulphides, e.g .,
gold sol on the 1 mL of 10% sodium chloride solution is
sol AS2S3 , Sb 2 S3, CdS sol
known as gold number. The smaller the value of gold
number of lyophilic sol, the greater is the protective Haemoglobin Sols of starch, gum, gelatin,
action. clay, charcoal
SURFACE CHEMISTRY 281

Exercise 1 : NCERT Based Topic-wise MCQs


12. Adsorption is accompanied by
Topic 1: Adsorption (a) decrease in enthalpy and increase in entropy
1. Which is not correct regarding the physical adsorption of (b) increase in enthalpy and increase in entropy
a gas on surface of solid? (c) decrease in enthalpy and decrease in entropy
(a) On increasing temperature, adsorption increases (d) increase in enthalpy and decrease in entropy
continuously 13. Choose the incorrect statement in respect of physisorption?
(b) Enthalpy and entropy changes are negative (a) It is not specific in nature
(c) Adsorption is more for some specific substance (b) It arises because of van der Waal’s force
(c) It is reversible in nature
(d) Reversible
(d) Enthalpy of adsorption is in the range 80-240 kJ mol–1
2. How many layers are adsorbed in chemical adsorption? 14. If x is amount of adsorbate and m is amount of adsorbent,
(a) One (b) Two (c) Many (d) Zero which of the following relations is not related to adsorption
3. Adsorption due to strong chemical forces is called process ?
(a) Chemisorption (b) Physisorption (a) x / m = f (p) at constant T.
(c) Reversible adsorption (d) Both (b) and (c) (b) x / m = f (T) at constant p.
4. In physical adsorption, gas molecules are bound on the (c) p = f (T) at constant (x / m).
solid surface by x
(d) = p× T
(a) chemical forces (b) electrostatic forces m
(c) gravitational forces (d) van der Waal’s forces 15. Which is adsorbed in maximum amount by activated charcoal?
5. Which of the following statements is not correct ? (a) N2 (b) CO2 (c) Cl2 (d) O2
(a) Physical adsorption is due to van der Waal’s forces 16. Which is not correct regarding the adsorption of a gas on
(b) Chemical adsorption first decreases with increase in surface of solid?
(a) On increasing temperature, adsorption increases
temperature. continuously
(c) Physical adsorption is reversible (b) Enthalpy and entropy changes are –ve
(d) Adsorption energy for a chemical adsorption is (c) Adsorption is more for some specific substance
generally greater than that of physical adsorption. (d) This Phenomenon is reversible
6. Adsorption of gases on solid surface is exothermic reaction 17. Which of the following is related to adsorption?
because (i) DH = – ve (ii) DS = – ve
(a) free energy increases (b) enthalpy is positive (iii) –TDS = – ve (iv) DG = – ve
(c) entropy increases (d) enthalpy is negative (a) (i), (ii) and (iv) (b) (ii) and (iii)
7. For adsorption of a gas on a solid, the plot of log x/m vs (c) (iii) only (d) (i), (iii) and (iv)
log P is linear with slope equal to (‘n’ being whole number) 18. Which of the following phenomenon occurs when the
1 precipitate of Mg(OH)2 attains blue colour in presence of
(a) k (b) log k (c) n (d) magneson reagent?
n (i) Absorption of solvent
8. In Freundlich adsorption isotherm, the value of 1/n is : (ii) Adsorption of coloured substance
(a) between 0 and 1 in all cases (iii) Absorption and adsorption both of solvent
(b) between 2 and 4 in all cases (iv) Adsorption of solvent
(c) 1 in case of physical adsorption (a) (i) and (ii) (b) (ii) only
(d) 1 in case of chemisorption (c) (ii) and (iv) (d) (iii) only
9. Which is correct about physical adsorption? x
(a) High temperature and high pressure favour adsorption 19. Assertion : The relation = k.p1/ n is known as
(b) High temperature and low pressure favour adsorption m
Freundlich adsorption isotherm, where ‘x’ is the mass of
(c) Low temperature and high pressure favour adsorption gas adsorbed by ‘m’ grams of adsorbate, ‘p’ is the
(d) Low temperature and low pressure favour adsorption equilibrium pressure, k and n are constants for given system
10. Methylene blue, from its aqueous solution, is adsorbed and temperature.
on activated charcoal at 25°C. For this process, which of Reason : When several substances have same value of
the following statement is correct ? 1
(a) The adsorption requires activation at 25°C , the lines by which their adsorption isotherms can be
n
(b) The adsorption is accompanied by a decrease in represented will meet at a point.
enthalpy (a) If both Assertion and Reason are correct and Reason
(c) The adsorption increases with increase of temperature is the correct explanation of Assertion.
(d) The adsorption is irreversible (b) If both Assertion and Reason are correct, but Reason
11. The gas which is least adsorbed on charcoal (under is not the correct explanation of Assertion.
identical conditions) is (c) If Assertion is correct but Reason is incorrect.
(a) HCl (b) O2 (c) CO2 (d) NH3 (d) If both the Assertion and Reason are incorrect.
EBD_8350
282 CHEMISTRY

20. Which of the following statements is incorrect regarding (c) increasing activation energy
physisorptions? (d) decreasing equilibrium constant
(a) More easily liquefiable gases are adsorbed readily. 28. A catalyst can affect reversible reaction by
(b) Under high pressure it results into multimolecular layer (a) changing equilibrium constant
on adsorbent surface. (b) slowing forward reaction
(c) Enthalpy of adsorption (DH adsorption) is low and (c) attaining equilibria in both directions
positive. (d) None of these
(d) It occurs because of van der Waal’s forces. 29. The action of enzymes in living system is to :
21. In the adsorption of a gas on solid, Freundlich isotherm is (a) supply energy to tissues
obeyed. The slope of the plot is zero. Then the extent of
(b) enhance immunity
adsorption is
(c) circulate oxygen
(a) directly proportional to the pressure of the gas
(d) enhance the rate of biochemical reactions.
(b) inversely proportional to the pressure of the gas
(c) directly proportional to the square root of the pressure 30. Hydrolysis of urea is an example of
of the gas (a) homogenous catalysis (b) heterogenous catalysis
(d) independent of the pressure of the gas (c) biochemical catalysis (d) zeolite catalysis
22. Assertion : The enthalpy of physisorption is greater than 31. The efficiency of an enzyme in catalysing a reaction is due
chemisorption. to its capacity
Reason : Molecules of adsorbate and adsorbent are held (a) to form a strong enzyme-substrate complex
by van der Waal’s forces in physisorption and by chemical (b) to decrease the bond energies of substrate molecule
bonds in chemisorption. (c) to change the shape of the substrate molecule
(d) to lower the activation energy of the reaction
(a) If both Assertion and Reason are correct and Reason
32. Catalytic poisons act by :
is the correct explanation of Assertion.
(a) making the products chemically inactive.
(b) If both Assertion and Reason are correct, but Reason
is not the correct explanation of Assertion. (b) increasing the rate of the backward reaction.
(c) If Assertion is correct but Reason is incorrect. (c) chemical combination with any one of the reactants.
(d) If Assertion is incorrect and Reason is correct. (d) preferential adsorption on the catalyst surface.
23. Which of the following statements regarding difference 33. Which of the following kind of catalysis can be explained
between adsorption and absorption is incorrect? by the adsorption theory ?
(a) Adsorption is a surface whereas absorption is a bulk (a) Homogeneous catalysis (b) Acid - base catalysis
phenomena. (c) Heterogeneous catalysis (d) Enzyme catalysis
(b) Water vapours are absorbed by anhydrous CaCl2 34. Which one of the following is an example of homogeneous
but adsorbed by silica gel. catalysis?
(c) Adsorption and absorption take place individually. (a) Haber’s process of synthesis of ammonia
They can not occur simultaneously. (b) Catalytic conversion of SO2 to SO3 in contact process
(d) All of the above statements are correct. (c) Catalytic hydrogenation of oils
(d) Acid hydrolysis of methyl acetate.
Topic 2: Catalysis 35. Identify the correct statement regarding enzymes
(a) Enzymes are specific biological catalysts that cannot
24. The role of a catalyst in a reversible reaction is to
be poisoned.
(a) increase the rate of forward reaction (b) Enzymes are normally heterogeneous catalysts that
(b) decrease the rate of backward reaction are very specific in their action.
(c) alter the equilibrium constant of the reaction (c) Enzymes are specific biological catalysts that can
(d) allow the equilibrium to be achieved quickly normally function at very high temperatures (T » 1000K).
25. A catalyst : (d) Enzymes are specific biological catalysts that possess
(a) lowers the activation energy well-defined active sites.
(b) changes the rate constant 36. What is the role of molybdenum in Haber’s process for
(c) changes the product manufacture of ammonia?
(d) itself destroyed in the reaction (a) As catalytic poison (b) As a catalytic promoter
26. Active charcoal is a good catalyst because it (c) As a catalyst (d) As a reactant
(a) is made up of carbon atoms. 37. Which of the following step(s) is/are not involved in the
(b) is very reactive. mechanism of adsorption theory of heterogeneous catalyst?
(c) has more adsorption power. (i) Diffusion of reactants to the surface of the catalyst.
(d) has inert nature toward reagents. (ii) Sorption of reactant molecules on the surface of the
27. Catalyst increases the rate of reaction by catalyst.
(iii) Occurrence of chemical reaction on the catalyst’s
(a) decreasing threshold energy
surface through formation of an intermediate.
(b) decreasing activation energy
SURFACE CHEMISTRY 283

(iv) Desorption of reaction products from the catalyst’s


Topic 3: Colloids and Classification of Colloids
surface.
(v) Diffusion of reaction products away from the 43. Milk is a colloid in which a
catalyst’s surface. (a) liquid is dispersed in a liquid
(a) (i) only (b) (ii) and (iv) (b) solid is dispersed in a liquid
(c) (ii) only (d) (i), (ii) and (v) (c) gas is dispersed in a liquid
38. Which of the following statement(s) is/are correct about (d) sugar is dispersed in a liquid
solid catalyst? 44. An example of dispersion of a liquid in a gas is :
(i) Same reactants may give different product by using (a) milk (b) vegetable oil
different catalysts. (c) foam (d) mist
(ii) Catalyst is required in large quantities to catalyse 45. Alloy is an example of
reactions. (a) gel (b) solidified emulsion
(iii) Catalyst does not change DH of reaction. (c) solid solution (d) sol
(iv) Catalytic activity of a solid catalyst does not depend 46. If dispersed phase is a liquid and the dispersion medium is
upon the strength of chemisorption. a solid, the colloid is known as
(a) (i) and (iii) (b) (i) only (a) a sol (b) a gel
(c) (ii), (iii) and (iv) (d) (iii) and (iv) (c) an emulsion (d) a foam
39. Match the columns 47. Hair cream is an example of
Column-I Column-II (a) gel (b) sol (c) aerosol (d) emulsion
(Biochemical reactions) (Enzymes) 48. Which one of the following in not a colloidal solution?
(A) C6H12O6(aq) ¾® (p) Zymase (a) Smoke (b) Ink (c) Blood (d) Air
2C2H5OH(aq) + 2CO2(g) 49. Small liquid droplets dispersed in another liquid is called
(B) NH2CONH2(aq) + H2O(l) ¾® (q) Pepsin (a) gel (b) suspension
(c) emulsion (d) true solution
2NH3(g) + CO2(g) 50. When dispersed phase is liquid and dispersion medium is
(C) Proteins ¾® Peptides (r) Urease gas then the colloidal system is called
(D) C12H22O11(aq) + H2O(l) ¾® (s) Invertase (a) Smoke (b) Clouds (c) Jellies (d) Emulsions
C6H12O6(aq) + C6H12O6(aq) 51. How non-polar and polar part in micelle are arranged ?
(a) Polar at outer surface and non-polar at inner surface
(a) A – (p), B – (r), C – (q), D – (s) (b) Polar at inner surface and non-polar at outer surface
(b) A – (p), B – (q), C – (r), D – (s) (c) Both polar and non-polar at inner surface
(c) A – (r), B – (p), C – (q), D – (s) (d) Distributed all over the surface
(d) A – (p), B – (r), C – (s), D – (q) 52. Lyophobic colloids are :
40. Which of the following statements about a catalyst is true? (a) gun proteins (b) protective colloids
(a) A catalyst accelerates the reaction by bringing down (c) irreversible colloids (d) reversible colloids
the free energy of activation 53. Associated colloid among the following is
(b) A catalyst also takes part in the reaction mechanism (a) enzymes (b) proteins
(c) A catalyst makes the reaction more feasible by making (c) cellulose (d) sodium stearate
the DGº more negative 54. The formation of micelles takes place only above
(d) A catalyst makes the equilibrium constant of the (a) inversion temperature (b) Boyle temperature
reaction more favourable for the forward reaction (c) critical temperature (d) Kraft temperature
41. Which one of the following, statements is incorrect about 55. Which of the following is used for neutralising charge on
enzyme catalysis? colloidal solution?
(a) Enzymes are mostly protenious in nature. (a) Electrons (b) Electrolytes
(b) Enzyme action is specific. (c) Positively charged ions (d) Compounds
(c) Enzymes are denaturated by ultraviolet rays and at 56. Pure water can be obtained from sea water by
high temperature. (a) Centrifugation (b) Plasmolysis
(d) Enzymes are least reactive at optimum temperature. (c) Reverse osmosis (d) Sedimentation
42. Which of the following statement(s) is/are correct? 57. Blood may be purified by
(i) Zeolites are good shape selective catalysts because (a) Dialysis (b) Electro-osmosis
of their honeycomb-like structures. (c) Coagulation (d) Filtration
(ii) All zeolites are naturally occurring substance. 58. The movement of colloidal particles towards their
(iii) An important zeolite catalyst used in the petroleum respective electrodes in the presence of an electric field is
industry in ZSM-5. known as :
(a) (i) only (b) (ii) only (a) electrolysis (b) brownian movement
(c) (i) and (iii) (d) (ii) and (iii) (c) electrodialysis (d) electrophoresis
EBD_8350
284 CHEMISTRY

59. Peptization involves 73. Surface tension of lyophilic sols is


(a) precipitation of colloidal particles (a) lower than that of H2O
(b) disintegration of colloidal aggregates (b) more than that of H2O
(c) evaporation of dispersion medium (c) equal to that of H2O
(d) impact of molecules of the dispersion medium on the (d) either less or more than H2O depending upon the
colloidal particles nature of disperse phase
60. Hardy-Schulze rule explains the effect of electrolytes on 74. Which of the following is a lyophilic colloid ?
the coagulation of colloidal solution. According to this
(a) Milk (b) Gum (c) Fog (d) Blood
rule, coagulation power of cations follow the order
75. Which one is an example of multimolecular colloid system
(a) Ba+2 > Na+ > Al+3 (b) Al+3 > Na+ > Ba+2
+3
(c) Al > Ba > Na +2 + (d) Ba+2 > Al+3 > Na+ (a) Soap dispersed in water (b) Protein dispersed in water
61. Tyndall effect is shown by (c) Gold dispersed in water (d) Gum dispersed in water
(a) sol (b) solution 76. Example of intrinsic colloid is
(c) plasma (d) precipitate (a) glue (b) sulphur (c) Fe (d) As 2S 3
62. Which of the following acts as protective colloid? 77. During dialysis
(a) Silica gel (b) Gelatin (a) only solvent molecules can diffuse
(c) Sodium acetate (d) None of these (b) solvent molecules, ions and colloidal particles can diffuse
63. Which of the following is not a property of colloidal solution? (c) all kinds of particles can diffuse through the semi-
(a) Heterogenity (b) Particle size > 100 nm permeable membrane
(c) Tyndall effect (d) Brownian movement (d) solvent molecules and ions can diffuse
64. Which of the following method is used for coagulation of 78. The electrolytic impurities of a sol can most easily be
the sol? separated by
(a) By mixing two oppositely charged sols. (a) dialysis (b) electrosmosis
(b) By electrophoresis. (c) electrophoresis (d) electrodialysis
(c) By addition of electrolytes. 79. The separation of colloidal particles from particles of
(d) All of the above.
molecular dimensions is known as
65. The dispersed phase and dispersion medium in soap lather
(a) sedimentation (b) dispersion
are respectively
(a) gas and liquid (b) liquid and gas (c) pyrolysis (d) dialysis
(c) solid and gas (d) solid and liquid 80. Colloidal gold is prepared by
66. Which of the following is not a colloid ? (a) Mechanical dispersion (b) Peptisation
(a) Chlorophyll (b) Smoke (c) Bredig’s Arc method (d) Hydrolysis
(c) Ruby glass (d) Milk 81. The cause of Brownian movement is
67. Which of the following forms a colloidal solution in water? (a) heat changes in liquid state
(a) NaCl (b) Glucose (b) convectional currents
(c) Starch (d) Barium nitrate (c) the impact of molecules of the dispersion medium on
68. Which one of the following isan examplefor multimolecular the colloidal particles.
colloid? (d) attractive forces between the colloidal particles and
(a) Sulphur sol molecules of dispersion medium.
(b) Aqueous enzyme sol 82. The charge on colloidal particles is due to
(c) Alcoholic polystyrene sol (a) presence of electrolyte
(d) Aqueous sol of sodium laurylsulphate (b) very small size of particles
69. Point out the false statement :
(c) adsorption of ions from the solution
(a) The colloidal solution of a liquid in liquid is called gel
(d) None of these
(b) Hardy Schulze rule is related with coagulation
(c) Brownian movement and Tyndall effect are shown by 83. In which of the following, Tyndall effect is not observed ?
colloidal system (a) Suspensions (b) Emulsions
(d) Gold number is a measure of the protective power of (c) Sugar solution (d) Gold sol
lyophilic colloid 84. Which of the following is not the condition for Tyndall
70. In Brownian movement or motion, the paths of the particles effect?
are (a) The refractive indices of dispersed phase and
(a) Linear (b) Zig-zag (c) Uncertain (d) Curved dispersion medium should differ greatly in magnitude.
71. The size of colloidal particles is between (b) The diameter of the dispersed particles is not much
(a) 10–7 – 10–9 cm (b) 10–9 – 10–11 cm smaller than the wavelength of light used.
–5
(c) 10 – 10 cm –7 (d) 10–2 – 10–3 cm (c) Tyndall effect is observed only when viewed from
72. Which one of the following is correctly matched? the direction of the passage of light.
(a) Emulsion-smoke (b) Gel-butter (d) All of these are required conditions for Tyndall
(c) Aerosol-hair cream (d) Sol-whipped cream effect.
SURFACE CHEMISTRY 285

85. Match the columns 92. Assertion: An ordinary filter paper impregnated with
Column-I Column-II collodion solution stops the flow of colloidal particles.
(A) As2S3 sol (p) Bredig’s Arc method Reason: Pore size of the filter paper becomes more than
(B) Fe(OH)3 sol (q) Double decomposition the size of colloidal particle.
(C) Colloidal sols of metals (r) Peptization (a) If both Assertion and Reason are correct and Reason
like Au, Ag, Pt, etc. is the correct explanation of Assertion.
(D) Conversion of freshly (s) Hydrolysis (b) If both Assertion and Reason are correct, but Reason
prepared precipitate is not the correct explanation of Assertion.
into a colloidal sol (c) If Assertion is correct but Reason is incorrect.
(a) A – (q), B – (s), C – (r), D – (p) (d) If both the Assertion and Reason are incorrect.
(b) A – (q), B – (p), C – (s), D – (r) 93. Match the columns
(c) A – (s), B – (q), C – (p), D – (r) Column-I Column-II
(d) A – (q), B – (s), C – (p), D – (r) (A) Argyrol (p) Kalazar
86. Match the columns (B) Antimony (q) Intramuscular injection
Column-I Column-II (C) Colloidal gold (r) Stomach disorders
(A) Protective colloid (p) FeCl3 + NaOH (D) Milk of magnesia (s) Eye lotion
(B) Liquid - liquid colloid (q) Lyophilic colloids (a) A – (r), B – (p), C – (s), D – (q)
(C) Positively charged colloid (r) Emulsion (b) A – (r), B – (p), C – (q), D – (s)
(D) Negatively charged colloid (s) FeCl3 + hot water (c) A – (s), B – (q), C – (p), D – (s)
(a) A – (q), B – (r), C – (p), D – (s) (d) A – (s), B – (p), C – (q), D – (r)
94. Assertion : The value of colligative properties are of small
(b) A – (p), B – (r), C – (s), D – (q)
order for colloids as compared to true solution.
(c) A – (q), B – (r), C – (s), D – (p)
Reason : Number of particles in colloidal solution is
(d) A – (r), B – (q), C – (s), D – (p)
comparatively smaller than true solutions.
87. Bredig arc method cannot be used to prepare colloidal (a) If both Assertion and Reason are correct and Reason
solution of which of the following is the correct explanation of Assertion.
(a) Pt (b) Fe (c) Ag (d) Au (b) If both Assertion and Reason are correct, but Reason
88. Colloidal solutions are not purified by is not the correct explanation of Assertion.
(a) Dialysis (b) Electrodialysis (c) If Assertion is correct but Reason is incorrect.
(c) Ultrafiltration (d) Electrophoresis (d) If both the Assertion and Reason are incorrect.
89. Assertion: Detergents with low CMC are more economical 95. Which of the following forms cationic micelles above certain
to use. concentration?
Reason: Cleansing action of detergents involves the (a) Sodium dodecyl sulphate
formation of micelles. These are formed when the (b) Sodium acetate
concentration of detergents becomes equal to CMC. (c) Urea
(a) If both Assertion and Reason are correct and Reason (d) Cetyl trimethyl ammonium bromide
is the correct explanation of Assertion. 96. Which of the following does not contain a hydrophobic
(b) If both Assertion and Reason are correct, but Reason structure?
is not the correct explanation of Assertion. (a) Linseed oil (b) Lanolin
(c) If Assertion is correct but Reason is incorrect. (c) Glycogen (d) Rubber
(d) If both the Assertion and Reason are incorrect. 97. Which of the following ions can cause coagulation of
proteins ?
90. Which of the following will be most effective in the
(a) Ag+ (b) Na+ (c) Mg2+ (d) Ca2+
coagulation of Al(OH)3 sol ?
98. Which of the following statements is incorrect?
(a) KCN (b) BaCl2
(a) Colloidal gold is used for intramuscular injection.
(c) NaCl (d) Mg3(PO4)2 (b) Colloidal solution of latex is used in preparation of
91. Match the columns rubber.
Column-I Column-II (c) Photographic films are prepared by coating an
(A) Butter (p) dispersion of liquid in liquid emulsion of AgBr in gelatin over glass plate.
(B) Pumice stone (q) dispersion of solid in liquid (d) Tannin used in leather industry contains positively
(C) Milk (r) dispersion of gas in solid charged colloidal particles.
(D) Paints (s) dispersion of liquid in solid 99. How many of the following are negatively charged sols?
(a) A – (r), B – (s), C – (p), D – (q) Eosin dye, sol of charcoal, haemoglobin, Al2O3.xH2O,
(b) A – (s), B – (r), C – (p), D – (q) As2S3, TiO2 sol, copper sol
(c) A – (q), B – (r), C – (p), D – (s) (a) 3 (b) 4
(d) A – (s), B – (r), C – (q), D – (p) (c) 6 (d) All of these
EBD_8350
286 CHEMISTRY

(c) the electrolyte present in river water coagulate the clay


Topic 4: Emulsion and Colloids Around Us
(d) Both (a) and (c) are correct
100. Match the columns 102. .......... is a silver sol used as an eye lotion. Fill in the blank
Column-I Column-II with an appropriate answer.
(A) Oil in water emulsion (p) Clouds (a) Amytal (b) Argyrol
(B) Aerosols containing (q) Vanishing cream (c) Ciprofloxacin (d) Both (a) and (b)
small droplets of water 103. Emulsions can be broken into constituent liquid
suspended in air by ______.
(C) When river water meets (r) Smoke (a) heating (b) freezing
the sea water (c) centrifuging (d) All of these
(D) Colloidal solution of (s) Formation of delta 104. Which of the following is not true about the emulsion?
carbon, arsenic compounds, (a) Emulsion can be broken into constituent liquids by
dust etc. in air heat, freezing
(a) A – (q), B – (p), C – (s), D – (r) (b) Emulsion of oil in water is generally unstable
(b) A – (p), B – (q), C – (s), D – (r) (c) Emulsion do not show the Tyndall effect
(c) A – (q), B – (s), C – (p), D – (r) (d) They show brownian motion
(d) A – (q), B – (p), C – (r), D – (s) 105. Which of the following is not emulsifying agent for W/O
101. How does a delta form at the meeting place of sea and river emulsion?
water? (a) Lampblack
(a) The electrolyte present in sea water coagulate the clay (b) Long chain alcohol
(b) the electrolyte present in sea water has no role (c) Proteins
(d) Heavy metal salts of fatty acids

1. How many of the following are colloid?


Sodium chloride, Starch solution, Cane sugar and Blood æxö
2. How many of the following substances gives a positive 9. Graph between log ç ÷ and log P is a straight line at
èmø
charged sol?
Gold, a metal sulphide, ferric hydroxide and silver chloride angle 45° with intercept OA as shown.
3. How many of the following are not a colloid?
Chlorophyll, Smoke, Ruby glass and Milk
4. For the coagulation of 500 mL of arsenious sulphide sol,
2 mL of 1 M NaCl is required. What is the flocculation
value of NaCl?
5. The gold number of gelatin is 0.01. What the amount of
gelatin to be added to 1000 mL of a colloidal sol of gold to
prevent its coagulation, before adding 1 mL of 10% NaCl
solution.
æxö
6. 526.3 mL of 0.5 m HCl is shaken with 0.5 g of activated Calculate ç ÷ at a pressure of 2 atm.
charcoal and filtered. The concentration of the filtrate is èmø
reduced to 0.4 m. Calculate the amount of adsorption (x/m). (Given: k = 2)
7. How many colloidal systems exist in nature in which liquid 10. On addition of 1 mL of 10% NaCl solution to 10 mL gold sol
is dispersed phase? in the presence of 0.025 g of starch, the coagulation is just
8. For soaps critical micelle concentration (CMC) is 10–x prevented. Find the gold number of starch.
(min.) to 10–y (max.) mol/L. What is the value of x?

Exercise 3 : NCERT Exemplar & Past Year MCQs


NCERT Exemplar MCQs 2. At the equilibrium position in the process of adsorption
................. .
1. Which of the following process does not occur at the (a) DH > 0
interface of phases? (b) DH = TDS
(a) Crystallisation (b) Heterogeneous catalysis (c) DH > TDS
(c) Homogeneous catalysis (d) Corrosion (d) DH < TDS
SURFACE CHEMISTRY 287

3. Which of the following interface cannot be obtained? Pt ( s )


(a) Liquid-liquid (b) Solid-liquid
(ii) 2SO2 ( g ) ¾¾¾® 2SO3 ( g )
(c) Liquid-gas (d) Gas-gas Fe( s )
(iii) N 2 ( g ) + 3 H 2 ( g ) ¾¾¾® 2NH3 ( g )
4. The term 'sorption' stands for ................ .
()
® CH3COOH ( aq )
HCl l
(a) absorption (iv) CH 3COOCH3 (l) + H 2O ( l ) ¾¾¾¾
(b) adsorption + CH3 OH ( aq )
(c) Both absorption and adsorption (a) (ii), (iii) (b) (ii), (iii) and (iv)
(d) desorption (c) (i), (ii) and (iii) (d) (iv)
5. Extent of physisorption of a gas increases with ............. . 14. Which of the following will show Tyndall effect?
(a) increase in temperature (a) Aqueous solution of soap below critical micelle
(b) decrease in temperature concentration
(c) decrease in surface area of adsorbent (b) Aqueous solution of soap above critical micelle
(d) decrease in strength of van der Waal's forces concentration
6. Extent of adsorption of adsorbate from solution phase (c) Aqueous solution of sodium chloride
increases with ............... . (d) Aqueos solution of sugar
(a) increase in amount of adsorbate in solution 15. Freshly prepared precipitate sometimes gets converted to
(b) decrease in surface area of adsorbate collodial solution by ................... .
(a) coagulation (b) electrolysis
(c) increase in temperature of solution
(c) diffusion (d) peptisation
(d) decrease in amount of adsorbate in solution
16. Which of the following electrolytes will have maximum
7. Which one of the following is not applicable to the coagulating value for Ag/Ag+ sol?
phenomenon of adsorption? (a) Na2S (b) Na3PO4 (c) Na2SO4 (d) NaCl
(a) DH > 0 (b) DG < 0 (c) DS < 0 (d) DH < 0 17. A colloidal system having a solid substance as a dispersed
8. Which of the following is not a favourable condition for phase and a liquid as a dispersion medium is classified
physical adsorption? as .............. .
(a) High pressure (a) solid sol (b) gel
(b) Negative DH (c) emulsion (d) sol
(c) Higher critical temperature of adsorbate 18. At high concentration of soap in water, soap behaves as......
(d) High temperature (a) molecular colloid
9. Physical adsorption of a gaseous species may change to (b) associated colloid
chemical adsorption with ............. . (c) macromolecular colloid
(a) decrease in temperature (d) lyophilic colloid
(b) increase in temperature 19. Method by which lyophobic sol can be protected
(c) increase in surface area of adsorbent (a) by addition of oppositely charged sol
(b) by addition of an electrolyte
(d) decrease in surface area of adsorbent
(c) by addition of lyophilic sol
10. In physisorption, adsorbent does not show specificity for
(d) by boiling
any particular gas because ............. . 20. The values of colligative properties of colloidal solution
(a) involved van der Waal's forces are universal are of small order in comparison to those shown by true
(b) gases involved behave like ideal gases solutions of same concentration because of colloidal
(c) enthalpy of adsorption is low particles ............
(d) it is a reversible process (a) exhibit enormous surface area
11. Which of the following is an example of absorption? (b) remain suspended in the dispersion medium
(a) Water on silica gel (c) form lyophilic colloids
(b) Water on calcium chloride (d) are comparatively less in number
(c) Hydrogen on finely divided nickel 21. Arrange the following diagrams in correct sequence of steps
(d) Oxygen on metal surface involved in the mechanism of catalysis, in accordance with
12. On the basis of data given below predict which of the modern adsorption theory.
following gases shows least adsorption on a definite A
amount of charcoal? | | |
— O —O —O —
Gas CO 2 SO 2 CH 4 H2 | | |
(I) —O—O—O—
Critical temp./K 304 630 190 33 | | |
(a) CO2 (b) SO2 (c) CH4 (d) H2 B
13. In which of the following reactions heterogeneous | | |
catalysis is involved? A — O —O —O —
NO( g ) | | |
(i) 2SO2 ( g ) + O 2 ( g ) ¾¾¾¾ ® 2SO3 ( g ) (II) B — O—O—O—
| | |
EBD_8350
288 CHEMISTRY

25. Which of the following process is not responsible for the


| | |
A — O —O —O — presence of electric charge on the sol particles?
| | | (a) Electron capture by sol particles
(III) B —O—O—O— (b) Adsorption of ionic species from solution
| | | (c) Formation of Helmholtz electrical double layer
A (d) Absorption of ionic species from solution
| | | |
— O — O —O — Past Year MCQs
B
(IV) | | | 26. Which of the following statements is correct for the
— O—O—O—
spontaneous adsorption of a gas? [AIPMT 2014, C]
| | |
(a) DS is negative and, therefore, DH should be highly
| | | positive
A — O —O —O —
| + | | | (b) DS is negative and therefore, DH should be highly
(V) B — O—O—O— negative
| | | (c) DS is positive and, therefore, DH should be negative
(a) I ® II ® III ® IV ® V (b) I ® III ® II ® IV ® V (d) DS is positive and, therefore, DH should also be highly
(c) I ® III ® II ® V ® IV (d) I ® II ® III ® V ® IV positive
22. Which of the following process is responsible for the 27. Which property of colloids is not dependent on the charge
formation of delta at a place where rivers meet the sea? on colloidal particles? [AIPMT 2014, 2015, C]
(a) Emulsification (b) Colloid formation (a) Coagulation (b) Electrophoresis
(c) Coagulation (d) Peptisation (c) Electro - osmosis (d) Tyndall effect
23. Which of the following curves is in according with 28. Which one of the following characteristics is associated
Freundlich adsorption isotherm? with adsorption? [NEET 2016, C]
(a) DG is negative but DH and DS are positive
(b) DG, DH and DS all are negative
(c) DG and DH are negative but DS is positive
®

(d) DG and DS are negative but DH is positive


log x/m

log x/m

(a) (b) 29. Fog is colloidal solution of [NEET 2016, C]


log p ® log p ®
(a) Liquid in gas (b) Gas in liquid
(c) Solid in gas (d) Gas in gas
30. For a linear plot of log (x/m) versus log p in a Freundlich
®

adsorption isotherm, which of the following statements is


log x/m

log x/m

(c) (d) correct? (k and n are constants) [JEE M 2016, S]


(a) Only 1/n appears as the slope.
log p ® log p ® (b) log (1/n) appears as the intercept.
(c) Both k and 1/n appear in the slope term.
24. Which of the following phenomenon is applicable to the (d) 1/n appears as the intercept.
process shown in the figure? 31. Which one of the following statements is not correct?
[NEET 2017, C]
(a) The value of equilibrium constant is changed in the
presence of a catalyst in the reaction at equilibrium
Solution of raw sugar (b) Enzymes catalyse mainly bio-chemical reactions
yellowish brown (c) Coenzymes increase the catalytic activity of enzyme
(d) Catalyst does not initiate any reaction
32. The Tyndall effect is observed only when following
conditions are satisfied : [JEE M 2017, S]
Column of (1) The diameter of the dispersed particles is much smaller
animal
charcoal than the wavelength of the light used.
(2) The diameter of the dispersed particle is not much
smaller than the wavelength of the light used.
(3) The refractive indices of the dispersed phase and
Colourless
solution of
dispersion medium are almost similar in magnitude.
sugar (4) The refractive indices of the dispersed phase and
dispersion medium differ greatly in magnitude.
(a) (1) and (4) (b) (2) and (4)
(a) Absorption (b) Adsorption
(c) Coagulation (d) Emulsification (c) (1) and (3) (d) (2) and (3)
SURFACE CHEMISTRY 289

33. On which of the following properties does the coagulating


power of an ion depend? [NEET 2018, C]
(a) The magnitude of the charge on the ion alone
(b) Size of the ion alone
(c) The sign of charge on the ion alone
(d) Both magnitude and sign of the charge on the ion
34. Which mixture of the solutions will lead to the formation (a) p 2 (b) p 1/4 (c) p 1/2 (d) p
of negatively charged colloidal [AgI]I– sol. ? 36. The aerosol is a kind of colloid in which: [JEE M 2019, S]
[NEET 2019, C] (a) solid is dispersed in gas
(a) 50 mL of 1 M AgNO3 + 50 mL of 1.5 M KI (b) gas is dispersed in solid
(b) 50 mL of 1 M AgNO3 + 50 mL of 2 M KI (c) gas is dispersed in liquid
(c) 50 mL of 2 M AgNO3 + 50 mL of 1.5 M KI (d) liquid is dispersed in water
37. Measuring Zeta potential is useful in determining which
(d) 50 mL of 0.1 M AgNO3 + 50 mL of 0.l M KI
property of colloidal solution? [NEET 2020]
35. Adsorption of a gas follows Freundlich adsorption (a) Solubility
isotherm. In the given plot, x is the mass of the gas (b) Stability of the colloidal particles
x (c) Size of the colloidal particles
adsorbed on mass m of the adsorbent at pressure p. is
m (d) Viscosity
proportional to: [JEE M 2019, S]

Exercise 4 : Problem Solving Skill Enhancer MCQs


1. For the graph below, select correct order of temperature? (a) 50 mL of 0.1 M AgNO3 + 50 mL of 0.1 M KI
(b) 50 mL of 0.1 M AgNO3 + 50 mL of 0.2 M KI
Ps T1 (c) 50 mL of 0.2 M AgNO3 + 50 mL of 0.1 M KI
Extent of T2 (d) none of these
adsorption 6. A sol is prepared by addition of excess of AgNO3 solution
(x/m) T3 in KI solution. The charge likely to develop on colloidal
particle is:
(a) Positive (b) Negative
Pressure (P)
(c) No change (d) Both charges
(a) T1 > T2 > T3 (b) T2 > T3 > T1 7. The disperse phase in colloidal iron (III) hydroxide and
(c) T3 > T2 > T1 (d) T1 = T2 = T3 colloidal gold is positively and negatively charged,
2. Coagulation value of the electrolytes AlCl3 and NaCl for respectively. Which of the following statements is NOT
As2S3 sol are 0.093 and 52 respectively. How many times correct?
AlCl3 has greater coagulating power than NaCl ? (a) Coagulation in both sols can be brought about by
electrophoresis
(a) 930 (b) 520
(b) Mixing the sols has no effect
(c) 560 (d) None of these (c) Sodium sulphate solution causes coagulation in both
3. 3.6 gram of oxygen is adsorbed on 1.2 g of metal powder. sols
What volume of oxygen adsorbed per gram of the (d) Magnesium chloride solution coagulates, the gold
adsorbent at 1 atm and 273 K? sol more readily than the iron (III) hydroxide sol
(a) 0.19 Lg–1 (b) 1 Lg–1 8. A lot of scum formation was observed during washing
(c) 2.1 L g–1 (d) None of these with hard water. The washing powder could be
(a) C17 H 33COONa
4. The density of gold is 19 g/cm3. If 1.9 ´ 10 -4 g of gold is
dispersed in one litre of water to give a sol having spherical O
gold particles of radius 10 nm, then the number of gold ||
(b) CH 3 (CH 2 )11 O S - O - Na +
particles per mm3 of the sol will be : ||
12 14 O
(a) 1.9 ´10 (b) 6.3 ´ 10 O
10 ||
(c) 6.3 ´10 (d) 2.4 ´ 10 6
(c) CH 3 (CH 2 )11 - - S - O - Na +
5. On adding AgNO3 solution into KI solution, a negatively ||
charged colloidal sol is obtained when they are in : (d) none of these O
EBD_8350
290 CHEMISTRY

9. Flocculation value of BaCl2 is much less than that of KCl (a) 0.6 g (b) 0.3 g
for sol ‘A’ and flocculation value of Na2SO4 is much less (c) 1.2 g (d) None of these
than that of NaBr for sol ‘B’. The correct statement among
12. A colloidal solution is subjected to an electric field. The
the following is :
particles move towards anode. The coagulation of same
(a) Both the sols ‘A’ and ‘B’ are negatively charged.
sol is studied using NaCl, BaCl2 and AlCl3 solutions. The
(b) Sol ‘A’ is positively charged arid sol ‘B’ is negatively
charged. order of their coagulation power should be –
(c) Both the sols ‘A’ and ‘B’ are positively charged. (a) NaCl > BaCl2 > AlCl3 (b) BaCl2 > AlCl3 > NaCl
(d) Sol ‘A’ is negatively charged and sol ‘B’ is positively (c) AlCl3 > BaCl2 > NaCl (d) BaCl2 > NaCl > AlCl3
charged. 13. Which of the following feature of catalysts is described
10. Which of the following adsorption isotherms represents in reactions given below?
the absorption of a gas by a solid involving multilayers of
Cu / ZnO-Cr O
formation? (Ps = saturation pressure) (i) CO ( g ) + 2H2 ( g ) ¾¾¾¾¾¾¾ 3 ( )
2 3 ® CH OH g

CO ( g ) + H 2 ( g ) ¾¾® HCHO ( g )
Cu
Ps Ps (ii)

(iii) CO ( g ) + 3H2 ( g ) ¾¾
Ni
® CH4 ( g ) + H2O ( g )
x x
(a) (b) (a) Activity (b) Selectivity
m m
(c) Catalytic promoter (d) Catalytic poison
P P
14. Gold numbers of protective colloids A, B, C and D are 0.50,
0.01, 0.10 and 0.005, respectively. The correct order of their
Ps Ps
protective powers is
x x (a) D < A < C < B (b) C < B < D < A
(c) (d) (c) A < C < B < D (d) B < D < A < C
m m
15. Which one of the following impurities present in colloidal
P P solution cannot be removed by electrodialysis?
11. 100 mL of 0.6 M acetic acid is shaken with 2 g activated (a) Sodium chloride (b) Potassium sulphate
carbon. The final concentration of the solution after (c) Urea (d) Calcium chloride
adsorption is 0.5 M. What is the amount of acetic acid
adsorbed per gram of carbon?

ANSW ER KEY
Exercise 1 : NCERT Based Topic-wise MCQs
1 (a) 12 (c) 23 (c) 34 (d) 45 (c) 56 (c) 67 (c) 78 (d) 89 (a) 100 (a)
2 (a) 13 (d) 24 (d) 35 (d) 46 (b) 57 (a) 68 (a) 79 (d) 90 (d) 101 (a)
3 (a) 14 (d) 25 (a) 36 (b) 47 (d) 58 (d) 69 (a) 80 (c) 91 (b) 102 (b)
4 (d) 15 (b) 26 (c) 37 (c) 48 (d) 59 (b) 70 (b) 81 (c) 92 (c) 103 (d)
5 (b) 16 (a) 27 (b) 38 (a) 49 (c) 60 (c) 71 (c) 82 (c) 93 (d) 104 (c)
6 (d) 17 (a) 28 (c) 39 (a) 50 (b) 61 (a) 72 (b) 83 (c) 94 (a) 105 (c)
7 (d) 18 (c) 29 (d) 40 (a) 51 (a) 62 (b) 73 (a) 84 (c) 95 (d)
8 (a) 19 (c) 30 (c) 41 (d) 52 (c) 63 (b) 74 (b) 85 (d) 96 (c)
9 (c) 20 (c) 31 (d) 42 (c) 53 (d) 64 (d) 75 (c) 86 (c) 97 (a)
10 (b) 21 (d) 32 (d) 43 (a) 54 (d) 65 (a) 76 (a) 87 (b) 98 (d)
11 (b) 22 (d) 33 (c) 44 (d) 55 (b) 66 (a) 77 (d) 88 (d) 99 (b)
Exercise 2 : Numeric/Integer Answer Questions
1 (2) 2 (2) 3 (1) 4 (4) 5 (1) 6 (4) 7 (3) 8 (4) 9 (4) 10 (25)
Exercise 3 : NCERT Exemplar & Past Year MCQs
1 (c) 5 (b) 9 (b) 13 (a) 17 (d) 21 (b) 25 (d) 29 (a) 33 (d) 37 (b)
2 (b) 6 (a) 10 (a) 14 (b) 18 (b) 22 (c) 26 (b) 30 (a) 34 (b)
3 (d) 7 (a) 11 (b) 15 (d) 19 (c) 23 (c) 27 (d) 31 (a) 35 (c)
4 (c) 8 (d) 12 (d) 16 (b) 20 (d) 24 (b) 28 (b) 32 (b) 36 (a)
Exercise 4 : Problem Solving Skill Enhancer MCQs
1 (c) 3 (c) 5 (b) 7 (b) 9 (d) 11 (b) 13 (b) 15 (c)
2 (c) 4 (d) 6 (a) 8 (a) 10 (a) 12 (c) 14 (c)
General Principles and
20 Processes of Isolation
of Elements
Trend Buster NEET & JEE Main

Number of Questions from 2020-15 7 5 Minimum one question has been


Weightage 2.2% 2.4% asked in NEET every year.

The most Important Concepts that Cover Maximum number of Questions asked in past 6 years.

Occurance of Metals 2 3
Thermodynamic Principles of Metallurgy 2 —

Less Important Concepts that Cover 1 or 2 Questions asked in past 6 years.


NEET JEE
Concentration of Ores — 1
Electrochemical Principles of Mettallurgy 1 —
Oxidation Reduction — 1
Refining 1 —
Extraction of Crude Metal from Concentrated 1 —
Ore and Uses of Al, Cu, Zn, & Fe
NEET JEE

2020 Thermodynamic Purest form of Iron / 1 Average 1 Easy


Principles of Metallurgy / Extraction of crude metal
Extraction of crude metal from concentrated ore
from concentrated ore
2019 Occerance of Metals Ores of Different Metals 1 Easy 2 Easy
2018 Thermodynamic Ellingham Diagram 1 Average — —
Principles of Metallurgy
2017 Oxidation Reduction Extraction of Gold & Silver 1 Average — —
2016 Refining Process / Different Extraction Process & 1 Average 1 Easy
Concentration of Ores Zone Refining / Froth Floatation
Method
2015 Occerance of Metal / Stability of Nitrate Ore of Metal / 2 Average 1 Average
Thermodynamic Copper Extraction / Halt
Principles of Metallurgy / Heroults Process
Electrochemical
Principles of Mettalurgy
EBD_8350
292 CHEMISTRY
GENERAL PRINCIPLES AND PROCESSES OF ISOLATION OF ELEMENTS 293
EBD_8350
294 CHEMISTRY

Problem Solving Tips/ Tricks/ Points to Remember

4 % of Carbon in different type of Iron 4 Froth stabiliser : Which stabilise the froth e.g. cresoles
Name % of C and aniline.
Wrought iron/Meleable 0.1 to 0.25 4 Depressants : Depressants prevent the formation of froth
Steel 0.25 to 2.0 eg NaCN, when added to ore containing ZnS and PbS
Pig Iron 2.3 to 4.6
Cast Iron 2.6 to 4.3 form a complex with ZnS as Na 2 [Zn (CN )4 ] and prevent
4 Sulphide ores are concentrated by froth floatation method. it from forming froth. PbS is then easily separated from
4 Oxide and carbonate ores are concentrated by ZnS.
gravitational method. 4 Amalgamation Process : Ag and Au are obtained by
4 Liquation process is used for the concentration of ores leaching process using solution of KCN or NaCN to form
which are having lesser melting point than impurities argento cyanide or aurocyanide. Ag and Au is precipitated
4 Matte is a mixture of Cu2S containing little FeS. by adding Zn dust.
4 Zone refining is used to obtain metals of very high purity. 2Na[Ag(CN )2 ] + Zn ® Na 2 [Zn(CN )4 ] + 2Ag
4 Cupellation : This is the method of purifying silver
2K[Au(CN )2 ] + Zn ® K 2 [Zn(CN )4 ]+ 2Au
containing lead as impurity. Impure silver is heated in a Soluble complex
shallow (Cupel) which is made of bone ash under blast of
air. 4 Bessemerisation :The oxidation of impurities by passing
4 Desilverisation of lead: the hot blast of air through molten metal in bessemer
(a) Parke’s process : Lead containing silver is melted converter is called bessemerisation . Pig iron and copper
in iron pots and 1% Zn is added then cooled. Zn - are purified by this method.
Ag alloy solidifies and being light floats over molten
4 Basic concepts of thermodynamics is quite useful in
lead and removed.
selecting a suitable reducing agent for a particular oxide.
(b) Pattison’s process : Lead containing less than 2.5%
In the oxidation reduction reaction, DfG – should be
of Ag melts at lower temperature than lead. Alloy of
negative
Pb-Ag containing more lead is melted then allowed
to cool slowly, pure lead separates. DfG – = DfH – – TDfS–
4 Sintering :The conversion of small pieces of substances The concept is graphically displayed by – DfG– vs T (in K)
into larger one by partial fusion is known as sintering. plots, i.e. Ellingham diagram.
4 The three most common elements in the universe by mass
4 Goldschmidt aluminothermic process is used in case of
fraction are: Hydrogen, Helium and then oxygen.
those metals that have very high melting points and are
4 The four most abundant elements in the earth’s crust are:
to be extracted from their oxides.
oxygen, silicon, aluminium and then iron.
4 Furnaces : 4 Presence of impurity in certain cases is useful e.g. The
a. Reverberatory furnace: Used for roasting, slag obtained from gangue, helps in preventing the
smelting etc. oxidation of molten metal in a blast furnace where it floats
b. Muffle furnace: Used when high temperature is
over the molten metal.
required.
c. Blast furnace: Temperature at bottom is 1500 °C 4 Copper is extracted by hydrometallurgy from low grade ores.
and at the top is 200–300 °C. The solution containing Cu2+ is treated with iron or H2:
Cu2+ (aq) + H2(g) ® Cu(s) + 2H+ (aq)
4 Relative Abundance : Abundance of elements in the
4 The process of extracting a metal in the state of fusion is
earth’s crust (by weight)
called smelting. The metallic oxide is reduced by carbon
O > Si > Al > Fe > Ca > Na > K > Mg > H > Ti > Cl > F
and the metal may be obtained in the molten state or
4 Collectors : Which increase the non wettability of ore
vapours which are then condensed.
particles e.g. pine oil, xanthates and fatty acids.
GENERAL PRINCIPLES AND PROCESSES OF ISOLATION OF ELEMENTS 295

Exercise 1 : NCERT Based Topic-wise MCQs


(c) Magnesium Cassiterite
Topic 1: Occurrence of Metals and Concentration of Ores (d) Tin Azurite
1. Which one of the following is an ore of silver ? 17. Pyrolusite is a/an
(a) Argentite(b) Stibnite (c) Haematite(d) Bauxite (a) oxide ore (b) sulphide ore
2. Cinnabar is an ore of (c) carbide ore (d) Not an ore
(a) Hg (b) Cu (c) Pb (d) Zn 18. Sulphide ores of metals are usually concentrated by froth
3. An example of an oxide ore is flotation process. Which one of the following sulphide
(a) Bauxite (b) Malachite ores offer an exception and is concentrated by chemical
(c) Zinc blende (d) Feldspar leaching?
4. The natural materials from which an element can be (a) Galena (b) Copper pyrite
extracted economically are called (c) Sphalerite (d) Argentite
(a) ores (b) minerals 19. Composition of azurite mineral is
(c) gangue (d) None of these (a) CuCO3CuO (b) Cu(HCO3)2. Cu(OH)2
5. The most abundant metal on the surface of the earth is (c) 2CuCO3.Cu(OH)2 (d) CuCO3. 2Cu(OH)2
(a) Fe (b) Al (c) Ca (d) Na 20. The most abundant element in the earth’s crust (by weight) is
6. Which of the following is an ore of tin ? (a) Si (b) Al (c) O (d) Fe
(a) Carborundum (b) Epsomite 21. Which of the following pair is incorrectly matched ?
(c) Cassiterite (d) Spodumene (a) Magnetite – Fe3O4 (b) Copper glance – Cu2S
7. Which of the following is chalcopyrite? (c) Calamine – ZnCO3 (d) Zincite – ZnS
(a) CuFeS2 (b) FeS2 22. Cassiterite is concentrated by
(c) KMgCl3.6H2O (d) Al2O3.2H2O (a) levigation
(b) electromagnetic separation
8. Which one of the following is not a sulphide ore? (c) floatation (d) liquefaction
(a) Magnetite (b) Iron pyrites 23. Which one of the following ores is not concentrated by
(c) Copper glance (d) Sphalerite froth floatation process?
9. Which one of the following ores is best concentrated by (a) Copper pyrites (b) Pyrargyrite
froth-flotation method ? (c) Pyrolusite (d) Zinc blende
(a) Galena (b) Cassiterite 24. In froth flotation process, many chemicals (frother,
(c) Magnetite (d) Malachite collector, activator, and depressant) are used. Which of
10. While extracting an element from its ore, the ore is the following is a frother ?
grounded and leached with dil. potassium cyanide solution (a) CuSO4 (b) NaCN+ alkali
to form the soluble product potassium argento cyanide. (c) Pine oil (d) Potassium xanthate
The element is 25. Hydro-metallurgical process of extraction of metals is based on
(a) Lead (b) Chromium (a) complex formation (b) hydrolysis
(c) Manganese (d) Silver (c) dehydration (d) dehydrogenation
11. The method of concentrating the ore which makes use of 26. Assertion : Leaching is a process of reduction.
the difference in density between ore and impurities is Reason : Leaching involves treatment of the ore with a
called suitable reagent so as to make it soluble while impurities
(a) levigation (b) leaching remain insoluble.
(c) magnetic separation (d) liquifaction (a) Assertion is correct, reason is correct; reason is a
12. Leaching is a process of correct explanation for assertion.
(a) reduction (b) concentration (b) Assertion is correct, reason is correct; reason is not
(c) refining (d) oxidation a correct explanation for assertion
13. Which of the following metal is leached by cyanide process ? (c) Assertion is correct, reason is incorrect
(a) Ag (b) Na (c) Al (d) Cu (d) Assertion is incorrect, reason is correct.
14. Froth flotation process is based on 27. Which ore contains both iron and copper?
(a) wetting properties of ore particle (a) Cuprite (b) Chalcocite
(b) specific gravity of ore particles (c) Chalcopyrite (d) Malachite
(c) magnetic properties of ore particles 28. Assertion : Levigation is used for the separation of oxide
(d) electrical properties of ore particles ores from impurities.
15. Copper can be extracted from Reason : Ore particles are removed by washing in a current
(a) Kupfernical (b) Dolomite of water.
(c) Malachite (d) Galena (a) Assertion is correct, reason is correct; reason is a
16. Which of the following metal is correctly matched with its correct explanation for assertion.
ore? (b) Assertion is correct, reason is correct; reason is not
Metal Ore a correct explanation for assertion
(a) Zinc Calamine (c) Assertion is correct, reason is incorrect
(b) Silver Ilmenite (d) Assertion is incorrect, reason is correct.
EBD_8350
296 CHEMISTRY

29. Which one of the following does not occur as sulphide 41. Which of the following condition favours the reduction
ore? of a metal oxide to metal?
(a) Zn (b) Cr (c) Ag (d) Fe (a) DH = +ve, TDS = + ve at low temperature
(b) DH = +ve, TDS = – ve at any temperature
Topic 2 : Extraction of Crude Metal from Concentrated Ore (c) DH = –ve, TDS = – ve at high temperature
(d) DH = –ve, TDS = + ve at any temperature
30. Main function of roasting is 42. Ellingham diagram normally consists of plots of
(a) to remove volatile substances (a) DSº vs T (b) D fGº vs DSº
(b) oxidation (c) DGº vs T (d) DHº vs DT
(c) reduction 43. Furnaces are lined with calcium oxide because
(d) slag formation (a) it gives off oxygen on heating
31. Roasting is generally done in case of the (b) it gives strong light on heating
(a) oxide ores (b) silicate ores (c) it is refractory and basic
(c) sulphide ores (d) carbonate ores (d) it is not affected by acids
32. Process followed before reduction of carbonate ore is 44. The following reactions take place in the blast furnace in
(a) calcination (b) roasting the preparation of impure iron. Identify the reaction
(c) liquation (d) polling pertaining to the formation of the slag.
33. When a metal is to be extracted from its ore and the gangue (a) Fe2O3(s) + 3 CO(g) ®2 Fe (l) + 3 CO2 (g)
associated with the ore is silica, then (b) CaCO3 (s) ® CaO (s) + CO2 (g)
(a) an acidic flux is needed (c) CaO (s) + SiO2(s) ® CaSiO3(s)
(b) a basic flux is needed (d) 2C(s) + O2 (g) ®2 CO(g)
(c) both acidic and basic fluxes are needed 45. Refractory materials are generally used in furnaces because
(d) Neither of them is needed (a) they possess great structural strength
34. Which of the following reactions is an example for (b) they can withstand high temperature
calcination process ? (c) they are chemically inert
(a) 2Ag + 2HCl + (O ) ® 2AgCl + H 2 O (d) they do not require replacement
46. Blister copper is
(b) 2Zn + O 2 ® 2 ZnO (a) Impure Cu (b) Cu alloy
(c) 2ZnS + 3O 2 ® 2ZnO + 2SO 2 (c) Pure Cu (d) Cu having 1% impurity
47. In the blast furnace iron oxide is reduced by
(d) MgCO3 ® MgO + CO 2 (a) silica (b) CO
35. 2CuFeS2 + O 2 ¾¾ ® Cu 2 S + 2FeS + SO 2 (c) carbon (d) limestone
Which process of metallurgy of copper is represented by 48. Extraction of zinc from zinc blende is achieved by
above equation? (a) electrolytic reduction
(a) Concentration (b) Roasting (b) roasting followed by reduction with carbon
(c) Reduction (d) Purification (c) roasting followed by reduction with another metal
36. Function of the flux added during smelting is (d) roasting followed by self-reduction
(a) to make ore porous (b) to remove gangue 49. Aluminothermic process is used for the extraction of metals,
(c) to precipitate slag (d) to make reduction easier whose oxides are
(a) fusible
37. Calcination is the process in which :
(b) not easily reduced by carbon
(a) ore is heated above its melting point to expel H2O or
(c) not easily reduced by hydrogen
CO2 or SO2 (d) strongly basic
(b) ore is heated below its melting point to expel volatile 50. The furnace used to prepare commercial iron is lined with
impurities which of the following ?
(c) ore is heated above its melting point to remove S, As (a) Haematite (b) Magnetite
and Sb as SO2 ,As2O3 and Sb2O3 respectively (c) Ironpyrites (d) Both (a) and (b)
(d) ore is heated below its melting point to expel H2O or 51. Which form of the iron contains 4% carbon ?
CO2 (a) Cast iron (b) Pig iron
38. Which of the following fluxes is used to remove acidic (c) Wrought iron (d) Both (a) and (b)
impurities in metallurgical process? 52. Which of the following reaction takes place in blast furnace
(a) Silica (b) Lime stone during extraction of copper ?
(c) Sodium chloride (d) Sodium carbonate (a) 2Cu 2S + 3O 2 ¾¾ ® 2Cu 2O + 2SO 2
39. After partial roasting, the sulphide of copper is reduced
(b) 2FeS + 3O 2 ¾¾ ® 2FeO + 2SO 2
by
(a) cyanide process (b) electrolysis (c) 2Cu 2 O + Cu 2S ¾¾ ® 6Cu + SO 2
(c) reduction with carbon (d) self reduction (d) All of these
53. Which of the following statement(s) is/are correct ?
Topic 3: Thermodynamic Principles of Metallurgy (i) Cast iron is used in the manufacture of railway sleepers
(ii) Wrought iron is used in the manufacture of anchors,
40. According to Ellingham diagram, the oxidation reaction of bolts, chains etc.
carbon to carbon monoxide may be used to reduce which (iii) Nickel steel is used in making pendulums.
one of the following oxides at the lowest temperature ? (a) Only (i) (b) (i) and (ii)
(a) Al2O3 (b) Cu2O (c) MgO (d) ZnO (c) (i), (ii) and (iii) (d) Only (iii)
GENERAL PRINCIPLES AND PROCESSES OF ISOLATION OF ELEMENTS 297

54. Match the columns (iii) Diagrams similar to Ellingham can be constructed for
Column - I Column - II sulphides and halides which clearly indicates why
D
(A) Fe 2 O3 .xH 2O(s) ¾¾ (p) Slag formation reduction of MxS is difficult in comparison to MxO.
®
(iv) Ellingham diagrams predicts the tendency of
Fe2O3 (s) + xH 2O(g) reduction with a reducing agent and kinetics of the
(B) FeO + SiO2 ¾¾® FeSiO3 (q) Reduction of reduction process.
iron oxide (a) (i), (ii) and (iii) (b) (i) and (iii)
(C) Discharge gas produced (r) Calcination (c) (i), (ii) and (iv) (d) (ii) and (iv)
during this process is utilised 61. D¦G– vs T plot in the Ellingham diagram slopes downward
in manufacture of H2SO4. for the reaction
(D) Fe2 O3 + 3C ¾¾ ® 2Fe + 3CO (s) Roasting 1
(a) A– (r), B – (p), C – (s), D – (q) (a) Mg + O2 ® MgO
2
(b) A– (p), B – (r), C – (s), D – (q)
(c) A– (r), B – (s), C – (p), D – (q) 1
(b) 2Ag + O 2 ® Ag 2O
(d) A– (r), B – (p), C – (q), D – (s) 2
55. Assertion : Copper obtained after bessemerization is
1
known as blister copper. (c) C + O2 ® CO
Reason : Blisters are produced on the surface of the metal 2
due to escaping of dissolved SO2. 1
(a) Assertion is correct, reason is correct; reason is a (d) CO + O 2 ® CO 2
correct explanation for assertion. 2
(b) Assertion is correct, reason is correct; reason is not 62. Which of the following statements, about the advantage
a correct explanation for assertion of roasting of sulphide ore before reduction is not true?
(c) Assertion is correct, reason is incorrect (a) The DG of of the sulphide is greater than those for
(d) Assertion is incorrect, reason is correct. CS2 and H2S.
56. Consider the following reactions at 1000°C (b) The DGof is negative for roasting of sulphide ore to
1 oxide.
A. Zn(s) + O (g) ® ZnO(s); DG° = -360kJ mol-1
2 (c) Roasting of the sulphide to the oxide is thermo-
2
1 dynamically feasible.
B. C(gr) + O 2 (g) ® CO(g); DG° = -460 kJ mol –1 (d) Carbon and hydrogen are suitable reducing agents
2 for reduction of metal sulphides.
Choose the correct statement at 1000°C
(a) zinc can be oxidised by carbon monoxide. 63. The value of Df Gº for formation of Cr2O3 is – 540 kJmol–1
(b) zinc oxide can be reduced by graphite and that of Al2O3 is – 827 kJ mol–1. What is the value of
(c) carbon monoxide can be reduced by zinc. DrG° for the reaction?
(d) both statements (a) and (b) are true 4 2 2
57. Which of the following statement is not correct about Al(s) + Cr2 O 3 (s) ® Al 2 O 3 (s) + 4 Cr(s).
3 3 3 3
Ellingham diagram?
(a) DG increases with an increase in temperature (a) – 191 kJ mol–1 (b) –287 kJ mol–1
(b) It consists of plots of DfGº vs T for formation of oxides (c) + 191 kJ mol–1 (d) +287 kJ mol–1
(c) A coupling reaction can be well expressed by this 64. Before introducing FeO in blast furnace , it is converted to
diagram Fe2O3 by roasting so that
(d) It expresses the kinetics of the reduction process (a) it may not be removed as slag with silica
58. A coupled reaction takes place as follow– (b) it may not evaporate in the furnace
A + B ––® C + D DGº = + x kJ (c) presence of it may increase the m.pt. of charge
D + E ––® F DGº = – y kJ (d) None of these.
for the spontaneity of reaction A + B + E –––® C + F, 65. Which of the following statements regarding metallurgy
which of the following is correct? of iron is incorrect ?
(a) 2x = y (b) x < y (a) Reaction Fe3O 4 + 4CO ¾¾ ® 3Fe + 4CO 2 belongs
(c) x > y (d) x = (y) × TDS to lower temperature range (500 – 800K) of the blast
59. In the extraction of Cu, the metal is formed in the bessemer furnace.
converter due to the reaction : (b) Reaction FeO + CO ¾¾ ® Fe + CO 2 belongs to
(a) Cu2S + 2Cu2O ¾¾ ® 6Cu + SO2 higher temperature range (900 – 1500K) of the blast
(b) Cu2S ¾¾ ® 2Cu + S furnace.
(c) Fe + Cu2O ¾¾ ® 2Cu + FeO (c) The iron obtained from blast furnace is cast iron with
(d) 2Cu2O ¾¾ ® 4Cu + O2 3% carbon.
60. Which of the following statements related to Ellingham (d) For reduction of iron oxide to occur DG of the couple
diagrams are correct ? of following reactions should be negative
(i) It provides a sound basis for the choice of reducing 1
agent in the reduction of oxides. FeO(s) ¾¾ ® Fe(s) + O 2 (g)
(ii) Each Ellingham plot is represented by a straight line 2
1
untill unless there is some change in phase i.e. solid C(s) + O2 (g) ¾¾ ® CO(g)
® liquid, liquid ® gas and gas ® liquid occurs. 2
EBD_8350
298 CHEMISTRY

Topic 4: Electrochemical Principles of Metallurgy Topic 5: Refining, uses of Aluminium, Copper, Zinc and Iron
and Oxidation Reduction
75. The process of zone refining is used in the purification of
66. Electrometallurgical process is used to extract (a) Si (b) Al (c) Ag (d) Cu
(a) Fe (b) Pb (c) Na (d) Ag 76. van Arkel method of purification of metals involves
67. The electrolytic method of reduction is employed for the
converting the metal to a
preparation of metals that
(a) are weakly electropositive (a) volatile stable compound
(b) are moderately electropositive (b) volatile unstable compound
(c) are strongly electropositive (c) non volatile stable compound
(d) form oxides (d) None of the above
68. Aluminium is extracted from alumina (Al2O3) by electrolysis 77. The method not used in metallurgy to refine the impure
of a molten mixture of metal is
(a) Al2O3 + HF + NaAlF4 (a) Mond’s process (b) Van–Arkel process
(b) Al2O3 + CaF2 + NaAlF4 (c) Amalgamation process (d) Liquation
(c) Al2O3 + Na3AlF6 + CaF2 78. Which of the following pairs of metals is purified by van
(d) Al2O3 + KF + Na3AlF6 Arkel method ?
69. In the extraction of aluminium by Hall-Heroult process,
(a) Ga and In (b) Zr and Ti
purified Al2O3 is mixed with CaF2 to
(i) lower the melting point of Al2O3. (c) Ag and Au (d) Ni and Fe
(ii) increase the conductivity of molten mixture. 79. What is anode mud?
(iii) reduce Al3+ into Al(s). (a) Fan of anode
(iv) acts as catalyst. (b) Metal of anode
(a) (i) and (ii) (b) (i), (ii) and (iii) (c) Impurities collected at anode in electrolysis during
(c) (iii) and (iv) (d) (ii), (iii) and (iv) purification of metals
70. Match the columns. (d) All of these
Column-I Column-II 80. Which of the following metal is used in the manufacture of
(A) Blisterred Cu (p) Aluminium dye-stuffs and paints ?
(B) Blast furnace (q) 2Cu2O + Cu2S ® 6Cu + SO2 (a) Copper (b) Zinc
(C) Reverberatory (r) Iron (c) Aluminium (d) Magnesium
furnace
81. Match the columns.
(D) Hall-Heroult (s) FeO + SiO2 ® FeSiO3
Column-I Column-II
process (t) 2Cu2S + 3O2 ® 2Cu2O + 2SO2
(a) A – (q), B – (r), C – (s), D – (p) (A) Coloured bands (p) Zone refining
(b) A – (p), B – (q), C – (r), D – (t) (B) Impure metal to volatile (q) Fractional
(c) A – (t), B – (s), C – (r), D – (q) complex distillation
(d) A – (s), B – (t), C – (r), D – (q) (C) Purification of Ge and Si (r) Mond Process
71. Assertion : Zinc can be used while copper cannot be used (D) Purification of mercury (s) Chromatography
in the recovery of Ag from the complex [Ag(CN)2]–. (t) Liquation
Reason : Zinc is a powerful reducing agent than copper. (a) A – (p), B – (q), C – (s), D – (t)
(a) Assertion is correct, reason is correct; reason is a (b) A – (s), B – (r), C – (p), D – (q)
correct explanation for assertion. (c) A – (r), B – (s), C – (p), D – (q)
(b) Assertion is correct, reason is correct; reason is not (d) A – (t), B – (s), C – (r), D – (q)
a correct explanation for assertion 82. In electro-refining of metal, the impure metal is used to
(c) Assertion is correct, reason is incorrect make the anode and a strip of pure metal as the cathode,
(d) Assertion is incorrect, reason is correct. during the electrolysis of an aqueous solution of a complex
72. In the cyanide extraction process of silver from argentite metal salt. This method cannot be used for refining of
ore, the oxidising and reducing agents used are (a) Silver (b) Copper
(a) O2 and CO respectively (c) Aluminium (d) Sodium
(b) O2 and Zn dust respectively 83. During the process of electrolytic refining of copper, some
(c) HNO3 and Zn dust respectively metals present as impurity settle as ‘anode mud’. These
(d) HNO3 and CO respectively are
73. Extraction of which of the following is based on oxidation? (a) Fe and Ni (b) Ag and Au
(a) Highly reactive metals (c) Pb and Zn (d) Sn and Ag
(b) Moderately reactive metals 84. If the impurities in a metal has a greater affinity for oxygen
(c) Non-metals and is more easily oxidised than the metal, then the
(d) Both (a) and (c) purification of metal may be carried out by
74. In Hall-Heroult process, how much carbon anode is burnt (a) distillation (b) zone refining
away to produce each 1kg of aluminium ? (c) electrolytic refining (d) cupellation
(a) 0.3 kg (b) 0.5 kg (c) 1 kg (d) 0.1 kg
GENERAL PRINCIPLES AND PROCESSES OF ISOLATION OF ELEMENTS 299

85. Match the columns 88. Which of the following statements regarding electrolytic
Column-I Column-II refining of copper is incorrect ?
(A) Cyanide process (p) Ultrapure Ge (a) In this process anode is made up of impure copper
(B) Floatation process (q) Pine oil and pure copper strips are taken as cathode.
(C) Electrolytic reduction (r) Extraction of Al
(b) Acidic or basic solution of copper sulphate is used
(D) Zone refining (s) Extraction of Au
(a) A – (r), B – (p), C – (s), D – (q) as electrolyte
(b) A – (s), B – (q), C – (r), D – (p) (c) Antimony, tellurium, silver and gold are some of the
(c) A – (r), B – (q), C – (s), D – (p) metals deposits as anode mud during this process
(d) A – (s), B – (p), C – (r), D – (q) (d) Zinc can be also refined by electrolytic refining
86. Germanium of very high purity is obtained by method.
(a) liquation 89. Which of the following is incorrectly matched ?
(b) vapour phase refining Metal Uses
(c) distillation (a) Wrought iron Casting stoves, gutter
(d) zone refining
pipes, toys etc.
87. Match the columns
Column-I Column-II (b) Copper Coinage alloy
(A) This metal is used in (p) Zinc (c) Aluminium Extraction of chromium and
extraction of chromium manganese
and manganese. (d) Nickel steel Measuring tapes
(B) Common metal in brass (q) Aluminium 90. Assertion : Lead, tin and bismuth are purified by liquation
and bronze. method.
(C) Common metal in brass (r) Copper Reason : Lead, tin and bismuth have low m.p. as compared
and german silver. to impurities.
(D) Substance used in making (s) Stainless steel
(a) Assertion is correct, reason is correct; reason is a
cycles, automobiles,
utensils, etc. correct explanation for assertion.
(a) A – (q), B – (r), C – (p), D – (s) (b) Assertion is correct, reason is correct; reason is not
(b) A – (r), B – (q), C – (p), D – (s) a correct explanation for assertion
(c) A – (q), B – (p), C – (r), D – (s) (c) Assertion is correct, reason is incorrect
(d) A – (q), B – (r), C – (s), D – (p) (d) Assertion is incorrect, reason is correct.

NCERT Exemplar MCQs 5. In the metallurgy of aluminium ..............


(a) Al3+ is oxidised to Al (s).
1. In the extraction of chlorine by electrolysis of brine ........... (b) graphite anode is oxidised to carbon monoxide and
(a) oxidation of Cl– ion to chlorine gas occurs carbon dioxide.
(b) reduction of Cl– ion to chlorine gas occurs (c) oxidation state of oxygen changes in the reaction at
(c) for overall reaction D G s has negative value anode.
(d) a displacement reaction takes place (d) oxidation state of oxygen changes in the overall
2. Which of the following reactions is an example of auto- reaction involved in the process.
reduction? 6. Electrolytic refining is used to purify which of the following
metals?
(a) Fe3 O 4 + 4CO ¾¾ ® 3Fe + 4CO 2 (a) Cu and Zn (b) Ge and Si
(b) Cu 2 O + C ¾¾ ® 2Cu + CO (c) Zr and Ti (d) Zn and Hg
7. Extraction of gold and silver involves leaching the metal
(c) Cu 2+ (aq) + Fe(s) ¾¾
® Cu(s) + Fe 2 + (aq) with CN – ion. The metal is recovered by ..........
1 1 (a) displacement of metal by some other metal from the
(d) Cu 2 O + Cu 2S ¾¾ ® 3Cu + SO 2 complex ion.
2 2 (b) roasting of metal complex.
3. A number of elements are available in earth's crust but (c) calcination followed by roasting.
most abundant elements are ................ (d) thermal decomposition of metal complex.
(a) Al and Fe (b) Al and Cu 8. When copper ore is mixed with silica in a reverberatory
(c) Fe and Cu (d) Cu and Ag furnace copper matte is produced. The copper matte
4. In the extraction of copper from its sulphide ore, the metal contains
is formed by the reduction of Cu2O with (a) sulphides of copper (II) and iron (II)
(b) sulphides of copper (II) and iron (III)
(a) FeS (b) CO
(c) sulphides of copper (I) and iron (II)
(c) Cu2S (d) SO2 (d) sulphides of copper (I) and iron (III)
EBD_8350
300 CHEMISTRY

9. Zone refining is based on the principle that .................. Past Year MCQs
(a) impurities of low boiling metals can be separated by
distillation. 14. The metal that cannot be obtained by electrolysis of an
(b) impurities are more soluble in molten metal than in aqueous solution of its salts is: [JEE M 2014, C]
solid metal. (a) Ag (b) Ca (c) Cu (d) Cr
(c) different components of a mixture are differently 15. "Metals are usually not found as nitrates in their ores".
adsorbed on an adsorbent. Out of the following two ((i) and (ii)) reasons which is/are
(d) vapours of volatile compound can be decomposed in true for the above observation ? [AIPMT 2015, C]
pure metal. (i) Metal nitrates are highly unstable.
10. Brine is electrolysed by using inert electrodes. The reaction (ii) Metal nitrates are highly soluble in water.
at anode is ................. (a) (i) and (ii) are false
(b) (i) is false but (ii) is true
1
(a) Cl - ( aq ) ¾¾
® Cl2 ( g ) + e - ; E ºcell = 1.36 V (c) (i) is true but (ii) is false
2 (d) (i) and (ii) are true
16. In the extraction of copper from its sulphide ore, the metal
(b) 2H 2 O ( l ) ¾¾
® O 2 ( g ) + 4H + + 4e - ; E ºcell = 1.23 V
finally obtained by the reduction of cuprous oxide with :
[AIPMT 2015 RS, C]
(c) Na + ( aq ) + e - ¾¾
® Na ( s ) ; E ºcell = 2.71 V
(a) iron (II) sulphide (b) carbon monoxide
1 (c) copper (I) sulphide (d) sulphur dioxide
(d) H + ( aq ) + e - ¾¾
® H 2 ( g ) ; E ºcell = 0.00 V 17. In the context of the Hall - Heroult process for the extraction
2
Direction (Q. No. 11-13) Answer the questions on the basis of of Al, which of the following statements is false ?
figure. [JEE M 2015, C]
(a) Al3+ is reduced at the cathode to form Al
–200
. . eO (b) Na3AlF6 serves as the electrolyte
. A D. ®
2F
(c) CO and CO2 are produced in this process
DG°/kJ mol of O2 ®

–300 O2

. ... E e+
2 F C + O2 ® CO2
(d) Al2O3 is mixed with CaF2 which lowers the melting
point of the mixture and brings conductivity
. B
–400
18. Match items of Column I with the items of Column II and
–1

.
2C
–500 O2 +O assign the correct code : [NEET 2016, A]
+ 2
O O 2 ®
2 C 2C 2C Column-I Column-II
–600 ® O
(A) Cyanide process (i) Ultrapure Ge
–700 (B) Froth floatation process (ii) Dressing of ZnS
0 400 800 1200 1600 2000 (C) Electrolytic reduction (iii) Extraction of Al
Temperature (°C) ® (D) Zone refining (iv) Extraction of Au
11. Choose the correct option of temperature at which carbon (v) Purification of Ni
reduces FeO to iron and produces CO. Code :
(a) Below temperature at point A (A) (B) (C) (D)
(b) Approximately at the temperature corresponding to (a) (iv) (ii) (iii) (i)
point A (b) (ii) (iii) (i) (v)
(c) Above temperature at point A but below temperature (c) (i) (ii) (iii) (iv)
at point D (d) (iii) (iv) (v) (i)
(d) Above temperature at point A
19. Which one of the following ores is best concentrated by
12. Below point 'A' FeO can ..................... froth floatation method? [JEE M 2016, C]
(a) be reduced by carbon monoxide only.
(a) Galena (b) Malachite
(b) be reduced by both carbon monoxide and carbon.
(c) be reduced by carbon only. (c) Magnetite (d) Siderite
(d) not be reduced by both carbon and carbon monoxide. 20. Extraction of gold and silver involves leaching with CN–
13. For the reduction of FeO at the temperature corresponding ion. Silver is later recovered by [NEET 2017, C]
to point D, which of the following statements is correct? (a) distillation (b) zone refining
(a) DG value for the overall reduction reaction with (c) displacement with Zn (d) liquation
carbon monoxide is zero. 21. Considering Ellingham diagram, which of the following
(b) DG value for the overall reduction reaction with a metals can be used to reduce alumina? [NEET 2018, C]
mixture of 1 mol carbon and 1 mol oxygen is positive. (a) Fe (b) Zn (c) Cu (d) Mg
(c) DG value for the overall reduction reaction with a 22. Which one is malachite from the following ?
mixture of 2 mol carbon and 1 mol oxygen will be
[NEET 2019, C]
positive.
(d) DG value for the overall reduction reaction with (a) CuFeS2 (b) Cu(OH)2
carbon monoxide is negative. (c) Fe3O4 (d) CuCO3.Cu(OH)2
GENERAL PRINCIPLES AND PROCESSES OF ISOLATION OF ELEMENTS 301

23. The ore that contains both iron and copper is: 25. Identify the correct statement from the following :
[JEE M 2019, C] [NEET 2020, S]
(a) copper pyrites (b) malachite (a) Blister copper has blistered appearance due to
(c) dolomite (d) azurite evolution of CO2.
24. The ore that contains the metal in the form of fluoride is: (b) Vapour phase refining is carried out for Nickel by
[JEE M 2019, C] Van Arkel method.
(a) cryolite (b) malachite (c) Pig iron can be moulded into a variety of shapes.
(c) magnetite (d) sphalerite (d) Wrought iron is impure iron with 4% carbon.
26. The purest form of commercial iron is: [JEE M 2020, C]
(a) pig iron (b) wrought iron
(c) cast iron (d) scrap iron and pig iron

1. Which of the following combination represents the correct


6. XCl2 ( excess ) + YCl 2 ¾¾
® XCl 4 + Y ¯ ;
matching of metals with the most commonly employed
ores for their extraction? D 1
Fe Zn Cu Al YO ¾¾¾¾ ® O + Y, Ore of Y would be:
> 400° 2 2
(a) Haematite Sphalerite Copper Bauxite
(a) Siderite (b) Cinnabar
pyrites
(c) Malachite (d) Hornsilver
(b) Iron pyrites Zincite Cuprite Clay
7. A sulphide ore is generally roasted to the oxide before
(c) Siderite Calamine Malachite Alumminium
reduction, because
phosphate
(a) the enthalpy of formation of CO2 is more than that of
(d) Chalcocite Magnetite Copper Bauxite
CS2
glance
(b) a metal sulphide is generally more stable than the
2. Select the property which forms the basis of hydromet-
metal oxide
allurgical process that is used for extraction of gold.
(a) Gold is electropositive metal (c) no reducing agent is found suitable for reducing a
(b) Gold is less reactive metal sulphide ore
(c) Gold forms complexes that are water soluble (d) a sulphide ore cannot be reduced at all
(d) Gold forms salts, which are water soluble 8. Complexes formed in the following methods are
3. Consider the following reactions : I. Mond's process for purification of nickel.
D II. Removal of lead poisoning from the body.
2XS + 3O 2 ¾¾® 2XO + 2SO 2
III. Cyanide process for extraction of silver.
D IV. Froth flotation process for separation of ZnS from
2XO + XS ¾¾® 3' X '+ SO 2
galena ore by using depressant.
Then 'X' can not be:
I II III IV
(a) Hg (b) Pb (c) Zn (d) Cu 2– –
4. When CuSO4 is electrolysed using platinum electrodes, (a) [Ni(CO)4] [Pb(EDTA)] [Ag(CN)2] [Zn(CN)2]
(a) copper is liberated at cathode, sulphur at anode (b) [Ni(CO)4] [Pb(EDTA)]2– [Ag(CN)2]– [Zn(CN)4]2–
(b) copper is liberated at cathode, oxygen at anode (c) [Ni(CO)6] [Pb(EDTA)]4– [Ag(CN)2]– [Zn(CN)6]4–
(c) sulphur is liberated at cathode, oxygen at anode (d) [Ni(CO)4] [Pb(EDTA)]2– [Ag(CN)4]3– [Zn(CN)4]2–
(d) oxygen is liberated at cathode, copper at anode 9. Four metals and their methods of refinement are given
5. Which of the following statements is correct ? i. Ni, Cu, Zr, Ga
(a) Gangues are carefully chosen to combine with the ii. Electrolysis, van Arkel process, zone refining, Mond's
slag present in the ore to produce easily fusible flux process
to carry away the impurities Choose the right method for each :
(b) Slags are carefully chosen to combine with the flux (a) Ni: Electrolysis; Cu: van Arkel process; Zr: Zone
present in the ore to produce easily fusible gangue to refining; Ga: Mond's process.
carry away the impurities
(b) Ni: Mond's process; Cu: Electrolysis; Zr: van Arkel
(c) Gangues are carefully chosen to combine with the
process; Ga: Zone refining
flux present in the ore to produce easily fusible slag
to carry away the impurities (c) Ni: Mond's process; Cu: van Arkel process; Zr: Zone
(d) Fluxes are carefully chosen to combine with the refining; Ga: Electrolysis.
gangue present in the ore to produce easily fusible (d) Ni: Electrolysis; Cu: Zone refining; Zr: van Arkel
slag to carry away the impurities process; Ga: Mond's process
EBD_8350
302 CHEMISTRY

10. The main reactions occurring in blast furnace during (b) the Hall-Heroult process is used for the production
extraction of iron from haematite are________. of aluminium and iron.
(i) Fe2O3 + 3CO —® 2Fe + 3CO2 (c) pig iron is obtained from cast iron.
(ii) FeO + SiO2 —® FeSiO3 (d) the blistered appearance of copper during the metal-
(iii) Fe2O3 + 3C —® 2Fe + 3CO lurgical process is due to the evolution of CO2.
(iv) CaO + SiO2 —® CaSiO3 14. The pair that does NOT require calcination is :
(a) ZnO and MgO
(a) (i) and (iii) (b) (ii) and (iv)
(b) ZnO and Fe2O3·xH2O
(c) (i) and (iv) (d) (i), (ii) and (iii)
(c) ZnCO3 and CaO
11. The correct statement is : (d) Fe2O3and CaCO3·MgCO3
(a) aniline is a froth stabilizer. 15. Consider the following statements –
(b) zincite is a carbonate ore. (A) In the Aluminothermite process, aluminium acts as
(c) sodium cyanide cannot be used in the metallurgy of
reducing agent.
silver.
(d) zone refining process is used for the refining of (B) The process of extraction of gold involves the
titanium. formation of [Au(CN)2]– and [Zn(CN)4]2–.
12. The idea of froth floatation method came from a person X (C) In the extractive metallurgy of zinc, partial fusion of
and this method is related to the process Y of ores, X and ZnO with coke is called sintering and reduction of ore
Y, respectively, are : to the molten metal is called smelting.
(a) fisher woman and concentration
(D) Extractive metallurgy of silver from its ore argentine
(b) washer woman and concentration
(c) fisher man and reduction involves complex formation and displacement by more
(d) washer man and reduction electropositive metal.
13. The correct statement is : Choose the correct options –
(a) leaching of bauxite using concentrated NaOH (a) A and B (b) B and C
solution gives sodium aluminate and sodium silicate. (c) A, B and C (d) A, B, C and D

ANSW ER KEY
Exercis e 1 : NCERT Based Topic-wise MCQs
1 (a) 10 (d) 19 (c) 28 (c) 37 (d) 46 (d) 55 (a) 64 (a) 73 (c) 82 (d)
2 (a) 11 (a) 20 (c) 29 (b) 38 (b) 47 (b) 56 (b) 65 (c) 74 (b) 83 (b)
3 (a) 12 (b) 21 (d) 30 (a) 39 (d) 48 (b) 57 (d) 66 (c) 75 (a) 84 (d)
4 (a) 13 (a) 22 (b) 31 (c) 40 (b) 49 (b) 58 (b) 67 (c) 76 (a) 85 (b)
5 (b) 14 (a) 23 (c) 32 (a) 41 (d) 50 (a) 59 (a) 68 (c) 77 (c) 86 (d)
6 (c) 15 (c) 24 (c) 33 (b) 42 (c) 51 (b) 60 (b) 69 (a) 78 (b) 87 (a)
7 (a) 16 (a) 25 (a) 34 (d) 43 (c) 52 (d) 61 (c) 70 (a) 79 (c) 88 (b)
8 (a) 17 (a) 26 (d) 35 (b) 44 (c) 53 (c) 62 (d) 71 (a) 80 (b) 89 (a)
9 (a) 18 (d) 27 (c) 36 (b) 45 (b) 54 (a) 63 (a) 72 (b) 81 (b) 90 (a)
Exercis e 2 : NCERT Exemplar & Past Year MCQs
1 (a) 4 (c) 7 (a) 10 (a) 13 (a) 16 (c) 19 (a) 22 (d) 25 (c)
2 (d) 5 (b) 8 (c) 11 (d) 14 (b) 17 (b) 20 (c) 23 (a) 26 (b)
3 (a) 6 (a) 9 (b) 12 (a) 15 (b) 18 (a) 21 (d) 24 (a)
Exercis e 3 : Problem Solving Skill Enhancer MCQs
1 (a) 3 (c) 5 (d) 7 (b) 9 (b) 11 (a) 13 (a) 15 (d)
2 (c) 4 (b) 6 (b) 8 (b) 10 (c) 12 (b) 14 (a)
The p-Block Elements
21 (Group 15, 16, 17 and 18)

Trend Buster NEET & JEE Main

Number of Questions from 2020-15 16 9 Minimum two questions in NEET and


Weightage 5.0% 5.1% 1 question in JEE asked every year.

The most Important Concepts that Cover Maximum number of Questions asked in past 6 years.

Oxoacids of phosphorus / sulphur 4 2


Shapes and hybridisation of noble gas compounds / 2 —
Interhalogen compounds
Bond dissociation enthalpy / reactivity / boiling point 2 1
of halogen/interhalogen/ hydrogen halide
Mixed properties of oxides of N,S / physical 3 1
properties of noble gases
Less Important Concepts that Cover 1 or 2 Questions asked in past 6 years.

Ionic radii trend / electron gain enthalpy of


halogens / thermal stability of gp.16 hydrides 1 2
Oxidation state trend in N compounds 1 1
Preparation of dinitrogen/chemical reactions of 3 2
nitrogen and Cl / preparation processes
NEET JEE

2020 Group 15 / Group 16 / Electron gain enthalpy of halogens / 2 Easy / 1 Easy


Group 17 Oxoacids of sulphur / reactions of Average
nitrogen compounds
2019 Group 15 / Group 16 / Thermal stability of hydrides of group 2 Easy 1 Easy
Group 17 16 / preparation processes / oxidation
states of nitrogen compounds
2018 Group 15 / Group 17 Oxidation state of nitrogen 2 Easy / 1 Average
compounds / mixed property trend of Average
halogens / preparation of dinitrogen
2017 Group 16 / Group 17 Oxoacids of sulphur and phosphorus/ 2 Average 1 Average
shapes and hybridisation of
interhalogen compounds / chemical
properties of Cl / shape and
hybridisation of noble gas compounds
2016 Group 15 / Group 17 Oxoacids of phophorus / 4 Average 1 Easy
Group 18 chemical properties of nitrogen /
bond dissociation enthalpy of
halogen molecules
2015 Group 15 / Group 16 Oxoacids of phosphorus / mixed 4 Average 4 Easy
Group 17 properties of oxides / boiling point of
hydrogen halides / ionic radii trend /
reactivity trend
EBD_8350
304 CHEMISTRY
THE p-BLOCK ELEMENTS (GROUP 15, 16, 17 AND 18) 305
EBD_8350
306 CHEMISTRY

Problem Solving Tips/ Tricks/ Points to Remember

4 Hydracid of halogens HF forms two series of salts


O O
(NaHF2 and NaF). S O O O
O O
4 Silver halide insoluble in ammonia : AgI S S S
O S S O
O O O
Asthma patient for respiration uses a mixture of : O O
4 O
O O
O O
Helium + Oxygen
g-form b- and a-form
4 Metals and non-metals react with HNO3 under different 4 SF6 is used in high voltage tranformers because of its
conditions to give NO or NO2. insulating property.
4 In laboratory, dioxygen is prepared by heating KClO3 in 4 Liquid oxygen mixed with finely divided carbon is used
presence of MnO2. in place of dynamite in coal mining.
4 XeF2 is linear, XeF4 is square planar and XeF6 is dis- 4 A mixture of ozone and cyanogen is used as a rocket
torted octahedral. fuel.
4 XeF2, XeF4, and XeF6 are powerful fluorinating agents. 4 Gun powder is a mixture of sulphur, charcoal and KNO3.
4 Xe form two oxides such as XeO3 and XeO4. 4 S2Cl2 is used in the vulcanisation of rubber. It is heating
4 Freshly prepared phosphorous is colourless. On rubber with sulphur and was discovered by, Charles
standing acquires pale lemon colour due to formation of Goodyear in 1839.
red phosphorus on the surface. It is therefore called 4 Electrical conductivity of Se is negligible in dark but
yellow phosphorous. Due to its poisonous nature the increases on exposure to light. Due to this property, Se
jaw bones decay and disease is known as “Phossy jaw” is used in photo-electric cells and as a photoconductor
4 Smoke screen : Ca 3P2 is used smoke screen. PH3 in photocopying (Xerox) machines.
obtained from it catches fire to give the needed smoke. 4 Euchlorine
4 Rat poison : Zinc phosphide Zn3P2 is a rat poison, which It is a mixture of chlorine and chlorine dioxide and
gives PH3 obtained by heating KClO3 with conc. HCl
4 In solid state, PCl5 exists as an ionic solid, [PCl4]+ [PCl6]– 2KClO3 + 4HCl ® 2KCl + Cl 2 + 2ClO 2 + 2H 2O
in which the cation [PCl4]+ is tetrahedral and the anion
NaClO3 is a powerful weed killer and KClO3 (Berthelot’s
[PCl6]– is octahedral.
salt) is used in fire works and matches as oxidising agent.
4 SO3. is a gas, sp2 hybridised and planar in nature.
4 Clathrates
O O O A number of organic and inorganic compounds having
S S S gases trapped into the cavities of crystal lattices are
enclosure or clathrate compounds. They are known as
O O O O O O cage compounds also. The clathrates are non
In solid state it exist as a cyclic trimer (SO3)3 g-form or stoichiometric compounds. He and Ne do not form
as a linear chain cross linked sheets clathrates due to their small size.
THE p-BLOCK ELEMENTS (GROUP 15, 16, 17 AND 18) 307

Exercise 1 : NCERT Based Topic-wise MCQs


Topic 1: Group 15 Elements 13. Which of the following can be used as an anaesthesia ?
(a) N2O (b) NO (c) NCl3 (d) NO2
1. Ionic radii (in Å) of As3+, Sb3+ and Bi3+ follow the order 14. A deep brown gas is formed by mixing two colourless gases
(a) As3+ > Sb3+ > Bi3+ (b) Sb3+ > Bi3+ >As3+ which are
3+
(c) Bi > As > Sb3+ 3+ (d) Bi3+ > Sb3+ > As3+ (a) NO2 and O2 (b) N2O and NO
2. Which of the following statements is not correct for (c) NO and O2 (d) NH3 and HCl
nitrogen?
(a) Its electronegativity is very high 15. The catalyst used in the manufacture of HNO 3 by
(b) d-orbitals are available for bonding Ostwald’s process is :
(c) It is a typical non-metal (a) platinum gauze (b) vanadium pentoxide
(d) Its molecular size is small (c) finely divided nickel (d) platinum black .
3. Collectively the elements of group 15 are called –
16. Concentrated nitric acid, upon long standing, turns yellow
(a) pnicogens (b) pnicopens
brown due to the formation of
(c) nicopen (d) None of these
4. Which one of the following elements is most metallic ? (a) NO (b) NO2 (c) N2O (d) N2O4
(a) P (b) As (c) Sb (d) Bi 17. One mole of calcium phosphide on reaction with excess
5. The three important oxidation states of phosphorus are water gives
(a) –3, +3 and +5 (b) –3, +3 and –5 (a) one mole of phosphine
(c) –3, +3 and +2 (d) –3, +3 and +4 (b) two moles of phosphoric acid
6. Nitrogen is relatively inactive element because (c) two moles of phosphine
(a) its atom has a stable electronic configuration (d) one mole of phosphorus pentoxide
(b) it has low atomic radius 18. P2O5 is heated with water to give
(c) its electronegativity is fairly high (a) hypophosphorous acid(b) phosphorous acid
(d) dissociation energy of its molecule is fairly high (c) hypophosphoric acid (d) orthophosphoric acid
7. Which of the following species has the highest dipole 19. Basicity of orthophosphoric acid is
moment? (a) 2 (b) 3 (c) 4 (d) 5
(a) NH3 (b) PH3 (c) AsH3 (d) SbH3 20. What is hybridization of P in PCl5 ?
8. The correct decreasing order of basic strength is: (a) sp3 (b) sp3d 2 (c) sp3d (d) sp2
(a) AsH3 > SbH3 > PH3 > NH3 21. Among the oxyacids of phosphorus, the dibasic acid is
(b) SbH 3 > AsH 3 > PH 3 > NH 3 (a) H4P2O7 (b) H3PO2 (c) HPO3 (d) H3PO3
(c) NH 3 > PH 3 > AsH 3 > SbH 3 22. Which of these doesn’t exist?
(a) PH3 (b) PH5 (c) LuH3 (d) PF5
(d) PH 3 > AsH 3 > SbH 3 > NH 3
23. In the reaction
9. Ammonia is generally manufactured for fertilizers by the
reaction HNO3 + P4 O10 ® HPO3 + X , the product X is
(a) N2O5 (b) N2O3 (c) NO2 (d) H2O
(a) 2NH 4Cl + Ca(OH) 2 ® CaCl 2 + 2H 2O + 2NH3
24. The structural formula of hypophosphorous acid is
(b) By passing an electric discharge in a mixture of N2
and H2 O O
(c) By passing a mixture of N2 and H2 under high pressure P P
and moderate temperature over a catalyst (a) H (b) H
OH OH
(d) None of these H OH
10. Which of the following oxides of nitrogen is a coloured
gas? O O
(a) N2O (b) NO (c) N2O5 (d) NO2 P P
(c) HO (d) H
11. In which one of the following oxides of nitrogen, one OH OOH
nitrogen atom is not directly linked to oxygen? OH OH
(a) NO (b) N2O4 (c) N2O (d) N2O3 25. Ammonia on catalytic oxidation gives an oxide from which
12. Which of the following oxides of nitrogen reacts with nitric acid is obtained. The oxide is :
FeSO4 to form a dark brown compound (a) N 2 O3 (b) NO (c) NO 2 (d) N 2 O5
(a) N2O (b) NO (c) NO2 (d) N2O3
EBD_8350
308 CHEMISTRY

26. In which of the following equations the product formed 38. Which of the following is the strongest reducing agent ?
has similar oxidation state for nitrogen? (a) NH3 (b) PH3 (c) BiH3 (d) SbH3
D 39. The nitrogen oxides that contain(s) N–N bond(s) is /are
(i) NH4NO3 ¾¾ ® N2O + 2H2O (i) N2O (ii) N2O3 (iii) N2O4 (iv) N2O5
673K
(ii) 2Pb(NO3)2 ¾¾¾® 4NO2 + 2PbO + O2 (a) (i) , (ii) (b) (ii) , (iii) , (iv)
(c) (iii) , (iv) (d) (i), (ii) and (iii)
(iii) 4HNO3 + P4O10 ¾¾ ® 4HPO3 + 2N2O5 40. Which of the following is incorrect for white and red
Cool phosphorus ?
ˆˆˆˆ† N O
(iv) 2NO2 ‡ˆˆˆˆ (a) They are both soluble in CS2
2 4
Heat (b) They can be oxidised by heating in air
(a) (i) and (iii) (b) (ii) and (iv) (c) They consist of the same kind of atoms
(c) (i) and (v) (d) (iii) and (iv) (d) They can be converted into one another
27. Which of the following group 15 element forms metallic 41. Which property of white phosphorus is common to
bonds in elemental state ? red phosphorous ?
(a) As (b) P (c) Sb (d) Bi (a) It burns when heated in air.
28. Which of the following has the highest pp – pp bonding (b) It reacts with hot caustic soda solution to give
tendency ? phosphine.
(a) N (b) P (c) As (d) Sb (c) It shows chemiluminescence.
29. Pick out the wrong statement. (d) It is soluble in carbon disulphide.
(a) Nitrogen has the ability to form pp-pp bonds with itself. 42. Which of the following statements regarding allotropic
(b) Bismuth forms metallic bonds in elemental state. forms of phosphorus is incorrect?
(c) Catenation tendency is higher in nitrogen when (a) White phosphorus is more reactive than red and black
compared with other elements of the same group. due to high angular strain.
(d) Nitrogen has higher first ionisation enthalpy when (b) Red phosphorus on heating catches fire and give
compared with other elements of the same group. dense red fumes of P4O10.
30. Among the 15th group elements, as we move from nitrogen (c) Red phosphorus is polymeric in nature consisting of
to bismuth, the pentavalency becomes less pronounced chains of P4 tetrahedral.
and trivalency becomes more pronounced due to (d) Black phosphorus has two forms a-black and b-black
phosphorus
(a) Non metallic character (b) Inert pair effect
43. When orthophosphoric acid is heated to 600°C, the product
(c) High electronegativity (d) Large ionization energy
formed is
31. Pure nitrogen is prepared in the laboratory by heating a mixture (a) PH3 (b) P2O5 (c) H3PO3 (d) HPO3
of 44. PCl3 reacts with water to form
(a) NH4OH + NaCl (b) NH4NO3 + NaCl (a) PH3 (b) H3PO4 and HCl
(c) NH4Cl + NaOH (d) NH4Cl + NaNO2. (c) POCl3 (d) H3PO4
32. Most acidic oxide among the following is – 45. The number of hydrogen atom(s) attached to phosphorus
(a) N2O5 (b) P2O5 (c) N2O4 (d) As2O 3 atom in hypophosphorous acid is
33. Liquid ammonia bottles are opened after cooling them in (a) three (b) one (c) two (d) zero
ice for sometime. It is because liquid NH3 46. The number of P – O – P bonds in cyclic metaphosphoric
(a) Brings tears to the eyes acid is
(b) Has a high vapour pressure (a) zero (b) two (c) three (d) four
(c) Is a corrosive liquid 47. Oxidation states of P in H4 P2O5 , H4 P2O6 , and H4 P2O7 ,
(d) Is a mild explosive are respectively:
34. Nitrogen dioxide cannot be obtained by heating : (a) + 3, + 5, + 4 (b) + 5, + 3, + 4
(a) KNO3 (b) Pb(NO3)2 (c) + 5, + 4, + 3 (d) + 3, + 4, + 5
(c) Cu(NO3)2 (d) AgNO3 48. How many bridging oxygen atoms are present in P4O10?
35. Which of the following oxides is neutral ? (a) 5 (b) 6 (c) 4 (d) 2
(a) N2O3 (b) N2O4 (c) N2O5 (d) N2O 49. Which of the following statements is not valid for oxoacids
36. Which oxide of nitrogen is obtained on heating ammonium of phosphorus?
nitrate at 250ºC ? (a) Orthophosphoric acid is used in the manufacture of
triple superphosphate.
(a) Nitric oxide (b) Nitrous oxide
(b) Hypophosphorous acid is a diprotic acid.
(c) Nitrogen dioxide (d) Dinitrogen tetraoxide
(c) All oxoacids contain tetrahedral four coordinated
37. Which of the following trihalide is unstable? phosphorus.
(a) NF3` (b) AsCl3 (c) SbBr3 (d) NCl3 (d) All oxoacids contain atleast one P = O and one P —
OH group.
THE p-BLOCK ELEMENTS (GROUP 15, 16, 17 AND 18) 309

50. Which of the following is a cyclic phosphate ? 59. Assertion : When a metal is treated with conc. HNO3 it
(a) H3P3O10 (b) H6P4O13 generally yields a nitrate, NO2 and H2O.
(c) H5P5O15 (d) H7P5O16 Reason : Conc. HNO3 reacts with metal and first produces
51. P—O—P bond is present in a metal nitrate and nascent hydrogen. The nascent
(a) H4P2O6 (b) H4P2O5 hydrogen then further reduces HNO3 to NO2.
(c) Both (a) and (b) (d) Neither (a) nor (b) (a) Assertion is correct, reason is correct; reason is a
52. The basicity of pyrophosphorus acid is correct explanation for assertion.
(a) 2 (b) 4 (c) 1 (d) 5 (b) Assertion is correct, reason is correct; reason is not
53. The oxidation state of phosphorus in cyclotrimeta- a correct explanation for assertion
phosphoric acid is (c) Assertion is correct, reason is incorrect
(a) +3 (b) +5 (c) –3 (d) +2 (d) Assertion is incorrect, reason is correct.
54. Which acid has P – P linkage ? 60. Nitrogen forms N2, but phosphorus is converted into P4
(a) Hypophosphoric acid (b) Pyrophosphoric acid from P, the reason is
(c) Metaphosphoric acid (d) Orthophosphoric acid (a) Triple bond is present between phosphorus atom
55. Which of the following statements are correct? (b) pp – pp bonding is strong
(i) All the three N—O bond lengths in HNO3 are equal. (c) pp – pp bonding is weak
(ii) All P—Cl bond lengths in PCl5 molecule in gaseous (d) Multiple bond is formed easily
state are equal. 61. Zinc on reaction with dilute HNO3 gives x and zinc on
(iii) P4 molecule in white phohsphorus have angular strain reaction with concentrated HNO3 gives y. Identify x and y.
therefore white phosphorus is very reactive. (a) x = NO2 , y = N2O (b) x = N2O , y = NO
(iv) PCl5 is ionic in solid state in which cation is tetrahedral (c) x = NO , y = NO2 (d) x = N2O , y = NO2
and anion is octahedral. 62. Phosphine is not evolved when
(a) (i) and (iv) (b) (iii) and (iv) (a) white phosphorus is boiled with a strong solution of
(c) (ii) and (iii) (d) (ii) only Ba(OH)2
56. Match the columns (b) phosphorus acid is heated
Column-I Column-II (c) calcium hypophosphite is heated
(A) 2Pb(NO3)2 ¾¾¾®673K (p) High pressure favours (d) metaphosphoric acid is heated.
63. Pure phosphine is not combustible while impure phosphine
4NO2 + 2PbO + O2 the formation of
product is combustible, this combustibility is due to presence of
(B) N2(g) + O2(g) ƒ (q) Product formed is acidic (a) P2 H 4 (b) N2 (c) PH 5 (d) P2 O5
2NO(g) brown gas 64. Assertion : Dinitrogen is inert at room temperature.
D Reason : Dinitrogen directly combines with lithium to form
(C) NH4NO3 ¾¾ ® (r) This reaction occurs at
ionic nitrides.
N2O + 2H2O a high temperature (a) Assertion is correct, reason is correct; reason is a
about 2000 K correct explanation for assertion.
(D) N2(g) + 3H2(g) ƒ (s) Product formed is a (b) Assertion is correct, reason is correct; reason is not
2NH3(g) neutral colourless gas a correct explanation for assertion
(a) A – (r, s), B – (q), C – (s), D – (p) (c) Assertion is correct, reason is incorrect
(b) A – (q), B – (r,s), C – (s), D – (p) (d) Assertion is incorrect, reason is correct.
(c) A – (q), B – (s), C – (r, s), D – (p) 65. In nitrogen family, the H-M-H bond angle in the hydrides
(d) A – (q), B – (r, s), C – (p), D – (s) gradually becomes closer to 90º on going from N to Sb.
57. Blue solid which is obtained on reacting equimolar amounts This shows that gradually
of two gases at 245K is? (a) The basic strength of the hydrides increases
(a) N2O (b) N2O3 (c) N2O4 (d) N2O5 (b) Almost pure p-orbitals are used for M-H bonding
58. Match the columns (c) The bond energies of M-H bonds increase
Column - I Column - II (d) The bond pairs of electrons become nearer to the
(A) Used in manufacture (p) Ammonia central atom
of calcium cyanamide 66. The deep blue colour produced on adding excess of
(B) Used in manufacture (q) Nitric acid ammonia to copper sulphate is due to presence of
of nitric acid 2+
(b) ëê Cu ( NH 3 ) 4 ( H 2 O )2 ûú
(C) Used in pickling of (r) Dinitrogen 2+ é ù
(a) Cu
stainless steel
2+ 2+
(a) A – (r), B – (p), C – (q) (b) A – (p), B – (r), C – (q) (c) é Cu ( NH ) ù (d) éê Cu ( NH 3 ) 2 ( H 2 O )4 ùú
(c) A – (r), B – (q), C – (p) (d) A – (q), B – (p), C – (r) ëê 3 6 ûú ë û
EBD_8350
310 CHEMISTRY

67. Match the columns. 80. It is possible to obtain oxygen from air by fractional
Column-I Column-II distillation because
(a) oxygen is in a different group of the periodic table
(Oxyacid) (Materials for
from nitrogen
preparation)
(b) oxygen is more reactive than nitrogen
(A) H3PO2 (p) Red P + alkali (c) oxygen has higher b.p. than nitrogen
(B) H3PO3 (q) P4O10 + H2O (d) oxygen has a lower density than nitrogen.
(C) H3PO4 (r) P2O3 + H2O 81. Electronegativity of oxygen is more than sulphur yet H2S is
(D) H4P2O6 (s) White P + alkali acidic while water is neutral. This is because
(a) water is highly associated compound
(a) (A) – (s), (B) – (r), (C) – (q), (D) – (p) (b) molecular mass of H2S is more than H2O
(b) (A) – (p), (B) – (r), (C) – (q), (D) – (s) (c) H2S is gas while H2O is a liquid
(c) (A) – (s), (B) – (r), (C) – (p), (D) – (q) (d) H–S bond is weaker than H–O bond
(d) (A) – (q), (B) – (r), (C) – (p), (D) – (s) 82. Which of the following oxide is amphoteric ?
68. What is Z in following reaction (a) SnO2 (b) CaO (c) SiO2 (d) CO2
CuSO4 + Z ® Cu3P2 +H2SO4 83. Which of the following is not correctly matched ?
(a) SF4 – gas (b) SeF4 – liquid
HgCl2 + Z ® Hg3P2 +HCl
(c) TeF4 – solid (d) SF6 – solid
(a) White phosphorus (b) Red phosphorus
84. The compound which gives off oxygen on moderate
(c) Phosphine (d) Orthophosphoric acid
heating is :
(a) cupric oxide (b) mercuric oxide
Topic 2: Group 16 Elements
(c) zinc oxide (d) aluminium oxide
69. Which of the following hydrides has the lowest boiling 85. Which of the following on thermal decomposition gives
point? oxygen gas ?
(a) H2O (b) H2S (c) H2Se (d) H2Te (a) Ag2O (b) Pb3O4
70. Which of the following hydrides is most acidic ? (c) PbO2 (d) All of these
(a) H2Te (b) H2Se (c) H2O (d) H2S 86. Which of the following form of the sulphur shows
paramagnetic behaviour ?
71. Which of the following hydrides shows the highest boiling
(a) S8 (b) S6
point ?
(c) S2 (d) All of these
(a) H2O (b) H2S (c) H2Se (d) H2Te
87. Which of the following oxo acid of sulphur has O–O bond ?
72. Which is the best oxidising agent among the following ? (a) H2S2O7 (b) H2S2O8 (c) H2S2O6 (d) H2S2O5
(a) S (b) O (c) Se (d) Te 88. Which of the following is the key step in the manufacture
73. Which of the following is not oxidized by O3 ? of sulphuric acid ?
(a) KI (b) FeSO4 (c) KMnO4 (d) K2MnO4 (a) Burning of sulphur or sulphide ores in air to generate
74. Oxygen gas can be prepared from solid KMnO4 by : SO2
(b) Conversion of SO2 to SO3 by the reaction with oxygen
(a) treating the solid with H2 gas
in presence of catalyst.
(b) strongly heating the solid (c) Absorption of SO3 in H2SO4 to give oleum.
(c) dissolving the solid in dil. H2SO4 (d) Both (b) and (c)
(d) dissolving solid in dil. HCl 89. The acid which has a peroxy linkage is
75. Which of the following is an acidic oxide? (a) Sulphurous acid (b) Pyrosulphuric acid
(a) Mn 2O7 (b) Na2O (a) N2O (b) BaO (c) Dithionic acid (d) Caro’s acid
76. Atomicity of sulphur in rhombic sulphur is 90. S – S bond is not present in
(a) 1 (b) 2 (c) 8 (d) 6 (a) S 2 O 24 - (b) S2 O 52 - (c) S2 O 32 - (d) S2 O 27 -
77. What is X in the following reaction ?
91. Sulphuric acid reacts with PCl5 to give
X
2SO2(g) + O2(g) ¾¾® 2SO3(g) (a) thionyl chloride (b) sulphur monochloride
(a) V2O5 (b) CuO (c) CuCl2 (d) MnO2 (c) sulphuryl chloride (d) sulphur tetrachloride
78. Carbohydrates on reaction with conc. H2SO4 becomes 92. The correct statement(s) about O3 is (are)
charred due to (i) O—O bond lengths are equal
(a) hydrolysis (b) dehydration (ii) Thermal decomposition of O3 is endothermic
(c) hydration (d) oxidation (iii) O3 is diamagnetic in nature
79. Hybridization of S in SO3 is (iv) O3 has a bent structure
(a) (i) and (iii) (b) (ii) and (iii)
(a) sp2 (b) sp3 (c) sp2d (d) sp3d 2
(c) (i), (iii) and (iv) (d) (i) and (iv)
THE p-BLOCK ELEMENTS (GROUP 15, 16, 17 AND 18) 311

93. Assertion : Bond angle of H2S is smaller than H2O. (b) Both oxidized and reduced
Reason : Electronegativity of the central atom increases, (c) Oxidized only
bond angle decreases. (d) Reduced only
(a) Assertion is correct, reason is correct; reason is a 103. At room temperature, HCl is a gas while HF is a low boiling
correct explanation for assertion. liquid. This is because
(b) Assertion is correct, reason is correct; reason is not (a) H- F bond is covalent (b) H- F bond is ionic
a correct explanation for assertion (c) HF has metallic bond (d) HF has hydrogen bond
(c) Assertion is correct, reason is incorrect 104. Gaseous HCl is a poor conductor of electricity while its
(d) Assertion is incorrect, reason is correct. aqueous solution is a good conductor; this is because
94. Assertion : Both rhombic and monoclinic sulphur exist as (a) H 2 O is a good conductor of electricity
S8 but oxygen exists as O2.
(b) a gas cannot conduct electricity but a liquid can
Reason : Oxygen forms pp – pp multiple bond due to small
(c) HCl gas does not obey Ohm’s law, whereas the
size and small bond length but pp – pp bonding is not
solution does
possible in sulphur.
(d) HCl ionises in aqueous solution
(a) Assertion is correct, reason is correct; reason is a
105. In which of the following reactions chlorine is both reduced
correct explanation for assertion.
and oxidized?
(b) Assertion is correct, reason is correct; reason is not
(a) 2KMnO4 + 16HCl¾¾ ® 2KCl + 2MnCl2 + 8H2O + 5Cl2
a correct explanation for assertion
(c) Assertion is correct, reason is incorrect (b) 6NaOH + 3Cl2 ¾¾ ® 5NaCl + NaClO3 + 3H2O
(d) Assertion is incorrect, reason is correct. (c) NH3 + 3Cl2 ¾¾ ® NCl3 + 3HCl
(d) I2 + 6H2O + 5Cl2 ¾¾ ® 2HIO3 + 10HCl
Topic 3: Group 17 Elements 106. Fluorine is a stronger oxidising agent than chlorine in
95. Among F, Cl, Br and I the lowest ionization potential will aqueous solution. This is attributed to many factors except
be of (a) heat of dissociation (b) ionisation potential
(a) fluorine (b) chlorine (c) bromine (d) iodine (c) heat of hydration (d) electron affinity
96. The electronegativity follows the order 107. The correct order of heat of formation of halogen acids is
(a) HI > HBr > HCl > HF (b) HF > HCl > HBr > HI
(a) F > O > Cl > Br (b) F > Cl > Br > O
(c) HCl > HF > HBr > HI (d) HCl > HBr > HF > HI
(c) O > F > Cl > Br (d) Cl > F > O > Br
108. Which of the following is most volatile ?
97. Electron gain enthalpy with negative sign of fluorine is
(a) HI (b) HBr (c) HCl (d) HF
less than that of chlorine due to :
109. The bleaching action of chlorine is due to
(a) High ionization enthalpy of fluorine
(a) reduction (b) hydrogenation
(b) Smaller size of chlorine atom
(c) chlorination (d) oxidation
(c) Smaller size of fluorine atom 110. Cl2 reacts with hot and conc. NaOH to give –
(d) Bigger size of 2p orbital of fluorine (a) NaClO (b) NaClO3 (c) NaClO2 (d) NaClO4
98. Which one of the following order is correct for the bond 111. Which of the following is used in the preparation of chlorine?
energies of halogen molecules ? (a) Only MnO2
(a) I2 > Cl2 > Br2 (b) Br2 > Cl2 > I2 (b) Only KMnO4
(c) I2 > Br2 > Cl2 (d) Cl2 > Br2 > I2 (c) Both MnO2 and KMnO4
99. The correct order of reactivity of halogens with alkalies is (d) Either MnO2 or KMnO4
(a) F > Cl > Br > I (b) F < Cl > Br < I 112. The reaction of KMnO4 and HCl results in
(c) F < Cl < Br < I (d) F < Cl < Br > I (a) oxidation of Mn in KMnO4 and production of Cl2
100. The correct order of increasing oxidising power is (b) reduction of Mn in KMnO4 and production of H2
(a) F2 > Br2 > Cl2 > I2 (b) F2 < Cl2 < Br2 < I2 (c) oxidation of Mn in KMnO4 and production of H2
(c) Cl2> Br2 > F2 > I2 (d) I2 < Br2 < Cl2 < F2 (d) reduction of Mn in KMnO4 and production of Cl2
101. Fluorine exhibits an oxidation state of only –1 because 113. Hydrochloric acid at 25ºC is
(a) it can readily accept an electron (a) ionic and liquid (b) covalent and liquid
(b) it is very strongly electronegative (c) ionic and gas (d) None of these
114. Interhalogen compounds are more reactive than the
(c) it is a non-metal
individual halogen because
(d) it belongs to halogen family
(a) two halogens are present in place of one
102. Which of the following is the best description for the (b) they are more ionic
behaviour of bromine in the reaction given below ? (c) their bond energy is less than the bond energy of the
H 2 O + Br2 ® HOBr + HBr halogen molecule
(a) Proton acceptor only (d) they carry more energy
EBD_8350
312 CHEMISTRY

115. Which of the following statements regarding properties 123. Which one is most stable to heat –
of halogens are correct? (a) HClO (b) HClO2 (c) HClO3 (d) HClO4
(i) Due to small size electron gain enthalpy of fluorine is 124. Which of the following is not the characteristic of
less than that of chlorine. interhalogen compounds ?
(ii) Iodine has same physical state but different colour (a) They are more reactive than halogens
as compare to other members of the group. (b) They are quite unstable but none of them is explosive
(iii) Fluorine shows no positive oxidation state.
(c) They are covalent in nature
(iv) In X2(g) + H2O(l) ¾¾ ® HX(aq) + HOX(aq)
(d) They have low boiling points and are highly volatile.
(where X2 = Cl or Br)
125. Which of the following order is/are incorrect regarding
(a) (i), (ii) and (iv) (b) (i), (iii) and (iv)
the property indicated against it?
(c) (ii), (iii) and (iv) (d) (iii) and (iv)
(i) HF > HI > HBr > HCl : Boiling point
116. Which of the following statements are correct?
(i) Among halogens, radius ratio between iodine and (ii) Cl2O7 > Cl2O6 > ClO2 > Cl2O : Acidic character
fluorine is maximum. (iii) SbCl3 > SbCl5 : Covalent character
(ii) Leaving F—F bond, all halogens have weaker X—X (iv) MCl > MBr : Ionic character
bond than X—X¢ bond in interhalogens. (a) (iii) only (b) (ii) only
(iii) Among interhalogen compounds maximum number (c) (i) and (iii) (d) (ii) and (iv)
of atoms are present in iodine fluoride.
(iv) Interhalogen compounds are more reactive than Topic 4: Group 18 Elements
halogen compounds.
126. In the following four elements, the ionisation potential of
(a) (i) and (ii) (b) (i), (ii) and (iii)
which one is the highest ?
(c) (ii) and (iii) (d) (i), (iii) and (iv)
117. Which one of the following arrangements does not give (a) Oxygen (b) Argon (c) Barium (d) Cesium
the correct picture of the trends indicated against it ? 127. Gradual addition of electronic shells in the noble gases
(i) F2 > Cl2 > Br2 > I2 : Oxidizing power causes a decrease in their
(ii) F2 > Cl2 > Br2 > I2 : Electron gain enthalpy (a) ionisation energy (b) atomic radius
(iii) F2 > Cl2 > Br2 > I2 : Bond dissociation energy (c) boiling point (d) density
(iv) F2 > Cl2 > Br2 > I2 : Electronegativity. 128. The most abundant inert gas in the atmosphere is
(a) (ii) and (iv) (b) (i) and (iii) (a) He (b) Ne (c) Ar (d) Kr
(c) (ii) and (iii) (d) (ii), (iii) and (iv)
129. The lowest boiling point of helium is due to its
118. The correct order of the thermal stability of hydrogen
halides (H–X) is (a) inertness
(a) HI > HCl < HF > HBr (b) HCl< HF > HBr < HI (b) gaseous nature
(c) HF > HCl > HBr > HI (d) HI < HBr > HCl < HF (c) high polarisability
119. In the case of alkali metals, the covalent character decreases (d) weak van der Waal’s forces between atoms
in the order: 130. The noble gas which was discovered first in the sun and
(a) MF > MCl > MBr > MI (b) MF > MCl > MI > MBr then on the earth
(c) MI > MBr > MCl > MF (d) MCl > MI > MBr > MF (a) argon (b) xenon (c) neon (d) helium
120. Which of the following is correct about the reaction? 131. Which one of the following elements is most reactive ?
3NaClO ¾¾¾ heat
® NaClO3 + 2NaCl (a) He (b) Ne (c) Ar (d) Xe
132. Noble gases are group of elements which exhibit very
(a) It is disproportionation reaction
(a) high chemical activity
(b) Oxidation number of Cl decreases as well as increases
in this reaction (b) low chemical activity
(c) This reaction is used for the manufacture of halates (c) minimum electronegativity
(d) All of these (d) much paramagnetic properties
121. Which pair gives Cl2 at room temperature : 133. In XeF2, XeF4, XeF6 the number of lone pairs on Xe are
respectively
(a) NaCl + Conc. H 2SO 4 (b) Conc. HCl + KMnO4
(a) 2, 3, 1 (b) 1, 2, 3 (c) 4, 1, 2 (d) 3, 2, 1.
(c) NaCl + Conc. HNO 3 (d) NaCl + MnO2 134. Total number of lone pair of electrons in XeOF4 is
122. Bleaching powder on standing forms mixture of : (a) 0 (b) 1 (c) 2 (d) 3
(a) CaO + Cl 2 (b) CaO + CaCl 2 135. Which of the following has maximum number of lone pairs
associated with Xe ?
(c) HOCl + Cl 2 (d) CaCl 2 + Ca (ClO3 ) 2
(a) XeF4 (b) XeF6 (c) XeF2 (d) XeO3
THE p-BLOCK ELEMENTS (GROUP 15, 16, 17 AND 18) 313

136. Argon is used 153. Match the columns


(a) in filling airships Column - I Column - II
(b) to obtain low temperature (A) XeF4 (p) Contains similar types
(c) in high temperature welding
of bonds
(d) in radiotherapy for treatment of cancer
137. Noble gases are used in discharge tubes to gives different (B) XeOF4 (q) Contains maximum lone
colours. Reddish orange glow is due to pair
(a) Ar (b) Ne (c) Xe (d) Kr (C) XeF2 (r) Square pyramidal
138. Sea divers go deep in the sea water with a mixture of which geometry
of the following gases (D) XeO3 (s) Contains one lone pair
(a) O2 and He (b) O2 and Ar (a) A – (p), B – (r, s), C – (p, q), D – (p, s)
(c) O2 and CO2 (d) CO2 and Ar (b) A – (r, s), B – (p), C – (r, s), D – (p, s)
139. Which of the following is the life saving mixture for an
(c) A – (p), B – (p, q), C – (r, s), D – (p, s)
asthma patient ?
(a) Mixture of helium and oxygen (d) A – (p), B – (r, s), C – (p, s), D – (p, q)
(b) Mixture of neon and oxygen 154. Which of the following noble gases has the highest positive
(c) Mixture of xenon and nitrogen electron gain enthalpy value?
(d) Mixture of argon and oxygen (a) Helium (b) Krypton (c) Argon (d) Neon
140. Which of the following noble gas is least polarisable? 155. The correct order of solubility in water for He, Ne, Ar, Kr,
(a) He (b) Xe (c) Ar (d) Ne Xe is
141. In which of the following groups, when He is placed, its (a) He > Ne > Ar > Kr > Xe
all the properties are satisfied
(b) Ne > Ar > Kr > He > Xe
(a) with alkali metals (b) with halogens
(c) Xe > Kr > Ar > Ne > He
(c) with inert gases (d) None of these
142. Which inert gas show abnormal behaviour on liquefaction (d) Ar > Ne > He > Kr > Xe
(a) Xe (b) He (c) Ar (d) Kr 156. Which one of the following reactions of xenon compounds
143. Which has trigonal pyramidal shape ? is not feasible?
(a) XeOF4 (b) XeO3 (c) XeO3F2 (d) XeOF2 (a) 3XeF4 + 6H 2 O ¾¾ ® 2Xe + XeO 3 +12HF +1.5O 2
144. End-product of the hydrolysis of XeF6 is (b) 2XeF2 + 2H 2O ¾¾
® 2Xe + 4HF + O 2
(a) XeF4O (b) XeF2O2 (c) XeO3 (d) XeO3–
(c) XeF6 + RbF ¾¾ ® Rb[XeF7 ]
145. The shape of XeO2F2 molecule is
(a) trigonal bipyramidal (b) square planar (d) XeO3 + 6HF ¾¾ ® XeF6 + 3H 2O
(c) tetrahedral (d) see-saw 157. Which element out of He, Ar, Kr and Xe forms least number
146. XeF4 on partial hydrolysis produces of compounds ?
(a) XeF4 (b) XeOF2 (c) XeOF4 (d) XeO3 (a) He (b) Ar (c) Kr (d) Xe
147. Shape of XeOF4 is 158. The element which has not yet been reacted with F2 is
(a) octahedral (b) square pyramidal (a) Ar (b) Xe (c) Kr (d) Rn
(c) pyramidal (d) T-shaped 159. Which statement about noble gases is not correct?
148. Which is a planar molecule ? (a) Xe forms XeF6
(a) XeO4 (b) XeF4 (c) XeOF4 (d) XeO2F2
(b) Ar is used in electric bulbs
149. Which of the following has sp3 hybridization ?
(c) Kr is obtained during radioactive disintegration
(a) XeO3 (b) BCl3 (c) XeF4 (d) BBr3
(d) He has the lowest b.pt among all the noble gases
150. The ease of liquefaction of noble gases increases in the
order 160. Trigonal bipyramidal geometry is shown by :
(a) He < Ne < Ar < Kr < Xe (a) XeOF4 (b) XeO3F2
(b) Xe < Kr < Ne < Ar < He (c) FXeOSO2F (d) [XeF8]2–
(c) Kr < Xe < He < Ne < Ar 161. Which of the following statements are true?
(d) Ar < Kr < Xe < Ne < He (i) Only type of interactions between particles of noble
151. Which of the following element has the property of gases are due to weak dispersion forces.
diffusing through most commonly used laboratory (ii) Ionisation enthalpy of molecular oxygen is very close
materials such as rubber, glass or plastics? to that of xenon.
(a) Xe (b) Rn (c) He (d) Ar (iii) Hydrolysis of XeF6 is redox reaction.
152. Which of the following is used to produce and sustain (iv) Xenon fluorides are not reactive.
powerful superconducting magnets to form an essential (a) (i) and (iii) (b) (i) and (ii)
part of NMR spectrometer ? (c) (ii) and (iii) (d) (iii) and (iv)
(a) Ar (b) Ne (c) Rn (d) He
EBD_8350
314 CHEMISTRY

162. Which of the following statements are correct? 163. What are the products formed in the reaction of xenon
(i) Natural abundance of noble gases is ~ 1% by volume hexafluoride with silicon dioxide ?
of which Ar is the major constituent. (a) XeSiO4 + HF (b) XeF2 + SiF4
(ii) Noble gases have high positive values of electron (c) XeOF4 + SiF4 (d) XeO3 + SiF2
gain enthalpy.
164. XeO4 molecule is tetrahedral having :
(iii) Preparation of XeF2 requires F2 in excess amount.
(iv) Complete hydrolysis of all three XeF2, XeF4 and XeF6 (a) Two pp – dp bonds (b) One pp – dp bonds
gives Xe as one of product. (c) Four pp – dp bonds (d) Three pp – dp bonds
(a) (i) and (iii) (b) (ii) and (iv) 165. Which of the following fluorides of xenon is impossible?
(c) (i) and (ii) (d) (ii) and (iii) (a) XeF2 (b) XeF3 (c) XeF4 (d) XeF6

1. Find the number of sigma bonds in P4O10. 7. What is the number of P–O–P bonds in cyclic
2. How many bonding electron pairs are there in white metaphosphoric acid?
phosphorus ? 8. Find the number of P–O–P bonds in P4O10
3. Find the number of P – O – P bonds in cyclic metaphosphoric 9. 25 mL of household bleach solution was mixed with 30 mL
acid. of 0.50 M KI and 10 mL of 4N acetic acid. In the titration of
4. Calculate the total number of bond pairs and lone pairs of the liberated iodine, 48 mL of 0.25 N Na2S2O3 was used to
electrons present in OF2 molecule. reach the end point. Calculate the molarity of the household
5. How many forms of SO3 exists bleach solution.
6. What is the number of S–S bonds in sulphur trioxide trimer 10. What is the value of n in the molecular formula BenAl2Si6O18?
(S3O9)?

Exercise 3 : NCERT Exemplar & Past Year MCQs


NCERT Exemplar MCQs (a) ClO -4 > IO 4- < BrO 4- (b) IO 4- > BrO -4 < ClO 4-
1. Affinity for hydrogen decreases in the group from fluorine (c) BrO 4- > IO-4 < ClO4- (d) BrO4- > ClO 4- < IO4-
to iodine. Which of the halogen acids should have highest 5. Which of the following is isoelectronic pair?
bond dissociation enthalpy? (a) ICl2, ClO2 (b) BrO -2 , BrF2+
(a) HF (b) HCl (c) HBr (d) HI
(c) ClO2, BrF (d) CN–, O3
2. In the preparation of HNO3 , we get NO gas by catalytic
6. On addition of conc. H2SO4 to a chloride salt, colourless
oxidation of ammonia. The moles of NO produced by the
fumes are evolved but in case of iodide salt, violet fumes
oxidation of two moles of NH3 will be ................. .
come out. This is because
(a) 2 (b) 3 (c) 4 (d) 6
(a) H2SO4 reduces HI to I2 (b) HI is of violet colour
3. The oxidation state of central atom in the anion of
(c) HI gets oxidised to I2 (d) HI changes to HIO3
compound NaH2PO2 will be ................. .
7. Which of the following elements can be involved in
(a) + 3 (b) + 5 (c) + 1 (d) – 3 pp – dp bonding?
4. Reduction potentials of some ions are given below. Arrange (a) Carbon (b) Nitrogen
them in decreasing order of oxidising power. (c) Phosphorus (d) Boron
- - - 8. Which of the following pairs of ions are isoelectronic and
Ion ClO4 IO4 BrO4
Reduction potential isostructural?
E° = 1.19V E° = 1.65V E° = 1.74V
E° / V (a) CO32 - , NO3- (b) ClO3- , CO32-

(c) SO32- , NO3- (d) ClO3- , SO32 -


THE p-BLOCK ELEMENTS (GROUP 15, 16, 17 AND 18) 315

9. Bond dissociation enthalpy of E — H (E = element) bonds 21. A black compound of manganese reacts with a halogen
is given below. Which of the following compounds will acid to give greenish yellow gas. When excess of this gas
acts as strongest reducing agent? reacts with NH3, an unstable trihalide is formed. In this
Compound NH 3 PH 3 AsH 3 SbH 3 process, the oxidation state of nitrogen changes from ...........
(a) – 3 to + 3 (b) – 3 to 0
D diss ( E — H ) / kJ mol-1 389 322 297 255 (c) – 3 to + 5 (d) 0 to – 3
(a) NH3 (b) PH3 22. In the preparation of compounds of Xe, Bartlett had taken
-
(c) AsH3 (d) SbH3 O+2 Pt F6 as a base compound. This is because
10. On heating with concentrated NaOH solution in an inert (a) both O2 and Xe have same size.
atmosphere of CO2, white phosphorus gives a gas. Which (b) both O2 and Xe have same electron gain enthalpy.
of the following statement is incorrect about the gas?
(c) both O2 and Xe have almost same ionisation enthalpy.
(a) It is highly poisonous and has smell like rotten fish
(d) both Xe and O2 are gases.
(b) It's solution in water decomposes in the presence of
light 23. In solid state PCl5 is a ................. .
(c) It is more basic than NH3 (a) covalent solid
(d) It is less basic than NH3 (b) octahedral structure
11. Which of the following acids forms three series of salts? (c) ionic solid with [PCl6]+ octahedral and [PCl 4]–
(a) H3PO2 (b) H3BO3 tetrahedral
(c) H3PO4 (d) H3PO3 (d) ionic solid with [PCl4]+ tetrahedral and [PCl 6]–
12. Strong reducing behaviour of H3PO2 is due to octahedral
(a) low oxidation state of phosphorus 24. In qualitative analysis when H2S is passed through an
(b) presence of two — OH groups and one P — H bond aqueous solution of salt acidified with dil. HCl, a black
(c) presence of one — OH group and two P — H bonds precipitate is obtained. On boiling the precipitate with
(d) high electron gain enthalpy of phosphorus dil. HNO3, it forms a solution of blue colour. Addition of
13. On heating lead nitrate forms oxides of nitrogen and lead. excess of aqueous solution of ammonia to this solution
The oxides formed are ................. . gives ............ .
(a) N2O, PbO (b) NO2, PbO (a) deep blue precipitate of Cu(OH)2
(c) NO, PbO (d) NO, PbO2 (b) deep blue solution of [Cu(NH3)4]2+
14. Which of the following elements does not show allotropy? (c) deep blue solution of Cu(NO3)2
(a) Nitrogen (b) Bismuth (d) deep blue solution of Cu(OH)2. Cu(NO3)2
(c) Antimony (d) Arsenic
25. In a cyclotrimetaphosphoric acid molecule, how many
15. Maximum covalency of nitrogen is ................... .
single and double bonds are present?
(a) 3 (b) 5 (c) 4 (d) 6
(a) 3 double bonds; 9 single bonds
16. A brown ring is formed in the ring test for NO3- ion. It is
(b) 6 double bonds; 6 single bonds
due to the formation of
(c) 3 double bonds; 12 single bonds
(a) [Fe (H2O)5 (NO)]2+ (b) FeSO4 . NO2
(c) [Fe (H2O)4 (NO)2]2+ (d) FeSO4 . HNO3 (d) Zero double bond; 12 single bonds
26. Which of the following statements is wrong?
17. Elements of group- 15 form compounds in +5 oxidation
(a) Single N — N bond is stronger than the single P — P
state. However, bismuth forms only one well characterised
bond.
compound in +5 oxidation state. The compound is
(b) PH3 can act as a ligand in the formation of coordination
(a) Bi2O5 (b) BiF5 (c) BiCl5 (d) Bi2S5
18. On heating ammonium dichromate and barium azide compounds with transition elements.
separately we get (c) NO2 is paramagnetic in nature.
(a) N2 in both the cases (d) Covalency of nitrogen in N2O3 is four.
(b) N2 with ammonium dichromate and NO with barium Past Year MCQs
azide
(c) N2O with ammonium dichromate and N2 with barium 27. Acidity of diprotic acids in aqueous solutions increases in
azide the order : [AIPMT 2014, C]
(d) N2O with ammonium dichromate and NO2 with barium (a) H2S < H2Se < H2Te (b) H2Se < H2S < H2Te
azide (c) H2Te < H2S < H2Se (d) H2Se < H2Te < H2S
19. Which of the following is not tetrahedral in shape? 28. Among the following oxoacids, the correct decreasing
(a) NH+4 (b) SiCl4 order of acid strength is: [JEE M 2014, C]
(c) SF4 (d) SO42– (a) HOCl > HClO2 > HClO3 > HClO4
20. Hot conc. H2SO4 acts as moderately strong oxidising
agent. It oxidises both metals and non-metals. Which of (b) HClO4 > HOCl > HClO 2 > HClO3
the following element is oxidised by conc. H2SO4 into two
(c) HClO4 > HClO3 > HClO2 > HOCl
gaseous products?
(a) Cu (b) S (c) C (d) Zn (d) HClO2 > HClO 4 > HClO3 > HOCl
EBD_8350
316 CHEMISTRY

29. Strong reducing behaviour of H3PO2 is due to 39. Match the compounds given in column I with the
[AIPMT 2015 RS, C] hybridisation and shape given in column II and mark the
(a) presence of one –OH group and two P–H bonds correct option. [NEET 2016, C]
(b) high electron gain enthalpy of phosphorus Column-I Column-II
(c) high oxidation state of phosphorus 1. XeF6 (i) Distorted octahedral
(d) presence of two –OH groups and one P–H bond. 2. XeO3 (ii) Square planar
30. Nitrogen dioxide and sulphur dioxide have some properties 3. XeOF4 (iii) Pyramidal
in common. Which property is shown by one of these 4. XeF4 (iv) Square pyramidal
compounds, but not by the other? [AIPMT 2015, C] Code :
(a) is a reducing agent 1 2 3 4
(a) (i) (iii) (iv) (ii)
(b) is soluble in water
(b) (i) (ii) (iv) (iii)
(c) is used as a food-preservative
(c) (iv) (iii) (i) (ii)
(d) forms 'acid-rain'
(d) (iv) (i) (ii) (iii)
31. Which of the statements given below is incorrect?
40. The pair in which phosphorous atoms have a formal
[AIPMT 2015 RS, C]
oxidation state of + 3 is : [JEE M 2016, C]
(a) Cl2O7 is an anhydride of perchloric acid
(a) Orthophosphorous and hypophosphoric acids
(b) O3 molecule is bent (b) Pyrophosphorous and pyrophosphoric acids
(c) ONF is isoelectronic with O2N–. (c) Orthophosphorous and pyrophosphorous acids
(d) OF2 is an oxide of fluorine (d) Pyrophosphorous and hypophosphoric acids
32. The variation of the boiling point of the hydrogen halides 41. In which pair of ions, both the species contain
is in the order HF > HI > HBr > HCl. [AIPMT 2015 RS, C] S – S bond? [NEET 2017, S]
What explains the higher boiling point of hydrogen fluoride?
(a) The electronegativity of fluorine is much higher than (a) S4 O62- ,S2 O32 - (b) S2 O72- ,S2 O82 -
for other elements in the group.
(c) S4 O62- ,S2 O72 - (d) S2 O72- ,S2 O32 -
(b) There is strong hydrogen bonding between HF
molecules 42. Match the interhalogen compounds of column-I with the
(c) The bond energy of HF molecules is greater than in geometry in column II and assign the correct code.
other hydrogen halides. [NEET 2017, C]
(d) The effect of nuclear shielding is much reduced in Column-I Column-II
fluorine which polarises the HF molecule. 1. XX' (i) T-shape
33. The ionic radii (in Å) of N3–, O2– and F– are respectively : 2. XX'3 (ii) Pentagonal bipyramidal
[JEEM 2015, C] 3. XX'5 (iii) Linear
(a) 1.71, 1.40 and 1.36 (b) 1.71, 1.36 and 1.40 4. XX'7 (iv) Square pyramidal
(c) 1.36, 1.40 and 1.71 (d) 1.36, 1.71 and 1.40 (v) Tetrahedral
Code :
34. Which among the following is the most reactive ?
1 2 3 4
[JEEM 2015, C]
(a) (iii) (i) (iv) (ii)
(a) I2 (b) IC1 (c) Cl2 (d) Br2
(b) (v) (iv) (iii) (ii)
35. Which one has the highest boiling point?
(c) (iv) (iii) (ii) (i)
[JEE M 2015, C]
(d) (iii) (iv) (i) (ii)
(a) Kr (b) Xe (c) He (d) Ne
43. The products obtained when chlorine gas reacts with cold
36. Which is the correct statement for the given acids?
and dilute aqueous NaOH are : [JEE M 2017, S]
[NEET 2016, C] (a) ClO– and ClO3– (b) ClO2– and ClO3–
(a) Phosphinic acid is a diprotic acid while phosphonic (c) Cl– and ClO– (d) Cl– and ClO2–
acid is a monoprotic acid 44. The correct order of N-compounds in its decreasing order
(b) Phosphinic acid is a monoprotic acid while of oxidation states is [NEET 2018,
phosphonic acid is a diprotic acid C]
(c) Both are triprotic acids (a) HNO3, NO, N2, NH4Cl
(d) Both are diprotic acids (b) HNO3, NO, NH4Cl, N2
37. The product obtained as a result of a reaction of nitrogen (c) NH4Cl, N2, NO, HNO3
with CaC2 is [NEET 2016, C] (d) HNO3, NH4Cl, NO, N2
(a) CaCN2 (b) CaCN (c) CaCN3 (d) Ca2CN 45. Which of the following statements is not true for
38. Which one of the following orders is correct for the bond halogens? [NEET 2018, C]
dissociation enthalpy of halogen molecules? (a) All form monobasic oxyacids
[NEET 2016, C] (b) All are oxidizing agents
(a) I2 > Br2 > Cl2 > F2 (b) Cl2 > Br2 > F2 > I2 (c) Chlorine has the highest electron-gain enthalpy
(c) Br2 > I2 > F2 > Cl2 (d) F2 > Cl2 > Br2 > I2 (d) All but fluorine shows positive oxidation states
THE p-BLOCK ELEMENTS (GROUP 15, 16, 17 AND 18) 317

46. The compound that does not produce nitrogen gas by the (a) NO2 < NO < N2O3 < N2O
thermal decomposition is : [JEE M 2018, A]
(b) NO2 < N2O3 < NO < N2O
(a) Ba(N3)2 (b) (NH4)2Cr2O7
(c) NH4NO2 (d) (NH4)2SO4 (c) N2O < N2O3 < NO < NO2
47. Which is the correct thermal stability order for H2E (E = O, (d) N2O < NO < N2O3 < NO2
S, Se, Te and Po) ? [NEET 2019, C] 50. Which of the following oxoacid of sulphur has – O – O –
(a) H2S < H2O < H2Se < H2Te < H2Po linkage? [NEET 2020, C]
(b) H2O < H2S < H2Se < H2Te < H2PO (a) H2SO4, sulphuric acid
(c) H2Po < H2Te < H2Se < H2S < H2O (b) H2S2O8, peroxodisulphuric acid
(d) H2Se < H2Te < H2Po < H2O < H2S (c) H2S2O7, pyrosulphuric acid
48. Match the following: [NEET 2019, C] (d) H2SO3, sulphurous acid
(a) Pure nitrogen (i) Chlorine 51. Urea reacts with water to form A which will decompose to
(b) Haber process (ii) Sulphuric acid form B. B when passed through Cu2+ (aq), deep blue colour
(c) Contact proces (iii) Ammonia solution C is formed. What is the formula of C from the
(d) Deacon’s process (iv) Sodium azide or following ? [NEET 2020, S]
Barium azide 2+
(a) [Cu(NH3)4] (b) Cu(OH)2
Which of the following is the correct option ?
(a) (b) (c) (d) (c) CuCO3 Cu(OH)2 (d) CuSO4
(a) (i) (ii) (iii) (iv) 52. The electron gain enthalpy (in kJ/mol) of fluorine, chlorine,
(b) (ii) (iv) (i) (iii) bromine and iodine, respectively, are: [JEE M 2020, C]
(c) (iii) (iv) (ii) (i) (a) –296, –325, –333 and –349
(d) (iv) (iii) (ii) (i) (b) –349, –333, –325 and –296
49. The correct order of the oxidation states of nitrogen in (c) –333, –349, –325 and –296
(d) –333, –325, –349 and –296
NO, N2O, NO2 and N2O3 is: [JEE M 2019, S]

Exercise 4 : Problem Solving Skill Enhancer MCQs


1. When SO2 gas is passed through an acidified solution of 5. A greenish yellow gas reacts with an alkali metal hydroxide
K2Cr2O7 to form a halate which can be used in fireworks and safety
(a) the solution becomes blue matches. The gas and the halate are
(b) the solution becomes colourless (a) Br2, KBrO3 (b) Cl2, KClO3
(c) SO2 is reduced (c) I2, NaIO3 (d) I2, KIO3
6. In the solid state, SO3 may have structure
(d) green Cr2(SO4)3 is formed
2. NH 4 ClO4 + HNO 3 (dil.) ¾¾® HClO 4 + [X] O O
D S
[X] ¾¾
® Y(g)
O O
[X] and [Y] are respectively – (a) O O
(a) NH4NO3 & N2O (b) NH4NO2 & N2 S S
O O O
(c) HNO4 & O2 (d) None of these
3. A metal ‘M’ reacts with nitrogen gas to give ‘M3N’. ‘M3N’ O O O
on heating at high temperature gives back ‘M’ and on
reaction with water produces a gas ‘B’. Gas ‘B’ reacts with (b) S S S
aqueous solution of CuSO4 to form a deep blue compound. O O O O O O
‘M’ and ‘B’ respectively are : (c) a & b both (d) None of these
(a) Li and NH3 (b) Ba and N2 7. A certain salt (X) gives the following tests :
(c) Na and NH3 (d) Al and N2 (i) Its aqueous solution is alkaline to litmus.
4. An inorganic compound reacts with SO2 in aqueous (ii) On strongly heating it swells to give a glassy bead
medium, produces (A). (A) on reaction with Na2CO3 gives (iii) When concentrated sulphuric acid is added to a hot
the compound (B) which with sulphur give a substance concentrated solution of (X), crystals of H3BO3
(C) used in photography. The compound (C) is separate out. Identify the colour of these crystals.
(a) Na2S2O3 (b) Na2SO4 (a) White (b) Blue
(c) Na2S (d) Na2S2O7 (c) Brown (d) Violet
EBD_8350
318 CHEMISTRY

8. Ordinary strong solution of HCl, HNO3 and H2SO4 (a) K2CrO3 (b) KI (c) KBr (d) K3PO4
contains roughly
(a) 1/5, 2/3 and 3/3 fractions of pure acid respectively 12. (
® ' A ' M + = Alkali metal cation
MF + XeF4 ¾¾ )
(b) 2/3, 1/5 and 3/3 fractions of pure acid respectively The state of hybridisation of the central atom in 'A' and
(c) 2/3, 3/3 and 1/5 fractions of pure acid respectively shape of the species are:
(d) None (a) sp3d, TBP
9. The correct order of S—S bond length in following (b) sp3d3, distorted octahedral
oxyanions is: (c) sp3d3, pentagonal planar
(d) No compound formed at all
(I) S2 O24- (II) S2 O52- (III) S2 O62- 13. All the three atoms of ozone are used up when it reacts
with
(a) I > II > III (b) I > III > II (a) H2O2 (b) PbS (c) KI (d) SO2
(c) III > II > I (d) III > I > II 14. P2H4 can be removed from phosphine containing traces of
10. A yellow metallic powder when burnt in a stream of fluorine it :
produced a colourless, thermally stable and chemically inert (a) by passing impure PH3 gas through a freezing mixture
gas ‘X’. A gas ‘Y’, which is colourless and consists of the (b) by passing the impure PH3 gas through HI and then
same elements as are present in gas ‘X’, is obtained by its treatment with KOH (aq)
heating together sulphur dichloride and sodium fluoride. (c) by both (a) and (b)
Gases ‘X’ and ‘Y’ respectively are (d) by none of these
(a) SF4 and SF6 (b) SF4 and S2F2 15. Amatol that is used as an explosive has the composition
(c) SF6 and SF4 (d) None of these (a) 80% of NH4NO3 + 20% TNT
11. A certain compound (X) when treated with copper (b) 20% of NH4NO3 + 80% TNT
sulphate solution, yields a brown precipitate. On adding (c) 30% of NH4NO2 + 20% TNT
hypo solution the precipitate turns white. The compound (d) 20% of NH4NO2 +80% TNT
is
ANSW ER KEY
Exercise 1 : NCERT Based Topic-wise MCQs
1 (d) 18 (d) 35 (d) 52 (a) 69 (b) 86 (c) 103 (d) 120 (d) 137 (b) 154 (d)
2 (b) 19 (b) 36 (b) 53 (b) 70 (a) 87 (b) 104 (d) 121 (b) 138 (a) 155 (c)
3 (a) 20 (c) 37 (d) 54 (a) 71 (a) 88 (b) 105 (b) 122 (d) 139 (a) 156 (d)
4 (d) 21 (d) 38 (c) 55 (b) 72 (b) 89 (d) 106 (b) 123 (d) 140 (a) 157 (a)
5 (a) 22 (b) 39 (d) 56 (b) 73 (c) 90 (d) 107 (b) 124 (d) 141 (c) 158 (a)
6 (d) 23 (a) 40 (a) 57 (b) 74 (b) 91 (c) 108 (c) 125 (a) 142 (b) 159 (c)
7 (a) 24 (a) 41 (a) 58 (a) 75 (a) 92 (c) 109 (d) 126 (b) 143 (b) 160 (b)
8 (c) 25 (b) 42 (b) 59 (a) 76 (c) 93 (c) 110 (b) 127 (a) 144 (c) 161 (b)
9 (c) 26 (b) 43 (d) 60 (c) 77 (a) 94 (a) 111 (d) 128 (c) 145 (d) 162 (c)
10 (d) 27 (d) 44 (b) 61 (d) 78 (b) 95 (d) 112 (d) 129 (d) 146 (b) 163 (c)
11 (c) 28 (a) 45 (c) 62 (d) 79 (a) 96 (a) 113 (d) 130 (d) 147 (b) 164 (c)
12 (b) 29 (c) 46 (c) 63 (a) 80 (c) 97 (c) 114 (c) 131 (d) 148 (b) 165 (b)
13 (a) 30 (b) 47 (d) 64 (c) 81 (d) 98 (d) 115 (b) 132 (b) 149 (a)
14 (c) 31 (d) 48 (b) 65 (b) 82 (a) 99 (a) 116 (d) 133 (d) 150 (a)
15 (a) 32 (a) 49 (b) 66 (b) 83 (d) 100 (d) 117 (c) 134 (b) 151 (c)
16 (b) 33 (b) 50 (c) 67 (a) 84 (b) 101 (b) 118 (c) 135 (c) 152 (d)
17 (c) 34 (a) 51 (b) 68 (c) 85 (d) 102 (b) 119 (c) 136 (c) 153 (a)
Exercise 2 : Numeric/Integer Answer Questions
1 (16) 2 (3) 3 (3) 4 (10) 5 (3) 6 (0) 7 (3) 8 (6) 9 (3) 10 (3)
Exercise 3 : NCERT Exemplar & Past Year MCQs
1 (a) 7 (c) 13 (b) 19 (c) 25 (c) 31 (d) 37 (a) 43 (c) 49 (d)
2 (a) 8 (a) 14 (a) 20 (c) 26 (a) 32 (b) 38 (b) 44 (a) 50 (b)
3 (c) 9 (d) 15 (c) 21 (a) 27 (a) 33 (a) 39 (a) 45 (N) 51 (a)
4 (c) 10 (c) 16 (a) 22 (c) 28 (c) 34 (b) 40 (c) 46 (d) 52 (c)
5 (b) 11 (c) 17 (b) 23 (d) 29 (a) 35 (b) 41 (a) 47 (c)
6 (c) 12 (c) 18 (a) 24 (b) 30 (c) 36 (b) 42 (a) 48 (d)
Exercise 4 : Problem Solving Skill Enhancer MCQs
1 (d) 3 (a) 5 (b) 7 (a) 9 (a) 11 (b) 13 (d) 15 (a)
2 (a) 4 (a) 6 (a) 8 (a) 10 (c) 12 (c) 14 (c)
The d-and f-Block
22 Element
Trend Buster NEET & JEE Main

Number of Questions from 2020-15 12 6 This chapter has more weightage


Weightage 3.8% 2.9% for NEET than JEEM.

The most Important Concepts that Cover Maximum number of Questions asked in past 6 years.

General Properties & Electronic Configuration of 6 3


d-Block Elements
Some Important Compounds of Transition Elements 3 —
The Lanthanoids and Actinoids 3 1

Less Important Concepts that Cover 1 or 2 Questions asked in past 6 years.

Some Application of d-Block and f-Block Elements — 2

NEET JEE
Concept Used

2020 The Lanthanoids and Lanthanoids Contraction / 1 Easy 1 Easy


Actinoids / General properties General properties of
of d-block elements d-block elements
2019 Some Application of d-Block Catalytic Properties of Compounds — — 2 Easy
and f-Block Elements / of d-Block Elements / Magnetic
General Properties & Properties
Electronic Configuration of
d-Block Elements
2018 General Properties & d-d Transition / Magnetic 2 Average — —
Electronic Configuration Properties
of d-Block Elements
2017 Some Important Compounds Properties of KMnO 4 / Oxidation 2 Average — —
of Transition Elements / The State of Actinoids
Lanthanoids and Actinoids
2016 Some Important Compounds Chemical properties of K 2Cr2O7 / 2 Average 1 Average
of Transition Elements / The Electronic Configuration of
Lanthanoids and Actinoids / lanthanoids / Magnetic
General Properties & Properties
Electronic Configuration of
d-Block Elements
2015 General Properties & Magnetic Properties / Electronic 5 Easy / 2 Average /
Electronic Configuration of Configuration of Iron / KMnO4 Average Difficult
d-Block Elements / Some (Oxidising Properties) /
Important Compounds of Lanthanoids Contraction /
Transition Elements / The Catalytic Properties of
Lanthanoids and Actinoids / compounds of d-Block
DTP : MANOJ
Some Application of d-Block Elements / CT Spectra of KMnO 4 3rd Print
and f-Block Elements
EBD_8350
320 CHEMISTRY
THE d-AND f-BLOCK ELEMENT 321
EBD_8350
322 CHEMISTRY

Problem Solving Tips/ Tricks/ Points to Remember


4 Fe3+ is more stable than Fe2+, hence Fe2+ act as reducing Maximum melting point : Cr, Mo and W in 3d, 4d and 5d
agent. Cr3+ is more stabe than Cr2+, hence Cr2+ act as series respectively.
reducing agent. Mn2+ is more stable than Mn3+, hence 4 The enthalpy of vapourisation is maximum for V, then
Mn3+ act as oxidising agent. decreases for Cr and minimum for Mn in 3d-series
4 At pH about 4, dichromate ion ( Cr2 O 72 - ) and chromate (except Zn).
exist in equilibrium. These are interconvertible.
4 The oxides of metals in the intermediate oxidation states,
4 Most common oxidation states of 3d-series metals :
are generally amphoteric. For example, CuO, Cr2O3, MnO2 Sc (+3); Ti (+4); V (+5); Cr (+3, +6);
etc. are amphoteric. Mn (+2, +7); Fe (+2, +3); Co (+2, +3);
4 Potassium permanganate (KMnO4) is the salt of an Ni (+2); Cu (+2) and Zn (+2)
unstable acid HMnO4 (permanganic acid). 4 In 3d-series, halide with highest oxidation number is CrF6
4 When a solid chloride salt is heated with potassium but MnF7 is not known. Similarly CrO3 and Mn2O7 are
dichromate and conc. H2SO4, orange coloured vapours known compounds.
of chromyl chloride is obtained. 4 Volumetric titrations with KMnO4 are done only in the
4 Trivalent ions of lanthanoids (except La3+ and Lu3+) are
presence of dil. H2SO4 but not in the presence of HCl or
paramagnetic.
4 Direct consequences of the lanthanoid contraction are HNO3 because HCl is oxidised to Cl2 and HNO3 is itself
(a) almost equal size of Hf 4+ and Zr4+. a strong oxidising agent.
(b) decrease in the basic character of lanthanoid 4 In volumetric titrations, K2Cr2O7 is a primary and KMnO4
hydroxides with increase in the atomic number. is a secondary standard.
(c) La(OH)3 is the most basic, while Lu(OH)3 is least 4 KMnO4 is a stronger oxidising agent than K2Cr2O7 in
basic. acidic medium.
4 TiCl4 and TiO2 are used in smoke screens.
4 The first synthetic element is Tc (Z = 43) (technetium ° °
EMnO - 2 + = +1.51 V, E
Cr2O72 - /Cr3+ = +1.33 V
4 /Mn
means artificial).
4 Alnico which is an alloy of Al (12%) , Co (50%), Ni (20%) 4 Except promethium, lanthanoids are non-radioactive.
and rest iron is used to make permanent magnets. 4 Some important electronic configurations:
4 Lanthanoids show limited number of O.S. because there 1 2 1 2
is a large energy gap between 4f and 5d subshells. 57La : [Xe] 4f ° 5d 6s ; 89Ac : [Rn] 5f ° 6d 7s ;

4 Lanthanoids are used in the production of alloy steel 90Th: [Rn] 5f ° 6d 2 7s2; 90Th3+ : [Rn] 5f 1
known as mischmetal, which is used in making Mg- 4 The most stable oxidation state of thorium is +4.
based alloy, known as pyrophoric alloys. 4 Applications of d- and f -block elements:
4 Equivalent weight of KMnO4 in different medium. (i) TiO in pigment industry, (ii) MnO2 in dry battery cells,
(a) Equivalent weight in acid medium (iii) Zn, Ni, Cd in battery industry, (iv) TiCl4 with Al(CH3)3
= Molecular weight /5 to prepare Ziegler catalysts, (v) catalysts such as PdCl2
(b) Equivalent weight in alkaline medium in Wacker process.
= Molecular weight /1 4 Color of KMnO4: The purple color in KMnO4 arises from
(c) Equivalent weight in neutral medium
an electronic transition, but it is actually not a d-d
= Molecular weight /3
transition, since the Mn in this compound has no d
4 The magnetic moment is calculated by the number of electron (Mnn+7 Þ 3d° 4s2). It arises from a charge transfer
unpaired electrons and by spin only formula. reaction within the molecule, in which photons promote
µ = n ( n + 2 ) BM (Bohr magneton) an electron from the highest energy molecular orbital in
one of the Mn-O bonds to an empty d-orbital on the Mn.
4 In any transition series, the melting point of metals rises
Therefore, charge transfer occurs from ligand (O2–) to
to a maximum at middle except for anomalous low values the metal atom (L ® m).
of Mn, Tc and Re in 3d, 4d and 5d series respectively.
THE d-AND f-BLOCK ELEMENT 323

Exercise 1 : NCERT Based Topic-wise MCQs


Topic 1: General Properties and Electronic 15. Which of the following transition element shows the
highest oxidation state?
Configuration of d–Block Elements (a) Mn (b) Fe (c) V (d) Cr
1. The transition elements have a general electronic configu- 16. Which of the following ions has the maximum magnetic
ration moment?
(a) ns2, np6, nd1–10 (a) Mn 2+ (b) Fe2+ (c) Ti2+ (d) Cr2+
(b) (n – 1)d1 – 10, ns0 – 2, np0 – 6 17. A compound of a metal ion Mx+ (Z = 24) has a spin only
(c) (n – 1)d1 – 10, ns1 – 2 magnetic moment of 15 Bohr Magnetons. The number
(d) nd1 – 10, ns1 – 2 of unpaired electrons in the compound are
2. Number of unpaired electrons in Ni 2+(Z = 28) is (a) 2 (b) 4 (c) 5 (d) 3
(a) 4 (b) 2 (c) 6 (d) 8 18. A gas metal in bivalent state has approximately 23e–, what
3. Which of the following element is not a member of transition
elements ? is spin magnetic moment in elemental state?
(a) Hf (b) Pt (c) Ce (d) Mo (a) 2.87 (b) 5.5 (c) 5.9 (d) 4.9
4. The number of unpaired electrons in gaseous species of 19. Transition elements show magnetic moment due to spin
Mn3+, Cr 3+ and V3+ respectively are. and orbital motion of electrons. Which of the following
(a) 4, 3 and 3 (b) 3, 3 and 2 metallic ions have almost same spin only magnetic
(c) 4, 3 and 2 (d) 3, 3 and 3 moment ?
5. Which of the following has most unpaired d-electrons? (i) Co2+ (ii) Cr2+ (iii) Mn 2+ (iv) Cr3+
(a) Zn+ (b) Fe2+ (c) Ni+ (d) Cu+ (a) (i) and (iii) (b) (i) and (iv)
6. Correct electronic configuration of Cr (Z = 24) is (c) (ii) and (iii) (d) (ii) and (iv)
(a) 1s2 2s2 2p6 3s2 3p6 3d 7 4s1 20. The aqueous solution containing which one of the
(b) 1s2 2s2 2p6 3s2 3p6 3d 5 4s1 following ions will be colourless? (Atomic number:
(c) 1s2 2s2 2p6 3s2 3p6 3d 4 4s2 Sc = 21, Fe = 26, Ti = 22, Mn = 25)
(d) 1s2 2s2 2p6 3s2 3p6 3d 6 4s2 (a) Sc3+ (b) Fe2+ (c) Ti 3+ (d) Mn 2+
7. Which one of the following ions has electronic 21. Transition elements form coloured ions due to
configuration [Ar] 3d 6 ? (a) d-d transition
(b) fully filled d-orbitals
(a) Ni3+ (b) Mn 3+ (c) Fe3+ (d) Co3+
(c) smaller atomic radii
(At. Nos. Mn = 25, Fe = 26, Co = 27, Ni = 28)
(d) availability of s-electrons
8. (n–1) d10 ns2 is the general electronic configuration of
22. The catalytic activity of transition metals and their
(a) Fe, Co, Ni (b) Cu, Ag, Au compounds is mainly due to
(c) Zn, Cd, Hg (d) Se, Y, La (a) their magnetic behaviour
9. Which one of the following is an example of non-typical (b) their unfilled d-orbitals
transition elements ? (c) their ability to adopt variable oxidation state
(a) Li, K, Na (b) Be, Al, Pb (d) their chemical reactivity
(c) Zn, Cd, Hg (d) Ba, Ga, Sr. 23. Assertion : Transition metals are good catalysts.
10. Which of the following has the maximum number of Reason : V2O5 or Pt is used in the preparation of H2SO4 by
unpaired electrons? contact process.
(a) Ti2+ (b) Fe2+ (c) Cr+ (d) Cu+ (a) Assertion is correct, reason is correct; reason is a
11. Maximum oxidation state is shown by correct explanation for assertion.
(a) Os (b) Mn (c) Co (d) Cr (b) Assertion is correct, reason is correct; reason is not
12. Which form coloured salts : a correct explanation for assertion
(a) Non-metals (b) Metals (c) Assertion is correct, reason is incorrect
(c) p-block elements (d) Transitional elements (d) Assertion is incorrect, reason is correct.
13. The number of elements present in the d-block of the 24. Which of the following transition metal ion is colourless in
periodic table is aqueous solution?
(a) 40 (b) 41 (c) 45 (d) 46 (a) Ti4+ (b) Zn 2+
14. Zinc and mercury do not show variable valency like (c) V4+ (d) Both (a) and (b)
d-block elements because 25. Among the following series of transition metal ions, the
(a) they are soft one where all metal ions have 3d2 electronic configuration
(b) their d-shells are complete is (At. nos. Ti = 22; V = 23; Cr = 24; Mn = 25)
(c) they have only two electrons in the outermost (a) Ti 3+ , V 2+ , Cr 3+ , Mn 4+ (b) Ti + , V 4+ , Cr 6+ , Mn 7+
subshell 2+ 3+ 4+ 5+
(d) their d-shells are incomplete (c) Ti 4+ , V 3+ , Cr 2+ , Mn 3+ (d) Ti , V , Cr , Mn
EBD_8350
324 CHEMISTRY

26. In which of the following pairs both the ions are coloured (At. no. Ti = 22, V = 23, Cr = 24, Mn = 25)
in aqueous solutions? 38. Which of the following arrangements does not represent
(a) Sc3+, Ti3+ (b) Sc3+, Co2+ the correct order of the property stated against it ?
2+
(c) Ni , Cu + (d) Ni2+, Ti3+ (a) V2+ < Cr2+ < Mn2+ < Fe2+ : paramagnetic behaviour
(At. no. : Sc = 21, Ti = 22, Ni = 28, Cu = 29, Co = 27) (b) Ni2+ < Co2+ < Fe2+ < Mn2+ : ionic size
27. Cuprous ion is colourless while cupric ion is coloured (c) Co3+ < Fe3+ < Cr3+ < Sc3+ : stability in aqueous solution
because (d) Sc < Ti < Cr < Mn : number of oxidation states
(a) both have half filled p-and d-orbitals
39. Which one of the following does not correctly represent
(b) cuprous ion has incomplete d-orbital and cupric ion
the correct order of the property indicated against it?
has a complete d-orbital
(a) Ti < V < Cr < Mn : increasing number of oxidation states
(c) both have unpaired electrons in the d-orbitals
(d) cuprous ion has complete d-orbital and cupric ion (b) Ti3+ < V3+ < Cr3+ < Mn3+ : increasing magnetic moment
has an imcomplete d-orbital. (c) Ti < V < Cr < Mn : increasing melting points
28. Which of the following species is/are paramagnetic? (d) Ti < V < Mn < Cr : increasing 2nd ionization enthalpy
Fe2+, Zn0, Hg2+, Ti4+ 40. Which of the following ion(s) is/are oxidising in nature?
(a) Fe2+ only
(c) Fe2+ and Hg2+
(b) Zn 0 and Ti4+
(d) Zn 0 and Hg2+
(i) ( °
V2+ EM 2+ / M = -1.18 )
29. In first transition series, the melting point of Mn is low because
(a) due to d10 configuration, metallic bonds are strong (ii) Mn3+ E° ( M3+ / M 2 +
= +1.57 )
(b) due to d7 configuration, metallic bonds are weak
(c) due to d5 configuration, metallic bonds are weak
(d) None of these
(iii) Cr 2+ E°( M2+ / M
= -0.91 )
30. Assertion : Transition metals show variable valency. (a) (i) and (iii) (b) only (ii)
Reason : Transition metals have a large energy difference (c) (ii) and (iii) (d) only (iii)
between the ns2 and (n – 1)d electrons. 41. Which of the following ions will exhibit colour in aqueous
(a) Assertion is correct, reason is correct; reason is a solutions?
correct explanation for assertion. (a) La3+ (Z = 57) (b) Ti3+ (Z = 22)
3+
(c) Lu (Z = 71) (d) Sc3+ (Z = 21)
(b) Assertion is correct, reason is correct; reason is not
a correct explanation for assertion 42. Mark the correct statement(s).
(c) Assertion is correct, reason is incorrect (i) Manganese exhibits +7 oxidation state
(d) Assertion is incorrect, reason is correct. (ii) Zinc forms coloured ions
31. The common oxidation states of Ti are (iii) [CoF6]3– is diamagnetic
(a) + 2 and + 3 (b) + 3 and + 4 (iv) Sc forms +4 oxidation state
(c) – 3 and – 4 (d) + 2, + 3 and + 4 (v) Zn exhibits only +2 oxidation state
32. Which of the following pairs has the same size? (a) (i) and (ii) (b) (i) and (v)
(a) Fe2+, Ni2+ (b) Zr4+, Ti4+ (c) (ii) and (iv) (d) (iii) and (iv)
4+
(c) Zr , Hf 4+ (d) Zn 2+, Hf 4+ 43. Which of the following statements are correct?
33. For the four successive transition elements (Cr, Mn, Fe (i) The maximum oxidation state of Mn with the oxygen
and Co), the stability of +2 oxidation state will be there in is +VII while with fluorine is +IV.
which of the following order? (ii) Fluorine is more oxidizing in nature than oxygen.
(a) Mn > Fe > Cr > Co (b) Fe > Mn > Co > Cr (iii) Fluorine exhibit an oxidation state of –1.
(c) Co > Mn > Fe > Cr (d) Cr > Mn > Co > Fe
(iv) Seven fluorine cannot be accommodated around Mn.
34. Of the following outer electronic configurations of atoms,
(a) (i), (ii) and (iii) (b) (ii), (iii) and (iv)
the highest oxidation state is achieved by which one of them?
(c) (i) and (iv) (d) (i), (ii), (iii) and (iv)
(a) (n – 1)d 3 ns2 (b) (n – 1)d 5 ns1
8 2 44. For the ions Zn2+, Ni2+ and Cr3+ which among the following
(c) (n – 1)d ns (d) (n – 1)d 5 ns2
35. Which of the following does not represent the correct order statements is correct?
of the properties indicated ? (atomic number of Zn = 30, Ni = 28 and Cr = 24)
(a) Ni2+ > Cr2+ > Fe2+ > Mn2+ (size) (a) All these are colourless
(b) Sc > Ti > Cr > Mn (size) (b) All these are coloured
(c) Mn2+ > Ni2+ < Co2+ <Fe2+ (unpaired electron) (c) Only Ni2+ is coloured and Zn2+ and Cr3+ are colourless
(d) Fe2+ > Co2+ > Ni2+ > Cu2+ (unpaired electron) (d) Only Zn2+ is colourless and Ni2+ and Cr3+ are coloured
36. Which one of the following transition elements does not 45. Iron exhibits +2 and +3 oxidation states. Which of the
exhibit variable oxidation state? following statements about iron is incorrect?
(a) Ni (b) Cu (c) Fe (d) Sc (a) Ferrous oxide is more basic in nature than the ferric
37. Which one of the following ions is the most stable in oxide.
aqueous solution? (b) Ferrous compounds are relatively more ionic than the
(a) V3+ (b) Ti3+ (c) Mn 3+ (d) Cr3+ corresponding ferric compounds.
THE d-AND f-BLOCK ELEMENT 325

(c) Ferrous compounds are less volatile than the 54. A series-1 metal ion, M(II) aqueous solution react with
corresponding ferric compounds. the KI to form iodine and a precipitate is formed, this
(d) Ferrous compounds are more easily hydrolysed than M(II) can be:
the corresponding ferric compounds. (a) Zn2+ (b) Mn 2+ (c) Cu2+ (d) Ni2+
46. AgNO3 does not decompose in/on: 55. The colour of the following ions V , V , V4+, Fe2+, Fe3+
2+ 3+

(a) U.V. radiation (b) Skin (human) are respectively


(c) Water at 25°C (d) Glucose (a) green, violet, blue, green, yellow
47. On passing H2S gas in the following test tubes precipita- (b) yellow, green, violet, green, blue
(c) violet, green, yellow, green, blue
tion occurs. Find the correct match?
(d) violet, green, blue, green, yellow
56. Which group contains coloured ions out of
(i) Cu2+ (ii) Ti4+ (iii) Co2+ (iv) Fe2+
(a) (i), (ii), (iii), (iv) (b) (i), (iii), (iv)
(c) (ii), (iii) (d) (i), (ii)
57. Match the columns
Column-I Column-II
(Ion) (Mcalculated)
(A) Ti2+ (p) 2.84
(B) Zn2+ (q) 5.92
(C) Mn 2+ (r) 0
(D) Sc3+ (s) 4.90
Yellow Black Orange Brown
(a) A – (s), B – (p), C – (q), D – (r)
Cu, Sb, Zn, Pb, Sn, Ni (b) A – (r), B – (p), C – (q), D – (s)
(a) Cd – Black (b) Sb – Orange (c) A – (p), B – (r), C – (q), D – (s)
(c) Ni – Yellow (d) Zn – Brown (d) A – (p), B – (s), C – (q), D – (r)
48. For d block elements the first ionization potential is of the
Topic 2: Some Important Compounds
order
(a) Zn > Fe > Cu > Cr (b) Sc = Ti < V = Cr of Transition Elements
(c) Zn < Cu < Ni < Co (d) V > Cr > Mn > Fe 58. Choose the correct increasing order of the oxidation state
49. Which of the following statements is incorrect? of the central metal atom in the following oxoanions.
(a) Zn,Cd and Hg due to presence of completely filled
VO2+ , VO2 + ,TiO2 + ,CrO42 -
d-orbitals [(n–1)d10ns2] are not transition metals.
(b) Zn, Cd and Hg have low m.p and are comparitively (a) VO2 + ; VO2+ < TiO2 + < CrO 24-
softer than other transition metals. (b) VO2+ ; TiO 2+ < VO +2 < CrO 24-
(c) Metallic bond made by elements with d5 configuration
(c) CrO24 - < TiO2 + < VO2+ < VO2 +
is stronger as compared to metalic bond made by
elements with d3 configuration. (d) TiO2 + < VO2 + ; VO2+ < CrO 24-
(d) Metals of 5d series forms strong metallic bonds as 59. Match the columns
compared with metals of 3d series. Column-I Column-II
50. Which of the following is incorrect? (A) Metal of the 3d-series (p) Manganese
(a) Mn shows oxidation state of +7 in MnF7 which does not form
(b) Fe and Co shows +3 oxidation state in FeX3 and CoF3. MO type oxide.
(c) V shows oxidation state of + 5 in VF5. (B) Metal of the 3d-series (q) Vanadium
(d) Cu does not shows +2 oxidation state with I–. which forms most
covalent oxide.
51. Four successive members of the first row transition
(C) Metal of the 3d-series (r) Scandium
elements are listed below with atomic numbers. Which one which forms the
of them is expected to have the highest E ° 3+ 2 + value? amphoteric oxide.
M /M
(a) Cr (Z = 24) (b) Mn (Z = 25) (a) A – (p), B – (r), C – (q) (b) A – (r), B – (p), C – (q)
(c) Fe (Z = 26) (d) Co (Z = 27) (c) A – (r), B – (q), C – (p) (d) A – (q), B – (p), C – (r)
52. If a non metal is added to the interstital sites of a metal, 60. Acidified K2Cr2O7 solution turns green when Na2SO3 is
then the metal becomes added to it. This is due to the formation of :
(a) softer (b) less tensile (a) Cr2(SO4)3 (b) CrO42–
(c) less malleable (d) more ductile (c) CrO3 (d) CrSO4
53. Which of the following transition metal acts as a catalysis 61. Identify the product and its colour when MnO2 is fused
in the reaction between iodide and persulphate ion? with solid KOH in the presence of O2.
(a) Fe2+ (b) Fe3+ (a) KMnO4, purple (b) K2MnO4, dark green
(c) Ni 2+ (d) Both (a) and (c) (c) MnO, colourless (d) Mn2O3, brown
EBD_8350
326 CHEMISTRY

62. Arrange the following increasing order of acidic character? (b) Na2Cr2O7 is preferred over K2Cr2O7 in volumetric
Mn2O7(A), Mn2O3(B), MnO(C)? analysis.
(a) C, A, B (b) A, C, B (c) B, A, C (d) C, B, A (c) K2Cr2O7 solution in acidic medium is orange.
63. K2Cr2O7 on heating with aqueous NaOH gives (d) K2Cr2O7 solution becomes yellow on increasing the
pH beyond 7.
(a) CrO 24- (b) Cr(OH)3 (c) Cr2O 72- (d) Cr(OH)2
76. Which one of the following is an amphoteric oxide?
64. CrO3 dissolves in aqueous NaOH to give (i) Mn 2O7 (ii) CrO
(a) Cr2O72– (b) CrO42– (c) Cr(OH)3 (d) Cr(OH)2
(iii) V2O4 (iv) Cr2O3
65. The oxidation state of chromium in the final product formed
by the reaction between KI and acidified potassium (a) (i) and (ii) (b) (ii), (iii) and (iv)
dichromate solution is (c) (iii) and (iv) (d) (ii) and (iv)
(a) + 3 (b) + 2 (c) + 6 (d) + 4 77. Which of the following oxidising reaction of KMnO4 occurs
66. In acidic medium which of the following does not change in acidic medium?
(i) Fe2+ (green) is converted to Fe3+ (yellow).
its colour?
(ii) Iodide is converted to iodate.
(a) MnO4– (b) MnO42– (c) CrO42– (d) FeO42– (iii) Thiosulphate oxidised to sulphate.
67. The bonds present in the structure of dichromate ion are (iv) Nitrite is oxidised to nitrate.
(a) four equivalent Cr – O bonds only
(a) (i) and (iii) (b) (i) and (iv)
(b) six equivalent Cr – O bonds and one O – O bond
(c) (iv) only (d) (ii) and (iv)
(c) six equivalent Cr – O bonds and one Cr – Cr bond
(d) six equivalent Cr – O bonds and one Cr – O – Cr bond 78. Dichromate [Cr(VI) ] is a strong oxidizing agent whereas
68. Potassium dichromate when heated with concentrated Mo(VI) and W(VI) are found to be not. This is due to
sulphuric acid and a soluble chloride, gives brown-red (a) Lanthanoid contraction
vapours of (b) Down the group metallic character increases
(a) CrO3 (b) CrCl3 (c) CrO2Cl2 (d) Cr2O3 (c) Down the group metallic character decreases
69. Which of the following oxides of Cr is amphoteric (d) Both (a) and (b)
(a) Cr O2 (b) Cr2O3 (c) CrO5 (d) CrO3 79. In the form of dichromate, Cr (VI) is a strong oxidising
70. In neutral or faintly alkaline medium, thiosulphate is agent in acidic medium but Mo (VI) in MoO3 and W (VI) in
quantitatively oxidized by KMnO4 to WO3 are not because ____________ .
(a) SO32– (b) SO42– (c) SO2 (d) SO52– (i) Cr (VI) is more stable than Mo(VI) and W (VI).
71. In the laboratory, manganese (II) salt is oxidised to (ii) Mo (VI) and W(VI) are more stable than Cr(VI).
permanganate ion in aqueous solution by (iii) Higher oxidation states of heavier members of group-
(a) hydrogen peroxide (b) conc. nitric acid 6 of transition series are more stable.
(c) peroxy disulphate (d) dichromate
(iv) Lower oxidation states of heavier members of group-
72. The starting material for the manufacture of KMnO4 is
6 of transition series are more stable.
(a) pyrolusite (b) manganite
(a) (i) and (ii) (b) (ii) and (iii)
(c) magnatite (d) haematite
73. If KMnO4 is reduced by oxalic acid in an acidic medium (c) (i) and (iv) (d) (ii) and (iv)
then oxidation number of Mn changes from 80. The acidic, basic or amphoteric nature of Mn 2O7, V2O5
(a) 4 to 2 (b) 6 to 4 (c) 7 to 2 (d) 7 to 4 and CrO are respectively
74. Match the columns (a) acidic, acidic and basic
Column-I Column-II (b) basic, amphoteric and acidic
(A) Compound formed when (p) acidified (c) acidic, amphoteric and basic
yellow CrO24 - is acidified. MnO-4 (d) acidic, basic and amphoteric
81. KMnO4 can be prepared from K2MnO4 as per the reaction:
(B) reagent oxidises Fe2+ to Fe3+ (q) Cr2 O72-
3MnO 24 - + 2H 2 O 2MnO4 + MnO 2 + 4OH -
(C) Compound produced when (r) K2MnO4
MnO2 is fused with KNO3 The reaction can go to completion by removing OH– ions
(D) Compound having dark (s) KMnO4 and by adding.
purple crystals isostructural (a) KOH (b) CO2
with KClO4 (c) SO2 (d) HCl
(a) A – (q),B – (p), C – (r), D – (s) 82. Solution of oxalate is colourless. It is made acidic by adding
(b) A – (p),B – (q), C – (r), D – (s) excess of H+, then titrated with KMnO4. Now at a moment
(c) A – (q),B – (r), C – (p), D – (s) if someone has added large amount of KMnO4, in it then
(d) A – (q),B – (p), C – (s), D – (r) no. of possible products are
75. Which of the statements is not true? (a) CO2, Mn2+, H2O (b) CO2, MnO2, H2O
(a) On passing H2S through acidified K2Cr2O7 solution,
a milky colour is observed. (c) MnO2, H2O, CO2 (d) CO2, MnO2, H2O, Mn2+
THE d-AND f-BLOCK ELEMENT 327

83. Which of the following conversions can be carried out by (b) Ln (III) compounds are generally colourless.
both acidified K2Cr2O4 and acidified KMnO4? (c) Ln (III) hydroxide are mainly basic in character.
(d) Because of the large size of the Ln (III) ions the
(i) Fe2+ ® Fe3+ + e– (ii) I– ®
bonding in its compounds is predominantly ionic in
(iii) I – ® I2 (iv) H2S ® S character.
(a) (i) and (iii) (b) (ii) and (iv) 95. Assertion : Magnetic moment values of actinides are
(c) (i), (iii) and (iv) (d) (i), (ii) and (iii) lesser than the theoretically predicted values.
Reason : Actinide elements are strongly paramagnetic.
Topic 3: The Lanthanoids and Actinoids (a) Assertion is correct, reason is correct; reason is a
correct explanation for assertion.
84. A reduction in atomic size with increase in atomic number
is a characteristic of elements of (b) Assertion is correct, reason is correct; reason is not
(a) high atomic masses (b) d-block a correct explanation for assertion
(c) f-block (d) radioactive series (c) Assertion is correct, reason is incorrect
85. Which of the following oxidation states is the most common (d) Assertion is incorrect, reason is correct.
among the lanthanoids? 96. Lanthanum is grouped with f-block elements because
(a) 3 (b) 4 (c) 2 (d) 5 (a) it has partially filled f-orbitals
86. The increasing order of the shielding of electrons by the (b) it is just before Ce in the periodic table
orbitals ns,np,nd,nf is (c) it has both partially filled f and d-orbitals
(a) ns,np,nd,nf (b) np,ns,nd,nf (d) properties of lanthanum are very similar to the
(c) nd,nf,np,ns (d) nf,nd.np,ns elements of f-block
87. The approximate percentage of iron in mischmetal is 97. Identify the incorrect statement among the following:
(a) 10 (b) 20 (c) 50 (d) 5 (a) 4f and 5f orbitals are equally shielded.
88. Which of the following factors may be regarded as the (b) d-Block elements show irregular and erratic chemical
main cause of lanthanoid contraction? properties among themselves.
(a) Greater shielding of 5d electrons by 4f electrons (c) La and Lu have partially filled d-orbitals and no other
(b) Poorer shielding of 5d electrons by 4f electrons partially filled orbitals.
(c) Effective shielding of one of 4f electrons by another (d) The chemistry of various lanthanoids is very similar.
in the subshell 98. In context of the lanthanoids, which of the following
(d) Poor shielding of one of 4f electron by another in the statements is not correct?
subshell (a) There is a gradual decrease in the radii of the members
89. Lanthanoid which has the smallest size in +3 state is with increasing atomic number in the series.
(a) Tb (b) Er (c) Ce (d) Lu (b) All the members exhibit +3 oxidation state.
90. Identify the incorrect statement among the following : (c) Because of similar properties the separation of
(a) Lanthanoid contraction is the accumulation of lanthanoids is not easy.
successive shrinkages. (d) Availability of 4f electrons results in the formation of
(b) As a result of lanthanoid contraction, the properties compounds in +4 state for all the members of the
of 4d series of the transition elements have no series.
similarities with the 5d series of elements. 99. Which of the following exhibit only +3 oxidation state?
(c) Shielding power of 4f electrons is quite weak. (a) U (b) Th (c) Ac (d) Pa
(d) There is a decrease in the radii of the atoms or ions as 100. Larger number of oxidation states are exhibited by the
one proceeds from La to Lu. actinoids than those by the lanthanoids, the main reason
91. Which one of given elements shows maximum number of being
different oxidation states in its compounds? (a) 4f orbitals more diffused than the 5f orbitals
(a) Am (b) Fm (c) La (d) Gd (b) lesser energy difference between 5f and 6d than
92. Which of the following exhibits minimum number of between 4f and 5d orbitals
oxidation states ? (c) more energy difference between 5f and 6d than
between 4f and 5d orbitals
(a) Mn (b) Np (c) Th (d) Cr
(d) more reactive nature of the actinoids than the
93. Consider the following statements lanthanoids
(I) La(OH)3 is the least basic among hydroxides of 101. The +3 ion of which one of the following has half filled 4f
lanthanides. subshell?
(II) Zr4+ and Hf4+ posses almost the same ionic radii. (a) La (b) Lu (c) Gd (d) Ac
(III) Ce4+ can act as an oxidizing agent. 102. Although + 3 is the characteristic oxidation state for
Which of the above is/are true ? lanthanoids but cerium also shows + 4 oxidation state
(a) (I) and (III) (b) (II) and (III) because _________ .
(c) (II) only (d) (I) and (II) (i) it has variable ionisation enthalpy
94. Knowing that the chemistry of lanthanoids (Ln) is (ii) it has a tendency to attain noble gas configuration
dominated by its + 3 oxidation state, which of the following (iii) it has a tendency to attain f 0 configuration
statements is incorrect? (iv) it resembles Pb4+
(a) The ionic size of Ln (III) decrease in general with (a) (ii) and (iii) (b) (i) and (iv)
increasing atomic number (c) (ii) and (iv) (d) (i), (ii) and (iii)
EBD_8350
328 CHEMISTRY

103. The correct order of ionic radii of Y3+, La3+, Eu3+ and Lu3+ is 109. Which of the following is the use of mischmetall?
(a) La 3+ < Eu 3+ < Lu 3+ < Y 3+ (a) In bullets
(b) Y 3+ < La 3+ < Eu 3+ < Lu 3+ (b) In lighter flint
(c) As catalyst in petroleum cracking
(c) Y 3+ < Lu 3+ < Eu 3+ < La 3+ (d) Both (a) and (b)
(d) Lu 3+ < Eu 3+ < La 3+ < Y 3+ 110. Which one of the following is coinage metal?
(Atomic nos. Y =39, La = 57, Eu = 63, Lu = 71) (a) Zn (b) Cu (c) Sn (d) Pb
104. Which of the following lanthanoid ions is diamagnetic? 111. Which of the following is not correctly matched?
(At nos. Ce = 58, Sm = 62, Eu = 63, Yb = 70) Compound of Use
(a) Sm2+ (b) Eu2+ (c) Yb2+ (d) Ce2+ transition metal
105. Match the columns (a) TiO Pigment industry
Column-I Column-II (b) MnO2 Dry battery cell
(A) Lanthanide hard as steel. (p) Lu (c) V2O5 Manufacture of H2SO4
(B) Lanthanide with maximum (q) Tb
paramagnetic character in (d) PdCl2 Manufacture of polyethylene
Ln4+ state. 112. Cadmium is used in nuclear reactors for?
(C) Lanthanide with maximum (r) Sm (a) absorbing neutrons (b) cooling
value of E° for reaction (c) release neutrons (d) increase energy
Ln3+(aq)+3e– ® Ln(s). 113. K4[Fe(CN)6] is used in detecting.
(D) Lanthanide whose Ln 3+ ion is (s) Eu
diamagnetic in nature (a) Fe3+ ion (b) Cu+ ion (c) Cu3+ ion (d) Fe2+ ion
(a) A – (r), B – (s), C – (p), D – (q) 114. An alloy of transition metal containing a non transition
(b) A – (r), B – (q), C – (s), D – (p) metal as a constituent is
(c) A – (s), B – (r), C – (q), D – (p) (a) invar (b) bronze
(d) A – (r), B – (s), C – (q), D – (p) (c) chrome steel (d) stainless steel
115. Which of the following statements are correct?
Topic 4: Some Application of d–Block and f–Block Elements (i) Interstitial compounds contain non-metal atoms
106. Gun metal is an alloy of trapped inside the metal crystal whereas alloys are
(a) Cu and Al (b) Cu and Sn homogeneous blend of metals.
(c) Cu, Zn and Sn (d) Cu, Zn and Ni (ii) Steel and bronze are alloys of transition and non-
107. Brass is an alloy of transition metals.
(a) Zn and Sn (b) Zn and Cu (iii) Some boride containing interstitial compounds are
(c) Cu, Zn and Sn (d) Cu and Sn very hard comparable to that of diamond.
108. V2 O5 catalys t is used for the manufacture of (iv) Interstitial compounds are chemically more reactive
(a) HNO3 (b) Polyethylene than parent metal.
(c) H2SO4 (d) NH3 (a) (i) and (iii) (b) (ii) and (iv)
(c) (ii) and (iii) (d) (i), (ii) and (iii)

1. The colour of KMnO4 solution is decolourised by Fe2+ 6. Titanium shows magnetic moment of 1.73 B.M. in its
solution, one mole of Fe2+ reacts with x moles of KMnO4. compound. What is the oxidation number of Ti in the
Find x. compound?
2. Use Hund’s rule to derive the electronic configuration of 7. Four successive members of the first series of the transition
Ce3+ ion, and calculate its magnetic moment on the basis metals are listed below. How many of them have the standard
of ‘spin-only’ formula. ( )
potential E°M 2+ / M value with positive sign?
3. A compound of a metal ion M x+ ( Z = 24 ) has a spin only (I) Co (Z = 27) (II) Ni (Z = 28)
magnetic moment of 15 Bohr Magnetons. Find the (III) Cu (Z = 29) (IV) Fe (Z = 26)
number of unpaired electrons in the compound. 8. What is the approximate percentage of iron in mischmetal?
4. What are the number of ions formed on dissolving one 9. The standard redox potentials for the reactions
Mn2+ + 2e– ® Mn and Mn3+ + e– ® Mn2+ are –1.18 V and
molecule of FeSO 4 ( NH 4 ) SO 4 .6H 2O (Mohr’s salt)?
2 1.51 V respectively. What is the redox potential for the
5. In an alkaline condition KMnO4 reacts as follows: reaction Mn3+ + 3e– ® Mn?
2KMnO4 + 2KOH ® 2K 2 MnO 4 + H 2O + O pH = x pH = y
10. CrO24 - ¾¾¾¾
® CrO72 - ¾¾¾¾
® CrO 24 -. Find the
Calculate its equivalent weight. sum of x and y
THE d-AND f-BLOCK ELEMENT 329

Exercise 3 : NCERT Exemplar & Past Year MCQs


NCERT Exemplar MCQs 12. KMnO4 acts as an oxidising agent in acidic medium. The
number of moles of KMnO4 that will be needed to react
1. Electronic configuration of a transition element X in +3
oxidation state is [Ar]3d 5 . What is its atomic number? with one mole of sulphide ions in acidic solution is
(a) 25 (b) 26 (c) 27 (d) 24 2 3 4 1
(a) (b) (c) (d)
2. Generally, transition elements form coloured salts due to 5 5 5 5
the presence of unpaired electrons. Which of the following 13. Interstitial compounds are formed when small atoms are
compounds will be coloured in solid state? trapped inside the crystal lattice of metals. Which of the
(a) Ag2SO4 (b) CuF2 (c) ZnF2 (d) Cu2Cl2 following is not the characteristic property of interstitial
3. The magnetic nature of elements depends on the presence compounds?
of unpaired electrons. Identify the configuration of (a) They have high melting points in comparison to pure
transition element, which shows highest magnetic moment. metals
(a) 3d7 (b) 3d5 (c) 3d8 (d) 3d2
(b) They are very hard
4. Which of the following oxidation state is common for all
lanthanoids? (c) They retain metallic conductivity
(a) +2 (b) +3 (c) +4 (d) +5 (d) They are chemically very reactive
5. The magnetic moment is associated with its spin angular 14. KMnO4 acts as an oxidising agent in alkaline medium.
momentum and orbital angular momentum. Spin only When alkaline KMnO4 is treated with KI, iodide ion is
magnetic moment value of Cr3+ ion is oxidised to ................. .
(a) 2.87 BM (b) 3.87 BM (a) I2 (b) IO–
(c) 3.47 BM (d) 3.57 BM (c) IO3 - (d) IO-4
6. Which of the following statements is not correct?
15. Highest oxidation state of manganese in fluoride is
(a) Copper liberates hydrogen from acids
+4 (MnF4) but highest oxidation state in oxides is +7
(b) In its higher oxidation states, manganese forms stable
(Mn2O7) because
compounds with oxygen and fluorine
(c) Mn 3+ and Co3+ are oxidising agents in aqueous (a) fluorine is more electronegative than oxygen
solution (b) fluorine does not possess d-orbitals
(d) Ti2+ and Cr 2+ are reducing agents in aqueous solution (c) fluorine stabilises lower oxidation state
7. When acidified K2Cr2O7 solution is added to Sn2+ salt (d) in covalent compounds, fluorine can form single bond
then Sn2+ changes to only while oxygen forms double bond
(a) Sn (b) Sn3+ (c) Sn4+ (d) Sn+ 16. Although zirconium belongs to 4d transition series and
8. Metallic radii of some transition elements are given below. hafnium to 5d transition series even then they show similar
Which of these elements will have highest density? physical and chemical properties because ................ .
Element Fe Co Ni Cu (a) both belong to d-block
Metallic radii/pm 126 125 125 128 (b) both have same number of electrons
(c) both have similar atomic radius
(a) Fe (b) Ni (c) Co (d) Cu
(d) both belong to the same group of the periodic table
9. On addition of small amount of KMnO4 to concentrated
H2SO4, a green oily compound is obtained which is highly 17. Why is HCl not used to make the medium acidic in
explosive in nature. Identify the compound from the oxidation reactions of KMnO4 in acidic medium?
following. (a) Both HCl and KMnO4 act as oxidising agents
(a) Mn 2O7 (b) MnO2 (c) MnSO4 (d) Mn 2O3 (b) KMnO4 oxidises HCl into Cl2 which is also an
10. When KMnO4 solution is added to oxalic acid solution, oxidising agent
the decolourisation is slow in the beginning but becomes (c) KMnO4 is a weaker oxidising agent than HCl
instantaneous after some time because (d) KMnO4 acts as a reducing agent in the presence of HCl
(a) CO2 is formed as the product 18. The electronic configuration of Cu(II) is 3d 9 whereas that
(b) reaction is exothermic of Cu(I) is 3d10. Which of the following is correct.
(c) MnO -4 catalyses the reaction (a) Cu(II) is more stable
(d) Mn2+ acts as autocatalyst (b) Cu(II) is less stable
11. There are 14 elements in actinoid series. Which of the (c) Cu(I) and Cu(II) are equally stable
following elements does not belong to this series? (d) Stability of Cu(I) and Cu(II) depends on nature of
(a) U (b) Np (c) Tm (d) Fm copper salts
EBD_8350
330 CHEMISTRY

19. Which of the following reactions are disproportionation 28. The number of d-electrons in Fe2+ (Z = 26) is not equal to
reactions? the number of electrons in which one of the following?
(i) Cu+ ® Cu2+ + Cu [AIPMT 2015, S]
(a) p-electrons in Cl (Z = 17)
(ii) 3MnO 24 - + 4H + ® 2MnO 4- + MnO 2 + 2H 2 O (b) d-electrons in Fe (Z = 26)
(iii) 2KMnO4 ® K2MnO4 + MnO2 + O2 (c) p-electrons in Ne (Z = 10)
(d) s-electrons in Mg (Z = 12)
(iv) 2MnO 4- + 3Mn 2+ + 2H 2O ® 5MnO 2 + 4H + 29. Which is the correct order of increasing energy of the
listed orbitals in the atom of titanium?
(a) (i) and (ii) (b) (i), (ii) and (iii)
[AIPMT 2015 RS, C]
(c) (ii), (iii) and (iv) (d) (i) and (iv) (a) 3s 4s 3p 3d (b) 4s 3s 3p 3d
20. Which of the following oxides are amphoteric? (c) 3s 3p 3d 4s (d) 3s 3p 4s 3d
Mn2O7, CrO3, Cr2O3, CrO, V2O5, V2O4 30. Assuming complete ionization, same moles of which of
(a) V2O5, Cr2O3 (b) Mn2O7, CrO3 the following compounds will require the least amount of
acidified KMnO4 for complete oxidation
(c) CrO, V2O5 (d) V2O5, V2O4
[AIPMT 2015 RS, S]
21. Gadolinium belongs to 4f series. Its atomic number is 64. (a) FeSO4 (b) FeSO3 (c) FeC2O4 (d) Fe(NO2)2
Which of th e following is the correct electronic
31. Because of lanthanoid contraction, which of the following
configuration of gadolinium?
pairs of elements have nearly same atomic radii ? (Numbers
(a) [Xe]4f 7 5d16s2 (b) [Xe]4f 6 5d 26s2 in the parenthesis are atomic numbers). [AIPMT 2015, C]
(c) [Xe]4f 8 6d2 (d) [Xe]4f 9 5s1 (a) Zr (40) and Nb (41) (b) Zr (40) and Hf (72)
Past Year MCQs (c) Zr (40) and Ta (73) (d) Ti (22) and Zr (40)
32. Match the catalysts to the correct processes :
22. Reason of lanthanoid contraction is:- [AIPMT 2014, C] [JEE M 2015, A]
(a) Negligible screening effect of ‘f ’ orbitals Catalyst Process
(b) Increasing nuclear charge (A) TiCl4 (i) Wacker process
(B) PdCl2 (ii) Ziegler - Natta
(c) Decreasing nuclear charge polymerization
(d) Decreasing screening effect (C) CuCl2 (iii) Contact process
23. The pair of compounds that can exist together is: (D) V2O5 (iv) Deacon's process
[AIPMT 2014, A] (a) (A) - (ii), (B) - (iii), (C) - (iv), (D) - (i)
(a) FeCl3, SnCl2 (b) HgCl2, SnCl2 (b) (A) - (iii), (B) - (i), (C) - (ii), (D) - (iv)
(c) FeCl2, SnCl2 (d) FeCl3, KI (c) (A) - (iii), (B) - (ii), (C) - (iv), (D) - (i)
24. In acidic medium, H2O2 changes Cr2O7–2 to CrO5 which (d) (A) - (ii), (B) - (i), (C) - (iv), (D) - (iii)
has two (–O–O) bonds. Oxidation state of Cr in CrO5 is:- 33. The color of KMnO4 is due to : [JEE M 2015, C]
(a) L ® M charge transfer transition
[AIPMT 2014, S] (b) s - s* transition
(a) + 5 (b) + 3 (c) + 6 (d) – 10 (c) M ® L charge transfer transition
25. The reaction of aqueous KMnO4 with H2O2 in acidic (d) d – d transition
conditions gives: [AIPMT 2014, S] 34. Which one of the following statements is correct when
(a) Mn4+ and O2 (b) Mn2+ and O2 SO2 is passed through acidified K2Cr2O7 solution ?
(a) The solution turns blue [NEET 2016, S]
(c) Mn2+ and O3 (d) Mn4+ and MnO2
(b) The solution is decolourized
26. Which series of reactions correctly represents chemical (c) SO2 is reduced
reactions related to iron and its compound? (d) Green Cr2(SO4)3 is formed
[JEE M 2014, S] 35. The electronic configurations of Eu (Atomic No. 63), Gd
(a) dil . H 2SO 4 H 2SO 4 , O 2 (Atomic No. 64) and Tb (Atomic No. 65) are
Fe ¾¾¾¾¾
® FeSO 4 ¾¾¾¾¾®
heat
Fe 2 ( SO4 )3 ¾¾¾ ® Fe [NEET 2016, C]
O , heat dil. H SO (a) [Xe]4f 6s , [Xe]4f 6s and [Xe]4f 5d 16s2
7 2 8 2 8
(b) Fe ¾¾¾¾
2 ® FeO ¾¾¾¾¾
2 4®
heat (b) [Xe]4f 75d16s2, [Xe]4f 7 5d1 6s2 and [Xe]4f 96s2
FeSO 4 ¾¾¾ ® Fe
(c) [Xe]4f 65d16s2, [Xe]4f 75d16s2 and [Xe]4f 85d16s2
Cl2 , heat
(c) Fe ¾¾¾¾® heat , air Zn
FeCl3 ¾¾¾¾® FeCl2 ¾¾® Fe (d) [Xe]4f 76s2, [Xe]4f 75d16s2 and [Xe]4f 96s2
O2 , heat
(d) Fe¾¾¾¾ CO , 600°C 36. Which of the following compounds is metallic and
® Fe3O¾¾¾¾¾
4 ®
CO , 700°C ferromagnetic? [JEE M 2016, C]
FeO ¾¾¾¾¾ ® Fe (a) VO2 (b) MnO2 (c) TiO2 (d) CrO2
27. Magnetic moment 2.84 B.M. is given by :
37. Name the gas that can readily decolourise acidified KMnO4
(At. nos, Ni = 28, Ti = 22, Cr = 24, Co = 27)
solution : [NEET 2017, S]
[AIPMT 2015, C]
(a) SO2 (b) NO2 (c) P2O5 (d) CO2
(a) Ti3+ (b) Cr2+ (c) Co2+ (d) Ni2+
THE d-AND f-BLOCK ELEMENT 331

38. The reason for greater range of oxidation states in actinoids Column I Column II
is attributed to : [NEET 2017, C] Catalyst Product
(a) actinoid contraction (A) V2O5 (i) Polyethylene
(b) 5f, 6d and 7s levels having comparable energies (B) TiCl4/ Al(Me) 3 (ii) ethanol
(c) 4f and 5d levels being close in energies (C) PdCl2 (iii) H2SO4
(d) the redioactive nature of actinoids (D) Iron Oxide (iv) NH3
39. Which one of the following ions exhibits d-d transition (a) (A)-(iii); (B)-(iv); (C)-(i); (D)-(ii)
and paramagnetism as well? [NEET 2018, S] (b) (A)-(ii); (B)-(iii); (C)-(i); (D)-(iv)
– (c) (A)-(iii); (B)-(i); (C)-(ii); (D)-(iv)
(a) CrO2– (b) Cr2 O72– (c) MnO 2–4 (d) MnO4
4 (d) (A)-(iv); (B)-(iii); (C)-(ii); (D)-(i)
40. Match the metal ions given in Column I with the spin 42. The highest value of the calculated spin only magnetic
magnetic moments of the ions given in Column II and moment (in BM) among all the transition metal complexes
assign the correct code : [NEET 2018, S] is : [JEE M 2019, C]
Column I Column II (a) 5.92 (b) 6.93 (c) 3.87 (d) 4.90
(A) Co3+ (i) 8 BM 43. Identify the incorrect statement. [NEET 2020, S]
(B) Cr3+ (ii) 35 BM (a) The transition metals and their compounds are known
for their catalytic activity due to their ability to adopt
(C) Fe3+ (iii) 3 BM multiple oxidation states and to form complexes.
(D) Ni2+ (iv) 24 BM (b) Interstitial compounds are those that are formed when
small atoms like H, C or N are trapped inside the crystal
(v) 15 BM lattices of metals.
(A) (B) (C) (D) (c) The oxidation states of chromium in CrO24 - and
(a) (iv) (v) (ii) (i)
Cr2 O72 - are not the same.
(b) (i) (ii) (iii) (iv)
(c) (iii) (v) (i) (ii) (d) Cr2+ (d4) is a stronger reducing agent than Fe2+ (d6)
(d) (iv) (i) (ii) (iii) in water.
41. Match the catalysts (Column I) with products (Column II). 44. The atomic radius of Ag is closest to: [JEE M 2020, C]
[JEE M 2019, C] (a) Au (b) Ni (c) Cu (d) Hg

Exercise 4 : Problem Solving Skill Enhancer MCQs


1. Out of TiF62– , CoF63– , Cu2Cl2 and NiCl2– (b) Au3+ disproportionates into Au4+ and Au+ in aqueous
4 (Z of Ti = 22,
Co = 27, Cu = 29, Ni = 28), the colourless species are: solution
(c) Au+ disproportionates into Au3+ and Au in aqueous
(a) Cu2Cl2 and NiCl 2– 4 (b) TiF62– ,and Cu2Cl2 solution
(c) CoF63– ,and NiCl 2– 4 (d) TiF62– ,and CoF63– , (d) Au+ disproportionates into Au2+ and Au in aqueous
2. The basic character of the transition metal monoxides solution
follows the order 6. A metal gives two chlorides ‘X’ and ‘Y’. The chloride ‘X’
(Atomic Nos.,Ti = 22, V = 23, Cr = 24, Fe = 26) gives a black precipitate with NH4OH. The chloride ‘Y’
(a) TiO > VO > CrO > FeO (b) VO > CrO > TiO > FeO gives a white precipitate with NH4OH. ‘Y’ gives red
(c) CrO > VO > FeO > TiO (d) TiO > FeO > VO > CrO precipitate with KI which is soluble in excess of KI. The
3. When (NH4)2Cr2O7 is heated, the gas evolved is formula of chlorides ‘X’ and ‘Y’ respectively are
(a) Hg2Cl2 and HgCl2 (b) HgCl2 and ZnCl2
(a) N2 (b) NO2 (c) O2 (d) Na2O
(c) ZnCl2 and Hg2Cl2 (d) None of these
4. In the extraction of silver from argentite ore the ore is treated 7. First four ionization energies of Ni and Pt are given below:
with dilute solution of NaCN in water in the presence of Y,
(IE1 + IE2) (IE3 + IE4)
whereby the following reaction takes place 3 –1
Ni 2.49 ×10 kJ mol 8.8 × 103 kJ mol–1
Ag 2 X + 4NaCN + 2Y ® 2Na éëAg (CN )2 ùû + Na 2XO 4 3
Pt 2.66 × 10 kJ mol –1 6.7 × 103kJ mol–1
X and Y in this reaction are represented by From the data it can be concluded that
(a) Cl and S (b) S and O2 (c) O and O2 (d) O and S (a) Ni(II) compounds are thermodynamically more stable
5. Given E ° ° 3+ + = 1.36V . Point
= 1.52V and EAu than Pt(II) compounds
Au 3+ /Au /Au (b) Pt(IV) compounds are thermodynamically more stable
out the correct statement of the following than Ni(II) compounds.
(a) Au 3+ disproportionates into Au 4+ and Au 2+ in (c) both correct
aqueous solution (d) none of these is correct
EBD_8350
332 CHEMISTRY

8. Which of the following statement is incorrect? (a) [Cu(CN)4 ]3- , (CN) 2 (b) CuCN, (CN)2
(a) Across a period from Sc to Cu the densities increase
with increasing atomic number. (c) [Cu(CN)4 ]2- , (CN) 2 (d) Cu (CN)2, K2S
(b) The melting point of transition elements rise to a maximum 12. The dichromate ion is in equilibrium with chromate ion in
from Sc to Cr and then decreases from Fe to Zn. aqueous solution as :
(c) Transition elements have high enthalpies of atomization ˆˆ† 2CrO24 - (aq) + 2H + (aq)
Cr2O27 - (aq) + H 2O ‡ˆˆ
and in 3d series increase regularly from Sc to Cu. The oxoanion has
(d) On going down a group from 3d to 6d series, the (a) same oxidizing property in acidic and alkaline solutions
stability of higher oxidation state increases with (b) better oxidizing property in acidic solution
increasing atomic number. (c) better oxidizing property in alkaline solution
9. Which metal M and its compound can give the following (d) no oxidizing property in acidic or alkaline solution
observable changes in a consequence of reactions 13. A blue colouration is not obtained when
é ù aqueous é White ù (a) ammonium hydroxide dissolves in copper sulphate
® êColourless
dilute
M ¾¾¾¾ solutions ú ¾¾¾¾ ®ê ú
HNO3 ë û NaOH ë precipitate û (b) copper sulphate solution reacts with K4[Fe(CN)6]
(c) ferric chloride reacts with sod. ferrocyanide
é White ù H 2S éColourless ù excess (d) anhydrous CuSO4 is dissolved in water
ê precipitate ú ¬¾¾ ¾ ê solutions ú ¬¾¾¾¾¾
( aq.)
ë û ë û NaOH 14. In nitroprusside ion the iron and NO exist as FeII and NO+
(a) Mg (b) Pb (c) Zn (d) Sn rather than FeIII and NO. These forms can be differentiated by
( X ) + K 2 CO3 + Air ¾¾¾heat (a) estimating the concentration of iron
10. ® ( Y)
(b) measuring the concentration of CN
( Y ) + Cl2 ¾¾ ® ( Z ) Pink (c) measuring the solid state magnetic moment
Which of the following is correct? (d) thermally decomposing the compound.
(a) X = black, MnO2, Y = Blue, K2CrO4, Z = KMnO4 15. Cerium (Z = 58) is an important member of the lanthanoids.
(b) X = green, Cr2O3, Y = Yellow, K2CrO4, Z = K2Cr2O7 Which of the following statements about cerium is incorrect?
(c) X = black, MnO2, Y = green, K2MnO4, Z = KMnO4 (a) The +4 oxidation state of cerium is not known in solutions
(d) X = black, Bi2O3, Y = colourless KBiO2, Z = KBiO3 (b) The +3 oxidation state of cerium is more stable than
H S­ Excess the +4 oxidation state
CuSO4 (aq.) ¾¾¾® M ¯ ¾¾¾¾ ®N+O
2
11. of KCN (c) The common oxidation states of cerium are +3 and +4
Then final products N and O are respectively. (d) Cerium (IV) acts as an oxidizing agent
ANSWER KEY
Exercise 1 : NCERT Based Topic-wise MCQs
1 (c) 13 (a) 25 (d) 37 (d) 49 (c) 61 (b) 73 (c) 85 (a) 97 (a) 109 (d)
2 (b) 14 (b) 26 (d) 38 (a) 50 (a) 62 (d) 74 (a) 86 (d) 98 (d) 110 (b)
3 (c) 15 (a) 27 (d) 39 (c) 51 (d) 63 (a) 75 (b) 87 (d) 99 (c) 111 (d)
4 (c) 16 (a) 28 (a) 40 (b) 52 (c) 64 (b) 76 (c) 88 (b) 100 (b) 112 (a)
5 (b) 17 (d) 29 (c) 41 (b) 53 (b) 65 (a) 77 (b) 89 (d) 101 (c) 113 (a)
6 (b) 18 (c) 30 (c) 42 (b) 54 (c) 66 (a) 78 (b) 90 (b) 102 (a) 114 (b)
7 (d) 19 (b) 31 (d) 43 (d) 55 (d) 67 (d) 79 (b) 91 (a) 103 (c) 115 (a)
8 (c) 20 (a) 32 (c) 44 (d) 56 (b) 68 (c) 80 (c) 92 (c) 104 (c)
9 (c) 21 (a) 33 (a) 45 (d) 57 (c) 69 (b) 81 (b) 93 (b) 105 (b)
10 (c) 22 (c) 34 (d) 46 (c) 58 (b) 70 (b) 82 (d) 94 (b) 106 (c)
11 (a) 23 (b) 35 (a) 47 (b) 59(b) 71(c) 83
(c) 95 (b) 107 (b)
12 (d) 24 (d) 36 (d) 48 (a) 60 (a) 72 (a) 84 (c) 96 (d) 108 (d)
Exercise 2 : Numeric/Integer Answer Questions
1 (0.2) 2 (1.73) 3 (3) 4 (5) 5 (158) 6 (3) 7 (1) 8 (5) 9 (-0.28) 10 (11)
Exercise 3 : NCERT Exemplar & Past Year MCQs
1 (b) 6 (a) 11 (c) 16 (c) 21 (a) 26 (d) 31 (b) 36 (d) 41 (c)
2 (b) 7 (c) 12 (a) 17 (b) 22 (a) 27 (d) 32 (d) 37 (a) 42 (a)
3 (b) 8 (d) 13 (d) 18 (a) 23 (c) 28 (a) 33 (a) 38 (b) 43 (c)
4 (b) 9 (a) 14 (c) 19 (a) 24 (c) 29 (d) 34 (d) 39 (c) 44 (a)
5 (b) 10 (d) 15 (d) 20 (a) 25 (b) 30 (a) 35 (d) 40 (a)
Exercise 4 : Problem Solving Skill Enhancer MCQs
1 (b) 3 (a) 5 (c) 7 (c) 9 (c) 11 (a) 13 (b) 15 (a)
2 (a) 4 (b) 6 (a) 8 (c) 10 (c) 12 (b) 14 (c)
23 Coordination Compounds

Trend Buster NEET & JEE Main

Number of Questions from 2020-15 17 11 Chapter has equal importance in


Weightage 5.3% 6.2% NEET & JEE M.

The most Important Concepts that Cover Maximum number of Questions asked in past 6 years.

Bonding in coordination compounds / valence bond 10 6


theory/crystal field theory/stability / magnetic moment
Isomerism in coordination compounds/IUPAC name 3 5
Werner's theory 3 1

Less Important Concepts that Cover 1 or 2 Questions asked in past 6 years.

Applications of coordination compounds 1 —

NEET JEE
Concept Used

2020 Nomenclature of IUPAC nomenclature / 2 Average 2 Average


coordination compounds/ bonding in metal
bonding in coordination carbonyls / magnetic moment /
compounds field strength
2019 Isomerism in coordination Crystal field theory / isomerism / 1 Average 3 Average
compound / bonding in optical isomerism
coordination compounds
2018 Isomerism in coordination Bonding in metal carbonyls / 3 Average 1 Difficult
compound/bonding in isomerism / Valence
coordination compounds bond theory
2017 Werner's theory of Stability of coordination 5 Average 1 Difficult
coordination compounds/ compounds / colours in the
bonding in coordination coordination compounds /
compounds / applicatins werner's theory / valence bond
and importance of theory / organometallic
coordination compounds compounds
2016 Isomerism in coordination bonding in metal carbonyls / 1 Average 2 Average
compound / bonding in isomerism / magnetic
coordination compounds properties
2015 Werner's theory of Werner's theory / coordination 5 Average 2 Difficult
coordination compounds/ number / isomerism / valence
bonding in coordination bond theory / colours in the
compounds / isomerism in coordination compounds
DTP : JOSHIcoordination compounds 2nd Print
EBD_8350
334 CHEMISTRY
COORDINATION COMPOUNDS 335
EBD_8350
336 CHEMISTRY

Problem Solving Tips/ Tricks/ Points to Remember

4 The ligand such as NH2NH2 does not act as chelating Where p and q are the number of electrons in t 2g and
ligands due to the formation of three membered ring and eg orbitals respectively..
locked structure, respectively. Only the chelate complexes m = Number of pairs of electrons caused by the ligand
with 5 or 6 membered ring are more stable. p = pairing energy
4 Square planar complexes [ (Ma 4 )n ± , (Ma 3 b) n ± 4 For a dn tetrahedral complex with e p t q2 configuration.
n± n±
[M(AA) 2 ] ,[M(AA)ab] ,[M(AA)a 2 ] n±
and CFSE = [ -0.6p + 0.4q]D t
octahedral complex 4 4
= [-0.6p + 0.4q] ´ D 0 (Q D t = D 0 )
[(Ma 6 ) n ± , (Masb)n ± , (M(AA)3 ) n ± ] 9 9
do not exhibit geometrical isomerism because all the = [ -0.27p + 0.18q]D 0
possible arrangement of the ligands round the metal Since tetrahedral complexes are high spin and no pairing
cation is the same. of d-electrons occurs (Dt < p). Therefore no pair energy
is in the above equation.
4 MnO4– , Cr2O72– , CrO2– 3
4 show sd hybridization, due to 4 The common coordination number of Cu2+ is 6 not 4. In
highest oxidation state of the metal ion orbitals become aqueous solution, it exists as [Cu(H2O)6]2+ complex.
empty and further combination gives four sd3 hybrid 2+
orbitals. é Cu ( H 2O) ù small amount of NH3
¾¾¾¾¾¾¾¾¾ ® éëCu ( H 2O ) 4 ( OH )2 ùû ¯
ë 6û ammonia acting as base
4 I- < Br - < S2– < SCN - < Cl– < N3- < F- < urea < ( light blue) light blue ppt .
2+
® éëCu ( NH3 ) 4 ( H 2O) 2 ùû
- Excess NH3
OH - < C 2 H 5 - OH < C 2 O 4 < O 2– < H 2 O ¾¾¾¾¾¾¾¾¾
ligandexchange reaction
( dark blue solution )
< NCS < gly < NH3 , Py < en, SO32– < NH 2OH < bpy,

4 Double salts such as FeSO4 (NH4)2 SO4. 6H2O dissociate
phen < NO2– < CH 3– < C6H5– < R 3P < CN – < CO into simple ions when dissolved in water, i.e. Fe2+, SO42–
4 The ability of CO molecule to accept electron-density and NH4+. However, complex compounds such as
back into its empty orbitals is called p-acidity (or back
donation). K4[Fe(CN)6] will dissociate into K+ and complex ion
4 The stability of a complex depends upon the following [Fe(CN)6]4–.
factors. 4 Square planar complex of the type [M abcd] can show 3
(a) Nature of the central ion – High charge, small size geometrical isomers out of these three, two are cis and
favours the stability of the complex. one is trans for any two ligands.
(b) Nature of the ligand – More basic ligand forms 4 Theoretically, tetrahedral complex of the type [M abcd]
more stable complex. can show optical isomerism because of absence of plane
4 Ambidenate ligand : Ligands containing more than one of symmetry. However, due to identical positions, these
coordinating atoms. four ligands rapidly interchange their positions. This is
(ambidentate) : CN– : NC– the reason why these isomers are not resolvable.
4 The complex [Pt(py)(NH3)(NO2) Cl Br I] is expected to
[Cr(H 2O)5 CN]2+ and [Cr(H 2 O)5 NC]2 + give 15 geometric isomers. Each of these forms will show
4 Haemoglobin : complex of Iron (II) an optical isomer, thus 15 pairs of enantiomers.
4 Chlorophyll : complex of Mg 4 In structural isomerism, the primary valence and
4 Vitamin B12 : complex of cobalt secondary valence shoule remain same for all isomers.
Zeigler Natta (Al (C 2H 5) 3 + TiCl 3) is used in
ë (
NH3 )3 ( CN )3 ù é Co
ûë (
NH3 )3 ( CN )3 ù is
4 é Cr
III III
polymerisation of ethene. 4 û not a
4 Tetraethyl lead is used as antiknock compound coordination isomer of [Cr(NH3)6][Co(CN)6] because the
Pb(C2H5)4. cation and anion become neutral.
4 Wilkinson’s catalyst (Ph3P)3 RhCl. for selective 4 d4 to d7 coordination entities are more stable for strong
hydrogenation. field as compare to weak field ligands.
4 C2H5HgCl (ethyl mercury chloride) as fungicide. 4 Both prussian blue and Turnbull’s blue are ferric
4 Effective atomic number (EAN) : It can be obtained from
potassium ferrocyanide K Fe éë Fe ( CN )6 ùû . However,,
III III
the following simple expression
EAN = Z – O.N + 2 (CN)
Where Z = Atomic number of central metal atom colour of Turnbull’s blue is less intense due to impurity
O.N. = Oxidation number of central metal atom by different preparation method.
CN = Coordination number of central metal atom. 4 Hardness of water is estimated by titration with Na2 EDTA,
It is equal to the number of monodentate ligands, twice as Ca2+ and Mg2+ ions form stable complexes.
the number of bidentate ligands and so on. 4 Electroplating of metals involves the use of complex salt
4 For a, dn high spin complex with electronic configuration as electrolyte e.g. K [Ag (CN)2] in silver plating.
4 Estimation and detection of metal ions uses complexes
t p2g , eqg e.g. dimethyl glyoxime is used in estimation of Ni2+ ion.
4 Metals in toxic proportions can be removed by chelate
CFSE = [ -0.4p + 0.6q]D 0 + mp therapy by using chelating ligands.
COORDINATION COMPOUNDS 337

Exercise 1 : NCERT Based Topic-wise MCQs


Topic 1: Werner’s Theory of Coordination Compounds 9. An ambident ligand is one which
(a) is linked to the metal atom through two donor atoms
Definitions of Some Important Terms Pertaining to
(b) has two donor atoms, but only one of them has the
Coordination Compounds capacity to form a coordinate bond [or a sigma (s)
bond]
1. The number of ions formed on dissolving one molecule of (c) has two donor atoms, but either of two can form a
FeSO4(NH4)2SO4.6H2O in water is: coordinate bond
(a) 4 (b) 5 (c) 3 (d) 6 (d) forms chelate rings.
2. Which of the following postulates of Werner’s theory is 10. Which one of the following is NOT a ligand ?
incorrect? (a) PH3 (b) NO+ (c) Na+ (d) F–
(a) Primary valencies are satisfied by negative ions. 11. Which of the following complexes are heteroleptic ?
(b) Secondary valencies are satisfied by neutral (i) [Cr(NH3)6]3+ (ii) [Fe(NH3)4Cl2]+
molecules or negative ions. (iii) [Mn(CN)6] 4– (iv) [Co(NH3)4Cl2]
(c) Secondary valence is equal to the coordination
(a) (i), (iv) (b) (ii) and (iv)
number and it depends upon the nature of ligand
(c) (i) and (ii) (d) (i) and (iv)
attached to metal.
(d) The ions/ groups bound by the secondary linkages 12. One mole of the complex compound Co(NH3)5Cl3, gives 3
moles of ions on dissolution in water. One mole of the
to the metal have charecteristic spatial arrangements.
same complex reacts with two moles of AgNO3 solution to
3. The coordination number of a central metal atom in a
yield two moles of AgCl (s). The formula of the complex is
complex is determined by
(a) the number of ligands around a metal ion bonded by (a) [Co(NH3)3Cl3]. 2 NH3 (b) [Co(NH3)4Cl2] Cl . NH3
sigma and pi-bonds both (c) [Co(NH3)4Cl] Cl2. NH3 (d) [Co(NH3)5Cl] Cl2
(b) the number of ligands around a metal ion bonded by 13. Some salts containing two different metallic elements give
pi-bonds test for only one of them in solution, such salts are
(c) the number of ligands around a metal ion bonded by (a) double salts (b) normal salts
sigma bonds (c) complex salts (d) None of these
(d) the number of only anionic ligands bonded to the 14. Which of the following represents hexadentate ligand?
metal ion. (a) EDTA (b) DMG
4. The oxidation state of Cr in [Cr(NH3)4Cl2]+ is (c) Ethylenediamine (d) None of the above
(a) 0 (b) + 1 (c) + 2 (d) + 3 15. Which of the following does not form a chelate ?
5. [EDTA]4– is a : (a) EDTA (b) Oxalate
(a) monodentate ligand (b) bidentate ligand (c) Pyridine (d) Ethylenediamine
(c) quadridentate ligand (d) hexadentate ligand 16. Identify the correct statements for the behaviour of
6. Some salts although containing two different metallic ethane- 1, 2-diamine as a ligand.
elements give test for only one of them in solution. Such (i) It is a neutral ligand.
salts are (ii) It is a bidentate ligand.
(a) complex (b) double salts (iii) It is a chelating ligand.
(c) normal salts (d) None of these (iv) It is a unidentate ligand.
7. Ligand in a complex salt are (a) (i), (ii) and (iii) (b) (ii) and (iii)
(a) anions linked by coordinate bonds to a central metal (c) (iii) and (iv) (d) (i), (iii) and (iv)
atom or ion 17. Match the columns.
(b) cations linked by coordinate bonds to a central metal Column-I Column-II
or ion
(Ligand) (Type of ligand)
(c) molecules linked by coordinate bonds to a central
metal or ion (A) Triphenylphosphine (p) Unidenate
(d) ions or molecules linked by coordinate bonds to a (B) BF3 (q) Didentate
central atom or ion (C) Ethylenediamine (r) Not a ligand
8. A bidenate ligand always (D) Ethylenediaminetetracetateion (s) Hexadenate
(a) has bonds formed to two metals ions (a) A – (p), B – (r), C – (q), D – (s)
(b) has a charge of +2 or – 2 (b) A – (p), B – (q), C – (r), D – (s)
(c) forms complex ions with a charge of +2 or –2
(c) A – (p), B – (r), C – (q), D – (s)
(d) has two donor atoms forming simultaneously two
sigma (s) bonds. (d) A – (p), B – (q), C – (s), D – (p)
EBD_8350
338 CHEMISTRY

18. O2 is a 26. The hypothetical complex chlorodiaquatriamminecobalt


(a) Monodentate ligand (b) Bidenate ligand (III) chloride can be represented as
(c) Tridentate ligand (d) Hexadenate ligand (a) [CoCl(NH3)3(H2O)2]Cl2 (b) [Co(NH3)3(H2O)Cl3]
19. The coordination number and the oxidation state of the (c) [Co(NH2)3(H2O)2 Cl] (d) [Co(NH3)3(H2O)3]Cl3
element ‘E’ in the complex [E(en)2(C2O4)]NO2 (where (en) 27. The IUPAC name of the coordination compound
is ethylene diamine) are, respectively, K3[Fe(CN)6] is
(a) 6 and 2 (b) 4 and 2 (a) Tripotassium hexacyanoiron (II)
(c) 4 and 3 (d) 6 and 3 (b) Potassium hexacyanoiron (II)
20. Assertion : Ethylenediaminetetraacetate ion forms an (c) Potassium hexacyanoferrate (III)
octahedral complex with the metal ion. (d) Potassium hexacyanoferrate (II)
Reason : It has six donor atoms which coordinate 28. The complex, [Pt(py)(NH3)BrCl] will have how many
simultaneously to the metal ion. geometrical isomers ?
(a) If both Assertion and Reason are correct and Reason (a) 3 (b) 4 (c) 0 (d) 2
is the correct explanation of Assertion. 29. The number of geometrical isomers from [Co(NH3)3(NO2)3]
(b) If both Assertion and Reason are correct, but Reason is
is not the correct explanation of Assertion. (a) 2 (b) 3 (c) 4 (d) 0
(c) If Assertion is correct but Reason is incorrect. 30. The compounds [PtCl2(NH3)4]Br2 and [PtBr2(NH3)4]Cl2
(d) If both the Assertion and Reason are incorrect. constitutes a pair of
21. 0.02 mole of [Co(NH3) 5Br]Cl 2 and 0.02 mole of (a) coordination isomers (b) linkage isomers
[Co(NH3)5Cl]SO4 are present in 200 cc of a solution X. The (c) ionization isomers (d) optical isomers
number of moles of the precipitates Y and Z that are formed 31. Which of the following type of isomerism is shown by
when the solution X is treated with excess silver nitrate given complex compound?
and excess barium chloride are respectively NH3
(a) 0.02, 0.02 (b) 0.01, 0.02 H3N NO2
(c) 0.02, 0.04 (d) 0.04, 0.02 Co
22. Assertion : NF3 is a weaker ligand than N(CH3)3.
O2N NO2
Reason : NF3 ionizes to give F– ions in aqueous solution.
NH3
(a) Assertion is correct, reason is correct; reason is a
correct explanation for assertion. (a) Facial (b) Meridional
(b) Assertion is correct, reason is correct; reason is not (c) Cis (d) Both b and c
a correct explanation for assertion 32. The formula for the complex, dichlorobis (urea) copper (II)
(c) Assertion is correct, reason is incorrect is
(d) Assertion is incorrect, reason is correct. (a) [Cu{O = C (NH2)2}] Cl2
23. What is the secondary valence of following compounds (b) [Cu{O = C (NH2)2}Cl]Cl
PtCl2.2NH3, CoCl3.4NH3 and NiCl2.6H2O (c) [CuCl2 {O = C(NH2)2}2]
if moles of AgCl precipitated per mole of the given (d) [CuCl2] [{O = C (NH2)2}]2
compounds with excess AgNO3 respectively are: 0, 1 and 2 33. Among the following coordination compounds/ions
(a) 6, 4, 4 (b) 4, 6, 6 (c) 4, 4, 6 (d) 2, 4, 6 (i) [Fe(CN)6]3– (ii) [Pt(NH3)2Cl2]
24. Suppose someone made aqueous solution of NiCl2 and (iii) [Co(NH3)6]3+ (iv) [Cr(H2O)6]Cl3
recrystallized its aqueous solution in excess of water and Which species exhibit geometrical isomerism?
if two moles of precipitate AgCl was formed on treatment (a) (ii) only (b) (i) and (ii)
with AgNO3, what is the most probable structure of the (c) (ii) and (iv) (d) (i) and (iii)
compound ? 34. For which value of the x, and y, the following square
(a) [Ni(Cl)2(H2O)4] (b) [Ni (H2O)6]Cl2 planar compound shows geometrical isomers
(c) [Ni(H2O)5Cl] (d) [Ni (H2O)4Cl2].2H2O [Pt (Cl)x (Br)y]2–
(a) 1, 3 (b) 3, 1 (c) 2, 2 (d) 1, 1
Topic 2: Nomenclature of Coordination Compounds 35. The IUPAC name for the complex [Co(ONO)(NH3)5]Cl2 is
Isomerism in Coordination Compounds (a) pentaamminenitrito-O-cobalt(II) chloride
(b) pentaamminenitrito-O-cobalt(III) chloride
25. K 3[Al(C 2 O 4 ) 3 ] is called (c) nitrito-N-pentaamminecobalt(III) chloride
(a) Potassium aluminooxalate (d) nitrito-N-pentaamminecobalt(II) chloride
(b) Potassium trioxalatoaluminate (III) 36. Chemical formula for iron (III) hexacyanoferrate (II) is
(c) Potassium aluminium (III) oxalate (a) Fe[Fe(CN)6] (b) Fe3[Fe(CN)6]
(d) Potassium trioxalatoaluminate (VI) (c) Fe3[Fe(CN)6]4 (d) Fe4[Fe(CN)6]3
COORDINATION COMPOUNDS 339

37. The type of isomerism present in Pentamminenitro- (a) A – (s), B – (p), C – (q), D – (r)
chromium (III) chloride is (b) A – (p), B – (r), C – (q), D – (s)
(a) optical (b) linkage (c) A – (q), B – (s), C – (p), D – (r)
(c) ionisation (d) polymerisation. (d) A – (p), B – (r), C – (s), D – (q)
38. Which of the following compounds shows optical 48. Which of the following will give a pair of enantiomorphs?
isomerism? (a) [Cr(NH3)6][Co(CN)6] (b) [Co(en)2Cl2]Cl
(a) [Co(CN)6]3– (b) [Cr(C2O4)3]3–
2– (c) [Pt(NH3)4] [PtCl6] (d) [Co(NH3)4Cl2]NO2
(c) [ZnCl4] (d) [Cu(NH3)4]2+
39. Which of the following does not show optical isomerism? (en =NH2CH2CH2NH2)
(a) [Co(NH3)3Cl3]0 (b) [Co (en) Cl2 (NH3)2]+ 49. [Co(NH3)4 (NO2)2] Cl exhibits
(c) [Co (en)3] 3+ (d) [Co (en)2Cl2]+ (a) linkage isomerism, ionization isomerism and
(en = ethylenediamine) geometrical isomerism
40. Which of the following has a square planar geometry? (b) ionization isomerism, geometrical isomerism and
(a) [PtCl4]2– (b) [CoCl4]2– optical isomerism
(c) [FeCl4] 2– (d) [NiCl4]2– (c) linkage isomerism, geometrical isomerism and optical
(At. nos.: Fe = 26, Co = 27, Ni = 28, Pt = 78) isomerism
41. Which one of the following has an optical isomer? (d) linkage isomerism, ionization isomerism and optical
(a) [Zn(en) (NH3)2]2+ (b) [Co(en)3]3+ isomerism
(c) [Co(H2O)4(en)] 3+ (d) [Zn(en)2]2+ 50. The complex given is
3+
(en = ethylenediamine) en
42. The ionisation isomer of [Cr(H2O)4Cl(NO2)]Cl is
(a) [Cr(H2O)4(O2N)]Cl2
(b) [Cr(H2O)4Cl2](NO2)
Co
(c) [Cr(H2O)4Cl(ONO)]Cl
(d) [Cr(H2O)4Cl2(NO2)].H2O en
en
43. Isomerism exhibited by [Cr(NH3)2(H2O)2Cl2]+ are –
(i) non-superimposable on its mirror images
(a) ionization, optical (b) hydrate, optical
(ii) optically inactive
(c) geometrical, optical (d) coordinate, geometrical
(iii) rotate plane polarised light
44. Which of the following coordination compounds would
(iv) planar
exhibit optical isomerism?
(a) (i) and (ii) (b) (i) and (iv)
(a) pentamminenitrocobalt (III) iodide
(c) (i) and (iii) (d) (ii) only
(b) diamminedichloroplatinum (II)
51. Which of the following will give maximum number of
(c) trans-dicyanobis (ethylenediamine) chromium (III)
chloride isomers?
(d) tris-(ethylendiamine) cobalt (III) bromide (a) [Ni(C2O4) (en)2]2– (b) [Ni(en) (NH3)4]2+
(c) [Cr(SCN)2 (NH3)4] + (d) [Co(NH3)4 Cl2]
45. Which of the following complex will show geometrical as
well as optical isomerism (en=ethylenediammine) 52. The terahedral complex [M(A)(B)(X)(Y)], where A, B, X
(a) [Pt(NH3)2Cl2] (b) [Pt(NH3)Cl4] and Y are different ligands and M is a metal ion is
(c) [Pt(en)3]4+ (d) [Pt(en)2Cl2] (a) optically inactive
46. Which one of the following will not show geometrical (b) rotate plane polarized light
isomerism ? (c) incomplete information
(a) [Cr(NH3)4Cl2]Cl (b) [Co(en)2Cl2]Cl (d) can’t be said
(c) [Co(NH3)5NO2]Cl2 (d) [Pt(NH3)2Cl2 ] Topic 3: Bonding in Coordination Compounds
47. Match the complex species given in Column-I with the
isomerism exhibited in Column-II and assign the correct 53. [Sc(H2O)6]3+ ion is :
code: (a) colourless and diamagnetic
Column-I Column-II (b) coloured and octahedral
(Complex species) (Isomerism) (c) colourless and paramagnetic
(A) [Co(NH3)4Cl2]+ (p) optical (d) coloured and paramagnetic
54. Which complex of Co2+ will have the weakest crystal field
(B) cis-[Co(en)2Cl2]+ (q) ionisation
splitting –
(C) [Co(NH3)5(NO2)]Cl2 (r) coordination (a) [CoCl6]4– (b) [Co(CN)6]4–
(D) [Co(NH3)6][Cr(CN)6] (s) geometrical (c) [Co(NH3)6]2+ (d) [Co(en)3]2+
EBD_8350
340 CHEMISTRY

55. In which of the following complexes of the Co (at. no. 27), 64. Assertion : [Cr(H2O)6]+2 ¾® [Cr(H2O)6]+3 while converting,
will the magnitude of Do be the highest? colour continuously changes.
(a) [Co(CN)6]3– (b) [Co(C2O4)3]3–
Reason : CFSE increase during change.
(c) [Co(H2O)6]3+ (d) [Co(NH3)6]3+
(a) If both assertion and reason are true and reason is
56. Among the ligands NH3, en, CN– and CO the correct order
of their increasing field strength, is : the correct explanation of assertion.
(a) NH3 < en < CN – < CO (b) CN – < NH3 < NH3 < en (b) If both assertion and reason are true but reason is not
(c) en < CN – < NH3 < CO (d) CO < NH3 < en < CN – the correct explanation of assertion.
57. The geometry of [Ni(CO)4] and [Ni(PPh3)2Cl2] are (c) If assertion is true but reason is false.
(a) both square planar (d) If both assertion and reason are false.
(b) tetrahedral and square planar 65. Which one of the following cyano complexes would exhibit
(c) both tetrahedral the lowest value of paramagnetic behaviour ?
(d) None of these (a) [Co(CN) 6 ]3 - (b) [Fe(CN) 6 ]3 -
58. Which of the following facts about the complex
(c) [Mn (CN) 6 ]3 - (d) [Cr (CN) 6 ]3 -
[Cr(NH3)6]Cl3 is wrong?
(a) The complex involves d 2sp3 hybridisation and is (At. Nos : Cr = 24, Mn = 25, Fe = 26, Co = 27)
octahedral in shape. 66. When an aqueous solution of copper (II) sulphate is
(b) The complex is paramagnetic. saturated with ammonia, the blue compound crystallises
on evaporation. The formula of this blue compound is:
(c) The complex is an outer orbital complex
(a) [Cu(NH3)4]SO4. H2O (square planar)
(d) The complex gives white precipitate with silver nitrate
solution. (b) [Cu(NH3)4]SO4 (Tetrahedral)
59. Which of the following statements is correct ? (c) [Cu(NH3)6]SO4 (Octahedral)
(Atomic number of Ni = 28) (d) [Cu(SO4) (NH3)5] (Octahedral)
(a) [Ni(CO)4] is diamagnetic and [NiCl4]2– and [Ni(CN)4]2– 67. Atomic number of Cr and Fe are respectively 25 and 26,
are paramagnetic which of the following is paramagnetic?
(b) [Ni(CO) 4] and [Ni(CN)4]2– are diamagnetic and (a) [Cr(CO)6] (b) [Fe(CO)5]
[NiCl4]2– is paramagnetic (c) [Fe(CN)6]4– (d) [Cr(NH3)6]3+
(c) [Ni(CO) 4] and [NiCl 4 ] 2– are diamagnetic and 68. Which one of the following cyano complexes would exhibit
[Ni(CN)4]2– is paramagnetic the lowest value of paramagnetic behaviour ?
(d) [NiCl 4]2– and [Ni(CN)4]2– are diamagnetic and (a) [Co(CN)6]3+ (b) [Fe(CN)6]3–
(c) Mn(CN)6] 3– (d) [Cr(CN)6]3–
[Ni(CO)4] is paramagnetic
60. Assertion : [Fe(CN)6]3– is weakly paramagnetic while (At. Nos : Cr = 24, Mn = 25, Fe = 26, Co = 27)
[Fe(CN)6]4– is diamagnetic. 69. Which of the following is paramagnetic?
Reason : [Fe(CN) 6]3– has +3 oxidation state while (a) [Fe (CN )6 ]4- (b) [Ni (CO )4 ]
[Fe(CN)6]4– has +2 oxidation state. (c) [Ni (CO )4 ]2+ (d) [CoF6 ]3-
(a) If both Assertion and Reason are correct and Reason 70. The correct order of magnetic moments (spin only values
is the correct explanation of Assertion. in B.M.) among is
(b) If both Assertion and Reason are correct, but Reason (a) [Fe(CN)6 ]4 - > [MnCl 4 ]2- > [CoCl 4 ]2 -
is not the correct explanation of Assertion. (b) [MnCl 4 ]2 - > [Fe(CN)6 ]4- > [CoCl 4 ]2 -
(c) If Assertion is correct but Reason is incorrect.
(d) If both the Assertion and Reason are incorrect. (c) [MnCl 4 ]2 - > [CoCl 4 ]2- > [Fe(CN) 6 ]4 -
61. Wh ich of the following complex shows sp 3 d 2 (d) [Fe(CN)6 ]4 - > [CoCl 4 ]2 - > [MnCl 4 ]2 -
hybridization? (Atomic nos. : Mn = 25, Fe = 26, Co = 27)
(a) [Cr(NO2)6]3– (b) [Fe(CN)6]4–
71. Which of the following is paramagnetic?
(c) [CoF6]3– (d) [Ni(CO)4] (a) [Fe(CN)6]4– (b) [Ni(CO)4]
62. Which of the following complexes has square planar
structure? (c) [Ni(CN)4]2– (d) [CoF6]3–
72. The compound which is not coloured is
(a) [Ni(CO)4] (b) [ Ni(CN) 4 ]2 - (a) K4[Fe(CN)6] (b) K3[Fe(CN)6]
(c) [ Ni(Cl) 4 ] (d) [ Zn(NH3 )4 ]
2- 2+
(c) Na2[CdCl4] (d) Na2[CuCl4]
63. Cr3+ in aquous medium forms 5 green coloured complex 73. The color of CoCl3.5NH3.H2O is
with NH3 ligand. How many ligands are associated with it? (a) red (b) orange
(a) 3 (b) 4 (c) 5 (d) 6 (c) orange - yellow (d) pink
COORDINATION COMPOUNDS 341

74. The deep blue colour produced on adding excess of (iii) Optical isomerism is common in octahedral complexes
ammonia to copper sulphate is due to presence of involving bidentate ligands.
(a) Cu 2 + (b) [ Cu(NH3 )4 ]2+ (iv) [Co(NH3)4Cl (NO2)]Cl show linkage isomerism.
2+
(c) [ Cu(NH 3 ) 6 ]2 + (d) [ Cu(NH3 )2 ] (v) Hydrate isomerism is another name of solvate
75. Which of the following complex ion is not expected to isomerism.
absorb visible light ? (a) (i), (ii) and (iii) (b) (i), (iii) and (iv)
(a) [Ni(CN)4]2– (b) [Cr(NH3)6]3+ (c) (ii), (iii) and (v) (d) (iii), (iv) and (v)
85. Which of the following statement(s) is/are incorrect?
(c) [Fe(H2O)6]2+ (d) [Ni(H2O)6]2+
(i) In metal carbonyls M–C s bond is formed by the
76. Which one of the following is an outer orbital complex and
donation of lone pair of electrons on the carbonyl
exhibits paramagnetic behaviour ?
carbon into a vacant orbital of metal.
(a) [Ni(NH3)6]2+ (b) [Zn(NH3)6)]2+ (ii) M—C p bond is formed by the donation of a pair of
(c) [Cr(NH3)6]3+ (d) [Co(NH3)6]3+ electrons from a filled d orbital of metal into the vacant
77. Which of the following options are correct for [Fe(CN)6]3– antibonding p* orbital of CO.
complex ? (iii) Bonding in metal carbonyls is called synergic
(i) Possess d2sp3 hybridisation bonding.
(ii) Possess sp3d2 hybridisation (a) (i) and (ii) (b) (iii) only
(iii) It is paramagnetic (c) (ii) only (d) None of these
(iv) It is diamagnetic 86. Assertion : [Ti(H2O)6] is coloured while [Sc(H2O)6]3+ is
3+
(a) (i) and (iii) (b) (ii) and (iii) colourless.
(c) (i) and (iv) (d) (ii) and (iv) Reason : d-d transition is not possible in [Sc(H2O)6]3+.
78. Consider the following complex [Co(NH3)5CO3]ClO4. The (a) Assertion is correct, reason is correct; reason is a
coordination number, oxidation number, number of correct explanation for assertion.
d-electrons and number of unpaired d-electrons on the
(b) Assertion is correct, reason is correct; reason is not
metal are respectively
a correct explanation for assertion
(a) 6, 3, 6, 0 (b) 7, 2, 7, 1 (c) 7, 1, 6, 4 (d) 6, 2, 7, 3
79. For which of the following ligands magnitude of the crystal (c) Assertion is correct, reason is incorrect
field splitting (Do) will be greater than pairing energy (P)? (d) Assertion is incorrect, reason is correct.
(a) Cl– (b) SCN– (c) CO (d) S2– 87. The d-electron configurations of Cr 2+, Mn2+, Fe2+ and Co2+
80. Which of the following is the limitation of crystal field are d 4, d 5, d 6 and d 7, respectively. Which one of the
theory? following will exhibit minimum paramagnetic behaviour?
(i) Ligands are assumed as point charges. (a) [Mn(H2O)6]2+ (b) [Fe(H2O)6]2+
(ii) It does not accounts for the covalent character of (c) [Co(H2O)6]2+ (d) [Cr(H2O)6]2+
bonding between the ligand and the central atom. (At, nos. Cr = 24, Mn = 25, Fe = 26, Co = 27)
(iii) It does not explain how colour of coordination 88. Correct statements about the following complexes
compounds depends on ligand attached to central [MnCl6]3– and [Mn(CN)6]3– respectively are.
metal atom/ion. (a) Magnetic moment is 4.8 and 2.8
(a) (i) and (ii) (b) (ii) and (iii) (b) inner sphere and outer sphere complexes.
(c) (ii) only (d) (i), (ii) and (iii) (c) sp3d 2 and d 2 sp3 complexes.
81. Which of the following does not have a metal- carbon
(d) Both (a) and (c).
bond?
89. Which of the following complex ions is expected to absorb
(a) Al(OC2H5)3 (b) C2H5MgBr
visible light?
(c) K[Pt(C2H4)Cl3] (d) [Ni(CO)4]
82. In [Fe(CO)5], the Fe – C bond possesses (a) [Ti (en)2(NH3)2]4+ (b) [Cr (NH3)6]3+
(c) [Zn (NH3)6] 2+ (d) [Sc (H2O)3 (NH3)3]3+
(a) ionic character (b) s-character only
(c) p-character (d) both s and p characters (At. no. Zn = 30, Sc = 21, Ti = 22, Cr = 24)
83. The correct structure of [Fe(CO)5] is (Z=26 for Fe) 90. For [Co2(CO)8], what is the total number of metal – carbon
(a) octahedral (b) tetrahedral bonds and number of metal–metal bonds.
(c) square pyramidal (d) trigonal bipyramidal (a) 10 ,1 (b) 8, 2 (c) 8, 1 (d) 10, 0
84. Which of the following statements are correct? 91. CN– is a strong field ligand. This is due to the fact that
(i) Square planar complexes of MABXL type show three (a) it carries negative charge
isomers-two cis and one trans. (b) it is a pseudohalide
(ii) Complexes of Ma3B3 type show three isomers-two (c) it can accept electrons from metal species
cis and one trans. (d) it forms high spin complexes with metal species
EBD_8350
342 CHEMISTRY

92. Which of the following complexes exhibits the highest (a) (i), (ii) and (iii) (b) (ii) and (iii)
paramagnetic behaviour ? (c) (i), and (ii) (d) (ii), (iii) and (iv)
(a) [V(gly)2(OH)2(NH3)2]+ (b) [Fe(en)(bpy)(NH3)2]2+ 98. Match the columns.
(c) [Co(ox)2(OH)2]2– (d) [Ti(NH3)6]3+ Column-I Column-II
where gly = glycine, en = ethylenediamine and bpy = (Complexes) (Absorbed Light)
bipyridyl moities) (A) [Ni(H2O)4(en)] (aq) 2+ (p) Yellow Orange
(At. nos Ti = 22, V = 23, Fe = 26, Co = 27) (B) [Ni(H2O)4(en)2]2+(aq) (q) Blue–Green
93. Which of the following statements is incorrect?
(C) [Ni(en)3]2+(aq) (r) Red
(a) [MnCl6]3– is more paramagnetic than [Mn(CN)6]3–
(a) A – (r), B – (q), C – (p) (b) A – (p), B – (r), C – (q)
(b) Both [Co(C2O4)3]3– and [CoF6]3– are paramagnetie.
(c) A – (q), B – (r), C – (p) (d) A – (r), B – (p), C – (q)
(c) [Fe(CN)6]3– forms inner orbital complex whereas
99. The value of the ‘spin only’ magnetic moment for one of
[FeF6]3– forms outer orbital complex.
the following configurations is 2.84 BM. The correct one
(d) Both (a) and (b).
is
94. Which of the following are inner orbital complex (i.e.,
(a) d 5 (in strong ligand field)
involving d2sp3 hybridisation) and is paramagnetic in
(b) d 3 (in weak as well as in strong fields)
nature?
(c) d 4 (in weak ligand fields)
(a) [Mn(CN)6]3– , [Fe(CN)6]3–, [Co(C2O4)3]3–
(d) d 4 (in strong ligand fields)
(b) [MnCl6]3–, [FeF6]3–, [CoF6]3–
100. Low spin complex of d 6 -cation in an octahedral field will
(c) [Mn(CN)6]3–, [Fe(CN)6]3–
have the following energy :
(d) [MnCl6]3– , [Fe(CN)6]3–, [Co(C2O4)3]3–
-12 -12
95. Which of the following is the limitation of valence bond (a) D0 + P (b) D 0 + 3P
theory? 5 5
(a) It does not distinguish between weak and strong -2 -2
(c) D 0 + 2P (d) D0 + P
ligands. 5 5
(b) It does not give quantitative interpretation of (D0= Crystal Field Splitting Energy in an octahedral field,
magnetic data. P = Electron pairing energy)
(c) It does not explain the colour exhibited by 101. Which of the following carbonyls will have the strongest
coordination compounds C – O bond ?
(d) All of these (a) [Mn (CO)6]+ (b) [Cr (CO)6]
96. Which of the following statements related to crystal field (c) [V (CO)6]– (d) [Fe (CO)5]
splitting in octahedral coordination entities is incorrect?
102. In the isoelectronic series of metal carbonyl, the CO bond
(a) The d 2 2 and dz 2 orbitals has more energy as strength is expected to increase in the order:
x –y
compared to dxy, dyz and dxz orbitals. (a) [Mn(CO) 6 ] + < [Cr(CO)6 ] < [V (CO) 6 ] -
(b) Crystal field spitting energy (Do) depends directly (d) [V (CO)6 ] - < [Cr(CO)6 ] < [Mn(CO)6 ] +
on the charge of the metal ion and on the field (c) [V (CO)6 ] - < [Mn(CO)6 ] + < [Cr(CO) 6 ]
produced by the ligand. (d) [Cr(CO)6 ] < [Mn(CO)6 ] + < [V (CO) 6 ] -
(c) In the presence of Br– as a ligand the distribution of
3 Topic 4: Stability of Coordination Compounds,
electrons for d4 configuration will be t2g , eg1,
Importance and Application of Coordination Compounds
(d) In the presence of CN– as a ligand Do < P.
97. Which of the following statements are correct? 103. Which one of the following coordination compounds is
(i) When light of wavelength 600nm is absorbed by used to inhibit the growth of tumours?
complex [Ti(H2O)6]3+ its configuration changes from (a) Trans-platin (b) EDTAcomplex of calcium
(c) [(Ph3P)3RhCl] (d) Cis-platin
t12 g e0g ¾¾
® t20g e1g and it appears violet in colour..
104. C63H88CoN14O14P is the formulae of the Cyanocobalamine,
(ii) Anhydrous CuSO4 is white but CuSO4 .5H2O is blue (vitamin B12) it contains CN– and CN– is very poisonous,
in colour as presence of H2O as a ligand causes then why this compound does not prove to be fatal for
crystal field spitting. us? (it inhibit the electron transport chain?)
(iii) Ruby is aluminum oxide containing 0.5 – 1% Cr 3+ (a) CN– forms covalent bond
ions with d3 configuration. (b) CN– is coordinating to the cobalt as the ligand
(iv) Crystal field theory predict correctly that anionic (c) CN– hydrolysis immediately
ligands should exert the greater splitting effect. (d) All of these
COORDINATION COMPOUNDS 343

105. Coordination compounds have great importance in (a) A – (r) , B – (p), C – (s), D – (t), E– (q)
biological systems. In this context which of the following (b) A – (p) , B – (r), C – (s), D – (t), E– (q)
statements is incorrect ? (c) A – (r) , B – (s), C – (p), D – (t), E– (q)
(a) Cyanocobalamin is B12 and contains cobalt (d) A – (r) , B – (p), C – (s), D – (q), E– (t)
(b) Haemoglobin is the red pigment of blood and contains 109. Match the columns.
iron Column-I Column-II
(c) Chlorophylls are green pigments in plants and contain (Coordination compound) (Central metal atom)
calcium (A) Chlorophyll (p) Rhodium
(d) Carboxypeptidase - A is an exzyme and contains zinc. (B) Blood pigment (q) Cobalt
ˆˆ† ML4
106. For the reaction of the type M + 4 L ‡ˆˆ (C) Wilkinson catalyst (r) Calcium
(D) Vitamin B12 (s) Iron
(a) larger the stability constant, lower the proportion of (t) Magnesium
ML4 that exists in solution (a) A – (t), B – (s), C – (p), D – (q)
(b) larger the stability constant, higher the proportion of (b) A – (s), B – (q), C – (p), D – (r)
ML4 that exists in solution (c) A – (p), B – (q), C – (r), D – (s)
(c) smaller the stability constant, higher the proportion (d) A – (r), B – (t), C – (p), D – (q)
of ML4 that exists in solution 110. Consider the following reactions.
(d) None of the above [XY]
107. Calculate the value of log K3 when log values of K2, K1, K4 X + Y ƒ XY K1=
[X][Y]
and b4 respectively are 4.0, 3.20, 4.0 and 11.9 ?
(a) 2.0 (b) 2.7 [XY2 ]
XY + Y ƒ XY2 K2 = [XY][Y]
(c) 3.0 (d) 2.5
108. Match the columns. [ XY3 ]
Column-I Column-I XY2 + Y ƒ XY3 K3 = XY Y
[ 2 ][ ]
(A) Estimation of (p) [Ag(CN)2]–
On the basis of reactions above which of the following is
water hardness. incorrect?
(B) Extraction of silver. (q) [Ni(CO)4] (a) Overall stability constant = K1K2K3
(C) Hydrogenation of (r) Na2EDTA [XY3 ]
alkenes. (b) K1K2K3 =
[X][Y]3
(D) Photography (s) [(Ph3P)3RhCl] 1
(E) Purification of (t) [Ag(S2O3)2]3– (c) Dissociation constant = .
Formation constant
Nickel. (d) All of the above are correct.

1. 0.02 mole of [Co(NH3) 5Br]Cl 2 and 0.02 mole of 6. What is the crystal field stabilization energy for high spin
[Co(NH3)5Cl]SO4 are present in 200 cc of a solution X. d 6 octahedral complex?
Calculate the number of moles of the precipitate Y formed 7. How many of the following complexes are optically
when the solution X is treated with excess silver nitrate.
inactive?
2. 50 mL of 0.2 M solution of a compound with empirical
(i) trans-[Co(NH3)4I2]+ (ii) cis-[Co(NH3)2(en)2]3+
formula CoCl 3.4NH3 on treatment with excess of 3+
AgNO3(aq) yields 1.435g of AgCl. Ammonia is not removed (iii) trans-[Co(NH3)2(en)2]
by treatment with concentrated H2SO4. How many ionisable 8. What is the coordination number of Fe(II) in
chlorine atoms are present in the formula of the compound? oxyhaemoglobin?
3. The ion Co (en )Cl 2 Br2- is expected to have x isomers. 9. If PQRS are four different ligands, then how many
geometric isomers will be found for square planar [Pt.
Find the value of x.
4. How many of the following compounds have optical isomers? PQRS]2+?
(I) [Co(en)3]Br3 (III) [Co(NH3)3Br3] 10. How many of the following complexes (K-P) are
(II) [Co(en)2Br2]Br (IV) [Co(en)(NH3)2Br2]Br diamagnetic?
5. Considering H2O as a weak field ligand, find the number of K 3 [ F e( C N ) 6 ] (K), [ C o ( NH 3 ) 6 ] Cl 3 ( L ) ,
unpaired electrons in [Mn(H2O)6]2+. Na 3 [C o( ox al at e) 3 ] (M ), t he [ Ni(H 2 O) 6 ]Cl 2 (N),
(At. no. of Mn = 25) K 2[Pt(CN)4] (O) and [Zn(H2O) 6](NO3)2 (P)
EBD_8350
344 CHEMISTRY

Exercise 3 : NCERT Exemplar & Past Year MCQs


NCERT Exemplar MCQs (a) 1 : 3 electrolyte (b) 1 : 2 electrolyte
1. Which of the following complexes formed by Cu2+ ions is (c) 1 : 1 electrolyte (d) 3 : 1 electrolyte
most stable? 10. When 1 mole of CrCl3 . 6H2O is treated with excess of AgNO3,
(a) Cu2+ + 4NH3 ® [Cu(NH3)4]2+, log K = 11.6 3 moles of AgCl are obtained. The formula of the complex
(b) Cu + 4CN ® [Cu(CN)4] ,
2+ – 2– log K = 27.3 is
(c) Cu2+ + 2en ® [Cu(en)2]2+, lok K = 15.4 (a) [CrCl3(H2O)3] . 3H2O
(d) Cu2+ + 4H2O ® [Cu(H2O)4]2+, log K = 8.9 (b) [CrCl2(H2O)4]Cl . 2H2O
2. The correct IUPAC name of [Pt (NH3)2Cl2] is (c) [CrCl(H2O)5]Cl2 . H2O
(a) Diamminedichloridoplatinum (II) (d) [Cr(H2O)6]Cl3
(b) Diamminedichloridoplatinum (IV) 11. The stabilisation of coordination compounds due to
(c) Diamminedichloridoplatinum (0) chelation is called the chelating effect. Which of the
(d) Dichloridodiammineplatinum (IV) following is the most stable complex species?
3. Due to the presence of ambidentate ligands coordination (a) [Fe(CO)5] (b) [Fe(CN)6]3–
compounds show isomerism. Palladium complexes of the (c) [Fe(C2O4)3]3– (d) [Fe(H2O)6]3+
type [Pd(C6H5)2 (SCN)2] and [Pd(C6H5)2 (NCS)2] are 12. Indicate the complex ion which shows geometrical
(a) linkage isomers (b) coordination isomers isomerism.
(c) ionisation isomers (d) geometrical isomers (a) [Cr(H2O)4Cl2]+ (b) [Pt(NH3)3Cl]3–
(c) [Co(NH3)6] 3– (d) [Co(CN)5(NC)]3–
4. The compounds [Co(SO4) (NH3)5] Br and [Co(SO4) (NH3)5]
Cl represent 13. The CFSE for octahedral [CoCl6]4– is 18,000 cm–1. The CFSE
(a) linkage isomerism (b) ionisation isomerism for tetrahedral [CoCl4]2– will be
(c) coordination isomerism (d) no isomerism (a) 18,000 cm–1 (b) 16,000 cm–1
5. A chelating agent has two or more than two donor atoms (c) 8,000 cm–1 (d) 20,000 cm–1
to bind to a single metal ion. Which of the following is not 14. Which of the following species is not expected to be a ligand?
a chelating agent? (a) NO (b) NH+4
(a) Thiosulphato (b) Oxalato (c) NH2CH2CH2NH2 (d) CO
(c) Glycinato (d) Ethane-1, 2-diamine
6. What kind of isomerism exists between [Cr(H2O)6] Cl3 Past Year MCQs
(violet) and [Cr (H2O)5Cl] Cl2 . H2O (greyish-green)? 15. Among the following complexes the one which shows zero
(a) Linkage isomerism (b) Solvate isomerism crystal field stabilization energy (CFSE):
(c) Ionisation isomerism (d) Coordination isomerism [AIPMT 2014, A]
7. IUPAC name of [Pt(NH3)2Cl(NO2)] is (a) [Mn(H2O)6]3+ (b) [Fe(H2O)6]3+
(a) Platinum diamminechloronitrite (c) [Co(H2O)6]2+ (d) [Co(H2O)6]3+
(b) Chloronitrito-N-ammineplatinum (II) 16. Which of the following complexes is used as an anti-cancer
(c) Diamminechloridonitrito-N-platinum (II) agent: [AIPMT 2014, A]
(d) Diamminechloronitrito-N-plantinate (II)
(a) mer-[Co(NH3)3Cl3]
8. The colour of the coordination compounds depends on
(b) cis-[PtCl2(NH3)2]
the crystal field splitting. What will be the correct order of
absorption of wavelength of light in the visible region, for (c) cis-K2[PtCl2Br2]
the complexes, [Co(NH3)6]3+, [Co(CN)6]3–, [Co(H2O)6]3+. (d) Na2CoCl4
(a) [Co(CN)6]3– > [Co(NH3)6]3+ > [Co(H2O)6]3+ 17. The octahedral complex of a metal ion M3+ with four
(b) [Co(NH3)6]3+ > [Co(H2O)6]3+ > [Co(CN)6]3– monodentate ligands L1, L2, L3 and L4 absorb wavelengths
in the region of red, green, yellow and blue, respectively.
(c) [Co(H2O)6]3+ > [Co(NH3)6]3+ > [Co(CN)6]3–
The increasing order of ligand strength of the four ligands
(d) [Co(CN)6]3– > [Co(NH3)6]3+ < [Co(H2O)6]3+ is: [JEE M 2014, A]
9. When 0.1 mol CoCl3 (NH3)5 is treated with excess of
(a) L4 < L3 < L2 < L1 (b) L1 < L3 < L2 < L4
AgNO3, 0.2 mol of AgCl are obtained. The conductivity of
solution will correspond to (c) L3 < L2 < L4 < L1 (d) L1 < L2 < L4 < L3
COORDINATION COMPOUNDS 345

18. The equation which is balanced and represents the correct (a) trans-[Co(en)2Cl2]Cl (b) [Co(NH3)4Cl2]Cl
product(s) is: [JEE M 2014, S] (c) [Co(NH3)3Cl3] (d) cis-[Co(en)2Cl2]Cl
(a) Li2O + 2KCl ® 2LiCl + K2O
28. The pair having the same magnetic moment is:
(b) [CoCl(NH3)5]+ + 5H+ ® Co2+ + 5NH+4 + Cl– [At. No.: Cr = 24, Mn = 25, Fe = 26, Co = 27]
excess NaOH
(c) [Mg(H2O)6]2+ + (EDTA)4– ¾¾¾¾¾¾ ® [JEE M 2016, S]
[Mg(EDTA)]2+ + 6H2O (a) [Mn(H2O)6]2+ and [Cr(H2O)6]2+
(d) CuSO4 + 4KCN ® K2[Cu(CN)4] + K2SO4 (b) [CoCl4]2– and [Fe(H2O)6]2+
19. Cobalt (III) chloride forms several octahedral complexes
(c) [Cr(H2O)6]2+ and [CoCl4]2–
with ammonia. Which of the following will not give test of
chloride ions with silver nitrate at 25ºC ? (d) [Cr(H2O)6]2+ and [Fe(H2O)6]2+
[AIPMT 2015, A] 29. HgCl2 and I2 both when dissolved in water containing I
(a) CoCl3·4NH3 (b) CoCl3·5NH3 ions the pair of species formed is : [NEET 2017, S]
(a) HgI2, I – 2
(b) HgI4 , I3-
-
(c) CoCl3·6NH3 (d) CoCl3·3NH3
20. The sum of coordination number and oxidation number of (c) Hg2I2, I– (d) HgI2 , I3-
the metal M in the complex [M(en)2(C2O4)]Cl (where en is 30. The correct order of the stoichiometries of AgCl formed
ethylenediamine) is: [AIPMT 2015 RS, S] when AgNO3 in excess is treated with the complexes :
(a) 9 (b) 6 (c) 7 (d) 8 CoCl3.6NH3, CoCl3.5NH3, CoCl3.4NH3 respectively is :-
21. The name of complex ion, [Fe(CN)6]3– is : [NEET 2017, A]
(a) Hexacyanoiron (III) ion [AIPMT 2015 RS, S] (a) 3 AgCl, 1 AgCl, 2 AgCl
(b) Hexacyanitoferrate (III) ion (b) 3 AgCl, 2 AgCl, 1 AgCl
(c) Tricyanoferrate (III) ion (c) 2 AgCl, 3 AgCl, 1 AgCl
(d) Hexacyanidoferrate (III) ion (d) 1 AgCl, 3 AgCl, 2 AgCl
22. Number of possible isomers for the complex [Co(en)2Cl2]Cl 31. Correct increasing order for the wavelengths of absorption
will be (en = ethylenediamine) [AIPMT 2015 RS, S] in the visible region the complexes of Co3+ is :
(a) 2 (b) 1 (c) 3 (d) 4 [NEET 2017, S]
23. Which of these statements about [Co(CN)6]3– is true ? (a) [Co(H2O)6] , [Co(en)3] , [Co(NH3)6]3+
3+ 3+

[AIPMT 2015, C] (b) [Co(H2O)6]3+, [Co(NH3)6]3+, [Co(en)3]3+


3–
(a) [Co(CN)6] has four unpaired electrons and will be (c) [Co(NH3)6]3+, [Co(en)3]3+, [Co(H2O)6]3+
in a low-spin configuration. (d) [Co(en)3]3+, [Co(NH3)6]3+, [Co(H2O)6]3+
(b) [Co(CN)6]3– has four unpaired electrons and will be 32. Pick out the correct statement with respect to [Mn(CN)6]3–
in a high spin configuration.
[NEET 2017, S]
(c) [Co(CN)6]3– has no unpaired electrons and will be in (a) It is sp3d2 hybridised and tetrahedral
a high-spin configuration.
(b) It is d2sp3 hybridised and octahedral
(d) [Co(CN)6]3– has no unpaired electrons and will be in (c) It is dsp2 hybridised and square planar
a low-spin configuration.
(d) It is sp3d2 hybridised and octahedral
24. Which of the following compounds is not colored yellow? 33. An example of a sigma bonded organometallic compound
[JEE M 2015, A] is : [NEET 2017, A]
(a) (NH4)3[As(Mo3O10)4] (b) BaCrO4 (a) Grignard's reagent (b) Ferrocene
(c) Zn2[Fe(CN)6] (d) K3[Co(NO2)6] (c) Cobaltocene (d) Ruthenocene
34. On treatment of 100 mL of 0.1 M solution of CoCl3 . 6H2O
25. The number of geometric isomers that can exist for square
with excess AgNO3; 1.2 × 1022 ions are precipitated. The
planar complex [Pt(Cl)(py)(NH3)(NH2OH)]+ is (py = pyridine).
complex is : [JEE M 2017, S]
[JEE M 2015, S] (a) [Co(H2O)4 Cl2]Cl.2H2O
(a) 4 (b) 6 (c) 2 (d) 3 (b) [Co(H2O)3Cl3].3H2O
26. Which of the following has longest C–O bond length? (c) [Co(H2O)6]Cl3
(Free C–O bond length in CO is 1.128Å) [NEET 2016, S]
(d) [Co(H2O)5Cl]Cl2.H2O
(a) [Ni(CO)4] (b) [Co(CO)4]–
(c) [Fe(CO)4] 2– (d) [Mn(CO)6]+ 35. Iron carbonyl, [Fe(CO)5] is [NEET 2018, C]
(a) Tetranuclear (b) Mononuclear
27. Which one of the following complexes shows optical
(c) Dinuclear (d) Trinuclear
isomerism? [JEE M 2016, S]
EBD_8350
346 CHEMISTRY

36. The type of isomerism shown by the complex [CoCl2(en)2]


is [NEET 2018, C]
(a) Geometrical isomerism (d)
(b) Coordination isomerism
(c) Linkage isomerism
(d) Ionization isomerism
41. The degenerate orbitals of [Cr(H2O) 6]3+ are:
37. The geometry and magnetic behaviour of the complex
[JEE M 2019, C]
[Ni(CO)4] are [NEET 2018, C]
(a) dxz and dyz
(a) Square planar geometry and diamagnetic
(b) Tetrahedral geometry and diamagnetic (b) dyz and dz 2
(c) Tetrahedral geometry and paramagnetic (c) dz2 and dxz
(d) Square planar geometry and paramagnetic (d) dx2- y2 and dxy
38. Consider the following reaction and statements: 42. Two complexes [Cr (H2O)6] Cl3 (A) and [Cr (NH3)6] Cl3
[Co(NH3)4Br2]+ + Br– ® [Co(NH3)3Br3] + NH3 (B) are violet and yellow coloured, respectively. The
[JEE M 2018, C] incorrect statement regarding them is:
[JEE M 2019, S]
(I) Two isomers are produced if the reactant complex ion
(a) D0 values of (A) and (B) are calculated from the
is a cis-isomer.
energies of violet and yellow light, respectively.
(II) Two isomers are produced if the reactant complex ion
(b) both are paramagnetic with three unpaired electrons.
is a trans -isomer.
(c) both absorb energies corresponding to their
(III) Only one isomer is produced if the reactant complex complementary colors.
ion is a trans -isomer. (d) D0 value for (A) is less than that of (B).
(IV) Only one isomer is produced if the reactant complex 43. Which of the following is the correct order of increasing
ion is a cis-isomer. field strength of ligands to form coordination compounds?
The correct statements are: [NEET 2020, S]
(a) (I) and (II) (b) (I) and (III) (a) SCN– < F– < CN– < C2 O 24 -
(c) (III) and (IV) (d) (II) and (IV)
(b) F– < SCN–<C2 O 24 - < CN–
39. What is the correct electronic configuration of the central
atom in K4[Fe(CN)6] based on crystal field theory ? (c) CN– < C2 O 24 - < SCN– < F–
[NEET 2019, A] (d) SCN– < F– < C2 O 24 - < CN–
44. The calculated spin only magnetic moment of Cr2+ ion is
(a) t 42g e 2g (b) t 62g e 0g [NEET 2020, A]
(a) 4.90 BM (b) 5.92 BM
(c) e3 t 32 (d) e4 t 22
(c) 2.84 BM (d) 3.87 BM
40. The one that will show optical activity is: 45. The IUPAC name of the complex [Pt (NH3)2Cl(NH2CH3)]Cl
(en = ethane 1, 2-diamine) [JEE M 2019, C] is: [JEE M 2020, C]
(a) Diamminechlorido (methanamine) platinum (II)
chloride
(a)
(b) Diammine (methanamine) chlorido platinum (II)
chloride
(c) Diamminechlorido (aminomethane) platinum (II)
chloride
(d) Bisammine (methanamine) chlorido platinum (II)
chloride
(b)
46. The theory that can completely/properly explain the nature
of bonding in [Ni(CO)4] is: [JEE M 2020, S]
(a) Werner’s theory
(b) Molecular orbital theory
(c) (c) Crystal field theory
(d) Valence bond theory
COORDINATION COMPOUNDS 347

Exercise 4 : Problem Solving Skill Enhancer MCQs


1. Incorrect matching for given complex compound/ion and (a) K, L, M, N (b) K, M, O, P
its characteristics: (c) L, M, O, P (d) L, M, N, O
(a) [CrBrCl(en)2]Br Ionization and optical 7. Which of the following complex compound(s) is/are
isomerism paramagnetic and low spin?
(b) [CoBr3(H2O)3] Fac-mer and hydrate
isomerism (I) K 3 ëé Fe ( CN 6 ) ûù (II) [ Ni(CO)4 ]0
3+ 4–
(c) [PtCl2(NH3)4] Linkage isomerism and (III) [ Cr(NH3 )6 ] (IV) [ Mn(CN)6 ]
[Co(SCN)4] paramagnetic character Choose the correct code:
(d) [Co(ox)3]3– Inner orbital complex and (a) I only (b) II and III
optical isomerism (c) I and IV (d) IV only
2. [ Ni(NH 3 )6 ]2++ en ¾¾® X + 2NH 3 8. An aqueous solution of titanium chloride, when subjected
to magnetic measurement, measured zero magnetic
[ Ni(NH 3 )6 ] + 2en ¾¾® Y + 4NH 3 moment. Assuming the octahedral complex in aqueous
[ Ni(NH 3 )6 ] + 3en ¾¾® Z + 6NH 3 solution, the formulae of the complex is:
Which of them show optical as well as geometrical (a) [Ti(H2O)6 ] Cl2 (b) [Ti(H2 O)6 ] Cl4
isomerism?
(a) X (b) Y (c) [TiCl3 (H 2O)3 ] (d) [TiCl2 (H2 O)4 ]
(c) Z (d) All of these 9. For complex ion/compound formation reactions
3. [Cr(H2O)6]Cl3 (atomic number of Cr = 24) has a magnetic (I) Co3+ (aq.) + EDTA4- ¾¾
®P
moment of 3.83 BM. The correct distribution of 3d-
electrons in the chromium present in the complex is (II) Ni 2+ (aq.) + dmg (excess) ¾¾¾¾
NH 4OH
®Q

(a) 3d 1xy ,3d 1 yz ,3d 1zx (III) Zn 2 + (aq.) + gly(excess) ¾¾


®R
(b) 3d 1xy ,3d 1 yz ,3d 1 (IV) Pt 4 + (aq.) + en(excess) ¾¾®S
z2

(c) 3d 2 2 ,3d 2 ,3d 1zx


1 1 which of the following complex ion/compound does not
(x -y ) z
exhibit optical activity?
(d) 3d xy ,3d 2 2 ,3d 1 xz
1 1
(a) P (b) Q
(x -y )
4. An octahedral complex with molecular composition (c) R (d) S
M.5 NH3.Cl.SO4 has two isomers, A and B. The solution of 10. Which of the following order of stability of complex ion is
A gives a white precipitate with AgNO3 solution and the Incorrect?
3- 3+
solution of B gives white precipitate with BaCl2 solution. (a) [Fe(C 2 O 4 )3 ] < [Fe(H 2 O) 6 ]
The type of isomerism exhibited by the complex is:
(a) Linkage isomerism (b) Ionisation isomerism
(b) [ Fe(edta)]- > [ Fe(en)3 ]3+
(c) Coordinate isomerism (d) Geometrical isomerism (c) [ Ni(en)2 ]2+ > [ Ni(DMG) 2 ]
5. Which of the following is correctly matched? (d) [ Fe(CN)6 ]3- > [ Fe(CN)6 ]4-
Column - I Column - II Column - III 11. Which of the following complex ions has electrons that
are symmetrically filled in both t2g and eg orbitals ?
(a) [Cr(CO)6 ] Paramagnetic 3 2
Octahedral, sp d
(a) [FeF3]3– (b) [Mn(CN)6]4–
(b) [ Fe(CO)5 ] Paramagnetic Trigonal bipyramidal, sp3d
(c) [CoF6]3– (d) [CO(NH3)6]2+
(c) [Co(CO)4 ] Paramagnetic Square planar, dsp 2
12. Which of the characteristic is not common between
(d) [ Ni(CO)4 ] Diamagnetic Square planar, dsp 2 [Cu(en)2)2+ and [Ni(dmg)2] ?
6. Among the following complexes (K-P) (a) Geometry of complexes
K3[Fe(CN)6] (K), [Co(NH3)6]Cl3 (L), Na3[Co(oxalate)3] (M), (b) Hybridization of central metal cation
[Ni(H2O)6]Cl2 (N), K2[Pt(CN)4] (O) and [Zn(H2O)6](NO3)2 (P) (c) Magnetic behaviour
the diamagnetic complexes are (d) Number of stereoisomers
EBD_8350
348 CHEMISTRY

13. [NiCl2 {P(C2H5)2(C6H5)}2] exhibits temperature dependent (a) The oxidation states of Co in (A), (B) and (C) are +3,
magnetic behaviour (paramagnetic/diamagnetic). The +2 and +3, respectively.
coordination geometries of Ni2+ in the paramagnetic and (b) Complexes (A) and (B) have sp3d2 hybridisation state
diamagnetic states are respectively while (C) has dsp3 hybridisation state.
(a) tetrahedral and tetrahedral (c) The spin multiplicities of Co in (A), (B) and (C) are 3,
(b) square planar and square planar 2 and 1, respectively.
(c) tetrahedral and square planar (d) The oxidation states of Co in (A), (B) and (C) are +6,
+8 and +1, respectively.
(d) square planar and tetrahedral
15. Among the following pairs of complexes, in which case
14. The magnetic moment of a certain complex (A) of Co was
the D0 value is higher for the first one?
found to be 4.89 BM and the EAN as 36. Co also forms
complex (B) with magnetic moment 3.87 BM and EAN as (a) [Co(NH3 )6 ]3+ and [Co(CN) 6 ]3-
37, and complex (C) with EAN as 36 but diamagnetic. (b) [Co(F6 )]3– and [Co(NH3 )6 ]3+
Which of the following statements is true regarding the
above observation?
(c) [Co(H 2O)6 ]2+ and [Co(H 2 O)6 ]3+
(d) [ Rh(H 2O)6 ]3+ and [Co(H 2O)6 ]3+

ANSWER KEY
Exercise 1 : NCERT Based Topic-wis e MCQs
1 (b) 12 (d) 23 (b) 34 (c) 45 (d) 56 (a) 67 (d) 78 (a) 89 (b) 100 (b)
2 (c) 13 (c) 24 (b) 35 (b) 46 (c) 57 (c) 68 (a) 79 (c) 90 (a) 101 (a)
3 (c) 14 (a) 25 (b) 36 (d) 47 (a) 58 (c) 69 (d) 80 (a) 91 (b) 102 (b)
4 (d) 15 (c) 26 (a) 37 (b) 48 (b) 59 (b) 70 (c) 81 (a) 92 (c) 103 (d)
5 (d) 16 (a) 27 (c) 38 (b) 49 (a) 60 (b) 71 (d) 82 (d) 93 (b) 104 (b)
6 (a) 17 (c) 28 (a) 39 (a) 50 (c) 61 (c) 72 (c) 83 (d) 94 (c) 105 (c)
7 (d) 18 (b) 29 (a) 40 (a) 51 (c) 62 (b) 73 (d) 84 (b) 95 (d) 106 (b)
8 (d) 19 (d) 30 (c) 41 (b) 52 (b) 63 (d) 74 (b) 85 (d) 96 (d) 107 (b)
9 (c) 20 (a) 31 (b) 42 (b) 53 (a) 64 (a) 75 (a) 86 (a) 97 (b) 108 (a)
10 (c) 21 (d) 32 (c) 43 (c) 54 (a) 65 (a) 76 (a) 87 (c) 98 (d) 109 (a)
11 (b) 22 (c) 33 (a) 44 (d) 55 (a) 66 (a) 77 (a) 88 (d) 99 (d) 110 (d)
Exercise 2 : Numeric/Integer Answer Ques tions
1 (0.04) 2 (1) 3 (4) 4 (1) 5 (5) 6 (0) 7 (2) 8 (6) 9 (3) 10 (4)
Exercis e 3 : NCERT Exemplar & Past Year MCQs
1 (b) 6 (b) 11 (c) 16 (b) 21 (d) 26 (c) 31 (d) 36 (a) 41 (a) 46 (b)
2 (a) 7 (c) 12 (a) 17 (b) 22 (c) 27 (d) 32 (b) 37 (b) 42 (a)
3 (a) 8 (c) 13 (c) 18 (b) 23 (d) 28 (d) 33 (a) 38 (b) 43 (d)
4 (d) 9 (b) 14 (b) 19 (d) 24 (c) 29 (b) 34 (d) 39 (b) 44 (a)
5 (a) 10 (d) 15 (b) 20 (a) 25 (d) 30 (b) 35 (b) 40 (c) 45 (a)
Exercise 4 : Problem Solving Skill Enhancer MCQs
1 (b) 3 (a) 5 (c) 7 (c) 9 (b) 11 (a) 13 (c) 15 (d)
2 (b) 4 (b) 6 (c) 8 (b) 10 (c) 12 (c) 14 (a)
Haloalkenes and
24 Haloarenes
Trend Buster NEET & JEE Main

Number of Questions from 2020-15 8 6 Minimum one question has been asked in
Weightage 2.5% 3% NEET and JEE M from this chapter every year.

The most Important Concepts that Cover Maximum number of Questions asked in past 6 years.

Physical and Chemical Properties of Haloalkanes & 8 5


Haloarenes
Less Important Concepts that Cover 1 or 2 Questions asked in past 6 years.

Methods of Preparation of Haloalkanes & Haloarenes — 1


Classification & Nomenclature / Polyhalogen Compounds — —

NEET JEE
Concept Used

2020 Physical and Chemical Elimination Reactions / Wurtz 2 Average 1 Average


Properties of Haloalkanes reaction
& Haloarenes
2019 Physical and Chemical Addition and Substtution — — 1 Average
Properties of Haloalkanes Reactions
& Haloarenes
2018 Physical and Chemical Electrophilic Substitution 1 Average 1 Average
Properties of Haloalkanes reactions / Substitution &
& Haloarenes Elimination Reactions
2017 Physical and Chemical Substitution & Elimination — — 2 Average
Properties of Haloalkanes Reactions / Reactivity order of
& Haloarenes SN 1 Reactions
2016 Physical and Chemical Addition, Elimination & 2 Average — —
Properties of Haloalkanes Substitution Reactions /
& Haloarenes Optical Activity
2015 Physical and Chemical SN1 & SN2 reaction Mechanism / 3 Easy / 1 Easy
Properties of Haloalkanes Stereochemistry of SN 1 Average
& Haloarenes / Methods of Reactions / Optical Isomerism /
Preparation of Haloalkanes Swarts Reaction
& Haloarenes
EBD_8350
350 CHEMISTRY
HALOALKENES AND HALOARENES 351
EBD_8350
352 CHEMISTRY

Problem Solving Tips/ Tricks/ Points to Remember


4 Prediction of number of optical isomers in compounds (ii) With increase in electronegativity, nucleophilicity
having chiral centre : decreases from left to right in the periodic table.
(i) When the molecule is chiral. – –
OH > F ; NH 3 > H 2O ; R 3P > R 2S
Number of d and l isomers (a) = 2n , Number of meso
forms (m) = 0 (iii) Nucleophilicity increases down the periodic table,
\ Total number of optical isomers = (a + m) = 2n following the increase in size and polarizability.
Where n is the number of differently substituted chiral I– > Br– > Cl– > F–;
carbon atom(s). –SeH > –SH > –OH;
Common example is CH3×CHBr×CHBr×COOH
(ii) When the molecule is symmetrical and has even R3P: > R3N:
number of chiral carbon atoms. The rate of SN1 is independent of the nature of the
Number of d and l isomers (a) = 2(n–1) , Number of meso nucleophile (stronger or weaker) because here the
forms (m) = 2(n/2) – 1 nucleophile attacks on the carbocation (a fast step). The
\ Total number of optical isomers = (a +m) net result is that, other things being equal, a strong
Common example is tartaric acid, nucleophile favours the SN2 reaction, and a weak
HOOC×CHOH×CHOH×COOH
nucleophile favours the SN1 reaction.
(iii) When the molecule is symmetrical and has an
odd number of chiral carbon atoms. 4 Intermolecular versus intramolecular nucleophilic
Number of d and l forms (a) = 2(n–1) – 2(n/2) – 0.5 substitution
Number of meso forms (m) = 2(n/2) – 0.5 A molecule having two functional groups one of which
\ Total number of opital isomers = (a + m) = 2(n–1) acts as nucleophile and other leaving group can undergo
Common example is intermolecular or intramolecular nucleophilic substitution.
* * * If the two such groups are separated from each other by
HOOC - CHOH - CHOH - CHOH - COOH
A carbon atom which is attached to two different atoms four or five carbon atoms, intramolecular reaction occurs
or groups as well as two identical carbon atoms is called leading to the formation of five or six membered (stable)
pseudo chiral carbon atom. For example, in rings (intramolecular SN).
trihydroxyglutaric acid, the central carbon atom is pseudo
chiral while the terminal carbon atoms are chiral. –
O ¾® + Br–
• Nucleophilicity of some common nucleophiles in water Br
and alcohol O
Class of Nucleophile Relative
¾® + Br–
nucleophile reactivity Br O–
Very good R3P:, I–, HS–, RS– > 105 O
– Since three- and four-membered rings are strained
Good R2NH, HO–, RO–, CN–, N 3 104
– – – (unstable) when the two groups are separated by one,
Fair Br , NH3, Cl , RCOO 103

two or three carbon atoms, intermolecular nucleophilic
Weak F , H2O, ROH 1
substitution takes place.
Very weak RCOOH 10–2

4 Trends in nucleophilicity : O– + Br O ¾®
Br
(i) A species with a negative charge is a stronger
nucleophile than a similar neutral species. In particular, a O– –
base is a stronger nucleophile than its conjugate acid. Br O– + Br
– – –
OH > H2O ; SH > H2S ; NH 2 > NH 3
HALOALKENES AND HALOARENES 353

Exercise 1 : NCERT Based Topic-wise MCQs


Topic 1. Classification and Nomenclature 7. IUPAC name of (CH3)3CCl is
(a) 3-Chlorobutane
1. When two halogen atoms are attached to same carbon (b) 2-Chloro-2-methylpropane
atom then it is : (c) t-butyl chloride
(a) vic-dihalide (b) gem-dihalide (d) n-butyl chloride
(c) a, w-halide (d) a, b-halide 8. IUPAC name of
2. Read the following statements and choose the correct CH3CH2C(Br) = CH – Cl is
option. (a) 2-bromo-1-chlorobutene
(i) The general formula of alkyl halides is CnH2n+1 X (b) 1-chloro-2-bromobutene
(ii) The general formula of aryl halides is Cn Hn–1 X (c) 3-chloro-2-bromobutene
(iii) In alkyl halides halogen atom(s) is attached to sp2 (d) None of the above
hybridised carbon atom 9. Which of the following halide is 2° ?
(iv) In aryl halides halogen atom(s) is attached to sp2 (a) Isopropyl chloride (b) Isobutyl chloride
hybridised carbon atom. (c) n-Propyl chloride (d) n-Butyl chloride
10. Phosgene is a common name for
(a) (i), (ii) and (iii) are correct
(a) phosphoryl chloride
(b) (i), (ii) and (iv) are correct
(b) thionyl chloride
(c) (ii), (iii) and (iv) are correct
(c) carbon dioxide and phosphine
(d) (i), (ii), (iii) and (iv) are correct
(d) carbonyl chloride
3. Match the columns
Column - I Column - II Topic 2. Methods of Preparation of
(A) CH2 = CH – CH2Cl (p) gem-Dichloride Haloalkanes and Haloarenes
(B) CH2 = CHX (q) Vinylic halide
11. Halogenation of alkanes is
(C) CH3CHCl2 (r) vic-Dichloride (a) a reductive process (b) an oxidative process
(D) CH2Cl CH2Cl (s) Allylic halide (c) an isothermal process (d) an endothermal process
(a) A – (r), B – (q), C – (p), D – (s) 12. Ethylene dichloride can be prepared by adding HCl to
(b) A – (q), B – (p), C – (s), D – (r) (a) ethane (b) ethylene
(c) A – (s), B – (q), C – (p), D – (r) (c) acetylene (d) ethylene glycol
13. The best method for the conversion of an alcohol into an
(d) A – (r), B – (p), C – (s), D – (q)
alkyl chloride is by treating the alcohol with
4. The IUPAC name of the compound shown below is (a) PCl5
Cl (b) dry HCl in the presence of anhydrous ZnCl2
(c) SOCl2 in presence of pyridine
(d) none of these
14. In the preparation of chlorobenzene from aniline, the most
suitable reagent is
Br
(a) chlorine in the presence of ultraviolet light
(a) 2-bromo-6-chlorocyclohex-1-ene (b) chlorine in the presence of AlCl3
(b) 6-bromo-2-chlorocyclohexene (c) nitrous acid followed by heating with Cu2Cl2
(c) 3-bromo-1-chlorocyclohexene (d) hCl and Cu2Cl2
(d) 1-bromo-3-chlorocyclohexene 15. Conant-Finkelstein reaction for the preparation of alkyl
5. A compound is formed by substitution of two chlorine for iodide is based upon the fact that
two hydrogens in propane. The number of possible (a) sodium iodide is soluble in methanol, while sodium
isomeric compounds is chloride is insoluble in methanol
(a) 4 (b) 3 (c) 5 (d) 2 (b) sodium iodide is soluble in methanol, while NaCl and
6. Which one of the following is not an allylic halide? NaBr are insoluble in methanol
(a) 4-Bromopent-2-ene (c) sodium iodide is insoluble in methanol, while NaCl
(b) 3-Bromo-2-methylbut-1-ene and NaBr are soluble
(c) 1-Bromobut-2-ene (d) the three halogens differ considerably in their
(d) 4-Bromobut-1-ene electronegativity
EBD_8350
354 CHEMISTRY

16. Aryl halides can not be prepared by the reaction of aryl D


alcohols with PCl3, PCl5 or SOCl2 because
(c) H (d) both (a) and (b)
(a) phenols are highly stable compounds.
(b) carbon-oxygen bond in phenols has a partial double CH3
bond character. Cl
(c) carbon-oxygen bond is highly polar 20. Ethyl alcohol reacts with thionyl chloride in the presence
(d) all of these of pyridine to give
17. Which of the following is the best method for synthesis
(a) CH3CH2Cl + H2O + SO2
of 1-bromo-3-chlorobenzene?
(b) CH3CH2Cl + HCl
Cl
HNO Zn HONO Cu Br (c) CH3CH2Cl + HCl + SO2
(a) ¾¾®
3
¾¾® ¾¾® ¾2¾®
2
H 2SO 4 HCl Cold (d) CH3CH2Cl + SO2 + Cl2
21. Following statements are given regarding the preparation
NO2
Cl Cu Br of aryl halides from toluene. Read the following statements
Zn HONO
(b) ¾ ¾2 ® ¾¾®
AlCl HCl
¾¾®
Cold
¾2¾®
2
and choose the correct options.
3
(i) Aryl chlorides and bromides can be easily prepared
Br by this method.
SO Cl2 H2SO4
(c) ¾¾3® ¾¾® (ii) The ortho and para isomers formed in the reaction
H 2SO 4 AlCl3 60°
can not be separated easily due to small difference in
their melting point.
Br
Cl (iii) Reactions with iodine are reversible in nature and
(d) ¾¾2 ® require the presence of an oxidising agent.
AlCl3
(iv) Fluoro compounds are not prepared by this method
18. Which observations will be correct about the major due to low reactivity of fluorine.
products X and Y of the following reaction ? (a) (i) and (iii) are correct
D D (b) (ii) and (iv) are correct
SO2Cl2 alc KOH,D (c) (i), (ii), and (iii) are correct
¾¾ ® X ¾¾¾¾® Y
peroxide (d) All statements are correct
D D 22. Match the columns
Column - I Column - II
D Cl D D
Cl Cl /UV light
(A) C2H6 ¾¾¾¾¾¾
2
C H Cl
® (p) Finkelstein reaction
(i) X is (ii) X is 2 5

D D D D (B) C6H5NH2 (q) Free radical


(±)
NaNO + HCl/Cu Cl
¾¾¾¾¾¾¾¾
2 2 2
® substitution
D 273- 278K
D D C6H5Cl
(C) CH3Cl + NaI ¾® (r) Swarts reaction
(iii) Y is (iv) Y is CH3I + NaCl
D D D D (D) CH3 – Br + AgF ¾® (s) Sandmeyer’s reaction
(±) CH3F + AgBr
(a) (i) and (ii) (b) (ii) and (iv)
(a) A – (q), B – (s), C – (p), D – (r)
(c) (i) and (iii) (d) (i) and (iv)
19. The major product of the following reaction is (b) A – (q), B – (r), C – (p), D – (s)
(c) A – (r), B – (p), C – (s), D – (q)
DCl
CH3 ¾¾® (d) A – (s), B – (r), C – (p), D – (q)
23. Hydrocarbon (CH3)3CH undergoes reaction with Br2 and
Cl2 in the presence of sunlight, if the reaction with Cl is
CH3 Cl highly reactive and that with Br is highly selective so
(a) H (b) H no.of possible products respectively is (are)
Cl CH3 (a) 2, 2 (b) 2, 1
D D (c) 1, 2 (d) 1, 1
HALOALKENES AND HALOARENES 355

24. Fluorobenzene (C 6H5F) can be synthesized in the (c) conversion of configuration


laboratory (d) Both (a) and (b)
(a) by direct fluorination of benzene with F2 gas 32. The order of reactivities of the following alkyl halides for a
(b) by reacting bromobenzene with NaF solution SN2 reaction is
(c) by heating phenol with HF and KF (a) RF > RCl > RBr > RI
(d) from aniline by diazotisation followed by heating the (b) RF > RBr > RCl > RI
diazonium salt with HBF4 (c) RCl > RBr > RF > RI
(d) RI > RBr > RCl > RF
Topic 3. Physical and Chemical Properties of Haloalkanes 33. Which among MeX, RCH2X, R2CHX and R3CX is most
and Haloarenes reactive towards SN2 reaction?
25. The decreasing order of boiling points of alkyl halides is (a) MeX (b) RCH2X
(a) RF > RCl > RBr > RI (c) R2CHX (d) R3CX
(b) RBr > RCl > RI > RF 34. Isopropyl chloride undergoes hydrolysis by
(c) RI > RBr > RCl > RF (a) SN1 mechanism
(d) RCl > RF > RI > RBr (b) SN2 mechanism
26. Which of the following is liquid at room temperature (b.p. (c) SN1 and SN2 mechanisms
is shown against it)? (d) Neither SN1 nor SN2 mechanism
(a) CH3I 42 ºC 35. Tertiary alkyl halides are practically inert to substitution
by SN2 mechanism because of
(b) CH3Br 3 ºC
(a) steric hindrance (b) inductive effect
(c) C2H5Cl 12 ºC (c) instability (d) insolubility
(d) CH3F –78 ºC 36. Which of the following is an optically active compound?
27. Which of the following possesses highest melting point? (a) 1-Butanol (b) 1-Propanol
(a) Chlorobenzene (b) m-dichlorobenzene (c) 2-Chlorobutane (d) 4-Hydroxyheptane
(c) o-dichlorobenzene (d) p-dichlorobenzene 37. An important chemical method to resolve a racemic mixture
28. Read the following statements and choose the correct makes use of the formation of
answer (a) a meso compound (b) enantiomers
(i) The boiling points of isomeric haloalkanes decrease (c) diastereomers (d) racemates
with increase in branching. 38. The process of separation of a racemic modification into
(ii) Among isomeric dihalobenzenes the para-isomers d-and l-enantiomers is called
have higher melting point than their ortho and meta- (a) Resolution
isomers. (b) Dehydration
(iii) The isomeric dihalobenzenes have large difference (c) Revolution
in their boiling and melting points (d) Dehydrohalogenation
(iv) The isomeric dihalobenzenes have nearly same 39. Which of the following will have a mesoisomer also?
boiling point. (a) 2, 3-Dichloropentane
(a) (i), (ii) and (iii) are correct (b) 2, 3-Dichlorobutane
(b) (i) and (iii) are correct (c) 2-Chlorobutane
(c) (ii) and (iv) are correct (d) 2-Hydroxypropanoic acid
(d) (i), (ii) and (iv) are correct 40. Racemic compound has
29. Which one is most reactive towards SN1 reaction? (a) equimolar mixture of enantiomers
(a) C6H5CH(C6H5)Br (b) 1 : 1 mixture of enantiomer and diastereomer
(b) C6H5CH(CH3)Br (c) 1 : 1 mixture of diastereomers
(d) 1 : 2 mixture of enantiomers
(c) C6H5C(CH3)(C6H5)Br
41. An organic molecule necessarily shows optical activity if
(d) C6H5CH2Br it
30. A Grignard reagent may be made by reacting magnesium (a) contains asymmetric carbon atoms
with (b) is non-polar
(a) Methyl amine (b) Diethyl ether (c) is non-superimposable on its mirror image
(c) Ethyl iodide (d) Ethyl alcohol (d) is superimposable on its mirror image
31. In SN1 reaction, the recemization takes place. It is due to 42. Mg reacts with RBr best in
(a) inversion of configuration (a) C2H5OC2H5 (b) C6H5OCH3
(b) retention of configuration (c) C6H5N(CH3)2 (d) Equally in all the three
EBD_8350
356 CHEMISTRY

43. 2-Bromopentane is heated with potassium ethoxide in 51. An aromatic chiral compound G(C8H8Br2) on treatment
ethanol. The major product obtained is with aqueous NaOH gives H(C8H9BrO). On heating G
(a) 2-ethoxypentane (b) pentene-1 with potassium tert-butoxide, I(C8H7Br) is formed. With
(c) trans-2-pentene (d) cis-pentene-2 one equivalent methyl magnesium bromide in ether
44. An alkyl halide reacts with metallic sodium in dry ether. p-bromoisopropylbenzene is formed. Which
The reaction is known as : observation(s) is/are correct about these reaction ?
(a) (I) is an optically inactive aromatic alkene and (H)
(a) Frankland’s reaction (b) Sandmeyer’s reaction
is an optically active alcohol.
(c) Wurtz reaction (d) Kolbe’s reaction
45. When 2-Bromobutane reacts with alcoholic KOH, the CH3
reaction is called
(a) halogenation (b) chlorination (b) G = Br
(c) hydrogenation (d) dehydrohalogenation Br
46. An alkyl halide by formation of its Grignard reagent and
(c) Benzylic halide gives SN1 reaction at a faster rate
heating with water yields propane. What is the original
than alkyl halide.
alkyl halide ?
(d) All are correct.
(a) Methyl iodide (b) Ethyl iodide * *
(c) Ethyl bromide (d) Propyl bromide 52. H3C - CH 2 - CH - CH3 ® H 3C - CH 2 - CH - CH 2 - Cl
| |
47. An organic compound A (C4H9Cl) on reaction with Na/
Cl Cl
diethyl ether gives a hydrocarbon which on mono- What is true about the stereochemistry of the products
chlorination gives only one chloro derivative, then A is formed in this reaction ?
(a) tert-butyl chloride (b) sec-butyl chloride (a) Racemised product
(c) isobutyl chloride (d) n-butyl chloride (b) Inverted product
48. The major product of the following reaction is (c) Retained product
Na–ether Br 2/hv C2H 5O/C2H5OH, D (d) More inverted than retained
Br Cl
53. Which of following can be used as solvent for Grignard
reagent?
(a) H2O (b) C2H5OH
(a) (b)
(c) CH3OH (d) C2H5OC2H5
54. Chlorobenzene reacts with Mg in dry ether to give a
compound (A) which further reacts with ethanol to yield
(c) (d) (a) Phenol (b) Benzene
(c) Ethylbenzene (d) Phenyl ether
55. Benzene reacts with n-propyl chloride in the presence of
Mg ether D2O Na ether
49. Br Cl ¾ ¾ ® A ¾ ¾ ® B ¾ ¾ ® C, C is anhydrous AlCl3 to give
(a) 3 – Propyl – 1 – chlorobenzene
(b) n-Propylbenzene
(c) No reaction
(a) D
(d) Isopropylbenzene
56. C – Cl bond of chlorobenzene in comparison to C – Cl
bond in methyl chloride is
(b) D D (a) Longer and weaker
(b) Shorter and weaker
(c) Shorter and stronger
(c) (d) Longer and stronger
57. Which of the following is not used in Friedel-Craft’s
(d) None of these. reaction?
50. A dihalogen derivative ‘X’ of a hydrocarbon with three (a) N–Phenyl acetanilide (b) Bromobenzene
carbon atoms reacts with alcoholic KOH and produces (c) Benzene (d) Chlorobenzene
another hydrocarbon which forms a red precipitate with 58. Which one of the following is most reactive towards
ammoniacal Cu2Cl2, ‘X ’ gives an aldehyde on reaction nucleophilic substitution reaction?
with aqueous KOH. The compound ‘X’ is (a) CH2 = CH – Cl (b) C6H5Cl
(a) 1, 3-Dichloropropane (b) 1, 2-Dichloropropane
(c) CH3CH = CH – Cl (d) ClCH2 – CH = CH2
(c) 2, 2-Dichloropropane (d) 1, 1-Dichloropropane
HALOALKENES AND HALOARENES 357

59. Haloarenes are ortho and para directing due to (a) B > C > A (b) B > A > C
(a) Resonance in aryl halide (c) C > B > A (d) A > B > C
(b) –I effect of halogen atom 65. Consider the reactions :
C H OH
(c) +I effect of halogen atom (i) (CH 3 )2 CH - CH 2 Br ¾¾¾¾®
2 5

(d) Both (a) and (b) (CH3)2CH – CH2OC2H5 + HBr


60. Read the following statements and choose the correct code C H O-
(ii) (CH 3 )2 CH - CH 2Br ¾¾¾¾®
2 5
(i) SN2 reactions follows a second order kinetics whereas
SN1 reactions follows the first order kinetics (CH3)2CH – CH2OC2H5 + Br–
(ii) SN1 reactions follows the second order kinetics The mechanisms of reactions (i) and (ii) are respectively :
whereas SN2 follows the first order kinetics (a) SN1 and SN2 (b) SN1 and SN1
(iii) SN2 reactions take place in a single step whereas (c) SN2 and SN2 (d) SN2 and SN1
SN1 reactions take place in two steps
66. Br + NaOH Solvent OH
(iv) Tertiary alkyl halides are least reactive towards SN2
reactions but we observe high reactivity towards SN1 For which solvent rate of SN2 will be maximum?
reaction. (a) Benzene
(a) (ii) and (iv) are correct (b) 100% H2O
(b) (i), (iii) and (iv) are correct (c) 100% acetone
(c) (i), (ii) and (iv) are correct (d) 75% H2O + 25% acetone
(d) (ii), (iii) and (iv) are correct
67. Br
61. Read the following statements and choose the correct
Br
option.
+ CH3OH
(i) SN1 reactions are carried out through formation of
carbocation as an intermediate. N Br
(ii) SN1 reactions are two step reactions in which step 1 (A), (A) is
H
is fast and irreversible.
(Excess)
(iii) Step 1 involves breaking of C–Br bond which obtain
energy through solvation of halide. OCH3 Br
(iv) SN1 reactions are two step reactions in which step 2 Br Br
is slow and reversible.
(a) (b)
(v) Allylic and benzylic halides show high reactivity N Br N Br
toward SN1 reactions.
(a) (i), (iii) and (v) are correct H OCH3
(b) (ii), (iii) and (v) are correct Br Br
(c) (i), (iii) and (iv) are correct OCH3 Br
(d) (i), (ii) and (iv) are correct (c) (d)
62. CH 3 - CH 2 - C H - CH 3 obtained by chlorination of N Br N OCH3
|
Cl H H
n-butane, will be
(a) l-form (b) d-form 68. The product (X) of the following reaction is
(c) Meso form (d) Racemic mixture
63. The number of possible enantiomeric pairs that can be CH2OTs
KCN
(X)
produced during monochlorination of 2-methylbutane is DMSO
(a) 2 (b) 3
(c) 4 (d) 1 CN
64. Consider the following bromides :
Me Me (a) (b) CH3
Me Br Me
CN
Br Br
(A) (B) (C)
(c) CH2CN (d) CH2NC
The correct order of SN1 reactivity is
EBD_8350
358 CHEMISTRY

74. Which among the following will give enantiomeric pair on


OH
69. CH3—F
—— CH3—OH ....... (A) heating with water?
(i) NaI Ph
CH3—OH ....... (B) CH3
(ii) OH
(a) I (b) I
The best way to convert CH3F into CH3OH is H Et
D CH2CH3
(a) A (b) B
(c) Both are same (d) None NO 2 CH2CH3
70. Which reaction condition is suitable for the following (c) I (d) H C
reaction? Et 3 C6H5
I
D D Br
OH Br 75. OH SH
H H NaOH (1 Mole)
(X)
(a) HBr solution (b) SOBr2 (Major)

(c) PBr3 (d) Br2/CH2Cl2 OH


(a) HO SH
(X)
Ag2O
OH
71. CH3CH2Br (b)
Moist S
Ag2O (Y)
SH
Select correct statement in the following statements (c)
O
(a) Both (X) and (Y) are same
(b) When dil H2SO4 is mixed in (X), (Y) is obtained
SH
(c) (Y) can decolourise Br2 water (d) HO
(d) (X) can release H2 gas on mixing with Na metal
72. Order of reactivity in SN1 reaction is 76. In which case first halide is more reactive towards SN2
than second halide?
O Cl Cl Cl

Cl
Cl (a)
Cl
(1) (2) (3) (4)
OCH3
(a) 4 > 3 > 2 > 1 (b) 2 > 1 > 4 > 3 (b) Cl Cl
(c) 2 > 4 > 1 > 3 (d) 2 > 4 > 3 > 1
(c)
73. NaBr (1 mole) Cl Cl
¾¾¾¾¾®
CH2Cl in DMF Cl Cl
Cl
CH3
Expected product from this reaction is (d)

Cl OCH3
(a) (b) 77. Which of the following reaction will go faster when con-
CH2Br CH2Br centration of attacking nucleophile is increased?
Cl Cl
CH3 CH3 Ph Ph
CH2Br (a) CH—Br + NH 3 CH—NH2
Ph Ph
(c) CH2Cl (d) OH
Br Cl (b) CH2Br + NaOH ¾ ®
CH3 CH3
HALOALKENES AND HALOARENES 359

(a) A – (s), B – (p), C – (r), D – (q)


(c) Br + NaCN ¾ ® CN
(b) A – (r), B – (p), C – (s), D – (q)
(c) A – (q), B – (s), C – (p), D – (q)
(d) CH3OH
Br + CH3ONa OCH3 (d) A – (r), B – (s), C – (p), D – (q)
86. Which of the following are the harmful effects of methylene
Topic 4. Polyhalogen Compounds chloride?
(a) Impaired hearing and vision
78. CFxCly [where x + y = 4]. These compounds are not used (b) Dizziness, nausea and tingling
because (c) Skin burning
(a) these are fluorocarbons (d) All of these
(b) these are difficult to synthesise 87. If chloroform is left open in air in the presence of sunlight,
(c) they deplete ozone layer it gives
(d) None of the these (a) carbon tetrachloride
79. Which of the following is used in fire extinguishers
(b) carbonyl chloride
(a) CH4 (b) CHCl3
(c) mustard gas
(c) CH2Cl2 (d) CCl4
(d) lewisite
80. Match the columns
88. Full name of DDT is
Column - I Column - II
(Haloalkane/arene) (Applications) (a) 1, 1, 1-trichloro-2, 2-bis(p-chlorophenyl) ethane
(A) Iodoform (p) CF4 (b) 1, 1-dichloro-2, 2-diphenyl trimethylethane
(B) BHC (q) Antiseptic (c) 1, 1-dichloro-2, 2-diphenyl trichloroethane
(C) Freon - 14 (r) Moth repellant (d) None of these
(D) Halothanes (s) Inhalative anesthetic 89. Assertion : Exposure of ultraviolet rays to human causes
(E) p-Dichlorobenzene (t) Termite pesticide the skin cancer, disorder and disrupt the immune system.
(a) A – (q), B – (s), C – (t), D – (r), E – (p) Reason : Carbon tetrachloride is released into air it rises to
(b) A – (q), B – (t), C – (p), D – (s), E – (r) atmosphere and deplets the ozone layer.
(c) A – (r), B – (s), C – (q), D – (p), E – (t) (a) Assertion is correct, reason is correct; reason is a
(d) A – (p), B – (r), C – (t), D – (q), E – (s) correct explanation for assertion.
81. Solvent which is used in the synthesis of chlorofluoro- (b) Assertion is correct, reason is correct; reason is not
carbons a correct explanation for assertion
(a) iodoform (b) chloroform (c) Assertion is correct, reason is incorrect
(c) carbon tetrachloride (d) methylene chloride (d) Assertion is incorrect, reason is correct.
82. Uses of dichloromethane is 90. Which one is correct ?
(a) paint remover (a) Freon-14 is CF4, Freon-13 is CF3Cl, Freon-12 is CF2Cl2
(b) solvent in drugs manufacturing and Freon-11 is CFCl3.
(c) metal cleansing and finishing solvent (b) Freons are chloroflurocarbons.
(d) All of the above (c) Freons are used as refrigerants.
83. Haloforms are trihalogen derivatives of (d) All the above.
(a) Ethane (b) Methane 91. A small amount of the alcohol is usually added to CHCl3
(c) Propane (d) Benzene bottles because
84. AgNO3 does not give precipitate with CHCl3 because (a) it retards the anasthetic property of CHCl3
(a) CHCl3 does not ionise in water (b) it retards the oxidation of CHCl3 to phosgene
(b) CHCl3 does not react with AgNO3 (c) it converts any phosgene formed to harmless ethyl
(c) CHCl3 is chemically inert carbonate
(d) None of these (d) both (b) and (c)
85. Match the columns 92. When CCl4 is boiled with KOH, the product formed is
Column - I Column - II (a) formic acid
(A) Chloroform (p) Antiseptic (b) methyl alcohol
(B) Iodoform (q) Insecticide (c) formaldehyde
(C) Trichloromethane (r) Anesthetic (d) carbon dioxide
(D) DDT (s) Propellant
EBD_8350
360 CHEMISTRY

93. An organic compound X when exposed to sunlight and (a) fluoroform (b) fluoric monoxide
air, slowly decomposes into a poisonous gas Y and HCl. (c) fluorine dioxide (d) fluoro methanol
To prevent from decomposition X is stored in dark brown 97. Which one of the following has antiseptic property?
coloured bottle and in 1% ethyl alcohol. This retards the (a) Dichloromethane (b) Trifluoromethane
decomposition and converts Y into harmless Z. Identify X, (c) Triiodomethane (d) Tetrachloromethane
Y and Z. 98. Chloroform cannot be prepared from which of the
(a) CCl4, COCl2, C2H5CO3 following?
(b) CHCl3, COCl2, (C2H5)2CO3 (a) CH3OH (b) C2H5OH
(c) CCl4, CO2, (C2H5)2CO3 (c) CH3CHO (d) (CH3)2CO
(d) CHCl3, COCl2, C2H5CO3 99. Exposure of CCl4 causes
(a) Liver cancer in human (b) Damage to nerve cells
94. Formula for tear gas is
(c) Coma, unconsciousness (d) All of these
(a) COCl2 (b) CCl3NO2
100. Assertion : CCl4 is not a fire extinguisher.
(c) N2O (d) None of these
Reason : CCl4 is insoluble in water.
95. Which of the following has highest chlorine content ?
(a) Assertion is correct, reason is correct; reason is a
(a) Chloral (b) DDT correct explanation for assertion.
(c) Pyrene (d) Gammexane (b) Assertion is correct, reason is correct; reason is not a
96. The compounds CHCl3 and HF lead to the formatioin of a correct explanation for assertion
compound of fluorine of molecular weight 70. The (c) Assertion is correct, reason is incorrect
compound is (d) Assertion is incorrect, reason is correct.

1. Fin d th e sum of total number of structural and 5. 2-Bromopentane is heated with KOH in alcoholic and
configurational isomers of a bromo compound, C5H9Br, aqueous medium respectively. Find the total number of
formed by the addition of HBr to 2-pentyne respectively. products.
6. In the following sequence of reactions

KCN H O+
3 ® B ¾¾¾¾
4® C LiAlH
CH3 - Br ¾¾¾® A ¾¾¾¾
ether
2. How many sp3 hybridized atoms are present in product C.
NaCN Ni/H Acetic
7. CH3CH 2Cl ¾¾¾¾
® X ¾¾¾¾
2
® Y ¾¾¾¾
®Z
anhydride

Find the sum of carbon and hydrogen atoms in product Z.


Find the sum of total number of isomeric chlorides obtained 8. Consider the following reaction.
in these reactions (consider only the major products).
Alc. KOH Br KCN
3. How many total number of alkenes are possible by C 2 H 5 I ¾¾ ¾ ¾
¾® X ¾¾
¾2 ® Y ¾¾¾® Z

dehydrobromination of 3-bromo-3-cyclopentylhexane
using alcoholic KOH? How many CN group are present in product Z?
4. How many of the following compounds will give white 9. How many following reactions are correct?
precipitate with aqueous AgNO3? (I) RX + AgCN ¾® RNC
(II) RX + KCN ¾® RCN

(III) RX + KNO2 ¾®

(IV) RX + AgNO2 ¾® R–O–N = O


10. In the following sequence of reactions
P +I Mg HCHO
CH 3CH 2 OH ¾¾¾
2 ® A ¾¾¾® B ¾¾¾¾
®
ether
H O
C ¾¾¾
2 ®D
HALOALKENES AND HALOARENES 361

NCERT Exemplar MCQs 5. Which of the following is halogen exchange reactions?


1. The order of reactivity of following alcohols with halogen (a) RX + NaI ® RI + NaX
acids is .............. . C C + HX C C
(b)

||

|
(A) CH3CH 2 — CH 2 — OH | |
H X
(B) CH3CH 2 - CH - OH ZnCl
| (c) R — OH + HX ¾¾¾¾
2®R — X + H O
2
CH3
CH3 CH3
Fe
CH3 + X 2 ¾¾¾ ®
| (d) dark
(C) CH3CH2 – CH – OH X
| CH3
CH3
+
(a) (A) > (B) > (C) (b) (C) > (B) > (A)
X
(c) (B) > (A) > (C) (d) (A) > (C) > (B)
2. Which of the following alcohols will yield th e 6. Which reagent will you use for the following reaction?
corresponding alkyl chloride on reaction with CH3CH2CH2CH3 ® CH3CH2CH2CH2Cl + CH3CH2CHClCH3
concentrated HCl at room temperature? (a) Cl2 /UV light
(a) CH3CH 2 — CH 2 — OH (b) NaCl + H2SO4
(b) CH3CH 2 - CH - OH (c) Cl2 gas in dark
| (d) Cl2 gas in the presence of iron in dark
CH3 7. Arrange the following compounds in the increasing order
(c) CH3CH2 – CH—CH2OH of their densities.
|
CH3 Cl
CH3 (i) (ii)
|
(d) CH3CH2 – C – OH
|
CH3 Cl Br
3. Identify the compound Y in the following reaction.
+ (iii) (iv)
NH3 N 2 Cl
Na NO 2 + HCl Cl Cl
¾¾¾¾¾¾ ® ¾¾¾¾
Cu Cl
2 2 ®Y + N
273- 278 K 2
(a) (i) < (ii) < (iii) < (iv) (b) (i) < (iii) < (iv) < (ii)
Cl (c) (iv) < (iii) < (ii) < (i) (d) (ii) < (iv) < (iii) < (i)
(a) (b) 8. Arrange the following compounds in increasing order of
their boiling points.
Cl
Cl CH3
(i) CH — CH2Br
CH3
(c) (d) (ii) CH3CH2CH2CH2Br
Cl
Cl CH3
4. Toluene reacts with a halogen in the presence of iron (III) |
chloride giving ortho and para halo compounds. The (iii) H3C — C — CH 3
reaction is |
Br
(a) electrophilic elimination reaction
(b) electrophilic substitution reaction (a) (ii) < (i) < (iii) (b) (i) < (ii) < (iii)
(c) free radical addition reaction (c) (iii) < (i) < (ii) (d) (iii) < (ii) < (i)
(d) nucleophilic substitution reaction
EBD_8350
362 CHEMISTRY

9. In which of the following molecules carbon atom marked


(a) CH2 — CH — CH2
with asterisk ( * ) is asymmetric?
Cl
H D

(i) C* (ii) C*
I Cl I Cl
Br Br (b) CH2 — CH2 — CH2 — Cl

H H

(iii)
C* (iv) H
C*
HO CH3 CH3
C2H5 C2H 5 (c) CH2 — CH — CH3
(a) (i), (ii), (iii) and (iv) (b) (i), (ii) and (iii)
Cl
(c) (ii), (iii) and (iv) (d) (i), (iii) and (iv)
10. Which of the following structures is enantiomeric with
the molecule (A) given below?
H
(d) Cl
CH3
C CH — CH2 — CH3
Br
H5C2
(A)

CH3
H 16. A primary alkyl halide would prefer to undergo .................
C2H5 H
(a) C (b) C
Br (a) SN1 reaction (b) SN2 reaction
H3C Br C2H5
(c) a-elimination (d) racemisation
H Br 17. Which of the following alkyl halides will undergo SN1
Br H reaction most readily?
(c) C (d) C
H3C C2H5 (a) (CH3 )3 C — F
H5C2 CH3
(b) (CH3 )3 C — Cl
11. Which of the following is an example of vic-dihalide?
(c) (CH3 )3 C — Br
(a) Dichloromethane
(d) (CH3 )3 C — I
(b) 1, 2-dichloroethane
18. Which is the correct IUPAC name for
(c) Ethylidene chloride
(d) Allyl chloride CH3 — CH — CH 2 — Br ?
|
12. The position of Br in the compound CH3CH = CHC(Br) C2 H5
(CH3)2 can be classified as .............. . (a) 1-bromo-2-ethylpropane
(a) allyl (b) aryl (b) 1-bromo-2-ethyl-2-mehylethane
(c) vinyl (d) secondary (c) 1-bromo-2-methylbutane
13. Chlorobenzene is formed by reaction of chlorine with (d) 2-methyl-1-bromobutane
benzene in the presence of AlCl3. Which of the following 19. What should be the correct IUPAC name for diethylbromo-
species attacks the benzene ring in this reaction? methane?
(a) Cl– (b) Cl+ (a) 1-bromo-1, 1-diethylmethane
(c) AlCl3 (d) [AlCl4]– (b) 3-bromopentane
14. Ethylidene chloride is a/an ................ . (c) 1-bromo-1-ethylpropane
(a) vic-dihalide (b) gem-dihalide (d) 1-bromopentane
(c) allylic halide (d) vinylic halide 20. The reaction of toluene with chlorine in the presence of
15. What is 'A' in the following reaction? iron and in the absence of light yields ............ .
CH2 — CH — CH2 + HCl ¾® A CH2Cl

(a)
HALOALKENES AND HALOARENES 363

CH3
(a) (i) (b) (i), (ii) and (iii)
(c) (ii) and (iii) (d) (i) and (iii)
Cl
Direction (Q. No. 26 to 29) : In the questions 26 to 29 arrange
(b) the compounds in increasing order of rate of reaction towards
nucleophilic substitution.
26.
(c) H3C Cl
Cl Cl Cl
(d) Mixture of (b) and (c) NO2
21. Chloromethane on treatment with excess of ammonia (i) (ii) (iii)
yields mainly NO2
æ CH3ö (a) (i) < (ii) < (iii) (b) (iii) < (ii) < (i)
(a) N, N-dimethylmethanamine çCH3 —N ÷ (c) (i) < (iii) < (ii) (d) (iii) < (i) < (ii)
è CH3ø
(b) N - methylmethanamine (CH3 — NH — CH3) Cl Cl Cl
(c) methanamine (CH3NH2) CH3
(d) mixture containing all these in equal proportion 27. (i) (ii) (iii)
22. Molecules whose mirror image is non – superimposable CH3
over them are known as chiral. Which of the following
molecules is chiral in nature? (a) (i) < (ii) < (iii) (b) (i) < (iii) < (ii)
(a) 2 - bromobutane (c) (iii) < (ii) < (i) (d) (ii) < (iii) < (i)
28.
(b) 1 - bromobutane Cl Cl Cl
(c) 2 - bromopropane
O2N NO2
(d) 2 -bromopropan - 2 - ol
(i) (ii) (iii)
23. Reaction of C6H5CH2Br with aqueous sodium hydroxide
follows .......... .
(a) SN1 mechanism NO2 NO2
(b) SN2 mechanism (a) (iii) < (ii) < (i) (b) (ii) < (iii) < (i)
(c) Any of the above two depen ding upon the (c) (i) < (iii) < (ii) (d) (i) < (ii) < (iii)
temperature of reaction 29.
(d) Saytzeff rule
Cl Cl Cl
24. Which of the carbon atoms presents in the molecule given
below are asymmetric? CH3
(i) (ii) (iii)
OH H O
HO | | | ||
1 2 4
|| C — C — C — C |
3
CH3 CH3
| | H
O H OH (a) (iii) < (ii) < (i) (b) (ii) < (i) < (iii)
(a) 1, 2, 3, 4 (b) 2, 3 (c) (iii) < (ii) < (i) (d) (i) < (iii) < (ii)
(c) 1, 4 (d) 1, 2, 3 30. Which is the correct increasing order of boiling points of
the following compounds?
25. Which of the following compounds will give racemic
mixture on nucleophilic substitution by OH – ion? 1 - iodobutane, 1 - bromobutane, 1 - chlorobutane, Butane
(a) Butane < 1-chlorobutane < 1- bromobutane <
Br 1 - iodobutabe
|
(i) CH3 — CH — Br (ii) CH3 — CH — CH3 (b) 1 - iodobutane < 1 - bromobutane < 1 - chloroubutane
| | < Butane
C2H5 C2H5 (c) Butane < 1 - iodobutane < 1 - bromobutane <
1 - chlorobutane
(iii) CH 3 — CH — CH 2 Br (d) Butane < 1 - chlorobutane < 1 - iodobutane <
| 1 - bromobutane
C2 H 5
EBD_8350
364 CHEMISTRY

31. Which is the correct increasing order of boiling points of (a) (i) and (ii) (b) (ii) and (iv)
the following compounds? (c) (iii) and (iv) (d) (iv)
1 - bromoethane, 1 - bromopropane, 1 - bromobutane,
34. In SN2 reactions, the correct order of reactivity for the
Bromobenzene
(a) Bromobenzene < 1 - bromobutane < 1 - bromopropane following compounds: [JEE M 2014, S]
< 1- bromoethane CH3Cl , CH3CH 2 Cl , (CH3 )2 CHCl and (CH3)3CCl is:
(b) Bromobenzene < 1 - bromobutane < 1 -
bromopropane < 1- bromobutane (a) CH3 Cl > ( CH3 )2 CHCl
(c) 1 - bromopropane < 1 - bromorpropane < 1 - > CH3 CH 2 Cl > ( CH3 )3 CCl
bromoethane < Bromobenzene
(b) CH3Cl > CH 3CH 2 Cl
(d) 1 - bromoethane < 1 - bromopropane < 1 -
> ( CH3 ) 2 CHCl > ( CH 3 )3 CCl
bromobutane < Bromobenzene
(c) CH3CH 2 Cl > CH3Cl
Past Year MCQs
> ( CH3 ) 2 CHCl > ( CH 3 )3 CCl
32. What products are formed when the following compounds (d) ( CH3 )2 CHCl > CH3CH 2Cl
is treated with Br2 in the presence of FeBr3? > CH 3Cl > ( CH3 )3 CCl
[AIPMT 2014, A] 35. In which of the following compounds, the C - Cl bond
CH3 ionisation shall give most stable carbonium ion?
[AIPMT 2015, C]
H
CH3
CH — Cl
CH3 CH3
H3C
Br C — Cl
(a) (a) H3C (b)
and CH3
CH3 CH3
H H 3C H
Br (c) C — Cl (d) C — Cl
O 2NH2C H H 3C
CH3 CH3
36. In an SN1 reaction on chiral centres there is :
Br Br
(b) [AIPMT 2015 RS, A]
and
(a) 100 % racemization
CH3 CH3
(b) inversion more than retention leading to partial
CH3 CH3 racemization
Br (c) 100 % retention
(c) and (d) 100 % inversion
CH3 CH3 37. Two possible stereo-structures of CH3CHOHCOOH, which
Br are optically active, are called. [AIPMT 2015 RS, C]
CH3 CH3 (a) Diastereomers (b) Atropisomers
(c) Enantiomers (d) Mesomers
(d) and 38. The synthesis of alkyl fluorides is best accomplished by :
CH3 Br CH3 [JEE M 2015, C]
(a) Finkelstein reaction
Br (b) Swarts reaction
33. Which of the following compounds will undergo (c) Free radical fluorination
racemisation when solution of KOH hydrolyses? (d) Sandmeyer's reaction
[AIPMT 2014, S] 39. For the following reactions : [NEET 2016, A]
CH2Cl
(1) CH3CH2CH2Br + KOH ¾®
(i) (ii) CH3CH2CH2Cl
CH3CH = CH2+ KBr + H2O
CH3 (2)
CH3 H3C CH3 H3C CH3
| C + KOH ¾® + KBr
(iii) H3C - C H - CH2Cl (iv) H Cl
C2H3 Br OH
HALOALKENES AND HALOARENES 365

Br 44. The major product of the following reaction is :


(3) + Br2 ¾® [JEE M 2018, A]
Br
Which of the following statements is correct?
(a) (1) and (2) are elimination reaction and (3) is addition
reaction
(b) (1) is elimination, (2) is substitution and (3) is addition (a) (b)
reaction
(c) (1) is elimination, (2) and (3) are substitution reactions
(d) (1) is substitution, (2) and (3) are addition reaction (c) (d)
40. Which of the following biphenyls is optically active?
[NEET 2016, A] 45. The major product of the following reaction is:
[JEE M 2019 A]
O2N Br Br
(i) Br
¾¾¾®
2
(ii) EtOH
(a) (b)
I I I OEt OEt

I CH3 Br
(a) (b)
(c) (d)
I CH3
Br OEt
41. The major product obtained in the following reaction is :
[JEE M 2017, A] OEt OEt
(c) (d)
Br
H
C 6H 5
C 6H 5 t-BuOK
¾¾¾® 46. Elimination reaction of 2-Bromo-pentane to form pent-2-
D ene is: [NEET 2020, C]
(a) (±)C6H5CH(OtBu)CH2C6H5 (A) b-Elimination reaction
(b) C6H5CH = CHC6H5 (B) Follows Zaitsev rule
(C) Dehydrohalogenation reaction
(c) (+)C6H5CH(OtBu)CH2C6H5
(D) Dehydration reaction
(d) (–)C6H5CH(OtBu)CH2C6H5 (a) (A), (C), (D) (b) (B), (C), (D)
42. The increasing order of the reactivity of the following (c) (A), (B), (D) (d) (A), (B), (C)
halides for the SN1 reaction is [JEE M 2017, S]
47. Which of the following alkane cannot be made in good
CH3CHCH2CH3 CH3CH 2CH2Cl yield by Wurtz reaction? [NEET 2020, C]
| (II)
Cl (a) 2,3-Dimethylbutane (b) n-Heptane
(I) (c) n-Butane (d) n-Hexane
p-H3CO–C6H4–CH2Cl 48. Consider the following reactions: [JEE M 2020 A]
(III) conc.H 2SO 4
(1) (CH3)3CCH(OH)CH3 ¾¾¾¾¾ ¾®
(a) (III) < (II) < (I) (b) (II) < (I) < (III)
alc.KOH
(2) (CH3)2CHCH(Br)CH3 ¾¾¾¾®
(c) (I) < (III) < (II) (d) (II) < (III) < (I)
43. The compound C7H8 undergoes the following reactions: - Å
(CH 3 )3O K
(3) (CH3)2CHCH(Br)CH3 ¾¾¾¾¾¾
3Cl2 / D Br2 /Fe Zn /HCl ®
C7 H8 ¾¾¾¾ ® A ¾¾¾¾ ® B ¾¾¾¾® C
D
The product 'C' is [NEET 2018, S] (4) (CH3)2C – CH2– CHO ¾¾
®
(a) m-bromotoluene OH
(b) o-bromotoluene Which of these reaction(s) will not produce Saytzeff
(c) p-bromotoluene product?
(d) 3-bromo-2,4,6-trichlorotoluene (a) (1), (3) and (4) (b) (4) only
(c) (3) only (d) (2) and (4)
EBD_8350
366 CHEMISTRY

Exercise 4 : Problem Solving Skill Enhancer MCQs

Br 4. In the following reaction, compound (B) is

Dry D
1.
CH3OH
(X), (X) is Br + Mg ¾ ¾ ¾® A ¾ ¾® B
(1 eq.) ether
Br
OCH3
(a) (b) Br
Br
(a) (b)
OCH3 Br
MgBr

(c) (d) (c) (d)


OCH3 OCH3 5. Correct order of the rate of solvolysis of following
2. Consider the following anions. compounds is

O O Br
Br
CF3 S O– C6H5 S O–

O O
(I) (II)

O O

CH3 C O– (1) (2)


(III) (IV)

When attached to sp3-hydridised carbon, their leaving Br


group ability in nucleophilic substitution reaction Br
Br
decreases in the order:
(a) I > II > III > IV
(b) I > II > IV > III (3) (4) (5)
(c) IV > I > II > III
(a) 3>2>5>4>1
(d) IV > III > II > I
(b) 2>3>5>4>1
3. Among the given halides, which one will give same product (c) 2>1>3>5>4
in both SN1 and SN2 reactions. (d) 3>2>4>5>1
CH3
CH3
(I) CH3 CH CH2 CH CH3
H Br CH3COCH3
Br 6. H + IV (X) , (X) is
CH3 (Excess)
CH3 Cl
(II) (III) Br
Cl CH3 CH3
(IV) CH3 CH Br (a) C==C
H H
Et
CH3 H
(a) (III) only (b) (I) and (II) C==C
(b)
(c) (III) and (IV) (d) (I), (III) and (IV) H CH3
HALOALKENES AND HALOARENES 367

(a) (ii) (b) (ii) and (iii)


CH3
(c) All the three (d) None of the three
10. The replacement of chlorine of chlorobenzene to give
I H
(c) phenol requires drastic conditions, but the chlorine of 2,
H I 4-dinitrochlorobenzene is readily replaced since,
(a) nitro groups make the aromatic ring electron rich at
CH3 ortho/para positions
(b) nitro groups withdraw electrons from the meta
CH3 position of the aromatic ring
(c) nitro groups donate electrons at meta position
H I
(d) (d) nitro groups withdraw electrons from ortho/para
H CH3 positions of the aromatic ring
11.
I
CH3 O

N—Br
H2O
7. CH—CH3 ¾¾® Product KCN (1 Mole)
O
(X) (Y) ,
2 moles CH 3—CO—CH3
Br CH2CH3
If during this transformation ring expansion takes place (Y) is
then product of this reaction can be best explained as
(a) Racemic mixture CN Br
(b) Diastereomers
(c) Meso form
(d) Only one enantiomer (a) (b)
8. Which reaction results in the formation of a pair of
enantiomers? CN CN
Br
H H O Br
(a) ¾¾®
2

CN
(b) MeOH
H Br O

(c) (d)
N
Br
NaOH
(c) Br
CH3 H 2O
CH3 O

12. Compound(s) that on hydrogenation produce(s) optically


NaCN active compound(s) is (are)
(d)
H Br DMF
H Br H Br
9. Which of the statement(s) is/are true, regarding following
reaction? (i) (ii) CH3
H 3C CH3 H 2C
R R
Nu – H Br
R' CBr ¾® R' CNu + Br– Br H
R'' R'' (iii) H2C (iv) H C CH 3
CH3 2
(i) The reaction involves the formation of transition
CH3
state.
(ii) Higher the nucleophilic character of the nucleophile, (a) (i) only (b) (i), (ii) and (iii)
faster will be the reaction. (c) (i) and (iii) (d) (ii) and (iv)
(iii) The product is always optically inactive.
EBD_8350
368 CHEMISTRY

13. Identify correct reactivity order for SN1 reaction 15. Identify which of the strucutre below are meso structures?

Cl Cl Cl O O
(1) (2)
(i) (ii) (iii) O O
O N B
H H O
(a) (i) > (ii) > (iii) (b) (ii) > (iii) > (i) O
O O
(c) (i) > (iii) > (ii) (d) (ii) > (i) > (iii) (3) (4)
O O
14. It required 0.7 g of hydrocarbon (A) to react completely
with 2 g of Br 2. (A) on reaction with HBr gives (B) which O O
is monobromo alkane. (B) was obtained when (A) reacts
with HBr and peroxide. (A) is O

O O
(a) (b) (5)

(c) (d) (a) 1 and 3 (b) 1, 3 and 5


(c) 1, 3 and 4 (d) 2 and 5

ANSW ER KEYS
Exercis e 1 : NCERT Based Topic-wis e MCQs
1 (b) 11 (b) 21 (a) 31 (d) 41 (c) 51 (d) 61 (a) 71 (b) 81 (c) 91 (d)
2 (b) 12 (d) 22 (a) 32 (d) 42 (a) 52 (c) 62 (d) 72 (d) 82 (d) 92 (d)
3 (c) 13 (c) 23 (b) 33 (a) 43 (c) 53 (d) 63 (a) 73 (b) 83 (b) 93 (b)
4 (c) 14 (c) 24 (d) 34 (c) 44 (c) 54 (b) 64 (a) 74 (d) 84 (a) 94 (b)
5 (c) 15 (b) 25 (c) 35 (a) 45 (d) 55 (d) 65 (a) 75 (b) 85 (b) 95 (c)
6 (d) 16 (b) 26 (a) 36 (c) 46 (d) 56 (c) 66 (c) 76 (d) 86 (d) 96 (a)
7 (b) 17 (b) 27 (d) 37 (c) 47 (a) 57 (a) 67 (d) 77 (c) 87 (b) 97 (c)
8 (a) 18 (b) 28 (d) 38 (a) 48 (b) 58 (d) 68 (c) 78 (c) 88 (a) 98 (a)
9 (a) 19 (d) 29 (c) 39 (b) 49 (b) 59 (d) 69 (b) 79 (d) 89 (b) 99 (d)
10 (d) 20 (c) 30 (c) 40 (a) 50 (d) 60 (b) 70 (b) 80 (b) 90 (d) 100 (d)
Exercise 2 : Numeric/Integer Answer Ques tions
1 (6) 2 (4) 3 (5) 4 (5) 5 (5) 6 (3) 7 (16) 8 (2) 9 (2) 10 (1)
Exercis e 3 : NCERT Exemplar & Past Year MCQs
1 (b) 6 (a) 11 (b) 16 (b) 21 (c) 26 (c) 31 (d) 36 (b) 41 (b) 46 (d)
2 (d) 7 (a) 12 (a) 17 (d) 22 (a) 27 (d) 32 (c) 37 (c) 42 (b) 47 (b)
3 (a) 8 (c) 13 (b) 18 (c) 23 (a) 28 (d) 33 (d) 38 (b) 43 (a) 48 (c)
4 (b) 9 (b) 14 (b) 19 (b) 24 (b) 29 (c) 34 (b) 39 (b) 44 (b)
5 (a) 10 (a) 15 (c) 20 (d) 25 (a) 30 (a) 35 (a) 40 (b) 45 (a)
Exercise 4 : Problem Solving Sk ill Enhancer MCQs
1 (d) 3 (c) 5 (d) 7 (b) 9 (d) 11 (d) 13 (d) 15 (a)
2 (b) 4 (c) 6 (a) 8 (b) 10 (d) 12 (c) 14 (d)
Alcohols, Phenols
25 and Ethers
Trend Buster NEET & JEE Main

Number of Questions from 2020-15 16 5 Minimum 2 questions in NEET and


minimum 1 question in JEE main has
Weightage 5.0% 4% been asked every year.

The most Important Concepts that Cover Maximum number of Questions asked in past 6 years.

Williamson synthesis 2 —
Reimer- Tieman Reaction / Kolbe's reaction 2 1
Chemical properties of ether / acidic cleavage 4 2
Chemical properties of phenol / alcohol 5 1

Less Important Concepts that Cover 1 or 2 Questions asked in past 6 years.

Preparation of phenol/dehydration of phenol / 3 —


physical properties of phenol derivatives
Chemical properties of epoxide — 1

NEET JEE

2020 Ether Acidic cleavage of epoxide 1 Average 1 Average

2019 Alcohol and phenols / ether Preparation of phenol / 2 Average — —


protonation of ether
2018 Alcohol and phenols/ether Chemical properties of 4 Average 3 Average
alcohol / Reimer-Tieman
reaction / Haloform reaction
of alcohol/Kolbe's reaction /
acidic cleavage of ether
2017 Alcohol and phenols / ether Physical properties of 3 Average 1 Difficult
phenols / acidic cleavage of
ether / substitution reaction
of ether/derivatives of ether
2016 Alcohol and phenols / ether W illiamson synthesis / cis & 2 Difficult — —
trans diol
2015 Alcohol and phenols / ether W illiamson synthesis/Reimer- 4 Average — —
Tieman reaction/chemical
properties of alcohol /
dehydration of alcohols
EBD_8350
370 CHEMISTRY
ALCOHOLS, PHENOLS AND ETHERS 371
EBD_8350
372 CHEMISTRY

Problem Solving Tips/ Tricks/ Points to Remember

4 Reduction of alcohols: Normally an alcohol cannot be (b) If HX is hot :


D
directly reduced to an alkane in one step CH3– CH2– O – CH2– CH3+2H–X ¾¾®
OH 2CH3– CH2 –X+H2O
D
LiAlH 4
CH3– O – CH2– CH3 + 2H–X ¾¾®
¾¾®
× CH3 – X + C2H5X + H2O
The above reaction is called ‘Ziesel’s method estimation’.
The –OH group is a poor leaving group. It is converted
into other superior leaving groups e.g. tosylate group. 4 In presence of sodium hydroxide, phenol generates
O phenoxide ion which is even more reactive than phenol.
OH Thus, in alkaline medium, phenol undergoes Kolbe’s
TsCl.py O S CH3 reaction.
¾¾®
O 4 The C–O bond in ethers can be cleaved by hydrogen
Cyclohexyl tosylate halides.
LiAlH H 4 Dehydration :
¾¾®
4

When vapours of ether are passed at 380 ºC over heated


4 Methylated spirit: The rectified spirit rendered poisonous alumina, then alkene is formed by the elimination of water.
by addition of 4-5% methyl alcohol, traces of pyridine 31 ´ Wt. of AgI ´100
4 % of –OCH3 group =
and some copper sulphate is known as methylated spirit Mol. mass of AgI ´ Wt. of ether
or denatured alcohol. 4 Distinction Between Primary, Secondary and Tertiary
4 Power alcohol: When alcohol is mixed with petrol and Alcohols
used in internal combustion engines then, it is known as (i) Lucas test. Lucas reagent (a mixture of conc HCl and
power alcohol. zinc chloride) reacts with alcohols to form corresponding
alkyl chlorides which are insoluble. Formation of a
4 Proof-spirit: An aqueous solution of 57.1% alcohol by chloride from an alcohol is indicated by the cloudiness
volume or 49.3% alcohol by volume called proof spirit. that appars when the chloride separates from the
4 Condensation with HCHO (Lederer-Manasse reaction): solution. Hence the time required for cloudiness to appear
Phenol react with HCHO in weak acidic or alkaline medium is a measure of reactivity of the alcohol.
A tertiary alcohol reacts immediately, a secondary alcohol
to form a mixture of o- and p-hydroxy benzyl alcohol. reacts within five minutes, and a primary alcohol does
Reaction is called Lederer-Manasse reaction. Condense not react appreciably at room temperature. However,
to give a crossed linked polymer as bakelite. remember that allyl alcohol, CH2 = CHCH2OH reacts as
4 Tests of Phenols rapidly as tertiary alcohols with the Lucas reagent to
form soluble allyl chloride.
(i) Ferric chloride test: Phenol + 1% FeCl3 solution (ii) Victor Meyer test. This test is based upon the fact that
® Violet colour the three types (1°, 2° or 3°) of nitroalkanes (formed by
(ii) Bromine water: Phenol + bromine water ® Curdy alcohols) react differently with nitrous acid followed by
sodium hydroxide. The three types of alcohols are first
precipitate converted to corresponding nitro compounds.
(iii) Phenolphthalein test : (Phenol + phthalic P/I2 AgNO2
heat —C—OH ¾¾® —C—I ¾¾¾® —C—NO2
anhydride + conc. H2SO4) + NaOH ¾¾¾ ® Pink
colour Nitroalkane
4 Test for peroxide linkage in ether Nitroalkane, so obtained, is treated first with nitrous acid
and then with sodium hydroxide to get different colours
[Ether + freshly prepared FeSO4 solution + Few drops of at the end.
KCNS] ® Red colour [Fe(CNS)3] HONO NaOH
CH3CH2NO2 ¾¾¾¾
® CH3 CNO2 ¾¾¾¾
® CH3 CNO2
Appearance of red colour confirms the presence of || ||
peroxide, i.e., NOH NOHa
peroxide 3KCNS 1° Nitroalkane Nitrolic acid (blue) Sod. nitrolate
Fe2+ ¾¾¾¾ ® Fe3+ ¾¾¾¾ ® Red colour [Fe(CNS)3] (red)
(iii) Reaction of Ether with HX (from 1° alcohol)
(a) If HX is cold : HONO NaOH
(CH3)2CHNO2 ¾¾¾¾
® (CH3)2 CNO2 ¾¾¾¾
® No reaction
CH3 – O – CH2 – CH3 + HX (cold) ¾¾® |
CH3 – X + C2H5OH NO
2° Nitroalkane Pseudonitrol (blue)
If we take unsymmetrical ether with cold H—X, then smaller (from 2° alcohol)
NaOH
alkyl group forms alkyl halide. (CH3)3CNO2 ¾¾¾¾
® No reaction (no colour)
3° Nitroalkane
ALCOHOLS, PHENOLS AND ETHERS 373

Exercise 1 : NCERT Based Topic-wise MCQs


Topic 1: Classification and Nomenclature CH 3
|
1. Which of the following is dihydric alcohol ? 10. The IUPAC name of CH 3 - CH - CH 2 - C - CH 3 is
(a) Glycerol (b) Ethylene glycol | |
(c) Catechol (d) Resorcinol OH OH
2. An example of a compound with functional group – O – is
: (a) 1, 1-dimethyl-1, 3-butanediol
(a) acetic acid (b) methyl alcohol (b) 2-methyl-2, 4-pentanediol
(c) diethyl ether (d) acetone (c) 4-methyl-2, 4-pentanediol
3. Butane-2-ol is
(a) primary alcohol (b) secondary alcohol (d) 1, 3, 3-trimethyl-1, 3-propanediol
(c) tertiary alcohol (d) aldehyde 11. Match the columns
4. Cresol has Column-I Column-II
(a) Alcoholic – OH (b) Phenolic – OH OH
(c) – COOH (d) – CHO
5. How many isomers of C5H11OH will be primary alcohols ? (A) (p) Quinol
(a) 5 (b) 4 (c) 2 (d) 3
6. In which of the following structures hydroxyl group is
attached to sp2 carbon atom? OH
OH OH
CH2OH CH (B) (q) Phenol
OH
(a) (b)
OH

OH CH(CH3)OH
(C) (r) Catechol
CH3
(c) (d) OH

OH
7. Which of the following is an example of unsymmetrical
ether?
(a) C2H5OC2H5 (b) C6H5OC6H5 (D) (s) Resorcinol
(c) C6H5OC2H5 (d) CH3OCH3
8. Vinyl carbinol is OH
(a) HO - CH 2 - CH = CH 2 (a) A – (q), B – (p), C – (s), D – (r)
(b) A – (r), B – (p), C – (s), D – (q)
(b) CH 3C(OH ) = CH 2
(c) A – (s), B – (q), C – (p), D – (r)
(c) CH 3 - CH = CH - OH
(d) A – (q), B – (r), C – (s), D – (p)
(d) CH 3 - C(CH 2 OH) = CH 2 12. Assertion : The general formula for a dihydric alcohol is
9. Which of the following shows structure of allylic alcohol? CnH2n(OH)2
(i) CH 2 = CH - CH 2OH Reason : Ethylene glycol is a dihydric alcohol.
(ii) CH 2 = CH - OH (a) Assertion is correct, reason is correct; reason is a
(iii) CH 2 = CH - CH ( CH3 ) OH correct explanation for assertion.
(b) Assertion is correct, reason is correct; reason is not
(iv) CH 2 = CH - C ( CH3 ) 2OH a correct explanation for assertion
(a) (i), (iii) and (iv) (b) (i), (ii) and (iv) (c) Assertion is correct, reason is incorrect
(c) (ii), (iii) and (iv) (d) (i), (ii), (iii) and (iv) (d) Assertion is incorrect, reason is correct.
EBD_8350
374 CHEMISTRY

13. Assertion : The bond angle in alcohols is slightly less


than the tetrahedral angle. 23. OH A
Reason : In alcohols, the oxygen of –OH group is attached
to sp3 hybridized carbon atom.
(a) Assertion is correct, reason is correct; reason is a C
OH
correct explanation for assertion.
(b) Assertion is correct, reason is correct; reason is not
a correct explanation for assertion OH B

(c) Assertion is correct, reason is incorrect


Correct order of basic character of hydroxy groups is
(d) Assertion is incorrect, reason is correct.
(a) A > B > C (b) B > C > A
(c) B > A > C (d) C > A > B
Topic 2: Alcohols and Phenols
24. Which of the following is correct ?
14. Acid catalyzed hydration of alkenes except ethene leads (a) On reduction of any aldehyde, secondary alcohol is
to the formation of
formed
(a) primary alcohol
(b) Reaction of vegetable oil with H2SO4 gives glycerine
(b) secondary or tertiary alcohol
(c) mixture of primary and secondary alcohols (c) Sucrose on reaction with NaCl gives invert sugar
(d) mixture of secondary and tertiary alcohols (d) Alcoholic iodine gives iodoform with NaOH
15. Ethyl alcohol can be prepared from Grignard reagent by 25. Which of the following is not true in case of reaction with
the reaction of : heated copper at 300°C?
(a) HCHO (b) R2CO (c) RCN (d) RCOCl (a) Phenol ¾ ¾® Benzyl alcohol
16. Isopropyl alcohol is obtained by reacting which of the
(b) Secondary alcohol ¾ ¾® Ketone
following alkenes with concentrated H2SO4 followed by
boiling with H2O? (c) Primary alcohol ¾ ¾® Aldehyde
(a) Ethylene (b) Propylene (d) Tertiary alcohol ¾ ¾® Olefin
(c) 2-Methylpropene (d) Isoprene 26. Which one of the following compounds has the most
17. Which of the following reacts with NaOH to give an acidic nature?
alcohol?
(a) Propene (b) Butene (c) Ethanal (d) Methanal CH2OH OH
18. Commercially carboxylic acids are reduced to alcohols by (a) (b)
converting them to the ______.
(a) esters (b) aldehydes OH
(c) ketones (d) amines
OH CH
19. In the reaction, RNH 2 ¾¾¾¾¾
HNO
2 ® ROH + H O + C ­; (c) (d)
273- 278K 2
C is (where R = C6H5)
27. Which of the following compounds is oxidised to prepare
(a) NH3 (b) N2
methyl ethyl ketone?
(c) O2 (b) CO2
(a) 2-Propanol (b) l-Butanol
20. Alcohols of low molecular weight are
(c) 2-Butanol (d) t-Butyl alcohol
(a) soluble in water
28. The compound which reacts fastest with Lucas reagent at
(b) soluble in water on heating
room temperature is
(c) insoluble in water
(a) butan-1-ol (b) butan-2-ol
(d) insoluble in all solvents
21. Which of the following has lowest boiling point ? (c) 2-methyl propan-1-ol (d) 2-methylpropan-2-ol
(a) p-Nitrophenol (b) m-Nitrophenol 29. When phenol is treated with excess bromine water, it gives:
(c) o-Nitrophenol (d) Phenol (a) m-bromophenol (b) o- and p-bromophenol
22. Which statement is not correct about alcohol? (c) 2, 4-dibromophenol (d) 2, 4, 6-tribromophenol
(a) Molecular weight of alcohol is higher than water 30. When phenol is heated with CHCl3 and alcoholic KOH
(b) Alcohol of less no. of carbon atoms is less soluble in salicyladehyde is produced. This reaction is known as
water than alcohol of more no. of carbon atoms (a) Rosenmund’s reaction (b) Reimer-Tiemann reaction
(c) Alcohol evaporates quickly (c) Friedel-Crafts reaction (d) Sommelet reaction
(d) All of the above
ALCOHOLS, PHENOLS AND ETHERS 375

31. Assertion : In Lucas test, 3º alcohols react immediately. (a) A – (s), B – (q), C – (p), D – (r)
Reason : An equimolar mixture of anhyd. ZnCl2 and conc. (b) A – (r), B – (s), C – (q), D – (p)
HCl is called Lucas reagent. (c) A – (s), B – (q), C – (r), D – (p)
(a) Assertion is correct, reason is correct; reason is a (d) A – (p), B – (r), C – (q), D – (s)
correct explanation for assertion. 39. Which of the following statements are correct ?
(b) Assertion is correct, reason is correct; reason is not (i) Ethanol mixed with methanol is called denatured
a correct explanation for assertion. alcohol.
(ii) Excess of methanol in body may cause blindness.
(c) Assertion is correct, reason is incorrect.
(iii) In the body methanol is oxidised to methanoic acid.
(d) Assertion is incorrect, reason is correct. (iv) A methanol poisoned patient is treated by giving
32. To distinguish between salicylic acid and phenol, one can intravenous injections of ethanoic acid.
use (a) (i), (ii) and (iii) (b) (ii), (iii) and (iv)
(a) NaHCO3 solution (b) 5% NaOH solution (c) (i) and (v) (d) (i), (iii) and (iv)
(c) neutral FeCl3 (d) bromine water 40. Assertion : Phenol is more reactive than benzene towards
33. Ethyl alcohol exhibits acidic character on reacting it with electrophilic substitution reaction.
(a) acetic acid (b) sodium metal Reason : In the case of phenol, the intermediate carbocation
(c) hydrogen chloride (d) acidic K2Cr2O7 is more resonance stabilized.
34. Phenol, when first reacts with concentrated sulphuric acid (a) Assertion is correct, reason is correct; reason is a
and then with concentrated nitric acid, it gives correct explanation for assertion.
(b) Assertion is correct, reason is correct; reason is not
(a) 2, 4, 6-trinitrobenzene (b) o-nitrophenol
a correct explanation for assertion.
(c) p-nitrophenol (d) nitrobenzene
(c) Assertion is correct, reason is incorrect.
35. 3 moles of ethanol react with one mole of phosphorus (d) Assertion is incorrect, reason is correct.
tribromide to form 3 moles of bromoethane and one mole
of X. Which of the following is X? 41. Propene, CH3CH = CH 2 can be converted into 1-
(a) H3PO4 (b) H3PO2 (c) HPO3 (d) H3PO3 propanol by oxidation. Indicate which set of reagents
36. Which of the following statements are correct ? amongst the following is ideal to effect the above
conversion ?
(i) Alcohols react as nucleophiles in the reactions
(a) KMnO4 (alkaline)
involving cleavage of O–H bond.
(ii) Alcohols react as electrophiles in the reactions (b) Osmium tetraoxide (OsO4/CH2Cl2)
involving cleavage of O–H bond. (c) B2H6 and alk. H2O2
(iii) Alcohols react as nucleophile in the reaction (d) O3/Zn
involving cleavage of C–O bond. X
42. C6 H5 - CH = CHCHO ¾¾® C6 H5 CH = CHCH 2 OH .
(iv) Alcohols react as electrophiles in the reactions
involving C–O bond. In the above sequence, X can be
(a) (i) only (b) (i) and (iv) (a) N2 / Ni (b) NaBH4
(c) (ii) and (iii) (d) (ii) only (c) K2Cr2O7 / H+ (d) Both (a) and (b)
37. Which of the following statements are correct ? 43. The hydroboration of an alkene is carried out, followed by
(i) In phenols, the —OH group is attached to sp2 oxidation with hydrogen peroxide, the alcohol so obtained
hybridised carbon of an aromatic ring is achiral. Possible structure of alkene is (are) :
(ii) The carbon – oxygen bond length (136 pm) in phenol (i) 2, 3– dimethylbut–2–ene.
is slightly more than that in methanol (ii) 3–methylpent–2–ene.
(iii) Partial double bond character is due to the (iii) 2–methyl–but–2–ene.
conjugation of unshared electron pair of oxygen with (iv) 2–methylpropene.
the aromatic ring. (a) (i) and (iv) (b) (ii) and (iii)
(iv) Phenol has sp2 hybridised state of carbon to which (c) (iii) and (iv) (d) (i) and (iii)
oxygen is attached. 44. Assertion : In case of phenol, bromination takes place
(a) (i), (ii) and (v) (b) (i), (ii) and (iii) even in absence of Lewis acid whereas bromination of
(c) (i), (iii) and (iv) (d) (i) and (iv) benzene takes place in presence of Lewis acid like FeBr 3.
38. Match the columns Reason : – OH group attached to benzene ring is highly
Column-I Column-II deactivating.
(A) Antifreeze used in (p) Methanol (a) Assertion is correct, reason is correct; reason is a
car engine correct explanation for assertion.
(B) Solvent used in (q) Phenol (b) Assertion is correct, reason is correct; reason is not
perfumes a correct explanation for assertion.
(C) Starting material for (r) Ethlene glycol (c) Assertion is correct, reason is incorrect.
picric acid (d) Assertion is incorrect, reason is correct.
(D) Wood spirit (s) Ethanol
EBD_8350
376 CHEMISTRY

45. Mechanism of acid catalysed hydration reaction involves


(i) Protonation of alkene to form carbocation by O R O
52. , Reagent R is
electrophilic attack of H3O+ O O
(ii) Nucleophilic attack of water on carbocation. COOH CH2OH
(iii) Deprotonation to form alcohol.
(a) NaBH4 / Ether / HÅ (b) LiAlH4 / Ether / HÅ
(a) (i) and (ii) (b) (i) and (iii)
(c) B2H6 / H2O (d) Red P / HI
(c) (i), (ii) and (iii) (d) (ii) and (iii)
53. Suggest the appropriate reagent for following conversion
46. Consider the following reaction:

Phenol ¾¾¾¾
Zn dust
® X ¾¾¾¾¾¾¾
3 ®Y
CH Cl O
Anhydrous AlCl3
C—H CH2OH
Alk. KMnO
¾¾¾¾¾¾
4®Z Reagent

The product Z is Ph Ph
(a) benzaldehyde (b) benzoic acid
(c) benzene (d) toluene
(a) H2 / Pd, D (b) NaBH4 / Ether / HÅ
47. The reagent used for dehydration of an alcohol is
(a) phosphorus pentachloride
(b) calcium chloride (c) LiAlH4 / Ether / HÅ (d) Both (b) and (c)
(c) aluminium oxide
(d) sodium chloride H
Major product
48. The alcohol which does not give a stable compound on 54. D
dehydration is OH
(a) ethyl alcohol (b) methyl alcohol
(c) n-Propyl alcohol (d) n-Butyl alcohol (a) (b)
49. A compound of the formula C4H10O reacts with sodium
and undergoes oxidation to give a carbonyl compound
which does not reduce Tollen’s reagent, the original (c) (d)
compound is
(a) Diethyl ether (b) n-Butyl alcohol 55. Which among the following optically active amine will
(c) Isobutyl alcohol (d) sec-Butyl alcohol produce optically inactive alcohol on treatment with
50. In the following sequence of reactions,
P +I Mg HCHO
KNO2 + HCl ?
CH 3CH 2 OH ¾¾¾
2 ® A ¾¾¾® B ¾¾¾¾
®
ether
H O
C ¾¾¾
2 ®D
(a) (b)
the compound D is NH2
NH2
(a) propanal (b) butanal
(c) n-butyl alcohol (d) n-propyl alcohol.
51. (c) (d) both (b) and (c)
NH2
OH
56. Which product is not expected for
CrO 3 CH 3MgBr B2 D 6
[P],
H
H3PO4
H3O D H2O2/OH
V
Product ?
[P] is
OH OH
CH3 CH3
D H
(a) OH (b) OH (a) (b)
H D COCH3 O
CH3 CH3
OH OH
(c) (d) (b) and (c)
(c) D (d) D COCH3
H H
ALCOHOLS, PHENOLS AND ETHERS 377

57. Decreasing order of acid strength of different OH groups 62. Identify (A) in the following scheme.
is
+
OH 1 3 OH OH H,D 1. Hg(OAc)2, H2O
4 A(C7H14O) B(C7H12)
2. NaBH 4
2° alcohol

O/Zn, CH3COOH
C(C7H14O)
HO 6-oxoheptanal an isomer of A
2
(a) 1 > 2 > 3 > 4 (b) 1 > 3 > 4 > 2
(c) 1 > 4 > 3 > 2 (d) 4 > 3 > 1 > 2 CH3
58. Geminal diol are unstable and loose water as shown below CH3
(a) (b)
OH HO CH3
H2O + O OH
OH
Arrange the following diols in order of their rate of CH3
dehydration CH3
(c) (d) OH
OH
OH OH OH
OH
CH3
OH OH
(x) (y) (z) 63. The end product of the following scheme of reactions
(a) x > y > z (b) y > z > x (c) z > x > y (d) z > y > x K Cr O , H SO
CH3CH2CH2CH2OH + NaBr + HÅ ¾¾ ¾¾¾¾¾¾¾
2 2 7 2 4®
59. ® OH H 2O
Identify product and mechanism of the reaction
CH OH (excess) (i) 2CH MgBr
(a) ¾¾¾¾¾¾¾
3 ® ¾¾¾¾¾¾
3 ®
Br and SN1 +
H +
(ii) H3O
(b) Br and SN2
OH OCH3
(a) (b)
(c) and SN1
Br OH
(c) (d)
(d) and SN2
Br 64. Which of the following reactions would convert
2-butanol into deuterated compound
OH
CH3 - CH 2 - CH - CH3 ?
PCC
[A] |
60. D
OH OH
(a) H SO (i) BD THF
[A] will show ¾¾¾¾
2 4 ® ¾¾¾¾¾¾¾
3 ®
heat (ii) H 2O 2 , NaOH
(a) Br2 water test (b) Tollen’s test
OH H SO D Pt
(c) Victor maeyer test (d) Lucas test (b) ¾¾¾¾
2 4 ® ¾¾¾¾
2 ®
heat
61. An unknown compound has molecular weight 92 and on
acetylation its molecular weight becomes 218. Number of OH PBr (i) Mg, diethyl ether
‘OH’ groups present in unknown compound are (c) ¾¾¾
3 ® ¾¾¾¾¾¾¾¾
®
(ii) D 2O
(a) 2 (b) 3
(c) 4 (d) 1 OH
(d) PBr NaOD, D O
¾¾¾
3 ® ¾¾¾¾¾®
2
EBD_8350
378 CHEMISTRY

65. Which of the following synthesis gives 3-methyl-1-hexanol? 69. An industrial method of preparation of methanol is :
(a) catalytic reduction of carbon monoxide in presence
Mg
(a) 2-Bromohexane ¾¾¾¾¾¾ (i) H 2C =O
® ¾¾¾¾¾ ® of ZnO–Cr2O3
+
C2H5OC2 H5 (ii) H 2O (b) by reacting methane with steam at 900 ºC with a nickel
catalyst
O
Mg (c) by reducing formaldehyde with lithium aluminium
(b) 2-Bromopentane ¾¾¾¾¾¾ ® ¾¾¾¾¾ ®
C H OC H +
2 5 2 5 (ii) H 2O hydride
(d) by reacting formaldehyde with aqueous sodium
Mg (i) CH CH = O hydroxide solution
(c) 3-Bromopentane ¾¾¾¾¾¾
® ¾¾¾¾¾¾
3
+
® 70. ‘Drinking alcohol’ is very harmful and it ruins the health.
C2 H5OC2H5 (ii) H 2O
Mg (i) CH3COCH3 ‘Drinking alcohol’ stands for
(d) 1-Bromobutane ¾¾¾¾¾¾
C2H5OC2 H5
® ¾¾¾¾¾¾®
(ii) H O+
(a) drinking methyl alcohol
2
(b) drinking ethyl alcohol
66. Identify the product ‘X’ in the following reaction. (c) drinking propyl alcohol
(d) drinking isopropyl alcohol
OH
71. In the commercial manufacture of ethyl alcohol from starchy
substances by fermentation method. Which enzymes
MeI NaOH (excess)
¾¾¾¾¾¾¾¾
®X stepwise complete the fermentation reaction
HO OH (a) Diastase, maltase and zymase
(b) Maltase, zymase and invertase
OMe O (c) Diastase, zymase and lactase
Me OMe (d) Diastase, invertase and zymase
Me Me
72. Select the incorrect statement about the fermentation.
(a) (b) Me Me
(a) When grapes are crushed, sugar and the enzyme come
MeO OMe O O in contact and fermentation starts
Me Me OMe (b) Fermentation takes place in anaerobic conditions
(c) Carbon monoxide is released during fermentation
O O (d) If air gets into fermentation mixture, the oxygen of air
Me Me Me oxidises ethanol to ethanoic acid which in turn
Me destroys the taste of alcoholic drinks
(c) Me Me (d) Me
73. Denaturation of alcohol is the
O O OMe OMe (a) mixing of CuSO4 (a foul smelling solid) and pyridine
Me Me Me Me (to give the colour) to make the commercial alcohol
unfit for drinking
67. The most unlikely representation of resonance structures (b) mixing of CuSO4 (to give the colour) and pyridine (a
of p-nitrophenoxide ion is foul smelling solid) to make the commercial alcohol
– – – unfit for drinking
O + O O + O
N N (c) mixing of Cu(OAc) 2 and ammonia to make the
commercial alcohol unfit for drinking
(d) mixing of Cu(OAc)2 and pyridine to make the
(a) (b) commercial alcohol unfit for drinking

– Topic 4: Ethers
O O

O O –
74. Which of the following cannot be made by using
+ O O
N + Williamson’s synthesis?
N (a) Methoxybenzene
(b) Benzyl p-nitrophenyl ether
(c) (d) (c) Methyl tertiary butyl ether

(d) Di-tert-butyl ether
– 75. Ethanol and dimethyl ether form a pair of functional isomers.
O
O The boiling point of ethanol is higher than that of dimethyl
Topic 3: Some commercially important alcohols ether, due to the presence of
68. Which enzyme converts glucose and fructose both into (a) H-bonding in ethanol
ethanol ? (b) H-bonding in dimethyl ether
(a) Diastase (b) Invertase (c) CH3 group in ethanol
(c) Zymase (d) Maltase (d) CH3 group in dimethyl ether
ALCOHOLS, PHENOLS AND ETHERS 379

76. An ether is more volatile than an alcohol having the same


molecular formula. This is due to (a) (b)
(a) dipolar character of ethers
(b) alcohols having resonance structures ONa
(c) inter-molecular hydrogen bonding in ethers (c) (d)
(d) inter-molecular hydrogen bonding in alcohols I O
77. Which of the following has strongest hydrogen bonding? O
(a) Ethyl amine (b) Ethanal
(c) Ethyl alcohol (d) Diethyl ether 87. + HI
78. Diethyl ether can be decomposed by heating with
(a) HI (b) NaOH (c) Water (d) KMnO4 OH I
79. The major organic product in the reaction, I
CH3 — O — CH(CH3)2 + HI ® Product is (a) (b)
(a) ICH2OCH(CH3)2 (b) CH 3O C( CH 3 ) 2 OH
|
I OH I
(c) CH3I + (CH3)2CHOH (d) CH3OH + (CH3)2CHI OH
80. An aromatic ether is not cleaved by HI even at 525 K. The (c) I (d)
compound is
(a) C6H5OCH3 (b) C6H5OC6H5 HI NaOH
88. [A] [X] [A]
(c) C6H5OC3H7 (d) Tetrahydrofuran (Major)
81. Assertion : ter - Butyl methyl ether is not prepared by the
Compound [A] is
reaction of ter-butyl bromide with sodium methoxide.
Reason : Sodium methoxide is a strong nucleophile.
(a) Assertion is correct, reason is correct; reason is a (a) (b)
correct explanation for assertion. O
I
(b) Assertion is correct, reason is correct; reason is not
a correct explanation for assertion.
(c) Assertion is correct, reason is incorrect. (c) (d)
(d) Assertion is incorrect, reason is correct.
82. Which of the following compounds is resistant to 89. Which method is useful for the synthesis of ether ?
nucleophilic attack by hydroxyl ions?
ONa
(a) Methyl acetate (b) Acetonitrile 30°C
(c) Acetamide (d) Diethyl ether (a) Br +
83. The major product of the reaction between tert-butyl
chloride and sodium ethoxide is Cl
(a) 2-methylprop-1-ene (b) 1-butene 30°C
(c) 2-butene (d) ethene (b) +
84. In Williamson synthesis if tertiary alkyl halide is used than ONa
(a) ether is obtained in good yield (c) CH3ONa
(b) ether is obtained in poor yield
+ CH3CH2—O—SO2 — 30°C
(c) alkene is the only reaction product —CH3
(d) a mixture of alkene as a major product and ether as a
minor product forms. CH3
85. Assertion : Ethers behave as bases in the presence of H2SO4
(d) CH3CH—OH
mineral acids. 443K
Reason : Due to the presence of lone pairs of electrons on
oxygen. 90. CH3 — —CH2—O—CH2— —NO2
(a) Assertion is correct, reason is correct; reason is a
correct explanation for assertion. + HI (1 mole)
(b) Assertion is correct, reason is correct; reason is not (Cold and dil.)
a correct explanation for assertion.
(c) Assertion is correct, reason is incorrect. Most probable product of this reaction is
(d) Assertion is incorrect, reason is correct. CH2OH CH2I

Na (a)
86. + HI [X]
(Excess) T.H.F
O
[X] is CH3 NO2
EBD_8350
380 CHEMISTRY

CH2I CH2OH 623K


(b) Ph—Cl + CH3ONa 300 atm

(b)
(c) —OH + (CH3)2SO4 ¾®

CH3 NO2
CH2I CH2I (d) —OH + CH3MgBr ¾®

93. Increasing order of the reactivity of the following alkyl


(c)
halides in the Williamson’s synthesis is
I. CH2 = CHCH2Cl II. CH3CH2CH2Br
CH3 NO2
III. (CH3)3CCH2Br IV. CH3CH2CH2Cl
CH2OH CH2OH
(a) II < III < IV < I (b) III < II < IV < I
(c) IV < III < I < II (d) III < III < II < I
(d) 94. The acidic hydrolysis of ether (X) shown below is fastest
when
CH3 NO2

O 18 V
OH
91. CH3—CH—CH—NO2 [X] Product [X] is acid
CH 3OH OR ¾¾¾
® OH + ROH
18
OH OH

(a) CH3—CH—CH—NO2 (X)


18 (a) one phenyl group is replaced by a methyl group
OH OH (b) one phenyl group is replaced by a para-
methoxyphenyl group
(b) CH3—CH—CH—NO2 (c) two phenyl group are replaced by two para-
OH OH methoxyphenyl group
(d) no structural change is made to X
(c) CH3—CH—CH—NO2 95. Assertion: Benzyl ethyl ether on reaction with dilute and
cold HI produces benzyl iodide
(d) CH3—CH—CH—NO2 Reason: This reaction occurs via SN2 pathway
(a) Assertion is true, Reason is true: Reason is a correct
OH OH explanation for Assertion
18 18 (b) Assertion is true, Reason is true: Reason is not a
92. Which will not result in the formation of anisole ? correct explanation for Assertion
V (c) Assertion is true, Reason is false
OH
(a) PhOH + CH3Cl (d) Assertion is false, Reason is true

1. Among the given hydroxyl compound how many will give 3. An ether (A), C5H12O, when heated with excess of hot
precipitate immediately when treated with concentrated concentrated HI produced two alkyl halides which when
hydrochloric acid and anhydrous zinc chloride. treated with NaOH yielded compounds (B) and (C).
3-methyl-2-butanol; 3-methyl-1-butanol; 1-butanol; 2-methyl- Oxidation of (B) and (C) gave a propanone and an ethanoic
2-butanol; 2, 3-dimethyl-2-butanol, 2, 3-dimethyl-1-butanol acid respectively.
OH In the compound A, at which carbon of alkane is the ether
NaOH CH =CHCH Cl group attached?
2. ¾ ¾® [X] ¾¾¾¾¾
2 2
® [Y]. 4. How many of the given reaction will form tert-Butyl ethyl
ether?
How many products are formed in above reaction?
ALCOHOLS, PHENOLS AND ETHERS 381

H+ O
(I) tert - Butanol + Ethanol ¾¾¾ ® O
(II) tert-Butyl bromide + Sodium ethoxide ¾® (I) R=
(III) Sodium tert-butoxide + Ethyl bromide ¾® O
+
H
(IV) Isobutene + Ethanol ¾¾¾
® O
CH2 – C – Cl
5. Which of the following statements is/ are true? (II) Q =
(I) Ethers are soluble in conc. H2SO4 but separate out OH O
on addition of water.
O
(II) Ethers are used as solvents for BF3 and Grignard C=O
(III) Q =
reagent.
(III) Mononitration of p-methylanisole gives 2-nitro- OH CH2Cl
4-methylanisole
OH
(IV) Monobromination of p-ethoxyphenol gives 2-
bromo-4-ethoxyphenol (IV) S =
(V) 4-Chlorophenol (I) will dissolve in NaOH but O – CH2 – CH2 – OH
4-chloro-1-methyl benzene (II) will not.
9. The reactivity of compound Z with different halogens
(VI) 4-Methyl benzoic (III) acid will dissolve in aq.
under appropriate conditions is given below:
NaHCO3 but 4-methyl phenol (IV) will not.
(VII) 2,4,6-Trinitrophenol (V) will dissolve in aq. NaHCO3 mono halo substituted
but 4-methyl phenol (VI) will not. OH
derivative when X2 = I2
(VIII) 4-Ethyl phenol (VII) will dissolve in aq. NaOH but
ethyl phenyl ether (VIII) will not. X2 di halo substituted
6. Generally, phenols, 1,3,1,4-benzenediols and 1,3,5- C(CH3)3 derivative when X2 = Br2
benzenetriols do not react with NaBH4/H3O+. However, Z
tri halo substituted
1,3,5-benzenetriol (phloroglucinol) gives a high yield of derivative when X2 = Cl2
product (B).
OH How many among the below given statements correctly
explains pattern of elctrophilic substitution?
+
NaBH4/H3O (I) The steric effect of the halogen.
¾¾¾¾¾® (B)
HO OH (II) The steric effect of the tert-butyl group.
Phloroglucinol (III) The electronic effect of the phenolic group.
(A) (IV) The electronic effect of the tert-butyl group.
The compound (B) has how many functional groups on (V) The mesomeric effect of tert-butyl group.
benzene ring? 10. When phenol is reacted with CHCl 3 and NaOH followed
by acidification, salicylaldehyde is obtained. How many
7. A (C5 H10 O ) ¾¾¾¾
Hot HI
® C5 H10 I2 (B) ¾ Hot
¾¾¾¾
KOH + EtOH among following species are involved in the above
(Chiral ether)
mentioned reaction as intermediates?
2 4
1 3 5 O OH
Me
(C) H CHCl2
-
1,3-Pentadiene (I) CCl 2 (II)
How many members are there in the ring of structure of A?
Cl O O–
CH 2 C Cl H CHCl2
OH (III) (IV)
CHCl
8. NaOH O
¾¾¾® P ¾¾¾¾¾¾®
OH
OH
NaOH LiAlH4 O–
Q ¾¾¾¾® R ¾¾¾¾® S
CHO
How many among the below given compounds correctly
(V)
match the products of the given reaction sequence?
EBD_8350
382 CHEMISTRY

Exercise 3 : NCERT Exemplar & Past Year MCQs


NCERT Exemplar MCQs 9. IUPAC name of the compound CH3 —CH — OCH3
is ............... . |
1. Monochlorination of toluene in sunlight followed by CH3
(a) 1-methoxy-1methylethane
hydrolysis with aq. NaOH yields
(a) o – cresol (b) 2-methoxy-2-methylethane
(b) m – cresol (c) 2-methoxypropane
(c) 2, 4 – dihydroxytoluene (d) isopropylmethyl ether
(d) benzyl alcohol 10. Which of the following species can act as the strongest
2. How many alcohols with molecular formula C4H10O are base?
chiral in nature? (a) –OH (b) –OR
(a) 1 (b) 2 (c) 3 (d) 4
3. What is the correct order of reactivity of alcohols in the (c) –OC H (d)

O
6 5
following reaction?
ZnCl NO2
R — OH + HCl ¾¾¾¾
2 ® R – Cl + H O
2
(a) 1° > 2° > 3° (b) 1° < 2° < 3° 11. Which of the following compounds will react with sodium
(c) 3° > 2° > 1° (d) 3° > 1° > 2° hydroxide solution in water?
4. CH3CH2OH can be converted into CH3CHO by ........ . (a) C6H5OH (b) C6H5CH2OH
(a) catalytic hydrogenation (c) (CH3)3COH (d) C2H5OH
(b) treatment with LiAlH4
(c) treatment with pyridinium chlorochromate 12. Phenol is less acidic than ............. .
(d) treatment with KMnO4 (a) ethanol (b) o - nitrophenol
5. The process of converting alkyl halides into alcohols (c) o-methylphenol (d) o-methoxyphenol
involves ............ . 13. Which of the following is most acidic?
(a) addition reaction (b) substitution reaction (a) Benzyl alcohol (b) Cyclohexanol
(c) dehydrohalogenation (d) rearrangement reaction (c) Phenol (d) m-chlorophenol
14. Mark the correct order of decreasing acid strength of the
6. Which of the following compounds is aromatic alcohol? following compounds.
CH2OH CH2OH OH OH CH2OH OH OH
OH OH

OCH3 NO2
CH3 CH3 NO2 OCH3
(A) (B) (C) (D) (i) (ii) (iii) (iv) (v)
(a) A, B, C, D (b) A, D (a) V > IV > II > I > III (b) II > IV > I > III > V
(c) B, C (d) A (c) IV > V > III > II > I (d) V > IV > III > II > I
7. Give IUPAC name of the compound given below. 15. Mark the correct increasing order of reactivity of the
following compounds with HBr/HCl.
CH3 — CH — CH 2 — CH 2 — CH— CH3
| |
Cl OH CH2OH CH2OH CH2OH
(a) 2-chloro-5-hydroxyhexane
(b) 2-hydroxy-5-chlorohexane
(c) 5-chlorohexan-2-ol
(d) 2-chlorohexan-5-ol NO2 Cl
(I) (II) (III)
8. IUPAC name of m-cresol is ........... .
(a) 3-methylphenol (b) 3-chlorophenol (a) I < II < III (b) II < I < III
(c) 3-methoxyphenol (d) benzene-1,3-diol (c) II < III < I (d) III < II < I
ALCOHOLS, PHENOLS AND ETHERS 383

16. Arrange the following compounds in increasing order of 22. The reaction
boiling point. CH3 CH3
Propan - 1- ol, butan - 1 - ol, butan - 2 - ol, pentan - 1 - ol
(a) Propan-1-ol, butan-2-ol, butan-1-ol, pentan-1-ol CH3–C–ONa + CH3CH2Cl –—® CH3–C–O–CH2–CH3
–NaCl
(b) Propan-1-ol, butan-1-ol, butan-2-ol, pentan-1-ol CH3 CH3
(c) Pentan-1-ol, butan-2-ol, butan-1-ol, propan-1-ol
(d) Pentan-1-ol, butan-1-ol, butan-2-ol, propan-1-ol is called :- [AIPMT 2015, A]
(a) Williamson continuous etherification process
Past Year MCQs (b) Etard reaction
17. Among the following sets of reactants which one produces (c) Gatterman - Koch reaction
anisole? [AIPMT 2014, A] (d) Williamson synthesis
(a) CH3CHO; RMgX 23. Reaction of phenol with chloroform in presence of dilute
(b) C6H5OH; NaOH ; CH3I sodium hydroxide finally introduces which one of the
following functional group? [AIPMT 2015 RS, S]
(c) C6H5OH; neutral FeCl3
(a) – CH2Cl (b) – COOH (c) – CHCl2 (d) – CHO
(d) C6H5–CH3; CH3COCl; AlCl3 24. Which of the following reaction (s) can be used for the
18. Which of the following will not be soluble in sodium preparation of alkyl halides ? [AIPMT 2015 RS, A]
hydrogen carbonate? [AIPMT 2014, S] anhy.ZnCl2
(I) CH3CH2OH + HCl ¾¾¾¾¾ ®
(a) 2, 4, 6-trinitrophenol (b) Benzoic acid
(c) o-Nitrophenol (d) Benzenesulphonic acid (II) CH3CH2OH + HCl ¾¾
®
19. Identify Z in the sequence of reactions: [AIPMT 2014, S]
(III) (CH3)3COH + HCl ¾¾
®
HBr/H 2 O 2 C 2 H 5ONa
CH3 CH 2 CH=CH 2 ¾¾¾¾¾® Y ¾¾¾¾¾
®Z anhy.ZnCl2
(IV) (CH3)2CHOH + HCl ¾¾¾¾¾ ®
(a) CH3—(CH2)3 —O—CH2CH3 (a) (I), (III) and (IV) only (b) (I) and (II) only
(b) (CH3)2 CH2—O—CH2CH3 (c) (IV) only (d) (III) and (IV) only
(c) CH3(CH2)4—O—CH3
(d) CH3CH2—CH(CH3)—O—CH2CH3 25. Which of the following is not the product of dehydration
20. The most suitable reagent for the conversion of
R - CH 2 - OH ¾¾ ® R - CHO is: [JEE M 2014, S]
(a) KMnO4 of OH ? [AIPMT 2015 RS, S]
(b) K2Cr2O7
(c) CrO3
(d) PCC (Pyridinium chlorochromate) (a) (b)
21. Sodium phenoxide when heated with CO2 under pressure
at 125ºC yields a product which on acetylation produces C

ONa 125°C H+
(c) (d)
+ CO 2 ¾¾¾¾
® B ¾¾¾® C
5 atm Ac2 O
26. Which of the following reagents would distinguish cis-
The major product C would be [JEE M 2014, A] cyclopenta-1,2-diol from the trans-isomer? [NEET 2016, S]
(a) Acetone (b) Ozone
OCOCH3 OH
(c) MnO2 (d) Aluminium isopropxide
COOH COCH3 27. The reaction
(a) (b)
NaOH – +
Me–I Me
OH ¾¾® O Na ¾¾® O
COCH3
Can be classified as :- [NEET 2016, A]
OH OCOCH3 (a) Williamson ether synthesis reaction
(b) Alcohol formation reaction
COOCH3 (c) Dehydration reaction
(c) (d)
COOH (d) Williamson alcohol synthesis reaction
EBD_8350
384 CHEMISTRY

28. Which one is the most acidic compound ? [NEET 2017, S] 32. The compound A on treatment with Na gives B, and with
OH PCl5 gives C. B and C react together to give diethyl ether.
OH
A, B and C are in the order [NEET 2018, S]
(a) (b) (a) C2H5OH, C2H6, C2H5Cl
(b) C2H5OH, C2H5Cl, C2H5ONa
NO2 (c) C2H5OH, C2H5ONa, C2H5Cl
OH (d) C2H5Cl, C2H6, C2H5OH
OH
33. In the reaction
(c) O2N NO2 (d)
OH O– Na+

CH3 CHO
NO2 + CHCl3+ NaOH ¾®
29. The heating of phenyl-methyl ethers with HI produces
[NEET 2017, S] The electrophile involved is [NEET 2018, A]
(a) Iodobenzene
(c) Benzene
(b) Phenol
(d) Ethyl chlorides
(
(a) Dichloromethyl cation C HCl2
Å
)
30. Identify A and predict the type of reaction
[NEET 2017, A] (Å )
(b) Formyl cation C HO
OCH3 (c) Dichlorocarbene (: CCl 2 )
NaNH2
A (
(d) Dichloromethyl anion CHCl 2
!
)
Br 34. Compound A, C8H10 O, is found to react with NaOI
OCH3 (produced by reacting Y with NaOH) and yields a yellow
precipitate with characteristic smell. [NEET 2018, S]
NH2
(a) and elimination addition reaction A and Y are respectively

(a) H3C CH2 – OH and I2


OCH3
Br
(b) CH2 – CH2 – OH and I2
(b) and cine substitution reaction
CH3
OCH3
(c) CH3 OH and I2
(c) and cine substitution reaction
(d) CH – CH3 and I2
OCH3
OH
35. Identify the major products P, Q and R in the following
(d) and substitution reaction sequence of reactions: [NEET 2018, A]
NH2
Anhydrous
31. Which of the following , upon treatment with tert-BuONa AlCl
+ CH3CH2CH2Cl ¾¾¾¾®
3
P
followed by addition of bromine water, fails to decolourize (i) O
the colour of bromine? [JEE M 2017, A] P ¾¾¾¾®
2
+ Q+R
(ii) H3O /D
O C6H5 P Q R
(a) (b) CH2CH2CH3 CHO
Br Br
O (a) , , CH3CH2 – OH
O
(c) (d) CH2CH2CH3 CHO COOH
Br Br
(b) , ,
ALCOHOLS, PHENOLS AND ETHERS 385

38. The major product formed in the following reaction is :


CH(CH3)2 OH [JEE M 2018, A]
(c) , , CH3 – CO – CH3

CH(CH3)2
OH (a) (b)
(d) , , CH3CH(OH)CH3

36. Phenol on treatment with CO2 in the presence of NaOH


followed by acidification produces compound X as the major (c) (d)
product. X on treatment with (CH3CO)2O in the presence of
catalytic amount of H2SO4 produces : [JEE M 2018, S] 39. The compound that is most difficult to protonate is :
[NEET 2019C]
O O
(a) H H (b) H 3C H
O O
(a) (b) (c) H 3C CH3 (d) Ph H
40. The structure of intermediate A in the following reaction, is:
CH3
CH OH
CH3 O
O +
H
¾®
2 A
¾¾® + H3C CH3
H O
2

(c) (d) [NEET 2019C]

CH3
O CH
37. Phenol reacts with methyl chloroformate in the presence
CH3
of NaOH to form product A. A reacts with Br2 to form
product B. A and B are respectively : [JEE M 2018, S] (a)

CH3
(a) and H3C – C – O – O – H

(b)

CH3
(b) and
O – O – CH
CH3
(c)
(c) and

CH3 – O – O – H
HC
CH3
(d) and
(d)
EBD_8350
386 CHEMISTRY

41. Anisole on cleavage with HI gives [NEET 2020, C] 42. 1-methyl ethylene oxide when treated with an excess of
HBr produces: [JEE M 2020A]
I OH
Br
Br Br
(a) (b) (a) (b)
+ CH3OH + C2H5I CH3
CH3

I OH Br
(c) Br CH3 (d)
Br
(c) + C2H5OH (d) + CH3I

Exercise 4 : Problem Solving Skill Enhancer MCQs

18
5. An alcohol (A) on heating with Cu/300°C gives (B). (B)
H
1. CH3COOH + CH3OH [X] + [Y], { O = O } [X] on oxidation gives (C). (C) on reaction with CH3MgBr/
and [Y] are H3O+ gives (A). (A) will be

OH
(a) CH3COOCH3 + H2O (b) CH3COOCH3 + H2O OH
(a) (b)
(c) CH3COOCH3 + H2O (d) CH3COOCH3 + H2O

(c) OH (d) OH
2. ® HO — (CH2)6 — OH, this conversion can be
6. Which is most stable conformation of butan 2, 3-diol ?
achieved by OH OH
(a) O3, Zn, then LiAlH4 CH3 H CH3 H
(b) O3/H2O2, then LiAlH4 (a) (b)
(c) cold dil. KMnO4, HlO4, then LiAlH4 H CH3 CH3 H
OH OH
(d) All of these
3. Which type of carbocation is/are formed when OH OH
CH3 OH CH3 OH
OH is treated with an acid? (c) (d)
CH3 H H CH3
(a) 1º (b) 2º H H
(c) 3º (d) All the three
7. Optically active alcohol is formed in
4. ClCH2CH2OH is stronger acid than CH3CH2OH because of:
D
(a) – I effect of Cl increases negative charge on O atom (a)
H2O
of alcohol
H
(b) – I effect of Cl disperses negative charge on O atom (b)
D2O
to produce more stable cation
H
(c)
(c) – I effect of Cl disperses negative charge on O atom H2O
to produce more stable anion (1) H3O (2) PCC
(d) None of these (d)
(3) LiAlH 4/H
ALCOHOLS, PHENOLS AND ETHERS 387

11. Maximum number of moles of G.R that can be consumed


CH3
by 1 mole of following compound are
8. CH3—C—CH2—O—CH3 + HI
(Cold & dil) O
CH3
Cl OEt
Most probable products of this reaction are O
O
CH3
(a) 2 (b) 3
(a) CH3—C—CH2—CH3 + CH3OH (c) 4 (d) 5
I B2H6 NaOH
12. V
[X] , [X] may be
H2O2/OH
CH3
Br
(b) CH3—C—CH2CH3 + CH3I
O
OH (a) (b)
Br ONa
CH3

(c) CH3—C—CH2I + CH3OH (c) (d)


HO OH O
CH3
O O dil. H2SO4
CH3 13. [X]

(d) CH3—C—CH2OH + CH3I Which functional group(s) is / are present in [X] ?


CH3 (a) Ketone and alcohol
(b) Aldehyde and alcohol
9. Which of the following is the correct order for ease of
dehydration– (c) Only alcohol
(d) Carboxylic acid and alcohol
OH
| OD
CH3 - CH 2 - CH 2 - CH 2 CH3 - CH 2 - CH - CH3
| D (i) CO2
(II) NaOH
(I) OH 14. [Intermediate] P . Here P is
(ii) D+

OH OH OD
(a) HOOC
| D
CH 2 = CH - CH - CH3
(III)
(IV)
OD
(a) I > III > II > IV (b) IV > III > II > I (b) DOOC D
(c) IV > II > III > I (d) III > IV > II > I
10.
OH OH OH OH
(c) DOOC D
+
Suggest suitable reagent/condition for the above
transformation (d) Reaction not possible
(a) dil. NaOH (b) Sunlight
(c) Al2O3 / D (d) dil. acid
EBD_8350
388 CHEMISTRY

OH O
O (1) Mg/THF
¾¾¾¾¾ ® (X) (b)
15. (2) H OÅ
,
3
Cl

O O
CH 3MgBr
¾¾¾¾® (Y) (c) ,
(X) and (Y) are
OH
O , OH
(a) O (d) ,
Cl

ANSW ER KEY
Exercise 1 : NCERT Based Topic-wise MCQs
1 (b) 11 (d) 21 (c) 31 (b) 41 (c) 51 (a) 61 (b) 71 (a) 81 (b) 91 (a)
2 (c) 12 (b) 22 (b) 32 (a) 42 (b) 52 (c) 62 (c) 72 (c) 82 (d) 92 (d)
3 (b) 13 (b) 23 (a) 33 (b) 43 (a) 53 (b) 63 (a) 73 (b) 83 (a) 93 (d)
4 (b) 14 (b) 24 (d) 34 (b) 44 (c) 54 (d) 64 (c) 74 (d) 84 (c) 94 (c)
5 (b) 15 (a) 25 (a) 35 (d) 45 (c) 55 (c) 65 (b) 75 (a) 85 (a) 95 (c)
6 (c) 16 (b) 26 (b) 36 (b) 46 (b) 56 (c) 66 (c) 76 (d) 86 (a)
7 (c) 17 (d) 27 (c) 37 (c) 47 (c) 57 (c) 67 (c) 77 (c) 87 (c)
8 (a) 18 (a) 28 (d) 38 (b) 48 (b) 58 (d) 68 (c) 78 (a) 88 (b)
9 (a) 19 (b) 29 (d) 39 (a) 49 (d) 59 (b) 69 (a) 79 (c) 89 (c)
10 (b) 20 (a) 30 (b) 40 (a) 50 (d) 60 (d) 70 (b) 80 (b) 90 (b)
Exercise 2 : Numeric/ Integer Answer Questions
1 (2) 2 (3) 3 (2) 4 (3) 5 (8) 6 (2) 7 (5) 8 (3) 9 (3) 10 (2)
Exercise 3 : NCERT Exemplar & Past Year MCQs
1 (d) 6 (c) 11 (a) 16 (a) 21 (a) 26 (a) 31 (a) 36 (a) 41 (d)
2 (a) 7 (c) 12 (b) 17 (b) 22 (d) 27 (a) 32 (c) 37 (c) 42 (b)
3 (c) 8 (a) 13 (d) 18 (c) 23 (d) 28 (c) 33 (c) 38 (d)
4 (c) 9 (c) 14 (b) 19 (a) 24 (a) 29 (b) 34 (d) 39 (d)
5 (b) 10 (b) 15 (c) 20 (d) 25 (b) 30 (d) 35 (c) 40 (b)
Exercise 4 : Problem Solving Skill Enhancer MCQs
1 (c) 3 (d) 5 (c) 7 (a) 9 (b) 11 (d) 13 (a) 15 (b)
2 (d) 4 (c) 6 (d) 8 (d) 10 (d) 12 (a) 14 (c)
Aldehydes, Ketones and
26 Carboxylic Acids
Trend Buster NEET & JEE Main

Number of Questions from 2020-15 14 9 Conversion is generally asked in


Weightage 4.4% 5.10% NEET & JEE from this chapter.

The most Important Concepts that Cover Maximum number of Questions asked in past 6 years.

Preparation of Aldehydes & Ketones 1 4


Properties of Aldehydes & Ketones 10 1
Preparation of Carboxylic Acids 1 2
Properties of Carboxylic Acids & their Derivatives 2 2

Less Important Concepts that Cover 1 or 2 Questions asked in past 6 years.

Nomenclature & Structure of Carbonyl Group — —


Nomenclature & Structure of Carboxyl Group — —

NEET JEE
Concept Used

2020 Preparation of Carboxylic Acids / Conversion:- Reduction of 3 Average 1 Average


Preparation and properties of carbonyl compounds followed
aldehydes & ketones by Grignard reaction / reaction
with Grignard reagent /
Preparation of benzaldehyde /
cross aldol condensation
2019 Preparation & Properties of Phthalimide preparation / 1 Average 5 Average /
Carboxylic Acids and their Reduction with DIBAL-H / Difficult
Derivatives / Properties of Conversion:- SN 1 followed by
Aldehydes & Ketones Oxidation of alcohols &
dehydration of Acids / Reduction
with LiALH4 / Oxidation with
KMnO4 / Acidity Order
2018 Properties of Carboxylic Acids Intermolecular H-bonding of 1 Easy — —
and their Derivatives Carboxylic Acids
2017 Properties of Aldehydes & Conversion:-Tollen's test, Aldol 2 Difficult 2 Difficult
Ketones condensation & semicarbazone
formation /Addition reaction with
Grignard reagent /Reduction
with DIBAL-H
2016 Properties of Aldehydes & Keto-enol tautomerism / 2 Easy / — —
Ketones Schiff's base Average
2015 Preparation & Properties of Iodoform & Tollen's test / 5 Easy / 1 Average
Aldehydes & Ketones / Addition reaction with Average
Preparation of Carboxylic Acids methyl lithium /Keto-enol
tautomerism / Addition-elimination
reaction / Alkaline ester
hydrolysis / Ozonolysis
EBD_8350
390 CHEMISTRY
ALDEHYDES, KETONES AND CARBOXYLIC ACIDS 391
EBD_8350
392 CHEMISTRY

Problem Solving Tips/ Tricks/ Points to Remember

4 Catalytic acidic hydration of alkynes : Acetaldehyde


is formed by passing acetylene in 40% aqueous solution O O
ôô D ôô
of H2SO4 at 60 °C in the presence of 1% HgSO4. The CH3 - C - CH 2 - COOH ¾¾ ® CH3 - C - CH3 + CO 2
reaction is called "Kucherov reaction" b a Propanone
dil. H2SO4 /HgSO4 3-Oxobutanoic acid
H – C º CH + HOH ¾¾¾¾¾¾¾¾¾
60°C
®

O
ˆˆˆ
†
é H - C = CH2 ù ‡ˆˆˆ H - C - CH 3 ôô
ê ô ú ôô D
HO - C- CH 2 - COOH ¾¾ ® CH3 - COOH + CO 2
ë O û O b a
Unstable Acetic acid
Malonic acid
4 In case of unsymmetrical ketones, the keto group stays
preferentially with the smaller alkyl group (Popoff’s rule). 4 Claisen condensation :
4 With Schiff 's reagent : Schiff's reagent is a solution of Esters containing a-H-atoms undergo self-condensation
magneta dye (rosaniline hydrochloride or fuchsine) in presence of a strong base such as sodium ethoxide to
decolourised by passing SO2. form b-keto esters. For ex :
4 Harpp reaction is the modification of HVZ reaction. This
O
reaction involves a-halogenation of carboxylic acids with P C2 H5ONa
N-halosuccinimide in presence of HCl or HBr and SOCl2. CH3 - C -OC2 H5 + H - CH 2COOC2 H5 ¾¾¾¾¾ ®
4 Formic Acid Ethyl acetate
It shows acidic character due to presence of – COOH (2 molecules)
group and reducing character due to presence of – CHO O
group. P
CH3 - C - CH 2 COOC 2 H5 + C 2 H5OH
It reduces Tollen's reagent and forms silver mirror. Ethylacetoacetate
It reduces Fehling’s solution. (b -ketoester)
4 Test : Benzoic acid + Neutral FeCl3 ¾® Red brown ppt. 4 Comparative reactivity of acid derivatives :
4 Beckmann rearrangement : The relative reactivities of different acid derivatives
Oximes on treatment with catalysts such as conc. H2SO4, towards nucleophilic acyl substitution reactions follow the
SOCl 2, PCl 5, etc. undergo rearrangement to form order :
substituted amides. This reaction is called Beckmann
O O O O O
rearrangement. P P P P P
O R - C - Cl > R - C - O - C - R ' > R - C - OR ' > R - C - NH 2
C 6H 5 (i) conc. H 2SO 4 Acid chloride Anhydride Ester
(i) Amide
¾¾¾¾¾¾¾ ®
:

C=N (ii) H 2O CH3 – C – NHC6H5


H 3C This order can be explained in terms of relative basic
OH Acetanilide
strength of the leaving groups:
O NH -2 > R 'O - > R 'COO - > Cl - . Since stronger base is
H5C6 CH 3 (i) conc. H 2SO 4 a poor LG.
(ii)
HO
C=N .. ¾¾¾¾¾¾¾
(ii) H 2O
® C6H5 – C – NHCH3
N-Methylbenzamide 4 Inductive effect is stronger at a-position than at b-position,
similarly at b-position it is more stronger than at g-position.
4 Decarboxylation of b-keto acids :
Carboxylic acids containing an EWG such as CO , –
COOH or –NO2 at b-carbon atom w.r.t. the – COOH group
readily undergo decarboxylation on heating. For ex :

a b g
Example : CH3 - CH 2 - CH - COOH > CH3 - CH - CH 2 - COOH > CH 2 - CH 2 - CH 2 - COOH
ô ô ô
Cl Cl Cl
ALDEHYDES, KETONES AND CARBOXYLIC ACIDS 393

Exercise 1 : NCERT Based Topic-wise MCQs


Topic 1: Nomenclature and Structure of Carbonyl Group 7. IUPAC name of the following compound is

1. Choose the correct IUPAC name for O


CH3 - C H - CHO
| H
CH2 -CH 3
(a) Butan - 2- aldehyde
(b) 2- methylbutanal (a) 2-(2-propenyl) butanal
(c) 3- methylisobutyraldehyde (b) 2-(1-propenyl) butanal
(d) 2- ethylpropanal (c) 4-formyl 4-ethyl but-2-ene
2. The IUPAC name of CH3COCH(CH3)2 is (d) 2-ethyl pent-3-en-l-al
(a) 2-methyl-3-butanone
(b) 4-methylisopropyl ketone Topic 2: Preparation of Aldehydes and Ketones
(c) 3-methyl-2-butanone
(d) Isopropylmethyl ketone 8. Which alkene on ozonolysis gives CH3CH2CHO and
3. IUPAC name of following will be CH3CCH3
CHO ||
O
CH3
CH3 (a) CH3CH2CH = C
CH3
(b) CH3CH2CH = CHCH2CH3
(c) CH3CH2CH = CH CH3
OH (d) CH 3 - C = CHCH 3
(a) 4-formyl 3-methyl 1-hydroxy benzene |
(b) 4-formyl 3-methyl phenol CH3
(c) 4-hydroxy 2-methyl benzaldehyde 9. An aldehyde group can be present
(d) 4-hydroxy 2-methyl carbaldehyde (a) in between carbon chain
4. In > C = O group sigma bond is formed by (b) at any position in carbon atom
(a) sp2-p-overlapping (b) sp3-p-overlapping (c) only at the end of carbon chain
(c) sp-p-overlapping (d) s-p-overlapping
(d) at the second carbon atom of the carbon chain
5. Which of the following is correct for carbonyl compounds?
10. Benzaldehyde is obtained from Rosenmund’s reduction
R R d+ d– of
d– d+
(a) C=O (b) C=O
R¢ R¢ O O

(c) d+ d– (d) d+ d– (a) CH3 (b) Cl


R–C=O R–C=O

R¢ R¢ O
6. Match the columns Cl
Column-I Column-II (c) Cl (d) OH
(Common names) (IUPAC names)
(A) Cinnamaldehyde (p) Pentanal
(B) Acetophenone (q) Prop-2-enal 11. Product of the following reaction is
(C) Valeraldehyde (r) 4-Methylpent-3-en-2-one CN
(D) Acrolein (s) 3-Phenylprop-2-enal SnCl +HCl
(E) Mesityl oxide (t) 1-Phenylethanone + 2(H) ¾¾¾¾®
2
?
H O+ 3
(a) A – (s), B – (t), C – (p), D – (q), E – (r)
(b) A – (p), B – (q), C – (s), D – (t), E – (r) COOH CHO
(c) A – (t), B – (s), C – (p), D – (r), E – (q)
(d) A – (q), B – (t), C – (r), D – (s), E – (p) (a) (b)
EBD_8350
394 CHEMISTRY

O (iii) Both can be prepared by the oxidation of respective


NH2 alcohol with acidic dichromate.
C ¾ CH3 (a) (i) only (b) (ii) and (iii)
(c) (d)
(c) (i) and (iii) (d) All the three
12. Find out B in the given reactions 19. Product of the following reaction is
CN MgX
AlCl3 CrO 3 in (CH 3CO) 2O
+ CH 3 – X A B +
dry ether
H3O+ +
H3O
(a) acetophenone O
(b) benzaldehyde
(c) cyclohexyl carbaldehyde
(d) benzoic acid (a) (b) OH
13. The reaction
O
CHO
AlCl
+ CO + HCl ¾¾¾®
3

(c) (d)
(a) Rosenmund’s reaction (b) Stephen’s reaction
(c) Cannizzaro’s reaction (d) Gatterman-Koch reaction 20. An ester (A) with molecular formula C9H10O2 was treated
14. Which aldehyde cannot be obtained by Rosenmund’s with excess of CH3MgBr and the compound so formed
reaction? was treated with conc. H2SO 4 to form olefin (B).
(a) CH3CHO (b) HCHO Ozonolysis of (B) gave ketone with formula C8H8O which
(c) CH3CH2CHO (d) All of these shows positive iodoform test. The structure of (A) is
15. The conversion PhCN ® PhCOCH 3 , can be achieved (a) C6H5COOC2H5
(b) CH3OCH2COC6H5
most conveniently by reaction with
(c) CH3CO – C6H4 – COCH3
(a) CH3MgBr followed by hydrolysis
(d) C6H5COOC6H5
(b) I2 – NaOH, CH3I
21. Consider the following statements : Acetophenone can
(c) dil. H2SO4 followed by reaction with CH2N2
be prepared by
(d) LiAlH4 followed by reaction with CH3I
I. oxidation of 1-phenyl ethanol
16. Which of the following is used to prepare ketone in good
II. reaction of benzaldehyde with methyl magnesium
yield from acyl chloride ?
bromide
(a) R-MgX (b) R2Cd
III. Friedel’s–Crafts reaction of benzene with acetyl
(c) CO + HCl (d) CrO3
chloride
17. In the given reaction
IV. distillation of calcium benzoate
CH3 CHO (a) II and III (b) I and IV
Cl2/hn H2O
¾¾¾® A ¾¾®
373K
(c) I and III (d) III and IV
22. The suitable reaction steps to carry out the following
A is transformation are
CH2Cl CHCl2 ? ? H
¾¾
® ¾¾
®
(a) (b)
O
(i) BH , THF PCC
CCl3 CH3 (a) ¾¾¾¾¾¾¾
3 ® ¾¾¾¾®
(ii) H 2O 2 , NaOH CH 2Cl2
(c) (d)
(i) BH , THF HIO
Cl (b) ¾¾¾¾¾¾¾
3 ® ¾¾¾

(ii) H 2O 2 , NaOH
18. Which of the following statement(s) is/are true regarding
preparation of aldehydes and ketones? H O, H SO PCC
(c) ¾¾¾¾¾¾
2 2 4 ® ¾¾¾¾®
CH2 Cl2
(i) Both can be prepared by the oxidation of the
concerned alcohol with copper at about 250ºC. OsO K Cr O , H SO
(ii) Both can be prepared by the oxidation of the (d) ¾¾¾¾¾¾¾¾¾
4
-
® ¾¾¾¾¾¾¾
2 2 7 2 4®
(CH3 )3 CCOOH, OH H 2O
concerned alcohol by Oppenauer oxidation.
ALDEHYDES, KETONES AND CARBOXYLIC ACIDS 395

23. An organic compound ‘A’ C8H12 on reaction with ozone (b) Aldol condensation is given by aldehydes in presence
followed by Zn gave one mole each of (CHO) 2 and of alkali
CH3COCH2CH2COCH3. The structure of ‘A’ is (c) Aldol condensation is given by aldehydes and
ketones in presence of acids
(a) CH3 - C - CH 2CH 2 - C - CH3 (d) None of the above
|| |
28. If formaldehyde and KOH are heated, then we get
CH 2 CH 2
(a) methane (b) methyl alcohol
(c) ethyl formate (d) acetylene
(b) CH3 CH3 29. The reagent which can be used to distinguish
acetophenone from benzophenone is
(a) 2,4- dinitrophenylhydrazine
(c) H 3C CH3
(b) aqueous solution of NaHSO3
(c) benedict reagent
(d) H 3C CH3 (d) I2and Na2CO3
30. Acetaldehyde reacts with semicarbazide and forms
Topic 3: Properties of Aldehydes and Ketones semicarbazone. Its structure is
(a) CH3CH = NNHCON = CHCH3
24. Propanal and propanone, both have same molecular
(b) CH3CH = NNHCONH2
formula(C3H6O), what do you expect about their boiling
points? (c) CH 3CH = N — N — CONH 2
|
(a) Both have same boiling point OH
(b) Boiling point of propanal is higher than the boiling (d) CH3CH = N—CONHNH2
point of propanone. 31. Phenylmethyl ketone can be converted into ethylbenzene
(c) Boiling point of propanal is lower than the boiling in one step by which of the following reagents?
point of propanone (a) LiAlH4 (b) Zn-Hg/HCl
(d) Nothing can be predicted (c) NaBH4 (d) CH3MgI
25. The boiling points of aldehydes and ketones lie in between 32. Which of the following compound will show positive silver
alkanes and alcohols of comparable masses because mirror test ?
(a) alkanes are polar (a) HCOOH
(b) aldehydes and ketones are non-polar (b) CH3(CHOH)3CHO
(c) alkanes are polar and aldehydes and ketones contain (c) CH3CO(CHOH)CH3
(d) Both (a) and (b)
C = O group and lower alcohols have H- 33. A compound does not react with 2, 4 dinitrophenyl-
polar
hydrazine, the compound is :
bonding. (a) Acetone (b) Acetaldehdye
(d) alkanes are held together by weak van der Waal’s (c) CH3OH (d) CH3CH2COCH3
forces (being polar), aldehydes and ketones contain 34. Which gives lactic acid on hydrolysis after reacting with
HCN ?
polar C = O group and held together by strong (a) HCHO (b) CH3CHO
(c) C6H5CHO (d) CH3COCH3
dipole-dipole attraction and lower alcohols have 35. The most appropriate reagent to distinguish between
H-bonding, which is stronger than dipole-dipole acetaldehyde and formaldehyde is :
attraction. (a) Fehling’s solution
26. Less reactivity of ketone is due to (b) Tollen’s reagent
(a) + I inductive effect decreases positive charge on (c) Schiff’s reagent
carbonyl carbon atom (d) Iodine in presence of base
(b) steric effect of two bulky alkyl groups 36. 2-Pentanone and 3-pentanone can be distinguished by :
(c) sp2 hybridised carbon atom of carbonyl carbon atom (a) Cannizzaro's reaction
(d) Both (a) and (b) (b) Aldol condensation
27. Which of the following statement is false ? (c) Iodoform reaction
(a) Cannizzaro reaction is given by aldehydes in presence (d) Clemmensen's reduction
of alkali
EBD_8350
396 CHEMISTRY

37. Cross aldol condensation occurs between X


46. C6H5CH=CHCHO ¾¾® C6H5CH=CHCH2OH
(a) two same aldehydes In the above sequence X can be :
(b) two same ketones (a) H2/Ni (b) NaBH4
(c) two different aldehydes and ketones (c) K2Cr2O7/H+ (d) Both (a) and (b)
(d) None of these
47. Two compounds benzyl alcohol and benzoic acid are
38. When acetaldehyde reacts with alcohol then produce
formed from a compound, when it is heated in the presence
(a) Acetal (b) Ketal (c) Acetone (d) None
of conc.NaOH, this compound is.
39. The product formed in Aldol condensation is
(a) Benzaldehyde (b) Benzylalcohol
(a) a beta-hydroxy aldehyde or a beta-hydroxy ketone
(c) Acetophenone (d) Benzophenone
(b) an alpha-hydroxy aldehyde or ketone
48. Which of the following compounds will give butanone on
(c) an alpha, beta unsaturated ester
oxidation with alkaline KMnO4 solution?
(d) a beta-hydroxy acid
40. Which of the following products is formed when (a) Butan-1-ol (b) Butan-2-ol
benzaldehyde is treated with CH3MgBr and the addition (c) Both of these (d) None of these
product so obtained is subjected to acid hydrolysis ? 49. Read the following statements and choose the correct
(a) A secondary alcohol option
(b) A primary alcohol (i) The carbonyl carbon atom is sp2 -hybridised
(c) Phenol (ii) The carbonyl carbon is an electrophilic (Lewis acid)
(d) tert-Butyl alcohol centre
41. During reduction of aldehydes with hydrazine and (iii) The carbonyl oxygen is a nucleophilic (Lewis base)
potassium hydroxide, the first step is the formation of centre
(iv) Carbonyl compounds are non- polar in nature.
(a) R — CH — — N — NH 2 (b) R — C º N
(a) (i), (ii) and (iv) are correct
(c) R — C— NH 2 (d) R — CH—
— NH (b) (i), (ii) and (iii) are correct
||
O (c) (ii), (iii) and (iv) are correct
42. A and B in the following reactions are (d) (ii) and (iv) are correct
OH 50. Aldehydes are generally more reactive than ketones in
HCN B
R–C–R' ¾¾® A ¾® R– C nucleophilic addition reactions. Which of the following
KCN
R' CH2NH2 statements accounts for this ?
O
CN (i) Sterically, the presence of two relatively large
(a) A = RR'C , B = LiAlH4 substituents in ketones hinders the approach of
OH nucleophile to carbonyl carbon
OH (ii) Aldehydes show resonance whereas ketones do not
(b) A = RR'C , B = NH3 (iii) Electronically, the presence of two alkyl groups reduce
COOH the electrophilicity of the carbonyl carbon more
effectively.
CN
(iv) Electronically carbonyl carbon atom in ketones is more
(c) A = RR'C , B = H 3O Å electrophilic than in aldehydes
OH
(a) (i) and (iii) (b) (i) and (iv)
(d) A = RR'CH2CN, B = NaOH
(c) (ii) and (iii) (d) (ii) and (iv)
43. In the reaction of NaHSO3 with carbonyl compounds to
OH -
form bisulphite product, the nucleophile is 51. 2C6 H5CHO ¾¾¾® C6 H5CH 2OH + C6 H5COO -
H 2O
(a) HSO3– (b) SO3Na
Which of the following statements are correct regarding
(c) SO32– (d) None of the above the above reduction of benzaldehyde to benzyl alcohol?
44. Imine derivatives of aldehyde and ketone is called as (i) One hydrogen is coming from H2O as H+ and another
(a) Schiff’s reagent (b) Fehling’s reagent from C6H5CHO as H–
(c) Schiff’s base (d) Schiff’s acid (ii) One hydrogen is coming from H2O as H– and another
45. Which reaction is used for detecting the presence of from C6H5CHO as H+
carbonyl group? (iii) One hydrogen from H2O and another from C6H5CHO,
(a) Reaction with hydrazine both in the form of H–
(iv) The reduction is an example of disproportionation
(b) Reaction with phenylhydrazine
reaction
(c) Reaction with hydroxylamine (a) (i), (ii) and (iii) (b) (i) and (iv)
(d) All of the above (c) (ii), (iii) and (iv) (d) (iii) and (iv)
ALDEHYDES, KETONES AND CARBOXYLIC ACIDS 397

52. Match the columns 56. Match the columns


Column-I Column-II Column-I Column-II
(A) R - CO - CH 3 ¾¾Zn - Hg / HCl R
¾ ¾ ¾® (p) Friedel-Craft’s
(A) C = NH (p) Oxime
R - CH 2 - CH 3 reaction R
NaOH R
(B) 2C6 H 5 CHO ¾¾¾® (q) Kolbe’s
(B) C = NOH (q) Semicarbazone
C6H5COONa + C6H5CH2OH reaction R
Anhyd. R
(C) C6 H 6 + CH 3 COCl ¾¾¾¾
® (r) Clemmensen’s
C = N – NH2
AlCl3 (C) (r) Imine
R
C6 H5 COCH3 reaction
O
(D) C 6 H 5 OH + CO 2 + NaOH ® (s) Cannizzaro’s R
HOC6 H 4 COONa reaction (D) C = N – NH – C – NH 2 (s) Hydrazone
R
(a) A – (p), B – (q), C – (r), D – (s)
(a) A – (q), B – (s), C – (p), D – (r)
(b) A – (r), B – (p), C – (q), D – (s)
(b) A – (r), B – (p), C– (s), D– (q)
(c) A – (r), B – (s), C – (p), D – (q)
(c) A – (r), B – (s), C – (p), D– (q)
(d) A – (s), B – (r), C – (p), D – (q)
(d) A – (s), B – (r), C – (q), D– (p)
53. Match the columns
Column-I Column-II 57. Acetal formation is a reversible reaction
+
(Reactions) (Reagents) R H R OH R'OH, H +
(A) Benzophenone ® (p) LiAlH4 C = O + R'OH C
H H OR'
Diphenylmethane
R OR'
(B) Benzaldehyde ® (q) DIBAL–H C + H2O
1-Phenylethanol H OR'
(C) Cyclohexanone ® (r) Zn(Hg)/conc. HCl Under what conditions, the reaction can be forced to
Cyclohexanol proceed only in right (forward) direction ?
(D) Phenyl benzoate ® (s) CH3MgBr Benzaldehyde (a) Using excess of alcohol
(a) A – (p), B – (s), C – (r), D – (q) (b) Using high temperature
(b) A – (q), B – (s), C – (p), D – (r) (c) Using dilute acid and excess of alcohol
(c) A – (s), B – (r), C – (q), D – (p) (d) Using dry acid and excess of alcohol
(d) A – (r), B – (s), C – (p), D – (q) 58. When ethanal reacts with propanal in the presence of a
54. Observe the following structures and pick up the correct base, the number of products formed is
statement. (a) 2 (b) 3
+ (c) 4 (d) 5
C=O C = OH
R NH3 Hydrolysis
I II 59. HCN
C = O ¾¾® (A) ¾¾® (B) ¾¾¾¾® (C)
(a) Carbonyl carbon of I is more electrophilic than that R
of II Compound (C) in above reaction is
(b) Carbonyl carbon of I is less electrophilic than that (a) a-hydroxy acid (b) a-amino acid
of II (c) a-amino alkanol (d) a-amino b-hydroxy acid
(c) Carbonyl carbon of both structures have equal 60. Benzaldehyde is less reactive than propanal because
electrophilic character (i) the carbon atom of the carbonyl group of
(d) It depends upon the complete structure of the benzaldehyde is less electrophilic as in propanal.
compound (ii) the carbon atom of the carbonyl group of
55. Which of the following reagent reacts in different ways benzaldehyde is more electrophilic as in propanal.
with CH3CHO, HCHO and C6H5CHO ? (iii) carbonyl group in benzaldehyde is more polar due to
(a) Fehling solution resonance
(b) C6H5NHNH2 (iv) carbonyl group in benzaldehyde is less polar due to
(c) Ammonia resonance
(a) (i) and (iii) (b) (i) and (iv)
(d) HCl
(c) (i) only (d) (iv) only
EBD_8350
398 CHEMISTRY

61. Addition of hydrogen cyanide to aldehydes and ketones


occurs in presence of a base.The role of base is to H /D (i) O3/H2O/Zn/D
(i) catalyse the reaction 67. [X] [Y]
(ii) KOH, D
(Major) (Major)
(ii) generate CN– ion
(iii) slow down the reaction OH
(iv) to stabilize the cyanohydrins [Y] is
(a) (i) and (iii) (b) (i) and (ii) O O
(c) (i) and (iv) (d) (ii) and (iv)
62. Suppose the reaction of compound containing ketone as (a) (b)
functional group is carried in basic medium of NaOH.
Which of the following will one use to protect the unwanted O
O
reaction due to presence of carbonyl moiety?
(a) NaHSO3 (c) (d)
(b) HCN
(c) ethylene glycol and HCl
(d) None of these 68. Arrange the following in order of rate of Cannizaro’s
63. A compound C5H10O forms orange–red precipitate upon reaction (Max. to min.)
reaction with 2,4–DNP, but does not give positive Tollen’s
O O O
test and iodoform test. Possible compound is
(a) 2, 2–dimethylpropanal (b) 3–methylbutan–2–one H—C—H D—C—D T—C—T
(c) Pentan–3–one (d) None of the above (1) (2) (3)
64. Nitration of the compound is carried out, this compound
gives red–orange ppt. with 2,4–DNP, this compound (a) 1 > 2 > 3 (b) 2 > 3 > 1
undergoes Cannizzaro reaction but not aldol, then possible (c) 1 = 2 = 3 (d) 1 > 2 = 3
product due to nitration is 69. Best Hydride donar is
(a) 3–nitroacetophenone V V
O O
(b) (2–nitro)–2–phenylethanal
(c) (2–nitro)–1–phenylpropan–2–one (a) H—C—H (b) CH3—C—CH3
(d) 3–nitrobezaldehyde
OH OV
O
V V
O O
65. [A] + aq. K2CO3 25°C [A] will be
(c) CH3—C—H (d) CH3—C—H
O OH
OV OH
70. Which molecule on intramolecular aldol candensation will
produce [X]?
(a) (b) O
O HO O
O
O
O
[X]
(c) (d) COCH3 CHO
O O
66. Which among the following will form most stable enolate? (a) (b)
O
O CH3
O
CHO COCH3
(a) (b) O Ac
O O
O O O (c) (d)

Ac
(c) (d) O O
O O
ALDEHYDES, KETONES AND CARBOXYLIC ACIDS 399

OH
V O
71. OHC(CH2)3COCH3 Major product is
D H
O O
OHC—CH2—
(a) (b) Ac O

COCH3 (a)

(c) O CH3 (d)


(b)

CH2—CH3
(c)
CH==C—C—H
72. is obtained from [X] and [Y]
O

when reacted in presence of base. (d)


[X] and [Y] are
CHO 75. Which will have maximum enol content?
(a) + CH3CH2CHO
(a) (b) O
CHO
O O
(b) + CH3(CH2)2CHO
(c) O (d)
CHO O
(c) + CH3CH2—COCH3 O
76. Arrange the following in correct order of rate of N. A.R
(nucleophilic addition reaction)
CH2CHO
(1) HCHO (2) CH3CHO
(d) + Propanal CHO CHO

73. Cyanide after acid catalysed hydrolysis gives RCOOH (3) (4)
H3O
R—CN RCOOH CH3
NO2
What would be the expected product of the following
reaction? (a) 1 > 2 > 3 > 4 (b) 1 > 3 > 2 > 4
(c) 4 > 3 > 2 > 1 (d) 3 > 1 > 2 > 4
CN (i) NaOH
CN (ii) H3 O /D
77. In [X], carbonyl groups are numbered as 1, 2, 3 and 4
O O O
4
3
(a) (b) NH O
2
COOH O O==C 1 O
COOH CH3
(c) NH (d)
Arrange these carbonyl groups in decreasing order of
nucleophilic addition reaction
74. Which starting material should be used to produce the
(a) 3 > 2 > 4 > 1 (b) 3 > 4 > 2 > 1
compound shown below?
(c) 1 > 3 > 4 > 2 (d) 3 > 2 > 1 > 4
EBD_8350
400 CHEMISTRY

O 82. Structure of the compound whose IUPAC name is 3-ethyl-


2-hydroxy-4-methylhex-3-en-5-ynoic acid is :
OH dry HCl gas (i) LiAlD 4
78. + [Y], OH OH
OH (ii) H 2 O/H
(a) COOH (b) COOH
O
[Y] is
OH OD OH
D COOH
(c) (d) COOH
(a) (b) OH
83. Match the columns
O O Column-I Column-II
O O COOH
(A) CH2 (p) Glutaric acid
(c) (d) COOH
D CH2 COOH
OH OD
(B) (q) Adipic acid
79. An organic compound ‘X’ on reductive ozonolysis CH2 COOH
O
followed by heating with NaOH gives (C) CH2 COOH (r) Succinic acid

[X] will be CH2

(a) (b) CH2 COOH

(D) CH2 COOH (s) Malonic acid


(c) (d)
CH2
80. Choose correct statement about cannizaro and cross
CH2
cannizaro reaction
(I) Both are the examples of disproportionation COOH
CH2
(II) Both are the examples of redox
(III) Disproportionation occurs only in Cannizaro (a) A – (q), B – (p), C – (s), D – (r)
(IV) Both occurs in strongly alkaline medium (b) A – (r), B – (p), C – (s), D – (q)
(c) A – (s), B – (r), C – (p), D – (q)
(a) I, II, III, IV (b) I, II, IV
(d) A – (r), B – (q), C – (s), D – (p)
(c) II, IV (d) II, III, IV
Topic 5: Preparation of Carboxylic Acids
Topic 4: Nomenclature and Structure of Carboxyl Group
84. Which of the following can not be oxidised to give
81. Which of the following best represents the structure of carboxylic acid?
the carboxylate ion ?
– –
CH2CH2CH3
d d CH3
O O
(a) R– C (b) R – C (a) (b)
O+ O–
d d
CH3
d+ CH3
O HC CH3 C CH3
(c) R– C (d) None of these CH3
O
(c) (d)
d+
ALDEHYDES, KETONES AND CARBOXYLIC ACIDS 401

90. Ethanoic acid can’t be obtained by which of the following


85. MgBr reaction ?
(i) KCN
(i) C2 H5Cl ¾¾¾¾¾

(i) CO 2 (ii) H3O
¾¾¾¾
® P
(ii) H 3O Å (i) AgCN
(ii) CH3Cl ¾¾¾¾¾
+
®
(ii) H3O
In the above reaction product 'P' is
KMnO / OH -
CHO COOH (iii) CH 3CH = CH 2 ¾¾¾¾¾¾
4 ®
heat
(i) Mg
(iv) CH3Br ¾¾¾¾¾
®
(a) (b) (ii) CO2
(iii) H3O +
(a) (iii) and (iv) (b) (i) and (ii)
(c) (ii) and (iii) (d) (i) and (iv)
OH
91. Acetic acid is obtained when
O (a) methyl alcohol is oxidised with potassium
||
(c) (d) C6 H5 - C - C 6 H5 permanganate
(b) calcium acetate is distilled in the presence of calcium
formate
86. The end product B in the sequence of reactions, (c) acetaldehyde is oxidised with potassium dichromate
- and sulphuric acid
CN NaOH
R - X ¾¾¾® A ¾¾¾¾ ® B is
(d) glycerol is heated with sulphuric acid
(a) an alkane 92. Cyanohydrin of which of the following forms lactic acid?
(b) a carboxylic acid (a) HCHO (b) CH3COCH3
(c) sodium salt of carboxylic acid (c) CH3CHO (d) CH3CH2CHO
(d) a ketone 93. Identify the product C in the series :
87. In the given reaction,
Na C H OH HNO Cr O 2 - H +
(C6 H5 CO) 2
H2O
O ¾¾¾ ®I CH 3CN ¾¾¾¾¾¾
2 5
® A ¾¾¾¾
2 ® B ¾¾¾¾¾®
2 7
C
H O (a) CH3COOH (b) CH3CH2NHOH
C6 H 5 COOCOCH 3 ¾¾¾ 2 ® II
(c) CH3CONH2 (d) CH3CHO
Identify the product(s) formed in the given reaction.
I II 94. Identify the method by which Me3CCO2H can be prepared
(a) 2 molecules of benzoic acid 2 molecules of ethanoic (a) Treating 1 mol of MeCOMe with 2 moles of MeMgI.
acid (b) Treating 1 mol of MeCO2Me with 3 moles of MeMgI.
(b) 2 molecules of benzoic acid 1 molecule of benzoic acid (c) Treating 1 mol of MeCHO with 3 moles of MeMgI.
and 1 molecule of ethanoic acid (d) Treating 1 mol of dry ice with 1 mole of Me3CMgI.
(c) 1 molecule of ethanoic acid 1 molecule of benzoic 95. Pyruvic acid is obtained by
acid
(a) oxidation of formaldehyde cyanohydrin
(d) 1 molecule of benzoic acid 1 molecule of butanoic
acid (b) oxidation of acetaldehyde cyanohydrin
88. Select the acid(s) which cannot be prepared by Grignard (c) oxidation of benzaldehyde cyanohydrin
reagent. (d) oxidation of acetone cyanohydrin
(a) Acetic acid (b) Succinic acid
(c) Formic acid (d) All of the above Topic 6: Properties of Carboxylic Acids and their Derivatives
89. Through which of the following reactions number of carbon
96. Lower carboxylic acids are soluble in water due to
atoms can be increased in the chain?
(a) low molecular weight (b) hydrogen bonding
(i) Grignard reaction
(c) dissociation into ions (d) easy hydrolysis
(ii) Cannizzaro’s reaction
97. Dimerisation of carboxylic acids is due to
(iii) Aldol condensation (a) ionic bond
(iv) HVZ reaction (b) covalent bond
Choose the correct option. (c) coordinate bond
(a) Only (iii) and (i) (b) Only (iii) and (ii) (d) intermolecular hydrogen bond
(c) Only (iii) and (iv) (d) (i), (ii), (iii) and (iv)
EBD_8350
402 CHEMISTRY

98. Boiling points of carboxylic acids are 1. COOH Oxalic acid


(a) lower than corresponding alcohols
(b) higher than corresponding alcohols
COOH
(c) equal to that of corresponding alcohols
2. HOOC – CH2 – COOH Malonic acid
(d) None of the above
99. In the anion HCOO– the two carbon-oxygen bonds are 3. CH2– COOH Succinic acid
found to be of equal length. What is the reason for it?
(a) Electronic orbitals of carbon atom are hybridised CH2– COOH
(b) The C=O bond is weaker than the C–C bond (a) 3 > 2 > 1 (b) 1 > 2 > 3
(c) The anion HCOO– has two reasonating structures (c) 2 > 3 > 1 (d) 2 > 1 > 3
(d) The anion is obtained by removal of a proton from
111. Assertion : Compounds containing –CHO group are easily
the acid molecule
oxidised to corresponding carboxylic acids.
100. Carboxylic acids are more acidic than phenol and alcohol
Reason : Carboxylic acids can be reduced to alcohols by
because of
treatment with LiAlH4.
(a) intermolecular hydrogen bonding
(b) formation of dimers (a) Assertion is correct, reason is correct; reason is a
(c) highly acidic hydrogen correct explanation for assertion.
(d) resonance stabilization of their conjugate base (b) Assertion is correct, reason is correct; reason is not a
101. Which of the following has the maximum acidic strength ? correct explanation for assertion.
(a) o-nitrobenzoic acid (b) m-nitrobenzoic acid (c) Assertion is correct, reason is incorrect.
(c) p-nitrobenzoic acid (d) p-nitrophenol (d) Assertion is incorrect, reason is correct.
102. Which one of the following esters is obtained by the 112. Which of the following is the correct decreasing order of
esterification of propan-2-ol with ethanoic acid ? acidic strength of
(a) (CH3)2CHCOOCH3 (b) CH3COOCH2CH3 (i) Methanoic acid (ii) Ethanoic acid
(c) CH3COOCH(CH3)2 (d) (CH3)2CHCOOCH2CH3 (iii) Propanoic acid (iv) Butanoic acid
103. Among the following acids which has the lowest pK a (a) (i) > (ii) > (iii) > (iv) (b) (ii) > (iii) > (iv) > (i)
value? (c) (i) > (iv) > (iii) > (ii) (d) (iv) > (i) > (iii) > (ii)
(a) CH 3 CH 2 COOH (b) (CH3 ) 2 CH - COOH 113. In a set of the given reactions, acetic acid yielded a
(c) HCOOH (d) CH 3 COOH product C.
104. Which reagent can convert acetic acid into ethanol ? CH3 COOH + PCl5 ¾¾
® A ¾¾¾®
6 6 CH
B ¾¾¾¾¾
2 5 ®C
C H MgBr
(a) Na + alcohol (b) LiAIH4 + ether Anh.AlCl 3 Ether
(c) H2 + Pt (d) Sn + HCl
Product C would be
105. Benzoic acid may be converted to ethyl benzoate by
reaction with C2 H 5
(a) sodium ethoxide (b) ethyl chloride |
(c) dry HCl—C2H5OH (d) ethanol (a) CH 3 - C (OH)C 6 H 5 (b) CH 3 CH (OH)C 2 H 5
106. Benzoic acid reacts with conc. HNO3 and H2SO4 to give : (c) CH 3 COC 6 H 5 (d) CH 3 CH (OH ) C 6 H 5
(a) 3-Nitrobenzoic acid 114. Kolbe’s electrolytic method can be applied on
(b) 4-Benzene sulphonic acid
(c) 4-Nitrobenzoic acid (i) CH 2 COONa (ii) CHCOONa
(d) 2-Nitrobenzoic acid | ||
107. The yield of ester in esterification can be increased by CH 2 COONa CHCOONa

CH 3 CH 2 OH + CH 3COOH ‡ˆˆ ˆˆ† (iii) C6 H5COOK (iv) CH3COOK


CH3COOCH2CH3 + H 2O
(a) (i), (ii) and (iv) (b) (i), (ii) and (iii)
(a) removing water
(b) taking ethanol in excess (c) (ii), (iii) and (iv) (d) (iii) and (iv)
(c) taking acetic acid in excess 115. Which of the following represents the correct order of the
(d) all the above factors acidity in the given compounds?
108. A carboxylic acid can best be converted into acid chloride (a) FCH2 COOH > CH 3 COOH > BrCH 2COOH >
by using ClCH2COOH
(a) PCl5 (b) SOCl2 (b) BrCH2COOH > ClCH2COOH > FCH2COOH >
(c) HCl (d) ClCOCOCl CH3COOH
109. The strongest acid among the following is –
(a) Salicylic acid (b) m-hydroxybenzoic acid (c) FCH2COOH > ClCH2COOH > BrCH2COOH >
(c) p-hydroxybenzoic acid (d) Benzoic acid CH3COOH
110. Arrange the following carboxylic acid in their decreasing (d) CH3 COOH > BrCH2COOH > ClCH2COOH >
acidity. FCH2COOH
ALDEHYDES, KETONES AND CARBOXYLIC ACIDS 403

116. The correct order for the acidic character of the following
OCH3
carboxylic acids is OCH3
COOH
COOH COOH (c) (d)
OH COOH COOH
121. NH2CH2CH2NH2 is called ethylene diamine. What would
OH
I II be the final product of the reaction when ethylene diamine
III reacts with di ethyl oxalate is?
COOH COOH H H
CH3 HO OH N O
N OEt
(a) (b)
N OEt N O
IV V
(a) IV > I > II > III > V (b) V > II > III > I > IV H H
H
(c) V > II > IV > III > I (d) V > II > IV > I > III COOEt
N == C N COOEt
P + Cl2 (Excess) SOCl2 H2
117. CH3COOH ¾¾¾¾¾® ¾¾¾® ¾¾¾® (c) COOEt (d)
Pd/BaSO 4
OH NH2 N COOEt
¾¾¾®
Pd/BaSO4
H
Final product of the reaction is
122. Consider the following processes
(a) CCl3CH2OH (b) CCl 3COO Process (1) C2H5OH/HÅ, LiAlH4, PCC
(c) CHCl3 (d) Both (a) and (b) Process (2) NaBH4, Cu/300°C
118. Which of the following carboxylic acid can be esterified Process (3) PCl5, LiAlH4, CrO3/H2SO4
most readily?
Process (4) SOCl2, Pd/BaSO4/H2
(a) CH3COOH
The process by which acetic acid can be converted into
(b) (CH3)2CHCOOH acetaldehyde is/are
(c) (CH3)2CCOOH (a) 1, 2, 3, 4 (b) 1, 4
CH3 (c) 1, 3, 4 (d) 2
(d) CH — CH2COOH O
H Ph
CH3 OH0 /H 2O
123. Ph—C 18 ¾¾¾¾¾ ® P1 + P2
Br O CH3
(1) Mg (Excess)
¾¾¾¾¾¾¾ Products P1 and P2 are
119. Å ® (X) would be
(2) CO2 /H3O /D
I O OH
COOH 18 V
(a) Ph—C—O and Ph
(a)
COOH H
18
COOH OH
(b) V
(b) PhCOO and Ph
I
COOH H
18
(c) OH
Br V
(d) None of these (c) PhCOO and H
120. Which will undergo decarboxylation rapidly?
Ph
OH
V 18
(a) (b) CH3 (d) PhCOO and CH3
COOH COOH H Ph
EBD_8350
404 CHEMISTRY

COOH
COOH
124. COOH D
¾¾ ® Product
COOH (b)
Final product of the reaction is
O

COOH

(a) (b)
(c) CH3O
CH3
O

COOH COOH
(c) (d) COOH
(d)
Å
O
P/ Br KCN HO
125. Propionic acid¾¾¾®
H O
2
[W] ¾¾¾® [X]¾¾¾
3
¾
® [Y],
2
[Y] is C2H5
(a) COOH heat
129. Assertion: CH3—C—COOH ¾¾¾
® Product,
COOH
(b) COOH
COOH
OH In this reaction two products are formed.
(c) COOH Reason: Due to rearrangement CH3—C(COOH)2
(d) COOH C2 H5
OH converts into the mixture of
COOH
® [ X ] , [ X ] is
PCl CH3—C(COOH)2 and HOOCCH2—CH—COOH
126. ¾¾¾
5

C2H5 C 2 H5
Cl OH
COOH COCl (a) Assertion is true, Reason is true: Reason is a correct
(a) (b) explanation for assertion.
(b) Assertion is true, Reason is true: Reason is a not a
Cl correct explanation for assertion.
COCl (c) Assertion is true, Reason is false
(c) (d) (d) Assertion is false, Reason is true

COOH

127. Consider the two carboxylic acids sodalime


Decalin.
(CH3)3CCOOH (CH3)3SiCOOH 130. Assertion:
(i) (ii)
Br
The correct statement is
(a) (pKa)2 > (pKa)1 (b) (pKa)1 > (pKa)2 Reason: Carbanion is produced as intermediate, which
gives SN2 reaction with Br group.
(c) (pKa)1 = (pKa)2 (d) None of these
(a) Assertion is true, Reason is true: Reason is a correct
CH3O COOC2H5 explanation for assertion.
H OÅ
128. ¾¾¾¾
3 ® [ X ] , [ X ] is (b) Assertion is true, Reason is true: Reason is a not a
CH3O COOC2H5 D
correct explanation for assertion.
O (c) Assertion is true, Reason is false
(a) (d) Assertion is false, Reason is true
ALDEHYDES, KETONES AND CARBOXYLIC ACIDS 405

1. Acetic acid is dissolved in water having oxygen as 18O. 7. In the scheme given below, how many number of
How many 18O atom will be present in the product intramolecular aldol condensation products formed from
molecule? ‘Y’?

OMe
(i) H + H O 1. O
¾¾¾ 3
¾® Y ¾¾¾¾¾®
1. NaOH (aq)
2. ¾¾¾¾¾ 2 ®
Z. 2. Zn, H 2O 2. heat
(ii) heat
OMe
COOC2H5 8. Among the following, how many number of reaction(s)
produce(s) benzaldehyde?
How many number of sp2 hybridised carbon atoms are
there in the product Z.
3. How many membered ring will be formed in the product P ¾¾¾¾¾¾¾¾¾
®
CO, HCl
I. Anhydrous AlCl3 /CuCl
of the following reaction.

O
OH (i) NaBH
¾¾¾¾¾

+ P H O
(ii) H3O II. ¾¾¾®
2
O 100°C

4. Compound A is obtained by following reaction.


(i) C H ONa
CH3COOC2H5 ¾¾¾¾¾¾ ® [A]
2 5
+ H
(ii) H3O ¾¾¾¾¾
2 ®
III. Pd - BaSO 4
How many following statement(s) is(are) true about A ?
(i) It gives red colour with blue litmus solution.
(ii) It decomposes NaHCO 3 solution and evolves DIBAL–H
¾¾¾¾¾¾®
CO2 gas. IV. Toluene,–78°C
H 2O
(iii) It decolourises bromine water colour.
(iv) It reacts with 2,4-dinitrophenyl-hydrazine 9. Find the number of reactants among the given options
(v) Product A will undergo acid catalysed halogenation. which does not undergo Michael addition?
5. How many of the following compound is decarboxylated (I) C6 H5CH = CHCOCH3 + C6 H5COCH3
on heating ?
(II) C6 H5CH = CHCH 2COC6H5 + CH 2 (COOC2H5 ) 2
CH 2 COOH
(I) ½ (II) C2H5CH(COOH)2
CH 2 COOH (III) + Br COCH = CH

O (IV) CH 2 = CHCOCH3 + CH3COCH3


COOH
(III) CH3COCH2COOH (IV) 10. How many number of aldol reaction(s) occurs in the given
transformation?

6. How many of the following compounds can be used as an OH OH


acylating agent ? conc. aq. NaOH
CH3CHO + 4HCHO ¾¾¾¾¾¾®
(I) CH3COCl (II) (CH3CO)2O
HO
(III) CH3COOH (IV) CH3CH2COCl. OH
EBD_8350
406 CHEMISTRY

NCERT Exemplar MCQs 6. Cannizzaro's reaction is not given by ........ .


1. Addition of water to alkynes occurs in acidic medium and CHO
in the presence of Hg2+ ions as a catalyst. Which of the (a) (b) CHO
following products will be formed on addition of water to CH3
but - 1-yne under these conditions? (c) HCHO (d) CH3CHO
O
|| 7. Which product is formed when the compound
(a) CH 3 — CH 2 — CH 2 — C— H
O CHO is treated with concentrated aqueous KOH
||
(b ) CH3 — CH 2 — C — CH 3 solution?
O
|| + –
(a) KO CHO
(c) CH3 — CH 2 — C — OH + CO 2

O O O
|| || || - +
(d) CH 3 — C— OH + H — C — H (b) C– OK + CH2OH
2. Which of the following compounds is most reactive
towards nucleophilic addition reactions? O
+ – || - + + - – +
O O (c) KO C O K+ K O OK
|| ||
(a) CH 3 — C — H (b) CH 3 — C — CH 3

O O O
|| || || - +
–+
(c) —C—H (d) — C — CH 3 (d) C— O K + OK

3. The correct order of increasing acidic strength is ........ .


40% H SO
(a) phenol < ethanol < chloroacetic acid < acetic acid 8. CH3 — C º CH ¾¾¾¾¾®
2 4 A
1% HgSO 4
(b) ethanol < phenol < chloroacetic acid < acetic acid
(c) ethanol < phenol < acetic acid < chloroacetic acid Isomerisation
¾¾¾¾¾¾ ® CH3 — C — CH3
(d) chloroacetic acid < acetic acid < phenol < ethanol ||
O
O
|| structure of 'A' and type of isomerism in the above reaction
4. Compound Ph — O — C— Ph can be prepared by the are respectively
reactions of ............ . (a) Prop-1-en-2-ol, metamerism
(a) phenol and benzoic acid in the presence of NaOH (b) Prop-1-en-1-ol, tautomerism
(b) phenol and benzoyl chloride in the presence of (c) Prop-2-en-2-ol, geometrical isomerism
pyridine (d) Prop -1-en-2-ol, tautomerism
(c) phenol and benzoyl chloride in the presence of ZnCl2
9. Compounds A and C in following reaction are ........... .
(d) phenol and benzaldehyde in the presence of palladium
5. The reagent which does not react with both, acetone and ( i ) CH MgBr H SO ,D
CH 3 CHO ¾¾¾¾¾¾
( )
3 ® ( A ) ¾¾¾¾¾
2 4 ® (B)
benzaldehyde? ii H 2O

(a) Sodium hydrogen sulphite


Hydroboration oxidation
(b) Phenyl hydrazine ¾¾¾¾¾¾¾¾¾® ( C )
(c) Fehling's solution (a) identical (b) positional isomers
(d) Grignard reagent (c) functional isomers (d) optical isomers
ALDEHYDES, KETONES AND CARBOXYLIC ACIDS 407

10. Which is the most suitable reagent for the following OH


conversion?
COCH3
O (b)
||
CH3 — CH = CH — CH 2 — C— CH 3 ¾¾
® COCH3

O OH
||
CH3 — CH = CH — CH 2 — C — OH COOCH3
(c)
(a) Tollen's reagent
(b) Benzoyl peroxide
(c) I2 and NaOH solution OCOCH3
(d) Sn and NaOH solution
11. Which of the following compounds will give butanone on (d)
COOH
oxidation with alkaline KMnO4 solution?
(a) Butan-1-ol (b) Butan-2-ol
15. In the reaction,
(c) Both (a) and (b) (d) None of these
LiAlH PCl alc.KOH
12. In Clemmensen reduction, carbonyl compounds is treated CH 3 COOH ¾¾¾¾
4 ® A ¾¾¾
5 ® B ¾¾¾¾® C

with ......... .
the product C is: [JEE M 2014, S]
(a) zinc amalgam + HCl
(a) Acetaldehyde (b) Acetylene
(b) sodium amalgam + HCl
(c) Ethylene (d) Acetyl chloride
(c) zinc amalgan + nitric acid
16. An organic compound 'X' having molecular formula C5H10O
(d) sodium amalgam + HNO3
yields phenyl hydrazone and gives negative response to
Past Year MCQs the Iodoform test and Tollen's test. It produces n-pentane
on reduction. 'X' could be :- [AIPMT 2015, S]
13. Which one is most reactive towards Nucleophilic addition (a) 2-pentanone (b) 3-pentanone
reaction? [AIPMT 2014, S] (c) n-amyl alcohol (d) pentanal
CHO COCH3
(a) (b)
17. Treatment of cyclopentanone =O with methyl

CHO CHO lithium gives which of the following species?


[AIPMT 2015, C]
(c) (d) (a) Cyclopentanonyl cation
(b) Cyclopentanonyl radical
CH3 NO2 (c) Cyclopentanonyl biradical
(d) Cyclopentanonyl anion
14. Sodium phenoxide when heated with CO2 under pressure
18. The enolic form of ethyl acetoacetate as below has:
at 125 ºC yields a product which on acetylation produces C.
[AIPMT 2015, S]
[JEE M 2014, S]
H H2
ONa H+ H3C C O H3C C O
125 °C
+ CO2 ¾¾¾
5 Atm
¾®A ¾¾¾® C C C C C
Ac2 O

The major product C would be OH OC2H5 O OC2H5

OCOCH3 (a) 16 sigma bonds and 1 pi - bond


(b) 9 sigma bonds and 2 pi - bonds
COOH (c) 9 sigma bonds and 1 pi - bond
(a)
(d) 18 sigma bonds and 2 pi - bonds
EBD_8350
408 CHEMISTRY

19. Reaction of a carbonyl compound with one of the following (d) a carbonyl compound with a hydrogen atom on its
reagents involves nucleophilic addition followed by alpha-carbon rapidly equilibrates with its
elimination of water. The reagent is : [AIPMT 2015 RS, A] corresponding enol and this process is known as
(a) a Grignard reagent keto-enol tautomerism.
(b) hydrazine in presence of feebly acidic solution 23. The product formed by the reaction of an aldehyde with a
(c) hydrocyanic acid primary amine is [NEET 2016, A]
(d) sodium hydrogen sulphite
(a) schiff base (b) ketone
20. Which one of the following esters gets hydrolysed most
(c) carboxylic acid (d) aromatic acid
easily under alkaline conditions? [AIPMT 2015 RS, A]
24. Consider the reactions :- [NEET 2017, A]
OCOCH3
+
Cu [Ag(NH ) ]
X ¾¾® A ¾¾¾¾¾® Silver mirror
3 2
(a) 573K OH /D
O 2N (C2H 6O) observed
OH /D
Y
OCOCH3 NH 2NHCONH2
Z
(b) Identify A, X, Y and Z
H3CO
(a) A-Methoxymethane, X-Ethanol, Y-Ethanoic acid, Z-
OCOCH3 Semicarbazide.
(c) (b) A-Ethanal, X-Ethanol, Y-But-2-enal, Z-Semicarbazone
(c) A-Ethanol, X-Acetaldehyde, Y- Butanone, Z-
Hydrazone
OCOCH3
(d) A-Methoxymethane, X-Ethanoic acid, Y-Acetate ion,
(d) Z-hydrazine.
Cl 25. Of the following, which is the product formed when
21. Which compound would give 5 - keto - 2 - methylhexanal cyclohexanone undergoes aldol condensation followed by
upon ozonolysis ? [JEE M 2015, S] heating ? [NEET 2017, A]

CH3
(a) (b)
CH3

(a) (b) H3C O OH

CH3 O

CH3 (c) (d)


CH3

CH3 O O OH
(c) (d)
CH3 26. The correct sequence of reagents for the following
conversion will be : [JEE M 2017, S]
22. The correct statement regarding a carbonyl compound with
a hydrogen atom on its alpha carbon, is : [NEET 2016, C] O HO CH3
(a) a carbonyl compound with a hydrogen atom on its
alpha-carbon never equilibrates with its corresponding
enol.
HO–CH3
(b) a carbonyl compound with a hydrgen atom on its CHO CH3
alpha-carbon rapidly equilibrates with its
corresponding enol and this process is known as (a) [Ag(NH3)2]+ OH–, H+/CH3OH, CH3MgBr
aldehyde-ketone equilibration. (b) CH3MgBr, H+/CH3OH, [Ag(NH3)2]+ OH–
(c) a carbonyl compound with a hydrogen atom on its
(c) CH3MgBr, [Ag(NH3)2]+ OH–, H+/CH3OH
alpha-carbon rapidly equilibrates with its
corresponding enol and this process is known as (d) [Ag(NH3)2]+ OH–, CH3MgBr, H+/CH3OH
carbonylation.
ALDEHYDES, KETONES AND CARBOXYLIC ACIDS 409

27. The major product obtained in the following reaction is : 30. The major product of the following reaction is:
[JEE M 2017, S]
(i) AlH (i–Bu)
R - C º N ¾¾¾¾¾¾®
2
[JEE M 2019, A]
(ii) H2 O
O
(a) RCOOH (b) RCONH2
O
(c) RCHO (d) RCH2NH2
31. The major product of the following reaction is:
DIBAL - H
¾¾¾¾¾
®
(1) KOH (aqueous)
COOH Br ¾¾¾¾¾¾
+
®
(2) CrO3/H
(3) H 2SO4/D
Br
OH OH [JEE M 2019, S]

(a) CHO (b) CHO O O

COOH CHO (a) (b)

Br Br

(c) CHO (d) CHO


O O
COOH CHO
28. Carboxylic acids have higher boiling points than aldehydes, (c) (d)
ketones and even alcohols of comparable molecular mass. HO HO
It is due to their [NEET 2018, C]
(a) Formation of intramolecular H-bonding 32. The major product of the following reaction is:
(b) Formation of carboxylate ion
LiAIH 4
CH3CH = CHCO2CH3 ¾¾¾¾ [JEE M 2019, A]
(c) Formation of intermolecular H-bonding ®
(d) More extensive association of carboxylic acid via van (a) CH3CH2CH2CO2CH3 (b) CH3CH = CHCH2OH
der Waals force of attraction (c) CH3CH2CH2CH2OH (d) CH3CH2CH2CHO
29. The major product of the following reaction is : 33. The major product of the following reaction is:
[NEET 2019, A] [JEE M 2019, A]
COOH
strong heating CH2CH3
+ NH3 ¾¾¾¾¾®
COOH (i) alkaline KMnO
¾¾¾¾¾¾®
+
4
(ii) H3O

COOH COOH CH2CHO


NH
(a) (b) (a) (b)
CONH2
O

CH2COOH COCH3
COOH NH2
(c) (d)
(c) (d)
NH2 NH2
EBD_8350
410 CHEMISTRY

34. The correct decreasing order for acid strength is:


CCl2 Cl
[JEE M 2019, A]
(a) NO2CH2COOH > FCH2COOH >
(c) (d)
CNCH2COOH > ClCH2COOH
(b) FCH2COOH > NCCH2COOH >
NO2CH2COOH > ClCH2COOH 37. Reaction between benzaldehyde and acetophenone in
(c) CNCH2COOH > NO2CH2COOH > presence of dilute NaOH is known as [NEET 2020]
FCH2COOH > ClCH2COOH (a) Cannizzaro's reaction
(d) NO2CH2COOH > NCCH2COOH > (b) Cross Cannizzaro's reaction
FCH2COOH > ClCH2COOH (c) Cross Aldol condensation
35. Reaction between acetone and methylmagnesium chloride (d) Aldol condensation
followed by hydrolysis will give : [NEET 2020] 38. What is the product of following reaction?
(a) Sec. butyl alcohol
(i) NaBH
4
(b) Tert. butyl alcohol Hex-3-ynal ¾ ¾ ¾ ¾ ® ? [JEE M 2020, S]
(ii) PBr3
(c) Isobutyl alcohol (iii) Mg/ether
(d) Isopropyl alcohol (iv) CO2/H3O +

36. Identify compound X in the following sequence of COOH


(a)
reactions [NEET 2020]
(b) COOH
CHO
(c) COOH
Cl /hv H O
¾¾
2
® X ¾ 2¾ ® (d) COOH
373K

CH2Cl CHCl2

(a) (b)

Exercise 4 : Problem Solving Skill Enhancer MCQs


1. Product, X of the following reaction is 3. When o-hydroxybenzaldehyde is heated with ethanoic
D
CDO + NaOH (Conc.) ¾¾ ® [X] (78%) anhydride in presence sodium ethanoate, compound
(1 : 1) formed during the reaction is
CHO
COO COO OH
(a) (b)
CH2OH CH2OD (a)
COOCH3
COO COO
(c) (d)
OH
CHDOH CHDOD
(b)
OH
– COOH
2. The reacion is an
O O
O O
CHO
example of (c)
(a) oxidation reaction (b) reduction
(c) both (d) aldol condensation (d) Both (b) and (c)
ALDEHYDES, KETONES AND CARBOXYLIC ACIDS 411

4. A new carbon - carbon bond is formed in 10. The major product of the following reaction is
Aldol condensati on Kolbe' s reaction CH2OH
I II

Reimer - Tiemann reaction (1) PCC


Wurtz Fittig reaction V
III IV (2) OH / D
(a) I, III (b) II, III (3) H / D
(c) I, III, IV (d) All the four CH2OH
5. The correct stability order of the following three quinones
V
is COO

O O (a)
O
O O
CH2OH
O
I II III
(b)
(a) I > III > II (b) I = III > II
(c) I = II > III (d) III > I > II O O
OH - ,Cl 2
6. CH 3COCH 2 Cl ¾¾¾¾¾
® [P], [P] is COOH
(a) ClCH2COCH2Cl
(b) CH3COCHCl2 (c)
(c) both (a) and (b)
(d) ClCH2COOH + CH3Cl CH2OH
7. Which of the following is an example of nucleophilic
addition ?
(d)
O NNH 2
O
|| NH NH ,H +
2 ¾2¾¾® C H C CH
||
(a) C 6 H 5 C CH 3 ¾¾ ¾ 6 5 3 O

O OH O
|| | OH
LiAlH
4 ® C H C HCH
CHO (1 mole)
(b) C 6 H 5 C CH 3 ¾¾ ¾¾ 6 5 3 11. OH NaBH4 H3 O
,
[X]
dry HCl gas
(c) Both (a) and (b)
(d) None of the two [X] is
O OH
O
|| CH2OH CHO
NH OH (a) (b)
8. CH3 C C2 H5 + NaCN + NH 4Cl ¾¾¾¾
4 ® Z ; Z is

CH3 OH CH3 OH
C C OH O
(a) (b)
C2H3 COOH C2H5 CN CH2OH CHO
(c) (d)
CH3 NH2 CH3 CN
(c) C (d) C
C2H5 CN C2H5 COOH O
9. Total number of possible aldol products (including
12. CHO Reagents O
stereoisomers) are
CH2OH
NaOH(dil)
C6H5CHO + CH3CHO ¾¾¾¾¾
65°C
® Reagents for this conversion should be
(a) 2 (b) 3 (a) MnO–4 then H+/D (b) Cr2O72– then H+/D
(c) 4 (d) 5 (c) Cu2+/OH– then H–/D (d) PCC then OH–/D
EBD_8350
412 CHEMISTRY

13. Correct order of rate of decarboxylation of following (b) Give effervescence of CO2 on reaction with NaHCO3
carboxylate ions is (c) Show Fehling test
(d) Lucas test
O O
COOH CH3 OH
D [W] [X]
O O 15.
F Cl COOH
SOCl2 CH3 NH 2
(1) (2) ¾¾¾¾ ® [Y] ¾¾¾¾® [Z]
Final product (Z) is
O O
COCl
O O (a)
CONHCH3
NO2
CONHCH3
(3) (4)
(b)
(a) 3 > 1 > 2 > 4 (b) 3 > 2 > 1 > 4 COOCH3
(c) 1 > 3 > 2 > 4 (d) 2 > 3 > 1 > 4 CO
O (c) N—CH3
CO
H OÅ
14. O ¾¾¾¾
3
D
® (X), (X) can COCl
O (d)
(a) Show 2, 4 DNP test CONH2

ANSW ER KEY
Exercise 1 : NCERT Based Topic-wise MCQs
1 (b) 14 (b) 27 (d) 40 (a) 53 (d) 66 (c) 79 (b) 92 (c) 105 (c) 118 (a)
2 (c) 15 (a) 28 (b) 41 (a) 54 (b) 67 (d) 80 (d) 93 (a) 106 (a) 119 (d)
3 (c) 16 (b) 29 (d) 42 (a) 55 (c) 68 (a) 81 (b) 94 (d) 107 (d) 120 (c)
4 (a) 17 (b) 30 (b) 43 (c) 56 (b) 69 (c) 82 (a) 95 (b) 108 (d) 121 (b)
5 (b) 18 (a) 31 (b) 44 (c) 57 (d) 70 (d) 83 (c) 96 (b) 109 (a) 122 (b)
6 (a) 19 (a) 32 (d) 45 (d) 58 (c) 71 (a) 84 (d) 97 (d) 110 (b) 123 (b)
7 (d) 20 (a) 33 (c) 46 (b) 59 (b) 72 (b) 85 (b) 98 (b) 111 (b) 124 (b)
8 (a) 21 (c) 34 (b) 47 (a) 60 (b) 73 (a) 86 (c) 99 (c) 112 (a) 125 (d)
9 (c) 22 (a) 35 (d) 48 (b) 61 (b) 74 (b) 87 (b) 100 (d) 113 (a) 126 (b)
10 (b) 23 (d) 36 (c) 49 (b) 62 (c) 75 (c) 88 (c) 101 (a) 114 (a) 127 (a)
11 (b) 24 (c) 37 (c) 50 (a) 63 (c) 76 (a) 89 (a) 102 (c) 115 (c) 128 (b)
12 (b) 25 (d) 38 (a) 51 (b) 64 (d) 77 (a) 90 (b) 103 (c) 116 (d) 129 (c)
13 (d) 26 (d) 39 (a) 52 (c) 65 (a) 78 (a) 91 (c) 104 (b) 117 (c) 130 (a)
Exercise 2 : Numeric/Integer Answer Questions
1 (2) 2 (1) 3 (5) 4 (4) 5 (3) 6 (3) 7 (1) 8 (4) 9 (2) 10 (3)
Exercise 3 : NCERT Exemplar & Past Year MCQs
1 (b) 5 (c) 9 (b) 13 (d) 17 (d) 21 (d) 25 (a) 29 (b) 33 (a) 37 (c)
2 (a) 6 (d) 10 (c) 14 (a) 18 (d) 22 (d) 26 (a) 30 (c) 34 (d) 38 (d)
3 (c) 7 (b) 11 (b) 15 (c) 19 (b) 23 (a) 27 (b) 31 (a) 35 (b)
4 (b) 8 (d) 12 (a) 16 (b) 20 (a) 24 (b) 28 (c) 32 (b) 36 (b)
Exercise 4 : Problem Solving Skill Enhancer MCQs
1 (c) 3 (d) 5 (d) 7 (c) 9 (c) 11 (b) 13 (b) 15 (b)
2 (d) 4 (d) 6 (b) 8 (c) 10 (d) 12 (c) 14 (a)
27 Amines

Trend Buster NEET & JEE Main

Number of Questions from 2020-15 9 8 Minimum one Qns has been asked in
Weightage 2.8% 4.6% NEET & JEE M. from this chapter every year.

The most Important Concepts that Cover Maximum number of Questions asked in past 6 years.

Preparation of Amines 3 1
Chemical Reactions of Amines 5 4
Properties of Diazonium Salts — 3

Less Important Concepts that Cover 1 or 2 Questions asked in past 6 years.

Structure, Classification & Nomenclature of Amines 1 —


Physical properties of Amines / Preparation of — —
Diazonium Salts
NEET JEE
Concept Used

2020 Chemical reactions of Basicity order / 1 Easy 2 Average


Amines / Properties of Coupling reaction /
Diazonium Salts Carbylamine reaction
2019 Chemical reaction of Basicity order / Coupling 1 Easy 3 Easy /
Amines / Properties of reaction / Qualitative Analysis Average
Diazonium Salts
2018 — — — — — —
2017 Preparation of Amines / Hoffman bromamide degra- 2 Easy 1 Average
Chemical Reactions of dation reaction / Basicity
Amines order / Nitration of Anilines
2016 Chemical reactions of Basicity order / Hoffman 1 Average 1 Easy
Amines / Preparation of bromamide degradation
Amines reaction
2015 Preparation of Amines / Aniline Preparation / Schotten 4 Easy / 1 Average
Chemical reactions of Baumann recation / Reduction Average
Amines / Str. of Amines / Pro- of nitrobenzene / Structural
perties of Diazonium Salts isomersim/Sandmeyer reaction
EBD_8350
414 CHEMISTRY
AMINES 415
EBD_8350
416 CHEMISTRY

Problem Solving Tips/ Tricks/ Points to Remember


4 Schotten-Baumann Reaction : Reaction of acid Mechanism :
chloride or sulphonyl chloride of aromatic carboxylic H
acids or aromatic sulphonic acid with an amine (1° or 2°; –
Cl
+
alphatic or aromatic) in presence of aqueous sodium R–N2 + : CCl2 ¾¾
® R–N–C ¾¾
®
hydroxide or pyridine to form substituted amides is Cl
known as Schotten-Baumann reaction. H

pyridine Cl + Cl
C6H5NH2 + C6H5COCl ¾¾¾¾® C6H5CONHC6H5
R–N–C ¾¾
® R–N–C
Benzanilide (–Cl)
4 Reaction with Aldehydes and Ketones : Primary amines Cl H
H H H
add (as nucleophiles) to the carbonyl group of an
Cl
aldehyde or a ketone to form carbinolamines, which
¾¾ ® R–N=C ¾¾
®
then dehydrate to form imines (also known as Schiff’s (–H) (– Cl)
bases) as the final product. This reaction is not given H
by tert-amines.
+ OH
Reaction with Carbon Disulphide : Primary amines
R – N = C – H ¾¾ ® R–N=C–H+HO
4
2
when warmed with carbon disulphide form a
dithiocarbamic acid which is decomposed by mercuric 4 Chemistry of the Hinsberg test :
chloride to give alkyl isothiocyanates, having mustard OH -
RNH2 + C6H5SO2Cl ¾¾¾® [C6H5SO2NHR]
oil-like smell, along with black precipitate of mercuric
1° Amine N-Substituted sulphonamide
sulphide. This reaction, known as Hofmann mustard (acidic H on N)
oil reaction, is used as a test for 1° amines.
KOH H+
NHR HgCl 2
¾¾¾® C6H5SO2N–RK+ ¾¾¾
® C6H5SO2NHR
RNH2 + S = C = S ¾¾® S = C ¾¾® Clear solution Precipitate
SH OH –

Dithiocarbamic acid R 2 NH + C6H5SO2Cl ¾ ¾ ¾®


2º Amine
R—N = C = S + HgS + 2HCl
Isothiocyana te Black KOH
C6 H5SO2 NR 2 ¾¾¾® Precipitate
Secondary amines also react with CS 2 to form N, N-Disubstituted
dithiocarbamic acids, but the latter do not react with sulphonamide
mercuric chloride (difference from 1° amines). (no acidic H)
NR2 HgCl 2 H+
¾¾® ¾¾¾¾¾® No reaction
R2NH + S = C = S ¾¾® S=C room temp.
SH Remember that N, N-disubstituted sulphonamides are
No reaction hydrolysed to give parent amine when heated with an
Tertiary amines do not react with carbon disulphide. acid.
(i) H O + , heat
4 Carbylamine Reaction of 1° Amines : C6H5SO2NR2 ¾¾¾¾¾¾
3
-
® C6H5SO2O– + R2NH
(ii) OH
RNH2 + CHCl3 + 3KOH ¾® R—N+ º C– : + 3 KCl + 3H2O OH
R 3 N + C H SO Cl ¾¾¾
-
An isocyanide 6 5 2 ®
3º Amine
(foul smelling)
HCl + -
Recall that here the nucleophile RNH2 attacks the R 3 N ¾¾¾ ® R 3 N Cl
electrophilic intermediate (dichlorocarbene, : CCl2) Insoluble Clear solution
formed by the action of a base (OH–) on CHCl3. 4 Reaction with HNO2 :
OH - -Cl-
HCCl3 ¾¾¾® :CCl3– ¾¾¾® :CCl2 (i) Primary amines react with nitrous acid to produce nitrogen
Dichlorocarbene gas [seen as bubbles]:
Since carbylamines have unpleasant smell which can
be easily detected, the reaction is used to identify C2H5NH2 + HONO ¾® C2H5OH + N2­ + H2O
primary amines.
AMINES 417

(ii) Secondary amines react with nitrous acid to produce a Selective reduction of one nitro group of a dinitro
yellow oily layer. compound can often be achieved by ammonium
bisulfide (NH4 HS) or through the use of carefully
(C2H5)2 N – H + HONO ¾® (C2H5)2N – N = O + H2O
measured amount of hydrogen sulphide in aqueous (or
N–Nitrosodiethyl amine alcoholic) ammonia
(yellow oil)
(iii) Tertiary amines react with nitrous acid to form soluble NO2 NH2
nitrite :
NH HS or
¾¾¾¾¾¾¾®
4
Å H2S, NH3, C2H5OH
(C2H5)3 N + HONO ¾® (C2H5)3 NHNO 2
– NO2 NO2
m-Dinitrobenzene m-Nitroaniline
Triethylammonium Nitrite
(soluble) Remember that LiAlH4 does not reduce PhNO 2 to
PhNH2.

4 Reduction of nitrobenzene in different mediums.

NH2

Sn – HCl, Fe – HCl
Acidic
N N
Zndust / NaOH
CH3OH
Azobenzene

Basic NH NH
Zndust / NaOH

Hydroazobenzene

NO2 Na3AsO 3 / NaOH O–


or glucose + NaOH
N = N+
¾¾
Zn/NH 4Cl Azoxybenzene
Neutral
NHOH

Phenyl hydroxy amine

NH2

Weakly acidic
Medium

Electrolytic
NHOH OH
Reduction

Strongly acidic Rearrangement


Medium

NH2
p-Amine phenol
EBD_8350
418 CHEMISTRY

Exercise 1 : NCERT Based Topic-wise MCQs


8. IUPAC name of the following compound is
Topic 1: Structure of Amines, Classification and Nomenclature
1. The total number of electrons around the nitrogen atom
in amines are NH2
(a) 8 (b) 7 (c) 4 (d) 3 Cl
2. Read the following statements and choose the correct (a) 2-chloro pentanamine
option. (b) 4-chloro pentan-1-amine
(i) Nitrogen atom in amines is sp3-hybridised. (c) 4-chloro pent-2-en-1-amine
(ii) The geometry of amines is pyramidal. (d) 2-chloro pent-3-en-5-amine
(iii) The angle C–N–C or C–N–H is slightly more than 9. Which of the following is the correct IUPAC name of the
109.5°. compound ?
(a) (i), (ii) and (iii) (b) (i) and (ii)
(c) (i) and (iii) (d) (ii) and (iii) N (CH 3 ) 2
3. A secondary amine is
(a) a compound with two carbon atoms and an –NH2
group. Cl
(b) a compound containing two –NH2 groups. Cl
(c) a compound in which hydrogens of NH3 have been
(a) 1, 2-dichloro-4-(N, N-dimethyl) aniline
replaced by two alkyl groups.
(b) Dimethyl – (3, 4-dichlorophenyl) amine
(d) a compound with an –NH2 group on carbon atom in
number two position. (c) 3, 4-dichloro - N, N-dimethyl aniline
4. The number of primary amines of formula C4H11N is : (d) N, N-dimethylamino - 3, 4-dichlorobenzene
(a) 1 (b) 3 (c) 4 (d) 2 Topic 2: Preparation of Amines
5. The IUPAC name of the compound having formula,
10. The general formula of quaternary ammonium compound
O C CH CH2 is is
(a) R–NH2 (b) R3N (c) R4N+ X– (d) NH4X
OH NH2 OH 11. Which of the following reactions will not give a primary
(a) 3-amino-hydroxy propine acid amine?
(b) 2-amino-propan-3-oic acid (a) Br / KOH
CH 3 CONH 2 ¾¾2¾ ¾
¾®
(c) amino hydroxy propanoic acid LiAlH
(b) CH 3CN ¾¾ ¾¾

(d) 2-amino-3-hydroxy propanoic acid
(c) LiAlH
6. What is the IUPAC name of the following compound ? CH 3 NC ¾¾ ¾¾

(d) LiAlH
NH 2 CH 3CONH 2 ¾¾ ¾¾

12. Propionamide on Hoffmann degradation gives –


(a) methyl amine (b) ethyl amine
(a) 2-methyl-4-hexanamine (c) propyl amine (d) ethyl cyanide
(b) 5-methyl-3-hexanamine 13. Secondary amines could be prepared by
(c) 2-methyl-4-amino hexane (a) reduction of nitriles
(d) 5-methyl-3-amino hexane (b) Hofmann bromamide reaction
7. The IUPAC name of diethyl isopropyl amine is (c) reduction of amides
(d) reduction of isonitriles
(a) N, N-diethylpropan-2-amine
14. Gabriel’s phthalimide synthesis is used for the preparation of
(b) N, N-diethylpropan-1-amine (a) Primary aromatic amines
(c) N, N-diethylisopropylamine (b) Secondary amines
(d) N, N-diethylaminopropane (c) Primary aliphatic amines
(d) Tertiary amines
AMINES 419

15. For alkylation of ammonia which of the following is not (b) Assertion is true, Reason is true: Reason is not a
used ? corret explanation for Assertion.
(a) CH3–X (b) CH3–CH2–X (c) Assertion is true, Reason is false.
(c) (CH3)2CH–X (d) (CH3)3C–X (d) Assertion is false, Reason is true.
16. The reduction of nitro compounds is most preferred in
the presence of Topic 3: Physical Properties
(a) Pd/H2 in ethanol (b) Sn + HCl 24. Amines have
(c) finely divided Ni (d) iron scrap and HCl.
(a) Garlic odour (b) Fishy odour
17. An alkyl or benzyl halide on reaction with an ethanolic
(c) Jasmine odour (d) Bitter almonds odour
solution of ammonia undergoes
(a) electrophilic substitution reaction 25. Aniline is less soluble in water than ethyl amine due to
(b) nucleophilic substitution reaction. (a) resonance stablization of benzene ring
(c) free radical mechanism. (b) resonance stabilization of anilium ion
(d) nucleophilic addition reaction. (c) more hydrophobic nature of C6H5 group than C2H5
18. In the ammonolysis of alkyl halides the halogen atom is group
replaced by an amino(–NH2) group which of the following (d) more hydrophobic nature of C2H5 group than C6H5
represent the correct order of reactivity of halides with amines. group
(a) RBr > RI > RCl (b) RI > RCl > RBr 26. Which of the following is not characteristic of amines?
(c) RI > RBr > RCl (d) RCl > RBr > RI (a) They smell like ammonia
19. Which of the following will give primary amine only ? (b) They are inflammable in air
(i) ammonia + propylchloride (c) They show the property of hydrogen bonding
(ii) potassium pthalimide + ethylchloride (d) They are amphoteric in nature
(iii) potassium pthalimide + chlorobenzene 27. Which of the following statements are correct ?
(a) (i) and (ii) (b) (i) and (iii)
(i) Lower aliphatic amines are soluble in water.
(c) (ii) and (iii) (d) (i), (ii) and (iii)
20. A primary amine is formed by an amide on treatment with (ii) Solubility increases with increase in molar mass of
bromine and alkali. The primary amine has amines.
(a) 1 carbon atom less than amide (iii) Higher amines are insoluble in water.
(b) 1 carbon atom more than amide (iv) Amines are soluble in organic solvents.
(c) 1 hydrogen atom less than amide (a) (i), (ii) and (iii) (b) (i), (iii) and (iv)
(d) 1 hydrogen atom more than amide (c) (ii), (iii) and (iv) (d) (i) and (iv)
21 CH3CH2NHCH3 can be prepared by reductive amination 28. Arrange the following compounds in decreasing order of
by using boiling point
(a) (CH3)3N < CH3NHEt < PrNH2 < PrOH
CH3
(a) C==O, NH3 (b) CH3CHO, CH3NH2 (b) (CH3)3N < PrNH2 < PrOH < MeNHEt
CH3
(c) PrNH2 < MeNHEt < Me3N < PrOH
(c) CH3CH2CHO, NH3 (d) HCHO, Et2NH (d) Me3N < MeNHEt < PrOH < PrNH2
22. Which of the following reaction does not give amine? 29. Assertion: Secondary amines have more boiling point than
D tertiary amines having same mol. wt
(a) RNCO + KOH
Reason: Molecules of secondary amines held with one
O another by H-bonding.
LiAlH4
(b) R—N—C—CH3 (a) Assertion is true, Reason is true: Reason is a correct
explanation for Assertion.
H (b) Assertion is true, Reason is true: Reason is not a
CO correct explanation for Assertion.
(c) N—CH3 + NH2—NH2 (c) Assertion is true, Reason is false.
CO (d) Assertion is false, Reason is true.
(d) all will produce amine
Topic 4: Chemical Reactions
23. Assertion: Aniline can not be prepared by Gabriel
phthalamide reaction while phenyl methyl amine can. 30. Aliphatic amines are.....basic than NH3 but aromatic amines
Reason: Aniline is less basic than phenyl methyl aniline are......basic than NH3.
(a) Assertion is true, Reason is true: Reason is a correct (a) more, less (b) less, more
explanation for Assertion. (c) both (a) and (b) (d) None of these
EBD_8350
420 CHEMISTRY

31. Substitution of one alkyl group by replacing hydrogen of 39. In the reaction,
primary amines HNO
(a) increases the base strength RNH 2 ¾¾¾¾ 2 ® ROH + H O + A ­ ; A is
2
(b) decreases the base strength (a) NH3 (b) N2 (c) O2 (d) CO2
(c) remains the same 40. Primary amines can be distinguished from secondary and
(d) None of the above tertiary amines by reacting with
32. The conjugate base of (CH3)2NH+2 is (a) Chloroform and alcoholic KOH
(a) (CH3)2NH (b) (CH3)2N+ (b) Methyl iodide
(c) (CH3)3N + (d) (CH3)2N– (c) Chloroform alone
33. High basicity of Me2 NH relative to Me3N in aqueous (d) Zinc dust
solution is attributed to: 41. The amine that does not react with acetyl chloride is
(a) effect of solvent (b) inductive effect of Me (a) CH3NH2 (b) (CH3)2NH
(c) shape of Me2NH (d) shape of Me3N (c) (CH3)3N (d) None of these
34. The correct order of basicity of the following compounds 42. In order to distinguish between C2H5–NH2 and C6H5–NH2,
which of the following reagent is useful
NH2 NH2 NH2 (a) Hinsberg’s reagent (b) HNO2
(c) CHCl3 + KOH (d) NaOH
43. Strong activating effect of –NH2 group is reduced by using
(a) CH3COCl (b) CH3Cl
NO2 (c) CH3ONa (d) CH3–CHO
(A) (B) (C) 44. When bromination of aniline is carried out by protecting
–NH2. The product is
(a) B > A > C (b) A > B > C
(a) o-bromoaniline
(c) C > A > B (d) C > B > A
(b) 2, 4, 6 tribromoaniline
35. Which of the following factors affect the basic strength
(c) p-bromoaniline
of amine?
(d) mixture of o-and p-bromoanilines
(i) Inductive effect
45. Hinsberg’s method to separate amines is based on the
(ii) Steric hinderance
use of
(iii) Solvation effect
(a) benzene sulphonyl chloride
(iv) Solubility in organic solvents.
(b) benzene sulphonic acid
(a) (i) and (iv) (b) (i), (ii) and (iii)
(c) ethyl oxalate
(c) (ii) and (iii) (d) (ii) and (iv)
(d) acetyl chloride
36. Which of the following statements about primary amines
46. Which of the following statements is not correct regarding
is ‘False’ ?
aniline ?
(a) Alkyl amines are stronger bases than aryl amines
(a) It is less basic than ethylamine
(b) Alkyl amines react with nitrous acid to produce
(b) It can be steam-distilled
alcohols
(c) It reacts with sodium to give hydrogen
(c) Aryl amines react with nitrous acid to produce phenols
(d) It is soluble in water
(d) Alkyl amines are stronger bases than ammonia
47. Aniline and other arylamines are usually colourless but
37. The compound obtained by heating a mixture of a primary
get coloured on storage due to___________.
amine and chloroform with ethanolic potassium hydroxide
(a) hydrolysis (b) dehydration
(KOH) is
(c) reduction (d) atmospheric oxidation
(a) an alkyl cyanide
48. The acylation reaction of amines is carried out in presence
(b) a nitro compound of pyridine because
(c) an alkyl isocyanide (i) pyridine is stronger base than amine.
(d) an amide (ii) pyridine is weaker base than amine.
38. R - NH 2 + CH3COCl ¾¾
®A (iii) pyridine removes HCl formed and shifts the
(excess) equilibrium to the right hand side.
The product (A) will be – (iv) pyridine removes HCl formed and shifts the
equilibrium to the left hand side.
(a) RNHCOCH3 (b) RN(COCH3)2
(a) (i) and (iii) (b) (ii) and (iv)
+ (c) (ii) and (iii) (d) (i) and (iv)
(c) RN(COCH3 )3 Cl- (d) R – CONH2
AMINES 421

49. N– Ethylbenzene sulphonyl amide is strongly acidic and (a) 1 > 3 > 2 (b) 3 > 2 > 1
soluble in alkali due to presence of (c) 1 > 2 > 3 (d) 2 > 1 > 3
(a) strong electron donating sulphonyl group. 54. The correct order of decreasing basic character is
(b) strong electron withdrawing sulphonyl group.
C6 H5 NH 2 , C6 H5CH 2 NH 2 ,
(c) weak electron donating sulphonyl group. I II
(d) weak electron withdrawing sulphonyl group.
( C6 H5 )2 NH , C6 H11 NH 2
50. Arrange the following in increasing order of their basic III IV
strength? (a) II > I > III > IV (b) IV > II > I > III
p–nitroaniline (1); m–nitroaniline (2); 2,4,6–trimethylaniline
(c) IV > III > II > I (d) IV > II > III > I
(3); 3–methylanline (4).
(a) 1, 3, 2, 4 (b) 2, 3, 4, 1 55. Aniline when treated with conc. HNO3 gives
(c) 3, 1, 2, 4 (d) 1, 2, 4, 3 (a) p-Phenylenediamine (b) m-Nitroaniline
51. Which of the following is/are correct regarding nitration (c) p-Benzoquinone (d) Nitrobenzene
of aniline with conc. HNO3 and conc. H2SO4 ? NH2
NH2 NH2 NH2
NO2 56. 3 (CH CO) O
2 ® (X) ¾¾¾¾ HNO
3 ® (Y) ¾¾¾ H+
conc. HNO
¾¾¾¾¾¾ ® (Z)
(i) ¾¾¾¾¾¾
3
® + H 2SO 4 H 2O
conc. H 2SO4

Product Z of the reaction is


NO2
NH2 NH2
+ +
NH3 NH3
(a) (b)
(ii) ¾¾
® NO2
NO2
NO2
(iii) The substitution can be explained on the basis of
inductive effect (– I) NO2 NO2
(iv) The substitution can be influenced by +M and +E
effects.
(a) (i), (ii) and (iii) (b) (ii) and (iii) (c) (d)
(c) (ii) and (iv) (d) (i) and (iii)
NO2
52. Assertion : Nitrating mixture used for carrying out nitration
of benzene consists of conc. HNO3 + conc. H2SO4. NO2
Reason : In presence of H2SO4, HNO3 acts as a base and NH2
produces NO2+ ions.
(a) Assertion is correct, reason is correct; reason is a Br - H O H O+
57. 3
¾¾¾¾¾
CH COCl
2 2 ® B ¾¾¾¾
® A ¾¾¾¾¾ 3 ®C
correct explanation for assertion.
(b) Assertion is correct, reason is correct; reason is not
a correct explanation for assertion C (major product) is –
(c) Assertion is correct, reason is incorrect
NH2 NH2
(d) Assertion is incorrect, reason is correct.
53. Arrange the following amines in the decreasing order of Br
their basicity (a) (b)
NH2 Br

(1) (2) NH2


NH2

(c) (d) None of these


CH2 – CH2 – NH2

(3) Br
EBD_8350
422 CHEMISTRY

58. A compound of molecular formulae C3H6N shows following ONHCONHNH2


characteristics (c)
(i) Get dissolved in acidic medium. COCH3
(ii) Does not react with benzoyl chloride OH
(iii) Does not give carbylamine test (d)
C== NCONHNH2
(iv) Does not evolve nitrogen gas on reacting with HNO 2.
Structure of the compound is CH3
(a) trimethylamine (b) isopropylamine 63. Which of the following is the weakest bronsted base?
(c) propylamine (d) None of these
N N
59. Which has more basic nitrogen than CH3—C— NH2 ? (a) (b)

O
O O
(a) CH3—C— NH —CN O
O
(c)
N (d) N
O O

(b) CH3—C— NH —C—CH3 O


64. Strongest acid among the following is
NH2 H
(c) NH2
N
(a) (b)
O
N
C— NH —COCH3
O
(d)
SO2NH2
(c) H—N (d)
60. Correct order of increasing basic character is
(a) NH3 < PhNH2 < Et2NH < EtNH2 < Et3N 65. Identify (X) in the following reaction.
(b) PhNH2 < NH3 < Et3N < Et2NH
(i) LAH (ii) H / heat
(c) PhNH2 < NH3 < Et2NH < Et3N ClCOCH2CH2CH2NH2 (X)
(iii) NaOH
(d) Et3N < PhNH2 < Et2NH < NH3
H
C2H5 ONa
61. Urea (A) + Diethyl malonate (B)
110°C N
Which is correct about the basic strength of A and B?
(a) N—CH3 (b) CH3 CH3
(a) (pKb)A = (pKb)B (b) (pKb)A > (pKb)B
(c) (pKb)A < (pKb)B (d) (pKb)A ; (pKb)B H
N CH3
62. Identify (X) in the following reaction
(c) (d)
N—H
OH C2 H5 OH
+ NH 2NHCONH 2 (X)
75°C, 8 hr 66. Assertion: After acylation of aniline, benzene nucleus
COCH3
becomes more reactive towards electrophile.
ONHNHCONH2 Reason: After acylation electron density inside benzene
(a) ring decreases.
COCH3
(a) Assertion is true, Reason is true: Reason is a correct
OH explanation for Assertion.
(b) (b) Assertion is true, Reason is true: Reason is not a
C== NHNCONH2
correct explanation for Assertion.
CH3 (c) Assertion is true, Reason is false.
(d) Assertion is false, Reason is true.
AMINES 423

P2 O5 –
67. CH2—CH2 + NH3 Product, Product of this N2+Cl Cl
D
O Conc. HCl + N2
72. + Cu2Cl2 ¾¾¾¾
¾®
2 : 1
reaction will be Chlorobenzene

O Above reaction is known as:


(a) (b) (a) Strecker's reaction
N N (b) Sandmeyer's reaction
H H (c) Wohl-Ziegler reaction
(d) Stephen's reaction
O
73. Which of the following reagents will convert
(c) (d)
N p-methylbenzenediazonium chloride into p-cresol?
N
(a) Cu powder
OH CH2OH (b) H2O
(c) H3PO2
Topic 5: Methods of Preparation of Diazonium Salts
(d) C6H5OH
68. Diazonium salt is obtained when aniline reacts with : 74. In the chemical reactions,
(a) cold NaOH
NH2
(b) NaNO2 and HCl (0–5°C)
(c) SnCl2 at 10°C NaNO
2 4 HBF
¾¾¾¾® A ¾¾® B
HCl, 278 K
(d) N2O at (0 – 5°C)
69. In the diazotization of arylamines with sodium nitrite and the compounds ‘A’ and ‘B’ respectively are
hydrochloric acid, an excess of hydrochloric acid is used (a) nitrobenzene and fluorobenzene
primarily to (b) phenol and benzene
(a) supress the concentration of free aniline available (c) benzene diazonium chloride and fluorobenzene
for coupling (d) nitrobenzene and chlorobenzene
(b) supress hydrolysis of phenol 75. Which of the following statements are correct ?
(c) ensure a stoichiometric amount of nitrous acid (i) In Sandmeyer reaction nucleophiles like Cl–, Br– and
(d) neutralise the base liberated CN– are indroduced in benzene ring in the presence
of Cu+ ion
Topic 6: Properties of Diazonium Salts (ii) In Gattermann reaction nucleophiles are introduced
70. Azo dye is prepared by the coupling of phenol and in benzene ring in the presence of copper powder and
HCl.
(a) benzene diazonium chloride
(iii) The yield in Gattermann reaction is found to be better
(b) o-nitroaniline
than Sandmayer reaction.
(c) benzoic acid (a) (i) and (ii) (b) (i), (ii) and (iii)
(d) chlorobenzene (c) (ii) and (iii) (d) (i) and (iii)
71. In the reaction sequence 76. Match the columns
Column-I Column-II
NH2
(A) ArN +2 Cl- ¾¾
® ArOH (p) HBF4 / NaNO 2
NaNO 2 ,HCl CuCN LiAlH 4
¾¾¾¾¾
0° C
¾ ® A ¾¾¾¾
® B ¾¾¾¾
® C, (B) ArN +2 Cl- ¾¾
® ArNO 2 (q) H2O
(C) ArN+2 Cl- ¾¾
® ArH (r) HBF4
the product ‘C’ is: + -
(D) ArN 2 Cl ¾¾® ArF (s) CH3CH2OH
(a) benzonitrile (a) A – (q), B – (p), C – (s), D – (r)
(b) benzaldehyde
(b) A – (s), B – (p), C – (q), D – (r)
(c) benzoic acid
(c) A – (q), B – (s), C – (p), D – (r)
(d) benzylamine
(d) A – (q), B – (s), C – (r), D – (p)
EBD_8350
424 CHEMISTRY

77. Match the columns 80. In a reaction of aniline-A coloured product C was obtained.
Column-I Column-II CH3
(A) Benzene sulphonyl (p) Zwitter ion –N
NH2
chloride CH
NaNO 3
(B) Sulphanilic acid (q) Hinsberg reagent ¾¾¾®
2
B ¾¾¾¾¾¾¾® C
HCl Cold
(C) Alkyl diazonium salts (r) Dyes
(D) Aryl diazonium salts (s) Conversion to alcohols A
(a) A – (s), B – (q), C – (r), D – (p) The structure of C would be :
(b) A – (q), B – (p), C – (s), D – (r)
(c) A – (r), B – (s), C – (p), D – (q) (a) – N = N – CH2– N –
(d) A – (s), B – (p), C – (r), D – (q)
78. The reagents that can be used to convert CH3
benzenediazonium chloride to benzene are _________.
CH3 CH3
(i) SnCl2/HCl (ii) CH3CH2OH
(iii) H3PO2 (iv) LiAlH4 –N=N–
(a) (i) and (ii) (b) (ii) and (iii) (b)
(c) (iii) and (iv) (d) (i) and (iii)
79. Nitration of nitrobenzence is carried out product is reduced CH3
with Fe/HCl, product so formed on reaction with HNO2 – NH – NH – –N
and than with H2O, forms (c) CH3
(a) 1, 3–dihydroxybenzene
(b) 3–nitrophenol CH3
(c) 2– nitrophenol (d) —N == N— —N
CH3
(d) 1, 2–dihydroxybenzene

NCERT Exemplar MCQs NH2


1. Which of the following is a 3° amine?
(c) (d) CH3NH2
(a) 1-methylcyclohexylamine
(b) Triethylamine 5. Benzylamine may by alkylated as shown in the following
(c) tert-butylamine equation?
(d) N-methylaniline C6H5CH2NH2 + R — X —® C6H5CH2NHR
2. The correct IUPAC name for CH2 = CHCH2NHCH3 is Which of the following alkyl halides is best suited for this
reaction through SN1 mechanism?
(a) allyl methylamine
(a) CH3 Br (b) C6 H5 Br
(b) 2-amino-4-pentene
(c) C6 H5CH2Br (d) C2 H5 Br
(c) 4-aminopent-1ene
6. Which of the following reagents would not be a good choice
(d) N-methylprop-2-en-1-amine for reducing an aryl nitro compound to an amine?
3. Amongst the following, the strongest base in aqueous (a) H2(excess) / Pt (b) LiAlH4 in ether
medium is ............... . (c) Fe and HCl (d) Sn and HCl
(a) CH3NH2 (b) NCCH2NH2 7. In order to prepare a 1° amine from an alkyl halide with
(c) (CH3)2NH (d) C6H5NHCH3 simultaneous addition of one CH2 group in the carbon
chain, the reagent used as a source of nitrogen is .......... .
4. Which of the following is the weakest Bronsted base?
(a) sodium amide, NaNH2
NH2 (b) sodium azide, NaN3
(a) (b) N—H (c) potassium cyanide, KCN
(d) potassium phthalimide, C6H4(CO)2N–K+
AMINES 425

8. The source of nitrogen in Gabriel synthesis of amines 17. Reduction of aromatic nitro compounds using Fe and HCl
is ............ . gives ............. .
(a) sodium azide, NaN3 (a) aromatic oxime (b) aromatic hydrocarbon
(b) sodium nitrite, NaNO2
(c) aromatic primary amine (d) aromatic amide
(c) potassium cyanide, KCN
18. The most reactive amine towards dilute hydrochloric acid
(d) potassium phthalimide, C6H4(CO)2N–K+
is ............. .
9. Amongst the given set of reactants, the most appropriate
for preparing 2° amine is ............ . CH3
(a) CH3 — NH2 (b) NH
(a) 2° R – Br + NH3 CH3
(b) 2° R – Br + NaCN followed by H2/Pt
(c) 1° R – NH2 + RCHO followed by H2/Pt NH2
CH 3
(d) 1° R – Br(2 mol) + potassium phthalimide followed by (c) N – CH3 (d)
H3O+/heat CH 3
10. The best reagent for converting 2-phenylpropanamide into
2-phenylpropanamine is ............ .
19. Acid anhydrides on reaction with primary amines
(a) excess H2 give ................. .
(b) Br2 in aqueous NaOH
(a) amide (b) imide
(c) iodine in the presence of red phosphorus
(c) secondary amine (d) imine
(d) LiAlH4 in ether
+
Cu/HCl
11. The best reagent for converting 2-phenylpropanamide into 20. The reaction Ar N 2Cl - ¾¾¾¾
® ArCl + N 2 + CuCl is
1-phenylethanamine is ............... .
named as ........... .
(a) excess H2 / Pt (b) NaOH / Br 2
(a) Sandmeyer reaction (b) Gattermann reaction
(c) NaBH4 / methanol (d) LiAlH4 / ether
12. Hoffmann bromamide degradation reaction is shown (c) Claisen reaction (d) Carbylamine reaction
by ............. . 21. Best method for preparing primary amines from alkyl halides
(a) ArNH2 (b) ArCONH2 without changing the number of carbon atoms in the chain
(c) ArNO2 (d) ArCH2NH2 is
13. The correct increasing order of basic strength for the (a) Hoffmann bromamide reaction
following compounds is .............. . (b) Gabriel phthalimide synthesis
NH2 NH2 NH2 (c) Sandmeyer reaction
(d) reaction with NH3
22. Which of the following compound will not undergo azo
coupling reaction with benzene diazonium chloride?
NO2 CH 3
(I) (II) (III) (a) Aniline (b) Phenol
(c) Anisole (d) Nitrobenzene
(a) II < III < I (b) III < I < II
(c) III < II < I (d) II < I < III 23. Which of the following compounds is the weakest
14. Methylamine reacts with HNO2 to form ........... . Bronsted base?

(a) CH3 – O – N = O (b) CH3 – O – CH3 NH2 NH2


(c) CH3OH (d) CH3CHO
15. The gas evolved when methylamine reacts with nitrous (a) (b)
acid is ................... .
(a) NH3 (b) N2 (c) H2 (d) C2H6 OH OH
16. In the nitration of benzene using a mixture of conc. H2SO4
and conc. HNO3, the species which initiates the reaction
is ............ . (c) (d)

(a) NO2 (b) NO+ (c) NO+2 (d) NO -2


EBD_8350
426 CHEMISTRY

24. Among the following amines, the strongest Bronsted base


is ............. . (d) N=N NH2
NH2 28. Which of the following will be most stable diazonium salt
RN2+X– ? [AIPMT 2014, C]
(a) (b) NH3 (a) CH3 N2+X– (b) C6H5N2+X–
(c) CH3CH2N2+X– (d) C6H5CH2N2+X–
H H 29. On heating an aliphatic primary amine with chloroform and
ethanolic potassium hydroxide, the organic compound
N N formed is: [JEE M 2014, A]
(a) an alkanol (b) an alkanediol
(c) (d)
(c) an alkyl cyanide (d) an alkyl isocyanide
30. Considering the basic strength of amines in aqueous
25. The correct decreasing order of basic strength of the solution, which one has the smallest pKb value?
following species is ........... . H2O, NH3, OH–, NH -2 [JEE M 2014, S]
(a) (CH3)2NH (b) CH3NH2
(a) NH 2– > OH – > NH3 > H 2 O
(c) (CH3)3N (d) C6H5NH2
(b) OH - > NH 2- > H 2O > NH3 31. Method by which Aniline cannot be prepared is:
[AIPMT 2015 RS]
(c) NH3 > H 2O > NH -2 > OH - (a) hydrolysis of phenylisocyanide with acidic solution
(b) degradation of benzamide with bromine in alkaline
(d) H 2 O > NH3 > OH - > NH -2 solution
(c) reduction of nitrobenzene with H2/Pd in ethanol
26. Which of the following should be most volatile?
(d) potassium salt of phthalimide treated with
I. CH3CH2CH2NH2 II. (CH3)3N chlorobenzene followed by hydrolysis with aqueous
NaOH solution.
III. CH3CH2 IV. CH3CH2CH3
NH 32. The following reaction
CH3
NH2
(a) II (b) IV (c) I (d) III + Cl ¾¾®

Past Year MCQs O NH


27. In the following reaction, the product (A)
O
[AIPMT 2014, A]
is known by the name : [AIPMT 2015 RS, A]
+ (a) Friedel-Craft's reaction (b) Perkin's reaction
NºNCl

NH2
(c) Acetylation reaction (d) Schotten-Baumen
33. The electrolytic reduction of nitrobenzene in strongly
+ H+ acidic medium produces : [AIPMT 2015, A]
¾¾® [A] ,[A] is :
Yellow dye (a) Azoxybenzene (b) Azobenzene
(c) Aniline (d) p-Aminophenol
34. The number of structural isomers possible from the
(a) N=N–NH molecular formula C3H9N is : [AIPMT 2015 RS, C]
(a) 4 (b) 5 (c) 2 (d) 3
NH2 35. In the reaction [JEE M 2015, A]
NH2
(b) N=N
NaNO2 /HCl CuCN/KCN
¾¾¾¾¾
0–5°C
® D ¾¾¾¾¾D
® E + N2
NH2

(c) CH3
N=N
the product E is :
AMINES 427

CH3 40. Which of the following compounds will form significant


CN
amount of meta product during mono-nitration reaction ?
[JEE M 2017, C]
(a) (b)
OH OCOCH3
CH3

COOH (a) (b)

(c)
NH2 NHCOCH3
CH3
(c) (d)
(d) H3C CH3

36. The correct statement regarding the basicity of arylamines 41. The correct order of the basic strength of methyl
is [NEET 2016, C]
substituted amines in aqueous solution is :
(a) Arylamines are generally less basic than alkylamines
because the nitrogen lone-pair electrons are [NEET 2019 A]
delocalized by interaction with the aromatic ring p (a) (CH3)2NH > CH3NH2 > (CH3)3N
electron system. (b) (CH3)3N > CH3NH2 > (CH3)2NH
(b) Arylamines are generally more basic than alkylamines
(c) (CH3)3N > (CH3)2NH > CH3NH2
because the nitrogen lone-pair electrons are not
delocalized by interaction with the aromatic ring p (d) CH3NH2 > (CH3)2NH>(CH3)3N
electron system. 42. Arrange the following amines in the decreasing order of
(c) Arylamines are generally more basic than alkylamines
basicity : [JEE M 2019 A]
because of aryl group.
(d) Arylamines are generally more basic than alkylamines,
because the nitrongen atom in arylamines is
sp-hybridized.
37. In the Hoffmann bromamide degradation reaction, the N
N N
number of moles of NaOH and Br 2 used per mole of amine H
produced are : [JEE M 2016, C] H
I II III
(a) Two moles of NaOH and two moles of Br 2.
(b) Four moles of NaOH and one mole of Br2. (a) I > II > III (b) III > I > II
(c) One mole of NaOH and one mole of Br 2. (c) III > II > I (d) I > III > II
(d) Four moles of NaOH and two moles of Br 2. 43. Aniline dissolved in dilute HCl is reacted with sodium
38. Which of the following reactions is appropriate for nitrate at 0 °C. This solution was added dropwise to a
converting acetamide to methanamine ? [NEET 2017, S]
solution containing equimolar mixture of aniline and
(a) Hoffmann Bromamide reaction
(b) Stephens reaction phenol in dil. HCl. The structure of the major product is:
(c) Gabriels phthalimide synthesis [JEE M 2019 A]
(d) Carbylamine reaction
39. The correct increasing order of basic strength for the (a)
following compounds is : [NEET 2017, C]

NH2 NH2
NH2 (b)
(I) (II) (III)
(c)
NO2 CH3
(a) III < I < II (b) III < II < I
(c) II < I < III (d) II < III < I (d)
EBD_8350
428 CHEMISTRY

44. The organic compound that gives following qualitative 46. The increasing order of pKb for the following compounds
analysis is: [JEE M 2019 S] will be: [JEE M 2020, A]
Test Inference
(a) Dil. HCl Insoluble
(b) NaOH solution soluble NH 2 – CH = NH, CH 3 NHCH3
(A) (C)
(c) Br2/water Decolourization
OH NH2
(a) (b) (B)

(a) (B) < (C) < (A) (b) (A) < (B) < (C)
NH2 OH
(c) (C) < (A) < (B) (d) (B) < (A) < (C)
(c) (d)
47. Consider the following reaction: [JEE M 2020, S]
45. Which of the following amine will give the carbylamine
test? [NEET 2020, C]
+
NHCH3 N(CH3)2

(a) (b)

The product ‘X’ is used:


NHC2H5 NH3 (a) in protein estimation as an alternative to ninhydrin
(b) in acid base titration as an indicator
(c) (d) (c) as food grade colourant
(d) in laboratory test for phenols

1. Consider the following reactions. 3. Which of the following imine is least stable?
KBrO
(1) EtCONH2 Product NH
HN
NaN3 / D
(2) CH3CH2COCl Product (a) (b)
H2 O
NH HN
LiAlH4
(3) AcNH2 Product
(i) NH3 NH NH
(4) CH3CHO Product
(ii) LiAlH4
The reaction in which ethyl amine forms as a product (c) (d)
is / are
(a) 1, 3 and 4 (b) 1, 2 and 4 NH NH
(c) 1 and 3 (d) 1, 2, 3 and 4
4. Introduction of a methyl group in ammonia markedly increases
2. Which among the following can not be prepared by the basic strength of ammonia in aq. solution, introduction
Gabriel-Phthalimide reaction? of the second methyl group increases only marginally the
CH3 CH2—NH2 basic strength of methyl amine in water. This is due to
(a) CH—NH2 (b) (a) different type of hybridisation in the two amines
CH3
(b) protonated dimethyl amine is more solvated than
methyl amine
NH2
(c) protonated dimethyl amine is more solvated than
CH2NH2
the protonated methyl amine
(c) (d) CH3
(d) protonated dimethyl amine is less stable than the
CH3
protonated methyl amine
AMINES 429

5. The correct order of decreasing basic character of the


three aliphatic primary amines is CH3

NH 2 NH 2 NH2 (a) CH3—C—NH2 , CH3 — N — CH3


I II III
(a) I > II > III (b) III > II > I CH3 Et
(c) I > II » III (d) I = II º III
6. The correct order of increasing basicity is CH3
NH2
NH O (b) , CH3—C—NH2
|| ||
CH 3CH 2 NH 2 CH 3 C NH 2 CH 3 C NH 2
I II III
CH3

(a) II < III < I (b) I » III < II


CH3
(c) I < II < III (d) III < I < II
7. Mitomycin C is an anticancer agent and used to treat (c) CH3CH—CH2NH2 , CH2NH2
stomach and colour couch. Its structure is
NH2 Et
2
O CH2OCONH2 (d) , CH3—CH—NH2
1
H2N OMe
V
(1) SOCl2 OBr
N 3 10. [X] [Y] [Z]
4 NH (2) NH3 + C2 H5 OH
O NaNO2
ortho cresol [X] will be
HCl
Which is correct about mitomycin C?
(a) o-Toluic acid (b) o-Chloro toluene
(a) It contains 5 chiral carbon atoms
(c) o-Bromo benzoic acid (d) o-nitro benzoic acid
(b) 3rd nitrogen is highly basic
O
(c) It contains 3 chiral carbon atoms ||
(i) OD - / Br
(d) 4th nitrogen is highly basic 11. (CH 3 )3 C - C - NH 2 ¾¾¾¾¾¾

Product P is
(ii)D2O
8. Consider the following amines.
(a) (CH3)3 CNH2 (b) (CH3)3 CNHD
Et CH 3 (c) (CH3)3CND2 (d) no reaction
12. Methyl ethyl propyl amine forms non super imposable
(1) CH3—C—Pr (2) CD 3CHCH2NH 2
mirror images but it does not show optical activity because
NH2 (a) of rapid flipping
(b) of absence of asymmetric nitrogen
CH2NH2 (c) of presence of plane of symmetry
(3) CHD2—CH2—CHDNH2 (4) (d) Amines are basic in nature

Amine/s which will exhibit optical isomerism after the (i) CH3I
treatment with NaNO2 + H2SO4 is / are 13. ¾¾¾¾¾® Product P is
N CH3 (ii) Ag 2O, heat
(a) 1, 2 (b) 1, 3 H
(c) 1, 3, 4 (d) Only 1
9.
CH3
NaNO2
(A) (C) –ve victor maeyer test (a) (b) N
HCl
N(CH3)2 CH3 CH3
+ve mustard oil test
(B) (D) Blue colour in victor maeyer test
NaNO2 CH3
HCl (c) N (d) None
CH3 CH3
(A) and (B) are respectively.
EBD_8350
430 CHEMISTRY

14. Which product will not form when primary amine reacts O
with sodium nitrite and HCl?
(a) Alcohol (b) Alkylhalide (b) NH
(c) Alkene (d) Chloramine O
15. Identify [Y] in the following reaction OH
O
(c) NH
O + NH3 [Y] O

O OH
O NH
(d)

(a) O O
NH OH

ANSWER KEY
Exercise 1 : NCERT Based Topic-wise MCQs
1 (a) 9 (c) 17 (b) 25 (c) 33 (a) 41 (c) 49 (b) 57 (c) 65 (c) 73 (b)
2 (b) 10 (c) 18 (c) 26 (d) 34 (c) 42 (b) 50 (d) 58 (a) 66 (d) 74 (c)
3 (c) 11 (c) 19 (a) 27 (b) 35 (b) 43 (a) 51 (b) 59 (c) 67 (b) 75 (a)
4 (c) 12 (b) 20 (a) 28 (a) 36 (c) 44 (d) 52 (a) 60 (b) 68 (b) 76 (a)
5 (d) 13 (d) 21 (b) 29 (a) 37 (c) 45 (a) 53 (a) 61 (c) 69 (a) 77 (b)
6 (b) 14 (c) 22 (d) 30 (a) 38 (a) 46 (d) 54 (b) 62 (b) 70 (a) 78 (b)
7 (a) 15 (d) 23 (b) 31 (a) 39 (b) 47 (d) 55 (b) 63 (a) 71 (d) 79 (b)
8 (c) 16 (d) 24 (b) 32 (a) 40 (a) 48 (a) 56 (b) 64 (d) 72 (b) 80 (d)
Exercise 2 : NCERT Exemplar & Past Year MCQs
1 (b) 6 (b) 11 (b) 16 (c) 21 (b) 26 (b) 31 (d) 36 (a) 41 (a) 46 (d)
2 (d) 7 (c) 12 (b) 17 (c) 22 (d) 27 (d) 32 (d) 37 (b) 42 (b) 47 (b)
3 (c) 8 (d) 13 (d) 18 (b) 23 (c) 28 (b) 33 (d) 38 (a) 43 (c)
4 (a) 9 (c) 14 (c) 19 (a) 24 (d) 29 (d) 34 (a) 39 (c) 44 (a)
5 (c) 10 (d) 15 (b) 20 (b) 25 (a) 30 (a) 35 (a) 40 (c) 45 (d)
Exercise 3 : Problem Solving Skill Enhancer MCQs
1 (d) 3 (d) 5 (a) 7 (b) 9 (c) 11 (c) 13 (b) 15 (d)
2 (d) 4 (d) 6 (d) 8 (b) 10 (a) 12 (a) 14 (d)
28 Biomolecules

Trend Buster NEET & JEE Main

Number of Questions from 2020-15 9 8 Minimum one Qns has been asked in
Weightage 2.8% 4.6% NEET & JEE from this chapter every year.

The most Important Concepts that Cover Maximum number of Questions asked in past 6 years.

Carbohydraes 3 3
Proteins 5 3
Vitamins — 2

Less Important Concepts that Cover 1 or 2 Questions asked in past 6 years.

Nucleic Acids 1 —
Proteins & Hormones — —

NEET JEE

2020 Vitamins / Carbohydrates / Deficiency of Vitamins / 2 Easy 1 Easy


Proteins hydrolysis of sucrose /
Amino acids
2019 Carbohydrates / Proteins Str. & properties of Sucrose / 1 Easy 2 Average
Non essential Amino acids &
structure of Amino acids
2018 Carbohydrates / Proteins Properties of Glucose & 2 Easy / 2 Average
Disaccharides / Zwitter ion & Average
str. of Amino acids
2017 Carbohydrates / Proteins Properties of different 1 Difficult 1 Difficult
Proteins / Properties of
Carbohydrates
2016 Carbohydrates / Proteins / Reducing & non reducing 3 Easy 1 Average
Nucleic acids Sugars / Structure of Amino
acids / Composition of
Nucleic acids
2015 Vitamins Solubility of Vitamins — — 1 Easy
EBD_8350
432 CHEMISTRY
BIOMOLECULES 433
EBD_8350
434 CHEMISTRY

Problem Solving Tips/ Tricks/ Points to Remember

4 Sucrose C12H22O11 • It is important to note that the replacement of just


It is non reducing sugar. It has glycosidic linkage between one amino acid in the sequence of a protein destroys
C1 of a-glucose and C2 of b-frucose. It is dextrorotatory its biological activity.
but on hydrolysis give laevorotatary invert sugar. (ii) Secondary structure :
4 Maltose C12H22O11 • The conformation which the polypeptide chains
It is reducing sugar. On hydrolysis it gives two a-D- assume as a result of H-bonding is called secondary
glucose units in which C1 of one glucose is linked to C4 structure of protein.
of another glucose. • The H-bonds are present between hydrogen of
amino group and oxygen atom of carboxylic acid
6 6 group.
CH2OH CH2OH
• This structure is of two types :
O 5 O
H 5 H H H
H H
4 1 4 1
OH H O
HO 3 2
OH H OH a-Helix b- Pleated sheet
3 2
H OH (a) a - Helix
H OH
(I) (II) • Chain is spiral.
a - D - Glucose Maltose a - D - Glucose • Intermolecular H-bonds are present.
e.g.: Myosin, Keratin etc.
4 Lactose C12H22O11
(b) b -Pleated sheet
It is commonly known as milk sugar due to presence in
• Structure of protein is not arranged in a sequence.
milk. The linkage is between C1 of b-D-galactose and C4
of b-D-glucose. It is also reducing sugar. • Polypeptide chains are parallel to each other.
4 Monosaccharides which differ in configuration at C1 in • Intermolecular H-bonds hold together the
aldoses and C2 in ketoses are called anomers. Thus a-D neighbouring polypeptide chains. e.g.: Silk fibres.
glucose and b-D glucose are anomers and so are a-D (iii) Tertiary structure :
fructose and b-D fructose. • In this structure of proteins, atoms are highly coiled
4 Epimers: Monosaccharides differing in configuration at and form a spherical form Ex. Albumin.
a carbon other than anomeric carbon are called epimers
• Tertiary structure refers to its three dimensional
e.g. glucose and galactose differ in configuration at C4
structure, i.e., folding and bonding of the long peptide
hence called C4 epimers.
chains. Three types of bonds are responsible for
4 Osazones: Monosaccharides and reducing disaccharides tertiary structure, viz. hydrogen, ionic and
react with excess of phenyl hydrazine to form crystalline hydrophobic.
substances of the structure known as osazones. Glucose
4 Contractile proteins : Found in muscles e.g. myosin,
and Fructose give same osazone.
actin.
4 Twenty five amino acids have been obtained from the
4 Tests of Protein:
hydrolysis of proteins. Except two (proline and
hydroxyproline) all are amino acids, the exceptional two (i) Biuret : Protein solution + NaOH + dil. CuSO4
are imino acids. ® Pink or violet colour
4 Configuration of Proteins : (ii) Ninhydrin : Protein solution + Ninhydrin ® Blue
colour
(iii) Hopkin’s cole : Protein solution + Glyoxalic acid +
Primary Secondary Tertiary Quaternary
conc. H2SO4 ® Blue - violet
(i) Primary structure : (iv) Million’s : Protein solution + Millon’s reagent
• This type of structure was given by Fredric Sanger ® Pink colour..
in 1953 in insulin (of one chain). Millon’s reagent : Solution of mercuric nitrate and
• Primary structure refers to the number, nature and nitrite in nitric acid containing traces of nitrous
sequence of the amino acids in protein molecule. acid.
BIOMOLECULES 435

(v) Xanthoproteic test : Protein solution + conc HNO3 protein part of enzymes is called apoenzyme.
NaOH 4 Vitamins:
® Yellow colour ¾¾ ¾
¾® Orange colour (i) Water soluble vitamins : Vitamin B and vitamins
4 Coenzymes : Non proteinous components required for C.
the activity of certain enzymes are known as coenzymes. (ii) Fat soluble vitamins : Vitamins A, D, E and K.

Sr.No. Name of Vitamins Source Deficiency Diseases

1 Vitamin A Cod liver oil, carrots, egg, Xerophthalmia (hardening of cornea of eye)
(Retinol) butter and milk Night blindness
2 Vitamin B1 Seeds, whole grains, pulses, nuts Beri beri (loss of appetite, retarded growth)
(Thiamine)
3 Vitamin B2 Milk, egg white, liver, kidney Cheilosis (fissuring at corners of mouth and lips),
(Riboflavin) digestive disorders and burning sensation of the skin
4 Vitamin B6 Yeast, milk, egg yolk, rice, cereals Anaemia
(Pyridoxine) and grams
5 Vitamin B12 Meat, fish, egg and curd Pernicious anaemia (RBC deficiency in haemoglobin)
(Cyanocobalamine)
6 Vitamin C Citrus fruits, amla and green leafy Scurvy (bleeding gums)
(Ascorbic acid) vegetables
7 Vitamin D Exposure to sunlight, fish Rickets (bone deformities in children) and osteomalacia
(Calciferol) and egg yolk (soft bones and joint pain in adults)
8 Vitamin E Vegetable oils like wheat germ Increased fragility of RBCs and muscular weakness,
(Tocoferol) oil, cotton seed oil, sunflower antifertility
oil, etc
9 Vitamin K Green leafy vegetables Increased blood clotting time
(Phyllo-quinone)

4 Types of Hormones Nucleotides : Nucleotides consist of 5 - carbon sugar


(i) Steroid hormones : (pentose) + nitrogenous base + 1-3 phosphate groups.
(a) Estrogens and Progesterones Nucleoside : Ribose + one base unit from AGCT or U
(b) Testosterone 4 Functions of Nucleic Acids
(c) Cortisone (i) To direct the synthesis of proteins.
(ii) Amino acid hormones : (ii) To transfer the genetic information (hereditary
(a) Thyroxine (b) Adrenaline characters).
(iii) Peptide hormones : 4 Replication : A molecule of DNA can exactly duplicate
(a) Insulin (b) Glucagon to itself.
(c) Oxytocin (d) Vasopressin 4 Template : It means pattern. In the process of replication
of DNA, the parent strand serves as template.
4 Nucleic Acids
4 Codons : The nucleotide bases in RNA function in groups
Nitrogenous base : Derived from purines having two
of three (triplet) in coding amino acids. These base triplets
rings in their structure Examples are adenine (A) and
are called codons.
guanine (G).
EBD_8350
436 CHEMISTRY

Exercise 1 : NCERT Based Topic-wise MCQs


9. Reduction of glucose by HI suggest that
Topic 1: Carbohydrates
(a) presence of OH groups
1. Which of the following statements is incorrect ? (b) presence of –CHO group
(a) Maltose gives two molecules of glucose only. (c) cyclic structure of glucose
(b) Cellulose and sucrose are polysaccharides. (d) six carbon atoms are arranged in straight chain
(c) Polysaccharides are not sweet in taste. 10. Which is the least stable form of glucose ?
(a) a-D-Glucose (b) b-D-Glucose
(d) Polysaccharides are also known as non-sugars.
(c) Open chain structure (d) All are equally stable
2. Which of the following properties of glucose cannot be
11. Isomerization of glucose produces
explained by its open chain structure?
(a) galactose (b) fructose
(i) Glucose does not form hydrogen sulphite with
(c) mannose (d) allose
NaHSO3
12. The number of chiral carbon atoms present in cyclic
(ii) On oxidation with HNO3 glucose gives saccharic acid. structure a-D(+) glucose
(iii) Glucose is found to exist in two different crystalline (a) 3 (b) 4 (c) 5 (d) 6
forms which are named as a and b. 13. The a-D glucose and b-D glucose differ from each other
(a) (ii) only (b) (i) and (iii) due to difference in carbon atom with respect to its
(c) (ii) and (iii) (d) (i) and (ii) (a) conformation (b) configuration
3. Which of the following statements is incorrect regarding (c) number of OH groups (d) size of hemiacetal ring
glucose? 14. Which of the following is the sweetest sugar?
(a) It is an aldohexose. (a) Sucrose (b) Glucose
(b) It is also known as dextrose (c) Fructose (d) Maltose
(c) It is monomer of cellulose. 15. Cellulose is a polymer of
(d) It is the least abundant organic compound on earth. (a) Glucose (b) Fructose
4. Glucose gives silver mirror test with Tollen’s reagent. It (c) Ribose (d) Sucrose
shows the presence of 16. For osazone formation, the effective structural unit
(a) acidic group (b) alcoholic group necessary is
(c) ketonic group (d) aldehyde group (a) CH2OCH3 (b) CH2OH

5. The symbols D and L represent


CO CO
(a) the optical activity of compounds.

(b) the relative configuration of a particular stereoisomer.


(c) CH2OH (d) CHO
(c) the dextrorotatory nature of molecule.

(d) the levorotatory nature of molecule CHOCH3 CHOCH3


6. Glucose is found to exist in two different a and b crystalline

forms. These forms can be obtained by. 17. Carbohydrates are stored in the body as
(i) The a form of glucose is obtained by crystallisation (a) sugars (b) starch
from concentrated solution of glucose at 303 K. (c) glucose (d) glycogen
(ii) The b form of glucose is obtained by crystallisation 18. Which of the following carbohydrate does not correspond
from concentrated solution of glucose at 303 K. to the general formula Cx(H2O)y ?
(iii) The b form is obtained by crystallisation from hot (a) Glucose (b) 2-Deoxyribose
and saturated aqueous solution at 371 K. (c) Fructose (d) Arabinose
(iv) The a form is obtained by crystallisation from hot 19. Lactose is made of
and saturated aqueous solution at 371 K. (a) a-D-glucose only
(a) (i) and (iii) (b) (ii) and (iv) (b) a-D-glucose and b-D-glucose
(c) (ii) and (iii) (d) (i) only (c) a-D-galactose and b-D-glucose
7. When glucose reacts with bromine water, the main product (d) b-D-galactose and b-D-glucose
is 20. Which of the following monosaccharides are present as
(a) gluconic acid (b) glyceraldehyde five membered cyclic structure (furanose structure)?
(c) saccharic acid (d) acetic acid (i) Ribose (ii) Glucose
8. Glucose does not react with (iii) Fructose (iv) Galactose
(a) Br2/H2O (b) H2NOH (a) (i) and (ii) (b) (i) and (iii)
(c) HI (d) NaHSO3 (c) (iii) and (iv) (d) (ii) and (iii)
BIOMOLECULES 437

21. Invert sugar is (iv) Maltose is a non-reducing sugar whereas sucrose is


(a) chemically inactive form of sugar a reducing disaccharide sugar.
(b) equimolecular mixture of glucose and fructose (a) (i), (ii) and (iii) (b) (i) and (iii)
(c) mixture of glucose and sucrose (c) (ii), (iii) and (iv) (d) (iii) and (iv)
(d) a variety of cane sugar 29. Read the following statements.
22. Sucrose which is dextrorotatory in nature after hydrolysis (i) Pyran is a cyclic organic compound with one oxygen
gives glucose and fructose, among which atom and five carbon atoms.
(i) Glucose is laevorotatory and fructose is (ii) The cyclic structure of glucose is correctly
dextrorotatory. represented by Haworth strucure.
(ii) Glucose is dextrorotatory and fructose is laevorotatory (iii) Five membered cyclic structure of glucose is called
(iii) The mixture is laevorotatory. pyranose structure.
(iv) Both are dextrorotatory. Which of the following statement(s) is/are true?
(a) (i) and (iii) (b) (ii) and (iii) (a) (i) and (iii) (b) (i) and (ii)
(c) (iii) and (iv) (d) (iii) only (c) Only (iii) (d) (i), (ii) and (iii)
23. Chemically amylose is a _________ with 200–1000 30. Read the following statements and choose the correct–
a-D-(+)-glucose units held by ______ glycosidic linkage option?
(a) long unbranched chain, C1– C6. (i) Starch is a polymer of a – glucose.
(b) branched chain, C1 – C4. (ii) Starch consists of amylose and amylopectin.
(c) long unbranched chain, C1– C4. (iii) Amylose is insoluble in water.
(d) branched chain, C1– C6. (iv) Amylopectin is soluble in water.
24. Amylopectin is a ________ polymer of a-D-glucose units (a) (i) (iii) and (iv) (b) (i), (ii) and (iii)
in which chain is formed by _______ glycosidic linkage (c) (i) and (ii) (d) (iii) and (iv)
whereas branching occurs by ________ glycosidic 31. Match the columns
linkage. Column - I Column - II
(a) branched chain, C1– C6, C1– C4. (Enzymes) (Reactions)
(b) branched chain, C1– C4, C1– C6.
(A) Invertase (p) Decomposition of urea into
(c) unbranched chain, C1– C4, C1– C6.
(d) unbranched chain, C1– C6, C1– C4. NH3 and CO2
25. Which of the following is incorrect about cellulose? (B) Maltase (q) Conversion of glucose into
(a) It is a major constituent of cell wall of plant cells. ethyl alcohol
(b) It is a branched chain disaccharide (C) Pepsin (r) Hydrolysis of maltose into
(c) It is composed of only b-D-glucose units. glucose
(d) The glycosidic linkage between two units is found (D) Urease (s) Hydrolysis of cane sugar
between C1 of one unit and C4 of next unit. (E) Zymase (t) Hydrolysis of proteins into
26. Read the following statements and choose the correct
peptides
answer?
(a) A – (s), B – (r), C – (t), D – (p), E – (q)
(i) All monosaccharides are reducing sugars.
(b) A – (r), B – (q), C – (s), D – (p), E – (t)
(ii) All monosaccharides are not reducing sugars.
(c) A – (q), B – (p), C – (r), D – (s), E – (t)
(iii) In disaccharides, if aldehydic or ketonic groups are
bonded, these are non– reducing sugars. (d) A – (s), B – (p), C – (t), D – (q), E – (r)
(iv) In disaccharides, if aldehydic or ketonic groups are 32. Which one of the following is the reagent used to identify
free, these are reducing sugars. glucose?
(a) Neutral ferric chloride
(a) (i), (iii) and (iv) (b) (ii), (iii) and (iv)
(b) Chloroform and alcoholic KOH
(c) (i) and (iv) (d) (ii) and (iv)
(c) Ammonical silver nitrate
27. Which of the following statement(s) is/are correct? (d) Sodium ethoxide
(i) Glucose is reducing sugar 33. Glucose molecule reacts with 'X' number of molecules of
(ii) Sucrose is reducing sugar phenylhydrazine to yield osazone. The value of 'X' is
(iii) Maltose is non reducing sugar (a) four (b) one
(iv) Lactose is reducing sugar (c) two (d) three
(a) (i) and (ii) only (b) (i) and (iii) only 34. Assertion : D(+)– Glucose is dextrorotatory in nature.
(c) (i) and (iv) only (d) All of these Reason : ‘D’ represents its dextrorotatory nature.
28. Which of the following statements regarding (a) Assertion is correct, reason is correct; reason is a
carbohydrates are correct? correct explanation for assertion.
(i) Lactose is the carbohydrate found in milk. (b) Assertion is correct, reason is correct; reason is not
(ii) More than 25 monosaccharides occur naturally. a correct explanation for assertion
(iii) Sucrose on hydrolysis gives one molecule each of (c) Assertion is correct, reason is incorrect
glucose and fructose. (d) Assertion is incorrect, reason is correct.
EBD_8350
438 CHEMISTRY

35. In the acetylation of glucose, which group is involved in (b) Assertion is correct, reason is correct; reason is not
the reaction a correct explanation for assertion
(a) CHO group (b) >C = O group (c) Assertion is correct, reason is incorrect
(c) alcoholic OH group (d) all of these (d) Assertion is incorrect, reason is correct.
36. Select the false statement about the cyclic glucose. 41. Sucrose in water is dextro-rotatory, [a]D= + 66.4º. When
(a) If the OH group is added to CHO group it will form boiled with dilute HCl, the solution becomes leavo-rotatory,
cyclic hemiacetal structure [a]D= –20º. In this process the sucrose molecule breaks
(b) Glucose form Six-membered ring in which –OH is at
into
C–5 position
(a) L-glucose + D-fructose
(c) Melting point of a-glucose is 423 K and of b-glucose
is 419 K (b) L-glucose + L-fructose
(c) D-glucose + D-fructose
O (d) D-glucose + L-fructose
1 1 1
H C OH H C HO C H
H 2 OH H 2 OH H 2 OH Topic 2: Proteins
(d) HO 3 H O HO 3 H HO 3 H O
H 4 OH H 4 OH H 4 OH 42. One of essential a-amino acids is
H 5 H 5 OH H 56
6 6 (a) lysine (b) serine
CH2OH CH2OH CH2OH (c) glycine (d) proline
a-D(+)-glucose a-D(+)-glucose 43. Two functional groups that are present in all amino acids are
37. Assertion : Sucrose is called an invert sugar. the
Reason : On hydrolysis, sucrose bring the change in the (a) hydroxy, amine (b) hydroxy, amide
sign of rotation from dextro (+) to laevo(–). (c) carboxyl, amino (d) carboxyl, amide
(a) Assertion is correct, reason is correct; reason is a 44. Which of the following is not an optically active amino
correct explanation for assertion. acid?
(b) Assertion is correct, reason is correct; reason is not (a) Valine (b) Glycine
a correct explanation for assertion (c) Leucine (d) Arginine
(c) Assertion is correct, reason is incorrect 45. Denaturation of proteins leads to loss of its biological
(d) Assertion is incorrect, reason is correct. activity by
38. When a-D-glucose and b-D-glucose are dissolved in water (a) Formation of amino acids
in two separate beakers I and II respectively and allowed (b) Loss of primary structure
to stand, then –
(c) Loss of both primary and secondary structures
(a) specific rotation in beaker I will decrease while in II
(d) Loss of both secondary and tertiary structures
will increase upto a constant value
46. Amino acids generally exist in the form of Zwitter ions.
(b) the specific rotation of equilibrium mixture in two
beakers will be different This means they contain
(c) the equilibrium mixture in both beakers will be (a) basic—NH2 +group and acidic —COOH group
leavorotatory (b) the basic— NH3 group and acidic —COO– group
(d) the equilibrium mixture in both beakers will contain (c) basic—NH2 and acidic —H+ group+
only cyclic form of glucose (d) basic –COO– group and acidic — N H 3 group
39. Cyclic structure of fructose resembles with 47. The structural feature which distinguishes proline from
(a) pyran (b) furan natural a-amino acids?
(a) Proline is optically inactive
(c) pyridine (d) oxiran
(b) Proline contains aromatic group
40. Assertion : b-Glycosidic linkage is present in maltose, (c) Proline is a dicarboxylic acid
CH2OH CH2OH (d) Proline is a secondary amine
H O H OH 48. Which one of the following structures represents the
H H peptide chain?
OH H O OH H
HO H OH H O
| | ||
H OH H OH
(a) - N - C - N - C - NH - C - NH -
Reason : Maltose is composed of two glucose units in || | |
which C–1 of one glucose unit is linked to C–4 of another O H
glucose unit. H H
| | | | | | | |
(a) Assertion is correct, reason is correct; reason is a (b) - N - C - C- C- C- N - C- C- C-
correct explanation for assertion. || | | | | | |
O
BIOMOLECULES 439

60. Primary structure of a protein is


H H H O
| | | | | | || (a) sequence in which a-amino acids are linked to one
(c) - N - C- C - N - C - C - N - C - C - another
| || | || | (b) sequence in which amino acids of one polypeptide
O O chain are joined to other chain
H O H (c) the folding patterns of polypeptide chains
| | | || | | | | | (d) the pattern in which the polypeptide chains are
(d) - N - C- C- C - N - C- C- N- C - C- C- arranged
| | | | | || | |
H O 61. The helical structure of protein is stabilized by
49. Simplest proteins has one peptide linkage. It is (a) dipeptide bonds (b) hydrogen bonds
(a) tripeptide (b) dipeptide (c) ether bonds (d) peptide bonds
(c) tetrapeptide (d) oligopeptide 62. Which of the following statements is incorrect?
50. Proteins are polypeptides of (a) In a-helix structure a polypeptide chain forms all
(a) b-amino acids (b) a-hydroxy acids possible hydrogen bonds by twisting into a right
(c) D-a-amino acids (d) L-a-amino acids handed screw.
51. Globular proteins are present in (b) In b-structure of proteins all peptide chains are
(a) blood (b) hair stretched out to nearly maximum extension.
(c) nails (d) all of these (c) During denaturation 1° and 2° structures are
52. In fibrous proteins, polypeptide chains are held together destroyed but 3° structure remains intact.
by (d) All the above statements are incorrect.
(a) van der waals forces 63. Which of the following indicates the order in which amino
(b) electrostatic forces of attraction acids are linked together in a protein ?
(c) hydrogen bonds (a) Primary structure (b) Secondary structure
(d) covalent bonds (c) Tertiary structure (d) Quaternary structure
53. Which of the following is not a fibrous protein?
64. Which of the following statement is not true about
(a) Keratin (b) Myosin
secondary structure of protein ?
(c) Insulin (d) Both (a) and (b)
(a) The alpha helix, beta pleated sheet and beta turns are
54. A polypeptide with more than hundred amino acid
examples of secondary structure of protein.
residues, having molecular mass higher than 10,000 u is
called_____. (b) The ability of peptide bonds to form intramolecular
(a) nucleic acid (b) hormone hydrogen bonds is important to secondary structure.
(c) protein (d) enzyme (c) The steric influence of amino acid residues is
55. An insulin is a ______ which contains _____ amino acids. important to secondary structure.
(a) protein, 74 (b) protein, 51 (d) The hydrophilic/ hydrophobic character of amino acid
(c) peptide hormone, 51 (d) peptide hormone, 74 residues is important to secondary structure.
56. Which of the following is an example of globular protein? 65. Which of the following terms indicates to the arrangement
(a) Glycine (b) Albumin of different protein subunits in a multiprotein complex ?
(c) Alanine (d) Both (a) and (b) (a) Primary structure (b) Secondary structure
57. Which of the following is not a characteristics of fibrous (c) Tertiary structure (d) Quaternary structure
proteins? 66. Secondary structure of protein is mainly governed by
(a) In the fibrous proteins, polypeptide chains are held (a) hydrogen bonds (b) covalent bonds
together by hydrogen and disulphide bonds. (c) ionic bonds (d) disulphide bonds
(b) These have fibre like structure. 67. The secondary structure of a protein refers to
(c) These are generally soluble in water.
(a) fixed configuration of the polypeptide backbone
(d) These have elongated shape.
(b) a – helical backbone
58. Which of the following statements is true about a peptide
bond (RCONHR)? (c) hydrophobic interactions
(a) It is non planar. (d) sequence of a – amino acids
(b) It is capable of forming a hydrogen bond. 68. Tertiary structure of protein arises due to
(c) The cis configuration is favoured over the trans (a) folding of polypeptide chain
configuration. (b) folding, coiling and bonding of polypeptide chain
(d) Single bond rotation is permitted between nitrogen (c) linear sequence of amino acid in polypeptide chain
and the carbonyl group. (d) denatured proteins
59. Proteins are condensation polymers of 69. Coagulation of protein is known as
(a) a-amino acids (b) b-amino acids (a) dehydration (b) decay
(c) a-hydroxy acids (d) b-hydroxy acids (c) deamination (d) denaturing
EBD_8350
440 CHEMISTRY

70. Which of the following terms refers to the overall three (b) Assertion is correct, reason is correct; reason is not
dimensional shape of a protein. a correct explanation for assertion
(a) Primary structure (c) Assertion is correct, reason is incorrect
(b) Secondary structure (d) Assertion is incorrect, reason is correct.
(c) Tertiary structure 78. The strongest form of intermolecular bonding that could
(d) Quaternary structure be formed involving the residue of the amino acid valine is
71. Which of the following indicates to ‘regions of ordered (a) ionic bond
structure within a protein’. (b) hydrogen bond
(a) Primary structure (b) Secondary structure (c) van der Waals interactions
(c) Tertiary structure (d) Quaternary structure (d) none of the above
72. The strongest form of intermolecular bonding that could 79. Which of the following molecules is capable of forming
be formed involving the residue of the amino acid serine Zwitter ion?
is. (a) NH2CH2COOH (b) CH3CH2NH2
(a) ionic bond (c) CH3CH2COOH (d) All of these
(b) hydrogen bond
(c) van der Waal’s interactions Topic 3: Enzymes
(d) None of the above 80. Which of the following is incorrect regarding enzymes?
73. Which of the following protein destroys the antigen when (a) Most of them are globular proteins.
it enters in body cell? (b) They are very specific for a particular reaction but
(a) Antibodies (b) Insulin not for a particular substrate.
(c) Chromoprotein (d) Phosphoprotein (c) They are generally named after the compound or class
74. Which among the following statements are true for of compounds upon which they work.
glycine? (d) All the above statements are incorrect.
(i) It exists in crystalline form 81. Enzymes take part in a reaction and
(ii) It is optically active (a) decrease the rate of a chemical reaction
(iii) It is soluble in water (b) increase the rate of a chemical reaction
(iv) It can form Zwitter ions (c) both (a) and (b)
(a) (i), (ii) and (iii) (b) (i), (ii) and (iv) (d) None of these
(c) (i), (iii) and (iv) (d) (ii), (iii) and (iv) 82. The enzyme which hydrolyses triglycerides to fatty acids
and glycerol is called
75. Which of the following statements are correct?
(a) Maltase (b) Lipase
(i) Proteins on hydrolysis gives only a-amino acids.
(c) Zymase (d) Pepsin
(ii) Gln stands for glutamic acid.
83. Which one of the following, statements is incorrect about
(iii) Amino acids with equal number of amino and carboxyl
enzyme catalysis?
groups are neutral.
(a) Enzymes are mostly proteinous in nature.
(iv) All naturally occuring a-amino acids are optically
(b) Enzyme action is specific.
active.
(c) Enzymes are denaturated by ultraviolet rays and at
(a) (i) and (iii) (b) (i), (ii) and (iv)
high temperature.
(c) (iii) and (iv) (d) (ii), (iii) and (iv)
(d) Enzymes are least reactive at optimum temperature.
76. Which of the statements about "Denaturation" given 84. Insulin production and its action in human body are
below are correct ? responsible for the level of diabetes. This compound belongs
(i) Denaturation of proteins causes loss of secondary to which of the following categories?
and tertiary structures of the proteins. (a) An enzyme (b) A hormone
(ii) Denaturation leads to the conversion of double strand (c) A co-enzyme (d) An antibiotic
of DNA into single strand 85. Enzymes are essential as biocatalysts. They function in
(iii) Denaturation affects primary strucrture which gets (a) aqueous medium, temp = 30–35ºC; pH=7
distorted (b) organic medium
(a) (ii) and (iii) (b) (i) and (iii) (c) aqueous medium under extreme pH conditions
(c) (i) and (ii) (d) (i), (ii) and (iii) (d) None of these
77. Assertion : At isoelectric point, the amino group do not 86. Which of the following statements is incorrect?
migrate under the influence of electric field. (a) Enzymes are organic catalysts
Reason : At isoelectric point, amino acid exists as a (b) Enzymes have a very large turnover number
zwitterion. (c) Enzymes action is specific
(a) Assertion is correct, reason is correct; reason is a (d) Enzymes always require a coenzyme in their catalytic
correct explanation for assertion. action.
BIOMOLECULES 441

87. Of the following statements about enzymes which ones 96. Match the columns
are true? Column - I Column - II
(i) Enzymes lack in nucleophilic groups (A) Vitamin A (p) Scurvy
(ii) Enzymes are highly specific both in binding chiral (B) Vitamin B12 (q) Hemorrhagic condition
substrates and in catalysing their reactions (C) Vitamin C (r) Sterility
(iii) Enzymes catalyse chemical reactions by lowering the (D) Vitamin E (s) Xerophthalmia
energy of activation (E) Vitamin K (t) Pernicious anaemia
(a) A – (t), B – (s), C – (p), D – (r), E – (r)
(iv) Pepsin is a proteolytic enzyme
(b) A – (s), B – (t), C – (p), D – (q), E – (r)
(a) (i) and (iv) (b) (i) and (iii)
(c) A – (s), B – (t), C – (p), D – (r), E – (q)
(c) (ii), (iii) and (iv) (d) (i)
(d) A – (t), B – (s), C – (p), D – (r), E – (q)
97. Assertion : Vitamin D cannot be stored in our body
Topic 4: Vitamins
Reason : Vitamin D is fat soluble vitamin and can not be
88. Among the following vitamins the one whose deficiency excreted from the body with urine.
causes rickets (bone deficiency) is : (a) Assertion is correct, reason is correct; reason is a
(a) Vitamin A (b) Vitamin B correct explanation for assertion.
(c) Vitamin D (d) Vitamin C (b) Assertion is correct, reason is correct; reason is not
89. Anaemia is caused by the deficiency of vitamin a correct explanation for assertion
(a) B6 (b) B1 (c) Assertion is correct, reason is incorrect
(c) B2 (d) B12 (d) Assertion is incorrect, reason is correct.
98. Vitamin C must be supplied regularly in diet because
90. Vegetable oils like wheat gram oil, sunflower oil etc. are the
good source of (a) it is water soluble hence excreted in urine and can’t
(a) vitamin K (b) vitamin E be stored in the body
(c) vitamin D (d) vitamin A (b) it is fat soluble hence stored in the body and cannot
91. Which is a fat soluble vitamin? be used on regular basis
(a) Vitamin A (b) Vitamin B6 (c) it is required in a large amount by the body hence
(c) Vitamin C (d) Vitamin B2 supplied regularly
92. Vitamin B2, a water soluble vitamin, is also known as (d) it is water soluble hence used by the body on daily
(a) ascorbic acid (b) riboflavin basis and is to be supplied regularly.
(c) thiamine (d) pyridoxine
93. Which of the following statements about vitamin B12 is Topic 5: Nucleic Acids
incorrect ? 99. The presence or absence of hydroxyl group on which
(a) It has a cobalt atom carbon atom of sugar differentiates RNA and DNA?
(b) It also occurs in plants (a) 1st (b) 2nd (c) 3rd (d) 4th
(c) It is also present in rain water 100. The couplings between base units of DNA is through :
(d) It is needed for human body in very small amounts (a) Hydrogen bonding (b) Electrostatic bonding
94. Which of the following statements are correct ? (c) Covalent bonding (d) van der Waals forces
(i) Vitamins A, D, E and K are insoluble in water. 101. Which of the following is correct about H-bonding in
(ii) Vitamins A, D, E and K are stored in liver and adipose nucleotide?
tissues. (a) A --- A and T --- T (b) G --- T and A --- C
(iii) Vitamin B and vitamin C are water soluble. (c) A --- G and T --- C (d) A --- T and G --- C
(iv) Water soluble vitamins should not be supplied 102. Structure of guanine is
regularly in diet.
NH2 O
(a) (i), (ii) and (iv) (b) (i), (ii) and (iii)
(c) (i) and (iv) (d) (ii) and (iv)
HC N N C NH
95. Match the columns
Column - I Column - II (a) HC C (b) HC
C C
(A) Vitamin B6 (p) Fat soluble N O N N NH2
(B) Vitamin K (q) Xerophthalmia H H
(C) Vitamin D (r) Convulsions O
(D) Vitamin A (s) Delayed blood clotting H3C
(a) A – (p,q), B – (p,s), C – (p), D – (r) (c) C N H (d) None
(b) A – (r), B – (p,s), C – (p), D – (p, q)
HC C
(c) A – (p,s), B – (r), C – (p), D – (p,q) N O
(d) A – (r), B – (p,s), C – (p,q), D – (p) H
EBD_8350
442 CHEMISTRY

103. In DNA the linkages between different nitrogenous bases 113. Which of the following statements regarding DNA
are : fingerprinting is incorrect?
(a) peptide linkage (b) phosphate linkage (a) It is used in forensic laboratories for identification of
(c) H-bonding (d) glycosidic linkage criminals.
104. DNA multiplication is called as (b) It cannot be altered by surgery.
(a) translation (b) transduction (c) It is different for every cell and cannot be altered by
(c) transcription (d) replication any known treatment.
105. Chromosomes are made from (d) It is used to determine paternity of an individual.
(a) proteins 114. Which of the following statement(s) is/are correct?
(b) nucleic acids (i) Information regarding the sequence of nucleotides in
(c) proteins and nucleic acids the chain of a nucleic acid is called its primary
(d) carbohydrates and nucleic acids structure.
106. The double helical structure of DNA was proposed by (ii) In secondary structure of DNA adenine forms
(a) Watson and Crick (b) Meichers hydrogen bonds with guanine whereas cytosine forms
(c) Emil Fischer (d) Khorana hydrogen bonds with thymine.
107. The chemical change in DNA molecule that could lead to (iii) RNA molecules are of three types m-RNA, r-RNA and
synthesis of protein with an altered amino acid sequence t-RNA and they all perform different functions.
is called (a) (ii) only (b) (i) and (iii)
(a) replication (b) lipid formation (c) (ii) and (iii) (d) (iii) only
(c) cellular membrane (d) mutation 115. In both DNA and RNA, heterocyclic base and phosphate
ester linkages are at –
108. The process by which synthesis of protein takes place
based on the genetic information present in m-RNA is (a) C5' and C1' respectively of the sugar molecule
called
(a) Translation (b) Transcription (b) C1' and C5' respectively of the sugar molecule
(c) Replication (d) Messenger hypothesis
(c) C '2 and C5' respectively of the sugar molecule
109. Which of the following structures represents thymine ?

O NH2 (d) C5' and C '2 respectively of the sugar molecule

HN N Topic 6: Hormones
(a) (b)
O N O N 116. Oxytocin hormone is produced by
H H (a) pituitary (b) adrenals
(c) hypothalamus (d) thyroid
NH2 OH 117. Ovulation in humans is controlled by
N N (a) FSH and LTH (b) FSH and G.H
CH3
(c) (d) (c) LTH and LH (d) FSH and LH
HO N HO N 118. Hormones of pituitary gland are
(a) some steroids and some proteins
110. When adenine is attached to ribose sugar, it is called (b) all proteins/peptides.
adenosine. To make a nucleotide from it, it would require (c) complex substances formed from proteins, steroids,
(a) oxygenation (b) addition of a base carbohydrates.
(c) addition of phosphate (d) hydrogenation (d) all steroids.
111. Which of the following is not present in a nucleotide? 119. Which of the followings is the more scientific definition of
hormone?
(a) Guanine (b) Cytosine
(a) They are extracellular messengers.
(c) Adenine (d) Tyrosine (b) They always act at distantly located target organ.
112. The function of DNA in an organism is (c) They are the products of well organized endocrine
(a) to assist in the synthesis of RNA molecule glands.
(b) to store information of heredity characteristics (d) They are non-nutrient chemicals that act as
intercellular messengers.
(c) to assist in the synthesis of proteins and 120. Ovulation and formation of corpus luteum is controlled by
polypeptides (a) ICSH (b) FSH
(d) All of these (c) thyroxine hormone (d) luteinizing hormone
BIOMOLECULES 443

1. For ‘invert sugar’, the total number of correct statement(s) 5. When the following aldohexose exists in its D-
is(are) configuration, the total number of stereoisomers in its
(Given : specific rotations of (+) -sucrose, (+)-maltose, pyranose form is :
L-(–)-glucose and L-(+) fructose in aqueous solution are CHO — CH2 — CHOH — CHOH — CHOH — CH2OH
+ 66°, +140°, –52° and +92°, respectively) 6. A tetrapeptide has —COOH group on alanine. This
(I) ‘invert sugar’ is prepared by acid catalyzed hydrolysis produces glycine (Gly), valine (Val), phenyl alanine (Phe)
and alanine (Ala), on complete hydrolysis. For this
of maltose.
tetrapeptide, find the number of possible sequences
(II) ‘invert sugar’ is an equimolar mixture of D-(+)-glucose
(primary structures) with — NH2 group attached to a chiral
and D-(-)-fructose. center.
(III) specific rotation of ‘invert sugar’ is –20°. 7. The specific optical rotations of pure a- and b-D-
(IV) on reaction with Br 2 water, ‘invert sugar’ forms mannopyranose are + 29.3° and –17.0, respectively. When
saccharic acid as one of the products. either form is dissolved in water, specific optical rotation
2. How many dipeptides are possible from two molecules of of the equilibrium mixture is found to be + 14.2°. Calculate
a typical a-amino acid ? the percentage of a anomer at equilibrium.
3. The total number of basic groups in the following form of 8. Penicillin, the common antibiotic, has following structure :
lysine is S
H
C6H5CH2CONHC
+ O
N
CH C O COOH

H2N O– Find the number of different functional groups present in


it.
4. A decapeptide (Mol. wt. 796) on complete hydrolysis gives 9. The enzyme cytochrome C, involved in oxidation-reduction
glycine (Mol. wt. 75), alanine and phenylalanine. Glycine processes in the living system, has 0.43% Fe and 1.48% S.
contributes 47.0% to the total weight of the hydrolysed Calculate the number of S atoms in the enzyme per Fe atom.
products. Calculate the number of glycine units present in 10. A strongly alkaline solution of a monoaminodicarboxylic
the decapeptide. acid contains how many basic groups ?

Exercise 3 : NCERT Exemplar & Past Year MCQs


NCERT Exemplar MCQs 4. Which of the following pairs represents anomers?
1. Glycogen is a branched chain polymer of a-D glucose units CHO CHO
in which chain is formed by C1–C4 glycosidic linkage where H OH HO H
as branching occurs by the formation of C1–C6 glycosidic HO H HO H
linkage. Structure of glycogen is similar to .......... . OH
(a) H OH H
(a) amylose (b) amylopectin
(c) cellulose (d) glucose H OH H OH
2. Which of the following polymer is strored in the liver of CH2OH CH2OH
animals ?
(a) Amylose (b) Cellulose
(c) Amylopectin (d) Glycogen CHO CHO
3. Sucrose (Cane sugar) is a disaccharide. One molecule of H OH HO H
sucrose on hydrolysis gives .......... . HO H H OH
(a) 2 molecules of glucose (b) H OH HO H
(b) 2 molecules of glucose + 1 molecule of fructose H OH HO H
(c) 1 molecule of glucose + 1 molecule of fructose CH2OH CH2OH
(d) 2 molecules of fructose
EBD_8350
444 CHEMISTRY

8. Dinucleotide is obtained by joining two nucleotides


H OH HO H
together by phosphodiester linkage. Between which
H OH H OH carbon atoms of pentose sugars of nucleotides are these
(c) HO H O HO H O linkages present?
H OH H OH (a) 5' and 3' (b) 1' and 5'
H H (c) 5' and 5' (d) 3' and 3'
CH2OH CH2OH 9. Nucleic acids are the polymers of......
(a) nucleosides (b) nucleotides
H OH HO H (c) bases (d) sugars
H OH HO H 10. Which of the following statements is not true about
(d) HO H O H OH O glucose?
H OH HO H (a) It is an aldohexose
H H (b) On heating with HI it forms n-hexane
CH2OH CH2OH (c) It is present in furanose form
(d) It does not give 2, 4-DNP test
5. Proteins are found to have two different types of secondary
11. Each polypeptide in a protein has amino acids linked with
structures viz. a-helix and b-pleated sheet structure. a- each other in a specific sequence. This sequence of amino
helix structure of protein is stabilised by: acids is said to be ......... .
(a) peptide bonds (a) primary structure of proteins
(b) van der Waal’s forces (b) secondary structure of proteins
(c) hydrogen bonds (c) tertiary structure of proteins
(d) dipole-dipole interactions (d) quateranary structure of proteins
6. In disacchar ides, if th e reducin g groups of
12. DNA and RNA contain four bases each. Which of the
monosaccharides, i.e., aldehydic or ketonic groups are following bases is not present in RNA?
bonded, these are non-reducing sugars. Which of the (a) Adenine (b) Uracil
following disaccharide is a non-reducing sugar?
(c) Thymine (d) Cytosine
CH2OH CH2OH 13. Which of the following B group vitamins can be stored in
H O H H O H our body?
H H (a) Vitamin B1 (b) Vitamin B2
(a) OH H O OH H
OH OH (c) Vitamin B6 (d) Vitamin B12
H OH H OH
14. Which of the following bases is not present in DNA?
(a) Adenine (b) Thymine
CH2OH (c) Cytosine (d) Uracil
H O H HOH2C O H 15. Three cyclic structures of monosaccharides are given
H below which of these are anomers.
OH H H OH
(b) OH O CH2OH HO H
OH H H OH HO H HO H
H OH
H OH H OH HO H O
HO H O HO H O H OH
CH2OH CH2OH
H OH H OH HO H
OH O OH O H
H H H H H
(c) OH H OH H CH2OH CH2OH CH2OH
H O OH
H (I) (II) (III)
H OH H OH
(a) I and II
(b) II and III
CH2 OH CH2OH
O O OH
(c) I and III
H
OH
H H (d) III is anomer of I and II
(d) OH H O OH H 16. Which of the following reactions of glucose can be
H
H H
explained only by its cyclic structure?
H OH H OH (a) Glucose forms pentaacetate
7. Which of the following acids is a vitamin? (b) Glucose reacts with hydroxylamine to form an oxime
(a) Aspartic acid (b) Ascorbic acid (c) Pentaacetate of glucose does not react with hydroxyl
(c) Adipic acid (d) Saccharic acid amine
(d) Glucose is oxidised by nitric acid to gluconic acid
BIOMOLECULES 445

17. Optical rotations of some compounds alongwith their (b) (A) and (B) are between C1 and C4, (C) is between C1
structures are given below which of them have D and C6
configuration. (c) (A) and (C) are between C1 and C4, (B) is between C1
and C6
CH2OH (d) (A) and (C) are between C1 and C6, (B) is between C1
CHO and C4
CHO
H OH C=O
H H Past Year MCQs
H OH HO HO
H OH H OH 20. D (+) glucose reacts with hydroxylamine and yields an
CH2OH H OH H OH oxime. The structure of the oxime would be
[AIPMT 2014, S]
CH2OH CH2OH
(+) rotation (+) rotation (–) rotation
(a) CH = NOH (b) CH = NOH
(I) (II) (III)
H – C – OH HO – C – H
(a) I, II, III (b) II, III
HO – C – H HO – C – H
(c) I, II (d) III
18. Structure of disaccharide formed by glucose and fructose HO – C – H H – C – OH
is given below. Identify anomeric carbon atoms in
monosaccharide units. H – C – OH H – C – OH
f CH2OH CH2OH
CH2OH
e a
O H O H
H HOH2C
H e
d a b CH = NOH CH = NOH
OH H H
OH
O CH2OH
c b c df HO – C – H H – C – OH
OH
H OH OH H
H – C – OH HO – C – H
(a) 'a' carbon of glucose and 'a' carbon of fructose
(b) 'a' carbon of glucose and 'e' carbon of fructose HO – C – H H – C – OH
(c) 'a' carbon of glucose and 'b' carbon of fructose (c) (d)
(d) 'f ' carbon of glucose and 'f ' carbon of fructose H – C – OH H – C – OH
19. Three structures are given below in which two glucose
CH2OH CH2OH
units are linked. Which of these linkages between glucose
units are between C1 and C4 and which linkages are between 21. Which of the following hormones is produced under the
C1 and C6? condition of stress which stimulates glycogenolysis in the
CH2OH CH2OH liver of human beings? [AIPMT 2014, C]
H O O OH
(a) Thyroxin (b) Insulin
H
H (A) H
OH H OH H (c) Adrenaline (d) Estradiol
OH O H
H 22. Which one of the following bases is not present in DNA?
H OH H OH
[JEE M 2014, C]
(I) (a) Quinoline (b) Adenine
CH2OH CH2OH
O H (c) Cytosine (d) Thymine
O OH
H
H H 23. Which of the vitamins given below is water soluble ?
OH H H
(C) OH H [JEE M 2015, C]
HO H
H OH CH2OH O H (a) Vitamin E (b) Vitamin K
OH
O (B) H O (c) Vitamin C (d) Vitamin D
H
24. Which one given below is a non-reducing sugar?
CH2 OH H
O H HO [NEET 2016, C]
H H
H H OH (a) Maltose (b) Lactose
OH H
OH OH (III) (c) Glucose (d) Sucrose
H OH 25. In a protein molecule various amino acids are linked
(II) together by [NEET 2016, S]
(a) (A) is between C1 and C4, (B) and (C) are between C1 (a) a-glycosidic bond (b) b-glycosidic bond
and C6 (c) peptide bond (d) dative bond
EBD_8350
446 CHEMISTRY

26. The correct statement regarding RNA and DNA, 31. Which of the following compounds can form a zwitterion?
respectively is [NEET 2016, S] [NEET 2018, S]
(a) The sugar component in RNA is arabinose and the (a) Aniline (b) Acetanilide
sugar component in DNA is 2'-deoxyribose. (c) Glycine (d) Benzoic acid
(b) The sugar component in RNA is ribose and the sugar 32. Glucose on prolonged heating with HI gives :
component in DNA is 2'-deoxyribose. [JEE M 2018, S]
(a) n-Hexane (b) 1- Hexene
(c) The sugar component in RNA is arabinose
(c) Hexanoic acid (d) 6-iodohexanal
(d) The sugar component in RNA is 2'-deoxyribose and 33. The predominant form of histamine present in human blood
the sugar component in DNA is arabinose is (pKa, Histidine – 6.0) [JEE M 2018, S]
27. Thiol group is present in : [JEE M 2016, C]
H H
(a) Cysteine (b) Methionine N +
N
(c) Cytosine (d) Cystine (a) (b) +
N
28. Which of the following statements is not correct : N H
[NEET 2017, C]
(a) Ovalbumin is a simple food reserve in egg-white H H
N N
(b) Blood proteins thrombin and fibrinogen are involved +
(c) + (d)
in blood clotting N N
H
(c) Denaturation makes the proteins more active
34. The non-essential amino acid among the following is:
(d) Insulin maintains sugar level in the blood of a human
[NEET 2019, C]
body
(a) valine (b) leucine
29. Which of the following compounds will behave as a
(c) alanine (d) lysine
reducing sugar in an aqueous KOH solution? 35. Which of the following statements is not true about
[JEE M 2017, A] sucrose? [JEE M 2019, C]
HOH2C CH2OH (a) It is a non reducing sugar
O
(b) The glycosidic linkage is present between C 1 of
(a) a-glucose and C-1 of b-fructose
HO OCOCH (c) It is also named as invert sugar
3
(d) On hydrolysis, it produces glucose and fructose
OH
36. The increasing order of pKa of the following amino acids
in aqueous solution is: [JEE M 2019, A]
HOH2C O CH2OH
Gly Asp Lys Arg
(a) Asp < Gly < Arg < Lys
(b) HO (b) Gly < Asp < Arg < Lys
(c) Asp < Gly < Lys < Arg
OH (d) Arg < Lys < Gly < Asp
37. Match the following: [JEE M 2019, A]
HOH2C O CH2OH A. Riboflavin (i) Beriberi
B. Thiamine (ii) Scurvy
(c) HO OCH C. Pyridoxine (iii) Cheilosis
3
D. Ascorbic acid (iv) Convulsions
OH (a) A – (i), B – (iv), C – (iii), D – (ii)
(b) A – (iii), B – (iv), C – (i), D – (ii)
HOH2C (c) A – (iii), B – (i), C – (iv), D – (ii)
O CH2OCH3 (d) A – (iv), B – (ii), C – (i), D – (iii)
38. Sucrose on hydrolysis gives: [NEET 2020, C]
(d) OH
(a) a-D-Glucose + b-D-Glucose
OH
OH (b) a-D-Glucose + b-D-Fructose
30. The difference between amylose and amylopectin is (c) a-D-Fructose + b-D-Fructose
[NEET 2018, C] (d) a-D-Glucose + b-D-Fructose
(a) Amylopectin has 1 ® 4 a-linkage and 1 ® 6 a-linkage 39. Which of the following is a basic amino acid ?
(b) Amylose has 1 ® 4 a-linkage and 1® 6 b-linkage [NEET 2020, C]
(c) Amylose is made up of glucose and galactose (a) Alanine (b) Tyrosine
(d) Amylopectin has 1 ® 4 a-linkage and 1® 6 b-linkage
(c) Lysine (d) Serine
BIOMOLECULES 447

Exercise 4 : Problem Solving Skill Enhancer MCQs


1.
5(CH CO) O
D - ( + ) - G l u c os e ¾¾¾¾¾¾
3 2 ® D- (+ )- Gl ucos e 4. What will happen when D-(+)-glucose is treated with
pentaacetate methanolic –HCl followed by Tollens’ reagent?
(a) A black ppt. will be formed
Which statement is true about glucose pentaacetate ?
(b) A red ppt. will be formed
(a) It will react with phenylhydrazine but not with (c) A green colour will appear
Tollens’ reagent. (d) No characteristic colour or ppt. will be formed.
(b) It will react with Tollens’ reagent but not with 5. Which is true about the acidic character of hydroxyl
phenylhydrazine. groups of sugars and hydroxyl group of an alcohol?
(a) The OH’s of sugars are more acidic than that of a
(c) It will react with both of the above mentioned typical alcohol.
reagents. (b) The OH’s of sugars are less acidic than that of a
typical alcohol.
(d) It will react neither with phenylhydrazine nor with (c) Both have similar acidic character.
Tollens’ reagent. (d) The OH’s of sugars are neutral while that of an
alcohol is acidic.
2. Which is not a reducing sugar? 6. Which of the following statement(s) is (are) true ?
(i) All amino acids contain one chiral center.
CH2OH CH2OH (ii) Some amino acids contain one, while some contain
OH H more chiral center or even no chiral center.
H H H H (iii) All amino acids found in proteins have L configuration.
O (iv) All amino acids found in proteins have 1° amino group.
(a) OH OH H H OH OH (a) (ii), (iii) & (iv) (b) (ii) & (iii)
(c) (i), (iii) & (iv) (d) (i) & (iv)
H OH OH H 7. Which of the nitrogen of histidine is first protonated ?
b +
N NH3
CH2OH CH2OH
CH2CHCOO–
H H OH H OH aN
H H
O (a) a (b) b
(b) OH OH H OH H
OH (c) both (d) None.
8. Histidine, a heterocyclic amino acid has following
H OH H OH structure at pH < 1.82,
+
CH2OH HN+ NH3
OH O CH2CHCOOH
H CH2OH N
H
OH H O CH2 H OH H
(c) OH OH at pH > 1.82 it should have which structure ?
H
+ +
H OH H H HN NH3
(a) CH2CHCOO–
CH2OH CH2OH N
OH H
H H O H + +
H HN NH3
O
(d) HO OH H H OH H
OH (b) + CH2CHCOOH
N
H2
H OH H OH
+
HN NH2
NaOH/CaO pass in
3. Amino acid ¾¾¾¾¾® gas ¾¾¾¾¾ ®
D lime water
(c) + CH2CHCOOH
(1 mole) evolved N
H2
0.1999 kg (salt)
+
Amino acid has N NH3
(a) Two NH2 groups (b) One COOH group
(d) + CH2CHCOOH
(c) Two COOH groups (d) Three COOH groups N
H2
EBD_8350
448 CHEMISTRY

9. Compound Property 12. Which statement is correct?


(A) Amylopectin (1) Milk sugar (a) Hormones not produced by body and may be stored
(B) Lactose (2) dextrorotatory in the body to fight out the diseases
(b) Vitamins may be stored in the body to fight out diseases
(C) Sucrose (3) 1, 4 and 1, 6 glycosidic
(c) Vitamins are not stored in the body but are
linkage
continuously produced
(D) Alanine (4) L - Configuration
(d) Both hormones and vitamins are stored in the body
Correct matching is
13. Number of peptide bonds in the compound
(a) (A) ® 3; (B) ® 4; (C) ® 1; (D) ® 2
(b) (A) ® 3; (B) ® 1; (C) ® 2; (D) ® 4 H O
O
(c) (A) ® 2; (B) ® 3; (C) ® 2; (D) ® 4 N
(d) (A) ® 3; (B) ® 4; (C) ® 2; (D) ® 1 CH3—C—NH NH—NH2
10. Vitamin Source
(A) E (1) Butter (a) 1 (b) 2
(B) K (2) Sunlight (c) 3 (d) 4 +
(C) D (3) Green leafy vegetable N H3
|
(D) A (4) Sunflower oil
14. Increase in pH of the solution converts RCHCOOH to
Correct matching is
(a) (A) ® 4; (B) ® 3; (C) ® 2; (D) ® 1 +
(b) (A) ® 3; (B) ® 2; (C) ® 1; (D) ® 4 NH 2 NH3


(a) (b)


(c) (A) ® 2; (B) ® 4; (C) ® 3; (D) ® 1 RCHCOOH RCHCOO–
(d) (A) ® 4; (B) ® 3; (C) ® 1; (D) ® 2
11. a-Amino acids are (c) Both (d) None
(a) acidic due to –COOH group and basic due to –NH2 15. Glucose-D has a great tendency to be converted into cyclic
group isomer. Which two carbon atoms get joined through ‘O’ to
(b) acidic due to – NH3+ group and basic due to – COO– form this hemiacetal ?
group.
(c) neither acidic nor basic. (a) C1 and C4 (b) C1 and C5
(d) none is true. (c) C1 and C6 (d) C2 and C6

ANSWER KEYS
Exercise 1 : NCERT Based Topic-wise MCQs
1 (b) 13 (b) 25 (b) 37 (d) 49 (b) 61 (b) 73 (a) 85 (a) 97 (d) 109 (d)
2 (b) 14 (c) 26 (a) 38 (a) 50 (d) 62 (c) 74 (c) 86 (d) 98 (a) 110 (c)
3 (d) 15 (a) 27 (c) 39 (b) 51 (a) 63 (a) 75 (a) 87 (c) 99 (b) 111 (d)
4 (d) 16 (b) 28 (b) 40 (d) 52 (c) 64 (d) 76 (c) 88 (c) 100 (a) 112 (d)
5 (b) 17 (d) 29 (b) 41 (c) 53 (c) 65 (d) 77 (a) 89 (d) 101 (d) 113 (c)
6 (a) 18 (b) 30 (c) 42 (a) 54 (c) 66 (a) 78 (c) 90 (b) 102 (a) 114 (b)
7 (a) 19 (d) 31 (a) 43 (c) 55 (c) 67 (b) 79 (a) 91 (a) 103 (c) 115 (b)
8 (d) 20 (b) 32 (c) 44 (b) 56 (b) 68 (b) 80 (b) 92 (b) 104 (d) 116 (c)
9 (d) 21 (b) 33 (d) 45 (d) 57 (c) 69 (d) 81 (b) 93 (c) 105 (c) 117 (d)
10 (c) 22 (b) 34 (c) 46 (d) 58 (b) 70 (c) 82 (b) 94 (b) 106 (a) 118 (b)
11 (b) 23 (c) 35 (c) 47 (d) 59 (a) 71 (a) 83 (d) 95 (b) 107 (d) 119 (d)
12 (c) 24 (b) 36 (c) 48 (c) 60 (a) 72 (b) 84 (b) 96 (c) 108 (a) 120 (b)
Exercise 2 : Numeric/Integer Answer Questions
1 (2) 2 (1) 3 (2) 4 (6) 5 (8) 6 (4) 7 (67) 8 (4) 9 (6) 10 (3)
Exercise 3 : NCERT Exemplar & Past Year MCQs
1 (b) 5 (c) 9 (b) 13 (d) 17 (a) 21 (c) 25 (c) 29 (a) 33 (d) 37 (c)
2 (d) 6 (b) 10 (c) 14 (d) 18 (c) 22 (a) 26 (b) 30 (a) 34 (c) 38 (b)
3 (c) 7 (b) 11 (a) 15 (a) 19 (c) 23 (c) 27 (a) 31 (c) 35 (b) 39 (c)
4 (c) 8 (a) 12 (c) 16 (c) 20 (d) 24 (d) 28 (c) 32 (a) 36 (c)
Exercise 4 : Problem Solving Skill Enhancer MCQs
1 (d) 3 (c) 5 (a) 7 (b) 9 (b) 11 (b) 13 (a) 15 (b)
2 (a) 4 (d) 6 (b) 8 (a) 10 (a) 12 (b) 14 (b)
29 Polymers

Trend Buster NEET & JEE Main

Number of Questions from 2020-15 6 5 Minimum 1 Question has been asked


Weightage 1.9% 3.3% in NEET & JEE Main every year.

The most Important Concepts that Cover Maximum number of Questions asked in past 6 years.

Preparation of Polymers 1 3
Biodegradable Polymers / Cross linked polymers 3 —
Free radical Polymerisation — 2

Less Important Concepts that Cover 1 or 2 Questions asked in past 6 years.

Uses of Polymers — 1
Types of Polymers 2 —

NEET JEE

2020 Classification of Polymers Natural Polymers 1 Easy — —

2019 Types of Polymerisation Free radical polymerisation — — 2 Difficult

Classification of Polymers Biodegradable polymers 1 Easy — —

2018 Classification of polymers Cross linked Polymers 1 Average — —

2017 Types of Polymerisation Types of Polymerisation — — 1 Average


reaction

2016 Classification of Polymers low density polythene / 1 Average


Natural rubber 1 Easy

2015 Classification of Polymers Uses of polymers / 2 Easy 1 Easy


Preparation of Nylon–6 /
Biodegradable Polymers
EBD_8350
450 CHEMISTRY
POLYMERS 451
EBD_8350
452 CHEMISTRY

Problem Solving Tips/ Tricks/ Points to Remember

4 Diphenylamine is added as an antioxidant to rubber to It is used for reinforcement of rubber tyres, manufacture
protect it from degradation on exposure to air. of parachute, safety belts, carpets and fabrics.
4 Super glue is a polymer of methyl a-cyanoacrylate. 4 Nylon 6 : Homopolymer of caprolactam (6C)
4 Ebonite is high sulphur (20–30% S) rubber and is obtained 4 Nylon 6, 10 : Copolymer of hexamethylene diamine (6C)
by vulcanization of natural rubber. and sebacoyl chloride (10C)
4 Rubber obtained from waste rubber articles is called 4 Kevlar: It is aromatic polyamide resembling nylons. It is
reclaimed rubber. used in aircraft industries, bullet proof vests, ropes cables
4 Polymethyl methacrylate, lucite or plexiglass : etc.
Used for making lenses, artificial eyes, dentures, 4 Polyurethanes : Polymers of diisocyanate. eg. Perlon-U,
aircraft windows. Substitute of leather.
4 Polyesters 4 Nylon-2-nylon-6
(i) Terylene (dacron) / Polyethylene Terephthalate Copolymer of glycine and amino caproic acid
(PET) : Ethylene glycol and Terephthalic acid are (H2N(CH2)5COOH). It is biodegradable.
monomers. It is used for blending with wool to 4 Plasticizers
provide better crease, in safety helmets and aircraft The substances added to resins to increase their plasticity
battery boxes. and flexibility are known as plasticizers. Examples are,
(ii) Glyptal or Alkyd resin : Ethylene glycol and vegetable oils phosphates.
Phthalic acid are monomers. For making good
insulators, sheets, rods, switches, lacquers and 8 Number average molecular weight (Mn) is given by
adherant paints.
N1M1 + N 2 M 2 + N 3 M 3 + ........... å N i M i
4 Rubbers Mn = =
N1 + N 2 + N 3 + ........... å Ni
(i) Buna - S or SBR or GSR (Government styrene
rubber): Styrene + Butadine Where N1 is number of molecules having molecular weight
(ii) Nitrile rubber (GR-A or Buna - N) : Butadiene + M1
Acrylonitrile
8 Mass - average molecular weight (M w ) is given by
(iii) Neoprene: Chloroprene
(iv) Heavy rubber : cis polyisoprene
(v) Gutta percha : trans polyisoprene N1M12 + N 2 M 22 + N3 M32 + ........... å N i M i2
Mw = =
4 Nylon 6,6 : Copolymer of adipic acid (6C) and N1M1 + N 2 M 2 + N3 M3 + ........... å Ni M i
hexamethylene diamine (6C).
8 Polydispersity index
n HOOC(CH 2 ) 4 COOH + nH 2 N(CH 2 )6 NH 2 ®
Adipic acid Hexamethylene diamine
It is the ratio of mass average molecular weight M w to
the number average molecular weight M n.
O O
|| ||
-( C - (CH 2 )4 - C - NH - (CH 2 ) 6 - NH -) n
Nylon 6, 6
POLYMERS 453

Exercise 1 : NCERT Based Topic-wise MCQs


(b) These possess very weak intramolecular forces or
Topic 1: Classification of Polymers
attractions between polymer chains.
1. Which is not true about polymers?
(c) Vulcanised rubber is an example of elastomer.
(a) Polymers do not carry any charge
(d) All of the above
(b) Polymers have high viscosity
12. Among the following polymers the strongest molecular
(c) Polymers scatter light forces are present in
(d) Polymers have low molecular weight
(a) elastomers (b) fibres
2. Which of the following polymers do not involve cross
(c) thermoplastics (d) thermosetting polymers
linkages?
13. Three dimensional molecular structure with cross links
(a) Melmac (b) Bakelite
are formed in the case of a
(c) Polythene (d) Vulcanised rubber
(a) thermoplastic (b) thermosetting plastic
3. Which of the following natural products is not a polymer?
(c) Both (a) and (b) (d) None of these
(a) DNA (b) Cellulose
14. Which of the following statements is not correct for fibres?
(c) ATP (d) Insulin
(a) Fibres possess high tensile strength and high
4. Rayon is : modulus.
(a) synthetic plastic (b) natural rubber (b) Fibres impart crystalline nature.
(c) natural silk (d) artificial silk (c) Characteristic features of fibres are due to strong
5. The turbidity of a polymer solution measures intermolecular forces like hydrogen bonding.
(a) the light scattered by a solution (d) None of these
(b) the light absorbed by a solution 15. Which of the following can be repeatedly soften on
(c) the light transmitted by a solution heating?
(d) none of the above (i) Polystyrene (ii) Melamine
6. Natural silk is a (iii) Polyesters (iv) Polyethylene
(a) polyester (b) polyamide (v) Neoprene
(c) polyacid (d) polysaccharide (a) (i) and (iii) (b) (i) and (iv)
7. Which of the following is/are a semisynthetic polymers? (c) (iii), (iv) and (v) (d) (ii) and (iv)
(a) Cellulose acetate (b) Polyvinyl chloride 16. Which of the following is a cross linked polymer?
(c) Cellulose nitrate (d) Both (a) and (c) (a) PVC (b) Bakelite
8. Which of the following polymer is an example of fibre? (c) Polyethylene (d) Rubber
(a) Silk (b) Dacron 17. Fibres that have good resistance to stains, chemicals,
(c) Nylon-66 (d) All of these insects and fungi is
9. Ebonite is (a) Acrylic (b) Terylene
(a) Natural rubber (c) Nylon (d) All of these
(b) Synthetic rubber 18. Match the columns.
(c) Highly vulcanized rubber Column-I Column-II
(d) Polypropene (A) Linear polymer (p) Melamine
10. In addition polymer monomer used is (B) Semisynthetic polymer (q) Polyvinyl chloride
(a) unsaturated compounds (C) Branched chain polymer (r) LDPE
(b) saturated compounds (D) Network polymer (s) Cellulose nitrate
(c) bifunctional saturated compounds (a) A– (s), B – (q), C – (r), D – (p)
(d) trifunctional saturated compounds (b) A– (q), B – (s), C – (r), D – (p)
11. Which is/are true for elastomers? (c) A– (q), B – (r), C – (s), D – (p)
(a) These are synthetic polymers possessing elasticity. (d) A– (q), B – (s), C – (p), D – (r)
EBD_8350
454 CHEMISTRY

Topic 2: Types of Polymerisation 29. Interparticle forces present in nylon-6, 6 are


(a) van der waal’s
19. Low density polythene is prepared by
(b) hydrogen bonding
(a) Free radical polymerisation
(c) dipole-dipole interactions
(b) Cationic polymerisation
(d) None of these
(c) Anionic polymerisation
30. Acetic acid is added in the preparation of nylon-6 to
(d) Ziegler-Natta polymerisation
(a) initiate polymerisation
20. The monomer of teflon is
(b) avoid polymerisation at first step
(a) CHF == CH2 (b) CF2 == CF2
(c) avoid oxidation
(c) CHC1 == CHCl (d) CHF == CHC1
(d) removal of water
21. Which of the following does not undergo addition
polymerization? 31. Which of the following polymer is used for making
phonograph records ?
(a) Vinylchloride
(a) Bakelite (b) Dacron (c) Teflon (d) PVC
(b) Butadiene
32. Dacron is a –
(c) Styrene
(a) crease resistant
(d) All of the above undergoes addition polymerizations
(b) polyamide
22. Which compound/set of compounds is used in the
(c) addition polymer
manufacture of nylon-6,6?
(d) polymer of ethylene glycol and phthalic acid
(a) HOOC(CH2)4 COOH + H2N(CH2)6NH2
33. The monomeric units of terylene are glycol and which of
(b) CH2 = CH–C(CH) = CH2 the following
(c) CH2 = CH2 OH
OH
(d) HOOC COOH + HOCH2 – CH2OH (a) (b)
OH OH
23. The monomer(s) used in the preparation of orlon, a
substitute for wool is/are
HO
(a) caprolactam
(c) HOOC COOH (d) OH
(b) styrene and 1, 3-butadiene
HO
(c) tetrafluoroethene
(d) acrylonitrile 34. Soft drinks and baby feeding bottles are generally made
24. Which of the following polymer is used for manufacturing up of
of buckets, dustbins, pipes etc ? (a) Polystyrene (b) Polyurethane
(c) Polyurea (d) Polyamide
(a) Low density polythene(b) High density polythene
(c) Teflon (d) Polyacrylonitrile 35. Polymerisation in which two or more chemically different
monomers take part is called:
25. Which of the following catalyst is used in preparation of
(a) addition polymerisation (b) copolymerisation
high density polythene ?
(c) chain polymerisation (d) homo polymerisation
(a) Peroxide catalyst (b) Ziegler - Natta catalyst
36. Which one of the following statement is not true?
(c) Wilkinson’s catalyst (d) Pd - catalyst (a) In vulcanization the formation of sulphur bridges
26. Which of the following statements is false? between different chains make rubber harder and
(a) Artificial silk is derived from cellulose. stronger.
(b) Nylon-66 is an example of elastomer. (b) Natural rubber has the trans -configuration at every
(c) The repeat unit in natural rubber is isoprene. double bond.
(c) Buna-S is a copolymer of butadiene and styrene.
(d) Both starch and cellulose are polymers of glucose.
(d) Natural rubber is a 1, 4 - polymer of isoprene.
27. Melamine plastic crockery is a copolymer of:
(a) HCHO and melamine (b) HCHO and ethylene 37. The repeating unit present in Nylon 6 is
(c) melamine and ethylene (d) None of these (a) — [NH(CH2)6NHCO(CH2)4CO] —
28. Polymer used in bullet proof glass is (b) — [CO(CH2)5NH] —
(a) Lexan (b) PMMA (c) — [CO (CH2)6NH] —
(c) Nomex (d) Kevlar (d) — [CO (CH2)4NH] —
POLYMERS 455

38. The process involving heating of rubber with sulphur is (a) (i), (ii) and (iii) (b) (ii), (iii) and (iv)
called (c) (iii) and (iv) (d) (ii) and (iii)
(a) Galvanisation (b) Vulcanization 45. Which of the following statements are correct?
(c) Bessemerisation (d) Sulphonation (i) Buna–N being resistant to the action of petrol,
39. Which of the following are examples of synthetic rubber? lubricating oil and organic solvents is used in making
(i) Polychloroprene (ii) polyacrylonitrile oil seals.
(iii) Buna-N (iv) cis-polyisoprene (ii) Biodegradable polymers are manufactured because
of low chemical resistance, strength and durability
(a) (i) and (iii) (b) (i) and (ii) of conventional polymers.
(c) (iii) and (iv) (d) (ii) and (iii) (iii) PHBV is a copolymer used in the manufacture of
40. Buna-N synthetic rubber is a copolymer of : orthopaedic devices.
(a) H2C = CH – CH = CH2 and H5C6 – CH = CH2 (iv) Nylon 2–nylon 6 is a biodegradable polymer.
(b) H2C = CH – CN and H2C = CH – CH = CH2 (a) (i), (ii) and (iii) (b) (ii), (iii) and (iv)
(c) (i), (iii) and (iv) (d) (i) and (iv)
(c) H2C = CH – CN and H 2 C = CH – C = CH 2
| 46. Match the columns.
CH3 Column-I Column-II
Cl
| (A) Highly branched (p) Teflon
(d) H 2 C = CH – C = CH 2 and H 2C = CH – CH = CH 2
chemically inert polymer
41. Which of the following is not a copolymer? used in the insulation of
(a) Buna–S (b) Baketite electric wires.
(c) Neoprene (d) Dacron (B) Linear polymer (q) Polyacrylonitrile
42. Assertion : Bakelite is formed when novolac is heated prepared in presence of
with formaldehyde which is a thermosetting polymer. Al(C2H5)3 and TiCl4.
Reason : Bakelite is infusible solid mass. (C) Corrosion resistant (r) HDPE
(a) If both assertion and reason are true and reason is polymer used in
the correct explanation of assertion. manufacture of non–stick
(b) If both assertion and reason are true but reason is not surface coated utensils.
the correct explanation of assertion.
(D) Addition polymer (s) LDPE
(c) If assertion is true but reason is false.
used as a substitute for wool.
(d) If both assertion and reason are false. (a) A–(s), B–(r), C–(q), D–(p)
43. Two condensation polymers are made (b) A–(s), B–(p), C–(r), D–(q)
(1) ethylene diamine + ethane–1, 2– dicarboxylic acid
(c) A–(s), B–(r), C–(p), D–(q)
(2) trimethylenediamine + ethane–1, 2– dicarboxylic acid (d) A–(r), B–(s), C–(p), D–(q)
if both polymers of same molecular weight are
obtained then which of the following statements is/ 47. Match the columns
are correct ? Column-I Column-II
(i) Polymer (1) is found to melt at lower temperature. (A) Polyester of glycol and (p) Novalac
(ii) Polymer (2) is found to melt at lower temperature. phthalic acid
(iii) H-bonding is major factor. (B) Copolymer of 1, 3-butadiene (q) Glyptal
and styrene
(a) (i), (ii) and (iii) (b) Only (ii)
(C) Phenol and formaldehyde (r) Buna-S
(c) (i) and (iii) (d) (ii) and (iii)
resin
44. Which of the following statements are correct?
(D) Polyester of glycol and (s) Buna-N
(i) A polyamide nylon 6,6 prepared by the condensation
polymerisation of hexamethylene diamine with adipic terephthalic acid
acid is used in the manufacture of tyre cords. (E) Copolymer of 1, 3-butadiene (t) Dacron
(ii) Terylene is crease resistant and is blended with and acrylonitrile
cotton and wool fibres for various applications. (a) A – (q), B – (s), C – (p), D – (t), E – (r)
(iii) Condensation reaction of phenol and formaldehyde (b) A – (q), B – (r), C – (p), D – (t), E – (s)
to form novolac can be catalysed either by acid or base. (c) A – (r), B – (p), C – (t), D – (s), E – (q)
(iv) Melamine formaldehyde polymer is mainly used in (d) A – (p), B – (s), C – (q), D – (t), E – (r)
the manufacture of electrical switches.
EBD_8350
456 CHEMISTRY

48. Match the polymers given in Column-I with their chemical (b) Assertion is correct, reason is correct; reason is not a
names given in Column-II correct explanation for assertion.
Column-I Column-II (c) Assertion is correct, reason is incorrect.
(A) Nylon 6 (p) Polyvinyl chloride (d) Assertion is incorrect, reason is correct.
(B) PVC (q) Polyacrylonitrile 54. When condensation product of hexamethylenediamine and
(C) Acralin (r) Polycaprolactam adipic acid is heated to 525K in an atmosphere of nitrogen
for about 4-5 hours, the product obtained is
(D) Natural rubber (s) Low density
Polythene (a) solid polymer of nylon 6, 6
(E) LDPE (t) cis-Polyisoprene (b) liquid polymer of nylon 6, 6
(c) gaseous polymer of nylon 6, 6
(a) A – (r), B – (p), C – (q), D – (t), E – (s)
(d) liquid polymer of nylon-6
(b) A – (s), B – (q), C – (t), D – (r), E – (p)
55. Assertion : Bakelite is a thermosetting polymer.
(c) A – (t), B – (s), C – (p), D – (q), E – (r)
Reason : Bakelite can be melted again and again without
(d) A – (s), B – (t), C – (r), D – (q), E – (p) any change.
49. Match the columns (a) Assertion is correct, reason is correct; reason is a
Column-I Column-II correct explanation for assertion.
(A) Polymer of styrene (p) used in making handles (b) Assertion is correct, reason is correct; reason is not a
of utensils and computer correct explanation for assertion.
discs (c) Assertion is correct, reason is incorrect.
(B) Polymer of ethylene (q) used as an insulator (d) Assertion is incorrect, reason is correct.
glycol and phthalic acid 56. Identify A, B and C in the following sequence of reactions
(C) Polymer of phenol (r) used in making paints
O O O
and formaldehyde and lacquers.
(D) Polymer of vinyl (s) used in manufacture of C6H5–C–O—O–C–C6H5 ® 2 C6H5–C–O ® 2A
chloride rain coats and flooring.
A + CH2 = CH2 ¾¾
®B
(a) A – (q), B – (r), C– (p), D – (s)
(b) A – (r), B – (q), C– (p), D – (s) B + nCH2 = CH2 ¾¾
®C
(c) A – (q), B – (p), C– (r), D – (s) C + C ¾¾
® C6H5–(CH2– CH2)n—CH2– CH2 – CH2
(d) A – (q), B – (r), C– (s), D – (p) CH2 –(CH2–CH2)n– C6H5
50. The polymer containing strong intermolecular forces e.g. · ·
hydrogen bonding, is (a) A = C 6H5, B = C6H5 – CH2– CH 2 and
(a) teflon (b) nylon 6, 6 ·
(c) polystyrene (d) natural rubber C = C6H5 –(CH2 – CH2)n—CH2 – CH 2
· ·
51. Assertion : Olefinic monomers undergo addition
(b) A = C6H5 C O, B = C6H5 – CH2– CH 2 and
polymerisation.
·
Reason : Polymerisation of vinylchloride is initiated by C = C6H5 –(CH2–CH2)n—CH2– CH 2
peroxides/ persulphates.
· ·
(a) Assertion is correct, reason is correct; reason is a (c) A = C 6H5, B = C6H5 – CH – CH3 and
correct explanation for assertion. ·
(b) Assertion is correct, reason is correct; reason is not a C = C6H5—(CH2 –CH2)n—CH2 CH 2
correct explanation for assertion. · ·
(c) Assertion is correct, reason is incorrect. (d) A= C 6H5, B=C6H5 –CH2– CH 2 and
(d) Assertion is incorrect, reason is correct. C = C6H5 –(CH2– CH2)n—C6H5
52. Among cellulose, poly vinyl chloride, nylon and natural 57. Assertion : In vulcanisation of rubber, sulphur cross links
rubber, the polymer in which the intermolecular force of are introduced.
attraction is weakest is Reason : Vulcanisation is a free radical initiated chain
(a) nylon (b) poly (vinyl chloride) reaction.
(c) cellulose (d) natural rubber (a) Assertion is correct, reason is correct; reason is a
53. Assertion : Teflon has high thermal stability and chemical correct explanation for assertion.
inertness. (b) Assertion is correct, reason is correct; reason is not a
Reason : Teflon is a thermoplastic. correct explanation for assertion.
(a) Assertion is correct, reason is correct; reason is a (c) Assertion is correct, reason is incorrect.
correct explanation for assertion. (d) Assertion is incorrect, reason is correct.
POLYMERS 457

58. Which one of the following monomers gives the polymer O O


neoprene on polymerization ?
(a) CF2 = CF2 (b) CH2 = CHCl (c) Nylon-6, 6 –NH–(CH2)6–NH –C –(CH2)4– C–O –n
Cl CH3


(c) CCl2 = CCl2 (d) CH2==C–CH==CH2 (d) PMMA – CH2–C
59. Which of the following is not correctly matched? COOCH3 n
O O
60. Which of the following monomers form biodegradable
(a) Terylene –OCH2–CH2–C– –C – polymers?
n (a) 3-hydroxybutanoic acid + 3-hydroxypentanoic acid
(b) Glycine + amino caproic acid
(b) Neoprene — CH2 — C = CH — CH2 —
(c) Ethylene glycol + phthalic acid
Cl n
(d) Both (a) and (b)

NCERT Exemplar MCQs


1. Which of the following polymers of glucose is stored by
(c) (CH 2 CH= CH CH 2 CH CH 2 )n
animals?
(a) Cellulose (b) Amylose (d) (O CH CH 2 C O CH CH 2 C )n
|

|
|

|
(c) Amylopectin (d) Glycogen | || | ||
CH 3 O CH2CH3 O
2. Which of the following is not a semisynthetic polymer?
6. Which of the following statements is not true about low
(a) cis-polyisoprene (b) Cellulose nitrate density polyethene?
(c) Cellulose acetate (d) Vulcanised rubber (a) Tough
3. The commercial name of polyacrylonitrile is ....... . (b) Hard
(a) dacron (b) orlon (acrilan) (c) Poor conductor of electricity
(c) PVC (d) bakelite (d) Highly branched structure
4. Which of the following polymer is biodegradable? CH3 CH3
| |
(a) –( CH2 C = CH CH2 n
–(
7. ( CH2 C CH2 C )n is a polymer having monomer
| |
Cl CN CH3 CH3
units ......... .
(b) –( CH2 CH = CH CH2 CH2 CH –)n
(a) (b)
(c) –( O CH CH 2 C O CH CH 2 C –)n H

CH3 O CH 2CH 3 O (c) (d)


H
H H O O 8. Which of the following polymer can be formed by using
the following monomer unit?
(d) –( N (CH2)6 N C (CH2)4 C –)n
H
5. In which of the following polymers ethylene glycol is one |
of the monomer units? N O
H2C C
||

(a) (OCH2 CH 2 OOC CO )n


H2C CH2
H2C CH2
(a) Nylon-6, 6 (b) Nylon-2-nylon-6
(b) ( CH 2 — CH 2 )n
(c) Melamine polymer (d) Nylon-6
EBD_8350
458 CHEMISTRY

Past Year MCQs (c) Its synthesis requires high pressure.


9. Which one of th e followin g is an example of a (d) It is a poor conductor of electricity.
thermosetting polymer? [AIPMT 2014, S] 17. The formation of which of the following polymers involves
hydrolysis reaction? [JEE M 2017, A]
(a) ( CH 2 - C = CH - CH 2 ) n (a) Nylon 6 (b) Bakelite
|
Cl (c) Nylon 6, 6 (d) Terylene
(b) ( CH 2 - CH ) n 18. Regarding cross-linked or network polymers, which of the
| following statements is incorrect? [NEET 2018, C]
Cl
H H O O (a) They contain covalent bonds between various linear
| | || || polymer chains.
(c) ( N - (CH 2 )6 - N - C - (CH 2 )4 - C )n (b) They are formed from bi- and tri-functional monomers.
(c) They contain strong covalents bonds in their polymer
OH OH
chains.
CH2 CH2
(d) Examples are bakelite and melamine.
(d)
19. The biodegradable polymer is: [NEET 2019, C]
n
(a) nylon-6, 6 (b) nylon 2-nylon 6
10. Which of the following organic compounds polymerizes (c) nylon-6 (d) Buna-S
to form the polyester Dacron? [AIPMT 2014, S] 20. The major product of the following reaction is:
(a) Propylene and para HO—(C6H4)— OH [JEE M 2019, A]
(b) Benzoic acid and ethanol
(c) Terephthalic acid and ethylene glycol (1) KOH (alc.)
¾¾¾¾¾¾®
(d) Benzoic acid and para HO–(C6H4)—OH Cl (2) Free radical
11. Which one is classified as a condensation polymer? Cl polymerisation

[JEE M 2014, S] CH3


(a) Dacron (b) Neoprene
(c) Teflon (d) Acrylonitrile
n n
12. Biodegradable polymer which can be produced from glycine
and aminocaproic acid is : [AIPMT 2015, A] (a) (b)
(a) PHBV (b) Buna - N
(c) Nylon 6, 6 (d) Nylon 2- nylon 6 OH OH
13. Caprolactum is used for the manufacture of : CH3
[AIPMT 2015 RS, S]
(a) Nylon - 6 (b) Teflon n
(c) Terylene (d) Nylon - 6,6 n
14. Which polymer is used in the manufacture of paints and (c) (d)
lacquers ? [JEE M 2015, A]
(a) Polypropene (b) Polyvinyl chloride Cl
Cl
(c) Bakelite (d) Glyptal 21. Major product of the following reaction is:
15. Natural rubber has [NEET 2016, C] [JEE M 2019, A]
(a) all cis-configuration (1) Et3N
Cl NH2 ¾ ¾¾¾¾¾®
(b) all trans-configuration (2) Free radical
Cl + H2N polymerisation
(c) alternate cis-and trans-configuration O
(d) random cis-and trans-configuration O
16. Which of the following statements about low density
polythene is FALSE? [JEE M 2016, C] Cl O
(a) Its synthesis requires dioxygen or a peroxide initiator
n
as a catalyst. (a) O
(b) It is used in the manufacture of buckets, dustbins etc. HN
NH2
POLYMERS 459

O Cl Cl

n n
(d) NH2
(b) N
HN
NH2 H
22. Which of the following is a natural polymer?
Cl [NEET 2020, S]
(a) poly (Butadiene-styrene)
n O
(b) polybutadiene
(c) NH2
N (c) poly (Butadiene-acrylonitrile)
O
H (d) cis-1, 4-polyisoprene

1. Formation of polyethylene from calcium carbide takes 5. The monomer of the polymer;
place as follows CH3 CH3
|
CaC 2 + 2H 2 O ¾
¾® Ca (OH ) 2 + C 2 H 2 ÚÚÚÚÚCH 2 - C - CH 2- C +
| is
C2H2 + H2 ¾
¾® C 2 H 4 CH3 CH3
nC 2 H 4 ¾
¾® ( - CH 2 - CH 2 -) n (a) H2C = C(CH3)2 (b) CH3CH = CHCH3
The amount of polyethylene obtained from 64 kg of CaC2 is (c) CH3CH = CH2 (d) (CH3)2C = C(CH3)2
(a) 7 kg (b) 14 kg 6. Head-to-tail addition takes place in chain-growth
(c) 21 kg (d) 28 kg polymerisation, when monomer is
2. Ethylene-propylene rubber (EPR) is (a) CH2 = CHPh
(a) unsaturated and stereoregular (b) CH 2 = CH - CH = CH 2
(b) saturated and stereoregular
(c) CH 2 < C , COCH3
(c) atactic and unsaturated | P
(d) syndiotactic and unsaturated CH3 O
3. Structure of some important polymers are given. Which
one represents buna-S? (d) CH 2 < CH , C º N
CH3 7. Select the correct statement.
(a) Vinyon is a copolymer of vinyl chloride and vinyl
(a) (– CH2 – C = CH – CH2 –) n acetate.
(b) (– CH2 – CH = CH – CH2 – CH – CH2 –) n (b) Saran is a copolymer of vinyl chloride and vinylidine
chloride.
C6H5
(c) Butyl rubber is a copolymer of isobutylene and
(c) (– CH 2 – CH = CH – C H 2 – CH(CN) – CH 2 –) n isoprene.
(d) (– C H2 – (Cl)C = CH – C H2 –) n (d) All of the above are correct.
4. Acrilan is a hard, horny and a high melting material. Which 8. Which of the following has been used in the manufacture
of the following represents its structure? of non-inflammable photographic films?
(a) — CH2 — CH —n (b) — CH2 — CH —n (a) Cellulose nitrate (b) Cellulose xanthate

(c) Cellulose perchlorate (d) Cellulose acetate


Cl CN
9. The polymer used in making synthetic hair wigs is made up
CH3
of

(c) — CH2 — CH —n (d) — CH2 — CH —n (a) CH2 = CHCl (b) CH2 = CHCOOCH3

COOCH3 COOC2H5 (c) C6H5CH = CH2 (d) CH2 = CH – CH=CH2


EBD_8350
460 CHEMISTRY

10. The condensation of hexamethylenediamine with sebacoyl (b) methyl acrylate


chloride at 525 K gives (c) methyl methacrylate
(a) nylon-6,20 (b) nylon-6,01 (d) None of these
(c) nylon-6,10 (d) None of these 14. In which of the following polymers, empirical formula re-
11. Which of the following statements is not correct for fibres? sembles with monomer?
(a) Fibres possess high tensile strength and high modulus. (a) Bakelite (b) Teflon
(b) Fibres impart crystalline nature. (c) Nylon-6, 6 (d) Dacron
(c) Characteristic features of fibres are due to strong
intermolecular forces like hydrogen bonding. CH3
(d) All are correct. 15. Monomer of — C — CH2 — is
12. Perlon is CH3 n
(a) Rubber (b) Nylon-6
(c) Terylene (d) Orlon (a) 2-methylpropene (b) styrene
13. Plexiglas (PMMA) is a polymer of (c) propylene (d) ethene
(a) acrylic acid

ANSWER KEY
Exercise 1 : NCERT Based Topic-wise MCQs
1 (d) 7 (d) 13 (b) 19 (a) 25 (b) 31 (a) 37 (b) 43 (d) 49 (a) 55 (c)
2 (c) 8 (d) 14 (d) 20 (b) 26 (b) 32 (a) 38 (b) 44 (d) 50 (b) 56 (a)
3 (c) 9 (c) 15 (b) 21 (d) 27 (a) 33 (c) 39 (a) 45 (c) 51 (b) 57 (b)
4 (d) 10 (a) 16 (b) 22 (a) 28 (b) 34 (a) 40 (b) 46 (c) 52 (d) 58 (d)
5 (a) 11 (d) 17 (a) 23 (d) 29 (b) 35 (b) 41 (c) 47 (b) 53 (b) 59 (a)
6 (b) 12 (d) 18 (b) 24 (b) 30 (b) 36 (b) 42 (b) 48 (a) 54 (b) 60 (d)
Exercise 2 : NCERT Exemplar & Past Year MCQs
1 (d) 4 (c) 7 (a) 10 (c) 13 (a) 16 (b) 19 (b) 22 (d)
2 (a) 5 (a) 8 (d) 11 (a) 14 (d) 17 (a) 20 (d)
3 (b) 6 (b) 9 (d) 12 (d) 15 (a) 18 (c) 21 (d)
Exercise 3 : Problem Solving Skill Enhancer MCQs
1 (d) 3 (b) 5 (a) 7 (d) 9 (a) 11 (d) 13 (c) 15 (a)
2 (b) 4 (b) 6 (a) 8 (d) 10 (c) 12 (b) 14 (b)
Chemistry in Everyday
30 Life
Trend Buster NEET & JEE Main

Number of Questions from 2020-15 5 4 Minimum 1 Question has been asked


Weightage 1.5% 2.2% every year.

The most Important Concepts that Cover Maximum number of Questions asked in past 6 years.

Therapeutic action of drug 2 1


Drug structure — 2

Less Important Concepts that Cover 1 or 2 Questions asked in past 6 years.

Cleansing agents 2 1
Drugs and their classification 1 —

NEET JEE

2020 Drug structure / Cleansing Structure of chloramphenical / 1 Easy 1 Difficult


agents Cationic Detergent
2019 Drug and their classification / Broad and narrow spectrum 1 Easy 1 Difficult
Drug structure antibiotics
2018 — — — — — —
2017 Therapeutic action of drugs Therapeutic action of drug 1 Easy —
2016 Cleansing agents / Anionic detergent / 1 Easy 1 Average
Therapeutic action of drug Analgesic drug
2015 Therapeutic action of drug / Antacid drug / 1 Average 1 Easy
Cleansing agent Soaps and detergents
EBD_8350
462 CHEMISTRY
CHEMISTRY IN EVERYDAY LIFE 463
EBD_8350
464 CHEMISTRY

Problem Solving Tips/ Tricks/ Points to Remember

4 0.2% solution of phenol acts as antiseptic and its 1% 4 Antacids are substances that decrease gastric acidity by
solution acts as disinfectant. neutralising hydrochloric acid.
4 Dettol is an antiseptic. It is a mixutre of chloroxylenol and (i) Systemic antacids : Systemic antacids like sodium
terpineol in a suitable solvent. bicarbonate decrease acidity but cause systemic
Bithional is an antiseptic which is generally added to alkalosis due to absorption of bicarbonate. Hence
medicated soaps to reduce the odour produced by acid-base balance is disturbed.
bacterial decomposition of organic matter on the skin. (ii) Non-systemic antacids : These may contain one
4 The chemical substances used to cure mental diseases or more of the following compounds : Al(OH)3,
are called tranquilizers. Mg(OH)2, MgCO3, CaCO3, magnesium trisilicate,
(i) Hypnotics : These are also known as tranquilizers magaldrate etc. These antacids are not absorbed
and are used to reduce mental tension and anxiety. after administration and hence acid-base
These induce sleep. These are components of imbalance is minimal.
sleeping pills (sedatives)
4 The most common preservative used is sodium benzoate,
C6H5COONa. It is metabolized by conversion to hippuric
O O
Ex. || C6H5 || acid, C6H5CONHCH2COOH which ultimately is excreted
N–H | in the urine. Salts of propionic acid and sorbic acid are
N–H
C2H5 also used as preservatives.
||

||

O N O N 4 The first popular artificial sweetener was saccharin.


| | Saccharin is approximately 600 times sweeter than cane
H H
Barbituric acid Luminal sugar.
Aspartame is unstable at cooking temperatures, limiting
its use as a sugar substitute to cold foods and soft drinks.
(ii) Non Hypnotics : They reduce tension and anxiety. Alitame is more stable than aspartame during cooking.
These do not induce sleep. Equanil is also an One potential problem with alitame and similar type of
important tranquilizer used in depression and high-potency sweetners is the difficulty in controlling
hypertension. sweetness of food. Sucralose is predicted to become a
Ex.Chlordiazepoxide, Meprobamate. great commercial success.
4 Chemical substances which are used to control pregnancy 4 Cationic detergents :
in women are called anti-fertility drugs e.g., cetyltrimethylammonium bromide, i.e.,
For example, norethindrone, mestranol
4 The drugs which have been used to fight allergy are CH3
+ –
called antihistamines. These are so called because they CH3 – (CH2)15 – N – CH3 Br
check the production of histamines. Cetrizine,
Chloropheniramine, Promethazine hydrochloride etc. CH3
Cetyl trimethyl ammonium bromide
(cationic detergent used in hair conditioners)
CHEMISTRY IN EVERYDAY LIFE 465

Exercise 1 : NCERT Based Topic-wise MCQs


(a) (ii) only (b) (iii) only
Topic 1 : Drugs and their Classification, Drug-Target Interaction
(c) (i) and (iii) (d) (ii) and (iii)
1. The use of chemicals for treatment of diseases is called as
(a) isothermotherapy (b) angiotherapy Topic 2 : Therapeutic Action of Different Classes of Drugs
(c) physiotherapy (d) chemotherapy
2. Which of the following method of classification of drugs H
is useful for medicinal chemists? N
(a) On the basis of molecular targets. 7. The drug is an
(b) On the basis of chemical structure. CH2
N
(c) On the basis of drug action.
(d) All of these. CH2 NH2
3. Which of the following statements is true about the
(a) Antacid (b) Analgesic
catalytic activity of enzyme?
(a) Enzyme holds the substrate for a biochemical reaction. (c) Antimicrobial (d) Antiseptic
(b) Enzyme binds substrate through a variety of 8. Which one of the following is employed as a tranquilizer?
interactions such as ionic bonding, hydrogen (a) Naproxen (b) Tetracycline
bonding, van der Waal’s interaction or dipole – dipole (c) Chlorpheninamine (d) Equanil
interaction. 9. Terfenadine is commonly used as a/an
(c) Enzyme provides functional group that will attack the (a) tranquilizer (b) antihistamine
substrate and carry out biochemical reaction. (c) antimicrobial (d) antibiotic
(d) All of the above. 10. Which one of the following is not a tranquilizer?
4. Which of the following statements are incorrect about (a) Equanil (b) Veronal
receptor proteins? (c) Salvarsan (d) Serotonin
(i) Majority of receptor proteins are embedded in the 11. Tranquillizers are substances used for the treatment of
cell membranes. (a) cancer (b) AIDS
(ii) The active site of receptor proteins opens on the (c) mental diseases (d) physical disorders
inside region of the cell. 12. The drug used as an antidepressant is
(iii) Chemical messengers are received at the binding sites (a) Luminol (b) Tofranil
of receptor proteins. (c) Mescaline (d) Sulphadiazine
(iv) Shape of receptor doesn’t change during attachment 13. Barbituric acid and its derivatives are well known
of messenger. (a) antipyretics (b) analgesics
(a) (i), (ii) and (iii) (b) (ii) and (iv) (c) antiseptics (d) tranquillizers
(c) (ii), (iii) and (iv) (d) (i) and (iv) 14. Which of the following is a hypnotic drug?
5. Assertion : Non-competitive inhibitor inhibits the catalyic (a) luminal (b) salol
activity of enzyme by binding with its active site. (c) catechol (d) chemisol
Reason : Non-competitive inhibitor changes the shape of the 15. Aspirin is
active site in such a way that substrate can’t recognise it. (a) antibiotic (b) antipyretic
(a) Assertion is correct, reason is correct; reason is a (c) sedative (d) psychedelic
correct explanation for assertion. 16. Sulpha drugs are used for
(b) Assertion is correct, reason is correct; reason is not (a) precipitating bacteria
a correct explanation for assertion (b) removing bacteria
(c) Assertion is correct, reason is incorrect (c) decreasing the size of bacteria
(d) Assertion is incorrect, reason is correct. (d) stopping the growth of bacteria
6. Which of the following is not correctly matched? 17. Which one among the following is not an analgesic?
(i) Proteins that are – Receptors (a) Ibuprofen (b) Naproxen
crucial to body’s (c) Aspirin (d) Valium
communication process. 18. Salol can be used as
(ii) Drugs that mimic – Antagonists (a) antiseptic (b) antipyretic
the natural messenger by (c) analgesic (d) None of these
switching on the receptor. 19. Various phenol derivatives, tincture of iodine (2 – 3%) I2 in
(iii) Drugs that bind to – Agonists (water / alcohol) and some dyes like methylene blue are
the receptor site and inhibit (a) antiseptics (b) disinfectants
its natural function. (c) analgesics (d) antipyretics
EBD_8350
466 CHEMISTRY

20. Streptomycin is effective in the treatment of Reason : Sedatives are used to suppress the activities of
(a) tuberculosis (b) malaria central nervous system.
(c) typhoid (d) cholera (a) Assertion is correct, reason is correct; reason is a
21. An antibiotic with a broad spectrum correct explanation for assertion.
(a) kills the antibodies (b) Assertion is correct, reason is correct; reason is not
a correct explanation for assertion
(b) acts on a specific antigen
(c) Assertion is correct, reason is incorrect
(c) acts on different antigens
(d) Assertion is incorrect, reason is correct.
(d) acts on both the antigens and antibodies 32. Antiseptics and disinfectants either kill or prevent growth
22. Which of the following is not an antiseptic drug? of microorganisms. Identify which of the following
(a) Iodoform (b) Dettol statements is not true:
(c) Gammexene (d) Genatian violet (a) Chlorine and iodine are used as strong disinfectants.
23. Veronal, a barbiturate drug is used as (b) Dilute solutions of boric acid and hydrogen peroxide
(a) anaesthetic (b) sedative are strong antiseptics.
(c) antiseptic (d) None of these (c) Disinfectants harm the living tissues.
24. A drug effective in the treatment of pneumonia, bronchitis, (d) A 0.2% solution of phenol is an antiseptic while 1%
etc, is solution acts as a disinfectant.
33. Novestrol is an
(a) streptomycin (b) chloramphenicol
(a) antibiotic (b) analgesic
(c) penicillin (d) sulphaguanidine
(c) antacid (d) antifertility drug
25. Commonly used antiseptic 'Dettol' is a mixture of 34. Which of the following is an essential component of
(a) o-chlorophenozylenol + terpeneol sleeping pills?
(b) o-cresol + terpeneol (a) Analgesics (b) Tranquilizers
(c) phenol + terpeneol (c) Antihistamines (d) Both (b) and (c)
(d) chloroxylenol + terpeneol 35. Which type of drugs inhibit the enzymes which catalyse
26. Chloroamphenicol is an : the degradation of noradrenaline?
(a) antifertility drug (a) Narcotic analgesics
(b) antihistaminic (b) Antacids
(c) antiseptic and disinfectant (c) Antidepressants
(d) antibiotic-broad spectrum (d) Non–narcotic analgesic.
27. The drug which is effective in curing malaria is 36. Which of the following is/are example(s) of narcotic
(a) quinine (b) aspirin (c) analgin (d) equanil analgesics?
28. The structure given below is known as (a) Morphine (b) Heroin
(c) Codeine (d) All of these
O 37. Which of the following was the first effective treatment
P H H S CH3 discovered for syphilis?
CH 2 - C - NH
CH3 (a) Penicillin (b) Arsphenamine
N COOH (c) Chloramphenicol (d) Sulphanilamide
O H 38. Match the columns
(a) Penicillin F (b) Penicillin G Column-I Column-II
(c) Penicillin K (d) Ampicillin (A) Sodium perborate (p) Disinfectant
29. Assertion : Tetracycline is a broad spectrum antibiotic. (B) Chlorine (q) Antiseptic
Reason : Tetracycline is effective against a number of (C) Bithional (r) Milk bleaching agent
types of bacteria, large viruses and typhus fever. (D) Potassium stearate (s) Soap
(a) Assertion is correct, reason is correct; reason is a
(a) A – (p), B – (q), C – (r), D – (s)
correct explanation for assertion.
(b) Assertion is correct, reason is correct; reason is not (b) A – (q), B – (r), C – (s), D – (p)
a correct explanation for assertion (c) A – (r), B – (p), C – (q), D – (s)
(c) Assertion is correct, reason is incorrect (d) A – (s), B – (p), C – (q), D – (r)
(d) Assertion is incorrect, reason is correct. 39. Match the columns
30. Bithional is an example of Column-I Column-II
(a) disinfectant (b) antiseptic (A) Ranitidine (p) Tranquilizer
(c) antibiotic (d) analgesic
(B) Furacine (q) Antibiotic
31. Assertion : Sedatives are given to patients who are
mentally agitated and violent. (C) Phenelzine (r) Antihistamine
(D) Chloramphenicol (s) Antiseptic
CHEMISTRY IN EVERYDAY LIFE 467

(a) A – (r), B – (s), C – (p), D – (q) (c) X = Narrow spectrum antibiotics.


(b) A – (s), B – (p), C – (q), D – (r) Y = Limited spectrum antibiotics.
(c) A – (p), B – (q), C – (r), D – (s) (d) X = Narrow spectrum antibiotics.
(d) A – (q), B – (r), C – (s), D – (p) Y = Broad spectrum antibiotics.
47. Which of the following is an example of synthetic
40. Assertion : Equanil is a tranquilizer. progesterone derivative which is most widely used as
Reason : Equanil is used to cure depression and antifertility drug?
hypertension. (a) Norethindrone (b) Novestrol
(a) Assertion is correct, reason is correct; reason is a (c) Ethynylestradiol (d) All of these
correct explanation for assertion.
(b) Assertion is correct, reason is correct; reason is not Topic 3: Chemicals in Food
a correct explanation for assertion
(c) Assertion is correct, reason is incorrect 48. Which is correct about saccharin?
(d) Assertion is incorrect, reason is correct. O
41. Assertion : Antiseptics are applied to living tissues. C
Reason : Iodine is a powerful antiseptic. (a) It is NH
(a) Assertion is correct, reason is correct; reason is a SO2
correct explanation for assertion. (b) It is 600 times sweeter than sugar
(b) Assertion is correct, reason is correct; reason is not (c) It is used as sweetening agent
a correct explanation for assertion (d) All of these
(c) Assertion is correct, reason is incorrect 49. Arrange the following artificial sweeteners in increasing
(d) Assertion is incorrect, reason is correct. order of their sweetness value?
42. Among the following antihistamines, which are antacids (a) Sucralose < Saccharin < Alitame < Aspartame
(i) Ranitidine (ii) Brompheniramine (b) Aspartame < Saccharin < Sucralose < Alitame
(iii) Terfenadine (iv) Cimetidine (c) Aspartame < Sucralose < Saccharine < Alitame
(d) Saccharine < Aspartame < Sucralose < Alitame
(a) (i) and (iii) (b) (i), (ii) and (iv)
50. Which of the following artificial sweetener does not
(c) (i) and (iv) (d) (ii) and (iii)
provide calories?
43. Select the incorrect statement. (a) Alitame (b) Aspartame
(a) Equanil is used to control depression and (c) Sucralose (c) Both (b) and (c)
hypertension. 51. Structuraly biodegradable detergent should contain
(b) Mifepristone is a synthetic steroid used as “morning (a) normal alkyl chain (b) branched alkyl chain
after pill”. (c) phenyl side chain (d) cyclohexyl side chain
(c) 0.2 percent solution of phenol is an antiseptic while 52. Substance used for the preservation of coloured fruit juices
its 1.0 percent solution is a disinfectant. is
(d) A drug which kills the organism in the body is called (a) benzene (b) benzoic acid
bacteriostatic. (c) phenol (d) sodium meta bisulphite
44. Bactericidal antibiotics are those which
(a) have inhibitory effect on microbes. Topic 4: Cleansing Agents
(b) have killing effect on microbes.
(c) have both inhibitory and killing effect on microbes. 53. Detergents are prepared by the action of H2SO4 on
(d) intervene in metabolic process of microorganism. which of the following?
45. Which of the following antibiotics is not correctly (a) Cholesterol (b) Lauryl alcohol
classified? (c) Cyclohexanol (d) p-Nitrophenol
Bactericidal Bacteriostatic 54. Sodium alkyl benzene sulphonate is used as
(A) Penicillin Erythromycin (a) soap (b) fertilizers
(B) Aminoglycosides Tetracycline (c) pesticides (d) detergents
(C) Chloramphenicol Ofloxacin 55. Which of the following represents a synthetic detergent?
(a) A and B (b) C only (c) B and C (d) B only (a) C15H31COOK
46. Antibiotics that are effective against Gram-positive or (b) CH3[CH2]16COONa
Gram-negative bacteria X. Antibiotics that are effective
against a single organism or disease are Y (c) C12H25 SO3Na
What is X and Y ?
(a) X = Broad spectrum antibiotics. (d) None of these
Y = Narrow spectrum antibiotics. 56. Which of the following represents soap
(b) X = Broad spectrum antibiotics. (a) C17H35COOK (b) C17H35COOH
Y = Limited spectrum antibiotics. (c) C15H31COOH (d) (C17H35COO)2Ca
EBD_8350
468 CHEMISTRY

57. Polyethyleneglycols are used in the preparation of which (C) C17H35COO–Na+ +


type of detergents? Na2CO3 + Resin (r) Hair conditioners
(a) Cationic detergents (b) Anionic detergents (D) CH3(CH2)16COO(CH2CH2O)nCH2CH2OH
(c) Non-ionic detergents (d) Soaps (s) Toothpaste
58. Which of the following statement is incorrect? (a) A – (p), B – (q), C – (r), D – (s)
(a) Potassium soaps are soft to the skin than sodium (b) A – (q), B – (r), C – (p), D – (s)
soaps.
(c) A – (r), B – (s), C – (q), D – (p)
(b) Shaving soaps contain ethanol to prevent rapid (d) A – (p), B – (r), C – (q), D – (s)
drying.
60. Match the columns
(c) Builders like Na2CO3 and Na3PO4 make soaps act more
Column -I Column -II
rapidly.
(A) Toilet soap (p) Made by beating
(d) Hard water contains Ca2+ and Mg2+ ions which form tiny air bubbles before
insoluble Ca2+ and Mg2+ soaps and separate out as their hardening.
scum.
(B) Transparent soap (q) Contain glycerol
59. Match the columns to prevent rapid drying.
Column-I Column-II (C) Shaving soaps (r) Prepared by
+ using better grades of
CH3 fats and oils.

(A) CH3(CH2)15–N–CH3 Br (p) Dishwashing powder (D) Soaps that float in (s) Made by dissolving
water the soap in ethanol
CH3
and then evaporating
excess alkali.
– +
(B) CH3– (CH2)11 SO3 Na (a) A – (s), B – (p), C – (q), D – (r)
(b) A – (r), B – (s), C – (q), D – (p)
(q) Laundry soap (c) A – (r), B – (q), C – (p), D – (s)
(d) A – (q), B – (s), C – (p), D – (r)

NCERT Exemplar MCQs (c) on the basis of molecular targets


(d) on the basis of pharmacological effect
1. Which of the following statements is not correct? 5. Which of the following statements is correct?
(a) Some antiseptics can be added to soaps (a) Some tranquilizers function by inhibiting the enzymes
(b) Dilute solutions of some disinfectants can be used which catalyse the degradation of noradrenaline
as antiseptic (b) Tranquilizers are narcotic drugs
(c) Disinfectants are antimicrobial drugs (c) Tranquilizers are chemical compounds that do not
(d) Antiseptic medicines can be ingested affect the message transfer from nerve to receptor
2. Which is the correct statement about birth control pills? (d) Tranquilizers are chemical compounds that can relieve
(a) Contain estrogen only pain and fever
(b) Contain progesterone only 6. Salvarsan is arsenic containing drug which was first used
(c) Contain a mixture of estrogen and progesterone for the treatment of ............... .
derivatives (a) syphilis (b) typhoid
(d) Progesterone enhances ovulation (c) meningitis (d) dysentery
3. Which statement about aspirin is not true? 7. A narrow spectrum antibiotic is active against .......... .
(a) Aspirin belong to narcotic analgesics (a) gram positive or gram negative bacteria
(b) It is effective in relieving pain (b) gram negative bacteria only
(c) It has antiblood clotting action (c) single organism or one disease
(d) It is a neurologically active drug (d) both gram positive and gram negative bacteria
4. The most useful classfication of drugs for medicinal 8. The compound that causes general antidepressant action
chemists is .......... . on the central nervous system belong to the class of ......
(a) on the basis of chemical structure (a) analgesics (b) tranquilizers
(b) on the basis of drug action (c) narcotic analgesics (d) antihistamines
CHEMISTRY IN EVERYDAY LIFE 469

9. Compound which is added to soap to impart antiseptic Past Year MCQs


properties is ..........
(a) sodium lauryl sulphate 19. Artificial sweetner which is stable under cold conditions
(b) sodium dodecylbenzenesulphonate only is : [AIPMT 2014, A]
(c) rosin (a) Saccharine (b) Sucralose
(d) bithional (c) Aspartame (d) Alitame
10. Equanil is ........... 20. Bithional is generally added to the soaps as an additive to
(a) artificial sweetener (b) tranquilizer function as a/an [AIPMT 2015, A]
(c) antihistamine (d) antifertility drug (a) dryer (b) buffering agent
11. Which of the following enhances leathering property of (c) antiseptic (d) softner
soap? 21. Which of the following compounds is not an antacid ?
(a) Sodium carbonate (b) Sodium rosinate [JEE M 2015, A]
(c) Sodium strearate (d) Trisodium phosphate (a) Phenelzine (b) Ranitidine
12. Glycerol is added to soap. It functions .............. (c) Aluminium hydroxide (d) Cimetidine
(a) as a filler (b) to increase leathering 22. Which of the following is an analgesic? [NEET 2016, A]
(c) to prevent rapid drying (d) to make soap granules (a) Novalgin (b) Penicillin
13. Which of the following is an example of liquid dishwashing (c) Streptomycin (d) Chloromycetin
detergent? 23. Which of the following is an anionic detergent?
[JEE M 2016, A]
(a) CH3 ( CH 2 )10 — CH 2 OSO3- Na +
(a) Cetyltrimethyl ammonium bromide.
(b) Glyceryl oleate.
(b) C9H19 O ( CH 2 – CH 2 – O )–5 – CH 2CH2OH
(c) Sodium stearate.
– +
(d) Sodium lauryl sulphate.
(c) CH 3 SO3 Na 24. Mixture of chloroxylenol and terpineol acts as:
[NEET 2017, S]
+ (a) antiseptic (b) antipyretic
CH3
| (c) antibiotic (d) analgesic
(d) CH3 ( CH)15 — N – CH3 Br - 25. Among the following, the narrow spectrum antibiotic is:
|
CH3 [NEET 2019, C]
(a) penicillin G (b) ampicillin
14. Polyethyleneglycols are used in the preparation of which (c) amoxycillin (d) chloramphenicol
type of detergents? 26. The correct match between Item-I and Item-II is:
(a) Cationic detergents [JEE M 2019, A]
(b) Anionic detergents Item-I (drug) Item-II (test)
(c) Non - ionic detergents
A Chloroxylenol P Carbylamine test
(d) Soaps
B Norethindrone Q Sodium
15. Which of the following is not a target molecule for drug
function in body? hydrogen
(a) Carbohydrates (b) Lipids carbonate test
(c) Vitamins (d) Proteins C Sulphapyridine R Ferric chloride test
16. Which of the following statements is not true about D Penicillin S Bayer’s test
enzyme inhibitors? (a) A ® R ; B ® P ; C ® S ; D ® Q
(a) Inhibit the catalytic activity of the enzyme (b) A ® Q ; B ® S ; C ® P ; D ® R
(b) Prevent the binding of substrate (c) A ® R ; B ® S ; C ® P ; D ® Q
(c) Generally a strong covalent bond is formed between (d) A ® Q ; B ® P ; C ® S ; D ® R
an inhibitor and an enzyme 27. Which of the following is a cationic detergent?
(d) Inhibitors can be competitive or non – competitive
[NEET 2020, S]
17. Which of the following chemicals can be added for
sweetening of food items at cooking temperature and does (a) Sodium stearate
not provides calories? (b) Cetyltrimethyl ammonium bromide
(a) Sucrose (b) Glucose (c) Sodium dodecylbenzene sulphonate
(c) Aspartame (d) Sucralose
(d) Sodium lauryl sulphate
18. Which of the following will not enhance nutritional value
of food? 28. The number of chiral carbons in chloramphenicol
(a) Minerals (b) Artificial sweetners is ________. [NV, JEE M 2020, S]
(c) Vitamins (d) Amino acids
EBD_8350
470 CHEMISTRY

1. Aspirin can be prepared by the reaction of 5. Which is incorrect?


(a) Salicyldehyde with acetic anhydride in presence of (a) Novestrol - Antifertility
H2SO4 (b) Serotonine - Tranquilizer
(b) Salicylic acid with methanol in presence of H2SO4 (c) Narrow spectrum - Chloramphenicol
(c) Salicylic acid with acetic anhydride in presence of (d) Ranitidine - antacid
H2SO4 6. Which of the following is a diamine?
(d) Cinnamic acid with acetic anhydride in presence of (a) Dopamine (b) Histamine
H2SO4
(c) Meprobamate (d) Chlorphenamine
2. The correct structure of the drug paracetamol is
7. Which of the following is an alkaloid?
OCH3 OH (a) Nicotine (b) Piperine
(c) Cocaine (d) All of these
8. Which of the following can possibly be used as analgesic
(a) (b)
without causing addiction and modification?
(a) morphine
(b) N-acetyl-para-aminophenol
CONH2 NHCOCH3
(c) diazepam
(d) tetrahydrocatenol
OCH3 OH 9. The following compound is used as

O
(c) (d)
O – C – CH3

CONH2 COCH3 COOH


3. The insecticide containing 99% g – isomer of benzene (a) an anti-inflammatory compound
hexachloride is known as
(b) analgesic
(a) lindane (b) TNT
(c) malathion (d) methoxychlor (c) hypnotic
4. Chemically heroin is (d) antiseptic
(a) morphine monoacetate 10. An antibiotic contains nitro group attached to aromatic
(b) morphine dibenzoate nucleus. It is
(c) morphine diacetate (a) penicillin (b) streptomycin
(d) morphine monobenzoate (c) tetracycline (d) chloramphenicol

ANSW ER KEY
Exercise 1 : NCERT Based Topic-wise MCQs
1 (d) 7 (a) 13 (d) 19 (a) 25 (d) 31 (a) 37 (b) 43 (d) 49 (b) 55 (c)
2 (a) 8 (d) 14 (a) 20 (a) 26 (d) 32 (b) 38 (c) 44 (b) 50 (c) 56 (a)
3 (d) 9 (b) 15 (b) 21 (c) 27 (a) 33 (d) 39 (a) 45 (b) 51 (b) 57 (c)
4 (b) 10 (c) 16 (d) 22 (c) 28 (b) 34 (b) 40 (a) 46 (c) 52 (b) 58 (b)
5 (d) 11 (c) 17 (d) 23 (b) 29 (a) 35 (c) 41 (b) 47 (a) 53 (b) 59 (c)
6 (d) 12 (b) 18 (a) 24 (c) 30 (b) 36 (d) 42 (c) 48 (d) 54 (d) 60 (b)
Exercise 2 : NCERT Exemplar & Past Year MCQs
1 (d) 4 (c) 7 (a) 10 (b) 13 (b) 16 (c) 19 (c) 22 (a) 25 (a) 28 (2.00)
2 (c) 5 (a) 8 (b) 11 (b) 14 (c) 17 (d) 20 (c) 23 (d) 26 (c)
3 (a) 6 (a) 9 (d) 12 (c) 15 (c) 18 (b) 21 (a) 24 (a) 27 (b)
Exercise 3 : Problem Solving Skill Enhancer MCQs
1 (c) 2 (b) 3 (a) 4 (c) 5 (c) 6 (c) 7 (d) 8 (b) 9 (b) 10 (d)
ANSWER KEY AND EXPLANATIONS
EBD_8350
1 Some Basic Concepts of Chemistry
Exercise - 1 Hence, both law of conservation of mass and constant
composition is obeyed.
1. (d) Smallest and largest amount of energy respectively 14. (c) The H : O ratio in water is fixed, irrespective of its
are eV and L-atm. source. Hence, it explains law of constant composition.
1 eV = 1.6 × 10–19 J 15. (a) Law of constant proportions according to which a
pure chemical compound always contains same elements
1L -atm = 101.325 J
combined together in the same definite proportion of
5 5 weight.
2. (c) °C = (°F - 32 ) = ( 41 - 32 ) = 5 °C
9 9 16. (c) Equal moles of different substances contain same
It will be cold. number of constituent particles but equal weights of
3. (b) Given P = 0.0030 m, Q = 2.40 m & R = 3000 m. In different substances do not contain the same number of
P (0.0030) initial zeros after the decimal point are not consituent particles.
significant. Therefore, significant figures in P(0.0030) are 17. (d) SnCl2 SnCl4
2. Similarly in Q (2.40) significant figures are 3 as in this 119 : 2 × 35.5 119 : 4 × 35.5
case final zero is significant. In R = (3000) all the zeros are Chlorine ratio in both compounds is
significant hence, in R significant figures are 4 because = 2 × 35.5 : 4 × 35.5 = 1 : 2
they come from a measurement. 18. (c)
4. (d) We know that all non-zero digits are significant and 19. (b) Law of multiple proportion. As the ratio of oxygen
the zeros at the beginning of a number are not significant. which combine with fix weights of 1 g of nitrogen bears a
Therefore number 161 cm, 0.161 cm and 0.0161cm have 3, 3 simple whole number ratio
and 3 significant figures respectively. 0.57 : 1.12 : 1.703 = 1 : 2 : 3
5. (c) On calculation we find 20. (c) According to Avogadro's law "equal volumes of all
gases contain equal number of molecules under similar
(29.2 – 20.2)(1.79 ´ 105 ) conditions of temperature and pressure". Thus if 1 L of
= 1.17×106
1.37 one gas contains N molecules, 2 L of any other gas under
As the least precise number contains 3 significant the same conditions of temperature and pressure will
figures therefore, answers should also contains 3 contain 2N molecules.
significant figures. 21. (b) PV = nRT, Here V, R and T are constant. Thus, P µ n.
6. (a) 2.6 has two significant figures. Number of as (n) and number of molecules are equal to
0.260 has three significant figures. keep pressure contant. Thus molecular mass should be
also equal. Hence, X is CO.
0.002600 has four significant figures.
22. (d) Avogadro’s law is independent of the reactive or
2.6000 has five significant figures.
unreactive nature of the gases.
7. (d) Both Y and X are neither precise nor accurate as the
According to Avogadro’s law equal volumes of gases
two values in each of them are not close. With respect to
at the same temperature and pressure should contain
X & Y, the values of Z are close & agree with the true equal number of molecules.
value. Hence, both precise & accurate.
23. (c) For statement (i) : H, O, C, N = All have different
8. (a) Out of two 3.929 g is more accurate and will be reported chemical properties.
as 3.93 after rounding off. For statement (ii) : It is true as per Dalton’s postulate.
9. (a) 1 mL = 3.12 g (given) For statement (iii) : N : O = 1 : 1 (NO)
1.5 mL = 4.68 g = 4.7 g For statement (iv) : Dalton’s postulates says, atoms
10. (c) can neither be created nor destroyed.
11. (c) Formation of CO and CO2 illustrates the law of 24. (c) This is Avogadro’s hypothesis.
multiple proportion that is constant mass of C reacts with According to this, equal volume of all gases contain
different masses of oxygen. These masses here bears equal no. of molecules under similar condition of
simple ratio of 1 : 2. temperature and pressure.
12. (a) 25. (c)
mass of H 2 combined 4 1 26. (b) Average atomic mass of neon
13. (c) I experiment : = =
mass of O2 combined 32 8 = 20 × 0.9051 + 21 × 0.0027 + 22 × 0.0922 = 20.187 u
mass of H 2 combined 50 1 27. (a) (78.9183361) × (0.5069) + (80.916289) × (0.4931) = 79.9
II experiment : = =
mass of O 2 combined 400 8
EBD_8350
472 CHEMISTRY

28. (a) Mass of oxygen atom is 16 amu, and 1 amu 842


= 1.66 × 10–24 g. Mass of one atom = = 1.4 ´ 10 - 21 g
23
6.023 ´ 10
= 16 × 1.66 × 10–24 g = 2.656 × 10 g.
42. (a) No. of molecules in different cases
2.324784 ´ 10-23 (a) Q 22.4 litre at STP contains
29. (d) = 14 amu = 6.023 × 1023 molecules of H2
1.66056 ´ 10–24
Where 1.66056 × 10–24 is equal to one atomic mass (amu) 15
\ 15 litre at STP contains = ´ 6.023 ´ 10 23
30. (b) Molecular masses of different molecules. 22.4
31. (b) Gram molecular weight of CO = 12 + 16 = 28 g = 4.03 × 1023 molecules of H2
6.023 × 1023 molecules of CO weigh 28 g (b) Q 22.4 litre at STP contains
= 6.023×1023 molecules of N2
28 = 4.65 ´10-23 g 5
1 molecule of CO weigh = Q 5 litre at STP contains =
6.02´1023 ´ 6.023 ´ 10 23
22.4
32. (b) Molecular weight of SO2 = 32 + 2 × 16 = 64 = 1.344 × 1023 molecules of N2
64 g of SO2 occupies 22.4 litre at STP (c) Q 2 g of H2= 6.023×1023 molecules of H2
22.4 0.5
240 g of SO2 occupies = ´ 240 = 84 litre at STP Q 0.5 g of H2= ´ 6.023 ´10 23
64 2
33. (a) 1 mol CCl4 vapour = 12 + 4 × 35.5 = 1.505 × 1023 molecules of H2
= 154 g º 22.4 L at STP (d) Similarly 10 g of O2 gas
10
\ Density =
154
gL-1 = 6.875 gL-1 = ´ 6.023 ´ 10 23 molecules of O2
22 .4 32
34. (b) 2g of H2 means one mole of H2, hence contains = 1.88 × 1023 molecules of O2
6.023 × 1023 molecules. Others have less than one mole, so Thus, (a) will have maximum number of molecules
have less no. of molecules. 43. (b) The number of atoms in 0.1 mole of a triatomic gas
35. (c) M. Wt of Na2SO4.10H2O is 322 g which contains 224 = 0.1 × 3 × 6.023 × 1023.
g oxygen. = 1.806 × 1023
\ 32.2 g will contain 22.4 g oxygen. 44. (b) 6.02 × 1023 molecules of CO =1mole of CO
36. (d) At S.T.P. 22.4 litre of gas contains 6.023 × 1023 6.02 × 1024 CO molecules = 10 moles CO
molecules = 10 g atoms of O = 5 g molecules of O2
\ Molecules in 8.96 litre of gas 4.4
45. (a) 4.4 g CO2 = = 0.1 mol CO2 (mol. wt. of CO2 = 44)
6.023 ´10 23 ´ 8.96 44
= = 24.08 ´10 22
22.4 = 6 × 1022 molecules = 2 × 6 × 1022 atoms of O.
37. (b) Mass of one molecule of water 558.5
46. (a) Fe (no. of moles) = = 10 moles = 10NA atoms.
18 -23 -26 55.85
= = 3 ´10 g = 3 ´10 kg No. of moles in 60 g of C = 60/12 = 5 moles = 5NA atoms.
6.023 ´1023
38. (d) 1 molecule of CO2 has one atom of C and two atoms 47. (a) 21% of 1 litre is 0.21 litre.
of oxygen. 22.4 litres = 1 mole at STP
\ 1 mole of CO2 has = 6.02 × 1023 atoms of C 0.21
\ 0.21 litre = = 0.0093 mol
= 2 × 6.02 × 1023 atoms of O 22.4
39. (a) Given, V = 1.12 × 10–7 cm3 48. (b) Total atoms in 1 molecule of C12H22O11
22400 cm3 at NTP = 6.02 × 1023 molecules = 12 + 22 + 11 = 45
6.02 ´ 1023 \ Total atoms in 1 mole of C12H22O11
\ 1.12 × 10–7 cm3 at NTP = ´1.12 ´10-7 = 45 × 6.02 × 1023 atoms/mol.
22400
= 3.01 × 1012 molecules. 49. (c) Mass of 1 electron = 9.11 × 10–28 g
weight 50 \ Mass of 1 mole (6.02 × 1023) electrons
40. (d) No. of moles = = = 0.14 mole
mol. wt . 342 = 9.11 × 10–28 × 6.02 × 1023g
41. (b) Molecular weight of C60H122 = (12 × 60) + 122 = 842. = 55 × 10–5 g = 55 × 10–5 × 103 mg = 0.55 mg.
Therefore weight of one molecule 50. (d) We know that from the reaction H2 + Cl2 ® 2HCl that
the ratio of the volume of gaseous reactants and products
Molecular weight of C 60 H 122
= is in agreement with their molar ratio.The ratio of H2 : Cl2 :
Avagadro' s number HCl volume is 1: 1: 2 which is the same as their molar ratio.
SOME BASIC CONCEPTS OF CHEMISTRY 473

Thus volume of gas is directly related to the number of Molecular mass of the compound = 42
moles. Therefore, the assertion is false but reason is true. \ n = 42/14 = 3
51. (d) 1 Mole of Mg3(PO4)2 contains 8 mole of oxygen atoms
\ Hence molecular formula = C3H6
\ 8 mole of oxygen atoms º 1 mole of Mg3(PO4)2
58. (c) The acid with empirical formula CH2O2 is formic acid,
1
0.25 mole of oxygen atom º ´ 0.25 mole of Mg3(PO4)2 HCOOH.
8 59. (a)
–2
= 3.125 × 10 mole of Mg3(PO4)2
52. (c) \ Volume occupied by 1 gram water = 1 cm3 Atomic Atomic Simple
Element Percentage
weight ratio ratio
or Volume occupied by
38.71 3.23
6.023 ´ 1023 C 38.71 12 = 3.23 =1
molecules of water = 1 cm3 12 3.23
18 9.67 9.67
H 9.67 1 = 9.67 =3
1 1 3.23
[\ 1g water = moles of water]
18 100 - 51.62 3.23
Thus volume occupied by 1 molecule of water O (38.71 + 9.67) 16 = 3.23 =1
= 51.62 16 3.23
1 ´ 18
= cm 3 = 3.0×10–23 cm3. Thus empirical formula is CH3O.
6.023 ´ 10 23
60. (c) Element Ratio Atomic Simplest
53. (d) At NTP, 22400 c c of N2O = 6.02 × 1023 molecules
ratio ratio
6.02 ´ 1023 9 3
\ 1 cc N2O = molecules C 9 = 3
22400 12 4
1
3 ´ 6.02 ´ 10 23 1.8 H 1 =1 4
= atoms = ´ 1022 atoms 1
22400 224 3.5 1
No. of electrons in a molecule of N 2O = 7 + 7 + 8 = 22 N 3.5 = 1
Hence no. of electrons 14 4
Empirical formula = C3H4N
6.02 ´10 23 23
= ´ 22 electrons = 1.32 ´ 10 (C3H4N)n = 108
22400 224 n ×(12 × 3 + 4 × 1 + 14) = 108
54. (b) If 1020 grains are distributed in one sec, 6.023 × 1023 n × (54) = 108
grains will be distributed in 108
n= =2
6.023 ´ 1023 ´ 1 54
= 1.673 hrs \ molecular formula = C6 H8N2
1020 ´ 60 ´ 60 61. (d) Q 18 g, H2O contains = 2 g H
1
55. (b) H2 + O ¾¾ ® H2 O 2
2 2 \ 0.72 g H2O contains = ´ 0.72 g = 0.08 g H
10g 64g 18
æ 10 ö æ 64 ö
Q 44 g CO2 contains = 12 g C
ç =5 mol ÷ ç = 2 mol ÷
è2 ø è 32 ø 12
In this reaction oxygen is the limiting agent. Hence amount \ 3.08 g CO2 contains = ´ 3.08 = 0.84 g C
44
of H2O produced depends on the amount of O2 taken 0.84 0.08
Q 0.5 mole of O2 gives H2O = 1 mol \ C:H= : = 0.07 : 0.08 = 7 : 8
12 1
\ 2 mole of O2 gives H2O = 4 mol \ Empirical formula = C7H8
56. (c) (a) Weight of H2 = mole × molecular wt. 62. (a) Let 100 g of compound be there.
= 0.2 × 2 = 0.4 g 35
Number of moles of Nitrogen = = 2.5
(b) 6.023 × 1023 = 1 mole 14
5
Thus 6.023 × 1022 = 0.1 mole Number of moles of Hydrogen = = 4.9
1.008
Weight of N2 = 0.1 × 28 = 2.8 g
60
(c) Weight of silver = 0.1 g Number of moles of Carbon = = 4.9
12.01
(d) Weight of oxygen = 32 × 0.1 = 3.2 g Since, 2.5 is the smallest value, division by it gives ratio
57. (a) Empirical formula of compound = CH2, Empirical N : H : C = 1 : 1.96 : 1.96 = 1 : 2 : 2
Empirical formula = C2H2N
mass = 14
Empirical formula weight = 2 × 12 + 2 + 14 = 40
EBD_8350
474 CHEMISTRY

Molecular mass = 80
6 ´ 10 –3
80 \ ppm = ´ 10 6 = 6.
n= 1000
40 72. (c)
Molecular formula = n (C2H2N) = 2 (C2H2N) = C4H4N2
73. (c) BaCO3 ® BaO + CO2
63. (b) Let 100 g of compound be there.
197 g
77.43g Q 197 g of BaCO3 released carbon dioxide
Number of moles of C = = 6.44
12.01g / mol = 22.4 litre at STP
7.53g 22.4
Number of moles of H = = 7.47 \ 1 g of BaCO3 released carbon dioxide = litre
1.008g / mol 197
\ 9.85 g of BaCO3 released carbon dioxide
15.05g
Number of moles of N = = 1.075 22.4
14.00g / mol = ´ 9.85 = 1.12 litre
1.075 is the smallest value, division by it gives a ratio of 197
C : H : N = 5.9 : 6.9 : 1 = 6 : 7 : 1 74. (b) Ba(OH)2 + CO2 ¾¾
® BaCO3 + H2 O
Empirical formula = C6H7N n mol n mol
Empirical formula weight = 6 × 12 + 7 + 14 = 93 n mol Ba(OH)2 = n mol BaCO3
Molecular mass \ 0.205 mol Ba (OH) 2 º 0.205 mol BaCO3
n= =1
Empirial formula weight Wt. of substance = No. of moles × Molecular mass
Molecular formula = 1 × C6H7N = C6H7N = 0.205 × 197.3 = 40.5 g
64. (a) Given, mass ratio is C:H:O (6:1:24) so, molar ratio will 75. (a) We know that
be 6/12:1/1:24/16 = 1:2:3 N2 + 3H2 ® 2NH3
therefore, HO-(C=O)-OH has molar ratio 1:2:3 28 g 6 g 34 g
65. (b) The equation for the formation of Al 2O3 can be 14 g 3 g 17 g
represented as Here given H2 is 3 kg and N2 is 20 kg but 3 kg of H2 can only
2Al + 3 / 2O 2 ¾¾ ® Al2 O3 react with 14 g of N2 and thus the obtained NH3 will be of
2 moles 1.5 moles 1 mole 17 kg.
Thus, 1 mole of alumina is obtained by the reaction of 1.5 76. (a) Let the amount of Na2CO3 present in the mixture be x
moles of oxygen and 2 moles of aluminium. Thus, the g. Na2SO4 will not react with H2SO4. 250 mL contains x g of
amount of aluminium Na2CO3, thus, 25 mL solution contains = Na2CO3.
= 2 × 27 g = 54 g. [mol. mass of Al = 27]
x
66. (b) Mg + 2 HCl ® MgCl 2 + H 2 ­ Moles of Na2CO3 = .
10 ´106
1 mole 1 mole
1 1 1
mole mole (12g of Mg = mol ) x 20 0.1
2 2 2 Thus = ´
10 ´ 106 1000 2
67. (c) C2H4 + 3 O2 ¾
¾® 2CO2 + 2H2O
28 g 96 g x 20 ´ 0.1 ´ 10
= \ x = 1.06g
Q 28 g of C2H4 undergo complete combustion by 53 1000
= 96 g of O2
1.06 ´ 100
\ 2.8 kg of C2H4 undergo complete combustion by \ Percentage of Na2CO3 = = 84.8%
= 9.6 kg of O2. 1.25
68. (c) According to stoichiometry, they should react as follow 77. (a) 5 M H2SO4 = 10 N H2SO4, (Q Basicity of H2SO4 = 2)
4NH 3 + 5O 2 ¾¾ ® 4NO + 6H 2 O N1V1 = N2V2,
4 mole of NH3 requires 5 mole of O2. 10 × 1 = N2 × 10 or N2 = 1 N
5 6.02 ´ 1020
1 mole of NH3 requires = = 1.25 mole of O2. 78. (b) Moles of urea present in 100 mL of sol.=
4
6.02 ´ 10 23
Hence, O2 is consumed completely.
69. (a) Molarity = Normality ÷ n factor = 0.2 ÷ 2 = 0.1 M 6.02 ´ 1020 ´ 1000
\M = = 0.01M
70. (c) Molarity = Normality 6.02 ´ 1023 ´ 100
Replaceable hydrogen atom [Q M = Moles of solute present in 1L of solution]
Q H2SO4 is dibasic acid. 79. (b) From the molarity equation.
M1V1 + M2V2 = MV
\ Molar solution of H2SO4 = 2N H2SO4
Let M be the molarity of final mixture,
Mass of solute M V + M 2V2
71. (b) ppm = ´ 106
Mass of solution M= 1 1 where V = V1 + V2
V
SOME BASIC CONCEPTS OF CHEMISTRY 475

480 ´ 1.5 + 520 ´ 1.2 No. of moles of solute


M = = 1.344 M 85. (a) Molarity (M) =
480 + 520 Volume of solution in litres
80. (a) Fe2O3 + 3CO ® 2Fe + 3CO2 Molarity µ nsolute
1 vol. 3 vol. 2 vol. 3 vol.
1 mol. 3 mol. 2 mol. 3 mol. 25
nNaOH = = 0.625
(Q vol% = mol%) 40
One gram mol of any gas occupies 22.4 litre at NTP.1 mol 25
nLiOH = = 1.04
of Fe2O3 requires 3 mol of CO for its reduction i.e., 1 mol 24
of Fe2O3 requires 3 × 22.4 litre or 67.2 dm 3 CO to get itself 25
reduced. nAl(OH)3 = = 0.32
(17 + 3 ´17)
163 25
81. (a) C57H110O6 + O2 ® 57CO2 + 55H2O nKOH = = 0.45
2 (39 + 17)
890 gram of fat produces = 990 gram of H2O 25
nB(OH)3 = = 0.403
(11 + 17 ´ 3)
450 gram fat produces = æç ö
990
´ 450 ÷ = 500.56 g of H2O
è 890 ø 26
86. (c) Wt. of NH3 = 26 g = g eq = 1.53 g eq
500.56 g 17
Moles of H2O = = 27.80 Vol. of soln. = 100 mL = 0.1 L
18 g mol
1.53
nC =
26g \ Normality = = 15.3 N
82. (d) = 2.16 0.1
12 g/mol
87. (b) Given N1 = 10N, V1 = 10 mL, N2 = 0.1N, V2 = ?
20 g
nO2 = = 0.625 N1V1 = N 2V2
32 g/mol
or 10 × 10 = 0.1 × V2
O2 will be a limiting reagent in reaction (i)
10 ´ 10
60g or V2 = , V2 = 1000 mL
nN 2 = = 2.14 0.1
28g/mol Volume of water to be added
nH 2 = 40 = V2 – V1 = 1000 – 10 = 990 mL.
According to balanced equation, 88. (c) N1V1 = N2V2
1 mol of N2 requires 3 mole of H2 (0.025 N) (0.050 L) = (N2) (0.025 L)
2.14 mol of N2 require 6.42 mol of H2 N2 = 0.05 N
N2 will be a limiting reagent in reaction (ii)
For acids, Normality = Molarity × basicity
100g H2SO4 is dibasic acid
nP4 = = 0.86; nO2 = 6.25
4 ´ 31 0.05
According to balanced equation \ Molarity = = 0.025 M
2
1 mol of P4 require 3 mol of O2 89. (a) A + 2HCl ® ACl2 + H2
0.86 mol of P4 require 2.58 mol of O2 x x
So, P4 is a limiting reagent in reaction (iii) mole =
15 15
83. (a) From molarity equation B+ 2 HCl ® BCl 2 + H 2
M1V1 + M2V2 = MV(total)
2-x 2-x
10 200 210 mole =
2´ + 0.5 ´ =M´ 30 30
1000 1000 1000 x 2-x 2.24 1
120 = M × 210 Mole of H2 = + = =
15 30 22.4 10
120 x x 1 1
M= = 0.57 M – = –
210 15 30 10 15
84. (a) 1 ppm = 1 mg / 1 litre (for liquids) x=1g
4 ppm = 4 mg / 1 litre 90. (b) Na2CO3 + HCl —® NaHCO3 + NaCl
1 litre contains 4 mg of fluoride ions NaHCO3 + HCl —® NaCl + H2O + CO2
4 From the above reactios ‘R’ is clearly true that 2 moles of
10 mL contains ´ 10 = 0.04 mg HCl are required for complete neutralisation of Na2CO3.
1000
0.04g Further the titre value using methyl orange corresponds
Number of moles of fluoride = = 2.10 × 10–3 to complete neutralisation of Na2CO3; the titre value using
19g / mol
phenolphthalein corresponds only to the neutralisation of
EBD_8350
476 CHEMISTRY

Na2CO3 to NaHCO3, i.e., half of the value required by Then M.Wt. of metal = 100 – (12 + 3 × 16) = 40 g
Na2CO3 solution. Hence ‘A’ is also true. But ‘R’ is not the (wt. of carbonate, CO32–)
correct explanation of ‘A’.
Molecular weight 40
Equivalent weight = = = 20
Exercise - 2 Valency 2
6. (40.08) Following Dulong-Pettit law, approx. atomic mass
1. (495)
6. 4 6.4
For the given reaction: = = = 40
Specific heat 0.16
2C57H110O6(s) + 163 O2(g) ¾® 114 CO2(g) + 110 H2O(l)
445 40 40
n= = 0.5 Valency of the metal = = =2
890 Equiv. mass 20.04
110 Correct atomic mass = valency × eq. mass = 2 × 20.04 = 40.08 g
Now, moles of water = ´ 0.5 = 27.5
2 7. (10) By the law of equivalents :
\ Mass of water = 27.5 × 18 = 495 g
E + Eq. mass of OH - mass of metal hydroxide
2. (68.4) =
E + Eq. mass of O mass of metal oxide
As we know, (E = Eq. mass of metal)
No.of moles of sugar E + 17 1.5
Molarity = or = Þ E = 10
Volumeof solution (in L) E+8 1
No.of moles of sugar 8. (178.57) The equation representing thermal decomposi-
0.1 =
2L tion of limestone is:
So, no. of moles of sugar = 0.2 mol CaCO3 ¾® CaO + CO2
\ Mass of sugar = No. of moles of sugar × 100 g 56 g 44 g

Molar mass of sugar It is seen from the above equation that 56 g of quicklime is
= 0.2 × 342 = 68.4 g obtained from 100 g of limestone. Thefore 100 kg of CaO
3. (15) Molar mass of A = 5 × 10–3 is obtained from
Molar mass of B = 10 × 10–3 100
× 100 × 103 = 178.57 × 103 g
5 mol AB2 weighs 125 g 56
\ AB2 = 25 g/mol = 178.57 kg of limestone.
10 mol A2B2 weighs 300 g 9. (84) CO + 1 O 2 ® CO 2 ;
\ A2B2 = 30 g/mol 2
\ Molar mass of A(MA) = 5 g CO 2 + 2KOH ® K 2CO 3 + H 2O
Molar mass of B (MB) = 10 g 28
Þ 5 + 10 = 15 g Moles of KOH = = 0.50
56
4. (24) Using the relationship It corresponds to 0.25 mol of CO2
Mol. mass of oxide mass of oxide Hence mol of CO = 1 – 0.25 = 0.75 º mole of CO2 formed
=
Mass of metal in molar mass mass of metal Mol of KOH requred = 2 × 0.75 = 1.5 = 1.5 × 56 = 84 g
10. (0.25) ˆˆ† 2NH 3 ,
N 2 + 3H 2 ‡ˆˆ
4 x + 96 10
= Þ x = 24
4x 5 NH 3 + HCl ¾¾® NH 4 Cl
5. (20) Divalent metal carbonate means MCO3. Moles of NH3 formed º 1 L of 1 M HCl = 1 mol
D
MCO3 ¾¾® MO + CO 2 1
1mol 22400cc Moles of N2 left = 1 – Moles of N2 reacted = 1 – = 0.5
2g 448cc 2
448 cc evolves from 2g 3
Moles of H2 left = 4 – Moles of H2 reacted = 4 – = 2.5
2 2
1cc evolves from g Moles of NH 3
448 Hence, mole fraction of NH3 =
2 Total moles
22400 cc will evolve from ´ 22400 g = 100 g 1
448 = = 0.25
i.e. 100 g is the molecular weight of the carbonate. 1 + 0.5 + 2.5
SOME BASIC CONCEPTS OF CHEMISTRY 477

Exercise - 3 7. (d) Moles of solute


Molality ( m) =
Mass of solvent ( in kg )
3.01 + 2.99
1. (b) Average of readings of student A = = 3.00 Q Molecular weight of HCl = 36.5 g
2 18.25
\ Moles of HCl = = 0.5
3.05 + 2.95 36.5
Average of readings of student B = = 3.00
2 0.5 ´ 1000
m= = 1 molal
Correct reading = 3.00 500
Since, average value in both the cases is close to the 8. (b) Molecular mass of CO2 = 44 g
correct value. Hence, readings of both are accurate. Q 44 g of CO2 contain = 12 g atoms of carbon
Readings of student A differ only by 0.02 and also close 12
\ % of C in CO2 = ´ 100 = 27.27%
to the correct reading hence, readings are precise too. But 44
readings of student B differ by 0.1 and hence are not i.e. mass percent of carbon in CO2 is 27.27%.
precise.
9. (c) Empirical formula mass of CH2O = 30
2. (c) The relation between the temperatures on two scales
Molecular mass = 180
is given by the following relationship :
(Given)
9
°F = T °C + 32 Molecular mass
5 n= 180
= =6
5 200 - 32 Empirical formula mass 30
\ T °C = ( °F - 32) ´ = ´5
9 9 \ Molecular formula = n × empirical formula
168 ´ 5
Þ T°C = = 93.3 °C = 6 × CH2O = C6H12O6
9
10. (a) For a solution, Mass = volume × density
weight ´ 1000
3. (c) Molarity = molecular weight ´ volume ( mL ) = 1.5 mL × 3.12 g mL– 1= 4.68 g
The digit 1.5 has only two significant figures, so the answer
5.85 ´ 1000 must also be limited to two significant figures. So, it is
= = 0.2 mol L–1
58.5 ´ 500 rounded off to 4.7 g.
4. (b) For dilution, a general formula is 11. (c) The properties of a compound are quite different from
M1 V 1 = M2 V 2 the properties of constituent elements. e.g., ammonia is a
(Before dilution) (After dilution) compound containing hydrogen and nitrogen combined
500 × 5M = 1500 × M2 together in a fixed proportion. But the properties of
5 ammonia are completely different from its constituents,
M2 = = 1.66 M hydrogen and nitrogen.
3
12. (b) In this equation
5. (d) number of atoms = No. of moles × NA
4 C3H8 (g) + O 2 (g) ¾¾
® CO 2 (g) + H 2O (g)
Moles of 4 g He = = 1 mol Þ NA atoms 44g 32g 44g 18g
4
46 i.e. mass of reactants ¹ mass of products
46 g Na = = 2 mol Hence, law of conservation of mass is not obeyed.
23
Þ 2 NA atoms 13. (a) Number of millimoles of H2SO4.
= molarity × volume in mL
0.40
0.40 g Ca = = 0.1 mol Þ 0.1 NA atoms = 0.02 × 100 = 2 millimoles = 2 × 10–3 mol
40
Number of molecules = Number of moles × NA.
12 = 2 × 10–3 × 6.022 × 1023 = 12.044 × 1020 molecules
12 g He = = 3 mol Þ 3 NA atoms
4 14. (a) According to the law of conservation of mass,
i.e.12 g He contains greatest number of atoms. Total mass of reactants = Total mass of products
6. (c) In the given question, 0.9 g L–1 means that 1000 mL 15. (b) In CO2, 12 parts by mass of carbon combine with 32
(or 1L) solution contains 0.9 g of glucose. parts by mass of oxygen while in CO, 12 parts by mass of
Molecular mass of glucose (C6H12O6) = 180 u carbon combine with 16 parts by mass of oxygen.
0.9 Therefore, the masses of oxygen combine with a fixed
\ Number of moles of glucose = = 0.005 moles mass of carbon (12 parts) in CO2 and CO are 32 and 16
180
–3
= 5×10 mol glucose respectively. These masses of oxygen bear a simple ratio
Hence, 1000 mL or 1L solution contains 0.005 mole glucose of 32 : 16 or 2 : 1 to each other.
or the molarity of glucose is 0.005 M. This is an example of law of multiple proportion.
EBD_8350
478 CHEMISTRY

16. (a) H2 + Cl2 ¾® 2HCl AgNO3 + NaCl ® AgCl¯ + Na+ + Cl–


t = 0 or 22.4 lit 11.2 lit 0 1 mole 1 mole 1 mole
t=0 1 mole 0.5 mole 0 \ 0.049 mole 0.049 mole 0.049 mole of AgCl
at time t (1 – 0.5) 0 0.5 × 2
= 0.5 = 1 mole
w
n= Þ w = (nAgCl) × Molecular Mass
M
1
17. (a) Initially Mg + O ¾® MgO = (0.049) × (107.8 + 35.5) = 7.02 g
1g 2 2
0.56 24. (b) 2 mole of water softner require 1 mole of Ca 2+ ion
1 0.56 1
or mole mole So, 1 mole of water softner require mole of Ca2+ ion
24 32 2
0.0416 mole 0.0175 mole 1 1
(0.0416 – 2 × 0.0175) mole 0 2 × 0.0175 Thus, = mol / g will be maximum uptake
2 ´ 206 412
\ Mass of Mg = 0.0066 × 24 = 0.158 » 0.16g
æ 4x + y ö y
18. (b) Number of moles of O 2 =
w 25. (d) CxHy(g) + çè ÷ø O2(g) ® xCO2(g) + H2O(l)
4 2
32
4w w 20
Number of moles of N 2 = = Volume of O2 used = 375 ´ = 75 mL
28 7 100
\ From the reaction of combustion
\ Ratio = w : w = 7 : 32
32 7 4x + y
1 mL CxHy requires = mL O2
19. (a) Ratio of weight of gases = w H2 : w O2 = 1 : 4 4
1 4 æ 4x + y ö
15 mL = 15 ç = 75
Ratio of moles of gases = n H2 : n O2 = :
2 32 è 4 ÷ø
1 32 So, 4x + y = 20
\ Molar Ratio = ´ = 4 :1 x=3
2 4
y=8
20. (d) MgCO3 ¾¾ ® MgO + CO 2 \ C3H8 will be the formula of hydro carbon.
84 g of MgCO3 form 40 g of MgO 26. (c) Percentage (by mass) of elements given in the body
40 ´ 20 of a healthy human adult is :-
\ 20g of MgCO3 form g of MgO = 9.52 g of MgO
84 Oxygen = 61.4%, Carbon = 22.9%,
Since 8.0 g of MgO is formed
Hydrogen = 10.0% and Nitrogen = 2.6%
8
Purity of sample = ´ 100 = 84.0% Q Total weight of person = 75 kg
9.52
10
21. (d) No. of moles of water \ Mass due to 1H is = 75 ´ = 7.5 kg
100
In 1.8 g of H2O = 0.1 moles
If 1H atoms are replaced by 2H atoms.
In 18 g of H2O = 1 mole
Mass gain by person would be = 7.5 kg
1 mole contain 6.022 × 1023 molecules of water therefore
27. (b) Given chemical eqn
maximum number of molecules is in 18 moles of water.
M2CO3 + 2HCl ® 2MCl + H2O + CO2
22. (b) If 6.022 × 1023 changes to 6.022 × 1020/mol than this
1g 0.01186 mole
would change mass of one mole of carbon.
from the balanced chemical eqn.
23. (c) 50 mL of 16.9% solution of AgNO3 nM2CO3 = nCO2
æ 16.9 ö
ç 100 ´ 50 ÷ = 8.45 g of Ag NO3 1
= 0.01186
è ø
M 2 CO3
8.45g
nmole = 1
(107.8 + 14 + 16 ´ 3) g / mol \ M 2 CO3 =
0.01186
æ 8.45 g ö Þ M = 84.3 g/mol

è 169.8g / mol ÷ø
= 0.0497 moles
H2SO4
28. (d) HCOOH ¾¾¾¾® CO + H2O
50 mL of 5.8% solution of NaCl contain Dehydrating
agent [H 2O absorbed
æ 5.8 ö by H 2SO 4]
NaCl = ç ´ 50 ÷ = 2.9 g 2.3 1
è 100 ø At start = = 0 0
46 20
2.9g (moles)
nNaCl = = 0.0495 moles 1 1
(23 + 35.5) g/ mol Final moles 0
20 20
SOME BASIC CONCEPTS OF CHEMISTRY 479

H2SO4
H2C2O4 ¾¾¾® CO + CO2 + H2O W
33. (c) Number of atoms = ´ N A ´ atomicity
[H 2O absorbed Molar mass
by H 2SO 4]
4.5 1 1
At start = = 0 0 0 (a) Number of Mg atoms = ´ NA ´1
90 20
(moles) 24
1 1 1
Final moles 0 1
20 20 20 (b) Number of O atoms = ´ NA ´ 2
CO2 is absorbed by KOH. 32
So the remaining product is only CO. 1
1 1 1 (c) Number of Li atoms = ´ N A ´1
Moles of CO formed from both reactions = + = 7
20 20 10
1 1
Left mass of CO = moles × molar mass = ´ 28 = 2.8 g (d) Number of Ag atoms = ´ N A ´1
10 108
29. (a) 34. (c) Except (c) all postulates was given by the Dalton.
(a) Mass of water = 18 × 1 = 18 g
18 Exercise - 4
Molecules of water = mole × NA = N = NA
18 A
(b) Molecules of water = mole × NA 1. (a) 14 g of N3– ions have = 8NA valence electrons.
0.18
= N = 10–2 NA 8N A
18 A 4.2 g of N3– ions have = ´ 4.2 = 2.4 N A
14
(c) Molecules of water = mole × NA = 10–3 NA
2. (d) 2C 6 H 6 + 15O 2 (g ) ® 12 CO 2 (g ) + 6 H 2 O( g )
0.00224 2 ( 78) 15( 32)
(d) Moles of water = = 10–4
22.4 Q156 g of benzene required oxygen = 15 × 22.4 litre
Molecules of water = mole × NA = 10–4 NA
15 ´ 22.4
\1 g of benzene required oxygen = litre
12 x 6 156
30. (c) = ; 2x = y for given organic compound (CxHyOz)
y 1 \ 39 g of benzene required oxygen
æ yö y 15 ´ 22.4 ´ 39
C x H y (g) + ç x + ÷ O2 (g) ¾¾® xCO2 (g) + H 2 O(l) = = 84.0 litre
è 4 ø 2 156
Number of oxygen atom in CxHyOz = z 3. (a) 0.478 mL = 0.478 g of water; 109 g water contain 0.10
Number of oxygen atom required for CxHy combustion is g CHCl3
æ yö æ yö 0.1
ç x + 4 ÷ ´ 2 = ç 2x + 2 ÷ \ 0.478 g water contain ´ 0.478 g CHCl3
è ø è ø 109
1æ yö y 0.1 0.478
So z = ç 2 x + ÷ ; z = x + \ nCHCl = ´
2è 2ø 4 3 9 119.5
10
2 x 3x (mass of CHCl3 = 119.5 g/mol)
z = x+ = ;
4 2 0.1
0.478
3x \ No. of molecules = ´
´ N A = 4 × 10–13 × NA
9
ratio of number of atoms = x : y : z = x : 2 x : =2:4:3 10 119.5
2
Hence, empirical formula of compound is C2H4O3. 4. (d) Volume of gold foil = 25 × 40 × 0.25 mm3
= 250 × 10–3 cm3
31. (c) N2 + 3H2 ¾® 2NH3
Mass of gold foil = 19.32 × 250 × 10–3 g = 4.83 g
3
1 Mol NH3 = mol H2 4.83
2 No. of gold atoms = ´ N A = 1.47 × 1022
197
3
20 mol NH3 = × 20 mol H2 = 30 mol H2
2 nSolute
5. (d) M = ´ 1000 ....(1)
\ 30 moles of H2 are required. Vol. of solution ( in mL )
32. (a) According to the stoichiometry of balanced equation
28 g N2 react with 6 g H2 nSolute
m= ´ 1000 ....(2)
Weight of solution ( in g )
N 2 + 3H 2 ¾¾
® 2NH3
1 mole 3 mole nSolute
28 g 6g m= ´ 1000
For 56 g of N2, 12 g of H2 is required. Weight of solution ( in g ) - wt of solute ( in g )
EBD_8350
480 CHEMISTRY

M ´ vol. of solution ( in mL ) ´1000 40


m= \ % of MgO = ´ 100 = 12.5%
1000 ´ ( wt of solution (in g ) - wt of solute (in g ) ) 320
10. (b) MnO4 + 5Fe2+ + 8H+ ® Mn2+ + 5Fe3+ + 4H2O
M ´ 1000 nf (KMnO4) = 7 – 2 = 5
m=
wt.of solutions(in g) nf (FeSO4) = 3 – 2 = 1
1000 ´ - 1000
vol.of solution (in m L) m-eq. of FeSO4.7H2O in 1 litre = m – eq. of KMnO4
wt.of solute(ing)
´ æ 1000 ö W
vol.of solution (in mL) Þ 20 × 0.02 × 5 × ç ÷= ´ 1 ´ 1000
è 25 ø 278
[Mass of FeSO4.7H2O = 278 g/mol]
M ´1000
m= Þ W = 22.24
1000 ´ density of solution (g/mL) – M.M1
Mass % of FeSO4.7H2O in given sample
1000 M
m= 22.24
1000 P – MM1 = ´ 100 = 69.5
32
28 11. (c) By equating moles of mixture
6. (c) Molarity = = 2.5 M
11.2 V ´ 45 (800 - V ) ´ 20 800 ´ 29.875
1000 mL of solution contain 2.5 mole H2O2, + =
100 100 100
1 density = 1.25 g/mL 9V V
Þ + 160 - = 239 Þ V = 316 mL
Thus wt. of solution of 1000 mL = 1250 g 20 5
wt. of solute (H2O2) in 1000 mL = 34 × 2.5 = 85 g
12. (d) Cr2 O27 - + 6Fe2+ (n = 1) (Mohr 's salt) + 14H + (n = 6)
wt/ pf solvent = (1250 – 85)g
® 2Cr 3+ + 6Fe3+ + 7H 2 O
¾¾
2.5
Molality, m = ´ 1000 = 2.15 m Equivalent of Mohr's salt = equivalents of K2Cr2O7
1250 - 85
Moles of Mohr’s salt × nf = Moles of K2Cr2O7 × nf
PM dRT 2.28 ´ 0.0821 ´ 300
7. (c) d= ÞM = = Þ n × 1 = (1 × 500 × 10–3) × 6 = 3
RT P 1
Þ n=3
= 56.15 g/mol Hence, mole per cent of Mohr's salt
3
85.7 14.3 = ´100 = 75 (total moles in mixture = 4)
Atomic ratio = : = 7.14 :14.3 = 1: 2; 4
12 1 1
\ E.F is CH2; 13. (d) For 3 mol of A : mol of P = ´ 3 = 1.5; mol of
2
M.F. = (CH2)n 2
Q = ´3 = 3
56.15 2
where n = ; 4; For 2 mol of B : mol of P = 2 ; mol of Q = 2 × 2 = 4
12 + 2
1
\ M.F. is C4H8 For 6 mol of C : mol of P = ´ 6 = 2;
3
28
8. (b) Mol. of solute in 100 g solution = =1 2
28 mol of Q = ´6 = 4
3
Mol. of water in 100 g solution = 100 - 28 = 4 A is limiting reactant. Hence, mol of P = 1.5 and mol of Q = 3
18 14. (a) The precipitation reaction is:
1
Mol. fraction of solute = = 0. 2 ; Ag + (aq) + Br - (aq) ¾¾
® AgBr(s)
1+ 4
In this problem, the relative amounts of NaBr and CaBr 2
Molality = 1´ 1000 = 125 are not known. However, the total amount of Br – in the
72 9
mixture can be determined from the amount of AgBr
9. (a) m-moles of HCl = 20 × 0.1= 2 produced. Let’s find the number of moles of Br –.

m-moles of MgO reacted with HCl =


2
Þ1 1 mol AgBr 1 mol Br -
2 1.6930 g AgBr ´ ´
187.8 g AgBr 1 mol AgBr
mass of MgO present = 1 × 40 mg
= 9.0149 ´ 10–3 mol Br–
SOME BASIC CONCEPTS OF CHEMISTRY 481

The amount of Br– comes from both NaBr and CaBr 2. Let The percentage by mass of NaBr in the mixture is:
x = number of moles NaBr. Then, the number of moles of
0.51448g
% NaBr = ´ 100% = 56.18% NaBr
9.0149 ´ 10-3 mol - x 0.9157 g
CaBr 2 = . The mole of CaBr2 are
2 15. (a) X + 2Y ¾® XY2
divided by 2, because 1 mole of CaBr 2 produces 2 moles 0.1 mol of XY2 = 10 g
of Br–. The sum of the NaBr and CaBr 2 masses must equal 1 mol of XY2 = 100 g
the mass of the mixture, 0.9157 g. We can write: 3X + 2Y ¾® X3Y2
mass NaBr + mass CaBr2 = 0.9157 g 0.05 mol of X3Y2 = 9Y
é 102.89 g NaBr ù 9
ê x mol NaBr + ú 1 mol of X3Y2 = = 180 g
1 mol NaBr û 0.05
ë
Molecular wt. of XY2 = 100 g
éæ 9.0149 ´ 10 -3 - x ö 199.88 g CaBr2 ù
Molecular wt. of X3Y2 = 180 g
+ êç ÷ mol CaBr2 ´ ú \ X + 2Y = 100 g
êëè 2 ø 1 mol CaBr2 úû
3X + 2Y = 180 g
= 0.9157 g Solving for X and Y we get
2.95x = 0.014751 X = 40 g
x = 5.0003 ´ 10–3 = moles NaBr Y = 30 g
Converting moles to grams:
mass of NaBr = (5.0003 ´ 10–3 mol NaBr)
102.89 g NaBr
´ = 0.51448 g
1 mol NaBr
EBD_8350
482 CHEMISTRY

2 Structure of Atom
23. (d) Atomic number is equal to number of protons or
Exercise - 1
number of electrons. Thus, if two species have different
1. (b) By his study of electrical discharge. atomic number they must contain different number of
2. (b) Cathode rays are never electromagnetic waves. protons and electrons. Number of neutrons = atomic
3. (c) mass – atomic number. Therefore, due to difference of
4. (c) Moves with speed less than that of light. atomic numbers two species also have different number
5. (c) The electrical discharge through the gases could be of neutrons.
observed only at low pressure and high voltage. 24. (a) Isoelectronic species
6. (b) The cathode rays (negatively charged particles 25. (b) Isotopes have same atomic number. Isobars have
stream) originates from cathode and move towards anode. same mass number, whereas isoelectronic species have
7. (b) Millikan determined the value of charge on the same number of electrons although the (A) has same
electron by using oil drop experiment. number of electrons but the protons they carry are same
8. (d) Proton is the nucleus of H-atom (H-atom devoid of its while in case of isolelectronic species number of protons
electron) they carry are different.
9. (c) As the neutron is a chargeless particle, hence, the 26. (d) Atomic number = No. of protons = 8
beam of neutrons is not deflected by electrical or magnetic Mass number = No. of protons + No. of neutrons
field. = 8 + 8 = 16
10. (a) James Chadwick in 1932 discovered the neutrons. Since the no. of electrons are two more than the no. of
11. (c) protons, hence, it is a binegative species. Thus, the species
12. (b) N3– , the amount of deviation depends upon the is 16 O82 -.
magnitude of negative charge on the particle. 27. (a) Isotopes have different physical properties due to
e 0 difference in mass.
13. (d) for (i) neutron = = 0 c
m 1 28. (d) n1 =
ll
2
(ii) a-particle = = 0.5 c c
4 n2 = =
l 2 3l1
1
(iii) proton = = 1 n 2 - n1
1 % change in frequency = ´ 100
n1
1
(iv) electron = = 1837 c c 2c
1 1837 - - ´ 100
14. (d) The lesser is the mass of particle, greater is the 3l1 l1 3l1
= ´ 100 = = – 66%
deflection. c c
15. (c) Deuteron and an a-particle have identical values of l1 l1
e/m.
29. (a) Energy is always absorbed or emitted in whole
16. (c) Rutherford’s a-ray scattering experiment first showed
number or multiples of quantum.
the existence of a small positivily charged entity in the
centre of atom, called nucleus. hc c
30. (d) E = hn = or l =
17. (d) The nucleus occupies much smaller volume compared l n
to the volume of the atom. 3 ´108
18. (a) All positive ions are deposited at small part. (nucleus Þl= = 3.75 ´10 -8 m
of atom). 8 ´1015
19. (d) Electrons are revolving around the nucleus, and In nanometer l = 3.75× 10 nm
centrifugal force is balancing the force of attraction. This which is closest to 4 × 101 nm
was explained by Rutherford. 31. (c) K.E. of emitted electron
20. (c) 18Ar40 contains 22 neutrons and 21Sc40 contains 19 = hv - hv0 (i.e. smaller than hv ).
neutrons. The number of neutrons = (A – Z) 32. (a) At a frequency n > n0, the ejected electrons come out
21. (b) Number of p = number of e– = 89 and neutrons with certain kinetic energy.
231 – 89 = 142. 33. (c) The oscillating fields are perpendicular to the
22. 35 37
(d) 17 Cl and 17 Cl are isotopes, so they will have direction of propagation.
same chemical properties. 34. (b) Energy increases with increase in temperature and so
the frequency.
STRUCTURE OF ATOM 483

35. (d) E = hn 41. (b) Angular momentum of an electron in nth orbit is given
by
æcö
and n=ç ÷ nh
èlø mvr =
2p
na = 1015, nb = 1014,
For n = 5, we have
nc = 1017, nd = 0.85 × 1015
5h 2.5h
and ne = 10 × 1015, Angular momentum of electron = =
2p p
36. (b) For statement (ii) there is no time lag between striking 42. (b)
of light beam and the ejection of electrons. Kinetic energy 43. (d) Isotopes discovered by soddy.
of ejected electron depends on the frequency of incident
44. (a) Radius of nth orbit = r n2 . (for H-atom)
radiation.
37. (c) Electromagnetic waves shown in figure (A) has 45. (a) For hydrogen atom (n) = 1 (due to ground state)
higher wavelength in comparison to EM waves shown Radius of hydrogen atom (r) = 0.53 Å.
in figure (B). Atomic number of Li (Z) = 3.
Thus, these waves also differ in frequency and energy. n2 (1)2
c Radius of Li2+ ion = r1 ´ = 0.53 ´ = 0.17 Å
n= Z 3
l RH
l1 46. (b) We know that En = kJ mol -1
n2
hc
(A) Þ E1 = –R H
l1 Þ –328 =
22
Þ RH = 1312
l2 -1312
hc Thus, E 4 = = –82 kJ mol–1
(B) Þ E2 = 42
l2
13.6Z 2
47. (d) For H atom, E n = - eV
l1 > l2 Þ E1 < E2 n2
hc For second orbit, n = 2
38. (c) Energy of a photon, E = Z = At. no. = 1 (for hydrogen)
l
13.6 ´ (1)2 -13.6
6.6 ´10-34 (Js) ´ 3 ´108 (ms-1 ) \ E2 = - = eV
= 2 4
= 6× 10–19 J (2)
331.3 ´10-9 (m)
-13.6 ´1.6 ´ 10-19 -19
No. of photons emitted per second = J = -5.44 ´ 10 J
4
600 (J) - E0
= =1021 [1 watt – second = joule]
6 ´ 10-19 (J) 48. (a) Energy of an electron En =
n2
1 2 [E0 corresponds to n = 1 ® n = ¥]
39. (a) mv = h n - h n0 For energy level (n = 2)
2
13.6 -13.6
1 En = - = = -3.4 eV.
Þ mv 2 = h ( n - n 0 ) (2) 2 4
2
49. (a) H, He+ and Li2+ are single electron species thus, show
2h similar line spectra.
Þ v= ( n - n0 )
m 1 æ 1 1 ö
50. (c) = Rç 2 - 2 ÷
l è n1 n2 ø
2 ´ 6.626 ´ 10-34
Þ v=
9.1 ´ 10-31
( 2 ´ 1015
– 7 ´ 1014 ) 1 æ1 1 ö
= 1.097 ´ 107 ç - ÷ = 1.097 ´107 m -1
l è1 ¥ ø
l = 91.15 ´ 10 -9 m » 91nm
6
= 1.89 ´ 1012 = 1.37 × 10 ms.
40. (c) Since the energy difference between two consecutive 1 é1 1ù
Bohr orbits is quantized and the energy of higher orbit is 51. (c) n = ´ IE ´ ê 2 - 2 ú
h ëê n1 n2 ûú
more than that of lower orbit, so an electron from one Bohr
stationary orbit can go to next higher orbit by absorption 2.18 ´ 10-18 é1 1 ù
of electromagnetic radiation of particular wavelength or = ´ ê - ú = 3.08 ´ 1015 s -1
-34 ë1 16 û
6.625 ´ 10
frequency.
EBD_8350
484 CHEMISTRY

1 é1 1 ù h
52. (a) u= = Rê 2 – 2 ú & me v 2 r2 = n2
l ë n1 n2 û 2p
For second line in lyman series me v1r1 n1 h 2p
n2 = 3 = ´
me v2 r2 n2 2p h
1 é1 1 ù é1 1 ù 8R
\ = Rê 2 - 2 ú = Rê – ú = Given, r1 = 5 r2, n1 = 5, n2 = 4
l ë1 3 û ë1 9 û 9
53. (c) Bohr model can explain spectrum of any atom or ion me ´ v1 ´ 5r2 5
=
containing one electron only (that is H-like species) me ´ v2 ´ r2 4
54. (d) Uncertainty principle which was given by Heisenberg
and not Bohr’s postulate. v1 5 1
Þ = = = 1: 4
55. (d) Bohr’s model can be applied to one electron system v2 4 ´ 5 4
only. 65. (b) Z = 3 for Li2+ ions
56. (b) Bohr model can only explain one electron system
57. (a) Both assertion and reason are true and reason is the 52.9 ´ n2
correct explanation of assertion. So rn = pm
Z
n = 3, Z = 3
n2 h2 n2
Radius, rn = = ´ 0.529Å.
52.9 ´ ( 3)
2
4pe2 mZ Z
rn = pm = 158.7 pm
For first orbit of H-atom 3
n=1 Also, linear momentum (mv) = 7.3 × 10–34 kg ms–1
(1)2 Then angular momentum will be
r1 = ´ 0.529Å = 0.529Å w = (mv) × r
1
58. (d) Given : Radius of hydrogen atom = 0.530 Å, Number = (7.3 × 10–34 kg ms–1) (158.7 pm)
of excited state (n) = 2 and atomic number of hydrogen = 7.3 × 10–34 kg ms–1 × (158.7 × 10–12 m)
atom (Z) = 1. We know that the Bohr radius = 11.58 × 10–48 kg m2 s–1
n2 (2) 2 = 11.58 × 10–45 g m2s–1
(r ) = ´ radius of atom = ´ 0.530
Z 1
= 4 ´ 0.530 = 2.12 Å Z2
66. (c) Energy of electron in 2nd orbit of Li+2 = -13.6
59. (d) Except Al3+, all contain one electron and Bohr’s n2
model could explain the spectra for one electron system, - 13.6 ´ (3) 2
Bohr’s model was not able to explain the spectra of = = –30.6 eV
multielectron system. (2) 2
60. (c) Energy of free e– at rest (n = ¥) is zero. Energy required = 0 – (–30.6) = 30.6 eV
67. (c) (i) As wavelength decreases, wave number will
æ 1 1 ö
61. (c) D E for two energy levels = 21.79 ç - ÷ J/atom . increase because n = 1/l. Wave number is defined as
ç n2 n2 ÷ number of wavelengths per unit length. Thus, lines in the
è 1 2 ø
Emmitted energy decreases for electron transition between series will converge.
two immediate states as we move to higher principle states. (ii) n1 = 2 is fixed for Balmer series.
Mathematically also, value of (iii) For calculation of ionization energy, n1 = 1 and n2 = ¥.
Thus, it cannot be calculated from the wave number of
æ 1 1ö lines for Balmer series.
ç 2 - 2 ÷ will decrease.
è n1 n2 ø (iv) Longest wavelength corresponds to lowest energy, which
will be the first line of Balmer series, i.e. n1 = 2 and n2 = 3.
62. (d) The shortest wavelength in hydrogen spectrum of
Hence, (i), (ii) and (iv) are correct statements.
Lyman series is given by formula :
1 RH RH 109678 68. (a) l = h/ mv ; for the same velocity, l varies inversely
= = 2 = with the mass of the particle.
l n2 1 1
Þ l = 9.117 × 10–6 cm h 6.6 ´ 10 -34
69. (d) l= = = 10 -33 m
= 911.7 × 10–10 m = 911.7 Å. mv 60 ´ 10 -3 ´ 10
63. (b) An e– absorbs energy to transit to higher state. 70. (d) Given DE = 4.4 × 10–4 J , l = ?
64. (d) From the expression of Bohr’s theory, we know that
hc 6.6 ´ 10 -34 ´ 3 ´ 108
h l= = = 4.5 ´ 10 -22 m
me v1r1 = n1 DE -4
4.4 ´ 10
2p
STRUCTURE OF ATOM 485

h h v2 9
71. (c) We know that l = ; \m = =
mv vl v1 4
The velocity of photon (v) = 3 × 108 m sec–1
l = 1.54 ´10-8 cm = 1.54 ´10 -10 meter v1 4
=
v2 9
6.626 ´ 10-34 Js
\m =
1.54 ´ 10 -10 m ´ 3 ´ 108 m sec -1 1 2
KE = mv
= 1.4285 ´ 10 -32 kg 2
2
h KE1 m1 v12 9 æ 4 ö 16
72. (b) l = = ´ = ´ç ÷ =
mv KE2 m2 v 22 1 è 9 ø 9
h = 6.6 × 10–34 J/s
m = 1000 kg 80. (a) Given mass of an electron(m) = 9.1´10 -28 g;

36 ´ 103 Velocity of electron (v) = 3 ´ 104 cm/s;


v = 36 km/hr = m/sec = 10 m/sec
60 ´ 60
Accuracy in velocity = 0.001% = 0.001 ;
6.6 ´ 10-34 100
\ l= = 6.6 ´ 10-38 m Actual velocity of the electron
103 ´ 10
0.001
73. (d) Heisenberg's uncertainty principle is applicable to any ( Dv) = 3 ´ 10 4 ´ = 0.3 cm/s .
moving object. 100
h Planck’s constant (h) = 6.626×10–27erg-sec.
74. (d) By Heisenberg uncertainty principle, Dx ´ Dp =
4p \ Uncertainty in the position of the electron
(which is constant)
As Dx for electron and helium atom is same thus momentum h 6.626´10,27 ´ 7
( Dx ) = <
of electron and helium will also be same therefore the 4pmDv 4´ 22´ (9.1´10,28 ) ´ 0.3
momentum of helium atom is equal to 5 × 10–26 kg. m.s–1.
=1.93 cm
75. (b) Dx.Dv value will be large for object of smallest mass 81. (a) Dp = mDv
and is therefore, the most significant for calculating Substituting the given values of DP and m, we get
uncertainity. Object B has smallest mass i.e., g.11 × 10–28g 1×10–18 g cm s–1 = 9×10–28 g × Dv
= g.11 × 10–31 Kg. So it will be most significant.
76. (d) The asseration is false but the reason is true. Exact 1 ´ 10-18
or Dv =
position and exact momentum of an electron can never be 9 ´ 10-28
determined according to Heisenberg’s uncertainty = 1.1 × 109 cm s–1 ; 1×109 cm s–1
principle. Even not with the help of electron microscope i.e. option (a) is correct.
because when electron beam of electron microscope strikes 82. (b) According to Heisenberg uncertainty principle.
the target electron of atom, the impact causes the change h h
in velocity and position of electron. Dx.mDv = Þ Dx =
4p 4 pm Dv
77. (c) The wavelengths of elements decreases with increase
600 ´ 0.005
æ h ö Here, Dv = = 0.03
in their mass. çèQ l = ÷ 100
mv ø
6.6 ´ 10 -34
78. (d) Given, vA = 0.1 ms–1 and vB = 0.05 ms–1 also, mB = 5mA So, Dx =
h 4 ´ 3.14 ´ 9.1´ 10-31 ´ 0.03
de-Broglie wavelength, l = = 1.92 × 10–3 meter
mv
83. (b) Magnetic quantum no. represents the orientation of
l h / mA v A mB v B atomic orbitals in an atom. For example px, py & pz have
\ A = =
l B h / mB v B m A v A orientation along X-axis, Y-axis & Z-axis respectively.
5m A ´ 0.05 84. (b) The sub-shell are 3d, 4d, 4p and 4s, 4d has highest
= = 5 ´ 0.5 = 2.5 = 5 / 2 energy as n + l value is maximum for this.
m A ´ 0.1
85. (a) The possible quantum numbers for 4f electron are
\ lA : lB = 5 : 2 1
h n = 4, l = 3, m = – 3, –2 –1, 0, 1 , 2 , 3 and s = ±
79. (d) de Broglie wavelength l = 2
mv Of various possiblities only option (a) is possible.
l1 m2 v 2 1 1 v 2 86. (b) n = 4 represents 4th orbit
= ; = ´
l 2 m1v1 4 9 v1 l = 3 represents f subshell
EBD_8350
486 CHEMISTRY

m = – 2 represents orientation of f-orbital 104. (b) According to Aufbau principle, the orbital of lower
s = 1/2 represents direction of spin of electron. energy (2s) should be fully filled before the filling of orbital
\ The orbital is 4f. of higher energy starts.
87. (b) For 4d orbitals, n = 4, l = 2 Option (a) violates Hund’s rule.
é For s orbital l = 0 ù 105. (d) The number of sub shell is (2 l + 1). The maximum
ê For p orbital l = 1 ú number of electrons in the sub shell is 2 (2 l + 1)
ê ú = (4 l + 2).
êë For d orbital l = 2 úû 106. (b) 5p : 5 + 1 = 6
m = –2, –1, 0, +1 or + 2 n + l = 6s : 6 + 0 = 6
1 1 4f : 4 + 3 = 7
s = + and -
2 2 5d : 5 + 2 = 7
-1 When values of (n + l) are equal, then, the orbital with
Thus choice b having n = 4, l = 2, m = 1 and s = is higher value of n will have higher energy.
2
correct. Hence, the order is 5p < 6s < 4f < 5d.
88. (c) n = 2, l = 1 means 2p–orbital. Electrons that can be 107. (d) 3d electron is in the highest energy orbital.
accommodated = 6 as p sub-shell has 3 orbital and each 108. (d) The d-orbital represented by option (d) will become
orbital contains 2 electrons. completely filled after gaining an electron. Therefore,
89. (b) m = – l to +l, through zero thus for l = 2, values of m option (d) is correct.
will be – 2, –1, 0, + 1, + 2. 109. (c)
Therefore for l = 2, m cannot have the value –3.
110. (b) Correct shape of orbitals.
90. (b) Value of l = 0 ..........(n – 1)
l cannot be equal to n. 111. (c) n = 3 and l = 2 means the orbital is in 3d-subshell.
91. (b) For n = 5, l = n – 1 = 5 – 1 = 4 For a d-orbital : m = 0 for dz2 orbital
m = 2l + 1 = 2(4) + 1 = 9 m = ±1 for dyz and dzx orbitals.
Sum of values of l and m = 9 + 4 = 13 and m = ±2 for dxy and dx2 – y2 orbital.
92. (d) The orbitals which have same energy are called Thus, m = + 2 is possible for 2 orbitals.
degenerate orbitals eg. px, py and pz. 112. (d) We know that atomic number of gadolinium is 64.
93. (a) The number of allowed orbitals are given by n2. Therefore, the electronic configuration of gadolinium is
Thus, when n = 5, number of orbitals = (5)2 = 25 [Xe] 4f 7 5d1 6s2. Because the half filled and fully filled
94. (d) This is as per the definition of Pauli’s exclusion principle. orbitals are more stable.
95. (b) Electronic configuration of Cu (29) is 1s2 2s 2 2p6 3s2
113. (b) (i) n = 5 means l = 0, 1, 2, 3, 4
3p6 3d 10 4s1 and not 1s2, 2s2 2p6 3s2 3p6 3d 94s2 due to extra
stability of fully filled orbitals. since m = + 1
96. (a) hence total number of electrons will be
97. (c) Possible values of l and m depend upon the value of n = 0 (from s) + 2 (from p) + 2 (from d)
l = 0 to (n – 1) + 2 (from f) + 2 (from g)
m = –l to + l through zero
=0+2+2+2+2=8
1 1
s = + and - (ii) n = 2, l = 1, ml = –1, ms = –1/2 represent 2p orbital with
2 2 one electron
Thus, for n = 3,
l may be 0, 1 or 2; but not 3 114. (a) For s-electron, l = 0
98. (c) First four orbitals contain four lobes, while fifth orbital h
\ Orbital angular momentum = 0(0 + 1)
=0
consists of only two lobes. The lobes of dxy orbital lie 2p
between x and y axis. Similarly in the case of dyz and dzx.
115. (d) The component values of orbital angular momentum
their lobes lie between yz and zx axis respectively. Four lobes
of d 2 2 orbital are lying along x and y axis while two lobes h
x -y in z-direction = ml ´
2p
of d 2 orbital are lying along z axis.
z
99. (b) y = 0 at only one point, at which y2 = 0. Exercise - 2
100. (a) As n – l – 1 = 5 or 8 – l – 1 = 5 Þ l = 2.
101. (b) According to given information n = 5 and l = 3. 1. (–6.04) According to Bohr’s model energy in nth state
102. (c) Fe2+ (26 – 2 = 24) = 1s2 2s2 2p6 3s2 3p6 4s0 3d 6 hence no.
Z2
of d electrons retained is 6. [Two 4s electron are removed] = -13.6 ´ eV
103. (b) This configuration represents ground state electronic n2
configuration of Cr. (due to stability of half-filled For second excited state, of He+, n = 3
d-orbital).
22
1s2 2s2 2p6 3s23p63d 5 4s1 \ E3(He+) = -13.6 ´ eV = –6.04 eV
32
STRUCTURE OF ATOM 487

hc or l1
2. (2.1) E = hn = =3
l l
6.626 ´ 10 -34 ´ 3 ´ 108 or l1 = 3l
E= = 3.1 eV 6. (2) 2pr = 4l; n = 4
4000 ´ 10 -10 ´ 1.6 ´ 10 -19
Total energy required + total energy released = 0
1 2 1 2 ´ 4.526 ´ NA + 2 ´ 13.6 ´ NA + 2 ´ 13.6
=mv = ´ 9 ´ 10-31 ´ 36 ´ 1010 J
2 2 æ 1ö
–19 × ç1 - ÷ × NA – 30.87 × x × NA = 0
= 1.62 × 10 J = 1 eV è 16 ø
According to photoelectric effect x= 2
K.E. = hn – hn0 \ moles of x required = 2
hn0 = hn – K.E. h
7. (1.77) de-Broglie wavelength l =
Work function (W0) = E – K.E. = 3.1 – 1 = 2.1 eV mn
n (n - 1) l1 m 2 n2 1 1 n2
3. (97.25) = 6; n = 4, = ; = ´
2 l 2 m1n1 4 9 n1
n = 4, E4 = – 0.85 eV
n = 1, E1 = – 13.6 eV n2 9
=
n1 4
\ DE = 12.75 eV
1240 eVnm n1 4
12.75 eV = (since h = 1240 eVnm) =
l n2 9
l = 97.25 nm 1 2
KE = mn
hc 2
4. (26.9) E = = 2.9 ´ 10–19 J
l 2
Total energy of 10 quanta KE1 m1 n12 9 æ 4 ö 16
= ´ 2 = ´ç ÷ = = 1.77
Þ 10 ´ 2.9 ´ 10–19 Þ 29 ´ 10–19 J KE 2 m 2 n2 1 è 9 ø 9

112 ´ 4.18 ´ 103 0.001


Energy stored for process = 8. (2) Dv = × 30,000 = 0.3 cm sec–1
6 ´ 1023 100
= 7.80 ´ 10–19 J According to uncertainty principle,

7.8 ´ 10 -19 Dx . Dp »
h
; Dx.Dv »
h
% efficiency = ´ 100 Þ 26.9% 4p 4pm
29 ´ 10 -19
5. (3) From the given data, when the system moves from 2E 6.625 ´ 10 27 ´ 7
Dx × 9.1 × 10–28 × 0.3 »
level to E level, we have 4 ´ 22
hc Dx » 1.93 cm. » 2
2E– E = 9. (6) Radial nodes = n – l – 1
l
hc Þ (I) n = 2, l = 1, radial node = 2 – 1 – 1 = 0
or E = Angular node = 1 (YZ plane)
l
4E (II) n = 2, l = 0, Radial node = 2 – 0 – 1 = 1
When the system moves from level to E level, Angular node = 0
3
we have (III) n = 3, l = 2, Radial node = 3 – 2 – 1 = 0, and
4 hc Angular node = 2 (XY and YZ planes)
E-E= (IV) n = 3, l = 2, Radial node = 0, and
3 l1
Angular nodes = 2
[l1 is the wave length of photon emitted]
Total = I + II + III + IV = 1 + 1 + 2 + 2 = 6
E hc 10. (3) In the 1st Bohr orbit of H :
or =
3 l1 n = 2.18 ´ 106 ms–1.
é hc ù Now, let us consider that in Li 2+ the electron is
êëQ E = l úû
or hc hc
= in nth orbit. Speed of electron in n th Bohr orbit of Li2+ is
l.3 l1
1 1 3
or = n = (Li2+) = 2.18 ´ 106 ´
3l l1 n
EBD_8350
488 CHEMISTRY

Now, applying the condition of equal speed : 1


For same speed of different particles, l µ
3 m
2.18 × 106 × = 2.18 × 106 Þ n = 3
n As h is constant, greater the mass of matter waves, lesser
Exercise - 3 is wavelength and vice-versa. Among these matter waves,
alpha particle (He2+ ) has higher mass, therefore, shortest
1. (c) The concept of electrons move in a circular path of wavelength.
fixed energy called orbits was given by Bohr and not 14. (d) The probability density of electrons in 2s orbital first
derived from Rutherford's scattering experiment. increases then decreases and after that it increases again
2. (b) The correct configuration for copper (z = 29) should as distance increases from nucleus.
be 1s22s22p63s23p63d104s1. Due to extra stability of fully 15. (b) 24Cr = [Ar]3d5, 4s1 Cr3+ = [Ar]3d3
filled orbital of d-subshell, the last electron enter into d- 6 2 Fe3+ = [Ar]3d5
26Fe = [Ar]3d , 4s
orbital instead of s-orbital. 5 2
25Mn = [Ar]3d , 4s Mn2+ = [Ar]3d5
3. (d) The characteristics of cathode rays do not depend 7 2
27Co = [Ar]3d , 4s Co3+ = [Ar]3d6
upon the material of electrodes and the nature of the gas 1 2
present in the cathode ray tube. 21Sc = [Ar]3d , 4s Sc3+ = [Ar]
4. (b) The mass of electron is very small as compared to Thus, Fe 3+ and Mn 2+ have the same electronic
the mass of the neutron. configuration.
16. (d) For the two electrons of 2s orbital, the value of ms is
Mass of electron = 9.1 × 10–31 kg
Mass of neutron = 1.67 × 10–27 kg 1 1
+ and - .
5. (a) J. J. Thomson, in 1898, proposed plum pudding model 2 2
of atom. An important feature of this model is that the 17. (a) Given: n = 3, l = 1, m = 0
mass of the atom is assumed to be uniformly distributed Hence, orbital is 3p
over the atom. This model was able to explain the overall –1 0 +1
neutrality of the atom.
6. (d) Isobars have the same mass number (i.e., sum of
protons and neutrons) but different atomic number (i.e., hence, the number of orbital identified by m = 0 can be one
number of protons) e.g., 26Fe58 and 27Ni58 are isobars. only.
7. (d) number of radial nodes = n – l – 1, l = 1 for p-orbital hc 6.63 ´ 10 - 34 ´ 3 ´ 108
18. (d) E = = = 4.42 × 10– 18 J
Number of radial nodes for 3p orbital = 3 – 1 – 1 = 1 l 45 ´ 10 - 9
8. (c) Number of angular nodes = l
19. (a) The electronic configuration of Rubidium (Rb = 37) is
l = 2 for d-orbital
\ Number of angular nodes = 2 1s 2 2s 2 2 p 6 3s 2 3 p6 3d 10 4s 2 4 p 6 5s1
9. (b) The important implications of Heisenberg uncertainty Since last electron enters in 5s orbital
principle is that it rules out existence of definite paths or 1
trajectories of electrons and other similar particles. Hence n = 5, l = 0, m = 0, s = ±
2
10. (c) Total number of orbitals associated with nth shell
= n2 –13.6 Z2
20. (a) Total energy = eV
\ Total number of orbitals associated with third shell n2
= (3)2 = 9 where n = 2, 3, 4 ....
Putting n = 2
h
11. (a) Orbital angular momentum, mvr = l ( l + 1) -13.6
2p ET = = -3.4eV
Hence, it depends only on 'l', l can have values ranging 4
from 0 to (n – 1). 21. (d) Two electrons occupying the same orbital should
have opposite spins i.e. they differ in spin quantum number.
12. (c) The fractional atomic mass (35.5) of chlorine is due
22. (b) As electron of charge 'e' is passed through 'V' volt,
to the fact that in ordinary chlorine atom, Cl-37 and Cl-35 kinetic energy of electron will be eV
are present in the ratio of 1 : 3.
h
\ Average atomic mass of Cl Wavelength of electron wave (l) =
2 m.K.E
3 ´ 35 + 1 ´ 37 h
= = 35.5 amu l=
4 2 meV
13. (b) From de-Broglie equation, wavelength, h
h Þ\ = 2meV
l= l
mv
STRUCTURE OF ATOM 489

23. (c) For hydrogen like atoms, energy of 2s-orbital and 30. (b) The possible number of orbitals in a shell in term of
2p-orbital is equal. 'n' is n2
24. (d) Radius of nth Bohr orbit in H-atom = 0.53 n2Å n = 5 ; n2 = 25
Radius of IInd Bohr orbit = 0.53 × (2)2 = 2.12 Å Exercise - 4
V 1
25. (d) Angular speed is . Vn µ and rn µ n 2 . 6.626 ´ 10-34 ´ 3 ´ 108
r n hc
1. (d) E=n Þ 2 × 10–17 = n ×
\ Angular speed is inversely proportional to n3. l 595.2 ´ 10-9
26. (a) 5f 5+3=8 Þ n = 60
6p 6+1=7 2. (a) The energy of an orbital is given by (n + l). In (D) and
5p 5+1=6 (E), (n + l) value is (3 + 2) = 5, hence they will have same
4d 4+2=6 energy, since here n values are also same.
5f > 6p > 5p > 4d (h = 6.63 × 10–34 Js)
27. (b) Balmer series 1 æ ö
28. (b) n µ DE 3. (d) = R H Z2 ç 1 - 1 ÷
l
For H-atom è n12 n2 2 ø
To calculate shortest wavelength take n2 = ¥ and longest
é1 1 ù
n = Rê - ú wavelength take nearest value of n2.
2 2
ëê n1 n2 ûú For H-atom, to calculate
For Lyman series, 1
, n2 = ¥ , Z = 1, n1 = 1
æ 1ö l shortest
n (max) = 13.6 ç1 - ÷
è ¥ø
1
æ 1ö \ = RH (Lyman series)
n (min) = 13.6 ç1 - ÷ x
è 4ø 1
For l longest for Li 2+ , Z = 3, n1 = 2, n2
æ 1ö
\ n max - n min = 13.6 ç ÷
è 4ø = 3 (Balmer series)
For Balmer series, 1 1 æ 1 1 ö 5
= × 3 2 çç 2 - 2 ÷÷ =
æ1 1ö l longest x è2 3 ø 4x
n (max) = 13.6 ç - ÷
è 4 ¥ø 4x
\ l longest =
æ 1 1ö 5
n (min) = 13.6 ç - ÷ 4. (a) Total number of spectral lines given by
è 4 9ø
1
\
æ 1ö
n max - n min = 13.6 ç ÷ 2
[ n - 1] ´ n = 15; \ n = 6
è 9ø
Thus, electron is excited upto 6th energy level from ground
Dn Lyman 9 state. Therefore,
=
Dn Balmer 4 1 é1 1ù 35
= R H ê - ú = 109737 ´ ;
29. (d) As we know, l ë12 2
6 û 36
æ 1 1ö l = 9.373 × 10–6 cm = 937.3 Å
n = – RH ç 2 - 2 ÷ Z2 (where, Z = 1)
è n2 n1 ø 5. (a) There is only one type of transition from n = 2 to
After putting the values, we get n = 1 and hence emitted radiation will constitute only one
æ 1 1ö frequency.
n = – RH çè 2 - 2 ÷ø 6. (c) As per Einstein's equation of photoelectric effect
n 8
hv = hvo + K.E.
RH RH
Þ n = - 1 2 hc hc
64 n 2 \ mv = hv - hvo = -
2 l lo
Comparing to y = mx + c, we get
1 2hc æ 1 1 ö
x = 2 and m = – RH (slope) v2 = - ;
n m çè l l o ÷ø
EBD_8350
490 CHEMISTRY

1/2 h
é 2hc æ 1 1 ö ù (for Li 3+ )
v=ê l Li3+ =
ç - ÷ú 2(3eV)mLi
ëê m è l l o ø ûú
1/2
é 2hc æ l o - l ö ù l Li3+ 2Vem p 1
Þ ê ç ÷ú \ = =
êë m è ll o ø úû lp 6eVm p ´ 9 3 3
7. (c) n1 + n2 = 4 ............. (i) n2 – n1 = 2 ........... (ii)
n1 = 1, n2 = 3 12. (b) Let wavelength of particle be x
x
æ 1 1ö So, velocity (v) =
2 100
n = RH Z ç 2 - 2 ÷
è n1 n2 ø
h h ´ 100
l= ; x=
é1 1 ù mv m´x
= R H ´ 32 ´ ê - ú = 8 R H
ë11 32 û h h
x 2 = 100 or x = 10
h 1 m m
8. (a) We know l = and K.E. = mv 2
mv 2 13. (a) En – E2 = 10 + 17 = 27 eV ....(i)
From the above relation, we get En – E3 = 4.25 + 5.95 = 10.2 eV ....(ii)
hv Substracting eq. (ii) from eq. (i):
K.E.=
2λ E3 – E2 = 16.8 eV
é 1 2 ù 13.6 Z 2 13.6 Z 2
êQ K .E. = 2 mv ú Þ - + = 16.8
ê ú 32 22
ê \ mv = 2 K .E. ; l = hv or K .E. =
hv ú Þ Z= 3
ëê v 2 K .E. 2l úû From eq. (i):
Since K.E. for two particles is same, so l µ v i.e., option (a). 13.6 Z 2 13.6 Z 2
9. (b) Y2 gives the probabilitiy of finding an electron in a - + = 27
n2 4
given volume of space. The 1s orbital is spherically
13.6 ´ 9 13.6 ´ 9
symmetrical so the probability of finding electron at any Þ - + = 27
given point depends only on its distance from the nucleus. n2 4
The probability density is greatest at r = 0 (at nucleus) and Þ n2 = 36 Þ n = 6
decreases with increasing distance. 14. (a) Number of moles of He gas
In contrast, the surface area of each spherical shell is
equal to 4pr2, which increases very rapidly with increasing æ 560 ö
çè ÷ ´ 1.5
r. The radial probability distribution function 4pr2y2 is PV 760 ø
n= = = 0.027 moles
equal to zero at nucleus (r = 0). RT 0.0822 ´ 500
90
10. (d) Energy of photon corresponding to second line of Number of moles of He+ ions = 0.027 × = 0.024
100
Balmer series for Li2+ ion
Number of moles in n = 3 level = 0.02 moles
2é 1 1ù 27 Number of moles in n = 2 level = 0.002 moles
= (13.6 ) ´ ( 3 ) ê 2 - 2 ú = 13.6 ´
ë2 4 û 16
Energy needed to eject electron from n = 2 level in æ 1 1ö
H-atom; Energy evolved DE = 2.18 × 10–18 × Z2 ç 2 - 2 ÷ J
è n1 n2 ø
2 é 1 1 ù 13.6
= 13.6 ´ 1 ´ ê 2 - 2 ú Þ Energy evolved from transition n = 3 to n = 1
ë2 ¥ û 4
K.E. of ejected electron æ1 1ö
DE = 2.18 × 10–18 × 22 çè 2 - 2 ÷ø J
9 ´ 3 13.6 æ 27 - 4 ö 1 3
= 13.6 ´ - = 13.6 ´ ç ÷ Þ 19.55 eV DE = 7.75 × 10–18 J
16 4 è 16 ø
11. (a) From de-Broglie equation, From 0.02 moles:
h DE1 = 0.02 × 6.023 × 1023 × 7.75 × 10–18 J
l= ;
2me v DE1 = 93.36 kJ
h Energy evolved from transition n = 2 to n = 1
lp = (for proton)
2mp ev æ1 1ö
DE = 2.18 × 10–18 × 22 çè 2 - 2 ÷ø J
1 2
STRUCTURE OF ATOM 491

DE = 6.54 × 10–18J
From 0.002 moles:
DE2 = 0.002 × 6.023 × 1023 × 6.54 × 10–18J 3d 3p 3s
DE2 = 7.88 kJ
Total energy evolved = DE1 + DE2 = (93.36 + 7.88)kJ

4p r2y 2
» 101 kJ
15. (c) The 3s-orbital is more extended/spread out in space
than the 3p-orbital which is in turn more extended than 3d-
orbital. Thus, radii of maximum probability decreases in
the order : (rmax)3s > (rmax)3p > (rmax)3d
r
EBD_8350
492 CHEMISTRY

3 Classification of Elements and Periodicity in Properties

Exercise - 1 23. (b) Its valence shell has 5 electrons (ns2, np3). It belongs
to 15th group of the periodic table.
1. (c) Periodic classification of elements follow a logical 24. (c)
consequence of electronic configuration of atoms which 25. (c) Element with Z = 33
is used to examine the physical and chemical properties of 2 2 6 2 6 10 2 3
the elements. (1s 2 s p 3s p d 4 s p ) lies in fifth (or 15th) group.
2. (c) According to Dobereneir’s triad the atomic mass of 26. (c) Elements (a), (b) and (d) belong to the same group
Br will be average of the atomic masses of Cl & I since each one of them has two electrons in the s sub shell.
35.5 + 127 In contrast, element (c) has seven electrons in the valence
= = 81.25 shell and hence does not lie in the same group in which
2
elements (a), (b) and (d) lie.
3. (a) Down is Gr. 15 : P, As, Sb
27. (c) The elements have 7 electrons in the valence shell.
4. (b) Lothar Meyer plotted the physical properties such as
28. (b) Na and Cl both belongs to III period.
atomic volume, melting point and boiling point against
29. (b) 30. (b)
atomic weight and obtained a periodically repeated pattern.
5. (a) 6. (b) 31. (b) 1s 2 , 2s 2 2 p6 , 3s 2 3 p 4
1424 3
7. (b) They have similar properties.
¯
8. (b) Correct order is Dobereiner, Chancourtois, Newlands,
Number of shell = 3
Mendeleev.
(Principal quantum number)
9. (d) Chloride formulas
Number of period = 3
(i) Eka-Aluminium = GaCl3(ECl3)
Valence electrons = 6 i.e., 2 + 4
(ii) Eka-Silicon = GeCl4(ECl4)
Mendeleev arranged elements in horizontal rows and Number of group = 16
vertical columns of a table in order of their increasing 32. (a) Metallic character decreases down a group and
atomic weights. increases along a period.
10. (a) Iodine with lower atomic weight than that of tellurium 33. (b)
(group VI) was placed in group VII along with fluorine, 34. (b) Non-metals are mainly placed in p-block elements.
chlorine, bromine because of similarities in properties. 35. (b) Non-metallic character increases on moving from left
11. (d) Oxides of Eka-Aluminium = Ga2O3 to right in a period.
Oxides of Eka-Silicon = SiO2 36. (d)
Melting point of Eka-Aluminium = Low (302 K) (i) Metals are on the left side of periodic table.
Melting point of Eka-Silicon = High (1231 K) (ii) O belongs to 2nd and P, S belong to 3rd period.
12. (c) Moseley discovered that atomic number is more 37. (d)
fundamental property than atomic mass. 38. (a) Seventh period includes most of the man-made radio-
13. (d) 6th period consists of 32 elements. active elements.
14. (b) 15. (b) 16. (b) 39. (a) Atomic number of the given element is 15 and it
17. (a) Atomic number (Z) = 120 belongs to 15th group. Therefore, atomic number of the
IUPAC name = Unbinilium element below the above element
Symbol = Ubn = 15 + 18 = 33.
18. (c) Digit Name 40. (b) These are characteristic properties of d-block
1 un elements.
4 quad 41. (b) X has + 2 charge and Mg can also form Mg2+ ion.
Using above notation IUPAC name of element 114 is 42. (c)
Ununquadium. 43. (c) Number of elements in each period is twice the number
19. (b) Element with atomic number 104 was named by of atomic orbitals available in the energy level that is being
American society as Rutherfordium and Kurchatovium filled.
by soviet society. 44. (a) By observing principal quantum number (n). Orbital
20. (b) (s, p, d, f ) and equating no. of e–’s we are able to find the
21. (c) Halogens are most electronegative elements i.e., they period, block and group of element in periodic table.
are likely to form anions most readily. 45. (d) Barium has atomic number 56. It is an alkaline earth
22. (c)
metal i.e., found in s-block.
CLASSIFICATION OF ELEMENTS AND PERIODICITY IN PROPERTIES 493

46. (b) 65. (c) All the given species are isoelectronic. In case of
47. (d) In fourth period filling up of 3d orbital becomes isoelectronic species, ionic radii increases with increase
energetically favourable before the 4p orbital is filled. in negative charge on anions.
48. (c) 66. (c) The increase in the electronegativities across a period
49. (c) He (1s2) should be placed along with s-block elements is accompanied by an increase in non-metallic properties
because of its electronic configuration but it has a (or decrease in metallic properties) of elements.
completely filled valence shell and as a result it exhibits 67. (c) 5B : 1s22s22p1
properties of noble gases, thus it is placed along with noble 2 2
4Be : 1s 2s
gases (ns2, np6). First ionisation enthalpy of B is lower than Be because Be
50. (b) 51. (d) has a stable electronic configuration. It required more
52. (d) Neptunium and plutonium with actinium and energy to remove the first electron from 2s (in Be) than 2p
protoactinium are also found in pitchblende.
(in B) because 2s e– has more penetration power than 2p.
53. (b) Elements A.No. Period Therefore options (I), (II) and (III) are correct. Atomic radius
A 2(He) 1 of B is less than Be.
B 3(Li) 2 68. (d) All the given species contains 10 e – each i.e.
C 7(N) 2 isoelectronic.
D 10(Ne) 2 For isoelectronic species anion having high negative
E 30(Zn) 4 charge is largest in size and the cation having high positive
Thus, B, C and D belong to the same period. charge is smallest.
54. (a) These are isoclectronic ions. 69. (c) Assertion is true but reason is false.
55. (d) The size of an anion will be larger than that of the Electronegativity refers to the tendency of atom to attract
parent atom because the addition of one or more electron(s) bonding electrons.
would result in increased repulsion among the electrons 70. (d) On moving down in a group atomic radii increases
and a decrease in effective nuclear charge. due to successive addition of extra shell hence
56. (d) ns1 configuration and lesser IE. O < S < Se
57. (a) The halogen (group-17) and the chalcogens Further As is in group 15 having one less electron in its p
(group-16) are two groups of elements having highly orbital hence have higher atomic radii than group 16
negative electron gain enthalpies. elements.
58. (d) i.e., O < S < Se < As
59. (d) Noble gases have positive values of electron gain 71. (c) highest positive charge on C2+.
enthalpy because the anion is higher in energy than the 72. (c) I.E. decreases down the group.
isolated atom and electron. 73. (a) Mg = 1s2 2s2 2p6 3s2
60. (b) It is electronic configuration of alkali metal. Hence it After removing of 2 electron, the magnesium acquired noble
will form basic oxide. gas configuration hence removing of 3rd electron will
61. (d) require large amount of energy.
62. (b) On passing from left to right in a period acidic 74. (b) As I.E. of Mg is more.
character of the normal oxides of the elements increases 75. (b) O < S < F < Cl
with increase in electronegativity. Electron gain enthalpy –141, –200, – 333, – 349 kJ mol–1
63. (c)
76. (a) Adding an electron to the 3p3 stable orbital leads to
64. (b) A. Li+ < Al2+ < Mg2+ < K+
greater repulsion than adding an electron to the 3p4 orbital.
The cation with the greater positive charge will have a
Hence, phosphorus has the least negative electron gain
smaller radius because of the greater attraction of the
electrons to the nucleus. Anion with the greater negative enthalpy.
charge will have the larger radius. 77. (a) Electronegativity increases along a period.
78. (a) Electronegativity values of given elements are as
1 follows:
Positive charge µ
ionic radius Be – 1.5 (I) Mg – 1.2 (IV)
Negative charge µ ionic radius O – 3.5 (II) N – 3.0 (III)
B. Greater positive charge, increases effective nuclear i.e. II > III > I > IV
charge in case of isoelectronic species. While for same 79. (d) Helium (He) 1s2 ® Highest ionisation
group elements effective nuclear charge decreases down energy due to noble
the groups. gas in nature.
C. Cl > F > Br > I Fluorine (F) 1s2, 2s22p3 ® High electronegativity
electron affinity of Cl is highest in halogen family. in nature due to small
D. F > Cl > Br > I size and –1 oxidation
electronegativity of fluorine (F) is higher than Cl, Br and I. state.
EBD_8350
494 CHEMISTRY

Rubidium (Rb) ® Most electropositive 4. (c) In case of transition element, the order of filling of
element due to large electrons in various orbital is 3p < 4s < 3d. Thus, 3d orbital
atomic size. is filled only when 4s orbital gets completely filled.
Lithium (Li) ® Strongest reducing 5. (c) As we move in a group from Cl to I, the electron gain
agent due to greater enthalpy (i.e., energy released in electron gain) become
hydration energy. less and less negative due to corresponding increase in
80. (b) Second ionization enthalpy will be higher than the the atomic size.
first ionization enthalpy but lower than the third ionization However, the electron gain enthalpy of F is less negative
enthalpy. than that of Cl due to its small size. Thus, the negative
81. (b) In case of halogens covalent radius is considered electron gain enthalpy among halogens follows the order:
this bond is formed by overlapping of electron clouds; F < Cl > Br > I
while noble gases remain monoatomic, in this case only 6. (c) As each period starts with the filling of electrons in a
way to obtain radius is through van der Waal radii. new principal quantum number, so, the period number in
82. (c) Covalent radius is radius of an atom in its bound the long form of the periodic table refers to the maximum
state i.e., in fluorine it is half of distance between two principal quantum number (n) of any element in the period.
covalently bonded fluorine atoms; van der Waal radii is 7. (c) The elements in which electrons are filled in 4f-orbital
one-half of the distance between the nuclei of two identical are called lanthanoids. Lanthanoids consist of elements
non-bonded isolated atoms. These atoms are attracted from Z = 58 (cerium) to 71 (lutetium).
toward each other through weak van der Waal’s force 8. (d) Generally, cations are smaller in size while anions are
hence van der Waal radii are very large. bigger in size than the neutral atom.
83. (c) I represents Li, II represents K 9. (a) Electronic configuration for the given elements will
III represents Br, IV represents I be :
V represents He Na = [Ne]3s1, Mg = [Ne]3s2, Al = [Ne]3s23p1,
So, amongst these, II represents most reactive metal and V Si = [Ne] 3s23p2
represents least reactive non-metal. Ionisation enthalpy increases along a period but I.E of
84. (b) Because of stable configuration in +1 state. Mg is higher than Al because of completely filled 3s orbital
85. (b) in Mg.
86. (d) Atomic size generally decreases along a period. 10. (c) O2– has noble gas configuration and isoelectronic
87. (c) Electron affinity of 9F is less than that of 17Cl. with neon but its formation is unfavourable due to strong
88. (b) Fully filled electronic configuration. electronic repulsion between the negatively charged
89. (b) On moving along the period, ionization enthalpy O– ion and the electron being added.
increases. Thus, the electron repulsion will be more than the
In second period, the order of ionization enthalpy should stability gained by achieving noble gas configuration.
be as follows : 11. (c) The element with atomic number 57 belongs to
F > O > N. d-block element as the last electron enters into the 5d-
But N has half-filled structure, therefore, it is more stable orbital against the aufbau principle. This anomalous
than O. That is why its ionization enthalpy is higher than behaviour can be explained on the basis of greater stability
O. Thus, the correct order of IE is of the xenon (inert gas) core.
F > N > O. After barium (Z = 56), the addition of the next electron
90. (d) With the exception of lithium and beryllium should occur in 4f-orbital in accordance with aufbau
compounds of s-block elements are predominantly ionic. principle. This will however, tend to destabilize the xenon
core (Z = 54), [Kr] (4d10 4f 0 5s2 5p6 5d0) since the 4f-
Exercise - 2 orbitals lie inside the core.
1. (b) In case of isoelectronic species Therefore, the 57th electron prefers to enter 5d-orbital
1 which lies outside the xenon core and whose energy is
ionic radii µ only slightly higher than that of 4f-orbital.
atomic number
Thus, the outer electronic configuration of La(Z = 57) is
\ The correct order of increasing ionic radii will be :
5d1 6s2 rather than the expected 4f 1 6s2.
Ionic radii Mg 2 + < Na + < F- < O2 - 12. (c) Each period starts with the filling of electrons in a
Atomic number (12) (11) (9) (8 ) new principal energy shell. Therefore, 6th period starts
with the filling of 6s-orbital and ends when 6p-orbitals are
2. (d) Elements with atomic number, Z = 90 to 103 are called completely filled.
actinoids. Terbium belongs to lanthanoids. Thus, the outermost electronic configuration of the last
3. (a) For a given shell, screening effect decreases in the element of the p-block in the 6th period is represented by
order : s > p > d > f. 6s2 4f 14 5d10 6p6 or 4f 14 5d10 6s2 6p6.
CLASSIFICATION OF ELEMENTS AND PERIODICITY IN PROPERTIES 495

13. (c) The long form of the periodic table contains element 22. (b) Alkali metals have the lowest ionization energy in
with atomic number 1 to 118. each period on the other hand Sc is a d-block element.
14. (a) The fifth period begins with Rb (Z = 37) and ends at Transition metals have smaller atomic radii and higher
Xe (Z = 54). Thus, the element with Z = 43 lies in the 5th nuclear charge leading to high ionisation energy.
period. Since, the 4th period has 18 elements, thus, the 23. (a) Z = 114 belong to Group 14, carbon family
atomic number of the element which lies immediately above Electronic configuration = [Rn]5f 146d107s27p2
the element with atomic number 43 will be 43 – 18 = 25. 24. (a) Isoelectronic species have same no. of electrons.
The electronic configuration of the element with Ions O2– F– Na+ Mg2+

No. of e = 8 + 2 9 + 111 – 1 12 – 2
Z = 25 is
1s2 2s2 2p6 3s2 3p6 3d 5 4s2 (i.e., Mn). 10 10 10 10
15. (b) The elements with atomic numbers 35(36 – 1), \ O2–, F–, Na+ , Mg2+ are isoelectronic
53( 54 – 1) and 85(86 – 1), lie in a group before noble 25. (a) Mg can form basic carbonate like
gases, i.e., belongs to halogens (group 17). 3MgCO3 . Mg(OH)2 . 3H2O
16. (a) Electronic configuration of given elements indicate while Li can form only carbonate (Li2CO3) not basic
that A is a noble gas (i.e., Ne), B is oxygen, C is sodium carbonate.
metal and D is fluorine. 26. (b) Consider the stability of electronic configuration after
(i) Noble gases have no tendency to gain electrons loss of one electron.
since all their orbitals are completely filled. 27. (d) Alkali metals have high difference in the first and
second ionisation energy as they achieve stable noble
\ element A will have the least electron gain enthalpy.
gas configuration after first ionisation.
(ii) Element D has one electron less and element B has 28. (c) Generally, electronegativity decreases down the group
two electrons less than the corresponding noble gas as the size increases. This can also be formulated as:
configuration, hence, element D will have the highest
electron gain enthalpy in comparison to element B. 1
Electronegativity µ
(iii) Since, element C has one electron in the s-orbital and size
need one more electron to complete its configuration, 175
therefore, electron gain enthalpy of C is less than that of 29. (d) 71 Lu

element B. So, we can conclude that the electron gain No. of protons = No. of electrons = 71
enthalpies of the four elements increases in the order : No. of neutrons = Mass no. – No. of protons
A < C < B < D. = 175 – 71 = 104
17. (N) All options are incorrect. 30. (c) Unununium (Z = 111), it is Roentgenium (Rg) not
18. (b) In isoelectronic species the radius decrease with Darmstadtium.
increase in nuclear charge hence increasing order of radius 31. (a) CO : Neutral oxide
is Ca2+ < K+ < Ar BaO : Basic oxide
19. (a) Incoming electrons occupies the smaller n = 2 shell, Al2O3 : Amphoteric oxide
also negative charge on oxygen (O–) is another factor due Cl2O7 : Acidic oxide
to which incoming electron feel repulsion. 32. (c) Chlorine has highest electron gain enthalpy (most
Hence electron repulsion outweigh the stability gained negative) among the given elements, the electron gain
by achieving noble gas configuration. enthalpy decreases down the group i.e., moves to least
20. (a) For isoelectronic species, size of anion increases as negative.
negative charge increases. Thus, the correct order is
Exercise - 3
N3– O2– F–
1.71 1.40 1.36 1. (a) IE1 is always less than IE2.
21. (b, c) The correct order is B < C < O < N 2. (b) Ionization energy is not the only criteria for the stability
Generally ionisation energy increases across a period. But
of an oxidation state.
here first I.E. of O is less than the first I.E. of N. This is due
to the half-filled 2p orbital in N (1s2, 2s2, 2p3) which is more 3. (d) O(g) + 2e - ® O2- , DH = 603kJ mol-1 ......(i);
stable than the 2p orbital in O (1s2, 2s2, 2p4).
The correct order of electron affinity is: I < Br < F < Cl O(g) + e - ® O- , DH = -141 kJ mol–1 ......(ii)
Halogens have high electron affinities which decreases
on moving down the group. However, fluorine has lower (i) - (ii) gives :
value than chlorine which is due to its small size and O- (g) + e - ® O 2 - (g),
repulsion between the electron added and electrons already
present. DH = 603 - (-141) = 744kJ mol–1
EBD_8350
496 CHEMISTRY

Order of energy releases is : DH4 > DH1 > DH3 > DH2
1
4. (d) Number of moles =
35.5 So, P - (g) ¾¾
® P(g) + e - requires least energy..
Given, 1 eV = 23.06 kcal mol–1 9. (d) For option (a) and (b) use (z/e) concept for
3.7 eV = 3.7 × 23.06 kcal mol–1 isoelectronic species.
i.e. 1 mole realease energy = 3.7 × 23.06 kcal In option (c) size of neutral atom is greater than its cation.
In option (d) Se2– and As3– are related with 4th period,
1
\ Energy released = ´ 3.7 ´ 23.06 kcal = 2.4 kcal while Ba2+ and Cs+ related with 6th period (these are not
35.5
isoelectronic species).
5. (d) As the size increases, the basic nature of oxides 10. (d)
changes to acidic nature i.e., acidic nature increases.
W : Phosphorus Y : Oxygen X : Sulphur Z : Chlorine
SO 2 > P 2 O 3 > SiO 2 > Al 2 O 3 Electronegativity O > Cl > S > P Catenation : S > P > O > Cl
Acidic Weak Amphoteric Electron Affinity : Cl > O > S > P Oxygen exhibits covalency of two only
acidic
11. (a) The electronegativity difference between M1 and O
SO2 and P2O3 are acidic as their corresponding acids H2SO3
and H3PO3 are strong acids. is 0.1, which indicates M1– O bond will be covalent, since
O–H bond having more ionic character thus bond will
6. (d) Successive electron affinity always decrease
break and H+ ions release and acidic solution is formed
+e
O ¾¾® O- DH eg = (-) ve and whereas difference between electronegativity of
+e M2-O is 2.3, thus, M2–OH bond will break. Hence, solution
O - ¾¾® O 2 - DH eg = (+ ) ve
will be basic in nature.
+2e
® O2-
O ¾¾¾ DH eg (Total) = ( + ) ve 12. (b) The third period element Cl has large size and vacant
Thus, O2– will show most positive DHeg, while O2+
will 3d orbitals, which can accommodate incoming electron
show most negative value due to small size. easily compare to second period element (F).
7. (b) Mg2+ and O2– ions have higher charge, and size of Li + So order is F (g) < Cl (g).
and F– are smaller than K+ and Br – . 13. (b) He (Z = 2) is a noble gas and has highest I.E. The I.E.
8. (b) Let us consider the opposite process : of Be (Z = 4) is more than that of Li (Z = 3)
14. (c) Generally, non-metal oxides are acidic in nature and
(a) F(g) + e - ¾¾
® F- (g) ; DH1
metal oxides are basic in nature, Al2O3 is amphoteric.
(b) ® P - (g) ; DH2
P(g) + e - ¾¾ 15. (d) Nuclear charge : B > Be
Be = 1s2 2s2 (more stable)
(c) S(g) + e - ¾¾
® S- (g) ; DH3 B = 1s2 2s2 2p1
Ionisation energy of Be is greater than B due to ns2 outer
(d) Cl(g) + e - ¾¾
® Cl - (g) ; DH4 electronic configuration.
CHEMICAL BONDING AND MOLECULAR STRUCTURE 497

4 Chemical Bonding and Molecular Structure

Exercise - 1 9. (a) PCl5 does not follow octet rule, it has 10 electrons in
its valence shell.
1. (d) The evolution of various theories of valence and the 10. (a) Atoms combine either by transfer of valence
interpretation of the nature of chemical bonds have closely electrons from one atom to another or by sharing of
been related to the developments in the understanding of valence electrons to have an octet in their valence shell.
structure of atom, electronic configuration of elements and 11. (c) Each combining atom contributes at least one
Periodic Table. electron to the shared pair.
2. (b) Kossel and Lewis provide some logical explanation 12. (a) Only (I) is correct. The correct Lewis representation
of valence which was based on the intertness of noble for (II) and (III) are:
gases.
Å Å
3. (b) Langmuir (1919) refined the Lewis postulations by

: :

: :
O O=N–O–H
abandoning the idea of the stationary cubical arrangement

: —

O :O:
of the octet, and by introducing the term covalent bond. :O:

:
4. (b) Electrovalent bond is formed as a result of II
electrostatic attraction between the positive and negative
ions. .. .. .. .. ..
13. (d) O .. S .. O..
5. (d) When a metal for example Na combines with a non ..
metal e.g., Cl2. Following reaction occurs
Total no. of valence electron around sulphur in SO2 is 10,
2Na + Cl 2 ¾¾ ® 2NaCl while in case of other molecules total no. of 8 electrons
In this process Na loses one electron to form Na+ and Cl are present in each.
accepts one electron to form Cl–
Å
® Na + + e-
Na ¾¾ 14. (a) N = O and O = Cl – O have all electrons.
Cl + e - ¾¾
® Cl - 15. (c) Sulphur forms many compounds in which the octet
Therefore, in this process Cl gains electron and hence, its rule is obeyed. For example, SCl2 has an octet of
size increases. electrons around it.
6. (a) The lowest energy structure is the one with the 16. (d) Lattice enthalpy is required to completely separate
smallest formal charges on the atoms. one mole of a solid ionic compound into gaseous
1 constituent ions.
:

O 17. (c) Higher the difference in electronegativity between the


7. (a) 2 3 two atoms, more will be electrovalent character of the bond.
: :
: :

O O:
Among given choices, calcium and hydrogen have
Formal charge = (total number of valence e– in the free maximum difference in their electronegativities.
1 18. (b) Ionic bonds will be formed more easily between
atom) – (total number of non-bonding e–) – (total number
2 elements with comparatively low ionization enthalpies
of bonding e–) and elements with comparatively high negative value of
1 electron gain enthalpy.
Formal charge on O1 = 6 – 2 – (6) = +1
2 19. (a) In ionic solids, the sum of the electron gain
1 enthalpy and the ionization enthalpy may be positive
Formal charge on O2 = 6 – 4 – (4) = 0 but still the crystal structure gets stabilized due to the
2
1 energy released in the formation of the crystal lattice.
Formal charge on O3 = 6 – 6 – × 2 = –1 20. (b) For compounds containing cations of same charge,
2
Hence, correct representation of O3 is lattice energy increases as the size of the cation decrease.
(+1) Thus, NaF has highest lattice energy. The size of cations
is in the order Na+ < K+ < Rb+ < Cs+
:

O
(0) (–1) 21. (b) The stability of the ionic bond depends upon the
: :
: :

O O: lattice energy which is expected to be more between Mg


8. (b) The compounds in which octet of central atom is and F due to +2 charge on Mg atom.
incomplete are known as electron deficient compounds. 22. (c) Because of the same bonded atoms, CO2 and CCl4 are
Hence, B2H6 is an electron deficient compound. non-polar molecules.
EBD_8350
498 CHEMISTRY

23. (d) Assertion is false but reason is true. 39. (b) Sigma electrons are not mobile e–.
The greater the lattice enthalpy, more stable is the ionic 40. (c) According to Fajan's rule, smaller size and greater
compound. charge on cation favour the formation of partial covalent
24. (c) I and II structure shown above constitute the character in ionic bonds.
cannonical structure. III structure represents the structure 41. (a) According to Fajan’s rule, as the charge on the cation
of O3 more accurately. This is also called resonance hybrid. increases, and size decreases, its tendency to polarise the
25. (c) The dipole moment of symmetrical molecules is zero. anion increases. This brings more and more covalent nature
F to electrovalent compounds. Hence AlCl3 shows maximum
| covalent character.
| B 42. (c) In case of SF6 resultant dipole moment is zero
F F
|

Triangular planar (symmetrical molecule)


(symmetrical shape) while all other possess dipole moment.
43. (b) Covalent radius is half of the distance between atoms
d+ d-
26. (a) C - F in bonding state, while van der Waal radius is half of the
Because difference between electronegativity of carbon distance between atoms in its non bonding state.
and flourine is highest. 44. (d) The electronegativity difference is maximum in O—H
27. (c) Ionic bond with small cation and large anion has bond hence O—H bond length is the smallest among the
covalent character. given set.
28. (d) Assertion is false but reason is true. 45. (c) CCl4 being symmetrical has zero dipole moment. H2O,
CH2Cl2 is polar while CCl4 is non-polar because in CCl4 net CHCl3 and NH3 have dipole moments of 1.84 D, 1.01 D and
dipole moment cancels. 1.46 D respectively. Thus, among the given molecules H2O
has highest dipole moment.
29. (c) As difference of electronegativity increases % ionic
character increases and covalent character decreases i.e., 46. (d) According to Fajan's rule :
electronegativity difference decreases, covalent character 1
Covalent character µ
increases. size of cation
Further, greater the charge on the cation and smaller the µ size of anion
size, more will be its polarising power. Hence, covalent Among the given species order of size of cations
character increases. N3+ < O2+ < Pb2+ < Ba2+
30. (d) In case of anions having same charge as the size of order of size of anions O2– > Cl–.
anion increases, polarisibility of anion also increases. Hence the order of covalent character is
31. (a) A gaseous HCl molecule has hydrogen and chlorine NCl3 > Cl2O > PbCl2 > BaCl2
linked by a covalent bond. Here, electronegativity of BaCl2 is least covalent in nature.
chlorine is greater than that of hydrogen. Due to this, the 47. (a) The number of lone pairs of electrons on central atom
shared pair of electron is more attracted towards chlorine. in various given species are
Thus, chlorine end of molecule has higher electron density Species Number of lone pairs on
and becomes slightly negative and the hydrogen slightly central atom
positive. Hence, the covalent bond in HCl has a polar
IF7 nil
character as shown below:
+ -
IF5 1
d d ClF3 2
H Cl
XeF2 3
32. (b) O C O
Thus the correct increasing order is
33. (d) Bond lengths are measured by spectroscopic, X-ray
diffraction and electron diffraction techniques. IF7 < IF5 < ClF3 < XeF2
34. (c) The covalent radius is measured approximately as 0 1 2 3
..
the radius of an atom’s core which is in contact with the S
core of an adjacent atom in a bonded situation. 48. (d)
O O
35. (d) All of the given statements are correct. lp = 1
36. (b) Both NO2 and O3 have angular shape and hence will bp = 4
have net dipole moment. 49. (a) V-shaped H2O like structure.
37. (c) Both representation of resonating structures in 50. (a) BF3 is sp2 hybridised. So, it is trigonal planner. NH3,
molecules of CO2 and CO2– 3 are correct. PCl3 has sp3 hybridisation hence has trigonal pyramidal
38. (a) The molecule does not exist for a certain fraction of shape, IF3, has sp3d hydridization and has T shape.
time in one cannonical form and for other fractions of ..
Sn
time in other cannonical forms. It exists as a resonance 51. (d) Bent
hybrid. Cl Cl
CHEMICAL BONDING AND MOLECULAR STRUCTURE 499

52. (d) The geometry of a covalent molecule depends on the 60. (a) Both assertion and reason are true and reason is the
sigma bond and lone pair of electrons left with central correct explanation of assertion.
atom. lone pair – lone pair repulsion > lone pair – bond pair
53. (b) According to VSEPR theory, the molecule is of AB3E repulsion > bond pair – bond pair repulsion. In the ammonia
type in which arrangement of electron pairs are in tetrahedral molecule, NH3 there are three bond pairs and one lone pair.
shape. But due to a lone pair of electrons, the actual shape The three N – H bond pairs are pushed closer because of
is trigonal pyramidal. the lone pair – bond pair repulsion, and HNH bond angle
gets reduced from 109°23’ (the tetrahedral angle) to 107°.
61. (c) Trigonal planar = BF3
Cl Tetrahedral = NH4+
O–
O O Trigonal bipyramidal = PCl5
54. (b) NH3 ® 1lp, 3bp ® Trigonal pyramidal Octahedral = SF6
SO2 ® 1lp, 2bp ® Bent 62. (a) It is to minimize lone pair – bond pair repulsions only.
SF4 ® 1lp, 4bp ® See-saw The three possible structures of BrF3 molecule are:
• One lp – lp repulsion at ~ 120°
ClF3 ® 2lp, 3bp ® T-shape
• Two lp – bp repulsions at ~ 120°
55. (d) In BrF5 molecule, there are 5 bond pair and one lone
pair of electrons with the central atom. • Four lp – bp repulsions at 90°
• Two bp – bp repulsions at 90° and
F
F F • One bp – bp repulsion at 180°
F
Br :
F F Br F
:
:

Thus, this molecule has octahedral geometry and square F


pyramidal shape.
(I)
56. (a) While the lone pairs are localised on the central
• One lp – lp repulsion at 90°
atom, each bonded pair is shared between two atoms. As
• Three lp – bp repulsions at 90°
a result, the lone pair electrons in a molecule occupy
• Two lp – bp repulsions at ~ 120°
more space as compared to the bonding pairs of
• One lp – bp repulsions at 180°
electrons. This results in greater repulsion between lone
• Two bp – bp repulsions at 90° and
pairs of electrons as compared to the lone pair -bond
• One bp – bp repulsion at ~ 120°
pair and bond pair - bond pair repulsions.
57. (b) SO2 – bent F
SF4 – see-saw :
ClF3 – T-shape Br F
BrF5 – square pyramidal F
:

XeF4 – square planar. (II)


58. (b) X • One lp – lp repulsion at 180°
• Six lp – bp repulsions at 90° and
..

• Three bp – bp repulsions at 120°


M X
F
:
..

X F Br
Number of lone pair = 2 F
:

1 (III)
59. (b) XeF4 hybridisation is = (V + X - C + A)
2 Now, it is given that the BrF3 molecule prefers structure (I). In
hence V = 8 (no. of valence e– )
this structure, (a) lp – lp repulsions are not minimum; it is
X = 4 (no. of monovalent atom)
minimum in structure (III). (b) lp – bp repulsions are less
1 3 2 than structure (III). (c) bp – bp repulsions are more than
H = (8 + 4 + 0 - 0) = 6 sp d
2 structure (III).
F F 63. (a) I is the most stable geometry because both the lone
:

The shape is Xe square planar shape. pairs are present at equatorial position. Due to which, repulsion
is minimum in molecule as compared to the repulsion in other
F F molecules where lone pair is in axial position.
:
EBD_8350
500 CHEMISTRY

64. (d) Structure of acetylene molecule overlapping may result in s bond if overlapping takes place
1s
along internuclear axis or may result in p–bond if sideways
1s 1s
H— C º C— H overlapping takes place. In any case, the extent of
2p
overlapping is lesser in p - p than that of the other two,
Thus, acetylene molecule has 3s bonds and 2p bonds.
s-s and s-p. Hence, the correct order is
s s s s s s s s s
65. (b) H— C—
p C— C —
p C— C —
p C— C —
p C—H s-s>s-p>p-p
s s s s s s s s
H H H H H H H H 77. (a) Bond angle increases with increase in s-character of
1, 3,5, 7 - octatetraene
hybridised orbital. The table given below shows the
17 s and 4p hybridised orbitals, their % s-chatracter and bond angles.
66. (b) Allyl cyanide is : Hybridised % s-character Bond
H H orbitals angle
sp 3 25 109.5°
H s s s s
s s
C p
C s C C N sp2 33 120°
H s s p,p
sp 50 180°
H
\ It contains 9 sigma bonds, 3 pi bonds and 1 lone pair of 78. (d) The hybridisation in a molecule is given by
electrons. 1
H = [V + M - C + A] .......(i)
67. (a) Cl2 : Cl – Cl (1s , No p) 2
O 2 : O = O (1s , 1p) where V = no. of valency e– in central atom
N 2 : N º N (1s, 2p) M = no. of monovalent atoms around central atom
CO2 : O = C = O (2s, 2p) C = charge on cation, A = charge on anion
68. (a) The given molecule is For NH3; V = 5, M = 3, C = 0, A = 0
s Putting these values in (i), we get
s s s s
H C p C C p C C C H 1
2p H = [5 + 3 - 0 + 0] = 4
H H H H 2
For H = 4, the hybridisation in molecule is sp3.
The number of C—C ‘s’ bonds = 5
79. (d) Hybridisaiton of SO42– ion is given by
The number of C—C ‘p’ bonds = 4
The number of C—H ‘s’ bonds = 6 1
H = [V + M + A– C]
2
69. (a) Where V = valency of central metal atom
s-orbital p-orbital M = no. of monovalent atoms surrounding central metal
The overlap between s- and p-orbitals occurs along atom.
internuclear axis and hence the angle is 180°. A = charge on anion, C = charge on cation
70. (a) Linear combination of two hybridized orbitals leads For SO2– 4 ; V = 6, M = 0, A = 2, C = 0
to the formation of sigma bond. 1
\ H = [6 + 0 + 2 – 0] = 4
71. (b) Sigma bond is stronger than p-bond. The electrons in 2
the p bond are loosely held. The bond is easily broken and i.e., sp3 hybridisation and tetrahedral shape.
is more reactive than s -bond. Energy released during 80. (c) In case of Xenon compounds
sigma bond formation is always more than p bond because 1
of greater extent of overlapping Hybridization = [No. of valence electr ons of
2
Xe + number of monovalent atoms surrounding–charge
72. (b) The minimum in the energy curve corresponds to
on cation + charge on anion]
the most stable state of H2.
1
O–H In XeF6 = [8 + 6 – 0 + 0] = 7 i.e. sp3d3 hybridisaiton.
2
73. (a) In case of SF6, Sulphur is sp3d2 hybridised.
CH3 - C = CH 2 has 9s,1p and 2lone pairs.
In case of BrF5, Bromine atom is sp3d2 hybridised.
74. (c) pi bonds are formed by the overlapping of p-p orbitals In PCl5, P is sp3d hybridised.
perpendicular to their axis i.e., sidewise overlap. 81. (a) For linear arrangement of atoms the hybridisation
75. (c) Due to different orientation of direction of approach, should be sp (linear shape, 180° angle). Only H2S has sp3-
there is zero overlap between orbitals. hybridization and hence has angular shape while C2H2,
76. (d) The strength of a bond depends upon the extent of BeH2 and CO2 all involve sp - hybridization and hence,
overlapping. s-s and s-p overlapping results in the formation have linear arrangement of atoms.
of s bond but extent of overlapping along internuclear axis 82. (b) BF3 involves sp2-hybridisation.
is more in case of s-s overlapping than in s-p. p-p
CHEMICAL BONDING AND MOLECULAR STRUCTURE 501

sp 3 sp sp sp 3 I
:
83. (b) H3 C — C º C — C H3 linear part of molecule : I :
84. (b) Equilateral or triangular planar shape involves sp2
hybridisation. I
85. (b) In CH3 CH2OH underlined C is forming 4 σ bonds, 95. (d) Statement (d) is incorrect.
hence sp3 hybridisation. In others, it is sp2 hybridised (due 96. (a) SF6 Þ sp3d 2
to 3 σ bonds). PF5 Þ sp3d
86. (a) Each sp3-hybrid orbital has 25% s-character and 75% BCl3 Þ sp2
p-character.
C2H6 Þ sp3
sp 2 sp sp 2
97. (b) Hybridisation of the central atom in compound is
87. (a) (b) CH 2 = C = CH 2
given by
sp3 1
sp 3 sp3 H = [V + M - C + A]
2
(c) sp3 sp3 1
3 · For NO -2 , H = [5 + 0 - 0 + 1] = 3
sp 2
3
sp2 hybridisation
sp sp 2 sp 2 sp3
(d) CH - CH = CH - CH 1
3 3 · For SF4, H = [6 + 4 - 0 + 0] = 5
2
sp 2 sp 2 sp 2 sp 2 sp3d hybridisation
(a) CH 2 = CH - CH = CH 2 1
1, 3-butadiene · For PF6–, H = [5 + 6 - 0 + 1] = 6
88. (d) Hybridisation present in a molecule can be find out 2
by the following formula. sp3d 2 hybridisation.
98. (a) XeF6 having one lone pair of electron, show distorted
1 pentagonal bipyramidal shape and sp3d3 hybridisation.
H= (V + M - C + A)
2 39. (b) List I List II
Where V = No. of electrons in valence shell of central atom Compound Structure
M = No. of singly charged atoms
(A) ClF3 T-shaped
C = charge on cation
(B) PCl5 Trigonal bipyramidal
A = charge on anion
(C) IF5 Square pyramidal
1 (D) CCl4 Tetrahedral
So, Hybridisation (in ClF3) = [7 + 3 - 0 + 0] = 5
2 (E) XeF4 Square planar
Þ sp3d Hybridisation. + 100. (a) Only those d orbitals whose lobes are directed along
89. (a) Number of sigma bonds in CH3 are 3, thus, X, Y and Z directions hybridise with s and p orbitals. In
hybridisation is sp2. other three d orbitals namely dxy, dyz and dxz, the lobes are
Number of sigma bonds in CH4 are 4, thus, hybridisation is sp3. at an angle of 45° from both axis, hence the extent of their
sp 2 sp sp 2 overlap with s and p orbitals is much lesser than d 2 2
x -y
90. (c) (a) CH 2 = C = CH 2 and d 2 orbitals.
sp3 sp 2 sp 2
sp 2 z
(b) CH3 — CH = CH— CH 2+ 101. (c) Let amount of compound = 100 g
sp 2 92.3
sp3 sp sp No. of moles of C = = 7.69 = 7.7
(c) CH3 — C º C — CH 2+ 12
7.7
sp3 sp 2 sp 2 sp3 No. of moles of H = = 7.7
1
(d) CH3 — CH = CH — CH -2
Empirical formula = CH
91. (a) According to VSEPR theory, trigonal bipyramidal Empirical formula mass = 12 + 1 = 13 g/mol
geometry results from sp3d or dsp3 hybridisation.
Molecular mass = 52 g/mol
dsp2 hybridisation results in square planar geometry, while
d2sp3 leads to octahedral shape. 52
n = =4
sp 3 sp 2 sp sp 2 sp3 13
92. (c) H3C - CH = C = CH - CH3 \ Molecular foumula = Empirical formula × 4 = C4H4
93. (d) All the hybrid orbitals are equivalent. Possible structures
94. (c) I–3 has sp3d hybridisation and has linear structure.
EBD_8350
502 CHEMISTRY
H
H H 118. (d) In this configuration, there are four completely filled
sp sp H
C C C C H C C C C bonding molecular orbitals and one completely filled
, H, antibonding molecular orbital. So that Nb = 8 and Na = 2.
H sp 2
sp2
H 2 2
sp sp sp sp
1 1
H H \ Bond order = ( N b - N a ) = (8 - 2) = 3.
2
2 2
sp C C sp2 119. (c) N 2 + = 7 + 7 – 1 = 13 electrons
Configuration is
H C C H
s1s 2 s *1s 2 s2 s 2 s * 2s 2 p2 p x 2 =
= p2 p y 2s 2 p1z
sp2 sp2 Bond order =
102. (c) Molecular orbital theory was given by Mulliken.
1 æ No. of e- s in bonding - No. of e -s in antibonding ö
103. (c) Triple bond is stronger than double bond and which
is stronger than single bond. 2 çè molecular orbital molecular orbital ÷
ø
104. (a) Bonding influence is stranger if Nx > Ny . 1 1
= (9 - 4) = ´ 5 = 2.5
105. (a) Both assertion and reason are true and reason is the 2 2
correct explanation of assertion. 120. (a, b) The molecular orbital structures of C2 and N2 are
Helium molecule is formed by linking two helium atoms.
N2 = s1s 2 s *1s2 s2s 2 s *2s 2 s2 px2p2 py2 p2 pz2
Both have 1s orbitals. These will combine to form two
molecular orbitals s (1s) and s* (1s). Four available C2 = s1s 2 s *1s 2 s2s 2 s * 2s 2 p2 py 2p2Pz2
electrons are accommodated as s (1s)2 and s* (1s)2. Both N2 and C2 have paired electrons, hence they are
106. (b) One bonding M.O. and one anti-bonding M.O. diamagnetic.
107. (d) Molecules having unpaired electrons show
ìp 2 p 2x ì p* 2 p1x
paramagnetism. 2 * 2 2 * 2 2 ï ï
121. (b) O : s1s s 1 s s 2 s s 2 s s 2 p z í
108. (a) Oxygen molucules (O2) : 2 2 í * 1
ïîp 2 p y ïî p 2 p y
O2 : s1s2 s*1s2 s2s2 s*2s2 s2pz2 (p2p2x = p2p2y) 10 - 6
(p*2p1x = p*2p1y) Bond order = =2
2
109. (d) For oxygen, correct increasing order is (two unpaired electrons in antibonding molecular orbital)
s1s < s*1s < s2s < s*2s < s2pz < (p2px = p2py) < (p*2px ìï p 2 px2 ìï p* 2 p1x
O+2 : s1s 2 s*1s 2 s 2s 2 s* 2s 2 s 2 p2z í
= p*2py) < s*2pz 2í * 0
110. (b) There is a nodal plane between two nuclei. ïî p 2 p y ïîp 2 p y
111. (b) Atomic orbital is monocentric because an electron in 10 - 5
it, is influenced by one nucleus. While molecular orbital Bond order = = 2. 5
2
is polycentric as it is influenced by two or more nuclei (One unpaired electron in antibonding molecular orbital)
depending upon the number of atoms in the molecule. Hence, O2 as well as O +2 both are paramagnetic, and bond
112. (c) Stabilization energy decreases the energy of molecular
orbital formed. order of O +2 is greater than that of O2.
113. (b) Atomic orbitals having same or nearly same energy 122. (d) As the bond order decreases, bond length increases
will not combine if they do not have the same symmetry. Bond order
2pz orbital of one atom cannot combine with 2px or 2py No. of bonding e -s - No. of antibonding e -s
orbital of other atom because of their different symmetries. =
2
114. (c) The electron density in a bonding molecular orbital
is located between the nuclei of the bonded atoms For N2, electronic configuration is
because of which the repulsion between the nuclei is
very less, while in case of an antibonding molecular s1s 2 < s*1s 2 < s2s 2 < s* 2s 2 < (p2 px2 = p2 p 2y ) < 2 p 2z
orbital, most of the electron density is located away from 10 - 4
the space between the nuclei. (i) Bond order of N2 = =3
2
Electrons placed in a bonding molecular orbital tend to 9-4
hold the nuclei together and stabilise a molecule. (ii) Bond order of N +2 = = 2.5
2
115. (c) He2 = s1s2, s*1s2
2+ 8 - 4
1 (iii) Bond order of N 2 = =2
B.O. = [ Nb - Na ] = 0 ; cannot exist 2
2 Hence, order of Bond length will be,
116. (a) N2 has triple bond between the atoms. N2 < N+2 < N22+
117. (c) According to molecular orbital theory, bond order of Li2 123. (d) On calculating bond order of species given in
is 1 , while in all other cases bond order is 0, so they do not question
exist. Li2 molecules are known to exist in the vapour phase.
CHEMICAL BONDING AND MOLECULAR STRUCTURE 503

C2 = 2 C22- = 3 H2 = s1s2 (no anti-bonding electron)


B+2 = 0.5 B2 = 1 Li2 = s1s2 s*1s2 s2s2(two anti-bonding electrons)
Li+2 = 0.5 Li2 = 1
1
B2 = s1s2 s*1s2 s2s2 s*2s2 p2p y = p2pz
1
{ }
(4 anti-bonding electrons)
O2 = 2.0 O 2- = 1.5
Though the bond order of all the species are same (B.O = 1)
1 but stability is different. This is due to difference in the
Bond length µ
Bond order presence of no. of anti-bonding electrons.
\ O -2 > O 2 Higher the no. of anti-bonding electron, lower is the stability,
124. (a) Molecular orbital electronic configuration of these hence the correct order is H2 > Li2 > B2
species are : 129. (c) H-bonding is maximum in the solid state and
O2- (17e - ) = s1s 2 s *1s 2 s 2s 2 s * 2s 2 s 2 p 2z minimum in gaseous state due to different molecular
distances.
O2 (16e - ) = s1s 2 s *1s 2 s 2 s 2 s * 2 s 2 s 2 p z2
130. (c) H2O shows intermolecular hydrogen bonding while
p2 px2 = p 2 p2y p * 2 p1x = p * 2 p1y
o-nitrophenol shows intramolecular H-bonding.
O 22 - (18e - ) = s1s 2 s *1s 2 s 2s 2 s * 2s 2 s2 p z2 d– d+ d+
O H H
p2 px2 = p 2 p 2y p * 2 p 2x = p *2 p 2y
d+ d+
Hence, number of antibonding electrons are 7, 6 and 8 H H O
d–
respectively. Intermolecular H–bonding
125. (a) Molecular orbital configuration of O
N 2–
2
2 2
= s1s s *1s s2 s s * 2 s - 2 2 +
N
O–
ìï p 2 px2 ìï p * 2 p1x d+ d+
í s2 p2z í O H
2 1
îï p 2 p y îïp * 2 p y Interamolecular H–bonding
10 – 6 131. (a) Due to strong hydrogen bonding in HF and than NH3.
Bond order = =2
2 132. (b) Methanol and ethanol are soluble because of the
ìp2 px2 ìï p * 2 p1x hydrogen bonding.
N 2– = s1s 2 s *1s 2 s2s 2s * 2s 2 ï í s2 pz2 í 133. (d) F—H----F bond is shortest, because with the increase
2 0
10 - 5 ïîp2 p y ïî p *2 p y of electronegativity and decrease in size of the atom to
Bond order = = 2.5 which hydrogen is linked, the strength of the hydrogen
2
ìï p2 px2 bond increases.
N 2 = s1s 2 s *1s 2 s 2 s 2 s * 2 s 2 í , s 2 pz2 134. (a) Hydrogen bonding is possible only in compounds
2
îï p 2 p y having hydrogen attached with F, O or N.
10 – 4
Bond order = =3 C2 H 5 - OH CH 3 - O - CH 3
2
(H-bonding possible) (H-bonding not possible)
\ The correct order is = N 2– –
2 < N2 < N2
O O
126. (d) H +2 : (s 1s1) || ||
1 1 CH3 - C - CH 3 CH 3 - C - H
Bond order = (1 - 0) = (H-bonding not possible) (H-bonding not possible)
2 2
H -2 2 *
: (s1s ) (s 1s ) 1 O
1 1 +
Bond order = (2 - 1) = N
2 2 135. (a) O–
The bond order of H +2 and H 2- are same but H +2 is more d+
O H
d+

stable than H 2- . In H 2- the antibonding orbital is filled with O-nitrophenol


1 electron so this causes instability. (Intramolecular hydrogen bonding)
127. (b) The removal of an electron from a diatomic molecule 136. (c) ortho-Nitrophenol has intramolecular H-bonding
may increase the bond order as in the conversion .
. . OH
O2 (2) ¾® O +2 ( 2 .5)
¾ or decrease the bond order as in the
. O and para-nitrophenol has
conversion, N 2 (3.0) ¾¾ ® N 2+ (2.5). As a result, the
||

. N
bond energy may increase or decrease. Thus, statement O
(b) is incorrect. intermolecular H-bonding.
128. (b) The molecular orbital configuration of the given 137. (d) Hydrogen bonding increases the boiling point of
molecules is compound.
EBD_8350
504 CHEMISTRY

138. (a) Hydrogen bond is formed when hydrogen is attached Decrease in s-character = 33.3 – 25 = 8.3
with the atom which is highly electronegative and having Actual decrease in bond angle
small radius. = 109.5° – 105° = 4.5°
139. (c) Ice has many hydrogen bonds which give rise to cage Expected decrease in s-character
like structure of water molecules. This structure possess 8.3
larger volume and thus makes the density of ice low. = ´ 4.5 = 3.56%
10.5
140. (c) The b.p. of p-nitrophenol is higher than that of Thus, the s-character should decrease by about 3.56% i.e.,
o-nitrophenol because in p-nitrophenol there is s-character = 25 – 3.56 = 21.44%
intermolecular H-bonding but in o-nitrophenol it is 4. (21) Molecular orbital electronic configuration of these
intramolecular H-bonding. species are :
141. (c) The strength of the interactions follows the order van
der Waal’s < dipole-dipole < hydrogen – bonding < covalent. O -2 (17e - ) : s1s 2 , s *1s 2 , s 2s 2 , s *2s2 , s 2p 2z ,
142. (b) Both assertion and reason are true but reason is not p2p2x = p 2p 2y , p * 2px2 = p * 2p1y
the correct explanation of assertion.
O2 (16e - ) : s1s 2 , s *1s 2 , s 2s 2 , s *2s 2 , s2p 2z ,
Water is excellent solvent because it has high value of
dielectric constant. Due to high value of dielectric constant, p2p2x = p 2p 2y , p * 2p1x = p * 2p1y
the electrostatic force of attraction between the ions
decrease and these ions get separated and ultimately get O22 - (18e - ) : s1s 2 , s *1s 2 , s2s 2 , s *2s2 , s2p 2z ,
solvated by the solvent molecules. p 2p2x : p2p2y , p * 2p x2 = p * 2p 2y
143. (b) It forms intermolecular hydrogen bonding with itself. Hence number of antibonding electrons are 7, 6 and 8
144. (c) Each H2O molecule can form two H-bonds through respectively.
H-atoms and two H-bonds through two lp of e– s on
O-atom.
5. (2) I3- Þ sp3 d
145. (c) B, C and D form intermolecular hydrogen bonding Group A l.p.
while A form intramolecular hydrogen bonding due to Px4 1
proximity of oxygen and hydrogen. Qy3 2
Rz2 3
Exercise - 2 x = Total l.p. = 6
Group B l.p.
1. (25) m = e × d Sx 4 0
Ty3 1
m 1.2 D 1.2 ´ 10 -18 esu cm Uz2 2
\ e = d = 1.0 A =
1.0 ´ 10 -8 cm y = Total l.p. = 3
x/y = 6/3 = 2
= 1.2 × 10–10 esu 6. (9) n = 1, then X = H; A = N
Percentage of electronic charge
(i) NH-2 (ii) NH3 (iii) NH+4
1.2 ´ 10 -10 esu lone pair x = 2, y = 1, z = 0
= ´ 100 = 25%
4.8 ´ 10-10 esu
x3 + y 2 + z = (2)3 + (1)2 + (0) = 9
2. (76.94) Dipole moment = electric charge × bond length
7. (3) –
= 4.8 ´ 10 -10 ´ 2.82 ´ 10 -8 = 13.53D O
Now % ionic character P
Actual dipole moment of the bond H–O O
= ´ 100 H
Dipole moment of pure ionic compound
x = 7, y = 1, z = 5
10.41 7+1–5=3
then % ionic character in AB = ´ 100 = 76.94%
13.53 8. (8) (i) IF5 (sp3d) (ii) ClI-4 (sp3d2)
3. (21.44) s-character µ bond angle
(iii) XeO2F2(sp3d) (iv) NH 2- (sp3)
For 25% s character (as in sp3 hybrid orbital), bond angle is
109.5°, for 33.3% s character (as in sp2 hybrid orbital), bond (v) BCl3(sp2) (vi) BeCl2 (sp)
+ 3
(vii) AsCl 4 (sp ) (viii) B(OH)3 (sp2)
angle is 120° and for 50% s character (as in sp hybrid orbital),
bond angle is 180°.
(ix) NO2- (sp2) (x) ClO+2 (sp2)
Similarly, when the bond angle decreases below 109.5°,
the s-character will decrease accordingly. x = ( sp3 )2 + sp3d (1) + sp3 d 2 (2) = 5
Decreasing in angle = 120° – 109.5° = 10.5° y = 4, z = 1; \ x + y – z = 5 + 4 – 1 = 8
CHEMICAL BONDING AND MOLECULAR STRUCTURE 505

H H
9. (2) XeO 4 , XeF7+
F F H H
O F F
Xe I
Sn H H
O F F
F Cl Cl
H H
sp3d sp2 sp3d 2
Now, there are 5 p-bonds and 19 s-bonds present in the
Polar Polar Polar
above molecule.
O 5. (b) HCl, HI and H2S do not from H-bonds. Only H2O
+ forms hydrogen bonds and each H2O molecule is linked
Xe
I with four H2O molecules.
O O
I I O H
H
O
O
sp 3 sp 3
H
Polar Non-polar H H H
F + O
F
F F F H H H H
Sn F Xe Se O O
F F F
O O F F 6. (d) The given electronic configuration shows that the
sp 3 sp3d 3 sp3d
Polar
element belongs to d-block of the periodic table and known
Non-polar Polar
to be a transition element. In transition elements, electrons
10. (2) 3s : 0 Nodal plane
of ns and (n – 1)d subshell take part in bond formation.
4d z 2 : 0 Nodal plane 7. (b) For sp2 hybridisation, generally the geometry is
4s : 0 Nodal plane usually taken to be triangular planar.
2px : 1 Nodal plane Y
3dx2 - y 2 : 2 Nodal plane
12
4pz : 1 Nodal plane 0°
4dxy : 2 Nodal plane X
3 py : 1 Nodal plane
Y Y
4d xy 4 2 , 3d , (Two)
z x2 - y 2 Thus, bond angle should be 120°.
8. (a) The given electronic configuration of A shows that it
Exercise - 3 is a noble gas because the octet is complete and hence it
1. (c) CO2 being symmetrical has zero dipole moment. will be the stable form.
Amongst HI, SO2 and H2O; H2O exhibit the highest dipole 9. (b) The electronic configuration of C represents chlorine.
moment as the central atom in H2O contains 2 lone pairs. Its stable form is Cl2 i.e., C2.
.. 10. (d) The electronic configurations show that B represents
..

O
..

S phosphorus and C represents chlorine. The stable


H—I H H O O compound formed will be PCl3 i.e., BC3.
m = 0.38D m = 1.84 D m = 1.61 D
11. (b) The bond between B and C will be covalent as both B
2. (c) 3-
In PO4 ion, formal charge on each O-atom of P – O and C are non-metal atoms.
Total charge 3 12. (a) Out of the given electronic configuration, 2s22p5
bond = = - = - 0.75
Number of O atom 4 represents fluorine which is the most electronegative
element, while 3s23p5 4s24p5 and 5s25p5 represents
3. (a) BH -4 Þ 4 bond pairs and 0 lone pair ® sp3 hybridised. chlorine, bromine and iodine respectively.
\ Tetrahedral geometry. 13. (b) The electronic configuration of (b) and (d) have
4. (c) The correct structure of the given compound will be exactly half-filled p-orbitals but (b) being smaller in size
as follows : than (d) will have the highest ionisation enthalpy.
14. (b) BF4- and NH +4 both the species are tetrahedral and
sp3 hybridised.
EBD_8350
506 CHEMISTRY

15. (b) Applying the formula to find the hybridisation of 4-4


central atom (nitrogen) : = =0
2
1
[No. of valence e – of central atom + Bond order of Be2 is zero thus, it does not exist.
2
no. of monovalent atoms attached to it + (b) He2 = s1s2 s*1s2
(–ve) charge if any – (+ve) charge if any] 2-2
B.O = =0
1 2
For, NO2+ = [5 + 0 + 0 – 1] = 2 Þ sp-hybridised
2 Now, He+2 = s1s 2 s *1s1
1
NO3– = [5 + 0 + 1 – 0] = 3 Þ sp2-hybridised
2 2 -1
B.O = = 0.5
1 2
NH4+ = [5 + 4 + 0 – 1] = 4 Þ sp3-hybridised
2 As the bond order is not zero, He2+ molecule is expected to exist.
16. (b) Strength of H-bonding is in the order : (c) N2 (7 + 7 = 14) = s1s2s*1s2s2s2s*2s2
HF > H2O > NH3
p2 px2 » p 2 p 2y s 2 p2z
Also, each H2O molecule is linked to four other H2O
molecules through H-bonds while each HF molecule is 10 - 4
linked only to two other HF molecules. B.O = =3
2
Hence, decreasing order of b.p. will be : H2O > HF > NH3 Thus, dinitrogen (N2 ) molecule contains triple bond, while no
17. (d) In NO3– ion, molecule of second period have more than double bond. Thus,
number of bond pairs (or shared pairs) = 4 bond strength of N2 is maximum amongst the homonuclear
number of lone pairs = 0 diatomic molecules belonging to the second period.
+ (d) The correct order of energies of molecular orbitals in
O N O– N2 molecule is
s1s < s*1s < s2s < s*2s < (p2px ; p2py ) < s2pz
O–
< (p*2px » p*2py) < s*2pz
18. (c) The electronic configuration of the given molecules
are : 22. (b) E.C of O2 :
N +2 = s1s 2s *1s 2s 2s 2 s * 2s 2 p 2 p2x = pp 2y s 2 p1z ; 2 2 2
s1s2 s*1s2 s2s2 s*2s2 s 2 pz p 2 px » p 2 p y
1 unpaired e–
p *2 p1x » p *2 p1y
O2 = s1s 2 s *1s 2 s2s 2s * 2s 2s 2 pz2 p 2 px2 » p 2 p2y
p * 2 p1x » p * 2 p1y ; 2 unpaired e–s 1 1
B.O = ( Nb - Na ) = (10 - 6) = 2
O22 - 2 2 2
= s1s s *1s s 2s s * 2s 2
s2 pz2 p 2 p 2x » p 2 p 2y 2 2
p*2 px2 » p*2 p 2y ; no unpaired e–s E.C of O+2 :
B2 = s1s 2s *1s 2 s2 s 2s * 2s 2 p 2 p1x
» p 2 p1y ;
– s1s 2 s *1s 2 s2s 2s * 2s 2 s 2 pz2 p 2 px2 » p 2 p 2y
2 unpaired e s
Trick : We know that O2 molecule has 2 unpaired electrons. p *2 p1x » p *2 p0y
Thus, O2–
2 will not have any unpaired electrons.
19. (c) XeF4 Þ square planar, all bonds are equal. 1
B.O = (10 - 5) = 2.5
BF4- Þ tetrahedral, all bonds are equal. 2
H E.C of O-2 :
H
C2H4 Þ C=C , C = C bond is not equal to
H
H
C – H bond. s1s 2 s *1s 2 s2s 2s * 2s 2 s 2 pz2 p 2 px2 » p 2 p 2y
SiF4 Þ tetrahedral, all bonds are equal.
p *2 px2 » p * 2 p1y
20. (a) The correct increasing order of energies of molecular
orbitals of N2 is : 1
Bond order = (10 - 7) = 1.5
s1s < s*1s < s2s < s*2s < (p2px » p2py ) < s2pz 2
< (p*2px ; p*2py ) < s*2pz Hence, the correct order of bond order will be :
21. (d) (a) Be2 = s1s s*1s s1s s* 2s2
2 2 2 O -2 < O 2 < O2+ .
1 23. (b) Be2+ = (4 – 2) = 2
(B.O) = [Number of bonding electrons (Nb ) –
2 is isoelectronic with Li+ (3 – 1 = 2)
Number of anti-bonding electrons (Na) ] Since both have same number of electrons in their
outermost shell.
CHEMICAL BONDING AND MOLECULAR STRUCTURE 507

24. (c) Dipole moment of NH3 > NF3 29. (c) ClO3– and SO2– 3 both have same number of electrons
(42) and central atom in each being sp3 hybridised. Both
are having one lone pair on central atom hence, they are
pyramidal.
N N
H H 30. (b) NO2+ has sp hybridisation so, it is linear with bond
F F
angle = 180°.
H F
O
m = 1.4 D m = 2.3 D
(F is more electronegative than N) 31. (a) Xe
1 O O O
25. (b) Hybridization of NO3– = (5 + 0 + 1 - 0)
2 Number of s bonds = 4
6 2
= = 3 = sp hence geometry is trigonal planar.. Number of p bonds = 4
2
32. (a) According to molecular orbital theory as bond order
NO2– (nitrite ion) also has sp2 hybridisation and gives a
decreases, stability of the molecule decreases
trigonal planar geometry but because there are only two
outer atoms, the molecular shape is bent with Ð120º bond 1
Bond order = ( N – Na)
angles. 2 b
26. (a) Nitric oxide is paramagnetic in the gaseous state 1
because of the presence of one unpaired electron in its Bond order for O+2 = (10- 5) = 2.5
2
outermost shell.
1
The electronic configuration of NO is Bond order for O 2 = (10 - 6) = 2
2
s1s2 s*s2 s2s2 s*2s2 s2pz2 p2px2 = p2py2 p*2px1
1
Bond order for O -2 = (10 - 7) = 1.5
O O Net dipole 2
27. (d) moment 1
2-
H H Bond order for O2 = (10 - 8) = 1.0
2
Hence, the correct order is
S S Net dipole
moment
O+2 > O 2 > O2– > O 2–
2
H H
In both the molecules, the bond moments are not canceling 33. (d) XeF4, XeO4
O
with each other and hence, the molecules has a resultant
dipole and hence, the molecule is polar. F F Xe
Xe
28. (c) O +2 ion - Total number of electrons (16 – 1) = 15. O O
Electronic configuration F F O
σ1s 2 < σ*1s 2 < σ2s 2 < σ* 2s 2 < σ2p 2z (Square planar) (Tetrahedral)

2 * 1 34. (b) H
< π2 px2 = π2 p y < π 2 px O 109°28’
C N
Nb - N a 10 - 5 5 1 10 H
Bond order = = = =2 H H H H 7
2 2 2 2 104.5° H H H °
O2– (Super oxide ion): Total number of electrons H 2O CH4 NH3
Bent Tetrahedral Trigonalpyramidal
(16 + 1) = 17 . Electronic configuration
Note: The geometry of H2O should have been tetrahedral
2 * 2 2 *
σ1s < σ 1s < σ2s < σ 2 s < σ2 p z2
2 if there are all bond pairs. But due to presence of two lone
pairs the shape is distorted tetrahedral. Hence, bond angle
< π2 p 2x = π2 p 2y < π* 2 p 2x = π* 2 p1y reduced to 104.5° from 109.5°.
( N b - N a ) 10 - 7 3 1 35. (a) According to VSEPR theory, order of repulsion in
Bond order = = = =1 between lp – lp, lp – bp and bp – bp is as under
2 2 2 2
+2
O2 ion: Total number of electrons = (16 – 2) = 14 lp – lp > lp – bp > bp – bp
Electronic configuration 1
36. (c) Hybridization (H) = [no. of valence electrons of
s1s2 < s*1s2 < s2s2 < s*2s2 < s2pz2 < p2px2 = p2py2 2
central atom + no. of Monovalent atoms attached to it
( Nb – Na ) 10 – 4 6 + (–ve charge if any) – (+ve charge if any)]
Bond order = = = =3
2 2 2 1
– + 2+
So bond order: O2 < O2 < O2 NO2+ = [5 + 0 + 0 - 1] = 2 i.e. sp hybridisation
2
EBD_8350
508 CHEMISTRY

1 45. (c) KCl is ionic compound while other are covalent


NO–2 = [5 + 0 + 1 - 0] = 3 i.e. sp2 hybridisation compounds.
2
1 46. (a) One double bond and one triple bond means 3p bonds/
NO–3 = [5 + 0 + 1 - 0] = 3 i.e. sp2 hybridisation option (1).
2
The lewis structure of NO 2 shows a bent molecular H H
1 2 3 4 5
geometry with trigonal planar electron pair geometry hence H–C–C C– C C–H
the hybridization will be sp2.
37. (b) CN– and CO have same no. of electrons and have H H
same bond order equal to 3. Pent-2-en-4-yne

38. (b) IBr–2, XeF2 O


47. (a) O
Total number of valence electrons are equal in both the
O
+6
species and both the species exhibit linear shape. O +4 +6
Br Br Br O
39. (c) BCl3 is trigonal planar and hence, the bond angle is 120°. O O
Cl O
120° Br3O8
B
Cl 3x – (2 × 8) = 0
Cl
40. (b) NO ® s1s2, s*1s2, s2s2, s*2s2, s2pz2 , p2p2x » p2p2y, p*2p1x 3x – 16 = 0
(a) one unpaired electron is present in p* molecular orbit Þ 3x = 16
hence, paramagnetic. 16
x=
(b) CO ® s1s 2 , s *1s 2 , s 2 s 2 , s * 2 s 2 , p 2 p x2 » p 2 p y2 , s 2 p 2z 3
no unpaired electron hence, diamagnetic. Fractional oxidation state is the average oxidation state of
(c) O2 ® s1s 2 , s *1s 2 , s 2 s 2 , s * 2 s 2 , s 2 pz2 , p 2 p x2 » p 2 p 2y , the element.
p *2 p1x » p *2 p1y 6 + 4 + 6 16
=
two unpaired electron hence, paramagnetic. 3 3
(d) B2 ® s1s 2 , s *1s 2 , s 2s 2 , s *2s 2 , p 2 p1x » p2 p1y O O–
B2 contains two unpaired electrons hence, paramagnetic
Mn Mn
41. (b) The structure of ClF3 is
F 48. (a) O – O
O– O O O
Manganate ion Permanganate ion
Cl F 49. (c) Only p bond is present in C2 molecule.
s1s2 s*1s2 s2s2 s*2s2p2p2x = p2py2
F 50. (a) Configuration of C2 =
The number of lone pair of electrons on central Cl is 2.
42. (b) Species Total elec. B.O. s1s2 s*1s2 s2s2 s*2s2 p2p2x = p2p2y
NO 15 2.5 Configuration of C–2 =
CN 13 2.5 s1s2 s*1s2 s2s2 s*2s2 p2p2x = p2p2y s2p1z
CN– 14 3 Bond order
CN+ 12 2
43. (c) No.of bounding e - - No.of antibonding e -
I =
2
C2 has s-p mixing and the HOMO is p2px = p2py and LUMO
I is s2pz. So, the extra electron will occupy bonding molecular
orbital and this will lead to an increase in bond order.
I C2– has more bond order than C2.
\ Total number of lone pair of electrons is 9 51. (c) Electronic configuratios of Li+2 and Li2– :
Li+2 : s1s2 s*1s2 s2s1
44. (d) The electronic configuration of is H 2–
2
Þ s1s s*1s
2 2 Li2– : s1s2 s*1s2 s2s2 s*2s1
1 1
Nb - N a 2 - 2 Now, Bond order of Li+2 = (3 - 2) =
Bond order = = =0 2 2
2 2
1 1
Hence, H 2–
2 does not exist due to zero bond order..
Bond order of Li2– = (4 - 3) =
2 2
CHEMICAL BONDING AND MOLECULAR STRUCTURE 509

Here, both Li+2 and Li2– have positive bond order, thus both 4. (a) For NO
are stable. Total no. of electrons = 15
52. (c) BF3, BeF2, CO2 and 1, 4-dichlorobenzene all are B.O = 2.5
symmetrical molecules. Mag behaviour = paramagnetic
F For NO+
µ=0 Total no. of electrons = 14
B B.O = 3
F F Magnetic behaviour = diamagnetic
Be µ=0 Bond order has increased, thus the bond dissociation
F F
energy also increased.
O C O µ=0 5. (c)
6. (c)
Cl Cl µ = 0 (a) The S-atom in the excited state (3s 2 3 p3 3d 1 ) has sp3
53. (d) For He 2 molecule, Electronic configuration is hybridisation. The three hybrid orbitals form three (S-O)
s-bonds and the 3d pure orbital gives (S-O) p-bond.
s1s 2 , s *1s 2
(b) In PO 34- , P-atom in its excited state (3s1 3 p 3 3d 1 ) is
1 1 sp 3 hybridised. Four hybrid orbitals forms (P-O)
Bond order = ( N b - N a ) = (2 - 2) = 0
2 2 s-bonds whereas 3d1 gives (P-O) p -bond.
Since, bond order of He2 is zero, so it does not exist. (c) In NO 3- , N-atom is sp2 hybridised.
54. (d) m CCl4 = m CH 4 = 0 due to symmetrical structure but Completely filled sp2 hybrid orbital on N-atom donates an
electron pair to the vacant 2p orbital of O atom to form
m CHCl3 ¹ 0 . So dipole moment order is :
dative bond. Other two hybrid orbitals form
CHCl3 > CH4 = CCl4. (N-O) s -bonds. The pure 2 p1z orbital forms (N-O) p -bond.
55. (d) Among given intermolecular forces, ionic interactions (d) In XeOF4, Xe-atom in its excited state (5s 2 5 p3 5d 3 )
are stronger as compared to van der Waal interaction is sp3 d 2 hybridised. The singly filled five hybrid orbitals
Thus, correct order is ion-ion > ion-dipole > dipole-dipole form s -bonds with 4F atoms and O atom. The pure 5d
orbital forms p -bond with 2p orbital of O-atom.
Exercise - 4 7. (d) It is most appropriate because in it the formal charge
1. (b) H3C × C º C × CH3 symmetrical and linear. is zero on all the atoms.
Hence, D.M. (µ) = 0. 8. (c) In nitrogen molecule, both the nitrogen atoms have
2. (c) According to Fajan’s rule, greater the polarising same electronegativity. So, it has zero polarity and hence,
power of cation greater would be the covalent character. less tendency to break away and form ions.
Since, Be2+ has maximum polarising power among given
cations. Therefore, BeX2 would be most covalent alkaline
earth metal halides among the given halides. 9. (b) (i) O–

O
F Cl
O Cl PO34- HPO32 - H 2 PO2-
3. (d) Xe P Cl 5 4 3
B.O. = B.O. = B.O. =
O Cl 4 3 2
F Cl –
Bond order : PO43– < HPO32– < H2PO2
3-atoms 4-atoms
Thus, P – O bond length order:
+ –
H PO43– > HPO32– > H2PO2
F F
F sp3 hybridisation H
As Xe tetrahedral shape
q1 q2
H H F F C C
H (ii) H H F F
3-atoms 5-atoms H F
Maximum atoms that ® 3 4 3 5 (iii) H – C º C – H H H
sp sp
can lie in a plane C = C2
sp2 sp
H H
EBD_8350
510 CHEMISTRY

Higher s-character means stronger bond. Thus, C–H bond Number of atoms in (X-Y) plane = 5
is stronger in C2H2. (c) Shape: Pentagonal bipyramidal
In CH3F, due to less difference in electronegativity between
C and H, the bond pairs are in between C and H atoms. F
Whereas in CHF 3 , due to large differen ce in F F
electronegativity between C and F, the bond pairs are IF7 I F
towards F atoms. F F
Therefore, bp–bp repulsions in CH3F is more than that in F
CHF3. We can conclude that HCH angle is more than FCF
angle. Number of atoms in (X-Y) plane = 6
Both the molecules are in tetrahedral shape. Hence, angle,
11. (b) dx 2 - y2 + py ( side wise overlap ) ® Non - bonding.
FCH in CH3F (q1) must be less than angle, HCF in CHF3
(q2), i.e. q2 > q1. 12. (c) Hybridisation of N in NH3 is sp3 and in F3B ¬ NH3
10. (b) also the hybridisation is sp3.
(a) Arrangement: Pentagonal bipyramid shape: pentago- 13. (b) In CrO2Cl2, the central atom Cr forms 4 sigma bonds and
nal planar 2 pi bonds. Thus, the structure must be distorted tetrahedral.

F F Cl
Xe F
XeF5-
F F Cr
O Cl
Number of atoms in ‘X-Y’ plane = 6 O
(b) Shape: Square bipyramidal
14. (b) B.O. of CN–, CN and CN+ are 3, 2.5 and 2 respectively.
F
15. (a) Other than NH4+, all are very reactive cations.
F F
S
SF6
F F
F
STATES OF MATTER 511

5 States of Matter
Exercise - 1 P1V1 = P2V2
or 750 × 600 = P2 × 500
1. (c) Attractive forces between an ion and a dipole are
or P2 = 750 ´ 600 = 900 mm of Hg
known as ion - dipole forces and these are not van der 500
Waals forces. Therefore, increase of pressure = (900 – 750) = 150 mm of Hg.
2. (d) Fritz London explained the concept of dispersion 17. (a)
force. 18. (b) The lowest hypothetical or imaginary temperature at
3. (a) Dipole-dipole interaction is stronger than the London which gases are supposed to occupy zero volume is called
forces but is weaker than ion-ion interaction because only absolute zero.
partial charges are involved e.g., HCl molecules. The 19. (b) Given initial volume (V1) = 500 mL ; Initial temperature
attractive force decreases with the increase of distance (T1) = 27ºC = 300 K and final temperature (T2) = –5ºC
between the dipoles. = 268 K.
4. (c) Nobel gases has only london forces due to neutral From Charle’s law :
atoms.
5. (c) Gases have much lower density than the solids and V1 V2 500 V2
= or =
liquids. T1 T2 300 268
6. (b) This type of attractive force operates between the Where V2 = New volume of gas
polar molecules having permanent dipole and the molecules
lacking permanent dipole. 500
V2 = ´ 268 = 446.66 mL
HCl is polar (m ¹ 0) and He is non polar (m = 0), thus gives 300
dipole-induced dipole interaction. 20. (b)
7. (a) Solid, liquid and gas as the three states. 21. (d) The mathematical relationship between pressure and
8. (d) All of the given statements are correct for hydrogen temperature was given by Gay Lussac's law.
bond. 22. (a) V1 = 2 L, T2 = (26.1 + 273) K = 299.1 K, V2 = ?
9. (a) Gases expand and occupy all the space available to T1 = (23.4 + 273) K = 296.4 K
them because there is no force of attraction between the
V1 V2 VT
particles of a gas at ordinary temperature and pressure. From Charle's law, = Þ V2 = 1 2
10. (c) In case of HCl molecules their is dipole-dipole T1 T2 T1
interaction which is stronger than London forces as in
2L ´ 299.1K
case of SF6 (due to symmetry SF6 does not have Þ V2 = = 2L ´ 1.009 = 2.018 L
permanent dipole). Now between HCl and NaCl the ion- 296.4K
ion interaction present in NaCl is far more stronger than 23. (d) According to Boyle’s law at constant temperature,
dipole-dipole interaction of HCl. 1
11. (d) Vµ or PV = constant
P
12. (c) Gaseous state of substance has the maximum thermal
24. (c) At any given pressure, graph of volume vs
energy.
temperature (in °C) is a straight line and on extending to
13. (c) Gases do not have definite shape and volume. Their
zero volume each line intercepts the temperature axis at –
volume is equal to the volume of the container.
273.15° C.
14. (c) Robert Boyle made first reliable measurement on
properties of gases. æ MP ö
25. (a) d µ P, Boyle’s law, ç d = ÷ . At sea level, pressure
è RT ø
15. (c)
®

P P is more, hence density of air is more.


V1 V 2
26. (c) = at const. pressure
T1 T2
V® 1/V ®
Curve I Curve III 22.4 V2
Þ = , V2 = 30.6 litre
Both these graphs represent Boyle's law. 273 373
16. (a) Given initial volume (V1) = 600 c.c.; Initial pressure 27. (a) Applying Boyle's law P1V1 = P2V2 for both gases
(P1) = 750 mm of Hg and final volume (V2) = 500 c.c.
500 200
according to Boyle’s law, ´ 400 = P ´ 3 Þ P =
1000 3
EBD_8350
512 CHEMISTRY

40. (a) By Dalton's Law.


666.6 400
600 ´ = P ¢ ´ 3 Þ P¢ = nRT 6 ´ 8.314 ´ 402
1000 3 41. (d) P = = ; 41648 Pa
V 16.02 0.03
200 400 600 42. (a) Given nCO = n N 2
Þ PT = P + P = + = = 200 torr
3 3 3 PCO + PN2 = 1 atm
28. (b) According to Boyle's law, PV = constant
Partial pressure of a gas = mole fraction of gas × total
\ log P + logV = constant pressure
logP = – logV + constant nN 2 nN 2 1
Hence, the plot of log P vs logV is straight line with \ PN2 = ´1 = ´ 1 = = 0.5 atm.
nCO + nN 2n N 2 2
negative slope. 2
43. (a) Given
P1 = 1.5 bar, T1 = 273 + 15 = 288 K, V1 = V
®

P2 = 1.0 bar ,T1 = 273 + 25 = 298K, V2 = ?


logP

PV
1 1 PV
= 2 2
T1 T2
logV ®
29. (d) 44 g at STP occupies volume 22.4 litre, which is 1.5 ´ V 1 ´ V2
=
molecular mass of CO2. Molecular mass occupies 22.4 litre 288 298
at STP. V2 = 1.55 V i.e., volume of bubble will be almost 1.6 time to
30. (b) An ideal gas obeys the gas laws under all experimental initial volume of bubble.
conditions. 44. (a) Partial pressure = total pressure × mole fraction
31. (c) In the equation PV = nRT, n moles of the gas have 64
Moles of oxygen = =2
volume V. 32
32. (b) R = 0.082 litre atm K–1 mol–1 . 60
Moles of neon = =3
m 20
33. (d) PV = nRT = RT
M 2 2
PM Mole fraction of oxygen = =
m 2+3 5
or PM = RT = dRT Þ d =
V RT 2
PO2 = ´ 10 = 4 bar
PV
1 1 T1 760 ´ 273 ´ (273 + 273) 5
34. (c) V2 = = ´ 273 3 3
T1 P2 273 ´ 600 Mole fraction of neon = =
2+3 5
= 691.6 mL
3
35. (b) Because H2 and Cl2 gases may react with each other PNe = ´ 10 = 6 bar
to produce HCl gas hence Dalton’s law is not applicable. 5
45. (d) Value of gas constant depends only upon units of
16 1
36. (d) n of O 2 = = measurement.
32 2 46. (a) Given conditions
3 V1 = 16.4 L, V2 = 5 L
n of H 2 =
2 P1 = 1.5 atm, P2 = 4.1 atm
3 1 T1 = 273 + 27 = 300 K,
Total no. of moles = + = 2
2 2 T2 = 273 + 227 = 500 K
nRT 2 ´ .082 ´ 273
V= = = 44.8L = 44800 mL PV
1 1 nT
P 1 Applying gas equation, = 11
PM P2V2 n2T2
37. (b) d =
RT n1 PV T
It means density of gas is directly proportional to pressure = 1 1 2
n2 P2V2T1
and inversely proportional to temperature.
Density of neon will be maximum at highest pressure and 1.5 ´16.4 ´ 500 2
lowest temperature. \ =
4.1 ´ 5 ´ 300 1
38. (a) At STP, molar volume of an ideal gas or a combination
of ideal gases is 22.71098 L mol–1. 47. (d) From the ideal gas equation :
PV = nRT (Volume of water is negligible in comparison to
4 2 volume of flask)
39. (a) Total moles = + = 1.125; PV = nRT
32 2
PV 3170×10-3
Þ P = 1.125 ´ 0.0821´ 273 or n = = = 1.27 × 10–3
RT 8.314×300
P = 25.215 atm
STATES OF MATTER 513

48. (d) All the gases occupy the available volume and will Temperature = 300 K
form homogeneous mixture. R = 0.0821 L atm K–1 mol–1
49. (a) From ideal gas equation, PV = nRT Mass of ideal gas = 50.5 – 50 = 0.5 g
æ mö RT According to ideal gas equation,
PV = çè ÷ø RT or M = m w
M PV PV = nRT = RT
Let the molecular wt. of A and B be MA and MB respectively. M
0.5
RT 3 ´ RT 1 × 200 × 10–3 = ´ 0.0821 ´ 300
Then MA = 2 ; MB = M
1´V 0.5 ´ V 0.5 ´ 0.0821´ 300
M= = 61.575
MA 2 RT 0.5V 2 ´ 0.5 1 200 ´10–3
\ = ´ = =
MB V 3 RT 3 3 PAV A 8 ´12 96
55. (c) Moles of A, (nA) = = =
Therefore, the ratio MA : MB = 1 : 3 RT RT RT

70.6 g PBVB 8 ´ 5 40
50. (d) Number of moles of O2 = = 2.21 mole Moles of B, (nB) = = =
32 g mol -1 RT RT RT
Total pressure × total volume = (nA + nB) × RT
Number of moles of Ne = 167.5 g = 8.375 mole P ´ (12 + 8) =
1
(96 + 40) RT
20 g mol -1 RT
P = 6.8
2.21
Mole fraction of O2 = = 0.21 mole Partial pressure of A = P × mole fraction of A
2.21 + 8.375
æ 96 96 + 40 ö
Mole fraction of Ne = 1 – 0.21 = 0.79 = 6.8 ç ÷ = 4.8 atm
Partial pressure of a gas = Mole fraction × total pressure è RT RT ø
Partial pressure of O2 = 0.21 × 25 = 5.25 bar Partial pressure of B = 6.8 – 4.8 = 2 atm.
56. (b) According to kinetic theory, the gas molecules are in
Partial pressure of Ne = 0.79 × 25 = 19.75 bar
a state of constant rapid motion in all possible directions
51. (a) Extent of diffusion H2 > CH4 > SO2 because rate of colloiding in a random manner with one another and with
1 the walls of the container. Between two successive
diffusion µ collisions, molecules travel in a straight line path but show
molar mass
Order of partial pressure after diffusion is haphazard motion due to collisions.
57. (c) Given statement explain the great compressibility of
pSO2 > pCH 4 > pH 2 gases.
52. (c) Number of moles, temperature and volume are same. 58. (b) Particles of a gas are always in constant and random
53. (a) Since atmospheric pressure remain constant motion. If the particles were at rest and occupy fixed
positions, then a gas would have a fixed shape which is
V1 V2 V1 2800 m3 not observed.
= Þ =
T1 T2 298 K 372 K 59. (d) Kinetic theory of gases proves all the given gas laws.
V1 = 2243 m3 60. (d) Molecules move very fast in all directions in a straight
line by colliding with each other but with different velocity.
2800 m3 volume of inflated balloon.
61. (b) At low temperature and high pressure.
Mass of air in inflated ballon 62. (a) In van der Waal’s equation, ‘b’ is for volume
= 2800 m3 × 0.94 kg m–3 = 2632 kg correction.
Keeping the volume same = 2800 m3
The mass of air, which occupies it with density æ a ö æ a ö
63. (c) ç P + 2 ÷ (V – b) = RT; Here ç P + 2 ÷ represents
(1.2 kg/m3) is 2800 × 1.2 = 3360 kg è V ø è V ø
Amount of air which had been escaped the intermolecular forces.
= 3360 – 2632 = 728 kg 64. (d) Both graphs coincide at low pressure and high
54. (a) Given weight of empty glass vessel = 50 g volume.
Weight of vessel filled with liquid = 144 g n2 a PV 2
65. (a) Unit of P º ;aº = atm dm6 mol -2
\ Weight of liquid = 144 – 50 = 94 g. V2 n2
Volume of liquid = Mass/density = 94/0.47 66. (c) Easily liquefiable gases have greater intermolecular
forces which is represented by high value of 'a'. The greater
= 200 mL = 200 × 10–3 L.
the value of 'a' more will be liquefiability.
Given, pressure of ideal gas = 760 mm Hg = 1 atm So, the order is Q < S < P < R.
EBD_8350
514 CHEMISTRY

84. (c) The correct order of viscosity of the given liquids is


æ a ö a
67. (c) çè P + 2 ÷ø (V – b ) = RT at high pressure 2 can be dimethyl ether < methyl alcohol < water < glycerol, due to
V V increasing H-bonding.
neglected 85. (d) Liquids tend to have minimum number of molecules
PV – Pb = RT and PV = RT + Pb at their surface due to surface tension. Also, spherical
PV Pb shape has minimum surface area.
= 1+ 86. (b) As the intermolecular forces of attraction decrease
RT RT
with the rise in temperature, the surface tension of a liquid,
Pb thus, decreases with increase in temperature.
Z = 1+ ; Z > 1 at high pressure
RT 87. (b) The surface tension of water is more than that of oil.
68. (b) When a drop of oil is placed on the surface of water, the
(i) At very large molar volume greater value of surface tension of water pulls oil in all
a directions and it spreads as a very thin layer.
P + 2 » P and Vm - b = Vm
Vm 88. (b) Viscosity of liquids decreases as the temperature rises
because at high temperature, molecules have high kinetic
(iii) According to van der Waal’s equation 'a' and 'b' are
energy and can overcome the intermolecular forces to slip
independent of temperature.
past one another between the layers.
an2 For gases, the molecules are already for apart to flow easily.
69. (a) For statement (ii), Preal = Pideal - Here, the dominating factor is the velocity of molecules
V2
which increases the strike rate per unit area. This increases
For statement (iii), value of 'a' is independent of temperature with the rise in temperature and which hinders the flow of
and pressure. gases. Hence, viscosity of gases increases with rise in
70. (c) Due to small size of these species (H2 and He) temperature.
intermolecular interactions (van der Waal forces) are
89. (c) Mercury has metallic bonding between their atoms
very low, therefore it is difficult to compress these.
whereas water has hydrogen bonding between their
71. (d) In the ideal gas, the intermolecular forces of attraction
molecules. Thus, intermolecular forces are high in
are negligible and hence, it cannot be liquefied.
mercury. Due to this, surface tension is also high in
72. (c) It brings molecules of gases closer.
73. (d) Above Boyle point, real gases show positive mercury which overcomes the gravity to maintain
deviation from ideality and Z values are greater than one. spherical shape of its droplets.
74. (a) For statement (iii), we can move down from G towards 90. (a) Force is required to maintain the flow of layer which
D by lowering the temperature. is inversely proportional to the area of contact of layer.
For statement (iv), we get liquid as soon as we cross point H. Therefore, flow in B is greater than that in A as the area
75. (b) At 1 atm pressure, boiling temperature is called normal of contact is greater in A. Also viscosity of the fluid
boiling point. If pressure is 1 bar, then the boiling point is decreases with increase in temperature, therefore, liquid
called standard boiling point of the liquid. flow increases.
76. (c) At high altitude, atmospheric pressure is low.
Exercise - 2
77. (b) 1 bar pressure is slightly less than 1atm pressure.
78. (d) A substance exists as a liquid above its m. p. and 1. (14.1) According to Graham’s Law of Diffusion:
below its b. p.
79. (a) Since, surface tension depends on the attractive r1 d2
forces between the molecules, and hydrogen bonding a r2 = d1
special type of dipole-dipole interactions in (b), (c) and
(d) which is stronger than London forces of attraction in Vol. of gas diffused (V)
Since rate of diffusion =
hexane. Time taken for diffusion(t)
80. (a) When a needle placed in water carefully, it can float r1 V /t
on water. Small weight of needle does not break the surface \ = 1 1
r2 V 2 /t 2
tension of water, hence it floats.
81. (d) All these phenomena are caused by surface tension. r1 V1 /t1 d2 20/60 16/2 1
or r = V /t = d = V /30 = 32/2 = 2
82. (c) Due to intermolecular H-bonding, the surface tension 2 2 2 1 2
of H2O is more than other liquids. One H2O molecule is Q Mol. wt = 2 × V.D
joined with 4 another H2O molecule through H–bonds.
Hydrogen bonding is in order H2O > C2H5OH > CH3OH. Mol.wt
\ V.D =
83. (b) As intermolecular forces are least in case of nail polish 2
remover. Thus, it has highest rate of evaporation. On calculating, V2 = 14.1
STATES OF MATTER 515

2. (0.92) Let the degree of dissociation be a, then


PCl5 ¾® PCl3 + Cl2 r1 M2
Also we know that =
(1 – a) a a r2 M1
(M) Molecular weight of PCl5 = 31 + 5 × 35.5 = 208.5
Total number of molecules before dissociation = 1 2 2
æ r1 ö æ 0.7 ö
\ M2 = ç ÷ M1 or M2 = ç
è 0.0308 ÷ø
Total number of molecules after dissociation × 2 = 1033
è r2 ø
= 1– a+a+a=1+a
Thus each gram mole changes to (1 + a) gram-mole.
5. (30) P 4V
W h
Thus PV = (1 + a)RT
T
M V p
p+ h
(V = volume after dissociation) 10
1700 4.5 é 1700 ù P1V1 = P2V2
or 1 × = (1 + a)RTT ê1700ml = L
1000 208.5 ë 1000 úû æ hö
or a = 0.921 P × 4V = P ç 1 + ÷ V
è 10 ø
3. (10) No. of balloons that can be filled
h
V of H 2 available 4 = 1+
= 10
V of one balloon
h = 30 m
Calculation of total volume of hydrogen in the cylinder at
6. (25) Let initial mixture contains n 1 and n 2 moles of
N.T.P.
NH3 and N2H4 respectively.
P1V1 P2 V2 Total moles of gases originally present = n1 + n2
= Total moles of gases after decomposition of gases
T1 T2
= 2n 1 + 3n2
P1 = 1 atm P2 = 20 atm
0.5 × V = (n1 + n2)R × 300
V1 = ? V2 = 2.82 l
4.5 × V = (2n1 + 3n2) R × 1200
T1 = 273 K T2 = 273 + 27 = 300 K
2n1 + 3n 2 9
20 ´ 2.82 ´ 273
\ V1 = = 51.324 l = 51324 mL n1 + n2 = 4
300 ´ 1
Actual volume to be transferred into balloons n2 1
= 51324 – 2820 mL = 48504 mL =
n1 3
[Q 2820 mL of H2 will remain in cylinder]
3 n2
4 3 4 22 æ 21ö × 100 = 25%
Volume of one balloon = pr = ´ ´ ç ÷ n1 + n 2
3 3 7 è 2ø
æ 708.2 ö
é diameter ù çè ´ 0.25÷
êëQ r = 2 úû 7. (870.6) n =
760 ø
= 9.36 × 10–3
N2 0.0821 ´ 303
= 4851 mL = 4.851L
No. of balloons that can be filled up
æ 756.2 ö
48504 çè ÷ ´ 0.3
= = 9.999 » 10 nN = 760 ø
4851 2
( 0.0821) ´ 298
4. (1033) Using gas equation; PV = nRT
= 0.0122
Total no. of moles of gases in the mixture (n)
ntotal moles = 0.02156
PV 6´3 pressure in final vessel = p
= = = 0.7308 moles.
RT 0.0821 ´ 300
( n total ) RT 0.02156 ´ 0.0821 ´ 308
=
Thus no. of moles of unknown gas = 0.7308 – 0.7
V 0.5
= 0.0308 moles
Now we know that p = 1.09 atm or 828.4 torr
Ptotal = p( N + Ne ) + V.p. of H 2 O
r1 moles of hydrogen gas 0.7 2
= =
r2 moles of unknown gas 0.0308 = 828.4 + 42.2 = 870.6 torr
EBD_8350
516 CHEMISTRY

PM 4. (c) Let the number of moles of dihydrogen and dioxygen


8. (112) Density (r) = (1 bar = 0.987 atm) be 1 and 4.
RT
4 ´ 0.987 ´ 28 4
rN = Mole fraction of O2 =
2 R ´ 300 5
Let the molar mass of gas be x Partial pressure of O2 = Mole fraction of O2 × total pressure
of mixture
2 ´ 0.987 ´ x
rgas = 4
R ´ 300 = ´ 1 atm = 0.8 atm
Given rgas = rN × 2 5
2 = 0.8 × 105 Nm–2 = 8 × 104 Nm–2
2 × 0.987 × x 4 × 0.987 × 28 5. (a) From Gay-Lussac's law, at constant volume, as the
= ×2
R × 300 R × 300 temperature is increased, pressure increases.
\ x = 112 g/mol dv f × dx
9. (0.49) Given T = 27 °C = 27 + 273 = 300 K 6. (b) As we know that, f = hA Þ h=
dx A × dx
V = 10.0 L
where, f = force
Mass of He = 0.4 g
h = viscosity coefficient
Mass of oxygen = 1.6 g
Mass of nitrogen = 1.4 g dv
= velocity gradient
nHe = 0.4/4 = 0.1 dx
nO = 1.6/32 = 0.05 Substitute SI units of f = N, x = m, A = m2 and v = ms–1
2
nN = 1.4/28 = 0.05 in above equation, we get,
2
n total = n He + nO + nN = 0.1 + 0.05 + 0.05 = 0.2 N´m
2 2
h= = Nm -2s
nRT 0.2 ´ 0.082 ´ 300
= m ´ ms -1
2
P= = 0.49 atm
V 10 Hence, the SI unit of h is Nsm–2
3 7. (a) Among all the four cities, Shimla has the lowest
10. (58.33) PO = × PT; atmospheric pressure. Since, lower the atmospheric
2 10
pressure, lower is the boiling point. Thus, at Shimla liquid
After removing 2 mole of O2,
will boil first.
1 8. (c) Kinetic energy of the molecules of liquid increases
P¢O = × PT;
2 10 with increase of temperature, which can overcome the
Decrease in partial pressure of O2 intermolecular forces. Hence, the liquid starts flowing.
In other words, the viscosity of a liquid decreases with
3PT PT increase in temperature.
-
10 8 ´ 100 9. (b) The surface tension of liquids generally decreases
= 3P
T with increase of temperature and becomes zero at the
10 critical temperature. This is due to the fact that, with
= 58.33 % increase of temperature, the kinetic energy of the molecules
increases and therefore, the intermolecular attraction
decreases.
Exercise - 3 10. (c) According to Boyle’s law at a particular temperature,
1. (c) At high altitude, pressure is low, hence, boiling point PV = constant
is low due to which things take more time to cook. However, Thus, P1V1 = P2V2 = P3V3 = P4V4
in a pressure cooker, pressure is increased and hence, As V1 > V2 > V3 > V4
boiling point increases. Thus, in a pressure cooker food Therefore, P1 < P2 < P3 < P4
cook faster i.e. a less period of time. 11. (c) London dispersion force is a temporary attractive
2. (b) The property of surface tension explains the spherical force that results when the electrons in two adjacent atom
shape of rain droplets. Surface tension tries to decrease occupy position that makes the atoms form temporary
the surface area of the liquid to the minimum. The rain dipoles. The energy of interaction varies as
droplets are spherical because for a given volume, a sphere 1
has minimum surface area. .
3. (c) Dipole-dipole forces act between the molecules ( distance between two interacting particles ) 6
possessing permanent dipole and ends of dipoles possess Larger or more complex are the molecules, greater is the
'partial charges'. Partial charges present on ends of a dipole magnitude of London forces. This is obviously due to the
are always less than the unit electronic charge. fact that the large electron clouds are easily distorted or
STATES OF MATTER 517

polarised. Hence, greater the polarisability of the Hence Z > 1 at all pressures.
interacting particles, greater is the magnitude of the This means , repulsive tendencies will be dominant when
interaction energy. interatomic distances are small.
12. (d) Higher the critical temperature, more easily is the gas This means, interatomic potential is never negative but
get liquified. Hence, order of liquefaction starting with becomes positive at small interatomic distances.
the gas liquefying first will be : O2, N2, H2, He.
13. (a) For ideal gas, PV = constant at all pressures. 19. (a) Given, nH2 = nO2 and tH 2 = tO2
Therefore, only B represents ideal gas. According to Graham's law of diffusion for two different
14. (c) According to Avogadro’s law "At same temperature gases.
and pressure
Volume µ no. of moles" rH 2 v /t MO 2 32
= 1 1 Þ =
w w w rO2 v2 / t2 MH2 2
nH 2 = ; nO = ; nCH =
2 2 32 4 16
1/ 2
Q VH : VO : VCH = nH 2 : nO 2 : nCH 4 = 16 = 4 [t1 = t2]
2 2 4 1/ x
w w w x
= : : = 16 : 1 : 2 =4
2 32 16 2
PV \x=8
15. (b) Compressibility factor ( Z ) = \ Fraction of O2 = 1/8
RT
For one mole of real gas van der Waals equation is 20. (a) Bulbs are closed, hence,
Initial moles and final moles are equal (nT)i = (nT)f
a
(P + )(V - b) = RT PV PV Pf V Pf V
V2 i
+ i = +
RT1 RT1 RT1 RT2
At low pressure, volume is very large and hence, correction
term b can be neglected in comparison to very large volume Pi Pf Pf
of V. 2 = +
T1 T1 T2
i.e. V – b » V
2 PT
i 2
æ a ö Pf =
çP+ 2 ÷ V = RT T1 + T2
è V ø 21. (c) In real gas equation,
a van der Waal constant (a) µ forces of attraction.
PV + = RT 22. (a) van der Waal constant ‘a’, signifies intermolecular
V
forces of attraction.
a Higher is the value of ‘a’, easier will be the liquefaction of
PV = RT - gas.
V
PV a PV
= 1- 23. (c) Compressibility factor, Z =
RT VRT nRT
Given: At 350 K and 15 bar,
a molar volume < volume of ideal gas
Hence, Z = 1 - \ Z<1
VRT
Therefore, attractive forces are dominant and the gas can
16. (a) At high temperature and low pressure, the attraction be compressed easily.
forces between molecules tends to negligible. 24. (d) Ideal gas equation: PV = nRT
17. (b) Hydrogen bond is a type of strong electrostatic After putting the values we get,
dipole-dipole interaction and dependent on the inverse 200 × 10 = (0.5 + x) R 1000
cube of distance between the molecules. (total no. of moles are 0.5 + x)
18. (c) P(V–b) = RT 4-R
Þ PV – Pb = RT x=
2R
PV Pb 7 1
Þ = +1 25. (b) nN 2 = = = 0.25
RT RT 28 4
Pb 8 1
Þ Z = 1+ nAr = = = 0.20
RT 40 5
EBD_8350
518 CHEMISTRY

Now, applying Dalton's law of partial pressure, PV


i iT f
pN2 = (mole fraction)N 2 × PTotal Pf =
V f Ti
0.25 5 Vi (273 + 28.0)
´ 27 = ´ 27 = 15 bar = 3.21 ´ 105 Pa ´ ´
0.45 9 1.03Vi (273 - 5.00)
8a = 3.50 ´105 Pa
26. (a) Critical temperature =
27 Rb 3
7. (c) PO = ´ PT ;
2 10
a
Value of is highest for Kr. Therefore, Kr has greatest After removing 2 mole of O2,
b
1
value of critical temperature. P 'O = ´ PT
28
Exercise - 4 Decrease in partial pressure of O2
1. (c) Mass of 1 L of vapour = volume × density 3PT PT
-
= 1000 × 0.0006 = 0.6 g = 10 8 ´100
= 58.33 %
mass 3PT
0.6
Volume of liquid water =
density
= = 0.6 cm3 10
1
3
2. (c) Draw a line at constant P parallel to volume axis. Take 8. (d) K = nT RT Þ
volume corresponding to each temperature. 2
3
From volume axis, V1 > V2 > V3 (V µ T ) 3 ´ 103 = nT ´ 8 ´ 250 Þ nT = 1 mole
2
Hence, T1 > T2 > T3. Let x be the mass of Ne gas.
m
3. (c) Gas equation is PV = RT ...(i) x 30 - x
M + =1
20 40
P m 2 Þ x = 1/3 or 33.33%
Again V = 1 R. T ...(ii)
2 M 3 9. (c) For ideal gas PV = nRT
Divide (i) by (ii) Volume of 1 mole gas at 1 atm pressure and 273 K
is 22.4 L and at 300 K and 1 atm pressure, volume = 24.63
m 3 L. All other conditions show ideal behaviour.
2= ´
m1 2 10. (d) (c) At critical temperature Tc, the pressure will be
3 1 equal to critical pressure Pc and the volume will be equal
\ m1 = m . Gas escaped is then = m to critical volume Vc.
4 4
4. (a) If n is the total number of moles of gas and n1 moles (a) For a temperture less than Tc, the gas is liquefiable. A
are in the larger sphere and n2 moles in the smaller sphere. cubic equation has three roots, thus V can take 3 real values.
Then n = n1 + n2 and pV = nRT (b) For a temperature more than Tc, the gas is not
liquefiable. For a particular pressure P, gas can have only
pV p ¢V p ¢V one real value of V.
= +
RT1 RT1 2 RT2 (d) Hence, all the given statements are correct.
2 pT2 11. (c) Molar mass ­, ‘a’ increases
p¢ = size of molecule ­,‘b’ increase
2T2 + T1
b (L/mol) a (bar. L2/mol2)
1
5. (b) n and T are same hence, P µ , H2 ® 0.02661 CH4 ® 2.25
V He ® 0.0237 O2 ® 1.36
V1 = 1000 cm3 O2 ® 0.03183 H2 ® 0.244
V2 = p (10)2 × 10 = 1000 p cm3 CO2 ® 0.04267
4 4 3 ´ 8.21
V3 = p (10)3 = p 1000 cm3 12. (c) nAB = = 1;
3 3 0.0821 ´ 300
\ Pressure of the gas is minimum in (III) container, Pres-
sure of the gas is maximum in (I), 1 ´ 8.21
nB2 = = 0.25
The ratio of pressure in II and III container is 4 : 3 0.0821 ´ 400
6. (c) Because the number of moles is constant. 2AB + B2 ® 2AB2
PVi i
Pf V f Initial moles 1 0.25
= ;
Ti Tf After reaction 0.5 0 0.5
STATES OF MATTER 519

AB2 is solid at 250k, thus the final pressure will be exerted = 51.324 L = 51324 cm3
by only AB (g) (0.5 mole). PV = nRT When the balloons are being filled, the pressure in the
= PAB × (8.21 + 8.21) = 0.5 × 0.0821 × 250 cylinder will decrease. We can continue filling from the
cylinder till the pressure within the cylinder is also 1 atm.
Þ PAB = 0.625 atm
At this stage, the volume of 2820 cm3 of the gas will remain
60 within the cylinder.
13. (b) Vc = = 75cm3 mol–1;
0.80 Hence, Volume of the gas which can be transferred to
Vc balloons will be = 51324 cm3 – 2820 cm3 = 48504 cm3
b= = 25cm3 mol–1; = 0.25 L mol–1 Number of balloons that can be filled up
3

8a 48504 cm 3
\ Tc = = 10 balloons
27 Rb 4851 cm3 / balloon
1
4 ´ 105 8´ a 2 RT æ T ö2
= 15. (a) Vmp = \ ç ÷ µ Vmp
821 27 ´ 0.0821 ´ 0.25 M èMø
Þ a = 3.375 From curve,
14. (d) We have, (Vmp ) I < (Vmp ) II < (Vmp ) III
4 3 300 400 300
Volume of balloon = pr (Vmp ) N 2 µ , (Vmp )O2 µ , (Vmp ) H 2 µ
3 28 32 2
3
æ 4 ö æ 22 ö æ 21 cm ö \ (Vmp ) N 2 < (Vmp )O2 < (Vmp ) H 2
= ç ÷ç ÷ç = 4851 cm3
è 3 ø è 7 ø è 2 ÷ø (under given condition)
Total volume of the gas available at STP conditions is
p1V1 T0 (20 atm)(2.82 L) æ 273K ö
V= ´ = ´ç
T1 P0 (300 K) è 1 atm ÷ø
EBD_8350
520 CHEMISTRY

6 Thermodynamics
Solids and liquids do not suffer any significant volume
Exercise - 1
changes upon heating. The difference, however, becomes
1. (b) The laws of thermodynamics deal with energy significant when gases are involved.
changes of macroscopic systems involving a large number 20. (d) We know that
of molecules rather than microscopic systems containing DH = DE + PDV
a few molecules. In the reactions, H 2 + Br 2 ® 2HBr there is no
2. (c) Closed system can exchange energy but not matter change in volume or DV = 0.
with surroundings. Pressure cooker provides closed So, DH = DE for this reaction.
system. 1 1
3. (b) We can describe the state of a gas by quoting its 21. (b) Dn = - ; DH = DE - RT ; Þ DE > D H
2 2
pressure (P), volume (V), temperature (T ), amount (n) etc.
22. (c) DH = DE + DnRT
4. (c) 5. (d)
6. (c) Values of state functions depend only on the state of Dn = 3 – (1 + 5) = 3 – 6 = –3
the system and not on how it is reached. DH - DE = (-3RT )
7. (a) DE =DQ–W 23. (a) Volume depends upon mass. Hence, it is extensive
For adiabatic expansion, DQ = 0 property.
Þ DE = –W 24. (d) As DH = DE + DngRT
The negative sign shows decrease in Internal energy, if np < nr; D ng = np – nr = – ve.
which is equal to the work done by the system on the Hence D H < D E.
surroundings. 25. (a) The properties whose magnitude depends upon the
8. (a) It is fact that absolute values of internal energy of quantity of matter present in the system are called extensive
substances cannot be determined. It is also true that it is properties e.g., internal energy.
not possible to determine exact values of constitutent 26. (b) The mass and volume depend upon the quantity of
energies of a substance. matter so these are extensive properties while ratio of mass
9. (d) The positive sign expresses when work is done on to its volume does not depend upon the quantity of matter
the system. Similarly, negative sign expresses when work so this ratio is an intensive property.
is done by the system 27. (b) 1 calorie = 4.184 joule
10. (a) q (heat) and work (w) are not state functions but (q + w)
is a state function. H – TS (i.e. G) is also a state function. 5
R
CP 2 5
11. (c) Internal energy and molar enthalpy are state functions. 28. (d) = = = 1.67
Work (reversible or irreversible) is a path function. CV 3 3
R
12. (d) Mathematical expression of first law of 2
thermodynamics 29. (a) In case of electric fan, electrical energy is converted
DE = q + w, DE is a state function. into mechanical energy and in case of heater, electrical
13. (d) When work is done by the system, DU = q – W energy is converted into heat energy. Therefore, these
14. (c) It may involve increase or decrease in temperature of follow the first law of thermodynamics.
the system. Systems in which such processes occur, are 30. (b)
thermally insulated from the surroundings. 31. (b) DH = DE + PDV, for solid and liquid,
15. (a) The shaded area shows work done on an ideal gas in a DV = 0 or DH = DE + Dn RT, for solids and liquids Dn = 0.
cylinder when it is compressed by a constant external pressure. 32. (c) During isothermal expansion of an ideal gas,
16. (b) As volume is constant hence, work done in this D T = 0. Now H = E + PV
process is zero. Hence, heat supplied is equal to change Q DH = DE + D(PV)
in internal energy. \ DH = DE + D(nRT);
17. (c) For isothermal reversible expansion. Thus if DT = 0., DH = DE
V2 i.e., remain unaffected
w = – nRT ln
V1 33. (a) if DE = 0, DE = DQ – W Þ DQ = W
34. (a) A – (p), B – (s), C – (r), D – (q)
18. (c) W = -PDV = -105 (1 ´ 10 -2 - 1 ´ 10-3 ) = -900J
Expansion of a gas in vacuum (pext = 0) is called free
19. (d) The difference between DH and DU is not usually expansion.
significant for systems consisting of only solids or liquids. For isothermal irreversible change
THERMODYNAMICS 521

q = –W = pext(Vf – Vi) For the same expansion of volume, the work done in
for isothermal reversible change irreversible process is greater than that in reversible one
q = – W = nRT ln (Vf /Vi) because the system has to work against friction etc. Thus
= 2.303 nRT log Vf /Vi DWirreversible > DWreversible
For adiabatic change, q = 0, DU = Wad Þ –DUirreversible > –DUreversible
35. (b) In an isothermal process change in internal energy [from equation (i)]
(DE) is zero (as it is a function of temperature). Þ DUirreversible < DUreversible
\ According to first law of thermodynamics Þ DTirreversible < DTreversible
Q Q + W = DE. Hence Q = –W (if DE = 0) Þ Tf irreversible > Tf reversible
If a system undergoes a change in which internal
energy of the system remains constant (i.e. DE = 0) then – 43. (a) q p = DH = C p dT
W = Q. This means that work done by the system equals
J
the heat absorbed by the system. Þ q p = 75.32 ´ (299 - 298) K
36. (b) DH = DU + DnRT for N 2 + 3H 2 ¾ ¾® 2 NH 3 K mol
Dng = 2 – 4 = – 2
J
\ DH = DU - 2 RT or DU = DH + 2 RT \ DU > DH Þ q p = 75.32
K mol
37. (a) Mass independent properties (molar conductivity and
For 180 kg of water, no. of moles of water
electromotive force) are intensive properties. Resistance
and heat capacity are mass dependent, hence extensive 180 ´ 103 g
properties. = = 104 moles
18g / mol
38. (d) The magnitude of the heat capacity depends on the
size, composition and nature of the system. J
q p = 75.32 ´ 104 moles
V2 mol
39. (a) q = –W = 2.303nRT log
V1 = 753.2 × 103 J = 753.2 kJ
DH for ATP = 7 kcal / mol
40. (c) For a cyclic process the net change in the internal
= 7 × 4.184 kJ/mol = 29.2 kJ/mol
energy is zero because the change in internal energy
29.2 kJ produced from 6.022 × 1023 molecules
does not depend on the path.
75.8
reversible path 753.2 kJ produced from 6.022 × 1023 ×
29.2
= 1.5 × 1025 molecules
B
44. (a) 18 g of water at 100oC
A
10 g of Cu at 25oC is added.

irreversible path
o
41. (a) –Wirreversible = Pext (V2 – V1) 100 C
= 10 atm (2L – 1L) = 10 atm – L
V2 qp = Cp, m dT
–Wreversible = 2.303 nRT log V
1 J 18g
= 75.32 ´ ´ (373 - 298) K
2 K mol 18g / mol
= 1 ´ 2.303 ´ 0.0821 × 298 atm–L /K /mol × log
1
J
= 16.96 atm–L = 75.32 × 75 K = 5649 J
K
Wreversible 16.96
= = 1.69 » 1.7 If now 10g of copper is added Cp, m = 24.47 J/ mol K
Wirreversible 10.00 Amount of heat gained by Cu
42. (c) According to first law of thermodynamics J 10g
= 24.47 ´ (373 – 298) K = 291.3 J
DQ = DU + DW K mol 63g / mol
An isolated system is adiabatic. This means DQ = 0. Heat lost by water = 291.30 J
J
The first law in this case yields – 291.30 J = 75.32 ´ (T2 - 373K )
0 = DU + DW Þ DW = – DU ..... (i) K
For expansion, DW is positive and hence DU is Þ –3.947 K = T2 –373 K
negative. This means Tf is less than Ti in both the cases. Þ T2 = 369.05 K
EBD_8350
522 CHEMISTRY

45. (c) Given Cp = 75 JK–1 mol–1 Þ 40.79 kJ/mol = DU + (1) (8.314 JK–1 mol–1) (373 K)
100 8.314 ´ 373
n= mole , Q = 1000 J DT = ? o æ ö
18 Þ DU = çè 40.79 kJ/mol - 1000
kJ / mol ÷
ø
1000 ´ 18
Q = nCpDT Þ DT = = 2.4 K
100 ´ 75 kJ
46. (b) The coefficients in a balanced thermo-chemical = (40.79 – 3.10) kJ/mol = 37.69
mol
equation refer to the number of moles (not to molecules) of
Internal energy change for 36 g of water
reactants and products involved in the reaction.
47. (b) Enthalpy of formation of C 2H4,CO2 and H2O are kJ 36g
= 37.69 ´
52, – 394 and – 286 kJ/ mol respectively. (Given) mol 18g / mol
The reaction is
DU = 75.98 kJ
C 2 H 4 + 3O 2 ® 2CO 2 + 2H 2 O.
change in enthalpy, 57. (d) 4NO 2 (g) + O 2 (g) ® 2N 2 O5 (g), D r H = – 111 kJ
(DH) = DHproducts – DHreactants
= 2 ´ (-394) + 2 ´ (-286) - (52 + 0) = – 1412 kJ/ mol. – 54 kJ
D H’
48. (b) Enthalpy change is a state function. 2N 2 O5 (s)
49. (a) Fe2O3(s)+ CO(g) ¾¾ ® 2FeO(s)+ CO2(g) [(i) – 2 × (ii)]
DH = –26.8 + 33.0 = + 6.2 kJ – 111 – 54 = D H’
50. (d) When a solid melts, increase in enthalpy is observed. D H’ = – 165 kJ
51. (a) Hess’s law is used for calculating enthalpy of reaction. 58. (a) To calculate average enthalpy of C – H bond in
52. (c) The value of enthalpy of neutralisation of weak acid methane following information is needed
by strong base is less than 57.1 kJ. This is due to the (i) dissociation energy of H2 i.e.
reason that the part of energy liberated during combination 1
of H+ and OH– ions is utilised in the ionisation of weak H 2 (g) ¾¾ ® H(g); DH = x ( suppose )
2
acid. Given DH = B.E. (Reactant) – B.E. (Product)
1 1 ® CH 4 (g); DH = 75 kJ mol -1
C(graphite) + 2H 2 (g) ¾¾
53. (b) H 2 + Cl 2 ¾¾
® HCl
2 2 59. (a) Conc. of HCl = 0.25 mole
Conc. of NaOH = 0.25 mole
DH HCl = å B.E. of reactant - å B.E. of products Heat of neutralization of strong acid by strong base
1 1 = – 57.1 kJ
-90 = ´ 430 + ´ 240 - B.E. of HCl HCl + NaOH ¾ ¾® NaCl + H2O + 57.1kJ
2 2
1 mole of HCl neutralise 1 mole of NaOH, heat evolved
\ B.E. of HCl = 215 + 120 + 90 = 425 kJ mol–1
54. (b) Enthalpy of reaction = 57.1 kJ
= B.E(Reactant) – B.E(Product) \ 0.25 mole of HCl neutralise 0.25 mole of NaOH
\ Heat evolved = 57.1 × 0.25 = 14.275 kJ
= éë B.E (C= C) + 4 B.E.(C–H) + B.E.(H - H) ùû 60. (c)
61. (a) H 2O (l) H 2O (g) + Q ; Dng = 1
- éë B.E.(C - C) + 6 B.E.(C - H) ùû DH = DE + Dng RT
= [606.1 + (4 × 410.5) + 431.37)] – [336.49 + (6 × 410.5)] 40660 = DE + 8.314 × 373
= –120.0 kJ mol–1 D E = 37558 J / mol
55. (c) The standard enthalpy of reaction is the enthalpy D E = 37.56 kJ mol–1
change for a reaction when all the participating substances 62. (d) This reaction shows the formation of H2O, and the X2
are in their standard states. The standard state of a represents the enthalpy of formation of H2O because as
substance at a specified temperature is its pure form at 1 the definition suggests that the enthalpy of formation is
bar. For example, the standard state of liquid ethanol at 298 the heat evolved or absorbed when one mole of substance
K is pure liquid ethanol at 1 bar. Standard state of solid is formed from its constituent atoms.
iron at 298 K is pure iron at 1 bar. The standard conditions 63. (b) Metallic bonding breaks in this reaction.
are denoted by adding the superscript to the symbol 64. (b) For the equation
DH e.g., –DH . B 2 H 6 (g) + 3O 2 (g) ¾¾
® B 2 O 3 (g) + 3H 2 O(g)
56. (a) H2O (l ) ® H2O (g) Eqs. (i) + 3 (ii) + 3 (iii) – (iv)
DHvap = 40.79 kJ/mol DH = – 1273 + 3(–286) + 3(44) – 36
DH = DU + DngRT = – 1273 – 858 + 132 – 36 = – 2035 kJ/mol
THERMODYNAMICS 523

65. (d) A process is spontaneous only when there is decrease in 80. (a) DG = - RT ln Keq : Normal body temperature = 37oC
the value of free energy, i.e., DG is –ve.
66. (d) DG is negative for a spontaneous process. kJ J
67. (a) Crystallization of sucrose solution. Entropy is a Þ -50 = 8.314 ´ 310 ln Keq
mol K mol
measure of randomness during the crystallisation of Þ 19.39 = lnKeq Þ Keq = 2.6 × 108
sucrose solution liquid state is changing into solid state
81. (b) DG = DH – TDS
hence entropy decreases.
DG = – TDS (when DH = 0 and DS = +ve)
68. (a) Third law of Thermodynamics.
DG = –ve
69. (c) DS has negative value if number of gaseous moles
decreases during a reaction, Dng = –ve 82. (a) DG° = DH° – TDS°
For the reaction For a spontaneous reaction DG° < 0
2SO2 + O2 ¾® 2SO3 DH °
or DH° – TDS° < 0 Þ T >
Dng = 2 – 3 = –1 DS °
70. (b) Spontaneity of reaction depends on tendency to 179.3 ´ 103
Þ T> > 1117.9K » 1118K
acquire minimum energy state and maximum randomness. 160.2
For a spontaneous process in an isolated system, the 83. (c) For a spontaneous reaction
change in entropy is positive.
DG(–ve), which is possible if DS = +ve, DH = +ve
71. (b) For the reaction
and TDS > DH [As DG = DH – TDS]
PCl5 (g) ƒ PCl3 (g) + Cl2 (g) 84. (a) If DGsystem = 0, the system has attained equilibrium, is
The reaction given is an example of decomposition reaction right choice.
and we know that decomposition reactions are endothermic In it, alternative (d) is most confusing as when DG > 0, the
in nature, i.e, DH > 0. process may be spontaneous when it is coupled with a
Further reaction which has DG < 0 and total DG is negative, so
Dn = (1 + 1) – 1= + 1 right answer is (a).
Hence, more number of molecules are present in products
85. (a) For spontaneous reaction, dS > 0 and dG should be
which shows more randomness i.e. DS > 0 negative i.e. < 0.
72. (d) Exercise - 2
q 1. (395) Since, work is done against constant pressure
73. (a) DS =
T and thus, irreversible.
q ¾¾
® required heat per mole Given, DV = (6 – 2) = 4 L; P = 1 atm
T ¾¾ ® constant absolute temperature 1 ´ 4 ´ 1.987
Unit of entropy is JK–1 mol–1 \ W = – 1 × 4 L-atm = – cal
0.0821
74. (a) For a spontaneous process, DStotal is always positive. (since 0.0821 L-atm = 1.987 cal)
75. (b) The factor TDS increases with increase in temperature. = – 96.81 cal = – 96.81 × 4.184 J (Q1 cal = 4.184 J )
76. (d) We know that DG = DH – TDS = – 405.05 J
When DH < 0 and DS < 0 then DG will be negative at low Now from Ist law of thermodynamics
temperatures (positive at high temperature) and the reaction q = DU – W
will be spontaneous. 800 = DU + 405.05
77. (d) Since the process is at equilibrium DG = 0, for DG = 0, \ DU = 395 J
there should be DH > 0, DS > 0. 2. (11.4) Given;
78. (b) For spontaneity, DH – TDS < 0 NaOH + HNO3 ® NaNO3 + H2O
1 mole 1 mole DH = 57.0 kJ
79. (b) Both assertion and reason are true but reason is not 0.2 mole 0.5 mole DH = ?
the correct explanation of assertion.
Given heat of neutralisation of strong acid by strong base
For a process to be spontaneous DG must be negative.
= 57.0 kJ
DG = DH – TDS Q 0.2 mole NaOH is limiting reagent.
Exothermic process (DH is negative) is non-spontaneous \ Heat of neutralization = 0.2 × 57 = 11.4 kJ
if DS is negative and temperature is high because in
3. (–328) Applying Hess’s Law
such condition TDS > DH. (DG = DH – TDS = +ive). When
1
temperature is decreased, T DS < DH (DG = DH – TDS D f H° = D sub H + D diss H + I.E. + E.A + D lattice H
2
=–ive) and so the reaction becomes spontaneous.
EBD_8350
524 CHEMISTRY

-617 = 161 + 520 + 77 + E.A. + (–1047) 8. (83.8) Molar heat capacity at constant pressure,
E.A. = –617 + 289 = –328 kJ mol–1 Cp = 248.2 × M J kg–1
\ electron affinity of fluorine where M is the molar mass of the gas.
= –328 kJ mol–1 Similarly, Cv = 149 × M J kg–1
Cp – Cv = R
1 3 ˆˆ† NH3 ;
4. (–964) Given N 2 + H 2 ‡ˆˆ \ 248.2 × M – 149M = 8.314
2 2
8.314
DHf = - 46.0 kJ / mol M= = 0.0838 kg/mol
248.2 - 149
ˆˆ† H 2 ; DH f = - 436 kJ / mol
H + H ‡ˆˆ Molar mass of the gas = 83.8 g/mol
9. (2.76) Given :
ˆˆ† N 2 ; DH f = - 712 kJ / mol
N + N ‡ˆˆ DS°CO2 = 213.5 JK–1;
1 3
DHf ( NH3 ) = DH N - N + DH H - H - DH N - H DS°C(s) = 5.74 JK–1;
2 2
DS°O 2 = 205 JK–1
1 3
-46 = ( -712 ) + ( -436 ) - DH N - H
2 2 C(s) + O 2 (g) ¾¾
® CO 2 (g)
On calculation
DH N - H = - 964 kJ / mol Standard entropy of formation of CO 2 (g)

5. (20) +
HA + aq ® H(aq) -
+ A (aq) , D H = x kJ mol -1 = DS°CO2 – [ DS°C(s) + DS°O 2 ]
= 213.5 – [5.740 + 205] = 2.76 JK–1
+ - -1
H(aq) + OH(aq) ® H 2O(l ) D H = -57.3 kJ mol
10. (158) DH° for reaction
- -
Hence, HA + OH(aq) ® H 2O(l ) + A(aq), = é H°TiCl4 ( l ) + HO
°
( g ) - HTiO
°
- H°Cl2 ´ 2ù
ë 2 2 û
DH = x – 57.3
But D H = x - 57.3 = -56.1 (given), ë 804.2 + 0.0 - ( -944.7 ) - 0.0ùû = 140.5 kJ
= é-
Also, DS° for reaction
-1
x = 1.2 kJ mol
= éS°TiCl4 ( l ) + S°O2 ( g ) - STiO
°
( s ) - S°Cl2 ( g ) ´ 2ùû
if no self ionization of HA occurs at all, ë 2

= [252.3 + 205.1 – 50.3 – 2 × 233.0]


D H(ionization) = 1.5 kJ mol-1
= – 58.9 J = – 0.0589 kJ K–1
Hence, % ionization in 1 M solution
Now, DG° = DH° – TDS°
(1.5 - 1.2) = 140.5 – 298 × (– 0.0589) = 158 kJ
= ´ 100 = 20
1.5 Exercise - 3
6. (22.8) DH 2 - DH1 = DCp (T2 - T1) 1. (c) Thermodynamics deals with the energy change,
feasibility and extent of a reaction, but not with the rate
DH 2 - 24 = (0.031 - 0.055)(100 - 50)
and mechanism of a process.
Þ DH2 = 22.8 calg–1 2. (c) For a closed vessel made of copper, there will be no
exchange of matter between the system and th e
7. (–17.15) (i) CuSO 4 (s) + H 2 O ¾¾
® CuSO 4 (aq) surroundings but energy exchange can occur through its
-1.451 walls.
DH = = -7.255k cal mol-1 3. (d) The state of a gas can be described by quoting the
0.2
relationship between pressure, volume, temperature and
(ii) CuSO 4 .5H 2O(s) + H 2 O ¾¾
® CuSO 4 (aq) amount. The ideal gas equation is
PV = nRT
0.264
DH = = 1.32 k cal mol -1 4. (c) Specific heat is an intensive property which depends
0.2 only on the nature of the gas. Hence, if the volume of gas
From (i) – (ii), is reduced to half from its original volume the specific
CuSO4 (s) + 5H 2O(l) ¾¾
® CuSO 4 .5H 2O(s); heat will remain constant.
5. (c) The complete combustion of one mole of butane is
represented by
D H = -28.575 k cal mol-1
THERMODYNAMICS 525

11. (c) x > y because same bonds are formed in reaction (1)
13
C4 H10 ( g ) + O 2 ( g ) ® 4CO2 ( g ) + 5H 2 O ( l ) and (2) but no bonds are broken in reaction (1) whereas in
2 reaction (2) bonds in the reactant molecules are broken.
DcH should be negative and have a value of 2658 kJ mol–1. As energy is absorbed when bonds are broken, energy
6. (b) Df H° = Df U° + DngRT released in reaction (1) is greater than that in reaction (2).
For the reaction, 12. (c) Enthalpy of formation can be positive or negative.
C(s) + 2H2(g) ® CH4(g) For example :
D ng = 1 – 2 = –1 C+ O2 ® CO2 is exothermic whereas, C+ 2S ® CS2 is
\ Df H° = DU – 1 × RT endothermic.
\ Df H° < Df U° 13. (a) Enthalpy of sublimation of a substance is equal to
7. (c) For free expansion, W = 0 ; and enthalpy of fusion + enthalpy of vaporisation.
For Adiabatic process, q = 0 Sublimation is, direct conversion of solid to vapour.
According to first law of thermodynamics, Writing in two steps, we have solid ® liquid ® vapour
DU = q + W = 0 solid ® liquid requires enthalpy of fusion and
Since, there is no change in DU hence, temperature change liquid ® vapour requires enthalpy of vaporisation.
will be zero i.e., DT = 0 14. (b) DG gives a criterion for spontaneity at constant
8. (b) Area under the curve is always greater in irreversible pressure and temperature.
compression than that in reversible compression which
can be seen from given figure. (i) If DG is negative (< 0), the process is spontaneous.
(ii) If DG is positive (> 0), the process is non-
spontaneous.
(iii) If DG is zero then reaction is at equilibrium.
¾¾®

15. (b) For adsorption DS < 0 and for a spontaneous change


P DG = – ve
hence, DH should be highly negative which is clear from
the equation
DG = DH – TDS = – DH – T(– DS) = – DH + TDS
Vf Volume (V) Vi So, if DH is highly negative DG will be (– ve)
¾¾
Irreversible ¾®
compression
16. (b) Given DU = 2.1 k cal., DS = 20 cal. K– 1
T = 300 K
Q DH = DU + DngRT
Putting the values given in the equation
¾ ¾®

P
2
DH = 2.1 + 2 ´ ´ 300 = 2.1 + 1.2 = 3.3 k cal.
1000
20
Now, DG = DH – TDS = 3.3 - 300 ´ = - 2.7 kcal
1000
Vf Volume (V) Vi
¾¾¾® 17. (a) C 2 H 5 OH(l) + 3O 2 (g) ® 2CO (g) + 3H O(l)
2 2
Reversible compression
9. (c) During the process of freezing, energy is released Bomb calorimeter gives DU of the reaction
which is absorbed by the surroundings. Given, DU = –1364.47 kJ mol–1
q Dng = – 1
\ DSsys = - rev
T DH = DU + DngRT
qrev
DSsurr = i.e., on freezing, entropy of the system 1 ´ 8.314 ´ 298
T = -1364.47 - = – 1366.93 kJ mol–1
decreases and of surrounding increases. 1000
10. (c) 18. (a) C + O2 ® CO2 + 393.5 kJ/mol
(a) C (graphite) + O2 (g) ® CO2 (g); DrH = x kJ mol–1 … (i) 12g 44g
1 44g is formed from 12g of carbon
(b) C (graphite) + O 2 (g) ® CO (g); D r H = y kJ mol –1
2 12 ´ 35.2
… (ii) 35.2g is formed from g of C
44
On subtracting eqn (i) & (ii) we get = 9.6 g of C = 9.6/12 = 0.8 mole
1 From 1 mole, released heat = 393.5 kJ
CO ( g ) + O 2 ( g ) ® CO 2 ( g ) ; D r H = z kJ mol-1
2 0.8 mole, released heat = 393.5 × 0.8
Hence, x – y = z or x = y + z = 314.8 kJ » 315 kJ
EBD_8350
526 CHEMISTRY

19. (c) DG = DH – T · DS 26. (a) DG = –RT ln K Þ DH – TDS = –RT ln K


For a spontaneous reaction DG = –ve (always) – DH ° DS °
which is possible only if Þ ln K = +
RT R
DH < 0 and DS > 0 DH = –ve exothermic reaction
\ spontaneous at all temperatures.
– DH °
20. (b) Given slope = = + ve
C(s) + O2(g) ® CO2(g); DH = –393.5 kJ mol–1 …(i) R
So from graph, line should be A and B.
1
CO(g) + O (g) ® CO2(g); DH = –283.5 kJ mol–1 …(ii) 27. (d) In 2H(g) ¾® H2(g), no. of species decreases,
2 2
therefore entropy decreases.
\ Heat of formation of CO = eqn(i) – eqn(ii)
28. (a) W = – Pext (V2 – V1) (Irreversible isothermal expansion)
= –393.5 – (–283.5) = –110 kJ = –2 (0.25 – 0.1) = – 2 (0.15) = – 0.3 L Bar
21. (b) The system is in isolated state. = – 0.3 × 100 J = – 30 J
Q For an adiabatic process, q = 0 29. (a) We know that heat and work are not state functions
DU = q + w but q + w = DU is a state function. H – TS (i.e. G) is also a
\ DU = w = –PDV = –2.5 atm × (4.5 – 2.5) L state function.
= –2.5 × 2 L-atm = –5 × 101.3 J = – 506.5 J » – 505J 30. (b) For reversible isothermal expansion,
22. (a) Given DH = 35.5 kJ mol–1 V2
DS = 83.6 JK–1 mol–1 w = –nRT ln
V1
Q DG = DH – TDS
For a reaction to be spontaneous, DG = –ve V2
i.e., DH < TDS |w| = nRT ln
V1
DH 35.5 ´ 103 Jmol-1 |w| = nRT (lnV2 – lnV1)
\ T> =
DS 83.6 JK -1 |w| = nRT ln V2 – nRT V1
So, the given reaction will be spontaneous at T > 425 K y = mx + c
23. (c) From 1st law of thermodynamics So, slope of curve 2 is more than curve 1 and intercept of
curve 2 is more negative than curve 1.
DU = q + w
For adiabatic process : 31. (c) We know that, Cl 2 (g) ¾¾ ® 2Cl (g) is endothermic
q=0 reaction because it required energy to break bond.
\ DU = w
So reverse reaction, 2Cl (g) ¾¾
® Cl2 (g) will be
24. (d) Let B.E of X2, Y2 and XY are x kJ mol–1,
0.5 x kJ mol–1 and x kJ mol–1 respectively. exothermic, D r H < 0 .
Also, two gaseous atom combine together to form 1
1 1
X 2 + Y2 ® XY; DH = –200 kJ mol –1 gaseous molecule.
2 2
DH = –200 = S (B.E)Reactants – S(B.E)Product So, randomness decreases i.e., D r S < 0.
32. (d) Free expansion of ideal gas
é1 1 ù Pex = 0
= ê 2 ´ ( x ) + 2 ´ (0.5 x ) ú – [1 ´ ( x )]
ë û \ w = - Pex DV = 0
On solving, x = 800 kJ mol –1
Q Adiabatic process Þ q = 0
15
25. (d) C 6 H 6 (l) + O 2 (g) ® 6CO 2 (g) + 3H 2 O(l) DE = q + w (first law of thermodynamics)
2
\ DE = 0
Dng = 6 – 7.5 = –1.5
DE = nCv dT Þ DE = 0
DU or DE = – 3263.9 kJ
So, q = 0, DT = 0, w = 0.
DH = DU + Dng RT
33. (–2.70)
Dng = –1.5
DU = 2.1 kcal = 2.1 × 10 3 cal
R = 8.314 JK–1 mol–1 Dng = 2
T = 298 K DH = DU + DngRT = 2.1 × 103 + 2 × 2 × 300
So, DH = –3263.9 + –1.5 × 8.314 ×10–3 × 298 = 2100 + 1200 = 3300 cal
= –3267.6 kJ DG = DH – TDS = 3300 – 300 × 20
= 3300 – 6000 = –2700 cals = –2.7 kcal
THERMODYNAMICS 527

Kp = (2X)2 × X
Exercise - 4
Kp = 4X3
1. (c) I2(s) ¾¾® I2(g)
DG° = RT ln Kp
Heat of reaction depend upon temperature i.e., it varies
with temperature, as given by Kirchoff's equation, DG° = – RT ln (4X3)
T2 DG° = – RT ln 4 – 3RT ln X
DHT2 = DH T1 + ò DCp dT 6. (d) DH needed for change = DHheating + DHvaporisation
T1
= 50 ´ 103 ´ 1 ´ 90 + 540 ´ 50 ´ 103
where DCp = Cp of product – Cp of reactant = 31500 × 103 cal
\ DCp = 0.031 – 0.055 = – 0.024 cal/g
31500 ´ 103 ´ 100
Now, DHT - DHT = DCp ( T2 - T1 ) DH actually needed =
2 1 60
DH(250) – DH(200) = –0.024 (523 – 473)
(Since only 60% of heat is used to do so)
DH(250) = 24 – 50 × 0.024 = 22.8 cal/g
Now 368 ´ 103 cal heat is given on combustion by 22.4 litre
2. (c) (a) From charle’s law : V µ T C2H6
Þ V = k1 T (at cons. P) 31500 ´ 103 ´ 100
\ heat is given by
æ dV ö 60
Þ ç ÷ = k1 ¹ 0
è dT ø P 22.4 ´ 31500 ´ 103 ´ 100
=
(b) From Gay Lussac’s law : 60 ´ 368 ´ 103
PµT (at cons. V) = 3.196 × 103 litre = 3.196 m3
Þ P = K2 T
1 3
æ dP ö 7. (b) N 2 (g) + H 2 (g) ¾¾
® NH 3 (g)
Þ ç ÷ = k2 ¹ 0 2 2
è dT øV 1 3
(N º N) + (H– H) ¾¾ ® H – N– H
(c) For an ideal gas, change in internal energy is zero at 2 2 |
constant temperature. (dU)T = 0 H
1 3
æ dU ö DH °f = DH N º N + DH H - H - 3DH N - H
Þ ç ÷ =0 2 2
è dV øT 1 3
°
(d) DU = nCV DT (at cons. V) DH f = x1 + x2 - 3x3
2 2
æ dU ö CP
Þ ç = nCV ¹ 0 8. (b) = g and CP – CV = R
è dT ÷ø V CV
3. (c) DH = (n1Cp,m + n2Cp,m ) DT CP
1 2 CP - =R
æ 7 ö g
= ç 0.5 ´ R + 0.5 ´ 4 R ÷ ( -100 ) æ g - 1ö gR
è 2 ø Þ CP ç ÷ = R Þ CP =
è g ø g -1
= – 375 R (DH is defined at cons. P)
gCV - CV = R
4. (c) w = – Pext (Vf – Vi)
R
æ 60 ´10-3 40 ´10-3 100 ´10-3 ö Þ CV ( g - 1) = R Þ CV =
= -105 ç + - ÷ g -1
ç 0.60 1000 1000 ÷ø
è 9. (c) Let x mole of KOH be neutralized by the strong acid
= – 105 (100 × 10–3 + 0.04 × 10–3 – 0.1 × 10–3) HA. Then, moles neutralized by HB = 1 – x
|w| = 9994 J Hence, – 13.7 × x + (– 12.7) × (1 – x) = – 13.5
5. (d) DG° = – RT ln KP; initial mole x 0.8
Þ x = 0.8; = =4
mole at e.g. Mole fraction partial pressure 1 - x 0.2
ˆˆ† 2NH3 (g) + CO 2 (g) 10. (d) In case of dissociation of CH4, the dissociation
NH 2 COONH 4 (s) ‡ˆˆ
energy is equal to the energy required to break four C – H
1 0 0
H
1- z 2z z

2 1 bonds (H — C — H)
´ 3X ´ 3X

3 3
H
EBD_8350
528 CHEMISTRY

360 10-2
\ C – H bond energy = = 90 DG = V DP – 2(300 - 200) - (300 2 - 200 2 )
4 2
10-2
H H DG = 24.63(1.5 - 1) – 200 - (5 ´ 10 4 )
2


In case of dissociation of C2H6 (H — C — C — H) DG = 12.315 ´ 100 J – 450 J


H H DG = 781.5 J [1L-atm = 100 J]
6 C – H bonds and one C – C bond break
\ C – C bond energy = 620 – 6 × 90 14. (b) DG = – PDV = Work done
= 620 – 540 = 80 k cal/mol DV is the change in molar volume in the conversion of
1 graphite to diamond.
11. (a) N 2 (g) + O 2 ® N 2 O(g)
2 æ 12 12 ö
DV = ç - ´ 10-6 m3 = -1.708 ´ 10-6 m3
1 + – è 3.31 2.25 ÷ø
N º N(g) + (O = O) ® N = N = O (g)
2 Work done = –(–1.708 × 10–6) × PJ
DH°f = [Energy required for breaking of bonds ] 1895 Jmol–1 = – (– 1.708 × 10–6) × PJ
- [Energy released for forming of bonds] 1895J mol –1
\P = = 1109.4 ´ 106 Pa
-3
1.708 ´ 10
1
= (DH ( N º N ) + DH ( O = O ) - ( DH N = N + DH (N = O) ) = 11.094 × 108 Pa
2
15. (d) Given : H = f (P, T)
1
= (946 + ´ 498) - (418 + 607) = 170 kJ mol -1
2 æ ¶H ö æ ¶H ö
dH = ç × dT + ç × dP
Resonance energy = Observed Df H° – Calculated Df H° è ¶T ÷ø P è ¶P ÷ø T
= 87 – 170 = –88 kJ mol–1
DH
12. (b) DHS-S + 2DH H -S = 239 , 2DH H -S = 175 At constant pressure = CP ,
DT
Hence, DHS-S = 239 - 175 = 64 kcal mol -1
æ ¶H ö
Then, DH for 8S(g) ® S8 (g) is 8 × (–64) = –512k cal Thus, dH = CP × dT + ç ÷ × dp
è ¶P øT
P1 P2 We know that H = U + PV
13. (c) At constant volume, =
T1 T2
æ ¶H ö æ ¶H ö æ ¶V ö æ ¶P ö
300 3 ç ÷ =ç ÷ + Pç ÷ +V ç ÷
Þ P2 = 1 ´ = atm = 1.5 atm è ¶P øT è ¶P øT è ¶P øT è ¶P øT
200 2
and V1 = 24.63 L æ ¶H ö é ¶ æ RT öù
ç ÷ =0+Pê ç ÷ú + V
Q dG = Vdp - SdT è ¶P øT ë ¶P è P ø ûT
[At constant temperature DU = 0 and PV = RT]
2 2 300
ò dG = V ò dP - ò ( 2 + 10-2 T ) dT æ ¶H ö
ç
æ 1 ö
÷ = PRT ç - 2 ÷ + V
1 1 200 è ¶P øT è P ø
300
é 10-2 2 ù æ ¶H ö RT
G2 - G1 = V ( P2 - P1 ) - ê2T + T ú ç ÷ =- + V = -V + V = 0
2 è ¶P øT P
êë úû 200
EQUILIBRIUM 529

7 Equilibrium
Exercise - 1 13. ˆˆ† B + C
(b) Given reaction, 2A ‡ˆˆ
1. (c) [B][C]
Kc =
2. (a) A reaction is said to be in equilibrium when rate of [A]2
forward reaction is equal to the rate of backward reaction.
3. (c) When the watch glass is open to the atmosphere, the 2 ´ 10 -3 ´ 3 ´ 10 -3
Kc = =6
rate of evaporation remains constant but the molecules (10-3 )2
are dispersed into large volume of the room. As a
consequence the rate of condensation from vapour to ˆˆ† 2AB [AB]2
14. (c) A2 + B2 ‡ˆˆ Kc =
liquid state is much less than the rate of evaporation. [A 2 ][B2 ]
4. (c)
5. (b) At equilibrium, the rate of forward and backward (2.8 ´ 10-3 )2 (2.8)2
Kc = = = 0.62
reactions is equal. 3 ´ 10-3 ´ 4.2 ´ 10-3 3 ´ 4.2
6. (b) (A) Liquid ‡ˆˆ ˆˆ† Vapour, equilibrium exists at the 15. (b) For the reaction
boiling point. CaCO3 (s) ƒ CaO (s) + CO2 (g)
(B) Solid ‡ˆˆ ˆˆ† Liquid, equilibrium exists at the
On the basis of the stoichiometric equation, we can write,
melting point. Kc = [CaO(s)] [CO2(g)]/[CaCO3(s)]
(C) Solid ‡ˆˆ ˆˆ† Vapour, equilibrium exists at the
Since [CaCO3(s)] and [CaO(s)] are both constant, therefore,
sublimation point. modified equilibrium constant for th e thermal
(D) Solute ‡ˆˆˆˆ† Solute (solution), equilibrium exists decomposition of calcium carbonate will be
in a saturated solution.
Kc = [CO2(g)]
7. (b) The reaction mixtures starting either with H2 or D2
reach equilibrium with the same composition, except that Kp = [pCO (g)]
2
D2 and ND3 are present instead of H2 and NH3. ˆˆ† C , Dn = 1 – 2 = –1 (assuming A, B
8. (a) Equilibrium can be attained by either side of the 16. (a) For A + B ‡ˆˆ
reactions of equilibrium. and C are gases)
9. (d) (i) Solubility is high at high pressure. As the water Dn -1
é mol ù é mol ù
bottle is opened, solubility decreases. Unit of K c = ê ú =ê
(ii) The given equilibrium is governed by Henry’s law. ë litre û ë litre úû
(iii), (iv) The amount of CO2 gas dissolved in liquid = Litre mol–1
decreases with increase in temperature or it increases with 17. (d) 2C(s) + O2(g) ƒ 2CO2 (g)
decrease in temperature. Dn = 2 – 1 = + 1 ¹ 0
10. (c) N2 + 3H2 ‡ˆˆ ˆˆ† 2NH \ Kc and Kp are not equal.
3
\ K = [NH3]2 / [N2] [H2]3 .... (i) ( PCO ) 2 4´ 4
18. (c) Kp = ;Kp = = 8; C(s) = 1;
1 3
ˆˆ† NH3
N2 + H 2 ‡ˆˆ ( PCO2 ) 2
2 2 The concentration of solids and liquids are taken as unity.
\
[NH 3 ] .... (ii) 19. (c) Using the relation Kp = Kc. (RT)Dn, we get
K¢ =
[N 2 ]1/2 [H 2 ]3/2 Kp
= ( RT ) Dn
Dividing equation (i) by equation (ii), we get Kc
K' = K Kp
11. (d) Rate constant of forward reaction (Kf) = 1.1 × 10–2 and Thus will be highest for the reaction having highest
Kc
rate constant of backward reaction (Kb) = 1.5 × 10–3 per
minute. value of Dn.
The Dn values for various reactions are
Kf 1.1 ´ 10-2
Equilibrium constant (Kc) = = = 7.33 æ 1ö 1
Kb 1.5 ´ 10 -3 (a) Dn = 1– ç1 + ÷ = –
è 2 ø 2
12. (c) A + B ˆˆ†
‡ˆˆ C + D (b) Dn = 2 – (1 + 1) = 0
At eqm. x x 2x 2x (c) Dn = (1 +1) – 1= 1
2x × 2 x (d) Dn = (2 + 4) – (7 + 2) = – 3
Kc = =4 Thus maximum value of Dn = 1
x×x
EBD_8350
530 CHEMISTRY

20. (b) 3Fe(s) + 4H2O (steam) Fe3O4 (s) + 4 H2 (g) 29. (b) PCl5 ƒ PCl3 + Cl2
1 1 1
( pH 2 ) 4 Moles at
2 2 2
Kp = only gaseous products and reactants. equilibrium
( pH 2O ) 4 1 1 1
Mole fraction
[Ni(CO) 4 ] mol L-1 3 3 3
21. (a) K= = = (mol l -1 ) -3 at equilibrium
[CO]4 (mol L-1 ) 4 P P P
Partial pressure
3 3 3
22. (c) Reaction (iii) can be obtained by adding reactions (i) at equilibrium
and (ii) therefore, K3 = K1. K2 P P
Hence, (c) is the correct answer. ´
P Þ P = 3K
Kp = 3 3 =
1.44 ´ 10 -5
p
Kp P/3 3
23. (d) K c = =
( RT ) Dn (0.082 ´ 773) -2 30. (b) ˆˆ† 2NH 3 (g) + CO 2 (g)
NH 2COONH 4 (s) ‡ˆˆ
(R in L. atm. K–1 mole–1).
24. (a) Kp = Kc(RT)Dn ( PNH3 )2 ´ ( PCO2 )
Dn g
KP = = (PNH3 )2 ´ (PCO2 )
25. (c) As Kp = Kc ( RT ) PNH 2COONH 4 (s )
Here Dng = 1 As evident by the reaction, NH3 and CO2 are formed in
So, Kp = Kc when RT = 1 Þ 0.08 × T = 1 molar ratio of 2 : 1. Thus if P is the total pressure of the
Thus, T = 12.5 K system at equilibrium, then
26. (b) C(s) + CO2(g) ¾® 2CO(g) 2´ P 1´ P
PNH3 = ; PCO2 =
Apply law of mass action, 3 3
2
( P )2 (10 PCO2 ) 2
4 P3
K P = CO or 63 = æ 2P ö P
PCO2 PCO2 KP = ç ÷ ´ =
è 3 ø 3 27
(Given KP = 63) and PCO = 10PCO2 Given KP = 2.9 × 10–5
100( PCO2 )2 4P 3
or 63 = \ 2.9 ´ 10 -5 =
or 63 = 100 PCO2 27
PCO2
63 2.9 ´ 10 -5 ´ 27
PCO2 = = 0.63 atm P3 =
100 4
1
PCO = 10 PCO 2 = 10 × 0.63 = 6.3 atm æ 2.9 ´ 10 -5 ´ 27 ö 3
P=ç –2
÷ = 5.82 × 10 atm
Ptotal = PCO2 + PCO = 0.63 + 6.3 = 6.93 atm. è 4 ø
[Ni(CO)4 ] [CO]2
27. (a) K1 = ; K = 31. (b) (I) ˆˆˆ
K1
N 2 + 2O 2 ‡ˆˆ †
ˆ 2NO 2
[CO2 ]2 2 [CO2 ]
2 [NO 2 ]2
[Ni(CO) 4 ] [Ni(CO)4 ] æ [CO2 ] ö K1 = ...(i)
K= Þ K= × çç ÷ [N 2 ][O 2 ]2
[CO] 4
[CO2 ] 2 2÷
è [CO] ø
ˆˆˆ†
K2
(II) 2NO 2 ‡ˆˆˆ N 2 + 2O 2
K1
K= [N 2 ] [O2 ]2
K22 K2 = ...(ii)
[NO 2 ]2
28. (b) PCl5 ˆˆ† PCl3 + Cl2
‡ˆˆ 1
a (1-x) ax ax ˆˆˆ
K3
(III) NO2 ‡ˆˆˆ† N2 + O2
2
a = 2, x = 0.4, V = 2 L
[N 2 ]1/2 [O2 ]
2(1 - 0.4) K3 =
\ [ PCl5 ] = = 0.6 mol L-1 [NO 2 ]
2
[N 2 ][O2 ]2
2 ´ 0.4
[ PCl3 ] = [ Cl2 ] = 2 = 0.4 mol L-1 \ ( K3 ) 2 = ...(iii)
[NO2 ]2
\ from equations (i), (ii) and (iii)
\ K c = 0.4 ´ 0.4 = 0.267
0.6 1 1
K1 = =
K 2 ( K3 ) 2
EQUILIBRIUM 531

32. (b) For the reaction


KP1 a2 × P1 K P1 1 P
2AB2 (g) ƒ 2AB(g) + B2 (g) = or = × 1
KP2 2 K P2 4 P2
At eqm 2(1- x ) 2x x 4a × P2
[ AB] [ B2 ]
2 2
(2 x ) ´ x
= x3 1 P é KP 9ù
Kc = or Kc = or 9= × 1 ê\ 1 = ú
[ AB2 ]2 {2(1 - x )}2 4 P2 ëê KP2 1 úû
[(1 – x) can be neglected in denominator (1 – x) ; 1] P1 36
The partial pressure at equilibrium are calculated on the or = or P1 : P2 = 36 : 1
P2 1
basis of total number of moles at equilibrium.
Total number of moles = 2(1 – x) + 2x + x = (2 + x) 34. (d) Equilibrium constant (K) is independent of
concentrations of reactants and products.
2(1 - x ) 35. (b) Reaction proceed backward according to Le-
\ PAB2 = ´ P where P is the total pressure.
(2 + x ) chatelier’s principle.
2x x 36. (b)
PAB = ´ P , PB = ´P 37. (d) For reaction to proceed from right to left
(2 + x ) 2 (2 + x )
Q > Kc i.e., the reaction will be fast in backward direction.
Since x is very small so can be neglected in denominator
Thus, we get 38. (a) Equilibrium constant is not effected by change in
PAB2 = (1 – x) × P; PAB = x × P conditions like P and V. These changes can change only
the time required to attain equilibrium.
x
PB = ´ P 39. (b) The equilibrium constant helps in predicting the
2 2 direction in which a given reaction will proceed at any
( PAB )2 ( PB2 ) stage. For this purpose, we calculate the reaction quotient
Now, K P =
( PAB2 )2 Q. The reaction quotient Q(Qc with molar concentration
and Qp with partial pressures) is defined in the same way
x
( x )2 ´ P 2 × P ´ x3 × P3 as the equilibrium constant K c except that the
2
= = [\ 1 – x ; 1] concentrations in Qc are not necessary equilibrium values.
(1 - x )2 ´ P 2 2 ´ 1 ´ P2 1 40. (a) For Dn = 0, no effect of pressure.
x3 × P 2× Kp æ 2K p ö 3 41. (c) Both (a) and (b) are correct for the equation,
= or x3 = or x = ç ÷
2 P è P ø d
-
33. (c) Given reaction are K = e -DG / RT
42. (d) DG = 0 and DG° = –RTln K
X ƒY +Z ..... (i)
43. (d) According to Le-chatelier's principle, whenever a
and A ƒ 2 B .... (ii) constraint is applied to a system in equilibrium, the system
Let the total pressure for reaction (1) and (2) be P1 and P2 tends to readjust so as to nullify the effect of the constraint.
respectively, then 44. (a) If the volume is kept constant and an inert gas such
KP 9 as argon is added which does not take part in the reaction,
1 = (given) the equilibrium remains undisturbed. It is because the
KP 1 addition of an inert gas at constant volume does not change
2
After dissociation, the partial pressure or the molar concentration of the
X ƒ Y + Z substance involved in the reaction. The reaction quotient
At equilibrium (1– a) a a changes only if the added gas is reactant or product
[Let 1 mole of X dissociate with a as degree of dissociation] involved in the reaction.
Total number of moles = 1– a + a + a = (1+ a) 45. (d)
46. (d) Solid ‡ˆˆˆˆ† Liquid
æ1- a ö æ a ö
Thus PX = ç ÷.P ;P = ç ÷ P1; It is an endothermic process. So when temperature is raised,
è1+ a ø 1 Y è1+ a ø æ a ö
PZ = ç ÷ × P1 more liquid is formed. Hence, adding heat will shift the
è1+ a ø equilbrium in the forward direction.
47. (c) As in this, no. of moles are increasing hence, low
\ KP æ a ö a æ 1- a ö pressure will favour the forward direction.
=ç ÷ × P1 ´ ×P/ × P ... (i)
1 è1+ a ø (1 + a ) 1 çè 1 + a ÷ø 1 Dn = (1 + 1) – 1 = 1
48. (b) The most favourable conditions are :
Similarly for ˆˆ† 2B
A ‡ˆˆ (i) High pressure (Dn < 0)
At equilibrium (1 – a) 2a
(ii) Low temperature (Exothermic reaction)
We have, (iii) Catalyst Fe in presence of Mo.
2
æ 2aP2 ö æ 1 - a ö 49. (c) By increasing pressure at constant volume, molar
K P2 = ç ÷ /ç ÷ P2 ...(ii)
è 1 + a ø è 1+ a ø concentration remains same. Thus, the equilibrium position
Dividing (i) by (ii), we get remains same.
EBD_8350
532 CHEMISTRY

50. (d) A2(g) + B2(g) 3C (g) D(g) 64. (b) HA ¾® H+ + A–


step-1 step-2 [H + ][A - ]
Ka = , Q [H+] = 10–pH
since the steps 1 and 2 are exothermic hence low temprature [HA]
will favour both the reactions. In step - 1 moles are \ [H+] = 10–5 ; and at equilibrium [H+] = [A– ]
increasing hence low pressure will favour it. In step 2, 10 -5 ´ 10-5
moles are decreasing, hence high pressure will favour it. \ Ka = = 2 ´ 10 -8
0.0015
51. (a) The reaction given is an exothermic reaction thus
according to Le-chatalier ’s principle, lowering of Ka 2 ´10-8
a= = = 4 ´ 10-6 = 2 ´ 10-3
temperature, addition of F2 and / or Cl 2 favour the C .005
forward direction and hence, the production of ClF3. Percentage ionization = 0.2
52. (b) An equilibrium constant does not give any
information about the rate at which the equilibrium is 65. (b) Acidic strength µ Ka
reached. 66. (b) pKa = –log Ka
53. (a) Ice Water ; DH = +ve Smaller the value of pKa, stronger will be acid
(19.66 cm 3/mol) (18.02 cm 3/mol) \ Acid having pKa value of 10–8 is strongest acid.
(i) When temperature is increased, the ice-water system
will try to decrease this effect by decreasing the a1 Ka1 3.14 ´ 10-4
67. (b) = = = 4 :1
temperature. This can be done by forming more water as a2 K a2 1.96 ´ 10-5
this process absorbs energy. 2
æ 1.34 ö
(ii) When external pressure is increased, the ice-water 68. (a) K = ca 2 = 0.1 ´ ç = 1.8 ´ 10 -5
è 100 ÷ø
system oppose the external pressure by increasing its own
69. (c) Due to common ion effect addition of NH4Cl in group
pressure. It is like pushing a piston in a gaseous system, (III) suppresses the ionisation of NH4OH with the result,
that is in equilibrium, the system work to push back the concentration of OH– decreases.
piston. 70. (c) Solubility of weak electrolyte decreases in solvent
Now to increase the pressure on the external agent, the having common ion. So solubility of AgI in NaI solution is
ice-water system can decrease its volume by forming more less than in pure water because of common ion effect.
water. 71. (b) CH3COOH is weak acid while NaOH is strong base,
54. (d) Base accepts protons and acid donates protons. so one equivalent of NaOH can not be neutralized with
55. (b) Because NH3 after losing a proton (H+) gives NH2– one equivalent of CH3COOH. Hence the solution of one
NH3 + H2O ‡ˆˆ ˆˆ† NH – + H O + equivalent of each does not have pH value as 7. Its pH will
2 3
(Conjugate acid-base pair differ only by a proton) be towards basic side as NaOH is a strong base. Hence,
56. (d) (CH3)3 B - is an electron deficient, thus behave as a conc. of OH– will be more than the conc. of H+.
lewis acid. 72. (c) HSO4– accepts a proton to form H2SO4.
57. (c) Boron in B2H6 is electron deficient Thus, H2SO4 is the conjugate acid of HSO4–.
58. (a) Bronsted base is a substance which accepts proton. + H+
HSO4- ¾¾¾® H 2SO4
In option (a), H2O is accepting proton, i.e., acting as a base. base conjugate acid
59. (b) The value of ionic product of water changes with the of HSO-4
temperature. 73. (b) HClO4 is a strong acid
60. (b) HNO2 is a weak acid.
61. (b) Given [OH–] = 10–3 NH2– is a very good proton acceptor and thus, it is a strong
\ pOH = 3 base.
Q pH + pOH = 14 H2SO4 is a strong acid hence its conjugate base (HSO4–)
\ pH = 14 – 3 = 11 will be a weak base.
62. (a) Given [H3O+] = 1 × 10–10 M 74. (a) (i) H 3PO 4 + H 2 O ¾¾ ® H 3O + + H 2 PO 4-
at 25º [H3O+] [OH–] = 10–14 acid1 base2 acid 2 base1
-14
10 (ii) H 2 PO-4 + H 2O ¾¾ ® HPO42- + H3O+
\ [OH - ] = = 10-4
-10
10 acid1 base 2 base1 acid2
Now, [OH - ] = 10- p
OH
= 10 -4 -
(iii) H 2 PO 4 + OH ¾¾
-
® H 3 PO 4 + O 2 -
base acid 2 acid base2
\ pOH = 4 1 1
Hence, only in (ii) reaction H2PO4– is acting as an acid.
63. (a) Given Ka = 1.00 × 10–5, C = 0.100 mol for a weak
electrolyte, degree of dissociation 75. (c) On dilution [H+] = 10–6 M = 10–6 mol
Now dissociation of water cannot be neglected,
Ka 1 ´ 10–5 Total [H+] = 10–6 + 10–7 = 11 × 10–7
(a) = = = 10-2 = 1%
C 0.100 pH = –log [H+] = –log (11 × 10–7) = 5.98
EQUILIBRIUM 533

76. (d) Q pH = 1 ; H+ = 10–1 = 0.1 M


[H 3O+ ][F- ]
pH = 2 ; H+ = 10–2 = 0.01 M 82. (c) Ka = ...(i)
[HF]
\ M1 = 0.1 V1 = 1
[HF][OH - ]
M2 = 0.01 V2 = ? Kb = ...(ii)
From M1V1 = M2V2 [F - ]
From (i) and (ii), KaKb = [H3O+][OH–] = Kw
0.1 × 1 = 0.01 × V2
(ionic product of water)
V2 = 10 litres 83. (b) [H3O]+ for a solution having pH = 3 is given by
\ Volume of water added = 10 – 1 = 9 litres [H3O]+ = 1×10–3 moles/litre [\ [H3O]+ = 10–pH]
77. (a) Molarity (M) = 10M. HCl is a strong acid and it is Similarly for solution having pH = 4,
completely dissociated in aqueous solutions as :
ˆˆ† H+(10) + Cl–. [H3O]+ = 1 × 10–4 moles/ litre and for pH = 5
HCl (10) ‡ˆˆ
[H3O+] = 1×10–5 moles/ litre
So, for every moles of HCl, there is one H+. Therefore,
[H+] = [HCl] or [H+] = 10. Let the volume of each solution in mixture be IL, then total
volume of mixture solution L = (1 + 1 + 1) L = 3L
pH = – log[H+] = – log [10] = – 1.
Total [H 3O] + ion present in mixture solution
4
78. (d) No. of moles of NaOH = = 0.1 = (10–3 + 10–4 + 10–5) moles
40 Then [H3O]+ ion concentration of mixture solution
[Molecular weight of NaOH = 40]
No. of moles of OH– = 0.1 =
10 -3 + 10 -4 + 10 -5 0.00111
M= M
0.1 3 3
Concentration of OH– = = 0.1mol / L = 0.00037 M = 3.7 × 10 M. –4
1 litre
84. (c) In aqueous solution BA(salt) hydrolyses to give
As we know that, [ H + ] [OH - ] = 10 -14
BA + H2O ‡ˆˆ ˆˆ† BOH + HA
\ [H+] = 10–13 (Q OH– = 10–1)
Base acid
79. (c) M1V1 = M2V2 Now pH is given by
1 × 0.10 = M2 × 100 1 1 1
M2 = 0.001 = 10–3 pH = pK w + pKa - pK b
2 2 2
BOH ‡ˆˆˆˆ† B+ + OH - substituting given values, we get
C 0 0
C (1-a ) Ca Ca 1
pH = (14 + 4.80 - 4.78) = 7.01
Ca ´ Ca 2
Kb = 85. (b) Kw depends upon temperature as it is an equilibrium
C (1 - a)
constant.
Kb = Ca2 (Q1 - a » 1) 86. (a) IVth group needs higher S2– ion concentration. In
presence of HCl, the dissociation of H2S decreases
a = Kb / C hence produces less amount of sulphide ions due to
Kb common ion effect, thus HCl decreases the solubility
[OH - ] = Ca = ´ C = K bC of H2S which is sufficient to precipitate IInd group
C radicals.
= 10-5 ´10- 3 = 10- 4 87. (c) Number of meq. of the acid = 0.04 × 100 = 4
\ pH + pOH = 14 Number of meq. of the base = 0.02 × 100 = 2
\ Number of meq. of the acid left on mixing = 4 – 2 = 2
\ pH = 14 – 4 = 10
Total volume of the solution = 200 mL
80. (d) Given Kb = 1.0 × 10–12 \ No. of meq of the acid present in 1000 mL of the solution
[BOH] = 0.01 M ; [OH–] = ? = 10
ˆˆ† B+ or No. of eq. of the acid in 1000 mL of the solution
BOH ‡ˆˆ + OH -
t=0 c 0 0 10
= = 0.01
teq C (1- x ) Cx Cx 1000
Since the acid is monobasic and completely ionises in
C2x2 Cx 2 0.01x 2
Kb = = Þ 1.0 × 10–12 = solution
C (1 - x ) (1 - x ) (1 - x ) 0.01 N HCl = 0.01 M HCl
On calculation, we get, x = 1.0 × 10–5 Thus [H + ] = 0.01
Now, [OH–] = Cx = 0.01 × 10–5 = 1 × 10–7mol L–1 \ pH = – log (0.01) = – (– 2) = 2
81. (d) [Cu(H2O)4]2+ + 4NH3 ‡ˆˆ ˆˆ† [Cu(NH ) ]2+ + 4H O 88. (a) Kw at 25°C = 1 × 10–14
3 4 2
involves lose and gain of electrons. H2O is coordinated to At 25ºC
Cu by donating electrons (LHS). It is then removed by
withdrawing electrons. Kw = [H+] [OH–] = 10–14
EBD_8350
534 CHEMISTRY

At 100°C (given) CH3COOH is weak acid, so in it [CH3COOH] is equal to


Kw = [H+] [OH–] = 55 × 10–14 initial concentration. Hence
Q for a neutral solution (3.4 ´10 -4 )(3.4 ´ 10 -4 )
1.7 ´10 -5 =
[H+] = [OH–] [CH 3 COOH ]
\ [H+]2 = 55 × 10–14 3.4 ´10 -4 ´ 3.4 ´10 -4
or [H+] = (55 × 10–14)1/2 [CH 3 COOH] = = 6.8 × 10–3M
1.7 ´ 10-5
Q pH = – log [H+] 95. (c) Given, CH3COOH ƒ CH3COO– + H+ ;
On taking log on both side
K a1 , = 1.5 × 10– 5 ....(i)
– log [H+] = –log (55 × 10–14)1/2
1 HCN ƒ H++ CN–; K a2 = 4.5 × 10–10
pH = – log 55 + 7 log10
2 or H+ + CN– ƒ HCN;
pH = – 0.87 + 7 = 6.13
1 1
89. (c) Higher the value of Ka lower will be the value of pKa Ka' 2 = = ...(ii)
Ka2 4.5 ´ 10–10
i.e., higher will be the acidic nature. Further since,
CN–, F– and NO2– are conjugate base of the acids HCN, HF \ From (i) and (ii), we find that the equilibrium constant
and HNO2 respectively hence, the correct order of base (Ka) for the reaction,
strength will be CN– + CH3COOH ƒ CH3COO– + HCN, is
F– < NO–2 < CN– 1.5×10 –5
1
K a = K a1 ´ K a' 2 = ´ 105 = 3.33 ´ 10 4
=
(Q stronger the acid weaker will be its conjugate base) 4.5×10 3 –10
wt. of solute per litre of solution 96. (c) For weak acid dissociation equilibria, degree of
90. (d) As, molarity, =
Mol. wt. of solute dissociation a is given as :
1000 Ka Ka
Molarity of H2O = mole/litre a= \ %a = 100
18 C C
H2O ƒ H+ + OH– [H + ][A - ] [H + ]ca [H + ]a
c (1 – a) ca ca Also, Ka = = =
[HA] c (1 - a) (1 - a)
ca2 a
Thus, K a = = ca2 = 1.8 × 10–14 log K a = log H + + log
1- a 1- a
91. (d) BF3 is an electron deficient molecule and thus, it can 1- a
accept a pair of electrons. or pK a = pH + log
a
In stannic chloride and stannous chloride, the Sn atom 1- a
has vacant d-orbitals. Sn can increase its covalency to 6 pK a - pH = log
by accepting pair of electrons. Thus SnCl2 and SnCl4 can a
1- a pK a - pH
also act as Lewis acids. = 10
92. (c) Dissociation of a weak acid in water : a
1
ˆˆ† H3O+ (aq) +
HA(aq) + H 2 O(l ) ‡ˆˆ A – (aq) or = 10pK a - pH + 1
a
acid base conjugate acid conjugate base 1
If HA is a stronger acid than H3O+, then HA will donate \ a=
pKa - pH
protons and not H3O+, and the solution will mainly contain [1 + 10 ]
A– and H3O+ ions. Hence, the equilibrium moves in the 100
direction of formation of weaker acid and weaker base. or % a =
[1 + 10 pK a –pH ]
Similarly, this is true for dissociation of a weak base also.
93. (c) Strong base has higher tendency to accept the proton. 97. (c) (ii) Bond energy being directly related to bond
Increasing order of base and hence the order of accepting strength; increases with increase in bond strength.
tendency of proton is (ii) It is not always ture.
98. (b) Given [NH3] = 0.3 M, [NH4+] = 0.2 M,
I- < HS- < NH3 < RNH 2
Kb = 1.8 × 10–5.
94. (c) CH3COOH ‡ˆˆ ˆˆ† CH3COO– + H+ [salt]
pOH = pKb + log
[CH3COO - ][H + ] [base]
Ka =
[CH 3COOH] [pKb = –log Kb; pKb = –log 1.8 × 10–5]
Given that, \ pKb = 4.74
[CH 3 COO - ] = [H + ] = 3.4 ´ 10 -4 M 0.2
pOH = 4.74 + log = 4.74 + 0.3010 – 0.4771 = 4.56
Ka for CH3COOH = 1.7 × 10–5 0.3
pH = 14 – 4.56 = 9.43
EQUILIBRIUM 535

appreciable change in its pH value. Like wise if few drops


é Salt ù
99. (d) pH = pKa+ log ê ú of NaOH are added, the OH – ions will combine with H+
ë Acid û ions to form unionised water molecule. Thus pH of solution
é Salt ù will remain constant.
log [H+] = log Ka – log ê ú
ë Acid û 107. (b) ˆˆ† Mg 2 + + 2 OH -
Mg(OH) 2 ‡ˆˆ
é Acid ù Ksp = [Mg++][OH–]2
log [H+] = log Ka + log ê ú
ë Salt û 1.0 × 10–11 = 10–3 × [OH–]2
é Acid ù -5 0.1
= 9 × 10-6 M 10-11
[H+] = Ka ê ú = 1.8 × 10 ´
0.2 [OH- ] = = 10-4
ë Salt û -3
10
100. (d) HNO2 is a weak acid and NaNO2 is salt of that weak
acid and strong base (NaOH). \ pOH = 4
ˆˆ† Cr 3+ (aq.) + 3OH - (aq.)
\ pH + pOH = 14
101. (b) Cr(OH)3 (s) ‡ˆˆ
s 3s \ pH = 10
(s) (3s)3 = Ksp 108. (d) pH or [H+] of a buffer does not change with dilution.
109. (b) Given pH = 12
27 s 4 = K sp
or [H+] = 10–12
1/ 4 1/ 4
æ K sp ö æ 1.6 ´ 10-30 ö Since, [H+] [OH–] = 10–14
s=ç =ç ÷
è 27 ÷ø è 27 ø 10-14
102. (b) Solubility product is the product of ionic \ [OH–] = = 10–2
concentration in a saturated solution of an electrolyte at a
10-12
given temperature. Ba(OH)2 Ba 2+ + 2OH
103. (c) Let binary electrolyte be AB s 2s
[OH–] = 10–2
ˆˆ† A+ + B -
AB ‡ˆˆ 2s = 10–2
s s s
10-2
Hence, solubility product of AB s=
2
Ksp = [A+] [B–] æ 10 -2 ö
3

S = [s.] [s] Þ s = S½ Ksp = 4s3 = 4´ç ÷ = 5 × 10–7


è 2 ø
104. (a) The pOH of a buffer consisting of NH3 (i.e., NH4OH)
and salt NH4Cl (salt) is given by the equation 110. (b) Ksp = [Fe3+].[3OH–]
[ 0.1] So molar solubility of Fe3+ = s and [3OH–] = 3s
[ammonia]
pOH = pKb + log = 5.0 + log Fe(OH)3 ƒ Fe 3+ + 3OH -
[salt] [1.0]
[s] [3s]
= 5.0 – log 10 = 5 – 1 = 4.0
\ pH = 14 – pOH = 14 – 4.0 = 10 1.0 ×10–38 = [s] [3s]3
é salt ù 1.0 ×10–38 = s4 × 27
105. (d) For acidic buffer pH = pKa + log ê ú
ë acid û 1.0 ´ 10-38
s4 =
27
[A - ] s4 = 3.703 × 10–40
or pH = pK a + log
[ HA ] 1

Given pKa = 4.5 and acid is 50% ionised. (


s = 3.703 ´ 10 - 40 ) 4 = 1.386 × 10–10

[HA] = [A–] (when acid is 50% ionised) 111. (a) Ksp of As2S3 is less than ZnS. In acid medium,
ionisation of H S is suppresed (common ion effect), thus,
\ pH = pKa + log 1 2
\ pH = pKa = 4.5 it an precipitate AS2S3 only.
pOH = 14 – pH = 14 – 4.5 = 9.5 112. (a) Given Na2CO3 = 1.0 × 10–4 M
106. (a) Lets take an example of an acidic buffer CH3COOH \ [CO32 – ] = 1.0 × 10–4 M
and CH3COONa. i.e., S = 1.0 × 10–4 M
CH3COOH ‡ˆˆ ˆˆ† CH COO – + H + ; At equilibrium
3
CH3COONa ‡ˆˆ ˆˆ† CH COO– + Na+ [Ba2+] [CO32 –] = Ksp of BaCO3
3
when few drops of HCl are added to this buffer, the H+ of K sp 5.1 ´ 10 -9
HCl immediatly combine with CH3COO– ions to form [Ba2+] = = = 5.1 × 10–5 M
undissociated acetic acid molecules. Thus there will be no [CO32 - ] 1.0 ´ 10 -4
EBD_8350
536 CHEMISTRY

113. (d) Total volume = 100 mL ˆˆ†


BOH (g) ‡ˆˆ B+ + OH–
1.0 C 0 0
[acid] = 10 mL ´ = 0.1
100 C (1 – a) Ca Ca
0.5 Ca ´ Ca
[salt] = 20 mL ´ = 0.1
Kb =
100
C(1 - a )
[ salt ] Kb = Ca2 (Q1 - a » 1)
pH of acidic buffer = pK a + log acid
[ ]
a = Kb / C
0.1
= 4.76 + log = 4.76
0.1 Kb
[OH - ] = Ca = ´ C = K bC
114. (b) AgBr ‡ˆˆˆˆ† Ag + + Br - C
Ksp = [Ag+] [Br–]
= 10-5 ´ 10-3 = 10 -4
For precipitation to occur \ pH + pOH = 14
Ionic product > Solubility product \ pH = 14 – 4 = 10
K sp 5 ´ 10 -13 28 6
[Br - ] = = = 10 -11 4. (0.592) Moles of N2 = = 1, Moles of H2 = = 3
+ 0.05 28 2
[Ag ]
i.e., precipitation just starts when 10–11 moles of KBr is 500 ´ 1
added to 1L AgNO3 solution Moles of H2SO4 required = = 0.5
1000
\ Number of moles of Br– needed from
Moles of NH3 neutralised by H2SO4 = 1.0
KBr = 10–11
\ Mass of KBr = 10–11 × 120 = 1.2 × 10–9 g
115. (c) Ksp = [Ag+] [Cl–] for AgCl Ag+ + Cl–
( 2NH 3 + H 2SO 4 ¾¾
® (NH 4 ) 2 SO 4 )
1.8 × 10–10 = [Ag+] [0.1] Hence 1 mole of NH3 by the reaction between N2 and H2.
[Ag+] = 1.8 × 10–9 M N2 + 3H2 ˆˆ† 2NH3
‡ˆˆ
initial conc. 1 3 0
Ksp = [Pb2+] [Cl–]2 for PbCl2 Pb2+ + 2Cl–
at equilibrium 1 – 0.5 3 – 0.5 × 3 1
1.7 × 10–5 = [Pb+2] [0.1]2 conc.
[Pb2+] = 1.7 × 10–3 M 1´ 1
Kc = = 0.592 mol–2 L2
Exercise - 2 0.5 ´ (1.5)3
5. (4) Hydrolysis of a salt is reverse reaction of acid base
1. (20) ˆˆ† CaO(s) + CO2(g)
CaCO3(s) ‡ˆˆ neutralization reaction.
K w 10-14
Kp = PCO = 0.82 atm ; \ Kh = = -7 = 10–7
2 Ka 10
Kh
n CO 2 =
PV
=
0.82× 20
= 0.2 mole [OH - ] = h = C × = C × Kh
RT 0.082×1000 C
Mole of CaCO3 dissociated = n CO 2 = 0.2 = 10-8 = 10- 4
Amount dissociated = 0.2 × 100 = 20 g Þ pOH– = –log [OH–]
= – log [10–4] = 4
ˆˆ† NH3 (g) + H 2S(g) 6. (6.93) C(s) + CO2(g) ¾® 2CO(g)
2. (0.75) NH 4HS(s) ‡ˆˆ
Apply law of mass action,
Initial 1.0 atm 0 atm
At eqm. 1.0 + x atm x atm (P )2 (10PCO2 ) 2
K P = CO or 63 =
Then 1.0 + x + x = 2x + 1.0 = 2.0 (given) PCO 2 PCO2
Þ x = 0.5 atm (Given KP = 63) and PCO = 10PCO2
p NH3 = 1.0 + 0.5 = 1.5 atm ; pH 2S = 0.5 atm 2
100(PCO 2 )2
or 63 = or 63 = 100 PCO2
2 2 PCO2
K= p NH3 ´ pH 2S = 1.5 ´ 0.5 atm = 0.75 atm
63
3. (10) M1V1 = M2V2 PCO 2 = = 0.63 atm
100
1 × 0.10 = M2 × 100 PCO = 10PCO2 = 10 × 0.63 = 6.3 atm
M2 = 0.001 = 10–3 Ptotal = PCO + PCO = 0.63 + 6.3 = 6.93 atm
2
EQUILIBRIUM 537

7. (6.5) HCOONH 4 is a salt of weak acid and weak base; 11. (4.76) Total volume = 100 mL

1 1 1 1.0
pH = pKw + pKa – pKb [acid] = 10 mL ´ = 0.1
2 2 2 100
0.5
\ pH =
1 1 1
´ 14 + ´ 3.8 - ´ 4.8 ; pH = 6.5
[salt] = 20 mL ´ = 0.1
100
2 2 2
8. (4) AB(g) ¾® A(g) + B(g) [ salt ]
pH of acidic buffer = pK a + log
Applying law of mass action [ acid ]
a2p 0.1
Kp = (given p = 1 atm)
1 - a2 = 4.76 + log
= 4.76
0.1
12. (0.134) ˆˆ† SO (g) + NO (g)
SO2 (g) + NO2 (g) ‡ˆˆ
a2 3
\ = 1.6 ´ 10-3 Þ a2 = 1.6 × 10–3 Initial moles 2 2 2 2
1- a at eqm. 2– x 2– x 2+x 2+x
(Q a < < < 1)
(Q Q p < K p )
Þ a = 1.6 ´ 10-3 Þ a = 0.04
Total no. of moles of gases at equilibrium
% age dissociation = 4%. = 8 + 2 = 10
9. ˆˆ† CH COONa + H O
(8.22) CH3COOH + NaOH ‡ˆˆ PSO3 × PNO
3 2
Let acid be = V mL Kp =
PSO2 × PNO2
V mL of 0.01 M CH3COOH will require V mL of 0.01 M
NaOH. But CH3COONa formed will make solution alkaline 2
due to hydrolysis. æ 2+ x ö
çè ´ P÷
10 ø
ˆˆ† CH COOH + NaOH
CH3COONa + H2O ‡ˆˆ Þ 25 =
3 2
æ2-x ö
0.01 çè ´ P÷
[CH3COONa] = = 0.005 M 10 ø
2
Using equation for pH of salt of weak acid and strong 2+ x
base. Þ 5= ; x = 1.33
2-x
pK a log C 4.74 log 0.005 2–x
pH = 7 + + =7+ + ´ Ptotal
2 2 2 2 Partial Pressure of NO2 =
10
= 8.22
10. (71) N2O4 (g) ˆˆ†
‡ˆˆ 2NO2 (g) 2 - 1.33
= ´2
Initial moles 1 0 10
Moles at eqm. (1 – a) 2a
0.67
(a = degree of dissociation) = ´ 2 = 0.134 atm
Total number of moles at equilibrium 10
= (1 – a) + 2a
= (1 + a) [CH3COO - ]
13. (5.0) pH = pKa + log
(1 – a) [CH 3COOH]
p N 2O 4 = ´P
(1 + a) pKa = – log (1.8 × 10–5) = 4.7447
2a [CH3COO–] = 2 × [(CH3COO)2Ba] = 0.2 M
p NO2 = ×P
(1+ a) [CH3COOH] = 0.1 M
2
æ 2a ö
2 çè (1+ a ) ø÷× P 0.2
(p NO 2 ) 4a 2 P pH = 4.7447 + log = 5.046 » 5.0
KP = = = 0.1
PN 2O4 æ1– aö 1 – a2
çè ÷ø ´ P 14. (99.9) [Ag + ] required for commencement of
1+ a
Given, KP = 2, P = 0.5 atm K sp (AgCl) 1.0 ´ 10 -10
precipitation of AgCl = - =
[Cl ] 0.1
4a 2 P 4a 2 ´ 0.5
\ KP = =
1 – a2 1– a 2 = 1.0 ´ 10 -9 M
a = 0.707 » 0.71 [Br–] remaining at this stage
\ Percentage dissociation = 0.71 × 100 = 71
EBD_8350
538 CHEMISTRY

ˆˆ† H + + S2 -
HS- ‡ˆˆ
K sp (AgBr) 1.0 ´ 10 -13
= = = 1.0 ´ 10 -4 M [ H + ][S2– ]
[Ag + ] 1.0 ´ 10 -9
K a2 = … (ii)
% of Br– remaining unprecipitated [ HS– ]
1.0 ´ 10-4 [ H + ]2 [S2– ]
= ´ 100 = 0.1 K a3 = … (iii)
0.1 [ H 2S]
% of Br– precipitated = 100 – 0.1 = 99.9
Hence, K a3 = K a1 ´ K a2
15. (0.05) K a (H 2S) = 4 ´ 10 -21 7. (c) According to GN Lewis, an acid is a species which
accepts an electron pair and base which donates an
[H + ]2 [S2 - ] 0.12 ´ [S2 - ] electron pair. Since BF3 is an electron deficient species,
= =
[H 2S] 0.1 hence, it is a Lewis acid.
8. (a) DG° = –RT ln K
Þ [S2 - ] = 4 ´ 10 -20 M At the stage of half completion of the reaction,
Ksp(CoS) = 2×10–21 = [Co2+] [S2–] [A] = [B] and K = [B]/[A]
= [Co2+] × 4 × 10–20 Þ [Co2+] = 0.05 M Therefore, K = 1
As we know that
Exercise - 3 –
DG = –RT ln K
1. (d) Relationship between Kc and Kp is given as : \ DG = 0
Kp = Kc (RT)Dn 9. (b) The order of boiling point of compounds given is
Water > acetone > ether
where, Dn = (number of moles of gaseous products) –
Lower the boiling point, higher is the vapour pressure of
(number of moles of gaseous reactants)
the solvent. Hence, the correct order of vapour pressure
For given reaction,
will be
ˆˆ† NH 3 (g) + HCl (g)
NH 4 Cl (s) ‡ˆˆ Water < acetone < ether
Dn = 2 – 0 = 2
1 1
2. (d) The relationship between DG° and K is : 10. (a) For the reaction, ˆˆ† HI ( g )
H 2 ( g ) + I 2 ( g ) ‡ˆˆ
DG° = –RT ln K 2 2
When G° > 0 it implies DG° must be positive, which will be [ HI ]
possible when lnK is negative i.e., K < 1. Kc = =5
3. (c) At the stage of equilibria physical processes does [ H 2 ]1/2[ I 2 ]1/2
not stop but forward and reverse process occur at the Thus, for the reaction,
same rate. 2HI(g) ƒ H2(g) + I2(g)
4. (b) For the given reaction, [ H 2 ][ I2 ]
ˆˆ† PCl3 + Cl2
PCl5 ‡ˆˆ Kc¢ =
At 500 K in a closed container, [ HI]2
[PCl5] = 0.8 × 10–3 mol L–1 2 2
æ 1 ö æ1ö 1
[PCl3] = 1.2 × 10–3 mol L–1 K¢c = ç
K ÷ = çè 5 ÷ø = 25 = 0.04
è cø
[Cl2] = 1.2 × 10–3 mol L–1 11. (d) Addition of inert gas at constant volume does not

Kc =
[ PCl3 ][Cl2 ] = (1.2 ´ 10-3 ) ´ (1.2 ´ 10-3 ) affect any equilibrium.
= 1.8 × 10–3 12. (a) For the given reaction,
[ PCl5 ] (0.8 ´10-3 )
ˆˆ†
[Co(H 2O)6 ]3+ ( aq ) + 4Cl - ( aq ) ‡ˆˆ
5. (b) With the increase in acidity or Ka value of the given Pink
acids pH decreases, hence the order of pH value of the [CoCl4 ]2- ( aq ) + 6H 2O ( l )
acids will be : Blue
hypochlorous acid > acetic acid > formic acid Equilibrium shifts to backward direction on cooling where
as on heating, it shifts to forward direction. Thus, reaction
(3.8 ´ 10-8 ) (1.74 ´10 -5 ) (1.8 ´ 10-4 ) is endothermic. i.e., DH > 0.
6. (a) For the reaction, 13. (c) pH of neutral water at 25°C = 7.0
ˆˆ† H + + HS-
H 2S ‡ˆˆ At 25°C, [H+] = [OH–] = 10–7 and
[ H + ][ HS – ] Kw = [H+] [OH – ] = 10–14
Ka1 = … (i)
[ H 2S] On heating, Kw increases, i.e., [H+] [OH–] > 10–14
For the reaction, As [H+] = [OH–]
EQUILIBRIUM 539

\ [H+]2 > 10–14. NaCl = Strong acid + Strong base


or, [H+] > 10–7 M ® neutral solution (pH » 7)
\ pH < 7 CuSO4 = Strong acid + weak base
® Acidic solution (pH < 7)
As temperature increases, pH of pure water decreases
hence, it will become less than 7 at 60°C. 23. (b) DG = – 2.303 RT log K
14. (c) When the concentration of weak base (NH4OH) is here K = [Ag+]2 [CO32–] = Ksp
higher than the strong acid (HCl) a mixture of weak base \ 63.3 × 10– 3 = – 2.303 × 8.314× 298 log Ksp
and its conjugate acid is obtained, which acts as basic
63.3 ´ 10- 3
buffer. \ log Ksp = - = - 11.09
5705.8
15. (d) AgCl, react with aqueous ammonia to form a complex,
[Ag(NH3)2]+ Cl–. \ Ksp = Antilog (– 11.09) = 8 × 10– 12
16. (a) Given, Ka = 1.74 × 10–5 1
24. (b) ˆˆ† SO3 (g)
SO 2 (g) + O 2 (g) ‡ˆˆ
Concentration CH3COOH (C) = 0.01 mol dm–3 2
KP = KC(RT)x
[H + ] = K a × C = 1.74 ´ 10-5 ´ 0.01 = 4.17 ´ 10 -4 where x = Dng = number of gaseous moles in product
pH = – log [H+] = – log (4.17 × 10–4) = 3.4
– number of gaseous moles in reactant
17. (c) Given Ka for CH3COOH = 1.8 × 10–5
æ 1ö 3 1
Kb for NH4OH = 1.8 × 10–5. = 1 - ç1 + ÷ = 1 - = -
è 2 ø 2 2
Ammonium acetate is a salt of weak acid and weak base.
For such salts 25. (b) Equilibrium constant for reaction:
1 [Product]
pH = 7 + ( pK a - pK b ) K = 1.6 × 1012 =
2 [Reactant]
éë - log1.8 ´ 10 -5 ùû - éë - log1.8 ´ 10 -5 ùû The value of K is very high so the system will contain
= 7+ mostly products at equilibrium.
2
4.74 - 4.74 26. (a) N2(g) + O2(g) 2NO(g)
= 7+ = 7.00
2 [NO]2
K=
18. (a) In the reaction N 2 (g) + 3H 2 ( g ) ‡ˆˆ ˆˆ† 2NH 3 ( g ) [N 2 ][O 2 ]
If the total pressure at which the equilibrium is established, 1 1
is increased without changing the temperature, K will N 2 + O2 ‡ˆˆ ˆˆ† NO
2 2
remain same but the equilibrium concentration will change.
[NO]
ˆˆ† éë Fe ( SCN ) ùû 2+ K¢ = = K1 2
19. (b) Fe3+ + SCN - ‡ˆˆ 12
[N 2 ] [O 2 ]12
(Red)
When oxalic acid is added to a solution containing iron 27. (b) DG° = –2.30RT log K
(III) nitrate and potassium thiocynate then, equilibrium because at equilibrium DG = 0
shifts towards backward direction and intensity of red 28. (a) g eq. of NaOH = 0.1 × V = 0.1V
colour decreases. g eq. of HCl = 0.01 × V = 0.01V
Oxalic acid reacts with Fe3+ ions to form a stable complex g eq. of NaOH > g eq. HCl
ion [Fe (C2O4)3]3–. Thus, decreasing the concentration of Hence resultant solution should be basic.
Fe3+ ions. M1V1 – M2V2 = MV
20. (d) Given reaction is exothermic reaction. Hence,
0.1V – 0.01V = MV
according to Le-Chatelier's principle low temperature
0.09
favours the forward reaction and on increasing pressure M= = 0.045 = 4.5 × 10–2
equilibrium will shift, towards lesser number of moles i.e., 2
forward direction. Now, pOH = – log [OH–] = – log 4.5 × 10–2 = 1.34
Q pH + pOH = 14
21. (a) In exothermic reactions on increasing temperature
\ pH = 14 – 1.34 = 12.65
value of Kp decreases
Trick: Only one option has pH > 7, thus, option (a) must
\ Kp > Kp¢ (Assuming T1 < T2) be the answer.
22. (c) Na2CO3 is a salt of strong base (NaOH) and weak 29. (c) Ag2CrO4
acid (H2CO3). On hydrolysis this salt will produce strongly Ksp = [Ag+]2 [Cr2O2– –12
4 ] = 1.1 × 10
basic solution. i.e., pH will be highest (pH > 7) for this
sotluion. Others are combination of 1.1 ´ 10 –12
KCl = Strong acid + Strong base [Ag + ] =
éCr2 O4–2 ù
® neutral solution (pH » 7) ë û
EBD_8350
540 CHEMISTRY

AgCl, Ksp = [Ag+] [Cl–] = 1.8 × 10–10 34. ˆˆ† M + + Y –


(b) MY ‡ˆˆ
s s
1.8 ´ 10 –10 KSP = s2 = 6.2 × 10–13
[Ag+] =
[Cl – ]
AgBr, Ksp = [Ag+] [Br–] = 5.0 × 10–13 s = 6.2 ´10-13 Þ s = 7.87 × 10–7 mol L–1
5.3 ´ 10 –13 ˆˆ† N3+ + 3Y –
NY3 ‡ˆˆ
[Ag + ] = Ksp = s × (3s)3 = 27s4 = 6.2 × 10–13
[Br – ]
AgI, Ksp = [Ag+] [I–] = 8.3 × 10–17 æ 6.2 ´ 10-13 ö
1/4

8.3 ´ 10 –17 s= ç ÷
[Ag + ] = è 27 ø
[I – ] s = 3.89 × 10 mol L–1
–4
– – –
If we take [Cr2O2– 4 ] = [Cl ] = [Br ] = [I ] = 1 \ molar solubility of NY3 is more than MY in water.
than maximum [Ag+] will be required in case of Ag2CrO4. 35. (b) The presence of large amount of KHSO4 will decrease
30. (a) Among the given acids, HClO4 is a very strong acid, ionisation of H2SO4 that result in lesser ionisation of nitric
completely dissociates acid and lesser formation of nitronium ion [NO2+]. Hence,
\ [HA] ® 0 in this case, hence cannot be used for acidic the rate of nitration will be slower.
buffer. 36. (a) Given,
31. (b) DG°NO(g) = 86.6 kJ/mol = 86600 J/mol A + B ‡ˆˆ ˆˆ† C + D
T = 298, KP = 1.6 × 1012 No. of moles initially 1 1 1 1
DG° = –RT ln KP At equilibrium 1–x 1–x 1+x 1+x
2
Given equation, æ 1+ xö
\ Kc = ç
è 1 - x ÷ø
= 100
ˆˆ† 2NO (g)
2NO(g) + O2 (g) ‡ˆˆ 2
1+ x
\ °
2DGNO – 2DG°NO = – R (298) ln (1.6 × 1012) \ = 10
2 1- x
° On solving, x = 0.81
2DGNO 2
– 2 × 86600 = – R (298) ln (1.6 × 1012)
°
[D]At eqm. = 1 + x = 1 + 0.81 = 1.81
2DGNO = 2 × 86600 – R (298) ln (1.6 × 1012)
[ NH3 ]2
2
° 1 ˆˆ† 2NH3 ; K1 =
2DGNO = [2 × 86600 – R(298) ln (1.6 × 1012] 37. (a) (i) N 2 + 3H 2 ‡ˆˆ
2 2 [ N 2 ][ H 2 ]3
= 0.5 [2 × 86600 – R (298) ln (1.6 × 1012)]
[ NO ]2
32. (d) DG° = 2494.2 J (ii) ˆˆ† 2NO; K 2 =
N 2 + O 2 ‡ˆˆ
2A ‡ˆˆˆˆ† B + C. [ N 2 ][ O2 ]
1 [H 2O]
R = 8.314 J/Kl mol. ˆˆ† H 2O; K3 =
(iii) H 2 + O2 ‡ˆˆ
1 1 2 [H 2 ][O2 ]1/2
[A] = , [B] = 2, [C] =
2 2 Applying (ii + 3 × iii – i) we will get
[ B ] [C ] 2 ´1 2 5 ˆˆˆK
†
= 2NH 3 + O 2 ‡ˆˆ ˆ 2NO + 3H 2 O;
Q= 2 2 =4 2
[ A] æ1ö
ç ÷
è2ø
[ H 2 O]3
DG° = – 2.303 RT log KC. K=
[ NO ]2
´
[ H 2 ] ´ [ O2 ]
3 3/2

2494.2 J = – 2.303 × (8.314 J/K mol) × (300 K) log KC [ N 2 ][ O2 ] [ NH3 ]2


2494.2 J [ N 2 ][ H 2 ]3
Þ log KC = – 3
K2 K3
2.303 ´ 8.314 J / K mol ´ 300 K \ K=
K1
Þ log KC = – 0.4341
38. (d) Max. pressure of CO2 = Pressure of CO2 at equilibrium
KC = 0.37
For reaction,
Q > KC.
SrCO3(s) SrO(s) + CO2(g)
Hence, the reacion proceeds in reverse direction.
33. (d) Clausius – Clapeyron's equation Kp = PCO2 = 1.6 atm = maximum pressure of CO2
volume of container at this stage.
d ln P DH v
=- nRT
dT RT 2 V= …(i)
P
EQUILIBRIUM 541

Since container is sealed and reaction was not earlier at 45. (b) CH3COOK is a salt of weak acid (CH3COOH) and
equilibrium. strong base (KOH)
\ n = constant.
CH3COOK + H2O ¾¾
® CH3COOH + KOH
PV 0.4 ´ 20 Weak acid Strong base
n= = …(ii)
RT RT Hence, nature of solution is basic
Put equation (ii) in equation (i) 46. (b) [H2S] = 0.10 M
[HCl] = 0.20 M ; [H+] = 0.2 M
é 0.4 ´ 20 ù RT
V= ê ú = 5L ˆˆ† HS– + H + ; K1 = 1.0 × 10–7
(i) H 2S ‡ˆˆ
ë RT û 1.6
ˆˆ† S2– + H + ; K2 = 1.2 × 10–13
(ii) HS – ‡ˆˆ
39. (c) ˆˆ† 2Ag + ( aq ) + C2 O42– ( aq )
Ag 2 C2 O4 (s) ‡ˆˆ ˆˆ† S2– + H + ; K1 × K2 = 1.2 × 10–20
(iii) H 2S ‡ˆˆ
2s s
Ksp = [Ag+]2 [C2O2– 1.2 ´ 10–20 ´ [H 2S]
4 ] Hence, [S2–] =
+ –4
[Ag ] = 2.2 × 10 M [H + ]2
Given that: 1.2 ´ 10–20 ´ 10–1
\ Concentration of C2O42– ions, = = 3 × 10–20 M
4 ´ 10 -2
-4
éë C2 O24 - ùû = 2.2 ´ 10 M = 1.1 ´ 10-4 M 47. (c)
2 1M + Final
Ba2+ ¾®
\ Ksp = (2.2 × 10–4)2 (1.1 × 10–4) = 5.324 × 10–12 Na2SO4 Solution

40. (a) Ammonia is a weak base and a salt containing its 50 mL 450 mL 500 mL
conjugate acid, the ammonium cation, such as NH4OH Concentration of SO42– in Ba2+ solution
functions as a buffer solution when they are present M1V1 = M2V2
together in a solution.
1 × 50 = M2 × 500
41. (b) Given, pKa (HA) = 3.2
pKb (BOH) = 3.4 1
M2 =
The salt (AB) given is a salt of weak acid and weak base. 10
Hence the pH can be calculated by the formula For just precipitation
1 1 1 1 I.P = Ksp
\ pH = 7 + pK a – pK b = 7 + (3.2) – (3.4) = 6.9
2 2 2 2 [Ba2+] [SO42–] = Ksp(BaSO4)

ˆˆ† X 2 (g); DH = - x kJ 1
42. (a) A2 (g) + B2 (g) ‡ˆˆ [Ba2+] × = 10–10
10
On increasing pressure, equilibrium shifts in a direction
[Ba2+] = 10–9 M in 500 mL solution
where number of moles decreases i.e., forward direction.
On decreasing temperature, equilibrium shifts in Thus, [Ba2+] in original solution (450 mL)
exothermic direction i.e., forward direction. Þ M1 × 450 = 10–9 × 500
So, high pressure and low temperature favours maximum [where M1 = molarity in original solution]
formation of product. 500
43. (a) Solubility of BaSO4 = 2.42 × 10–3 gL–1 M1 = ´ 10 –9 = 1.11 ´ 10 -9 M
450
2.42 ´ 10-3 48. (c) When a proton is removed from an acid, we obtain its
\s= = 1.038 ´ 10-5 mol L-1 conjugate base.
233
ˆˆ† H + + OH -
H 2 O ‡ˆˆ
Ksp = s2 = (1.038 × 10–5)2 = 1.08 × 10–10 mol2 L–2
ˆˆ† H + + F -
HF ‡ˆˆ
1
44. (c) Meq. of HCl = 75 ´ ´ 1 = 15
5 49. (a) ˆˆ† Ca 2 + + 2OH -
Ca(OH) 2 ‡ˆˆ
1 pH = 9, pOH = 14 – 9 = 5
Meq. of NaOH = 25 ´ ´ 1= 5
5 [OH–] = 10–5
Meq. of HCl in resulting solution = 10
10-5
10 1 [Ca 2+ ] =
Molarity of [H+] in resulting mixture = = 2
100 10
æ 10-5 ö -5 2
Ksp = [Ca2+] [OH–]2 =ç ÷ ´ (10 ) = 0.5 × 10–15
é1ù è 2 ø
pH = –log[H+] = –log ê ú = 1.0
ë 10 û
EBD_8350
542 CHEMISTRY

50. (c) Basic Buffer is made up of weak base & its salt. o
HCl + NH4OH ¾® NH4Cl + H2O So, D r G = - RT ln K eq
Initial 10 m mol 20 m mol 0
Final 0 10 m mol 10 m mol
D r G o = -8.314 J mol–1 K–1 × 300 K × ln (2 × 1013)
51. (c) Due to the resonance stabilisation of the conjugate 55. (10.60)
base, CH(CN)3 is the strongest acid amongst the given 9.8
compounds. MH2SO4 = = 10–3 M
98 ´ 100
4
MNaOH = = 10–3 M
40 ´ 100
After neutralisation [OH–] can be calculated as
º ¬¾® º (40 ´ 10-3 ) - (2 ´ 10 -3 ´ 10) 20
[OH–] = = × 10–3
º 50 50
2
[OH–] = × 10–3
5
º

¬¾® º ¬¾® º pOH = 3.397


pH = 14 – pOH = 14 – 3.397 = 10.603

º
Exercise - 4
The conjugate bases of CHBr3 and CHl3 are stabilised by
inductive effect of halogens. This is why, they are less 1. (c) PCl5(g) PCl3(g) + Cl2(g)
1–x x x
stable. Also, the conjugate base of CHCl3 involves back- Total moles after dissociation
bonding between 2p and 3p orbitals. 1–x+x+x=1+x
52. (b) m. mol of H2SO4 = 20 × 0.1 = 2 æ x ö
p = mole fraction of PCl3 × Total pressure = ç ÷P
m. mol of NH4OH = 30 × 0.2 = 6 PCl 3 è1+ x ø
H2SO4 + 2NH4OH ® (NH4)2 SO4 + 2H2O
Kc =
[ NO2 ]2
Initial 2 m mol 6 mmol 0 2. (b)
Final (2-2) (6 – 2 × 2) 2 m mol [ N 2O 4 ]
= 0 m mol = 2 m mol If [NO2] ® at y-axis
[NH4OH]left = 2 m mol [N2O4] ® at x-axis
[(NH4)2 SO4] = 2 m mol Þ y = Kc.x2
+
[NH 4 ] = 2 × 2 = 4 m mol Parabolic curve.
Total Volume = 30 + 20 = 50 mL 3. (c) CaCO3(s) ƒ CaO (s) + CO2(g)
K P = PCO2 = 1 atm (given)
é Salt ù 4 /50
pOH = pKb + log ê ú = 4.7 + log 8400 8400
ë Base û 2 /50 \ By ln Kp = 7 – Þ 0=7- [ln 1 = 0]
= 4.7 + log 2 = 5 T T
pH = 14 – pOH Þ pH = 14 – 5 = 9 8400
ÞT= = 1200 K = (1200 – 273) = 927 °C
7
53. (d) ˆˆ† Ni 2 + (aq) + 2OH - (aq)
Ni(OH)2 (s) ‡ˆˆ 4. (a) ˆˆ† NH3 (g) + H 2S(g)
NH 4 HS(s) ‡ˆˆ
Start 0.5 atm 0 atm
s s 2s At equib. 0.5 + x atm x atm
Then 0.5 + x + x = 2x + 0.5 = 0.84 (given)
® Na + (aq) + OH - (aq)
NaOH (aq) ¾¾ Þ x = 0.17 atm.
0.1 0.1 0.1 pNH3 = 0.5 + 0.17 = 0.67 atm ; pH 2S = 0.17 atm
Total (OH–) = 2s + 0.1 » 0.1
Ionic product = (Ni2+)(OH–)2 K = pNH3 ´ pH 2S = 0.67 × 0.17 = 0.1139 = 0.11 atm2
5. (a) ¾¾
Sb 2S3 (s) + 3H 2 (g) ¬¾
¾® 2Sb(s) + 3H 2S(g)
Þ 2 ´ 10-15 = s(0.1)2
0.01- a 0.01-3a 2a 3a
-13
Þ s = 2 ´ 10 M H2S + Pb2+ ¾® PbS + 2H+
no. of moles of PbS formed
54. (d) DG = DG o + RT ln Q
1.19
= = 0.005 mole = Moles of H2S
At equilibrium DG = 0, Q = Keq 238
EQUILIBRIUM 543

(a) is correct.
3a
[ H2S] = 250 = éê
0.005 ù
[H 2 ], (b) Value of Kp increases by increasing the temperature.
ë 250 úû Thus, forward reaction increases. So, the reaction is
0.01– 3a æ 0.01 - 0.005 ö 0.005 endothermic or DH is positive.
= =ç ÷ø =
250 è 250 250 (c) Unit of Kp = (atm)Dn = atm.
3 (d) By increasing the pressure, [N2O4] will increase.
[H 2S]3 æ 0.005 ö
Kc = Þ Kc = ç ÷ =1 11. (d) Ag (+aq ) + 2NH 3( aq ) [Ag(NH 3 ) 2 ](+aq )
[H 2 ]3 è 0.005 ø
1
6. (c) 2 AB3 (g) ƒ A2 (g) + 3B2 (g) Instability constant =
formation constant
Initial 8 0 0

At eqm.
8 - 2a a 3a
K r 2.0 ´ 10-2
1 1 1 = = = 2.0 ´ 10-8
Kf 1.0 ´ 106
Since volume = 1 dm3 = 1 L
12. (b) pH = 9; [H+] = 10–9; [OH–] = 10–5;
At eq. [A2] = 2 mol = a
[Mg2+] = 1 × 10–3; [Mg++][OH–]2 = 1 × 10–13
\ [AB3] = 8 – 2 × 2 = 4 M given Ksp of Mg(OH)2 = 8.9 × 10–12 which is more than 1 ×
[A2] = 2 M 10–13. Hence, Mg(OH)2 will not precipitate.
[B2] = 3 × 2 = 6 M -11 1/4
13. (c) (i) pH 2O = K 1/4
p = (8.1 ´ 10 ) = 3.0 ´ 10-3 atm

K=
[ B2 ]3 [ A2 ] = 6 ´ 6 ´ 6 ´ 2 = 27 mol2 L-2
(ii) pH 2O = (KP)1/5 = (3.2 × 10–9) = 2.0 × 10–2 atm
[ AB3 ]2 4´4

7. (b) 2NOBr (g) 2NO (g) + Br2 (g) (iii) pH 2O = (KP)1/10 = (1.0 × 10–30)1/10 = 1.0 × 10–3 atm
2P P Smaller is the equilibrium pH 2O , more effective will be the
x
9 9 lower hydrate or anhydrous salt as dehydrating agent.
Since total pressure is P.
This is because removal of water molecules is difficult, so
2P P 6P
So, x + + =P Þx= the hydrated salt is more stable. We can say that anhydraus
9 9 9 salt can easily absorb moisture to form hydrated salt.
2
æ 2P ö P Hence, Z2SO4.
2
p NO ´ pBr2 ç ÷ ´
è 9 ø 9 14. (c) DrG° = DGf (Products) – DGf (Reactants)
\ Kp = = 2
2
p NOBr æ 6P ö = 0 – 77.1 × 2 = – 154.2 kJ/mol
ç ÷
è 9 ø
Kp 1 [H + ]2 10 -6
\ = Q= = = 2 ´ 10 4
. [Ag + ]2 -10
P 81 PH 2 0.5 ´ (10 )
8. (a)
DnG = DrG° + RT ln Q
(a) The sample of ammonia obtained will be a mixture of
NH3, NT3, NH2T and NHT2. Thus, this mixture will be 8.314 ´ 300 ln (2 ´ 10 4 )
radioactive. DrG = –154.2 + = –129.5 kJ/mol
1000
(b) The position of equilibrium will not change by using
15. (c) The two Ksp values do not differ very much. So it is
isotopes.
a case of simultaneous equilibria, where the concentration
(c) The value of equilibrium constant also remains same.
of any species can not be neglected.
(d) Now the sample will contain tritium containing forms
of ammonia and hydrogen (H2, HT and T2). Thus, the K sp
[Sr 2+ ][F – ]2 SrF
2 7.9 ´ 10 -10
average molecular mass will increase. = = = 1.128
9. (b) For exothermic reaction, yield ­ as temperature ¯ [Sr 2+ ][CO32– ] K sp
SrCO
7.0 ´ 10 -10
2
i.e., T1 > T2 > T3 \ [F ] = 1.128 × 1.2 × 10–3 = 13.5 × 10–4
– 2

10. (a) Dn = 2 – 1 = 1 \ [F–] = (13.5 × 10–4)1/2 = 3.674 × 10–2 » 3.7 × 10–2 M.


That is, with the decrease of pressure, reaction shifts
towards right, i.e., proportions of NO2 increases. Statement
EBD_8350
544 CHEMISTRY

8 Redox Reactions
16. (d) OsO4
Exercise - 1
Let O.N. of Os be x then 1 × (x) + 4(–2) = 0
1. (c) Given reaction is oxidation reaction due to removal \ x=8
of electropositive element potassium from potassium
ferrocyanide. 17. (b) O.N. of iodine in I3- is –1/3
D 18. (c) CaOCl2 or Ca(OCl)Cl is the mixed salt of Ca(OH)2 with
2. (d) 2Na(s) + H 2 (g) ¾¾® 2NaH(s) HCl and HOCl.
With the careful application of the concept of 19. (c) Carbon has the maximum oxidation state of + 4,
electronegativity, we can find that sodium is oxidised and therefore carbon dioxide (CO2) cannot act as a reducing agent.
hydrogen is reduced. 20. (a) Let the oxidation state of S be x.
3. (b) Oxidation reaction (loss of 2e–)
S4O 62– Þ 4x – 12 = – 2 Þ 4x = 10 Þ x = 10/4 = 2.5
2– 21. (c) Oxidation number of oxygen in OF2 = + 2.
2Na(s) + S(s) ¾®(Na+)2S (s)
-1
Reduction (gain of 2e–)
In KO2 =
2
reaction
4. (b) 22. (c) K[Co(CO) 4 ]
5 (a) Sn2+ ® Sn4+ + 2e–. In this reaction Sn2+ change in Let O.N. of Co be x then
Sn4+ it is called an oxidation reaction. 1 × ( + 1) + x + 4 × (0) = 0
6. (b) Blue colour of the solution disappear due to for K for Co for CO
formation of Zn 2+. \ O.N. of Co is = –1
7. (d) NO3- ¾¾® N 2 H 4 So, for reduction of 1 mole of 23. (d) O.N. of Fe in (a), (b), (c) and (d) respectively are :
+5 -2 +3, +2, +2 and 0.
NO3– , number of electrons required is 7.
24. (c) O.N. of Mn in K2MnO4 is +6
25. (d) MnO 4– (O.S. of Mn +7); Cr (CN) 36– (O.S. of Cr +3),
8. (a) Oxidation/loss of 2e–
NiF62 – (O.S. of Ni +4) and CrO2Cl2 (O.S. of Cr +6)
Zn(s) + Cu2+(aq) Zn2+ (aq) + Cu(s)
26. (a) Ox. no. of Cr on both side is + 6.
Reduction/gain of 2e –
27. (b) Let the oxidation no. of Fe in Fe3O 4 = x
+ 2 -1 +1 + 6 - 2 + 2 +6 -2 +1 -1 \ 3x + (– 2 × 4) = 0 or 3x = 8
9. (c) Ba O 2 + H 2 S O 4 ¾¾ ® Ba S O 4 + H 2 O 2 8
\ x=
In this reaction, none of the elements undergoes a change in 3
oxidation number or valency. 28. (c) Let oxidation state of oxygen in OF2 = x
10. (d) Reduction potential of Cu(II) is greater than that of \ x + ( – 1 × 2) = 0
Zn(II) and Al(III) thus can be easily replaced by these \ x= +2
ions. Moreover, solution of copper is blue in color.
29. (b) In KMnO4 : Let O.N. of Mn be x
11. (d) Oxidation number of hydrogen when it is bonded to
Þ +1 + x + 4(–2) = 0 Þ x = +7
metals in binary compounds is –1.
In K2Cr2O7 : Let O.N. of Cr be x
12. (a) Let x = oxidation no. of Cr in K2Cr2O7.
Þ 2(1) + 2x + 7(–2) = 0 Þ x = +6
\ (2 × 1) + (2 × x) + 7 (– 2) = 0
30. (d) The element is Ti (At. no. 22). Electronic configuration
or 2 + 2x – 14 = 0 or x = + 6.
is 1s2, 2s2p6, 3s2p6d2, 4s2. the energy level of 3d and 4s is
13. (a) (i) Oxidation state of element in its free state is zero.
very close. It can have Ti4+ O.S.
(ii) Sum of oxidation states of all atoms in compound is zero.
31. (a) CuO Þ +2
O.N. of S in S8 = 0; O.N. of S in S2F2 = + 1;
MnO2 Þ +4
O.N. of S in H2S = –2;
HAuCl4 Þ +3
14. (d) (NH4)2 SO4 is split into ions. NH +4 . Let O.N. of N be Tl2O Þ +1
x then, 1 × (x) + 4 × (+1) = 1 \ x = –3 32. (c) Calculating the oxidation state of nitrogen in given
H +1 molecules;
+1 | Oxidation state of N in NH3 is
15. (c) H – C 22+- – Cl -1 O.N. of C is zero x + 3 × (+ 1) = 0 or x = – 3
| Oxidation state on N in NaNO3 is
Cl - 1 1 + x + 3 × (– 2) = 0 or x = + 5
REDOX REACTIONS 545

Oxidation state of N in NaN3 is 46. (c) O.N. of F is always –1.


1 47. (d) 8KMnO 4 + 3NH 3 ¾¾ ® 8MnO 2 + 3KNO 3 + 5KOH
+ 1 + 3x = 0 or x = –
3 +2H 2O
Oxidation state of N in Mg3N2 is
3 × 2 + 2x = 0 or x = –3 48. (c) On balancing the given reaction, we find
Thus, 3 molecules (i.e. NH3, NaN3 and Mg3N2) have 3Na2HAsO3 + NaBrO3 + 6HCl
nitrogen in negative oxidation state. ¾¾® 6NaCl + 3H3AsO4 + NaBr
33. (b) Oxidation no. of O are + 2, 0, – 1/2 and – 1 respectively. 49. (c)
0 +5 –1
34. (d) PO3–
4 = x + 4 (– 2) = – 3; x – 8 = – 3; x = + 5 ¾® ClO -3 + yCl- + 3H 2 O
x Cl 2+ 6OH –¾
SO 2–
4 = x + 4 (– 2) = – 2; x – 8 = – 2; x = + 6 change in oxidation number = – 1
on balancing the eq we get
Cr2 O72– = 2x + 7 (– 2) = – 2; 2x – 14 = – 2;
2x =12; x = + 6 3Cl 2 + 6OH - ¾¾ ¾ ® ClO 3– + 5Cl - + 3H 2 O
35. (b) HClO4 is stronger acid tham HClO3 because the anion 50. (b) H2O2 is strong oxidizing than I2, reduction potential
formed, ClO 4– is more stabilized due to 4 oxygen of H2O2 is greater than that of I2.
(electronegative) atoms than ClO–3 which has 3 oxygen 51. (b) On reaction with hot and concentrated alkali, a mixture
atoms. Negative charge is more delocalized in ClO4–. of chloride and chlorate is formed
36. (c) A reaction, in which a substance un dergoes D
-1 +5
simultaneous oxidation and reduction, is called 3Cl2 + 3 NaOH(excess) ¾¾® 5NaCl + NaClO3 + 3H 2O
disproportionation reaction. In these reactions, the same 52. (d) In (i) and (ii), both P and S are in highest oxidation
substance simultaneously acts as an oxidising agent and state. In (iii) and (iv) ; P has oxidation state of +4 which can
as a reducing agent. Here Cl undergoes simultaneous be oxidized to +5 state, while in case of NH3, nitrogen has
oxidation and reduction. oxidation state of –3 which can be oxidised.
0 –1 +1 53. (c) The redox reaction involve loss or gain of electron(s)
2KOH + Cl 2 ® KCl + KO C l + H 2O. i.e. change in oxidation state. Given reaction is not a redox
0 reaction as this reaction involves no change in oxidation
37. (d) 3B r2 + 6CO32– + 3H 2 O ® state of reactant or product.
54. (c) On balancing the given equations, we get
+5

5Br + Br O3– + 6HCO3– 2MnO 4– + 5C 2 O 42– + 16H + ¾¾
® 2Mn 2+
O.N. of Br2 changes from 0 to –1 and +5. +10CO 2 + 8H 2O
Hence, it is reduced as well as oxidised. So, x = 2, y = 5 & z = 16
38. (b) D
2H 2 O ¾¾® 2H 2 + O 2 55. (a) Given reaction is
IO3- + aI - + bH + ¾¾
® cH 2 O + dI 2
There is decomposition of H2O molecule into H2 and O2. st
I half reaction
39. (a) H+ ions are added to the expression on the
I - ¾¾
® I2 ...(i)
appropriate side so that the total ionic charges of
reactants and products become equal. –1 0 (oxidation)
40. (b) The balanced equation is IInd half reaction
IO 3- ¾¾
® I2 ...(ii)
2C2 H6 + 7O 2 ® 4CO2 + 6H 2O.
+5 0 (reduction)
Ratio of the coefficients of CO2 and H2O is 4 : 6 or 2 : 3. On balancing equation (ii) we have
41. (a) 2MnO 4– + 5H 2 O 2 + 6H + ® 2Mn 2+ + 5O 2 + 8H 2 O. 10e - + 2IO3- + 12H + ¾¾ ® I 2 + 6H 2 O ...(iii)
42. (b) Reduction Now, balance equation (i)
2I - ¾¾ ® I 2 + 2e - ....(iv)
2Fe3+ + Sn2+ ® 2Fe2+ + Sn4+ Multiply eqn (iv) by 5 and add it to eqn (iii), we get
2IO 3- + 10I - + 12H + ¾¾ ® 6I 2 + 6H 2 O
Oxidation
43. (b) or, IO3- + 5I - + 6H + ¾¾ ® 3I 2 + 3H 2 O
44. (b) Decomposition of calcium carbonate is not a redox Hence, a = 5, b = 6, c = 3, d = 3
reaction. 56. (b) If one uses HCl, HBr or HI, to make acidic medium for
45. (c) F2 being most electronegative element cannot KMnO4, then all the halide ion can be oxidized as the
exhibit any positive oxidation state. reduction potential of KMnO4 is very high in acidic
medium, while in case of H2SO4, sulphur is already in its
In ClO4– chlorine is present in its highest oxidation state
highest oxidation state, cannot be further oxidized.
i.e. +7. Therefore, it does not show disproportionation 57. (b) Phosphorus, sulphur and chlorine disproportionate
reaction. in the alkaline medium.
EBD_8350
546 CHEMISTRY

Pu(s) + 3OH–(aq) + 3H2O(l) ® PH3(g) + 3H2PO2– (aq), 70. (d) According to electrochemical series.
S8(s) + 12OH–(aq) ® 4S2–(aq) 2S2O32–(aq) + 6H2O(l), 71. (a) Standard electrode potential i.e. reduction potential
of A is minimum (–3.05V) i.e. its oxidation potential is
Cl2(g) + 2OH–(aq) ® ClO–(aq) + Cl–(aq) + H2O(l)
maximum which implies ‘A’ is most reactive chemically.
58. (c) The balanced equation : 72. (a) By convention E°H+/H = 0
2NH 3 + OCl – ¾¾
® N 2 H 4 + Cl – + H 2 O 2
73. (a) (i) Mn+ + ne– ƒ M, E° is very negative
59. (b) During disproportionation, same compound undergo
simultaneous oxidation reduction. This means that oxidation potential of M is very positive.
Oxidation Thus, M is a good reducing agent.
(ii) The standard reduction potential decreases from
- +1 -1 +5
chlorine to iodine or the oxidizing power also decreases.
3HO Cl ¾
¾® 2 HCl+ HClO 3
(iii) The ease of oxidation increases from F– to I– and thus
Reduction reducing power of hydrogen halides also increases from
60. (a) ClO3- ¾¾
® Cl02 hydrogen chloride to hydrogen iodide.
H+I– ® I2 (Eox is highest)
x - 6 = -1 x =0
x = +5 x = 0 ( x = oxidation number ) Exercise - 2
Molecular mass 1. (d) Following are the examples of redox reaction
Equivalent mass =
Oxidation number Red
84.45
= = 16.89 (a) CuO + H 2 ¾¾
® Cu + H 2 O
5
61. (a) Both Fe (II) and S (IV) in SO32 - can be oxidised to Fe (III) Oxi

and (SO4)2– respectively hence, (3/5)×0.5 =0.3 moles /litre. Oxi

é MnO-4 + 5e - + 8H + ® Mn 2 + + 4H 2O ù ´ 3 (b) Fe2 O3 + 3CO ¾¾


® 2Fe + 3CO2
ë û 5
Red
Fe2+ ¾¾ ® Fe3+ + le–
SO32- ¾¾ ® SO 24 - + 2e- Red

8 24 + (c) 2K + F2 ¾¾
® 2KF
MnO 4- + H + Fe2 + + SO32 -
5 5 Oxi
`
¾¾
® 3Mn 2+
+ 4H 2 O + Fe 3+
+ SO 24- Option (d) is not an example of redox reaction as there is
no change in oxidation number of any of the reactants.
62. (b) Strongest oxidising agent = Ag+/Ag(s) (E° is max.) 2. (d) Since the redox couple Ag+/Ag has the highest
Weakest oxidising agent = Mg2+/Mg(s) (E° is min.) positive value of Eo i.e., 0.80 V, therefore, Ag+ is the
63. (a) More is E°RP, more is the tendency to get itself reduced strongest oxidising agent.
or more is oxidising power. 3. (a) Oxidation number of hydrogen is not always +1. It is +1
64. (a) Since oxidation potential of Zn is highest hence in hydrogen halides, –1 in hydrides and zero in H2 molecule.
strongest reducing agent. 4. (b) Split NH4NO3 into two ions NH +4 and NO3- . Now
65. (a) Fe Al Br
0.77 –1.66 1.08 E°Red calculate the oxidation number of nitrogen in the two species.
+
–0.77 1.66 –1.08 E°Oxi Let, oxidation number of N in N H 4 is x.
Hence, reducing power Al > Fe2+ > Br– Þ x + (4 × 1) = +1 or x + 4 = + 1 or x = – 3
66. (d) More the negative reduction potential, more is the
Let, oxidation number of N in NO3- is y
tendency to lose electron. The reducing power increases
as the standard reduction potential becomes more and more Þ y + 3 × (– 2) = – 1 or y – 6 = – 1 or y = + 5
negative. 5. (d) Highest oxidation number of any transition element
Thus, Li is the strongest reducing agent as the standard is the sum of (n – 1) d-electrons and ns electrons. Hence,
reduction potential of Li+/ Li is most negative, –3.05 V. large the number of electrons in the 3d-orbitals, higher is
67. (a) Higher the value of reduction potential higher will be the maximum oxidation number.
the oxidising power whereas the lower the value of 6. (d) Given that E° values of
reduction potential higher will be the reducing power. Br2/Br– = + 1.09 V
68. (c) The redox couple with maximum reduction potential Ag+ / Ag(s) = + 0.80 V
will be best oxidising agent and with minimum reduction
Cu2+/Cu(s) = + 0.34V
potential will be best reducing agent.
69. (a) Lower the value of reduction potential higher will be I2 (s) / I– = + 0.54 V
reducing power hence the correct order will be The E° values show that copper will reduce Br 2, if the E°
Mn2+ < Cl– < Cr3+ < Cr of the following redox reaction is positive.
The electrode reactions will be
REDOX REACTIONS 547

13. (b) Reaction balance by oxidation number method


Cu ® Cu 2 + + 2e - ; E° = -0.34V +7 +2
- - ® Mn2+; 5e – gain
Mn O 4– ¾¾ ...(i)
Br2 + 2e ® 2Br ; E° = +1.09 V +3 +4
Cu + Br2 ® CuBr2 ; E° = + 0.75 V C2O 42– ¾¾ ® CO2 ; 2e - loss ...(ii)

Multiplying (i) by 2 and (ii) by 5 to balance e–
Since, E of the overall reaction is positive, therefore, Cu 2+
2 MnO 4– + 5 C 2 O 2–
4 ¾¾ ® 2 Mn + 10 CO2
can reduce Br2.
7. (d) On balancing charge;
(a) 2Fe3+ + 2e– ® 2Fe2+; E° = + 0.77 V 2 MnO 4– + 5 C2 O 42– + 16 H + ¾¾
®
2I– ® I2 + 2e–; E° = – 0.54 V 2 Mn 2 + + 10 CO 2 + 8 H 2 O
2Fe3+ + 2I– ® 2Fe2+ + I2; E°cell = + 0.23 V
14. (c) (i) [Cr(H 2 O)6 ]Cl3 :
(b) Cu ® Cu2+ + 2e–; E° = – 0.34 V
2Ag+ + 2e– ® 2Ag; E° = + 0.80 V x + 6 × 0 + (–1) × 3 = 0
Cu + 2Ag+ ® Cu2+ +2Ag; E° = + 0.46 V x =+3
(c) 2Fe3+ + 2e– ® 2Fe2+ ; E° = + 0.77V (ii) [Cr(C6H6)2]:
Cu ® Cu2+ + 2e– ; E° = – 0.34 V y+2×0=0
2Fe3+ + Cu ® 2Fe2+ + Cu2+ ; E° = + 0.43V y=0
(d) Ag ® Ag+ + e–; E° = – 0.80 V (iii) K2[Cr(CN)2(O)2 (O2)(NH3)] :
Fe3+ + e– ® Fe2+; E° = + 0.77 V 2 × 1 + z + 2 × (–1) + 2 × (–2) + 2 × (–1) + 0 = 0
Ag + Fe3+ ® Ag+ + Fe2+; E° = – 0.03 V z = +6
The oxidation states of Cr in given compounds are +3, 0
This reaction is not feasible since E°cell is negative.
and + 6 respectively.
+2 0 2.5 15. (a) In a disproportionation reaction, one species
8. (a) 2 S 2 O32- ( aq ) + I 2 ( s ) ® S4 O62 - ( aq ) + 2I - ( aq ) undergoes both oxidation and reduction.
+2 0
S2 O32 - ( aq ) + 2 Br2 ( l ) + 5H 2 O ( l ) ¾¾
® Reduction
+6
2S O 24- ( aq ) + 4Br - ( aq ) + 10H + ( aq ) 0

Hence, bromine is a stronger oxidising agent than I2, as it 2 Cu+ ¾¾¾¾® Cu2++ Cu
Oxidation
oxidises S of S2 O32 - to SO 24 - whereas I2 oxidises it only
into S4 O62 - ion. Reduction
9. (a) Calculating the oxidation number of central atom in +6 2– + +7 – +4
the given species. 3MnO4 + 4H ¾® 2MnO4 + MnO2 + 2H2O
+3 +5 +6 +7 Oxidation
Cr O -2 , Cl O 3- , Cr O 42- , MnO 4-
-4 +4
10. (d) Reactions in which the same substance undergo 16. (b) CH 4 (g) + 4Cl 2 (g) ¾¾
® CCl 4 (l ) + 4HCl (g)
oxidation as well as reduction are called disproportionation
reactions. Writing the O.N. of each element above its Change in oxidation state of carbon is –4 to +4.
symbol in the given reactions. 17. (b) K2O : 2x – 2 = 0 Þ x = +1
In reaction (d), N is both oxidised as well as reduced since K2O2 : 2x – 2 = 0 Þ x = +1
the O. N. of N increases from +4 in NO2 to +5 in NO3- and KO2 : x – 1 = 0 Þ x = +1
decreases from +4 in NO2 to +3 in NO2- . Thus, potassium shows +1 state in all its oxides,
superoxides and peroxides.
oxidation

+4 +3 +5
Exercise - 3
NO2 + 2OH - ¾¾
® NO-2 + NO-3 + H2O 1. (d)
reduction
I. Cs ¾¾ ® Cs+ + e- (Oxidation) ù
11. (c) Fluorine is the strongest oxidising agent. It does not ú
show positive oxidation state hence, it does n ot - – 1
( ) ú
H2O + e ¾¾ ® OH + H2 Reduction ú
disproportionate. 2 û
12. (a) In the reaction Hence, redox reaction.

e - + Cu 2+ ¾¾ ® Cu+ ( Reduction )ù
Oxidation
II. ú
+4 +1 +6 0
2I ¾¾ -
® I 2 + 2e (Oxidation) ú
XeF4 + O2F2 ® XeF6 + O2 û
Hence, redox reaction.
Reduction
EBD_8350
548 CHEMISTRY

12. (c) SO2 is an oxidising agent.


III. NH +4 ¾¾
® NH3 (No change in oxidation number)
4e– + SO2 ¾¾ ® S
x =-3 x = -3
Hence, reaction is not a redox reaction. 64
Equivalent of SO2 = = 16.
IV. No change in the oxidation number of either Fe2+ or 4
CN– in both reactant and product; hence, reaction is not a 13. (b) Reaction II is disproportionation, while I, III and IV
redox reaction. are not.
2. (d) II, III and IV are redox reactions. -3 +5 +1
D
+6 +6 (I) N H 4 N O3 ¾¾
® N2 O + H2O
3. (c) CrO 24- ¾¾
® Cr2O72- comproportionation
Since, oxidation state of Cr in both reactant and product 0 -3 +2
D
is same. (II) P4 ¾¾® P H3 + H P O2– disproportionation
4. (d) Intermolecular redox reaction involves oxidation of +5 +3
D
(III) P Cl ¾¾
one molecule and reduction of the other molecule like in 5 ® P Cl3 + Cl 2 reduction
redox reactions.
-1 0
+3 +5 +5 +2 (IV) H 2 O 2 ¾¾ ® O 2 + 2e - oxidisation
As 2 S3 + HNO3 ¾¾
® H3As O4 + H 2SO4 + NO
14. (c) (a) and (b) are disproportionation reactions.
5. (a) Oxidation state of C in CO32– is +4, which is maximum.
(c) is an oxidation reaction.
So, it will not be oxidised.
-1´2 g 2H - ¾¾ ® H 2 + 2e -
6. (c) Ba 2+ ( H 2 PO 2 )2 : H 2 PO 2- : 2+ x - 4 = -1
+2 +1 Al(OEt)
C — H ¾¾¾®
–1 3
Þ x = +1 (a) 2
\ Oxidation state of P = + 1 ||
7. (d) Zinc gives H2 gas with dil H2SO4/HCl but not with O–2
+1
HNO3 because in HNO3, NO3– ion is reduced and give
NH4NO3, N2O, NO and NO2 (based upon the concentration +1
of HNO3) H
+2 –1 +1 |
8. (d) C+1 — O — C–1 –1

|| –1 |
( )
+2
é +3 0
1- ù O–2 H+1
(a) êë Co N H3 5 Cl úû Cl 2 -1 ´ 2
+3 –1
NH3 : x + 3 = 0 Þ x = –3; Oxidation State of N = –3
(b) NH2OH : x + 3 – 2 = 0 Þ x = –1
Oxidation state of N = –1 (b) –2
O
(c) (N2H5)2+1 × 2SO42– : ||
+2 –1 +1 +1
C—H
N2H5+ : 2x + 5 = + 1 Þ x = – 2 | + OH – ¾®
+2 +1 – 1
Oxidation number of N = –2 C —O—H
||
+2×3 –3´2 –2 O +3
(d) Mg 3 N 2 : Oxidation state of N = –3

( )
–2
+1 H +1 O
9. (c) (a) NH 4 2 S2 O82– –1
+1 | –1 –1 || +3
H —–1C+1— O — H +2 C +1— O–
\ Oxidation state of S = + 6 | + | –1
+2 +1 – +2 –
(Since S2 O82– has one peroxide bond) C —O C+1— O
|| –1 || – 1 +3
(b) Oxidation state of Os = +8 +3 –2 O –2 O

(c) Oxidation state of S in H 2SO5 = +6 15. (d) Since reaction is occuring in basic medium, therefore,
(Since it has one peroxide bond) 2OH– are added on right side.
(d) K1+ O21–, oxidation state of O = –1/2 2MnO -4 (aq) + Br– (aq) ¾¾ ®
10. (a) S2O62– : 2x – 12 = –2 Þ x = 5 2MnO2(s) + BrO–3 (aq) + 2OH–(aq)
SO32– : x – 6 = –2 Þ x = 4 Now, hydrogen atoms can be balanced by adding one
H2O molecule to the left side
S2O42– : 2x – 8 = –2 Þ x = 3
2MnO 4- (aq) + Br - (aq) + H 2 O(l) ¾¾
®
11. (c) SO42– cannot be oxidised since the oxidation state
(+6) of S is highest. 2MnO2(s) + BrO3- (aq) + 2 OH - (aq)
9 Hydrogen

16. (d) Very pure hydrogen can be prepared by the action of


Exercise - 1
water on sodium hydride.
1. (d) Hydrogen is a non-metal while all other members of NaH + H2O ¾¾ ® NaOH + H2
group 1 (alkali metals) are metals. cobalt
2. (c) In metal hydrides the O.S. of hydrogen –1 otherwise 17. (b) CO(g) + 2H2(g) ¾¾¾¾ ® CH3OH(l)
catalyst
it is +1. 18. (c) Hydrogen is not used in the synthesis of
3. (a) Number of neutrons in protium, deuterium and hydroquinone and tartaric acid.
tritium respectively is = 0, 1 and 2 19. (d) H2 will not reduce heated Al2O3. As Al is more electro-
4. (b) The only pollutant in combustion of dihydrogen is positive than hydrogen. therefore, its oxide will not be
oxides of dinitrogen (due to the presence of dinitrogen reduced by hydrogen.
as impurity with dihydrogen). 20. (c) Hydrogenation occurs in presence of nickel as a
5. (d) It is for the first time in the history of India that a pilot catalyst.
project using dihydrogen as fuel was launched in October 21. (c) Mixture of CO and H2 is used in synthesis of
2005 for running automobiles. Initially 5% H2 has been methanol and a number of hydrocarbons due to this
mixed in CNG for use in four wheeler vehicles. reason it is also called syn gas.
6. (a) Due to extremely small size of H+ as compared to 22. (b) Electron deficient hydride = B2H6
normal atomic and ionic size H+ does not exist freely. Electron precise = CH4
7. (c) Hydrogen behaves as a non-metal under normal Electron rich = NH3 and HF
gaseous and liquid states. However, at very high pressure 23. (d)
of about 2 million bars, it becomes conducting in nature 24. (c) Fire due to action of water on saline hybrides
and behaves as a metal. cannot be extinguished with water or CO2. These
8. (d) Zn+ 2NaOH ¾¾ ® Na2ZnO2 + H2
hybrides can reduce CO2 at high temperature to produce
(Sodium zincate) O2.
25. (d) Due to its covalent nature MgH2 is Polymeric in
9. (c) Fe + dil. H 2SO 4 ® FeSO 4 + H 2 ­ nature.
26. (c) Ionic hydrides give the basic solution when it reacts
3Fe + 4H 2 O ® Fe 3O 4 + 4H 2 ­
Steam
with water, e.g.,
Cu + dil. HCl ® No reaction LiH + H2O ¾® LiOH + H2
27. (d) d- and f-block elements form metallic hydride.
Copper does not evolve H2 from acid as it is below
hydrogen in electrochemical series. While p-block elements form covalent hydrides, s-block
elements except Be and Mg form ionic hydrides. Hydrides
2Na + 2C 2 H5OH ® 2C 2 H5ONa + H 2 ­
of Be, Mg, Cu, Zn, Ca and Hg are intermediate hydride.
catalyst
10. (b) CO + H 2 + H 2O ¾¾¾® CO 2 + 2H2 28. (b) H - (aq) + H 2 O(l) ¾¾
® OH - (aq) + H 2 (g)
base 1 acid 1 base 2 acid2
water gas
In this reactionH– acts as bronsted base as it accepts one
11. (c) Except method given in statement (iii) all other are
proton (H+) from H2O and form H2.
commercial methods for production of dihydrogen.
29. (c) Tanks of metal alloy like NaNi5, Ti – TiH2, Mg – MgH2
12. (d) Carbon dioxide formed in water gas shift reaction is
etc., are used for storage of dihydrogen in small
removed by scrubbing with sodium arsenite solution. quantities.
13. (a) Mg + dil. HNO 3 ® Mg( NO 3 ) 2 + H 2 (Mg and Mn 30. (c) The unusual properties of water in the condensed
give H2 with dil. HNO3) phase (liquid and solid states) are due to the presence of
14. (a) Zinc has no action on cold water. extensive hydrogen bonding between the water molecules.
15. (a) Cu + 4HNO3(conc.) ¾® Cu(NO3)2 + 2NO2 + 2H2O 31. (b) In the structure of ice, each molecule of H2O is
surrounded by three H2O molecules in hexagonal honey
® C2 H 5 O - Na + + 1 / 2H 2 ­
C 2 H 5 OH + Na ¾¾ comb manner which results an open cage like structure.
As a result, there are a number of 'hole' or open spaces. In
Mg + 2H 2 O(steam) ¾¾
® Mg(OH) 2 + H 2 ­ such a structure, lesser number of molecules are packed
per ml. When ice melts, a large no. of hydrogen bonds are
® C6 H 5 O - Na + + 1 / 2H 2 ­
C6 H 5 OH + Na ¾¾
broken. The molecules therefore move into the holes or
EBD_8350
550 CHEMISTRY

open spaces and come closer to each other than they were 50. (d) H2O2 does not have basic properties.
in solid state. This results in increase in the density. 51. (d) Although H2O2 is a better polar solvent than H2O.
Therefore, ice has lower density than water. However, it cannot be used as such because of the strong
32. (b) Two ice cubes stick to each other due to H-bonding. auto-oxidation ability.
33. (b) Temporary hardness is due to presence of 52. (a)
bicarbonates of calcium and magnesium and permanent 53. (d) MnO2 , PbO2 and BaO will not give H2O2 with HCl.
hardness is due to the sulphates and chlorides of both of MnO2 and PbO2 will give Cl2 and BaO will react with HCl
calcium and magnesium. to give BaCl2 and water.
34. (a) 2C17H35COONa(aq) + M2+ (aq) ¾¾ ® 54. (d) H2O2 is not used as a moderator in nuclear reactors.
(C17H35COO)2M¯ (M = Ca or Mg) + 2Na+(aq) 55. (d) H2O2 show all these properties.
35. (b) Due to high dielectric constant, water acts as a good 56. (d) Dihedral angle of H2O2 in gas phase is 111.5° and
solvent therefore it is also called a universal solvent. in solid phase it is 90.2°
electrolysis
36. (c) Polyphosphates (sodium hexametaphosphates, 57. (b) 2HSO4– (aq) ¾¾¾¾¾ ® HO3SOOSO3H(aq)
sodium tripolyphosphate or STPP) form soluble complexes
® 2HSO -4 (aq) + 2H+(aq) + H2O2(aq)
Hydrolysis
¾¾¾¾¾
with Ca2+, Mg2+ present in hard water.
58. (a) As H2O2 is loosing electrons so it is acting as reducing
37. (b) When calgon added to hard water, following reactions agent.
takes place: 59. (d) I 2 (s) + H 2 O 2 (aq) + 2OH - (aq) ¾¾®
Na2 [Na4 (PO3)6] ® 2Na+ + [Na4 (PO3)6]2–
2I - (aq) + 2H 2 O(l) + O 2 (g)
Ca2+ + [Na4 (PO3)6]–2 ® [Na2Ca (PO3)6]2– + 2Na+
60. (a) CaC2 + 2D 2 O ® C 2 D 2 + Ca(OD) 2
This is an example of exchange of ions. 61. (b) D2O is used in nuclear reactors as moderator.
38. (d) Many salts can be crystallised as hydrated salts from 62. (b) Al4C3 + D2O ® 3CD4 + 4Al(OD)3
an aqueous solutions such an association of water is of 63. (d) The usual compounds with dentirium atom in place
different types viz., of hydrogen.
(i) Coordinated water e.g., [Cr(H2O)6]3+ 3Cl– 64. (d) H2O absorbs neutrons more than D2O and this
(ii) Interstitial water e.g., BaCl2 . 2H2O decreases the number of neutrons for the fission process.
(iii) Hydrogen-bonded water 65. (a) D2O actually has higher freezing point (3.8°C) than
e.g., [Cu(H2O)4]4+ SO2– 4. H2O in CuSO4.5H2O water H2O (0°C)
39. (b)
40. (c) Assertion is correct reason is incorrect. Exercise - 2
Correct reason : Calgon forms soluble complexes with Ca2+
and Mg2+ in which properties of these ions are marked. 1. (b) Hydrogen like halogens accept an electron readily to
41. (a) The complex salt of metaphosphoric acid sodium achieve nearest inert gas configuration.
hexametaphosphate (NaPO3)6, is known as calgon. It is 2. (d) Electron-precise hydrides are the type of hydrides in
represented as Na2[Na4(PO3)6] which the number of electrons present is equal to the
42. (a) H2SO4 + BaO2 ® BaSO4 + H2O2 number of electrons required.
43. (c) 30 vol of H2O2 means one volume of H2O 2 on 3. (c) The radioactive isotope of hydrogen is tritium. Its
decomposition will give 30 volume of oxygen. half life is about 12.32 years.
Reduction
vol. strength
44. (a) Normality of H2O2 =
®

5.6 -1 -1 0 -2
Volume of normal (1N) H2O2 solution = 5.6 volumes. 4. (b) (i) H 2 O2 + 2HI ¾¾
® I2 + 2H 2 O
®

\ Volume of strength of 1.5 N H2O2 Oxidation

= 1.5 × 5.6 = 8.4 volumes. Here, H2O2 oxidises HI into I2 hence, behaves as an
68 ´ 10 oxidising agent.
45. (b) Strength of 10V H2O2 = ´ 100 = 3.035%
22400 Reduction
®

H +1 -1 0
+ -
46. (b) O–O is the true structure of H2O2. (ii) HOCl + H 2 O 2 ¾¾
® H 3O + Cl + O 2
®

H Oxidation
47. (b) H 2 O2 + [O] ¾Oxidation
¾¾¾ ¾® H 2 O + O 2 ­ Here, H2O2 reduces HOCl to Cl, thus, behaves as reducing
48. (b) The value of dipole moment of H2O2 is 2.1 D, which agent.
suggest the structure of H2O2 cannot be planar. 5. (b) Oxides such as BaO2, Na 2O2 etc; which contain
An open-book structure is suggested for H2O2 in which peroxide linkage on treatment with dilute H2SO4 give H2O2
O – H bonds lie in different plane. but dioxides such as PbO2, MnO2, TiO2 do not give H2O2
49. (c) H2S is oxidised to S by H2O2. on treatment with dilute H2SO4.
HYDROGEN 551

16. (c) The production of syn gas or synthesis gas from


BaO2 .8H 2 O ( s ) + H 2SO4 ( aq ) ¾¾
®
Hydrated barium
coal is called coal gasification.
peroxide
C ( s ) + H 2 O ( g ) ¾¾¾¾ ® CO ( g ) + H 2 ( g )
1270 K
BaSO 4 (s) + H 2 O 2 ( aq ) + 8H 2O ( l ) Coal Steam Ni 1442443
Hydrogen Syn gas
peroxide In order to increase the production of hydrogen, steam is
Reduction added to carbon monoxide of the syn gas in the presence
of iron chromate as a catalyst at 673 K.
-1 0 -2
6. (c) 2I - + 2H + + H 2 O 2 ¾ ¾
® I2 + 2H 2 O
CO ( g ) + H 2 O ( g ) ¾¾¾¾¾¾® CO 2 ( g ) + H 2 ( g )
4 FaCrO . 673 K

®
Oxidation
CO2 is removed by scrubbing with a solution of sodium
Here H2O2 oxidises I– to I2, hence behaves as oxidising arsenite.
agent.
17. (c) Sodium hexametaphosphate also known as Calgon
7. (b) The given below reaction show the reducing action
is used.
in basic medium.
Reduction 2CaCl 2 + Na 6 P6O18 or Na 2 éë Na 4 ( PO3 ) 6 ùû ®

I20 +
–1
H 2 O2 + 2OH - ¾ ¾
® 2I- + 2H 2 O + O02 ( From hard
water ) Sodium
hexametaphosphate

Oxidation
Na 2 éë Ca 2 ( PO3 )6 ùû + 4NaCl
8. (c) Hydrogen peroxide acts as an oxidising as well as Complex Salt
reducing agent in both acidic and alkaline media.
18. (a) Only one element of group 6, i.e., Cr forms hydride.
9. (d) Na 2 O 2 + dil. H 2SO 4 ¾¾ ® Na 2SO 4 + H 2 O 2
Oxidation
10. (b) Hydrogen peroxide is manufactured by electrolysis
of 50% sulphuric acid followed by distillation. The distillate +1 –1 0, –2 0
19. (c) (i) H2O2 + O3 ¾® H2O + 2O2
is 30% hydrogen peroxide. The first product of electrolysis
is peroxy disulphuric acid.
Reduction
2H2SO4 ----® 2H+ + HSO4–

® H 2S2 O8 ( aq ) + 2e -
Oxidation
2HSO4- ( aq ) ¾¾¾¾¾
Electrolysis

H2S2O8 (aq) + 2H2O (l) ---® 2H2SO4(aq) + H2O2(aq) +1 –1 +1, –2 0 +1 –2 0


(ii) H2O2 + Ag2O ¾® 2Ag + H2O + O2
11. (d) The water gas is a combination of carbon and
hydrogen. It is used in manufacturing of methanol.
Reduction
CO ( g ) + 2H 2 ( g ) ¾¾¾¾
Cobalt
® CH3OH ( l)
Catalyst Hence, in both the reactions, H2O2 is acting as an reducing
12. (a) Bicarbonates, chlorides and sulphates of Ca and Mg agent.
are responsible for the hardness of water. 20. (d) The reducing agent loses electron during redox
13. (a) Elements of group 7, 8 and 9 of d-block do not form reaction i.e. oxidises itself.
hydrides at all. The inability of metals of group 7, 8 and 9 of –1 –2
periodic table to form hydrides is referred to as hydrides gap (i) H 2O 2 + 2H + + 2e - ¾¾
® 2H 2O (Re d.)
of d-block. In these compounds, H atoms are supposed to –1 0
occupy interstitial position in the metal lattices. They are (ii) ® O2 + 2H + + 2e- (Ox.)
H 2O 2 ¾¾
also called non-stoichiometric hydrides. –1 –2
14. (d) H+ ion always get associated with other atoms or (iii) H 2O 2 + 2e - ¾¾
® 2OH - (Re d.)
–1 0
molecules because loss of an electron from hydrogen atom -
® O2 + H 2O + 2e - (Ox.)
(iv) H 2O 2 + 2OH ¾¾
results in a nucleus of very small size as compared to other
atoms or ion. Due to small size it cannot exist free. 21. (c) H2O2 has oxidizing and reducing properties both.
15. (b) The stability of hydrides decreases from Li to Cs. It 22. (a, d)Among the three isotopes of hydrogen, Protium (1H1)
is because of the fact that M-H bond becomes weaker is most common. It is an energetic reducing agent. It reduces
due to increase in size of alkali metals down the group. oxides, chlorides and sulphides of certain metals and
Hence, ionic character increases as the size of the atom produce free metals at ordinary temperature.
increases i.e., the correct order of increasing ionic 23. (a) There is extensive intermolecular hydrogen bonding
character is in the condensed phase instead of intramolecular
LiH < NaH < KH < RbH < CsH H-bonding.
EBD_8350
552 CHEMISTRY

+2
24. (c) (i) [Fe (CN)6 ]4 - + H 2 O 2 W2 ´ 1000 34 ´ 1000 200
9. (a) N= = =
E.w 2 ´ VmL 17 ´ 1120 12
H+
+3 1 N of H2O2 = 5.6 vol strength
[Fe (CN)6 ]3- + H 2 O
+3 200 200
(ii) [Fe (CN)6 ]3- + H 2O 2 N of H 2 O2 = 5.6 ´ = 10 vol strength
12 12
OH– 10. (d) N of H2O2 = Volume strength/5.6 = x/5.6
+2 mEq. of H2O2 = mEq. of MnO4–
[Fe(CN)6 ]4 - + O2 + H 2O
\ N1V1 º N2V2
25. (b)
x 1
26. (c) Hydrogen has three isotopes: ´ 10 mL = ´ 10 mL
5.6 0.56
Protium (1H1), deuterium (1H2) and tritium (1H3).
\ x = '10' volume strength
27. (c) In CuSO4 × 5H2O, four H2O molecules are directly
11. (c) 1 N = 5.6 vol
coordinated to the central metal ion while one H2O
molecule is hydrogen bonded. Þ Normalities of two solutions are 1 N and 2 N
28. (b) Synthetic resin method is more efficient than zeolite 1´1 + 1´ 2 3
Normality of mixture = = = 1.5 N
process as it can exchange both cations as well as anions. 1 +1 2
3 16.8
Exercise - 3 Volume strength = ´ 5.6 = = 8.4 vol
2 2
12. (d) H2O2 º I2
1. (a) Zn + H2SO4 ® ZnSO4 + H2
Zn + 2NaOH ® Na2ZnO2 + H2 2KI + H2O2 + H2SO4 ® K2SO4 + 2H2O + I2
\ Ratio of volumes of H2 evolved is 1 : 1 mEq of H2O2 = 5 × N
Weight
2. (b) 2H 2SO 4 ® 2H + + 2HSO -4 mEq of I2 = ´ 1000
E.w
At cathode : 2H + + 2e - ® H 2­ 1.27
´ 1000 = 10 mEq
At anode : 2HSO -4 ¾¾
® H 2S2 O 8 + 2e - 127
\ mEq of H2O2 = mEq of I2
Peroxy disulphuric acid
3. (a) A fusion reaction between hydrogen nuclei is difficult 5 × N = 10 \ N = 2
because positively charged nuclei repel each other. 1 N of H2O2 = 1.7% of H2O2
However, at very high temperatures of the order of 106 to 2 N of H2O2 = 3.4% H2O2
107 K, the nuclei may have sufficient energy to overcome 13. (a) Volume of O2 at STP = 100 mL× 11.2 mL volume strength
the repulsive forces and thus fuse. This is why, fusion = 1120 mL of O2 at STP
reactions are also called thermonuclear reactions.
Since 1 N = 5.6 volume strength of H2O2
4. (a) 2 Na + 2H 2 O ® 2 NaOH + H 2 ­
2 N = 11.2 volume strength of H2O2
'A ' 'C' ' B'
Volume of O2 produced by H2O2 = 1120 mL
Zn + 2 NaOH ® Na 2 ZnO2 + H 2 ­
'D ' 'C' 'B'
Same volume of O2 will be produced by KMnO4 i.e. 1120 mL.
Total volume of O2 produced = 2240 mL = 2.24 L
Zn + dil. H 2SO 4 ® ZnSO 4 + H 2 ­
'D ' 'B'
14. (b) MnO4– + 5e– ® Mn2+ (acidic)
Na produces golden yellow colour with smokeless flame MnO4 + 3e ® MnO2
– – (basic)
of Bunsen burner. 100 mL H2O2 º 100 × 5N MnO4– º V × 3N MnO2
5. (a) Pure para form can be obtained at low temperature 500
(20K) but pure ortho form is very difficult to obtain. V= mL
3
6. (a) As Na+ ions in sodium zeolite are replaced by Ca2+
and Mg2+ ions present in hard water, these two ions will 15. (b) mEq of MnO4– º mEq of H2O2
not be present. M
7. (a) Soap contains salt of sodium such as sodium stearate 10 ´ ´ 5 º mEq of H2O2
5
which reacts with calcium and magnesium ions present in Þ 10 mEq º mEq of H2O2
hard water. Thus, calcium and magnesium salts precipitate out.
34
8. (b) H - (aq)+ H 2 O(l) ¾¾ ® OH - (aq)+ H 2 (g) Weight of H2O2 = 10 × 10–3 × = 0.17 g
base 1 acid 1 base 2 acid2 2
In this reaction H– acts as Bronsted base as it accepts one 100
Weight of impure H2O2 = ´ 0.17 = 0.2 g
proton (H+) from H2O and forms H2. 85
THE s-BLOCK ELEMENTS 553

10 The s-Block Elements


Na+ < K+ < Rb+ < Cs+
Excercise - 1
Hence, the lattice energy of NaF will be maximum.
1. (c) Francium is highly radioactive. 18. (c) As Cs+ ion has larger size than Li+ and I– has larger
2. (b) Alkali metals readily lose electron to give size than F–, therefore maximum distance between centres
monovalent M+ ion. Hence they are never found in free state of cations and anions is in CsI.
in nature. 19. (a) Larger cation (K+) develops less polarisation in anion
3. (a) Na ® Na+ + e– ; IE of Na = + ve and thus KI has more ionic nature and more soluble in water.
Na+ + e– ® Na ; E.A. of Na+ = – ve 20. (c) Na2CO3 is more stable than Li2CO3.
Both are equal but opposite in nature 21. (c) Because KO2 not only provides O2 but also removes
4. (a) Electrons can easily excite due to low I.E. CO2 as follows
5. (b) Sodium metal on burning in air gives sodium peroxide.
6. (c) All the alkali metals when heated with oxygen form 4KO2 + 2CO2 ® 2K 2 CO3 + 3O2
different types of oxides for example lithium forms lithium 4KO 2 + 4CO 2 + 2H 2 O ® KHCO3 + 3O 2
oxide (Li2O), sodium forms sodium peroxide (Na2O2), while 22. (d) LiI is more soluble as the degree of covalent character
K, Rb and Cs form their respective superoxides. is high due to larger size of anion i.e., iodide ion by greater
1 polarization of lithium cation.
2Li + O2 ® Li 2O
2 23. (b) During the dissolution of alkali metal hydroxides,
7. (d) Li, Na, K are lighter than water but Rb is heavier than energy is released in large amount, i.e., it is exothermic in
water. nature.
8. (d) Smaller the size of cation higher is its hydration energy 24. (d) (a) and (b) forms corresponding hydroxides (NaOH
and lesser is its ionic mobility hence the correct order is Li + and LiOH) in aqueous solution
< Na+ < K+ < Rb+
M 2 O + H 2O ¾¾ ® 2M + + 2OH - (M = Na or Li)
9. (d) The basic character and stability of hydrides decrease
down the group. Therefore, reaction of HI with (a), (b) and (c) is simply a
10. (a) For a given metal, order of stability of halides is neutralization reaction, while aqueous solution of (d) form
MF > MCl > MBr > MI H2O2 which act as oxidizing agent, hence convert Iodide
11. (a) A reducing agent is a substance which can loose to Iodine( I2).
electron and hence, a reducing agent should have low Na 2O 2 + 2H 2 O ¾¾ ® 2Na + + 2OH - + H 2 O 2
ionisation energy. Now since ionisation energy decreases 25. (a) 4LiNO3 ® 2Li 2 O + 4NO 2 + O 2
from Li to Cs, the reducing property should increase from
Li to Cs. The only exception to this is lithium. This is 26. (a) Lithium carbonate is unstable to heat; lithium being
because the net process of converting an atom to an ion very small in size polarises a large CO32 - ion leading to
takes place in 3 steps. the formation of more stable Li2O and CO2.
(i) M(s) ® M(g) DH = Sublimation energy 27. (c) 4LiNO3 ® 2LiO + 4NO2 + O2
(ii) M(g) ® M+(g) + e– DH = Ionisation energy 2 NaNO3 ® 2NaNO2 + O2
(iii) M+(g)+H2O ® M+(aq) DH = Hydration energy 28. (c) Li due to highest hydration energy among the alkali
The large amount of energy liberated in hydration of Li metals is the strongest reducing agent.
(because of its small size) makes the overall DH negative. 29. (c) The atom becomes larger on descending the group,
This accounts for the higher oxidation potential of lithium so the bonds become weaker (metallic bond), the cohesive
i.e., its high reducing power. force/energy decreases and accordingly melting point also
12. (d) Lithium with lead is used to make white metal. decreases.
13. (b) Liquid sodium is used as a coolant in fast breeder 30. (a) Li+ has maximum degree of hydration among other
nuclear reactors. alkali metals.
14. (d) Amongst alkali metal, Li ions are highly hydrated. 31. (b) Actually Li is harder than other alkali metals.
15. (a) All the given elements react with oxygen to form 32. (b) Lithium although has most negative E d- value
oxides but only Li also react with nitrogen to form Li3N. reacts less vigorously with water than other alkali metals
16. (a) Since the ionization energies of alkali metals decrease due to its small size and very high hydration energy.
down the group, the ionic character and consequently basic 33. (a) The ionisation potential value of lithium is maximum
property of their hydroxides increases in the same order, among alkali metals i.e., its tendency to ionise to give Li+
i.e. from LiOH to CsOH. ions should be the minimum i.e. Li should be the poorest
17. (c) With the same anion, smaller the size of the cation, reducing agent. But, lithium is the strongest reducing agent
higher is the lattice energy. The correct order of size of
in aq. solution. This is due to the largest value of hydration
cations is –
energy of Li+ ions.
EBD_8350
554 CHEMISTRY

I
34. (c) NH 4HCO3 + NaCl ¾¾
® NaHCO3 ¯ + NH 4Cl 11 Na ¾® 1s2 , 2 s2 2 p 6 , 3s1 ¾
¾ ¾®
Brine Sod.bicarbonate ppt. II
1s 2 , 2 s 2 2 p 6 , 3s 0 ¾¾®
35. (a) NH3 is recovered when the solution containing
NH4Cl is treated with Ca(OH)2 1s 2 , 2s 2 2 p5 , 3s 0
2NH4Cl + Ca(OH)2 ® 2NH3 + CaCl2 + H2O I
36. (b) Na metal is manufactured by electrolysis of fused NaCl 12 Mg ¾® 1s 2 , 2s 2 2 p6 , 3s 2 ¾
¾ ¾®
by two methods. II
1s2 , 2s2 2 p6 , 3s1 ¾¾® 1s 2 , 2s 2 2 p6 , 3s 0
(i) Castner's process
Here Na-I < Mg-I and Na-II > Mg-II.
(ii) Down's process
55. (c) As we go from group I element to group II element in
In both the above processes electrolysis of fused sodium
a period, an extra electron is added in same shell which
chloride produces Na at cathode. results in increase in nuclear charge due to which force of
37. (a) Baking powder has starch, NaHCO3 and Ca(H2PO2)2. attraction by the nucleus increases and hence ionic radii
38. (a) CO2 and NH3 formed are reused (See Solvay process) decreases and consequently I.E. increases.
39. (b) KHCO3 is more soluble than NaHCO3. 56. (a) Be(OH)2 is amphoteric while Ca(OH)2, Sr(OH)2 and
40. (d) Sodium sulphate, calcium chloride and magnesium Ba(OH)2 are all basic.
chloride are present as impurities in crude sodium 57. (a) Because of small atomic size and high I.E. Be forms
chloride. covalent chloride.
41. (c) NaCl (brine), NH3 and CO2 are raw materials. CaCO3 58. (a) As we move down the group, the lattice energies of
is source of CO2. carbonates remain approximately the same. However, the
42. (a) hydration energies of the metal cation decreases from Be2+
43. (c) Within a period, the size decreases from left to right, i.e., to Ba2+, hence the solubilities of carbonates of the alkaline
Na > Mg > Li > Be. Atomic size increases down the group. earth metal decrease down the group mainly due to
44. (d) Value of I.E. decreases down the group. decreasing hydration energies of the cations from Be2+ to
45. (d) 46. (d) Ba2+.
59. (c) Calcium carbonate on thermal decomposition gives
47. (d) Barium is most electropostive element among those
CaO (Basic oxide) and CO2 (Acidic oxide)
given. Hence it is most reactive.
D
48. (d) Because of larger size and smaller nuclear charge, alkali CaCO3 ¾® CaO + CO2­
metals have low ionization potential relative to alkaline Basic oxide Acidic oxide
earth metals. 60. (c) 2NaO2 + 2H2O ® 2NaOH + H2O2 + O2
49. (a) 61. (d) For a compoud to be soluble, the hydration energy
50. (b) 2Na + 2H 2 O ¾¾® 2NaOH + H 2 ­ must exceed lattice energy. For Gp.II hydroxides, lattice
energy decreases more rapidly than the hydration energy
2K + 2H 2O ¾¾® 2KOH + H 2 ­
and so their solubility increases down the group. Ba(OH)2>
All alkali metals decompose water with the evolution of
Sr(OH)2 > Ca(OH)2 > Mg(OH)2
hydrogen.
62. (a) Carbonates increases with increasing cationic size of
Ca + 2H 2 O ¾¾® C a(OH) 2 + H 2 ­ group 2 elements. The thermal stability of further, group 1
carbonates are more thermally stable than group 2
Sr + 2H 2O ¾¾® Sr(OH) 2 + H 2 ­
therefore, the order of thermal stability is
Be + 2H2O ¾¾® No reaction BeCO3 < MgCO3< CaCO3< K2CO3.
Ca, Sr, Ba and Ra decompose cold water readily with 63. (a) Lattice energy decreases more rapidly than hydration
evolution of hydrogen. Mg decomposes boiling water but energy for alkaline earth metal hydroxides. On moving down
Be is not attacked by water even at high temperatures as a group solubility of their hydroxides increases.
its oxidation potential is lower than the other members. 64. (d) Down the group solubility of sulphate decreases.
51. (b) Electrons in Mg due to its small size are tightly bound Thus, Ba2+ ions will precipitate out most easily.
so they cannot be excited by the flame. 65. (d) All alkaline earth metal oxides except BeO are basic
52. (d) All the given statements are correct. in nature. BeO is amphoteric in nature.
53. (d) K and Mg, both form oxides Hydrated halides of Ca, Sr and Ba on heating undergo
dehydration while corresponding hydrated halides of Be
K + O2 ® KO 2 ; 2Mg + O 2 ® 2MgO and Mg on heating suffer hydrolysis.
Mg form nitride also 3Mg + N 2 ® Mg3 N 2 66. (a) Mg burns in CO2 to give MgO and C.
67. (d) BeF2 is highly soluble in water due to the high
K does not form nitride. hydration enthalpy of the small Be2+ ion.
54. (d) The IInd ionisation potential of Na is higher than Mg 68. (d) Be(OH)2 is amphoteric, but the hydroxides of other
because it requires more energy to remove an electron alkaline earth metals are basic. The basic strength increases
from a saturated shell or stable (fully filled) orbital. gradually.
THE s-BLOCK ELEMENTS 555

69. (a) Calcium gives brick red colour and barium gives 2. (a) The reactivity of alkali metals with water increases
apple green colour in flame test. on moving down the group from Li to Cs due to increase
70. (a) Both are amphoteric in nature. in electropositive character.
71. (a) Because of high value of ionisation enthalpy and small 3. (a) The basic character of the given hydroxides increases
size, it forms compounds which are highly covalent in down the group due to decrease in ionization enthalpies.
nature, hence, they get hydrolysed easily. On moving down the group, the ionic size increases, so
72. (b) The Be(OH)2 and Al(OH)3 are amphoteric in nature. M–O bond in metal hydroxides gets weaker and easily
73. (a) Gypsum is CaSO4.2H2O gives OH– in the solution and I.E. further decreases.
1 Hence, Mg(OH)2 will be least basic. The order of basicity
74. (d) Plaster of Paris (CaSO4 . H2O) on making paste with
2 will be:
little water sets to a hard mass due to formation of gypsum Mg(OH)2 < Ca(OH)2 < Sr(OH)2 < Ba(OH)2
(CaSO4.2H2O). 4. (a) Ethanol is a covalent compound and only a compound
1 3 which has more covalent character can be dissolved in it.
CaSO4 . H2O + H2O ® CaSO4.2H2O + Heat Also the group ionic character increases and beryllium
2 2
Plaster of Paris Gypsum halide have covalent character due to its small size and
high effective nuclear charge. Hence, among the given
75. (b) CaCO 3 + 2HCl ¾¾ ® 2CaCl 2 + H 2 O + CO 2 halides, BeCl2 can be soluble in ethanol.
76. (a) During setting of cement, silicates and aluminates of 5. (c) On moving down the group from Li to Rb, increased
calcium are hydrated. Hydration is an exothermic process. atomic radii makes the removal of electron easier.
Therefore, setting of cement is exothermic process. Thus, the order of decreasing ionization enthalpy will be:
77. (c)
Li > Na > K > Rb
78. (d) When cold calcium hydroxide reacts with chlorine,
Also, LiF exhibit very high lattice energy.
then bleaching powder is obtained.
6. (a) Be(OH)2 is an amphoteric hydroxide thus gets
3Ca(OH) 2 + 2Cl 2 dissolve both in acids and alkalies.
slaked lim e
¾¾
® Ca(OCl)2 .Ca(OH) 2 .CaCl 2 .2H 2 O Basic nature :
Bleaching powder Be ( OH ) 2 + 2HCl ¾¾ ® BeCl 2 + 2H 2 O
79. (d) 80. (c) 81. (c)
82. (c) There are four chief minerals present in a Portland Acidic nature:
cement: tricalcium silicate (Ca3SiO5), dicalcium silicate Be ( OH ) 2 + 2NaOH ¾¾ ® Na 2 BeO 2 + 2H 2 O
(Ca2SiO4), tricalcium aluminate (Ca3Al2O6) and calcium Sodium beryllate
alumino-ferrite (Ca4AlnFe2-nO7). 7. (a) Alkali metals dissolve in liquid NH3 giving deep blue
83. (b) Quick lime is used for the manufacture of dyestuffs. coloured solution.
Plaster of Paris is used for setting of fractured bones. Na + ( x + y ) NH 3 ¾¾
®
Slaked lime is used for the manufacture of bleaching +
powder. éë Na ( NH3 ) x ùû + e - ( NH3 ) y
Limestone is a constituent of chewing gum. Ammoniated cation Ammoniated electron
84. (d) For a good quality cement, the ratio of silica (SiO2) When light fall on the solution, the ammoniated electrons
to alumina (Al2O3) should be between 2.5 and 4 and the get excite to higher energy level by absorbing certain
ratio of lime (CaO) to the total of the oxides of silica wavelength and transmit blue colour.
(SiO2), aluminia (Al2O3) and iron (Fe2O3) should be as 8. (a) On heating plaster of Paris at certain temperature, it
close as possible to 2. forms anhydrous calcium sulphate which is known to be
85. (c) Radium is used in treatment of cancer. dead burnt plaster and has no setting property as it
86. (c) Monovalent sodium and potassium ions and divalent absorbs water very slowly.
magnesium and calcium ions are found in large 1 200°C 1100° C
proportions in biological fluids. CaSO 4 . H 2O ¾¾¾¾ ® CaSO4 ¾¾¾¾ ® CaO + SO3
2 ( Anhydrous)
87. (b) 88. (a)
89. (b) Potassium ions are the most abundant cations 9. (c) Slaked lime [Ca(OH)2] is sparingly soluble in water
within cell fluids. and it forms a suspension of slaked lime in water which is
90. (a) The calcium concentration in plasma is regulated at called milk of lime. A clear solution obtained after the
about 100 mgL–1. It is maintained by two hormones : suspension settles is known as lime water.
calcitonin and parathyroid hormone. 10. (d) On heating washing soda, it loses its water of
crystallisation. Above 373 K, it becomes completely
Exercise - 2 anhydrous white powder called soda ash.
Above 373K
1. (d) Alkali metals have low melting and boiling points. Na 2CO3.10H 2O ¾¾¾¾¾¾
® Na 2CO3
The melting point of alkali metals decreases from Li to Cs Washing soda Soda ash
as cohesive force decreases with increase in atomic size. ( Anhydrous form)
EBD_8350
556 CHEMISTRY

11. (a) Ca(OH)2 is used in the manufacture of bleaching 18. (b) Gypsum (CaSO4.2H2O) is added to cement to increase
powder. its setting time so that it gets sufficiently hardened. Setting
Cold
2Ca ( OH ) 2 + 2Cl2 ¾¾¾ ® CaCl2 + Ca(OCl) 2 + 2H 2 O of cement is an exothermic process and involves hydration
Slaked Bleaching of calcium aluminates and silicates.
lime powder
19. (a) Except Be, all alkaline earth metals form hydrides
12. (c) To recover NH3 in Solvay process, Ca(OH)2 is used. (MH2) on directly heating with H2. BeH2 can't be prepared
On passing CO2 through Ca(OH)2, it turns milky due to
by direct heating but it can be prepared by the action of
the formation of CaCO3. Also, Ca(OH)2 is used for white
LiAlH4 on BeCl2.
washing due to its disinfectant nature.
Ca(OH)2 + CO2 ® CaCO3¯+ H2O 2BeCl2 + LiAlH 4 ® 2BeH 2 + LiCl + AlCl3
13. (c) Reducing character is measured by tendency of an 20. (b) Calcium gives brick red coloured flame. Also, calcium
element to lose electron in aqueous solution. Higher the nitrate on heating decomposes into calcium oxide and
° value, greater is the ability to lose electrons.
negative E RP evolution of a mixture of NO2 and O2 takes place.
° depends on: 2Ca(NO3)2 ® 2CaO + 4NO2 + O2
E RP
(i) enthalpy of sublimation brown coloured gas
(ii) ionisation of enthalpy 21. (d) Chlorides of alkaline earth metals are hydrated salts.
(iii) enthalpy of hydration Due to their hygroscopic nature, they can be used as a
Thus, in aqueous medium, order of reactivity of alkali dehydrating agent and can absorb moisture from air.
° value of Li is least (–
metals is Na < K < Rb < Cs < Li. E RP Extent of hydration decreases from Ca to Ba.
3.04 V) among all alkali metals. 22. (b) K+ ion is a constituent of sodium pump.
The formation of Li+ (aq) from Li involves following steps: 23. (d) Solubility of alkaline earth metal sulphates decreases
Sublimation down the group due to decrease in hydration energy.
(i) Li(s) ¾¾¾¾¾® Li(g);
DHs = Enthalpy of sublimation MgSO 4 > CaSO4 > SrSO 4 > BaSO 4
- Hydration Solubility
-e
(ii) Li(g) ¾¾¾ ® Li+ (g); IE1 = Ionisation enthalpy energy

+ 24. (c) Carbonates becomes more thermally stable down the


(iii) Li (g) ¾¾ ® Li + (aq); DHn = Enthalpy of hydration group, therefore MgCO3 will leave CO2 easily.
For alkali metals, enthalpies of sublimation are almost same. 25. (d) Solubility order
IE 1 value of Li is positive & highest and hydration BeSO 4 > MgSO 4 > CaSO 4 > SrSO 4 > BaSO 4
enthalpy is negative and maximum for smallest Li+ which 26. (c)
makes it strongest reducing agent. 27. (b) On heating with excess of air Li, Na and K forms
14. (d) BaCO3 is thermally most stable due to the small size following oxides
of resulting oxide ion upon decomposition. As the atomic 4 Li + O2 ¾® 2Li2O Lithium monoxide
number increases the stability of the metal oxide decreases
575 K
hence, stability of carbonates increases. 2Na + O2 ¾¾¾® Na2O2 Sodium peroxide
15. (a) BeCO3 is unstable due to high polarising power of K + O2 ¾® KO2 Potassium superoxide
smallest Be2+ ion. Also, it shows reversible reaction, 28. (b) In metals moving down the group metallic character
because of more stability of oxide formed. Hence, BeCO3 increases, so basic nature increases hence, most acidic
has least stability and it is stable only in CO2 atmosphere. will be BeO.
16. (b) Among fluorides, the order of solubility is LiF < NaF BeO < MgO < CaO < BaO
¾¾¾¾¾¾¾¾¾¾
®
< KF < RbF < CsF. On moving down the group solubility increasingbasic character
increases because lattice energy decreases. 29. (c) Element (X) electronic configuration is
17. (a) Sodium carbonate can be synthesised by Solvay 1s2 2s2 2p3
ammonia soda process. So, valency of X will be 3.
The reactions involved in the process are : Valency of Mg is 2.
Formula of compound formed by Mg and X will be Mg3X2.
NH3 + H 2O + CO2 ¾¾
® NH 4 HCO3 30. (a) BeH2 < CaH2 < BaH2
Ammonium bicarbonate
Smaller the size of cation, more will be its polarising power.
NaCl + NH 4 HCO3 ¾¾
® NaHCO3 ¯ + NH 4Cl Hence, BeH2 will be least ionic.
Sodium bicarbonate 31. (d) Amphoteric hydroxide means it can react with both
D acid and base.
2NaHCO3 ¾¾
® Na 2 CO3 + H 2O + CO 2
Sodium carbonate Be ( OH )2 + 2HCl ¾¾
® BeCl2 + 2H 2O
NH3 is recovered from NH4 HCO3 and NH4Cl
Be ( OH )2 + 2NaOH ¾¾
® Na 2 éë Be ( OH ) 4 ùû
D
NH 4 HCO3 ¾¾® NH 3 + H 2 O + CO 2
32. (b) Enzyme that utilise ATP in phosphate transfer require
2NH 4 Cl + Ca ( OH ) 2 ® 2NH3 + CaCl 2 + 2H 2 O an alkaline earth metal (M) Mg as the cofactor.
Ammonium chloride Ammonia Calcium chloride
THE s-BLOCK ELEMENTS 557

33. (b) Mg burns in air and produces a mixture of nitride and D


7. (d) ZnCl2 + 2NaHCO3 ¾¾® Zn ( HCO3 ) 2 + 2NaCl
oxide.
( A)
34. (d) The chances of formation of hydrate decreases with
the decrease in the charge density down the group. This is D
Zn ( HCO3 )2 ¾¾
® ZnCO3 + H 2O + CO2 ­
why, Ba(NO3)2 does not crystallise with water molecules. ( B) ( C)
(A)
35. (a) When HCl is passed through the solution Cl– ion
concentration increases. Hence ionic product becomes ZnCO3 + 2NaOH ® Na 2 ZnO2 + 2H 2 O
more than solubility product. Only NaCl is crystallised due ( B) ( D)
to less solubility than MgCl2 and CaCl2.
8. (d) K2Cr2O7 + 5H2SO4 + 4Na2O2 ® 2CrO5 + K2SO4 +
36. (c) Potassium (K) activates many enzymes to participate
(Blue)
in oxidation of glucose to produce ATP and helps in the
4Na2SO4 + 5H2O
transmission of nerve signal along with Na. CrO5 is an unstable peroxide which decomposes as:
4CrO5 + 6H2SO4 ® 2Cr2(SO4)3 + 6H2O + 7O2­
Exercise - 3
(green)
1. (b) As the size of alkali metal ion increases, lattice 9. (a) Na2SO4 is soluble in water because its hydration
enthalpy decreases and hence the stability of the enthalpy is more than the lattice enthalpy. BaSO4 is
corresponding metal hydride decreases. insoluble in water because its hydration enthalpy is less
than the lattice enthalpy.
2. (b) X ¾heat
¾® CO2 + Residue 10. (b) Potassium superoxide (KO2) is used in respiratory
Solid equipment because it liberates oxygen and removes carbon
boil H2O dioxide and water vapour.
excess CO2 4KO2 + 4CO2 + 2H2O ® 4KHCO3 + 3O2
Z ¬¾¾¾¾ Y
Clear solution 11. (c) MgSO4 is the only alkaline earth metal sulphate which
is soluble in water and for solubility hydration energy
The given properties coincide with CaCO3
should be greater than lattice energy.
CaCO3 ¾heat
¾® CO2 + CaO 12. (c) Ca and CaH2 both react with H2O to form H2 gas,
'X' Residue
Ca + 2 H 2 O ¾
¾® Ca ( OH ) 2 + H 2
boil H2O
excess
CaH 2 + 2H 2 O ¾¾® Ca (OH ) 2 + 2 H 2
(CaCO3)2 ¬CO
¾¾ CO2 + Ca(OH)2 whereas
2
'X' 'Y'
3. (a) Setting of cement is exothermic process which K gives H2 while KO2 gives O2 and H2O2
develops interlocking crystals of hydrated silicates 2K + 2 H2O ¾ ¾® 2KOH + H2
4. (a) The crystal structures of NaHCO3 and KHCO3 both 2KO2 + 2H 2O ¾¾
® 2KOH + O2 + H 2O2
show hydrogen bonding but are different. In NaHCO3, Similarly, Na gives H2 while Na2O2 gives H2O2
the HCO3– ions are linked into an infinite chain, while in 2 Na + 2 H 2 O ¾
¾® 2 NaOH + H 2
KHCO3, a dimeric anion is formed. Na 2 O 2 + 2H 2 O ¾
¾® 2 NaOH + H 2 O 2
5. (a) Basic strength of the oxides increases in the order
Likewise Ba gives H2 while BaO2 gives H2O2
Li2O < Na2O < K2O < Rb2O < Cs2O. The increase in basic
Ba + 2 H2O ¾ ¾® Ba (OH)2 + H2
strength is due to the decrease in I.E. and increase in
BaO2 + 2H2O ¾ ¾® Ba (OH)2 + H2 O2
electropositive character.
13. (b) 30 mL N/50 HCl º 30 mL N/50 Ca(HCO3)2 º 30 mL
The melting points of the halides decrease in the order
N/50 CaCO3 º 100 mL tap water
NaF > NaCl > NaBr > NaI, as the size of the halide ion Mass of CaCO3 in 100 mL tap water
increases. The decrease in melting point is due to increase
in the covalent character with increase in the size of anion E´N´V 50 ´ 30
= = = 0.03 g
according to Fajan's rule. 1000 50 ´ 1000

6. (b) Na 2CO3 + 2SO2 + H 2O ® 2NaHSO3 + CO 2 Þ hardness = 300 ppm


(A) 14. (a) The magnitude of hydration enthalpy depends upon
the square of the charge on the ion. Since, charge on the
2NaHSO3 + Na 2 CO3 ® 2Na 2SO3 + H 2O + CO2
divalent ion (alkaline earth metal cation) is double of the
( A) ( B)
Heat monovalent ion (alkali metal cation), hence the hydration
2Na 2SO3 + S ¾¾¾® 2Na 2S2 O3 enthalpy of alkaline earth metal cations (M2+) should be
(B) (C) nearly four times that of the alkali metal cations (M+).
Na 2S2O3 + 2AgNO3 ® Ag 2S2 O3 + 2NaH 2 15. (b)
( D)
EBD_8350
558 CHEMISTRY

11 The p-block Elements (Group 13 and 14)


D
Exercise - 1 Na 2 B4O7 ¾¾
® 2 NaBO2 + B2 O3
Anhydrous sod.metaborate Boricanhydride
1. (b) Aluminium does not occur in the free state in nature CuO + B2 O3 ¾¾
® Cu(BO 2 )2
but is most abundant metal in the earth’s crust. Cupric meta borate (blue beed)
2. (b) The order of ionisation enthalpies, as expected, is 24. (c) H3BO3 on heating at 373 K yields metaboric acid
DiH1 < DiH2 < DiH3. (HBO2)
3. (c) 373K
H3BO3 ¾¾¾® HBO2 + H2O
4. (b) Metallic character increases down the group. D
Metaboric acid
5. (a) Due to its small size and high ionization energy, boron (orthorombic form)
does not form B3+ ion. 25. (b) In Boric acid each B atom is sp2 hybridised and
6. (b) Thallium shows different (+1 and +3) oxidation states contains BO 33- units which are held together by hydrogen
because of inert pair effect. bonds.
26. (d)
7. (c) Because Tl5+ does not exist.
27. (b) 2BF3 + 3LiBH4 ® 2B2 H6 + 3LiF
8. (c) 2Al + Na2CO3 + 2H2O ® 2NaAlO2 + CO2 + 3H2
9. (b) Down the group basic character of oxides increases. H H
B2O3 – Acidc H B H H + B

+ H
Al2O3 – Amphoteric N N N N
Ga2O3 – Amphoteric 28. (b) B B B– B–
+
In2O3 – Basic H N H H N H
Tl2O3 – Basic H H
12 s bonds 12 s and 3 p bonds
10. (c) Aluminium chloride in acidified aqueous solution
forms octahedral [Al(H2O)6]3+ ion. 29. (b)
11. (b) AlCl3 + 3HOH ® Al(OH)3 + 3HCl 30. (c) The correct formula of inorganic benzene is B3N3H6
so (d) is incorrect statement
12. (a) 4Al + 3O2 ® 2Al2O3
OH
13. (a) Boron does not react with acids and alkalies. |
Boric acid (H3BO3 or B - OH ) is a lewis acid so (a) is
14. (d) B(OH)3 is acid because it can take OH– ions. |
H3BO3 or B(OH)3 + OH– ® B(OH) -4 incorrect statement.
OH

15. (a) B2O3 is acidic and Al2O3 is amphoteric. The coordination number exhibited by beryllium is 4 and
not 6 so statement (b) is incorrect.
16. (b) 2Al(s) + 2NaOH(aq) + 6H2O(l) ®
Both BeCl2 and AlCl3 exhibit bridged structures in solid
2Na+[Al(OH)4]–(aq) + 3H2 (g)
state so (c) is correct statement.
17. (d) Atomic radius of gallium is less than that of
aluminium. Cl m Cl Cl Cl Cl
2p
20
18. (d) The order of strength of Lewis acid character for boron Be 98° Be 263 pm Be Be
halides is, BF3 < BCl3 < BBr3 < BI3 (due to back bonding) 82°
19. (a) F3B+ : NH3 ® F3B ¬ NH3 Cl Cl Cl Cl Cl
20. (d) It is pp – pp back bonding involving B and F. The Cl Cl Cl
smaller atoms show more back bonding.
Al Al
21. (b) BCl3 forms [B(OH)4]– in which B is sp3 hybridized
and have +3 oxidation state. Cl Cl Cl
22. (a) Higher boranes are also spontaneously flammable 31. (a) H3BO3 is a weak monobasic acid.
in air. 32. (d) Boron nitride (BN) is known as inorganic graphite.
D The most stable form is hexagonal one. It has layered
23. (b) Na2B4O7. 10 H2O ¾¾¾¾® Na2B4O7
- 10H 2O structure similar to graphite.
THE p-BLOCK ELEMENTS (GROUP 13 AND 14) 559

42. (c) When diborane is hydrolysed, one can get both


N N+ orthoboric acid and H2.

+ B
B– –B B2H6 + 6H2O ® 2H3BO3 + 6H2
43. (c) Due to the low density of aluminium it is useful for
N+ +N N food packaging.
– +
–B B B– –B 44. (d) BF3 is a weak Lewis acid.
45. (a) The use of aluminium and its compounds for domestic
+N N N+ – + N purposes is now reduced considerably because of their
+
B– –B B toxic nature.
46. (c) CO2, SiO2 are acidic, CaO is basic and SnO2 is
N+ +N N
– + amphoteric.
B B– 47. (b) The inert pair effect is most prominent in the heavier
Boron nitride – Inorganic graphite
members of the group. Inert pair effect increases as we
move down the group.
100°C 160°C
33. (d) H3BO3 ¾¾¾® HBO2 ¾¾¾® 48. (c) Inert pair effect increases down the group. Hence,
H2B4O7 + H2O ¾¾¾¾
330°C
® 2B2O3 + H2O Pb2+ is most stable.
(boric acid 49. (d) PbO2 is a strong oxidising agent and is produced
anhydride) in situ in lead storage batteries. The anode is oxidized to
34. (c) Inorganic benzene, B3N3H6 PbO2 and cathode is reduced to spongy Pb.
It is isoelectronic and isostructural with benzene. 50. (d) The thermal stability of tetrahalides decreases in order
35. (d) In diborane (B2H6) structure there are four 2c-2e–
CX4 > SiX4 > GeX4 > SnX4 and in terms of same metal with
bonds and two 3c–2e– bonds (see structure of diborane).
different halides is in order of
Structure of B2H6 :
MF4 > MCl4 > MBr4 > MI4.
Hb
Therefore, GeBr 4 will be least stable.
Ht •• Ht 51. (a) Since bond energy of C-F >C-Cl > C-Br > C-I
B B Hence CF4 is most stable.
52. (a) The stability of dihalides (MX2) increases down the
Ht •• Ht group. Except C and Si, the other members form dihalides.
Hb 53. (b) SiO2 is acidic oxide.
excess NH 54. (d) Reluctance of valence shell electrons to participate in
(d) B2H6 + NH3 ¾¾¾¾¾¾¾
3 ®
36. low temperature B2H6.2NH3 bonding is called inert pair effect. The stability of lower
Diborane with ammonia gives B2H6.2NH3 that is formulated oxidation state (+2 for group 14 element) increases on
as [BH2(NH3)2]+[BH4]– which when heated to 473K, going down the group. So the correct order is
decomposes to give borazole. SiX2 < GeX2 < SnX2 < PbX2
H H H 55. (a) The stability of +2 and +4 O.S. follows the order
37. (d) B 97°83° B Pb2+ > Sn2+ > Ge2+ and Ge4+>Sn4+>Pb4+
H H H Hence, reducing power Ge > Sn > Pb
B is sp3 hybridised 56. (d) Si and Ge are semiconductors and are used in making
Only 12 bonding electrons are available transistors.
BHB angle is 83° not 180°. 57. (b) Both Ge and Si are extensively used as
D D semiconductors. Semiconductors conduct electricity when
38. (c) Na2B4O7.10H2O ¾¾
® Na2B4O7 ¾¾
®
(i) there is only a small difference in energy, called band gap,
2NaBO2 + B2O3 between the filled valency band of electrons and a
(ii) (iii) conduction band Since the band gap of Ge is less than Si,
Na2B4O7 + 7H2O ¾¾
® 4H3BO3 + 2NaOH it is a better element to be used as semiconductor.
(iv) (v) 58. (b) Ge 4+ is more stable than Ge 2+. Hence, GeCl4 is
39. (a) When borax is heated in a Bunsen burner flame more stable than GeCl2
with CoO on a loop of platinum wire, a blue coloured 59. (d) Carbon halides are not hydrolysed due to absence of
Co(BO2)2 bead is formed. d-orbitals. On the other hand, SiCl4 is easily hydrolysed
40. (b) The hybridizations of B in H3BO3 is sp2. due to the availability of d-orbitals in Si.
41. (c) Diborane is produced on industrial scale by the SiX4 + 4H2O ® Si(OH)4 + 4HX
reaction of BF3 with sodium hydride.
EBD_8350
560 CHEMISTRY

60. (d) In nature, Pb4+ is strong oxidant and I– is strong 81. (b) Quartz is crystalline form of silica.
reductant. Hence, PbI4 cannot exist. 82. (b) 6 HF + SiO 2 ® H 2SiF6 + 2 H 2 O
61. (b) F and Cl are more oxidising in nature and can 83. (a) Producer gas is a fuel gas and is mixture of CO and N2.
achieve Pb in (IV) O.S. but Br 2 and I2 can not achieve 84. (b) In silica (SiO2); each Si atom is surrounded by four
Pb in (IV) O.S. Secondly Pb4+ is strong in oxidising oxygen atom.
nature and in its presence, Br– and I– can not exist.

— —

— —

— —

— —
62. (c) Lead pipes are readily corroded by water containing — Si — O —Si — O — Si — O —Si —
organic acids. It forms lead acetate with acetic acid.
63. (b) 2Pb + 2H 2O + O 2 ® 2Pb(OH) 2 O O O O


64. (a) Carbon does not contain d-orbital hence, it cannot — Si — O —Si — O — Si — O —Si —


expand its octet.
65. (d) Ge(II) tends to acquire Ge (IV) state by loss of O O O O

— —

— —

— —

— —
electrons. Hence, it is reducing in nature. Pb (IV) tends to
— Si — O —Si — O — Si — O —Si —
acquire Pb (II) O.S. by gain of electrons. Hence, it is
oxidising in nature. This is due to inert pair effect. Structure of SiO 2
66. (c) CO2 forms carbonic acid H2CO3, when dissolved in Only Si – O bonds exist and no Si = O.
water. When group 2 hydroxide like Ca(OH)2 is added, a
85. (b) R 3SiCl + HOH ® R 3SiOH + HCl
precipitation will form: Ca(OH)2 + CO2 ® CaCO3¯ + H2O.
CO is neutral, whereas other two GeO2 and SnO2 are solids. R 3SiOH + HOSiR 3 ® R 3Si - O - SiR 3 + H 2O
67. (a) Lead in +2 oxidation is stable while Sn and C are both 86. (c) Water gas is CO + H2.
stable in +4 oxidation. 87. (c) CO is essential constituent of almost all fuel gases.
68. (c) Pb with dil HCl forms protective coating of PbCl2. 88. (c) CO 2 is incombustible and non supporter of
69. (a) PbI4 does not exist because Pb–I bond initially combustion.
formed during the reaction does not release enough 89. (b) CO r eact with haemoglobin , for ms carboxy
energy to unpair 6s2 electrons and excite one of them to haemoglobin and stops the supply of O2.
higher orbital to have four unpaired electrons around 90. (b) Carbon dioxide is not a poisonous gas.
lead atom.
91. (a) Being biocompatible, silicones are used in surgical
70. (c) Due to non-availability of vacant d-orbitals, it cannot and cosmetic plants industry.
exceed its coordination number more than four. Thus,
carbon never forms complexes e.g., [CCl6]2– does not exist 92. (b) ZSM – 5 type of zeolite is used to convert alcohols
but [SiCl6]2– exists. directly into gasoline.
71. (d) The more the bond energy, the more is the catenation. 93. (d) All the statements are correct.
72. (d) The order of tendency of catenation for elements of C 94. (b) Coal gas is a mixture of H2 + CO + N2 + CH4
family is 95. (d) H2CO3/HCO3– buffer system helps to maintain pH of
C >> Si > Ge » Sn > Pb blood between 7.26 to 7.42.
73. (b) Buckminster fullerene is C60. The molecule has shape
of soccer ball. Exercise - 2
74. (b) In fullerene, each carbon atom is bonded to three other 1. (c) Among these elements, in gallium, the crystal
carbon atoms and is sp2 hybridised. structure is different which suggests that Ga consists of
75. (d) In graphite, each carbon is sp2 -hybridised and the almost discrete Ga2 molecule, so its melting point is lowest.
single occupied unhybridised p-orbitals of C-atoms Ga exists as liquid from 30°C upto 2000°C and hence, it is
overlap side wise to give p -electron cloud which is used in high temperature measurement.
delocalized and thus, the electrons are spread out between 2. (a) Alkaline earth metals (Mg, Ca, Ba) form ionic chloride
the structure. whereas aluminium form covalent chloride. Despite of
76. (a) Graphite sp2, \ % s character = 33% sharing electrons with chlorine, the octet of aluminium is
Diamond sp3, \ % s character = 25% incomplete. To complete the octet, it needs electrons and
77. (b) 78. (c) thus, acts as a Lewis acid.
79. (d) Diamond and crystalline silicon are isomorphous. The 3. (a) On moving down the group the acidic nature of oxides
C atom in diamond and Si atom in crystalline silicon is changes from acidic to basic through amphoteric
coordinated to 4 other atoms, resulting in tetrahedral
B2 O3 , Al2 O3 and Ga 2O3 , In 2 O3 and Tl 2O 3
structure. { 14442444 3 144 42444 3
More acidic Amphoteric Basic
80. (a) Df HO for an element in its standard state is zero. less basic
THE p-BLOCK ELEMENTS (GROUP 13 AND 14) 561

4. (a) Among these elements boron has the lowest atomic 11. (a) (i) Reaction of ammonia with diborane gives
number. It has atomic number 5 and do not have vacant initially B 2 H6 .2NH 3 which is r epresen ted as
d-orbital. Boron can show maximum coordination number [BH2(NH3)2]+[BH4]– which on further heating gives
of 4. The element M in the complex ion MF63- has a borazine, B3N3H6 also called borazole.
coordination number of six. Hence, B can not form complex 473 K
3B2 H 6 + 6NH 3 ¾¾¾® 2B3 N 3H 6 + 12H 2
3-
of the type MF6 . Diborane Burazine
(X) (Y)
5. (b) Quartz is a crystalline form of silica. Quartz is
(Inorganic benzene)
extensively used as a piezoelectric material.
6. (c) Carbon dioxide can be obtained as a solid in the form (ii) Diborane can be prepared by the reaction of BF3 with
of dry ice, allowing the liquified CO2 to expand rapidly. lithium aluminium hydride in diethyl ether.
It is used to maintain the low temperature in laboratory. 4BF3 + 3LiAIH4 ¾® 2B2H6 + 3AIF3 + 3LiF
Dry ice is solid CO2 12. (d) Sn and Pb can show two oxidation states i.e. +2 and
-
+4. These elements in +4 oxidation state behave as
7. (a) Structure of B ( OH ) 4 is oxidising agent, whereas in +2 oxidation state behave as
OH reducing agent. The stability of +2 oxidation state
| s increases on moving down in a group i.e., reducing
HO B OH character decreases from Ge (II) to Pb (II). i.e., Germanium
|

|
OH salts are strongest reducing agent and Pb(II) salt the
4 bond pair + 0 lone pair weakest. Ge is much less abundant than tin in nature. Hence,
Hybridisation — sp3 Sn (II) salts are widely used as reducing agent.
Geometry —Tetrahedral 13. (b) Cement is obtained by combining a material rich in
lime (CaO) with other material like silica SiO2 along with
8. (c) Boric acid is a monobasic weak acid. It does not
the oxides of aluminium, iron and magnesium. The average
liberate H+ ion but accepts electrons from OH– ion.
composition of cement is
-
H3BO3+ H2O ¾® B ( OH ) 4 + H+ CaO (50 – 60%)
9. (b) The tendency of forming long open or closed chains SiO2 (20 – 25%)
by combination of some atoms in themselves is known as Al2O3 (5 – 10%)
catenation. The tendency of catenation is maximum in Fe2O3 (1 – 2%)
carbon and decreases on moving down the group.
SO2 (1 – 2%)
C > > Si > Ge » Sn > Pb
MgO (2 – 3)%
This is due to high bond energy of C — C bonds. On
Thus, it contains elements of group 2 (Ca), group
moving down the group, atomic size increases and
13 (Al) and group 14 (Si).
electronegativity decreases, hence, tendency to show
catenation decreases. 14. (c) Silicon has a strong tendency to form polymers like
silicones. The chain length of silicon polymer can be
10. (b) In diborane, there are 12 valence electrons, three from
controlled by adding (CH3)3SiCl which block the ends as
each boron atoms and six from the six hydrogen atoms.
shown below:
The structure of B2H6 can be represented as
CH3 CH3
H H H | |
n HO — Si — OH + Cl — Si — CH3
| |
B B CH3 CH3
–HCl Polymerisation

H H H CH3 CH3
Bonding in diborane | |
The four terminal hydrogen atoms and two boron atoms — O — Si — O — Si — CH3
lie in the same plane. These four terminal B-H bonds are | |
CH3 CH3
regular 2c-2e– bonds. Above and below the plane, there
Silicone
are two bridging hydrogen atoms. Each boron atom forms
four bonds even though it has only three electrons. The 15. (d) On moving down a group, the ionization enthalpies
terminal B — H bonds are regular bonds but the bridge generally decreases due to increase in atomic size and
B — H bonds are different. screening effect which is more than to compensate the
effect of increase in nuclear charge. Consequently, the
Each bridge hydrogen is bonded to the two boron atoms
electron becomes less and less tightly held by the nucleus
only by sharing of two electrons. Such covalent bond is
as we move down the group. The sharp decrease in
called banana bond (3c–2e–).
ionization enthalpy from B to Al is due to increase in size.
EBD_8350
562 CHEMISTRY

In case of Ga, there are 10 d-electrons in its inner electronic 28. (b) The carboxyhaemoglobin (haemoglobin bound to
configuration. Since the d-electrons shield the nuclear CO), is about 300 times more stable than
charge less effectively than the s & p electrons, the outer oxygaemoglobin.
electron are held strongly by the nucleus. As a result, the 29. (d)
ionization enthalpy increases slightly inspite of the (A) Water gas is a mixture of combustible gases (CO and
increase in atomic size as we move from Al to Ga. The H2).
similar increase is also observed from In to Tl, which is
(B) Temporary hardness of water is due to bicarbonates
due to presence of 14f electrons in inner electronic
of calcium and magnesium. It can be removed by
configuration of Tl which has poor shielding effect.
boiling of water.
16. (b) Lower oxidation states become more stable on moving
(C) Diborane (B2H6) is an electron deficient hydride.
down the group
(D) Hydrogen peroxide (H2O2) is a non-planar molecule
Al < Ga < ln < Tl
with (twisted symmetry). It is non-planar, non-linear
17. (d) Inertness of ns2 electrons of the valence shell to molecule with an open book structure.
participate in bonding on moving down the group in heavier
p-block elements is called inert pair effect. Exercise - 3
As a result, Pb (II) is more stable than Pb (IV) and Sn (IV)
is more stable than Sn (II) Pb (IV) is easily reduced to Pb
1. (b) Na 2 B4 O7 + H 2 O ® 2Na éë B ( OH ) 4 ùû + 2H 3BO3
(II) and can act as an oxidising agent whereas Sn (II) is
Salt with strong Weak acid
easily oxidised to Sn (IV) and can act as a reducing agent. base
18. (c) Due to poor screening effect of 3d10 electrons in Ga, 2H O
Ga < Al. 2. (c) (CH3)2 SiCl2 ¾¾¾¾
2 ®
–2HCl
(CH3)2Si(OH)2
19. (d) Boron belongs to 2nd period and it does not have
Condensation
vacant d-orbital. ¾¾¾¾¾¾
®
polymerisation
20. (d) BCl 3 & AlCl 3 both have vacant p-orbital and CH3 CH3
incomplete octet. So, they act as Lewis acid. | |
- O — Si — O — Si — (A straight chain silicone)
21. (c) NaOH
Al + 3H 2 O ¾¾¾® Al(OH)3 ¯ + 3 / 2H 2 (g) | |
(X) CH3 n CH3
White
gelatinous ppt. 3. (c) Sn4+ is more stable than Sn2+. Hence, Sn2+ will oxidise
to Sn4+ but it will not reduce to Sn.
[Soluble in excess of NaOH and form Na[Al(OH) 4]
D 4. (c) SnCl 2 + 2 NaOH ® Sn (OH) 2 + 2 NaCl
2Al(OH) 3 ¾¾
® Al 2O 3 + 3H 2O
(used as adsorbent in
chromatography)
Sn (OH) 2 + 2 NaOH ® Na 2SnO 2 + 2H 2O
Thus, metal is Al. 5. (a) The water repellency arises because a silicone chain
22. (a) PbF4 is ionic in nature. is surrounded by organic side groups and looks like an
23. (d) [SlCl6]2– does not exist because six large chloride ions alkane from the outside.
cannot be accommodated around Si4+ due its small size. 6. (d) Mg2C3 consists of C34 - type of anionic part.
24. (d) Fullerene (C60) contains 20 hexagons (six membered)
+
AlCl3.6H2O ¾¾¾¾¾® éëAlCl2 ( H 2O ) 4 ùû
rings and 12 pentagons (five membered rings): dissociation
7. (c)
25. (a) Due to the inert pair effect, thallium exists in more
-
than one oxidation state. Also, for thallium + 1 oxidation + éë AlCl4 ( H 2 O ) 2 ùû
state is more stable than +3 oxidation state.
HCl
26. (d) Silicones are polymers containing Si—O—Si linkages 8. (d) Borax ¾¾¾
® H 3 BO 3
with strong hydrophobic character.
B2 O3 ¾¾® B ( crystalline )
Al
Generally, they exhibit high thermal stability with high D
dielectric strength. Silicon greases are resistant to oxidation
® B ( Amorphous )
Mg
which are commonly used for greasing purposes. B2 O3 ¾¾¾
D
27. (c) 9. (c) Al(OH) 3 dissolves in NaOH solution to give
(i) Dry ice, CO2(s), is used as refrigerant. -
Al ( OH ) 4 ion which is supposed to have the octahedral
(ii) C60 contains 20 six membered rings, 12 five membered
rings. complex species [Al(OH)4(H2O)2]– in aqueous solution.
(iii) Ethanol, methanol and larger alcohols can be Al OH 33 ++ NaOH
Al(OH) aq ¾¾
NaOH (aq) ®é Al OH 4 H O ù
converted at 300-400° C in the presence of a zeolite [Al(OH4)(H2O)2)]–(aq) + Na+ (aq)
catalyst (ZSM-5).
THE p-BLOCK ELEMENTS (GROUP 13 AND 14) 563

13. (d)
10. (d)
(i) D
Mg3B2 + H3PO4 ¾¾ ® B2H6
ether
(ii) 4BX3 + 3LiAlH4 ¾¾¾¾¾®
X = F, Cl, Br 2B2H6 + 3Lix + 3Alx3

180°C
(iii) 2BF3(g) + 6NaH ¾¾¾® B2H6(g) + 6NaF
In diglyme
(iv) 2NaBH4 + I2 ¾¾¾¾¾
solution
® B2H6 + H2 + 2NaI
14. (c)
(i) Mg2Si + H2SO4 ¾¾
® mixture of silanes
Total No. oxygen atoms per silicon atom
1 1 1 (SiH4, Si2H6, Si3H8, etc.)
= + + + 1 = 2.5 (ii) SiCl4 + Li[AlH4] ¾¾
® SiH4 + AlCl3 + LiCl
2 2 2
\ Formula Si 2 O52 - . (iii) CH3SiCl3 + CH3MgCl ¾¾® (CH3)2 SiCl2 + MgCl2
Cu catalyst
11. (d) 2CaF2 + SiO 2 + H 2 SO 4 ¾¾
® (iv) Si + 2CH3Cl ¾¾¾¾¾
280 - 300° C
® (CH3)2 SiCl2
hydrolysis
SiF4 + H2 O + CaSO 4 ¾¾¾¾¾ 15. (b) Aluminium oxide is amphoteric oxide because it
® CaSiF6
shows the properties of the both acidic and basic oxides.
12. (a) Pb3 O 4 + 4HNO3 ® 2Pb(NO 3 ) 2 + PbO2 ¯ + H 2 O It reacts with both acids and bases to form salt and water.
Pb3O4 + H 2O 2 + 6HNO3 ® 3Pb(NO3 ) 2 + O 2 + 4H 2 O Al2O3. xH2O + 2NaOH ¾¾® NaAlO2 + H2O
Al2O3. xH2O + HCl ¾¾® AlCl3 + H2O
EBD_8350
564 CHEMISTRY

12 Organic Chemistry–Some Basic Principles & Techniques

Exercise - 1 CH 3
|
1. (c) A carbon having an sp hybrid orbital with 50% CH 3 C HCH 2 CH 2CH 3
s-character is more electronegative than carbon atoms
(d )
having sp2 and sp3 hybrid orbitals with 33% and 25% s-
character respectively. 9. (b) The compound is a derivative of butane.
In CH2 C CHCH3 10. (b) The compound contains longest chain of 6C atoms
Number of s bonds : and amino group. Hence it is an alkanamine.
sC – C = 3, s C – H = 6, total = 6 + 3 = 9 11. (d) The compound is an ester. Its IUPAC name is derived
Number of p bonds = 2 from alkyl alkanoate.
12. (d) The compound is an aldehyde containing longest
CH3 CH3 chain of 6 C-atoms and side chains.
sp3 sp2 sp
2. (d) CH3 C CH CH CH C CH Cl
7 6 5 4 3 2 1 |
1 2 3
CH3 13. (c) CH3 — C — CH3
x |
3. (c) See the number of s bonds formed by C in each case. In CH3
x x x x
HCOOH , (H 2 N) 2CO and CH 3C HO,C forms 3s bonds 2-Chloro-2-methyl propane
x 14. (a)
and 1 p bond, hence hybridisation is sp2. In (CH 3 )3C OH ,
x 15. (d) (CH 3 ) 2 CHCH 2 CH 2 -
C forms 4s bonds, so hybridisation will be sp3. 3-Methylbutyl group

H H H Ketonic group
| | |
4. (b) H - C- C = C- C- C- H ; 4 3 2 1
| | | | | 16. (a) CH3 CH CO CH3
H H H H H
2 - Pentene CH3 methyl group
No. of s bonds = 14, No. of p bond = 1 3-Methyl-2-butanone
5. (b) Correct order of decreasing priority is
– COOH, – SO3H, – COOR, – OH.
17. (a) CH3
6. (b) |
7. (d) 2, 2,3-trimethyl pentane 1 2 3
H3C- C - CH3
1° 1° |
CH3 CH3 CH3
1° 2° | | 1°
Neopentane
H3C - H 2 C - CH - C - CH 3
3° | 4°
or 2, 2- Dimethylpropane
CH3
1° 18. (b) The compound is a derivative of benzoic acid. The
positions of substituents attached to benzene nucleus are
CH3 CH3 represented by number of C-atoms and not by ortho, meta
| | and para.
8. (d) CH 3 - C - C - CH 2 CH 3 19. (c)
| |
CH3 CH3 20. (d) The correct name of the given compound is
(a ) 2, 5, 6 -trimethyloctane
21. (a) It is derivative of ethanamide having N-phenyl
group.
CH3 CH3 CH3 22. (b) The compound contains longest chain of 5C - atoms
| | |
CH 3 – C – CH 2 CH 3 CH C - C HCH CH and ‘e’ of ene is retained as the suffix name starts with constant
3 2 3
| | 23. (d) The compound contains longest chain of 3 C - atoms
CH3 CH3 and three -COOH groups and one -OH group attached to
(b) (c ) it (latest convention).
24. (a) 25. (c)
ORGANIC CHEMISTRY–SOME BASIC PRINCIPLES & TECHNIQUES 565

26. (a) Same number of carbon, hydrogen and oxygen atoms 41. (b) Homolytic fission of the C – C bond gives free radicals
are present in both the compounds. in which carbon is sp2- hybridised.
27. (b) 42. (b) The order of stability of carbocations is :
28. (b) Structures (a), (c) and (d) have the same molecular + + +
formula (C6H12O) while (b) has same molecular formula (CH 3 )3 C > (CH 3 ) 2 CH > CH 3
(C6H10O) as compound given. This order can be explained on the basis of hyper
29. (b) 2, 2, 4, 4 - Tetramethylhexane has 10 carbon atoms, conjugation and Inductive effect.
only 4-isopropylheptane has also 10 carbon atoms, so 43. (d) Higher stability of allyl and aryl substituted methyl
these two are isomers. carbocation is due to dispersal of positive charge due to
30. (d) resonance
31. (b) The two isomers differ in the position of the double + +
bond are called position isomers, so given compounds are CH 2 = CH - C H 2 ¬¾® CH 2 - CH = CH 2
position isomers. Resonating structures of allyl carbocation
32. (a) 33. (c) +
CH2 CH2 CH2 CH2
34. (b) C2H5SC2H5 and C3H7SCH3 are metamers.
Å
35. (d) C4H10O:
Å

(a) Metamerism
and Å
O O
Resonating structures of benzyl carbocation
(b) Functional isomerism
whereas in alkyl carbocations dispersal of positive charge
OH and O on different hydrogen atoms is due to hyperconjugation
(c) Position isomerism hence the correct order of stability will be
OH and Å
CH2
OH Å Å
Thus, option (d) is correct. > CH 2 = CH - C H 2 > CH3 - CH 2 - CH 2
36. (c) Metamerism shown among compounds of the same Allyl Propyl
functional group. Benzyl
37. (a) Carbocation are electron deficient species. In general, 44. (b) Structure (b) is a 3º carbocation, and more stable due
greater the number of alkyl groups attached to it, greater is to resonance with 2 Ph-ring while (a) is 2º (c) and (d) are 1º
the +I effect and stabilisation of the cation. carbocations; thus (b) is the most stable.
+ 45. (a)
CH3 ®- C -¬ CH3
| > Cl
­ – – – –
CH3 46. (b) C Cl > C6H5CH2 > (CH3)2 CH > (CH3)3C
3°carbocation Cl
(+vecharge dispersed to maximum extent) –ve charge –M effect +I effect of CH 3 group
(9 hyperconjugative H's) delocalises intensifies the –ve charge
highly dispersed
due to – I effect –ve charge
+ +
CH3 ®¾ C– H > CH3CH2 ®¾ CH2 47. (b) The order of stability of free radicals is
| • • • •
­ (C6 H 5 ) 3 C > (C 6 H 5 ) 2 CH > (CH 3 ) 3 C > (CH 3 ) 2 C H
CH2CH3
The stabilisation of first two is due to resonance and last
2° carbocation 1° carbocation
(5 hyper. H's) (+ve charge least dispersed) two is due to inductive effect (i.e., +R > +I).
(2 hyper. H's) 48. (c) The strength of nucleophile depends upon the nature
of alkyl group R on which nucleophile has to attack and
Alternatively, above order of stability order can be also on the nature of solvent. The order of strength of
explained in terms of hyperconjugation. nucleophiles follows the order :
38. (a) Higher the possibility of delocalisation, greater is its CN– > I– > C6H5O– > OH– > Br– > Cl–
+ 49. (c) 50. (d)
stability; in C6 H5CHC6 H5 , +ve charge can delocalise
51. (b) Electrophiles are electron deficient or positively
over two benzene rings.
charged species. In SO3, oxygen is more electronegative
39. (d) The organic reaction which proceed through
than sulphur and therefore can withdraw electron density
heterolytic bond cleavage are called ionic or heteropolar
towards themselves, creating positive charge on sulphur
or just polar reactions.
atom, hence SO3 becomes electron deficient and can act
40. (b) Methyl carbanion is sp3 hybridised, with three bond
as a electrophile.
pairs and one lone pair same is the case with NH3.
EBD_8350
566 CHEMISTRY

52. (d) BF3 and R3C – X are electrophiles, while (CH3)3N 66. (b) – I group destablises carbocation and since inductive
and C2H5O– are nucleophiles. effect decreases with increasing length of carbon chain.
53. (d) Due to – I effect of the – CHO group, oxygen acquires- Therefore (b) is the correct option.
d - charge and the terminal carbon acquires d + charge. 67. (d) –NO2 group, being strong electron-withdrawing,
d+ d- disperses the –ve charge, hence stabilizes the concerned
CH 2 = CH – C = O carbanion.
H 68. (a) Nucleophilicity increases down the group in the
54. (c) All resonating structures should have same number periodic table. I - > Br - > Cl - > F-
of electron pairs. +
55. (b) Only structure (b) has a conjugated system, which is 69. (d) CH3 Br CH3 + Br–
necessary for resonance.
56. (a) It is fact that aniline is better nucleophile than anilium CH3 Cl CH3 + Cl
ion. Anilium ion has +ve charge, which reduces the
70. (b) Higher the number of a – H, more the
tendency to donate lone pair of electron C6 H5 NH3+ . hyperconjugating structures, and hence more will be the
Anilium ion stability of the compound.
71. (c) The electronegataive atom in the carbon chain
57. (b) – OH shows + R effect while C = O shows – R effect. produces –I effect.
58. (d) 59. (b) 72. (a)
60. (b) The stability of carbocation on the basis of hyper- 73. (c) The molecule in which all the atoms have complete
conjugation can be explained as hyperconjugation stabilises octet is more stable. Also, more the number of resonating
the carbocation because electron density from the adjacent structures, more will be the stability.
s-bond helps in dispersing the positive charge. (a) and (b) have complete octet but in (c) and (d)
+ H H all atoms do not have complete octet. Hence (c) and (d)
H H H H H H
| |+ | + | | | | are unstable.
H - C- C « H - C = C « H C = C « H - C = C 74. (b) Electron releasing groups stabilize carbocations
| | | | | | |
H H H+ H
whereas electron withdrawing groups (–CF3) destabilize
H H H H
carbocation.
In general greater the number of alkyl groups attached to a
75. (b) (CH3)4N+ has no electron pair available for donation
positively charged carbon atom, the greater is the
to act as nucleophile. Valence shell of nitrogen is
hyperconjugation interaction and stabilisation of the cation.
completely filled and has no scope for acceptance of
Thus, we have the following relative stability of carbocation.
CH3 extra electrons to act as electrophile.
| + + + 76. (c)
CH3 - C + > (CH3 ) 2 CH > CH3 CH 2 > CH3 77. (c) Cyclopropylmethyl carbocation is especially stable
| because of conjugation between the bent orbitals of the
CH3 cyclopropyl ring and the vacant p-orbitals of the
Hence, stability of carbocation is directly proportional to cationic carbon.
number of alkyl group directly attached to carbocation. + + +
61. (b) Stability order of different alkyl carbocations on the Thus, CH2 > Ph3C > CH2 CHCH2 >
V IV I
basis of hyperconjugation is :
+ +
3° > 2° > 1° > methyl (CH3)3C > CH2 CH
In t-butyl cation, the C-atom bearing the positive charge is
III IV
attached to three methyl groups therefore it possesss nine
78. (d)
a-hydrogens. It will give maximum ten hyperconjugative
79. (c) (a) –COR, –COOR Þ –M ; –OR Þ +M
structures leading to maximum stability.
62. (a) Only option (i) is correct because in (ii) – CH3 is an (b) –Cl, –NH2 Þ +M ; –CHO Þ –M
electron donating group whereas in (iii) – OC6H5 is an (c) –NO2, –CN, –SO3H Þ –M
electron withdrawing group. (d) –OH, –NR, –SR Þ +M
63. (c) 80. (b) Among the given compounds naphthelene is volatile
64. (d) Resonance structures have the same number of but benzoic acid is non-volatile (it forms a dimer). So, the
electron pairs. However, they differ in the way of best method for their separation is sublimation, which is
distribution of electrons. applicable to compounds which can be converted directly
65. (c) Resonance effect is the polarity produced in the into the vapour phase from its solid state on heating and
molecule by the interactions of two p – bonds or back to the solid state on cooling. Hence it is the most
between a p – bond and a lone pair of electrons present appropriate method.
on an adjacent atom. 81. (d)
ORGANIC CHEMISTRY–SOME BASIC PRINCIPLES & TECHNIQUES 567

82. (b) If there is a small difference (10 or less) in the boiling 102. (d) To convert covalent compounds into ionic
points of liquids fractional distillation is used, e.g. acetone compounds such as NaCN, Na2S, NaX, etc.
b.p. 329 K and methanol b.p. 338 K. 103. (b) Na2S and NaCN, formed during fusion with metallic
83. (a) Fractional distillation is used for the distillation of sodium, must be removed before adding AgNO3, otherwise
petroleum. This method is used for separating a mixture of black ppt. due to Na2S or white precipitate due to AgCN
two or more miscible, volatile liquids having close (less will be formed and thus white precipitate of AgCl will not
than 40 degrees) boiling points. (For example, a mixture of be identified easily.
acetone, b.p., 56°C and methanol, b.p. 65°C)
Na 2S + 2AgNO3 ¾
¾® 2NaNO3 + Ag 2S ¯
84. (c) Glycerol decomposes at its boiling point, hence it Black
should be purified by distillation under reduced pressure.
NaCN + AgNO 3 ¾
¾® NaNO 3 + AgCN ¯
85. (c) White
86. (a) The latest technique for the purification of organic
compounds is chromatography. These are of various types NaCl + AgNO3 ¾¾
® NaNO3 + AgCl ¯
White
like column, paper and gas-chromatography. boil
87. (d) Both silica gel and alumina are used as adsorbents Na 2S + 2HNO 3 ¾¾
¾® 2 NaNO 3 + H 2S ­
in adsorption chromatography. boil
NaCN + HNO 3 ¾¾ ¾® NaNO 3 + HCN ­
88. (a) Chromatography paper contains water trapped in it,
which acts as the stationary phase. 104. (d)
89. (c) The mixture of sugars is a homogenous one. 105. (d) Kjeldahl method is not applicable to any of the
Homogeneous mixtures of a solvent and one or more given compounds. As nitrogen of these compounds
solutes (dissolved substances) are often separated by does not change to ammonium sulphate on heating with
chromatography. Chromatography works to separate a conc. H2SO4.
mixture because the components of a mixture distribute 106. (b)
themselves differently when they are in contact with a 107. (b) In Kjeldahl’s method nitrogen is converted into
“two phase system”. One phase is stationary and the other (NH4)2 SO4, then to NH3.
is moving or mobile. The stationary phase may be a solid 108. (d) To increase the bpt of H2SO4,K2SO4 is added.
packed in a tube or a piece of paper. The mobile phase may 109. (b) 120 mL 1.0 m NaOH @ 60 mL 1.0 m H2SO4
be liquid of gaseous. Volume of excess of acid used to neutralise NH3
90. (b) 91. (b) = 100 – 60 = 40 mL
Normality of acid used = 1.0 × 2 = 2 N
Distance travelled by substance from base line( x)
92. (a) Rf = 1.4 ´ V ´ N
Distance travelled by solvent from base line( z )
Percentage of N =
x y m
For x : Rf = ; For y : Rf =
z z 1.4 ´ 40 ´ 2
93. (d) Separating funnel is used when the two liquids are = = 22.4%
5.0
immiscible. 110. (b) Percentage of N in a compound
94. (b) Fractional distillation method is used if the difference
in boiling points of two liquids is not much. 1.4 ´ Normality of acid ´ Volume of acid used
=
95. (a) 96. (a) Mass of the substance taken
97. (d) The boiling point of o-nitrophenol is less than para- Given, 0.5 M H2SO4 is used.
nitrophenol due to presence of intramolecular hydrogen Normality = Molarity × n
bonding. Since p-nitrophenol is less volatile than o-nitrophenol
due to presence of inter molecular hydrogen bonding Mol.mass 98
where n = = =2
hence they can be separated by steam distillation. Eq.mass 49
98. (b) \ Normality = 0.5 × 2 = 1 N H2SO4
99. (b) Hydrazine (NH2NH2) does not contain carbon and Volume of acid used to neutralise NH3 = 10 cm3
hence on fusion with Na metal, it cannot form NaCN; Mass of organic compound taken = 0.25 g
consequently hydrazine does not show Lassaigne’s test
for nitrogen. 1.4 ´ 1 ´ 10
\ %N = = 56.
0.25
100. (a) Prussian blue Fe 4 [Fe(CN) 6 ]3 is formed in
Lassaigne test for nitrogen. 32 Wt.of BaSO 4
111. (b) % of S = ´ × 100
6NaCN + Fe2+ ¾® Na4[Fe(CN)6] + 2Na+ 233 Wt.of compound
3Na4[Fe(CN)6] + 4Fe3+ ¾® Fe4[Fe(CN)6]3 + 12Na+
Prussian blue 32 0.233
= ´ ´ 100 = 10 %
101. (c) 233 0.32
EBD_8350
568 CHEMISTRY

62 Wt.of Mg 2 P2 O7 2. (2)
112. (b) Percentage of P = × × 100 ..
222 Wt.of compound
..
OCH3

62 1.332 ..
=
222
×
2.79
× 100 = 13.33% C
+
..
OCH3

113. (b) V1 = 68.6 mL, P1 = 756 mm, T1 = 300 K OCH3 C=O


V2 = ?, P2 = 760 mm, T2 = 273 K CH3
P1V1 P2 V2
= +
T1 T2 3. (3) C = C - C, C = C - C, C = C-C
P1V1 T2 + +
At NTP, vol. of N2, V2 = T × P 8
1 2 4. (5) X = C4H7Cl Degree of unsaturation = 5 - = 1
756 ´ 68.6 273 2
= ´ = 62.09 mL
300 760
CH2Cl CH3 CH3 CH3
Percentage of nitrogen in organic compound Cl
Cl
28 V 28 62.09
= ´ 2 ´100 = ´ × 100 = 14.93%
22400 w 22400 0.52 Cl Cl
114. (b) Molar mass of AgBr = 108 + 80 = 188 g mol–1 cis– and trans–
80 0.12
Percentage of Br2 = ´ ´ 100 = 34.04% H CH3
188 0.15 H
H
115. (b) 5. (1) CH3 CH3 CH3 + Br –
116. (b) K 2SO 4 raises bpt. and CuSO 4 acts as catalyst. ¾ ¾® Br ¾¾ ®
Br
H
117. (c) 118. (b)
119. (b) Fluoride cannot be detected by this test because the Br
CuF2 formed loses fluorine only at a molten stage above CH3
Br
950°C, which is difficult to be obtained in the lab i.e., it is
H H Br
not volatile and hence, the presence of fluoride cannot be CH3
detected by this test. CH3 H Br
H
12 weight of CO2 Br CH3
120. (c) Percentage of C = ´ ´ 100 (meso)
44 weight of compound
+
12 0.66 6. (8) Among the given compounds/ions C H3 , BF3, AlCl3,
= ´ ´ 100 = 90%
44 0.20 GaCl3, SbCl5, PCl5, PCl3 and CH3 — Cl are electrophiles.
\ Percentage of H = 100 – percentage of C In CH3 — Cl, Cl — C bond is polar bond. Hence, carbon
= 100 – 90 = 10% will have low energy anti-bonding orbitals.

Exercise - 2 7. (6) CH3


1. (4) In diphenylmethane monochlorination at following O OH
positions will produce structured isomers *
*
4 CH3 O O 6-stereocentres
1 CH2 O
*
2 3 O
* * *
O OH
Cl CH2 CH2
OH CH3
(1) Cl (2)
Asymmetric compound
Cl
8. (5) Optically inactive: (i), (iv), (viii), (x), (xi)
CH2 CH 9. (4) C4H10O represents 4 isomeric alcohols :
(3) (4)
Cl CH3CH 2CH 2CH 2OH CH3CH 2CHOHCH3
butanol-1 butanol- 2
ORGANIC CHEMISTRY–SOME BASIC PRINCIPLES & TECHNIQUES 569

CH3 Thus, sp-carbon (CH3 – CH2 – C º *CH) has the highest


CH3 - CH - CH 2OH | electronegativity.
| CH3 - C -OH 5. (c) Two or more compounds having the same molecular
|
CH3 CH3 formula but different functional groups are called
2 -methyl- propanol -1 2 -methylpropanol- 2 functional isomers. Functional isomer of alcohol is ether,
Further, butanol-2 shows optical isomerism, as it has one aldehyde is ketone and cyanide is isocyanide. But alkyl
asymmetric carbon atom and exists into two optically active halides do not show functional isomerism.
forms shown below : 6. (d) Essential oils are insoluble in water and have high
vapour pressure at 373K but are miscible with water vapour
CH3 CH3
| | in vapour phase, it means these are steam volatile. Thus,
H — C — OH HO — C — H steam distillation technique is a suitable method for the
| |
C2H5 C2H5 extraction of essential oils.
7. (d) Thin layer chromatography (TLC) involves separation
Enantiomers
of substances of a mixture over a thin layer of an adsorbent
10. (3) Three of the given four names have only one possible coated on a glass plate.
structure. The fourth one, sec-pentanol has three possible A thin layer of an adsorbent is spread over a glass plate and
structures and hence it can not be ambiguous. glass plate is placed in an eluant. As eluant rises, components
Three structures possible for sec-pentanol are as follows: of the mixture move up along with the eluant to different
OH OH distances depending on their degree of adsorption and
| |
CH3CHCH 2CH 2CH3 , CH3CHCH(CH 3 ) 2
separation takes place. Therefore, TLC technique will give
the best results in identifying the different types of ink used
OH at different places in the documents.
| 8. (b) Partition chromatography is based on continuous
and CH3CH2CHCH 2CH3
differential partitioning of components of a mixture between
Exercise - 3 stationary and mobile phases. Paper chromatography is a
type of partition chromatography.
CH3 9. (a) Stability of the given cations can be understood by
7 6 5 4| 3 2 1 the following structures:
1. (a) CH3 – CH2 – CH2 – C – CH – CH2 – CH3 + +
| | ..
CH3 CH CH3 ; CH3 - C H - O . . - CH3 ;
CH3C2H5 (I) (II)
+ I – effect of the two Strong + R – effect of – OCH 3
3-Ethyl-4, 4-dimethyl heptane
methyl groups stabilises group stabilises the carbocation
O O the carbocation
|| 3
5 2 1 || +
2. 4
(d) CH3 - C - CH 2 - CH 2 - C - OH CH 3 - C H - CH 2 ®- OCH 3
(III)
4-oxopentanoic acid - I effect of -OCH 3 group
Note : Carboxylic acid (—COOH) has more priority than destabilises the carbocation
Hence, the correct order of decreasing stability of
ketone ( C= O).
carbocation will be : II > I > III
3. (b) Cl 10. (b) 4 3
1 NO2 H3C — CH — CH — CH3
| |
2 5 CH 2 CH2
2
3
| |
4 6 CH3 1 CH3

CH3 3, 4-dimethylhexane

1-chloro-4 -methyl-2 -nitrobenzene 11. (a) Electronegativity of Cl, Br, C and Mg follows the order
For tri or higher substituted benzene derivatives, the Cl > Br > C > Mg, thus chlorine has the greatest –I-effect
compounds are named by identifying substituent positions and disperse the positive charge on ‘C’ atom most
on the ring by following the lowest locant rule. effectively.
4. (c) Electronegativity of carbon atom depends on its state Hence, *CH3 — CH2 — Cl has the greatest positive charge.
of hybridisation. More the s-character more will be the 12. (d) In all the given ionic species, the negative charge is
electronegativity. dispersed which stabilises them. Here, the negative charge
sp3 < sp2 < sp is dispersed by two factors, i.e., +R-effect of the carboxylate
s-character : 25% 33% 50% ion (conjugation) and – I-effect of the halogens.
EBD_8350
570 CHEMISTRY

As it can be clearly seen in the given structures, that \ 20 × 10– 3 mol NH3 ¾® 20 × 10– 3 × 14 nitrogen
+R-effect is common in all the four species, therefore, \ 0.75 g of sample contains
overall dispersal of negative charge depends upon the
number of halogen atoms and their electronegativity. 14 ´ 20 ´ 10 - 3
= ´ 100 = 37.33%
Since, fluorine has the highest electronegativity and two 0.75
F-atoms are present in option (d), thus, dispersal of 17. (d) In both the molecules the bond moments are not
negative charge will be maximum in it, thus it is most stable. canceling with each other and hence the molecules has a

O O resultant dipole and hence the molecule is polar.
F || F



18. (c) All of these compounds show tautomerism
CH C—O CH C——O º
F F 19. (a) Nucleophile is a species that provide electron while
F O
species which are deficient of electrons are termed as lewis
CH C –
F O acid, hence nucleophiles are usually lewis bases.
Note : In structure (a), methyl group increases the density 20. (b) Only structure (III) has hydrogen which is in
on C-atom. conjugation with free radical. So, hyperconjugation is
13. (c) When an electrophile attacks CH3 – CH = CH2, there possible in (III) only.
are two possibilities of an intermediate formed: 21. (b) p bond is transferred after leaving Cl–

Å
+
CH3 CH CH2 + H+ CH3 CH CH3 CH3 – CH=CHCH2 — Cl ¾® CH3 – CH=CH—CH2 ¬®
(2° carbocation)
+
CH 3—CH—CH=CH 2
Å 22. (b) Wt. of organic substance = 0.25 g
CH3 CH CH2 + H+ CH3 CH2 CH2 V1 = 40 mL, T1 = 300 K
(1° carbocation)
P1 = 725 – 25 = 700 mm of Hg
As 2° carbocation is more stable than 1° carbocation thus
P2 = 760 mm of Hg (at STP)
first addition is more feasible.
T2 = 273 K
Note : Stability of carbocations is the basis of
Markownikoff's rule. P1V1 P2 V2
=
14. (b) Arrow denotes the direction of movement of T1 T2
electrons.
V2 (Volume of nitrogen at STP)
Å
CH 3 - Br ¾¾
® CH 3 + Br
273 ´ 700 ´ 40
Since, Br is more electronegative than carbon, hence = = 33.52 mL
300 ´ 760
heterolytic fission occurs in such a way that CH3 gets a
positive charge and Br gets a negative charge. Percentage of nitrogen
15. (b) Addition of HCl to an alkene takes place in two steps 28× volume of N 2 at STP ×100
as follows: = 22400 × wt. of organic substance
Step I : p-bonds creates an electron cloud, electrophile
(H+) from H — Cl attacks the electron cloud and a 28 ´ 33.52 ´ 100
= = 16.76%
carbocation is formed. 22400 ´ 0.25
H
+ —C ¾®
+ |
H C— C—C 23. (c) H3C — C = CH — CH 2

|
H Ph
(carbocation)
1- Phenyl-2-butene
Step II : The chloride ion attacks the carbocation formed. The two groups around each of the doubly bonded carbon
are different. Hence this compound can show cis-and trans-
Å –
C—C + Cl ¾® C—C isomerism.
| | |
H Cl H 24. (d) CH3— C º C–
16. (a) 10 mL, 1MH2SO4 = 20 mL, 1M NH3 No. of s bp - 1 ù
Q wt of N in one mole NH3 = 14 lp -1 û® 2 and hybridisation is sp.
ORGANIC CHEMISTRY–SOME BASIC PRINCIPLES & TECHNIQUES 571

25. (b) Pent-l-ye and glycerol are separated by distillation


under reduced pressure.
Under the reduced pressure the liquid boil at low I

temperature and the temperature of decomposition will not


reach. e.g. glycerol boils at 290 °C with decomposition but
at reduced pressure it boils at180 °C without decomposition.
26. (d) 27. (c)
28. (d) Compound (d) is nonaromatic and hence least
reasonance stabilized whereas other three are aromatic.
sp2 sp2 sp sp
29. (b) CH2 = CH – C CH
30. (d) –NO2 group is meta-directing group
NO2 NO2
+
¬¾®
+
Y H Y H
(Less stable due to more e– withdrawing effect of –NO 2) Exercise - 4
NO2 NO2 NO2 1. (a)
+ + 2. (a)
¬¾® ¬¾®
H H H OH 1 OH
Y + Y Y 4 2 2
OH 4 4 4
(More stable due to less e– withdrawing effect of –NO2) greater no. 2 15 2
5 5 3 1
of resonating structures. 3 1 3 3
31. (a) – I Effect increases on increasing electronegativity OH
of atom. So, correct order of – I effect is
–NH2 < – OR < – F. OH 1 3 1
2 3 2 4
*Most appropriate answer is option (a), however option 3 1
4
2 OH 4 2
1 3
(b) may also be correct answer. OH OH
32. (b) NO2 All have different IUPAC names. Thus, none of identical.
4
3 a
a
2 Cl
1 3. (c) (6a -H)
CH3
a
2-Chlor o-1-methyl-4-nitrobenzene
a
CH3 4. (a) 1, 3 and 4 compounds are 3-ethyl-4-methyl hexane.
33. (c) H3C – C – CH3 H3C – CH – CH2 – CH3 5. (c) CH3 H
Tertiary butyl carbocation Secondary butyl carbocation
(9 a -H atoms) (5 a -H atoms) C=C H

CH3 C
t-Butyl carbocation is more stable due to hyper-conjugation.
34. (a) Paper chromatography is a type of partition CH 3 COOH
chromatography.
It has chiral asymmetric carbon atom (marked *) so it shows
optical isomerism. Two groups attaced to C = C are same
35. (b) (–CH3), so it does not show geometrical isomerism.
6. (a) Stability of carbocation µ no. of µ–H present on
carbocation.
EBD_8350
572 CHEMISTRY

··+
O 2NO + 2Cu ¾ ¾® 2CuO + N2
7. (c) It has complete octet for all atoms. 2NO2 + 4Cu ¾ ¾® 4CuO + N2 etc.
Halogens will be removed as AgX. In case of sulphur SO2
8. (b) In general, the weaker the base, the better is its leaving formed will be removed as PbSO4.
tendency. Basic character of the species, in turn can be 13. (a) In the carbocation (a), the positive charge is located
ascertained by knowing the acidic character of the in a position relative to the nitrogen such that the lone pair
corresponding conjugate acids. of electrons on the nitrogen can be conjugated with
carbocation. Therefore, due to resonance effect this
F3CSO 2OH > H3C SO2OH carbocation is more stable among all these carbocations.
O – O –
O OH O OH
Va IVa
R R
+ ¬¾®
> H3 O > HF > NH 3 Å
Ia N OH NÅ OH
III a IIa
9. (c) In resonance, only the group which is coplanar with the H H
skeleton can participate in resonance, in case the group is out 14. (c) I and III will be most stable cabocation because in (I)
of plane of the ring (skeleton), it can’t participate in resonance. nitrogen can donate its lone pair of electrons by resonance
+
O I O O I O whereas in II allylic system is present which is more stable
+ + + +
N N N N than tertiary carbocation (IV).
–O
O– –O
O–

+ +
N N
O O– 15. (b)
–O O–
2,4,6 – Trinitroiodobenzene
(Picryl iodide)
Here only the p- nitro group is involved in resonance with
the benzene, the two o-nitro groups do not participate in
resonance as they are pushed out of the plane of the ring
by bulky iodine atom, hence the C–N bond length of the
o-nitro groups is very close to C–N single bond. Thus
a = b >c.
O 1 NHCH3
10. (c) 2' 2 4
3
Cl 1'
5
O
N-Methyl-2-(2'-chloroethyl)-3-keto-4-pentenamide
11. (a) More the number of H-atoms on the a-carbon of the
group R, more are the hyperconjugating structures, weaker
is the C—Br bond and hence, ethyl has two, iso-propyl
has one and tert-butyl grouphas no a-hydrogen therefore,
hyperconjugation effect decreases and hence the rate
decreases in the same order.
12. (d) When organic compound contains nitrogen, upon
combustion it will produce oxides of nitrogen soluble in Similarly CO32– and RSO3Å have all their resonating
KOH solution. The copper will convert them into N2 structures same (equivalent R.S)
13 Hydrocarbons
Exercise - 1 CH3
|
1. (c) Pentane (C5H12) exists as three chain isomers 9. (d) Neo-pentane, H3C – C – CH3, has only 1° hydrogen
|
CH3CH2CH2CH2CH3 CH3 – CH – CH2 – CH3 CH3
|
CH3 and hence gives only one monochloro derivative.
n-Pentane 2–Methylbutane (isopentane) 10. (d)
11. (a) Complete combustion of all organic compounds leads
to formation of CO2 + H2O.
CH3
| 12. (b) During cracking higher hydrocarbons (liquid) are
H3C – C – CH3 converted to lower gaseous hydrocarbons.
| 13. (c) n-Alkanes on heating with anhydrous aluminium
CH3 chloride and hydrogen chloride gas isomerise to branched
2,3–Dimethylpropane (neo pentane) chain alkanes.
14. (c) A conformation is defined as the relative arrangement
2. (c) 3. (d)
of atoms or groups around a central atom, obtained by the
4. (d) CH4, C2H6 and C3H8 can have only one structure but
free rotation of one part of the molecule with respect to
C4H10 can have more than one structure. Possible
rest of the molecule. For a complete rotation of 360º, one
structures of C4H10 are following
part may rotate through any degree say 0.1º, 0.5º, 1º etc.
H H H H giving rise to infinite number of relative arrangements of
1 2 3 4
H C C C C H group (atom) around a central atom, keeping other part
H H H H fixed.
Butane (n- butane), (b.p. 273 K)
15. (d) Spatial arrangements of atoms which can be
converted around a C – C single bond are called
H H H conformations or conformers or rotamers.
12 3
16. (c) In methane carbon atom is sp3 hybridised.
H C C C H 17. (b) Dihydrogen gas adds to alkenes and alkynes in the
HH C HH presence of finely divided catalysts like platinum, palladium
H or nickel to form alkanes. These metals adsorb dihydrogen
2-Methylpropane (isobutane) gas on their surfaces and activate the hydrogen-hydrogen
(b.p.261 K) bond. Platinum and palladium catalyse the reaction at room
5. (a) When alkyl halide is treated with sodium metal in temperature but relatively higher temperature and pressure
presence of ether, alkane is obtained, this reaction is called are required with nickel catalyst.
as Wurtz reaction. 18. (b) For statement (iii), fluorination is too violent to be
dry ether
controlled. For statement (iv), iodination is very slow and
R – X + 2Na + X – R' ¾¾¾¾® R - R ' + 2NaBr reversible reaction. It can be carried out in the presence of
dry ether oxidizing agents like HIO3 or HNO3.
CH3 – Br + 2Na + Br – CH3 ¾¾¾¾®
CH 3 - CH 3 + 2 NaBr ˆˆ† CH3I + HI
CH 4 + I2 ‡ˆˆ
6. (c) Methane cannot be prepared by Wurtz reaction, 5 HI + HIO3 ® 3I 2 + 3H 2 O
Kolbe's method and reduction with H2. Since CH4 has one
carbon atom, so pure methane can be producd by 19. (c) Pyrolysis and cracking are same process.
decarboxylation of CH3COONa. 20. (b) Rotation around a C-C single bond is not completely
7. (b) Sodium salts of carboxylic acids on heating with soda free. It is hindered by a small energy barrier of 1-20 kJ
lime (mixture of sodium hydroxide and calcium oxide) give mol–1 due to weak repulsive interaction between the
alkanes containing one carbon atom less than the carboxylic adjacent bonds. Such a type of repulsive interaction is
acid. This process of elimination of carbon dioxide from a called torsional strain.
carboxylic acid is known as decarboxylation 21. (b) Eclipsed form has maximum torsional strain, while
staggered form has the least.
CH3COO- Na + + NaOH ¾¾¾
CaO
® CH 4 + Na 2CO3 Pt/Pd/ Ni
D 22. (a) CH 2 = CH 2 + H 2 ¾¾¾¾¾
® CH3 - CH3
Sodium ethanoate Methane
Ethene Ethane
hn Zn, H +
8. (a) Cl2 ¾¾¾¾¾¾® 2Cl• CH3 - Cl + H 2 ¾¾¾¾
® CH 4 + HCl
Chain initiation
Chloromethane Methane
EBD_8350
574 CHEMISTRY

dry ether
CH3 Br + 2Na + BrCH 3 ¾¾¾¾¾¾® CH3 - CH 3 + 2NaBr
Bromomethane Wurtz reaction
Ethane
(i) (ii) (iii)
CH 3COO - Na + + NaOH ¾¾¾
CaO
® CH 4 + Na 2 CO3
D
23. (c)
24. (b) The assertion that chlorination of CH4 does not take
(iv) (v)
place in dark is correct because it is a free radical reaction
and free radicals are obtained in presence of sun light. 40. (c) C5H10 has 1º degree of unsaturation since the isomers
25. (b) Iodination is reversible since formed HI is a strong are acyclic, all of these are alkenes. For writing the isomers,
reducing agent and reduces the alkyl iodide back to alkane. first introduce the double bond at different possible
CH4 + I2 ƒ CH3I + HI positions, and then consider the possibility of branching
in the alkyl group.
Iodination can be done only in presence of strong oxidising
agents like iodic acid which destroys the hydriodic acid. CH3CH2CH2CH =CH2 CH3CH2CH=CHCH3
1-pentene (i) 2-pentene, (cis,- trans) (ii), (iii)
26. (d) Iso pentane (C5H12)
CH3 CH3
C5H12 + 8 O2 ¾® 5 CO2 + 6 H2O | |
CH 3- CH - CH = CH 2 CH 3 CH 2 C = CH 2
AlCl3 KMnO4
27. (c) HCl 3-methyl-1-butene, (iv ) 2-methyl-1-butene, (v)
OH
CH3
28. (a) It is obtained only in (a) |
CH 3 - C = CHCH 3
COOH
NaOH/Current 2-methyl-2-butene, (vi)
41. (c) The condition for geometrical isomerism is
Kolbey’s
COOH electrolysis a a a e
C=C or C=C
29. (c) CH3—CH2COOH
NaOH
CH3CH3 b b b d
CaO, D CH2 = C(Cl)CH3 does not follow above mention condition.
(CH3COO)2 Mn NaOH 42. (c)
CH3COOH CH4
CaO, D Br
For second reaction see NCERT, hydrocarbon. | alc. KOH
43. (d) CH3– CH – CH2 – CH3 ¾¾¾®
30. (a) For (b), (c) and (d) we have to take mixture of alkyl
2-Bromobutane
halides and hence many side products will also form along
with main products. Å –H+ H3C H
CH3–CH–CH2–CH3 ¾® C=C
31. (c) The reaction is complete oxidation or combustion of H CH3
alkanes. trans-2-butene
32. (a) 33. (d) 34. (d) 35. (a) conc. H SO
44. (c) C2 H5 OH ¾¾¾¾¾¾
2 4® C H + H O
2 4 2
36. (a) Since b (from bromo) comes earlier in alphabetical KMnO4
order than c (from chloro), the correct name should be 2, 3- Note : If ethyl alcohol is taken in excess and the reaction is
dibromo-1, 4-dichlorobutene-2 and not 1,4-dichloro-2, carried out at a temperature of 433-443 K diethyl ether is
3-dibromobutene-2. formed.
37. (d) Alkenes having double bonds with two different
conc. H SO
2 4 ® C H OC H + H O
groups on each end of the double bond show geometrical 2C2 H5OH ¾¾¾¾¾¾ 2 5 2 5 2
433-443 K
isomerism. A2b2c2, A2b2cd, A2bcde. (excess)
45. (a) Peroxide effect is observed only in case of HBr.
Br Br
C C 1,2-dibromopropene Therefore, addition of HCl to propene even in the presence
H CH3 of benzoyl peroxide occurs according to Markovnikov’s
rule :
Br Br HCl
C C 2,3-dibromobut-2-ene CH3 - CH = CH 2 ¾¾¾¾¾¾®
H3C CH3 (C6 H5CO)2 O 2

38. (b) The two isomers differ in the position of the double CH 3 - CHCl - CH 3
bond so they are called position isomers. 46. (b) Markonikov’s way of addition :
39. (a) As sketched in the above question , C5H10 may be
A + B-
monosubstituted as in (i) and (iv), disubstituted as in (ii) CH 3 - CH = CH 2 ¾¾¾¾
® CH3 - CH - CH 2
and (v) and trisubstituted as in (iii) | |
B A
HYDROCARBONS 575

47. (d) Since given alkene on ozonolysis gives 2 moles of In this case the major product formed is 2- bromo-2-
propanone, hence alkene should have a double bond methylbutane i.e. option (b) is correct answer.
between two equivalent C atoms i.e. the formula should be (Note: The unexpected product results from a rearrangement
of 2° carbocation to 3°. Please note that all carbocations
H3C CH3 do not rearrange.)
C C +O3 ¾¾¾¾
Zn/H 2O
®
H3C CH3 50. (c) Alkenes combine with hydrogen under pressure and
2, 3-dimethyl but-2-ene
O in presence of a catalyst (Ni, Pt or Pd) and form alkanes.
½½ H /Pd
2CH3 —C—CH3 Butene – 1 ¾¾¾¾
2 ® Butane
Propanone 51. (c)
48. (d) In presence of peroxide, HBr adds on alkenes in 52. (d) A doubly bonded carbon atom having an alkyl group
anti–markovnikov’s way, thus is oxidised to aldehyde which is further oxidised to
carboxylic acid.
Peroxide
H3CCH = CH 2 + HBr ¾¾¾¾® H3CCH 2CH 2 Br (i) KMnO ,OH -
4
CH3CH 2CH = CH CH3 ¾¾¾¾¾¾¾®
Pr opene n - propyl bromide + (ii) H
Kharasch observed that the addition of HBr to
unsymmetrical alkene in the presence of organic peroxides CH3CHO + CH3CH 2CHO
follows an opposite course to that suggested by ¯ ¯
Markovnikov. This is termed as anti-Markovnikov or CH3COOH CH3CH 2COOH
peroxide effect. 53. (d) Completing the sequence of given reactions,
49. (b) We know that in case of an unsymmetrical alkene
O
there is the possibility of forming two products. In such CH 3 – CH = CH - CH 3 ¾¾3¾
®
cases the formation of major product is decided on the
basis of Markovnikov’s rule which is rationalized in terms O
of stability of the intermediate carbocation. Also remember
Zn / H 2O
that 3° carbocation is more stable than 2° carbocation and CH3 – CH CH – CH3 ¾¾¾¾®
2° carbocation is more stable than 1° carbocation.
H3C O O
| CH – CH = CH2
|

‘A’
H3C 3– methyl-1-butene (ozonide)
CH3 + 2CH3CHO+ H 2 O + ZnO
CH – CH2 – CH2 (1°)
CH3 |
'B '
|

+H + 54. (c) The given molecular formula suggests that the


CH3 + aldehyde formed will be acetaldehyde hence the alkene
| CH – CH – CH3 (2°)
|

CH3 will be
Of the two possibilities 2° carbocation is more stable so (i) O
CH3CH = CHCH3 ¾¾¾¾¾
3
® 2CH3CHO
(ii) Zn/H 2O
the product of the reaction expected was predominantly 2 - Butene
one formed by 2° carbocation i.e. 55. (a) The addition of HBr to symmetrical alkenes is not
CH3 – CH – CH – CH3 affected by the presence or absence of peroxide.
| | OH –
CH3 Br 56. (d) CH 2 = CH 2 + KMnO4 ¾¾¾ ®
2– Bromo-3-methylbutane
CH2 - CH 2
However some electrophilic addition reaction form | | + MnO 2 + KOH
products that are clearly not the result of the addition of OH OH
electrophile to the sp 2 carbon bonded to the most Glycol
hydrogens and the addition of a nucleophile to the other 57. (b) When unsymmetrical unsaturated hydrocarbon reacts
sp2 carbon. with unsymmetrical reagent, then negative part of reagents
In the above cases the addition of HBr to 3-methyl-1-butene attacks that carbon which has less H-atom. [Markovnikov's
the two products formed are shown below. rule] I
CH3 |
| 400 °C
CH 3 - CH = CH 2 + HI ¾¾¾¾ ® CH 3 - CH - CH 3
CH3 – CH – CH = CH2 + HBr ¾®
Propene-2 2-Iodopropane
CH3 CH3
| | 58. (a) CH 2 = CH 2 + Br2 ¾¾¾
CCl
4 ® CH - CH
Ch3 – CH – CH – CH3 + CH3 – C – CH2 – CH3 2 2
| |
| |
Br Br Br
Br
1, 2-Dibromoethane
2-Bromo-3-methylbutane 2-Bromo-2-methylbutane
EBD_8350
576 CHEMISTRY

Conc. H SO 66. (c) In compounds


59. (b) CH3 - CH = CH 2 + H 2 O ¾¾¾¾¾¾¾¾
2 4 ®
Markovnikov addition CH3 CH3
CH 3 - CH - CH3
|
OH
Isopropyl alcohol
first has more dipole moment than second.
Therefore its boiling point will be higher. Melting point
60. (b) For statement (ii), Alkenes are also known as olefins. depends on symmetry therefore I has higher melting point
For statement (iv), Carbon–Carbon double bond in alkene than II. Steric crowding in II is more than I therefore I is
consists of one sigma and one p bond. more stable than II.
H s H 67. (b)
C C 68. (a) When 3, 3 dimethyl 2-butanol is heated with H2SO4
H H the major product obtained is 2, 3 dimethyl 2-butene.
61. (a) For statement (i), cis form of alkenes have significant
CH3
dipole moment whereas dipole moment of trans form is |
almost zero. For statement (iv), due to different Conc.H SO
CH3 - CH - C - CH3 ¾¾¾¾¾¾
2 4®
arrangements of atoms or groups in space cis and trans | |
isomers differ in their properties like melting point, boiling OH CH3
point, dipole moment, solubility etc. 3,3- Dimethyl 2-butanol

62. (a) Pd /C CH3 CH3


CH º CH + H 2 ¾¾¾® CH 2 = CH 2 +
Ethyne Ethene CH3 – CH – C – CH3 ¾® CH3 – CH – C – CH3 ¾®
H H H H + OH2 CH3 CH3
b a
alc.KOH
H C C H ¾¾¾¾ ® C =C (2° carbocation)
D
H X H H CH3 CH3
(X = Cl, Br, I) +
CH3 – CH – C – CH3 ¾® CH3 – C = C – CH3
CH 2 Br - CH 2 Br + Zn ¾¾
® CH 2 = CH 2 + ZnBr2 CH3 CH3
(3° carbocation) 2, 3 dimethyl -2-butene
H H
b a
Conc.H 2SO 4
H C C H ¾¾¾¾¾¾ ® CH 2 = CH 2 + H2 O CH 2 CH3
D H2 O
H OH Ethene 69. (a) P + H 2SO4 ® | ¾¾¾ ®
CH 2 CH 2 HSO 4
Ethanol
63. (a) This reaction follows anti Markovnikov’s rule CH3
| + H 2SO 4
Peroxide
CH3CH2CH = CH2 + HBr ¾¾¾¾® CH 2OH
CH3 CH2 CH2 CH2 Br Except ethyl alcohol, no other primary alcohol can be
In this reaction anti Markovnikov’s addition is explained prepared by this method as the addition of H2SO4 follows
on the basis of the fact that in the presence peroxides, Markovnikov’s rule. Generally secondary and tertiary
Addition takes place via free radical mechanism rather than alcohols are obtained.
the ionic mechanism. 70. (a) Given
64. (d) The two hydrogen atoms on first carbon and the two Cl alc./KOH O /H O
H-atoms on the third carbon atom lie in perpendicular A ¾¾¾
2 ® B ¾¾¾¾¾
® C ¾¾¾¾®
3 2 CH 2O
Hydrocarbon
planes. The central carbon atom is sp-hybridised while
terminal carbon atoms are sp2-hybridised Since hydrocarbon C give only CH2 on ozonolysis, C
should be CH2 = CH2 hence going backward A should be
65. (b) Stability of an alkene depends upon the heat of
ethane. Thus the reactions are
hydrogenation of an alkene. The heat of hydration is the
standard enthalpy change in hydrogenation of on alkene. Cl /hn alc.
CH3CH3 ¾¾¾¾
2 ® CH3CH 2Cl ¾¾¾® CH 2 = CH 2
The lower the heat of hydrogenation of an alkene higher KOH
(A) (B) (C)
will be the stability.
O /H O
Order of stability Heat of hydrogenation ¾¾3 ¾
D
2¾®
HCHO
( D)
(kJ/mol)
71. (c)
trans-2-butene – 115.5 72. (d) Nature of halogen atom and the alkyl group both
cis-2-butene – 119.6 and determine rate of the reaction.
1-butene – 126.8 respectively. 73. (a) Observe stability of C+ ion.
HYDROCARBONS 577

74. (d) As it is highly stable free radical. alc.KOH


75. (b) R - CH 2 - CCl 2 - R ¾¾¾¾® R - CH = CCl - R
NaNH
H ¾¾¾¾
2®R - C º C - R
76. (a) CH3CH = CHCH2CH2CH3 ¾®
2
CH3(CH2)4CH3
(X ) ( Y) 82. (c) CaC2 + 2H2O ¾
¾® HC º CH + Ca (OH)2
Calcium Acetylene
O3 carbide
O 83. (d) Among isomeric alkanes, the straight chain isomer
CH3CH CHCH2CH2CH3 ¾® CH3CHO has higher boiling point than the branched chain isomer.
O O (I) The greater the branching of the chain, the lower is the
+ boiling point. Further due to the presence of p electrons,
CH3CH2CH2CHO these molecules are slightly polar and hence have higher
(II) boiling points than the corrosponding alkanes.
84. (a) Terminal alkyenes give a white precipitate easily on
Br reaction with ammonical silver nitrate solution.
| dil H SO
alc. KOH 85. (b)
77. (c) CH3 – C – CH2CH3 ¾ ¾ ¾ ® CH3CH == CHCH3 CH º CH + H 2 O ¾¾¾¾¾¾
2 4 ®
HgSO 4 .60° C
| 2-Butene
H (main product) [CH 2 = CHOH] ¾¾
® CH3CHO
'X ' unstable

86. (b) HCl


CH3 CH3 CH º CH + HCl ® CH 2 = CH - Cl ¾¾¾
®
| | O
O3 Cl
CH3 – CH – CH = CH2 ¾® CH3 – CH – CH CH2 CH3 – CH
Cl
O O 1, 1-Dichloroethane
Zn/H2O 87. (b) Alkynes having terminal –C º H react with Na in
CH3 liquid ammonia to yield H2 gas.
| Na in
CH3 – CH – CHO + HCHO CH3CH 2 C º CH ¾¾¾¾¾
®
liquid NH 3
'Y ' 'Z '
1
78. (c) CH3CH 2C º C – Na + + H 2 ( g )
2
Br 88. (a) Acetylene reacts with the other three as:
CH3CH2CH = CHCH2CH3 ¾ ®2
CH3CH2CH – CHCH2CH3
(X) | | CH2 CH3
Na + HCl
HBr Br Br CH º CNa ¬¾¾¾ CH º CH ¾¾® ¾®
conc. H2SO4 liq. NH3
CHCl CHCl2
CH3CH2CH2CHCH2CH3 CH3CH2CH2CHCH2CH3
| | [AgNO + NH OH]
HSO4 (Y ) Br CH º CH ¾¾¾¾¾¾¾®
3 4
AgC º CAg + NH 4NO 3
white ppt.
NaOH
89. (c) 90. (d)
[O]
CH3CH2CH2 – C – CH2CH3 ¬ ¾ CH3CH2CH2CHCH2CH3 40% H SO
|| | 91. (c) CH3 - C º CH + H 2O ¾¾¾¾¾®
2 4
1% HgSO4
O OH
(Z ) Rearrangement
3-Hexanone CH3 – C = CH2 ¾¾¾¾¾¾® CH3 – C = CH3
| P
79. (c) When both double and triple bonds are present, then OH O
triple bond is considered as the principal group. Acetone
CH3 - CH = CH - C º CH 92. (a)
5 4 3 2 1
3CH 3 - C º CH ¾¾
® H3C H3C
80. (b) Three alkynes are possible for the formula C5H8.
(i) CH3CH2CH2C º CH (ii) CH3CH2C º CCH3
CH3
| H3C
(iii) CH3 – CH – C º CH
Mesitylene or 1, 3, 5-trimethyl benzene
81. (c) On heating an ethylene chloride (1, 1 dichloro ethane)
with alcoholic potash followed by sodamide alkyne is Lindlar's
93. (b) CH º CH + H 2 ¾¾¾¾¾
® CH 2 = CH 2
obtained Acetyle Catalyst
Pd. BaSO4 Ethylene
EBD_8350
578 CHEMISTRY

94. (d) Addition – 100. (a) The acidity of acetylene or 1–alkyne can be explained
Ni on the basis of molecular orbital concept according to
CH º CH + 3H 2 ¾¾® CH 3 - CH 3 which formation of C—H bond in acetylene involves sp-
Substitution – hybridised carbon atom. Now since s electrons are closer
1 to the nucleus than p electrons, the electrons present in a
CH º CH + Na ¾¾ ® CH º C- Na + + H 2 bond having more s character will be correspondingly more
2
closer to the nucleus.
Polymerization –
Thus owing to high s character of the C—H bond in alkynes
(s = 50%), the electrons constituting this bond are more
hot Cu tube strongly held by the carbon nucleus i.e., the acetylenic
3CH º CH ¾¾¾¾¾¾® C6H 6
Polymerization carbon atom or the sp orbital acts as more electronegative
Benzene
species than the sp2 and sp3 with the result the hydrogen
Br present on such a carbon atom (º C—H) can be easily
|
95. (b) CH3C º CH + 2HBr ¾¾
® CH3 - C - CH3 removed as a proton.
| 101. (a) Only terminal alkynes show acidic nature. Ethyne is
Br more acidic than propyne. But-2-yne is not acidic as it
96. (b) does not contain any hydrogen attached to sp hybridised
97. (b) Thin film of polyacetylene can be used as electrodes carbon.
in batteries. These films are good conductors, lighter and
cheaper than the metal conductors. H2
102. (a) H 3C – C º C – CH 3 ¾¾¾¾® C=C
98. (b) For statement (ii), alkynes on reduction with Pd/C Lindlar's
catalyst H (A)
H
form cis-alkenes. For statement (iv), propyne on reduction
with Pd/C forms propene. Propene does not show Ozonalysis
geometrical isomerism. Only those compounds show ¾¾¾¾® CH3CHO
geometrical isomerism which have two different atoms or (B)
groups attached to each carbon atom.
wacker
CH3 H C2H4 ¾¾¾¾
process
C C 103. (c) Due to the maximum percentage of s character (50%),
H H the sp hybridised orbitals of carbon atoms in ethyne
Propene
molecules have highest electronegativity; hence, these
99. (a) attract the shared electron pair of the C-H bond of ethyne
to a greater extent than that of the sp2 hybridised orbitals
(i) HC º C - CH2 - CH2 - CH2 - CH3
of carbon in ethene and the sp3 hybridised orbital of carbon
Hex -1- yne
in ethane. Thus in ethyne, hydrogen atoms can be liberated
(ii) CH3 - C º C - CH2 - CH2 - CH3 as protons more easily as compared to ethene and ethane.
Hex -2- yne 104. (c)
(iii) CH3 - CH 2 - C º C - CH 2 - CH3 KMnO4
Hex -3- yne C C—CH2—CH2—CH2—CH==CH
D

(iv) HC º C - CH - CH 2 - CH3
|
CH3 COOH + HOOC—CH2—CH2—CH2—COOH + HOOC
3- Methylpent -1-yne

(v) HC º C - CH 2 - CH - CH3 105. (b) CH3—CH==C==CH—CH3 ¾® 2 CH3COOH + CO2


| 106. (c)
CH3
4- Methylpent -1-yne
Na
O
Given compound V
(vi) CH3 - C º C - CH - CH3 Liq. C Na
NH3
| O
CH3 O
4- Methylpent -2-yne Br—CH2 —CH2 —I
CH3
| O Br
(vii) HC º C - C - CH 3
| 107. (c) It is an addition reaction, so it does not show the
CH3 acidic nature of ethyne.
3,3- Dimethylbut-1-yne
HYDROCARBONS 579

high temp.
108. (d) n CH2 = CH2 ¾¾¾¾¾¾®
pressure, catalyst
(—
CH2 – CH2 —
)n O O
O
A linear polyethene polymer is formed when ethene O
polymerises. 123. (c) + O3 ¾® O
109. (d) O
O
110. (c) They have a relatively high percentage of carbon. O O
111. (c)
Benzene triozonide
112. (c) Benzene does not show addition reactions like other
unsaturated hydrocarbons. However, it show substitution 124. (b) Friedel- Craft reaction occurs in presence of an
reactions. Due to resonance all the C – C bonds have the attacking reagent which is an electrophile (AlCl3).
same nature, which is possible because of the cyclic 125. (a) According to experimental evidences, electrophilic
delocalisation of p-electrons in benzene. Monosubstitution substitution reactions are supposed to proceed via the
will give only a single product. following three steps:
113. (d) Benzene can be obtained by all the compounds given. (1) Generation of the electrophile
(2) Formation of carbocation intermediate
Red hot tubs (3) Removal of proton from the carbocation intermediate
3CH º CH ¾¾¾¾®
873 K
..
OH 126. (b) HO3SO - H + H - O
.. - NO 2
H
D
+ Zn ¾® + -
H-O
.. - NO2 + HSO 4
H
Cl Å
+
H-O
.. - NO 2 H 2 O + NO 2
Protonated Nitronium
Ni-Al alloy
+ 2 [H] ¾¾¾¾® nitric acid ion
NaOH
In the process of generation of nitronium ion, sulphuric
114. (a) This is an example of decarboxylation reaction. acid serves as an acid and nitric acid as a base.
127. (a) p-Electrons of benzene rings are delocalised
COONa throughout the molecule. This makes the molecule very
stable. The stability resists breaking of double bonds for
NaOH + CaO addition.
¾¾¾¾¾® + Na2CO3 128. (d) Polynuclear hydrocarbons are not produced in human
body by any biochemical reaction However, when they
115. (a) Y = –COOH because it is meta directing group while enter into human body, they undergo various biochemical
–NH2, – OH and –Cl are o and p directing groups. reactions which finally damage DNA and cause cancer.
NO2 NO2 129. (a)
AlCl3
(A) C6 H6 + Cl 2 ¾¾¾® C6 H5Cl
HNO3 Br 2
116. (a) ¾¾® ¾¾® Benzene Chlorobenzene
H2SO4 FeBr3
Br AlCl3
(B) C6 H6 + CH3Cl ¾¾¾® C6 H5CH3 + HCl
117. (c) Cl exhibits –I effect and +M effect. Toluene

Ni (C) C6 H 6 + CH 3 COCl ¾¾
® C6 H 5COCH3 + HCl
118. (b) + 3H2 ¾¾®
Methyl phenyl
ketone
Benzene Cyclohexane
119. (d) (D) C6 H5 - CH3 ¾¾¾¾
KMnO
4 ® C H COOH
6 5
NaOH
CH3 Toulene
130. (d) 131. (a)
anhydrous AlCl 132. (c) In arenes, p electrons are delocalised, hence arenes do
120. (c) + CH3Cl ¾¾ ¾ ¾ ¾ ¾
¾3® + HCl not undergo addition reactions easily. Aromatic compounds
(arenes) are highly stable and show resonance. eg. benzene
121. (b) Benzene can be obtained by polymerisation of is the simplest example.
acetylene. 133. (d) In benzene due to delocalisation of p - electrons, all
the C – C bond lengths are equal as each C – C bond has
Red hot tube
3HC º CH ¾¾¾¾¾® some double bond character and thus the bond length is
500°C
between single and double bond, i.e., between C2H6 and
122. (d) C2H4.
EBD_8350
580 CHEMISTRY

134. (a) Cl
(2) ; 19s, 5p Þ 19/5 = 3.8
Cl2, anly AlCl3 (Substitution)
¾¾¾¾¾¾¾¾¾® + HCl

Chlarobenzne (3) ; 26s, 7p Þ 26/7 = 3.71


¾ Cl
Cl Cl
Benzene 3Cl2, sunlight (Addition)
¾¾¾¾¾¾¾¾¾®
Cl Cl
(4) ; 25s, 6p Þ 25/6 = 4.166
Cl
Benzene hexachloride
(BHC)
135. (b) –Cl group is o-, p-directing due to +R effect ; however Exercise - 2
it is deactivating due to strong –I effect of Cl (difference
d b
from other o-, p-directing groups which are activating).
e a
The net result is that chlorobenzene undergoes o, 1. (6) c
p-substitution, but with difficulty. f d
e
136. (b) C6 H5COOH + NaOH ® C6H5COONa + H2O
(X) There are six positions in the given compound which
distill will give monochloroderivatives.
C6 H5OH + Zn ¾¾¾® C6H 6 + ZnO
(Y) Cl Cl Cl Cl Cl
137. (a) Huckel rule is not obeyed. It has only four electrons. Cl
Further it does not have continous conjugation. 2. (4)
138. (d) Rate of F.C.R. will be highest in (III) because both the
Cl Cl
ortho psotions of benzene nucleus are connected with + R Cl
group. Rate of F.C.R. will be least in (I) because both the 3. (7) Seven possible isomers are
ortho positions of benzene ring are substituted by – R
groups.
139. (d) NO2 and SO3H are meta directing groups so (a), (b)
and (c) are wrong but isopropyl group is o/p directing
hence, (d) is correct.
140. (d) +R effect of CH3 is greater than —C—N—C—

O O
Å
group hence NO2 will attack at ortho position of CH3
group.
141. (a) As it has least deactivated benzene nucleus.
142. (a)
143. (a) Aromatic are most stable followed by non-aromatic
and anti-aromatics are least stable.

CH3
AlCl3 | +
144. (b) ¾¾® – CH3 – C – CH3 ¬¾
¬¾¾¾

– AlCl4
Cl |
H
1° carbocation
(less stable)
CH3
|
¬¾¾
+ CH3 – C+ – CH3
–H
3° carbocation
tert-Butylbenzene (more stable) 4. (5) 1, 3-Dichlorocyclobutane can exist in cis and trans
forms. trans-1, 2-Dichlorocyclobutane can exist
145. (c)
i n (+) – and (–) – forms. However, cis-1, 2
Dichlorocyclobutane has a plane of symmetry and
(1) ; 12s, 3p Þ 4
hence it can exist as meso isomer.
5. (4)
HYDROCARBONS 581

C C C
| | |
C - C- C º C, C = C- C = C, C-C= C= C

Exercise - 3
1. (d) We know that, as the number of carbon atom
increases, boiling point increases while for the same alkane,
boiling point decreases with branching.
2, 2-dimethyl propane, CH3
|
H3C – C – CH3
|
CH3
b.pt = 282.5 K
6. (4) All alkyl bromides having carbon skeleton of
isopentane (2-methylbutane (CH3)2CHCH2CH3) will n – pentane, b.pt = 309.1 K
give isopentane via Grignard reagent. 2-methyl butane, H3 C — H 2 C — HC — CH3
|
CH3
b.pt = 301 K
n-butane, b.pt = 273 K
CH3Br 2. (a) Reactivity of halogens decreases with decrease in
electronegativity which decreases down the group.
CH3CHCHCH3 Hence the order of reactivity of alkanes with halogens is
F2 > Cl2 > Br2 > I2.
3
. 1 Also with I2, the reaction is too slow that it requires a
catalyst.
7. (4) 3. (b) The reactivity of halogens with alkanes follows the
order : F2 > Cl2 > Br2 > I2. Further, the reactivity of reduction
2
4 of alkyl halides with Zn & dilute HCl increases as the
strength of C – X (X = F, Cl , Br, I) bond decreases. Hence,
reduction of alkyl halide with Zn and dilute HCl follows
Br the order :
¾¾¾¾
2 ®
8. (6) aq.NaCl R – I > R – Br > R – Cl.
4. (a) The correct IUPAC name of the alkane will be :

H3C – CH2 – CH – CH2 – CH2 – CH – CH2 – CH3


3 4 5 6 7 8
2
CH CH2
1
CH3 CH3 CH3

3, 6-Diethyl -2-methyloctane
H / hn Cl2 /hn
¾¾¾¾ ®
5. (a) 1-butene is an unsymmetrical alkene and will follow
9. (6) C ¾¾2 ¾¾
® C
(a ) (c) Markovnikov's rule to give major product.
(1) CH3 - CH 2 - CH = CH 2 + H - Br ¾¾ ®
I-Butene
Cl Å
(b) O 3 * + H – Br ¾® + Å
CH2 – CH2Cl CH – CH3
CHO C + C H
minor more stable (2°)
OHC C CHO + CH2O product(s) trigonal planar
(1) d + l (2) carbocation
CHO
¾¾

a = 1; b = 2; c = 3 Sum =1 + 2 + 3 = 6 H H Br Å
+ ¬¾¾
C C C
| | | Br Br
10. (6) C = C - C - C, C - C = C - C, C - C- C = C
Racemic mixtures
(major products)
EBD_8350
582 CHEMISTRY

6. (d) In option (d), a carbon with double bond has two


same functional groups (CH3) attached to it. The rotation CH3 CH3
around carbon will not produce a new compound. Hence, CH3 – C – CH = CH2+ H ¾® CH3 – C – CH +– CH3
+
geometrical isomerism is not possible.
7. (c) Lesser the bond energy of hydrogen halide more will CH3 CH3
be its reactivity. Hence, the order of reactivity of given Rearrangement
compounds with propene will be HI > HBr > HCl.
8. (b) + I-effect decreases the stability of carbanion. Since, CH3
CH3 CH3 CH3
– CH3 group shows + I-effect, therefore, it intensifies the C=C ¬¾ CH3 – C – C
negative charge and hence destabilises the carbanion (A) CH3 CH3 + CH3
H
relative to (B). 2, 3 - dimethyl – 2 - butene (Stable 3° carbocation)
Also, sp hybridised carbanion is more stabilised than sp3.
CH º C > CH3 – C º C – > CH3 – CH2
– 14. (c) 1-chloro-1-methylcyclohexane.
sp sp 3
(B) (A) (C) sp CH3
Hence, the order of decreasing stability will be : B > A > C Cl
9. (d) Alkyl halides on heating with alcoholic KOH
eliminates halogen from a-carbon atom and hydrogen is
eliminated from b-carbon atom to form an alkene.
CH2 CH3 CH3
3° b-carbon
(A) CH3 — CH — CH 2Br Å Cl
| + H – Cl ¾® + Cl ! ¾®
CH3
(A)
1° b-carbon CH3
(B) CH — CH — Br CH3 CH3
3 2 Å Cl
2° b - carbon H H
(C) CH3 — CH2 — CH2 — Br + H – Cl ¾® + Cl ! ¾®
More the number of b-substituents (alkyl groups) more
stable alkene will be formed on b-elimination, more will be 15. (d) H3C
the reactivity. Thus the correct order is A > C > B H3C
10. (c) During incomplete combustion of alkanes with O3 O=C
insufficient amount of air carbon black is formed which is CH3 ¾¾¾®
Zn + H O
CH3
used in the manufacture of ink, printer ink, black pigments
2
OHC
and as filters. 5-Keto-3-methylhexanal

Incomplete
16. (a) No. of double bonds = 4
CH 4 ( g ) + O2 ( g ) ¾¾¾¾¾® C( s) + 2H 2O ( l ) No. of p bond electrons
combustion
Carbon black = 2 × no. of double bond
11. (b) The combustion reaction of ethylene is =2× 4=8
5 17. (a) Enthalpy of hydrogenation
C2H 2 + O 2 ¾¾
® 2CO 2 + H 2O
2 1
µ
Both HC CH and CO2 have sp hybridisation. stability of alkene
Cl \ III > II > I
| VO
12. (c) 2Cl - C - CH3 + 6Ag ® CH 3C º C CH 3 + 6Ag Cl 18. (b) 2C6 H 6 (g) + 9O 2 (g) ¾¾¾
2 5¾
®
410°C
| 2- Butyne
Cl O
1,1,1-Trichloroethane
CH—C
13. (b) CH3C = C - CH3 O + 4CO2(g) + 4H2O(g)
| |
CH3 CH3 CH—C
2,3-Dimethyl-2-butene
O
(CH3)2CH – CH – CH = CH 2 Maleic anhydride
|
CH3 19. (d)
contain 7 carbon atoms 20. (d) When 1, 3-dimethylcyclopentene is heated with ozone
and then with zinc and acetic acid, oxidative cleavage leads
to keto - aldehyde.
HYDROCARBONS 583

CH3 CH3 O O
O 6 || 5 4 3 2 1 ||
(i) O – 78 °C CH3 – C – CH2 – CH2 – CH – C – H
¾¾¾¾¾¾¾
3
® O C–H |
(ii) Zn- CH3COOH
CH3
CH3 CH3 5-Keto-2-methylhexanal

+
21. (a) NaNH 3 2
2 ® HCº CNa ¾¾¾¾¾¾¾¾¾¾¾¾¾¾¾¾
HCº CH ¾¾¾¾¾¾¾¾¾¾¾
H C –CH - Br
® HC º C – CH 2 – CH3
liq. NH 3
(X)
1-Butyne NaNH2
liq. NH3
+
H 3C - CH 2 – Br
H3 C – CH 2 – Cº C – CH 2 – CH3 ¬¾¾¾¾¾¾¾¾¾¾¾¾¾¾¾¾ ¾ H3C – CH 2 – C º C N a ¬¾
3-Hexyne (Y)

22. (d) In staggered conformation any two hydrogen atoms 28. (d) If two chirality centres are created as a result of an
on adjacent carbon atoms are as far apart as possible there addition reaction four stereoisomers can be obtained as
by minimising repulsion between the electron clouds of products.
s-bonds of two non-bonded H-atomic (torsional strain) H3C CH3
23. (d) + C = C HBr
¾¾¾ ®
¾® CH2 – CH – CH3 –H O 2
+
H C2H5
sp3 CH = CH – CH + Cl ¾ Cl More stable intermediate Methyl pent-2-ene
2 3
+
¾® CH2 – CH – CH3 Br CH3
| |
Cl CH3 – CH – CH – CH2 – CH3
+
– * *
OH 2-Bromo-3-methyl pentane
CH2 – CH – CH3 ¾® CH2 – CH – CH3 (2 chiral centre)
Cl Cl OH Maximum no. of steroisomers = 2n
24. (d) N – bromosuccinimide results into bromination at (where n is the no. of chine centre)
allylic and benzylic positions Therefore, maximum no. of stereroisomers = 22 = 4
Br Na
29. (c) CH 4 ¾¾¾
2 ® CH – Br ¾¾¾
3 ® CH 3 – CH 3
¾® hv ether
(less than four 'C')
Na/liq.NH
NBS/hv 30. (c) ¾¾¾¾¾¾®
3
¾¾¾® Birch reduction
31. (b) Alkynes can be reduced to cis-alkenes with the use
¾® of Lindlar’s catalyst.
32. (c) CH3
More stable CH3 – C CH – CH3
Br HO O 3¾¾
HC

H O/K CO O
¾n/H 2 ½
NBS
¾¾® ¾¾¾¾® ¯
l

2 2 3
¬Z CH3
O
CH3 – C – CH3 CH3 – C CH2 – CH3
25. (c) There is no change in bond angles and bond lengths +
in the conformations of ethane. Cl
CH3CHO
26. (a) Correct order is
H - C º C - H > H3C - C º C - H > H 2C = CH 2 > CH3 - CH3 33. (d) CH3 - C º C - H ¾DCl
¾® CH3 - C = CH
1 eq

( Two acidic
hydrogens ) ( One acidic
hydrogen ) Cl D
¾¾ ®

27. (c) Hydration of alkynes give ketones. DI

OH I D
|
H3C – C º CH ¾® H3C – C = CH2 ¾¾ CH3 – C – CH
¾¾¾

O (A)
|| Tautomerism
Cl D
H3C – C – CH3¬¾¾¾¾ Both additions follow Markovnikov’s rule.
(B)
EBD_8350
584 CHEMISTRY

OCH3 CH3 Cl CN 3. (b) Cumulative dienes.


4. (b) Formation of carbocation is the rate determining step
34. (a) more the stable carbocation more is the reactivity toward HBr.
> > >
CH3 +
CH3 +
(+M) (+I and +H) (–I and +M) (–M, –I) C CH3 > C CH3 >
CH3O CH3
(b) (a)
O resonance stabilized (9a)
CH2– CH = CH 2 CH2– C – H CH3
O +
35. (b) (i) O 3
C CH3
¾¾¾®
(ii) Zn.H 2O
+ H–C–H CH3OCH2
(c)
Methanal (–I)
5. (c)
Exercise - 4
H ring –H
CH2OH CH2
1. (c) Since hydrocarbon does not react with Cl2 in dark it –H2O expansion

means it does not have multiple bond. It must be an alkane [X]

as it is showing free radical substitution reaction. H

Br
Cl V
Cl2 OH NBS
–H
UV

2. (a) Due to high selectivity of bromine it abstracts that H2


6. (a) CH3 CH2 CH CH CH2 CH3 Platinum
hydrogen which forms stable free- radical.
CH3 CH3 CH2 CH2 CH2 CH2
HBr
.
CH3
CH3 CH2 CH CH2 CH2 CH3
Br
CH3
3° free-radical

CH==CH2
H
—CH—CH3 ring
7. (d) expansion
(TS)1
CH3 CH3
V
H
Shift (TS)3
(TS)3 (TS)2
H
CH3 CH3
CH3
CH3OH O OCH3
–H

(TS)4

O
O3 2H O
8. (b) CH 3 - C º C - CH 2 - CH 3 ¾¾¾
® CH3 – C – C – CH2 – CH3 ¾¾®
2

O O
Pentyne ozonide
CH 3 - CO
(O)
| + H 2 O 2 ¾¾¾ ® CH3COOH + CH3CH2COOH
CH3CH 2CO Acetic acid Propionic acid
Ethylmethylglyoxal
HYDROCARBONS 585

The glyoxal formed as an intermediate is oxidised by H2O2 According to Baker-Nathan effect, hyperconjugation
to give the acids. dominates over inductive effect.
9. (d) Hence ortho-position of methyl group has more e– density
for EAS.
2 HBr Zn 11. (d) For a compound to be aromatic it must have
R2O2 D (4n + 2) pe– where n is an integer. (III) have 8 and (VI)
Br Br 'Y'
have 4e–. Hence are Antiaromatic.
10. (c) Cl2/hn Þ Free radical mechanism
Cl2/AlCl3 Þ Electrophilic aromatic substituion.

H CH2—CH2 —CH—CH—CH3
12. (c) — 2
CH2—CH2 —CH—CH—CH

CH3 CH3
3
4 3 2 1 V
4 2 CH2—CH2—C—CH2 —CH3 H Shift
–H
5 1 5
H CH3

13. (c) Here hydration occurs in anti-Markovnikov’s way + –


OH
because of electron-withdrawing nature of –COOH which HC CH – COOH ¾® HC = CH – COOH
destabilises carbocation I more than the II. II OH
+
H+ OHC – CH 2 – COOH
HC º C – COOH ¾® H 2C = C COOH
I (+ve charge 14. (d) C—H or C—D bond breaking is not rate determining
more intensified)
step (slow step)
+
or HC = CH COOH
II (+ve charge
less intensified)
More stable

15. (c) CH3CH2CH2Cl + AlCl3 AlCl4 + CH3CH2 CH2


OH
O
H Shift C6 H6 O2
CH3CH2CH2 CH3—CH—CH3 +
H3 O
cumene
Manufacture of phenol
EBD_8350
586 CHEMISTRY

14 Environmental Chemistry
Exercise - 1 NO 2 + O ¾
¾® NO + O 2
1. (d) DDT is a non-biodegradable pollutant. 2 O3 + h u ¾ ¾® 3 O 2 (Net reaction)
2. (b) Nitric oxide (NO) which may be produced at the ground The presence of oxides of nitrogen increase the
level due to human activity or natural sources or is decomposition of O3.
produced in large amounts in the exhaust gases by the hn •
engine of supersonic transport planes and introduced 27. (c) CF2 Cl 2 ¾¾® CF2 Cl + Cl
directly into the stratosphere. • •
NO + O3 ¾® NO2 + O2 Cl + O3 ¾ ¾® Cl O + O 2
3. (d) CO is highly toxic and impairs respiration. CO combine 28. (d) HCl(g) + ClONO2(g) ® Cl2(g) + HNO3(l) in the
with haemoglobin of blood and reduces its O2 carrying presence of sunlight, chlorine molecules dissociate and
capacity. catalyse ozone depletion.
4. (a) Green house gases such as CO2, ozone, methane, the 29. (d)
chlorofluorocarbon compounds and water vapour form a 30. (a) Classical smog is also called reducing smog.
thick cover around the earth which prevents the IR rays 31. (c) H2, He and ionic oxygen are present in exosphere.
emitted by the earth to escape. It gradually leads to increase 32. (a) The presence of particulate matter in polluted air
in temperature of atmosphere. catalyses the oxidation of SO2 to SO3.
5. (c) The irritant red haze in the traffic and congested places 33. (b) CO2 and water vapour can re-radiate I.R. rays.
is due to the presence of oxides of nitrogen. 34. (b) SPM (Suspended Particulate Matter) is defined as
6. (d) particles floating in the air with a diameter below 10
7. (b) mm. Studies have shown that high SPM concentrations
8. (a) Smog is caused by oxides of sulphur and nitrogen. in the air can have a detrimental impact on respiratory
9. (d) Ozone absorbs U.V. radiations harmful to human life. organs. SPM is generated from natural sources (e.g.,
10. (c) volcanoes or dust storms) and human activities
11. (d) London smog is formed in morning during winter. (vehicles, incinerators and industrial plants).
12. (a) Both NO2 and O3 are oxidising in nature. Catalytic converters is a device designed to reduce the
13. (d) Ozone is destroyed by pollutant gases. amount of emissions from automobiles. The current
14. (d) O3 and hydrocarbons react to form PAN. (so-called three-way) systems use a heated metal
15. (b) High concentration of SO2 leads to stiffness of catalyst to reduce the emissions of carbon monoxide
flower buds. (CO), hydrocarbons, and nitric oxide (NO), all of which
16. (b) CO2 is generally not regarded as pollutant. contribute to the formation of photochemical smog. In
17. (c) CO and oxides of Nitrogen are poisnous gases an automobile’s exhaust system, a catalytic converter
present in automobile exhaust gases. provides an environment for a chemical reaction where
18. (d) Rising temperature increases parasites. unburned hydrocarbons completely combust.
19. (a) Radiation coming from sun or outerspace have high 35. (d) 3CH4 + 2O3 ® 3CH2= O + 3H2O
energy or short wavelength, which are allowed to enter by Formaldehyde
green house gases. However, radiation emitted by earth is CH2 == CHCH == O; CH3COONO2
in infrared region, having long wavelength, are reflected Acrolein Peroxyacetyl nitrate (PAN)
back by the envelope of green house gases. 36. (c) Troposphere contains water vapour.
20. (d) Acid rain contains H2SO4 > HNO3> H2CO3. 37. (d) 38. (d)
21. (b) Large amounts of CH4 are released in paddy fields, 39. (a) PAH (Poly Aromatic Hydrocarbon)
coal mines and by fossil fuels. 40. (c) Usually catalytic converters are used in the
22. (a) 23. (a) 24. (a) automobiles, which prevent the release of nitrogen oxide
25. (a) A non-viable particle does not contain living micro and hydrocarbons to the atmosphere. Certain plants e.g.,
organism. Pinus, Juniparus. Quercus, Pyrus and Vitis can
26. (c) The ozone layer, existing between 20 to 35 km above metabolise nitrogen oxide and therefore, their plantation
the earth’s surface, shield the earth from the harmful could help in this matter. SO2 is not a content of
U. V. radiations from the sun. photochemical smog.
Depletion of ozone is caused by oxides of nitrogen 41. (d) 42. (a) 43. (b)
NO2 + h u ¾ ¾® NO + O 44. (a) PSCs react with chlorine nitrate and HCl to give HOCl
reactive nitric oxide 45. (a) The average residence time of NO is 4 days.
NO + O 3 ¾
¾® NO 2 + O2 46. (d) Ozone layer acts as a shield and does not allow
ultraviolet radiation from sun to reach earth.
O3 + h u ¾
¾® O 2 + O
ENVIRONMENTAL CHEMISTRY 587

47. (c) The ozone layer, existing between 20 to 35 km above With increasing eutrophication, the diversity of the
the earth’s surface, shield the earth from the harmful phytoplankton community of a lake increases and the lake
U. V. radiations from the sun. finally becomes dominated by blue - green algae.
Depletion of ozone is caused by oxides of nitrogen 69. (d) For statement (iii), Methemoglobinemia (blue baby
NO2 + h u ¾ ¾® NO + O syndrome) is caused due to excess of nitrate in drinking
reactive nitric oxide water.
NO + O 3 ¾
¾® NO 2 + O2 For statement (iv), Excessive sulphate (> 500 ppm) in
drinking water causes laxative effect, otherwise at
O3 + h u ¾
¾® O 2 + O
moderate levels it is harmless.
NO 2 + O ¾
¾® NO + O 2 70. (c) Because they are very sensitive to sulphur dioxide
and in cities the amount of SO2 is high so lichen do not
2 O3 + h u ¾ ¾® 3 O 2 (Net reaction)
grow in cities.
The presence of oxides of nitrogen increase the 71. (d) 72. (d)
decomposition of O3. 73. (b) Pesticides and synthetic fertilizers pollute the soil.
48. (d) Minamata is caused by Hg poisoning. 74. (a) Lower trophic level has lower toxins deposition than
49. (c) 50. (a) 51. (b) higher trophic level.
52. (a) Normal rain water has pH 5.6 Thunderstorm results 75. (a)
in the formation of HNO3 which lowers the pH. 76. (b) Thermal pollution is caused by power plants. Power
53. (d) Decrease in dissolved oxygen causes death of fish. plant requires a larger quantity of water for cooling. The
54. (b) Strength of sewage or degree of water pollution is water after cooling is left in the water body. The temperature
measured in terms of BOD (Biochemical oxygen demand) of left water is generally very high and affects aquatic life.
value. 77. (c) 78. (a)
55. (d) Biochemical and chemical oxygen demand increases. 79. (a) (a) Phosphate containing fertilizers enhances algae
56. (d) growth. This process in which nutrient enriched water
57. (c) The excess of fluorine in water causes fluorosis. The body support a dense plant population, which kills animal
symptoms of fluorosis are mottling of teeth (yellowish life by depriving it of oxygen and results in subsequent
streaks) and abnormal bones liable to fracture etc. It is an loss of biodiversity is known as Eutrophication.
example of endemic disease. (b) Azotobacter fix molecular n itrogen from the
58. (c) atmosphere without symbiotic relations with plants.
59. (b) The ideal value of DO for growth of fishes is 8 mg/L. Whereas Rhizobium fix atmospheric nitrogen in root
7 mg/L is desirable range, below this value fishes get nodules of plants by converting it to ammonia.
susceptible to disease. A value of 2 mg/L or below is lethal (c) Cyanobacteria are important for photosynthesis and
for fishes. nitrogen fixation.
60. (b) Water pollution is mainly caused by industrial wastes, (d) There are few varities of maize that do not require
sewage, insecticide, herbicides, etc. chemical fertilizers and are able to fix the atmospheric
61. (d) Addition of phosphate fertilizers to water leads to nitrogen.
nutrient enrichment (eutrophication). 80. (a) (A)Phosphate fertilisers promotes algae growth and
62. (a) BOD of pond is connected with microbes and organic which causes eutrophication. It increases biochemical
matter. oxygen demand of the water body.
63. (a) Eutrophication causes reduction in D.O (B) Methane is a greenhouse gas which causes global
64. (c) High BOD means high concentration of bacteria in warming.
water. (C) The bacteria responsible for degrading synthetic
65. (b) Domestic sewage constitute biodegradable biodegradable detergent feed on it and grow rapidly. While
pollutants. growing, they increases BOD of water.
66. (a) The F– ions make the enamel on teeth much harder (D) Nitrogen oxides form HNO3 and causes acid rain.
by converting hydroxyapatite, [3Ca3(PO4)2. Ca(OH)2]. 81. (b) This represents a great step forward for green chemistry.
the enamel on the surface of the teeth, into much harder 82. (d) All the given statements are true about waste
fluorapatite, [3Ca3(PO4)2. CaF2]. recycling.
67. (a) It affects oxygen carrying capacity of haemoglobin 83. (b) Green chemistry may be defined as the programme of
and parts of skin becomes blue in colour. developing new chemical products and chemical processes
68. (b) Eutrophication is a natural process which literally or making improvements in the already existing compounds
means well nourished or enriched. It is a natural state in and processes so as to make less harmful to human health
many lakes and ponds which have a rich supply of nutrients. and environment. This means the same as to reduce the
Eutroph ication become excessive, however when use and production of hazardous chemicals.
abnormally high amount of nutrient from sewage, fertilizers, 84. (b) Replacement of earlier used tetra-chloroethene as
animal wastage and detergent, enter streams and lakes solvent for dry cleaning by liquid CO2 results in less
causes excessive growth or blooms of microorganisms. harm to ground water.
EBD_8350
588 CHEMISTRY

85. (b) Ethanal is commerically prepared by one step .


Hydrocarbon + O ® RCO (free radicals)
oxidation of ethene in the presence of ionic catalyst in .
aqueous medium with an yield of 90%. RCO + O2 ® RCO3
Catalyst RCO3g + NO2 ® RCO3 NO 2
CH2 = CH2 + O2 ¾¾¾¾¾¾
Pb(II)| Cu(II)
® CH3CHO
Peroxyacetyl nitrate
(in water)
13. (d) The oxidised hydrocarbons and ozone in presence of
humidity cause photochemical smong.
Exercise - 2
Hydrocarbons + O2, NO2, NO, O, O3 ® Peroxides,
1. (c) The smog which is formed in presence of sunlight is formaldehyde, peroxyacetyl-nitrate (PAN), acrolein etc.
called photochemical smog. The main components of the Hence, chlorofluoro carbons are not common component
photochemical smog results from the action of sunlight of photochemical smog.
on unsaturated hydrocarbons and nitrogen oxides 14. (a) Nitrogen and oxygen in air do not react to form oxides
produced by automobiles and factories. of nitrogen in atmosphere because the reaction between
2. (a) Classical smog occurs in cold humid climate. It is a nitrogen and oxygen requires high temperature.
mixture of smoke, fog and sulphur dioxide. 15. (a) The maximum limit of nitrate in drinking water is
3. (c) The large population of bacteria decomposes organic
50 ppm. Excess nitrate in drinking water can cause disease
matter present in water. They consume oxygen dissolved
such as methemoglobinemia ('blue baby' syndrome).
in water. Hence, oxygen from water decreases. It is harmful
16. (a) Microorganisms present in the soil is a sink for CO.
for aquatic life.
4. (b) Photochemical smog has high concentration of 17. (c) Above 2 ppm concentration of F– in drinking water
oxidants such as O3, organic oxidant and is therefore cause brown mottling of teeth.
called as oxidising smog. 18. (a) Nitrous oxide (N2O) occurs naturally in environment.
5. (b) The lowest region of the atmosphere in which human In automobile engine, when fuel is burnt dinitrogen and
beings along with other organisms live is called dioxygen combine to yield NO and NO2.
troposhere. It extends upto the height of ~ 10 km from sea 19. (c) [3Ca 3 (PO 4 ) 2 .Ca(OH) 2 ] + 2F – ¾¾
®
level. Troposphere is a turbulent, dusty zone containing (upto 1ppm)
air, much water vapour and clouds. [3Ca 3 (PO4 ) 2 .CaF2 ]+ 2OH –
6. (c) Ozone hole is thinning of ozone layer of stratosphere (Harder teeth
at some places. NO and chlorofluorocarbon have been enamel)
found to be the most responsible for depleting the ozone
20. (b) The water sample containing Mn = 5 ppm is
layer.
unsuitable for drinking as the prescribed level for Mn in
7. (d) Using plastic cans for neatly storing substances will drinking water is 0.5 ppm.
not come under green chemistry. The plastic materials are
non-biodegradable. 21. (a) Global warming is caused by the emission of green
8. (a) Those gases which absorb sunlight near the earth's house gases. 72% of the totally emitted green house gases
surface and then radiates back to the earth are called green is CO2. Therefore, excessive release of CO2 is the main
house gases. cause of global warming.
Carbon dioxide, water vapour, methane, ozone, oxides of 22. (d)
nitrogen, chlorofluoro carbons (CFCs) etc; are green house
gases. CO is not a green house gas. Exercise - 3
9. (a) Water considered to be clean if it has BOD less than
1. (a) HNO3 and H2SO4 are formed by NO2 and SO2.
5 ppm whereas highly polluted water has BOD more than
2. (b) Replacement of halogenated solvent (tetrachlora
17 ppm.
ethane) by liquid CO2 will result in less harm to ground
Therefore, water having BOD less than 5 ppm is rich in
dissolved oxygen. water.
10. (a) O 3 is responsible for greenhouse effect. Its 3. (d) In coming years, when the ozone layer will become
contribution is about 8%. thinner or will have hole, ultraviolet radiations will
11. (a) Nitrogen and oxygen do not react with each other at come directly to the earth that will cause cancer
normal temperature. At high altitude when lightning especially related to skin like, melanoma.
strikes, they combine to form oxides of nitrogen. 4. (a) Water sample is reported to be highly polluted if BOD
3000° C
value of sample is more than 17 ppm.
N 2 ( g ) + O 2 ( g ) ¾¾¾¾® 2NO ( g ) 5. (b) In cold water, dissolved oxygen can reach a
12. (c) Hydrocarbons present in atmosphere combine with concentration upto 10 ppm, whereas oxygen in air
oxygen atom produced by the photolysis of NO2 to form is about 200, 000 ppm.
highly reactive intermediate called free radical. Free radical 6. (d) Growth of fishes are more in cold water.
initiates a series of reaction. 7. (d) Lead
Peroxyacetyl nitrates are formed, which can be said as 8. (b) CFC > N2O > CH4 > CO2
secondary pollutants.
15 The Solid State
21. (a) Diamond is like ZnS. In diamond cubic unit cell, there
Exercise - 1
are eight corner atoms, six face centered atoms and four
1. (b) Crystalline solids are anisotropic in nature that is more atoms inside the structure.
some of their physical properties like electrical resistance Number of atoms present in a diamond cubic cell
or refractive index show different values when measured 1 1
along different directions in the same crystals. = 8´ + 6´ + 4 = 8
8 2
2. (a) In crystalline solid, there is perfect arrangement of (corners) (face (inside
the constituent particles only at 0 K. As the temperature centered) body)
increases the chance that a lattice site may be unoccupied 1
22. (d) In a unit cell, W atoms at the corner = ´ 8 = 1
by an ion increases. As the number of defects increases 8
1
with temperature, solid changes into liquid. O-atoms at the centre of edges = ´ 12 = 3
3. (c) Glass is amorphous solid. 4
4. (b) Amorphous solids are isotropic, because these Na-atoms at the centre of the cube = 1
substances show same properties in all directions. W : O : Na = 1 : 3 : 1
5. (c) Amorphous silicon is used as best photovoltaic Hence, formula = NaWO3
material available for conversion of sunlight into electricity. 1
6. (c) Quartz glass is an example of amorphous solid and 23. (a) Number of A atoms = ´8 = 1
8
crystalline solids are anisotropic in nature.
7. (a) Amorphous solids are isotropic in nature. 1
Number of B atoms = ´6 = 3
8. (a) 2
9. (b) Among the given crystals, only silicon exists as a \ Formula AB3
covalent solid. It has diamond like structure.
1 1 1
10. (c) Covalent bond force 24. (b) A : B : C = ´8: ´ 2: ´ 4 = 1 : 1 : 2
11. (b) In graphite, the electrons are spread out between the 8 2 2
sheets. 25. (a) Given: Atoms are present in the corners of cube = A
12. (d) Methane is a non-polar covalent molecular which and atom present at body centre = B. We know that a cubic
forms molecular solid when solidified. unit cell has 8 corners. Therefore, contribution of each
13. (d) 1
atom at the corner = . Since number of atoms per unit cell
14. (b) The aqueous solution or melt has ions. 8
1
15. (a) Potassium crystallises in bcc lattice. is 8, therefore, total contribution = 8 ´ = 1 . We also know
8
16. (b) For tetragonal a = b ¹ c , a = b = g = 90° thats atom is in the body centre, therefore number of atoms
17. (c) per unit cell = 1. Thus formula of the compound is AB.
18. (a) There are two atoms in a bcc unit cell. 26. (b) hcp is a closed packed arrangement in which the unit
So, number of atoms in 12.08 × 1023 unit cells cell is hexagonal and coordination number is 12.
= 2 × 12.08 × 1023 = 24.16 × 1023 atoms. 27. (a) The number of octahedral voids is equal to number
19. (d) Total number of atoms per unit cell for a face centered of spheres. The number of tetrahedral voids is double the
cubic unit is 4. number of spheres.
The atom at the body center completely belongs to the 28. (d) It represents ccp arrangement.
unit cell in which it is present. 29. (a)
20. (d) The bcc cell consists of 8 atoms at the corners and 30. (d) Number of effective atoms in fcc arrangement
one atom at centre. Contribution of each atom at each æ 1ö æ 1ö
= ç8´ ÷ + ç 6 ´ ÷ = 4
1 è 8ø è 2ø
corner is equal to . Number of tetrahedral voids = 2 × 4 = 8
8
31. (b) Metals such as copper and silver crystallise in fcc
æ 1ö structure while metals Mg and Zn crystallise in hcp
\ n = ç8 ´ ÷ + 1 = 2
è 8ø structure.
The fcc cell consists of 8 atoms at the eight corners and 32. (a)
one atom at each of the six faces. This atom at the face is 33. (c) In diamond cubic unit cell, there are eight corner atoms,
shared by two unit cells. six face centered atoms and four more atoms are in the
1 æ 1ö tetrahedral voids.
\n = 8´ + ç6´ ÷ = 4 Number of effective atoms in fcc unit cell = 4
8 è 2ø
EBD_8350
590 CHEMISTRY

Number of octahedral voids = 4 2 ´ 3.14


Number of tetrahedral voids = 4 × 2 = 8 = ´ 100 = 60.43%
(a) In this type of structure, each atom is tetrahedrally 6´ 3

(d) For bcc, d = 3 a or a = 2d = 2 ´ 4.52 = 5.219Å


surrounded by four atoms.
42.
(b) Coordination number is four Number of Ge atoms in a 2 3 1.732
æ 1ö æ 1ö = 522 pm
unit cell = ç 8 ´ ÷ + ç 6 ´ ÷ + 4 = 8 z´M 2 ´ 39
è 8ø è 2ø r= 3 =
-30
a ´ N A ´10 (522) ´ (6.023 ´1023 ) ´ 10 -30
3
æ 1 ö
(d) Only 50% of the tetrahedral voids ç 8 ´ = 4 ÷ are = 0.91g / cm3 = 910 kg m -3
è 2 ø
occupied. All the octahedral voids remain unoccupied. ZM
34. (d) Tetrahedral holes are smaller in size than octahedral 43. (a) For b.c.c., Z = 2, Now, d = and V = a3
NV
holes. Cations usually occupy less space than anions.
2 ´ 56
35. (a) \ d= = 7.92 g ml–1
36. (b) Packing fraction is defined as the ratio of the volume (6.02 ´10 ) ´ (2.861´10-8 )3
23

of the unit cell that is occupied by the spheres to the total 44. (b) Density is given by
volume of the unit cell. Z ´M
P.F. for cpp and bcc are 0.74 and 0.68 respectively. d= ; where Z = number of formula units present in
N A a3
So, the free space in ccp and bcc are 26% & 32%
respectively. unit cell, which is 4 for fcc
37. (b) In AB AB packing (hcp), spheres occupy 74%. 26% is a = edge length of unit cell. M = Molecular mass
empty. 4´M
2.72 = (Q 1pm = 10 -10 cm)
( )
3
38. (a) For bcc lattice, body diagonal = a 3 . 6.02 ´ 10 ´ 404 ´ 10-10
23

The distance between the two oppositely charged ions =


a 2.72 ´ 6.02 ´ (404)3
3
2 M= = 26.99 = 27 g mole–1
4 ´107
387 ´ 1.732
= = 335pm
2 45. (b) Z M
r=
39. (d) Number of atoms per unit cell = 1 N AV
a
Atoms touch each other along edges. Hence r = rN AV 8.92 ´ 6.02 ´1023 ´ (362)3 ´10 -30
2 Z= = =4
( r = radius of atom and a = edge length) M 63.55
4 3
\ It has fcc unit cell.
pr 3
3 p d1 (a2 )3 z1 æ 3 ö 4
Therefore % fraction = 3
= 46. (c) d = ´ =ç ÷ ´ = 1.46
(2 r ) 6 2 (a1 )3 z2 è 3.5 ø 2
40. (b) P.F. for fcc lattice is 74%. 47. (a) Following generalization can be easily derived for
various types of lattice arrangements in cubic cells between
41. (a)
the edge length (a) of the cell and (r) the radius of the
sphere.
a
For simple cubic : a = 2r or r =
2
For body centred cubic :
Let radius of the sphere = r 4 3
a= r or r = a
Area occupied by sphere in hexagonal close packing 3 4
æ1 ö For face centred cubic :
pr 2 + 6 ´ ç ´ pr 2 ÷ = 2pr 2
è6 ø 1
é 3 ù
a = 2 2r or r = a
Area of hexagonal = 6 ´ ê ´ (2r )2 ú 2 2
ë 4 û Thus the ratio of radii of spheres for these will be simple :
3 a 3 1
= 6´ ´ 4r 2 = 6 3 ´ r 2 bcc : fcc = : a: a
4 2 4 2 2
2pr 2 i.e. option (a) is correct answer.
% occupied by = ´100
6 ´ 3 ´ r2
THE SOLID STATE 591

48. (a) In fcc structure one unit cell consist 4 atoms, hence 59. (d) Ionic solids must always maintain electrical neutrality.
density × a3 = The mass of four atoms Ionic solids show vacancy or interstitial defects as Frenkel
= 8.92 × (3.608 × 10–8)3 g = 4.18 × 10–22g and Schottky defect.
ZM 60. (d) When electrons are trapped in anion vacancies, these
49. (a) r = are called F-centre.
N AV
rN AV 2 ´ 6 ´1023 ´ (5 ´10-8 )3
Z= = +ve –ve
M 75
e– ion ion
Z = 2, which represents bcc structure
3 3 F- centre in crystal
\ r= a= ´ 5 = 2.165Å = 216.5 pm » 217 pm
4 4 61. (d) The appearance of colour in solid alkali metal halide
3
r ´ a ´ N A ´ 10 -30 is due to presence of F-centre found as defect in the crystal
50. (d) M= structure.
Z
62. (a) Stoichiometric Frenkel defects occur in those
3
10 ´ (100) ´ 6.02 ´ 10 23
´ 10 -30 compound which have
= = 15.05 (i) Low C.N.
4
(ii) Large difference in size of cations and anions
6.02 ´1023 63. (b) Transition metals exhibit this defect due to metal
\ Number of atoms in 100 g = ´100 = 4 × 1025 deficiency. The compound obtained are non stoichiometric
15.05
e.g. It is difficult to prepare ferrous oxide with the ideal
51. (b) The volume of the unit cell = (2.88 Å)3 composition of FeO; what we actually obtain is Fe0.95O or
= 23.9 × 10–24 cm3. FexO with x = 0.93 to 0.96
The volume of 100 g of the metal 64. (c) AgBr exhibit Frenkel defect.
M 100 65. (c)
= = = 13.9 cm3 66. (d) LiCl crystals are pink because its anionic sites are
r 7.20
occupied by unpaired e–. Zinc oxide shows metal excess
Number of unit cells in this volume defect due to presence of extra cations at interstitial sites.
13.9 cm3 67. (a)
= = 5.82 × 1023 68. (d) Excess of lithium makes LiCl crystal pink.
23.9 ´ 10 -24 cm3
69. (b) Assertion and reason, both are correct. Reason is not
52. (c) For body centred cubic lattice Z = 2 but in this case,
the correct explanation of assertion.
density of CsBr is asked, which is a molecule. Considering The correct reason for Schottky defect is loss of equal
CsBr as a particle, Z = 1. number of cations and anions from lattice sites. Here we
Atomic mass of unit cell = 133 + 80 = 213 a.m.u are only giving examples of such solids in which this defect
Volume of cell = (436.6 × 10–10)3 cm3 is observed.
ZM 1 ´ 213 70. (d) When insulators (non metal atoms) interact to form a
Density, r = = -10 3 solid, their atomic orbitals mix to form two bunch of orbitals,
a3 N A (436.6 ´ 10 ) ´ 6.02 ´ 10 23
separated by a large band gap. Electrons cannot therefore
Note : Z is number of formula units per unit cell.
be promoted to an empty level, where they could move
= 4.25 g/cm3
freely.
53. (d) If in an ionic crystal of the type A+ B–, equal number
of cations and anions are missing from their lattice sites so 71. (b) Doping by electron deficient atoms create p-type
that the electrical neutrality is maintained. The defect is semiconductors.
called Schottky defect. 72. (d) Extrinsic semiconductor
54. (b) More is the Schottky defect in crystal, more is the 73. (d) 74. (b)
decrease in density. 75. (d) Ferrimagnetism is observed when the magentic
55. (c) Frenkel and Schottky defects are crystal defects. It moments of the domains in the substance are aligned in
arises due to dislodgement of cation or anion from their parallel and antiparallel directions in unequal numbers.
places in the crystal lattice. 76. (a) In case of semiconductors, the gap between valence
56. (b) When equal number of cations (Na+) and anions (Cl–) band and the conduction band is small and therefore, some
are missing from their regular lattice positions, we have of the electrons may jump from valence band to conduction
Schottky defect. band and thus, on increasing temperature, conductivity is
57. (c) Frenkel defect is due to dislocation of ion from its also increased.
usual lattice site to interstitial position. 77. (a) MnO2
58. (a) Schottky defect is due to missing of equal number of 78. (c) CrO2 is ferromagnetic whereas O2 is paramagnetic.
cations and anions. 79. (c)
EBD_8350
592 CHEMISTRY

80. (b) Out of the given substances, only Li has high Thus, 10.9 × 10–30 = x × 10–30
electrical and thermal conductivity as Li is a metallic solid. x = 10.9
81. (c) Rhenium oxide ReO3 is like metallic copper in 5. (600) In a fcc lattice, the distance between the cation
conductivity. and anion is equal to the sum of their radii, which is equal
82. (a) Due to release of electrons, the crystal can conducts to half of the edge length of unit cell,
electricity but conductivity is not as high as that of metals. a
Its conductivity is very low and because conduction is i.e. r + + r - = (where a = edge length)
2
due to electrons so it is n-type semiconductor, also
r+ = 100 pm, r– = 200 pm
excitation of these electron give rise to yellow color in
Edge length = 2r+ + 2r– = (2 × 100 + 2 × 200) pm
crystal.
= (200 + 400) pm = 600 pm.
83. (a) BaTlO3
84. (d) Ferrimagnetic substance become para-magnetic on 3
6. (3.72) For bcc structure, atomic radius, r = a
heating. This is due to randomisation of spins on heating. 4
85. (d)
3
= × 4.3 = 1.86Å
Exercise - 2 4
Since, r = half the distance between two nearest
n´ M d ´ N A ´ a3 neighbouring atoms.
1. (5) d= or n =
N A ´ a3 M \ Shortest interionic distance = 2 × 1.86 = 3.72Å
7. (26) In AB AB packing spheres occupy 74%. 26% is
2 ´ 6 ´ 10 23 (5 ´ 10 -8 )3 empty.
Þ n= =2 8. (3) Volume of unit cell = a × b × c
75
= 5 × 10– 8 × 8 × 10– 8 × 4 × 10– 8
Therefore Metal crystallizes in bcc structure and for a bcc
= 1.6 × 10– 22 cm3
lattice 3a = 4r Mass of unit cell = 1.6 × 10– 22 × 5.2
= 8.32 × 10– 22 g
3 3´5 Number of molecules in one unit cell
r= a= = 2.165Å = 216.5 pm
4 4
8.32 ´10 – 22 g
216.5 = =3
x= 166.4g mol –1
43.3
9. (90) Volume occupied by atoms in solid 2
x= 5
2. (4) For an octhedral void a = 2 (r + R) 4 3 4
In fcc lattice the largest void present is octahedral void. If = pr + p(2r)3 = 12 pr3
3 3
the radius of void sphere is R and of lattice sphere is r,
Relationship between body diagonal and radius of atom
2 ´ 400 (r),
Then, r = = 141.42 pm (a = 400 pm)
4 6r = 3a
Applying condition for octahedral void,
2 (r + R) = a 6r
a=
\ 2 R = a – 2r = 400 – 2 × 141. 42 3
\ Diameter of greatest sphere = 117.16 pm vol. of particles
Packing efficiency =
117.16 vol. of unit cells
d= =4
29.29 12pr 3
Packing efficiency = 3
× 100 = 90%
3. (335) For bcc lattice body diagonal = a 3 . æ 6r ö
çè ÷ø
The distance between the two oppositely charged ions 3
a 387 ´ 1.732 10. (7) Unit cell No. of atoms
= 3 = = 335 pm
2 2 1
Simple cubic ´ 8 =1
4 ´ 96 8
4. (10.9) For bcc lattice, 3a = 4R Þ a = pm
3 1
bcc ´ 8 + 1 ´1 = 2
= 221.7 pm 8
(where R is the radius of Ca atom) 1 1
fcc ´8+ ´6 = 4
Volume of unit cell = a3 = (211.7 × 10–12)3 m3 8 2
= 10.9 × 10–30 m3 Thus, the sum is 7.
THE SOLID STATE 593

Exercise - 3 17. (a) When a ferromagnetic substance is placed in a


magnetic field it becomes a permanent magnet because all
1. (b) At low temperature existence of a substance in the the domains get oriented in the direction of magnetic field
solid state is due to slow molecular motion and strong even after the removal of applied magnetic field.
cohesive forces. 18. (b) Packing efficiency in different types of unit cells is as
2. (b) Crystalline solid is anisotropic in nature as it shows follows :
different physical properties such as electrical resistance,
refractive index in different directions. Unit cell Packing efficiency
3. (b) Quartz glass (SiO2) is an amorphous solid due to its
fcc 74%
short range order of constituent particles.
4. (d) Substances which have domain structure are bcc 68%
oppositely oriented and cancel each other's magnetic field Simple cubic 52%
are known as antiferromagnetic substances.
5. (a) Since, quartz glass is an amorphous solid having short Thus, the correct order of packing efficiency is: fcc (74%)
range order of constituents. Hence, value of refractive > bcc (68%) > simple cubic (52%).
index is same in all directions, measurable and not to be 19. (a) Conductivity of metal, insulator and semiconductors
equal to zero always. can be represented in the terms of k (Kappa) which
6. (a) Iodine molecules belongs to a class of non – polar depends upon energy gap between valence band and
molecular solids in which constituents molecule are held conduction band.
together by London or dispersion forces.
Conduction band
7. (a) Ionic solids get easily dissociated into its ions in
molten state and show high electrical conductivity. Empty
Empty
8. (b) In graphite, each carbon is sp2 hybridised having band
band
one free electron and due to the presence of free electrons Forbidden zone
(Large energy gap) Small energy gap
Energy

graphite behaves as a good conductor of electricity.


Filled
9. (d) Graphite is a covalent solid which is made up of carbon band
Overlapping
atoms covalently bonded to three carbon atoms. As the band bands
formation of covalent bond occurs throughout the crystal Metal Insulator Semiconductor
hence, it is a type of network solid. Also, due to the Value of k 10 4 – 107 10– 20 –10– 10 10– 6 to 10 4
presence of free electrons, graphite is a conducting solid. (in W – 1 m– 1)
10. (a) When smaller ion (usually a cation) is dislocated from Hence, correct order of conductivity in solids is :
its normal site in crystal and move to interstitial site, it is kmetals >> kinsulators < ksemiconductors
known as Frenkel defect. 20. (d) Amorphous solids are isotropic in nature and any of
11. (b) Schottky defect is observed in crystal when equal its physical property will be same in all direction.
number of cations and anions are missing from the lattice. 21. (b) Crystalline solids has a regular arrangement of
When lattice site is occupied by electron, this type of constituent particles observed over a long distance in the
defect is known as metal excess defect. crystal lattice due to which they exhibit sharp melting
12. (d) To get a n – type semiconductor from silicon, it should point.
be doped with a substance with valency 5. 22. (c) Diamond is a giant molecule in which constituent
e.g., silicon is doped with phosphorus to form a atoms are held together by covalent bonds.
n – type semiconductor. 23. (c) Iodine is a non–polar molecular solid and a non–
13. (b) As packing efficiency for hcp or ccp is calculated to conductor of electricity. Water is a hydrogen bonded
be 74%, it is maximum among all type of crystals. molecular solid in which H and O are held together by
14. (c) Packing efficiency for bcc arrangement is 68% which polar covalent bond and each water molecule is held
represents total filled space in the unit cell. Hence, empty together by hydrogen bonding. Also, due to its non-ionic
space in a bcc arrangement will be 100 – 68 = 32%. nature, it also behaves as a non-conductor of electricity.
15. (c) Coordination number in a square closed packed 24. (d) Metal oxides like CrO2, TiO and ReO3 show electrical
structure in 2-D must be equal to 4 as shown : conductivity similar to metals while SiO2, MgO and SO2
do not show electrical properties.
A 25. (c) Each lattice point in a pure crystal is either an atom or
A
A molecule or ion which are joined together by a straight
A line to bring out geometry of lattice in pure crystal
Cubic constituents which are arranged in fixed stoichiometric
Square closedclosed
packedstructure
structure
ratio.
16. (b) Sillicon doped with electron rich impurity forms a 26. (b) For the formation of p-type semiconductors, group
n – type semiconductor. 13 elements are doped with group 14 elements, which
EBD_8350
594 CHEMISTRY

creates a hole in a molecule but the molecule as a whole 36. (c) Certain metal oxides like VO2, VO3 and TiO3 etc. show
remains neutral. metallic or insulating property dependin g upon
27. (b) In an fcc unit cell, 8 tetrahedral voids are present at temperature which is due to variation in energy gap
the centre of each 8 smaller cube of the unit cell. between conduction band and valence band.

.. .. .. . ..
Eight tetrahedral voids 37. (d) The fraction of octahedral or tetrahedral voids
per fcc unit cell occupied depends upon the radii of the ions present at
1 2 the lattice points. As we know, the radii of octahedral or
3 tetrahedral void is related to radii of atoms (r) as

. .. . .
4
Radius of octahedral void (Ro) = 0.414 r

.
Radius of tetrahedral void (Rt) = 0.225 r
Tetrahedral Where, r = radius of bigger atom involved

.. .
void
38. (d) For bcc lattice, body diagonal = a 3
Each cube contain one tetrahedral The distance between the body centered atom and one
5 6 void at its body centre
7 8 3a
corner atom in cube will be =
Number of octahedral voids present = 4 2
Number of tetrahedral voids present = 8 Cl– Cl–
28. (a) In AgBr, both Ag+ and Br– ions are absent from the –
39. (c) Cl Cl
lattice causing Schottky defect. Also, Ag+ ions are mobile
so they have a tendency to move from their lattice site to
Cs+
interstitial site which causes Frenkel defect.
29. (d) Hexagonal close packing can be diagramatically
represented as : Cl– Cl–
Cl–
Cl–
B Relation between radius of cation, anion and edge length
A
B
of the cube
A
2r + 2r = 3a
Cs+ Cl -
We can see that lst layer (A) and 4th layer (B) are not
exactly aligned. 3a
r +r =
Cs+ Cl-2
30. (a) In NaCl crystal, Cl– ions are present at fcc lattice
points and face centre and Na+ occupies all the octahedral 40. (b) CsI3 dissociates as
voids of the unit cell. CsI3 ® Cs+ + I3–
Thus, coordination number of Na+ and Cl– is 6.
31. (d) When electron rich or electron deficient impurity is 2a 1.41 ´ 361
41. (a) Z = 4 for fcc For fcc : r = = = 127 pm
added to a perfect crystal, it introduces electronic defect 4 4
in it. 42. (d) Number of atoms per unit cell = 8 × (1/8) + 1 = 2
32. (b) Ferromagnetic substances can be permanently
3a
magnetised by applying magnetic field and they will exhibit r=
the magnetic properties even after the removal of magnetic 4 4
2 ´ pr 3
field. 3 3p
33. (a) Frenkel defect is also known as dislocation defect Packing Fraction (P.F.) = = = 0.68
(4r / 3)3 8
because in this defect, atoms present in crystal lattice are % P.F. = 68% ; % of vacant space = 100 – 68 = 32%
dislocated to interstitial site. 43. (c) In bcc, the atoms touch along body diagonal
34. (d) In the cubic close packing, the unit cell has 8 \ 2r + 2r = 3a
tetrahedral voids located at each eight smaller cube of an
unit cell. 3a 3 ´ 4.29
35. (a) Edge lengths for different types of unit cells are given as : \ r= = = 1.857Å
4 4
Types of unit cell Edge length 44. (d) For an fcc unit cell
fcc 2 2r 2a
r=
4 4
bcc r
3 2a a
\ closest distance (2r) = =
Simple cubic 2r 4 2
THE SOLID STATE 595

4r pr 2
45. (d) For bcc lattice : Z = 2, a =
3 5. (d) Area of circle inside the square = pr2 + 4 × = 2pr 2
4
For fcc lattice : Z = 4, a = 2 2r
4r
æ ZM ö Diagonal of square = r + 2r + r = 4r, Thus L = = 2 2r
ç 3 ÷
2
d25 °C çè NA a ÷ø 2 æ 2 2r ö
3
3 3
\ = bcc Area occupied by effective circles
= ç ÷ = Packing effieincy =
d900 °C æ ZM ö 4ç 4 r ÷ 4 2 Area of square
çç 3 ÷÷ ç 3 ÷
N a
è A ø fcc è ø
2 pr 2 2pr 2
= ´ 100 = ´100
46. (c) Cation dislocate from lattice to interstitial site. L2 (2 2r )2
47. (c) Oh void (C) : HCP (A)
p
75 = ´100 = 78.54%
No of ions Þ 6 × : 6 4
100
3 C D
: 1 6. (a)
4 B
3 : 4 C3 A 4
48. (d) For bcc, a
3a
3a = 4r Þ r = A E
4
Given, a = 288 pm
AB = a (nearest)
3 (next - nearest)
r= ´ 288 pm BD = 2a
4
49. (c) Quartz exhibits piezoelectricity and thus can be used CE = 3a (next - next - nearest)
as a piezoelectric material. 7. (a) Zeff for fcc = 4 and for bcc = 2/unit cell
3
Exercise - 4 dg d fcc Zeff (fcc ) æa ö
= = ´ ç bcc ÷
db d bcc Z eff ç afcc ÷
1. (b) M = 59g mol–1 , Z = 4 (for ccp), (bcc ) è ø
a = 3.5 Å = 3.5 × 10–8 cm, d = 9 g/cm3 3
Z ×M Z ×M 4 æ 290 ö
d= Þ NA = = ×ç ÷ = 0.9788
NA × a3
d × a3 2 è 386 ø
4 ´ 59 Z eff ´ M w
Þ NA = -8 3 8. (d) Density ( d ) =
9 ´ (3.5 ´ 10 ) N A ´ a3
Þ N A = 6.11´ 1023 (For antifluorite, Zeff = 4/unit cell)
1
2. (a) Number of A atoms = ´ 8 = 1 4 ´ [ 23´ 2 +16]
8 d=
1 6 ´10 ´ (100 ´10-10 cm)3
23
Number of B atoms = ´ 6 = 3
2
\ Formula AB3 é1Pm = 10-12 m = 10-10 cm ù
ë û
3. (c) CaO has higher lattice energy because of higher
charge on Ca2+ and O2–, which results in higher attraction. = 414.16 g cm–3
KI is more soluble in water because of low lattice energy Density remains same in Frenkel defect.
and higher hydration energy. Clearly (c) is wrong because rc 0.066
9. (c) = = 0.47
CaO has higher melting point as compared to KI. ra 0.140
1 1
4. (c) No. of Cu = 8 ´ + 6 ´ = 4 So, cation (Mg2+) fitted in octahedral holes of O2– present
8 2 at fcc lattice point.
1
No. of Ag = 12 ´ = 3 10. (d) Number of atoms = NA
4
(each edge is shared with 4 cells) NA
No. of Au = 1 Þ Number of unit cells =
4
Cu4Ag3Au1
[Q 4 atoms in each unit cell]
EBD_8350
596 CHEMISTRY

NA 7 pR 3
Volume of 1 mol lattice = × Volume of unit cell
4 3 7p 3
Desired ratio Þ =
64 3 64
6.023 × 1023 R
= × (400 × 10– 12)3 m3 = 9.64 mL 3 3
4
14. (a) In NaCl, Cl– ions are in fcc arrangement.
3 Effective number of remaining
11. (b) For bcc, r = a
4 é 1 1 ù 13
Edge length not covered by atom = a – 2r Cl– = 4 - ê 2 ´ + ú =
ë 8 2û 4
3 é2– 3ù Effective number of Na+ = 4
=a–2× a = aê ú
4 ë 2 û Occupied volume
Packing Fraction =
é2– 3 ù Total volume
aê ú
ë 2 û 13 4 3 4 13 3 16 3
\ % of fraction not covered = × 100 ´ pr- + 4 ´ pr+3 pr- + pr+
a 4 3 3
= 0.134 × 100 = 13.4% = = 3 3
12. (c) Among the three options KCl, NaCl and MgCl2, the {2(r+ + r- )}3
8(r+ + r- )3
size of anion is same. So, larger the cation, larger will be the 15. (d) ln Be2C3; Be3+ = In tetrahedral voids
cation/anion ratio i.e., KCl will have larger cation/anion
ratio among the three. So, we left with two options KCl and
CaF2. Among these two CaF2 will have maximum value of
cation/anion ratio because decrease in ionic radii of anion
from Cl– to F– does not overcome the effect of decrease in
Be 3+ =
ionic radii of cation from K+ to Ca2+.
13. (a) Effective no. of atoms of B present in a unit cell = 2 C 23 – =
Total volume of B unoccupied by A in a unit cell
4
(
= 2 ´ R3 - r3 ´ p
3
)
r = radius of A
7 pR 3 æ Rö
R = radius of B = çQ r = ÷ C32 - = In fcc arrangement (Lattice point at corner + at each face
3 è 2ø
Volume of unit cell = a3 centre)
3 Therefore, coordination number of Be3+ = 4
æ 4R ö
Þ ç
è 3ø
÷ =
64 3
3 3
R (Q 3a = 4 R ) And coordination number of C32 - = 8.
SOLUTIONS 597

16 Solutions
12. (c) In a given solution, sum of all the mole fraction is
Exercise - 1
unity i.e.,
1. (c) In homogeneous mixtures, composition and (a) x1 + x2 + x3 + x4 = 1
properties both are uniform throughout the mixture. n3
2. (b) Density = 1.17 g/cc (Given) (b) = x3
n1 + n2 + n3 + n4
Mass
As d < 13. (a) Mole fraction of any component A in solution
Volume
volume = 1cc \ mass = d = 1.17g No. of moles of A
x=
No. of moles 1.17 ´ 1000 Total No. of moles of solution
Molarity < =
Volume in litre 36.5 ´ 1 As total no. of moles of solution > No. of moles of A
Thus x can never be equal to one or zero.
1170
< = 32.05 M 14. (b) Let total moles in solution = 1, Moles of solute = 0.2,
36.5 Moles of solvent = 0.8, Mass of solvent
3. (c) From molarity equation = 0.8 × 78 × 10–3 kg.
M1V1 + M2V2 = M3(V1 + V2)
1× 2.5 + 0.5 × 3 = M3 × 5.5 moles of solute 0.2
Molality X = Mass of solvent = = 3.2
4 0.8 ´ 78 ´ 10-3
M3 < < 0.73 M
5.5 15. (a) For HCl
Number of moles M = N = 0.1
4. (c) Molarity = N1V1 = N2V2 ; 25 × N1 = 0.1 × 35
Volume of solution (L)
0.1 ´ 35 0.1 ´ 35
Mass 1000 N1 = ;\M = = 0.07 .
Moles of water = = = 55.6 25 25 ´ 2
Molar mass 18 16. (c) N1V1 + N2V2 = NV
4x + 10 (1 – x) = 6 × 1; –6x = –4 ; x = 0.67
55.6
Molarity = = 55.6 Thus 0.67 litre of 4N HCl
1 1 – x = 1 – 0.67 = 0.33 litre of 10 N HCl
5. (b) Q 10 g glucose is dissolved in = 100 ml solution. 17. (c)
\ 180 g (g mole) is dissolved in
Molarity
100 Molality (m) =
= ´ 180 = 1800 ml = 1.8 L Molarity ´ Molecular mass
10 Density -
1000
Mass of solute 18
6. (d) ppm = ´ 106 = = 500
Mass of solution 18 ´ 98
1.8 -
10 1000
\ ppm =
6
´ 106 = 10 ppm wt ´1000
10 18. (d) Molarity (M) =
mol. wt. ´ vol (mL)
5 wt. 1000
7. (b) Molarity = No. of moles of solute = =2M 2= ´
Volume in litres 2.5 63 250
8. (d) NV = N1V1 + N2V2 + N3V3 63
wt. = g = 31.5g
1 1 1 2
or, 1000 N = 1 ´ 5 + ´ 20 + ´ 30 or N = .
2 3 40 100
9. (d) H3PO4 is tribasic so N = 3 × 1N = 3N. wt. of 70% acid = ´ 31.5 = 45 g
70
10. (d) No. of millimoles = 500 × 0.2 = 100
19. (c) Applying the law of equivalence,
Thus, molarity of diluted solution
N1V1 + N 2V2 + N3V3 = N RVR
100
= N N N
700 ´ 50 + ´ 30 + ´ 10 = N R ´ 1000
(Molarity = No. of moles L–1 = No. of millimoles mL–1) 10 3 2
= 0.1428 M 5 N + 10 N + 5 N = 1000 ´ N R
11. (b) More than theoretical weight since impurity will not N
Þ NR =
contribute. 50
EBD_8350
598 CHEMISTRY

20. (a) Given w = 10 g Mol. mass = 40 35. (d) According to Henry’s law,
Weight of solvent = 1250 × 0.8 g = 1000 g = 1 kg solubility = k × p
10 = 1.4 × 10–3 × 0.5 mol/L
\ molality = = 0.25 = 7 × 10–4 mol/L
40 ´ 1
21. (a) Number of moles in 100 mL = 7 × 10–5
22. (c) Equivalent weight of orthophosphoric acid Mass of oxygen m = 7 × 10–5 × 32g = 2.24 mg
3 + 31 + 64 98 36. (d) To increase the solubility of CO2 in soft drinks and
(H3PO4) = = soda water, the bottle is sealed under high pressure.
3 3
37. (d) Scuba divers must cope with high concentrations of
Now 100 g solution contains 70 g H3PO4 dissolved gases while breathing air at high pressure
100 70 underwater. Increased pressure increases the solubility
litre of solution contains g equivalent of atmospheric gases in blood. When the divers come
1000 ´1.54 98 / 3
of H3PO4 towards surface, the pressure gradually decreases. This
Normality of solution releases the dissolved gases and leads to the formation
70 ´ 3 of bubbles of nitrogen in the blood. This blocks capillaries
98 70 ´ 3 and creates a medical condition known as bends, which
= = ´ 10 ´ 1.54 = 33 N are painful and dangerous to life. To avoid bends, as well
1 98
10 ´1.54 as, the toxic effects of high concentrations of nitrogen in
23. (d) 24. (d) the blood, the tanks used by scuba divers are filled with air
25. (b) An increase in temperature of the solution increases diluted with helium (11.7% helium, 56.2% nitrogen and 32.1%
the solubility of a solid solute. oxygen).
The amount of solute that dissolve depends on what type 38. (c) Solubility of gases increase with decrease of
of solute it is. temperature.
For solids and liquid solutes, changes in pressure have 39. (d) According to Henry’s law, the mass of a gas dissolved
practically no effect on solubility. per unit volume of solvent is proportional to the pressure
26. (b) Number of solute particles going into solution will be of the gas at constant temperature m = K. p i.e. as the
equal to the solute particles separating out and a state of solubility increases, value of Henry’s law constant
dynamic equilibrium is reached. decreases. Since, CO2 is most soluble in water among the
solute + solvent solution. given set of gases. Therefore, CO2 has the lowest value of
i.e., rate of dissolution = rate of unsaturation. Henry’s law constant.
27. (d) The maximum amount of solute dissolved in a 40. (d) Let the mass of methane and oxygen = m g.
given amount of solvent is its solubility. Mole fraction of O2
28. (d) Solubility is directly proportional to pressure.
29. (a) Solubility decreases and KH increases with increase Moles of O2
=
in temperature. Moles of O 2 + Moles of CH 4
30. (b) x µ 1/KH
31. (c) The partial pressure of the gas in vapour phase (p) is m / 32 m / 32 1
= = =
proportional to the mole fraction of the gas (x). m / 32 + m /16 3m / 32 3
p = KH .x Partial pressure of O2 = Total pressure × mole fraction of
Where KH is Henry’s constant. 1 1
32. (a) At high altitudes, the partial pressure of oxygen is O2 , PO2 = P × = P
3 3
less than that at the ground level. This leads to low
concentrations of oxygen in the blood and tissues of 41. (a) KH = 100 kbar = 105 bar, p = 1 bar
people living at high altitudes or climbers. Low blood p = KH × xA
oxygen causes climbers to become weak and unable to p 1
think clearly, symptoms of a condition known as anoxia. xA = K = = 10-5
H 100 ´ 103
33. (b) According to Le-chateliers principle, for an exothermic
reaction (DH < 0) increase in temperature, decreases the 1000
Moles of water = = 55.5
solubility. 18
34. (c) On increasing the pressure over the solution phase Weight of water = 1000 g (Q 1000 mL = 1000 g)
by compressing the gas to a smaller volume (in fig b),
x
increase the number of gaseous particles per unit volume Mole fraction = 10–5 =
over the solution and also the rate at which the gaseous 55.5 + x
particles are striking the surface of solution to enter it. The As 55.5 >>> x, thus neglecting x from denominator
solubility of the gas will increase until a new equilibrium is x
reached resulting in an increase in the pressure of a gas 10-5 = Þ x = 55.5 ´ 10 -5 moles
55.5
above the solution. Thus, its solubility increases. or 0.555 millimoles.
Trick: One of the options (c) or (d) must be the answer.
SOLUTIONS 599

42. (a) % of N2 in atmosphere = 78.9% by volume The value of R and T is same for all the solute however, all
% of O2 in atmosphere = 20.95% by volume of them undergo 100% dissociation
Mole fraction = volume fraction \ pµi×C
Partial pressure of N2 = 0.789 atm = 0.799 bar iKCl = 2, iNaCl = 2, iBaCl2 = 3 and iurea = 1.
Partial pressure of O2 = 0.2095 atm = 0.212 bar nKCl = 1/74.5
According to Henry’s law, 1 / 74.5
CKCl = ´ 1000 = 0.13
0.212 100
PO2 = ( K H )O2 x Þ = xO2 = 6.08 ´ 10-6 or pKCl = 2 × 0.13 = 0.26
(34.86 ´ 1000)
1
0.799 nNaCl=
PN 2 = ( KH ) N 2 x Þ = xN 2 = 1.0447 ´ 10 -5 58.5
(76.48 ´ 1000)
1/ 58.5
æ nO2 ö æ nN 2 ö CNaCl = ´ 1000 = 0.17
100
Þ ç ÷ :ç ÷
è nO2 + nH 2O + n N 2 ø è nO2 + nH 2O + n N 2 ø \ pNaCl = 2 × 0.17 = 0.34
Þ nO2 : n N 2 1
nBaCl2 =
208.4
6.08 ´ 10 -6 :1.04 ´10 -5
= 1 : 1.71 1/ 208.4
CBaCl2 = ´ 1000 = 0.048
43. (c) Like dissolves like. 100
44. (a) For example, decrease in the vapour pressure of water or pBaCl2 = 3 ´ 0.048 = 0.14
by adding 1.0 mol of sucrose to one kg of water is nearly
similar to that produced by adding 1.0 mol of urea to the 1
nurea =
same quantity of water at the same temperature. 60
45. (b) pA° = ? , Given p B° = 200 mm of Hg , xA = 0.6, 1/ 60
Curea = ´ 1000 = 0.16
xB = 1 – 0.6 = 0.4, P = 290 of Hg 100
\ purea = 1 × 0.16 = 0.16
P = PA + PB = PA°x A + PB°xB
\ pBaCl2 < purea < pKCl < pNaCl
Þ 290 = PA° × 0.6 + 200 × 0.4 \ p°A = 350 mm of Hg. or III < IV < I < II
46. (c) The vapour pressure of a liquid increases with 52. (b) ptotal = p°X x X + pY° xY
increase of temperature.
1 3
47. (d) At intersection P1 = P2 and x1 = x2 550 = p X° ´ + pY° ´
48. (a) Given V.PP = 80 torr 4 4
V.PQ = 60 torr p°X + 3 pY° = 550 ´ 4 ...(i)
Ptotal = V·PP × xp + V·Pq × xq In second case
é 3 2ù 1 4
= ê80 ´ + 60 ´ ú = 16 × 3 + 12 × 2 ptotal = p X° ´ + pY° ´
ë 5 5û 5 5
Ptotal = 48 + 24 = 72 torr p°X + 4 pY° = 560 ´ 5 ...(ii)
Subtract (i) from (ii)
49. (b) Moles of glucose = 18 = 0.1
180 \ pY° = 560 ´ 5 - 550 ´ 4 = 600
Q p°X = 400
Moles of water = 178.2 = 9.9 53. (a) Total vapour pressure = vapour pressure of pure
18
benzene + vapour pressure of toluene
Total moles = 0.1 + 9.9 = 10 = 100 + 50 = 150 mm
pH2O = Mole fraction × Total pressure = 9.9 ´ 760 We know,

PC°6H6 = P ´ xC6H 6
10
= 752.4 Torr
50. (d) P = PAXA + PBXB 100 = 150 ´ xC6H6
= PAXA + PB (1 – XA)
100
Þ PAXA + PB – PBXA xC6 H6 = = 0.67
Þ PB+ XA (PA – PB) 150
51. (d) 1% solution contains 1 g of the solute in 100 g of solution. 54. (a) According to idea of Raoult’s law “partial pressure
Osmotic pressure, p = CRT of one of the component is proportional to mole fraction
of that component in the solution.”
EBD_8350
600 CHEMISTRY

P = P1° x1 + P2° x 2 66. (d) Azeotropic mixture is constant boiling mixture, it is


not possible to separate the components of azeotropic
600 = 450 x1 + 700 x2
mixture by boiling.
4.5 x1 + 7x2 = 6
67. (c) Because it shows negative deviation from Raoult’s
Q x1 + x2 = 1, from the given options, it is possible
\ If x1 = 0.4 and x2 = 0.6 law.
Þ x1 = 0.4, x2 = 0.6 68. (a) Minimum boiling azeotrope is formed by solution
55. (a) The solutions (liquid mixture) which boils at constant showing positive deviation. e.g. acetone – ethanol.
temperature and can distil as such without any change in 69. (b)
composition are called azeotropes. 70. (a) According to Raoult’s law,
Solution of HNO3 and H2O will form maximum boiling point PT = xAp°A + xBp°B
azeotrope. Maximum boiling azeotropes show negative Given, PT1 = 500 mm Hg
deviation from Raoult’s law.
nA = 1 and nB = 2 \ xA = 1/3 and xB = 2/3
Composition (%) Boiling Point
HNO 3 68 .0 359 K 1 2
H 2O 32 .0 373 K Þ 500 = p°A + p°B
3 3
Boiling point of the azeotrope of these two solutions is
393.5 K. Þ 1500 = p°A + 2 p°B ...(i)
56. (c) These two components A and B follows the condition
of Raoult’s law if the force of attraction between A and B is Also given that, one more mole of B is added to the
equal to the force of attraction between A and A or B and B. solution, the pressure of the ideal solution increases by 25
mm Hg.
57. (c) For an ideal solution, DH = 0 , DV = 0
Hence, option (c) is incorrect. \ PT2 = 500 + 25 = 525 mm Hg
58. (d) C2H5I and C2H5OH form non-ideal solution, because Also, nB = 3 \ xA = 1/4 and xB = 3/4
of hydrogen bonding in ethyl alcohol.
59. (d) A solution containing A and B components shows 1 3
525 = p°A + p°B ...(ii)
negative deviation when A–A and B–B interactions are 4 4
weaker than that of A–B interactions. For such solutions.
DH = –ve and DV = –ve 2100 = p°A + 3 p°B Subtract (i) and (ii),
60. (b) p°B = 600 mm Hg
61. (d) CCl4 is non-polar and CHCl3 is polar. p°A + 2p°B = 1500 Þ p°A = 300 mm Hg.
62. (a) On adding acetone, its molecules get in between the 71. (b) According to Raoult's law, the relative lowering in
host molecules and break some of the hydrogen bonds vapour pressure of a dilute solution is equal to the mole
between them. Due to weakening of interactions, the fraction of the solute present in the solution.
solution shows positive deviation from Raoult's law. p° - p n
63. (b) Positive deviations are shown by such solutions in which = Mole fraction of solute =
p n+ N
solvent-solvent and solute-solute interactions are stronger
than the solute-solvent interactions. In such solution, the 72. (a) Given vapour pressure of pure solvent
interactions among molecules becomes weaker. Therefore their (P°) = 121.8 mm Hg; Weight of solute (w) = 15 g
escaping tendency increases which results in the increase in Weight of solvent (W) = 250 g; Vapour pressure of solution
their partial vapour pressures. (P) = 120.2 mm Hg and Molecular weight of solvent
In pure methanol, there exists intermolecular H–bonding. (M) = 78
---O – H--- O—H --- O—H--- From Raoult’s law
| | | Po - P w M
CH3 CH3 CH3 = o
= ´
P m W
On adding benzene, its molecules come between ethanol
molecules, thereby breaking H-bonds which weaken 121.8 - 120.2 15 78
= ´
intermolecular forces. This results in increase in vapour 121.8 m 250
pressure. 15 ´ 78 121.8
64. (c) Acetone and chloroform shows negative deviation or m= ´ = 356.2
250 1.6
from Raoult's law. When these are mixed, the hydrogen
bonding takes place between the two molecular species due 73. (d) P° - P n
=
to which escaping tendency of either liquid molecules P° N
becomes less and boiling point of solution increases. 640 - 600 2.5 x
65. (b) For this solution, intermolecular interactions between =
n-heptane and ethanol are weaker than n-heptane-n- 640 39 78
heptane & ethanol-ethanol interactions. Hence the 640 ´ 78 ´ 2.5
solution of n-heptane and ethanol is non-ideal and shows x= = 80
positive deviation from Raoult’s law. 39 ´ 40
SOLUTIONS 601

74. (c) According to Raoult's law Ca (NO3)2 ¾¾ ® Ca2+ + 2 NO3–


p° - p Total ions produced = 3
= xB
p° Na2SO4 ¾¾ ® 2 Na+ + SO42–
Total ions produced = 3
é .2 1ù
ê xB = Mole fraction of solute = .2 + .8 = 5 ú \ 0.1 M Ca (NO3)2 and 0.1 M Na2SO4 are isotonic.
ê ú 86. (a) As both the solutions are isotonic hence, there is no
ë p = 60 mm of Hg û net movement of the solvent occurs through the
p° - p 1 semipermeable membrane between two solutions.
= or 4 p° = ( p) ´ 5
p° 5 87. (d) All are colligative properties.
60≥5 88. (b) Relative lowering of vapour pressure is given by :
Þ p° < < 75mm of Hg P ° – Ps w/m
4 =
P° w/ m +W / M
P o - Ps n w M where, P° = vapour pressure of pure solvent
75. (c) = = ´
o N m W Ps = vapour pressure of solution
P
w = mass of solute
12 18
0.1 = ´ m = molecular mass of solute
m 108 W = mass of solvent
12 ´ 18 M = molecular mass of solvent
m= = 20 For dilute solution
0.1 ´ 108
121.8 - 120.2 15 / m 15 78
76. (a) The vapour pressure of a solution of glucose in water Þ = = ´ = 1.3 ´ 10-2
can be calculated using the relation 121.8 250 / 78 m 250
p° - ps Moles of glucose in solution Þ m = 356.265
= 89. (d) Mass of non-volatile solute = 1g
ps Moles of water in solution
Molar mass of solute = 250g mol–1
17.5 - ps 18/180 Mass of benzene = 51.2g, Kf = 5.12 K kg mol–1
or = [Q p° = 17.5 ]
ps 178.2/18 K f ´ 1000 ´ w2
DT f =
0.1´ ps M 2 ´ w1
or 17.5 – ps = or ps = 17.325 mm Hg.
9.9 where, w2= mass of the solute
77. (c) Osmotic pressure is a colligative property.
M2 = molar mass of solute
0.1 ´180 ´100 w1 = mass of the solvent
78. (c) Kb = =1 K / m
1.8 ´1000 5.12 ´1000 ´1
On substituting given values, DT f =
79. (c) 51.2 ´ 250
K f ´ W2 ´1000 1 1000 \ DT f = 0.4K
DT = K f m = = 5.12 ´ ´ = 0.4K
M 2W1 250 51.2 é P° - P ù w2 M 1
80. (b) DT f = K f ´ m = 1.86 ´ 0.5 = 0.93°C; T f = -0.93°C 90. (b) êë P° úû ´ 100 = M × w ´ 100 = 10
2 1
81. (d) Addition of solute to water decreases the freezing
point of water (pure solvent). w2 18
´ ´ 100 = 10 or w2 = 60 g
\ When 1% lead nitrate (solute) is added to water, the 60 180
freezing point of water will be below 0°C. Thus, 60 g of the solute must be added to 180 g of water so
1000W2 1.86×1000×68.5 that the vapour pressure of water is lowered by 10%.
82. (a) DT f = K f = = 0.372 91. (b) Benzoic acid exists as dimer in benzene.
M 2W1 342×1000
92. (a) The resulting osmotic pressure of the solution is given
Tf = T °f –DTf
p1 + p 2 1.64 + 2.46
Tf = – 0.372°C by p s = = = 2.05 atm
2 2
83. (b) Blood cells neither swell nor shrink in isotonic 93. (c) Isotonic solutions have same osmotic pressure
solution. As isotonic solutions have equal concentration, pglucose = punknown solute
therefore, there is no flow of solvent occurs and hence,
solvent neither enters nor flow out of the blood cells. m1 m 5 2
\ = 2 or = Þ M 2 = 72
84. (a) Isotonic solutions have same molar concentration at M1 M 2 180 M 2
given temperature, provided the van't Hoff factor (i) is
94. (a) The lowering of vapour pressure lowers the volatility
same.
85. (a) The solution which provide same number of ions are of liquid. This lowers the freezing point of solution.
isotonic. 95. (c) Solvent molecules flow from low conc. to high conc.
EBD_8350
602 CHEMISTRY

96. (a) From Raoult law D Tb = i × Kb × m


For lowering of vapour pressure,
p° – p No.of moles of solute
=
p° No. of moles of solvent+ No. of moles of solute °
Psolvent - Psolution æ n ö
= iç .
When the concentration of solute is much lower than the ° è N + n ÷ø
Psolvent
concentration of solvent,
101. (d)
p° - p No. of moles of solute
= 102. (a) Concentration of particles in CaCl2 solution will be
p° No. of moles of solvent maximum as i = 3 for CaCl2 and i = 2 for KCl.
DTb = Kb × m Glucose and Urea do not dissociate into ions, as they are
Number of moles of the solute nonelectrolytes.
m= ´ 1000 103. (a) K4[Fe(CN)6] and Al2(SO4)3 both dissociates to give 5
Mass of solvent in grams
ions or i = 5
Number of moles of the solute ˆˆ† 4K+ + [Fe(CN)6]4–
K4 [Fe(CN)6] ‡ˆˆ
DTb = K b ´ × 1000
Mass of solvent in grams ˆˆ† 2Al3+ + 3SO 2–
and Al 2 (SO4 )3 ‡ˆˆ 4
Number of moles of solute
DTb ´ Mass of solvent in grams 104. (c) DTb = Kb ´ m
= Elevation in boiling point is a colligative property, which
K b ´ 1000
depends upon the no. of particles (concentration of
2 ´100 solution). Thus, greater the number of particles, greater is
= = 0.26,
0.76 ´1000 the elevation in boiling point and hence, greater will be
100 its boiling point.
Number of moles of solvent = = 5.56
Na2SO4 ƒ 2Na+ + SO42–; i = 2
18
DTb = 3 × 0.01 × Kb
From equation (i) we get, 760 - p = 0.26 105. (b) DTb = Kb × i × m
760 5.56
Where DTb = Elevation in boiling point
On solving, p = 724.46 » 724
Kb = molal elevation constant
97. (d) (100 + DTb) – (0 – DTf ) = 105 i = van’t Hoff factor
DTb + DTf = 5 \ D Tb µ molality..
m (Kb + Kf ) = 5 106. (a) DTf = i × Kf × m
5 5 van't Hoff factor, i = 2 for NaCl, m = 0.01
m= i.e., moles in 1000 g water hence DTf = 0.02 Kf which is maximum in the present case.
2.37 2.37
Hence, DTf is maximum or freezing point is minimum.
5 +
(or) moles in 100 g water 107. (d) ( HX H + X - , i = 1.3); DTf = Kf × m × i
2.37 ´ 10 1- 0.3 0. 3 0.3
5 DTf = 1.85 × 0.2 × 1.3 = 0.481º C
\ Wt. of sucrose = ´ 342 = 72g
2.37 ´10 \ Tf = T f° - DT f = 0 – 0.481ºC = – 0.481ºC
98. (b) Given Kb = x K kg mol–1 108. (c) Na2SO4 ¾¾ ® 2Na+ + SO 24-
DTb = Kb × m
Mol. before dissociation 1 0 0
\ y=x×m
Mol. after dissociation 1–a 2a 1a
y i = 1 - a + 2a + a = 1 + 2a
m=
x
No. of particles after ionisation
We know 109. (a) (i) i =
DTf = Kf × m No. of particles before ionisation
On substituting value of m and Kf (ii) DTb = i × Kb × m
yz ® Cu 2+ + 2Cl –
CuCl2 ¾¾
DTf =
x 1 0 0
99. (c) If compound dissociates in solvent i > 1 and on (1 – α) α 2α
association i < 1.
1 + 2a
100. (a) Van’t Hoff equation is i= , i = 1 + 2a
1
pV = inRT Assuming 100% ionization
For depression in freezing point. So, i = 1 + 2 = 3
D Tf = i × Kf × m 13.44
DTb = 3 ´ 0.52 ´ 0.1 = 0.156 » 0.16 [m = = 0.1]
For elevation in boiling point. 134.4
SOLUTIONS 603

DT f = i .k f .m ; DTb = i.kb .m 5. (11.7) Mass of NaCl, w2 = ?, Volume of water = 1000 mL,


110. (d)
DTf = 0.744 K, Density of water = 1g mL–1
ΔT f kf So, mass of water w1 = 1000 mL × 1 g mL–1
= = 1000g = 1 kg
ΔTb kb
(w 2 / 58.5) iK f w 2
DT f = 0 - ( -0.186°C) = 0.186°C DTf = iK f m = iK f ´ =
w1 58.5 ´ w1
0.186 1.86 0.52 ´ 0.186 DTf ´ 58.5 ´ 1 0.744 ´ 58.5
= DT = = 0.052
DTb 0.52 Þ b 1.86
w2 =
i ´ Kf
=
2 ´ 1.86
= 11.7g

(Q i for NaCl = 2)
Exercise - 2 So, Mass of NaCl required = 11.7g.
1. (3) Let the solubility of Al(OH)3 in 0.2M NaOH w
solution be s. Pº - P m
6. (111) =
Then, Pº w W
+
m M
ˆˆ† Al3+ + 3OH -
Al(OH)3 ‡ˆˆ Let the initial (normal) pressure (Pº) = P
s 3s
75 3
ˆˆ† Na + + OH -
and NaOH ‡ˆˆ \ Pressure of solution = ×P= P
100 4
0.2 M 0.2 M 0.2 M
m = 60, M = 18, W = 100 g
[Al3+] = s and = 3s + 0.2 » 0.2
[OH–] 3
Ksp = 2.4 × 10 = [Al3+] [OH]3
–2 P- P
\ 4 = w / 60
2.4 × 10–2 = s(0.2)3 P w 100
+
60 18
2.4 ´ 10-2
s= = 3mol / L 1 w / 60 4w w
8 ´ 10 -3 = or = + 5.55
4 ( w / 60) + 5.55 60 60
2. (0.017) Relative lowering of vapour pressure, is given or w = 111 g
p° - p nA n 7. (12.65) Molarity
by, = xA = ; A
p° n A + nB nB Mass of solute /M. wt. of solute
= × 1000
Mass of solution/density of solution
0.60
Given, p° = 35mm Hg, nurea = , 86/98
60 M= × 1000
100/1.787
360
nwater = 0.8775
18 = × 1000 = 15.81 M
55.5
p° - p 0.6 ´ 18 1
= = Now,
35 60 ´ 360 2000 M1V1 = M2V2
Dp = p° - p = 0.017 \ 15.81 × V1 = 0.2 × 1000
3. (0.36) Dissociation of Potassium Sulphate (K2SO4), 0.2×1000
or V1 = = 12.65 mL
15.81
K2SO4 ¾¾ ® 2K + + SO 24 -
\ Amount of acid to be used to make 1 L of 0.2 M H2SO4
i (Van’t Hoff factor) = 3 = 12.65.
We know that, DTf = iKfm 8. (3.84) N1 = 1, V1 = ?, N2 = 26.7, V2 = 0.4
where, Kf is molal depression constant and m is N1V1 = N2V2
1 × V1 = 26.7 × 0.4
molality.
\ DTf = 3 × 4 × 0.03 = 0.36 K 26.7 ´ 0.4
V1 = = 10.68
1
4. (0.06) We know, p = iCRT; pxy = 4p BaCl
2 49g (Q eq wt of H2SO4 = 49) of H2SO4 will be neutralised
\ 2[XY] = 4 × (0.01) × 3 by 1N 1000 mL of NaOH.
[XY] = 0.06 \ 0.5g of H2SO4 will be neutralised by
= 6 × 10–2 mol/L = 0.06 mol/L 1000
= ´ 0.5 = 10.20 mL 1N NaOH
49
EBD_8350
604 CHEMISTRY

Volume of 1 N NaOH used by dissolved SO3 5. (b) At high altitude, the partial pressure of oxygen is
= 10.68 – 10.20 = 0.48 mL less than at the ground level. This decreased atmospheric
pressure causes release of oxygen from blood. Hence,
SO3 + 2NaOH ¾¾
® Na2SO4 + H2O people living at high altitude have low concentration of
Mol wt 80 oxygen in the blood and tissues.
\ Eq wt of SO3 = = = 40 6. (b) Colligative properties depend upon number of solute
2 2
particles in solution irrespective of their nature.
Wt of SO3 in 0.48 ml of 1 M solution 7. (b) Colligative properties depends upon the number of
40 particles. As we know, greater the value of van't Hoff factor,
= ´ 0.48 = 0.0192 g higher will be elevation in boiling point and hence, higher
1000
will be the boiling point of solution. Among the given
0.0192 solution 1.0M Na2SO4 have highest no. of particles i.e.
% of SO3 = ´ 100 = 3.84% highest value of ‘i’.
0.5
Hence, 1.0 M Na2SO4 has highest value of boiling point.
9. (65.25) According to Raoult's law
8. (a) Elevation in boiling point DTb = Kbm
P° - P w/m
= ΔTb
P° w/ m +W / M Kb =
m
Here, P° = 640 mm Hg, P = 600 mm Hg, w = 2.175g, unit of DTb K
W = 39.0 g, Unit of Kb = unit of m = molality
M = 78, m = Molecular weight of solute
K
Substituting the various values in the above equation,
= = K mol -1 kg
640 - 600 2.175 / m mol kg -1
=
640 2.175 / m + 39 / 78 9. (c) DTf = iKf m i.e., depression in freezing point is directly
m = 65.25 g related to van't Hoff factor (i). Glucose is a non-electrolyte
hence, will remain undissociated. Hence, for glucose, value
0.4 ´ 1000
10. (5.92) Molality (m) of solution = = 0.05 of i is 1, whereas for MgCl2 value of i is 3.
80 ´ 100 Hence, depression in freezing point of MgCl2 is about 3
ΔTf (normal) = Kf × m = 1.86 × 0.05 = 0.093 K times of glucose.
Van’t Hoff factor, 10. (a) Osmotic pressure (p) = CRT.
For concentrated solution, C has higher value than dilute
ΔTf (observed) 0.12 solution.
i= = = 1.290
ΔTf (normal) 0.093 Hence, as concentration of solution increases, osmotic
pressure also increases.
ˆˆ† H O+ + A-
HA + H2O ‡ˆˆ 11. (b) Number of total ions present in the solution is known
3

i – 1 1.290 –1 as van't Hoff factor (i).


a= = = 0.29 for KCl, i=2
n –1 2 –1
for NaCl, i=2
for K2SO4, i=3
Ca 2 0.05 ´ (0.29) 2
Ka = = = 5.92 ´ 10–3 12. (a) The value of Henry's constant (KH) increases with
1– a 1 – 0.29 increase in temperature.
Exercise - 3 13. (b) According to Henry's law,
pµx
1. (a) According to Henry's law partial pressure of a gas in Þ p = KHx Þ x = p/KH
the solution is proportional to the mole fraction of gas in As value of KH rises, solubility of gases decreases.
the solution. 14. (a) On adding salt to water to make the salt solution the
p = KH x ; KH = (Henry's constant) vapour pressure of solution gets lowered. This is due to
2. (d) Dissolution of sugar in water will be most rapid when the decrease in surface covered by solvent molecule which
powdered sugar is dissolved in hot water because in leads to decrease in number of solvent molecule escaping
powdered form it can easily insert in the vacancies of from the surface corresponding to pure solvent.
water particle. Hence, vapour pressure also get reduces.
Also, dissolution of sugar in water is an endothermic 15. (d) As we know, M1V1 = M2V2
process. So it is favourable at high temperature. On putting values, we get
3. (c) Rate of dissolution = Rate of unsaturation 0.02 × 4 L = M2 × 5L
4. (b) When solute gets precipitated in the solution, then 0.08
the solution is known as supersaturated solution. M2 = = 0.016 M
5
SOLUTIONS 605

16. (c) Maximum amount of solid that can be dissolved in a 26. (c) Value of KH depends upon nature of gases dissolved
specified amount of a given solvent does not depend upon in water. Higher the value of KH at a given temperature,
pressure. The reason being, solid and liquid are highly the lower is the solubility of the gas in the liquid. Hence,
incompressible and practically remain unaffected by correct order is :
change in pressure. Ar < CO2 < CH4 < HCHO.
17. (a) Mixture of methanol and acetone show a positive 27. (c) Colligative properties µ no. of particles. Since
deviation from Raoult’s law. Molecules in pure methanol Al2(SO4)3 contains maximum number of particles, hence
are hydrogen bonded. On adding acetone, its molecules will have the largest value of freezing point depression.
enters in between the host molecules and break some of 28. (a) p = i CRT
the hydrogen bonds between them. pC H OH = 1 × 0.500 × R × T = 0.5 RT
2 5
Therefore, the intermolecular attractive forces between the pMg (PO ) = 5 × 0.100 × R × T = 0.5 RT
solute-solvent molecules are weaker than those between the 3 42
pKBr = 2 × 0.250 × R × T = 0.5 RT
solute-solute and solvent-solvent molecules.
Other three remaining options will show negative pNa PO = 4 × 0.125 × RT = 0.5 RT
3 4
deviation. Since, the osmotic pressure of all the given solutions is
18. (d) When an unripe mango is placed in a concentrated equal. Hence, all are isotonic solution.
salt solution to prepare pickle then mango loose water ¾¾® 4K+ + [Fe(CN)6]–
29. (c) K4[Fe(CN)6] ¬¾
¾
due to osmosis and get shrivel.
19. (a) The value of molal depression constant, Kf depends and Al2(SO4)3 ® 2Al3+ + 3SO2– 4
upon nature of solvent. Therefore, two different solutions \ van’t Hoff factor is 5 for both Al2 (SO4)3 and K4[Fe(CN)6]
of sucrose of same molality prepared in different solvents 30. (d) DTb = iKb m
will have different depression in freezing point. Given, (DTb)x > (DTb)y
20. (b) In reverse osmosis, solvent molecules move \ ix Kb m > iyKb m
through a semipermeable membrane from a region of (Kb is same for same solvent)
higher concentration of solute to lower concentration. ix > iy
21. (b) If a pressure higher than the osmotic pressure is So, X is undergoing dissociation in water.
applied on the solution, the solvent will flow from the 31. (a) For an ideal solution, DSmix > 0
solution into the pure solvent through the semi-permeable W 1
membrane. This process is called reverse osmosis. 32. (d) Molality = ´
M V (kg)
Thus, in this case, if a pressure greater than osmotic n
pressure is applied on piston (B). Water will move from < (where nsolute = W/M)
V (kg)
side (B) to side (A). Thus, option (b) is also correct.
22. (c) As van’t hoff factor depends only on no. of n
1.00 m <
dissociated ions. Hence, i will be independent on V (kg) i.e., 1 mole in 1 kg of water
concentration of solutions, due to NaCl being a strong Moles of 1 kg H2O
electrolyte.
1000g
23. (b) A mixture of bromoethane and chloroethane is an < < 55.55 mole
18g / mol
example of ideal solution. For an ideal solution, the
A — A or B — B type intermolecular interaction is nearly Moles of solute = 1
equal to A — B type interaction. Mole fraction
Chloroform and acetone mixture is an example of nsolute 1
non-ideal solution having negative deviation while < <
nsolute + nwater (1 ∗ 55.55)
ethanol-acetone mixture shows positive deviation.
24. (d) If two liquids A and B form minimum boiling azeotrope = 0.01768 = 0.0177
at some specific composition then A — B interactions are 33. (c) Let the weight of acetic acid initially be w1 in 50 ml of
weaker than those of A — A and B — B. Because in case 0.060 N solution.
of positive deviation, we get minimum boiling azeotropes w1 ´ 1000
N= (Normality = 0.06 N)
whereas in case of negative deviation we get maximum M.wt. ´ 50
boiling azeotropes.
w1 ´ 1000
25. (a) At specific composition methanol-acetone mixture 0.06 =
will form minimum boiling azeotrope and will show positive 60 ´ 50
deviation. This is due to weaker A — B interaction than A 0.06 ´ 60 ´ 50
— A and B — B interaction. Þ w1 = = 0.18 g = 180 mg.
1000
EBD_8350
606 CHEMISTRY

After an hour, the strength of acetic acid = 0.042 N


W1 ´ 1000
so, let the weight of acetic acid be w2 DTb = m × Kb = M ´ W × Kb
1 2
w2 ´ 1000
N=
60 ´ 50 0.52 ´ 6.5 ´ 1000
DTb = = 1.06 °C
w2 ´ 1000 31.75 ´ 100
0.042 = \ boiling point of solution
3000
= 100°C + 1.06°C = 101°C
Þ w2 = 0.126 g = 126 mg
37. (a) According to Raoult's Law
So amount of acetic acid adsorbed per 3g
= 180 – 126 mg = 54 mg P° - Ps WB × M A
= …(i)
Amount of acetic acid adsorbed per g Ps M B ×WA
54 Here P° = Vapour pressure of pure solvent,
= = 18mg
3 Ps = Vapour pressure of solution
34. (d) Using relation, WB = Mass of solute, WA = Mass of solvent
MB = Molar mass of solute, MA = Molar Mass of solvent
p° - ps w2 M 1 Vapour pressure of pure water at 100° C (by assumption =
=
ps w1M 2 760 torr)
where w1, M1 = mass in g and mol. mass of solvent By substituting values in equation (i) we get,
w2, M2 = mass in g and mol. mass of solute 760 - Ps 18 ´18
Let M2 = x = …(ii)
Ps 180 ´178.2
p° = 185 torr
ps = 183 torr On solving (ii) we get
Ps = 752.4 torr
185 - 183 1.2 ´ 58 38. (c) Kf (molal depression constant) only depends on the
= (Mol. mass of acetone = 58)
183 100 x nature of the solvent and is independent of the
x = 64 concentration of the solution.
\ Molar mass of substance = 64 39. (a) Molarity depends on the volume of a solution which
35. (a) Let us consider that A is benzene and B is toluene can be changed with change in temperature.
1 : 1 molar mixture of A and B 40. (d) Colligative properties depends upon the no. of
particles. It cannot compare two pure solvents.
1 1
\ xA = and xB = 41. (d) In benzene
2 2 2CH3COOH ƒ (CH3COOH)2
Total pressure of solution (P) = PAo x A + PBo xB 1–a a/2
1 1 i = 1 – a + a/2 = 1 – a/2
P = 12.8 × + 3.85 × = 8.325 kPa
2 2 Here, a is degree of association
1 DTf = iKf m
12.8 ´
Po x 2 = 0.768
YA = A A =
P 8.325 æ 0.2 ö
çè ÷
\ yB = 1 – yA = 1 – 0.768 = 0.232 æ a ö 60 ø
0.45 = ç1 – ÷ (5.12)
so, the vapour will contain higher percentage of benzene. è 2ø 20
Trick: Benzene has higher vapour pressure. Thus, in 1 : 1 1000
mixture, the vapour will contain higher percentage of a
1 – = 0.527
benzene. 2
a = 0.945
æ P° - Ps ö n W1 M 2 % degree of association = 94.6%
36. (a) ç P° ÷ = N = M × W
è ø 42. (d) Compounds Number of particles (i)
1 2
Where, W1 = wt of solute (b) [Co(H2O)5Cl]Cl2.H2O 3
W2 = wt of solvent (c) [Co(H2O)4Cl2]Cl.2H2O 2
M1 = Mass of solute (d) [Co(H2O)3Cl3].3H2O 1
M2 = Mass of solvent (a) [Co(H2O)6]Cl3 4
at 100 °C, P° = 760 mm
DTf µ i where DTf = (Tf – T 'f )
760 - 732 6.5 ´ 18 Remember, the greater the no. of particles, the lower will be
=
760 M1 ´ 100 the freezing point. Compound (d) will have the highest
M1 = 31.75 g mol–1 freezing point due to least no of particle.
SOLUTIONS 607

43. (c) DHmix is zero at constant T and P. y A PA° x A 4 1 xA


= × Þ = ×
°
y B PB xB 3 3 (1 – x A )
44. (d) PMo = 450 mmHg, PNo = 700 mmHg
4 1
PM = PMo xM = yM PT xA = or xB =
5 5
yM
Þ P°M = x ( PT )
M PA PA° X A X' P°
yN 4. (c) X ¢A = = Þ A = A <1
Similarly, PNo = x ( PT ) P P XA P
N

Given, PMo < PNo (since the liquid A is less volatile, PA° < P )
yM yN P° – P n n
Þ x < xN
a 5. (b) = » (for very dilute solution)
M P° n+ N N
yM xM
Þ y < n n 2 1
N xN Þ 2´ = Þ =
NA NB w / MA w / MB
45. (a) The solubility of the gas in liquids decreases with
the increase in value of KH at a given pressure. Þ MB = 2MA
3-
46. (b) We know, p = iCRT; pxy = 4p BaCl2 6. (b) ˆˆ† 3K + + é Fe ( CN ) ù
K3 éë Fe ( CN )6 ùû ‡ˆˆ ë 6û

\ 2[XY] = 4 × (0.01) × 3 At t = 0 1 0 0

[XY] = 0.06 After ionisation (1 – a) 3a a


i = (1+3a) / 1 = 1 + 3a
= 6 × 10–2 mol/L
M normal
92 Since, i = ;
47. (b) Number of moles in 92 g of Na+ = = 4 moles M abnormal
23
Number of moles 1 + 3a M normal
Molality (m) = \ =
Mass of solvent (in kg) 1 M abnormal
4 5 5
\ m= = 4 mol kg–1 7. (c) Moles of urea = ; moles of fructose = ;
1 60 180
48. (b) DT f = k f m = 5.12 ´ 0.078 = 0.399 K = 0.40 K 5
moles of sucrose = ;
49. (d) Hydron bond of ethanol gets weakened by addition 342
of acetone. Thus the mixture of ethanol and acetone show 5 5
moles of KCl(effective) = 2 ´ =
positive deviation from Roult's law. 74.5 37.25
50. (b) Mixture of carbon disulphide and acetone will show Osmotic pressure p = CRT or p µ C
positive deviation from Raoult’s Law. Thus, p4 > p1 > p2 > p3
The dipolar interaction between solute (CS2) solvent 8. (b) Total V.P.
(acetone) molecules in solution are weaker. So the vapour 2 3
pressure of solution will be greater than the individual = PAº X A + PBº X B = 100 ´ + 150 ´ = 130 torr
5 5
vapoure pressure of pure components.
The observed vapour pressure is smaller than that
Exercise - 4 calculated from Raoult’s law (negative deviation). Hence,
interactions A – B > A – A or B – B.
1. (b) For an ideal solution, DHmixing = 0 9. (d) Only solvent molecules can passed through SPM
DH = DH1 + DH2 + DH3 (According to Hess's law) so, only dilution is possible.
i.e., for ideal solutions, there is no change in magnitude of 10. (c) Molarity (experimental)
the attractive forces in the two components present.
2. (d) Solution X is unsaturated so v.p. will be more, DT f3.82
solution Y and Z are saturated so v.p. of Y = v.p. of Z and = = = 2.054 mol/1000 g solvent
Kf 1.86
2 g of solute would be present in form of solid in system Z. mole of solute
Molarity (theoretical) = ´ 1000
PA P° x PB° xB wt. of solvent(g)
3. (a) yA = Þ A A and y B = ; 5 g /142 g / mole
P P P = ´ 1000
x
EBD_8350
608 CHEMISTRY

® 2Na + + SO 42 -
Na 2SO 4 ¾¾ Hence , Fe3+ and CNS– ions do not come in contact on
Moles before either side.
dissociation 1 0 0
Moles after 12. (c) DT f (normal) = K f m = 1.86 ´ 0.01 = 0.0186 ;
dissociation 1–x 2x x

Moles after dissociation ΔT f (obs) 0.0205


Von't Hoff Factor (i) = i= = = 1.10 = 1 + a ; a = 0.1
Moles before dissociation ΔT f (nor) 0.0186
(1 - x ) + 2 x + x
=
1 Ca 2 0.01´ 0.12 1
Ka = = = ´10 -3 ;
Na2SO4 is ionised 81.5% 1- a 1 - 0.1 9
means x = 0.815
Kw
(1 - 0.815) + 2 ´ 0.815 + 0.815 Kb = = 1.0 ´10 -14 ´ 9 ´103 = 9 ´ 10 -11
= = 2.63. Ka
1
Observed molarity 13. (b) Super cooled liquid has temperature below its freezing
i= point. On commencement of freezing, the latent heat of
Calculated molarity
freezing evolves causing the temperature to rise to F.Pt.
2.054 (temperature of liquid-solid equilibrium).
Þ 2.63 = Þ x = 45.07 g.
0.0352 14. (b) AlCl3 furnishes more ions than CaCl2 and thus,
´1000
x possess higher boiling point i.e., T1 > T2.
11. (d) Only solvent molecules and not the solute molecules 15. (d) mKf = DTf has the units of temperature. van’t Hoff
or ions can pass through the semipermeable membrane. factor, i, is dimensionless quantity.
17 Electrochemistry
Exercise - 1 17. (a) Higher the value of reduction potential higher will be
the oxidising power whereas the lower the value of
1. (a) Anode has negative polarity. reduction potential higher will be the reducing power.
2. (b) Oxidation takes place at zinc anode.
3. (d) When both the electrodes are kept in the same 0.059 1
18. (a) E = E° - log +
solution there will be no requirement of salt bridge. n [H ]
4. (d) Daniell cell is a type of galvanic cell. 0.059 1
5. (b) The cell in which Cu and Zn rods are dipped in its = 0- log -4 = -0.236V
solution is called Daniell cell. 1 10
6. (c) The magnitude of the electrode potential of a metal is 19. (b) For Zn2+ ® Zn
a measure of its relative tendency to loose or gain electrons.
i.e., it is a measure of the relative tendency to undergo E = E° -
2.303RT
log
[ Zn ]
Zn 2+ /Zn Zn 2+ /Zn
oxidation (loss of electrons) or reduction (gain of nF [Zn 2+ ]
electrons). 0.06 1
= -0.76 - log = -0.76 - 0.03
M ® M n + + ne - (oxidation potential) 2 [ 0.1]
M n + + ne - ® M (reduction potential) E
Zn 2 + / Zn
= -0.79V
7. (b) 2AgCl(s) + H 2 (g) ® 2HCl(aq) + 2Ag(s) 20. (c) Using the relation,
The activities of solids and liquids are taken as unity and
at low concentrations, the activity of a solute is 2.303 RT 0.0591
E°cell = log K c = log K c
approximated to its molarity. nF n
The cell reaction will be 0.0591
+ \ 0.295 V = log K c
Pt(s) | H 2 (g),1bar | H (aq)1M | AgCl(aq)1M | Ag(s) 2
8. (d) Cu is anode and Ag+ is cathode. 2 ´ 0.295
9. (d) Calomel electrode is used as reference electrode. or log Kc = = 10
10. (b) Given E 4+ 2+ = + 0.15 V 0.0591
Sn Sn or Kc = 1 × 1010
ECr3+ Cr = – 0.74 V 21. (b) Without losing its concentration, ZnCl 2 solution
Sn4+ will reduce and Cr will oxidize, as the standard cannot kept in contact with Al because Al is more reactive
reduction potential value is positive for Sn4+. than Zn due to its highly negative electrode reduction
potential.
° = Eox
Ecell ° + Ered
° = (0.74 + 0.15) V = 0.89 V
22. (b) Suppose A and B forms, A+ and B+ ions. From the
11. (d) As the reduction potential for second reaction is more given value of oxidation potentials, it is clear that A will
than that of first reaction, Ag2O will reduce and Zn will oxidize to A+ and B+ will reduce to B.
oxidize.
A + B+ ¾® A+ + B or
E°Cell = E°OP + E°RP = 0.76 + 0.34 = 1.10 V
12. (d) According to an accepted convention, anode is A + BX ¾® AX + B
written on the left side and cathode on the right while We can say that A will replace B from its solution.
representing the galvanic cell.
13. (a) Salt bridge allows the flow of current by completing 23. o
(c) Gold having higher ERed ® Fe 2 + .
and oxidises Fe ¾¾
circuit. No current will flow and voltage will drop to zero, if 0.0591
salt bridge is removed. 24. (d) E° = log Kc
n
14. (d) In electrolytic cell, the flow of electrons is from anode
to cathode through internal supply. Here, n = 2, E ° = 0.295
15. (c) Mg ¾® Mg2+ 2e– (oxidation at anode) 2 ´ 0.295
\ log Kc = = 9.98 » 10 or Kc = 1010
Ag+ + e– ¾® Ag (reduction at cathode) 0.0591
Anode is written on the left and cathode is written on the o RT
right side. 25. (d) Ecell = Ecell - ln Q
nF
16. (c) As the value of standard reduction potential decreases At equilibrium,
the reducing power increases i.e., Ecell = 0 and Q = Kc
Z > X > Y o
( -3.0) ( -1.2) ( +0.5) \ Ecell ¹ Ecell
EBD_8350
610 CHEMISTRY

26. (c) The E°cell is given by [Sn 2 + ] 0.01 ´ 2


or log = = 0.3 (Q Ecell = 0)
0.0591 [Pb 2 + ] 0.059
E°cell = log Kc
n
0.0591 [Sn 2+ ]
\ 0.591 = log Keq or = antilog (0.3)
1 [Pb 2 + ]
0.591 35. (d) Ecell = 0; when cell is completely discharged.
or log Kc = = 10
0.0591 æ é Zn 2 + ù ö
0.059
or Kc = 1 × 1010 Ecell = E°cell - log ç ë û÷
2 ç é 2+ ù ÷
27. (b) For the given cell ç Cu ÷
èë ûø
0.059V [Zn 2+ (aq)] æ é Zn 2 + ù ö
Ecell = E°cell - log
0.059
2 [Cu 2+ (aq)] or 0 = 1.1 - log ç ë û÷
ç + ÷
The cell potential decreases with increase in [Zn 2+ (aq)]
2 ç éCu ù ÷
2
èë ûø
and increases with increase in [Cu2+(aq)].
æ é Zn 2+ ù ö
-D G ° û ÷ = 2 ´1.1 = 37.3
28. (c) DG ° = - nFE °; E ° = ; log ç ë
nF ç é 2+ ù ÷ 0.059
ç Cu ÷
-(-50.61kJ) èë ûø
E° = = 0.26V
2 ´ 96500 ´10-3 [Zn 2 + ]
\ = 1037.3
29. (c) Because reduction potential of water is higher than [Cu 2 + ]
that of Na+ so H2 will be evolved and no reduction of 36. (a) The Half cell reaction is Zn 2+ + 2e - ¾¾ ® Zn .
metal ions occurs.
o 0.059 1 [E°cell = – E°ox]
30. (b) E° 2+ 4+ = -0.15V > E° 2+ Ecell = Ecell - log
2+ = - 0.92 2 [Zn 2+ ]
Sn / Sn Hg 2 / Hg

Hence, Sn 2+ is stronger reducing agent than Hg 22 + . = - 0.763 - 0.059 log 1 = -0.822V


2 0.01
0.591
31. (d)
o
Ered = Ered + log[ M n + ] Eoxi = 0.822 V
n
Lower the concentration of M n+, lower is the reduction 37. (d) 2H + + 2e - ¾
¾® H 2
potential.
Hence order of reduction potential is : Q > R > S > P o 0.059 1
ERed = ERed - log + 2 ;
32. (b) (i) A3+ + e– ––––® A2+, DG1 = (– 1) F y2 n [H ]
(ii) A2+ + 2e– ––––® A, DG2 = (–2)F(–y1) = 2Fy1
0.059 1
Add, (i) and (ii), we get E Red = 0 - log ; ERed = –0.059 V,,
A3+ + 3e– ––––® A ;
2 (0.1) 2
DG3 = DG1 + DG2 Eoxi = 0.059 V.
–3FE° = –Fy2 + 2Fy1 38. (d) Here n = 4, and [H+] = 10– 3 (as pH = 3)
–3FE° = –F (y2 – 2y1) Applying Nernst equation
y2 - 2 y1 0.059 [Fe 2 + ]2
E° = E = Eº – log
3 n [H + ]4 (pO2 )
33. (a) Zn(s) + 2H + (aq) Zn 2+ (aq) + H 2 (g)
0.059 (10-3 )2
2+ = 1.67 - log
Ecell = E °cell -
0.059
log
[Zn ][H 2 ] 4 (10-3 ) 4 ´ 0.1
2 [H + ]2 0.059
= 1.67 - log107 = 1.67 – 0.103 = 1.567 V
Addition of H2SO4 will increase [H+]and Ecell will also 4
increase and the equilibrium will shift towards RHS. 39. (a) Cell reaction
34. (a) Apply Nernst equation to the reaction
1 -
Pb + Sn2+ ® Pb2+ + Sn cathode : H 2 O(l) + O 2 (g) + 2e ¾¾ ® 2OH - (aq)
2
0.059 [Pb 2 + ] anode : H 2 (g) ¾¾ ® 2H + (aq) + 2e -
Ecell = E ° - ´ log
2 [Sn 2 + ] ––––––––––––––––––––––––––––––––––––––––
1
0.059 [Sn 2+ ] ® 2H + (aq) + 2OH - (aq)
H 2O(l) + O 2 (g) + H 2 (g) ¾¾
or E° + log = Ecell 2
2 [Pb 2 + ]
ELECTROCHEMISTRY 611

Also we have [G (conductance) – siemens or ohm–1(S).]


1 47. (d) Conductivity does not depend upon mass or weight
H 2 (g) + O 2 (g) ¾¾ ® H 2 O(l) ; of material.
2
DG of = -237.2 kJ / mol 48. (d) L °CH3COOH = L°CH3COONa + L°HCl - L °NaCl
® H + (aq) + OH - (aq) ; DG° = 80 kJ / mol
H 2 O(l) ¾¾ = 91 + 425.9 – 126.4 = 390.5 S cm2mol–1
Hence, for cell reaction Lm 9.54
49. (b) a = ¥ = = 0.04008 = 4.008 %.
Lm 238
DG° = -237.2 + (2 ´ 80) = -77.20 kJ / mol
50. (d) Cell constant = l/a
DG° 77200 Unit = m/m2 = m–1.
\ E° = - = = 0.40 V
nF 2 ´ 96500 51. (b) We know, R µ l or R = r æç l ö÷ , where proportionality
40. (c) When the concentration of all reacting species kept A è Aø
unity, then Ecell = E°cell and the given relation will constant r is called resistivity. If l = 1m and A = 1m2, then
R = r i.e., Resistance = Resistivity..
become D r G = -nFE°cell . e.g. redox reaction for Daniell
cell : Zn(s) + Cu2+ (aq) ® Zn2+ (aq) + Cu(s) 1000 1 l 1000
52. (a) L eq = k ´ = ´ ´
solutions of CuSO4 and ZnSO4 are the reacting species. N R a N
The Ecell for this cell : Ecell = E°cell 1 1000 1 1000
= ´ cell constant ´ = ´ 0.88 ´
RT [Zn 2 + ] R N 220 0.01
- ln Þ Ecell = Ecell = 400 mho cm2 g eq–1
nF [Cu 2 + ] 53. (a) The specific conductance increases with
if [Zn2+] = [Cu2+] =1 concentration. The number of ions per cm–3 increase with
41. (d) Molarity = 0.01 M ; Resistance = 40 ohm; increase of concentration.
l KA
-1 54. (d) Conductance G =
Cell constant = 0.4cm . l
A Molar conductivity of a solution at a given concentration
Specific conductivity (k ) is the conductance of the volume V of solution containing
one mole of electrolyte kept between two electrodes with
cell constant 0.4
= = = 0.01 ohm -1 cm -1 area of cross section A and distance of unit length.
resistance 40
KA
1000k Molar conductivity Ù m =
Molar conductance ( Ù m ) = l
Molarity Since, l = 1 and A = V (volume containing one mole of
1000 ´ 0.01 electrolyte) then Ù m = K .V
= = 103 ohm -1 cm 2 mol -1 If the concentration is C mol/litre then
0.01
42. (b) Specific conductance of the solution (k) = 0.012 Ùm = K / C
ohm–1 cm–1 and resistance (R) = 55 ohm. 55. (c) (i) represents weak electrolyte ( Ù m is never zero)
Cell constant = Specific conductance × Observed resistance (ii) represents strong electrolyte. ( Ù m can be represented
= 0.012 × 55= 0.66 cm–1. by a straight line equation.
43. (b) ohm–1 cm2 (geq)–1
44. (c) Molar conductance of solution is related to specific Ù m = Ù°m - AC1/2 )
conductance as follows : 56. (b) L¥
HCl = 426.2
1000
Ùm =k´ ....(a) (i) ¥
L AcONa = 91.0
C
where C is molar concentration. (ii) L¥
NaCl = 126.5
Putting k = 6.3 × 10–2 ohm–1 cm–1 and (iii) L ¥
AcOH = (i) + (ii) - (iii)
C = 0.1M
= [426.2 + 91.0 - 126.5] = 390.7
1000
Ùm = (6.3 × 10–2 ohm–1 cm–1) × 57. (d) Kohlrausch’s Law states that at infinite dilution, each
(0.1mol / cm3 ) ion migrates independent of its co-ion and contributes to
= 6.3 × 10–2 × 104 ohm–1cm2 mol–1 the total equivalent conductance of an electrolyte a definite
= 630 ohm–1 cm2 mol–1 share which depends only on its own nature.
45. (d) ohm–1 cm–1 From this definition we can see that option (d) is the correct
46. (d) Correct matching for pair (iii) will be answer.
EBD_8350
612 CHEMISTRY

58. (c) Equivalent conductance at infinite dilution of an l


electrolyte is the sum of equivalent conductance at infinite = 65m-1
dilutions of its constituent ions. A
k ´ 1000
L °Al2 (SO 4 )3 + L ° + L° L=
Al3+ SO 42 - molarity
[molarity is in moles/litre but 1000 is used to convert liter
59. (b) L° + L° = 248 ´ 10 -4 Sm 2 mol-1 into cm3]
Na + OH -

L° + L° = 126 ´ 10 -4 Sm 2 mol -1 æ 1 ö
Na + Cl - çè ´ 65 m -1 ÷ ´ 1000 cm 3
260 ø
=
L° + L° = 280 ´ 10 -4 S m 2 mol-1 0.4 moles
Ba 2 + 2Cl -

Now, L°Ba(OH)2 = L°BaCl2 + 2L °NaOH - 2L °NaCl 650 m-1 1


= ´ m3 = 6.25 × 10–4 S m2 mol–1
260 ´ 4 mol 1000
L°Ba(OH)2 = 280 ´ 10-4 + 2 ´ 248 ´ 10-4 - 2 ´ 126 ´ 10-4
° °
L °Ba(OH) 2 = 524 ´ 10- 4 Sm 2 mol- 1. 66. (b) L° for CH3COOH = l CH3COO - + l H+
60.
61.
(d) Larger the size, lower the speed.
(a) ˆˆ† x 2+ +
xy ‡ˆˆ
¥
y 2- L m = 57 + 73 = 130 Scm mol 2 -1
= l°(
CH COO-
3
) (° ° °
) ( °
+ l°Na + + l H+ + l Cl- - l Na + + l Cl- )
k ´ 1000 = 90 + 425 – 125 = 390 mho cm2 mol–1.
62. (d) Molar conductivity Ù m =
M
M = Molarity L cm 7.8
k = specific conductivity Degree of dissociation (a) = = = 0.02
L °m 390
1 1 l
k= ×´ cell constant; = × 2
R R A 67. (d) L C = L 0 ´ a = 400 ´ =8
100
1 1cm -6
= 3
´
2 = 2 ´10 W-1cm -1 or Scm -1 1 l 1000 1000 æ l ö
5 ´10 W 100 cm LC = ´ ´ Þ R= ´ç ÷
R A N LC ´ N è A ø
2 ´10-6 ´1000 2
= ;= S cm2 mole–1 0.4 ´ 1000
0.1 100 ÞR= = 500 Ohms
8 ´ 0.1
63. (b) Here, R = 31.6 ohm
68. (d) Charge on Mg and Ca ion is greater than that of Na
1 1
\ Conductance = = ohm -1 = 0.0316 ohm–1 and K, so Mg and Ca ions possess higher conductivity,
R 31.6 also solvation of metal ion decreases as we move down
Specific conductance = conductance × cell constant. the group, hence conductivity increases)
= 0.0316 ohm–1 × 0.367 cm–1 = 0.0116 ohm–1 cm–1 69. (d) The conductivity of electrolytic solution depends
Now, molar concentration = 0.5M (given) upon all of the given factors.
= 0.5 × 10–3 mole cm–3 70. (b) Given current (i) = 0.5 amp;
k 0.0116 Time (t) = 100 minutes × 60 = 6000 sec
\ =
Molar conductance = Equivalent weight of silver nitrate (E) = 108.
C 0.5 ´ 10 -3
According to Faraday's first law of electrolysis
= 23.2 S cm2 mol–1
64. (b) According to Kohlrausch’s law, molar conductivity Eit 108 ´ 0.5 ´ 6000
W= = = 3.3575 g.
of weak electrolyte acetic acid (CH3COOH) can be 96500 96500
calculated as follows: 71. (c) Electrorefining and electroplating are done by
L o
CH3COOH = L ( o
CH3COONa +L o
HCl ) - L NaCl
o electrolysis.
W A W B 1.6 Wt. of Ag
\ Value of L o NaCl should also be known for calculating 72. (d) = ; =
EA EB 8 108
value of LoCH 3COOH .
\ Wt . of Ag = 21.6 g
1 l 73. (b) Cu2+ + 2e– ® Cu (s)
65. (a) k = ´
R A 2 × 96500 C 63.6 g

1 l 63.6
1.3 = ´ 9650 C will deposit = ´ 9650 = 3.18 g
50 A 2 ´ 96500
ELECTROCHEMISTRY 613

74. (d) Ag + + e - ¾¾
® Ag 86. (d) Oxidation reaction at anode, upon electrolysis of water:
2H2O (l) ¾® O2 (g) + 4H+ (aq) + 4e–; E°cell = + 1.23 V
96500 coulombs deposit = 108 g of Ag
Thus, 1 mole of oxygen is liberated by 4 moles of electrons.
108 4 × 96500 coulombs electricity liberates = 22.4 L.O2 gas
\ 965 coulombs deposit = ´ 965 = 1.08 g Ag
96500 22.4
75. (a) According to Faraday law of electrolysis, amount of 9650 coulombs electricity liberates = ´ 9650
4 ´ 96500
electricity required to deposit 1 mole of metal = 96500 C = 1 F = 0.56 L. O2 gas
i.e., for deposition of 108g Ag, electricity required = 1 F
76. (c) 56
87. (b) Fe2+ + 2e- ¾¾
® Fe; EFe = = 28
77. (d) Charge (Coulombs) pass per second = 10–6 2
Number of electrons passed per second 1 Faraday liberates = 28 g of Fe
3 Faraday liberates = 3 × 28 = 84 g
10 -6 88. (d)
= -19
= 6.24 ´ 1012
1.602 ´ 10 89. (d) 1F ¾® 11.2 L Cl2 at STP (2Cl– ¾® Cl2 + 2e–)
78. (d) Charge on one mole of electrons = 96500 C. 9.65 ´ 5 ´ 60 ´ 60
79. (a) When platinum electrodes are dipped in dilute solution \ No. of Faradays = = 1.8
96500
of H2SO4 then H2 is evolved at cathode. \ Vol. of Cl2 = 1.8 × 11.2 L = 20.16
80. (a) When molten or fused NaCl is electrolysed, it yields 90. (b) W = Zit
metallic sodium and gaseous chlorine. Reactions involved where Z = Electrochemical equivalent
are as follows: 63
NaCl (s) ¾® Na+ (l) + Cl– (l) Eq. wt. of copper = = 31.5
2
Na+ (l) + e– ¾® Na (l) (at cathode) 31.5
Z=
Cl– (l) ¾® Cl (g) + e– (at anode) 96500
Cl (g) + Cl(g) ¾® Cl2 (g) 31.5
W = Zit = ´ 1.5 ´ 10 ´ 60 = 0.2938g
81. (b) Pt4+ + 4e–—® Pt 96500
4F electricity is required to deposit 1 mole of Pt. 91. (a) Electrolysis of these (i), (ii) and (iii) salt release chlorine
\ 0.80 F of electricity will deposit which is yellowish in colour while Br2 is reddish brown in
= 1/4 × 0.80 moles of Pt = 0.20 mol. colour
82. (b) During the electrolysis of sulphuric acid, the following 92. (b) Electronic conductance decreases with increase in
processes are possible at the anode : temperature whereas electrolytic conductance increases
with increase in temperature as no. of ions or charge
2H2O (l) ¾® O2 (g) + 4H+ (aq) + 4e–, E°cell = + 1.23 V
carriers increases with increase in temperature.
2SO42– (aq) ¾® S2 O82–(aq) + 2e–, E°cell = + 1.96 V
93. (a) Gold is an inert metal. Electrodes made up of inert
For dilute sulphuric acid, first reaction is preferred but at
metals does not participate in chemical reaction.
higher concentrations of H2SO 4, second reaction is
94. (a) 2H+ + 2e– ¾® H2
preferred.
83. (d) 1 mole of e– = 1F = 96500 C 2 22400
EH (Eq. wt) = =1 g = = 11200 mL (STP)
27g of Al is deposited by 3 × 96500 C 2 2
5120 g of Al will be deposited by
Total charge passed = 96500 ´112 = 965 coulom b
3 ´ 96500 ´ 5120 11200
= = 5.49 ´ 10 7 C
27 Q = It = 965 and t = 965s.
+4 +5
84. (a) VO32– ¾¾
® VO3– 965
4 This reaction involves transfer I= = 1amp.
965
of one electrons.
95. (c) Using Faraday’s second law of electrolysis,
Thus, for conversion of 1 mole, 1 F of electricity is required.
Weight of Cu deposited Equ. wt. of Cu
Charge = 0.2 × 1 Faraday = 0.2 × 96500 coulombs =
= 19300 = 1.93 × 104 coulombs Weight of Ag deposited Equ. wt. of Ag
85. (d) C 6 H 5 NO 2 + 6H + + 6e - ¾¾
® C 6 H 5 NH 2 + 2H 2 O wCu 63.5 1
Þ = ´
123 0.18 2 108
EC6 H5NO 2 (eq.wt) = = 20.5 63.5 ´ 18
6 Þ wCu = = 0.0529 g.
w ´ 96500 2 ´108 ´ 100
Number of coulombs required =
Eq. wt 96. (c) According to the definition, 1 F or 96500 C is the charge
12.3 ´ 96500 carried by 1 mol of electrons. When water is electrolysed:
= = 57900 C 2H2O ¾¾ ® 4H+ + O2 + 4e–
20.5
EBD_8350
614 CHEMISTRY

So, 4 Faraday of electricity liberate = 32 g of O2. Thus, 31.75 is the eq. mass of copper
Thus 1 Faraday of electricity liberate At. mass = Eq. mass × Valency
32 63.56 = 31.75 × x
= g of O2 = 8 g of O2 x = + 2.
4
+6
97. (b) ˆˆ† H + + OH -
H 2O ‡ˆˆ 2-
102. (b) Cr 2O7 + 6e ¾¾
-
® 2Cr3+
1 Total of 6 electrons are required to form 2 moles of Cr 3+
H + + e - ¾¾
® H2 therefore to form 1 mole of Cr 3+, 3F of charge is required.
2
\ 0.5 mole of H2 is liberated by 1 F = 96500 C 103. (c) 2NH 4 Cl + Zn ® 2NH 3 + ZnCl 2 + H 2 ­ .
96500 104. (b) In H2 – O2 fuel cell, the combustion of H2 occurs to
0.01 mole of H2 will be liberated by = ´ 0.01 = 1930 C create potential difference between the two electrodes.
0.5 105. (d) Primary cells are those cells, in which the reaction
Q=I×t occurs only once and after use over a period of time, it
Q 1930 C becomes dead and cannot be reused again. e.g., Leclanche
t= = = 19.3 ´ 10 4 sec cell and mercury cell.
I 10 ´ 10-3 A
106. (c) The electrolyte used in Leclanche cell is moist paste
- 1 - of NH4Cl and ZnCl2.
98. (b) At Anode, Cl ® Cl2 + e
2 107. (c)
35.5 ´ 2 108. (d) Prevention of corrosion by zinc coating is called
Equivalent wt. of chlorine ( ECl2 ) = = 35.5
2 galvanization.
ECl2 ´ I ´ t
35.5 ´ 2 ´ 30 ´ 60 109. (c) Mercury cell being primary in nature can be used
WCl2 = = = 1.32 g. only once.
96500 96500
110. (d) Fuel cells produce electricity with an efficiency of
99. (a) The charge carried by 1 mole of electrons is one about 70% compared to thermal plants whose efficiency
faraday. Thus for a reaction is about 40%.
M n + + ne - ¾¾
®M 111. (b) During charging, the lead storage battery behaves
nF = 1 mole of M like an electrolytic cell. So, at anode the reaction is
Al3+ + 3e - ¾¾
® Al PbSO 4 + 2H 2 O ¾¾ ® PbO 2 + 4H + + SO 42 - + 2e -
3F 1 mol
1F 1/3 mol 112. (b) H2 – O2 fuel cell supply power for pressure.
113. (a) Cathodic protection is best method to prevent iron
Cu 2+ + 2e- ¾¾
® Cu from rusting. In this method iron is made cathode by
2F 1 mol
1F 1/2 mol application of external current.
Saline water is highly conducting and hence accelerates
Na + + e - ¾¾
® Na
1F 1 mol the formation of rust.
The mole ratio of Al, Cu and Na deposited at the respective 114. (b) Magnesium provides cathodic protection and prevent
rusting or corrosion.
1 1
cathode is : :1 or 2 : 3 : 6. 115. (c)
3 2
116. (c) Reaction occurring at anode
W Q
100. (a) By Faraday's 1st Law of electrolysis, = ® 2Fe2 + + 4e -
2Fe ( s ) ¾¾
E 96500
(where Q = it = charge of ion ) Reaction occuring at cathode
We know that no. of gram equivalent O 2 (g) + 4H + (aq) + 4e - ¾¾
® 2H 2 O
W it 1 ´ 965 1 Discharge
= = = = ˆˆˆˆˆˆ†
ˆ
117. (d) Pb + PbO 2 + 2H 2SO4 ‡ˆˆˆˆˆˆ
ˆ 2PbSO 4 + 2H 2 O.
E 96500 96500 100 Recharge
(where i = 1 A, t = 16 × 60 + 5 = 965 sec.) Sulphuric acid is consumed on discharging.
Since, we know that 118. (d) This is because zinc has higher oxidation potential
1 than Ni, Cu and Sn. The process of coating of iron surface
No. of gram equivalent 100 with zinc is known as galvanization. Galvanized iron sheets
Normality = = = 0.01 N maintain their lustre due to the formation of protective
Volume (in litre) 1
layer of basic zinc carbonate.
101. (b) Quantity of charge passed = 0.5 × 30 × 60 = 900 119. (c) At cathode, reduction occurs according to following
coulomb reaction.
900 coulomb will deposit = 0.2964g of copper +4 +3
\ 96500 coulomb will deposit MnO 2 + NH +4 + e - ¾¾ ® MnO ( OH ) + NH 3
0.2964 120. (c) Corrosion of iron can be minimized by forming an
= ´ 96500 = 31.75 g of copper
900 impermeable barrier at its surface.
ELECTROCHEMISTRY 615

Exercise-2 ° q)CH COONa = l


L (e + l + = 91.0 ....(i)
3 CH3COO - Na

0.0591
1. (7) E°Cl - / AgCl/Ag = E °Ag/Ag + 1 log K sp ° q)HCl = l
L (e + l Cl¯ = 426.16 ....(ii)
1 H+

0.0591 ° q)NaCl = l
L (e + l Cl¯ = 126.45 ....(iii)
0.209 = 0.80 + log K sp Na +
1
Adding equations (i) and (ii) and substracting (iii),
-10
Ksp = 10 ; Let solubility of AgCl in 0.01 M solution is x l +l
CH3COO - Na + + l H+ + l Cl¯ – l Na + - l Cl¯
10–10 = x (x + 0.01)
= 91.0 + 426.16 – 126.45
x = 10–8 °
l - + l + = L (e q)CH3COOH
\ Moles of AgCl dissolved in 10L = 10–8 × 10 = 10–7 CH3COO H
2. (13) = 390.7 ohm–1 cm2 equiv–1
Electrolysis
KCl ¾¾¾¾¾® Cl2 (g) + H 2 (g) + OH -
9. (4)
¥
LM ( HA ) = L ¥M ( HCl) + L ¥M ( NaA ) - L M
¥
( NaCl)
at anode at cathode in solution = 425 + 100 – 125 = 400 S cm2mol–1
pH = 4, [H+] = 10–4 = aC
1F = 1 eq of H2(g) = 1 eq of Cl2(g) º 1 eq of OH– ions
2F = 2 eq of OH– L m 200
a= ¥ = = 0.5;
L m 400
2eq 2
[OH - ] = = = 10-1 N or M ( Ca ) × a = 10-4 ( 0.5) = 10-4 ;pK = 4
Volumein L 20L Ka =
(1 - a ) (1 - a ) a
\ pOH = –log (10–1) = 1
pH = 14 – 1 = 13 0.0591 æ 1 ö
10. (4) -0.12 - log ç ÷ = -0.24
1´ 1000 2 è Xø
3. (241.67) Molarity of BaCl2 = = 0.024 M
208 ´ 200
1 0.12 ´ 2
Also, Normality of BaCl2 = 0.024 × 2 = 0.048 N log = =4
(Q N = M × Valency factor) X 0.06
1000 0.0058 ´ 1000 X = 10–4
Now, L m = k ´ = = 241.67 S cm2 mol–1 11. (0.05) According to the Faraday’s law of electrolysis, nF
CM 0.024
of current is required for the deposition of 1 mole.
4. (12.5) Degree of dissociation is given by According to the reaction,
Lc
a= Ni(NO3 ) 2 ¾¾ ® Ni 2 + + 2NO 3-

2 F of current deposits = 1 mol
(i) Evaluation of L¥
CH3COOH \ 0.1 F of current deposits = = 0.05 mol

CH 3COOH = L
¥
+ L¥ + 12. (–384) DG° = –nFE°cell
CH 3COO - H
= 40.9 + 349.8 = 390.7 ohm–1 cm2 eq–1 = –2 ´ (96000) ´ 2 V = – 384000 J/mol
(ii) Evaluation of degree of dissociation = –384 kJ/mol
At C = 0.001 M,
13. ® Y; DG° = – 193 kJ mol–1
(4) X ¾¾
Lc 49.2
a= ¥ = = 0.125 i.e., 12.5%
L 390.7 M+ ¾¾
® M3+ + 2e– ; E° = – 0.25V
0.967 2 Hence DG° for oxidation will be
5. (3) ´2 = ´ nf
63.5 197 DG° = – nFE° = –2 × 96500 × (–0.25) = 48250 J = 48.25 kJ
nf = 3 48.25 kJ energy oxidises one mole M+
2 ( 0.34 - 0.15 )
6. (6) K c = 10 = 2.6 ´ 10 6 193
0.0591 \ 193 kJ energy oxidises mole M+ = 4 mole M+
48.25
7. (5) ˆˆ† MnO 2 + 2H 2O
4H + + MnO -4 ‡ˆˆ
14. (1.35) Maximum voltage E° = 0.49 + 0.86 = 1.35 V
æ 1.5 ´ 5 - 2 ´ 1.25 ö 15. (10.85) For this cell, reaction is; Zn + Fe2+ ® Zn2+ + Fe
DG° = -3 ´ F ´ ç ÷ø = – 5 F ; x = 5
è 3 0.0591 c 0.0591 c
E = E° - log 1 ; E° = E + log 1
8. (390.7) According to Kohlrausch’s law, n c2 n c2
EBD_8350
616 CHEMISTRY

0.0591 10 -2 L °m ( NaCl) = l °( + l °(
E° = 0.2905 + log -3 = 0.32 V. Na + ) Cl– )
2 10
...(iii)
0.0591 On adding (i) & (ii) and subtract (iii), we get :
E° = log K eq
2
L °m( NH + L °m( NaOH ) - L °m ( NaCl) = L °m( NH OH )
0.32 ´ 2 0.32 4Cl ) 4
log K eq = =
0.0591 0.0295 11. (b) Electrode potential for Mg electrode varies according
0.32 to the equation :
\ K eq = 10 0.0295 0.059 1
E = E° - log
Mg 2 + /Mg Mg 2 + /Mg
Comparing the value of 10x,
2 [Mg 2+ ]

0.059
0.32
= 10.847 » 10.85 E = E° + log [Mg 2 + ]
x= Mg 2 + /Mg Mg 2 + /Mg 2
0.0295
0.059
Exercise-3 E log [Mg 2 + ] + E ° 2 +
=
Mg 2 + /Mg 2 Mg /Mg
1. (c) Standard electrode potential of copper electrode can This equation represents equation of straight line.
be calculated by constructing a concentration cell 0.059
composed of two half cell reactions in which concentration Hence, intercept (c) = E °Mg 2 + /Mg and slope =
2
of species on left hand and right hand side are unity.
Thus, equation can be diagrammatically represented as:
Pt ( s) | H 2 ( g,1 bar ) | | H + ( aq., 1 M ) || Cu 2 + ( aq, 1 M ) | Cu
144444424444443 144424443
Oxidation half cell Reduction half cell

®
2. (b) Cell emf is the difference between the electrode E Mg2+/ Mg
potential of two electrodes when no current is drawn
through the cell.
3. (b) Among the given options chromium has highest log [Mg2+] ®
negative value of E° hence, it is the most strongest
reducing agent. 12. (c) Ecell is an intensive property as it is independent of
4. (c) Higher the positive value of standard reduction the mass of species but DrG of cell reaction is an extensive
potential of metal ion, higher will be its oxidising capacity. property because this depends upon mass of species.
Since, E° 13. (d) Inert electrode does not participate in redox reaction
- 2 + has highest positive value hence, it
MnO4 /Mn and acts only as source or sink for electrons. It provides
is the strongest oxidising agent. surface either for oxidation or for reduction reaction.
5. (b) On moving down in electrochemical series reducing 14. (c) If an external opposite potential is applied on the
power decreases as the value of electrode potential galvanic cell and increased slowly. It is observed that the
decreases. reaction continues to take place till the opposing voltage
E° reaches the value 1.1V. At this stage, no current flow
6. (d) MnO-4 /Mn 2 + has highest positive value. So, Mn 2+
through the cell. Any further increase in the external
is most stable ion in its reduced form. potential restarts the reaction but in the opposite direction.
7. (a) E ° 3+ has the lowest value of SRP. Hence, Cr 3+ Hence, now the cell will behave like an electrolytic cell.
Cr /Cr
15. (c) Greater the solvation of ions of an electrolyte, lesser
is the most stable oxidised species. will be the electrical conductivity of the solution.
+ 3e -
8. (c) Al3+ ( aq ) ¾¾¾® Al ( s ) 16. (a) When the lead storage battery is recharged, the
Hence, total 3F is required. reaction occurring on cell is reversed and PbSO4 (s) on
anode and cathode is converted into Pb and PbO 2
l respectively.
9. (d) Cell constant (G) =
A The electrode reactions are as follows :
where l = length of object and A = area of cross section Anode reaction:
Since, l and A remain constant for any particular object PbSO4 (s) +2e– ® Pb(s) + SO42– (aq) (Reduction)
hence, value of cell constant always remains constant. Cathode reaction:
PbSO4 (s) + 2H2O ® PbO2(s) + SO42–(aq) + 4H+ + 2e–
L ° m( NH = l° °
10. (b) 4Cl) ( NH 4+ ) + l ( Cl– ) ...(i)
Net reaction:
(Oxidation)

L °m ( NaOH ) = l °( + l (° 2PbSO4 (s) + 2H2O ® Pb(s) + PbO2(s) +


Na + ) OH + )
...(ii)
4H+ (aq) + 2SO42– (aq)
ELECTROCHEMISTRY 617

17. (d) In the electrolysis of aqueous NaCl, following 24. (b) HCl completely dissociates to give H+ and Cl– ions,
reactions are possible at anode. hence, act as very good electrolyte. While others are non-
1 electrolytes.
At anode; Cl - ( aq ) ¾¾
® Cl2 ( g ) + e - ; Ecell
°
= 1.36V 25. (d) A device that converts energy of combustion of fuels,
2 directly into electrical energy is known as fuel cell.
® O 2 ( g ) + 4H + ( aq ) + 4e - ;
At cathode; 2H 2 O ( l) ¾¾ 2+ –
26. (d) Cu + 2e ¾¾ ® Cu
°
E cell = 1.23 V 2F i.e. 2 × 96500 C deposit Cu = 1 mole = 63.5 g
27. (a) 2H+(aq) + 2e– ® H2(g)
The reaction at anode with lower value of E° is preferred
and therefore, water should get oxidised in preference to 0.0591 PH 2
Cl– (aq). However, on account of overpotential of oxygen, \ E = E0 – log
reaction (d) is preferred.
2 [ H + ]2
( +6) ( +7) PH2
18. (c) Mn O 24 - ¾® MnO -4 + e - 0 = 0 – 0.0295 log
0.1 mole (10-7 )2
PH2
Quantity of electricity required = 0.1F = 0.1 × 96500 = 9650 C =1
19. (d) wO = nO × 32
2 2
(10-7 )2
5600 PH 2 = 10–14 atm
wO = ´ 32 = 8g = 1 equivalent of O2
2 22400 28. (b) Galvanization is the process by which zinc is coated
= 1 equivalent of Ag = 108 g over corrosive (easily rusted) metals to prevent them from
20. (a) Given for 0.2 M solution corrosion.
R = 50 W 29. (c) Li+ being smallest, has maximum charge density.
k = 1.4 S m–1 = 1.4 × 10–2 S cm–1 \ Li+ is most heavily hydrated among all alkali metal
l l l l ions. Effective size of Li+ in aqueous solution is therefore,
Now, R = ρ = ´ Þ = R ´ k = 50 ´ 1.4 ´ 10 -2 largest. So, moves slowest under electric field.
a κ a a
For 0.5 M solution 30. (b) For cell,
R = 280 W Zn|ZnSO4(0.01 M) || CuSO4(1.0 M)|Cu
κ=? 2.303RT log [ Zn 2 + ]
Ecell = E °cell -
l
= 50 ´1.4 ´ 10-2 nF [Cu 2+ ]
a 2.303RT ( 0.01)
l l \ E1 = E °cell - ´ log
l 2´ F 1
Þ R=ρ = ´
a κ a When concentrations are changed for ZnSO4 and CuSO4,
1
Þ κ= ´ 50 ´ 1.4 ´ 10 -2 we can write
280 2.303 RT 1
1 E2 = E °cell - ´ log
= ´ 70 ´ 10-2 = 2.5 × 10–3 S cm–1 2F 0.01
280 \ E1 > E2
κ ´ 1000 2.5 ´ 10 -3 ´ 1000 31. (a) E° = 1.51V
Now, L m = = MnO 4– / Mn 2 +
M 0.5
= 5 S cm2 mol–1 = 5 × 10–4 S m2 mol–1 E° = 1.36V
Cl 2 / Cl –
21. (c) According to Debye Huckle Onsager equation, E° = 1.33V
Cr2 O72– / Cr 3+
λC = λ¥ - B C
E° = – 0.74V
Cr 3+ / Cr
22. (a) Mn 2 + + 2e- ® Mn; E 0 = -1.18 V; ... (i)
Since, Cr3+ having least reduction potential, so Cr is the
3+
Mn + e – ® Mn 2 + ; E 0
= -1.51 V ; ... (ii) best reducing agent.
Now multiplying equation (ii) by two and subtracting from 32. (c) Calculate E°cell corresponding to each compound
equation (i) undergoing disproportionation reaction. The reaction for
which E°cell comes out + ve is spontaneous.
3Mn 2+ ® Mn + + 2Mn3+ ;
HBrO ¾® Br2 E° = 1.595 V, SRP (cathode)
E 0 = EOx. + ERed. = – 1.18 + (– 1.51) = – 2.69 V HBrO ¾® BrO 3– E° = –1.5 V, SOP (anode)
(–ve value of EMF (i.e. DG = +ve) shows that the reaction
2HBrO ¾® Br2 + BrO 3–
is non-spontaneous)
23. (b) Ca is a reactive metal and extraction of reactive metal E°cell = SRP (cathode) – SRP (anode) = 1.595 – 1.5 = 0.095 V
is not possible by electrolysis of aq. salt solution. E°cell > 0 Þ DG° < 0 [spontaneous]
EBD_8350
618 CHEMISTRY

33. (c) B2 H 6 + 3O 2 ® B2 O3 + 3H 2O \ DG1° – DG2° = G3°


Moles of B2H6 = 27.6/27.6 = 1.0 Þ –FE° = –2F(0.34) + F(0.522)
moles of O2 required = 3x moles of B2H6 = 3 × 1.0 = 3 Þ E° = 0.68 – 0.522 = 0.158 V
2H2O ® O2 + 4H+ + 4e–
I ´t Exercise-4
= moles of O × 4
96500 2
1. (d) For spontaneous E° > 0,
100 ´ t 3 ´ 4 ´ 96500 11580
= 3 ´ 4; t = sec. = = 3.2 h Minimum possible spontaneous E° = 0.28 V
96500 100 3600
2. (b) For this cell, reaction is; Zn + Fe2+ ® Zn2+ + Fe
° =
2.303RT
34. (c) E cell log K 0.0591 c 0.0591 c
nF E = E° - log 1 ; E ° = E + log 1
n c2 n c2
° = 0.59 V ,
Given : E cell
n=1 0.0591 10 -2
E° = 0.2905 + log -3 = 0.32 V.
0.059 2 10
0.59 = log K
1 0.0591
E° = log K eq
0.59 2
= log K
0.059 0.32 ´ 2 0.32
log Kc = =
10 = log K 0.0591 0.0295
K = 1010 0.32
35. (a) DG = –nFE° = –2 × 96500 × 0.24 = – 46320 J/mol \ K c = 10 0.0295 .
= – 46.32 kJ/mol
36. (c) Half cell reaction: PbSO4 ¾® Pb4+ + 2e– 3. (b) In electrolysis of NaCl, when Pt electrode is taken,
According to the reaction: then H2 liberated at cathode, while with Hg cathode, it
PbSO4 ¾® Pb4+ + 2e– forms sodium amalgam because more voltage is required
We require 2F for the electrolysis of 1 mol or 303 g of PbSO4 to reduce H+ at Hg than at Pt.
\ Amount of PbSO4 electrolysed by 0.05F C2
4. (b) ˆˆ† Co 2+ ( aq ) ,Q =
Co2 + ( aq ) ‡ˆˆ
303 C1 C2 C1
= × .05 = 7.575 g » 7.6 g
2
37. (a) DG° = –nFE°cell C2
If < 1, D G < 0
= –2 × (96000) × 2 V = – 384000 J/mol = –384 kJ/mol C1
38. (d) 1 equivalent of any substance is deposited by 1 F of 5. (c) 2Mn 2+ ¾¾
® 2Mn 3+ + 2e - , DG1°
charge.
Mn 2+ + 2e - ® Mn, DG2°
20 g calcium contains,
Given mass 3Mn 2 + ( aq ) ® Mn (s ) + 2Mn 3+ ( aq )
Number of equivalents =
Equivalent mass – 2 × F × E°3 = – 2 ×F × [–1.51] – 2 × F × (–1.185)
Atomic mass 40 E3° = -2.695
(Equivalent mass of Ca = = = 20 )
Valency 2
0.0591
20 E3° = + log Kc ; Kc ; 6.3 ´10-92 .
= =1 2
20
So, 1 Faraday of charge is required to deposit 1 equivalent COONa COO–
Ù°eq | °
of Ca. 6. (a) = l eq | – +
39. (a) During the electrolysis of dil. sulphuric acid using Pt COOK COO
electrodes following reaction occurs.
At cathode : 4H + (aq) + 4e - ¾¾
® 2H 2 (g) l°eq ( Na + ) + l°eq ( K + )

® O2 (g) + 4H + (aq) + 4e -
At anode : 2H 2 O (l ) ¾¾ = (74 + 73 + 50) ohm–1 cm2 eq–1
= 197 cm2 ohm–1 eq–1
40. (b) Cu2+ + 2e– ¾® Cu, DG1° = – 2F(0.34) ...(i) Note : If equivalent conductance of cation and anion are
Cu+ + e– ¾® Cu, DG2° = – F(0.522) ...(ii) respectively l°+eq and l°–eq at infinite dilution, then that
Subtract (ii) from (i) of an electrolyte can be expressed as :
Cu2+ + e– ¾® Cu+, DG3° = –F(E0) l°eq = l°+eq + l°–eq
ELECTROCHEMISTRY 619

7. (d) Al(s) + 3Ag + (aq) ¾¾


® Al3+ (aq) + 3Ag(s) 0.059
0.59 = log K c \ K = 1010
1 c
[Al3+ ] 0.1
Q= = = 102 10. (c) A cation having highest reduction potential
+ 3 3
[Ag ] 0.1 will be reduced first and so on. However, Mg 2+
log10 Q = log10 102 = 2 log10 10 = 2 in aqueous solution will not be reduced
æ E° ö.
8. (a) ®Fe2+ + 2e-;
Fe ¾¾ -DG1° = 2E1° F ...(i) ç Mg 2+ /Mg
<E
H 2O/ H 2 + OH - ÷
1
è 2 ø
° °
Fe2+ ¾¾
® Fe3+ + e- ; -DG2 = E2 F ...(ii) Instead water would be reduced in preference.

®Fe3+ + 3e-;
Fe ¾¾ -DG3° = 3E3° F ...(iii) æ dE ö
DS = nF ç -4
11. (c) ÷ = 2 ´ 96500 ´ 1.5 ´ 10
Subtracting Eqs. (i) from (iii), è dT ø p

-DG3° = 3E3° F = 28.95 J/K mol


®Fe3+ + 3e-;
Fe ¾¾
12. (a) It is a concentration cell, therefore, E°cell = 0.
®Fe2+ + 2e-;
Fe ¾¾ -DG1° = 2E1° F 1
pH of weak acid = (pKa – log c)
2
Fe 2+ ¾¾
® Fe3+ + e - ; -DG3° + DG1° = 3E3° F - 2 E1° F
...(iv)
1
Comparing Eqs. (ii) and (iv) = (4.74 – log 10– 3) = 3.87
2
-DG2° = -DG3° + DG1° = 3E3° F - 2E1° F pH of NaOH = 14 – 3 = 11
\ E = – 0.059 (pHc – pHa)
+ E2° F = 3E3° F - 2 E1° F
= – 0.059 (11 – 3.87) = – 0.42 V
2 E1° + E2° k k
\ E3° = or 3E3° = 2 E1° + E2° 13. (c) Lm = ´1000; ( x + y ) = ´1000
3 M M
°
9. (c) Given, In 3+ + 2e- ¾¾
® In + ; E1 = -0.42 V ...(i) 1000 ´ k
Þ M =
° ( x + y)
In 2+ + e- ¾¾
® In + ; E2 = -0.40 V ...(ii) k ´ 1000 ´ 188
By subtracting Eq. (ii) from Eq. (i) a third half-cell reaction Solubility (in gL–1) = x+ y
can be obtained as: 14. (d) The source of SO42– ions is from two different species.
é where E ° ´ 1 ´ F = E ° ´ 2 ´ F - E ° ´ 1 ´ F ù Thus,
ê 3 1 2 ú
ê or E ° = 2 ´ ( -0.42) - 1 ´ ( -0.40) = -0.44 V ú L°eq [K2SO4.Al2(SO4)3.24H2O]
ë 3 û
°
In 3+ + e - ¾¾
® In 2+ ; E3 = -0.44 V = l°eq (K+) + l°eq (Al3+) + [ l°eq (SO42–)] × 2
For the reactions: Cu2+ + e- ¾¾
®Cu + ; E4° = 0.15 = 73.5 + 149 + 2 × 85.8
°
= 394.1
In 2+ ¾¾
® In 3+ + e - ; E3 = +0.44 15. (b) Upto equivalence point, highly mobile H+ get replaced
The net redox change: Cu 2 + + In 2 + ¾¾ ® Cu + + In 3+ ; by much less mobile K+ ions. Hence, resistance of solu-
E°cell = E°4+ E°3 = 0.15 + 0.44 = 0.59 V tion increases. After equivalence point, resistance de-
creases due to unreacted K+ and highly mobile OH– ions.
° 0.059
Also Ecell = log K c
1
EBD_8350
620 CHEMISTRY

18 Chemical Kinetics
Exercise - 1 14. (c) Rate of reaction does not remain constant during the
complete reaction because rate depends upon the
1. (b) 2. (c) concentration of reactants which decreases with time.
3. (a) If we write rate of reaction in terms of concentration 15. (d) Order of reaction may be zero, whole number or fractional.
of NH3 and H2,then 16. (b) The value of rate constant can be increased only by
increasing the temperature.
1 d[NH3 ] 1 d[H 2 ]
Rate of reaction = =- 17. (a) For a zero order reaction.
2 dt 3 dt rate =k[A]º i.e. rate = k
d[NH3 ] 2 d[H 2 ] hence unit of k = M.sec–1
So, =- For a first order reaction.
dt 3 dt
4. (d) 5. (c) 6. (a) 7. (b) 8. (c) 9. (d) rate = k [A]
10. (a) Average rate depends upon the chan ge in k = M.sec–1/M = sec–1
concentration of reactants or products and the time taken 18. (d) The reaction involving two different reactant can
never be unimolecular.
for that change to occur. Average rate cannot be used to
19. (b) For reaction 3A ¾ ¾® B + C
predict the rate of a reaction at a particular instant as it
If it is zero order reaction r = k [A]0, i.e the rate remains same
would be constant for the time interval for which it is
at any concentration of 'A'. i.e independent upon
calculated. So, to express the rate at a particular moment of
concentration of A.
time, we determine the instantaneous rate. It is obtained
20. (c) k = (mol L–1)1–n time–1. For given reaction n = 2.
when we consider the average rate at the smallest time
\ k = mol–1 L sec–1
interval say dt (i.e. when it approaches zero).
21. (b) Velocity constant ‘k’ is characteristic constant of a
dMnO -4 reaction and depends only on temperature and catalyst.
11. (b) Given - = 4.56 × 10–3 Ms–1
dt 22. (a) 2A + B ¾ ¾® A2B
1 dMnO -4 1 dI 2 r1 = k[A]2 [B] ...(i)
- =
2 dt 5 dt é Bù
When, [A] = [2A], [B] = ê ú
ë2û
5 dMnO-4 dI2
\ - = 2 éBù 2 [B]
2 dt dt r2 = k [ 2A ] ê ú = k 4[A]
ë û
2 2
On substituting the given value
dI2 4.56 ´ 10-3 ´ 5 = k 2[A]2[B] = 2r1 (
Q r1 = k[A]2[B] )
\ = = 1.14 × 10–2 M/s \ Rate of reaction is increased two times.
dt 2
23. (c) Let us consider a reaction,
12. (b) Rate of disappearance of reactants = Rate of
appearance of products x X + y Y ¾¾ ® aA+bB
rate = [X]x [Y]y
1 d(N 2 O5 ) 1 d(NO 2 ) d(O 2 ) It is given that order of reaction w.r.t. component Y is zero.
- = =
2 dt 4 dt dt Hence, rate = [X]x
1 1 i.e., rate becomes independent of the concentration of Y.
k (N 2 O 5 ) = k ¢(N 2 O 5 ) = k ¢¢(N 2 O 5 ) 24. (a) Rate = k[A]°
2 4
Unit of k = mol L–1 sec–1
k k¢
= = k ¢¢ 25. (a) Given r = k [A]3/2 [B]–1/2
2 4
3 -1 2
k Order = 3/2 – 1/2 = = =1
k¢ = 2k, k¢¢ = 2 2
2
26. (b) The order w.r.t. I2 is zero because the rate is not
13. (d) As the slowest step is the rate determining step, the dependent on the concentration of I2.
mechanism B will be more consistent with the given 27. (a) For given reaction x and y may or may not be equal to
information. It involves one molecule of H2 and one p and q respectively.
molecule of ICl thus, it can be expressed as : 28. (b) Given dx/dt = 2.400 × 10–5 mol litre–1 sec–1
r = k [H2][ICl] k = 3.10 × 10–5 sec–1 thus, it is a first order reaction.
Which shows that the reaction is first order w.r.t. both H2 For first order reaction
& ICl.
2N 2 O 5 ¾¾
® 2NO 2 + O 2
CHEMICAL KINETICS 621

dx x
= k [N 2 O 5 ] 1.25 ´ 10-3 é 0.005 ù

dt
5.0 ´ 10-3 ë 0.010 úû
or 2.4 × 10–5 = 3.0 × 10–5 [N2O5]
or 0.25 = (0.5)x or (0.5)2 = (0.5)x
2.4 ´ 10 -5 \ x=2
or [N2O5] = = 0.8 mol. litre–1 The order with respect to ‘A’ is 2.
3.0 ´ 10 -5
From the 1st and 3rd sets of data dividing eq. (1) by eq. (3)
29. (c) Rate1 = k [A]n [B]m; Rate2 = k [2A]n [½B]m
y
5.0 ´10- 3 é 0.005 ù
Rate 2 k [2A]n [½B]m =ê ú
\ = = [2]n [½]m = 2n.2–m = 2n–m 1.0 ´10 -2
ë 0.010 û
Rate1 k[A]n [B]m
or (0.5)1 = (0.5)y Þ y = 1
1
30. (c) N 2 O ¾¾® N 2 + O2 The order with respect to ‘B’ is 1
2 So, the order with respect to the reactants A, B and C are
dx 1
µ [N 2 O] 2, 1 and 0.
dt Fast Slow
37. (b) O 3 ¾¾
¾® O 2 + O; O + O 3 ¾¾¾® 2O 2
i.e. order of reaction = 1
31. (d) If rate = k[A]x [B]y [C]z [O 2 ][O]
k= (I) Rate = k' [O3][O] put [O] from (I)
From first two given data [O 3 ]
8.08 × 10–3 = k [0.2]x [0.1]y [0.02]z .... (i) k '[O3 ]K [O3 ]
–3 x y
2.01 × 10 = k [0.1] [0.2] [0.02] z .... (ii) r= = k[O 3 ]2 [O 2 ]-1
[O 2 ]
Divide (i) by (ii) we get, 4 = 2x (1/2)y Note : intermediates are never represented in rate law
Similarly, from second and third data equation.
(9)y (9)z = 3 38. (d) Overall order = sum of orders w.r.t each reactant.
2y + 2z = 1 . Let the order be x and y for G and H respectively
From first and fourth data 4z = 8 = 23 [G]mole [H]mole rate(mole
2z = 3. So z = 3/2, y = – 1, x = 1 Exp.No.
32. (c) As we know that, units of rate constant. litre-1 litre -1 litre- time -1 )
= (unit of conc.)1–n (unit of time)–1 1 a b r
= (mol L–1)1–n (sec)–1 2 2a 2b 8r
On comparing these units with the given units of rate
3 2a b 2r
constant, we get
(mol L–1)1–n (sec)–1 = L mol–1 sec–1 Q For (1) and (3), the rate is doubled when conc. of G is
Þ Ln–1 mol1–n sec–1 = L mol–1 sec–1
doubled keeping that of H constant i.e., rate µ [G]
On comparing the powers, we get
n–1=1Þn=2 \ x=1
So, reaction is of second order. From (2) and (3), y = 2
33. (d) In case of (II) and (III), keeping concentration of [A] \ Overall order is 3.
constant, when the concentration of [B] is doubled, the
39. (c) Rate law : - d [ A] = k [ A]x [ B ] y
rate quadruples. Hence, it is second order with respect to
dt
B. In case of I & IV, keeping the concentration of [B] Doubling [A], rate is doubled. Hence 2x = 2, x =1
constant, when the concentration of [A] is increased four
d [ A]
times, rate also increases four times. Hence, the order with Similarly y = 1; - = k [ A][ B ]
respect to A is one. Hence, dt
rate 2.0 ´ 10 -4
Rate = k [A] [B]2 k= = =1
[ A][ B ] 1 ´ 10 -2 ´ 2 ´ 10 -2
34. (b) Slow reaction is the rate determining step.
35. (a) Rate of reaction involving gaseous molecules depends (rate)2 k (2[ A])(2[b])
= =4
on pressure. (rate)1 k[ A][ B ]
36. (d) From 1st and 2nd sets of data - no change in rate is 40. (d) The rate of reaction is never negative. Minus sign
observed with the change in concentration of ‘C’. So the used in expressing the rate only shows that the
order with respect to ‘C’ is zero. concentration of the reactant is decreasing.
From 1st and 4th sets of data 41. (b) For zero order reaction,
Dividing eq. (4) by eq. (1)
-d [ R ]
rate, r = k[R]° Þ rate = = k ´1
dt
EBD_8350
622 CHEMISTRY

Þ d[R] = k dt 64
\ Amount left = =8 g
Þ [R] = – kt + R0 (2)3
where R0 is the concentration of reactant at time t = 0.
Thus, [R] decreases with time t. Slope = – k / 2.303
42. (a) Plots of conc. [A] Vs time, t
51. (b)
Zero First log C
order order
1 1
log [A]

[A] Second Third


[ A] [ A]2
order order t
t t t t
52. (b) A ® B For a first order reaction
1 Given a = 0.8 mol, (a – x) = 0.8 – 0.6 = 0.2
43. (d) t1/2 µ
a2 2.303 0.8
k= log or k = 2.303 log 4
1 1 0.2
We know that t1/2 µ n -1 again a = 0.9, a – x = 0.9 – 0.675 = 0.225
a
i.e. n = 3 2.303 0.9
Thus, reaction is of 3rd order. k= log
t 0.225
44. (a) For a zero order reaction
a 2.303
t1/ 2 = 2.303log 4 = log 4
2k t
45. (a) t1/2 = 4s T = 16s Hence, t = 1 hour
53. (b) For a first order reaction, A ® products
T 16
n= = =4 (\ T = n × t½) r
t1/ 2 4 r = k[A] or k =
[A]
n 4
æ1ö æ1ö 0.12 1.5 ´ 10-2
[ A] = [ A0] ç ÷ = 0.12 ´ ç ÷ = = 0.0075 M Þ k= = 3 × 10–2
2
è ø 2
è ø 16 0.5
Where [A0] = initial concentration and [A] 0.693 0.693
= concentration left after time t. Further, t1/ 2 = = = 23.1
k 3 ´ 10-2
0.693 54. (b) Given: 75% reaction gets completed in 32 min
46. (c) For first order reaction, k =
t1/ 2 2.303 a 2.303 100
where k = rate constant Thus, k = log = log
t (a - x ) 32 (100 - 75)
t1/2 = half life period = 480 sec.
2.303
0.693 = log 4 = 0.0433 min–1
\k = = 1.44 × 10–3 sec–1 32
480 Now we can use this value of k to get the value of time
n required for 50% completion of reaction
æ 1ö
47. (c) [A] = [A]0 ç ÷ 2.303 a 2.303 100
è 2ø t= log = log
k (a - x ) 0.0433 50
where [A]0 = initial concentration
2.303
n = log 2 = 16 min
1 æ 1ö 0.0433
= 1ç ÷ ; n = 3
8 è 2ø 2.303 1 2.303 4
55. (c) t1/ 4 = log = log
0.693 k 3/ 4 k 3
t1/ 2 = = 100 sec
6.9 ´ 10-3 =
2.303
(log 4 - log 3) =
2.303
(2 log 2 - log 3)
\ T = n ´ t1/ 2 = 3 × 100 = 300 sec. k k
48. (d) Unit of k for Ist order reaction is (Time)–1 2.303 0.29
= (2 ´ 0.301 - 0.4771) =
49. (a) As doubling the initial conc. doubles the rate of k k
reaction, order =1 56. (b) Since doubling the concentration of B does not
50. (d) t1/2 = 5 years, T = 15 years hence total number of half change half life, the reaction is of 1st order w.r.t. B.
15 Order of reaction with respect to A = 1 because rate of
life periods = =3. reaction doubles when concentration of A is doubled
5 keeping concentration of B constant.
CHEMICAL KINETICS 623

\ Order of reaction = 1 + 1 = 2 and units of second order [R]


64. (a) For a first order reaction log 0 varies linearly with
reaction are L mol–1 sec–1. [R]
2.303 a time as shown below.
57. (c) t = log
k a-x
2.303 2.303
or t = log a - log(a - x) slop = k/2.303

[R]0
k k

[R]
-2.303

log
slope =
k 0
Time
n -1 -2 ö n -1
(t1/ 2 )1 æ a2 ö 120 æ 4 ´ 10
58. (c) = ; =ç ÷ ;n = 2 65. (a) If a reactant is present in excess, order with respect to
(t1/ 2 )2 çè a1 ÷ø 240 è 8 ´ 10-2 ø that reactant is zero.
59. (d) The given reaction is : 2X + Y —® Z 66. (a) Rewriting the given data for the reaction
d[X] d[Z] H+
– = CH3COCH3 (aq) + Br2 (aq) ¾¾¾
®
2dt dt
CH3COCH 2 Br(aq) + H + (aq) + Br - (aq)
\ Rate of formation of Z is half of the rate of
disappearance of X. Actually this reaction is autocatalyzed and involves
complex calculation for concentration terms.
-d[X] - d[Y]
= We can look at the above results in a simple way to find
2dt dt the dependence of reaction rate (i.e., rate of disappearance
Rate of disappearrance of X is not equal to rate of
of Br2).
disappearance of Y.
From data (1) and (2) in which concentration of CH3COCH3
60. (d) For a zero order reaction, and H+ remain unchanged and only the concentration of
t1/2 µ a0 (initial concentration or initial pressure) Br2 is doubled, there is no change in rate of reaction. It
(t1/2)1 µ P1 means the rate of reaction is independent of concentration
(t1/2)2 µ P2 of Br2.
Again from (2) and (3) in which (CH3CO CH3) and (Br2)
(t1/2 )2 P2 ( t1/2 )2 16
= , = remain constant but H+ increases from 0.05 M to 0.10 i.e.
(t1/2 )1 P1 45 4 doubled, the rate of reaction changes from 5.7×10–5 to 1.2 ×
16 10–4 (or 12 × 10–5), thus it also becomes almost doubled. It
(t1/2)2 = ´ 45 = 180 min shows that rate of reaction is directly proportional to [H+].
4
61. (a) Given initial concentration (a) = 2.00 M; Time taken From (3) and (4), the rate should have doubled due to
(t) = 200 min and final concentration (a – x) = 0.15 M. increase in conc of [H+] from 0.10 M to 0.20 M but the rate
For a first order reaction rate constant, has changed from 1.2× 10–4 to 3.1×10–4. This is due to
change in concentration of CH3COCH3 from 0.30 M to 0.40
2.303 a 2.303 2.00
k= log = log M. Thus the rate is directly proportional to [CH3 COCH3].
t a-x 200 0.15 We now get
2.303 rate = k [CH3COCH3]1[Br2]0[H+]1
= ´ (0.301 + 0.824) = 1.29 ´ 10 -2 min -1 .
200 = k [CH3COCH3][H+].
Further Trick : only option (a) can be correct if rate does not
0.693 0.693 depend on [Br2].
(t1/ 2 ) = = = 53.72 min . 67. (b) In Haber’s process, ammonia is manufactured from
k 1.29 ´ 10-2
N2 and H2 using iron as catalyst with molybdenum as
62. (b) C2H4O (g) ¾® CH4 (g) + CO (g)
promoter at high temperature and pressure
80 – x x x t = 20 min.
Fe O (catalyst)
Let x torr of C2H4O decompose after 20 min. Then, N 2 + 3H 2 ¾¾¾¾¾¾¾¾¾
2 3 ® 2NH3
Mo (catalytic promoter)
80 – x + 2x =120 ; x = 40 torr = 50% of initial pressure.
Hence t 1/2 = 20 min. For 75% reaction, fraction left 68. (b) For a reversible reaction,
DH = Ea (forward) – Ea (backward)
2
25 1 æ 1 ö DH = 150 – 260 = –110 kJ mol–1
= = =ç ÷
100 4 è 2 ø 69. (b) In equation k = Ae - Ea / RT ; A = Frequency factor
No. of half lives = 2. Time needed for 75% reaction., k = velocity constant, R = gas constant and Ea = energy of
2 × 20 = 40 min activation
63. (a) If on increasing concentration, rate of reaction DH = Ea( f ) - Ea (b)
70. (b)
decreases then the order of reaction will be negative.
EBD_8350
624 CHEMISTRY

Thus energy of activation for reverse reaction depend 80. (c) The rate constant doubles for 10º C rise in
upon whether reaction is exothermic or endothermic. temperature.
If reaction is exothermic, DH = - ve , Ea(b) > Ea( f ) For 20º C rise, the rate constant will be 4 times
\ k1 = 4k2 or k2 = 0.25 k1
If reaction is endothermic, DH = + ve Ea(b) < Ea( f ) 81. (c)
82. (a) Threshold Energy = Energy of activation + Internal energy
71. (c)
83. (c) For the exothermic reaction the energy of products is
Ea 1 always less than the reactants. If E a is the energy of
72. (d) k = Ae- Ea / RT log k = log A – .
2.303 R T activation for the forward reaction, the energy of activation
- Ea for backward reaction is Ea + DH
Equation of straight line slope = .
2.303 R 84. (b) According to Arrhenius equation, k = Ae–Ea/RT
\ when Ea = 0, k = A
r æ T2 -T1 ö æ 100 -10 ö
100°C çè
10 ø
÷ çè
10 ø
÷ Also ln k us 1/T is a straight line with slope = –Ea/R.
73. (b) =2 =2 = 29 = 512 (where 2 is
r \ Statements (ii) and (v) are correct.
10°C
85. (d) The relevant expressions are as follows.
temperature coefficient of reaction)
DH 1
(i) log Kp = - +I
k2 Ea æ1 1 ö R T
74. (b) log = ç - ÷ (ii) log [X] = log [X]0 + kt
k1 2.303 R è T1 T2 ø
(iii) P/T = constant (V constant)
Ea é 1 1 ù (iv) PV = constant (T constant)
log 2 =
2.303 ´ 8.314 ê 293 - 308 ú 86. (c)
ë û
Ea 15 Ea
0.3 = × 87. (d) log k = log A - …(1)
2.303 ´ 8.314 293 ´ 308 2.303RT
1
0.3 ´ 2.303 ´ 8.314 ´ 293 ´ 308 Also given log k = 6.0 - (2000) …(2)
Ea = . T
15 On comparing equations, (1) and (2)
= 34673 J mole–1 = 34.7 J mole–1 log A = 6.0 Þ A = 106 s–1
75. (b) DH = E a f - Eab = 0 Ea
and = 2000 ;
76. (d) The activation energy depends upon the nature of 2.303 R
chemical bonds undergoing rupture during chemical Þ Ea = 2000 × 2.303 × 8.314 = 38.29 kJ mol–1
reaction and is independent of enthalpies of reactants and 1
products. 88. (b) A graph plotted between log k vs for calculating
T
– E /RT activation energy is shown as
77. (b) k = Ae a
lnk = ln A – Ea/RT
For ln k vs 1/T
log k
ln A = intercept
1/T
– Ea/R = slope = –2 × 104 K
from Arrhenius equation
\ Ea = 8.3 × 2 × 104 J mol–1
Ea
= 16.6 × 104 J mol–1 or 166 kJ mol–1 log k = log A -
2.303 RT
78. (a) As per Arrhenius equation (k = Ae - Ea / RT ) , the rate 89. (b) For 10°C rise of temperature, the rate is almost doubled.
- Ea / RT
constant increases exponentially with temperature. 90. (c) k1 = A1e 1 .........(i)
k 2 Ea é T2 - T1 ù -E / RT
79. (a) 2.303 log = k2 = A2e a2 ........(ii)
k1 R êë T1T2 úû
On dividing eqn (i) from eqn. (ii)
k2 9.0 ´103 é 308 - 298 ù
log = k1 A1
k1 2.303 ´ 2 êë 308 ´ 298 úû =
k2 A2
( Ea2 - Ea1 ) / RT ........(iii)
k2
= 1.63; k 2 = 1.63k1;
k1 Given Ea2 = 2Ea1 . Let A1/A2 = A
1.63k1 - k1
Increase in k1 = k2 - k1 ´ 100 = ´ 100 = 63.0% On substituting this value in eqn. (iii)
k1 k1 Ea / RT
k1 = k 2 A ´ e 1
CHEMICAL KINETICS 625

91. (b) Reactants may form upon releasing of energy from 103. (b) When the temperature is increased, energy in form of
activated complex. heat is supplied which increases the kinetic energy of the
92. (b) (rate)1 = (rate)2 and rate µ k reacting molecules. This will increase the number of
collisions and ultimately the rate of reaction will be
k2 (rate)2
Thus, k = (rate) = 1 enhanced.
1 1 104. (b) For a 10 K rise in temperature, collision frequency
Ea increases merely by 1 to 2% but the number of effective
ln k = ln A -
RT collisions increases by 100 to 200%.
Ea 2 – Ea1 105. (d)
k A
ln 2 = ln 2 +
k1 A1 RΤ Exercise - 2
A2 2R T A
ln 1 = ln – ln 2 = 2 1 d[X] d[Y] d[Z]
A1 RΤ A1 1. (0.1) Rate of reaction = - =– =+ ... (i)
2 dt dt dt
93. (b) For first reaction \ Rate of appearance of
k¢ E1 æ 1 1 ö
log 1 = ç - ÷ ... (i) d[Z]
k1 2.303 R è T1 T2 ø z= = 0.05 mol L–1 per min.
dt
For second reaction d[X]
\ Rate of disappearance of x = –
k¢ E2 æ 1 1 ö From equation (i), we get
dt
log 2 = ç - ÷ ... (ii)
k 2 2.303 R è T1 T2 ø
1 d[X] d[Z] d[X] d[Z]
Dividing eq. (i) by eq. (ii) : - = \ - = 2´
2 dt dt dt dt
( ) = E1
log k1¢ / k1
=2×
d[Z]
= 2 × 0.05 = 0.1 mol L–1 min–1
log ( k2¢ / k2 ) E2 dt

It is given that E1 > E2, thus 2. (1.053) NO(g) + O3(g) ¾¾


® NO2(g) + O2(g)
æ k¢ ö æ k¢ ö Changeof pressure DP
log ç 1 ÷ > log ç 2 ÷ Average rate = =
ç k1 ÷ ç k2 ÷ Changeof time Dt
è ø è ø
700 – 500 200 200
k1¢ k2¢ = = = atm s –1
Þ > 250 250 760 ´ 250
k1 k2
= 1.053 × 10–3 atm s–1
94. (b) T2 = T (say), T1 = 25°C = 298K,
0.69
Ea = 104.4 kJ mol–1 = 104.4 × 103 J mol–1 3. (2) t1/2 = , t 3/14 = t 75%
k
k1= 3 × 10 –4, k2 = ?,
2.303 a
t 3/4 = log
k2 Ea é 1 1 ù k æ 3a ö
log = - çè a - ÷ø
k1 2.303 R êë T1 T2 úû 4
k2 104.4 ´ 103 J mol-1 é 1 1ù 2.303
log 4 =
2.303
´ 2 ´ 0.3010 =
0.69 ´ 2
log = - ú =
-1 -1 ê k k k
3 ´10 -4 2.303 ´ (8.314 J K mol ) ë 298 T û
1 t 3/4 0.69 ´ 2 k
As T® ¥, ®0 = ´ Þ t 3/4 = 2t1/2
T t1/2 k 0.69
k2 104.4 ´ 103 J mol -1
t1/ 2 µ ( a )
1- n
\ log = 4. (2)
3 ´ 10-4 2.303 ´ 8.314 ´ 298
425 µ ( 350 )
1- n
For I Þ .... (i)
k2 k2
log = 18.297, = 1.98 ´ 1018
275 µ ( 540 )
1- n
3 ´ 10-4 3 ´ 10-4 For II Þ .... (ii)
941 µ (158)
1- n
k2 = (1.98 ´ 1018 ) ´ (3 ´ 10-4 ) = 6 ´ 1014 s -1 For III Þ .... (iii)
Trick : k = Ae–Ea/RT, when T ® ¥ then k = A 1- n
425 æ 350 ö
= ç
è 540 ÷ø
95. (b) 96. (c) 97. (a) 98. (a) 99. (c) By Eqns. (i) and (ii),
275
100. (c) Applicable to bimolecular reactions. 425 350
101. (b) 102. (a) Taking log Þ log = (1 - n ) log Þn=2
275 540
EBD_8350
626 CHEMISTRY

Ea (F.R.) = DH + E¢a (B.R.)


k2 Ea é T2 - T1 ù
5. (28.81) log k = R ´ 2.303 ê T T ú 150 = – 135 + E¢a (B.R.)
1 ë 1 2 û
E¢a (B.R.) = 285 kJ/mol
k2
Given = 3 ; R = 8.314 JK–1 mol–1; 11. (0.125) Given, t1/2 = 15 days
k1
T1 = 20 + 273 = 293 K and T2 = 50 + 273 = 323 K No = 2 g
Substituting the given values in the Arrhenius equation, From Ist Jan. 2009 to Ist March 2009
Ea é 323 - 293 ù no. of day = 60
log 3 = \ no. of half lives = 4
8.314 ´ 2.303 êë 323 ´ 293 úû
4
N æ 1ö N 1
2.303 ´ 8.314 ´ 323 ´ 293 ´ 0.477 \ =ç ÷ or =
Ea = No è 2 ø 2 16
30
= 28811.85 J mol–1 = 28.81 kJ mol–1 N = 0.125 g
6. (0.158) Decomposition of sucrose is first order reaction, n -1 n -1
(t1/2 )1 æ a2 ö 10 æ 300 ö
12. (0) = ; =ç ÷
2.303 [A ] (t1/2 )2 çè a1 ÷ø 15 è 200 ø
k = log 0 ....(i)
t [A]
1 n -1 1- n
For time, t1/2 = 3×0 hr æ 2ö æ 3ö æ 2ö
or, çè ÷ø = çè ÷ø =ç ÷
è 3ø
; 1 - n = 1, n = 0
3 2
0.693 0.693
k = = = 2×231 hr–1 13. (180) For a zero order reaction,
t1/2 3 hr
t1/2 µ a0 (initial concentration or initial pressure)
Substituting k = 0×231 hr–1 in (i) we get,
(t1/2)1 µ P1
2 × 303 [A ]
2×231 = log 0 (t1/2)2 µ P2
8 [ A]
(t1/2 )2 P2 (t1/2 )2 16
[ A0] = , =
log = 0×8024 (t1/2 )1 P1 45 4
[ A] 16
(t1/2)2 = ´ 45 = 180 min
[ A0 ] 4
= Antilog 0×8024 = 6×345 14. (2) A ¾® B
[ A]
Initial concentration Rate of reaction
[ A] 1
\ Fraction of sample remains = = = 0 ×158 2 × 10–3 M 2.40 × 10–4 Ms–1
[ A0 ] 6 × 345 1 × 10–3 M 0.60 × 10–4 Ms–1
7. (25) k = A.e - Ea /(RT) rate of reaction
\ Effective overall energy of activation r = k[A]x
where x = order of reaction
1 1
E a = Ea (2) - E a (3) + E a (1) - E a (5) hence
2 2 2.40 × 10–4 = k [2 × 10–3]x .....(i)
1 1 0.60 × 10–4 = k [1 × 10–3]x ....(ii)
= 60 - 50 + ´ 40 - ´ 10 = 25 kJ/mol
2 2 On dividing eqn.(i) from eqn. (ii) we get
t1/2 (1) n -1
880 é 364 ù
n -1 4 = (2)x
é p (2) ù
8. (2) =ê , =ê , \ x=2
t1/2 (2) ú
ë p (1) û 410 ë 170 úû
i.e. order of reaction = 2
[2.14]1 = [2.14]n -1 ; n – 1 = 1 ; n = 2 dMnO -4
15. (0.0114) = 4.56 × 10–3 Ms–1 (Given)
9. (8) The increase in pressure shows the increase in dt
conc. of Z. From the reaction given,
120 - 100 1 dMnO 4 – 4.56 ´ 10 -3
Rate of appearance of Z = = 4 mm min–1 - = Ms-1
5 2 dt 2
Rate of disappearance of X2 = 2 × rate of appearance of 1 dMnO-4 1 dI 2
Z = 2 × 4 mm min–1 = 8 mm min–1 - =
2 dt 5 dt
10. (285) X ¾® Y ; DH = –135 kJ/mol,
Ea = 150 kJ/mol 5 dMnO-4 dI2
\ - =
For an exothermic reaction 2 dt dt
CHEMICAL KINETICS 627

On substituting the given value 10. (d) The time in which the concentration of a reactant is
-3 reduced to half of its original value is called half-life period
dI 2 4.56 ´ 10 ´5
\ = = 1.14 × 10–2 M/s = 0.0114 Ms–1. of the reaction.
dt 2 But it is impossible to perform 100% of the first order
reaction. Because substance never reacts completely as
Exercise - 3 in every half-life, 50% of the substance reacts. So, time
1. (c) The role of a catalyst is to change the activation taken for 100% completion of a reaction is infinite.
energy of reaction. 11. (b) r = k [A]x[B]y
2. (c) The heat absorbed or evolved during the reaction x y
remains unchanged in presence of catalyst as there is no Rate of exp.1 [0.30] [ 0.30]
=
change in stability of reactant and product. Rate of exp. 2 [ 0.30] x [ 0.60] y
3. (b) Arrhenius equation relates activation energy of a y
0.10 [ 0.30 ]
chemical reaction with rate constant of a reaction at two =
0.40 [ 0.60 ] y
different temperatures.
y
æk ö E é1 1 ù 1 é1ù
ln ç 1 ÷ = a ê - ú =
4 êë 2 úû
è k 2 ø R ë T1 T2 û
2 y
4. (d) As per Arrhenius equation k = Ae -Ea / RT é1ù é1ù
êë 2 úû = êë 2 úû
So, k µ e - Ea
1 y=2
- x y
k µe T Rate of exp.1 [0.30 ] [0.30 ]
= x
µ eT Rate of exp. 3 [ y
0.60 ] [ 0.30 ]
These relations show that rate constant increases with x y
decreasing activation energy and increasing temperature. 0.10 é 0.30 ù é 0.30 ù
=
5. (c) Order of reaction may or may not be equal to sum of 0.20 êë 0.60 úû êë 0.30 úû
stoichiometric coefficients of reactants in the balanced 1 é1ù
x
chemical equation. = [1] y
6. (a) Concentration of reactant decreases with passage of 2 êë 2 úû
time, so rate of reaction decreases because rate changes x
1 é1ù
with change in concentration of reactant or product with =
2 êë 2 úû
time.
i.e., x = 1
7. (c) 5Br - ( aq ) + BrO 3- ( aq ) + 6H + ( aq ) ¾¾
® \ Rate = k [A]x[B]y
3Br2 ( aq ) + 3H 2O ( l ) Rate = k [A]1[B]2
Rate law expression as 12. (a) The minimum energy required to convert reactant
molecules into product molecules.
1 D [ Br - ] D éBrO 3– ûù -1 D [ H + ] +1 D [ Br2 ] i.e. Activation energy = E1 + E2
- =- ë = =
5 Dt Dt 6 Dt 3 Dt Energy of product is greater than the reactants so less
D [ Br ] D é BrO3 ùû -5 D [ H ]
- – + stable.
Þ – =- ë = 13. (b) A(g) ¾® B(g) + C(g)
Dt Dt 6 Dt Initially pi 0 0
D [ Br - ] 5 D [ H + ] At time t pi – x x x
Þ =
Dt 6 Dt Total pressue at equilibrium, pt = pi – x + x + x = pi + x
8. (b) Rate of reaction w.r.t B is of first order For first order reaction x = pt – pi
R1 = k [A] [B] 2.303 pi 2.303 pi
when concentration of reactant 'B' is doubled then rate k= log = log
t pi - x t pi - ( pt - pi )
(R2) will be :
R2= k [A] [2B] 2.303 pi
= log
R2= 2k [A] [B] t 2 pi - pt
R2 = 2 R1
14. (a) According to Arrhenius equation, k = Ae- Ea / RT
9. (c) In collision theory of reaction rates, the key factors

Taking log on both side ln k = ln ( Ae )


which determine whether a particular collision will result E
- a
in a reaction is the energy of collision and the orientation RT
of the collision.
EBD_8350
628 CHEMISTRY

Ea d[C ]
ln k = ln A - 21. (d) Let rate of reaction = = k[A]x [B]y
RT dt
Now from the given data
æ Ea ö 1
ln k = ç - ÷ ´ + ln A 1.2 × 10 – 3 = k [0.1]x[0.1]y .....(i)
è R ø T 1.2 × 10 – 3 = k [0.1]x[0.2]y .....(ii)
- Ea
From the graph, it is clear that slope = and intercept 2.4 × 10 – 3 = k [0.2]x[0.1]y .....(iii)
R
= ln A. Dividing equation (i) by (ii)
15. (c) Zn + dil. HCl ¾¾
® ZnCl 2 + H 2 ­ 1.2 ´10-3 k[0.1]x [0.1] y
Þ =
Rate of reaction 1.2 ´10-3 k [0.1]x [0.2] y
Change in concentration of H 2 V3 - 0 V3 We find, y = 0
= = =
Change in time 40 - 0 40 Now dividing equation (i) by (iii)
16. (b) Instantaneous rate of reaction 1.2 ´10-3 k[0.1]x [0.1] y
Þ =
Change in volume 2.4 ´10 -3 k [0.2] x [0.1] y
= Time interval close to 40s We find, x = 1
V4 - V2 V -V d [C ]
rinst ( 20s ) = incorrect, correct is 5 2 Hence, = k[ A]1 [ B ]0
50 - 30 50 - 30 dt
17. (a) Ea(F.R.) + DH = Ea(B.R.) Trick : From experiment (1) and (2), on doubling the [B],
rate of reaction remains same. From ex. (1) and (3), on
DH = – ve
doubling [A], rate also becomes two times. Thus order of
Activated complex
reaction w.r.t. A is one and that w.r.t. B is zero.
22. (d) Rate constant k = 0.6 × 10–3 mole per second. (unit
mole per second shows zero order reaction)
For a zero order reaction
Ea(B.R.) [A] = [A]0 – kt
Energy

Ea(F.R.) and [A0] – [A] = [B] = kt


DH = 0.6 × 10–3 × 20 × 60 = 0.72 M
0.693
23. (a) t1/2 =
k
Reaction coordinate
For first order t1/2 is independent of initial concentration
18. (b) Catalyst does not change Gibbs free energy because of reactant.
Gibbs free energy is related to concentration of reactant 24. (b) Arrhenius equation
and product which is not changed by catalyst. Ea
19. (b) A ® B K =A.e - Ea / RT Þ ln K = ln A –
RT
As time increases, concentration of reactant decreases
– Ea
and concentration of product increases exponentially. slope =
20. (b) Pseudo first order reaction is a chemical reaction in R
so, activation energy of reaction can be determined from
which rate of reaction depends upon concentration of
only one reactant while concentration of another reactant 1
the slope of ln K vs .
has no effect on rate of reaction. T
Hydrolysis of ester by dil. acid is an example of pseudo 25. (c) Reactions of higher order (>3) are very rare due to
first order reaction, i.e. very less chances of many molecules to undergo effective
H+ collisions.
CH 3 COOC2 H5 + H 2 O ¾¾¾
® CH 3 COOH + C 2 H 5OH
26. (a) For a first order reaction
Rate = k ¢ [CH3COOC2H5] [H2O]
But in this reaction mixture, water exists in large excess as
2.303 ( a - x1 ) 2.303 (rate)1
k = t - t log a - x = log
compared to ester. Hence, reaction obeys the following ( 2 1) ( 2) t2 - t1 (rate)2
first order kinetics : [rate µ conc.]
Rate = k[CH3COOC2H5] 2.303 æ 0.04 ö
k= ( log ç ÷
Rate constant k = k¢[H2O] or the value of rate constant 20 - 10 ) è 0.03 ø
depends on the concentration of reactants present in 2.303 ´ 0.1249
excess ([H2O]). k=
10
CHEMICAL KINETICS 629

0.6932 2.303 ´ 0.1249 – Ea / RT


t1/2
= so, k1 = A.e 1 .....(1)
10
0.6932 ´10 – Ea / RT
.....(2)
t1/2 = = 24.1 sec k2 = A.e 2
2.303 ´ 0.1249
( Ea – Ea )
27. (d) A catalyst provides an alternative route for the reaction k
1 2
with a lower activation energy. On dividing equation (2)/(1) Þ 2 = e RT
k1
1
28. (d) H2O2(aq) ® H2O(aq) + O (g) æ k ö E a – Ea 2 10, 000
2 2 ln ç 2 ÷ = 1 = =4
For a first order reaction k
è 1ø RT 8.314 ´ 300
2.303 a 32. (b) (t1/2)1st order = Independent of concentration
k= log
t (a – x )
1
Given a = 0.5, (a – x) = 0.125, t = 50 min (t1/2)2nd order µ
[A]0
2.303 0.5
\ k= log = 2.78 ´ 10–2 min–1 33. (a) CH 3 CHO ¾¾
® CH 4 + CO
50 0.125
r = k[H2O2] = 2.78 ´ 10–2 ´ 0.05 = 1.386 ´ 10–3 mol min–1 Generally r µ ( a – x ) m
d [ H 2O2 ] d [ H2 O] 2d [ O2 ] r1 = 1 torr sec–1, when 5% reacted
Now, - = = r2 = 0.5 torr sec–1, when 33% reacted
dt dt dt
2d [ O2 ] d [ H 2 O2 ] (a - x1 ) = 0.95(unreacted)
\ = - (a - x2 ) = 0.67(unreacted)
dt dt
d [ O2 ] 1 d[H 2 O2 ] 1.386 ´10-3 r1 é (a - x1 ) ù
m
1 æ 0.95 ö
m
\ ´= = =ê ; =
dt 2 dt 2 ú ç ÷
r2 ë (a - x2 ) û 0.5 è 0.67 ø
= 6.93 ´ 10–4 mol min–1
29. (c) The overall reaction rate depends on the rate of the 2 = (1.41)m Þ 2 = ( 2)m
slowest step. Þ m=2
i.e., Overall rate = Rate of slowest step (ii) = k[X][Y2] ...(1) 34. (c) For a first order reaction,
k = rate constant 2.303 [A]o
Assuming step (i) to be reversible, its equilibrium constant, t= log
k [A]
[ X ]2 for 99% completion of the reaction,
Þ [ X ] = keq [ X 2 ];
keq = 2
[X2 ] 2.303 100
t= log
1 1
k 100 - 99
[X] = 2
keq [X2 ]2 … (2)
t=
2.303
log 102
From eq (1) and (2) k
1 4.606
1 t=
Rate = kkeq 2 [ X 2 ] 2 [ Y2 ] k
35. ˆˆ† 2NH 3 (g)
(c) N 2 (g) + 3H 2 (g) ‡ˆˆ
1 3
Overall order = + 1 = = 1.5
2 2 - d [ N2 ] -1 d [ H 2 ] 1 d [ NH3 ]
= =
30. (a) Half life for a first order reaction, dt 3 dt 2 dt
0.693 36. (a) In graph (i), ln [Reactant vs time is linear with positive
t1/2 =
K intercept and negative slope. Hence it is 1st order In graph
0.693 (ii), [Reactant] vs time is linear with positive intercept and
So, t1/2 = sec . negative slope. Hence, it is zero order.
10-2
Also, for the reduction of 20 g of reactant to reduce to 5 g, 37. (a) From experiment 1 and II, it is observed that order of
two half lives will be required. reaction w.r.t. (c) is zero.
\ For 20 g of the reactant to reduce to 5g, time taken, From experiment II and III, a can be calculated as:
0.693 1.386 ´ 10-2 a
t=2× sec = 138.6 sec. æ 0.2 ö
10-2 -3 = çè 0.1 ÷ø \ a= 1
6.93 ´ 10
31. (d) From arrhenius equation,
Now, Rate = K[A]1
– Ea
or, 6.93 × 10–3 = K(0.1)
k = A.e RT
EBD_8350
630 CHEMISTRY

K = 6.93 × 10–2 0.27 0.29 0.29


For the reaction, 2A + B ® Products Þ = log Þ 0.117 = log
2.303 (a - x) ( a - x)
[A]0
2Kt = ln Þ (a – x) = 0.22 M.
[A]
2000 1000
- -
0.693 0.693 5. (d) Given, k1 = 1016.e T and k2 = 1015.e T
\ t1/2 = =
2K 0.693 ´10-2 ´ 2 when k1 and k2 are equal at any temperature T, we have
t1/2 = 5 2000 1000
- -
2.303 [A ] 1016 × e T 15
= 10 × e T
38. (b) First order rate equation is k = log 0
t [ A] 2000 1000
- -
-3 2.303 2 or 10 ´ 1015 × e T 15
= 10 × e T
Þ 4.606 ´ 10 = log
t 0.2 2000 1000
- -
2.303 103 or 10 × e T
=e T
Þ t= ´ log10 = = 500 sec
4.606 ´ 10-3 2 2000 1000
or ln 10 - =-
39. (c) The number of collisions per second per unit volume T T
of the reaction mixture is known as collision frequency (Z)
2000 1000
collision frequency µ no. of reacting molecules or atoms. or ln 10 = -
Higher the concentration of reactant molecules higher is T T
the probability of collision and so the collision frequency. 1000
or 2.303 log 10 =
40. (23.03) t1/2 = 6.93 years, T
a = 10–6 g or 2.303 ×1×T=1000 [\ log 10= 1]
0.693 1000
t1/2 = or T= K
K 2.303
0.693 0.693 6. (a)
Þ K= = = 0.1
t1/2 6.93
Ea = x kJ/mol
For Ist order reaction,
2.303 a
K= log
t a-x
2.303 a 2.303 10-6 – 40 kJ/mol
t= log = log -7
K a-x 0.1 10
2.303
= = 23.03 years æ Eforward ö
0.1 ç (A ®B) ÷ 2 x
ç ÷= =
E
ç backward ÷ 3 - 40 +x
Exercise - 4
è (B®A) ø
1. (d) At = A0e–kt where At = (a – x) = conc. of reactant Þ –80 + 2x = 3x
remained after time = t Þ x = –80 kJ/mol for forward
A0 = a = Initial concentration of reactant when t = 0 For backward = –40 + (–80) = –120 kJ/mol
n -1 Because activation energy cannot be negative
(t1/2 )1 æ a2 ö log (t1 / t2 ) so, for forward reaction Ea = 80 kJ/mol
2. (c) = ç ÷ ; n = 1+
(t1/2 )2 è a1 ø log (a2 / a1) for backward reaction Ea = 120 kJ/mol

3. (b) k =
0.693
min -1 =
2.303
log
a
or
7. (c) k = Ae- Ea / RT
45 t99.9% a - 0.999a Ea
2.303 ´ 45 when T ® ¥, - ® O or e- Ea / Rt ¾¾
®1
t99.9% = log103 = 448 min » 7.5 hrs RT
0.693 Þ k = A = 3.0 × 1015s–1
2.303 a Other given data are redundant.
4. (b) k= log
t (a - x )
k2 Ea æ 1 1 ö
(a – x) is the concentration left after 100 sec. 8. (a) log = ç - ÷
k1 2.303R è T1 T2 ø
2.303 0.29
2.7 ´ 10-3 = log
100 ( a - x) æ 2.2 ´10-5 ö
Þ log ç ÷
ç 2 ´10-6 ÷
è ø
CHEMICAL KINETICS 631

Ea ´ (303 - 287) kC 3.8 ´ 10 - 5


= -3 The fractional yield of C = =
2.303 ´ 2 ´10 kcal / mol – k ´ 287 ´ 303k k av 1.64 ´ 10 - 4
[T1 = 273 + 14 = 287k, T2 = 273 + 30 = 303 k] = 0.2317 = 23.17%
Ea ´ 16 14. (d) Since the slow step is the rate determining step hence
Þ log11 = if we consider option (A) we find
400.542kcal / mol
Þ Ea = 26 kcal/mol Rate = k [ Cl2 ][ H 2S]
Now if we consider option (B) we find
9. (d) 2N 2O5 ¾¾
® 4NO 2 + O2
Rate = k [ Cl2 ] éë HS ùû
-
...(i)
a 0 0 t =0
(a - x) (2 x) ( x / 2) time = t For equation,
ˆˆ† H+ + HS–
H2S ‡ˆˆ
Ct = Þ (a – x) =
C0e–kt ae–kt
Þ x = a – ae–kt Þ x = a (1 – e–kt) é H + ù é HS- ù
[NO2] = 2x = 2a(1–e–kt) K = ë ûë û
[H 2S]
10. (b) Use the formula:
K [ H 2 S]
é -ù
2.303 V0 1 V0 or ë HS û =
t= log V = ln V [H + ]
k t k t
Substituting this value in equation (i) we find
1 25mL log e 5
= –2
4.5 ´ 10 min –1 ln 5mL =
4.5×10 – 2
min
Rate = k [ Cl2 ] K
[ H 2S] = k ' [ Cl2 ][ H 2S]
a 1 1 H+ éH+ ù
ë û
11. (d) t100% = = min = ´ 60 sec = 100 sec.
k 0.6 0.6 Slow step should involve 1 molecule of Cl2 and 1 molecule
Reaction will be finish in 100 sec., so concentration of B of H2S only.
will reach maximum value 2 M after 100 sec. Hence, only mechanism (A) is consistent with the given
12. (c) First step is slow (require more activation energy), rate equation.
second step is fast (less activation energy) and overall 15. (b) T1 = 273 + 25 = 298 K
reaction is exothermic, so product energy level should be T2 = 273 + 60 = 333 K
less as compared to reactants.
13. (d) For two parallel reactions k2 Ea æ T2 – T1 ö
log = ç ÷
kaverage = k1 + k2 = 1.26 × 10–4 + 3.8 × 10–5 k1 2.3 R è T1T2 ø
= 1.64 × 10–4sec–1 k2 Ea æ T2 – T1 ö
or loge = ç ÷
kB k1 R è T1T2 ø
The fractional yield of B =
k av 2.1 ´ 10 – 2 Ea æ 35 ö
loge = ç ÷
1.26 ´ 10 - 4 1.5 ´ 10 – 3 R è 333 ´ 298 ø
= = 0.7683 = 76.83 %
1.64 ´ 10 - 4 298 ´ 333 21
\ Ea = × R × loge
35 1.5
EBD_8350
632 CHEMISTRY

19 Surface Chemistry
Exercise - 1 restricted i.e. DS is negative. Hence, adsorption
accompanied by decrease in enthalpy as well as decrease
1. (a) On increasing temperature, physical adsorption of a in entropy of the system.
gas on surface of solid decreases. Solid adsorb greater 13. (d) The heat evolved in physisorption is quite low
amount of gas at lower temperature. varying generally between 20-40 kJ mol–1.
2. (a) Chemical adsorption involves formation of monolayer. 14. (d) x/m ¹ P × T
3. (a) Chemisorption involves strong chemical forces. 15. (b) The gases having higher values for critical
4. (d) In physisorption, gas molecules are held on the solid temperature are easily liquified and are adsorbed to the
surface by weak van der Waal’s forces. greater extent. CO2 has highest critical temperature of 304K.
5. (b) Chemisorption increases with increase in temperature. 16. (a) On increasing temperature adsorption of a gas on
6. (d) Adsorption is accompanied by evolution of heat as surface of solid decreases. Solid adsorb greater amount of
the residual forces acting along the surface of adsorbent substances at lower temperature.
decrease i.e., adsorption is accompanied by decrease in 17. (a) For adsorption to occur DG = –ve, as in adsorption,
enthalpy. DH = –ve and DS = –ve, thus, –TDS is positive for
7. (d) According to Freundlich adsorption isotherm. adsorption.
At intermediate pressure, extent of adsorption 18. (c)
x x 1 19. (c) Assertion is true, reason is false. When several lines
= kP1/ n or log = log k + log P 1
m m n have the same value of , then the lines by which their
n
adsorption isotherms can be represented will be parallel
1 and will not meet at a point.
pe
= n 20. (c) Adsorption is an exothermic process, hence DH will
S lo always be negative.
m

21. (d) Freundlich’s isothermal adsorption equation can be


x

given as
log

x
= kp1/ n
m
Intercept = log k x 1 1
log = log k + log p; slope = = 0
log P m n n
x 1 x
plot of log vs log P is linear with slope = Thus, = kp0
m n m
8. (a) According to Freundlich adsorption isotherm 22. (d) Assertion is false but Reason is true. The enthalpy
1 of chemisorption is of the order of 40 - 400 kJmol–1 while
x for physical adsorption it is of the order of 20 - 40 kJmol–1.
= kP n
m 23. (c) Both adsorption and absorption can take place
The value of n varies between 0 to 1.
simultaneously. The term sorption is used to describe both
9. (c) Physical adsorption involves weak forces, physical in
the processes.
nature with small heat of adsorption. Thus, low temperature
24. (d) For a reaction in equilibrium, the increase in rate of
and high pressure favours physical adsorption.
reaction in forward direction by catalyst increases the
10. (b) The adsorption of methylene blue on activated
concentration of product(s) and thus the rate of backward
charcoal is an example of physiosorption which is
reaction also increases with the same magnitude and allow
exothermic, multilayer and does not have energy barrier.
the equilibrium to be achieved quickly.
11. (b) The more readily soluble and easily liquefiable gases
25. (a) A catalyst increases the rate of reaction by decreasing
such as NH3, HCl and SO2 are adsorbed more than the so
the activation energy.
called permanent gases like O2. This is due to the van der
26. (c) Active charcoal has more adsorption power due to
Waals or intermolecular forces which are involved in
greater surface area.
adsorption.
27. (b) Catalyst decreases the activation energy of the
12. (c) Adsorption is an exothermic process i.e. DH of
reaction by forming an intermediate product. So, no. of
adsorption is always negative. When a gas is adsorbed,
molecules having activation energy increases hence, rate
the freedom of movement of its molecules becomes
of reaction increases.
SURFACE CHEMISTRY 633

28. (c) A catalyst can affect reversible reaction by attaining


equilibria in both directions. Polar head
51. (a)
29. (d) Enzymes are biological catalysts and enhance the rate Non-polar tail
of biochemical reactions. (micelle)
30. (c) Hydrolysis of urea can be represented as follows
Urease 52. (c) Lyophobic colloids are irreversible colloids. They are
H 2 N - C - NH 2 + H 2O ¾¾¾¾® 2NH3 + CO 2 protected by lyophilic colloids.
|| (enzyme)
O 53. (d) Sodium stearate is a soap. Soaps and detergents are
Since, it involves biological catalyst (enzyme) so, it is an surface agents which when dissolved in a medium, forms
example of biochemical catalysis. aggregated particles, called associated colloids.
31. (d) Efficiency of catalysing property of a catalyst is 54. (d) The formation of micelles takes place only above a
inversely proportional to activation energy. particular temperature called kraft temperature (TK).
32. (d) The catalytic poisons decrease the activity of the 55. (b) Electrolytes are used for neutralising charge on
catalyst because they are preferentially adsorbed on the colloidal particles.
surface of catalyst. 56. (c) The osmotic pressure of sea water is 25 atm at 15°C.
33. (c) Adsorption theory is applied to heterogeneous When pressure greater than 26 atm is applied on sea water
catalysis. separated by a rigid semipermeable membrane, pure water
34. (d) In acid hydrolysis of methyl acetate, all are present in is obtained.
one phase (liquid). 57. (a) Blood is purified by dialysis
35. (d) Enzymes are specific biological catalysts possessing 58. (d)
well - defined active sites. 59. (b) Peptisation is disintegration of colloidal aggregates.
36. (b) Molybdenum acts as a promoter for iron which is 60. (c) According to this law, the coagulating effect of an
used as a catalyst in Haber’s process. ion on dispersed phase of opposite charge increases with
37. (c) Second step in volves adsorption of reactan t the increase in valency of the ion. The precipitating power
molecules on the surface of the catalyst. of Al3+ , Ba2+, Na+ ions is in order Al3+ > Ba2+ > Na+.
38. (a) (i) This property is called selectivity of catalyst. 61. (a) Tyndall effect is shown by sols.
(ii) Catalyst is required in small quantities. 62. (b)
(iii) Catalyst does not change DH of reaction as it 63. (b) The size of colloidal particles is between 1 nm and
does not affect the energy states of reactant 1000 nm i.e., it is not always greater than 100 nm. So, (b) is
and product. not a property of colloidal solution. All others are the
(iv) Catalytic activity depends on strength of properties of colloidal solution.
chemisorption. 64. (d)
39. (a) 40. (a) 65. (a) Soap lather is a colloid containing gas as a dispersed
41. (d) Enzymes are most reactive at optimum temperature. phase and liquid as a dispersion medium.
The optimum temperature for enzyme activity lies between 66. (a) Chlorophyll. Smoke is an example of solid-gas colloid
40°C to 60°C. system. Ruby glass is an example of solid-solid colloid
42. (c) Zeolites are found in nature as well as synthesised system. Milk is a liquid -liquid colloid system.
for catalytic selectivity. 67. (c) Starch molecules have colloidal dimensions whereas
43. (a) Milk is an emulsion in which oil is dispersed in water. NaCl, glucose and Ba(NO3)2 are crystalloids and soluble
44. (d) Mist is a colloid (aerosol) in which liquid is dispersed in water.
in gas. 68. (a) Multimolecular colloids consist of aggregates of
45. (c) Alloy is an example of solid solution. atoms or small molecules. Sulphur sol is an example of
46. (b) Colloid Dispersed phase Dispersion medium multimolecular colloids
Sol Solid Liquid 69. (a) Colloid of liquid in liquid is called emulsion. Colloid
Gel Liquid Solid of liquid in solid is gel.
Emulsion Liquid Liquid 70. (b) Brownian movement is zig zag motion of sol particles.
Foam Gas Liquid 71. (c) Size of colloidal particles is 10–5 – 10–7 cm.
47. (d) Emulsions are liquid-liquid colloidal systems, 72. (b) Butter is an example of gel.
Generally one of the two liquids is water. 73. (a) Surface tension of lyophilic sols is lower than water
48. (d) Air is a homogeneous mixture of gases, mainly (dispersion medium).
nitrogen and oxygen. 74. (b) Gum is lyophilic colloid.
49. (c) When the dispersed phase and dispersion medium 75. (c) Example of multimolecular colloid system is a gold
both are liquid, the colloidal system is called an emulsion dispersed in water, i.e., aggregate of gold atoms.
like milk, vasnishing cream etc. 76. (a) On shaking with the dispersion medium, colloids
50. (b) Cloud consists of fine droplets of water suspended directly form the colloidal sol. Hence, they are called intrinsic
in air. colloids. i.e., glue.
EBD_8350
634 CHEMISTRY

77. (d) The use of membrane for separating colloidal particles 97. (a) Proteins are coagulated by some heavy metal ions
is termed as dialysis. It is clear that colloidal particle cannot like Ag+, Hg2+ and Pb2+.
pass through animal membrane. Hence, only solvent 98. (d) Tannin used in leather industry contains negatively
molecules and ions (in case of electrodialysis) can diffuse. charged colloidal particles.
78. (d) Electrolytic (ionic) impurities can be most easily 99. (b) Eosin dye, sol of charcoal, As2S3 and copper sol are
removed on application of electric field. examples of negatively charged sol.
79. (d) The separation of colloidal particles of molecular 100. (a) 101. (a) 102. (b) 103. (d)
dimension is known as dialysis. It is a purification method 104. (c) Emulsion show the tyndall effect.
for colloid. 105. (c) Protein is an emulsifying agent for O/W emulsion.
80. (c) Sols of platinum, silver, copper or gold is prepared by
this method. Exercise-2
81. (c) It is due to impact of molecules of dispersion medium 1. (2) Starch molecules have colloidal dimensions. Blood is
on the colloidal particles. a –vely charged colloidal system. Rest of the compounds,
82. (c) i.e., NaCl, urea and cane sugar form true solution in water.
83. (c) Sugar forms homogeneous solution hence, no Tyndall
2. (2) Fe(OH)3 particles absorb Fe3+ ions and get peptized
effect is exhibited.
84. (c) Tyndall affect is observed when viewed from the right to give a positively charged sol. Similarly AgCl particles
angle to the passage of light. absorbs Ag+ ions to give a positively charged sol.
85. (d) 86. (c) 3. (1) Chlorophyll. Smoke is an example of solid-gas colloid
87. (b) Bredig’s arc method is suitable for the preparation of system Ruby glass is an example of solid-solid colloid
colloidal solution of metals like gold, silver, platinum etc. system. Milk is an liquid-liquid colloid system.
An arc is struck between the metal electrode under the
2
surface of water containing some stabilzing agent such as 4. (4) 2 mL of 1 M NaCl contains NaCl = = 2 m mole
a trace of KOH. However, Fe does not react with alkalies 1000
Thus 500 mL of As2S3 sol require NaCl for complete
that is why it is not obtained by Bredig’s-arc method.
88. (d) Colloidal solutions are not purified by electrophoresis. coagulation = 2 m mole
Movement of colloidal particles under the influence of electric Hence 1 L, i.e., 1000 mL of the sol require NaCl for complete
field is called electrophoresis. Electrophoresis is the property coagulation = 4 m mole
of colloids not the purification method. Therefore, flocculation value of NaCl = 4.
89. (a) 5. (1) Gold number of gelatin = 0.01
90. (d) Al(OH)3 is a positive sol so salt having anion with or 0.01 mg gelatin required to be added to 10 mL of gold sol to
maximum negative charged (i.e. phosphate ion) will be most completely prevent coagulation of 1 mL of 10% NaCl solution.
effective in coagulation. Therefore, gelatin added to 1000 mL of gold sol to prevent
91. (b)
92. (c) Colloidal particles can pass through ordinary filter 0.01 ´ 1000
coagulation = = 1 mg
paper because the pores are too large. However, the pores 10
of filter paper can be reduced by impregnating with 6. (4) Mass of HCl acid adsorbed by 10 g charcoal
‘collodion’ solution to stop the flow of colloidal particles. = 526.3 × 10–3 (0.5 – 0.4) × 38 » 2
Thus, pore size of filter paper becomes less than the size of (Mw of HCl = 38 g mol–1)
colloidal particles. x 2
93. (d) Argyrol is used as an eye lotion. The amount of adsorption, = =4
m 0.5
Antimony is used in Kalazar. 7. (3) Aerosol, Emulsion, Gels.
Collidal gold is used in intramuscular injection.
8. (4) CMC of soap
Milk of magnesia is used in stomach disorder (acidity).
10–4 M to 10–3 M
94. (a) Colligative properties depend upon number of
particles. x= 4
95. (d) Cetyl trimethyl ammonium bromide, x
+ - 9. (4) = k × p1 / n
[C16 H33 (CH3 )3 N Br ] is a cationic micelle. m
96. (c) (a) All oils have hydrophobic ends in their structure. x
= 2 ´ 21 = 4
(b) Lanolin is a wax secreted by glands of wool bearing m
animals. 10. (25) Gold Number is the minimum amount of lyophilic
(c) Glycogen is a polysaccharide of glucose and glucose colloide in miligrams (starch in given case) which
is highly polar in nature as it contains large number of prevents the flocculation of 10 ml gold sol by the addition
hydroxyl groups. Thus, it is hydrophilic in nature. of 1ml of 10% NaCl solution.
(d) Rubber is a h ydrophilic sol but it contains \ Gold number = miligram of starch
hydrophobic structure also. = 0.025 × 103 mg = 25
SURFACE CHEMISTRY 635

Exercise-3 Electrolytes Anionic part Charge on anion


1. (c) In homogeneous catalysis reactant and catalyst have Na 2S S2– 2
same phase so, homogeneous catalysis can not occur at
Na 3 PO4 PO3–
4 3
the inertface of phases
2. (b) Equilibrium DG = 0 Na 2 SO4 SO42– 2
DH – TDS = 0 –
NaCl Cl 1
DH = TDS
3. (d) Gas-gas interface can not be obtained as gases are Here, PO3– 3–
4 have highest charge. Hence, PO4 have
completely miscible in nature.
highest coagulating power.
4. (c) When both absorption and adsorption occur it is
17. (d) Sol is a colloidal system in which solid substance is a
called sorption:
+ + + + dispersed phase and a liquid is a dispersion medium e.g.,
paint, cell fluids etc.
+ + + + + +

+ + + + + +

+ + + +
+ + + + In paints, solid colouring particles are dissolved in liquid
+ + + + adsorption dispersion medium.
+ + + + 18. (b) At high concentration of soap in water, soap particles
+ + + +
present in the solution get associated and it leads to the
+ + + + formation of associated colloid.
absorption sorption
5. (b) In physisorption, particles are attached to the surface 19. (c) Lyophobic sol can be protected by addition of
by weak van der Waal's force of attraction so, on increasing lyophilic sol as lyophobic sols are readily precipitated on
temperature they get desorbed. addition of small amount of electrolytes or shaking or
6. (a) When amount of adsorbate increases the interaction heating. Hence, they are made stable by adding lyophillic
of adsorbate-adsorbent increases which lead to increase sol which stabilises the lyophobic sols.
in adsorption. 20. (d) The value of colligative properties of colloidal
7. (a) Enthalpy change during adsorption is negative. As solution are of small order in comparison to those of true
in adsorption, there is decrease in residual forces of the solutions of same concentration because colloidal
surface which result in decrease in surface energy in form
particles are comparatively less in number.
of heat.
21. (b) The Correct sequence of steps is
8. (d) Adsorbate get adsorbed on the adsorbent surface
by weak van der Waal's force of attraction in physisorption. I ® III ® II ® IV ® V
Rise in temperature will break the interaction between Each step represents a meaningful process as follows
adsorbate and adsorbent so, adsorbate particles get I ® adsorption of A and B on surface
desorbed.
9. (b) With increase in temperature physisorption changes III ® II interaction between A and B to form
to chemisorption, because energy of activation of intermediate
adsorbate particles increases which lead to formation of
chemical bond. II ® IV starting desorption of A – B
10. (a) The extent of van der Waal's interaction between IV ® V complete desorption
absorbate and adsorbent is constant for all gases. 22. (c) River water is a colloidal solution of clay and sea. Sea
11. (b) Absorption is a bulk phenomena in which particles water contains various electrolytes. When river water
goes to the bulk instead of residing only to surface. comes in contact with sea water, then the electrolytes
12. (d) Critical temperature of gases is directly proportional present in sea water coagulate the suspended colloidal
to the extent of adsorption. particles (clay) which ultimately settle down at the point
13. (a) Reaction in which catalyst and reactant/product is in
of contact.
different phase then it is known as heterogeneous
23. (c) According to the Freundlich adsorption iotherm
catalysis.
1
14. (b) Tyndall effect is a characteristic of colloidal solution, x
as aqueous solution of soap above cmc form a colloidal = kp n
m
solution.
15. (d) Peptisation is a process in which by addition of a x æ 1ö
Taking log on both side log = ç ÷ log p + log k
suitable peptising agent precipitate gets converted into m è nø
colloidal solution. This equation is comparable with straight line equation,
16. (b) According to Hardy-Schulze law, greater the charge y = mx + c, where m represents slope of the line and c
on anion greater will be its coagulating power for positive represents intercept on y axis.
charged colloid.
EBD_8350
636 CHEMISTRY

31. (a) A catalyst speeds up both forward and backward


reaction with the same rate.
So, equilibrium constant is not affected by the presence of
a catalyst at any given temperature.
log x /m
Slope = 1/n 32. (b)
33. (d) According to Hardy Schulze rule, coagulating power
log k (intercept) of an ion depends on both magnitude and sign of the
log p
charge on the ion.

24. (b) Above figure represent adsorption of yellowish 34. (b) AgNO3 + KI ¾¾
® AgI + KNO 3
Negatively
brown colour of raw sugar by animal charcoal. charged colloid
Here, aqueous solution of raw sugar is filtered by using
A solution of AgNO3 and KI will form a negatively charged
animal charcoal. Yellowish brown colour of raw sugar is
colloidal sol, [AgI]I–, only when KI is present in excess
adsorbed and filterate is colourles which gives white colour
(i.e., KI behaves as a solvent).
on cystallisation. Hence, this phenomenon is adsorption.
25. (d) Absorption of ionic species from solution is not Millimole of KI is maximum in option (b) (50 × 2 = 100)
responsible for the presence of electric charge on the sol 35. (c) In Freundlich adsorption isotherm the extent of
particles. Charge on the sol particles is due to adsorption (x/m) of a gas on the surface of a solid is related
(i) electrons capture by sol particles during electro to the pressure of the gas (P) which can be formulated as:
dispersion of metal. x
= k(p)1/n
(ii) preferential adsorption of ionic species from solution. m
(iii) formation of Helmholtz electrical double layer. x 1
Þ log = log k + log p
26. (b) For adsorption DS < 0 and for a spontaneous change m n
DG = – ve. x
In the given plot, the slope between log versus
Hence, DH should be highly negative which is clear from m
the equation 2 1
log P = =
DG = DH – TDS 4 2
27. (d) Tyndall effect is optical property, whereas other x
\ µ p1/2
properties are electrical properties. Hence, dependent on m
the charge on colloids. 36. (a) In aerosol, the dispersion medium is gas while the
28. (b) Adsorption is spontaneous process, therefore, change dispersed phase can be both solid or liquid.
in the free energy (DG) for the process is negative. 37. (b) In colloidal solution, the potential difference between
According to Gibbs's Helmholtz eqn. the fixed layer and the diffused layer of opposite charge is
DG = DH – TDS known as Zeta potential.
DS is negative because adhering of gas molecules to the Greater the Zeta potential more will be the stability of
surface lowers the randomness. colloidal particle.
\ DG can be –ve only when DH is –ve.
29. (a) Fog is a colloidal system having dispersed phase as Exercise-4
liquid and dispersion medium as gas. 1. (c) For ph ysisorption, the extent of adsorption
30. (a) According to Freundlich adsorption isotherm: decreases as the temperature increases. Thus, T3 > T2 > T1
x 1
log = log K + log p Coagulation power of AlCl3
m n 2. (c)
Thus, if a graph is plotted between log(x/m) and log p, a Coagulation power of NaCl
straight line will be obtained
Coagulation value of NaCl 52
= = = 559.13 » 560
Coagulation value of AlCl3 0.093
log (x/m) ¾®

n
1/
=
pe 3.6
slo 3. (c) Mass of O2 per gram of adsorbent = =3
1.2
3
No. of moles of O2 per gram of adsorbent =
Intercept = log K
32
Volume of O2 per gram of adsorbent
log p ¾® 3 0.0821 ´ 273
= ´ = 2.10
The slope of the line is equal to 1/n and the intercept on 32 1
log (x/m) axis will correspond to log K.
SURFACE CHEMISTRY 637

4. (d) Volume of gold dispersed in 1 L water 8. (a) Calcium and magnesium salts in hard water form
-4 insoluble salts of higher fatty acids.
Mass 1.9 ´10 g
= = = 1 × 10–5 cm3 2C17 H 33COONa(aq.) + Ca 2+ (aq.) (Hard water) ¾¾ ®
Density 19 g cm -3
Radius of gold sol particle (C17 H33 COO) 2 Ca(s) + 2Na + (aq.)
= 10 nm = 10 × 10–7 cm = 10–6 cm 9. (d) In first case, the given compounds have same anion
4 but different cations having different charge. Hence, they
Volume of gol sol particle = p r 3 will precipitate negatively charged sol i.e. ‘A’.
3
4 22 In second case the given compounds have similar cation
= ´ ´ (10-6 )3 = 4.19 × 10–18 cm3 but different anion with different charge. Hence, they will
3 7
precipitate positively charged sol. i.e. ‘B’. Also, flocculation
\ No. of gold sol particles in 1 × 10–5 cm3 power of Ba2+ will be more than K+ for negative charged
1´ 10-5 sol ‘A’ and that of SO42– is more than Br– for positive
= = 2.38 ´ 1012 charged sol ‘B’.
4.19 ´ 10 -18
= particles in 106 mm3 (1L) of sol

¾®
\ No. of gold sol particles in one mm3
10. (a) x
2.38 ´ 1012
= = 2.38 ´ 106 m
106
P ¾®
5. (b) AgNO3(aq) + KI(aq) ® AgI(s) + KNO3(aq) x
5 × 10–3 10 × 10–3 At PS , = constant
m
0 5 × 10–3 5 × 10–3 11. (b) Mass of acetic acid adsorbed by 2 g charcoal
AgI(s) + I–(aq) ® AgI : I–(aq) = 100 × 10–3 × (0.6 – 0.5) × 60 = 0.6 g
Negative charge colloids. x 0.6
Þ = Þ 0.3
6. (a) AgNO3(aq) + KI(aq) ® AgI(s) + KNO3(aq) m 2
x y 0 0 initial 12. (c) As colloidal particles move towards anode so these
x–y 0 y y final particles are negatively charged and coagulated by cations
AgI(s) + Ag+(aq) ® AgI : Ag+(sol) of electrolyte.
According to Hardy Schulze rule,
Positive charge colloid.
Coagulation power µ Charge on ion
7. (b) Colloidal iron (III) hydroxide – positive charged sol
\ Order of coagulation power is Al3+ > Ba2+ > Na+
Colloidal gold – negative charged sol
13. (b) Given reactions shows that the selectivity of different
(a) When electric field is applied, coagulation will occur catalysts for some reactants is different.
in both sols. 14. (c) For a protective colloid, lesser the value of gold
(b) When oppositely charged sols are mixed, their charges number, more will be the protective power. Thus, the correct
are neutralised. Both sols will be partially or completely order of protective power of A, B, C and D is
precipitated. Þ (A) < (C) < (B) < (D)
(c) Na2SO4 solution will cause coagulation in both sols. Gold number 0.50 0.10 0.01 0.005
Na+ will cause coagulation in negative charged sol and 15. (c) Electrodialysis involves movement of ions towards
SO42– will cause coagulation in positive charged sol. oppositely charged electrodes.
(d) MgCl2 solution will cause coagulation in both sol. Urea being a covalent compound does not dissociate
But due to more positive charge in Mg2+ than the negative to give ions and hence, it cannot be removed by
charge in Cl– ion, Mg2+ will coagulate the negative charge electrodialysis. However, all the other given compounds are
sol (gold sol) more than Cl– can coagulate the positive ionic which can undergo dissociation to give oppositely
charge sol (iron hydroxide sol). charged ions and thus, can be separated.
EBD_8350
638 CHEMISTRY

20 General Principles and Processes of Isolation of Elements

Exercise - 1 19. (c) Azurite is a basic carbonate ore of copper.


2CuCO3. Cu(OH)2
1. (a) Argentite or silver glance (Ag 2S) is an ore of Ag. 20. (c) Oxygen is about 46.6% by weight in earth’s crust.
21. (d) Zincite is ZnO.
2. (a) Cinnabar (HgS) is an ore of Hg.
3. (a) Bauxite ore of aluminium is Al2O3·xH2O. 22. (b) Cassiterite contains the magnetic impurities of FeSO 4
4. (a) All ores are minerals but not vice versa. and thus concentrated by electromagnetic separation.
5. (b) Al is most abundant metal on the surface of the earth. 23. (c) Pyrolusite is MnO2. Hence, not concentrated by froth
6. (c) Carborundum - SiC floatation process.
Epsomite or Epsom salt - MgSO4.7H2O 24. (c) Froth reduces the surface tension of water and the
Cassiterite - SnO2 solution forms froth.
Spodumene - Ore of lithium 25. (a) For example, Ag2S is converted into Na[Ag(CN)2].
7. (a) Chalcopyrite : CuFeS2 When Zn is added, Ag is displaced.
Fool's gold : FeS2 26. (d) Assertion is false but reason is true. Leaching is a
Carnalite : KMgCl3.6H2O process of concentration.
Bauxite : Al2O3.2H2O 27. (c) Cuprite : Cu2O; Chalcocite : Cu2S; Chalcopyrite : CuFeS2;
8. (a) The formula of magnetite is Fe3O4. Malachite: Cu(OH)2.CuCO3. We see that CuFeS2 contains
9. (a) Galena is PbS and thus purified by froth floatation both Cu and Fe.
method. 28. (c) Assertion is true but reason is false.
Froth flotation method is used to concentrate sulphide Oxide ores being heavier than the earthy or rocky gangue
ores. This method is based on the fact that the surface of particles, settle down while lighter impurities are washed
sulphide ores is preferentially wetted by oils while that of away.
gangue is preferentially wetted by water. 29. (b) Except chromium all the given metals exists as their
10. (d) Cyanide process is used in the metallurgy of Ag sulphides.
Zn exists as zinc blende ZnS.
2Ag 2S + 8KCN + O 2 + 2H 2O ¾¾ ®
Silver exists as silver glance Ag2S.
4K[Ag(CN) 2 ] + 4KOH + 2S
Iron exists as iron pyrites FeS2.
2K[Ag(CN) 2 ] + Zn ¾¾ ® K 2 [Zn(CN) 4 ] + 2Ag ¯ 30. (a) To remove moisture and non-metallic impurities like
11. (a) It is a type of gravity separation. S, P and As are oxidised and are removed as volatile
12. (b) Leaching is a process used for concentration of ore. substances.
In this process, a powdered ore is treated with a suitable S8 + 8O 2 ® 8SO 2 ­ ; P4 + 5O2 ® P4 O10 ­
reagent (such as acids, bases or other chemicals) which
can selectively dissolve the ore, but not the impurities. 4As + 3O2 ® 2As2O3 ­
13. (a) Ag is leached by cyanide process. 31. (c) In this process sulphide ores are converted into oxide
14. (a) Froth flotation process is based on wetting properties ores.
of ore particles. 2ZnS + 3O2 ® 2ZnO + 2SO2­
15. (c) Malachite (CuCO3 . Cu(OH)2) is an ore of copper. 32. (a) Calcination is heating ore in absence of air to remove
16. (a) (a) Zinc Calamine is ZnCO3 moisture and volatile impurities. Carbonate ores decomposed
(b) Silver Ilmenite is FeTiO3 to corresponding oxides as a result of calcination.
(c) Magnesium Cassiterite is SnO2 33. (b) Since silica is acidic impurity, the flux must be basic.
(d) Tin Azurite is CaO + SiO2 ® CaSiO3
[2CuCO3.Cu (OH)2] 34. (d) Decomposition of carbonates and hydrated oxides.
17. (a) MnO2 is pyrolusite (oxide ore). 35. (b) Ore is being heated in presence of air.
18. (d) Leaching is the selective dissolution of the desired 36. (b) Flux + Gangue ® Slag
mineral leaving behind the impurities in a suitable 37. (d) Calcination is a process of heating a substance to a
dissolving agent e.g., high temperature but below the melting or fusion point,
Argentitie or Silver glance, Ag2S is an ore of silver. Silver causing loss of moisture, reduction or oxidation and
is extracted from argentite by the mac-Arthur Forest process dissociation into simpler substances.
(leaching process). 38. (b) To remove acidic impurities basic flux is added which
Ag 2S + 4NaCN ® 2Na[Ag ( CN )2 ] + Na 2S is CaCO3.
39. (d) 2CuO + CuS ® 3Cu + SO 2 (Self - reduction)
4Au + 8KCN + 2H 2O + O 2 ® 4K[Au ( CN ) 2 ] + 4KOH
GENERAL PRINCIPLES AND PROCESSES OF ISOLATION OF ELEMENTS 639

40. (b) In the graph of DrG° vs T for formation of oxides, the


1
Cu2O line is almost at the top. So, it is quite easy to reduce 61. (c) C ( s ) + O2( g ) ® CO( g ) ; DS increases. Hence, as
2
oxide ores of copper directly to the metal by heating with
the temperature increases, TDS increases and hence DG
coke. Both the lines of (C, CO) and (C, CO2) are at much
(DH – TDS) decreases. In other words, the slope of the
lower temperature (500 - 600 K).
curve for formation of CO decreases. However, for all other
Cu 2 O + C ¾¾ ® 2Cu + CO oxides, it increases.
41. (d) DG = DH – TDS = –ive
62. (d) The sulphide ore is roasted to oxide before reduction
42. (c) Ellingham diagram normally consists of plots of DfGº
because the DGof of most of the sulphides are greater
Vs T for the formation of oxides of elements.
than those of CS2 and H2S, therefore neither C nor H can
43. (c) It can withstand high temperatures.
reduce metal sulphide to metal. Further, the standard free
44. (c) In blast furnace at about 1270 K, calcium carbonate is
energies of formation of oxide are much less than those of
almost completely decomposed to give CaO which acts as a
SO2. Hence oxidation of metal sulphides to metal oxide is
flux and combines with SiO2 present as impurity (gangue) in
thermodynamically favourable.
the ore to form calcium silicate (fusible slag)
3
CaO(s) (basic flux) + SiO2 (s) (acidic gangue) ¾¾ ® 63. (a) 2Cr(s) + O2(g) ® Cr2O3(s); Df G° = –540 kJ/mol
2
CaSiO3 (s) (slag)
45. (b) These are the substances which can withstand very 3
2Al(s) + O (g) ® Al2O3(s); Df G° = –827 kJ/mol
high temperature without melting or becoming soft. 2 2
46. (d) Blister-copper contains 1 – 2 % impurities. It is By substracting first equation from second:
obtained after bessemerisation of crude copper. 2Al(s) + Cr 2O3(s) ® Al2O3(s) + 2Cr(s)
47. (b) Fe2 O3 + 3CO ® 2Fe + 3CO 2 DrG° = –827 + 540 = –287 kJ/mol
48. (b) Extraction of Zn from ZnS (Zinc blende) is achieved 2
by roasting followed by reduction with carbon. Multiplying this equation by we get:
3
2ZnS + 3O2 ¾¾ ® 2ZnO + 2SO2 4 2 2 4
ZnO + C ¾¾ ® Zn + CO Al(s) + Cr2O3(s) ® Al2O3(s) + Cr(s)
3 3 3 3
49. (b) Aluminothermite process involves reduction of oxides 2
which are not satisfactorily reduced by carbon such as DrG° = × (–287) kJ/mol = –191 kJ/mol
3
Fe2O3, Mn3O4, Cr 2O3, etc. to metals with aluminium. 64. (a) FeO is capable of forming slag with SiO2:
Cr2 O3 + 2Al ® Al 2 O 3 + 2Cr; DH = - ve SiO 2 + FeO ® FeSiO 3
50. (a) Reverberatory furnace is lined with haematite. 65. (c) The iron obtained from blast furnace is pig iron with
51. (b) Pig iron contains 4% carbon and many impurities in 4% carbon and impurities like S, P, Mn etc., in small amount.
smaller amount. 66. (c) Because Na is very reactive and cannot be extracted
52. (d) The obtained copper is called blister copper. by means of the reduction by C, CO etc. So, it is extracted
53. (c) by electrolysis.
54. (a) SO2(g) produced is utilised for manufacturing H2SO4. 67. (c) Strongly electropositive metals cannot be easily
55. (a) Both assertion and reason are correct and reason is reduced by other reduction methods.
the correct explanation of assertion. 68. (c) Fused alumina (Al2O3) is a bad conductor of electricity.
56. (b) The value of DG° is more negative for reaction (B). Therefore, cryolite (Na3AlF6) and fluorspar (CaF2) are
Thus, C will oxidize to CO and ZnO will reduce to Zn. added to purified alumina which not only make alumina a
57. (d) Ellingham diagrams are based on thermodynamic good conductor of electricity but also reduce the melting
concepts. It does not tell anything about the kinetics of point of the mixture to around 1140 K.
the reduction process. 69. (a) 70. (a)
58. (b) For a spontaneous reaction , DGº must be negative 71. (a) SRP of Zn is –0.76 V.
and it can be possible only in this case when x < y 72. (b) The reactions involved in cyanide extraction process are:
59. (a) In the extraction of copper, metal is formed in the Ag 2S + 4NaCN ® 2Na [Ag(CN)2] + Na2S
bessemer converter due to self reduction. (argentite)

Cu 2S + 2Cu 2 O ¾¾
® 6Cu + SO 2 4Na2S + 5O 2 + 2H2O ® 2Na2SO4 + 4NaOH + 2S
60. (b) Ellingham diagram represents plot between DG and Oxiding
agent
T therefore with increase in temperature phase change Gas
® Liquid is not possible. Ellingham diagram does not give 2Na[Ag(CN)2] + Zn ® Na2 [Zn(CN)4] + 2 Ag ¯
any information about kinetics of the reduction reaction. (reducing
agent)
EBD_8350
640 CHEMISTRY

73. (c) Extraction of non-metals are based on oxidation. For 88. (b) During electrolytic refining of copper, electrolyte used
example extraction of chlorine from brine. is acidified solution of copper sulphate.
2Cl–(aq) + 2H2O(l) ® 2OH–(aq) + H2(g) + Cl2(g) 89. (a) Wrought iron is used in making anchors, wires, bolts
74. (b) Reaction at cathode: chains and agricultural implements.
Al3+ + 3e– ® Al ....(i) 90. (a) In this method, a low melting metal like Pb, Sn or Bi
Reaction at anode: can be made to flow on a sloping surface. In this way, it is
C + O2– ® CO + 2e– ....(ii) separated from higher melting impurities.
C + 2O2– ® CO2 + 4e– ....(iii)
By performing 2 × (i) + (ii) + (iii): Exercise - 2
2Al3+ + 2C + 3O2– ® 2Al + CO + CO2
1. (a) Chlorine extraction by electrolysis can be represented
(2 × 12 kg) (2 × 27 kg)
as :-
2 × 27 kg of Al is required = 2 × 12 kg of carbon
2H2O(l) + 2Cl–(aq) ® H2(g) + Cl2(g) + 2OH–(aq)
2 ´ 12
1 kg of Al is required = kg of carbon = 0.5 kg. So, the oxidation of Cl– to Cl2 occurs.
2 ´ 27
1 1
75. (a) To get very pure semiconducting material. 2. (d) Cu 2 O + Cu 2S ¾¾ ® 3Cu + SO 2
76. (a) 2 2
523K 1700K
In this given reaction, copper is reduced by itself hence, it
Ti + 2I2 ¾¾¾® TiI4 ¾¾¾¾
® Ti + 2I 2 is known as auto-reduction.
Volatile Pure metal 3. (a) The most abundant elements are aluminium (8.3% by
Stable compound
weight) and iron (4.2% by weight).
77. (c) Liquation process, Mond’s process and van Arkel
4. (c) In the extraction of copper from its sulphide ore, the
processes are the refining processes that are applied
metal is formed according to the reaction given :
depending upon the nature of the metal under treatment
and nature of the impurities whereas amalgamation process 1 1
Cu 2O + Cu 2S ¾¾ ® 3Cu + SO 2
is used for the extraction of noble metals like gold, silver, 2 2
etc. from native ores. The metal is recovered from the 5. (b) In metallurgy of aluminium, graphite anode is oxidised
amalgam by subjecting it to distillation, where the mercury to CO and CO2 and this process is known as Hall-Heroult
distils over leaving behind the metal. process.
Hg-vapours The electrolytic reactions the process are :
Distilled At cathode : Al3+ (melt) + 3e– ¾¾
Ore + Hg ® Amalgam ® Al(l)
Metal At anode : C(s) +O2– (melt) ¾¾ ® CO (g) + 2e–
78. (b) Zr and Ti are purified by van Arkel method. C(s) + 2O2– (melt) ¾¾ ® CO2 (g) + 4e–
870K
6. (a) Two metals which are generally purified by using
Zr(s) + 2I2 (g) ¾¾¾® ZrI4 (g) electrolytic refining are copper and zinc.
2075K 7. (a) In the process of extraction of Ag and Au involving
ZrI4 (g) ¾¾¾¾¾¾¾
® Zr(s) + 2I 2 (g)
Tugsten filament leaching with cyanide ion, the metal can be recovered by
523K
Ti(s) + 2I2(s) ¾¾¾ displacement of Ag/Au by some other metal in the complex
® TiI4(g)
ion. Reactions involved in the process are as follows :
1700K
TiI4 ¾¾¾¾ ® Ti(s) + 2I2(g) 4 Au (s) + 8CN– (aq) + 2 H2O (aq) + O2 (g) ¾¾ ®
Pure titanium 4 [Au (CN)2]– (aq) + 4 OH– (aq)
79. (c) -
80. (b) Zinc dust is used as a reducing agent in the 2 éë Au ( CN ) 2 ùû ( aq ) + Zn (s ) ¾¾
®
manufacture of dye-stuffs, paints etc. 2-
2Au ( s ) + éë Zn ( CN ) 4 ùû ( aq )
81. (b)
82. (d) Na reacts vigorously with water (exothermic process). 8. (c) Copper matte contains sulphide of copper (I) and
83. (b) During the process of electrolytic refining of copper, iron (II). Cu2S and FeS
Ag and Au are obtained as anode mud due to impurity. 9. (b) The principle behind zone refining is that the
84. (d) The impurities are oxidized and removed with blast of impurities are more soluble in molten state than in solid
air. state of the metal.
85. (b) Cyanide process is for gold (A - s); floatation process 10. (a) Possible reactions occuring at anode are :
1
- pine oil (B - q); Electrolytic reduction - Al (C - r); Zone (i) Cl - ( aq ) ¾¾® Cl2 ( g ) + e - ; E!
cell = 1.36 V
refining -Ge (D - p). 2
86. (d) Metals of high purity are obtained by zone refining (ii) 2H2O (l) ¾¾ ® O2 (g) + 4H+ + 4e– ; E! cell = 1.23 V
e.g., silicon, germanium, boron, gallium, indium.
87. (a) The reaction at anode with lower value of Es is preferred
and therefore water should get oxidised in preference to
GENERAL PRINCIPLES AND PROCESSES OF ISOLATION OF ELEMENTS 641

Cl– (aq). However, Cl2 is produced instead of O2. This 22. (d)
unexpected result is explained on the basis of the fact that 23. (a) Amongst the given ores, copper pyrite (CuFeS2),
water needs greater voltage for oxidation to O2 (over dolomite (MgCO3.CaCO3), malachite [CuCO3.Cu(OH)2], azur-
voltage) than that needed for oxidation of Cl– ions to Cl2. ite [2CuCO3.Cu (OH)2], only copper pyrite contains both
So reaction (i) is taking place at anode. copper and iron.
11. (d) Above point A : Df G°(C,CO) < Df G°(Fe, FeO) 24. (a) Magnetite Fe3O4
Therefore, reduction of FeO by carbon occurs above point Sphalerite ZnS
A. Cryolite Na3AlF6
12. (a) Gibbs free energy change for the formation of CO2 Malachite CuCO3.Cu(OH)2
from CO has lesser value than Gibbs free energy change 25. (c) The iron obtained from blast furnace contains about
for the formation of FeO (below point A). As a result , FeO 4% carbon and smaller amount of impurities like S, P, Si,
will be reduced by CO below point A. Mn. This form of iron is known as pig iron. It can be moulded
13. (a) DG curve for the formation of CO2 and FeO intersect into variety of shapes.
each other at point D thus, net reduction of FeO with CO 26. (b) Wrought iron is purest form of commercial iron.
is zero.
14. (b) On electrolysis of aqueous solution of s-block Exercise - 3
elements, H2 gas is discharged at cathode. 1. (a)
- 1 - 2. (c) When an ore containing gold is treated with KCN
At cathode: H 2 O + e ® H 2 + OH solution. (cyanide process for extraction of gold), a soluble
2
15. (b) Metal nitrates are highly soluble in water and are very complex of gold is formed.
stable for e.g. NaNO3 and KNO3. 3. (c) Zn is extracted from its sulphide ore by roasting
16. (c) 2 Cu2S (s) + 3O2 (g) ® 2Cu2O(s) + 2SO2(g) followed by carbon reduction. Hg, Pb and Cu are extracted
The unchanged Cu2S, mixed with Cu 2O and heated by self reduction from their sulphide ores.
strongly in absence of air 4. (b) ˆˆ† Cu 2+ + SO24-
CuSO 4 ‡ˆˆ
2 Cu2S + 2Cu2O ® 6Cu + SO2
17. (b) In the metallurgy of aluminium, purified Al2O3 is mixed ˆˆ† H + + OH -
H 2O ‡ˆˆ
with Na3AlF6 or CaF2 which lowers the melting point of At cathode : Cu 2 + + 2e - ® Cu
the mix and brings conductivity.
18. (a) Highly electropositive metals like Al, K, Na etc. are At anode : 4OH - ® 2H 2 O + O 2 + 4e -
extracted by the electrolytic reduction. Sulphate ion requires higher voltage, thus, OH– get oxidized
zone refining method is used for obtaining metals of high in preference to sulphate ions.
purity e.g. Ge 5. (d) For basic impurities (CaO, FeO), acidic flux (SiO2) is
Froth flotation process is suitable for sulphide ores. used and for acidic impurities (SiO2), basic flux (CaO) is
Cyanide process is used for the extraction of gold. used.
19. (a) Froth floatation method is mainly applicable for 6. (b) SnCl2 + HgCl2 ¾¾ ® SnCl 4 + Hg
sulphide ores. ( XCl2 ) ( YCl2 ) ( XCl4 ) (Y)
Malachite ore : Cu(OH)2 . CuCO3 D 1
Magnetite ore : Fe3O4 HgO + ¾¾¾¾® Hg + O 2
> 400°C 2
Siderite ore : FeCO3 HgS : Cinnabar
Galena ore : PbS (Sulphide Ore) 7. (b) A metal sulphide is generally more stable and difficult
20. (c) Zn being more reactive than Ag and Au, displaces to reduce than the metal oxide. Therefore, metal sulphide
them. is converted to metal oxide before reduction.
Leaching
4Ag + 8NaCN + 2H2O + O2 ¾¾¾¾® 8. (b)
(I) Mond's process
4Na[Ag(CN)2] + 4NaOH
50°C 200°C
Sodium dicyanoargentate (I) Ni + 4CO ¾¾¾® éë Ni ( CO ) 4 ùû ¾¾¾¾ ® Ni + 4CO ­
Soluble Impure Pure
Volatile
complex
Soluble cyanide compound can be treated with Zn to give
metal by displacement. (II) Removal of lead poisoning
2-
2Na[Ag(CN)2] + Zn ¾¾¾¾¾¾Displacement ˆˆ† éë Pb ( EDTA ) ùû
Pb 2 + + EDTA 4- ‡ˆˆ
®
Na2[Zn(CN)4] + 2Ag¯ (III) Cyanide process for extraction of silver
21. (d) Mg has more – DG° value than alumina. So it will be in 1
the lower part of Ellingham diagram. Metals which have 2Ag + 4NaCN + H 2 O + O 2 ‡ˆˆ ˆˆ†
2
more – DG° value can reduce those metal oxides which
2Na éë Ag ( CN ) 2 ùû + 2NaOH
have less – DG° value.
EBD_8350
642 CHEMISTRY

(IV) A depressant NaCN depresses the floating property aluminate (Na[Al(OH)4 ]) is formed and impurity (SiO2)
of ZnS so that PbS (galena) passes into the froth present in bauxite dissolves by forming sodium silicate
when air is blown in. This is because, NaCN reacts (Na2SiO3)
with ZnS, forming a zinc complex Na2 [Zn(CN)4]. Al2O3(s) + 2NaOH(aq) + 3H3O (l) ¾®
9. (b) 2Na [Al (OH)4] (aq)
10. (c) In blast furnace, following are the main reactions: Cast iron is obtained from pig iron.
(i) Reduction of Fe2O3 by CO: Blister of copper is due to evolution of SO2.
Fe2O3 + CO ® 2FeO + CO2 Hall Heroult process is used for the production of
2FeO + 2CO ® 2Fe + 2CO2 aluminium.
Fe2O3 + 3CO ® 2Fe + 3CO2 14. (a) Calcination involves the conversion of metal
carbonates or hydrated oxides into metal oxides. ZnO and
(iv) Slag formation:
D MgO are oxides, therefore, does not require calcination.
CaCO3 ¾¾ ® CaO + CO2­ +3
CaO + SiO2 ® CaSiO3 15. (d) (A) Cr2O3 + 2Al ¾¾
® Al 2O3 + 2Cr
(flux) (impurity) (slag) (B) Au + 2CN - + H 2O + 1 O 2
11. (a) Ti is refined by van Arkel method. Silver is leached 2
® [Au(CN) 2 ]- + 2OH -
¾¾
by dilute solution of NaCN. Zincite is oxide ore of zinc i.e.
ZnO. Aniline is a froth stabilizer. 2[Au(CN) 2 ] + Zn ® [Zn(CN) 4 ]2 - + 2Au
-

12. (b) Froth floatation was discovered by washer women. (C) Statement is true
O2
It is a method of concentration of ores. (D) Ag 2S + 4NaCN ¾¾¾ ® 2Na[Ag(CN) 2 ] + Na 2S
13. (a) During metallurgy of aluminium, when bauxite 2Na[Ag(CN) 2 ] + Zn ¾¾
® Na 2 [Zn(CN) 4 ] + 2Ag
(powdered ore) is treated with NaOH (conc), sodium
21 p-Block Elements (Group 15, 16, 17 and 18)
21. (d) Structures of given oxyacids are following
Exercise - 1
O O
1. (d) Ionic radii increases down the group
2. (b) In case of nitrogen, d-orbitals are not available. P P
3. (a) Collectively these elements are called pnicogens and (a) HO O OH
their compound pniconides. OH OH
4. (d) Metallic character increases down the group, Bi is H4P2O7
most metallic O
5. (a) –3, +3, +5 O
6. (d) N2 molecule contains triple bond between N atoms P
having very high dissociation energy (946 kJ mol–1) due to (b) (c) HO ¾ P ¾® O
HO H
which it is relatively inactive. HPO3
H
7. (a) Order of dipole moment H3PO2
NH3 > PH3 > AsH3 > SbH3 O O
(Based upon electronegativity)
8. (c) As the size of central atom increases, the lone pair of (d) P
electrons occupies a larger volume. In other words, electron P
HO OH H OH
density on the central atom decreases and consequently OH
its tendency to donate a pair of electrons decreases along OH
H 3PO 4 H3PO3
with basic character from NH3 to BiH3.
9. (c) By Haber’s process The H–atom of the –OH group is ionisable whereas
10. (d) NO2 is reddish brown coloured gas. Rest of the oxides H–atom which is directly linked to P–atom is non-ionisable.
are colourless. Thus, H3PO3 is dibasic acid.
11. (c) In N2O (nitrous oxide) two N atoms are covalently 22. (b) PH5 does not exist because d-orbital of ‘P’ interacts
bonded through triple bond with s-orbital of H. Bond formed is not stable and not
[ N º N ¾¾® O ] energetically favorable. It depends on size and orientation
of interaction.
12. (b) FeSO 4 + NO ® FeSO 4 .NO
23. (a) 4HNO3 + P4O10 ¾® 4HPO3 + 2N2O5
13. (a) N2O is used as an anaesthetic
24. (a) We know that empirical formula of hypophosphorus
14. (c) 2NO + O 2 ® 2 NO 2 (brown) acid is H3PO2. In this only one ionisable hydrogen atom is
Pt . gauge present i.e. it is monobasic. Therefore, option (a) is correct
15. (a) 4NH 3 + 5O 2 ¾¾¾¾® 4NO + 6H 2 O structural formula of it.
Catalyst
16. (b) The slow decomposition of HNO3 is represented by 25. (b) 4NH3(g) + 5O2(g) ¾¾¾¾ ® 4NO(g) + 6H2O(g)
the eqn. ˆˆ† 2NO (g)
2NO(g) + O2(g) ‡ˆˆ 2
4HNO3 ® 4NO2 + 2H2O + O2
3NO2(g) + H2O (l) ¾® 2HNO3(aq) + NO(g)
(yellow-brown)
17. (c) Ca3P2 + 6H2O ® 3Ca(OH)2 + 2PH3 ; i.e. 2 moles of 26. (b) NO2 and N2O4 has + 4 oxidation state for nitrogen.
phosphine are produced from one mole of calcium 27. (d) Bismuth forms metallic bonds in elemental state.
phosphide. 28. (a) Nitrogen due to small size is able to show pp-pp lateral
overlap forming N º N, rest elements due to bigger size are
D
18. (d)P2O5 + 3H 2O ¾¾® 2H3 PO4 not able to show pp-pp lateral overlap.
19. (b) Orthophosphoric acid, H 3PO 4 contains three 29. (c) Catenation tendency is higher in phosphorus when
compared with other elements of same group.
P – OH bonds and is therefore, tribasic.
30. (b) Due to inert pair effect, ns2 electrons do not have a
O tendency to form bond.
||
Heat
P 31. (d) NH 4 Cl + NaNO 2 ¾¾¾¾
® NH 4 NO 2
| OH - NaCl
|

OH Heat
¾¾¾® N 2 + 2 H 2 O.
orthophosphoric acid 32. (a) Oxide in which central atom has higher charge and
1 more electronegativity, is more acidic, i.e.
20. (c) Hybridisation in PCl5 = (5 + 5 + 0 - 0) = 5 ; sp3d N2O5 > N2O4 > P2O5 > As2O3.
2
EBD_8350
644 CHEMISTRY

33. (b) Liquid ammonia has high vapour pressure which is


lowered down by cooling, otherwise the liquid will bump.
O
34. (a) Only nitrates of heavy metals and lithium decompose
on heating to produce NO2. 49. (b) H P H Hypophosphorous acid (H3PO2) is a
35. (d) N2O3, N2O4 and N2O5 are acidic oxides. Only N2O is
neutral oxide. O
36. (b) NH4NO3 ¾® N2O + 2H2O H
37. (d) For nitrogen, only NF3 is known to be stable.
38. (c) BiH3 is the strongest reducing agent while NH3 is the
monobasic acid. i.e., it has only one ionisable hydrogen
weakest reducing agent.
atom or one OH is present.
O
O 50. (c) H5P5O15 (HPO3)5. It is metaphosphoric acid which is
=

39. (d) NºN ® O N–N = a cyclic phosphate.


=

(i) (ii) O 51. (b) H4P2O5 is pyrophosphorous acid it contains P–O–P


O O bond
O O
N–O–N ®
= ®

= ®

N–N ® 52. (a) Pyrophosphorous acid (H4P2O5) is a dibasic acid as


=

O it contains two P—OH bonds.


=

O O O
(iii) (iv) O O
40. (a) Both white and red phosphorus are not soluble in ½½ ½½
CS2, only white phosphorus is soluble in CS2. H—P—O—P—H
41. (a) Except (a) all other properties are shown by white ½ ½
OH OH
phosphorous.
53. (b) Formula of cyclotrimetaphosphoric acid is (HPO3)3
42. (b) White phosphorus on heating readily catches fire in
air to give dense white fumes of P4O10. Oxidation state of ‘P’ is 3(+ 1 + x + 3 (– 2)) = 0
x + (– 6) + 1 = 0 Þ x = + 5
P4 + 5O2 ¾¾
® P4O10. 54. (a) O O
600°C
43. (d) 2H 3 PO 4 ¾¾¾
¾® 2HPO 3 HO — P — P — OH
- 2H 2O
44. (b) PCl3 + H2O ¾¾ ® POCl3 + 2HCl HO OH
POCl3 + 3H2O ¾¾ ® H3PO4 + 3HCl Hypophosphoric acid
45. (c) Structure of hypophosphorous acid
H OH OH
|
H -O- P ® O
| O P O P O
H
Two H-atoms are attached to P atom. OH OH
46. (c) In cyclic metaphosporic acid, number of P–O–P Pyrophosphoric acid
bonds is three.
O OH HO O OH
P P P
O O O O O
O O
P P Metaphosphoric acid
O OH
HO O
47. (d) H4P2O5 : 4 + 2x + 5(–2) = 0 Þ x = + 3
H4P2O6 : 4 + 2x + 6(–2) = 0 Þ x = + 4 HO P OH
H4P2O7 : 4 + 2x + 7(–2) = 0 Þ x = +5
48. (b) O OH
Orthophosphoric acid
bridging P bridging
55. (b) (iii) White phosphorus consists of discrete
O tetrahedral P4 molecules. It is more reactive than the other
O solid phases because of angular strain in the molecule
O
where the angles are only 60°.
P O (iv) In the solid state, PCl5 exists as an ionic solid [PCl4]+
[PCl6]– in which the cation, [PCl4]+ is tetrahedral and the
O P
O anion [PCl6]– is octahedral.
O P O 56. (b)
bridging
57. (b) 2NO + N2O4 ¾¾¾ 250K
® 2N O
2 3
O bridging
oxygen
THE p-BLOCK ELEMENTS (GROUP 15, 16, 17 AND 18) 645

1100°C 67. (a) (A)P4 + 2Ca(OH)2 + 4H2O ¾® 2 Ca(H2PO2)2 + 2H2


58. (a) (A) CaC2 + N2 ¾¾¾¾ ® CaCN2 + C (white P) (hot) (calcium hypophosphite)
(B) NH3 is oxidized to NO, Which is used to prepare The salt is then treated with a strong non-oxidizing acid to
nitric acid. give hypophosphorous acid.
(C) Steel metal is dipped in a bath of nitric acid/
H2PO2– + H+ ¾® H3PO2
hydrofluoric acid. It strips the surface layer of steel,
removing impurities and free iron molecules; leaving an (B) P2O3 + 3H2O ¾® 2H3PO3
even, satin like finish. (C) P4O10 + 6H2O ¾® 4H3PO3
59. (a) Both assertion and reason are true and reason is the (D) P4 + 8H2O2 + 4NaOH ¾® 2Na2H2P2O6 + 8H2O
correct explanation of assertion. (red P) (dil.) (dil.)

M + HNO3 ¾¾
® MNO3 + H Na2H2P2O6 can be converted to H4P2O6 by passing it in
(metal) (conc.) (metal nitrate) (nascent hydrogen) an ion exchange column.
68. (c) 3CuSO4 + 2PH3 ® Cu3P2 + 3H2SO4
2HNO3 + 2H ¾¾
® 2NO2 + 2H 2O 3HgCl2 + 2PH3 ® Hg3P2 + 6HCl
(nascent hydrogen) 69. (b) H2O has exceptionally high b.p. due to hydrogen
60. (c) Nitrogen form N2 (i.e. NºN) but phosphorus form P4, bonding.
because in P2, pp — pp bonding is present which is a 70. (a) Acidity increases as we go down the group.
weaker bonding. 71. (a) H2O (due to intermolecular H - bonding)
61. (d) 4 Zn + 10 HNO3 (dil.) ® 4 Zn(NO3)2 + 5H2O + N2O 72. (b) Oxygen being more electronegative
Zn + 4 HNO3 (conc.) ® Zn(NO3)2 + 2H2O + 2NO2 73. (c) In KMnO4 manganese is already present in its highest
62. (d) PH3 is not obtained when metaphosphoric acid is possible oxidation state i.e. +7. So no further oxidation is
heated. possible.
(a) 2P4 + 3Ba (OH)2 + 6H2O ¾® D
(white P) 74. (b) 2KMnO 4 ¾¾® K 2 MnO4 + 4MnO 2 + O 2
2PH3 + 3 Ba (H2PO2)2 75. (a) Mn2O7 is an acidic oxide. BaO and Na2O are basic
(phosphine) (barium oxides while N2O is a neutral oxide.
hypophosphite) 76. (c) It is present as S8 molecules.
(b) 4H3 PO3 ¾® 3H3 PO4 + PH3
V2O5
D 77. (a) 2SO2(g) + O2(g) ¾¾¾® 2SO3(g)
(c) Ca(H2PO2)2 ¾¾ ® 78. (b) Conc. H2SO4 is a strong dehydrating agent due to
(calcium hypophosphite)
which carbohydrates becomes charred on reaction with
PH3 + CaHPO4
conc. H2SO4 acid.
(phosphine) (calcium hydrogen phosphate)
79. (a) In SO3, sp2 hybridisation is for Sulphur atom.
63. (a) The combustibility of PH3 is due to presence of P2H4.
80. (c) Air is liquified by making use of the Joule-Thompson
The pure PH3 is not combustible.
effect (cooling by expansion of the gas). Water vapour
64. (c) At higher temperatures, dinitrogen combines with
metals to form ionic nitrides. and CO2 are removed by solidification. The remaining major
65. (b) VSEPR theory breaks down for heavier elements. constituents of liquid air i.e., liquid oxygen and liquid
There is no significant s–p mixing for PH3 onwards down nitrogen are separated by means of fractional distillation
the group. For PH3, there is very little contribution from (b.p. of O2 = –183°C : b. P. of N2 = – 195.8°C)
the P 3s-orbit. We can assume the P–H bonds are 81. (d) S-H bond is weaker than, O–H bond. Hence, H2S will
predominantly P3p– H1s bonds. Thus, the geometry around furnish more H+ ions
P is determined mostly by the orientation of the p-orbitals. 82. (a) SnO2 is an amphoteric oxide because it reacts with
H acids as well as with bases to form corresponding salts.
6Py SnO2 + 2H2SO4(conc) ¾¾ ® Sn(SO4)2 + 2H2O
90°
SnO2 + 2NaOH ¾¾ ® Na2SnO3 + H2O
e.g. in BiH3, the H–Bi–H Bi H
6Px 83. (d) All hexafluorides of group 16 elements are gaseous in
6Pz nature.
angle » 90° H
84. (b) Oxygen can be prepared by heating oxides of Hg, Pb,
And the lone pair of Bi remains in 6s orbital. The Ag, Mn and Ba.
s-orbital is closer to the nucleus, which means that the
D
negative charge of the electrons is better stabilised. 2HgO ¾¾® 2Hg + O2
66. (b) [Cu(H2O)6]SO4 + 2NH4OH ¾® [Cu(H2O)4(OH)2] 85. (d) 2Ag2O (s) ® 4Ag (s) + O2 (g)
(pale blue) 2Pb3O4 (s) ® 6PbO (s) + O2 (g)
excess NH 4OH 2PbO2 (s) ® 2PbO (s) + O2 (g)
¾¾¾¾¾¾® [Cu(NH3)4 (H2O)2]2+ 86. (c) S2 is paramagnetic. It contains two unpaired electrons
(deep blue) in the antibonding p* orbital.
EBD_8350
646 CHEMISTRY

O O 103. (d) Due to hydrogen bonding, HF is a liquid.


104. (d) In gaseous state, HCl is covalent in nature, while in
=

=
87. (b) O = S–O–O–S=O
aqueous solution it ionises to give H+ and C l ions.
OH OH
0 -1 +5
Peroxodisulphuric acid 105. (b) 6NaOH + 3Cl2 ¾¾
® 5NaCl + NaCl O3 + 3H 2O
(H2S2O8)
88. (b) The key step in the manufacture of H2SO4 is catalytic 106. (b) Except ionisation potential, other factors are true to
oxidation of SO2 with O2 to give SO3 in presence of V2O5. explain the oxidising (strong) behaviour of F2 .
89. (d) Caro’s acid is H 2SO 5 which contains one 107. (b) On moving from top to bottom of halogen group the
S – O – O – H peroxy linkage. It is also known as bond dissociation energy of hydrogen halides decreases
permonosulphuric acids. and so the heat of formation of halogen acids also
O decreases.
|| 108. (c) Volatile character; HCl > HBr >HI > HF
H – O – O – S – OH 109. (d) Bleaching action of chlorine is due to oxidation in
|| presence of moisture.
O
Caro's acid Cl 2 + H 2O ® HCl + HClO
HClO ® HCl + O
O O
|| || Colouring matter + | O | ® Colourless matter
90. (d) S S
| 110. (b) 3Cl2 + 6NaOH ® 5NaCl + NaClO3 + 3H2O
||

||

O– O (hot & conc.)


|

O –
O O
111. (d) MnO2 or KMnO4 with conc HCl give Cl2.
91. (c) HO.SO 2OH + 2PCl5 ® ClSO2Cl + 2POCl3 + 2HCl
+7 +2
Sulphuryl chloride 112. (d) 2 KMnO 4 + 16HCl ® 2 MnCl 2 + 2 KCl + 8H 2 O + 5Cl 2
92. (c) Å Å
O.S of Mn changes from +7 to +2 hence reduction occurs
O O
¬¾® and Cl2 is formed.
O O– –O O 113. (d) HCl acid at 25º C is a gas and polar in nature.
Ozone is diamagnetic in nature (due to presence of paired 114. (c) The bond energy of interhalogen compounds is less
electron) and both the O – O bond length are equal. It has
than the bond energy of halogens.
a bent structure.
115. (b) Physical state of iodine is different from other
93. (c) Bond angle of H2S (92°) < H2O (104°31). As the
electronegativity of the central atom decreases, bond angle halogens as iodine is solid, bromine is a liquid whereas
decreases. In the present case, S is less electronegative fluorine and chlorine are gases.
than oxygen. Thus, bond pairs in H2S are more away from 116. (d) All halogens (leaving F-F) have stronger bond then
the central atom than in H2O and thus repulsive forces that in interhalogens.
between bond pairs are smaller, producing smaller bond 117. (c) From the given options we find option (i) is correct.
angle. The oxidising power of halogens follow the order
94. (a) F2 > Cl2 > Br2 > I2. Option (ii) is incorrect because it is not
95. (d) Ionisation potential decreases down the group. the correct order of electron gain enthalpy of halogens.
96. (a) The correct order is Cl2 > F2 > Br 2 > I2. The low value of F2
97. (c) The electron gain enthalpy order for halogens is than Cl2 is due to its small size.
Cl > F > Br > I
Option (iii) is incorrect. The correct order of bond
Due to small size of fluorine, the extra electron to be added
dissociation energies of halogens is
feels more electron-electron repulsion. Therefore, fluorine
has less value for electron affinity than chlorine. Cl2 > Br2 > F2 > I2.
98. (d) Bond energy decreases from Cl to I. Option (iv) is correct. It is the correct order of
99. (a) Reactivity follows the order F > Cl > Br > I electronegativity values of halogens.
100. (d) Oxidising power decreases down the group. 118. (c) The H–X bond strength decreases from HF to HI. i.e.
101. (b) It is the most electronegative element. HF > HCl > HBr > HI. Thus, HF is most stable while HI is
0 +1 -1 least stable. The decreasing stability of the hydrogen halide
102. (b) H 2O + Br2 ¾¾
® HOBr + HBr is also reflected in the values of dissociation energy of the
Thus, here oxidation number of Br increases from 0 to +1 H–X bond.
and also decreases from 0 to –1. Thus, it is oxidised as well H-F H - Cl H - Br H-I
as reduced. 135kcal mol-1 103kcal mol-1 87 kcal mol-1 71kcal mol-1
THE p-BLOCK ELEMENTS (GROUP 15, 16, 17 AND 18) 647

119. (c) MI > MBr > MCl > MF. As the size of the anion
decreases covalent character also decreases.
+1 +5 -1 XeO3 : Xe
120. (d) NaClO ¾¾ ® NaClO3 + 2NaCl All statements are
correct as evident from the reaction
O O
121. (b) 2KMnO4 + 16HCl ® 2KCl + 2MnCl2 + 8H2O + 5Cl2 O
122. (d) 3Ca(OCl)2 ® Ca(ClO3)2 + 2CaCl2 1lp
It is autooxidation and disproportionation. Hence XeF2 has maximum no. of lone pairs of electrons.
123. (d) As the oxidation state of the central halogen atom 136. (c) Argon is used in high temperature welding and other
increases, the halogen-oxygen bond becomes more and operations which require a non-oxidising atmosphere and
more covalent. As a result, the thermal stability of the the absence of nitrogen.
oxoacid increases. Thus, HClO4 is most stable to heat, 137. (b) Neon gives a distinct reddish glow when used in either
whereas HClO is least stable to heat. low-voltage neon glow lamps or in high voltage discharge
124. (d) Interhalogen compounds are not highly volatile. tube.
125. (a) Metal halides with higher oxidation state are more 138. (a) Breathing mixture is (O2 + He) due to low solubility of
covalent than the one in lower oxidation state. He in blood.
126. (b) Ionization potential of inert gases is highest in 139. (a) Mixture of (He + O2) is used for asthma patient.
periodic table due to stable electronic configuration. 140. (a) The smaller the size the least is the polarisability.
127. (a) Ionisation energy decreases as we move away from 141. (c) The differentiating electron enter in s subshell in case
nucleus due to less electrostatic attraction between of He, hence it is s- block element. Its electronic
electrons and nucleus configuration l s2 makes it inert in nature hence, it is placed
128. (c) Ar is the most abundant in atmosphere. with inert gases.
129. (d) Due to weak van der Waal’s forces, He has lowest 142. (b) At 2.2 K, liquid helium can flow.
boiling point. 143. (b) The shape of XeO3 is Trigonal Pyramidal.
130. (d) He was observed in the spectrum of the sun.
131. (d) Xe forms maximum compounds hence, it is most
Xe
reactive. (Trigonal Pyramidal Structure)
O O
132. (b) Noble gases exhibit low chemical activity O

133. (d) Xe has 8 e– in its valence shell. 144. (c) XeF6 + 3H2O ¾¾ ® XeO3 + 6HF
134. (b) In XeOF4, Xenon is sp3d 2 hybridised and has one \ Complete hydrolysis of XeF6 gives XeO3 (an explosive).
lone pair of electrons. 145. (d) XeO2F2 has trigonal bipyramidal geometry, but due
F to presence of lone pair of electrons on equatorial position,
its actual shape is see-saw.
F
O
135. (c) XeF2 : Xe Xe
O
F
F
146. (b) XeF4 + H 2O ® 2HF + XeOF2
3lp 147. (b) XeOF4 is square pyramidal.
148. (b) XeF4 is planar.
F F 149. (a) In XeO3 the hybridisation is sp3.
XeF4 : 150. (a) As size increases, van der Waal's forces of attraction
Xe
between noble gas atoms also increases. Consequently,
F F ease of their liquefaction increases.
151. (c) 152. (d) 153. (a)
2lp 154. (d) Electron gain enthalpy for noble gases is positive
F
and it becomes less positive with increase in size of atom.
F
Value of electron gain enthalpy
He – 48 kJ mol–1, Ne – 116 kJ mol–1
XeF6 : F Ar, Kr – 96 kJ mol–1, Xe – 77 kJ mol–1
Xe F
Hence, Ne has highest positive electron gain enthalpy.
155. (c) Solubility increases from He to Rn.
F F
EBD_8350
648 CHEMISTRY

156. (d) The products of the concerned reaction react each In second excited state, Xe can form XeF4
other forming back the reactants.
Xe =
XeF6 + 3H 2 O ¾¾ ® XeO3 + 6HF .
157. (a) No compound of He as yet been reported In third excited state, Xe can form XeF6
158. (a) No compound of Ar as yet been reported with F2
159. (c) The most important source of Kr is Earth's Xe =
atmosphere while radon is obtained as a byproduct of Hence, xenon can combine with only even number of
the decay of radioactive elements. fluorine atoms.
160. (b) The hybridization of XeO3F2 is sp3d and its structure
is trigonal bipyramidal in which oxygen atoms are situated Exercise -2
on the plane and the fluoride atoms are on the top and
bottom. F O
O
P
O Xe O O
1. (16) O
O
O P P O
F O
161. (b)
O P
162. (c) For statement (iii) preparation of XeF2 requires Xe in
excess amount O
O
673K,1 bar
Xe ( g ) + F2( g ) ¾¾¾¾¾® XeF2( s ) ..
P
2. (3)
( excess )
For statements (iv) P P
..

..
2XeF2(s) + 2H2O(l) ¾¾® 2Xe(g) + 4HF(aq) + O2(g)
..P
6XeF4 + 12H2O ¾¾ ® 4Xe + XeO + 24HF + 3O
3 2 P4 (White phosporus)
XeF6 + 3H2O ¾¾ ® XeO3 + 6HF Q Each P-atom forms 3 bonds. Thus it has 3 bond pairs of
163. (c) 2XeF6 + SiO 2 ® SiF4 + 2XeOF4 electrons.
164. (c) Xenon undergo sp3 hybridization. 3. (3) In cyclic metaphosporic acid number of P–O–P bonds
is three.
(ground
O OH
state)
5s 5p 5d P
O O
(third excited O O
P O P
state)
5s 5d HO OH
5p
In the fourth excited state, xenon atom has 8 unpaired 4. (10) F – O – F Number of bond pairs = 2, Number of lone
electrons pairs = 8.
5. (3) SO3 exists in 3 solid allotropic forms.
6. (0) S3O9 (also called a- sulphur trioxide) is an ice like
5s 5p 5d solid with the following molecular structure.
One s and three p orbital undergo sp3 hybridization. Four O O
sp3 hybrid orbitals form four s bonds with oxygen atoms. S
They are ssp3 – p. Four pp– dp bonds are also formed with O O
oxygen atoms by the unpaired electrons. O O
S S
O O O
165. (b) Xe = [Kr]4d105s2
5p There is no S–S bond is S3O9

O O
P
G.S.
5d O O
7. (3) O O
In first excited state, Xe can form XeF2 P P
– –
O O O
Xe = No. of P–O–P bonds = 3. It is the most stable among all
5p 5d the cyclic metaphosphates.
THE p-BLOCK ELEMENTS (GROUP 15, 16, 17 AND 18) 649

8. (6) There are 6 P–O–P bond in P4O10. NO3- = 7 + 8 × 3 + 1 = 32


9. (3) Number of millimole of hypo = 0.25 × 48 Hence, CO32 - and NO3- are isoelectronic. Also, both have
= 2 × millimole of Cl2 same type of hybridization (sp2) and have trigonal planar
structure. Hence, they are also isostructural.
0.25 ´ 48
\ Number of millimole of Cl2 = =6 9. (d) On moving down the group, size of central atom
2 increases thus, bond length of E—H bond increases and
millimole of Cl2 = millimole of CaOCl2 bond dissociation energy decreases. Hence, reducing
Molarity of bleaching solution nature increases in the order : NH3 < PH3 < AsH3 < SbH3
10. (c) White phosphorus on reaction with NaOH solution
Millimoles of CaOCl2 in the presence of inert atmosphere of CO2 produces
6
= = = 0.24 phosphine gas which is less basic than NH3.
Vol.(in mL)of CaOCl2 25
P4 + 3NaOH + 3H 2 O –® PH3 + 3 NaH 2 PO 2
10. (3) Total cationic charge = Total anionic charge (Sodium hypophosphite)
2n + 6 + 24 = 36
11. (c) Structure of H3PO4 is
n=3
O
Exercise -3 ||
P
1. (a) On moving down the group, size of halogen atom
HO OH
increases hence, the H–X bond length increases. As a OH
result, bond dissociation enthalpy decreases. H3PO4 has 3 – OH groups i.e., three ionisable H-atoms
Hence, the correct order of bond enthalpy is : and hence, it forms three series of salts: NaH2PO4,
H – F > H – Cl > H – Br > H – I. Na2HPO4 and Na3PO4
Pt/Rh gauge catalyst
12. (c) The acids which contain P–H bond have strong
2. (a) 4NH3 (g) + 5O2 (g) ¾¾¾¾¾¾¾¾
® reducing properties. Thus, H3PO2 is a strong reducing
500K, 9 bar
agent due to the presence of two P — H bonds and one —
4NO(g) + 6H2O(l) OH group
\ Two moles of NH3 will produce 2 moles of NO on
O
catalytic oxidation. ||
3. (c) Let oxidation state of P in NaH2PO2 is x. P H
1 + 2 × 1 + x + 2 × (– 2) = 0
1+2 +x–4 =0 H OH
+ x–1=0 Hypophosphorus acid
x= +1 13. (b) On heating, lead nitrate produces brown coloured
4. (c) Greater the SRP value, higher will be the oxidising nitrogen dioxide (NO2) and lead (II) oxide.
D
power. Hence, the correct order of oxidising power is: 2Pb ( NO3 )2 ¾¾® 4NO2 + 2PbO + O 2
BrO4- > IO-4 > ClO4-
5. (b) Isoelectronic pair have same number of electrons 14. (a) Nitrogen does not show allotropy due to its small
size and high electronegativity. The N–N bond is weak
Total number of electrons for the given compounds :
due to high inter–electronic repulsions among non-
BrO2– BrF2+ bonding electrons due to the small bond distance. Hence,
35 + 2 × 8 + 1 = 52 35 + 9 × 2 – 1 = 52 it does not show allotropy.
15. (c) Maximum covalency of nitrogen is 4 in which one
ICl2 ClO2 BrF electron is from s-orbital and 3 electrons are from p-orbitals.
53 + 2 × 17 = 87 17 + 16 = 33 35 + 9 = 44 Hence, total four electrons are available for bonding.
CN – O3 16. (a) When freshly prepared solution of FeSO4 is added to
6 + 7 + 1 = 14 8 × 3 = 24 an aqueous solution containing NO3- ion, it leads to the
formation of a brown coloured complex. This is known as
6. (c) Hydrogen iodide (HI) is stronger reducing agent than brown ring test for nitrate ion.
H2SO4. Hence, it reduces H2SO4 to SO2 and itself oxidises
to I2. NO3- + 3Fe2+ + 4H+ –® NO + 3Fe3+ + 2H2O
H2SO4 + 2HI –® SO2 + I2 + 2H2O 2+ 2+
éFe ( H2 O) ù + NO –® éëFe ( H 2O )5 ( NO ) ùû + H2 O
(Violet ë 6û
colour) Brown ring
7. (c) Among carbon, nitrogen, phosphorus and boron only 17. (b) The only well characterised compound having + 5
phosphorus has vacant d-orbital hence, only phosphorus oxidation state of Bi is BiF5. It is due to smaller size and
has the ability to form pp – dp bonding.
high electronegativity of fluorine.
8. (a) Both CO32 - and NO3- have same number of electrons. 18. (a) On heating ammonium dichromate and barium azide
both produces N2 gas separately.
CO32 - = 6 + 8 × 3 +2 = 32
EBD_8350
650 CHEMISTRY

( NH 4 )2Cr2 O 7 ¾¾
D
® N 2 + 4H 2 O + Cr2 O3 25. (c) O O H
P
Ba ( N3 )2 –® Ba + 3N2
O O O
O
19. (c) SF4 has sea-saw shape as shown below: P P
H O O O H
.. i.e, 3 double and 12 single bonds.
26. (a) The single N — N bond is weaker than the single
F S F P — P bond. This is why phosphorus show allotropy but
nitrogen does not.
F F 27. (a) The weakening of M—H bond with increase in size of
M (where M = S, Se, Te) explains the acid character of
It has trigonal bipyramidal geometry having sp 3 d hydrides. Since, on moving down the group atomic size
hybridisation. increases hence bond length increases and hence, removal
20. (c) Carbon on oxidation with H2SO4 produces two types tendency of H also increases.
of oxides CO2 and SO2. 28. (c) Acidic strength increases as the oxidation number of
C + 2H2SO4 (conc) –® CO2 + 2SO2 + 2H2O central atom increases.
21. (a) MnO2 reacts with HCl to produce greenish yellow Hence, acidic strength order is
coloured gas of Cl2. (+7) (+5) (+3) (+1)
HClO4 > HClO3 > HClO2 > HClO
MnO2 + 4HCl –® MnCl2 + 2H 2O + Cl 2 29. (a) The acids which contain P-H bond have strong
( Black ) (greenish
reducing properties. Thus H3PO2 acid is good reducing
yellow gas)
agent as it contains two P–H bonds and reduces, for
Cl2 on further treatment with NH3 produces NCl3. example, AgNO3 to metallic silver.
-3 +3 4 AgNO3 + 2sH2O + H3PO2 —®
N H3 + 3Cl2 –® NCl3 + 3HCl 4Ag + 4HNO3 + H3PO4
Hence, NH3 (– 3) changes to NCl3 (+ 3). 30. (c) SO2 is widely used in food and drinks industries for
its property as a preservative and antioxidant while NO2 is
22. (c) Bertlett had taken O+2 Pt F6- as a base compound not used as food preservative.
because O2 and Xe both have almost same ionisation 31. (d) OF2; among the following O and F, F is more
enthalpy. electronegative than oxygen.
23. (d) In solid state PCl5 exists as an ionic solid with the So OF2 cannot be called oxide because in that case O is in
cation [PCl 4]+ (tetrahedral) and the anion [PCl 6] – –2 oxidation state which is not possible, so OF2 is called
oxygen difluoride.
(octahedral).
32. (b) The H-bonding is present in HF due to high
Cl + electronegativity of fluorine atom while H-bonding is not
Cl
Cl Cl present in HI, HBr and HCl.
P P 33. (a) For isoelectronic species, size of anion increases as
Cl Cl Cl Cl negative charge increases. Thus, the correct order is
Cl Cl
[PCl6] –
[PCl4]+ N 3- > O 2 - > F -
(1.71) (1.40) (1.36)
octahedral tetrahedral
34. (b) ICl
24. (b) When H2S gas is passed through an aqueous solution
Order of reactivity of halogens
of copper sulphate acidified with dil. HCl, a black ppt. of
CuS is obtained. Cl2 > Br2 > I2
dil. HCl But the interhalogen compounds are generally more
CuSO 4 + H 2S ¾¾¾¾
® CuS + H 2SO 4
black ppt reactive than halogens (except F2), since the bond between
On boiling CuS with dil. HNO3, it forms a blue coloured two dissimilar electronegative elements is weaker than the
solution and the following reactions occur bond between two similar atoms i.e, X – X
3CuS + 8HNO3 –® 3Cu(NO3)2 + 2NO + 3S + 4H2O 35. (b) Xe. As we move down the group, the melting and
boiling points show a regular increase due to corresponding
2+
+
Cu 2aq + 4NH3 ( aq ) ® éë Cu ( NH3 ) 4 ùû increase in the magnitude of their van der Waal forces of
( )
(Deep blue solution) attraction as the size of the atom increases.
THE p-BLOCK ELEMENTS (GROUP 15, 16, 17 AND 18) 651

36. (b) Phosphinic acid as shown in structure below has one 42. (a) XX' ® Linear (e.g. ClF, BrF)
P—OH bond thus, it is monobasic or monoprotic. XX3' ® T-Shape (e.g. ClF3, BrF3)
O XX5' ® Square pyramidal (e.g. BrF5 , IF5)
XX7' ® Pentagonal bipyramidal (e.g. IF7)
P 43. (c) Cl2 + NaOH ® NaCl + NaClO + H2O
H OH (Monoprotic)
[cold and dilute]
H
+5 +2 0 -3
Phosphonic acid as shown in structure has two P–OH 44. (a) HNO3 , NO, N2 , NH4Cl
bonds thus, it is dibasic or diprotic.
45. (N) (a) All form monobasic oxyacids e.g. HOF, HOCl,
O
HOBr and HOI. But HOF is unstable at room
P temperature 2HOF ® 2HF + O2
(Diprotic acid)
H OH (b) All halogens are good oxidizing agents.
OH (c) Electron gain enthalpy order: Cl > F > Br > I
37. (a) CaC2 + N2 ® CaCN2 + C (d) Fluorine is the most electronegative atom, thus, it
38. (b) F – F bond dissociation enthalpy is smaller then shows only –1 oxidation state. The oxidation states of
Cl – Cl and even smaller than Br – Br. This is because F +1 O -1
atom is very small and hence, the three lone pairs of elements in HOF are H - O - F
electrons on each F atom repel the bond pair holding the All other halogens can show odd positive oxidation
F-atoms in F2 molecules.
number i.e. +1, +3, +5 and +7.
The increasing order of bond dissociation enthalphy is
D
I2 < F2 < Br2 < Cl2 46. (d) (b) (NH 4 )2 Cr2 O7 ¾¾ ® Cr2 O3 + N 2 + 4H 2 O
39. (a) XeF6 XeO3 D
(c) NH 4 NO 2 ¾¾ ® N 2 + 2H 2 O
F
D
F (d) (NH 4 ) 2 SO4 ¾¾ ® 2NH 3 + H 2SO4

Xe D
(a) Ba(N 3 ) 2 ¾¾ ® Ba + 3N 2
F Xe F
NH3 is evolved in case of (d) .
O O 47. (c) On going down the group, bond dissociation
F O enthalpy of the hydrides of oxygen family decreases.
F
1L.P. 1 L.P. Therefore, thermal stability also decreases.
distorted octahedral Pyramidal
48. (d)
XeO4 XeF4 49. (d) (Oxide) (Oxidation state)
F
N2O +1
F
O NO +2
F F Xe N2O3 +3
Xe NO2 +4
F F F F
So, N2O < NO < N2O3 < NO2
1 L.P. 2 L.P. O O
Square pyramidal Square planar
40. (c) Phosphorous acid contain P in +3 oxidation state. 50. (b) HO – S – O – O – S – OH
Acid Formula Oxidation O O
state of Peroxodisulphuric acid
Phosphorous
51. (a) NH2CONH2 + H2O ¾¾® (NH4)2CO3
Pyrophosphorous acid H4P2O5 +3
(A)
Pyrophosphoric acid H4P2O7 +5
Orthophosphorous acid H3PO3 +3 D
Hypophosphoric acid H4P2O6 +4 NH3 (g) + CO3 (g) + H3O (l)
(B)
O O O
2+
NH3 (g) ¾¾¾¾® [Cu(NH 3 ) 4 ]2 +
41. (a) – Cu (aq)
O –S–S–S–S–O S
O O S O
O (B) (C)
2- 2- [Blue coloured
S4 O6 S2 O3 solution]
EBD_8350
652 CHEMISTRY

52. (c) Chlorine has highest electron gain enthalpy (most Therefore, HCl contains » 20/100 = 1/5
negative) among the given elements, the electron gain HNO3 contains » 68/100 » 2/3
enthalpy decreases down the group i.e., moves to least H2SO4 contains » 98/100 » 3/3
negative. Thus, ordinary strong solution of HCl, HNO3 and H2SO4
contains roughly 1/5, 2/3 and 3/3 fractions of pure acid
Exercise - 4
respectively.
1. (d) K 2Cr2O7 + H 2SO4 + 3SO2 ® O
O O
K 2SO 4 + Cr2 (SO4 )3 + H 2O S X1 S X2
Green 9. (a) O S O S
2. (a) NH4ClO4 + HNO3 ® HClO4 + NH4NO3 O O
(X) O
D O
NH4NO3 ¾¾
® N2O + 2H2O S X3
O S
(Y) O
ˆˆˆˆˆˆˆ† 2Li3N (s) O
3. (a) Li (s) + N 2 (g) ‡ˆˆˆˆˆˆˆ
(M) high According to L.P – L.P > L.P – B.P > B.P – B.P repulsion,
temperature
the correct order of S-S bond length will be : x1 > x2 > x3
Li3N + 3H2O ¾® 3LiOH + NH3 heat
(B)
10. (c) S + 3F2 ¾¾¾
® SF6
(Yellow powder) 'X '

CuSO 4 + 4NH3 ¾¾
® [Cu(NH3 ) 4 ]SO4 heat
deep blue compound 3SCl2 + 4NaF ¾¾¾
® SF4 + S2Cl 2 + 4NaCl
'Y '
4. (a) Inorganic compound + SO2 in aq. medium ® (A)
11. (b) CuSO4 + 2KI ® CuI2 + K2SO4
(A) + Na2CO3 ® (B)
(X)
(B) + S ® (C) (Used in photography)
2CuI2 ® Cu 2I2 ¯ + I2 ­
Na 2CO3 + 2SO2 + H 2O ® 2NaHSO 4 + CO 2 White ppt Brown
(A)
2NaHSO 4 + Na 2CO3 ® 2Na 2SO3 + H 2 O + CO 2 I 2 + 2Na 2S2 O3 ® Na 2S4 O 6 + 2NaI
Colourless
( B) (hypo)
Na 2SO3 + S ® Na 2S2O3
(C) 12. (c) ® M + éë XeF5- ùû
MF + XeF4 ¾¾
5. (b) The halate used in fireworks and safety matches is
KClO3. Thus, the gas is Cl2. The hybridisation of Xe in XeF5– is sp3d3.
3Cl2 + 6KOH ® KClO3 + 5KCl + 3H2O The geometry is pentagonal bipyramidal.
Cl2 is greenish yellow gas. To minimise lp-lp repulsion, the two lone pairs of electrons
6. (a) SO3 forms trimer in solid state. occupy the axial positions. Thus, the shape of molecule is
7. (a) (X) is borax, Na2B4O7.10H2O pentagonal planar.
13. (d) In other reactions, oxygen is evolved but in reaction
ˆˆ† 2NaOH + 4H3 BO3
(i) Na 2 B 4O 7 + 7H 2O ‡ˆˆ
(Strong) with SO2, all oxygen is used up in the process of oxidation
(Weak)
Base acid and no oxygen gas is evolved.
Due to presence of NaOH, the aqueous solution is alkaline 3SO 2 + O 3 ¾¾® 3SO 3
to litmus.
14. (c) Both the methods (a) and (b) can be used.
Heat
(ii) Na 2 B4 O7 .10H 2 O ¾¾¾
® Na 2 B4 O7 When passed through freezing mixture the P2H4 present
(swells) condenses and pure PH3 is obtained.
+ 10H 2 O ¾¾
® 2NaBO 2 + B2 O 3 When passed through HI, PH3 is absorbed forming
Glassy bead PH4I.PH4I when treated with KOH (aq) yields pure
(iii) Na 2 B4 O 7 + H 2SO 4 + 5H 2 O ¾¾® phosphine.
Na 2SO 4 + 4H 3 BO 3
PH3 + HI ¾¾
® PH 4 I
white crystals
8. (a) HCl, HNO3 and H2SO4 form azeotrope at 20%, 68% PH 4 I + KOH(aq) ¾¾
® KI + H 2 O + PH 3 ­
and 98% respectively by mass of acid. 15. (a) Amatol is 80% NH4NO3 + 20% TNT
THE d-AND f-BLOCK ELEMENT 653

22 The d-and f-Block Element


p-block elements due to the presence of incomplete d-
Exercise - 1
subshell.
1. (c) General electronic configuration of transition elements 13. (a) 40 elements are present in d-block.
is (n - 1)d1 -10 ns1 - 2 14. (b) 30Zn and 80Hg have their d orbitals completely filled
2. (b) Atomic no. of Ni = 28 so they do not show any variable valency.
Ni (Ground state) = 1s2, 2s2, 2p6, 3s2, 3p6, 3d 8, 4s2 15. (a) Highest O.S. by Mn (+7)
16. (a) Mn2+ –5 unpaired electrons
Ni 2 + = 1s2, 2s2, 2p6, 3s2, 3p6, 3d 8, 4s0
Fe2+ – 4 unpaired electrons
3d
Ti2+ – 2 unpaired electrons
Cr2+ – 4 unpaired electrons
\ It has 2 unpaired electrons Hence, maximum no. of unpaired electron is present in
3. (c) Cerium (Ce) belongs to lanthanide series and is member Mn2+.
of inner-transition metals. Magnetic moment µ number of unpaired electrons
4. (c) Mn3+ = [Ar]3d 4 = [Ar] 17. (d) Magetic moment m = n ( n + 2 ) where n = number
Number of unpaired electrons = 4 of unpaired electrons 15 = n (n + 2 ) \ n = 3
Cr3+ = [Ar]3d 3 = [Ar] 18. (c) M2+ ® 23e–
M ® 25e– (It should be Mn)
No. of unpaired electrons = 3
3d 4s m = 5(5 + 2) = 35 = 5.9
5 2
V3+ = [Ar]3d 2 = [Ar] 19. (b) Both have 3 unpaired electrons.
No. of unpaired electrons = 2
20. (a) Sc3+® 3d 04s 0
5. (b) Zn + = [Ar]3d 10 4 s1, Fe 2+ = [Ar]3d 6 4 s 0 , Ni +
Fe2+® 3d 6 4s 0
= [Ar]3d 8 4s1, Cu + = [Ar]3d10 4s0 ;
2+
Fe contains maximum number of unpaired electrons. Ti3+® 3d 14s 0
6. (b) Cr (24) = 1s 2 , 2 s 2 2 p6 , 3s 2 3 p6 , 3d 5 , 4 s1
Mn2+ ® 3d 5 4s 0
7. (d) Ni3+ : [Ar] 3d 7
In Sc3+ there is/are no unpaired electrons. So the aqueous
Mn3+ : [Ar] 3d 4 solution of Sc3+ will be colourless.
Fe3+ : [Ar] 3d 5 21. (a) Transition elements form coloured ions due to d-d
Co3+ : [Ar] 3d6 transitions. In the presence of ligands, there is splitting of
8. (c) Due to completely filled d-orbitals, these metals are energy levels of d-orbitals. They no longer remain
not considered as transition metals. degenerated. So, electronic transition may occur between
9. (c) Zn, Cd, Hg do not show properties of transition two d-orbitals. The required amount of energy to do this is
elements hence they are known as non typical transition obtained by absorption of light of a particular wavelength
elements. in the region of visible light.
10. (c) The outer electronic configuration of the given ions is 22. (c) The transition metals and their compounds are used
as as catalysts. Because of the variable oxidation states, they
d s
may form intermediate compound with one of the reactants.
Ti2+ These intermediate provides a new path with low activation
d s energy. V2O5 + SO2 ® V2O4 + SO3 2V2O4+ O2 ® 2V2O5
Fe2+ 23. (b) Due to larger surface area and variable valencies to
d s form intermediate absorbed complex easily, transition metals
Cr+ are used as catalysts.
24. (d) Ti4+ (3d0) and Zn2+ (3d10) are colourless.
d s
25. (d) The electronic configuration of different species
Cu+ given in the question are
2+ 2 2 6 2 6 2
11. (a) Os shows maximum oxidation state of + 8. 22Ti : 1s 2s p 3s p d
3+ 2 2 6 2 6 2
12. (d) Most of the transition metal compounds (ionic as well 23V : 1s 2s p 3s p d
4+ 2 2 6 2 6 2
as covalent) are coloured both in the solid state and in 24Cr : 1s 2s p 3s p d
5+ 2 2 6 2 6 2
aqueous solution in contrast to the compounds of s and 25Mn : 1s 2s p 3s p d
EBD_8350
654 CHEMISTRY

26. (d) Sc3+ : 1s2, 2s2p6, 3s2p6d0, 4s0; no unpaired electron. 37. (d) For chromium ion +3 oxidation state is most stable.
Cu+ : 1s2, 2s2p6, 3s2p6d 10, 4s0; no unpaired electron. 38. (a)
Ni2+: 1s2, 2s2p6, 3s2p6d 8, 4s0; unpaired electrons are present. (a) V = 3d 3 4s 2 ; V2+ = 3d 3 = 3 unpaired electrons
Ti3+ : 1s2, 2s2p6, 3s2p6d 1, 4s 0; unpaired electron is present Cr = 3d 5 4s 1 ; Cr 2+ = 3d 4 = 4 unpaired electrons
Co2+ : 1s2, 2s2p6, 3s2p6d 7, 4s0; unpaired electrons are present Mn = 3d 54s2 ; Mn2+ = 3d 5 = 5 unpaired electrons
So from the given options the only correct combination is Fe = 3d 6 4s 2 ; Fe2+ = 3d 6 = 4 unpaired electrons
Ni2+ and Ti3+. Hence the correct order of paramagnetic behaviour
27. (d) In Cu +[Ar]3d 10 there is no unpaired electron, V2+ < Cr 2+ = Fe2+ < Mn2+
Cu 2+[Ar]3d 9 contains one unpaired electron hence (b) For the same oxidation state, the ionic radii generally
coloured. decreases as the atomic number increases in a particular
28. (a) The outermost electronic configuration of Fe is transition series. Hence the order is
Fe = [Ar] 3d6 4s2 Mn2+ > Fe2+ > Co2+ > Ni2+
Fe2+ = [Ar] 3d6 4s0 (c) In solution, the stability of the compound depends
upon electrode potentials, SEP of the transitions metal ions
are given as
Since Fe2+ has 4 unpaired electrons, it is paramagnetic in Co3+ / Co2+ = + 1.97, Fe3+ / Fe2+ = + 0.77;
nature. Cr / Cr 2+ = – 0.41, Sc3+ is highly stable as it does not
3+

Zn = [Ar] 3d10 4s2 —— no unpaired e– show + 2 O. S.


(d) Sc – (+ 2), (+ 3)
Hg2+ = [Ar] 4f 14 5d10 —— no unpaired e–
Ti – (+ 2), (+ 3), (+ 4)
Ti4+ = [Ar] 3d0 4s0 —— no unpaired e– Cr –, (+ 2), (+ 3), (+ 4), (+ 5), (+ 6)
29. (c) Due to d 5 configuration, Mn has exactly half filled Mn – (+ 2), (+ 3), (+ 4), (+ 5), (+ 6), (+ 7)
d-orbitals. As a result, the electronic configuration is stable i.e., Sc < Ti < Cr < Mn
which means 3d electrons are more tightly held by the 39. (c) The melting points of the transition element first rise
nucleus and this reduces the delocalization of electrons to a maximum and then fall as the atomic number increases;
resulting in weaker metallic bonding. manganese have abnormally low melting point.
30. (c) The assertion is correct but the reason is false. 40. (b) Only (ii) has positive reduction potential.
Actually transition metal show variable valency due to very 41. (b) La3+ : 54 e– = [Xe]
small difference between the ns2 and (n – 1)d electrons.
Ti3+ : 19 e– = [Ar] 3d1 (Coloured)
31. (d) The minimum oxidation state in transition metal is
Lu3+ : 68 e– = [Xe] 4f 14
equal to the number of electrons in 4s shell and the maximum
Sc3+ : 18 e– = [Ar]
oxidation state is equal to the sum of the 4s and 3d electrons.
Ti = [Ar] 3d24s2 42. (b) (i) Outer electronic configuration of Mn is 3d 54s2
Hence, minimum oxidation state is +2 and maximum oxidation and hence exhibits +7 oxidation state.
state is +4. Thus, the common oxidation states of Ti are +2, (ii) Zinc does not form coloured ions as it has completely
+3 and +4 filled 3d104s2 configuration.
32. (c) Due to lanthanide contraction, the size of Zr and Hf (iii) In [CoF6]3–, Co3+ is a d7 system. Fluoride is a weak
(atom and ions) become nearly similar. field ligand and hence, does not cause pairing of electrons.
33. (a) 24 Cr = [Ar] 3d 54s1 Co3+ ­¯ ­¯ ­ ­ ­ ; Paramagnetic
25 Mn = [Ar] 3d 54s2 (iv) Sc can form a maximum of +3 oxidation state as it has
26 Fe = [Ar] 3d6 4s2 an outer electronic configuration of 3d14s2.
27 Co = [Ar] 3d 7 4s2 (v) Zn exhibits only +2 oxidation state as this O.S. is the
Due to half-filled d-orbital in Mn2+, it is most stable. Thus, most stable one.
option (a) must be the answer. 43. (d) The ability of oxygen to stabilise high oxidation states
34. (d) (n – 1)d 5ns2 attains the maximum O.S. of + 7. exceeds that of fluorine. Thus, the highest fluoride with
35. (a) In a period on moving from left to right, ionic radii Mn is MnF4, whereas the highest oxide is Mn2O7. Also, it
decreases. is hard to accommodate 7 fluorine atoms around Mn.
(a) So order of cationic radii is 44. (d) The ions with unpaired electrons are coloured and
Cr2+ > Mn2+ > Fe2+ > Ni2+ and
those with paired electrons are colourless.
(b) Sc > Ti > Cr > Mn (correct order of atomic radii)
(c) For unpaired electrons Zn 2 + = 1s 2, 2s2p6, 3s2 p6 d10
(No. of e - s = 28)
Mn 2+ (Five) > Ni 2+ (Two)
Cr 3+ = 1s 2, 2s2p6, 3s2 p6 d3
< Co 2 + (Three) < Fe2 + (Four) -
(No. of e s = 21)
(d) For unpaired electrons >
Fe2+ (Four) > Co2+ (Three) > Ni 2 + = 1s2, 2s2p6, 3s2 p6 d8
-
(No. of e s = 26)
Ni2+ (Two) > Cu 2+ (One)
36. (d) Sc does not show variable valency because of only Thus Zn 2+, Cr 3+ and Ni2+ have zero, 3 and 2 unpaired
one electron in 3d-subshell : [Ar]3d14s2. electrons respectively.
THE d-AND f-BLOCK ELEMENT 655

45. (d) Fe3+ is easily hydrolysed than Fe2+ due to more 61. (b) 2MnO2 + 4KOH + O2 ¾¾
® 2K 2 MnO4 + 2H 2O
positive charge. dark green
46. (c) AgNO3 ® Ag is not decomposed by H2O at room 62. (d) As the oxidation state increases, the acidity increases.
temperature but reduces to silver in presence of light and 63. (a) Cr2 O 72– + 2OH – ¾¾ ® 2CrO 42 - + H 2 O
reducing agent like glucose. Hence, CrO 24- ion is obtained.
47. (b) CuS ® Black
SnS ® Brown 64. (b) CrO3 + 2NaOH ® Na 2CrO 4 + H 2 O
CdS ® Yellow 65. (a) Cr2O72 - + 6I - + 14H + ¾¾ ® 3I 2 + 7H 2O + 2Cr 3+
Sb2S3 ® Orange oxidation state of Cr is +3.
ZnS ® White 66. (a) MnO4– is stable in acidic medium,
NiS ® Black MnO42– disproportionates, CrO42– converts into Cr 2O72–
48. (a) The ionisation energies increase with increase in
and FeO42– decomposes.
atomic number. However, the trend is some irregular among
d-block elements. On the basis of electronic configuration, the 2–
67. (d) O O
Zn : 1s 2 2s 2 p 6 3s 2 p6 d 10 4s 2
Fe : 1s 2 2s 2 p 6 3s 2 p 6 d 6 4s 2 Cr Cr
2 2 6
Cu : 1s 2s p 3s p d 4s 2 6 10 1 OO O O O
Cr : 1s 2 2s 2 p6 3s 2 p 6 d 5 4s1 Dichromate ion
IE1 follows the order : Zn > Fe > Cu > Cr There are six equivalent Cr — O bonds and one
Cr — O — Cr bond.
49. (c) Metallic bond made by elements with d5 configuraiton 68. (c) Solid potassium dichromate when heated with
can be stronger (Cr) or weaker (Mn) than that with d3 concentrated sulphuric acid and a soluble chloride gives
configuration (V). orange red vapours of a volatile oily liquid CrO2Cl2.
50. (a) The +7 oxidation state of Mn is not represented in K2Cr2O7 + 4NaCl + 6H2SO4
simple halides but MnO3F is known. ¾® 2KHSO4 + 4NaHSO4 + 2CrO2Cl2
51. (d) E ° 3+ 2+ = – 0.41 V E ° 3+ 2+ = + 0.77 V chromyl
Cr /Cr Fe /Fe chloride
E° = + 1.57 V,, E° = + 1.97 V 69. (b) Cr2O3 is amphoteric in nature.
Mn 3+ /Mn 2 + Co3+ /Co 2+
70. (b) In neutral or faintly alkaline medium thiosulphate is
The high positive value of E ° 3+ 2+ is due to highest quantitatively oxidized by KMnO4 to SO42–
Co /Co
8KMnO4 + 3Na2S2O3 + H2O ¾®
negative enthalpy of hydration because of very small size
3K2SO4 + 8MnO2 + 3Na2SO4 + 2KOH
of Co3+ ion.
71. (c) In laboratory, manganese (II) ion salt is oxidised to
52. (c) If non metal is added to the interstital site, the metal permanganate ion in aqueous solution by peroxodisulphate.
becomes less malleable due to formation of covalent bond
2Mn 2 + + 5S2O82- + 8H 2O ® 2MnO-4 + 10SO42- + 16H +
between metal and non metal.
peroxodisulphate ion
53. (b) 2Fe3+ + 2I– ¾¾ 2+
® 2Fe + I2 72. (a) Pyrolusite (It is MnO2)
2Fe 2+ + S2 O82 - ¾¾
® 2Fe3+ + 2SO 24 - 73. (c) In acid medium
54. (c) Only Cu in its +2 oxidation state is able to oxidizes MnO-4 + 8H + + 5e - ® Mn 2+ + 4H 2O
the I– to I2
(O.S. of Mn changes form +7 to +2)
55. (d) V2+ – violet, V3+ – green, V4+ – blue, 74. (a)
Fe2+ – green, Fe3+ – yellow 75. (b) Na2Cr2O7 is hygroscopic.
56. (b) Cu 2+ [ Ar ] 3d 9 , Ti 4+ [ Ar ] 3d 0 , Co2+ [ Ar ] 3d 7 , 76. (c) (i) Mn2O7 ® acidic
(ii) CrO ® basic
Fe2+ [Ar]3d 6
1, 3, 4 are coloured ions. (iii) V2O4 ® amphoteric
57. (c) Depends on unpaired electrons. (iv) Cr 2O3 ® amphoteric
+4 +4 +5 +6 77. (b) 5Fe2+ + MnO-4 + 8H+ ¾¾ ® Mn2+ + 4H2O + 5Fe3+
58. (b) VO2+ ; Ti O2+ < VO+2 < Cr O24-
59. (b) (A) Sc only shows +3 oxidation state, thus, cannot 5NO -2 + 2MnO 4- + 6H + ¾® 2Mn 2+ + 5NO 3- + 3H 2O
form SCO type oxide. 78. (b) Down the group metallic character increases hence
(B) Mn forms Mn2O7 covalent oxide. tendency to loose electron increases.
(C) V2O5 is an amphoteric oxide. 79. (b) Opposite to the trend in p-block elements, higher
60. (a) The green colour appears due to the formation of oxidation states of heavier numbers (Mo, W) of transition
elements are more stable than that of lighter memebrs (Cr).
Cr3+ion
80. (c) Mn2O7 is acidic, V2O5 is amphoteric acid and CrO is
Cr2 O72– + 3SO32– + 8H + ¾¾
® 3SO 2–
4 + 2Cr
3+
+ 4H 2 O basic.
EBD_8350
656 CHEMISTRY

81. (b) HCl and SO2 are reducing agents and can reduce In case of actinoids, we can remove electrons from 5f as
MnO4–. CO2 which is neither oxidising and nor reducing well as froms 6d and due to this actinoids exhibit larger
will provide only acidic medium. It can shift reaction in number of oxidation states than lanthanoids.
forward direction and reaction can go to completion. 101. (c) Gd 3+ = [Xe] 4f 7
82. (d) If KMnO4 was added slowly than option a was correct,
102. (a) Ce3+ = [Xe] 4f 1; Ce4+ = [Xe]4f °
but at a moment due to addition of large amount of KMnO4,
reduction of whole KMnO4 added does not take place, it Thus, statement (ii) and (iii) are correct.
also react with Mn2+ which had formed in the solution to 103. (c) In lanthanide series there is a regular decrease in the
give MnO2. atomic as well as ionic radii of trivalent ions (M3+) as the
2MnO4– + 3Mn2+ + 2H2O ¾¾ ® 5MnO2 + 4H+ atomic number increases. Although the atomic radii do
- show some irregularities but ionic radii decreases from La
83. (c) I– is converted to IO3 by neutral or faintly alkaline (103 pm) to Lu (86pm). Y3+ belong to second transition
MnO 4- as shown below.. series therefore have greater ionic radii then other ions of
2MnO -4 + H 2 O + I - ¾¾
® 2MnO 2 + 2OH - + IO 3- third transition series.
84. (c) Lanthanide contraction results into decrease in atomic 104. (c) Sm2+(Z = 62)
and ionic radii. [Xe]4f6 6s2 – 6 unpaired e–
85. (a) 86. (d) Eu2+(Z = 63)
[Xe]4f7 6s2 – 7 unpaired e–
87. (d) Mischmetal is an alloy which contains rare earth
Yb2+(Z = 70)
elements (94-95%), iron (5%) and traces of sulphur, carbon,
[Xe]4f14 6s2 – 0 unpaired e–
silicon, calcium and aluminium. It is used in gas lighters,
Ce2+(Z = 58)
tracer bullets and shells.
[Xe]4f1 5d1 6s2 – 2 unpaired e–
88. (b) In lanthanides, there is poorer shielding of 5d electrons
Only Yb2+ is diamagnetic.
by 4f electrons resulting in greater attraction of the nucleus
over 5 d electrons and contraction of the atomic radii. 105. (b) (A) Sm is hard as steel.
89. (d) On going from left to right in lanthanoid series ionic, (B) Tb4+ has 4f7 configuration (7 unpaired electrons)
size decreases i.e., and thus maximum paramagnetic character.
Ce3+ > Tb3+ > Er3+ > Lu3+. (C) Eu has maximum value of E° for reaction:
90. (b) As a result of lanthanoid contraction, change in ionic Eu3+ (aq) + 3– ® Eu(s); E° = –2.0 V.
radii on going from elements of 4d to 5d transition series is (D) Lu3+ is diamagnetic as it has 4f14 configuration.
very small. Thus, chemical properties of 4d and 5d series 106. (c) Gun metal is an alloy of Cu, Zn and Sn. It contains
of transition elements are similar. 88% Cu, 10% Sn and 2% Zn.
91. (a) Am shows maximum number of oxidation states,
107. (b) Brass is an alloy of Cu and Zn
+ 3, + 4, + 5, + 6
108. (d) V2 O5 catalyses the oxidation of SO2 in the manufacture
92. (c) Thorium can show normally +4 oxidation state
whereas Mn and Cr can show large number of O.S., Np of H2SO4.
belong to 5f series and shows variable O.S. 109. (d)
93. (b) As a result of lanthanide contraction Zr 4+ and Hf 4+ 110. (b) Cu, Ag and Au are called coinage metals.
possess almost the same ionic radii. Ce4+ is an oxidising 111. (d) PdCl2 is used as a catalyst in Wacker’s process.
112. (a) Control rods slowdown the motion of neutrons and
agent. Ce4+ gains electron to acquire more stable Ce3+state.
help in controlling the rate of fission. Cadmium is efficient
La(OH)3 is the most basic among lanthanide hydroxides. for this purpose.
94. (b) Most of the Ln3+ compounds except La3+ and Lu3+ 113. (a) Fe3+ ion can be detected by K4[Fe(CN)6]
are coloured due to the presence of f-electrons. 4Fe3+ + 3K4[Fe(CN)6] ¾® Fe4[Fe(CN)6]3 + 12K+
95. (b) The magnetic moment values are lesser than the 114. (b) Bronze - 10% Sn, 90% Cu
theoretically predicted values due to the fact that 5f
electrons of actinides are less effectively shielded which (Sn is a non transition element)
results in quenching of orbital contribution. 115. (a) Steel is an alloy of Fe and C (non-metal). Interstitial
96. (d) La doesn’t have partially filled f-orbital. compounds are chemically inert.
97. (a) 4f orbital is nearer to nucleus as compared to 5f orbital
Exercise - 2
therefore, shielding of 4 f is more than 5 f.
98. (d) All members do not show +4 oxidation state. 1. (0.2)MnO4– + 5Fe2+ + 8H+ ¾® 5Fe3+ + Mn2+ + 4H2O
99. (c) Ac (89) = [Rn] [6d1] [7s2] In the above reaction 1 mole of MnO4– reacts with 5 mole of
100. (b) The main reason for exhibiting larger number of Fe2+.
oxidation states by actinoids as compared to lanthanoids or 1 mole of Fe2+ reacts with 0.2 mole of MnO4–.
is lesser energy difference between 5 f and 6d orbitals as 2. (1.73) The electronic configuration of Ce3+ (atomic no. of
compared to that between 4f and 5d orbitals. Ce = 58) is
THE d-AND f-BLOCK ELEMENT 657

1s2 2s2 2p6 3s2 3p6 4s2 3d10 4p6 5s2 4d10 5p6 6s1 Exercise - 3
There is only one unpaired electron in Ce3+ ion. The spin-
only formula for calculating the magnetic moment is 1. (b) Electronic configuration of X 3+ is [Ar]3d 5
Atomic number of X = 18 + 5 + 3 = 26
m= n (n + 2) BM 2. (b) CuF2 is coloured in solid state because Cu(II) has d9
where n is the number of unpaired electrons in the species. configuration thus, due to the presence of unpaired
Here, n = 1, electrons, it exists as a coloured salt.
3. (b) Greater the number of unpaired electrons, higher will
So, m = 1´ 3 BM = 1.73 BM
be the value of magnetic moment.
3. (3) Magetic moment m = n (n + 2 ) where n = number of
Configuration No. of Unpaired e –s

unpaired electrons 15 = n(n + 2 ) \ n = 3 3d 7 3


5
3d 5
4. (5) FeSO 4 . ( NH 4 )2SO 4 .6H 2O in solution will give
8
3d 2
( ) ( )
Fe2 + , 2 SO 24- , 2 NH 4+ hence total number of ions is 5.
3d 2 2
5. (158) Change in O.S. is by 1.
4. (b) Lanthanoids show common oxidation state of +3.
158
Hence eq. wt. is = 158 g 5. (b) Spin only magnetic moment (m ) = n ( n + 2 ) BM
1

6. (3) Magnetic moment m = n (n + 2 )BM = 3 ( 3 + 2 ) = 15 = 3.87 BM


6. (a) Copper lies below hydrogen in the electrochemical
1.73 = n ( n + 2) \ n = 1 , it has one unpaired electron hence series and hence does not liberate H2 from acids.
electronic configuration is [ Ar ] 3d 1 and electronic 7. (c) As mentioned in the reaction below Sn 2+ changes to
Sn4+ :
configuration for Z = 22 is [ Ar ] 3d 2 4 s 2 . Hence charge on Oxidation

Ti is +3. Cr2+ 6 O72 - + 14H + + 3Sn 2+ ® 2Cr3+ + 3Sn 4+ + 7H 2 O


(1) E°Cu 2+ /Cu = 0.34 volt,
Reduction
7.
other has – ve E°R.P. 8. (d) Along the period, decrease in metallic radius is
coupled with increase in atomic mass therefore results in
E° = – 0.28 increase in density of metal. Hence, Cu will exhibit highest
Co 2 + /Co
density.
E° = – 0.25 9. (a) The compound formed can be explained by the
Ni 2 + / Ni
following reaction :
E° = – 0.44 2KMnO4 + 2H2SO4 (conc.) ® Mn2O7 + 2KHSO4 + H2O
Fe2 + /Fe
(green oily)
8. (5) Mischmetal is an alloy which contains rare earth 10. (d) Mn2+ acts as autocatalyst.
elements (94-95%), iron (5%) and traces of sulphur, carbon,
2MnO-4 + 16H+ + 5C2O24- ® 2Mn 2+ + 10CO2 + 8H2O
silicon, calcium and aluminium. It is used in gas lighters,
tracer bullets and shells. 11. (c) Tm; Thulium Z = 69 does not belong to actinoid series.
9. (–0.28) 12. (a) Reaction of KMnO4 with sulphide ions in acidic
medium is as follows :
Mn 2 + + 2e - ¾¾
® Mn, DG1 = - nFE1 = -2 ´ F ´ ( -1.18) 2KMnO4 + 3H2SO4 ® K2SO4 + 2MnSO4 + 3H2O + 5[O]
Mn3+ + e - ¾¾
® Mn 2 + , DG 2 = -nFE 2 = -1 ´ F ´ (1.51) [H 2S +[O] ® H 2O +S]´5
____________________________________________ 2KMnO4 + 3H 2SO4 +5H 2S ® K 2SO4 + 2MnSO4 +8H 2O + 5S
Adding, Mn 3+ + 3e - ¾¾® Mn, DG 3 = -3FE 3 5 moles of S2– ions react with 2 moles of KMnO4
DG3 = DG1 + DG 2 2
-3FE3 = -2 ´ F ´ (-1.18) - (1) ´ F´ (1.51) thus, 1 mole of S2– ion will react with
moles of KMnO4.
5
E3 = - 0.28 V 13. (d) Interstitial compounds are chemically inert.
pH = 3 pH = 8 14. (c) When alkaline KMnO4 an oxidising agent, is treated
10. (11) CrO42– ¾¾¾® CrO72– ¾¾¾® CrO42–
with KI, iodide ion is oxidised to IO3- .
\ Sum of x and y is 11.
2KMnO4 + H2O + KI ® 2MnO2 + 2KOH + KIO3
EBD_8350
658 CHEMISTRY

15. (d) In covalent compounds fluorine can form only single Hence, CrO5 has two peroxy linkages.
bond while oxygen forms double bond. Now suppose the oxi. no. of Cr is x then
16. (c) Zr and Hf possess almost same atomic and ionic radii x + (– 1 × 4) + (– 2) = 0
because of lanthanoid contraction. Hence, they exhibit \ x = +6
similar properties. 25. (b) 2 KMnO4 + 3H2SO4 + 5H2O2 ¾®
17. (b) HCl is not used in oxidation reactions of KMnO4 in K2SO4 + 2MnSO4 + 8H2O + 5O2
acidic medium because KMnO4 + HCl produce nascent 2+
i.e., Mn ion and O2.
oxygen which will partly oxidise HCl to Cl2. So the end
D
point will not be correct in the titration. 26. (d) 3Fe + 2O2 ¾¾
® Fe3O4
18. (a) Cation carrying a higher charge and smaller size is 600°C
associated with greater lattice and hydration energy. But Fe3O4 + CO ¾¾¾® 3FeO + CO2
it is expected on the basis of electronic configuration that FeO + CO ¾¾¾® 700°C Fe + CO2
Cu+ should be more stable because it has completely filled 27. (d) Magnetic moment = 2.84 B.M. This indicates that 2
orbitals. But the lattice and hydration energy factors unpaired electrons are present.
dominate and as a result, Cu2+ is more stable.
19. (a) (i) The reaction in which oxidation as well as For Ni2+ = 4s0 3d8
reduction of same metal occur is known as Number of unpaired electrons (n) = 2
disproportionation reaction. Hence Ni2+ gives magnetic moment 2.84 B.M.
Oxidation 28. (a) Fe2+ = 3d 6 (number of ‘d’ electrons = 6)
Cu
+
Cu + Cu
2+ In Cl = 1s2 2s2 2p6 3s2 3p5
Reduction total p electrons = 11, which are not equal to number of ‘d’
electrons in Fe2+
+6 +7 +4 p-electrons in Ne = 1s2 2s2 2p6 = 6
+
(ii) 3MnO2– –
4 + 4H ® 2 MnO4 + MnO 2 + 2H 2O s-electrons in Mg = 1s2 2s2 2p6 3s2 = 6
Thus, this is also a disproportionation reaction. 29. (d)(n + l) rule can be used. Titanium is a multi electron system
(n + l)
+7 +6 +4
3s < 3p < 4s < 3d
(iii) 2K MnO 4 ® K 2 MnO 4 + MnO 2 + O 2
¯ ¯ ¯ ¯
Only reduction takes place.
+7 +2 +4
(3 + 0) (3 + 1) (4 + 0) (3 + 2)
(iv) 2 MnO4– + 3Mn 2+ + 2H 2O ® 5MnO 2 + 4H + || || || ||
3 4 4 5
It is an example of comproportionation where two If (n + l) values are same, then value of "n" has to be
reactants, each containing the same element but with a considered.
different oxidation number, form a product in which the 30. (a) Considering the same moles of compounds
element involved reach the same oxidation number. ( +6)
20. (a) Amphoteric oxide react with alkalies as well as acids. (a) Fe2+ SO2–
4
, only Fe2+ is oxidised by KMnO4. SO2– 4
alkali
V2O5 ¾¾¾ 3- in which sulphur is in highest oxidation state cannot be
® VO 4
oxidised.
acid
V2O5 ¾¾¾ +
® VO 4 (b) Fe 2 + SO32– ¾¾
® Fe3+
acid
Cr2O3 ¾¾¾ 3+
® [Cr(H2O)6] (+4) (+6)
2– 2– –
Cr2O3 ¾¾¾® alkali [CrO2]– SO3 —® SO4 + 2e
21. (a) The E.C. of gadolinium is : Sulphur can be oxidised to (+6) i.e., to SO 2–
4
+3 +4
Gd = [Xe]4f 75d16s2
(c) In FeC 2 O4 ¾¾ ® CO2
22. (a) The shape of f-orbitals is very much diffused and
they have poor shielding effect. The effective nuclear Carbon can be oxidised to (+4) i.e., to CO2
charge increases which causes the contraction in the size
® Fe2 +
(d) Fe(NO 2 ) 2 ¾¾
of electron charge cloud. This contraction in size is quite
regular and known as lanthanoid contraction. (+3)
23. (c) Both are reducing agents. – Nitrogen can be
+ 2NO2
24. (c) The structure of CrO5 is oxidised to (+5)
¯ – – state
O O 2NO3 + 2e
­
Cr (+5)
Due to minimum change in oxidation number in option (a),
O O least amount of acidified KMnO4 will be required for
O oxidation.
THE d-AND f-BLOCK ELEMENT 659

31. (b) Due to lanthanoid contraction atomic radii of Zr and 41. (c) (A) V2O5 ® Preparation of H2SO4 in contact process
Hf is almost similar. (B) TiCl4+ Al(Me)3 ® Polyethylene
32. (d) (A) - (ii), (B) - (i), (C) - (iv), (D) - (iii)
33. (a) L ® M charge transfer spectra. KMnO4 is colored (Ziegler-Natta catalyst)
because it absorbs light in the visible ran ge of (C) PdCl2 ® Ethanal (Wacker’s process)
electromagnetic radiation. The permanganate ion is the (D) Iron oxide ® NH3 in (Haber’s process)
source of color, as a ligand to metal, (L ® M) charge
transfer takes place between oxygen's p orbitals and the 42. (a) Magnetic moment, m = n(n + 2) BM (where, n = no.
empty d-orbitals on the metal. This charge transfer takes
place when a photon of light is absorbed, which leads to of unpaired electrons)
the purple color of the compound. As transition metal atom/ion in a complex may have
34. (d) K2Cr2O7 + SO2 + H2SO4 ¾® unpaired electrons ranging from zero to 5. So, maximum
K2SO4 + Cr 2(SO4)3 + H2O number of unpaired electrons that may be present in a
green colour complex is 5.
35. (d) Eu (63) = [Xe] 4 f 7 6s2 \ Maximum value of magnetic moment among all the
Gd (64) = [Xe] 4 f 7 5d1 6s2 transition metal complexes is = 5(5 + 2) = 35 = 5.92 BM
Tb (65) = [Xe] 4 f 9 6s2
36. (d) Out of all the four given metallic oxides CrO2 is 43. (c) Oxidation state of Cr in CrO 24- and Cr2 O72- is + 6 i.e.
attracted by magnetic field very strongly. The effect oxidation states are same.
persists even when the magnetic field is removed. Thus 44. (a) Atomic size of elements of 4d and 5d transition series
CrO2 is metallic and ferromagnetic in nature. are nearly same due to lanthanide contraction.
37. (a) Potassium permanganate has a purple colour. When
sulphur dioxide reacts with potassium permanganate the Exercise - 4
solution decolourises. 1. (b) The colour exhibited by transition metal ions is due
SO2 + 2KMnO4 + 2H2O ® 2H2SO4 + 2MnSO4 + K2SO4 to the presence of unpaired electrons in d-orbitals which
38. (b) Minimum or comparable energy gap between 5f, 6d permits the d-d excitation of electrons.
and 7s subshell makes electron excitation easier, hence
there is a greater range of oxidation states in actinoids. In TiF62– ,– Ti is in + 4 O.S. ; 3d0 – colourless
39. (c) CrO2–
4 Cr6+ diamagnetic In CoF63– ,– Co is in + 3 O.S ; 3d6 – coloured
In Cu2Cl2 – Cu is in +1 O.S. ; 3d10 – colourless
Cr2O2–
7 Cr6+ diamagnetic
In NiCl2– 8
4 – Ni is in + 2 O.S ; 3d – coloured
MnO4– Mn7+ diamagnetic 2. (a) The order of basic character of the transition metal
monoxide is TiO > VO > CrO > FeO because basic character
MnO 2–
4 Mn6+ paramagnetic
of oxides decreases with increase in atomic number.
3d 3. (a) Due to the formation of Cr 3+ from Cr2O72– :
D
(NH 4 ) 2 Cr2 O7 ¾¾
® Cr2 O3 + N 2 + 4H 2 O
Thus, unpaired electron is present, so d–d transition is (orange) (green)
possible.
4. (b) Ag 2S + 4 NaCN + 2O 2 ® 2 Na[Ag(CN )2 ] + Na 2SO 4
t2g t 2g
X is S and Y is O2.
3d
eg eg 5. (c) 2Au3+ + 6e - ¾¾
® 2Au, DG° = - 6 ´ 1.52F ......(i)

Before After 3Au3+ + 6e - ¾¾


® 3Au + , DG° = -6 ´1.36F ......(ii)
transition transition For the reaction
40. (a) Co3+ = [Ar] 3d 6 , unpaired e–(n) = 4
3Au + ¾¾ ® 2Au + Au3+ , ΔG° = (i) – (ii)
Spin magnetic moment = 4(4 + 2) = 24 B.M.
= 8.16 F – 9.12 F = – 0.96 F (negative)
Cr3+ = [Ar] 3d 3 , unpaired e–(n) = 3 Trick : The reduction potential for Au3+/Au+ is less than
Spin magnetic moment = 3(3 + 2) = 15 B.M. that for Au 3+ /Au. Thus, Au + is unstable and
disproportionate to Au3+ and Au in aqueous solution.
Fe3+ = [Ar] 3d 5 , unpaired e–(n) = 5
Spin magnetic moment = 5(5 + 2) = 35 B.M. 6. (a) Hg 2Cl 2 + 2NH 4OH ¾¾
®
'X' H2N
Ni2+ = [Ar] 3d 8 , unpaired e–(n) = 2 Hg + NH Cl + 2H O
Cl 4 2
Spin magnetic moment = 2(2 + 2) = 8 B.M.
Black ppt
EBD_8350
660 CHEMISTRY

H2N If acidity or [H+] increased, then equilibrium will shift


HgCl 2 + 2NH 4 OH ¾¾
® Hg + NH4Cl + 2H2O towards Cr2O72– and the solution has more oxidizing
'Y' Cl property.
White ppt
13. (b) 2 CuSO 4 + K 4 [ Fe (CN ) 6 ] ¾¾®
HgCl2 + 2kI ® HgI 2 ¯ +2kCl Cu 2 [ Fe (CN ) 6 ] + 2 K 2SO 4
(red) Chocolate ppt.
HgI2 + 2kI ® k2 HgI 4 (soluble) 14. (c) The magnetic moment (m) of a species is related to its
7. (c) Smaller is the ionization energy of a metal to give a number of unpaired electrons (n) in form of following
particular oxidation state, greater will be thermodynamic expressions.
stability of that oxidation state. m= n( n + 2) B.M
8. (c) (a), (b) and (d) are correct statements.
The number of unpaired electrons in the given pairs are as
(c) There is drop in enthalpy of atomization at manganese follows:
because of half filled stable electronic configuration leading
to less mobilisation of electrons. This results in weaker Fe2 + = 1s 2 , 2s 2 2 p 6 ,3s 2 3 p6 3d 6
metallic bonds. 3d
dil. HNO3 NaOH or Thus here n = 4.
9. (c) Zn ¾¾¾¾¾
® Zn(NO3 )2 ¾¾¾¾ ® Zn(OH)2
(M) aq.
Colourless White ppt.
solution
Fe3+ = 1s 2 ,2 s 2 2 p6 ,3s 2 3 p6 3d 5
Excess
¾¾¾¾
NaOH
® Na 2 [ Zn ( OH )4 ] ¾¾¾® H 2S
ZnS 3d
Soluble White ppt.
or Thus here n = 5.
D
10. (c) 2MnO 2 + 2K 2CO 3 + O 2 ¾¾® 2K2 MnO 4 + 2CO 2 ­
+ +
(X) (air)
green
(Y ) && :
NO or N = O n =0
xx
´ ..
2K 2 MnO4 + Cl 2 ¾¾
® 2KMnO4 + 2KCl NO or ´´ N —
—O: n =1
(Y) (Z) Pink
The given combinations differ in the number of
H S KCN
11. (a) CuSO 4 (aq.) ¾¾¾
2 ® 2CuS ¾¾¾¾
® unpaired electrons. Hence, these can be differentiated by
Black ppt. Excess
(M) the measurement on the solid state magnetic moment of
nitroprusside ion.
[ Cu(CN)4 ]3– + (CN) 2 ­ 15. (a) The +4 oxidation state of cerium is also known in
(N) (O)
solution.
12. (b) Eº = 1.33 V; E º = – 0.11 V
Cr2O 72 - /Cr 3+ CrO 2- / Cr 3+
4
Cr2O72–(aq) + H2O(l) 2CrO42–(aq) + 2H+(aq)
23 Coordination Compounds

Exercise - 1
H2N NH2
1. (b) It is a double salt:
FeSO 4 .( NH 4 )2 SO4 .6H 2 O ¾¾
®
17. (c) (A) Triphenylphosphine
2+
Fe + 2SO24 - + 2NH +4 Ph
2. (c) Secondary valence does not depend upon the nature P Ph
of ligand but depends on number of ligands attached to
metal. Ph
unidentate
3. (c) The coordination number of central metal atom in a
complex is equal to number of monovalent ligands, twice (B) BF3 Electron deficient molecule
the number of bidentate ligands and so on, around the (C) Ethylene diamine (en) didentate ligand
metal ion bonded by coordinate bonds. (D) Ethylenediaminetetraacetate ion (EDTA) 4–
Hence coordination number = no. of s bonds formed by hexadentate ligand
metals with ligands 18. (b) In bridging complexes, O2 can act as a bidentate
ligand.
4. (d) Oxidation state of Cr in [ Cr ( NH 3 ) 4 Cl 2 ] + . 19. (d) In the given complex we have two bidentate ligands
Let it be x, 1 × x + 4 × 0 + 2 × (–1) = 1 Therefore x =3. (i.e en and C2O4), so coordination number of E is 6
5. (d) [EDTA]4– is a hexadentate ligand, because it has six (2 × 2 + 1 × 2 = 6)
donor atoms and donate 6 pairs of electrons to central Let the oxidation state of E in complex be x, then
metal atom in the complex. [x + (–2) = 1] or x – 2 = 1
6. (a) Salt may be complex salt. Metal atom present in the or x = + 3, so its oxidation state is + 3
coordination sphere appears in the form of complex ion Thus option (d) is correct.
and not as simple cation. 20. (a) Reason is the correct explanation of Assertion.
7. (d) Ligand can be anion, neutral molecule or even cation 21. (d) When excess of AgNO3 and BaCl2 are added to
also linked by coordinate bonds to central atom or ion. solution X.
8. (d) A bidentate ligand like glycinato [Co(NH3 )5 Br]Cl2 + 2AgNO3 ®
( NH2 – CH 2 – COO –) forms simultaneously two 1mole 2 moles
0.02 mole
coordinate bonds with two donor atoms.
9. (c) Ambident ligand has two donor atoms, either of two [Co(NH3 ) 5 Br](NO3 ) 2 + 2AgCl(ppt.)(Y)
1mole 2moles
can form a coordinate bond. 0.02 ´ 2 = 0.04 mole
10. (c) Na+ is not a ligand.
11. (b) Two different types of ligands are present, i.e. NH3 [Co(NH3 )5 Cl]SO 4 + BaCl2 ®
and Cl. 1mole
0.02mole
12. (d) Co (NH3)5 Cl3 [Co(NH3)5Cl]+2 + 2Cl-
[Co(NH3 )5 Cl]Cl 2 + BaSO 4 (ppt.) (Z)
\ Structure is [Co (NH3)5 Cl] Cl2. 1mole
0.02mole
Now [Co(NH3 ) 5 Cl]Cl 2 + 2AgNO3 ¾¾
® 22. (c) It is correct statement that NF3 is a weaker ligand
[Co(NH3 )5 Cl](NO3 )2 + 2AgCl than N(CH3)3, the reason is that fluorine is highly
electronegative therefore, it with draw electrons from
13. (c) Complex compounds contains two different metallic nitrogen atom. Hence, the lone pair of nitrogen atom cannot
elements but give test only for one of them. Because be ligated. While N(CH3)3 is a strong ligand because CH3
complex ions such as [Fe (CN)6]4– of K4 [Fe (CN)6], do not is electron releasing group.
dissociate into Fe2+ and CN– ions.
23. (b) Pt Cl2.2NH3= [Pt(NH3)2Cl2]
14. (a) EDTA is hexadentate ligand
CoCl3.4NH3= [Co(NH3)4Cl2] Cl
– –
OOCH2C •• •• CH2 – COO NiCl2.6H2O = [Ni(H2O)6]Cl2.
– N – CH2 – CH2– N 24. (b) Since complex compound gives 2 moles of AgCl on
OOCH2C CH2 – COO–
treatment with AgNO3.
15. (c) Pyridine (C5H5N:) is a neutral unidentate ligand.
\ most probable structure is (b).
16. (a) Neutral, bidentate and chilating ligand.
25. (b) IUPAC name is Potassium trioxalatoaluminate (III).
EBD_8350
662 CHEMISTRY

26. (a) Chlorodiaquatriamminecobalt (III) chloride is 41. (b) Option (b) shows optical isomerism [Co(en)3]3+
[CoCl( NH 3 )3 (H 2O) 2 ]Cl 2 . 3+ 3+
en en
27. (c) K 3[Fe(CN ) 6 ] is Potassium hexacyanoferrate (III).
28. (a) Complexes of the type MABCD may exist in three en Co Co en
isomeric forms.
A B A C A C en
en

d–form Mirror –form


M M M
Complexes of Zn++ cannot show optical isomerism as they
are tetrahedral complexes with plane of symmetry.
D C D B B D
(I) (II) (III) [Co(H2 O)4 (en)]3+ have two planes of symmetry hence it
Similarly, [Pt (py) (NH3) BrCl] may exist in three isomeric is also optically inactive.
form in which [Zn(en)2]2+ cannot show optical isomerism
M = Pt, A = Py, B = NH3, C = Br, D = Cl. 42. (b) Ionisation isomer of [Cr(H2O) 4 Cl(NO 2)]Cl is
29. (a) [Co(NH3)3(NO2)3] is of the type MA3B3. They give [Cr(H2O)4Cl2]NO2.
two geometrical Isomers. 43. (c) [Ma 2b2c2] can show both optical & geometrical
30. (c) [PtCl2(NH3)4]Br2 and [PtBr2(NH3)4]Cl2 are ionisation isomerism.
isomers. 44. (d) The optical isomers are pair of molecules which are
31. (b) Given compound shows meridional isomerism. non superimposable mirror images of each other.
32. (c) [CuCl2{(O = C(NH2)2}2] en en
33. (a) Geometrical isomerism is possible only in square
planar complexes of the type [MA2B2] and [MA2BC] and
for octahedral complexes of the type [MA4B2] and en Co Co en
[MA4BC]. Hence, only (ii) will show geometrical isomerism.
34. (c) Geometrical isomers of following type of square
planar complexes is possible. [Ma2b2] type, [Ma2bc] en en
type and [Mabcd] type. 3+ 3+
35. (b) [Co(ONO)(NH3)5]Cl2 [Co(en)3] [Co(en)3]
(dextro) Mirror
(laevo)
pentaamminenitrito-O-cobalt (III) chloride
The two optically active isomers are collectivity called
36. (d) Chemical formula of iron (III) hexacynaoferrate (II) is
enantiomers.
Fe4[Fe(CN)6]3
45. (d) [Pt(en)2Cl2] is a complex of the type M(AA)2 B2 which
37. (b) The chemical formula of Pentamminenitrochromium is octahedral. Such compounds exhibit optical and
(III) chloride is geometrical isomerism both.
éCr ( NH3 ) NO2 ù Cl2 46. (c) Octahedral complex of the type MA5B do not show
ë 5 û
geometrical isomerism
It can exist in following two structures
47. (a) (A)
é Cr ( NH 3 ) NO 2 ù Cl2 and Cl
ë 5 û Cl
é Cr ( NH3 ) ONO ù Cl 2 NH3
ë 5 û H3N H3N Cl
Therefore the type of isomerism found in this compound
is linkage isomerism as nitro group is linked through N as CO CO
–NO2 or through O as – ONO. H3N H3N NH3
NH3
3–
38. (b) [Cr (C2 O4)3] or [Cr (ox) 3]3– Cl NH3
ox ox trans cis
ox (B)
Cr ox Cr en
en
ox ox Cl Cl
d- and l- forms
CO CO
39. (a) The octahedral coordination compounds of the type
MA3B3 exhibit fac-mer isomerism. Cl Cl
40. (a) Complexes with dsp2 hybridisation are square planar. en en
So, [PtCl4]2– is square planar in shape. d- and l- forms
COORDINATION COMPOUNDS 663

(C) [CO (NH3)5 (NO2)] Cl2 and 52. (b) Non –superimposable mirror images are optically
[CO (NH3)5 Cl] (NO2) Cl are ionisation isomers. active, hence rotate plane polarized light.
(D) [CO (NH3)6] [Cr (CN)6] and
[Cr (NH3)6] [CO (CN)6] are coordination isomers. A A
48. (b) Non superimposable mirror images are called optical
isomers and may be described as “chiral’. They are also M M
called enantiomers and rotate plane polarised light in Y B
B Y
opposite directions. X X
53. (a) Sc = [Ar] 3d 1, 4s2
Cl Cl Oxidation state of Sc in [Sc(H2O)6]3+ is Sc3+
Cl Cl Sc3+ = [Ar] 3d 0, 4s0.
Q It does not have unpaired electron
en Co Co en \ Sc3+ is diamagnetic and colourless.
54. (a) Cl– is a weak field ligand.
55. (a) In octahedral complex the magnitude of Do will be
en en highest in a complex having strongest ligand. Out of the
given ligands CN– is strongest. So, Do will be highest for
49. (a) The given compound may have linkage isomerism [Co(CN)6]3–. Thus option (a) is correct.
due to presence of NO2 group which may be in the form – 56. (a) Ligands can be arranged in a series in the orders of
NO2 or –ONO. increasing field strength as given below :
It may have ionisation isomerism due to presence of two Weak field ligands :
ionisable group –NO2 & –Cl. It may have geometrical I- < Br - < S2 - < SCN < Cl - < N3- , F -
isomerism in the form of cis-trans form as follows : < Urea, OH– < oxalate
[Co(NH3)4Cl(NO2)]NO2 & [Co(NH3) (NO2)2]Cl Strong field ligands
––– Ionisation isomers.
O -- < H 2O < NCS- < EDTA < Py, NH3 <
[Co(NH3)5(NO2)2]Cl & [Co(NH3)5(ONO)2]Cl
––– Linkage isomers en = SO3– < bipy, Phen < NO2- < CH3-
NO2 NO2 < C6 H 5- < CN - < CO
Such a series is termed as spectrochemical series. It is an
H3N NH3 H3N NO2 experimentally determined series based on the absorption
Co Co of light by complexes with different ligands.
57. (c) [Ni(CO)4] [Ni(PPh3)2Cl2]
H3N NH3 H3N NH3
O.S. Ni0 Ni2+
NO2 NH3 E.C. [Ar]3d 84s2 [Ar]3d 84s0
trans-form cis-form
Pairing of e– No pairing of e–
Geometrical isomers
50. (c) Complex is not superimposable on its mirror image Hybridization sp3 (tetrahedral) sp3 (tetrahedral)
hence optically active i.e., rotate plane polarized light.
58. (c) [Cr(NH3)6]Cl3 is an inner orbital complex, because in
180° this complex inner d-orbital is used for hybridisation (d2sp3)
59. (b)
3+ 3+
en en Atom/Ion Configuration
No. of
Magnetic
Complex unpaired
electrons nature
en Co Co en 2+ 8
3d 4s 4p
Ni (d ) 2
2–
en en [NiCl4]
2
180°
cannot be sp3
superimposed en 2–
3+ [Ni(CN)4] 0
non-superimposable Rearrangement
en Co dsp
2

en Ni (d s ) 8 2 2
[Ni(CO)4]
0

51. +
(c) [Cr(SCN ) 2 ( NH 3 ) 4 ] shows linkage and geometrical Rearrangement 3
sp
isomerism. Hence, produces maximum no. of isomers.
EBD_8350
664 CHEMISTRY

60. (b) Both Assertion and Reason are true but Reason is 69. (d) Fe2+ has electronic configuration
not the correct explanation of statement-1. [Fe(CN)6]3– is 3d 4s
weakly paramagnetic as it has unpaired electrons while due to strong ligand
[Fe(CN)6]4– has no unpaired electron.
\ It is diamagnetic. Ni has electronic configuration
61. (c) Among these ligands, ‘F’ is a weak field ligand, makes 3d 4s
only high spin complexes which has sp3d2 hybridization. due to strong ligand
62. (b) In [Ni(CO)4]; [Ni(Cl)4]–2; [Zn(NH3)4]+2, all the central
Ni2+ has electronic configuration
atoms have sp3 hybridisation, while in [Ni(CN)4]–2, its central
3d 4s
atom has dsp2 hybridisation due to strong field ligand (CN).
due to strong ligand
Hence, [Ni(CN)4]–2 has square planar structure.
63. (d) [Cr(H2O)6]+3 + 6NH3 ® [Cr(NH3)6]+3 + 6H2O
Co 3+ has electronic configuration
Cr+3 show six C.N. with NH3
2+
64. (a) Oxidation state of Cr in éë Cr ( H 2 O )6 ùû is + 2. 3d 4s

Electronic configuration of Cr No pairing of electrons due


= (Ar)18 4s1 3d 5
3-
Electonic configuration of Cr 2+ = (Ar)16 3d 4 to weak ligand hence éëCoF6 ùû is paramagnetic.
H2O is a weak field ligand.
70. (c) Number of unpaired electrons in central atom
CFSE value [- 0.4n t 2g + 0.6n eg ]D0 + nP
[Fe(CN)6 ]4- ,[CoCl4 ]2- ,[MnCl 4 ]2 -
= [ -0.4 ´ 3 + 0.6 ´1] D 0 + 0 = – 0.6 D0 zero three five
3+
Oxidation state of Cr in éë Cr ( H 2 O )6 ùû is + 3 The greater the number of unpaired electrons, the higher
Electronic configuration of Cr3+ = (Ar)16 3d 3 the value of magnetic moment
3+ 71. (d) The electronic configuration of Fe2+ is [Ar] 3d6. Since
CFSE value of éë Cr ( H 2 O )6 ùû
CN is strong field ligand d electrons are paired. In Ni(CO)4
= éëê -0.4n t 2g + 0.6n e g ùûú D 0 + nP O. S. of Ni is zero, electronic configuration is [Ar]3d8 4s2.
In presence of CO it is [Ar] 3d10 4s0, electrons are paired.
= [ -0.4 ´ 3 + 0] D0 + 0 = – 1.2 D0 Electronic configuration of Ni2+ [Ar]3d8 4s0, due to CN–
: ligand, all electrons are paired. Co3+ is [Ar] 3d6 since F is
65. (a) Co3+
weak ligand hence, paramagnetic.
[Co(CN)6]3– : 72. (c) In Na2[CdCl4], Cd has oxidation state +2.
So, its electronic configuration is 4d105s0
CN– is a strong field ligand and it causes pairing of or all the 4d orbitals are fully filled.
electrons; as a result number of unpaired electrons in Co3+ Hence, there will be no d-d transition. So, it is colourless.
becomes zero and hence it has lowest value of
73. (d) CoCl3.5NH3.H2O is pink in colour
paramagnetic behaviour.
74. (b) CuSO 4 + 4 NH 3 ® [Cu(NH3 ) 4 ]SO 4
66. (a) NH3 is a stronger ligand than H2O. When aqueous
solution of copper (II) sulphate or [Cu(H2O)4]SO4. H2O is Blue complex due to [ Cu(NH 3 ) 4 ]2 +
a saturated with NH3, then NH3 replaces H2O and form a
blue complex [Cu(NH3)4]SO4. H2O
75. (a) [ Ni(CN)4 ]2- : Number of unpaired electrons = 0
67. (d) Cr3+ has 4s0 3d 3 electronic configuration with 3 [Cr(NH3 )6 ]3+ : Number of unpaired electrons = 3
unpaired electrons, hence paramagnetic. In other cases,
pairing of d-electrons take place in presence of strong field [ Fe(H 2 O)6 ]2+ : Number of unpaired electrons = 4
ligands such as CO or CN–.
In [Cr(CO)6] molecule, 12 electrons are contributed by CO [ Ni(H 2O)6 ]2+ : Number of unpaired electrons = 2
group and it contains no odd electron. 76. (a) [Ni(NH3)6]2+
6 2
68. (a) Co3+ : Ni 2+ = 3d 8 , according to CFT = t 2g eg therefore,
hybridisation is sp3d 2 and complex is paramagnetic.
[Co(CN)6]3– :
77. (a) [Fe (CN)6]3–
CN– is a strong field ligand and it causes pairing of Iron is in + 3 oxidation state.
electrons as a result, number of unpaired electrons in Co3+
becomes zero and hence it has lowest value of Fe3+ =
paramagnetic behaviour. 3d 4s
COORDINATION COMPOUNDS 665

In the presence of a strong field ligand CN–,


Fe2+ d6 4
[Fe (CN)6]3– = 14444
42444443
d2sp3 Co2+ d7 3
Hence, this is a paramagnetic complex with d2sp 3 Minimum paramagnetic behaviour = [Co (H2O)6]2+
hydridization. 88. (d) Number of unpaired electrons in [MnCl 6]3– and
78. (a) [Co(NH3)5CO3]ClO4. Six monodentate ligands are [Mn(CN)6]3– respectively are 4 and 2
attached to Co hence C. N. of Co = 6; \ Magnetic moment will respectively be 4.8 and 2.8
O. N. = x + 5 × (0) + 1 × (–2) + 1× (–1) = 0 \ x = + 3 ; [MnCl6]3– is sp3d2 hybridised and [Mn(CN)6]3– will be
electronic configuration of Co3+[Ar] 3d64s0 hence, number d2sp3 hybridised.
of d electrons is 6. All d-electrons are paired due to strong
89. (b) Since Cr3+ in the complex has unpaired electrons in
ligand hence, unpaired electron is zero.
the d orbital, hence it will absorb visible light and will be
79. (c) CO is a strong field ligand and for strong field ligands coloured.
Do > P.
Ti = [Ar]3d 2 4 s2 ; Ti4 + = 3d 0
80. (a)
81. (a) Triethoxyaluminium has no Al – C linkage Cr = [Ar] 3d 5 4s1; Cr3+ = 3d 3
10
Zn= [Ar] 3d 4s ; 2 Zn2+= 3d 10
O - CH 2 CH 3 Sc = [Ar] 3d 1 4s2; Sc3+ = 3d 0
Al O - CH 2 CH 3 90. (a) Structure of [Co2(CO)8]
O - CH 2 CH 3 O

82. (d) Due to some backbonding by sidewise overlapping C CO


OC
between d-orbitals of metal and p-orbital of carbon, the
Fe–C bond in [Fe(CO)5] has both s and p character. OC Co Co CO
83. (d) [Fe(CO)5] (Z=26) O. S. of Fe is zero . Electronic OC C CO
configuaration is [Ar] 3d6, 4s24p0. After pairing of electrons
of d and s orbitals, we have one d atomic orbital empty. C.
O
N. is 5 so hybridisation is dsp 3 which is trigonal Total M – C bonds = 10, Total M – M bonds = 1
bipyramidal. 91. (b) CN– is a strong field ligand as it is a pseudohalide ion.
84. (b) Complexes of [Ma3B3] type shows fac– meridional These ions are strong coordinating ligands and hence have
isomerism. the tendency to form s-bond (from the pseudo halide to
Solvate isomerism is refered to as a hydrate isomerism the metal) and p-bond (from the metal to pseudo halide)
when solvent is water. 92. (c) [CO(Ox)2 (OH)2]2–
85. (d) All the statements are correct. If oxidation state of Co is x, then:
x + 2(–2) + 2(–1) = –2
p p*
x = +4
s 4+
CO =
M CºO
3d 4s
p Ox2– and OH–, both are weak field ligands. Thus, the 3d
p*
electrons will remain unpaired and n = 5. This complex will
Synergic bonding
show highest paramagnetic behaviour.
86. (a) Both Assertion and Reason are true and Reason is 93. (b) [Co(C2O4 )3]3– is dimagnetic as oxalate is a strong
the correct explanation of statement-1. [Sc(H2O6]3+ has no ligand causing pairing of 3d electrons in Co3+ thereby
unpaired electron in its d subshell and thus d–d transition leading to d2sp3 hybridisation.
is not possible whereas [Ti(H2O)6]3+ has one unpaired 3d 4s 4p
electron in its d subshell which gives rise to d–d transition Orbitals of
to impart colour. Co3+ ion
3d 4s 4p
87. (c) Cr2+ d4 4 d 2sp 3 hybridised
oribitals of Co3+
Mn2+ d5 5
d 2sp3 hybrid
EBD_8350
666 CHEMISTRY

94. (c) [Mn(CN)6] 3– and [Fe(CN) 6]3– are inner orbital of C – O bond). Hence, the C – O bond would be strongest
complexes and paramagnetic while [Co(C 2O4)3]3– is in [Mn(CO)6]+.
diamagnetic in nature. 102. (b) CO bond strength is reciprocal to the extent of back
95. (d) donation involved in synergic bonding.
96. (d) CN– is a strong field ligand and form low spin 103. (d)
complexes thus Do > P. 104. (b) CN– is coordinated to cobalt as the ligand and
coordinated compounds have different properties than
97. (b) [Ti(H 2 O)6 ]3+ gives violet colour if light corresponding
the individual species.
to the energy of blue-green region of wavelength 498 nm 105. (c) The chlorophyll molecule plays an important role in
is absorbed by it. photosynthesis, contain porphyrin ring and the metal Mg
Irrespective of prediction of crystal field theory; on the not Ca.
basis of experimental observation shows that anionic
ligands are found at the low end of the spectrochemical 106. (b) For the reaction of the type M + 4L ‡ˆˆ ˆˆ† ML4 ,
series. larger the stability constant, the higher the proportion of
98. (d) Crystal field splitting energy increases with increase ML4 that exists in solution.
in ligand field strength i.e., with increase in no. of ‘en’ 107. (b) b = K1K2K3K4
groups and wavelength of absorbed light decrease with log b = log (K1 K2 K3 K4)
increase in ligand field strength log b = logK1 + logK2 + logK3 + logK4
\ [Ni(H2O)4(en)]2+(aq) will absorb light of higher logK3 = 11.9– (3.20 + 2.0 + 4.0)
wavelength i.e., Red. logK3 = 2.7
[Ni(en)3]2+ will absorb light of lower wavelength i.e., blue- 108. (a) 109. (a)
green and [Ni(H2O)4(en)2]2+ will absorb yellow orange 110. (d) Overall stability constant = K1 K2 K3
light.
99. (d) d 5 –––– strong ligand field [ XY ] ´ [ XY2 ] ´ [ XY3 ]
=
[ X ][ Y] [ XY][ Y] [ XY2 ][ Y ]
[ XY3 ]
t2g eg = = Formation constant for the reaction
[ X ][ Y]3
m = n(n + 2) = 3 = 1.73 B.M.
ˆˆ† XY
X + 3Y ‡ˆˆ
d 3–– in weak as well as in strong field 3
1
Dissociation constant =
Formation Constant
t2g eg
Exercise - 2
m = 3(5) = 15 = 3.87 B.M. 1. (0.04) When excess of AgNO3 and BaCl2 are added to
d 4– in weak ligand field solution X.
[Co(NH3 )5 Br]Cl2 + 2AgNO3 ®
1mol 2mol
t2g eg 0.02mol

m = 4(4+2) = 24 = 4.89 [Co(NH3 )5 Br](NO3 ) 2 + 2AgCl(ppt.)(Y)


1mol 2 mol
d 4– in strong ligand field 0.02 ´ 2 = 0.04 mol
1.435
2. (1) Moles of AgCl = = 0.01; 50 mL of 0.2 M
143.5
t2g eg complex º 0.01 mol
m = 2(4) = 8 = 2.82. [Co(NH3 ) 4Cl 2 ]Cl ® [Co(NH3 ) 4Cl 2 ]+ + Cl -
100. (b) d 6 – t2g 2, 2, 2 eg 0,0 (in low spin) Cl
12 Br
C.F.S.E = – 0.4 × 6D0 + 3P = - D + 3P
5 0 3. (4) en Co
101. (a) As positive charge on the central metal atom
increases, the less readily the metal can donate electron Br
density into the p* orbitals of CO ligand (donation of
electron density into p* orbitals of CO result in weakening Cl
COORDINATION COMPOUNDS 667

Br (ii) Cl (chlorido; di prefixed to represent two ligands.)


Cl The oxidation number of platinum in the compound is 2.
Hence, correct IUPAC name is
en Co Diamminedichloridoplantinum (II)
Cl 3. (a) NCS (thiocyanate) can bind to the metal ion in two
ways :
Br M ¬ NCS or M ¬ SNC
Cl Cl Thus, coordination compounds [Pd(C6H5)2 (SCN)2] and
[Pd(C6H5)2 (NSC)2] are linkage isomers.
Cl Br
4. (d) Isomers are the compounds having same molecular
en Co en Co formula but different structural formula. [Co(SO4)2 (NH3)5]
Br and [Co(SO4)(NH3)5]Cl do not have same molecular
Br Cl formula. Hence, they are not isomers.
Br Br 5. (a) Chelating ligand has two or more donor atoms to a
single metal ion e.g.,
4. (1) [Co(NH3)3 Br3] shows optical isomerism.
5. (5) Since H2O is a weak ligand, it will not cause pairing of NH2
electrons in the metal ion Mn 2+ . Thus electronic O NH2 CH2
configuration of the metal (Mn 2+) in the complex will be
2+ C O CH2
25 Mn :1s 2 2s 2 2 p 6 3s 2 3 p6 3d 5 CH2
i.e. 5 unpaired electrons. C O C O NH2
6. (0) d 6 in high spin octahedral complex
O
eg O
Oxalato Glycinato Ethane-1, 2 diamine
t 2g Here (¬) denotes binding site.
CFSE = [0.6 × 2] + [–0.4 × 4] = 0 S
7. (2) (i) & (ii) are optically inactive. || .. –
|| |....–
S O
.. ..
8. (6) The C. N. of Fe(II) in oxyhaemoglobin is 6.
9. (3) O .. O |
10. (4) Complex No. of electrons No. of unpaired ( )
Thiosulphato S2O32- is not a chelating ligand because
in outer d electron (s)
geometrically it is not favourable for S2 O32- to chelate a
orbital
metal ion.
3– 5 –
[Fe(CN)6 ] 3d 1 (CN causes 6. (b) The compounds having same molecular formula but
pairing of differ in ligands inside & outside the coordination sphere
electrons) are solvate (hydration) isomers.
[Co(NH3 )6 ]
3+
3d
6 – Coordination compound [Cr(H2O)6]Cl3 and [Cr(H2O)5Cl]
3– 6 –
H2O × Cl2 are solvate isomers, because water is exchanged
[Co(oxal.)3 ] 3d by chloride ion. This is why both of them show different
[Ni(H2O)6]
2+
3d
8 2 colour on exposure to sunlight.
[Pt(CN)4]
2–
5d
8 –
D (CN causes 7. (c) Ligands (based on priority) present are :
pairing of (i) NH3
electrons) (ii) ClS
[Zn(H2 O)6 ]
2+
3d
10 – (iii) NOS2
So, IUPAC name will be Diamminechloridonitrito-N-
Thus L, M, O and P are diamagnetic.
platinum (II).
Exercise - 3 8. (c) Strength of ligand increases as crystal field splitting
energy increases.
1. (b) Higher the value of log K, higher will be stability of Ligand strength order : CN– > NH3 > H2O
complex compound formed. As energy separation increases, wavelength decreases.
2. (a) Ligands present in the compound are Thus, the correct order is :
(i) NH3 [Co(H2O)6]3+ > [Co(NH3)6]3+ > [Co(CN)6]3–
EBD_8350
668 CHEMISTRY

9. (b) In above reaction, when 0.1 mol CoCl3 (NH3)5 is thus lower frequency. Conversely, stronger field ligands
treated with excess of AgNO3, 0.2 mol of AgCl are obtained create a larger D, absorb light of shorter l and thus higher
thus, there must be two free chloride ions in the solution v i.e. higher energy.
of electrolyte as one mole of AgNO3 precipitates one mole Red < Yellow < Green < Blue
of chloride. So, molecular formula of complex will be l = 650 nm 570 nm 490 nm 450 nm
[Co(NH3)5 Cl] Cl2 and electrolytic solution must contain So order of ligand strength is
[Co(NH3)5Cl]2+ and two Cl– as constituent ions. Thus, it L1 < L3 < L2 < L4
is 1 : 2 electrolyte.
18. (b) The complex [CoCl(NH3 )5 ]+ decomposes under

éëCo( NH3 )5 Clùû Cl2 ® éëCo( NH3 )5 Clùû (aq) + 2Cl (aq)
-
acidic medium, so
10. (d) 1 mole of AgNO3 precipitates one free chloride ion ® Co 2+ + 5NH 4 + + Cl -
[CoCl(NH3 )5 ]+ + 5H + ¾¾
(Cl–). Here, 3 moles of AgCl are precipitated by excess of
AgNO3. Hence, there must be three free Cl– ions. 19. (d) CoCl3 . 3NH3 will not give test for chloride ions with
So, the formula of the complex can be [Cr(H2O)6]Cl3 silver nitrate due to absence of ionisable chloride atoms.
11. (c) Ligand which chelates the metal ion are known as CoCl3 . 3NH3 Þ [Co(NH3)3 Cl3]
chelating ligand. Here, only oxalate ion is a chelating AgNO 3
[Co(NH3)3 Cl3] ¾¾¾¾ ® no ppt
ligand. Hence, it stabilises coordination compound by
20. (a) [M (en)2 (C2O4)] Cl
chelating Fe3+ ion.
12. (a) [Cr(H2O)4Cl2]+ shows geometrical isomerism. The C2 O24 - = bidentate ligand, carry – 2 charge
possible geometrical isomers are en = bidentate ligand, carry 0 charge
Cl Cl \ M carry + 3 charge; coordination number = 6
H2O OH 2 H 2O Cl \ Sum = + 3 + 6 = 9
Cr Cr 21. (d) Hexacyanidoferrate (III) ion.
22. (c) [Co (en)2 Cl2] Cl
H2O OH2 H2O OH2
Cl H2O Cl N
trans-isomer cis-isomer N N N N
13. (c) Relation between CFSE of octahedral and tetrahedral Co and Co
N N N Cl
4
complexes is D t = Do Cl Cl
9 trans cis
According to question, Do = 18,000 cm–1 (I) (II)
4 4
\ Dt = D o = ´ 18, 000cm –1 Mirror image of (II)
9 9 N
= 4 × 2,000 cm–1 = 8,000 cm–1
N N
14. (b) Ligand should have a pair of electron which is loosely Co
held and form a M – L bond. Cl N
e.g., :N O, NH2CH 2CH 2NH 2, :CO,
:

:
:

Cl
||

+ 23. (d) In [Co(CN)6]–3, O.N. of Co is +3


Amongst the species given NH 4 does not have any pair
\ Co+3 = 3d 6 4s0
of electron. CN– is a strong field ligand
So, it is not expected to be a ligand. \ Pairing of electrons occurs so in this complex no
15. (b) Due to d 5 configuration CFSE is zero. unpaired electron is present and it is low spin complex.
16. (b)
24. (c) Zn2 [Fe (CN)6] = Zn2+ [Fe (CN)6]4–
17. (b) B Zn2+ = [Ar] 3d10
V G Fe2+ = [Ar] 3d6
In the presence of strong field ligand CN–, all 6 electrons
of 3d electrons will pair. Thus, the compound does not
have any unpaired electrons and the colour will be white.
R Y 25. (d) Square planar complexes of type M[ABCD] form three
O isomers. Their position may be obtained by fixing the
position of one ligand and placing at the trans position
For a given metal ion, weak field ligands create a complex any one of the remaining three ligands one by one.
with smaller D, which will absorbs light of longer l and
COORDINATION COMPOUNDS 669

HOH 2N Cl HOH 2N
NH3 [Co(NH3 )4 Cl2 ] Cl ¾¾¾¾
AgNO3
®1 mole of AgCl
Pt Pt Complexes are respectively [Co(NH 3 ) 6]Cl 3 ,
[Co(NH3)5Cl]Cl2 and [Co(NH3)4Cl2]Cl
py NH3 py Cl
trans cis 31. (d) The order of the ligand in the spectrochemical series
HOH 2N
is:
NH3 H2O < NH3 < en
Pt Thus, wavelength absorbed will be in the opposite order
i.e., [Co(en)3]3+, [Co(NH3)6]3+, [Co(H2O)6]3+
Cl py
cis 32. (b) In the complex [Mn(CN)6]3–, O.S. of Mn is + 3
26. (c) [Fe(CO)4]2– E.C. of Mn3+ ® 3d 4
Since metal atom is carrying maximum –ve charge therefore 3d 4s 4p
it would show maximum synergic bonding as a resultant
C—O bond length would be maximum.
27. (d) Optical isomerism occurs when a molecule is non- The presence of a strong field ligand CN– causes pairing
super imposable with its mirror image hence the complex of electrons.
cis-[Co(en)2Cl2]Cl is optically active.
+ +
en en
d 2sp 3
Cl Cl
As, coordination number of Mn = 6, so it will form an
Co Co octahedral complex.
\ [Mn(CN)6]3– =
Cl Cl
en en
´´ ´´ ´´ ´´ ´´ ´´
cis-d-isomer Mirror cis-l-isomer

28. (d) Complex d 2sp 3

Configuration Metal ion Magnetic 33. (a) Grignard's reagent (RMgX) is a s-bonded
organometallic compound.
moment
Molarity ´ Volume(mL)
m= 34. (d) Moles of complex =
n(n + 2) 1000
(a) [Cr(H2O)6]2+ Cr2+ d4 100 ´ 0.1
24 = = 0.01 mole
1000
(b) [Fe(H2O)6]2+ Fe2+ d6 24 Moles of ions precipitated with excess of
(c) [CoCl4]2– Co2+ d7 15 1.2 ´ 1022
AgNO3 = = 0.02 moles
(d) [Mn(H2O)6]2+ Mn 2+ d 5 35 6.02 ´ 1023
Since (a) and (b), each has 4 unpaired electron they will 0.01 × n = 0.02
have same magnetic moment. \ n=2
It means 2Cl– ions present in ionization sphere
29. (b) In a solution containing HgCl2, I2 and I–, both HgCl2
\ complex is [Co(H2O)5Cl]Cl2.H2O
and I2 compete for I–.
35. (b) [Fe(CO)5]
Since formation constant of [HgI 4] 2– is very large
(1.9 × 1030) as compared with I–3 (Kf = 700) EAN = Z – O.N. + 2(C.N.) = 26 – 0 + 2(5) = 26 + 10 = 36
\ I– will preferentially combine with HgCl2. Only one central metal atom/ion is present and it follows
EAN rule, so it is mononuclear.
HgCl2 + 2I– ® HgI2 ¯ + 2Cl–
36. (a) In the given complex, the CN of Co is 6, and the
Red ppt
complex has octahedral geometry.
2-
HgI2 + 2I- ® [ HgI 4 ] en
Cl
soluble

30. (b) [ ] AgNO3


Co(NH3 )6 Cl3 ¾¾¾¾ ® 3 mole of AgCl en Co en Co en
Cl
[Co(NH3 )5 Cl] Cl2 ¾¾¾¾
AgNO3
® 2 mole of AgCl Cl Cl
trans (optically inactive) cis (optically active)
EBD_8350
670 CHEMISTRY

37. (b) Ni(28) : [Ar]3d8 4s2


42. (a)
Q CO is a strong field ligand, so unpaired electrons get
paired. Hence, configuration would be:

sp3–hybridisation

CO CO CO CO

For, four ‘CO’ligands hybridisation would be sp3 and thus


the complex would be diamagnetic and of tetrahedral
geometry.
38. (b) NH3 –

Br NH3 Br
H3N NH3 Here, both the complexes (A) and (B) are paramagnetic
Co3+ 3+
Br– NH3 Co with 3 unpaired electrons each. Also H2O is a weak field
H3N NH3
NH3 – ligand which causes lesser splitting than NH3 which is
cis
Br
comparatively stronger field ligand. Hence, the (D0) value

–NH3 +Br trans of (A) and (B) are calculated from the wavelengths of

–NH3 +Br light absorbed and not from the wavelengths of light emit-

– Br NH3 NH3 ted.

Br NH3 Br– NH3 Br NH3 43. (d) According to spectrochemical series, the order of
3+
– Co –
Co3+ – Co
3+
ligand field strength is
Br NH3 Br Br– Br Br

– – – 2– – 2–
NH3 NH3 NH3 I < Br < SCN < Cl < S < F < OH < C2O4
Facial Meridionial Only meridionial 4–
< H2O < NCS < EDTA < NH 3 < en < CN < CO
4- + 4-
39. (b) K 4 éë Fe ( CN )6 ùû ¾¾
® 4K + [Fe(CN) 6 ]
So, correct order is SCN - < F- < C2O 42- < CN -
x–6=–4 44. (a) Electronic configuration of Cr2+ – [Ar] 3d4
x=+2 n=4
Fe: 4s2 3d6
m n = n(n + 2)
Fe2+ : 3d6
eg \ mn = 4(4 + 2) = 24 BM = 4.9 BM
45. (a) [Pt (NH3)2Cl(NH2CH3)]Cl
t2g
In spherical field Diamine chlorido (methanamine) platinum (II) chloride
In the presence of

6CN (strong field ligand) 46. (b) The covalant character of the bonding (M – C s and
M – C p bonding) which exists between the metal and the
carbon atom of the CO can only be explained by the
molecular orbital theory.
40. (c)
Exercise - 4

No plane of symmetry or centre of symmetry 1. (b) [CoBr3(H2O)3] It does not exhibit hydrate isomerism.
Hence it is optically active. 2+
41. (a) Cr3+ has d3 configuration and forms an octahedral 2. (b) X = éë Ni ( en ) ( NH3 ) 4 ùû neither G.I. nor O.I.
inner orbitals complex. 2+
Y = éë Ni ( en ) 2 ( NH3 )2 ùû G.I. as well as O.I.
The set of degenerate orbitals are ( d xy , dyz and dxz ) and
( dx 2 - y 2 and d 2 ). Z = [ Ni ( en )3 ]2 + does not show G.I. while show O.I.
z
COORDINATION COMPOUNDS 671

3. (a) Magnetic moment indicates that there are three 3d 4s 4p


unpaired electrons present in chromium. These must be
present in lower energy orbitals which are 3dxy, 3dyz and
3dxz.
d2 sp3
4. (b) The two possible isomers for the given octahedral
complex are [M(NH3)5 SO4] Cl and [M(NH3)5 Cl] SO4. They Þ low spin complex µ = 1.732 BM
respectively give chloride ion (indicated by precipitation 8. (b) [Ti(H2O)6]Cl4
Coordination number 6 Þ octahedral complex
with BaCl2) and SO42– ion (indicated by precipitation with
Ti is in +4 oxidations state Þ no unpaired electrons
AgNO3). Hence, the type of isomerism exhibited by the
Þ magnetic moment = 0 B.M.
complex is ionization isomerism. 9. (b) Ni2+ (aq) with DMG (dimethylglyoxime) form square
5. (c) [Co ( CO )4 ] ® Paramagnetic, dsp2 (square planer) planar complex which is optically inactive. While
[Co(EDTA)]–, [Zn(gly)2]2+ and [Pt(en)3]4+ are optically
active as they do not have P.O.S.
6. (c) Complex No. of electrons No. of unpaired 10. (c) As number of stable rings increases stability of
in outer d electron (s) complex compound also increases due to chelation effect.
orbital Number of rings in [Ni(en)2]2+ = 2
3– 5 – Number of rings in [Ni(dmg)2] = 4
[Fe(CN)6 ] 3d 1 (CN causes
11. (a) Symmetrically filled t2g and eg are those, which contain
pairing of electrons)
equal distribution of electrons.
3+ 6 0
[Co(NH3 )6 ] 3d [FeF3]3– º º eg ì equal distribution of
ïelectrons in orbitals
ü
ï
[Co(oxal.)3 ]
3–
3d
6 0 í ý
º º º ït2 g and eg
t2 g î þï
2+ 8 2
[Ni(H2 O)6 ] 3d
2– 8 –
[Pt(CN)4 ] 5d 0 (CN causes [Mn(CN)6]4– eg unequal
pairing of electrons) distribution
2+ 10
t2 g of electrons
[Zn(H2 O)6 ] 3d 0
in orbitals

Thus L, M, O and P are diamagnetic. [CoF6]3– eg


7. (c) Fe3+ in[Fe(CN)6]3– unequal
t2 g distribution
3d 4s 4p of electrons

[Co(NH3)6]2+ eg unequal
distribution
d 2 sp3 t2 g of electrons
Þ low spin complex µ = 1.732 BM in orbitals
Ni in [Ni(CO)4] 12. (c)
4p (a) Geometry of complexes ® Both square planer
3d 4s
(b) Hybridisation of central metal cation ® dsp2
µ=0
(c) Magnetic behaviour [Cu(en)2)]2 ® Paramagnetic;
sp3 [Ni(dmg)2] ® Diamagnetic
(d) Number of stereoisomers = 0
Cr3+ in [Cr(NH3)6]3+ 13. (c) In both states (paramagnetic and diamagnetic) of the
3d 4s 4p given complex, Ni exists as Ni 2+ whose electronic
configuration is [Ar] 3d 84s0.
3d 4s 4p
2 3
d sp Ni2+ :
Þ low and high spin complex is applicable for d4 to d7 sp3
configuration
In the above paramagnetic state, geometry of the complex
Mn2+ in [Mn(CN)6]4– is sp3 giving tetrahedral geometry.
The diamagnetic state is achieved by pairing of electrons
in 3d orbital.
EBD_8350
672 CHEMISTRY

3d 4s 4p

dsp2
Thus the geometry of the complex will be dsp2 giving square planar geometry.

14. (a)
M.M. No. of unpaired electron Oxidation state
3d
Complex (A) 4.89 BM 4 Co ® 3d 6 +3

3d
Complex (B) 3.87 BM 3 Co ® 3d 7 +2

3d
Complex (C) 0.0 BM 0 Co ® 3d 6 +3

15. (d) In complexes [Rh(H2O6)]3+ and [Co(H2O)6]3+, central metal cations have same oxidation state as well as same ligands
and they fall in same group, but D0 of [Rh(H2O)]63+ > D0 of [Co(H2O)]63+ because Rh3+ has high zeff value than Co3+.
24 Haloalkenes and Haloarenes

Exercise - 1 2°
9. (a) In isopropyl chloride CH3 - CH - CH3 , chlorine atom
|
1. (b) CH 2 Cl CHCl 2 Cl
| | is attached to 2° carbon atom.
CH 2 Cl CH3
10. (d)
(vic-dihalide) (gem-dihalide)
UV light
2. (b) In alkyl halides halogen atom(s) is attached to sp3 11. (b) RH + Cl 2 ¾¾¾¾® RCl + HCl
hybridised carbon atom. Oxidation No. of Cl is decreasing from 0 (in Cl2) to –1
3. (c) In allylic halides hydrogen atom is bonded to sp3 (in RCl)
hybridised carbon atom. Whereas in vinylic halide,
12. (d) Ethylene dichloride can be prepared by adding HCl
hydrogen atom is bonded to sp2 hybridised carbon atom.
CH 2 - CH 2 to ethylene glycol (CH2OH. CH2OH).
CH3CHCl2
| |
Ethylidene chloride 13. Pyridine RCl + SO ­ + HCl­
(c) R – OH + SOCl2 ¾¾¾¾¾
Cl Cl ® 2
(gem-dihalide) Ethylene dichloride (vic-dihalide)
SO2 and HCl being gases escape leaving behind pure alkyl
Cl halide.
1 14. (c)
2 +
4. (c) NH2 N º NCl– Cl
3 Br
HNO /HCl Cu Cl
5. (c) The compound is C3H 6Cl 2 and the number of ¾¾¾¾®
2
¾¾¾¾¾¾¾®
2 2
(Diazotisation) Sandmeyer’s reaction
possible isomeric compunds is 5
asymmetric carbon atom
acetone
H H H 15. (b) R-X + NaI ¾¾¾¾
® R -I + NaX ¯
H H H
| | | | | | Soluble in
H-C-C-C-H H - C - C*- C - H R-X + NaI
(CH 3OH, Me 2CO)
¾¾¾¾
® R -I + NaX ¯
| | | | | |
Cl Cl H Insoluble in
Cl H Cl (CH 3OH, Me 2CO)
(d-l-pair)
(where X = Cl or Br)
H H H H Cl H 16. (b) This method is not applicable for the preparation of
| | | | | | aryl halides because the C–O bond in phenol has a partial
Cl - C - C - C - H H- C- C- C - H
| | | | | | double bond character and is difficult to break being
Cl H H H Cl H stronger than a single bond.

6. (d) 4-Bromobut-l-ene is not an allylic halide NO2 NO2 NH2


BrH 2 C —CH 2 —CH==CH 2
Cl Zn HONO
4 -Bromobut -1-ene ¾¾2 ® ¾¾® ¾¾®
17. (b) AlCl3 HCl Cold
1 Cl Cl
CH 3
| + –
7. (b) H 3C - 2C - Cl N2Cl Br
|
3 CH
3 Cu Br
¾2¾®
2

IUPAC name : 2-Chloro-2-methylpropane.


Cl Cl
Br
|
8. (a) CH3CH2C = CH – Cl
4 3 2
2-Bromo-1-chloro but-1-ene
EBD_8350
674 CHEMISTRY

23. (b) Chlorine atom is highly reactive so it will react with


D D D D all types of hydrogen available while the Br atom is
Cl highly selective so it will react with that hydrogen which
SO Cl / peroxide
18. (b) ¾ ¾2¾2¾ ¾ ¾ ®
free radic-al substition
* gives the highly stabilised tertiary alkyl radical so only
one product is formed.
D D D D +
(±)
recemic mixture
NH2 N2 Cl–
(X)
Na NO + HCl HBF
D D 24. (d) ¾¾ ¾ ¾2¾ ¾¾® ¾¾¾

0 - 5º diazotisation
alc. KOH
¾ ¾¾®
N2+BF4– F
D D
(±)
D
(Y) ¾¾¾¾¾¾¾® + BF3+ N2
(Balz-Schiemann
reaction)
Benzene diazonium
CH3 CH3 tetrafluoroborate

+
+

25. (c) For the same alkyl group, the boiling points of alkyl
19. (d) ¾D® Cl
D ¾® halides decrease in the order :
RI > RBr > RCl > RF
CH3 Cl This is because with the increase in size and mass of
H + H halogen atom, the magnitude of van der Waal's forces
Cl CH3 increases.
D D 26. (a) Boiling point of CH3I is 42°C which indicates that it
Carbocation produced is planar hence the nucleophile is liquid at room temperature. CH3I is larger molecule so it
attacks from either side of the plane. has stronger vander Waal’s force of attraction than others.
20. (c) Ethyl alcohol forms ethyl chloride with thionyl 27. (d) Para-dichlorobenzene has most symmetrical structure
chloride in presence of pyridine. than others. It is found as crystalline lattice form, therefore,
Pyridine it has highest melting point (52°C) due to symmetrical
CH 3CH 2OH + SOCl2 ¾¾¾¾® CH 3CH 2 Cl + SO2 + HCl structure.
21. (a) Aryl chlorides and bromides can be easily prepared
Cl
by electrophilic substitution of arenes with chlorine and
bromine respectively in the presence of Lewis acid catalyst
like iron or iron (III) chloride.
The ortho and para isomers can be easily separated due
to large difference in their melting points. Reactions with Cl
iodine are reversible in nature and require the presence of
an oxidising agent (HNO3, HIO4) to oxidise the HI formed 28. (d) The boiling points of isomeric haloalkanes decrease
with increase in branching. For example, 2-bromo-2-
during iodination. Fluoro compounds are not prepared
methylpropane has the lowest boiling point among the
by this method due to high reactivity of fluorine.
three isomers.
22. (a) Alkyl iodides are often prepared by the reaction of
alkyl chlorides/bromides with NaI in dry acetone. This CH3
reaction is known as Finkelstein reaction. |
CH3 CH 2CH 2 CH 2 Br CH3CH 2CH CH3 H3C - C - CH3
R - X + NaI ¾¾ ® R - I + NaX | |
X = Cl, Br Br Br
b . p . / K 375 364 346
NaCl or NaBr thus formed is precipitated in dry acetone.
It facilitates the forword reaction according to Le Boiling points of isomeric dihalobenzenes are very nearly
Chatelier’s principle. The synthesis of alkyl fluorides is the same. However, the para-isomers have higher melting
best accomplished by heating an alkyl chloride/bromide point as compared to their ortho and meta-isomers. It is
in the presence of a metallic fluoride such as AgF, Hg 2F2, due to symmetry of para-isomers that fits in crystal lattice
better as compared to ortho- and meta-isomers.
CoF2 or SbF3. The reaction is termed as Swarts reaction.
H 3C - Br + AgF ¾¾
® H 3C - F + AgBr
HALOALKENES AND HALOARENES 675

40. (a) A mixture of equal amounts of the two enantiomers is


Cl Cl Cl called a racemic mixture.
Cl 41. (c)
42. (a) Although all the three compounds can be used for
preparing Grignard reagents, diethyl ether is considered
Cl as the best because it provides electron pairs to Mg of the
Cl reagent fully for coordination, in case of C6H5OCH3 and
b.p/K 453 446 448 C6H5N(CH3)2 electron pair on O and N are partialy
delocalised over the benzene and hence are less available
m.p.K 256 249 323
for coordination with Mg.
29. (c) SN1 reactions involve the formation of carbocations, R O(C2H5)2
hence higher the stability of carbocation, more will be Mg
reactivity of the parent alkyl halide. Thus tertiary X O(C2H5)2
carbocation formed from (c) is stabilized by two phenyl
groups and one methyl group, hence most stable. 43. (c) Potassium ethoxide is a strong base, and 2- bromo-
pentane is a 2º bromide, so elimination raction predomi-
Dry
30. (c) CH3 CH 2 I + Mg ¾¾¾® CH3CH 2 MgI nates
ether
Ethyl magnesium iodide OC H -
CH 3 CH ( Br ) CH 2 CH 2 CH 3 ¾¾ ¾
2 ¾

31. (d) SN1 reaction involves carbocation which are planar
(sp2 hybridised) and thus can be attacked on either face of CH 3CH = CHCH 2CH 3 + CH 2 = CHCH 2 CH 2 CH 3
trans -2-Pentene (major) Pentene -1(minor)
the carbon.
32. (d) Weaker the C–X bond, greater is the reactivity. Since trans- alkene is more stable than cis. Thus
trans-pentene -2 is the main product.
33. (a) 1° Alkyl halides (having least steric hindrance at
44. (c) Wurtz reaction : It involves the interaction of two
a-carbon atom) are most reactive towards SN2 reaction.
molecules of an alkyl halide (preferably bromide or iodide)
34. (c) Isopropyl chloride, being 2° alkyl halides, can with metallic sodium in presence of dry ether to form
undergo SN1 as well as SN2 mechanism. symmetrical alkanes containing double the number of
35. (a) Due to steric hindrance tertiary alkyl halide do not carbon atoms present in the alkyl halide. For example,
react by SN2 mechanism they react by SN1 mechanism. Dry ether
R - X + 2Na + X - R ¾¾¾¾® R - R + 2NaX
SN2 mechanisam is followed in case of primary and Alkyl halide Alkane
secondary alkyl halides of Br
CH3 – X > CH3 – CH2X > (CH3)2 – CH.X > (CH3)3– C–X |
alc KOH
45. (d) CH 3 - CH - CH 2 - CH 3 ¾¾¾¾®
H
| CH 2 = CH - CH 2 - CH 3
36. (c) H3C - CH 2 - C*- CH3
| In this reaction both hydrogen and halogen atom has been
Cl removed, so it is known as dehydrohalogenation reaction.
The compound containing a chiral carbon atom (a carbon 46. (d) CH 3 CH 2 CH 2 Br + Mg ¾¾ ® CH3CH2CH2MgBr
(Propyl bromide)
atom attached to four different atoms is optically active.
37. (c) Diastereomers since they have different melting CH3CH 2CH 2 MgBr + H2O ¾¾
®
points, boiling points, solubilities etc. CH3CH2CH3 + MgBrOH
38. (a) 47. (a) CH3 CH3
39. (b) The compound has two similar asymmetric CH3 – C – Cl + 2Na + Cl – C – CH3
C-atoms. It has plane of symmetry and exist in meso form.
CH3 CH3
t-Butyl chloride
Wurtz
Rxn

Plane of symmetry
CH3 CH3 CH3 CH3
CH3 – C – C –CH2Cl ¬¾¾ CH3– C – C – CH3
Mono
meso - 2, 3 - Dichlorobutane CH3CH3 chlorination CH3 CH3
EBD_8350
676 CHEMISTRY

56. (c) Due to resonance in chlorobenzene.


Na/ether
48. (b) Br Cl ¾¾® 57. (a) N-Phenylacetanilide, C6H5N(C6H5)COCH3,
precipitates out to a complex with anhydrous AlCl3.
58. (d) More the stability of the carbocation, higher will be

Br2/hv C 2H 5O /C 2H 5OH the reactivity of the parent chloride. Allyl chloride > vinyl
¾¾® Br ¾¾¾¾¾¾ ®
D chloride > chlorobenzene.
59. (d) Due to resonance, the electron density increases more
Mg/ether at ortho- and para-positions than at meta-positions.
49. (b) Cl Br ¾ ¾ ¾ ® Cl MgBr
Further, the halogen atom because of its – I effect has
(A) some tendency to withdraw electrons from the benzene
ring. As a result, the ring gets somewhat deactivated as
D 2O compared to benzene and hence the electrophilic
substitution reactions in haloarenes occur slowly and
require more drastic conditions as compared to those in
Na/ether benzene.
D D
¾¾¾® Cl D
60. (b) SN2 reaction follow a 2nd order kinetic, i.e., the rate
(C) (B) depends upon the concentration of both the reactants,
where in SN1 reactions rate depends only upon the
50. (d) The reaction sequence is as follows
concentration of only one reactant.
CH3CH2CHO
aq. KOH
CH3CH2CHCl2
alc. KOH The order of reactivity order of alkyl halides for SN1
Aldehyde 1, 1-Dichloropropane (X)
reaction 3° > 2° > 1° and for SN2 reactions 3° < 2° < 1°
61. (a) In SN1 reactions step 1 is slow and reversible, and
Cu2Cl2/NH 3 the slowest step is the rate determining step.
red ppt. CH2 – C –
– CH + 2KCl + H2O 62. (d) Chlorination of n-butane takes place via free radical
Hydrocarbon
hn • •
51. (d) formation i.e., Cl2 ¾¾® Cl + Cl
Cl / hn
CH3 — CH2 — CH2 — CH3¾¾2¾¾®
CH = CH2
(CH3)3COK
CH3 CH3
Br (I)
H — C — Cl + Cl — C — H
OH
Br
CH – CH3 C2H5 C2H5
CH aq. NaOH
d l
CH3 Racemic mixture
Br 50% d form + 50% l form
(H)
Br
(G) CH3 Cl• may attack on either side and give a racemic mixture of
2-chlorobutane which contain 50% d form and 50% l-form.
CH – CH3 63. (a) First draw possible different structures obtained on
CH 3MgBr (1 eq)/ether
monochlorination of 2-methylbutane,
Br CH3CH(CH3)CH2CH3.

CH3 CH3
52. (c) No bond to the chiral centre has cleaved. Terefore, *
| |
the configuration of the reactant is retained. (i) ClCH 2 - C HCH 2 CH3 (ii) CH3 - C - CH 2 CH3
53. (d) Except (d) all contain abstractable proton |
Cl
Mg
54. (b) C6 H 5 Cl ¾¾¾
® C6 H 5 MgCl Optically active Optically inactive

CH 3CH 2OH
¾¾¾¾¾¾ ® C6 H6 + CH3CH 2 OMgCl CH3 Cl CH3
Anhyd. | | |
55. (d) C6 H 6 + CH3CH 2 CH 2Cl ¾¾¾¾
® *
(iii) CH3 - CH - CH CH3 (iv) CH 3 - CH - CH 2 CH2Cl
AlCl3 CH 3
| Optically active Optically inactive
C6 H 5 - CH - CH 3
Isopropyl benzene
HALOALKENES AND HALOARENES 677

Thus structures (i) and (iii) are optically active, each has
R1
one chiral carbon; so each structure will give one
enantiomeric pair; thus total enantiomeric pairs will be SOBr2 (SN2)
two. Pyridine or
H (Inversion)
64. (a) any amine R2 Br
Å 71. (b) 2CH3CH2Br + Ag2O ¾® 2 AgBr + Et—O—Et
Ionisation + Br
Me Br ¾ ¾ ¾ ¾ ® Me (X)
(A)
CH3CH2Br + AgOH or Ag2O ¾®
Me Me Me (Moist)
Ionisation
¾¾¾¾® ¬® + Br
Å AgBr + CH3CH2OH
Br 2° (resonance
(B) stabilised) (Y)
Å
H
Me Me Et—O—Et + H2O ¾¾¾® 2CH3CH2OH
Ionisation + Br
Me ¾ ¾ ¾ ¾ ® Me Å ( X) (Y)
Br
(C)
2° (no resonance) 72. (d)
Since SN1 reactions involve the formation of carbo-cation 73. (b) It is SN2 reaction, BrV will attack at less hindered site.
as intermediate in the rate determining step, more is the 74. (d) Here (b) and (c) are not chiral substrate while (a) is
stability of carbocation higher will be the reactivity of primary halide, so it is not suitable for SN1 reaction, hence
alkyl halides towards SN1 route. Now we know that only (d) can form enantiomeric pair.
stability of carbocations follows the order : 3° > 2° > 1°, so
75. (b) SH bond is weaker than OH bond, so base abstracts
SN1 reactivity should also follow the same order.
proton from SH group.
3° > 2° > 1° > Methyl (SN1 reactivity)
65. (a) A strong nucleophile favours the SN2 reaction and a Br V
OH
weak nucleophile favours the SN1 reaction. HO SH
–H2 O
First reaction is SN1 reaction because C2H5OH is used as
solvent which is a weak nucleophile. Br V
Second reaction is SN2 reaction because C2H5O– is strong HO S
nucleophile.
66. (c) For SN2 reaction polar aprotic solvent is needed. OH
¾®
67. (d) It is an example of SN1 reaction. S
Rate of SN1 µ stability of CÅ ion
68. (c) It is an example of SN2 reaction, so ring expansion ++
+++
+++ Cl ++ Cl
does not occur. Cyanide ion is ambident nucleophile and it More Less
attacks with carbon site as it is stronger base, hence (c) 76. (d) electrophilic electrophilic
will be the product, X. carbon carbon
69. (b) A good nucleophile possesses two properties
(i) It is a good leaving group Cl OCH3
(ii) It is a good attacking reagent (+R/–I) (+R/–I)
(–I is stronger than + R)
‘F’ is not a good leaving group, hence to convert R—F
into R—OH, first it is treated with NaI to convert it into 77. (c) Rate of SN2 = k [R–X] [Base]
R—I (I is good base) and then R—I is treated with NaOH
to produce R—OH (I is good leaving group) only (c) is occuring via SN2 pathway .
78. (c)
R1 R1
79. (d) CCl4 vapours are non-inflammable (i.e. do not catch
P + Br2 fire). Hence used as fire extinguishers under the name
70. (b) OH H (SN2)
or PBr3 pyrene.
H Br (Inversion)
R2 or PBr5 R2 80. (b)
R1 81. (c) Tetrachloromethane (carbon tetrachloride) is also
used as feedstock in the synthesis of chlorofluorocarbons
SOBr2 (SN1) and other pharmaceutical manufacturing and general
Br (retention)
solvents etc.
R2 H
EBD_8350
678 CHEMISTRY

82. (d) Dichloromethane is widely used as solvent as a paint In this reaction the compound obtained is fluoroform
remover, as a propellant in aerosols and as a process (CHF3).
solvent in the manufacture of drugs. It is also used as a Molecular weight of fluoroform (CHF3)
metal cleansing and finishing solvent. = 12 + 1 + (3 × 19) = 70
83. (b) Haloforms (CHX3) are trihalogen derivatives of 97. (c) Triiodomethane (CHI3) when comes in contact with
methane. organic matter decomposes easily to free iodine which has
Example : Chloroform CHCl3. antiseptic property.
oxidation
84. (a) Chloroform is an organic compound which does not 98. (a) CH3CH 2OH + Cl2 ¾ ¾¾¾¾® CH3CHO + 2HCl
ionise in water. Since it can not provide Cl–, therefore it is Ethanol Ethanal
not precipitated with AgNO3.
CH3CHO + 3Cl2 ¾¾® CCl3CHO + 3HCl
85. (b) 86. (d) Ethanal Chloral

87. (b) 2CHCl3 + O2 ¾¾¾


light
® 2COCl2 + 2HCl 2CCl3CHO + Ca(OH)2 ¾¾®
Phosgene 2CHCl3 (HCOO) 2 Ca
+ Cal. formate
2 3 Chloroform

Cl 1 4 CH3COCH3 + 3Cl2 ¾¾® CCl3COCH3 + 3HCl


Cl
|1 2 Acetone Trichloroacetone
88. (a) Cl – C – CH
| 4 2CCl3CO.CH3 + Ca(OH)2 ¾¾®
Cl 2CHCl3 + (CH3COO)2Ca
Cl 1
2 3 99. (d) Exposure of carbon tetrachloride causes liver cancer
1,1,1-trichloro-2, 2-bis(p-chlorophenyl) ethane in humans. The most common effects are dizziness, light
headedness, nausea and vomiting which cause permanent
89. (b) Carbon tetrachloride rises to atmosphere and deplete
damage to nerve cells. In severe cases, these effects can
the ozone layer. This depletion of ozone layer increases
lead rapidly to stupor, coma, unconsciousness or death.
exposure of UV rays to human being which lead to increase
100. (d) CCl4 is used as a fire extinguisher. The dense, non
of skin cancer, eye diseases and disorder with discruption
combustible vapours cover the burning substance and
of the immune system.
prevents the availability of oxygen around burning material.
90. (d) All the given points about freons are correct.
91. (d) Ethyl alcohol retards the oxidation of CHCl 3 to Exercise - 2
phosgene as well as it converts any phosgene formed to
harmless ethyl carbonate. 1. (6) Addition of HBr to 2-pentyne gives two structural
isomers (I) and (II)
COCl2 + 2C2 H 5OH ¾¾
® (C 2 H5 ) 2 CO3 + 2HCl
HBr
Ethyl carbonate CH 3 - C º C - CH 2 CH 3 ¾¾¾
®
-4KCl HO OH CH 3C(Br) = CHCH 2 CH 3 + CH 3CH = C(Br)CH 2CH 3
92. (d) CCl 4 + 4KOH ¾¾¾¾
® C (I) (II)
HO OH
Each one of these will exist as a pair of geometrical isomers.
-2H O
¾¾¾¾
2 ® CO
2 Thus, there are two structural and four configurational
isomers. Hence, total number of isomers is 6.
93. (b) CHCl3 + [O] ¾¾
® COCl2 + HCl 2. (4)
X Y

COCl2 + 2C 2 H 5 OH ¾¾
® (C 2 H 5 ) 2 CO3 + 2HCl
Z
94. (b) When treated with concentrated nitric acid, chloroform
gives chloropicrin or nitrochloroform (CCl3NO2) known as
tear gas.
95. (c) Pyrene is the technical name of CCl4 which has highest
chlorine content.
96. (a)
CHCl3 + HF ¾¾
® CHF3 + 3HCl
(Chloroform) (Hydrogen (Fluoroform) (Hydrochloric
fluoride) acid)

Total no. of products is 4.


HALOALKENES AND HALOARENES 679

3. (5) Total no. of alkenes will be = 5


Br
5. (5)
H3C –– CH2 –– CH2 –– C –– CH2 –– CH3

alc.KOH
¾¾¾®

H3C –– CH2 –– CH2 –– C == CH –– CH3

(E & Z)

or H3C –– CH2 –– CH == C –– CH2 –– CH3

(E & Z)
+

or H3C –– CH2 –– CH2 –– C –– CH2 –– CH3 Hence, 5 products will formed.

(Only 1) KCN H O+
6. (3) 3 ®
CH3 - Br ¾¾¾® CH3 - CN ¾¾¾¾
(A)

LiAlH
4 ® CH - CH - OH
CH3 - COOH ¾¾¾¾ 3 2
4. (5) (B) (C)
Ethyl alcohol

NaCN
7. (16) CH3CH2Cl ¾¾¾® CH3CH2CN + HCN
(X)
¾®

AgNO
3
Ni/H2
¾¾¾® (CH3CO)2O
CH3COOH CH3CH2CH2NH2
¾¾¾¾®
+
Cl + (Y)
(Aromatic)
CH3CH2CH2NHCOCH3
(Z)

alc. KOH Br
8. (2) C 2 H 5 I ¾¾¾¾® CH 2 = CH 2 ¾¾¾

KCN
BrCH 2 - CH 2 Br ¾¾¾® CNCH 2 .CH 2 CN
9. (2) (I) RX + AgCN ¾® RNC (correct)
(II) RX + KCN ¾® RCN (correct)
(III) RX + KNO2 ¾® RNO2 (wrong)
When haloalkanes is treated with potassium nitrite (KNO2),
alkyl nitrite is formed as major product because since the
bond between K–O is ionic in nature, the negative charge
on oxygen serves as an attacking site.
EBD_8350
680 CHEMISTRY

R–X + KNO2 ¾® RONO + KX The attack of nucleophile to the carbocation is possible


(IV) RX + AgNO2 ¾® RONO (wrong) only on stable carbocation as at room temperature only 3°
carbocation is stable.
On treating haloalkanes with silver nitrite (Ag–O–N = O),
Step I
nitroalkanes is formed because since the bond between
Ag–O is covalent, the lone pair on nitrogen acts as an CH3 CH3
attacking site for nucleophilic substitution. | | –
CH3 CH 2 C OH ® CH3 CH2 CÅ + OH
RX + AgNO2 ¾® R–NO2 + AgX | |
CH3 CH3
P+ I
10. (1) CH3CH 2OH ¾¾¾
2 ® CH CH I
3 2 Step II
(A)
Mg HCHO
¾¾¾® CH3CH 2 MgI ¾¾¾¾ CH3 CH3
®

— —

— —
Ether –
(B) +
CH3 — CH2 — C + Cl ® H3C — CH2 — C — Cl
CH 2CH3 CH 2CH3 CH3 CH3
| H2 O
|
H - C - OMgI ¾¾¾ ® H - C - OH
| | 3. (a) It is an example of Sandmeyer’s Reaction
H H +
(C) (D) NH3 N2Cl
n - propylalcohol NaNO +HCl
¾¾¾¾¾
-
2
®
273 278 K

Exercise - 3
Cl
Cu Cl
1. (b) Alchols and halogen acid react through S N 1 ¾¾¾®
2 2
+ N2
mechanism.
(Y)
CH 3 — CH 2 — CH 2 — OH ¾
®
+ 4. (b)
CH 3 — CH 2 — CH + OH -
In this case, 1° carbocation is formed. It is least stable. So, FeCl3 Å ! Å !
here SN2 mechanism is followed. In this SN2 mechanism a Cl – Cl Cl – Cl – FeCl3 Cl + FeCl4
transition state is observed in which a - carbon is linked CH3
with two nucleophilies. CH3 CH3
Å ! rds
H
CH3 — CH2 — CH — OH ¾® + Cl – Cl – FeCl3 ¾® Cl or Å
| Å

CH3 H Cl
+
CH3 — CH2 — CH + OH –
| CH3 CH3
CH3 H
2° Carbocation (more stable than 1° carbocation) + Cl or +
CH3 H Cl
|
CH3 — CH2 — C — OH ¾® ! !
| FeCl4 FeCl4
CH3
CH3 CH3
|+ CH3
H3C — CH2 — C + OH – Cl
|
CH3
3° carbocation (most stable) Cl
Greater the stability of carbocation, greater will be the As electrophile Cl+ attacks on
electron rich benzene ring
possibilities of attack of X– ion on the carbocation. Order and substitutes hydrogen on ortho and para position
of stability of carbocation is : 3° > 2° > 1. w.r.to. – CH3 group. So, the reaction is electrophilic
substitution reaction.
So order of reactivity will be C > B > A.
5. (a) Halogen exchange reactions are those reactions in
2. (d) Reaction of alcohols with conc. HCl at room which one hailde replaces another (Finkelstein reaction).
temperature follows SN1 mechanism.
HALOALKENES AND HALOARENES 681

6. (a) It is a substitution reaction which involves the 11. (b) Halides in which two halogen atoms are present on
replacement of 1° and 2° hydrogen of alkanes by chlorine. the two adjacent carbon atoms are known as vic -
It occurs in presence of ultraviolet light. dihalides.
UV ·
Step 1 : Cl — Cl ¾¾¾
® 2Cl H
light | CH 2 — CH 2
· Cl — C— Cl | |
C l + CH3 — CH 2 — CH 2 — CH3 ¾¾
® | Cl Cl
H
·
CH3CH 2CH 2 — C H 2 + HCl Dichloromethane 1, 2 - Dichloroethane

Step 2 :
· H Cl Cl
CH3 — CH 2 — CH 2 — C H 2 + Cl2 ¾¾
® | | |
H — C— C — H CH 2 — CH = CH3
· | |
CH3 — CH 2 — CH 2 — CH 2 Cl + C l H Cl
· Ethylidene chloride Allyl chloride
CH3 – CH2 – C H – CH3 + Cl2 ®
Cl
| · 12. (a) Compounds in which the halogen atom is bonded to
CH3 – CH2 – CH – CH3 – C l sp3 hybridised carbon atom next to carbon carbon-double
bond are known as allyl halides.
Step 3 :
· · CH3 CH = CHC ( Br ) ( CH3 ) 2
CH3 — CH 2 — CH 2 — C H 2 + C l ®
-
CH3CH 2 CH 2 CH 2 Cl 13. ® [ AlCl 4 ] + Cl+
(b) AlCl3 + Cl2 ¾¾
· · Cl
CH3 - CH 2 - C H - CH3 + Cl ® Cl +
| + Cl ¾®
CH3CH2 CH – CH3
Chlorobenzene

7. (a) Density is directly related to molecular mass. Higher 14. (b) If halogen atoms present on the same carbon atom
the molecular mass, higher will be the density of the then they are known as gem – dihalides or alkylidene
compound. The order of molecular mass is halides.
benzene < chlorobenzene < dichlorobenzene < 15. (c) Addition of HCl takes place in accordance with
bromochlorbenzene Markovnikov's rule.
8. (c) Greater the surface area, greater will be the boiling
point of a compound. Surface area decreases with CH2 – CH = CH2 CH2 – CH – CH3
|
increases in branching. Cl
Increasing order of boiling point + HCl ¾®
CH3
H3C 16. (b) It would prefer to undergo SN2 reaction.
H3C C CH3 < CH — CH2Br
H3C SN2 reactions occur if there is less steric crowding on a -
Br carbon of alkyl halide. In case of primary aklyl halides,
< CH3CH 2 CH 2CH 2 Br steric crowding is very less. So, it would prefer to undergo
SN2 reaction.
9. (b) Carbon atom in which all four valencies are different
is known as Asymmetrical/chiral. 17. (d) All compounds are tertiary alkyl but bond formed
10. (a) Enantiomers are the steroismers which are related to between carbon and iodine (C — I) is weakest bond due
each other as non – superimposable mirror images. to higher difference in size of carbon and iodine.
The enantiomer of molecule (A) is 18. (c) IUPAC name
Mirror 2 1
CH3 — CH — CH2 — Br
3
|
H H CH2
CH3 H3C |
C C CH3
4
H5C2 Br Br C2H5
1-Bromo-2-methylbuta ne
EBD_8350
682 CHEMISTRY

Br
reaction. In compound (iii) electron releasing group is
| present at meta position w.r.t. chlorine, so the impact is
19. (b) H3 C— H 2 C— H C — C H 2 — C H3 nothing but in compound (ii) it is present at ortho position.
1 2 3 4 5
28. (d) If electron withdrawing group is present at ortho and
IUPAC name is 3-bromopentane. para position, then the nucleophilic substitution reaction
20. (d) Toluene react with chlorine in presence of iron and in rate increases.
absence of light, by substitution on benzene ring.
29. (c) If electron releasing group is present at ortho and
CH3 CH3 CH3 para position w.r.t. to chlorine it decreases the rate of
nucleophilic substitution reaction.
Cl
Fe 30. (a) Greater the surface area, greater will be the
+ Cl2 ¾ ¾® + intermolecular forces of attraction and intermolecular
forces of attraction is directly proportional to boiling
Cl point. Surface area is larger for larger size of halogen.
Toluene o-Chlorotoluene p-Chlorotoluene 31. (d) Boiling point is directly proportional to size of the
molecule. All contains same halogen atom but different
21. (c) CH3Cl + NH3 ® CH3 NH 2 + HCl hydrocarbon part. Larger the different hydrocarbon part
Excess Methanamine
larger the boiling point.
However, if the two reactants are present in the same
32. (c) Methyl group is ortho para directing but due to steric
amount, then the mixture of amines (i.e., primary, secondary
hindrance effect, generated by two CH3 groups substitution
and tertiary) are obtained.
will not take place on position (I). Hence only two products
22. (a) Carbon in which four bonds are different is known as
are possible.
Chiral carbon.
1 2 3 4 CH3
H3C — CH — CH2 — CH3
| III I
Br
2- Bromobutane
II
CH3
Å
23. (a) In C6H5CH2Br carbocation is C6 H5 C H 2 which is i.e.,
stable due to resonance. CH3 CH3 CH3
24. (b) If carbon atom has all four valencies with four different Br
groups then it is called as asymmetric/chiral carbon. Br
¾ ¾2® +
25. (a) All those compounds which follow SN1 mechanism FeBr2
during nucleophilic subsitution reaction will give racemic CH3 CH3 CH3
mixture. Br
(A) (B)
Order of reactivity of alkyl halides for SN1.
33. (d) Out of the given four compounds only (iv) compound
3° > 2° > 1° CH3 X
is chiral and hence only this compound will undergo
Thus, CH 3 — CH — Br contains a 2° carbon so gives a racemisation.
|
C2 H5 34. (b) Steric congestion around the carbon atom undergoing
the inversion process will slow down the SN2 reaction,
racemic product.
hence lesser the congestion, faster will be the reaction. So,
26. (c) The substitution is faster, if the electron withdrawing the order is
group is at ortho and para position, because electron CH3Cl > CH3CH2Cl > (CH3)2CH Cl > (CH3)3CCl
density is high at these positions as chlorine is electron
donating group which increase electron density at ortho 35. (a) Tertiary butyl chloride will give the most stable tertiary
and para position. Therefore, it has partial double bond carbonium ion among the other given compounds
character, and is not easy to break.
In compound (ii) and (iii) both has one electron H3C ! H3C
– Cl +
withdrawing group but in compound (ii) electron C Cl ¾¾® C
withdrawing (— NO2) group is present at ortho position, H3C H3C
so rate of reaction in compound (ii) is more than that of CH3 CH3
t-butyl carbonium ion
(iii) while (i) has no electron withdrawing group.
27. (d) If electron releasing group is present at ortho or para In general cases benzyl carbocation is more stable than all
position, it decreases the rate of nucleophilic substitution tertiary carbocation but in case of t-butyl carbocation it is
HALOALKENES AND HALOARENES 683

more stable than benzyl carbocation due to


Br H
hyperconjugation while benzyl carbocation is stabilised
by conjugation with benzene ring only, so t-butyl C6H5
carbocation is more stable. C6H5 ¾ ¾t-BuOK
¾¾¾®
1, 2 elimination
H
36. (b) In SN1 reaction, carbocation, a planar species as H –
intermediate is formed. OtBu

R1 C 6H 5
Nu
C 6H 5 + t-BuOH + Br–
Å
C R3 42. (b) Since S N1 reactions involve the formation of
carbocation as intermediate in the rate determining step,
Nu
R2 more is the stability of carbocation higher will be the
So attack from below or above the plane can take place. reactivity of alkyl halides towards SN1 route.
If 50% attack below and above the plane of carbocation Hence correct order is II < I < III
take place than 100% racemization occurs but it may not
be highly probable. CH3 CCl3
i.e. if inversion occurs more than retention leading to partial
racemization. 43. (a) 3Cl Br
¾¾®
D
2
¾¾®
2
Fe
37. (c) Four different Substituent, only one chiral centre.
Hence only enantiomers are possible.
CCl3 CH3
38. (b) Alkyl fluorides are more conveniently prepared by
heating suitable chloro – or bromo-alkanes with organic
Zn
fluorides such as AsF3, SbF3, CoF2, AgF, Hg2F2 etc. This ¾¾®
HCl
reaction is called swarts reaction. Br Br
m-Bromotoluene
CH 3Br + AgF ¾¾
® CH 3 F + AgBr
OMe
2CH3CH 2 Cl + Hg 2 F2 ¾¾
® 2CH 3CH 2 F + Hg 2Cl 2 44. (b) ¾¾¾® + NaBr + MeOH
MeOH
39. (b) (1) CH3CH2CH2–Br + KOH
¾® CH3CH=CH2 + KBr + H2O (dehydrohalogenation reaction)
45. (a) Mechanism involved for the given reaction is:
This is dehydrohalogenation reaction which is an example
of elimination reaction.
(2) H3C CH3 H3C CH3
C + KOH ® C + KBr
Br OH
– –
Replacement of Br by OH is substitution reaction thus it
is a nucleophilic substitution reaction.
Br
(3) + Br2 ¾®
Br
1 2 3 4 5
Elimination
Above reaction involves addition of Br2 across double 46. (d) C H3 - C H - CH 2 - C H 2 - C H3 ¾¾¾¾¾®
|
bond. Thus it is called addition reaction. Br
40. (b) Compound (b) is optically active due to absence of 2-Bromo-pentane
plane of symmetry and center of symmetry. CH3 - CH = CH - CH 2 - CH 3
41. (b) Elimination reaction is highly favoured if Mechanism :
a b
(a) Bulkier base is used CH3 – CH – CH – CH2 – CH3
(b) Higher temperature is used
Hence in given reaction biomolecular elimination reaction Br H
provides major product.
Base
CH3 – CH = CH – CH2 – CH3
EBD_8350
684 CHEMISTRY

It is an b elimination reaction as b hydrogen is abstracted O O


and results more substituted alkene, which fallows
Zaitsev’s rule. 2. (b) CF3 S O > Ph S O > CH3 COO > PhO
Since ‘H’ and ‘Br’ are removed, it is dehydro-halogenation.
O O
47. (b) Wurtz reaction is used to prepare symmetrical alkanes L.G. Ability
like R1 – R1, as
3. (c) SN2 and SN1 same, if CÅ not rearrange.
Dry ether
R1 - X + 2Na + X - R1 ¾¾¾¾
® R1 - R1 + 2NaX

d+
d MgBr D
If R1 and R2 are different, then mixture of alkanes may be 4. (c) ¾ ¾®
obtained as
Dry ether Br
R1 - X + 2Na + R 2 - X ¾¾¾¾
®
Heat favours elimination.
R1 - R1 + R1 - R 2 + R 2 - R 2 + 2NaX
5. (d) Solvolysis is SN1 reaction.
Hence, n-heptane cannot be made in good yield using
6. (a) In finkelstein reaction inversion in configuration
Wurtz reaction, since it is unsymmetrical alkene.
occurs
conc.H 2SO 4
48. (c) (A) (CH3)2CCH(OH)CH3 ¾¾¾¾¾¾ ®
CH3 CH3
CH3

CH3 — C == C — CH3
H Br I I H
H CH3 Inversion I CH3

CH3
Br H
alc.KOH
(B) (CH3)2CHCH(Br)CH3 ¾¾¾¾®
CH3
CH3
CH3 I
CH3 C CH CH3
Or
– +
t - BuO K
(C) (CH3)2CHCH(Br)CH3 ¾¾¾¾¾ I H
®
CH3 CH3
CH3 CH CH CH2
Due to bulky nature of tertiary butoxide, the least hindered
hydrogen is eliminated. Therefore, Hoffman product is
formed.
CH3 SN 1
D
(D) (CH3)2 C CH 2 CHO ¾¾
® CH3 C CH CHO 7. (b)
CH—Br CH—CH3
OH
CH3
Exercise - 4

1. (d) It is the case of SN1 reaction, Ring


expansion
Br CH—CH3 CH3 CH3
V
–Br H H

H 2O HO H3C
CH3 CH3

CH3OH –H
CH3 OH
Diastereomers

8. (b) In (d) SN2 occurs, in (a) substrate is optically inactive


OCH3 OCH3
and in (c) diastereomers will be obtained like
H
HALOALKENES AND HALOARENES 685

12. (c)
OH OH
and
CH3 CH3 (i) H /Ni
¾¾®
2
*
CH3 CH3
H Br H Br
H /Ni
(ii) ¾¾®
2
In (b) SN1 occurs and enantiomeric pair will be produced (Optically inactive)

OH and
H OH H /Ni
H (iii) H2C ¾¾®
2
H3C
CH3 * CH3
9. (d) tert-Alkyl halides undergo SN1 reactions, hence they CH3 CH3
involve the formation of quite stable carbocations, and
not the transition state. In SN1 reactions, the nucleophile
is not involved in rate determining (first) step, hence its
H /Ni
stronger or weaker nature does not influence the reaction (iv) H 2C CH3 ¾¾® H 3C
2 CH3
rate. In SN1, the product has more percentage of the inverted
configuration than the retained configuration, i.e. only
partial racemization takes place, hence the product will be 13. (d)
+ + +
having some optical activity.
10. (d) —NO2 group is electron attractive group, so it is able
O –I max N –I B
to deactivate the benzene ring.
Hence withdrawl of electrons from ortho and para position H H
cause easy removal of –Cl atom due to development of Aromatic Aromatic Anti-aromatic
+ve charge on – o- and – p positions.
11. (d) 14. (d) 2 g Br2 reacts with = 0.7 g of (A)
0.7 ´160
O Less hindered 160 g Br2 reacts with = = 56 g of (A)
Br 2
N—Br So molecular weight of (A) = 56
(2 mole)
Hence (A) is C4H8. Since (A) gives same product with HBr
O
similar to N.B.S
or HBr/peroxide hence double bond will be present at the
centre hence (A) =
Br 15. (a) Meso compounds are those having 2 or more chiral
More hindered centers along with symmetry.
CN
V
CN O O * POS
O
(S N2) (1) (2)
* * O *
O
Br
POS O
Meso
EBD_8350
686 CHEMISTRY

25 Alcohols, Phenols and Ethers

Exercise - 1 H H
| CH3MgBr
|
15. (a) H C = O ¾¾¾ ¾¾® H C OMgBr

|
|
1. (b) Glycols are dihydric alcohols (having two hydroxyl

|
|
groups). Ethylene glycol is the first member of this series. CH3
CH 2 OH H
| H O + |
CH 2 OH ¾¾3 ¾¾® H C OH

|
Ethylene glycol |
CH 3
2. (c) Ethers contain the functional group – O –
16. (b) Since the compound is formed by hydration of an
3. (b) CH3CH2CH(OH)CH3 is a secondary alcohol alkene, to get the structure of alkene remove a molecule of
4. (b) Cresol has phenolic– OH group water from the alcohol.
5. (b) Four primary alcohols of C5H11OH are possible. These
-H O
are: CH3 C H CH3 ¾¾¾¾
2 ® CH = CHCH
2 3
|
(i) CH 3 CH 2 CH 2 CH 2 CH 2 OH OH
(ii) CH 3 CH 2 - CH - CH 2 OH Isopropyl alcohol Propylene
| 17. (d) The aldehydes which do not have a -hydrogen atom
CH 3
react with NaOH when half of molecules are reduced to
(iii) CH 3 - CH - CH 2CH 2 OH
alcohol and other half of molecules are oxidised to acid
|
CH 3 (Cannizzaro reaction).
2 HCHO ¾¾® CH 3OH + HCOONa
CH 3 Methanol Methyl alcohol Sod. formate
|
(iv) CH 3 - C - CH 2 OH 18. (a) Commercially, acids are reduced to alcohols by
| converting them to the esters, followed by their reduction
CH 3 using hydrogen in the presence of catalyst (catalytic
6. (c) In this structure –OH group is directly attached to double hydrogenation).
bonded carbon atom i.e. sp2 hybridised carbon atom.
R'OH H
7. (c) If two groups attached to the oxygen atom are different RCOOH ¾¾®
+
RCOOR ¢ ¾¾¾
2 ® RCH OH + R ¢OH
2
H Catalyst
then ethers are known as unsymmetrical or mixed ethers.
8. (a) Methyl alcohol (CH3OH) is also known as carbinol. 19. (b) A diazonium salt is formed by treating an aromatic
Hence vinyl carbinol is CH2 = CH – CH2OH. primary amine with nitrous acid (NaNO2 + HCl) at
273-278 K. Diazonium salts are hydrolysed to phenols by
9. (a) CH 2 = CH - OH represents vinylic alcohol. In warming with water or by treating with dilute acids.
vinylic alcohols, – OH group is attached to sp2 hybridised
carbon whereas in allylic alcohols, – OH group is attached NH2 N2 Cl OH
to sp3 hybridised carbon.
NaNO2 H2O
CH 3 +HCl Warm + N2 + HCl
5 4 3 |2 1
10. (b) C H3 – CH – CH 2 – C – CH 3
| | Aniline Benzene diazonium
OH OH chloride

2-Methyl-2, 4-pentanediol. 20. (a) The lower alcohols are readily soluble in water and
11. (d) 12. (b) the solubility decreases with the increase in molecular
:O : weight. The solubility of alcohols in water can be explained
13. (b) The bond angle in alcohols is slightly less
due to the formation of hydrogen bond between the highly
C H
than the tetrahedral angle (109°-28¢). It is due to the polarised —OH groups present both in alcohol and water.
repulsion between the unshared electron pairs of oxygen. 21. (c) o-Nitrophenol has intramolecular H-bonding.
14. (b) 22. (b) The solubility of alcohols depend on number of
C-atoms of alcohols. The solubility of alcohols in water
ALCOHOLS, PHENOLS AND ETHERS 687

decreases with the increase in number of C-atoms of


alcohol. As resulting molecular weight increases, the polar 29. (d) OH OH
| Br Br
nature of – OH bond decreases and hence strength of
3Br2 (aq.)
hydrogen bond decreases.
23. (a) Basic nature of OH group µ stability of C+ ion
produced after ejection of H2O Br
2, 4,6-Tribromophenol
24. (d) C 2 H 5OH + 4I 2 + 6NaOH ¾¾
®
Note : The –OH group in phenol, being activating group,
CHI 3 ¯+ HCOONa + 5NaI + 5H 2 O facilitates substitution in the o- and p-positions.
Iodoform
25. (a) When primary (1°) alcohols are treated with copper at OH OH
CHCl2
300°C, then aldehydes are obtained by dehydrogenation of
30. (b) + CHCl3
alcohols. Similarly secondary (2°) alcohols form ketone and
alkene is obtained by dehydration of tertiary
(3°) - alcohols. But phenol does not respond to this test.
Cu OH OH
R - CH 2 - CH2 - OH ¾¾® R - CH 2 - CHO + H 2 CH(OH)2 CHO
1° Alcohol 300°C Aldehyde
2KOH
H –H2O
| Cu
R – C – OH ¾¾¾®
300°C
R – C – R + H2
This set of reactions is known as Reimer-Tiemann reaction.
|
R O 31. (b) The correct explanation is : In Lucas test, tertiary
2° Alcohol Ketone alcohols react immediately because of the formation of the
CH3 more stable tertiary carbocations.
| Cu
R – C – OH ¾¾¾®
300°C
R - C = CH 2 + H2O 32. (a) OH OH
| |
CH3 CH 3 COOH
Salicyclic acid Phenol
3° Alcohol Alkene (evolves CO 2 with NaHCO3) (No reaction with NaHCO3)

Cu 33. (b) Other options are acids, only Na metal is a base.


C 6 H 5 – OH ¾¾¾® 300°C
No reaction
34. (b) Phenol on reaction with conc. H2SO4 gives a mixture
26. (b) Phenol is most acidic because its conjugate base is
of o- and p- products (i.e., –SO3H group, occupies
stabilised due to resonance, while the rest three compounds
o-, p- position). At room temperature o-product is more
are alcohols, hence, their corrosponding conjugate bases
stable, which on treatment with conc. HNO3 will yield
do not exhibit resonance.
o-nitrophenol.
27. (c) Secondary alcohols oxidise to produce ketones.
OH OH OH
( O) SO3H
CH3CHOHCH2CH3 ¾¾
¾® CH3COCH2CH3 Conc. H SO
¾¾¾¾®
2 4
+
2-Butanol Ethyl methyl ketone
28. (d) The rates of reaction with Lucas reagent follows the SO3H
o-product p-product
order.
At room temperature o-product is more stable
3° alcohol > 2° alcohol > 1° alcohol
Since carbocations are formed as intermediate, more stable OH OH
SO3H NO2
the carbocation, higher will be the reactivity of the parent Conc. HNO3
compound (alcohol). 2-Methylpropan-2-ol generates a 3º
carbocation, so it will react fastest; other three generate o- nitrophenol
either 1º or 2º carbocations.
CH3 35. (d) 3C 2 H 5OH + PBr3 ¾
¾® 3C 2 H 5Br + H 3PO 3
| H+
[X ]
CH3 - C - OH ¾¾¾ ® 36. (b) Alcohols are versatile compounds. They react both
|
CH3 as nucleophiles and electrophiles. The bond between
2-Methyl-2 -propanol O—H is broken when alcohols react as nucleophiles.
CH3 CH3 Alcohols as nucleophiles
| - |
Å Br
CH3 - C ¾¾¾® CH3 - C - Br H
| |
.. –C – ® R –O–C– +H
R –+O +
(i) R–O–H+ +C –
CH3 CH3
EBD_8350
688 CHEMISTRY

(ii) The bond between C — O is broken when they react 44. (c) The usual halogenation of benzene takes place in
as electrophiles. Protonated alcohols react in this manner. the presence of a Lewis acid, such as FeBr 3, which
Protonated alcohols as electrophiles polarises the halogen molecule. In case of phenol, the
polarisation of bromine molecule takes place even in the
+ +
R - CH 2 - OH + H ¾¾
®R - CH 2 - OH 2 absence of Lewis acid. It is due to the highly activating
effect of –OH group attached to the benzene ring.
45. (c) The mechanism of the reaction involves the following
+
Br – + CH2 – OH2 Br – CH2 + H2O three steps:
Step 1: Protonation of alkene to form carbocation by
R R
electrophilic attack of H3O+
37. (c) The C — O bond length (136 pm) in phenol is slightly H2O + H+ ® H3O+
less than that in methanol (142 pm).
38. (b) H H
39. (a) A methanol poisoned patient is treated by giving H + +
C=C + H — .O. — H —C—C + H2O
intravenous injection of ethanol.
40. (a) R is the correct explanation of A. Due to +M effect of H
..
-O
.. H , its intermediate carbocation is more stable than Step 2: Nucleophilic attack of water on carbocation.
the one in benzene.
41. (c) KMnO4 (alkaline) and OsO4 / CH2Cl2 are used for H H H
hydroxylation of double bond while O3 /Zn is used for + .. +
—C—C + H2O
.. — C — C — O —H
ozonolysis. Therefore, the right option is (c), i.e.,
BH in THF H H
3CH 3CH = CH 2 3 ¾¾
¾¾¾ ® ( CH 3CH 2 CH 2 )3 B
3H O Step 3: Deprotonation to form an alcohol.
¾¾2¾¾

NaOH
3CH 3CH 2CH 2OH + H3BO3
..
1-propanol H H H OH

..
42. (b) NaBH4 and LiAlH4 attack only carbonyl group and + .. +
reduce it into alcohol group. .. ¾¾
— C — C — O — H + H2O ® — C — C — +H3 O
..
H H
NaBH
C6 H5 - CH = CHCHO ¾¾¾¾4®

Cinnamic aldehyde C6 H 5 - CH = CH.CH 2OH OH


Cinnamyl alcohol
Zn CH Cl
CH3 CH3 46. (b) ¾¾® ¾¾¾¾®
3
H anhy AlCl3
(i) BH3 THF
43. (a) (i) ¾¾¾¾¾¾¾
-
® OH Phenol X
CH3 COOH
H CH H
(ii) H 2O2 , OH
CH3 3
2, 3 dimethylbut-2-ene
alk KMnO
¾¾¾¾¾

H
(i) BH THF * OH
(ii) ¾¾¾¾¾¾¾
3
-
® Y Z
CH3 (ii) H 2O2 , OH CH3 Al2O3 Al2O3
3-methylpent-2-ene 47. (c) CH 2 = CH 2 ¬¾¾¾ ¾ CH 3CH 2 OH ¾¾¾¾ ® CH 3CH 2OCH 2CH 3
650 K 525 K
CH3
CH3 H ® CH 3CH 2 OCH 2 CH 3
(i) BH3 THF
(iii) ¾¾¾¾¾¾¾
-
® 48. (b) Dehydration of CH3OH gives carbene (methylene),
(ii) H 2O2 , OH *
OH an unstable intermediate.
2-methylbut-2-ene H SO
2 4 ®[: CH ] + H O
CH3 CH3OH ¾¾¾¾ 2 2
CH3 (i) BH THF HO Carbene
(iv) ¾¾¾¾¾¾¾
3
-
® 49. (d) Since the compound (C4H10O) reacts with sodium, it
(ii) H 2O2 , OH H
H must be alcohol (option b, c, or d). As it is oxidised to
2-methylpropa ne carbonyl compound which does not reduce Tollen’s
In case of (ii) and (iii), the alcohol so obtained contains reagent, the carbonyl compound should be a ketone and
carbon which is attached to four different groups i.e., chiral thus C4H10O should be a secondary alcohol, i.e. sec-butyl
carbon while in case of (i) and (iv) achiral alcohol is alcohol; other two given alcohols are 1º.
obtained.
ALCOHOLS, PHENOLS AND ETHERS 689

P+IMg sp2
50. (d) CH3CH 2OH ¾¾¾
2 ® CH CH I
3 2 ¾¾¾ ® OH
ether
(A) 60° sp3 60° O
CH 2 CH3 OH
|
HCHO
CH3CH 2 MgI ¾¾¾¾ ® H - C - OMgI Less angular strain More angular strain
(B) | 109° – 60° = 49° 120° – 60° = 60°
H 59. (b) Primary alcohol will favour SN2 pathway as primary
(C)
CÅ ion is less stable.
CH 2 CH3
| OH
¾¾¾
H2 O
® H - C - OH 60. (d) [A] is O
|
H 61. (b) Increase in mol. wt = 218 – 92 = 126
(D) Molar mass of one CH3CO groups = 43
n - Propylalcohol Number of acyl group or
51. (a) 126
52. (c) B2H6 selectively reduces COOH and not other OH group in unknown compound = j 3 (approx.)
43
groups like COCl, CHO, NO2, etc. 62. (c)
53. (b) LiAlH4 can not reduce C—C multiple bond but it can CH3
CH3 + CH3
reduce C—C multiple bond when phenyl group is present H Hg(OAc)2
OH
in the conjugation of C==C bond. D D
OH
54. (d) (A) (B) (C)
HNO2
O 3/Zn, CH 3COOH
55. (c) –N2
O
NH2 N2
CH3
1, 2 shift H2O H
O
(More OH
stable) (Optically inactive) 63. (a)
K 2Cr2O7 COOH CH 3OH COOCH3
56. (c) H ring OH H2SO4
–H2O expansion (excess)
CH3MgBr
HO OH
OH
O OCH3
|| + |
OH O H3O
C – CH3 C – OMgBr
|
+ H CH3
CH3MgBr

H CH3 OMgBr CH3


–H2O Shift
| H3O
+ |
C – CH3 C – OH
HO | |
OH OH CH3 CH3

+ H OH Br MgBr
PBr3 Mg diethyl ether
64. (c) ¾¾
® ¾¾¾¾¾®
OH O
D2O
57. (c)
58. (d) Rate of dehydration will be least in ‘x’ because after
D
dehydration angular strain will be increased
EBD_8350
690 CHEMISTRY

65. (b) CH3 – CH2 – CH2 – CH – CH3 – Maltase


C12 H 22 O11 + H 2 O ¾¾¾¾¾ ® 2C6 H12 O6
| Mg (from yeast)
Glucose
CH – CH
Br Diethyl ether
2 3
Zymase
O
CH3 – CH2 – CH2 – CH – CH3 C6 H12 O 6 ¾¾¾¾¾
(from yeast)
® 2C 2 H 5 OH + 2CO 2
|
MgBr 72. (c) The quantity of sugar increases and yeast grows on
the outer skin as grapes ripen. When grapes are crushed,
CH3 – CH2 – CH2 – CH – CH2 – CH2 – CH2OMgBr sugar and the enzyme come in contact and fermentation
|
CH3 starts. Fermentation takes place in anaerobic contidions
i.e., in absence of air, CO2 is released during fermentation.
+
H2O If air gets into fermentation mixture, the oxygen of air
oxidises ethanol to ethanoic acid which in turn destroys
CH3 – CH2 – CH2 – CH – CH2 – CH2OH the taste of alcoholic drinks.
|
CH3 73. (b) The commercial alcohol is made unfit for drinking by
3-methyl-1-butanol
mixing in it some copper sulphate (to give it colour) and
pyridine (a foul smelling liquid). It is known as denaturation
66. (c) O–H O of alcohol.
74. (d) The two components should be (CH3)3CONa +
tautomerises
(CH3)3CBr. However, tert-alkyl halides tend to undergo
H–O O–H elimination reaction rather than substitution leading to the
O O
more stable formation of an alkene, Me2C = CH2
keto form 75. (a) Due to H-bonding, the boiling point of ethanol is
O O much higher than that of the isomeric diethyl ether.
H 76. (d) Due to inter-molecular hydrogen bonding in alcohols
– –
OH SN2
¾¾
® ¾¾® Me boiling point of alcohols is much higher than ether.
MeI
77. (c) Ethyl alcohol has strongest hydrogen bonding due
O O O O
to large electronegativity difference.
O 78. (a) Ethers are readily cleaved by HI as follows :
Me Me
H+ + I

Me MeI R–O–R R – O – R ¾¾ ® ROH + RI.


Me |

O O OH H
(excess) 79. (c) In case of unsymmetrical ethers, the site of cleavage
Me Me
depends on the nature of alkyl group e.g.,
The more stable keto form gives a-alkylation reaction.
373K
– CH 3O - CH(CH3 ) 2 + HI ¾¾¾® CH 3I + (CH3 )2CHOH
67. (c) O + O Methyl Isopropyl
N iodide alcohol
The alkyl halide is always formed from the smaller alkyl
group.
80. (b) Due to greater electronegativity of sp2-hybridized

carbon atoms of the benzene ring, diaryl ethers are not
O attacked by nucleophiles like I–.
In this structure valence shell of nitrogen and oxygen are 81. (b)
82. (d) Diethyl ether, being a Lewis base, is not attacked by
not stable.
nucleophiles, while all others contain electrophilic carbon,
68. (c) Glucose and fructose obtained by hydrolysis of hence attacked by nucleophiles like OH– ions.
sucrose, are converted into alcohol by enzyme zymase.
Od -
C 6 H12 O 6 ¾¾ ¾
zymase
¾® 2C 2 H 5 OH + 2CO 2 ÷÷ d+ d-
CH3 - C - OCH3 CH3 - C º N
Cr O – ZnO d+
CO + H2 + H2 ¾ ¾¾ ¾ ® CH3OH
2 3
69. (a)
300ºC ..
water gas Methanol C2H5 - O
. . - C 2 H5
70. (b)
Diastase 83. (a) In Williamson’s synthesis the reaction of alkyl halides
71. (a) 2(C6 H10O5 ) n + nH 2O ¾¾¾¾¾¾¾¾¾
(from germinated barley)
® with sodium alkoxides give ethers. However, if the alkyl
Starch
nC12 H 22 O11 halide is 3°, it undergoes elimination to give an alkene.
Maltose
ALCOHOLS, PHENOLS AND ETHERS 691

CH3 90. (b) Reaction will follow SN1 path IV goes with most stable
| +–
H3C – C – Cl + NaOC2H5 D CÅ ion.
| 91. (a) Both the sites of epoxide group is equally hindered.
CH3 V
tert-Butyl chloride Sod. ethoxide So, OH will attack on that site where electrophilicity is
CH2 maximum
||
CH3 – C – CH3 + C2H5OH + NaBr
O
2-Methyl-1-propene O 18
OH
84. (c) If a tertiary alkyl halide is used, an alkene is the only CH3—CH—CH NO2 CH3—CH—CH—NO2
reaction product and no ether is formed. For example, the
–Rgp OH
reaction of CH3ONa with (CH3)3C–Br gives exclusively 2- 18
methylpropene.
CH3OH
CH3 CH3—CH—CH—NO2
| + – OH OH
CH3 - C - Br + NaO – CH3 ¾¾¾ ®
: :

18
|
CH3 92. (d) In last case CH4 is produced.
CH3 - C = CH 2 + NaBr + CH 3OH 93. (d) Order of reactivity of different alkyl halides towards
| Williamson’s synthesis is Allyl > 1° > 2° > 3°.
CH3
2-Methylpropene For same alkyl group order of reactivity is
It is because alkoxides are not only nucleophiles but R – I > R – Br > R – Cl
strong bases as well. They react with alkyl halides leading
to elimination reactions. Thus order of reactivity of given alkyl halides towards
85. (a) R is the correct explanation of A. Williamson’s synthesis can be given as
+ HI
HI CH2 = CHCH2Cl (Allyl) > CH3CH2CH2Br (1° bromide)
86. (a) —H2O
HO I > CH3CH2CH2Cl (1° chloride) > (CH3)3CCH2Br
O
(Stearically hindered)
Wurtz
I I 94. (c) Rate of SN1 reaction is proportional to the stability of
87. (c) It will occur by SN1 pathway. carbocation. When two phenyl groups are replaced by

HI OH two MeO groups, the carbocation formed will


88. (b)
SN2
O I OH be more stable. Hence, the reaction is fastest.
95. (c) It is SN1.
I
O O Exercise - 2
89. (c) At 443K compound in (d) will produce propene. In
(a) alkene will be produced as tertiary halide and strong 1. (2) Structure of given alcohols
base favours elimination. In (b) reaction is not possible at
room temperature as due to resonance C—Cl bond has CH3
partial double bond character which is very difficult to |
break. CH 3 — CH — CH — CH3 CH3 — CH 2 — C — CH 3
| | |
O CH3 OH OH
3-methyl-2-butanol 2-methyl-2-butanol
CH3O + CH3CH— ( 2° alcohol ) ( 3° alcohol )
2 O—S— —CH3

O
CH3 —CH—CH 2 —CH 2 OH
|
Tosyl group
(good, L.G) CH3
3-methyl-1-butanol
OTs + CH3OCH2CH 3 (1° alcohol )
EBD_8350
692 CHEMISTRY

Therefore, ether group is attached at the 2nd carbon


CH3 CH3 of carbon chain.
| |
CH3 —CH—CH—CH 2OH 4. (3) (CH 3 )3CBr + NaOC2 H5 can’t be applied for
2,3-dimethyl-1-butanol synthesising the ether because sod. ethoxide, being a
(1° alcohol )
strong base, will preferentially cause elimination reaction.
-
OC H
CH3 CH3 (CH3 )3CBr ¾¾¾¾®
2 5
(CH 3 ) 2C = CH 2 + HBr
| |
CH3 — CH 2 — CH 2 — CH 2OH CH3—CH—C—CH3 Rest other forms tert-Butyl ethyl ether.
1-butanol
|
(1° alcohol ) OH 5. (8)
(2,3-dimethyl-2-butanol) (I) Water is stronger base than ether and removes proton
(3° alcohol)
from protonated ether, R2OH+
2, 3-dimethyl -2-butanol and 2-methyl-2-butanol being [R2OH]+ + H2O ¾® R2O + H3O+
tertiary alcohol gives precipitate immediately with Acid1 Base2 Base1 Acid2
conc. HCl and anhydrous ZnCl2. (II) Due to unshared electron pairs, ethers act as Lewis
2. (3) bases and hence easily react with Lewis acids like
BF3 and RMgBr to form coordinated compounds.
OH O– O O
.. .. – +
:– (C2H5)2 O : + BF3 ¾® (C2H5)2 O : BF3
¾® ¬¾® ¬¾®
.–. ..
2(C2H5)2 O : + RMgBr ¾®

R
(C2H5)2O ¾® Mg ¬¾ O(C2H5)2
Br
CH = CHCH Cl
2 2
¾¾¾¾¾¾ ® Note that two molecules of ether coordinate
tetrahedrally with one Mg2+.

mononitration
+ (III) H 3C OCH3 ¾¾¾¾¾®
(Strong
activator)
p-Methylanisole

3. (2)
Conc. HI H 3C OCH3
CH3CH2O CH(CH3)2 ¾¾¾® (CH3)2CHI + CH3CH2I
(A) (B) (C) NO2
2 Nitro-4-methylanisole
NaOH NaOH

(CH3)2CHOH CH3CH2OH
monobromination
(IV) HO OC2H5 ¾¾¾¾¾¾®
[O] [O]
(Strong
activator)
(CH3)2CO CH3COOH HO OC2H5
p-Ethoxyphenol
Hence, Br
2 1
2 Bromo-4-ethoxyphenol
CH3 — CH 2 — O — CH — CH3
½ (V) Due to –I effect of Cl, 4-chlorophenol is acidic and
3CH3
2-ethoxy propane dissolves in NaOH, but (II) is an aryl halide and
(A) does not dissolve.
ALCOHOLS, PHENOLS AND ETHERS 693

(VI) (III) is an acid and dissolves in NaHCO3 but (IV) is O


a phenol and does not dissolve.
C=O
¾¾®
(VII) Although (V) is a phenol, because of the presence –
O CH2 Cl
of (3NO2) group, it is exceptionally acidic (pKa = 0.38). (Q)
So it dissolves in NaHCO3, but (VI) is a phenol (pKa =
10.17) and will not dissolve. O
O
(VIII) (VII) is a phenol and dissolves in NaOH, but (VIII)
¾¾¾®
is an ether; and does not dissolve.
6. (2) O
(R)
OH OH OH
OH
BH –
¾¾¾
4
®
LiAlH
HO OH
HO
OH ¾¾¾¾

O OH
H O – CH2 – CH2 – OH
H + or (BH3) (S)

OH 9. (3) —OH group is strongly activating and o, p-directing


due to +M effect. Thus positions a, b and c are the sites for
attack by an electrophile. However, sites b and c are not
H2+O OH preferred by bulky electrophile due to steric crowding. Thus
H or (BH3)
–H2O or HOBH 3– more bulky electrophile (like I2) can attack only site a, which
is least sterically hindered, a bit smaller electrophile (Br2) can
O
attack at sites a and also b (relatively less sterically hindered
site) and the smallest electrophile (Cl2) can attack all the three
sites, viz., a, b and c (most sterically hindered site).
OH

Tautomerisation OH

a c
OH
C(CH3)3
b
OH Statements 1, 2 and 3 correctly explains the pattern of
(B)
electrophilic substitution.
7. (5) One degree of unsaturation in (A) and formation of 10. (2) The given reaction involves electrophilic
(B) with two I atoms suggest that (A) is a ring. The substitution on the highly reactive phenoxide ion.
formation of (C) (five C atoms) suggest that (A) is a five- Here the electrophile is dichlorocarbene formed by the
membered ring ether. action of strong alkali on chloroform.
O OH – –
Me CHCl3 + OH CCl3 + H2O
I HI
HI
¾ ¾® excess – –
Me : CCl3 : CCl2 + Cl
.. Dichlorocarbene
..
:O: –
1
O:
I
2 4 - 2HI
Me ¬¾ ¾ ¾ ¾ ¾ I H
3 5 Alc. KOH –
(C) Me + : CCl2 ¾® CCl2
.. –
O :O: –
O
– O
CHCl2 CH(OH)2
+ C –
8. (3) ¾® OH
¾®
OH Cl – CH2 Cl
(P)
EBD_8350
694 CHEMISTRY

6. (c) Compound (A) i.e., phenol and compound (D) i.e., a


O– OH derivative of phenol cannot be considered as aromatic
CHO CHO alcohol.
+
¾¾¾®
H
¾¾® On the other hand, in compounds (B) and (C), — OH
(–H2O) group is bonded to sp3 hybridised carbon which inturn is
Salicylaldehyde bonded to benzene ring. Hence, are considered as
(o-Hydroxybenzaldehyde) aromatic.
6 5 4 3 2 1
7. (c) CH3 — CH — CH2 — CH2 — CH — CH3
Exercise - 3


Cl OH
1. (d) 5-Chlorohexan-2-ol
CH3 CH2Cl CH2OH 8. (a) The structure of m – cresol is
CH3
Cl aq NaOH
¾¾¾¾
2
® ¾¾¾¾¾
® 3
sunlight 2
1
Toluene Benzyl chloride Benzyl alcohol
2. (a) Following are the three possible isomers of butanol m-cresol OH
Here –OH is the functional group and the methyl is
(i) CH3CH 2 — CH 2 — CH 2OH
substituents. Hence the IUPAC name is 3-methylphenol.
Butan -1-ol
no chiral carbon
9. (c) 3 2
H3C — CH — OCH3
* |
(ii) CH3 CH2 CH CH3 1
CH3
OH IUPAC name of the above compund is 2-methoxypropane.
Butan –2 – ol 10. (b) Weakest acid has the strongest conjugate base.
1-Chiral Carbon Among all these acids, ROH is the weakest acid.
(iii) CH3 Therefore, the strongest base is RO– .
11. (a) Phenol being more acidic in nature reacts with sodium
H3C C CH3 hydroxide solution and by the loss of one proton, it gives
phenoxide ion. This phenoxide ion is resonance stabilised.
OH OH ONa
2 – methylpropan – 2 – ol
No Chiral Carbon +H2 O
+NaOH
3. (c) HCl + An. ZnCl2 is known as lucas reagent. It is used
to determine degree of an alcohol.
12. (b) Presence of electron withdrawing group at ortho
The reaction follow nucleophilic substitution reaction in position increase the acidic strength. In o-nitrophenol,
which — OH group is replaced by — Cl. In this reaction nitro group is present at ortho position. On the other hand,
carbocation is formed as intermediate. Higher the stability in o-methylphenol and in o-methoxyphenol, electron
of intermediate carbocation higher will be the reactivity of releasing group (—CH3, —OCH3) are present.
reactant molecule. Since 3° carbocation is more stable than Presence of these groups at ortho and para positions of
2° carbocation as well as 1° carbocation, so the order of phenol decreases the acidic strength of phenols. So,
reactivity of alcohols is 3° > 2° > 1°. phenol is less acidic than o-nitrohenol.
4. (c) Less powerful oxidizing agent, Pyridinium 13. (d) Presence of electron withdrawing group increases
the acidic strength. So, m-chlorophenol is most acidic
chlorochromate (C 5 H 6 ClCrNO 3 ) oxidises primary
among all the given compounds.
alcohols to aldehydes. 14. (b) Electron withdrawing substituents increase the acidic
PCC strength of phenols. so, p-nitr ophenol (II) an d
CH3CH 2OH ¾¾¾
® CH3CHO
Ethanol Ethanal
m-nitrophenol (IV) are stronger acid than phenol (I). If —
5. (b) The process of conversion of alkyl halides into NO2 group is present at p-position, then it exerts both —
alcohols involves substitution reaction. I and — R effect, but if it is present at meta position, then
OH -
it exerts only–I effect. Therefore, p-nitrophenol is stronger
R — X ¾¾¾® R — OH than m-nitrophenol.
Alkyl halide Alcohol
ALCOHOLS, PHENOLS AND ETHERS 695

On the other hand, electron releasing substituents ONa OH


decreases the acidic strength of phenol. If — OCH3 group
is present at meta position, it will exert – I effect only. 21. (a) + CO2 ¾¾
®
But, if it is present at para position, it will exert + R and COONa
Sodium
– I effect. Thus, m - methoxy benzyl alcohol is more acidic Phenoxide
than p- methoxy phenol. Hence, the correct order of
decreasing acidic strength will be : II > IV > I > III > V. O 2 4
H SO
15. (c) Nucleophilic substitution reactions depend upon the ||
stability of carbocation. As, presence of electron O - C - CH3 OH
withdrawing group decreases the stability of carbocation (CH3CO)2 O
¬¾¾¾¾¾
¾
in compound (II) and (III). Therefore, they will give less
COOH COOH
stable carbocation than (I). Aspirin Salicylic acid
Since, NO2 group is a stronger EWG than — Cl. (Acetyl salicylate)
+ + CH3COOH
So, NO2 — C6H5 — C H 2 will be less stable than
+
Cl — C6H5 — C H 2 22. (d) Williamson synthesis is one of the best methods for
the preparation of symmetrical and unsymmetrical ethers.
Hence, the order of reactivity of carbocatiors will be: In this method, an alkyl halide is allowed to react with
+ + + sodium alkoxide.
O2N C H 2 < Cl C H2 < C H2 OH
Therefore, the correct order is II < III < I CHCl3 + aq. NaOH
23. (d)
16. (a) With increase in molecular mass boiling point
increases. Thus the b.p. of pentan-1-ol will be more than
– –
other given compounds. Now, among isomeric alcohols O Na+ O Na+
1° alcohols have higher boiling points than 2° alcohols CHCl2 CHO
NaOH
due to higher surface area in 1° alcohols.
Hence, increasing order of b.p. will be
Propan-1-ol < butan-2-ol < butan-1-ol < pentan-1-ol. Intermediate
H
+

17. (b) Phenols react with alkyl halides in alkaline medium to


form ethers. Therefore, OH
OH OCH3 CHO

(i) NaOH
Salicylaldehyde
(ii) CH3I

18. (c) o-Nitrophenol will not be soluble in NaHCO3. Due to Therefore functional group – CHO is introduced.
intramolecular hydrogen bonding hydrogen on OH is 24. (a) In presence of ZnCl2 (catalyst), the 1° and 2° alcohols
strongly bound. So it can not behave as an acid and can are easily react with HCl to give the corresponding
not react with sodium bicarbonate. products. While in case of 3° alcohols ZnCl 2 is not
HBr/H2O 2 required. In the absence of ZnCl2 formation of primary
19. (a) CH3 CH2 CH CH2
(Peroxide effect) carbocation is very difficult.
25. (b)
CH3 CH2 CH2 CH2
(Y) Å
Br OH +H O H
Å

H
CH3 (CH2)3 O CH2 CH3
(Z) H
Ha b
20. (d) An excellent reagent for oxidation of 1° alcohols to Å
a c
aldehydes is PCC.
R - CH 2 - OH ¾¾¾
PCC
® R - CHO b H
c
EBD_8350
696 CHEMISTRY

26. (a) cis-cyclopenta-1, 2-diol when reacts with acetone O O


forms cyclic ketal whereas trans-isomers can not form tert - BuONa
cyclic ketal. 31. (a) ¾¾¾¾¾
®
Br O-tBu
OH CH3 O CH3 (fails to decolorise the
colour of bromine due to
+O C absence of unsaturation)

OH CH3 O CH3 –
32. (c) Na
¾¾® C2H5O Na+
cis-cyclopenta-1, 2-diol cyclic ketal
C2H5OH (B)
PCl
OH CH3 (A) ¾¾®
5
C2H5Cl
+O C No reaction (C)

OH CH3 – S 2
C2H5O Na+ + C2H5Cl ¾¾®
N
C2H5OC2H5
trans-cyclopenta-1, 2-diol (B) (C)

27. (a) This is an example of Williamson ether synthesis –


reaction in which sodium alkoxide reacts with alkyl halide 33. (c) CHCl3 + NaOH ¾® CCl3 + H2O

¾®
and gives ether. – Cl (a-elimination)
28. (c) Electron withdrawing – NO2 group has very strong – : CCl2
I and –R effects, so compound (c) will be most acidic. dichlorocarbene
(electrophile)
29. (b) When Ar – O – R ethers are reacted with HI, they are
cleaved at weaker O – R bond to give phenol and alkyl 34. (d) Haloform reaction is shown by compounds having
iodide.
CH3 – C – or CH3 – CH – Group
O – CH3 OH || |
O OH
HI NaOI
+ CH3I CH – CH3 ¾¾¾¾®
or
| NaOH + I 2
OH
30. (d) This is a substitution reaction which goes by an (A)
elemination addition path way. – +
C – ONa + CHI3
OCH3 OCH3 || Yellow
H O ppt.
NH 2 35. (c) Mechanism :
AlCl
Br
Benzyne
CH 3 – CH 2 – CH 2 – Cl ¾¾¾®
3
+ –
OCH3 CH3 – CH 2 – CH 2 + AlCl4
NH2 1° Carbocation
¬¾¾¾


OCH3 a H shift
(Less stable) CH3 – CH – CH3
NH2 +
ESR
OCH3 OCH3 ¬¾¾¾ CH3 – CH – CH3
2° Carbocation
b
H — NH 2 (P)

NH2 NH2 CH3


(More stable as –ve charge (More stable) CH3 – CH – CH3
is close to electron
CH3–C– O– O–H
withdrawing group) +
O
¾¾®
D
2 H3O
Also, incoming nucleophile gets attached on same 'C' on D
which 'Br' (Leaving group) was present. So it is not a cine Cumene Cumene
substitution reaction. (P) hydroperoxide
OH
O
+ CH3 – C– CH3
Phenol Acetone
(Q) (R)
ALCOHOLS, PHENOLS AND ETHERS 697

OH O
||
(i) CO ,NaOH
36. (a) ¾¾¾¾¾¾ 2
® H+ CH3
(ii) Acidification ¾¾¾® + H3C
Kolbe reaction H2 O
Salicylic acid
(X)
..
:OCH3 HO – CH3
conc. H 2SO 4
+ (CH 3CO) 2 O ¾¾¾¾¾
®
(Acylation reaction) +d –d
H–I I
(X) 41. (d) + H
S N2

OH

Aspirin + CH3I
(non-narcotic analgesic)

CH3 CH3
NaOH
37. (c) + ClCOOMe ¾¾¾® 42. (b) CH
HBr
CH Br–
H2C H2C
(A) +

:
O O
1-Methylethylene oxide
H
Br2 OH CH3 Br CH3
¾¾® + HBr
HBr
CH2 CH CH2 CH
(A) Br
(B) Br Br

+ Exercise - 4
38. (d) ¾ ¾® 1. (c)

–CH3OH O OH OH
CH 3OH
H
CH3—C—O—H CH3—C—OH CH3—C—OH
+
O
H CH3
OH2

CH3—C==O + H + H2O CH3—C—OH


:

:O–H
OCH3 OCH3
39. (d) In lone pair of e–s present on oxygen atom is
2. (d)
involved in delocalisation. Therefore, it is most difficult
to protonate the phenol amongst the given compounds, O3 CHO LiAlH4 CH2 OH
Zn + H2O
due to less availability of e–s. CHO CH2 OH

CH3 O3/H2O2 COOH LAH CH2 OH


CH3

CH H3C–C– O–O–H COOH CH2 OH


CH3 OH
Cold dil.
40. (b) O2 KMnO4 HIO4 CHO
¾¾¾
®
CHO
Cumene Cumene OH
hydroperoxide LAH CH2OH
CH2OH
EBD_8350
698 CHEMISTRY

10. (d) First due to acid catalysed dehydration of alcohol


+
3. (d) H +
C+ ion is produced which has planar geometry so, H2O
OH OH2
can attack on it either from above the plane or from below
+
–H2O CH2 the plane.

11. (d) 2 moles


1º carbocation O O
(least stable)
Cl OEt
1,2–alkyl 1,2–alkyl + O
shift + shift
2 moles
2º carbocation 3º carbocation 1 moles
(more stable) (most stable)
B 2H6 OH
4. (c) ClCH 2 CH 2 OH is stronger acid than CH3CH 2 OH due
12. (a) H 2O 2/OH —H2O
to – I effect of Cl. Br Br OH
® Cl -¬ CH 2 CH 2 O - + H +
Cl -¬ CH 2 CH 2 OH ¾¾ O
Stronger acid - ve charge on O + Br
dispersed, hence
conjugate base stable Br O
CH3CH 2 OH ® CH3 ®- CH 2 O - + H +
¾¾
Weaker acid - ve charge intensified,
hence conjugate 13. (a) After hydrolysis
base unstable
HO O OH
O –
Cu KMnO4 OD O
5. (c) OH
300°C D 14. (c) H D H D
—CO2 NaOH
¾¾® ¬¾
®
CH 3MgBr
OH
H3O O O
6. (d) Stable due to H-Bonding between OH groups. H D CO2 – H D
.–. ¾® OOC
7. (a) Alcohols produced in a, b, c and d are given below
OH OD OH OH (C-D is stronger than C – H)
OH OH
CH2D –
D OOC D DOOC D
(a) (b) (c) (d) D+
¾®
8. (d) Reaction will occur by SN2 pathway.
9. (b) The ease of dehydration can be determined by seeing
O O
the stability of carbocation as intermediate.
Å Å
Mg
CH3 - CH 2 - CH 2 - CH 2 CH 3 - CH 2 - CH - CH 3 15. (b)
(I) (II) Cl MgCl
Å Å
CH 2 = CH - CH - CH3 OMgCl OH
(III) H3 O
(IV)
(IV) is most stable due to resonance followed by (III) which is
(X)
allylic carbocation. Next is (II) as it is a secondary carbocation
O
and least stable is (I) as it is a primary carbocation.
V
Alternatively, ease of dehydration can be determined from CH3 MgBr O MgBr
the stability of the product formed.
Cl Cl
> CH 2 = CH - CH = CH 2 > CH3 CH = CHCH3 > CH3 CH2 CH = CH 2

From IV From III From II From I –MgBrCl O


most stable conjugated more substituted Least stable
alkene (terminal alkene)
26 Aldehydes, Ketones and Carboxylic Acids
10. (b) Rosenmund’s reduction:
Exercise - 1
Acylchloride is hydrogenated over catalyst, Pd-BaSO4.
2 1
This reaction is called Rosenmund reduction.
1. (b) CH3 – CH – CHO
O
3 4
CH2 – CH3 C Cl H CHO
¾ ¾2 ®
2-methylbutanal Pd-BaSO4

O CH3 Benzyl chloride Benzaldehyde


1 2 || 3| 4
2. (c) CH 3 - C - C H - CH 3 11. (b)
3-methyl-2-butanone
CN CN = NH
3. (c) 4. (a)
+ SnCl2 + HCl
5. (b) O is more electronegative than C.
6. (a) (A) Cinnamaldehyde:
O H3O+
3
C
2 1 H CHO

3-Phenylprop-2-enal
(B) Acetophenoneone
CH3
O 12. (b) AlCl3
+ CH3X
Friedel
C
1 CH3 crafts
(A)
2 alkylation
H3O+ CrO 3 in
1-Phenylethan one Etard (CH3CO)2O
(C) Valeraldehyde reaction

5 4 3 2 1 CHO
CH3 CH 2 CH 2 CH 2 CHO
Pentanal
(D) Acrolein
13. (d)
3 2 1
CH 2 = CH – CHO 14. (b) Formyl chloride is unstable at room temperature.
15. (a) Alkanenitriles (other than methanenitrile) benzonitrile
Prop-2 -enal
(E) Mesityl oxide give ketones with Grignard reagents.
16. (b) R2Cd reacts with acid chlorides much more rapidly
5 4 3 2 1
H3C – C = CH CO CH 3 than with ketones so that in general very little of the tertiary
alcohol, formed by further reaction of R2Cd with the
ketone.
4-Methyl pent-3-en-2-one
17. (b)
7. (d) CH3 CHCl2 CHO
8. (a) Cl2/hu H2O
H CH3 H O CH3 373 K
O3
CH 3– CH 2– C C ¾® CH3– CH 2– C– C
CH3 Toluene Benzal chloride Benzaldehyde
CH3 O O
O O 18. (a) Primary alcohols on oxidation give carboxylic acids
CH 3– C – CH 3 + CH3 – CH2 – CHO as the final product, of course through aldehydes.
(–H2O) Oppenauer oxidation involves oxidation of 2º alcohols to
9. (c) ketones, and not for the oxidation of 1º alcohols.
EBD_8350
700 CHEMISTRY

19. (a) 23. (d)


OH CHO
2CH3MgBr |
COOC2H5 ¾¾H¾¾ ® C – CH3 ¾ H 3C
(i) O
CH3 ¾¾¾¾ 3®
H3C CH3 + CHO

¾¾¾¾
O 2 | (ii) Zn
A (C 9H 10O 2) CH3 OO
H2SO4 (conc.)
24. (c) Propanone has symmetrical structure.
(i) O3
C = CH2 25. (d) 26. (d)
(ii) ZnH2O | 27. (d) Cannizzaro reaction is given by aldehydes having no
(B) Alkene
CH3 a-hydrogen atom in the presence of conc. alkali, aldol
condensation is given by aldehydes and ketones having
C = O + HCHO at least one a-atom in presence of alkali or in presence of
| Formaldeyde acids
(C8 H8O) CH3 28. (b) Aldehydes containing no a-hydrogen atom on
20. (a) warming with 50% NaOH or KOH undergo
disproportionation i.e. self oxidation - reduction known as
OH Cannizzaro’s reaction.
2CH3MgBr |
COOC2H5 ¾¾H¾¾ ® C – CH3 ¾ 50% NaOH
2 HCHO ¾¾ ¾ ¾¾® HCOONa + CH 3OH

¾¾¾¾
O 2 |
A (C 9H 10O 2) CH3 29. (d) I2 and Na2CO3 react with acetophenone (C6H5COCH3)
to give yellow ppt. of CHI 3 but benzophenone
(i) O3 H2SO4 (conc.)
C = CH2 (C6H5COC6H5) does not and hence can be used to
(ii) ZnH2O | distinguish between them.
(B) Alkene
CH3 O
( - H 2O )
30. (b) CH3–C=O + H2N–NH–C–NH2 ¾ ¾ ¾ ¾®
C = O + HCHO O
| (Formaldeyde) H
(C8 H8O) CH3 CH3– CH=N–NH–C–NH2
21. (c) Acetaldehyde semicarbazone

O O
OH
C–CH3 CH2–CH3
CH – CH3 C – CH3
[O]
¾¾¾
® 31. (b) Zn-Hg/HCl
¾¾ ¾ ¾¾®

OH Phenyl methyl Ethyl


CHO ketone benzene
C–H This reaction is known as Clemmensen's reduction.
H3O + 32. (d) Compounds having – CHO group reduce Tollen’s
+ CH3MgBr ¾¾¾¾
® CH3 reagent to silver mirror. It is called silver mirror test.
O O
O || ||
H - C - OH CH3 - (CHOH) 3 - C - H
C – CH3 (a) (b)

+ CH3COCl
AlCl3
¾¾¾¾¾ ® O
Acylation
Both (a) and (b) have – C – H group, so both of them give
positive silver mirror test.
Distilation
(C6 H5CO2 ) 2 Ca ¾¾¾¾¾
® C6H 5 - C - C6H 5 + CaCO 3 33. (c) Only aldehydes and ketones react with
|| 2, 4-dinitrophenylhydrazine.
O
Benzophenone O OH
22. (a) || |
HCN
34. (b) CH3 – C – H ¾ ¾ ¾® CH3 - C - H
æ ö Acetaldehyde |
(i) BH , THF CN
¾ ¾ ¾ 3¾ ¾ ¾ ® ç ÷ B
è ø3 OH
Hydrolysis
|
H 2O 2, NaOH ¾¾¾¾¾® CH3 - C - H
|
H PCC
COOH
¬¾ ¾ ¾¾ OH Lactic acid
CH 2Cl2
O
ALDEHYDES, KETONES AND CARBOXYLIC ACIDS 701

35. (d) HCHO does not undergo iodoform test, while O OH


acetaldehyde undergoes iodoform test (I2 in presence of dil. NaOH
CH3 – C + HCH2CHO CH3 – C + CH2 – CHO
base) to form yellow precipitate of iodoform.
36. (c) Iodoform test is given by compounds which have H H
CH3CO group. Acetaldehyde Aldol
D
O CH3 – CH = CH.CHO
–H2O
|| Crotonaldehyde
CH 3 - CH 2 - CH 2 - C - CH 3 40. (a) Aldehydes, other than formaldehyde, when treated
2-Pentanone with RMgX give 2º alcohols.
O O
|| 41. (a) R C H + NH2 NH2 R C N NH2
CH 3 - CH 2 - C - CH 2 - CH 3 H
3-Pentanone Aldehyde Hydrazine Aldehyde hydrazone
Q 2-pentanone has CH3CO group, so it gives iodoform O OH
|| |
test, while 3-pentanone does not have CH3CO group, so it 42. HCN
(a) R– C –R ¢ ¾¾¾®R —C—CN
does not give iodoform test. KCN |
37. (c) In cross aldol condensation aromatic aldehydes or R¢
(A)
ketones (with or without a-hydrogen) react with aldehydes,
ketones or esters having a-hydrogen atoms in the presence OH
|
of dilute alkali to form a b-unsaturated carbonyl compound. LiAlH4 (B)
¾¾¾¾¾ ®R – C – CH 2 NH 2
Example, Re duction |

OH - 43. (c) The nucleophile is SO32 – not HSO3– , SO3Na
(i) C6 H5CHO + CH3CHO ¾¾¾®
– 2–
Benzaldehyde Acetaldehyde HSO 3 H + + SO 3
C6 H5CHOHCH 2 CHO O

SO3– HSO3–
– H2O C = O + :S – O C
O–

C6 H 5 CH = CHCHO SO 3
C + SO32–
Cinnamaldehyde OH
Addition
CH3 O product
| P
(ii) C6 H 5 - C = O + H 3C - C - C 6 H 5 44. (c)
Acetophenone 45. (d) These reactions lead to replacement of oxygen atom
CH3 of carbonyl group to form hydrazones and oximes.
O
| P 46. (b) NaBH4 selectively reduces the aldehyde group to
Aluminium
¾¾¾¾¾®C6 H5 - C = CH - C - C6 H5 alcohol without affecting double bond in an organic
t-butoxide compound. So, X is NaBH4.
Dypnone
38. (a) When acetaldehyde is treated with alcohol in the NaBH
C6 H5 CH = CHCHO ¾¾¾¾

presence of dry HCl , then acetal is formed C6 H5 CH = CHCH2OH
47. (a) Benzaldehyde undergoes Cannizzaro reaction, which
CH 3 CH 3 OR forms benzoic acid and benzylalcohol as the product.
dry HCl 48. (b)
C= O + ROH ¾¾ ¾¾® C
H H OH 49. (b) Carbonyl compounds have substantial dipole
Hemiacetal moments and are polar in nature. The high polarity of the
carbonyl group is due to resonance.
OR 50. (a) Aldehydes are generally more reactive than ketones
CH 3 in nucleophilic addition reactions due to steric and
ROH
H2O + C ¬¾¾ electronic reasons. Sterically, the presence of two relatively
H OR large substituents in ketones hinders the approach of
Acetal nucleophile to carbonyl carbon than in aldehydes having
only one such substituent. Electronically, aldehydes are
39. (a) Aldehydes and ketones havin g at least one more reactive than ketones because two alkyl groups reduce
a-hydrogen atom in presence of dilute alkali give the electrophilicity of the carbonyl carbon more effectively
b-hydroxy aldehyde or b-hydroxy ketone than in former.
EBD_8350
702 CHEMISTRY

51. (b) The hydrogen atom that is added to the carbonyl NH2 NH2
R R
carbon of the aldehyde in the reduction is derived directly HOH
from the other aldehyde molecule as a hydride ion. The C C
second hydrogen that is added to the negatively charged R CN
R COOH
oxygen is coming from the solvent (consult mechanism of 60. (b) The carbon atom of the carbonyl group of
Cannizzaro reaction). Oxidation of one molecule of the benzaldehyde is less electrophilic than carbon atom of the
compound at the expense of other molecule of the same carbonyl group present in propanal. The polarity of the
compound is known as disproportionation.
carbonyl group is reduced in benzaldehyde due to
52. (c) 53. (d) resonance and hence it is less reactive than propanal.
54. (b) In structure II, presence of positive charge on oxygen 61. (b) Aldehydes and ketones react with hydrogen cyanide
causes the displacement of p electrons toward oxygen, (HCN) to yield cyanohydrins. This reaction occurs very
making carbon more electron deficient than that in slowly with pure HCN. Therefore, it is catalysed by a base
unprotonated carbonyl group. and the generated cyanide ion (CN– ) being a stronger
55. (c) With ammonia, HCHO forms hexamethylenetetramine, nucleophile readily adds to carbonyl compounds to yield
CH3CHO gives acetaldehydeammonia addition product, corresponding cyanohydrins.
while C6H5CHO gives hydrobenzamide. 62. (c) Acetal formed upon reaction of ethylene glycol and HCl,
56. (b) which is unaffected by base hence unwanted reaction does
not occur due to presence of carbonyl group.
57. (d) Being reversible reaction, the backward reaction i.e.
63. (c) 2, 2-Dimethyl propanal gives Tollen’s test and
acetal -hemiacetal step can be restricted by minimizing water
3-methylbutan-2-one gives iodoform test.
content, i.e. by using dry HCl. The step hemiacetal -
aldehyde can be restricted by using excess of alcohol. 64. (d)
65. (a)
58. (c) 2CH3CHO ¾ ¾® 1st Product,
O O
2CH 3CH 2 CHO ¾¾® 2 nd Product
H 2O
a Product
rd
CH3CH 2CHO + CH3CHO ¾¾
®3 Product;
V
a O
th
CH3 C H 2 CHO + CH 3CHO ¾¾
® 4 Product
59. (b) R R OH
HCN NH
C = O ¾¾® C ¾¾®
3
66. (c)
R R CN

3
Å !
67. (d) H
¾¾ ® ¾¾® OH 1
$ 2
4
D CHO ¾ ¾ ¾®
Aldol 5 ¾
¾¾¾¾¾

reaction O CHO
O
OH
$ 1
O ! HO O 2
¬ ¾OH
¾¾ /D
¾ O H2O O (ii)
¬¾¾ 5
¬ ¾¾ ¾
Aldol 4
condensation 3

68. (a) C—H bond breaking occurs in rate determining step. COCH2
Order of stability of bonds C—T > C—D > C—H hence
rate of reaction will be C—H > C—D > C—T.
70. (d) ¾¾® Product (X)
69. (c) Ease of H donation µ negative charge density
O O
O
(b) is not the answer as it does not have —C—H linkage O O 6 5
1 2 3 4 5 4 6
71. (a) H—C—C—C—C—C—CH2 ¾¾®
which is necessary for H donation. 3 1
2
ALDEHYDES, KETONES AND CARBOXYLIC ACIDS 703

O O CHO
72. (b) Ph—C—H + CH3CH2CH—CHO Ph—CH—CH
CH2CH3
OH CHO
OH /D H 2O
Product –H2O
Ph—CH—CH
CH2CH3

73. (a) C N N H2 O NH H3 O O D O
CN
CN CN COOH
Hydrolysis is similar to Stephen’s reaction.

74. (b) After ozonolysis (b) will produce the desired Cross cannizaro reaction:
compound.
OH
75. (c) Enol form of (c) is aromatic. PhCHO + HCHO HCOO + PhHC2OH
76. (a) Aromatic aldehydes have less rate of N.A.R than Oxi.
aliphatic aldehydes and out of (3) and (4) former has more Red.
rate of N.A.R because –R effect of NO 2 increases Redox but not disproportionation
electrophilicity of CHO group. 81. (b) Both C–O bonds are identical and each O possesses
77. (a) 1, 2 and 4 are less reactive than ‘3’ because these are partial negative charge.
taking part in resonance with benzene ring which decreases OH
electrophilicity of carbon of carbonyl group. In between 2,
3 1
4 and 1, 2nd carbonyl group is more reactive because of –I 82. (a) H3C 2 COOH
effect of 3rd and 1st carbonyl group which increases its 6 4

electrophilicity. 5 CH3

O O IUPAC name of the structure is 3-ethyl-2-hydroxy


OH -4-methylhex-3-en-5-ynoic acid.
OH LiAlD4 83. (c)
78. (a)
H 84. (d) Primary and secondary alkyl groups are oxidised to
give carboxylic acid, while tertiary alkyl group remains
O O O unaffected.
Less hindered site 85. (b) Grignard reagent forms addition product with bubbled
carbon dioxide which on hydrolysis with HCl yields benzoic
O
D acid.
H3O O
Product
MgBr C OMgBr
O O (i) CO H O+
¾¾¾®
2
¾¾®
3

H O
79. (b) O3 /H2O/Zn/D NaOH/D
O
¾¾ ¾¾¾¾ ® O O ¾ ¾aldol
¾ ¾®
condensation
C O H
80. (d) All kind of Cannizaro reactions are redox as well as
disproportionation but in cross Cannizaro
disproportionation does not occur. + Mg(OH)Br
Cannizaro reaction: (P)
Benzoic acid
OH
2 PhCHO PhCH2OH + PhCOO
-
Red. CN NaOH
86. (c) R - X ¾¾ ¾® R - CN ¾¾¾¾
® RCOONa
Oxi.
(A) (B)
Redox as well as disproportionation
EBD_8350
704 CHEMISTRY

H O and higher inductive effect compared to -m and -p


87. (b) (C6 H5 CO)2 O ¾¾¾
2 ® 2C H – COOH
6 5 derivatives.
Benzoic anhydride Benzoic acid
O CH3
C6H 5COOCOCH3 ¾¾¾
H O
2 ® C H COOH + CH COOH || |
6 5 3
Bezoyl ethanoic Benzoic acid Ethanoic acid
102. (c) CH 3 - C - OH + HO– CHCH 3 ¾¾
®
anhydride Ethanoic acid Propan-2-ol
88. (c) Formic acid cannot be prepared by Grignard reagent. O CH3
89. (a) Grignard reagents and nitriles are useful for converting || |
alkyl halide into carboxylic acids having one carbon atom CH3 - C - O– CHCH3
more than that present in alkyl halides. 103. (c) pKa = –logKa; HCOOH is the strongest acid and hence
90. (b) it has the highest Ka or lowest pKa value.
K2Cr2O7 + H2SO4 104. (b) LiAlH4 in presence of ether can be used to convert
91. (c) CH 3CHO +[O] CH 3COOH acetic acid into ethanol.
Acetaldehyde Acetic acid
ether
92. (c) CH3CHO + HCN ¾® CH3COOH + 3LiAlH4 ¾¾¾®
Acetic acid
Acetaldehyde
[CH3CH2O]4AlLi + 2LiAlO2 + 4H2
OH OH H+
[CH3CH2O]4AlLi ¾¾
¾® CH3CH2OH
— —

— —
H2O/H+
CH3 — C — H CH3 — C — H Ethanol
COOH COOC2H5
CN COOH
Cyanohydrin Lactic acid
105. (c) HCl
Na/C 2H 5OH + C2H5OH ¾¾®
dry + H2O
93. (a) CH3CN Ethanol
HNO2
CH3 CH2NH2 CH3CH2OH This process is known as esterification.
(A) (B) 106. (a) –COOH group when attached to benzene ring
deactivates the ring and substitution occurs at
Cr2O72–/H +
m-position. (HNO 3 + H2 SO 4) is a source of + NO 2
(electrophile) which attacks at m-position.
CH3COOH
(C) COOH COOH
94. (d) O == C == O + Me3CMgI ¾® O == C — OMgI
H2SO4
+ HNO3 ¾¾¾¾

®
CMe3 NO2
H 3O + 3-Nitrobenzoic acid
O 107. (d) The yield of product in a reversible reaction can be
increased by (i) removing one of the products, (ii) taking
==

Me3C — C — OH either of the reactant in excess.


O 108. (d) Use of SOCl2 and ClCOCOCl forms gaseous by-
==

95. (b) Pyruvic acid is CH3 — C — COOH. products which can be easily removed, giving better yield
It can be prepared by oxidation of acetaldehyde of RCOCl. Further, oxalyl chloride is particularly easy to
cyanohydrin. use becasue any excess of it can be easily evaporated due
to its low b.p. (62ºC)
OH
O O O

CH3 — CH == O + HCN ¾® CH 3 — CH — CN || || ||
OH O R - C - OH + Cl - C - C - Cl ¾¾
®
O

==

H+ [O] ||
CH3 — CH — COOH CH3 — C — COOH R - C - Cl + HCl ­ + CO ­ +CO 2 ­
H 2O
Pyruvic acid
109. (a) Salicylic acid, because it stabilizes the corresponding
96. (b) 97. (d)
salicylate ion by intramolecular H-bonding.
98. (b) Due to intermolecular H-bonding. 110. (b) 111. (b)
99. (c) 112. (a) An electron releasing substituent (+I) intensify the
100. (d) In carboxylates (conjugate base of carboxylic acids), negative charge on the anion resulting in the decrease of
resonance is more significant because the two resonating stability and thus decreases the acidity of the acid. Hence
structures are similar, while in phenoxide, the resonating acid character decreases as the + I-effect of the alkyl group
structures are not equivalent, alkoxide ions do not show increases as
resonance. CH3– < CH3CH2– < CH3CH2CH2– < CH3CH2CH2CH2–
101. (a) -o-Nitrobenzoic acid is more acidic due to ortho effect Hence the order becomes : (i) > (ii) > (iii) > (iv)
ALDEHYDES, KETONES AND CARBOXYLIC ACIDS 705

OMgBr OH
COCH3 C2H5 – C – CH3 C2H5 – C – CH3
C H MgBrC2H 5 H+
113. (a) CH3COOH +PCl5 ¾ ¾ ® CH3COCl ¾ ¾ 6¾6¾ ®
Anhyd. AlCl ether
3
[A] (hydrolysis)
[B] [C]

114. (a) Br MgBr


Mg/THF CO2
115. (c) Electron withdrawing substituent (like halogen, — 119. (d)
NO2, C6H5 etc.) would disperse the negative charge and I MgI H3 O
hence stabilise the carboxylate ion and thus increases
acidity of the parent acid. On the other hand, COOH CO
D
electron-releasing substituents would intensify the O
–H2 O
negative charge, destabilise the carboxylate ion and thus COOH CO
decrease acidity of the parent acid.
120. (c) Rate of decarboxylation
Electronegativity decreases in order µ stability of carbanion produced as intermediate.
F > Cl > Br - CO
RCOOH ¾¾¾
2
® R0
and hence –I effect also decreases in the same order, (intermediate)
therefore the correct option is V
[FCH2COOH > ClCH2COOH > BrCH2COOH > CH3COOH]
116. (d) V is most stable because its anion is stabilized to a CH3
greater extent through H – bonding with H atom of OH V OCH3
present on both ortho-positions ; followed by II in which OCH3
one OH group is present. Compound IV comes next to II
because here –OCH3 group is present in ortho position
V V
which although is not capable of forming H–bonding yet
(More stable
more acidic than p-HOC6H4COOH (III) due to ortho effect. due to – I ef fect of OCH3)
Compound III is less acidic than benzoic acid because of
electron-releasing group in the para position. Thus O V
121. (b) O
C—O C2H5
:

COOH COOH NH2 NH2 OC2H5


HO OH OH
NH2 C—O C2H5 NH2 OC2H5
> >
:

O
V
V II H H H
N O N O
Å
COOH COOH COOH
– 2H Å
OCH3 N O N O
H H
> > H
122. (b)
IV I 123. (b) O
OH H
18
Ph
III
Ph—C—O OH

HVZ SOCl2
117. (c) CH 3COOH CCl3COOH
V
H2 O H 18 V
CCl3COCl CCl3CHO 18
Ph O
Pd/BaSO4
Ph—C—O PhCOOH +
OH OH H Ph
HCOO + CHCl3
18
118. (a) Rate of esterification is, OH
V
1° alc > 2° alc > 3° alc PhCOO +
So, CH3COOH will be esterified most readily. Ph
H
EBD_8350
706 CHEMISTRY

124. (b) Vicinal dicarboxylic acid on heating


Exercise-2
undergoes dehydration

O O
18
COOH 1. (2) CH3¾ C¾ OH ¾H¾
2O
® CH3¾ C¾ OH ¾ ¾ ®
COOH
COOH D
+
18O H2
– CO2
COOH COOH
O

¾¾¾
OH O
– H2O D
O ¬¾¾ CH3¾ C¾ OH ¾ ¾ ® CH3¾C = O

18OH 18OH
O 18
18O H O
18
Br H O 18
¾¾®
2
CH3¾C ¾OH ¾¾® CH3¾C¾OH
125. (d) COOH
P + Br2
COOH
H2 O 18OH
[W]
OMe OH
CN 2 H+/H O
2. (1) ¾¾¾®
KCN H3 O OMe OH
COOH
[X] COOC2H5 COOH

COOH
heat H
heat ¾¾®
COOH COOH H
[Y]
(Z)
126. (b) COOH
b-Keto acid
127. (a) ‘Si’ is less electronegative than ‘C’ hence +I effect of
Remember that b-keto acids undergo decarboxylation on
(CH3)3 Si group is greater than that of (CH3)3C group, so (i)
heating.
is more acidic than (ii). 3. (5)
128. (b) O O
OH NaBH4 OH
¾¾¾®
CH3O COOC2H5 HO COOH
H3 O
HO
O O
CH3O COOC2H5 COOH A g-hydroxy acid

heat COOH – H2 O H O+
O COOH O
¾¾¾¾
3
®
COOH cyclisation
O O
4. (4) (i), (iii), (iv), (v)
C2H5 C2H5 O
D
129. (c) CH3—C—COOH CO2 CH3CH—COOH CH3 C OC2H5 ¾¾®
*
COOH
é ù
Product is optically active and hence it will ê O O- ú
ê ½½ ½ ú
exist in two forms ‘d’ and ‘l’ ê - CH 2 ¾ C ¾ OC2H5 ¬¾® CH 2 = C ¾ OC2H5 ú
130. (a) Sodalime produces following carbanion, ê More stable ú
ë û
V OH
2 H+
SN ¾¾¾
® CH 2 = C OC 2H5 .
An enol

Decalin
5. (3) Dicarboxylic acids having two —COOH groups on
Br the same carbon atom ; and b-keto acids are easily
decarboxylated on heating.
ALDEHYDES, KETONES AND CARBOXYLIC ACIDS 707

6. (3) Stronger the basic nature of the leaving group, weaker CH2OH
will be its leavability. In CH3COOH, OH– is a strong base –
OH / HCHO
+
so it can’t be removed easily to form CH3 C O required for 3rd aldol HOCH2 C CHO
condensation
acetylation (acylation). CH2OH
7. (1)
O CH2OH
1. O3 NaOH(aq) –
¾¾¾ ¾® ¾¾¾¾® OH / HCHO
2. Zn, H2O HOCH2 C CH2OH
Cannizzaro
O reaction
CH2OH
OH
O Exercise-3
heat
¾¾® ¾¾®
–H2O
1. (b) But-1-yne on reaction with water in presence of Hg2+
(–) O O O ions as a catalyst produces butan-2-one.
CHO CH2 CH Hg2+
CH2 CH2
¾¾® C
H2 O CH3
CO, HCl CH3 C
8. (4) ¾¾¾¾¾¾¾ ® ;
Anhyd AlCl 3/CuCl But–1–yne OH
(Gatterman Koch Reaction)
Tautomerism CH2 CH3
CHO ¾¾¾¾¾¾ ®
OH
CH3 C
CHCl2 CH
; ¾¾¾® OH
100°C O
Butan-2-one
terman Koch Reaction)
OH 2. (a) The carbonyl group in ketones being influenced by
CH two alkyl group is less reactive than in aldehydes where
OH ¾¾¾® the carbonyl group is under the influence of one alkyl
100°C – H2O group only. As the number of alkyl group increases both
the +I effect and the steric hinderance get increases
COCl CHO preventing the attack of nucleophile.
H2 Now among benzaldehyde and acetaldehyde former is less
¾¾¾¾ ® ; electrophilic than carbon atom of carbonyl group present
Pd–BaSO4
(Rosenmund Reaction) in ethanal. The polarity of carbonyl group is reduced in
benzaldehyde due to resonance hence it is less reactive
than ethanal.
COOMe 3. (c) Due to the electron withdrawing characteristic of –
DIBAL–H Cl, chloroacetic acid is more acidic than acetic acid further,
¾¾¾¾¾¾¾¾®
Toluene, – 78°C H2O in phenol phenoxide ion obtained on ionisation of phenol
is stabilised due to resonance where as no such
CHO stabilisation occurs in case of ethanol. Hence phenol is
stronger acid than ethanol. Thus acidic strength increases
in the order.
9. (2) (II) does not have a, b-unsaturated carbonyl group, ethanol < phenol < acetic acid < chloro acetic acid
while (IV) can't form stable carbanion (from acetone). 4. (b) Compound Ph — COO — Ph can be prepared by the
Remember that cyclopentadiene is quite acidic in reaction of phenol and benzoyl chloride in the presence
nature, because its carbanion is stable due to of pyridine.
presence of conjugated system; hence it functions O
as the nucleophile required in Michael reaction. OH ||
Cl N
a + ¾¾¾®
OH – O
10. (3) CH3 – CHO + HCHO ¾¾¾¾
1st aldol
®
||
condensation Phenol Benzoyl
chloride C
a CH2 OH O
CH2 CHO –
OH / HCHO a
¾ ¾ ¾ ¾ ¾ ® CH CHO
2nd aldol
CH2 OH condensation
CH2 OH
This is an example of Schotten-Baumann reaction.
EBD_8350
708 CHEMISTRY

5. (c) Acetone and benzaldehyde both do not react with 13. (d) Any substituent in the carbonyl compound that
Fehling's solution. increases the positive charge on the carbonyl carbon will
6. (d) Cannizzaro reaction is given by those aldehydes increase reactivity towards nucleophilic addition.
which have absence of a-hydrogen atom. So, CH3CHO
will not give Cannizzaro reaction. O
7. (b) Benzaldehyde having no a-hydrogen on reaction ||
with aqueous KOH solution undergo Cannizzaro reaction OH O - C - CH3
and produces benzyl alcohol and potassium benzoate.
14. (a)
–+ COONa COOH
CHO CH2OH COOK (A)
Aspirin
(Acetyl salicylate)
KOH (aq ) (B)
¾¾¾¾¾ ® +

Benzaldehyde Benzyl Potassium


4 ® CH CH OH ¾¾¾LiA1H
5® PCl
alcohol benzoate 15. (c) CH3 COOH ¾¾¾¾ 3 2
8. (d) Chemical reaction can be shown as (A)

OH H alc.
| | CH3CH2Cl ¾¾¾ ® CH2 = CH2
40% H 2SO4 KOH
CH3 — C º CH ¾¾¾¾¾® CH3 — C = C H (B) (C)
1% HgSO4
Propyne Prop-1-en-2-ol

O 16. (b) It does not give iodoform test due to absence of group.
|| It also does, not give Tollen’s test. On reduction it gives n-
Keto-enol tautomerism
¾¾¾¾¾¾¾¾¾ ® CH3 — C— CH 3 pentane
Acetone
reduction
9. (b) Chemical reaction can be shown as H3C—CH 2—C—CH2—CH3 ¾¾¾¾®
Zn-Hg/HCl
(i) CH MgBr
3 H 2SO 4 , D
CH3CHO ¾¾¾¾¾¾ ® CH3— CH— OH ¾¾¾¾¾® O
Ethanal (ii) H 2O | Dehydration
CH3 CH 3—CH 2—CH2—CH 2—CH 3
n-pentane
Propan-2-ol
(A) 17. (d)
Hydroboration 18. (d) Enolic form of ethyl acetoacetate has 18 sigma and 2
CH3 — C = CH2 CH3 — CH2 — CH2OH pi-bonds as shown below:
| propan -1-ol
H
Propene
(C) H H
(B) s s s
s s s
10. (c) H C C p C s
C p
O
O s s H H
s
|| H Os H s s
I2 / NaOH solution s
CH3 — CH = CH — CH 2 — C— CH 3 ¾¾¾¾¾¾¾ ¾® Os C C H
( Iodoform reaction ) s
s
s
Hex - 4 - en - 2 - one H H
O
|| O
CH3 — CH = CH — CH 2 — C— OH ||
H+
19. (b) R — C— R ¢ + NH 2 — NH 2 ¾¾¾
®
Pent -3-en -1-oic acid

11. (b) 2° alcohols on oxidation with alkaline KMnO4 NH2


solution produce ketones. OH N

[O ] CH3 R—C—R¢
– H 2O
¾¾¾¾¾ ® R—C—R¢
¾¾¾¾® Elimination
KMnO 4
OH O NH—NH2
Butan-2-ol Butan-2-one (Addition)
(2° alcohol)
12. (a) Clemmensen reduction is used to convert carbonyl While in all other cases no elimination of water take place.
group as follows 20. (a) Among the substituent attached to the benzene ring,
Zn (Hg )+ HCl –NO 2 group is the most electron withdrawing, so
C = O ¾¾¾¾¾¾ ® CH2 withdraws electron density from carbonyl carbon, thus
Zinc amalgam and HCl act as reagent in this reaction. facilitate the attack of OH– ion.
ALDEHYDES, KETONES AND CARBOXYLIC ACIDS 709

21. (d) When 1, 3-dimethylcyclopentene is heated with ozone 27. (b) DIBAL-H is an electrophilic reducing agent. It reduces
and then with zinc and acetic acid, oxidative cleavage leads both ester and carboxylic group into an aldehyde at low
to keto - aldehyde. temperature.

CH3 6
CH3 O O
5
O OH O
(i) O3,- 78 °C
¾¾¾¾¾¾¾ ® 4 O C1 – H
(ii) Zn- CH 3COOH DIBAL - H H
3 2 ¾¾¾¾®
CH3
CH3 CHO
5- keto – 2 – methylhexanal
CO2H

22. (d) Keto-enol tautomerism is possible only in those 28. (c) Carboxylic acids have higher boiling points than
aldehydes and ketones which have at least one a-hydrogen aldehydes, ketones and even alcohols of comparable
atom, which can convert the ketonic group to the enolic molecular mass. This is due to more extensive association
group. through intermolecular H-bonding.
23. (a) Schiff base is formed when 1° amine reacts with O ¼¼¼H O
aldehydes. R C C R
R
H+
R O H ¼¼¼O
C = O + R' — NH 2 ¾ ¾¾ ® C = N – R'
H H COOH
Aldehyde Primary amine Schiff base 29. (b) + NH 3
COOH
24. (b) Since 'A' gives positive silver mirror test therefore, it
must be an aldehyde of a-Hydroxyketone. Also, reaction Å
CONH2
COONH4
with OH– i.e., aldol condensation (by assuming alkali to be D
¾¾¾¾ ®
dilute) indicates that A is aldehyde as aldol reaction of Å -2H 2O
COONH4 CONH2
ketones is reversible and carried out in special apparatus.
It indicates that A is an aldehyde
O
CH 3 - CHO CH3 CH2OH
Strong C
(A) (X) Heating
¾¾¾® NH
O –NH3 C
CH3 - CH = N - NH - C - NH 2 CH3 - CH = CH - CHO O
(Z) (Y)
Semicarbazone But-2-enal (i)AlH(i -Bu)
(c) R — C == N ¾¾¾¾¾¾ ® R — CHO
2
30. (ii)H O 2
O O
The reduction of nitriles to aldehydes can be done using
H
(i) OH
– DIBAL-H[AlH(i-Bu)2].
+ H ¾ ¾®
||

25. (a) (ii) D 31. (a) For the given reaction condition, the major product
is:
O

O O

[Ag(NH ) ]OH H+/CH OH Br OH


26. (a) ¾¾¾¾¾®
3 2 ¾¾¾¾¾
3
Tollens reagent (esterification)
Br Br
CHO CO2H
==
OH
HO CH3 O
Br D Br
CH3MgBr,
¬¾¾¾¾
+
H /CH3OH 32. (b) LiAlH4 reduces esters to alcohols but does not
H3C – C – CH3 reduce C = C.
C LiAlH
OH O OCH3 CH3CH = CH – CO2Me ¾¾¾¾
4
® CH3CH = CHCH2OH
EBD_8350
710 CHEMISTRY

38. (d)
33. (a) Alkaline KMnO 4 converts R with a CHO (i) NaBH 4

benzylic hydrogen into benzoic acid. Hex-3-ynal OH


(ii) PBr4
CH2CH3 COOH
Br
(i) alkaline KMnO (iii) Mg/ether
¾¾¾¾¾¾¾®
4
+ (ii) H3O MgBr

34. (d) The acidic strength of a compound or an acid depends


on the inductive effect (–I). Higher the (–I) effect of a
substituent higher will be acidic strength. Now, the
decreasing order of (–I) effect of the given substituents is H+ O
NO2 > CN > F > Cl. C
\ The correct decreasing order of acidic strength amongst COOH
O
the given carboxylic acids is:
NO 2CH2COOH > CNCH2COOH > FCH2COOH >
Exercise-4
ClCH2COOH
d– 0
35. (b) O d– d+ O MgBr 1. (c) C— D bond is stronger than C—H bond so H
CH3–MgBr migration will occur
CH3–C–CH3 CH3–C–CH3
d+ O
Acetone CH3
CDO 0
Hydrolysis + OH C—D
OH CHO
HO—C—H
CH3–C–CH3 0
0 O
O
CH3
(tert- Butyl alcohol) H + transfer
H—C—D CHDOH
OH 0
COOH COO
CH3 CHCl2 CH – OH 2. (d) It is an example of intramolecular aldol condensation.
The carbon atom labelled as a acts as a nucleophile.
Cl2/hv H 2O a
36. (b) –
373 K O ¾® OH O ¾®
(X)
H H
O
–H2O O O
OH
CHO
–H O
¾¾®
2

O
OH (CH CO) O
O O 3. (d) ¾¾¾¾¾
3 2
®
37. (c) CH3COONa

dil. OH CHO (Perkin reaction)
C–H+ C – CH3 ¾¾¾¾®
Cross Aldol o-Hydroxybenzaldehyde
reaction
OH H O OH
OH O O
C–C–C C=O
¾®
H H
O
–H2O 4. (d) Aldol condensation :
¾¾® C H = CH – C
OH New C–C bond

In the presence of dil. NaOH, C 6H5CHO and OH
2CH3CHO ¾ ® H3C–C–CH2CHO
C6H5COCH3 will react to undergo cross-aldol
condensation. H
ALDEHYDES, KETONES AND CARBOXYLIC ACIDS 711

Kolbe reaction : 7. (c) (a)


+ +
ONa OH New C–C bond H H
C = O + H2NNH2 ¾® C – OH ¾® C = NNH2
COOH
CO ,140 ºC NHNH2
¾¾¾¾®
2
Pressure
(b) In the reduction of carbonyl group with LiAlH4 or
Reimer-Tiemann reaction : NaBH4, a hydride ion is transferred from the metal to the
New C–C bond carbonyl carbon (nuclephilic addition)
OH OH
H
CHO – –
¾¾¾®
3CHCl
+ 3KCl + 2H2O C = O + H– AlH3 ¾ ® C – OAlH3
3KOH
8. (c)
Wurtz Fittig reaction :
Na
CH3 NH3 CH3 OH –H O CH3
H3CCl + 2Na + Cl ¾¾® C = O ¾¾® C
2
¾¾® C = NH
C2H5 C2H5 NH2 C2H5
H3C + 2NaCl –
– CH3 NH H O CH3 NH2
CN
¾® C ¾¾®
2
C
New C–C bond C2H5 CN C2H5 CN
5. (d) The keto group has considerable resonance energy ;
9. (c) Ph — CH == CH — CHO = 2 (Cis and trans)
hence more the number of keto groups in a compound,
higher is its stability. However, here all the three compounds CH3 — CH == CH — CHO = 2 (Cis and trans)
have same (two) number of keto groups. Compound II is 10. (d) Intramolecular cannizaro followed by esterification.
least stable because two adjacent keto groups destabilize
the molecule due to positive charge on the adjacent carbon CH2OH CHO
atoms. Relative stability of I and III can be ascertained by
PCC OH
the stability of their fully reduced forms (phenols), since
D
fully reduced form of III has two benzene rings, it will be
more stable than the I whose reduced form has only one CH2 OH CHO
benzene ring. Thus CH2OH CH2OH

d
O O – H/
d
d
+
d
+
O
O O > > 0
COO COOH
O
III I II Esterification –H2O
6. (b) The reaction is a part of haloform reaction.
O O
|| ||
OH - –
CH3 - C - CH 2Cl ¾¾¾® CH3 - C - CHCl
C HCl
O
I
O
O 11. (b)
- || O O
or C H 2 - C - CH 2 Cl O
OH
II CHO CH
OH
O
H
However, – I effect of –Cl makes the H’s of CH2Cl more
acidic, hence I is more likely to be formed. OH O
O NaBH4 O H3 O
||
H 3 C - C - CH 2 ¾¾
® Cl
OH
O O CHO OH
- || - ||
OH Cl2 +
Cl ¾¾¾® H3C - C - C H - Cl ¾¾¾
® H3C - C - CHCl2 OH
EBD_8350
712 CHEMISTRY

12. (c) Fehling solution (Cu2+/OH ) oxidises CHO 15. (b)
into COOH then esterification occurs in presence of H+.
13. (b) Rate of decarboxylation (RCOOH) a stability of
0 COOH CO
Carbanion (R) . CH3 OH
– H2 O O
14. (a) COOH CO

COOH COCl
O SOCl2
OH O
O
2
1
H
COOCH3 COOCH3
O H3O COOH D
3 b
a –CO2
O 4 OH –HCl CH3NH2
5 OH [X]

Ketone can show 2, 4- DNP test. CONH2

COOCH3
AMINES 713

27 Amines
O
Exercise - 1

1. (a) RX / DMF RNH


¾¾¾¾¾® NR ¾¾®
2

2. (b) The fourth orbital of nitrogen in all amines contains S N2 reaction


an unshared pair of electrons. Due to the presence of
unshared pair of electrons, the angle C–N–E, (where E is C O
N-alkyl phthalimide
or H) is less than 109.5°. O
3. (c) RNH 2 R 2 NH R3N
Primary amine Secondary amine Tertiary amine OH
+ H2N – R
4. (c) 1° amines have –NH2 group in their structure, 4
OH Primary
primary amines are possible for C4H11N. amine
CH 3 - CH 2 - CH 2 - CH 2 NH 2 CH 3 - CH 2 - CH - CH 3 O
| Phthalic acid
NH 2
15. (d)
(i) (ii) 16. (d) Reduction with iron scrap and hydrochloric acid is
CH3 CH3 preferred because FeCl2 formed gets hydrolysed to release
| | hydrochloric acid during the reaction. Thus, only a small
CH3 - C - CH 2 - NH 2 CH3 - C - CH3 amount of hydrochloric acid is required to initiate the
| |
NH 2 reaction.
H
17. (b) 18. (c) 19. (a)
(iii) (iv) 20. (a) The reaction is Hoffmann bromamide reaction
5. (d)
6. (b) The compound contains longest chain of 6C atoms O
and amino group. Hence it is an alkanamine. ||
R - C - NH 2 + Br2 + 4NaOH ¾¾
®
CH3 C2H5
7. (a) R - NH 2 + 2NaBr + Na 2CO3 + 2H 2O
HC – N
CH3 C2H5 O
||
N, N- Diethylpropan - 2-amine R – NH2 contains one carbon less than R - C - NH 2
8. (c)
21. (b)
9. (c) The compound is derivative of aniline. H
10. (c) CH3CHO + CH3NH2 D
CH3—CH== N—CH3
11. (c) CH3NC (methyl isocyanide) on reduction with LiAlH4 H2 / Ni
gives secondary amine D
CH3—CH2—NH—CH3
O
|| 22. (d) RNCO + 2KOH ¾® K2CO3 + RNH2
12. (b) CH 3 - CH 2 - C - NH 2 + Br2 + KOH
gg CO NH2
D
¾¾® CH3 - CH 2 - NH 2 N—R +
CO NH2
R - N ®C
4[H]
13. (d) ¾¾¾® RNH - CH3
Alkyl isocyanide Secondary amine CO NH
O O RNH2 +
CO NH
KOH 23. (b) Aromatic amines can not be prepared by Gabriel
14. (c) NH ¾¾® NK
Phthalimide reaction.
O O 24. (b) Amines possess fishy smell.
Phthalimide N-Potassium phthalimide 25. (c)
(a nucleophile) 26. (d) Amines are basic in nature
EBD_8350
714 CHEMISTRY

27. (b) Lower aliphatic amines are soluble in water and their 51. (b)
solubility decreases with increase in molar mass of amines.
52. (a) HNO3 + 2H 2SO4 2HSO4- + NO2+ + H3 O+
Higher amines are essentially insoluble in water.
28. (a) B.P. µ Intermolecular forces of attractions like H- 53. (a)
54. (b) Greater the delocalisation of electron pair on N, lesser is
Bonding.
its availability for protonation leading to lesser basic character.
29. (a) B.P. µ number of H-Bonds

:
NH2 CH2NH2
30. (a)
31. (a) Secondary amine is more basic than primary amine.
32. (a)
33. (a) Secondary amines are more basic than tertiary amines > >
due to stabilisation of 2° amine by hydrogen bonding with
IV II
solvent molecule. Delocalisation not Delocalisation not
34. (c) Aliphatic amines are more basic than aromatic amines. possible, cyclohexyl gp. possible, phenyl gp. is
Resonance decreases the basic character due to delocalisation is electron-releasing electron-withdrawing
of shared pair of electrons on nitrogen within benzene nucleus.
Further electron withdrawing if (–NO2) decreases basicity. H

:
NH2 :N
35. (b)
36. (c) Aryl amines produce diazonium salts on treatment
with nitrous acid
>
37. (c) We know that
CH 3CH 2 NH 2 + CHCl3 + 3KOH ¾¾ ® I III
CH3CH 2 NC + 3KCl + 3H 2O Delocalisation possible, more
in III than in I. Further C6H 5– gp. is
In this reaction, bad smelling compound ethyl isocyanide electron-withdrawing
(CH 3CH 2 NC) is produced. This equation is known as 55. (b) Although – NH2 group is o,p - directing but in presence
carbylamine reaction. +
of conc. HNO3 it undergoes protonation to form – N H3
38. (a) Acylation occurs in one step only because lone pair
of nitrogen is delocalized with acyl group. which, being electron - deficient, becomes m- directing.
+
:NH2
O O– NH3
..
R – NH – C – CH3 ¬¾® R – NH – C – CH3 conc. HNO3
¾¾ ¾¾ ¾ ®
conc. HNO3
¾¾ ¾¾ ¾ ®
HNO
39. (b) RNH2 ¾¾¾¾

ROH + H2O + N2 ­
– NH2 group is – NH3+ group is
(A) (B) (C) o, p - directing m - directing +
40. (a) 1° amines (aliphatic and aromatic) react with N H3
CHCl3/KOH to yield isocyanide (foul smelling) This is known
as carbylamine test which is not given by 2° and 3° amines.
41. (c) The compounds containing active H-atoms (H atoms
attached to N, O or S) react with CH3COCl to form acetyl NO2
derivatives. Protonated
m - nitroaniline
42. (b) 43. (a) 44. (d)
45. (a) Hinsberg’s method is based on the use of benzene NH2 NH – CO – CH3
sulphonyl chloride.
46. (d) Aniline is insoluble in water, because its –NH2 group (CH CO) O
56. (b) 3 2 ®
¾¾¾¾¾¾
can’t form H- bond with water due to bulky phenyl group.
47. (d)
48. (a) Pyridine is a stronger base than the amine, pyridine [X]
removes HCl formed in acylation reaction of amines and
shifts the equilibrium to the right hand side. NH – CO – CH3 NH2
49. (b) The hydrogen attached to nitrogen in sulphonamide
is strongly acidic due to the presence of strong electron
HNO H O+
withdrawing sulphonyl group. Hence, it is soluble in alkali. ¾¾¾¾

¾¾¾®
3

50. (d) In case of substituted aniline, electron releasing H 2SO 4


groups like –OCH3, –CH3 increase basic strength whereas
electron withdrawing groups like –NO2, –SO3H, –COOH, NO2 NO2
–X decrease it. [Y] [Z]
AMINES 715

NH2 66. (d) Benzene ring becomes less reactive towards


electrophile because + R effect of NH2 is greater than —
57. (c) 3CH COCl NH—COCH3
¾¾¾¾¾
®
67. (b)
O O

NH – C – CH3 NH – C – CH3 CH2 — CH2 + NH3 CH2 — CH2

Br – H O O O NH3
¾¾¾®
2 2

O
Br CH2 — CH2 ¾¾¾® CH2CH2 NH2CH2CH2OH
(A) (B) NH2

¾®
OH NH2 O
+
H O
¾¾®
3

O P2 O5
heat CH2—CH2—NH—CH2—CH2—OH
Br N –H2O
(C) OH
58. (a) It is a tertiary amine hence shows above observations. H
59. (c) Delocalisation of lone pair of electrons of nitrogen + -
NH2 N2 Cl
1
µ
Basic character
NaNO ,HCl
60. (b) 68. (b) ¾¾¾¾¾
2 ¾
®
(0 -5°C)

O Aniline Diazonium chloride

69. (a) Excess of HCl is used to convert free aniline to aniline


O C
hydrochloride otherwise free aniline would undergo
NH
61. (c) NH2—C—NH2 CH2 coupling reaction with benzenediazonium chloride.
(A) C==O 70. (a) Azo dye is prepared by diazo coupling reaction of
C NH phenol with diazonium salt.
O –
(B) N+2Cl OH
Lone pair of nitrogen in (B) are more delocalised and hence
these lone pairs of electrons are less available for protonation, + ¾® N=N OH
consequently basic character decreases or (pKb)B > (pKb)A Benzene p–Hydroxyazobenzene
62. (b) diazonium
chloride
63. (a) Lone pair of electrons are highly delocalised and
hence less available for protonation. (– N = N –) group is called azo – group.
64. (d) Acid strength µ Stability of conjugate base. + -
NH2 NºNCl
65. (c)
NaNO ,HCl CuCN
NH2 71. (d) 2
¾¾¾¾® ¾¾®
0°C
Aniline (diazotisation) (A)
LAH
ClCOCH2CH2CH2NH2 benzene
diazonium chloride
OH
H+ CN CH2NH2
NH2
LiAlH4
OH ¾¾¾®
– H2O
N N (B) (C)
Cyanobenzene Benzylamine
H H H
EBD_8350
716 CHEMISTRY

72. (b) The given reaction is known as Sandmeyer’s reaction. 80. (d) The reaction can be completed as follows:

+ –
N2 Cl OH

H O
73. (b) ¾¾2¾® + N2+HCl

CH3 CH3
p-Cresol

74. (c) Primary aromatic amines react with nitrous acid to


yield arene diazonium salts.
cold
ArNH2 + NaNO2 + 2HX ¾¾¾
®
Exercise-2
1° Aromatic amine
Ar—N = N+X– + NaX + 2H2O 1. (b) 3° amine is the amine in which nitrogen is attached
Arene diazonium salt to three alkyl groups whether they are same or different.
The diazonium group can be replaced by fluorine by

:
treating the diazonium salt with fluoroboric acid (HBF4).
The precipitated diazonium fluoroborate is isolated, dried CH3CH2 N CH2CH3
and heated until decomposition occurs to yield the aryl CH2CH3
fluoride. This reaction is known as Balz-Schiemann Triethylamine
(3° amine)
reaction.
3 2 1
HBF4 heat 2. (d) IUPAC name of CH 2 = C HC H 2 NHCH3 is
Ar—N2+X– ¾¾¾® Ar—N2+BF4–¯ ¾¾¾
®
N-methylprop-2-en-1-amine.
Ar—F + BF3 + N2 3. (c) More the hydrogen bonding more will be the basicity
75. (a) The yield in Sandmeyer reaction is found to be better in gaseous phase but in aqueous solution solvation effect
than Gattermann reaction. also play the role. By combining both the effects order of
76. (a) 77. (b) basicity in aqueous medium is 2° > 3° > 1° > NH3. The
inductive effect comes to play when order of amines is
+ –
78. (b) ArN 2Cl + H 3PO 2 + H 2O ¾® same as in case of (c) & (d) where both the amines are
secondary. +I group increase the basicity and –I group
ArH + N 2 + H3PO 3 + HCl will decrease the basicity of amines.
+ – 4. (a) In aniline, lone pair of nitrogen is not available for
ArN 2Cl + C 2H5OH ¾®
donation to the acid as it is involved in the resonance.
ArH + N2 + CH3CHO + HCl Thus, it will act as the weakest Bronsted base amongst
79. (b) the given compounds.
+ + +
NH2 NH2 NH2 NH2 NH2
NO2 NO2 NO2
– –
HNO3 Fe/HCl
¾¾¾® ¾¾¾® –
H2SO4
I II III IV V
NO2 NH2
5. (c) SN1 reaction involves the formation of carbocation,
HNO2 so, more stable carbocation will have higher reactivity
towards SN1 mechanism.
NO2 NO2 Å
C6H5CH2Br ¾® C6H5 – CH2 + Br
–N Å
¬¾¾¾
2
As C6H 5CH 2 is the most stabilised carbocation due to
H2O + –
OH N2Cl resonance, hence it will be most reactive towards SN1
mechanism.
AMINES 717

11. (b) Hoffmann bromamide reaction.

H2(excess)/Pt
NH2
® O
NO2 H3C ||
NH2 NH2 CH3
6. (b) Fe/HCl
®
NaOH/Br CH — NH2
Sn/HCl NH2 ¾¾¾¾®
2

®
2-phenylpropanamide 1-phenylethanamine
LiAlH4/ether
® No reaction 12. (b) Hoffmann bromamide degradation reaction:
O
7. (c) 1° amine with increased –CH2 group can be achieved ||
by following the given steps: Br2 / NaOH
NH2 ¾¾¾¾¾ ®
KCN Na/C O H
X ¾¾® C º N ¾¾¾®
2 5
CH2NH2 Benzamide
alkyl –KX NH2
halide (Nucleophilic (reduction of CN ) (1° amine)
substitution of
alkyl halide by CN )
+ Na2CO3 + NaBr + 2H2O
8. (d) Potassium phthalimide is the source of nitrogen in
Aniline
Gabriel phthalimide synthesis.
13. (d) NH2 NH2 NH2
O
||
C CO > >
–H O –+ R–X
NH + KOH (alc.) ¾¾®
2
NK ¾¾¾®
C CO
|| CH3 NO2
O III I II

Higher the electron density towards ring, higher will be


CO H +/H2O, D COOH
or its basic strength. Electron donating group increases the
N–R ¾¾¾¾¾® + RNH 2
CO HO –/H2O, D basic strength while electron withdrawing group
COOH 1° amine
Phthalic acid decreases the basic strength.
9. (c) Reaction can be represented as + –
14. (c) CH3 — NH2 + HNO2 ¾® CH3 — N2Cl
Methylamine Nitrous acid ¾¾®
R — NH2 + RCHO ¾® [R — N = C — R] H2O
|
¾¾®

1° amine
H2/Pt H H CH3OH
| Methanol
R—N—C—R
| | HNO
H H 15. (b) CH3 — NH2 ¾¾¾®
2
CH3 — N + º NCl –
¾¾®

2° amine Methylamine
D H 2O
10. (d) LiAlH4 in ether is the best reagent for converting
2-phenylpropanamide into 2-phenylpropanamine because CH3OH + N2­
Methanol
LiAlH4 will reduce carbonyl group but does not reduce
benzene ring. 16. (c) For the nitration of benzene, NOÅ 2 acts as an
electrophile (electrophilic substitution reaction).
O
|| Step 1: H2SO4 ¾® H+ + HSO4–
NH2 NH2 +
Step 2: H + H – O – NO2
+
H 2 O + NO2
nitronium ion
4LiAlH in ether
¾¾¾¾¾¾® NO2
NO +
¾¾®
2
2-Phenylpropanamide 2-Phenylpropanamine
EBD_8350
718 CHEMISTRY

17. (c) Reduction of aromatic nitro compounds gives 26. (b) On comparison to amines, hydrocarbons have lower
aromatic 1° amine. boiling point, hence they are more volatile whereas amines
are having higher boiling point due to H-bonding.
NO2 NH2
Fe / HCl
¾¾¾® NH2
27. (d) N2Cl ¾¾¾¾¾®
+
Aromatic Aromatic H
nitro compound primary amine

18. (b) Reactivity of a base towards dilute HCl is directly N=N NH2
proportional to the strength of the base.
Thus, as (CH3)2NH has the highest basic strength, so it 28. (b) Arene diazonium salts are most stable among the given
will have highest reactivity. options because of the dispersal of +ve charge on the
benzene ring due to resonance.
H 3C HCl H 3C Å H – 29. (d) R – CH2 – NH2 + CHCl3 + 3KOH (alc.)
N — H ¾¾¾ ® N + Cl
H 3C Carbylamine reaction
H 3C H ¾¾¾¾¾¾¾¾® R – CH2 – NC + 3KCl + 3H2O
R Alkyl isocyanide
Å
O O O NH2 O – 30. (a) Arylamines are less basic than alkyl amines and even
|| || ||
R – NH2 ammonia. This is due to resonance. In aryl amines the lone
R O R ¾¾¾® ¾®
19. (a) R O pair of electrons on N is partly shared with the ring and is
Acid R thus less available for sharing with a proton.
anhydride In alkylamines, the electron releasing alkyl group increases
O the electron density on nitrogen atom and thus also
||
Å O increases the ability of amine for protonation. Hence more
R—N R .. ||
the number of alkyl groups, higher should be the basicity

OOCR ¾® R — N R + COOH
H H of amine. But a slight discrepancy occurs in case of
HAmide R trimethyl amines due to steric effect. Hence the correct
20. (b) Gattermann reaction: order is
N +2 Cl- Cl (CH3 )2 NH > CH3 NH 2 > (CH3 )3 N > C 6 H5 NH 2
Cu / HCl
¾¾¾® + N2 + CuCl 31. (d) Because arylhalides do not undergo nucleophilic
substitution reaction with potassium phthalimide easily.
Benzene diazonium Chlorobenzene
chloride 32. (d) It is an example of Schotten-Baumen reaction.
21. (b) Gabriel phthalimide synthesis is the best method of 33. (d)
preparing primary amines from alkyl halides without
NO2 HN—OH NH2
changing the number of carbon atoms in the chain.
Electrolytic
22. (d) Diazonium cation is a weak E+ so it will not react with reduction Rearrangement
compound containing electron withdrawing group. Also, ¾¾¾¾¾¾® ¾ ¾¾¾¾¾®
(Strongly acidic
NO2 group is an electron withdrawing group thus, medium)
nitrobenzene Phenyl
nitrobenzene will not undergo azo coupling reaction with hydroxylamine OH
benzene diazonium chloride.
23. (c) Phenol has the least tendency to accept a proton 34. (a) C3H9N
and also exhibit acidic character due to polar O – H bond CH3—CH2—CH2—NH2
hence, it is a weak Bronsted base. CH3 – NH – CH2 – CH3.
24. (d) Pyrrolidine will be the strongest base as the lone pair CH 3 — CH — CH 3 , CH 3 —N — CH 3
on nitrogen are not involved in resonance and are easily | |
available for the donation. So, basic strength becomes NH2 CH3
high. 35. (a)

25. (a) Amide ion (NH2–) is the most basic among the given NH2 N2+ Cl CN
species due to the presence of negative charge and two
NaNO / HCl CuCN/KCN
lone pairs of electron on nitrogen atom. The species which ¾¾¾¾¾¾
2
® ¾¾¾¾¾®
0 – 5 °C D
are having lone pair of electrons are more basic than neutral
species. So, NH3 and H2O are less basic than NH2– and CH3 CH3 CH3
OH–. Also, nitrogen is less electronegative than oxygen 36. (a) Arylamines are generally less basic than alkylamines
hence it can easily donate e–s. due to following factors
\ The correct order of basic strength will be: (i) Due to resonance in aromatic amines.
NH2– > OH– > NH3 > H2O (ii) Lower stability of anilinium ion
AMINES 719

37. (b) 4 moles of NaOH and one mole of Br 2 is used for the
synthesis of one mole of amine during Hoffmann's
bromamide degradation reaction.
O
38. D
(a) CH3 - C - NH 2 + Br2 + 4NaOH ¾¾ 44. (a)
®
Acetamide
CH3 – NH2 + 2NaBr + Na2CO3 + 3H2O
Methanamine
It is called Hoffmann Bromamide reaction.
R–NH2 + Na2CO3 + 2NaBr + 2H2O
39. (c) – NO2 group has strong – R effect and –CH3 shows
+R effect. 45. (d) Aliphatic and aromatic primary amines give
\ Order of basic strength is carbylamine reaction. Since aniline is primary aromatic
NH2 NH2 NH2 amine it gives carbylamine test.
46. (d) Conjugate acid of guanadine(B) is resonance
< < stabilised and have 2 resonance structure.
Similarly conjugate acid of (A) is also resonance stabilised
NO2 CH3 and have one resonance structure. (C) does not exhibit
resonance structure.
40. (c) Nitration takes place in presence of concentrated Therefore the basic order is, Kb : (B) > (A) > (C)
HNO3 + concentrated H2SO4. \ pKb : (B) < (A) < (C)
In strongly acidic nitration medium, the aniline is converted
into anilinium ion (– NH3+); substitution is thus controlled 47. (b)
not by – NH2 group but by – NH3+ group because of its
positive charge and directs the entering group to the meta-
position instead of ortho, and para.
+
: NH2 NH3

conc. HNO
¾¾¾¾®
3
conc. H2SO4
Methyl orange is used as an indicator in acid base
– NH2 gp : o, p-directing – NH3+ gp : m-directing titrations.
41. (a) Account for the inductive effect, solvation effect
(H-bonding.) and steric hinderance for basic character in Exercise-3
aqueous solutions
(CH3)2 NH > CH3 NH2 > (CH3)3 N 1. (d)
2. (d) Aromatic amines can not be prepared by Gabriel
42. (b) Compound, III is most basic as the lone pair of Phthalimide.
3. (d) Most reactive imine will be least stable. Emine in (d)
is more reactive towards H/ ion because after protonation
nitrogen is easily available for the donation.
it is stabilized by resonance.

NH NH2
In case of compound (I) lone pair is not involved in
resonance but nitrogen atom is sp2 hybridsed whereas in
+ H
compound II the lone pair of nitrogen is involved in
aromaticity which makes it least basic.
43. (c) In acidic medium aniline is more reactive than phenol NH NH
+ NH2
that’s why electrophilic aromatic substitution of Ph––N2
takes place with aniline

NH
EBD_8350
720 CHEMISTRY

4. (d) The basic character of an amine in water is determined O


by (i) electron availability on the N atom and (ii) the extent .. ..
of stabilization of the cation (protonated amine) due to CH3 C NH2 < CH3CH2 NH2
solvation by hydrogen bonding III I ..
NH
H OH2 ..
H + +
< CH 3 C NH 2
CH3 N H ¾® CH3 N H OH2 ; II
7. (b)
H H OH
2
+ 8. (b) Alcohols obtained from (1), (2), (3) and (4) are
Methyl amine Protonated methyl amine, CH3NH 3
Et CH3
(Highly stabilized)
*
(1) CH3—C—Pr (2) CD3—C—CH3
H OH2
OH OH
+
H + (Optically active) (Inactive)
CH3 N H ¾® CH3 N H OH2 H
CH3 CH3 *
(3) CHD2—C—CH (4)
2D OH
Dimethyl amine Protonated dimethyl amine
+
(Lesser stabilized than CH 3NH 3) OH
(Active) (Inactive)
5. (a) Note the point of difference in the given compounds CH3 CH3
which here lies at b-carbon. In I, II, III, the b-carbon atoms HONO
9. (c) CH3—CH—CH2NH2 CH3—CH—CH2
are sp3, sp2 and sp hybridised respectively which in turn (A)
cause the difference in their s character. We know that CH3
more is the s character of an atom, greater will be its electron- H2 O
withdrawing nature. Thus sp (50% s character) hybridised ¾® CH3—C—CH3
carbon is most electron-withdrawing, while sp3 (25% s OH
character) is least electron-withdrawing. Further, we know (C)
that presence of an electron-withdrawing group decreases ¾® –ve Victor Mayer’s test
basicity of an amine. Thus
HONO
OH
NH2 > NH2 > NH2 CH2 NH2
b b b
I (b – C sp 3 hybridised) II (b – C sp 2) III (b – C sp)

6. (d) II is most basic because delocalisation of electron


pair leads to negative charge on N making it electron-rich Blue colour in
and hence liable to be attacked by proton very easily. victor maeyer’s
Moreover, the corresponding protonated species is very test
10. (a)
much stable because of equivalent contributing structures.
CH3 CH3
.. .–. COOH COCl NH3(alc.)
SOCl2
NH .NH
. +
H
CH3 C ¬¾® CH3 C ¾®
+
NH
.. 2 NH2 CH3 CH3
II CONH2 NH2
OBr –
+
NH2 NH2
CH3 C + ¬¾® CH3 C NaNO2
HCl
NH2 NH2
CH3
Species III is least basic because of delocalisation of
electron pair on N, making it less available for protonation. OH2
Species I lies in mid-way, thus
o-cresol
AMINES 721

11. (c) Remember that in Hofmann rearrangement, the two 14. (d)
original H atoms of the –CONH2 group are removed by
base (OH–) and new H’s are derived from H2O. HONO
R—CH2—CH2—NH2 R—CH2—CH2—N2
O
|| –N2
(i) OD – /Br
(CH3 )3 C - C - NH 2 ¾¾¾¾¾®
2 (CH3 )3 C - ND2
(ii)D 2O R—CH2—CH2
H2O Cl
–H
Me Me
12. (a) Et N N Et RCH2CH2OH R—CH==CH2 RCH2CH2—Cl
Pr Pr
(X) (Y)
15. (d)
Due to lone pair flipping (X) and (Y) are interconvertible.
Due to this at room temperature methyl ethyl propyl amine
exist in the form of mixture of (X) and (Y) and hence it is
optically inactive. O NH2 O
NH2

b OH O O
+
(i) CH3I a a b H
13. (b) ¾¾¾® +
N CH3 (ii) Ag 2O N C H
H O O
H H3C CH3 O

NH
heat OH
¾¾®
N
O
H3C CH3
EBD_8350
722 CHEMISTRY

28 Biomolecules
13. (b) a -D glucose and b–D glucose are the isomers which
Exercise - 1
differ in the orientation (configuration) of H and OH groups
1. (b) Sucrose is an oligosaccharide and cellulose is a around C1 atom.
polysaccharide.
2. (b) To explain the properties which can not be explained
by open chain structure of glucose it was proposed that H —C1—OH HO —C1—H
one of the –OH groups may add to the –CHO group and H —C2—OH H —C2—OH
forms a cyclic hemiacetal structure as shown below. O O
O HO —C3—H HO —C3—H
1 1
H—C—OH 1
H —C HO—C—H H —C4—OH H —C4—OH
2 2 2
H OH H OH H OH H —C
5
H —C
5
HO 3 O 3 HO 3 O 6 6
H HO H H
4 4 CH2OH CH2OH
H OH 4 H OH
H OH a– D - Glucose b– D - Glucose
H 5 5 H 5
H OH
6 6 6 14. (c) Fructose is the sweetest sugar.
CH2OH CH2OH CH2OH 15. (a) We know that cellulose (C 6H12O5)n is the chief
a – D – (+) – Glucose b – D – (+) – Glucose constituent of cell walls of plants. It is the most abundant
3. (d) It is the most abundant organic compound on earth. organic substance found in nature. It is a polymer of
4. (d) Tollen’s reagent is reduced by glucose due to
glucose with 3500 repeat units in a chain.
aldehydic group and gives grey colour as silver metal.
5. (b) The letter ‘D’ or ‘L’ before the name of any compound 16. (b)
indicate, the relative configuration of a particular 17. (d) Carbohydrates are stored in the body as glycogen.
stereoisomer. 18. (b) CH 2 OHCH 2 CHOHCHOHCH 2 OH does not
6. (a) correspond to Cx(H2O)y.
7. (a) Glucose contains an aldehyde group. It is oxidised 19. (d) Lactose (milk sugar) is a disaccharide, it is made of
into acidic group by bromine water and gluconic acid is b-D-galactose and b-D-glucose
formed
[O] CH2OH
CH 2 OH - (CHOH) 4 - CHO ¾¾¾ ® CH2OH O H
H
CH 2OH - (CHOH ) 4 - COOH HO O H
H O OH H
Br2 + H 2 O ¾¾ ® 2HBr + O OH
OH H
8. (d) Weak reagent like NaHSO3 is unable to open the chain H H
H OH
and can’t react with glucose. This explains the inability of H OH
glucose to form aldehyde bisulphite compound. 20. (b) 21. (b)
9. (d)
22. (b) Sucrose is dextrorotatory but after hydrolysis gives
10. (c) Open chain structure is unstable and converted to
cyclic. dextrorotatory glucose and laevorotatory fructose. Since
11. (b) the laevorotation of fructose (–92.4°) is more than
12. (c) dextrorotation of glucose (+52.5°), the mixture is
1 laevororatory.
H OH 23. (c) Chemically amylose is a long unbranched chain with
2 200-1000 a-D-(+)-glucose units held by C1-C4 glycosidic
H OH
O linkage.
3 24. (b) It is a branched chain polymer of a-D-glucose units
HO H
in which chain is formed by C1-C4 glycosidic linkage,
4
H OH whereas branching occurs by C1-C6 glycosidic linkage.
5 25. (b) Cellulose is a straight chain polysaccharide.
H 26. (a)
CH2OH 27. (c) Sucrose is non-reducing in nature. It does not contain
a -D-(+) - Glucose a free aldehydic or ketonic group. Maltose is a reducing
sugar.
BIOMOLECULES 723

28. (b) Naturally occurring monosaccharides are 20 only. a-D-glucose ƒ Open chain form ƒ b-D-glucose
Sucrose is a non–reducing sugar, whereas maltose is a (+111°) (+19°)
reducing sugar.
29. (b) The six membered cyclic structure of glucose is called Specific rotation of a-form falls until a constant value
pyranose structure (a or b), in analogy with pyran. Pyran of +52.5° is reached. On the other hand, specific rotation
is a cyclic organic compound with one oxygen atom and of b form increases again to +52.5°. Specific rotation of
five carbon atoms in the ring. The cyclic structure of glucose equilibrium mixture is 52.5°.
is correctly represented by Haworth structure. Five
6 1
membered cyclic structure of glucose is called furanose
39. (b) HOH2C O CH2OH
structure.
30. (c) Amylose is water soluble component which 5
H OH 2
constitutes about 15-20% of strach. Amylopectin is 4 3
H OH
insoluble in and constitutes about 80-85% of starch.
31. (a) OH H
32. (c) Glucose contains aldehyde group due to which it a-D-(–)-Fructofuranose
gives positive test with ammonical silver nitrate.
33. (d) We know that glucose reacts with one molecule of 6
phenylhydrazine to give phenylhydrazone. When warmed HOH2C O OH
with excess of phenylhydrazine, the secondary alcoholic 5
group adjacent to the aldehyde group is oxidised by another H OH 2
4 3
molecule of phenylhydrazine to a ketonic group. With this H CH2OH
1
ketonic group, the third molecule of phenylhydrazine
condenses to glucosazone. Therefore the value of X is 3 OH H
b-D-(–)-Fructofuranose
CHO + H2NNHC6H5 CH = NNHC6H5
40. (d) Maltose is compound of two a-D - glucose units in
CHOH CHOH which C1 of one glucose is linked to C4 of another glucose
warm
unit.
(CHOH)3 (CHOH)3
6 6
CH2OH CH2OH
CH2OH CH2OH O H
H 5 O H H 5
Glucose Glucose phenylhydrozone H H
4 1 O 4 1
H2NNHC6H5
HO OH
3
H
2
OH
3
H
2 OH
CH = NNHC6H5 CH = NNHC6H5
H OH H OH
H2NNHC6H5 I II
C = NNHC6H5 C=O a-D-glucose a-D-glucose
– H2O
41. (c) The hydrolysis of sucrose by boiling with mineral
(CHOH)3 (CHOH)3
acid or by enzyme invertase or sucrase produces a mixture
of equal molecules of D(+) glucose and
CH2OH CH2OH
D(–) Fructose.
Glucosazone Keto compound of glucose
phenylhydrazone HCl
C12H22O11 + H2O ¾ ¾ ® C6H12O6 + C6H12O6
34. (c)
ZnCl2 Sucrose D-Glucose D-Fructose
35. (c) OHC.(CHOH)4.CH2OH + 5Ac2O ¾¾¾¾ ® [a D]=+66.5° [a D]=+52.5° [a D]=–92°
Glucose Acetic anhydride
Invert sugar, [a D]=–20°
OHC. (CHOAc)4. CH2OAc
Glucose penta-acetate or 42. (a)
Penta-acetyl glucose
43. (c) Amino acids are the compounds having one or more
36. (c) Melting point of a-glucose® 419 K and b-glucose is
amino groups and one or more carboxyl groups in the
323 K.
same molecule.
37. (d) The hydrolysis of sucrose brings about a change in
the sign of rotation from dextro (+) to laevo (–) and the 44. (b) Except glycine, all other naturally occurring a- amino
product is named as invert sugar. acids are optically active, since the a-carbon atom is
38. (a) a-D-glucose or b-D-glucose when dissolved in water asymmetric.
and allowed to stand, following equilibrium is stablised, 45. (d)
which is called mutarotation.
EBD_8350
724 CHEMISTRY

46. (d) Zwitter ion contains both +ve and –ve charge. Proton 64. (d) The hydrophilic/ hydrophobic character of amino
of –COOH group is transferred to the –NH2 group. acid residues is important to tertiary structure of protein
—NH3+ group is acidic since it can donate a proton and — rather than to secondary structure. In secondary structure,
COO– group is basic since it can accept a proton. it is the steric size of the residues that is important and
47. (d) Proline is a secondary amine. residues are positioned to minimise interactions between
48. (c) The bond formed between two amino acids by the each other and the peptide chain.
elimination of a water molecule is called a peptide linkage 65. (d) Quaternary structure refers to the overall structure of
or bond. The peptide bond is simply another name for a multiprotein complex whereas primary, secondary and
tertiary structure refer to the different structural levels of a
amide bond.
single protein.
66. (a) The arrangement of polypeptide chains formed as a
- C OH + H — N— ¾¾
® — C– N— + H 2 O
÷÷ | || | result of hydrogen bonding is called secondary structure
O H O H of proteins.
Carboxyl group Amine group of Peptide bond
a-Helix is formed by intr amolecular H-bondin g.
of one amino acid other amino acid
b-Pleated sheet is formed by intermolecular H-bonding.
The product formed by linking amino acid molecules 67. (b) The secondary structure of a protein refers to the
through peptide linkages, —CO—NH—, is called a shape in which a long peptide chain can exist. There are
peptide. two different conformations of the peptide linkage present
49. (b) in protein, these are a-helix and b-conformation. The a-
50. (d) Proteins are highly complex, natural compounds, helix always has a right handed arrangement. In
composed of a large number of different a-amino-acids b-conformation, all peptide chains are streched out to nearly
maximum extension and then laid side by side and held
joined together with peptide linkage, i.e., they are naturally
together by intermolecular hydrogen bonds. The structure
occuring polypeptides.
resembles the pleated folds of drapery and therefore is
51. (a) Globular proteins is present in blood. These are soluble
known as b-pleated sheet.
in water. 68. (b) In this structure of protein, atoms are highly coiled
52. (c) Polypeptide chains, in fibrous proteins, are held and form a spherical form.
together by disulphide and hydrogen bonds. 69. (d) When a protein, in its native form, is subjected to a
53. (c) Insulin is an example of globular protein. physical change like change in temperature, or a chemical
54. (c) 55. (c) 56. (b) change like change in pH, the native conformation of the
57. (c) Fibrous proteins are generally insoluble in water. molecule is disrupted and proteins so formed are called
58. (b) The NH of the amide can act as a hydrogen bond denaturated proteins.
donor and the carbonyl group can act as a hydrogen bond The denaturation may be reversible or irreversible. The
acceptor. Statements (a), (c) and (d) are false. The peptide coagulation of egg on boiling is an example of irreversible
bond has double bond character due to the interaction of protein denaturation.
the nitrogen lone pair with the carbonyl group. This However, it has been shown now that in some cases, the
prevents bond rotation and makes the bond planar. The process is actually reversible. The reverse process is called
trans isomer is favoured over the cis isomer. renaturation.
59. (a) 70. (c) Tertiary structure indicates the overall structure of
60. (a) The sequence in which the a-amino acids are linked the protein.
to one another in a protein molecule is called its primary 71. (a)
structure. 72. (b) Serine contains a hydroxyl functional group on its
61. (b) The a-helix structure is formed when the chain of side chain and so the strongest possible interaction will
a-amino acids coils as a right handed screw (called be hydrogen bonding where the hydroxyl group could act
a-helix) because of the formation of hydrogen bonds as a hydrogen bond donor or hydrogen bond acceptor.
between amide groups of the same peptide chain, i.e., NH 73. (a) When antigens enter into the body cells and destroy
group in one unit is linked to carbonyl oxygen of the third them, then antibodies being proteins are synthesised in
unit by hydrogen bonding. This hydrogen bonding the body and combine with antigens and destroy these
between different units is responsible for holding helix in antigens by forming inactive complexes. Therefore
a position. antibodies protein destroy antigens.
62. (c) During denaturation 2° and 3° structures are
74. (c) Glycine is optically inactive.
destroyed but 1° structure remains intact.
75. (a) Gln stands for glutamine. Except glycine, all other
63. (a) Primary structure refers to the order of the amino acids
naturally occurring a–amino acids are optically active.
in a protein.
BIOMOLECULES 725

76. (c) When the proteins are subjected to the action of heat, 96. (c) Vitamin A - Xerophthalmia
mineral acids or alkali, the water soluble form of globular Vitamin B12 - Pernicious anaemia
protein changes to water insoluble fibrous protein. This is Vitamin C - Scurvy
called denaturation of proteins. During denaturation
Vitamin E - Sterility
secondary and tertiary structures of protein destroyed but
primary structures remains intact. Vitamin K - Haemorrhage
77. (a) R is the correct explanation of A. 97. (d) Vitamin D is a fat soluble vitamin and can be stored in
78. (c) Valine has no functional groups on its side chain. the body since it is not excreted out of the body.
There is only an alkyl group and so only van der Waals 98. (a) Vitamin C is water soluble. Therefore, it is readily
interactions are possible. excreted in urine and cannot be stored in our body and is
79. (a) Amino Acids are amphoteric in nature. So for it a supplied regularly with diet.
special term is coined called Zwitter ion. 99. (b) RNA has D (–) – Ribose and the DNA has 2–Deoxy
They have following structure in solution D (–) – ribose as the carbohydrate unit.
R 5 5
HOCH2 O OH HOCH 2 O OH
+
H3N–C–COO– 4 1 4 1
H H H H
H H H H H
3 2 3 2
[Zwitter Ion] OH H
OH OH
80. (b) Enzymes are highly specific for a particular reaction Ribose 2-Deoxy ribose
and also for a particular substrate. From the structures it is clear that 2nd carbon in DNA does
81. (b) Enzymes being biocatalysts can increase the rate of a not have OH group.
reaction upto 10 million times. Even very small amount can 100. (a) DNA consists of two polynucleotide chains, each
accelerate a reaction. chain forms a right handed helical spiral with ten bases in
82. (b) Triglycerides are lipids, hence these are hydrolysed one turn of the spiral. The two chains coil to double helix
by lipases to glycerol and fatty acids. and run in opposite directions held together by hydrogen
83. (d) Enzymes are most reactive at optimum temperature. bonding.
The optimum temperature for enzyme activity lies between
40°C to 60°C. 101. (d) O Deoxyribose-Adenine ... Thymine -Deoxyribose O
84. (b) Insulin is a biochemically active peptide hormone OH P Deoxyribose-Guanine ... Cytosine -Deoxyribose
P OH
secreted by pancreas. O O
P P
85. (a) OH Deoxyribose-Guanine ... Cytosine -Deoxyribose
OH
O O
86. (d) Enzymes may or may not require a coenzyme for their OH P Deoxyribose-Adenine ... Thymine -Deoxyribose
P OH
catalytic action.
87. (c) The hydrogen bonds are formed between the base (shown
88. (c) Deficiency of vitamin D causes rickets. by dotted lines). Because of size and geometrics of the
89. (d) Vitamin Disease caused by deficiency bases, the only possible pairing in DNA are between
B6 Dermatitis G(Guanine) and C(Cytosine) through three H-bonds and
B1 Beri-beri between A (Adenine) and T (Thymine) through two H-
B2 Photophobia, glossitis bonds.
B12 Pernicious anaemia 102. (a)
90. (b) Vitamin E is mainly present in vegetable oils like wheat 103. (c) The base pairs of the two strands of DNA are linked
gram oil, sunflower oil, etc. together through H-bonds.
91. (a) Vitamin A or retinol. 104. (d) DNA has the property of self - replication . It is
92. (b) therefore a reproducing molecule. This unique property of
93. (c) It is found in liver, egg, milk, meat, and fish. Minute DNA is at the root of all reproduction. Through its
amounts are probably present in all animal cells. Peculiarly, replication, DNA acts as the key to heredity. In the
unlike other vitamins, B12 is not found in significant replication of DNA, the two strands of a double helix
amounts in green plants. unwind and separate as a template for the formation of a
94. (b) Water soluble vitamins must be supplied regularly in new complementary strand.
diet because they are readily excreted in urine and cannot 105. (c) Each chromosome is made up of DNA tightly coiled
be stored (except vitamin B12) in our body. many times around proteins called histones that supports
95. (b) its structure.
EBD_8350
726 CHEMISTRY

106. (a) Total weight % of glycine (given) = 47%


107. (d) 958 ´ 47
Total weight of glycine in product = g = 450g
108. (a) Synthesis of polypeptide is known as translation. For 100
this process three types of RNA are essential. Molecular weight of glycine = 75 g/mol
109. (d) The correct structure of thymine is 450
OH Number of glycine molecules = =6
75
5. (8)
N CH3 1 1
CHO CHOH
6
CH2OH
HO N 2
CH2
2
CH2
110. (c) 5 O
111. (d) Tyrosine is an a-amino acid, and not a purine. 3 3 4
H
1
CHOH CHOH O or HO OH
112. (d)
113. (c) DNA fingerprinting is same for every cell and cannot 4
CHOH
4
CHOH 3 2
be altered by any known treatment. OH
5 5
114. (b) In secondary structure of DNA adenine forms H C OH H C
hydrogen bonds with thymine whereas cytosine forms
CH2OH CH2OH
hydrogen bonds with guanine. 6 6
115. (b) In DNA and RNA heterocyclic base and phosphate
D- Aldohexose D- Aldohexopyranose
ester are at C1' and C5' respectively of the sugar molecule.
Thus, total number of stereoisomers in pyranose form of
HO N D-configuration = 23 = 8
| N
5
HO – P – O – CH2 O N 6. (4) According to question C – Terminal must be alanine
4¢ 1 N
|| C C¢ and N – Terminal do have chiral carbon means it should
O H H not be glycine. So possible sequence is :
H | | H
C3
Val Phe Gly Ala
C2
| | Val Gly Phe Ala
OH OH Phe Val Gly Ala
116. (c) Oxytocin hormone is produced by hypothalamus Phe Gly Val Ala
gland and is secreted by the posterior pituitary gland into 7. (67) Let fraction of a-anomer = x
the blood stream. \ Fraction of b-anomer = (1 – x)
117. (d) 118. (b) 119. (d) 120. (b) Hence x (+ 29.3°) + (1 – x) (– 17.0°) = + 14.2°
29.3° x + (– 17° + 17°x) = + 14.2°
Exercise-2 29.3°x + 17°x = 14.2° + 17°
1. (2) Invert sugar is an equimolar mixture of D-(+) glucose 46.3°x = 31.2°
and D(–) glucose. x = 0.67
\ Fraction of a-anomer = 0.67 or 67%
8. (4) Amide, lactam, carboxylic and thioether linkage are
present.
9. (6) The minimum atom of Fe in the enzyme is one.
0.43 g of Fe is present in 100 g of enzyme
100
56 g of Fe is present in × 56
-92° + 52° 0.43
Specific rotation of invert sugar = = -20° = 1.3 × 104 g of enzyme
2 Thus the minimum mol. wt. of the enzyme = 13000
D-glucose on oxidation with Br 2-water produces gluconic 100 ´ 32
acid and not saccharic acid. Similarly, 32 g of S is present in = 2.16 × 103
1.48
2. (1) Two molecules of an a-amino acid will form only one Thus the minimum mol. wt. corresponding to 1 S atom
dipeptide, the four different dipeptides are formed when = 2160 13000
two a-amino acids are different. \ No of S atoms in the enzyme per Fe atom = »6
3. (2) The basic groups in the given form of lysine is NH2 2160
10. (3) In strongly alkaline solution of an amino acid, all of
+
(not NH 3 ) and CO-2 . its —COOH groups are converted into —COO–. Thus a
4. (6) Molecular weight of decapeptide = 796 g/mol strongly alkaline solution of a monoaminodicarboxylic acid
Total bonds to be hydrolysed = (10 – 1) = 9 per molecule will have one —NH2 and two —COO– groups, all of which
Total weight of H2O added = 9 × 18 = 162 g/mol are basic in nature. Further, —NH2 is more basic than a –
Total weight of hydrolysis products = 796 + 162 = 958 g COO– group.
BIOMOLECULES 727

Exercise-3 9. (b) The polymer of nucleotides in which nucleic acids


are linked together by phosphodiester linkage are known
1. (b) Structure of glycogen is similar to the structure of
as nucleic acid.
amylopectin.
CH2OH CH2OH 10. (c) Glucose is present in pyranose form,
H O H H O H
H H CH2OH
4
O
4 OH 1
O OH H 1 a-link O H
O
H
H
H OH H OH OH H
HO OH

CH2OH 6 CH2 CH2OH H OH


O H
H O H H5 O H H
H H
4 OH H 1
H
4 OH H 1 Pyranose means six membered ring containing oxygen.
4 OH 1
O O O
O 11. (a) Primary structure of proteins. When each polypeptide
a-link a-link
H OH H OH H OH in a protein has amino acids linked with each other in a
Structure of amylopectine specific sequence.
2. (d) Glycogen is stored in the liver, brain and muscles of 12. (c) RNA does not contain thymine.
animals. 13. (d) Vitamin B12 can be stored in our body because it is
H O
3. (c) C12 H 22 O11 ¾¾¾
2 ®C H O + C H O
+ 6 12 6 6 12 6 not water soluble.
H
Cane sugar D( +) glucose D ( -) fructose 14. (d) DNA does not contain uracil.
4. (c) Structures having different configuration at C-1 if they 15. (a) Cyclic structures of monosaccharides which differ in
are aldohexoses are known as anomers structure at carbon -1 are known as anomers.
5. (c) In a-helix structure, — NH group of one amino acid is
Here, I and II are anomer because they differ from each
hydrogen bonded to C = O group of adjacent amino other at carbon-1 only.
acid, forming a helix. 16. (c) "Pentaacetate of glucose does not react with
6. (b) Reducing groups of glucose and fructose are involved hydroxylamine" showing absence of free CHO group. This
in glycosidic bond formation.
can not be explained by open structure of glucose while
6
CH2OH 1 all other properties can be easily explained.
HOCH2
H O H O H 17. (a) CH2OH
5 CHO
2 5 CHO
4
OH H 1 H HO H OH C=O
O 3 4 CH2OH H H
HO 3 2
6 H OH HO HO
H OH OH H H OH H OH
a-D-glucose b-D-fructose CH2OH H OH H OH
CH2OH CH2OH
7. (b) Ascorbic acid is the chemical name of vitamin C.
(+) (+) (–)
8. (a) Dinucleotides are formed by phosphodiester linkage (iii)
(i) (ii)
between 5' and 3' carbon atom of pentose sugar.
When OH on lowest asymmetric carbon is written at right
O 5' end of chain
|| 5'
hand side, it is represented as D configuration and when
!
O P O CH2 Base OH is written on left hand side, it is represented as L
| O
O! 4' 1¢ configuration.
Sugar
18. (c) Carbon adjacent to oxygen atom in the cyclic structure
3' 2¢
of glucose or fructose is known as anomeric carbon.
O
|
! 6
O P =O 19. (c) 6
CH2 OH CH2 OH
| 5 5
O O O
H 1 (A) H H OH
H
Base 4 O 4 1
5'CH2 O OH H H OH H
OH H
4' 1¢ 3 2 3 2
Sugar H OH OH
3' C1 – C4 linkage

OH (I)

3' end of chain


EBD_8350
728 CHEMISTRY
6
6
CH2 OH CH2 OH 29. (a)
O O CH2OH
H 5 H H 5 OH HOCH2 CH2OH HOCH2
H H O O
4 1 4 1 aq.KOH
OH H (C) O
OH H O ¾¾¾¾¾ ®
OH H HO – CH3COOK HO –
3 2
6
CH2 OH 3 2 O – C – CH3 O
H OH 5 H OH
O
C1 – C6 linkage O (B) H C1 – C4 linkage OH OH
H 1
4
OH H Hemiketal
6
CH2 HO H
O 3 2 Ring opening
H 5 H H OH
H 1 (III)
4 OH H HOCH2 OH
OH OH CH2OH
3 2 positive silver
H OH Tollen 's HO O
mirror ¬¾¾¾
Reagent
¾
(II) test
20. (d) Glucose reacts with hydroxyl amine to form an oxime. OH
CHO CH=N–OH (a Reducing sugar) a-hydroxy ketone
H OH H OH 30. (a) Amylose and amylopectin are polymers of
HO H HO H a-D-glucose, so b-link is not possible. Amylose is linear
NH 2OH
H OH ¾¾¾® H OH with 1 ® 4 a-linkage whereas amylopectin is branched
H OH H OH and has both 1 ® 4 and 1 ® 6 a-linkages.
So option (a) should be the correct option.
CH2 – OH CH2 – OH
+
D(+) glucose
31. (c) ˆˆ† OOC – CH 2 – NH 3
HOOC – CH 2 – NH 2 ‡ˆˆ
21. (c) Adrenaline is a hormone produced by adrenal glands Glycine Zwitter ion
during high stress or exciting situations. This powerful CHO
hormone is part of the human body’s acute stress response H OH
system, also called the fight or flight response. HO H
HI/ D
32. (a) H OH ¾¾¾¾ ®
22. (a) DNA contains ATGC bases H OH
Reduction n-Hexane

So quinoline is not present in DNA.


CH2OH
23. (c) Water-soluble vitamins dissolve in water and are not Glucose
stored by the body. The water soluble vitamins include 33. (d) Structure of Histamine
the vitamin B-complex group and vitamin C. H
24. (d) Sucrose is non-reducing disaccharide as the two
N
monosaccharide units are linked through their respective
carbonyl groups. N
25. (c) Peptide bond 9.75 pKa (more basic)
\ Predominant structure of Histamine is
— C — NH — H
||
O N +
N
26. (b) Sugar in DNA is 2-deoxyribose whereas sugar in RNA
( Due to slightly basic nature of blood (pH = 7.35)
is ribose. 34. (c) Alanine is non-essential amino acid.
27. (a) Among 20 naturally occuring amino acids "Cysteine" 35. (b) Sucrose contains glycosidic link between C1 of a-D
has '– SH' or thiol functional group. glucose and C2 of b-D-fructose.
Þ General formula of amino acid ® R– CH – COOH C12H22O11 + H2O ¾® Glucose + Fructose
| 6
CH 2OH
NH2 O 1 O
H 5 H HOH2C H
H
Þ Value of R = – CH2 – SH in Cysteine. 4
OH H
1 2H HO 5
O
HO 3 2 3 4 CH 2OH
28. (c) Due to denaturation of proteins, helix get uncoiled 6
Glycosidic
and protein loses its biological activity. H OH OH H
linkage
a-D-Glucose b-D-Fructose
Sucrose
BIOMOLECULES 729

36. (c) Structure of the given -amino acids are: 2. (a) For reducing sugar, the sugar must have free hemi
acetal group. The sugar present in (a) is non reducing
becaue both the hemi acetal linkage are not free.

Here, aspartic acid is an acidic and glycine is a neutral


amino acid while lysine and arginine are basic amino acids. CH2OH CH2OH
Also, arginine is more basic due to the stronger basic H O H H O H
H O H
functional groups. OH H OH H Free hemi acetal linkages,
HO OH hence it is reducing sugar
\ The order of pKa value is directly proportional to the
H OH H OH
basic strength of amino acids, i.e. Arg > Lys > Gly > Asp.
3. (c) Evolved gas is CO2. Salt produced is CaCO3
37. (c) Vitamins Deficiency Diseases 0.199 kg = 0.2 kg = 200 g CaCO3 = 2 mole
Vitamin B1 (thiamine) Beri beri Since 1 mole of amino acid gives 2 moles of CaCO3 hence
Vitamin B2 (riboflavin) Cheilosis amino acid must have two COOH groups.
Vitamin B6 (pyridoxine) Convulsions 4. (d) Reaction of D-(+)-glucose with methanolic –HCl
Vitamin C (ascorbic acid) Scurvy leads to formation of methyl glucoside (C1–OH group is
methylated) which, being acetal, is not hydrolysable by
38. (b) Sucrose ¾¾¾¾®
Hydrolysis a-D-Glucose + b-D-Fructose base, so it will not respond Tollens’ reagent.
CH2OH 5. (a) The OH’s of sugars are m4ore acidic than that of a
O O typical alcohol because of their mutual electron-
H H HOH2C OH withdrawing inductive effect.
H
OH H H HO 6. (b) Although D-alanine is a constituent of a bacterial
HO O
CH OH 2 cell walls, it is not found in proteins.
7. (b) Protonation at b–N leads to imidazolium ion, which
H OH OH H
is stabilized by two equivalent resonating structures.
a-D-Glucose b-D-Fructose b +

:
39. (c) H2N – (CH2)4 – CH – COO– :N + HN HN
H
:

:
+
NH3 aN N N
Lysine H H H
Since it contains more number of –NH2 groups as compared Equivalent resonating structures
to –COOH groups, hence it is basic amino acid. 8. (a) On increasing the pH by adding an alkali ; H+ will be
lost from — COOH.
Exercise-4 9. (b) 10. (a)
11. (b) Amino acids exist as zwitterions in which acidic
1. (d) During acetylation of (+)-glucose, it is the C1 —OH character is due to - NH 3+ and basic due to – COO– group.
of the hemiacetal that is acetylated and not the C5—OH
R R
that forms the ring (cyclic structure). Since equilibrium with + | + |
acid
the open-chain aldehyde is prevented, the penta-acetate ¾ H3 N C HCOO-
H3 N C HCOOH ¬¾¾
does not respond the aldehydic reactions. R
|
base
® H 2 N C HCOO-
¾¾¾
12. (b)
Ac O H
¾¾¾
2 ®
O O
N
13. (a) CH3 —C—NH NH NH2

Peptide link +
14. (b) – COOH is a better proton donor than N H3 .
15. (b) Glucose exists as glucopyranose (a six membered ring
structure) which is formed through C1 and C5.
EBD_8350
730 CHEMISTRY

29 Polymers
Exercise - 1 27. (a) Melamine plastic crockery is a copolymer of HCHO
and Melamine.
1. (d) 28. (b) PMMA is used in bullet proof glass
2. (c) Polythene is a linear polymer 29. (b) 30. (b)
3. (c) ATP is a monomer molecule. 31. (a) Bakelite is used for making phonograph records.
4. (d) Rayon is manufactured from regenerated cellulosic
fiber. Rayon is produced from naturally occurring polymers OH OH OH
and therefore it is not a truly synthetic fibre nor is it a CH2OH

OH
natural fiber. It is known by the names viscose rayon and + HCHO ¾® +
artificial silk in the textile industry. So, option (d) is the
correct choice. CH2OH
5. (a) Molecules of a polymer, being large in size, scatter o & p-Hydroxymethylphenol
light (intermediate)
6. (b)
7. (d) Cellulose acetate also known as rayon and cellulose OH
OH OH
nitrate are semisynthetic polymers. CH2OH
8. (d) Silk is protein fibre. Dacron is polyester fibre and Polymerisation
n ¾¾¾¾¾® CH2
Nylon-66 is polyamide fibre. – (n – 1) H 2O
9. (c) Ebonite is a hard highly vulcanized rubber, containing
Linear polymer (Novalac)
20-30% sulphur.
10. (a) 32. (a) Dacron is a polyester and is the condensation polymer
11. (d) All these are characteristics of elastomers. of ethylene glycol and terephthalic acid. It is crease
12. (d) Thermosetting polymers have strongest molecular resistant.
forces. These are crosslinked polymers. 33. (c) Terylene is made from glycol and terephthalic acid
13. (b) Thermosetting plastics have three dimensional cross
linked structure. HO - CH 2 - CH 2 - OH and HOOC COOH
14. (d) All the given statements about fibres are correct. Glycol
15. (b) Polystyrene and polyethylene belong to the category Terephthalic acid
of thermoplastic polymers which are capable of repeatedly 34. (a)
softening on heating and harden on cooling. 35. (b) It is the definition of copolymerisation.
16. (b) 17. (a) 36. (b)
18. (b) A – (q), B – (s), C – (r), D – (p)
37. (b) Nylon 6 is
19. (a) Ethene on free radical polymerisation gives low
density polythene. O O
20. (b) Monomer of teflon is to tetrafluoroethylene C2F4. ½½ ½½
—(NH(CH 2 )5 —C— NH— (CH2)5 —C— )n
21. (d) Vinyl chloride, butadiene and styrene being
unsaturated undergoes addition polymerization. 38. (b) 39. (a)
22. (a) 40. (b) Buna – N is a copolymer of butadiene
23. (d) Acrylonitrile is the monomer used in the preparation (CH2= CH – CH = CH2) and acrylonitrile (CH2 = CHCN).
of orlon. 41. (c) Neoprene is a homopolymer of chloroprene.
24. (b) High density polythene is used for manufacturing of 42. (b)
buckets, dustbins, pipes etc. 43. (d) Number of hydrogen bonds is greater in polymer (1)
25. (b) High density polythene is formed when addition than in (2) as the density of amide bond is greater in (1)
polymerisation of ethene takes place in a hydrocarbon therefore the chain links to each other strongly in (1) than
solvent in presence of catalyst such as Ziegler-Natta in (2) hence (1) melts at higher temperature.
catalyst. 44. (d) Nylon–6 is used in the manufacturing of tyre cords
26. (b) Nylon-6, 6 is an example of first synthetic fibre while nylon–6,6 is used in making sheets, bristles for
produced from the simple molecules. It is prepared by brushes and in textile industry. Melamine –formaldehyde
condensation polymerisation of adipic acid and polymer is used in the manufacture of unbreakable
haxamethylene diamine. crockery.
POLYMERS 731

45. (c) Biodegradable polymers are manufactured because Cl


conventional polymers ar e quite resistant to the |
environmental degradation which leads to accumulation of (- CH 2 - CH = C - CH 2 - ) n
polymeric solid waste materials causing acute Neoprene
environmental problems. 59. (a) Terylene is prepared by condensing terephthalic acid
46. (c) A – (s), B – (r), C – (p), D – (q) and ethylene glycol
47. (b) A – (q), B – (r), C – (p), D – (t), E – (s)
48. (a) A – (r), B – (p), C – (q), D – (t), E – (s) nHOOC COOH + nHOCH2CH 2OH
49. (a) A – (q), B – (r), C – (p), D – (s)
Terephthalic acid Ethylene glycol
50. (b) Nylon 6, 6 has amide linkage capable of forming
hydrogen bonding. O O
51. (b) || ||
C C – OCH2 –CH2 – O
52. (d) Nylon and cellulose, both have intermolecular n
hydrogen bonding, polyvinyl chloride has dipole-dipole Terylene
interactions, while natural rubber has van der Waal forces 60. (d)
which are weakest.
53. (b) Due to the presence of strong C–F bonds, teflon has Exercise-2
high thermal stability and chemical inertness.
54. (b) The condensation polymerisation of hexamethylene 1. (d) Glycogen is a polymer of glucose found in liver, brain
diamine and adipic acid is done in solution form by interface and muscles of animals.
technique. In this liquid nylon polymer is obtained. 2. (a) cis-polyisoprene is a synthetic polymer.
3. (b) The commercial name of polyacrylonitrile is orlon
n.H 2 N - (CH 2 )6 - NH 2 + nHOOC – (CH 2 ) 4 – COOH
(acrilan).
Polymerisation
¾¾¾¾¾¾
- nH O
® [-HN - ( CH 2 )6 - NHCO- ( CH 2 ) 4 - CO-]n n CN
Nylon
2
|
55. (c) Bakelite can be heated only once. CN ¾¾¾¾¾¾¾ ® ( CH2 CH )n
Peroxide catalyst
PAN
O O or orlon
or acrilan
56. (a) C6H5 – C – O – O – C – C6H5 ¾¾® 4. (c) PHBV is an example of biodergradable polymer.
O
|| HOOC COOH
g g
2C6H5 - C- O ¾¾
® 2C6 H5
(A) 5. (a) n HOCH2 — CH2OH + n
Glycol
Phenyl radical Phthalic acid
g ·
O
C6 H5 + CH 2 = CH 2 ¾¾
® C6 H5 - CH 2 - C H 2 Zn (OCOCH3)2 + Sb2O3 ||
(A) 420-460 K
¾¾¾¾¾¾¾¾® (O CH2 CH2OOC C )n
· –nH2O
C6 H 5 - CH 2 - C H 2 + nCH 2 = CH 2 ¾¾
®
·
C6 H 5 - (CH 2 - CH 2 )n – CH 2 - C H 2 Terylene or dacron
(C) 6. (b) Low density polythene is not hard because branching
·
C6 H 5 — (CH 2 - CH 2 ) n —CH 2 - C H 2 + reduces the intermolecular forces of attraction.
(C)
· CH3
C6 H5 ¾ (CH 2 – CH 2 ) n ¾ CH 2CH 2 ¾¾
® 7. (a) n CH2 = C
(C) Monomer CH3
CH3 CH3
C6H5 – (CH2CH2)n – CH2CH2 – CH2CH2 – (CH2CH2)n – C6H5 | |
Polymerisation
57. (b) Vulcanisation is a process of treating natural rubber ¾¾¾¾¾® ( CH2 C CH2 C )n
with sulphur or some compounds of sulphur under heat | |
so as to modify its properties. This cross-linking gives CH3 CH3
Polymer
mechanical strength to the rubber.
H
Cl | O O
| N || ||
O or peroxides D, Polymerisation
58. (d) nCH 2 = CH - C = CH 2 ¾¾¾¾¾¾¾
2 ® 8. (d) ¾¾¾¾¾¾®
– ( n – 1)H O –
[ NH — (CH )
2 5 — C] n
2
Chloroprene Nylon-6
Caprolactum
EBD_8350
732 CHEMISTRY

9. (d) density polythene is used for insulating electric wires and


10. (c) in the manufacture of flexible pipes, toys, coats, bottles
etc.
D
n HOCH 2CH 2OH + n HOOC COOH ¾¾® 17. (a) Formation of Nylon-6 involves hydrolysis of
caprolactum, (its monomer) in initial state.
Ethylene glycol Terephthalic acid
O
O O
|| || NH 2 H O
¾¾® H2N(CH2)5COOH
-( O CH 2CH2–O– C C- )n D

Caprolactam e-Amino Caproic acid


Terylene
It is resistant to mineral and organic acids. It is used for O O
blending with wool to provide better crease, in safety D
¾¾¾¾¾® –(NH(CH 2)5–C–NH–(CH2)5–C)–n
helmets and aircraft battery boxes. Polymerise
Nylon-6
11. (a) Except Dacron all are additive polymers. Terephthalic 18. (c) Cross-linked or network polymers are usually formed
acid condenses with ethylene glycol to give Dacron.
from bi-functional and tri-functional monomers and contain
strong covalent bonds between various linear polymer
HOOC COOH + chains like melamine, bakelite etc.
19. (b)
Terephthalic acid
HO – CH2 – CH2 – OH ¾¾
® –
Alc. OH
Ethylene glycol 20. (d) ¾¾¾¾®
Cl Cl Cl
CH3
— CO COO – CH2 – CH2 – O —
n
CH—CH
n
Dacron (Polyester)

12. (d) H2 N—CH2—COOH + H2 N — (CH2)5—COOH ® polymerisation


¾¾ ¾¾®
Glycine Aminocaproic acid
(HN—CH2—CO—NH— (CH2)5—CO) — Cl
Nylon-2-nylon-6

O 21. (d)
O
NH H
13. (a) ¾¾¾® N
Nylon–6 n
Caprolactum ¾¾¾®
14. (d) Glyptal is used in the manufacture of paints and
lacquers. (2) Free radical
polymerisation
15. (a) Natural rubber is found to be a polymer of cis-isoprene ¾ ¾¾¾¾®
i.e. it is cis-polyisoprene
polymerisation
H2C C — CH CH2 ¾¾¾¾¾¾ ® 22. (d) cis-1,4-Polyisoprene is a nautral polymer.
|
CH3
Isoprene
Exercise-3
CH2 CH2
1. (d) The concerned chemical reactions are
H3C H (i) CaC 2 + 2H 2 O ® Ca ( OH ) 2 + C 2H2
64 kg Ethyne , 26kg
cis-polyisoprene
16. (b) High density polythene is used in the manufacture of (ii) C 2 H 2 + H 2 ® C2H 4
housewares like buckets, dustbins, bottles, pipes etc. Low Ethylene , 28 kg
POLYMERS 733

(iii) nC2 H 2 ® [ -CH 2 - CH 2 -]n 6. (a) 7. (d)


n ´ 28kg n ´ 28kg polythene
or 28kg or 28kg 8. (d) Cellulose acetate has been used in the manufacture
of non-inflammable photographic films.
Thus 64 kg of CaC2 gives 26 kg of acetylene which in
turn gives 28 kg of ethylene whose 28 kg gives 28 kg of the 9. (a) SARAN, a polymer of vinyl chloride (CH2=CHCl) and
polymer, polythene. vinylidene chloride, is used for making synthetic hair wigs.
2. (b) O O
|| ||
3. (b) CH = CH2
10. (c) H 2 N - (CH 2 )6– NH 2 + Cl - C - (CH 2 )8 - C - Cl
Hexamethylene diamine Sebacoyl chloride
Na, Heat
nCH 2 = CH - CH = CH 2 + n ¾¾¾¾¾® O O
Polymerisation
1, 3-Butadiene || ||
D
Styrene ¾¾
® -( HN - (CH 2 )6 - NH - C - (CH 2 )8 - C)-n
(– CH2 – CH = CH – CH2 – CH – CH2 –)n Nylon 6, 10

Butadiene - 11. (d) All the given statements about fibres are correct.
Styrene copolymer 12. (b) Perlon is nylon-6. It is prepared from a single monomer
(SBR or Buna - S)
(caprolactam) having a potential amino group at one end
4. (b) Acrilan is a polyacrylonitrile ( PAN). and a potential carbonyl group at other end.
5. (a) Addition of monomers follows isoprene rule 13. (c)
14. (b) In addition homopolymers such as teflon, empirical
CH3 CH3 formula resembles with monomer.
C = CH2 C = CH2
CH3 CH3 é CH3 ù
H T H T ê | ú
15. (a) Monomer of —
ê C — CH 2 ú polymer is

¯ ê | ú
êë CH3 úû n
CH3
H 3C +

C – CH2– C – CH2 2-methylpropene.


H 3C

CH3
EBD_8350
734 CHEMISTRY

30 Chemistry in Everyday Life


30. (b) Bithional is a well known antiseptic, added in soaps
Exercise - 1
to reduce odours produced by bacterial decomposition of
1. (d) 2. (a) 3. (d) 4. (b) organic matter of skin.
5. (d) Cl Cl
OH HO
6. (d) Drugs that mimic the natural messenger by switching
on the receptor are called agonists. While drugs that bind S
to the receptor site and inhibit its natural function are called
antagonists. Cl Cl
Bithional
7. (a) Given drug is used as antacid. 31. (a) A small quantity of sedative produces a feeling of
8. (d) Equanil is an important medicine used in depression relaxation, calmness and drowsiness.
and hypertension. 32. (b) Dilute solutions of boric acid and hydrogen peroxide
9. (b) Terfenadine is commonly used as antihistamine. are weak antiseptics.
10. (c) Salvarsan is an organoarsenic compound, used in the 33. (d) Novestrol is an antifertility drug.
treatment of syphilis. It was the first modern 34. (b)
chemotherapeutic agent. 35. (c) Antidepressant drugs inhibit the enzymes which
11. (c) 12. (b) 13. (d) catalyse the degradation of noradrenaline.
14. (a) These drugs induce sleep and are habit forming 36. (d)
common example of hypnotic drugs are luminal and saconal. 37. (b) Arsphenamine also known as Salvarsan was the first
effective treatment discovered for syphilis.
15. (b) Aspirin is antipyretic i.e., a drug which is responsible
38. (c) 39. (a)
for lowering the temperature of feverish organism to
40. (a) Tranquilizers are chemicals which are used to cure
normal, other antipyretic drugs are paracetamol,
mental diseases.
phenacetin.
41. (b) Antiseptics are those chemical which kill or prevent
16. (d) Sulpha drugs (antibacterial and antibiotic) are group the growth of micro organism. Antiseptics do not harm the
of drugs which are derivative of sulphanilamides. living tissues and can be applied on cuts and wounds.
17. (d) Valium is a tranquilizer and not an analgesic. It is used They help to reduce odour resulting from the bacterial
for treatment of stress, fatigue, mild and severe mental decomposition in the mouth and on the body.
diseases. 42. (c)
18. (a) Salol is phenyl salicylate used as antiseptic. 43. (d) Bacteriostatic drugs inhibit the growth of organism
19. (a) Antiseptic drugs cause destruction of micro-organism while bactericidal drugs kill the microorganisms.
that produce septic disease e.g. dettol, savlon, boric acid, 44. (b) Bactericidals have killing effect on microbes, while
phenol, iodoform, KMnO4 and some dye such as methylene bacteriostatic have inhibitory effect on microbes.
blue, gentin violet. 45. (b) Chloramphenicol is bacteriostatic antibiotic, while
20. (a) It is the very effective antibiotic for tuberculosis. ofloxacin is bactericidal type antibiotic.
21. (c) Broad spectrum antibiotics act on different antigens. 46. (c) Narrow spectrum antibiotics are effective against
22. (c) It is an insecticide. Gram–positive or Gram-negative bacteria. Limited spectrum
23. (b) antibiotics are effective against a single organism or
24. (c) Penicillin is an effective medicine for pneumonia disease.
disease. 47. (a) Norethindrone is an example of synthetic progestrone
25. (d) The mixture of chloroxylenol and terpenol is dettol derivative most widely used as antifertility drug.
which is used as antiseptic. 48. (d) All are characteristics of Saccharin.
49. (b) Artificial sweetener Sweetness value
26. (d) Chloroamphenicol is a broad spectrum antibiotic.
Aspartame 100
27. (a) Substances used for the treatment of malaria are
Saccharin 550
antimalarial e.g. quinine, chloroquine. Sucralose 600
28. (b) It is the known structure of penicillin G. Alitame 2000
29. (a) Broad spectrum antibiotics are those medicines which 50. (c) Sucralose does not provide calories.
are effective against several different types of harmful 51. (b) Structurally biodegradable detergents should contain
micro organisms. branched alkyl chain.
CHEMISTRY IN EVERYDAY LIFE 735

52. (b) Benzoic acid is used as preservative as sodium 6. (a) Paul Ehrlich first synthesised arsenic based
benzoate. structures in order to produce less toxic substances for
53. (b) the treatment of syphilis. Salvarsan also known as
54. (d) It is used as detergent. arsphenamine was the first effective medicine discovered
55. (c) The most widely used domestic detergent is the for syphilis. Salvarsan is toxic to human beings, but its
sodium dodecyl benzene sulphonate (SDS). effect on bacteria is much greater than on human being.
7. (a) Broad spectrum antibiotics are effective against a
CH3 – (CH2)11 SO3Na wide range of gram-positive and gram-negative bacteria
whereas narrow spectrum antibiotics are effective mainly
Sodium dodecyl benzene sulphonate
penicillin- G, against gram-positive or gram-negative
56. (a) Soaps are the sodium or potassium salts of higher bacteria antibiotic is a narrow spectrum antibiotic.
fatty acids e.g., C17H37COOK (Potassium stearate). These
are obtained by alkaline hydrolysis of oils and fats. The 8. (b) Tranquilizers are a class of chemical compounds
reaction is called saponification. used for the treatment of stress and mild or even severe
mental disease.
57. (c)
58. (b) Shaving soaps contain glycerol to prevent rapid 9. (d) Bithinol is added to soaps to impart antiseptic
drying. properties. Sodium laurysulphate and sodium dodecyl-
59. (c) 60. (b) benzenesulphonate are anionic detergents.
A gum rosin added to soap to make it leather well.
Exercise-2
10. (b) Equanil is a tranquilizer.
1. (d) Antiseptic is an antimicrobial drug which tends to 11. (b) While preparing soaps, a gum called rosin is added
inhibit the pathogenic action of microbes. They are to them. It forms sodium rosinate which leathers well.
applied to the living tissues such as wounds, cuts ulcers 12. (c) Gylcerol is added to shaving soap to prevent rapid
and diseased skin surface. drying.
So, antiseptic medicines cannot be ingested like 13. (b) Liquid dishwashing detergents are non-ionic.
antibiotics. Mechanism of cleansing action of this type of detergent
2. (c) Birth control pills contain a mixture of synthetic is same as that of soaps. These also remove grease and
estrogen and progesterone derivatives. These are sex oil by micelle formation.
hormones. Progesterone suppresses ovulation and
estrogen control the menstrual cycle. C 9H19 O (CH 2CH 2O)5 — CH 2CH 2OH
3. (a) Aspirin inhibits the synthesis of prostaglandins Liquid dishwashing detergent
which stimulate inflammation in the tissues and cause 14. (c) Polyethyleneglycols are used in the preparation of
pain. So, it is effective in relieving pain as it does not non-ionic detergents. Non-ionic detergents do not
make a person addictive and known to be a non-narcotic contain any ion in their constitution.
drug.
4. (c) Classification on the basis of molecular target: drug CH3 ( CH 2 )16 COOH + HO ( CH 2 CH 2O ) n CH 2 CH 2OH
usually interact with the biomolecules or biological Stearic acid Polyethylene glycol
macromolecules such as proteins, nucleic acids and - H 2O
® CH3 ( CH 2 )16 COO ( CH 2 CH 2 O )n CH 2CH 2 OH
¾¾¾¾
lipids. These are called drug tagets. Drug possessing
some common structural features may have the same 15. (c) Drugs usually interact with biomolecules such as
mechanism of action on a specific drug target. This carbohydrates, lipids, proteins and nucleic acid. These
classification is most useful fo the medicinal chemists. are called drug targets. Vitamins are not a target molecule
for drug function in body.
5. (a) Due to the low level of noradrenaline in the body
the message transfer process becomes slow and the 16. (c) Some drugs do not bind to the enzyme's active site
person suffers from depression. In such cases, but bind to a different site of enzyme which is called
tranquilizers are used. These drugs inhibits the enzymes allosteric site, which changes the shape of the active site,
which catalyse the degradation of noradrenaline. in such a way that substrate cannot recognize it. If the
bond formed between an enzyme and inhibitor is a strong
If the enzyme is inhibited, then the neurotransmitter covalent bond and cannot be broken easily the enzyme is
noradrenaline is slowly metabolised and can thus blocked permanently. The body then degrades the
activate the receptor for longer periods thereby enzyme-inhibitor complex and synthesises the new
counteracting the effect of depression. enzyme.
EBD_8350
736 CHEMISTRY

17. (d) Sucralose is trichloro derivative of sucrose. It is H H S


O CH3
stable at cooking temperture. It does not provide calories.
R C NH
18. (b) Artificial sweeteners are non-caloric substitutes for CH3
sugar. They are often more sweet than sugar but do not N COOH
O H
enhance nutritional value of food.
Pencillin
19. (c) Aspartame is stable under cold conditions.
20. (c) Bithionol is added to soaps to impart antiseptic CH3
properties.

21. (a) Phenelzine is an antidepressant, while others are 27. (b) CH3 – (CH2)15 – N – CH3 Br
antacids.
22. (a) Novalgin is most widely used as analgesic. CH3
Cetyltrimethyl ammonium bromide
Analgesics are pain releiving
23. (d) Sodium lauryl sulphate (C11H23CH2OSO3– Na+) is an
O
anionic detergent. Glyceryl oleate is a glyceryl ester of Cl
oleic acid. Sodium stearate (C17H35COO–Na+) is a soap. NH C CH
Cl
Cetyltrimethyl ammonium bromide 28. (2.00) O2N
*
CH CH CH2
*
OH
é + ù - OH
ê CH3 (CH 2 )15 N(CH3 )3 ú Br is a cationic detergent.
ë û
Exercise-3
24. (a) Dettol is a mixture of chloroxylenol and terpineol
which is a very commonly known antiseptic. 1. (c) Asprin is prepared by reaction of salicylic acid with
25. (a) acetic anhydride in presence of H2SO4.
26. (c) As chloroxylenol contains phenolic group so it gives 2. (b)
3. (a) Lindane or gammexane is g isomer of BHC.
positive ferric chloride test
4. (c) 5. (c)
6. (c) diamines are those compounds which contain two
amino groups.
HO NH2
H3C Dopamine -
HO
Chloroxylenol N NH2
Norethindrone has double bond, thus it will give Bayer’s Histamine -
N
test. H Histamine (HA)

O O
CºCH
Meprobamate - H2N O O NH2

Cl
N

Chlorphenamine - N
Sulphapyridine contains –NH2 group so it gives
carbylamine test.
7. (d) 8. (b)
9. (b) It is acetyl salicylic acid i.e., aspirin, used as an
analgesic and antipyretic.
10. (d) Chloramphenicol is
O
||
Penicillin contains –COOH group so it will give sodium O2N CH - CH - NH - C – CHCl2
| |
hydrogen carbonate (NaHCO3) test.
OH CH2OH
Answer Keys & Explanations
Mock Test-1
ANSWER KEYS
1 (c) 6 (d) 11 (c) 16 (a) 21 (d) 26 (a) 31 (d) 36 (a) 41 (b)
2 (d) 7 (a) 12 (c) 17 (d) 22 (b) 27 (b) 32 (b) 37 (a) 42 (c)
3 (c) 8 (a) 13 (d) 18 (b) 23 (a) 28 (b) 33 (b) 38 (c) 43 (d)
4 (c) 9 (c) 14 (a) 19 (c) 24 (b) 29 (b) 34 (a) 39 (a) 44 (a)
5 (d) 10 (c) 15 (b, c) 20 (c) 25 (d) 30 (d) 35 (b) 40 (d) 45 (d)

H 3C 8. (a) Addition of HCN to carbonyl compounds is


1. (c) C CH CH 2 CH 3 nucleophilic addition reaction. The order of reactivity of
H3C carbonyl compounds is :
Due to the presence of two similar methyl group at same HCHO > CH3COCH3 > PhCOCH3 > PhCOPh
carbon atom, above compound doesn’t show geometrical The lower reactivity of ketones is due to the presence of
isomerism. two alkyl group which shows + I effect. The reactivity of
2. (d) P = PA XA + PB XB ketones decreases as the size of alkyl group increases.
= PA XA + PB (1 – XA) 9. (c) Liquid - liquid system is known as emulsion.
= PA XA + PB – PB XA
10. (c) PCl5(g) PCl3(g) + Cl2(g)
= PB+XA (PA – PB) 1–x x x
3. (c) The cyclic portion contains more C-atoms than
acyclic portion. Hence, it is derivative of cyclopentane. Total moles after dissociation
1–x+x+x=1+x

p = mole fraction of PCl3 ×
1 PCl 3

1– (1¢-methyl) ethyl cyclopentane æ x ö


Total pressure = ç ÷P
è1+ x ø
5+5+2
4. (c) For SbCl52– , H = = 6 means sp3d 2 11. (c) PbS + 4H2O2 ¾® PbSO4 + 4H2O
2
12. (c) Since, rate of reaction becomes four times on doubling
hybridization. I3– , SF4 , and PCl5 ; all have sp3 d concentration of reactant, it is second order reaction.
hybridization.
1 mnu 2
5. (d) In éë Ni ( NH3 ) 2 Cl 2 ùû , Ni 2 + is in sp3 hybridisation, 13. (d) Pressure exerted by the gas, P = ...(i)
3 V
thus tetrahedral in shape. Hence, the four ligands are not
Here u = root mean square velocity
different to exhibit isomerism.
6. (d) Hemiacetal ® presence of alcohol and ether on same m = mass of a molecule, n = No. of molecules of the gas
carbon. Hence, (a) & (b) are clearly wrong.
7. (a) Given 3RT
Again u2 = [explained from (i)]
A ¾¾¾
Cl2
® B ¾¾¾¾¾
alc./KOH
® C ¾¾¾¾® CH 2O
O 3 /H 2O M
Hydrocarbon Here, M = Molecular wt. of the gas;
Since, hydrocarbon C give only CH2O on ozonolysis, C Hence, (c) is wrong.
should be CH2 = CH2. Hence, on going backward, A should 3
be ethane. Thus, the reactions are Further, Average K.E. = KT; Hence, (d) is true.
2
Cl2 / hn alc. KOH
CH3CH3 ¾¾¾¾ ® CH3CH 2 Cl ¾¾¾¾® 14. (a) Given, for reaction
(A) (B)
(i) ® H + (aq.) + OH – (aq.);
H2O (l) ¾¾
O /H2O
CH 2=CH 2 ¾¾3 ¾
D
¾® HCHO DH r = 57.32 kJ
(C)
EBD_8350
MT- 18 CHEMISTRY

1 Oxidation
(ii) H 2 (g) + O 2 (g) ¾¾
® H 2 O(l);
2
- + -1 +5
DH r = –286.20 kJ 3HOCl ¾
¾® 2HCl+ HClO 3
For reaction (i) reduction

DH r = DH °f (H +aq) + DH °f (OH – aq) – DH °f (H 2O,l) 19. (c) According to Arrhenius equation, the rate constant
of reaction is given by,
57.32 = 0 + DH °f (OH – ,aq) – DH °f (H 2O,l) …(iii) k = Ae – Ea /RT
where A is pre-exponential factor, Ea is activation energy
For reaction (ii) and T is absolute temperature.
DH r = DH °f (H 2O, l) – E
when T ® ¥, a ® 0
RT
1
DH °f (H 2 , g) – DH °f (O2 ,g) then k = Ae0
2
or k=A
–286.20 = DH °f (H 2O, l) 20. (c) Applying VSEPR theory, both NF3 and H2O are sp3
hybridized.
On replacing this value in equ. (iii) we have
21. (d) Al2O 3 .2H 2O ® Al 2O3 + 2H 2O is calcination.
57.32 = DH °f (OH – ,aq) – (–286.20) 22. (b) The main reason for exhibiting larger number of
oxidation states by actinoids as compared to lanthanoids
DH °f = –286.20 + 57.32
is lesser energy difference between 5 f and 6d orbitals as
= –228.88 kJ compared to that between 4f and 5d orbitals.
In case of actinoids, we can remove electrons from 5f as
15. (b and c) From the given options we find options (a) is well as from 6d and due to this, actinoids exhibit larger
correct. The oxidising power of halogen follow the order F2 number of oxidation states than lanthanoids.
> Cl2 > Br 2 > I2. Option (b) is incorrect because it is not the 23. (a) Anode (oxidation) on left and cathode (reduction) on
correct order of electron gain enthalpy of halogens. right.
The correct order is Cl2 > F2 > Br 2 > I2. The low value of F2 1
24. (b) Number of A ions in the unit cell. = ´8 =1
than Cl2 is due to its small size. 8
Option (c) is incorrect. The correct order of bond
1
dissociation energies of halogens is Cl2 > Br 2 > F2 > I2. Number of B ions in the unit cell =´6 =3
2
Option (d) is correct. It is the correct order of Hence, empirical formula of the compound = AB3
electronegativity values of halogens. Thus, option (b) and 25. (d) K2Cr2O7 + 14 HCl ® 3Cl2 ­ + 2CrCl3 + 2KCl + 7H2O
(c) are incorrect.
Cl 26. (a) Co3+ : 3d-orbital

(CH3 )2 NH
[Co(CN)6]3– : 3d-orbital
16. (a) + (CH3 )2 NLi ¾¾¾¾¾ ®
CN– is a strong field ligand and it causes pairing of
N(CH 3 ) 2 electrons, as a result, number of unpaired electrons in Co3+
becomes zero and hence, it has lowest value of
+ LiCl paramagnetic behaviour.
27. (b)
17. (d) Chloramphenicol is
H +
O ¾®
|| C6H5 C6H5
O2N +
CH - CH - NH - C – CHCl 2 OH
| |
OH CH 2 OH rearranges +
¾¾¾¾®
18. (b) During disproportionation, same compound undergo C6H5
Benzyl carbocation
simultaneous oxidation and reduction. is more stable
MOCK TEST 1 MT- 19

H H 35. (b) Protonation at b–N leads to imidazolium ion, which is


H stabilized by two equivalent resonating structures.
–H + + CH
¾® C6H 5 6 5
H b + ..
N HN HN

..
trans–(more stable due cis– (less stable due to bulky H
+
to steric relief) gps. lying on same side) ¾®
28. (b) Bond length order : Single bond > Double bond >
.. .. ..
aN N N
Triple bond H H H
Equivalent resonating structures
– – – –
O O O O
– – – – –
O O O O O O O O 36. (a) Eq. wt. of H2O2 = 17
Si P S Cl
30.36
O

O O O N= = 1.78 N
17
All four compounds are resonance hybrid of 4 resonance Volume strength = 5.6 × normality
structures. The compound in which the single bond
= 5.6 × 1.78 = 10 V
character is more, will have more X–O bond length.
37. (a)
O
ôô 2KMnO4 + 3H2SO4 K2SO4 + 2MnSO4 + 3H2O + 5(O)] × 3
29. (b) (4) Ph – C – NH 2 is weak base due to resonance. COO
2 Fe + 3(O) Fe2O3 + 2CO2 ]×5
(3) Due to –M effect of NO2 and ortho effect it is weak COO
base. 6KMnO4 + 9H2SO4 + 10 COO
(2) Strong base because lone pair is not involved in Fe 3K2SO4 +6MnSO4 + 9H2O
(6 moles) COO
resonance. + 5Fe2O3+ 10CO2
(10 moles)
30. (d) The metal placed below in electrochemical series does
not react with that metal salt solution whose metal is placed
From above equation, 6 moles of MnO4– required to oxidise
above in electrochemical series.
31. (d) 32. (b) 10 moles of oxalate.
33. (b) Tertiary alcohols react fastest with conc. HCl and Thus, number of moles of MnO4– required to oxidise one
anhydrous ZnCl 2 (lucas reagent) as its mechanism 6
proceeds through the formation of stable tertiary mole of oxalate = = 0.6 moles
10
carbocation.
Mechanism 38. (c) The correct order of acidic strength of the given
species is:
CH3
| HSO3 F > H 3 O + > HSO 4 - > HCO3-
Step 1 : CH3 — C — OH + H - Cl (iv) (ii) (iii) (i)
|
CH (i) < (iii) < (ii) < (iv)
3
2-Methylpropan-2-ol 39. (a) Addition of monomers follows isoprene rule (linkage
+ from tail to head) :
ˆˆ† (CH 3 )3 C — OH 2 + Cl-
‡ˆˆ
CH3 CH3
+ C = CH2 C = CH2
ˆˆ† (CH3 )3 C + + H 2 O
Step 2 : (CH3 )3 C - OH 2 ‡ˆˆ CH3 CH3
3° Carbocation
H T H T
ˆˆ† (CH 3 )3 C - Cl
Step 3 : (CH3 )3 C+ + Cl- ‡ˆˆ
t -Butyl chloride ¯

34. (a) C 6 H 5CH 2Cl ¾¾¾


NaCN
¾® C 6 H 5CH 2CN ¾¾2 ¾
¾®
H / Ni CH3
H 3C +

Benzyl chloride C – CH2– C – CH2


H 3C

C 6 H 5 CH 2 CH 2 NH 2 CH3
β - Phenylethy lamine
EBD_8350
MT- 20 CHEMISTRY

44. (a) As the nuclear charge increases, the force of attraction


1 between the nucleus and the incoming electron increases
40. (d) Rate µ . The smaller the value of M the more is
M and hence, the elecron gain enthalpy becomes more
the rate of diffusion. negative. Hence, the correct order is :
41. (b) Stability depends on number of hyperconjugative Ca < Al < C < O < F
structures, which depends on no. of a–H atoms.
NH3 ® B ¾¾ D Br
2 ® CH CH NH
42. (c) Due to positive charge on nucleus. 45. (d) A ¾¾¾ ® C ¾¾¾¾¾ 3 2 2
(I) II KOH,(III)
43. (d) Boron nitride (BN) is known as inorganic graphite.
The most stable form is hexagonal one. It has layered Reaction (III) is a Hofmann bromamide reaction. Now
structure similar to graphite. formation of CH3CH2NH2 is possible only from a
compound CH3CH2CONH2(C) which can be obtained from
the compound CH3CH2COO– NH+4 (B).
N N+

B–
+
–B B Thus (A) should be CH3CH2COOH
O
P
N+ +N N NH
3 ® CH CH COO - NH +
– CH3CH 2 - C - OH ¾¾¾ 3 2 4
+
–B B B– –B (A) (B)
D KOH
¾¾® CH 3CH 2 CONH 2 ¾¾¾® CH3CH 2 NH 2
+N N N+ +N Br2
+ – (C)
B– –B B

N+ +N N
– +
B B–
MT- 20 CHEMISTRY

Mock Test-2
ANSWER KEYS
1 (c) 6 (a) 11 (d) 16 (b) 21 (d) 26 (c) 31 (a) 36 (b) 41 (a)
2 (c) 7 (c) 12 (d) 17 (b) 22 (c) 27 (c) 32 (d) 37 (d) 42 (a)
3 (d) 8 (c) 13 (c) 18 (d) 23 (c) 28 (c) 33 (d) 38 (c) 43 (d)
4 (d) 9 (b) 14 (a) 19 (d) 24 (a) 29 (d) 34 (c) 39 (b) 44 (a)
5 (a) 10 (c) 15 (d) 20 (b) 25 (a) 30 (b) 35 (a) 40 (c) 45 (d)

1. (c) Butter of tin is SnCl4 .5H2O .


C2H5
H2O |
+3 +7 +6 –2
¾¾® CH3 – CH2 – C – OH + Mg(OH)Br
+ 4e – – H+
2. (c) Mn2O3 ¬¾ ¾ ¾ [KMnO7 ] ¾+¾e¾® [MnO4] |
C2H5
¾
¾
¾

+ 5e–
¾

3-ethyl pentan-3-ol
¾

+ 3e–
¾
®

+4
®

Mn2+ MnO2 8. (c) Reaction rate R1= k [A]2 [B]


Now increase conc. of A by three times and conc. of B by
3. (d) PH3 (Lewis base) can react with B2H6 (Lewis acid).
two times. Then new rate
4. (d) Q = m ´ L
R2 = k [3A]2 [2B]
where, L = latent heat of vapourisation of water
= 2260 kJ/kg R1 k [A]2 [B] 1 1 1
= = ´ =
= 2260 × 103 J/kg R2 k [3A] [2B] 32 2 18
2
Q = 70 × 10–3 × 2260 × 103 = 1,58,200 Joule
5. (a) CaO is basic as it form strong base Ca(OH)2 on R2 = 18 × R1
reaction with water. Hence, rate increases by 18 times.
CaO + H2O –––––® Ca(OH)2 3
9. (b) 2Al + O2 ® Al2O3 , DH = -1596 kJ .....(i)
CO2 is acidic as it dissolve in water forming unstable 2
carbonic acid. 3
2Cr + O2 ® Cr2O3 , DH = -1134 kJ ....(ii)
2
H2O + CO2 –––––® H2CO3
By (i) –(ii)
Silica (SiO2) is insoluble in water and acts as a very weak
2Al + Cr2O3 ® 2Cr + Al2O3 , DH = -462 kJ.
acid.
SnO2 is amphoteric as it reacts with both acid and base.
SnO2 + 2H2SO4–––––® Sn(SO4)2 + 2H2O ¬¾® ¾¾
®
10. (c)
SnO2 + 2KOH–––––® K2SnO3 + H2O +
+
6. (a) The sequence of bases in mRNA are read in a serial
order in groups of three at a time. Each triplet of nucleotides 3° 3° allylic (more stable)
(having a specific sequence of bases) is known as codon. 11. (d) Faraday’s laws of electrolysis are independent of
Each codon specifies one amino acid. Further since, there temp; nature of electrodes and electrolytes.
are four bases, therefore, 43 = 64 triplets or codons are
12. (d) When both double and triple bonds are present, then
possible.
double bond is considered as the principal group.
7. (c) CH3 – CH2 – C = O + C2H 5 – MgBr ¾® 13. (c) This is Avogadro’s hypothesis.
|
C 2H 5 C2H5 According to this, equal volume of all gases contain equal
|
CH3 – CH2 – C – O – MgBr no. of molecules under similar condition of temperature
| and pressure.
C2H5
EBD_8350
MOCK TEST 2 MT- 21

24. (a) It is due to movement of energised electrons


HBr
14. (a) (CH3 )2 C = CH 2 ¾¾¾
® (CH3 )2 CHCH 2 Br (KE µ T).
Isobutylene Isobutyl bromide

Note that, here HBr is added in anti-Markownikoff’s manner,
25. (a) CH2 — C — CH3 CH2 = C — CH3
so reaction should take place in presence of peroxide.
Hydroquinone and diphenylamine are not free-radical O O
producing substances but scavangers. –
I II
15. (d) In FeS2, Fe2+ is converting into Fe3+ and sulphur is
changing from –1 oxidation state to +4 oxidation state. 26. (c) Na3Al F6 increases conductivity and reduces the melt
S–
There are two and one Fe2+ in FeS2. Thus total no. of temperature.
electrons lost in the given reaction are 11. 27. (c) Patient feels like a medicine is given to him/her.
16. (b) It is an example of concentration cell, Ecell cannot be 28. (c) (a) Cr =
zero since [H+] are different (HCl is strong and CH3COOH 3d 4s
weak acid). In the presence of strong ligand (CO), the electrons
17. (b) Forward reaction is favoured by removal of products. will pair up :
18. (d) Interstitial compounds of transition metal exhibit
Cr =
metallic conductivity. 4s
3d
s
s C = C —H
s
19. (d) H— p [Cr(CO)6] =
s| |s
3d 4s 4p
H H
d 2sp3
20. (b) Liquid He behaves like a gas.
21. (d) Colligative properties depends upon the no. of Hence, this complex is octahedral and diamagnetic.
particles. Since, methanol is non electrolyte hence, cannot
be considered. (b) Fe =
3d 4s

[Fe(CO)5] =
AlCl3 3d 4s 4p
22. (c) + (CH3)2CHCl ¾¾¾®
dsp3

Hence, this complex is trigonal bipyramidal and


diamagnetic.
HNO Fe / HCl
¾¾¾¾

¾¾¾¾
® (c) Co =
H2SO4 NaOH
3d 4s

NO2 [Co(CO)4] =

dsp2
KCN
NaNO2/HCl
Hence, th is complex is square planar and
paramagnetic option(c) is correctly matched.
NH2
(d) Ni =
3d 4s

[Ni(CO)4] =
3d 4s 4p
sp3
CN
23. (c) Electrolytic reduction of nitrobenzene in weakly Hence, this complex is tetrahedral and diamagnetic.
acidic medium gives aniline, whereas in strongly acidic
CoCl
medium it gives p-hydroxyaniline 29. (d) 2CaOCl 2 ¾¾¾¾
2 ® 2CaCl + O
2 2
MT- 22 CHEMISTRY

37. (d) We can distinguish between formic acid and acetic


1
30. (b) After every 30 minutes the amount is reduced to acid by their action on Fehling’s solution. Formic acid gives
2
a red ppt of cuprous oxide but acetic acid does not give
therefore, t1/2 is 30 minutes. In 90 minutes the amount is red ppt.
1 1 38. (c) As MgO is a weak base hence, some energy got
reduced to i.e. n . Here, n = 3, which is true for 1st order
8 2 consumed to break MgO (s). Hence, enthalpy is less than
–57.33 kJ mol–1.
reaction.
31. (a) Normally NaBH4 as well as LiAlH4 reduce only – OH -
39. (b) PhCOCHBr2 ¾¾¾® PhCOCHO
CHO group without affecting carbon-carbon double bond.
(A)
However, when it is present in conjugation with benzene
ring and aldehydic group, it is also reduced along with O–
the reduction of –CHO group. OH
OH - +
NaBH 4 ¾¾¾® Ph – C – C H
¾¾® PhCH(OH)COOH
C6 H 5 CH = CH CHO ¾¾¾¾
®
O
C6 H5CH 2CH 2 CH 2 OH H
(B) (C)
32. (d)
1 40. (c) CsCl structure transforms to NaCl structure on
33. (d) He2 Bond order = (2 - 2 ) = 0 . Hence, cannot be
2 heating.
formed. 41. (a) Cl2 is obtained by electrolysis of (aqueous) NaCl.
42. (a) Calcium and magnesium form complexes with EDTA.
O
3 RT
43. (d) m rms = = 30 R1/2
COOH H —NH
M
D
34. (c) ¾¾®
H O
O ¾¾¾®
2
D
2 3R ´ (27 + 273)
COOH Þ = 30 ´ 30 ´ R
M
O
(A) 3 ´ 300
Þ M= = 1 kg / mol
succinic 30 ´ 30
anhydride
44. (a) ABS is acrylonitrile-butadiene-styrene rubber which
O is obtained by copolymerisation of acrylonitrile,
–+ 1, 3-butadiene and styrene.
Br +2 COOK
NH ¾¾¾®
KOH
NH2 CH2 = CH + CH 2 = CH + CH2 = CHC6H5 ¾¾
®
(C)
| | Styrene
O CN CH = CH 2
(B) Acrylonitrile 1,3-Butadiene
succinamide
35. (a) Smaller the charge on anion, lesser will be its
coagulating power. é CH 2 - CH - CH 2 CH = CHCH 2 - CH 2 - CH —
— ù
\ KBr have Br– with least charge of – 1 on Br, thus, KBr ê | | ú
êë CN C 6H5úû
is least effective in coagulating Fe(OH)3. n

ABS rubber
36. (b)
45. (d) Since, Sc3+ does not contain any unpaired electron it
Cl is colourless in water.
(A) (Z)
EBD_8350
Mock Test-3
ANSWER KEYS
1 (d) 6 (c) 11 (d) 16 (a) 21 (b) 26 (b) 31 (d) 36 (d) 41 (c)
2 (c) 7 (c) 12 (c) 17 (d) 22 (b) 27 (d) 32 (d) 37 (b) 42 (a)
3 (b) 8 (c) 13 (d) 18 (b) 23 (b) 28 (c) 33 (a) 38 (b) 43 (c)
4 (c) 9 (a) 14 (c) 19 (c) 24 (b) 29 (a) 34 (d) 39 (a) 44 (a)
5 (d) 10 (c) 15 (c) 20 (c) 25 (b) 30 (c) 35 (a) 40 (d) 45 (d)

1. (d) –OCH3 activates the benzene ring. –NO2 deactivates CuSO 4 + 4NH 3 ¾¾
® [Cu(NH 3 ) 4 ]SO 4
the ring. Hence, the reaction of the given compounds with Blue complex
electrophiles is in the order, I > II > III.
2. (c) It has centre of symmetry, so optically inactive. 9. (a) For bcc lattice, body diagonal = a 3 .
The distance between the two oppositely charged ions =
3. (b) Reaction (b) does not involve the formation of Cl +,
a
the electrophile. 3
2
4. (c) K2HgI4 + KOH is called Nessler’s reagent. It is used
for the detection of NH4+ in group analysis and in 387 ´ 1.732
= = 335 pm
quantitative determination of ammonia solution. It gives a 2
yellow or brown precipitate with concentrations as low as 10. (c) Given : Percentage of the iron = 0.334%; Molecular
1 ppm of NH3.a weight of the haemoglobin
5. (d) I2 and NaOH react with acetophenone (C6H5COCH3) = 67200 and atomic weight of the iron = 56. We know that
to give yellow ppt. of CHI 3 but benzophenone the number of iron atoms
(C6H5COC6H5) does not and hence, can be used to Molecular wt. of haemoglobin ´ % of iron
<
distinguish between them. Oxidation of methyl ketones 100 ´ Atomic weight of iron
with sodium Hypoiodite (NaOI) or (I2 + NaOH) gives 67200 ´ 0.334
iodoform (CHI3) but not in case of benzophenone. = =4
100 ´ 56
6. (c) Percentage of degree of ionization = 1.34%
11. (d) Octane number defines the knocking tendencies of
\ Degree of ionization (a ) = 0.0134 various hydrocarbons. Higher is the octane number, lesser
is the knocking tendency of fuel. Experimentally it is
ˆˆ† CH3 COO- + H +
CH3COOH ‡ˆˆ
Initial 0.1 0 0
observed that 2, 2, 4-trimethylpentane, a branched chain
At equilibrium 0.1(1 – 0.0134) 0.1 ´ 0.0134 0.1 ´ 0.0134 hydrocarbon has the highest anti-knocking property. It
has been given an octane number 100, while n-heptane, a
[CH3COO - ] [H + ] straight chain hydrocarbon has the highest knocking
K=
[CH 3COOH] property, assigned zero octane number.
12. (c)
0.1 ´ 0.0134 ´ 0.0134 ´ 0.1
= o
0.1 ´ (1 - 0.0134) 13. (d) Ecell = Eo + Eo first cell
Zn / Zn 2+ Cu 2+ / Cu
-5
= 0.182 ´ 10
7. (c) Let oxidation number of Co and O2 be x and y 1.10 = Eo + 0.34
Zn / Zn 2+
respectively.
EAN = Atomic number – oxidation number + (2 × Eo = 0.76 V
coordination number) Zn / Zn 2+
Þ 36 = 27 – x + 2 × 6 Þ x = + 3; o
Now, x + 2 (0) + y + 0 + (–1) = 0 Ecell = Eo + Eo second cell
Cu / Cu 2+ Ag + / Ag
Þ y = –2 or O22– is a peroxide ion.
8. (c) 3Mg + N 2 ¾¾ ® Mg3 N 2 0.46 = -0.34 + E o
Ag + / Ag
M
M Y

M g3 N 2 + 6H 2 O ¾¾
® 3Mg(OH) 2 + 2NH3 ­ Eo = 0.80 V
Ag + / Ag
Y colourless
For third cell
MT- 24 CHEMISTRY

o
Ecell = Eo + Eo 7 pR3
Zn / Zn 2+ Ag + / Ag
3 7p 3
= 0.76 + 0.80 Desired ratio = =
64 3 64
= 1.56 V R
3 3
14. (c) Ca and CaH2 both react with H2O to form H2 gas,
Ca + 2H2O ¾ ¾® Ca(OH)2 + H2 18. (b) Initial concentration = 10 mol L–1
\ Conc. after 20 min (two half lives) = 2.5 mol L–1
CaH2 + 2H2O ¾
¾® Ca(OH)2 + 2H2
0.693 0.693
whereas, Now, k = =
t1 / 2 10 min
K gives H2 while KO2 gives O2 and H2O2
or 0.0693 min–1
2K + 2 H2O ¾
¾® 2KOH + H2
\ rate = k × [reactant]
2KO2 + 2H2O ¾
¾® 2KOH + O2 + H2O2
= 0.0693 × 2.5 mol L–1 min–1
Similarly, Na gives H2 while Na2 O2 gives H2O2 19. (c) Among the isoelectronic species, size increases with
2 Na + 2 H 2 O ¾
¾® 2 NaOH + H 2 the increase in negative charge. Thus, S2– has the highest
negative charge and hence, largest in size, followed by
Na 2 O 2 + 2H 2 O ¾
¾® 2 NaOH + H 2 O 2 Cl–, K+ and Ca.
Likewise Ba gives H2 while BaO2 gives H2O2 U H2 TH 2 ´ 28 TH 2 1
20. (c) = = 7 Þ =
Ba + 2 H2O ¾
¾® Ba (OH)2 + H2 U N2 TN 2 ´ 2 TN 2 2
BaO2 + 2H2O ¾
¾® Ba (OH)2 + H2 O2
Þ TN 2 = 2TH2 \ TN2 > TH 2
CH3 CHO
21. (b) Reactivity decreases as the number of –NO2 group
CrO Cl w.r.t. –Br decreases. m-nitrobromobenzene is, however, less
15. (c) ¾¾¾®
2 2
in CCl4 reactive than the p-nitrobromobenzene since, the –NO2
Toluene Benzaldehyde group at the m-position cannot stabilize the intermediate
carbanion by resonance.
This reaction can not be given by aliphatic compounds. 22. (b) Halides Cl –, Br –, I – React with AgNO3 to give
NO 2 AgCl ¾¾ ® soluble in NH4OH
NO 2 AgBr ¾¾ ® spraringly soluble in NH4OH
16. (a) or AgI ¾¾ ® Insoluble
and the C – Cl bond is weakest in benzyl chloride [(sp3)
Hence, it is homocyclic (as the ring system is made of one hybridised carbon is attached to Cl]
type of atoms, i.e. carbon) but not aromatic.As it does not 23. (b) In froth floatation process, ore particles float because
have (4n +2)p electrons required for aromaticity. their surface is hydrophobic i.e., not easily wetted by oil.
17. (d) Effective no. of atoms of B present in a unit cell = 2 24. (b) The correct order of ionic radii is
Total volume of B unoccupied by A in a unit cell
Lu3+ < Eu3+ < La 3+ < Y39
3+
At. No. 71 63 57
= 2´
3
(
4 3 3
R -r ´p ) [In lanthanides, ionic radii (M3+) decreases with increase
r = radius of A in atomic number. Y3+ belongs to d-block].
R = radius of B 25. (b) No. of equivalents of caustic soda
= 20 × 10–3 × 0.5 = 10–2
7 pR3 æ Rö
= çQ r = 2 ÷ \ No. of equivalents of acid = 10–2
3 è ø
3 Wt.of acid ´ Basicity
Volume of unit cell = a Also, = No. of equiv..
MW
3
æ 4R ö
Þ ç
è 3ø
÷ =
64 3
3 3
R (Q 3a = 4 R ) Þ
0.45
´ x = 10-2 Þ x = 2
90
EBD_8350
MOCK TEST 3 MT- 25

26. (b) It is optically active when two Cl atoms are in cis 32. (d) From Hess’ law :
position. DHsoln = DH1 + DH2 + DH3
27. (d) Amino acid is 5% dissociated. 33. (a) The reason for the lesser basicity of aniline as
Thus, [A–] = [conjugate base] = 0.05 compared to that of cyclohexylamine is +R effect of –NH2
[HA] = [Amino acid] = 0.95 group (i.e., electron releasing resonance effect of – NH2
[ A- ] group).
pH = pKa + log Soda -lim e ( X )
[HA] 34. (d) C6 H 5COOH ¾¾¾¾¾¾®
æ1ö C6 H6 + Na 2CO3 + H 2O
= 9.15 + log ç ÷
è 19 ø Zn dust (Y )
C6 H 5OH ¾¾¾¾¾® C6 H 6 + ZnO
= 7.87 35. (a) Because at infinite dilution th e equivalent
28. (c) Molar mass ­, ‘a’ increases conductance of strong electrolytes furnishing same number
size of molecule ­,‘b’ increase of ions is same.
b (L/mol) a (bar. L2/mol2) 36. (d) Atomic radius decreases as atomic number increases
H2 ® 0.02661 CH4 ® 2.25 for lanthnides.
He ® 0.0237 O2 ® 1.36 37. (b) Elements X, Y, Z with atomic numbers 19, 37, 55 lie in
O2 ® 0.03183 H2 ® 0.244 group 1 (alkali metals). On moving down a group, the size
CO2 ® 0.04267 of atoms increases; the outermost electrons become less
29. (a) In alkaline medium, SnO2 is converted into negatively strongly held. So, the ionization energy decreases.
charged colloidal sol Therefore, I.E. of Y could be between those of X and Z.
X Y Z
SnO 2 + 2OH - ¾¾
® SnO32 - + H 2O
K (19) Rb (37) Cs (55)
SnO 2 + SnO32 - ¾¾
® [SnO 2 ]SnO32 - I.E. 4.3 4.2 3.9
144244 3 38. (b) We have to calculate the enthalpy of the reaction
anion migrates to anode
OH (g) ® O(g) + H(g)
30. (c) Chloramphenicol is a broad spectrum drug. From the given reactions, this can be obtained as follows.
Broad spectrum antibiotics are medicines effective against
gram positive as well as gram negative bacteria, e.g., é1 1 ù
– ê H 2 (g) + O 2 (g) ® OH(g) ú ; DH = –42.09 kJ mol–1
tetracycline, chloramphenicol, etc. ë2 2 û
1 1
31. (d) [ Co(CN)6 ] 4- +
2
[H2(g) ® 2H(g)]; DH = × 435.89 kJ mol–1
2
CO 2 + , d 7 , SFL 1 1
eg + [O (g) ® 2O(g)]; DH = × 495.05 kJ mol–1
2 2 2
Adding these three equations :
; DH =
D0 OH(g) ® H(g) + O(g) 423.38 kJ mol -1
39. (a) The boron atom in boric acid, H3BO3 is electron
t2g deficient i.e., boric acid is a Lewis acid with one p-orbital
vacant. There is no d-orbital of suitable energy in boron
Unpaired e– = 1
atom. So, it can accommodate only one additional electron
3+
éë Cr ( NH3 )6 ùû pair in its outermost shell.

Cr 3+ , d 3 , SFL
40. (d)
0.0591 [ Fe3+ ]
E = E° - log
eg 1 éë Ag + ùû [ Fe2 + ]
as Q ­ {[Ag+] ¯, [Fe3+] ­}, E¯
D0 15
41. (c) C6 H6 + O2 ¾¾ ® 6CO2 + 3H 2 O
2
1 mole º 3250 kJ
t2g 0.39 0.39
mole º 3250 ´ = 16.25 kJ
Unpaired e– = 3 78 78
MT- 26 CHEMISTRY

42. (a) Mac Arthur process involves extraction of gold from 45. (d) Given,
sulphide ore with the help of KCN. ˆˆ† 2NH 3 ; K1
N 2 + 3H 2 ‡ˆˆ ....(i)

NH2 ˆˆ† 2NO; K 2


N 2 + O 2 ‡ˆˆ ....(ii)
43. (c) (I) + R effect 1
H2 + ˆˆ† H 2 O; K 3
O2 ‡ˆˆ ....(iii)
2
the lone pair of electron is less easily available for We have to calculate
protonation.
4NH3 + 5O2 ¾¾
® 4NO + 6H 2O; K = ?
(II) 5
+ R effect or 2NH 3 + O2 ¾¾
® 2NO + 3H 2 O
2
N [NO]2 [H2O]3
H For this equation, K =
the lone pair of electron is not available for
[NH3 ]2 [O2 ]5 / 2
2
protonation. [NH3 ] [NO]2
but K1 = , K2 =
3 [N 2 ] [O2 ]
More +I, less H-bonding [N 2 ] [H 2 ]
N — CH3
(III) + I, H-bonding [H 2O] 3 [H 2O]3
and K 3 = or K 3 =
[H 2 ] [O2 ]½ [H 2 ]3 [O 2 ]3/2
+I more preferred than H-bonding, is due to steric
hindrance to H-bonding in 1° amine. K 2 . K 33
Now operate,
K1
(IV) + I, H-bonding
[NO]2 [H 2O]3 [N ] [H 2 ]3
NH2 = ´ . 2
[N 2 ] [O2 ] [H 2 ]3 [O2 ]3/ 2 [NH3 ]2
More H-bonding, less +I effect
Thus, the correct order is [NO]2 [H 2 O]3
= =K
(II) < (I) < (IV) < (III) [NH 3 ]2 [O 2 ]5 / 2
44. (a) 3MnO 2– –
4 + 2H 2O ® MnO 2 + 2MnO 4 + 4OH

K 2 . K 33
2 1 2 – 4 – \ K=
or MnO 2–
4 + H 2 O ® MnO 2 + MnO 4 + OH K1
3 3 3 3
EBD_8350
Mock Test-4
ANSWER KEYS
1 (a) 4 (a) 7 (b) 10 (d) 13 (a) 16 (b) 19 (c) 22 (0.78) 25 (1.7)
2 (c) 5 (b) 8 (a) 11 (d) 14 (a) 17 (d) 20 (b) 23 (10)
3 (c) 6 (a) 9 (a) 12 (a) 15 (c) 18 (d) 21 (200) 24 (2)

6. (a)
1. (a) H3C CH3 CH2 CH3
alc. KOH
¾¾¾¾® NO2 NO2
CH –
CH3 CH2
I NaOH CH -CHO
¾¾¾¾
® ¾¾¾¾¾
3
+
®
- H 2O H
SO Cl , heat
¾¾¾¾¾¾¾ 2 2 ® CH = CHCH Cl
allylic substitution 2 2
OH NO2
NO2
2. (c) Applying (n + l) rule CH2 = CH – CH3
CH2 – CH – CH3 D
(I) n + l = 3 + 2 = 5 (II) n + l = 5 + 0 = 5 ¾¾
®
(III) n + l = 4 + 1 = 5 (IV) n + l = 4 + 2 = 6
(V) n + l = 4 + 0 = 4
Higher the value of (n + l), more is the energy associated
7. (b) 58Ce is lanthanoid. Lanthanoids are from 57 to 71 and
with it. In case of same value of (n + l), the electron with
all are present in IIIrd group. Hence 58Ce forms 56X ele-
higher value of n has higher energy. ment on emission of one a particle which belongs to IInd
\ The order is : V < I < III < II < IV group.
3. (c) C6H6 is diamagnetic (i – 5) Å
8. (a) – CCl3, – NO2 and – NH are meta directing groups,
CrO2 is ferromagnetic (ii – 3) 3

MnO is antiferromagnetic (iii – 1) whereas – O is ortho-para directing group.

Fe3O4 is ferrimagnetic (iv – 2) 9. (a) Reactivity µ stability of TS.


10. (d) The products of the concerned reaction, react with
Fe3+ is paramagnetic with 5 unpaired electron (v – 4)
each other forming back the reactants.
4. (a) In NH3, the atomic dipole (orbital dipole due to lone XeF6 + 3H 2 O ¾¾ ® XeO3 + 6HF .
pair) and bond dipole are in the same direction whereas in 11. (d) Elemental sodium is easily oxidised ( has low I.P.) and
NF3, these are in opposite direction so, in the former case, acts as reductant.
they are added up whereas in the latter case, net result is 12. (a) A base can donate electrons hence, all are bases but
reduction of dipole moment. It has been shown in the
SO 32 - can be oxidised to SO 24 - . Hence, it is reducing also.
following figure :
13. (a) Since the sol particles migrate towards cathode, they
are positively charged. Hence, anions would be effective
N N in coagulation. Greater is the valence of effective ion,
smaller will be its coagulating value.
H H H F F F
14. (a) Symmetrically filled t2g and eg are those, which contain
equal distribution of electrons.
5. (b) At the same T and P, V µ n
[FeF3]3– ­ ­ eg ìequal distribution of ü
14 1 1 ïelectrons in orbitals ï
nN 2 = = ; VN 2 µ í ý
28 2 2 ­ ­ ­ t 2 g ïît2 g and eg ïþ

36 3 3
nO 3 = = ; VO 3 µ [Mn(CN)6]4–
48 4 4 eg unequal
distribution
t2 g of electrons
2
Hence , VN 2 / VO3 = , 3 VN 2 = 2 VO 3 in orbitals
3
MT- 28 CHEMISTRY

[CoF6]3– eg unequal R R
| |
distribution ¾¾® R – Si – O – Si – R
t2 g of electrons – H2O
| |
R R

[Co(NH3)6]2+ eg unequal Upon hydrolysis, it condensed to form a dimer.


distribution
21. (200)
t2 g of electrons
in orbitals At constant volume A ® B:
Heat change = n . DCV . DT

15. (c) + 3
=2× R × (600 – 200)
2
= 1200 R
Stabilised by extended conjugation.
At constant pressure C ® A:
16. (b) When benzaldehyde is refluxed with aqueous
Heat change = – n . DCP . DT
alcoholic potassium cyanide, two molecules of
benzaldehyde condense together to form benzoin. 5
=–2× R × (400 – 200)
2
H O
KCN (alc.) = – 1000 R
—C +C— ¾¾¾¾®
D Sum of heat change = 1200 R – 1000 R
O H = 200 R
22. (0.78)
H O 2 Fe3+ + 2 e– ® 2 Fe2+ reduction at cathode
—C –C— 2I– –® I2 + 2e– oxidation at anode
OH
Benzoin 0.0591
Eocell = log10 Kc
n
17. (d) 2Cu2S + 3O2 ® 2Cu2O + 2SO2
0.0591
3Cu 2 O + CH 4 ® 6Cu + 2H 2 O + CO = log10 108 = 0.24 V
2
(From green
logs of wood)
Eocell = E Fe3+ /Fe2+ - E I 2 /I-
18. (d) Green colour of CuCr2O7 is due to blue colour of
Cu2+ ions and yellow colour of Cr2 O 72– ions. 0.24 V = E Fe3+ /Fe2+ - E I 2 /I- – 0.54 V

19. (c) Higher the stability of the intermediate carbocation, E Fe3+ /Fe2+ = 0.78 V
more easily it is formed and more will be the ease of its 23. (10)
dehydration. Thus, order of stability of the carbocations Kf = 2 × 10–2 and Kb = 2 × 10–3
of the corresponding alcohols is Kc = Kf/Kb
+ + +
F3C CH2 < Cl3C CH 2 < H3C CH2 < C6H5CH 2
+
2 ´ 10-2
= = 10
Least stable due Most stable due 2 ´ 10-3
to intensification to resonance
of charge 24. (2) The repeating unit for (—CH2—CH2—O—)n is

F3 CCH 2 OH < Cl3CCH 2 OH < H 3CCH 2 OH (ethylene oxide).


O
< C 6 H 5 CH 2 OH (Ease of dehydration)
Hence, 2 carbons in the ring.
R R R 25. (1.7) P = KH . X or X = P/KH
| | | Mole fraction of CO2 gas remains same
HOH
20. (b) R – Si – Cl ¾¾® R – Si – O H + HO – Si –R
– HCl
| | | 0.3 P
R R R Thus, = Þ P = 1.7 atm
600 3400
EBD_8350
MT- 30 CHEMISTRY

Mock Test-5
ANSWER KEYS
1 (c) 4 (b) 7 (c) 10 (d) 13 (a) 16 (d) 19 (b) 22 (320) 25 (2)
2 (c) 5 (b) 8 (a) 11 (b) 14 (c) 17 (b) 20 (b) 23 (98)
3 (b) 6 (d) 9 (d) 12 (b) 15 (b ) 18 (c) 21 (191.47) 24 (64)

2.303 100 CH3 Cl H OH CH 3 OH


1. (c) t90% = log – 2HCl Si
k 100 - 90 Si + ¾¾®
CH3 Cl H OH CH 3 OH
Dialkyl silandiol
2.303 100
t50% = log
k 100 - 50 Polymerisation of dialkyl silandiol yields linear
thermoplastic polymer.
t90% log10
= t90% = 3.3t50%
t50% log 2 ; CH3 CH3

HO — Si — OH + H O — Si — OH ¾®
2. (c) CH3 H
C C H CH3 CH3
*
CH3 C
CH3 COOH CH3 CH3

It has asymmetric carbon atom (marked *) so, it shows HO — Si — O — Si — OH


optical isomerism. Two groups attached to C = C are same
(–CH3), so it does not show geometrical isomerism. CH3 CH3
3. (b) Be(OH)2 and Al(OH)3 are amphoteric in nature.
8. (a) CH2 CH2 – Cl CH2 – OH
4. (b) CaCO3 (s) ¾¾
D
® CO 2 (g) ­ + CaO(s) | Cl2 | |
dil. NaOH
(A) colourless CH ¾¾® CH ¾¾¾¾® CH
|| || ||
CH2 CH2 CH2
® Ca ( OH ) 2 (aq)
CaO(s) + H 2O(g) ¾¾
(propene) (allyl chloride) (allyl alcohol)
(B)

® Ca ( HCO3 )2 (aq)
Ca ( OH )2 (aq) + 2CO 2 (g) ¾¾
( B) (C ) CH2OH CH2OH
| |
D HOCl NaOH
Ca ( HCO3 ) 2 (s) ¾¾® CaCO3 (s) + CO 2(s) + H 2O(g) ¾¾¾® CHCl ¾¾¾® CHOH
(C ) ( A) || ||
CH2OH CH2OH
5. (b) (b-chlorohydrin) (glycerol)
6. (d) Number of atoms per unit cell = 1
9. (d) DTf = Kf × m × i. Since, Kf has different values for
a
Atoms touch each other along edges. Hence, r = different solvents, hence, even if the molality is same DTf
2
will be different.
( r = radius of atom and a = edge length)
10. (d) ¾® CH 3CH 2 P + Ph 3Br -
CH 3CH 2 Br + Ph 3P ¾
4 3 - +
pr C H Li or
p ¾¾6 ¾
5¾¾® CH 3 CH = PPh 3 ¬¾® CH 3 C H P Ph 3
Therefore, % fraction = 3 3 = = 0.52 C 2 H 5ONa
(2r ) 6
11. (b) Biodegradable polymer is Nylon-2-Nylon-6, which is
7. (c) Hydrolysis of substituted chlorosilanes yield copolymer of glycine (H 2N – CH 2– COOH) and
corresponding silanols which undergo polymerisation. aminocaproic acid (H2N–(CH2)5 – COOH).
MOCK TEST 5 MT- 31

nH2N – CH2 – COOH +


16
glycine nH2N – (CH2)5– COOH 16. (d) 16.0 g O3 = mole
48
aminocaproic acid
16
= ´ 6.023 ´ 1023 molecules
48

16
O O = 3´ ´ 6.023 ´ 1023 atoms
48
–( HN – CH2 – C – HN – (CH2)5 – C –) n = 6.023 × 1023 atoms
nylon – 2 – nylon – 6 28
28.0 g CO = mole = 1 mole
28
Lm 9.54
12. (b) a = = = 0.04008 = 4.008 %. = 1 × 6.023 × 1023 molecules

m
238
= 1 × 6.023 × 1023 atoms
13. (a) Conversion of C4H6 to C4H6Br2 indicates that the = 6.023 × 1023 atoms
compound is either butyne-1 or butyne-2. However, white
16 1
precipitate with ammonical silver nitrate solution indicates 16.0 g O 2 = mole = mole
32 2
that it is a terminal alkyne, i.e. butyne-1 and not butyne-2.
1
14. (c) Nickel ions are frequently detected by the formation = × 6.023 × 1023 molecules
2
of red precipitate of the complex of nickel dimethylglyoxime,
1
when heated with dimethylglyoxime. = × 2 × 6.023 × 1023 atoms
2
= 6.023 × 1023 atoms
CH3 C NOH 2+
+ Ni ¾¾® Therefore, the ratio is
CH3 C NOH 6.023 × 1023 : 6.023 × 1023 : 6.023 × 1023
Dimethylglyoxime i.e. 1 : 1 : 1
17. (b) The secondary structure of a protein refers to the
shape in which a long peptide chain can exist. There are
H-bonding two different conformations of the peptide linkage present
OH O
in protein, these are a-helix and b-conformation. The a-
­
CH3 C N N C CH3 helix always has a right handed arrangement. In b-
Ni conformation all peptide chains are streched out to nearly
CH3 C N N C CH3
¯ maximum extension and then laid side by side and held
O HO together by intermolecular hydrogen bonds. The structure
Nickel dimethylglyoxime
resembles the pleated folds of drapery and therefore is
known as b-pleated sheet.
[dsp2 hybridisation and square planar geometry] 18. (c) Structure I has two resonance forms, which make
the hydrogen bonding more stable while II molecule has
CºN
no such resonance stabilization.
+ CH3MgBr ¾¾
® 6Cl2 (g) + 6Ba(OH)2 (aq) ¾¾
®
15. (b) 19. (b)
OCH3
Ba (ClO3 ) 2 (aq) + 5BaCl2 + 6H 2 O
(X)

H3C – C = NMgBr COCH3


Ba(ClO 3 ) 2 (aq) + H 2SO 4 ¾¾
® 2HClO3 + BaSO 4 (s)
+ (X) (Y )
HO
¾¾¾¾¾¾
3
®
– Mg(OH)Br, – NH 3 D
OCH3 OCH3 2HClO3 ¾¾¾® Cl2O + H 2 O + 2O 2
Q P 365 K
(Y ) (Z )
EBD_8350
MT- 32 CHEMISTRY

20. (b) CH3 CH3 CH3 23. (98)

Br Br Z´M
Br2 Sn/HCl
¾¾® ¾¾¾® Theoretical density =
FeBr3
a3 ´ N A

NO2 NO2 NH2 4 ´ 24


= -10 3
(400 ´ 10 ) ´ 6.023 ´ 1023
CH3 CH3
Br = 2.5 g/cm3
Br NaNO2
HBr
¬¾¾ ¬¾¾¾¾ observed density
CuBr HCl
\ % occupancy = theoretical density × 100
Br N = NCl
2.45
21. (191.47) = × 100
2.5
V2 V2
DS = n R ln V = 2.303 n R log V = 98
2
1 1
24. (64)
1000
= 2.303 × 5 × 8.314 × log 2NO + O2 ® 2NO2
10
= 191.47 J mol–1 K–1 Rate of reaction = K [NO]2 [O2]
22. (320)
If volume of the vessel is diminished to 1/4, then the con-
mSO2 M Ne TSO2 1 10 TSO2
centration of each reactant will increase to 4 times.
´ ´
m Ne
= MSO 2 TNe Þ 2 = 64 200 New rate of reaction = K × 42 [NO]2 × 4 × [O2]
= 64K [NO]2 [O2]
Þ TSO = 320 K
2

25. (2)
NOTES
NOTES

EBD_8350
MOCK TEST - 1
Time : 1 hr. Max. Marks -180

1. Which of the following will not show cis-trans isomerism? 6. Which of following compound is hemiacetal?
(a) CH 3 — CH = CH — CH 3
HO OMe
(b) CH 3 — CH 2 — CH = CH — CH 2 — CH 3 O OH
(a) (b)
(c) CH3 — C = CH — CH 2 — CH3
|
CH3
H OH
(d) CH 3— CH — CH = CH — CH 2 — CH 3 (c) (d) all of these
| O
CH3
7. A hydrocarbon A on chlorination gives B which on heating
2. P A and P B are the vapour pressure of pure liquid
with alcoholic potassium hydroxide changes into another
components, A and B, respectively of an ideal binary
hydrocarbon C. The latter decolourises Baeyer's reagent
solution. If XA represents the mole fraction of component
and on ozonolysis forms formaldehyde only. A is
A, the total pressure of the solution will be.
(a) Ethane (b) Butane
(a) PA + XA (PB – PA) (b) PA + XA (PA – PB)
(c) PB + XA (PB – PA) (d) PB + XA (PA – PB) (c) Methane (d) Ethene
8. The increasing order of the rate of HCN addition to
3. What is the IUPAC name of the compound ?
compound A – D is
(A) HCHO (B) CH3COCH3
CH(CH3)2
(C) PhCOCH3 (D) PhCOPh
(a) D < C < B < A (b) C < D < B < A
(c) A < B < C < D (d) D < B < C < A
9. Point out the false statement
(a) Brownian movement and Tyndall effect are shown
(a) 1, 1 – dimethyl – 1 – cyclopentyl methane
by colloidal systems.
(b) 2 – cyclopentyl propane (b) Gold number is a measure of the protective power of
(c) 1 – (1' – methyl) ethyl cyclopentane a lyophillic colloid.
(d) Cumene. (c) The colloidal solution of a liquid in liquid is called
4. In which one of the following species, the central atom gel.
has the type of hybridization which is not the same as that (d) Hardy - Schulze rule is related with coagulation.
present in the other three? 10. Phosphorus pentachloride dissociates as follows, in a
closed reaction vessel
(a) SF4 (b) I3– (c) SbCl52– (d) PCl5
ˆˆ† PCl3(g) + Cl2(g)
PCl5(g) ‡ˆˆ
5. Which one of the following complexes is not expected to If total pressure at equilibrium of the reaction mixture is P
exhibit isomerism? and degree of dissociation of PCl5 is x, the partial pressure
(a) [ Ni(en)3 ]2+ of PCl3 will be
2+
(b) é Ni ( NH3 ) ( H 2 O ) ù æ x ö æ x ö
ë 4 2û
(a) ç x -1 ÷ P (b) ç ÷P
è ø è 1- x ø
(c) é Pt ( NH3 ) Cl 2 ù
ë 2 û
æ x ö æ 2x ö
(d) é Ni ( NH3 ) Cl 2 ù (c) ç x +1÷P (d) ç ÷P
ë 2 û è ø è 1- x ø
EBD_8350
MT- 2 CHEMISTRY

11. Which one of the following reactions does not form 17. An antibiotic contains nitro group attached to aromatic
gaseous product? nucleus. It is
(a) PbO2 + H2O2 (a) penicillin (b) streptomycin
(c) tetracycline (d) chloramphenicol
(b) Acidified KMnO4 + H2O2 18. What products are expected from the disproportionation
(c) PbS + H2O2 reaction of hypochlorous acid?
(d) Cl2 + H2O2 (a) HCl and Cl2O (b) HCl and HClO3
12. In a reaction, when the concentration of reactant is (c) HClO3 and Cl2O (d) HClO2 and HClO4
19. The rate constant of a reaction becomes equal to the
increased two times, the increase in rate of reaction was
pre-exponential factor when
four times. Order of reaction is :
(a) the absolute temperature is zero
(a) zero (b) 1
(b) the activation energy is infinity
(c) 2 (d) 3
(c) the absolute temperature is infinity
13. According to kinetic theory of gases, for a diatomic (d) the temperature in Celsius is zero.
molecule 20. In which of the following pair, both the species have sp3
(a) the pressure exerted by the gas is proportional to hybridization?
mean velocity of the molecule (a) H2S, BF3 (b) SiF4, BeH2
(b) the pressure exerted by the gas is proportional to the (c) NF3, H2O (d) NF3, BF3
root mean velocity of the molecule
21. The process of converting hydrated alumina into
(c) the root mean square velocity of the molecule is anhydrous alumina is called
inversely proportional to the temperature (a) roasting (b) smelting
(d) the mean translational kinetic energy of the molecule (c) dressing (d) calcination
is proportional to the absolute temperature. 22. Larger number of oxidation states are exhibited by the
14. On the basis of the following thermochemical data : actinoids than those by the lanthanoids, the main reason
being
(Δ f G °H + (aq) = 0)
(a) 4f orbitals more diffused than the 5f orbitals
H 2O(l) ® H + (aq) + OH – (aq); DH = 57.32 kJ (b) lesser energy difference between 5f and 6d than
between 4f and 5d orbitals
1 (c) more energy difference between 5f and 6d than
H2 (g)+ O2 (g) ¾¾ ® H2O(l); ΔH = – 286.20 kJ
2 between 4f and 5d orbitals
The value of enthalpy of formation of OH– ion at 25° C is: (d) more reactive nature of the actinoids than the
(a) –228.88 kJ (b) +228.88 kJ lanthanoids
23. The cell reaction Cu + 2Ag + ® Cu2+ + 2Ag is best
(c) –343.52 kJ (d) –22.88 kJ
represented by
15. Which one of the following arrangements does not give
the correct picture of the trends indicated against it ? (a) Cu(s) | Cu 2+ (aq) | | Ag + (aq) | Ag(s)
(a) F2 > Cl2 > Br2 > I2 : Oxidizing power
(b) Pt | Cu 2+ | | Ag + (aq) | Ag(s)
(b) F2 > Cl2 > Br2 > I2 : Electron gain enthalpy
(c) F2 > Cl2 > Br2 > I2 : Bond dissociation energy (c) Cu 2+ | Cu | | Pt | Ag
(d) F2 > Cl2 > Br2 > I2 : Electronegativity. (d) None of the above representations
16. What is the product of the following reaction ? 24. An ionic compound has a unit cell consisting of A ions at
the corners of a cube and B ions on the centres of the
Cl faces of the cube. The empirical formula for this compound
would be
(CH ) NLi
¾¾ ¾
3 ¾
2 ¾®
? (a) A3B (b) AB3
(CH 3 ) 2 NH
(c) A2B (d) AB
25. Chlorine is liberated when we heat
(a) N, N-dimethylaniline
(a) KMnO4 + NaCl (b) K2Cr2O7 + MnO2
(b) phenyllithium (C6H5Li) (c) Pb(NO3)2 + MnO2 (d) K2Cr2O7 + HCl
(c) para-chloro-N, N-dimethylaniline
(d) meta-chloro-N, N-dimethylaniline
MOCK TEST 1 MT- 3

26. Which one of the following cyano complexes would exhibit 30. The position of some metals in the electrochemical series
the lowest value of paramagnetic behaviour ? in decreasing electropositive character is given as
Mg > Al > Zn > Cu > Ag. What will happen, if a copper
(a) [Co(CN) 6 ]3 - (b) [Fe(CN) 6 ]3 - spoon is used to stir a solution of aluminium nitrate ?
(a) The spoon will get coated with Al
(c) [Mn (CN) 6 ]3 - (d) [Cr (CN) 6 ]3 -
(b) An alloy of Cu and Al is formed
(At. no : Cr = 24, Mn = 25, Fe = 26, Co = 27)
(c) The solution becomes blue
27. The following alcohol is treated with conc. H2SO4, the
(d) There is no reaction
major product obtained is
31. Which of the following practices will not come under green
conc. H2SO4 chemistry?
¾¾¾¾®
(–H2O)
C6H5 (a) If possible, making use of soap made of vegetable
OH oils instead of using synthetic detergents.
(b) Using H2O2 for bleaching purpose instead of using
chlorine based bleaching agents.
(a) C6H5 (c) Using bicycle for travelling small distances instead
of using petrol/diesel based vehicles.
H (d) Using plastic cans for neatly storing substances.
32. Which of the following graph correspond to one node
(b) C H
6 5
H
Y Y
H
H (a) (b)
(c) C6H5 a0
a0

28. Arrange the following ions in the order of decreasing


X – O bond length, where X is the central atom
Y Y
(a) ClO -4 ,SO 24 - , PO34- ,SiO -4
(c) (d)
(b) SiO44 - , PO34- ,SO24 - , ClO-4
a0 a0
(c) SiO 44 - , PO34- ,ClO4- ,SO42 -
33. From amongst the following alcohols the one that would
(d) SiO44 - ,SO42- , PO34- , ClO-4
react fastest with conc. HCl and anhydrous ZnCl2, is
29. Arrange the following compounds in order of increasing (a) 2-Butanol (b) 2- Methylpropan-2-ol
basic strength. (weakest ® strongest) (c) 2-Methylpropanol (d) 1- Butanol
34. The reaction of benzyl chloride with sodium cyanide
O followed by reduction with hydrogen in the presence of
nickel gives
NH2 CH2NH2 NH2 CNH2 (a) b-Phenylethylamine (b) N-Isobutylaniline
(c) Benzylamine (d) Aniline.
NO2
35. Which of the nitrogen of histidine is first protonated ?
+
b NH3
1 2 3 4 N
CH2CHCOO–
(a) 4 < 2 < 1 < 3 (b) 4 < 3 < 1 < 2 aN
H
(c) 4 < 1 < 3 < 2 (d) 2 < 1 < 3 < 4
(a) a (b) b (c) both (d) None
EBD_8350
MT- 4 CHEMISTRY

36. The strength in volumes of a solution containing 30.36 g/ 40. The rate of diffusion of SO2, CO2 , PCl3 and SO3 are in the
following order
L of H2O2 is
(a) PCl3 > SO3 > SO2 >CO2
(a) 10 V (b) 5 V
(b) CO2 > SO2 > PCl3 > SO3
(c) 20 V (d) None of these
(c) SO2 > SO3 > PCl3 > CO2
37. Number of moles of MnO-4 required to oxidize one mole of
(d) CO2 > SO2 > SO3 > PCl3
ferrous oxalate completely in acidic medium will be : 41. Arrange the following in increasing order of stability
(a) 0.6 moles (b) 0.4 moles + +
(A) (CH3 )2 C - CH 2 CH3 (B) (CH3 )3 C
(c) 7.5 moles (d) 0.2 moles + +
38. Four species are listed below: (C) (CH3 )2 CH (D) CH3 CH 2
+
i. HCO3– ii. H3 O+ (E) CH3
iii. HSO4– iv. HSO3F (a) E < D < C < B < A (b) E < D < C < A < B
Which one of the following is the correct sequence of (c) D < E < C < A < B (d) A < E < D < C < B
43. Which of the following structure is similar to graphite?
their acid strength? (a) B (b) B4C
(a) iv < ii < iii < i (b) ii < iii < i < iv (c) B2H6 (d) BN
(c) i < iii < ii < iv (d) iii < i < iv < ii 44. Which one of the following arrangements represents the
correct order of least negative to most negative electron
39. The monomer of the polymer; gain enthalpy for C, Ca, Al, F and O?
(a) Ca < Al < C < O < F (b) Al < Ca < O < C < F
CH3 CH3 (c) Al < O < C < Ca < F (d) C < F < O < Al < Ca
|
42. The order of ionisation potential between He+ ion and
ÚÚÚÚÚCH 2 - C - CH 2- C + is
| H-atom (both species are in gaseous state) is:
CH3 CH3 (a) I. P. (He+) = I.P. (H) (b) I. P. (He+) < I.P. (H)
(c) I. P. (He+) > I.P. (H) (d) cannot be compared
45. An organic compound A upon reacting with NH3 gives B.
CH3 On heating B gives C. C in presence of KOH reacts with
(a) H 2C = C (b) CH3CH = CHCH3 Br2 to given CH3CH2NH2. A is :
CH3 (a) CH3COOH (b) CH3CH2CH2COOH
(c) CH3 - CH - COOH (d) CH3CH2COOH
(c) CH3CH = CH2 (d) (CH3)2C = C(CH3)2 |
CH3
MOCK TEST - 2
Time : 1 hr. Max. Marks -180

1. Butter of tin is
(a) SnCl4 .6H 2O (b) SnCl4 .4H 2O
(c) SnCl4 .5H 2 O (d) SnCl4 .2H 2O H SO
2 4
10. ¾¾¾¾ ®
2. When KMnO4 acts as an oxidising agent and ultimately
forms [MnO4]2–, MnO2, Mn2O3, Mn2+ then the number of OH
electrons transferred in each case respectively is Which carbocation is involved in the above reaction ?
(a) 4, 3, 1, 5 (b) 1, 5, 3, 7 (c) 1, 3, 4, 5 (d) 3, 5, 7, 1.
3. Incorrect statement about PH3 is:
(a) It is produced by hydrolysis of Ca3P2 (a) (b)
(b) It gives black ppt. (Cu3P2) with CuSO4 solution +
+
(c) Spontaneously burns in presence of P2H4
(d) It does not react with B2H6 (c) Both (d) None
4. How many joules of heat are absorbed when 70.0 grams of 11. Faraday’s laws of electrolysis will fail when
water is completely vaporised at its boiling point? (a) temperature is increased
(a) 23,352 (b) 7,000 (b) inert electrodes are used
(c) 15,813 (d) 158, 200 (c) a mixture of electrolytes is used
5. Which of the following oxides is amphoteric in character? (d) in none of the above cases
(a) SnO2 (b) SiO2 12. The IUPAC name of the compound
(c) CO2 (d) CaO CH3CH = CHC º CH is
6. A sequence of how many nucleotides in messenger RNA (a) Pent-l-yn-3-ene (b) Pent-4-yn-2-ene
makes a codon for an amino acid? (c) Pent-3-en-1-yne (d) Pent-2-en-4-yne
(a) Three (b) Four 13. 100 mL O2 and H2 kept at same temperature and pressure.
(c) One (d) Two What is true about their number of molecules
7. To prepare 3-ethylpentan-3-ol, the reagents needed are – (a) NO > NH (b) NO < NH
2 2 2 2
(a) CH3CH2MgBr + CH3COCH2CH3 (c) NO = NH (d) NO + NH = 1 mole
2 2 2 2
(b) CH3MgBr + CH3CH2CH2COCH2CH3 14. Iso-butyl bromide may be obtained from iso-butylene and
(c) CH3CH2MgBr + CH3CH2COCH2CH3 HBr in the presence of
(d) CH3CH2CH2MgBr + CH3COCH2CH3 (a) peroxide (b) hydroquinone
8. Rate of a reaction can be expressed by following rate (c) diphenylamine (d) All of these
expression 15. FeS2 + O2 ® Fe2O3 + SO2
Rate = k[A]2 [B], if concentration of A is increased by 3 In the above equation, the number of electrons lost by
times, and concentration of B is increased by 2 times, how one molecule of FeS2 are –
many times rate of reaction increases? (a) 6 (b) 2
(a) 9 times (b) 27 times (c) 1 (d) 11
(c) 18 times (d) 8 times 16. An electrochemical cell is set up as follows :
9. The enthalpies of formation of Al2O3 and Cr 2O3 are –1596 Pt (H2, 1 atm) | 0.1 M HCl | 0.1 M acetic acid | (H2, 1 atm) Pt
kJ and –1134 kJ respectively. DH for the reaction EMF of this cell will not be zero because
(a) the temperature is constant
2Al + Cr2 O3 ® 2Cr + Al2 O3 is (b) the pH of 0.1 M HCl and 0.1 M acetic acid is not the
(a) – 2730 kJ (b) – 462 kJ same
(c) – 1365 kJ (d) + 2730 kJ (c) acids used in the two compartments are different
(d) EMF of a cell depends on molarities of the acids used
EBD_8350
MT- 6 CHEMISTRY

17. In lime kiln, the reversible reaction 23. Electrolytic reduction of nitrobenzene in weakly acidic
CaCO3 (s) CaO (s) + CO2 (g) medium gives
proceeds to completion because (a) N-Phenylhydroxylamine
(a) of high temperature (b) CO2 escaped out (b) Nitrosobenzene
(c) CaO is removed (d) of low temperature (c) Aniline
18. Which of the following is not a characteristic of interstitial (d) p-Hydroxyaniline
compounds of transition elements? 24. If one end of a piece of a metal is heated,1 the other end
(a) The formulae of these compounds do not correspond becomes hot after some time. This is due to
to any normal oxidation state (a) Energised electrons moving to the other part of the
(b) They have melting points higher than those of pure metal
elements (b) resistance of the metal
(c) They are very hard and some compounds approach (c) mobility of atoms, in the metal
diamond in hardness (d) minor perturbation in the energy of atoms.
..
(d) They are insulators in contrast to the transition metals 25. CH2 – C – CH3 and CH2 = C – CH3 are
|| |
19. The outer orbitals of C in ethene molecule can be :O
O . .:
considered to be hybridized to give three equivalent sp2 (a) resonating structures (b) tautomers
orbitals. The total number of sigma (s) and pi (p) bonds in (c) geometrical isomers (d) optical isomers
ethene molecule is 26. In the commercial electrochemical process for aluminium
(a) 1 sigma (s) and 2 pi (p) bonds extraction, the electrolyte used is
(b) 3 sigma (s) and 2 pi (p) bonds
(c) 4 sigma (s) and 1 pi (p) bonds (a) Al(OH )3 in NaOH solution
(d) 5 sigma (s) and 1 pi (p) bonds
20. Which inert gas show abnormal behaviour on liquefaction (b) An aqueous solution of Al 2 (SO 4 )3
(a) Xe (b) He (c) Ar (d) Kr (c) A molten mixture of Al2 O 3 and Na 3 AlF6
21. Which observation(s) reflect(s) colligative properties?
(i) a 0.5 m NaBr solution has a higher vapour pressure (d) A molten mixture of Al 2 O 3 and Al(OH )3
than a 0.5 m BaCl2 solution at the same temperature 27. Placebo is often given to patients. It is
(ii) pure water freezes at the higher temperature than pure (a) an antidepressant
methanol (b) a broad spectrum antibiotic
(iii) a 0.1 m NaOH solution freezes at a lower temperature (c) a sugar pill
than pure water (d) a tonic
Choose the correct answer from the codes given below 28. Which of the following is correctly matched?
(a) (i), (ii) and (iii) (b) (i) and (ii) Column - I Column - II Column - III
(c) (ii) and (iii) (d) (i) and (iii) (a) [ Cr ( CO )6 ] Paramagnetic Octahedral, sp3 d 2
22. Which sequence represents the best synthesis of 4-
isopropylbenzonitrile?
(b) [ Fe ( CO )5 ] Paramagnetic Trigonal bipyramidal, sp3 d
(c) [Co ( CO )4 ] Paramagnetic Square planar, dsp 2
(CH3)2CH C N (d) [ Ni ( CO )4 ]
Diamagnetic Square planar, dsp2
29. In the presence of cobalt chloride (CoCl2), bleaching
4-Isopropylbenzonitrile
powder decomposes to form
(a) Benzene + (CH3)2CHCl, AlCl3; Br2, FeBr3; KCN (a) CaCO3 and O3 (b) ClO2 and CaO
(b) Benzene + (CH3)2CHCl, AlCl3; HNO3, H2SO4; Fe, (c) Cl2O and CaO (d) CaCl2 and O2
HCl; NaOH; NaNO2, HCl, H2O 30. The reaction L ¾ ¾® M is started with 10.0 g of L. After
(c) Benzene + (CH3)2CHCl, AlCl3; HNO3, H2SO4; Fe, 30 and 90 minutes 5.0 g and 1.25 g of L respectively are left.
HCl; NaNO2/HCl; KCN The order of the reaction is
(a) 0 (b) 1
(d) Benzene + HNO3, H2SO4; (CH3)2CHCl, AlCl3; Fe, HCl;
NaNO2, HCl, H2O; CuCN (c) 2 (d) 3
MOCK TEST 2 MT- 7

31. The reaction of C6H5CH = CHCHO with NaBH4 gives 38. The absolute enthalpy of neutralisation of the reaction:
(a) C6H5CH2CH2CH2OH (b) C6H5CH = CHCH2OH MgO (s) + 2HCl (aq) ¾® MgCl2 (aq) + H2O (l) will be:
(c) C6H5CH2CH2CHO (d) C6H5CH2CHOHCH3 (a) -57.33 kJ mol-1
32. In Kjeldahl’s method for the estimation of nitrogen, the (b) greater than -57.33 kJ mol- 1
formula used is (c) less than -57.33 kJ mol-1
1.4VW
(a) % N = (b) % N = 1.4VW (d) 57.33 kJ mol-1
N V
1.4 VN
(c) % N = VNW (d) % N = 39.
OH-
PhCOCHBr2 ¾¾¾® A ¾¾¾® B ¾¾¾
®C
OH - H+
1.8 W
33. Which of the following cannot be formed ? The compound C is –
(a) He2+ (b) He+ (c) He (d) He2
(a) PhCH(OH)CHO (b) PhCH(OH)COOH
D NH3 Br2
34. Succinic acid ¾¾ ® (A) ¾¾¾ ® (B) ¾¾¾ ® (C) ;
D KOH
Product (C) will be: (c) PhCOCBr2 (d) Ph - C - CH 2 - OH
| ||
CH2 CO2H CH2 CO2H H O
(a) (b)
CH2 CH2 NH2 40. Which of the following is the incorrect statement?
CH2 NH2
(a) NaCl has 6 : 6 coordination and CsCl has 8 : 8
CH2

CO2 K+ CH2 CO2H coordination.
(c) (d) (b) In Na2O each oxide ion is coordinated by 8Na+ ions
CH2 NH2 CH2 CH2 Br
and each Na+ ion by 4 oxide ions
35. Which of the following electrolytes is least effective in (c) NaCl structure transforms to CsCl structure on
coagulating ferric hydroxide solution? heating
(a) KBr (b) K2SO4 (d) In CaF2 structure, each F– ion is coordinated by 4
(c) K2CrO4 (d) K4 [Fe(CN)6] Ca2+ and vice-versa.
41. Which of the following element is extracted commercially
HCl EtONa
36. ¾¾¾¾® (A) ¾¾¾¾
®(Z) (major) by the electrolysis of an aqueous solution of its compound?
peroxide D
(a) Chlorine (b) Bromine
Identify (Z) in the above sequence of reactions: (c) Sodium (d) Aluminium
42. Estimation of calcium and magnesium is done by
(a) (b) (a) EDTA (b) oxalate
(c) Phosphate (d) none of these
O CH3 43. If urms of a gas is 30 R1/2 ms–1 at 27 °C then molar mass of
(c) (d)
OEt gas is:
(a) 0.02 kg/mol (b) 0.001 kg/mol
37. The property which distinguishes formic acid from acetic
(c) 0.003 kg/mol (d) 1 kg/mol
acid is
44. Which is a polymer of three different monomers ?
(a) only ammonium salt of formic acid on heating gives
amide (a) ABS (b) SBR
(b) when heated with alcohol/H2SO4 only acetic acid (c) NBR (d) Nylon-26
forms ester 45. Which of the following is colourless in water?
(c) only acetic acid forms salts with alkali (a) Ti3+ (b) V3+
(c) Cu 3+ (d) Sc3+
(d) only formic acid reduces Fehling’s solution
EBD_8350
MT- 8 CHEMISTRY

MOCK TEST - 3
Time : 1 hr. Max. Marks -180

1. Among the following compounds (I - III), the ease of their 8. A metal M on heating in nitrogen gas gives Y. Y on
reaction with electrophiles is, treatment with H2O gives a colourless gas which when
OCH3 NO2 passed through CuSO4 solution gives a blue colour. Y is
(a) NH3 (b) Mg(NO3)2
(c) Mg3N2 (d) MgO
9. AB; crystallizes in a body centred cubic lattice with edge
length ‘a’ equal to 387 pm. The distance between two
I II III oppositely charged ions in the lattice is :
(a) II > III > I (b) III > II > I (a) 335 pm (b) 250 pm (c) 200 pm (d) 300 pm
(c) II > I > III (d) I > II > III 10. Haemoglobin contains 0.334% of iron by weight. The
2. Which of the following is optically inactive? molecular weight of haemoglobin is approximately 67200.
The number of iron atoms (at. wt. of Fe is 56) present in
H H one molecule of haemoglobin are
H 3C Cl Cl CH3
(a) (b) (a) 1 (b) 6 (c) 4 (d) 2
Cl CH3 H3C Cl
11. Which of the following has highest knocking property ?
H H
(a) Aromatic hydrocarbons
H (b) Olefins
H 3C Cl (c) Branched chain paraffins
(c) (d) none of these.
H 3C Cl (d) Straight chain paraffins
H 12. Black-foot disease is caused due to groundwater
3. Chlorination of benzene is not possible in the following contaminated with excess of
reaction : (a) Nitrate (b) Fluoride
FeCl (c) Arsenic (d) Sulphur
(a) C 6 H 6 + Cl 2 ¾¾ ¾3 ®
13. For the cell
(b) H+
C 6 H 6 + HOCl ¾¾
¾®
Zn | Zn 2+ (1M) || Cu 2 + (1M) | Cu, E o is 1.10 V,,
cell
ZnCl
(c) C 6 H 6 + I - Cl ¾¾ ¾
¾2® o
Ecu 2+ /Cu = 0.34V and for the cell Cu | Cu2+ (1M) || Ag+
(d) AlCl 3 o
C 6 H 6 + Cl 2 ¾¾ ¾® (1M) | Ag, Eocell = 0.46 V hence, Ecell of the cell
4. Nessler’s reagent is Zn | Zn 2 + (1M) || Ag + (1M) | Ag is
(a) KHgI4 (b) K2HgI4 + NH4OH (a) – 0.04 V (b) + 0.04 V (c) + 0.30 V (d) + 1.56 V
(c) K2HgI4 + KOH (d) KHgI4 + NH4OH 14. All of the following substances react with water. The pair
5. The reagent (s) which can be used to distinguish that gives the same gaseous product is
acetophenone from benzophenone is (are) (a) K and KO2 (b) Na and Na2O2
(a) 2,4- Dinitrophenylhydrazine (c) Ca and CaH2 (d) Ba and BaO2
(b) Aqueous solution of NaHSO3 15. Chromyl chloride oxidises toluene to benzaldehyde, this
(c) Benedict reagent reaction is known as
(d) I2and NaOH. (a) Rosenmund reaction (b) Wurtz reaction
6. If 0.1 M of a weak acid is taken, and its percentage of (c) Etard reaction (d) Fittig reaction
degree of ionization is 1.34%, then its ionization constant 16. An organic compound X (molecular formula C6H7O2N) has
will be : six carbon atoms in a ring system, two double bonds and
(a) 0.8 × 10–5 (b) 1.79 × 10–5 a nitro group as substituent, X is
(c) 0.182 × 10 –5 (d) none of the above (a) Homocyclic but not aromatic
7. EAN of cobalt is 36 in [Co(NH3)2O2(en)Cl]. Thus, O2 is (b) Aromatic but not homocyclic
(a) dioxide (b) superoxide ion (c) Homocyclic and aromatic
(c) peroxide ion (d) oxide (d) Heterocyclic and aromatic
MOCK TEST 3 MT- 9

17. A body centered cubic lattice is made up of hollow spheres 24. The correct order of ionic radii of Y3+, La3+, Eu3+ and Lu3+
of B. Spheres of solid A are present in hollow spheres of is
B. Radius of A is half of radius of B. What is the ratio of (a) Y3+< La3+ < Eu3+< Lu3+
total volume of spheres of B unoccupied by A in a unit (b) Lu3+< Eu3+ < La3+< Y3+
cell and volume of unit cell? (c) La3+< Eu3+ < Lu3+< Y3+
(d) Y3+< Lu3+ < Eu3+< La3 +
7 3p 7 3 25. 0.45 g of acid of molecular weight 90 was neutralized by 20
(a) (b)
64 128 ml. of a 0.5N caustic potash. The basicity of an acid is
(a) 1 (b) 2 (c) 3 (d) 4
7.p 26. Among the following complexes, optical activity is possible
(c) (d) None of these
24 in
18. T50 of first -order reaction is 10 min. Starting with 10 (a) [Co( NH3 ) 6 ]3+
mol L–1, rate after 20 min is (b) [ Co ( H 2 O ) 2 ( NH 3 ) 2 Cl 2 ] +
(a) 0.0693 mol L–1 min–1
(b) 0.0693 × 2.5 mol L–1 min–1
(c) [Cr(H 2 O) 2 Cl2 ]+
(c) 0.0693 × 5 mol L–2 min–1 (d) [ Co ( CN ) 5 NC]
(d) 0.0693 × 10 mol L–1 min–1 27. The pKa of an amino acid is 9.15. At what pH, amino acid is
19. The correct order of the decreasing ionic radii among the 5% dissociated ?
following isoelectronic species are : (a) 9.15 (b) 4.85 (c) 9.44 (d) 7.87
28. For real gases van der Waals equation is written as
(a)Ca 2+ > K + > S2– > Cl–1
(b) Cl – > S2- > Ca 2 + > K + æ an2 ö
ç p+ 2 ÷ (V– nb)= nRT
(c) S2– > Cl – > K + > Ca 2+ è V ø
(d) K + > Ca 2+ > Cl – > S 2– where ‘a’ and ‘b’ are van der Waals constants.
20. The r.m.s velocity of hydrogen is 7 times the r.m.s Two sets of gases are :
velocity of nitrogen. If T is the temperature of the gas , (I) O2, CO2, H2 and He
then (II) CH4, O2 and H2
(a) T(H2 ) = T(N2 ) (b) T(H2 ) > T(N 2 ) The gases given in set-I in increasing order of ‘b’ and
gases given in set-II in decreasing order of ‘a’, are arranged
(c) T(H2 ) < T(N 2 ) (d) T(H 2 ) = 7T(N 2 )
below. Select the correct order from the following :
21. The decreasing order of reactivity of (a) (I) He < H2 < CO2 < O2 (II) CH4 > H2 > O2
m-nitrobromobenzene (I); (b) (I) O2 < He < H2 < CO2 (II) H2 > O2 > CH4
2,4,6-trinitrobromobenzene (II); (c) (I) He < H2 < O2 < CO2 (II) CH4 > O2 > H2
p-nitrobromobenzene (III); and (d) (I) H2 < O2 < He < CO2 (II) O2 > CH4 > H2
2,4-dinitrobromobenzene (IV) 29. SnO2 is taken in basic medium and current is passed.
towards OH– ions is Colloidal sol migrates towards
(a) I > II > III > IV (b) II > IV > III > I (a) anode (+ plate) (b) cathode (– plate)
(c) IV > II > III > I (d) II > IV > I > III (c) both (a) and (b) (d) none of these
22. A compound on treatment with NaOH followed by addition 30. Which of the following is a broad spectrum drug?
of Ag NO 3 produces white precipitate at room (a) Plasmoquine (b) Chloroquine
(c) Chloramphenicol (d) D.D.T.
temperature. The precipitate is soluble in NH4OH. The
31. Pair of complexes, which do not have equal number of
compound is identified as
unpaired electron(s)?
(a) vinyl chloride (b) benzyl chloride
(c) chlorobenzene (d) ethyl bromide (a) [Co ( CO )4 ]- , [ Ni ( CN )4 ]4-
23. In the froth floatation process for the purification of ores,
3+ 2-
the ore particles float because (b) éë Co ( H 2 O )6 ùû , [ NiF6 ]
(a) They are light
(b) Their surface is hydrophobic i.e. not easily wetted (c) [ Ni ( en )3 ]2+ , [ Cr ( CN )6 ]4-
by water
(c) They bear electrostatic charge (d) [Co ( CN )6 ]4- , éëCr ( NH3 )6 ùû3+
(d) They are insoluble
EBD_8350
MT- 10 CHEMISTRY

32. Formation of a solution from two components can be 39. H3BO3 is


considered as (a) monobasic and weak Lewis acid
(i) pure solvent ® separated solvent molecules, DH1 (b) monobasic and weak Bronsted acid
(ii) pure solute ® separated solute molecules, DH2 (c) monobasic and strong Lewis acid
(iii) separated solvent and solute molecules ® solution, (d) tribasic and weak Bronsted acid
DH3 40. Consider an electrochemical cell in which the following
Solution so formed will be ideal if reaction occurs and predict which changes will decrease
(a) DHsoln = DH1 + DH2 – DH3 the cell voltage :
(b) DHsoln = DH1 – DH2 – DH3 Fe2+ (aq) + Ag+ (aq) ® Ag (s) + Fe3+ (aq)
(c) DHsoln = DH3 – DH1 – DH2
(I) decrease the [Ag+]
(d) DHsoln = DH1 + DH2 + DH3
(II) increase in [Fe3+]
33. The basicity of aniline is less than that of cyclohexylamine.
(III) increase the amount of Ag
This is due to
(a) + R effect of —NH2 group (a) I (b) II and III (c) II (d) I and II
(b) – I effect of —NH2 group 41. The enthalpy of combustion of C6H6 is –3250 kJ, when
(c) – R effect of —NH2 group 0.39 g of C6H6 is burnt in excess of oxygen in an open
(d) hyperconjugation effect vessel, the amount of heat evolved is
34. Benzoic acid gives benzene on being heated with X and (a) 8.32 kJ (b) 12.36 kJ
phenol gives benzene on being heated with Y. Therefore X (c) 16.25 kJ (d) 20.74 kJ
and Y are respectively 42. Mac Arthur process is used for the extraction of:
(a) Soda-lime and copper (a) Au (b) Pt (c) Cu (d) Zn
(b) Zn dust and NaOH 43. Among the following compounds, the increasing order of
(c) Zn dust and soda-lime their basic strength is :
(d) Soda-lime and zinc dust.
35. Equivalent conductance of an electrolyte containing NaF NH2
at infinite dilution is 90.1 Ohm–1cm2. If NaF is replaced by
(I) (II)
KF what is the value of equivalent conductance? N
(a) 90.1 Ohm–1cm2 (b) 111.2 Ohm–1cm2 H
(c) 0 (d) 222.4 Ohm–1cm2
CH3
36. In which of the following is there a consistent decrease in
N
atomic radius as the atomic number increases?
(III) (IV)
(a) halogens NH2
(b) representative elements
(c) transition elements (a) (I) < (II) < (IV) < (III) (b) (I) < (II) < (III) < (IV)
(d) lanthanides (c) (II) < (I) < (IV) < (III) (d) (II) < (I) < (III) < (IV)
37. Elements X, Y and Z have atomic numbers 19, 37 and 55 2–
44. MnO 4 (1 mole) in neutral aqueous medium
respectively. Which of the following statements is true disproportionates to
about them ?
(a) 2/3 mole of MnO4– and 1/3 mole of MnO2
(a) Their ionization potential would increase with
increasing atomic number (b) 1/3 mole of MnO4– and 2/3 mole of MnO2
(b) ‘Y’ would have an ionisation potential between those (c) 1/3 mole of Mn 2O7 and 1/3 mole of MnO2
of ‘X’ and ‘Z’ (d) 2/3 mole of Mn 2O7 and 1/3 mole of MnO2
(c) ‘Z’ would have the highest ionization potential 45. The following equilibrium constants are given:
(d) ‘Y’ would have the highest ionization potential. ˆˆ† 2NH3 ; K1
N 2 + 3H 2 ‡ˆˆ
38. The enthalpies of the following reactions are shown below. ˆˆ† 2NO; K 2
N2 + O2 ‡ˆˆ
1 1 1 ˆˆ†
H2(g) + O2(g) ® OH–(g) ; DH = 42.09 kJ mol–1 H2 + O2 ‡ˆˆ H2O; K3
2 2 2
H2(g) ® 2H(g) ; DH = 435.89 kJ mol–1 The equilibrium constant for the oxidation of NH3 by
O2(g) ® 2O(g) ; DH = 495.05 kJ mol–1 oxygen to give NO is
Calculate the O –– H bond energy for the hydroxyl radical.
(a) 223.18 kJ mol–1 (b) 423.38 kJ mol–1 K 2 K32 K 22 K3 K1 K 2 K 2 K33
(a) (b) (c) (d)
(c) 513.28 kJ mol –1 (d) 113.38 kJ mol–1 K1 K1 K3 K1
MOCK TEST - 4
Time : 1 hr. Max. Marks -100

Multiple Choice Questions (Qs. no. 1 - 20)

5. 14 g of N2 and 36 g of ozone are at the same pressure and


1. H3C CH3 ¾¾¾¾¾¾
(i) alc. KOH
® P is
(ii) SO2Cl2 , D temperature . Their volumes will be related as

I (a) 2VN 2 = 3VO3 (b) 3VN 2 = 2VO3

(a) 3-Chloropropene (b) 2-Chloropropene


(c) 3VN 2 = 4VO3 (d) 4VN 2 = 3VO3
(c) 1-Chloropropene (d) 1, 2-Dichloropropane
2. The electrons, identified by quantum numbers n and l,
NO2
(I) n = 3, l = 2 (II) n = 5, l = 0 CH3
(III) n = 4, l = 1 (IV) n = 4, l = 2 (i) NaOH
6. + CH3CHO ¾¾¾¾¾
® [X ]
(V) n = 4, l = 0 (ii) D

can be placed in order of increasing energy, as


(a) I < V < III < IV < II (b) I < V < III < II < IV In this reaction, [X] will be –
(c) V < I < III < II < IV (d) V < I < II < III < IV
NO2
3. Molecules/ions and their magnetic properties are given CH = CH – CH3
below.
(a)
Molecule/ion Magnetic property
(i) C6H6 (1) Antiferromagnetic
(ii) CrO2 (2) Ferrimagnetic
NH2
(iii) MnO (3) Ferromagnetic CH3
(iv) Fe3O4 (4) Paramagnetic
3+
(b)
(v) Fe (5) Diamagnetic
The correctly matched pairs in the above is
(a) i-5, ii-3, iii-2, iv-1, v-4 (b) i-3, ii-5, iii-1, iv-4, v-2
NO2
(c) i-5, ii-3, iii-1, iv-2, v-4 (d) i-5, ii-3, iii-1, iv-4, v-2 CH2 – CH2 – CH2OH
4. The electronegativity difference between N and F is greater
(c)
than that between N and H yet the dipole moment of
NH3 (1.5 D) is larger than that of NF3 (0.2D). This is because
(a) in NH3, the atomic dipole and bond dipole are in the NH2
same direction whereas in NF3, these are in opposite CH2 – CH2 – CHO
directions (d)
(b) in NH3 as well as NF3, the atomic dipole and bond
dipole are in opposite directions
(c) in NH3 the atomic dipole and bond dipole are in the 7. The Ce (Z = 58) belongs to IIIrd group of periodic table. If
opposite directions whereas in NF3, these are in the it furnish one a particle to form an element 'X', then X be-
same direction longs to
(d) in NH3 as well as in NF3, the atomic dipole and bond (a) IIIrd group (b) IInd group
st
(c) I group (d) zero group
dipole are in the same direction
EBD_8350
MT- 12 CHEMISTRY

+ 13. Under the influence of an electric field, the particles in a sol


8. In which case the NO2 will attack at the meta position migrate towards cathode. The coagulation of the same sol
is studied using NaCl, Na2SO4 and Na3PO4 solutions. Their
CCl3 NO2 coagulating values will be in the order
(a) NaCl > Na2SO4 > Na3PO4
(b) Na2SO4 > Na3PO4 > NaCl
(i) (ii)
(c) Na3PO4 > Na2SO4 > NaCl
(d) Na2SO4 > NaCl > Na3PO4
+
NH 3 O– 14. Which of the following complex ions has electrons that
are symmetrically filled in both t2g and eg orbitals ?
(a) [FeF3]3– (b) [Mn(CN)6]4–
(iii) ( iv) (c) [CoF6]3– (d) [CO(NH3)6]2+
15. Which carbocation is the most stabilized?
(a) i, ii, iii (b) ii, iv
(c) ii and iii only (d) ii only +
9. Arrange the following in the order of their reactivity of
(a) (b)
SN2 reaction +

(i) CH2Cl (ii) CH=CH—CH2Cl

(c) + (d)
(iii) CH 2 = CH - CH 2Cl +
(iv) CH 3CH 2CH 2Cl 16. Benzaldehyde reacts with ethanoic KCN to give
(a) iv > iii > i > ii (b) ii > i > iii > iv (a) C6H5CHOHCN
(c) i > ii > iv > iii (d) i > iii > iv > ii (b) C6H5CH(OH)COC6H5
10. Which one of the following reactions of xenon compounds
(c) C6H5CHOHCOOH
is not feasible?
(d) C6H5CHOHCHOHC6H5
3
(a) 3XeF4 + 6H 2 O ¾¾
® 2Xe + XeO3 +12HF + O 2 17. In order to refine “blister copper” it is melted in a furnace
2
and is stirred with green logs of wood. The purpose is
(b) 2XeF2 + 2H 2O ¾¾
® 2Xe + 4HF + O 2 (a) to expel the dissolved gases in blister copper.
(c) XeF6 + RbF ¾¾
® Rb[XeF7 ] (b) to bring the impurities to surface and oxidize them.
(d) XeO3 + 6HF ¾¾
® XeF6 + 3H 2O (c) to increase the carbon content of copper.
11. Identify the correct statement (d) to reduce the metallic oxide impurities with
(a) Elemental sodium can be prepared and isolated by hydrocarbon gases liberated from the wood.
electrolysing an aqueous solution of sodium chloride 18. The colour of CuCr2O7 solution in water is green because?
(b) Elemental sodium is a strong oxidising agent
(a) Cr2 O72 - ions are green
(c) Elemental sodium is insoluble in ammonia
(d) Elemental sodium is easily oxidised
(b) Cu 2+ ions are green
12. Which of the following species is basic & reducing?
(c) Both Cr2O2– 2+
7 and Cu ions are green
(a) SO 32 - (b) SO 24 -
(d) Cu2+ ions are blue and Cr2 O72 - ions are yellow
(c) S2 O 24 - (d) HSO -4
MOCK TEST 4 MT- 13

19. Arrange the following alcohols in order of increasing ease 22. The equilibrium constant of the following redox reaction
of dehydration at 298 K is 1 × 108 : 2 Fe3+ (aq.) + 2 I– (aq.) ƒ 2 Fe2+ (aq.)
CH 3CH 2 OH C 6 H 5 CH 2 OH + I2 (s). If the standard reduction potential of iodine
I II becoming iodide is +0.54 V, what is the standard reduction

Cl 3CCH 2 OH F3CCH 2 OH potential of Fe3+ /Fe 2 + ?


III IV
23. The rate constant for forward and backward reactions of
(a) II < I < IV < III (b) IV < III < II < I hydrolysis of ester are 2 × 10–2 and 2 × 10–3 per minute
(c) IV < III < I < II (d) II < I < III < IV respectively. What is the value of equilibrium constant?
20. Identify the incorrect statement : 24. How many carbon atom(s) is/are present in the ring
(a) In (Si3O9)6–, tetrahedral SiO4 units share two oxygen structure of mononer of following polymer
atoms. (polyethyleneglycol)?
(b) Trialkylchlorosilane on hydrolysis gives R3SiOH. {{—O—CH2—CH2—O—CH2—CH2—O—CH2—CH2—O—}}
(c) SiCl4 undergoes hydrolysis to give H4SiO4. 25. A solution of CO2 gas is prepared with water. At 273 K,
(d) (Si3O9)6– has cyclic structure. the value of Henry’s constant and pressure are
respectively 600 and 0.3 atm. At 340 K, Henry’s constant
Numerical Answer Questions (Qs. no. 21 - 25)
is 3400. What should be the pressure of CO2 gas at this
21. 2 moles of an ideal mono-atomic gas goes around the temperature to keep the same concentration of CO2 gas?
cycle as shown in this figure. The sum of heat change in
the processes A ® B and C ® A is q × R. What is the
magnitude of q in Joule?

B (600 K)
Pressure

A C (400 K)
(200 K)
Volume
EBD_8350
MT- 14 CHEMISTRY

MOCK TEST - 5
Time : 1 hr. Max. Marks -100

Multiple Choice Questions (Qs. no. 1 - 20)

1. The time taken for 90% of a first order reaction to complete (b) Ni: Mond's process; Cu: Electrolysis; Zr: van Arkel
is approximately process; Ga: Zone refining
(a) 1.1 times that of half-life (c) Ni: Mond's process; Cu: van Arkel process; Zr: Zone
refining; Ga: Electrolysis.
(b) 2.2 times that of half-life
(d) Ni: Electrolysis; Cu: Zone refining; Zr: van Arkel
(c) 3.3 times that of half-life
process; Ga: Mond's process
(d) 4.4 times that of half-life 6. The fraction of total volume occupied by the atoms present
2. The structure : in a simple cube is
CH3 H p p p p
(a) (b) (c) (d)
3 2 4 2 4 6
C C H
CH3 C 7. The straight chain polymer (silicones) is formed by
COOH (a) hydrolysis of CH3SiCl3 followed by condensation
CH3
polymerisation
shows (b) hydrolysis of (CH3)4Si by addition polymerisation
(a) tautomerism
(c) hydrolysis of (CH3)2SiCl2 followed by condensation
(b) geometrical isomerism
polymerisation
(c) optical isomerism
(d) hydrolysis of (CH3)3SiCl followed by condensation
(d) geometrical and optical isomerism
polymerisation
3. Beryllium shows diagonal relationship with aluminium.
8. In the synthesis of glycerol from propene, compounds
Which of the following similarity is incorrect ?
formed are
(a) Be forms beryllates and Al forms aluminates
(a) Glycerol, b-chlorohydrin and allyl chloride
(b) Be(OH)2, like Al(OH)3 is basic.
(b) Glyceryl trichloride and glycerol a-chlorohydrin
(c) Be like Al is rendered passive by HNO3.
(c) Allyl alcohol and a-chlorohydrin
(d) Be2C like Al4C3 yields methane on hydrolysis.
(d) Allyl alcohol and monosodium glycerolate
4. The compound A on heating gives a colourless gas and a
9. Which one of the following statements is FALSE?
residue that is dissolved in water to obtain B. When excess
of CO2 is bubbled through aqueous solution of B, C is (a) The correct order of osmotic pressure for 0.01 M
formed which is recovered in the solid form. Solid C on aqueous solution of each compound is
gentle heating gives back A. The compound is BaCl2 > KCl > CH3COOH > Sucrose
(a) CaSO4.2H2O (b) CaCO3 (b) The osmotic pressure (p) of a solution is given by the
equation p = MRT, where M is the molarity of the
(c) Na2CO3 (d) K2CO3
solution
5. Four metals and their methods of refinement are given
(c) Raoult’s law states that the vapour pressure of a
i. Ni, Cu, Zr, Ga
ii. Electrolysis, van Arkel process, zone refining, Mond's component over a solution is proportional to its mole
process fraction
Choose the right method for each : (d) Two sucrose solutions of same molality prepared in
(a) Ni: Electrolysis; Cu: van Arkel process; Zr: Zone different solvents will have the same freezing point
refining; Ga: Mond's process. depression
MOCK TEST 5 MT- 15

10. The missing reagents R1 and R2 in the following series of


reactions are CH(OH)CH3 COCH3
R R -
CH3CH 2Br ¾¾® 1 [ ] ¾¾¾2 ® CH CHP + Ph
3 3
(a) PhLi and Ph3P respectively (a) (b)
(b) Ph3P and PhLi respectively OCH3 OCH3
(c) Ph3P and C2H5ONa respectively
(d) Either (b) or (c)
11. Which one of the following sets forms the biodegradable CHO COOH
polymer?
(a) CH2 = CH – CN and CH2 = CH – CH = CH2
(b) H2N – CH2 – COOH and H2N–(CH2)5 – COOH (c) (d)
(c) HO – CH2 – CH2 – OH and OCH3 OCH3

HOOC 16. The ratio of number of oxygen atoms (O) in 16.0 g ozone
COOH
(O3), 28.0 g carbon monoxide (CO) and 16.0 g oxygen (O2)
is (Atomic mass : C = 12, O = 16 and Avogadro’s constant
NA = 6.0 × 1023 mol–1)
(d) CH = CH2 and CH2 = CH – CH = CH2
(a) 3 : 1 : 2 (b) 1:1 :2
(c) 3 : 1 : 1 (d) 1:1 :1
12. At 25°C, molar conductance of 0.1 molar aqueous
solution of ammonium hydroxide is 9.54 ohm–1 cm2mol–1 17. The secondary structure of a protein refers to
and at infinite dilution its molar conductance is (a) fixed configuration of the polypeptide backbone
238 ohm–1 cm2 mol–1. The degree of ionisation of ammonium (b) a– helical backbone
hydroxide at the same concentration and temperature is : (c) hydrophobic interactions
(a) 20.800% (b) 4.008% (c) 40.800% (d) (d) sequence of a– amino acids.
2.080% 18. Consider the two molecules :
13. An unknown compound A has a molecular formula C4H6,
H
when A is treated with an excess of Br 2, a new substance
(I) H3C C
B with formula C4H6Br2 is formed. A forms a white
C C CH3
precipitate with ammonical silver nitrate solution. A may
be O O
(a) Butyne-1 (b) Butyne-2 H
(c) Butene-1 (d) Butene-2
H H
14. Red precipitate is obtained when ethanol solution of (II) H3C C CH3.
dimethylglyoxime is added to ammoniacal Ni(II). Which of C C
the following statements is not true ? H
(a) Red complex has a square planar geometry. O O
H
(b) Complex has symmetrical H-bonding
(c) Red complex has a tetrahedral geometry. Which of the following statements about them is true ?
(d) Dimethylglyoxime functions as bidentate ligand. (a) II is capable of forming intramolecular hydrogen
bonding whereas I is not
C N
(b) Both are capable of forming equally stable
H O + intramolecular hydrogen bonding
15. + CH3MgBr ¾® Q ¾¾®
3
P
(c) I is likely to form more stable intramolecular hydrogen
OCH3
bonding than II
Product 'P' in the above reaction is (d) Both of them cannot form intramolecular hydrogen
bonding
EBD_8350
MT- 16 CHEMISTRY

19. Cl 2 (g) + Ba ( OH )2 ¾¾
® X (aq.) + BaCl2 + H 2O Numerical Answer Questions (Qs. no. 21 - 25)

X + H 2SO 4 ¾¾
® Y + BaSO 4 21. What is the value of entropy change involved in the
D isothermal reversible expansion of 5 moles of an ideal gas
Y ¾¾¾¾¾ ® Z + H 2O + O 2
D>365K
from a volume of 10 ml to 1000 ml at 20°C (in J mol–1 K–1)?
Y and Z are respectively :
(a) HClO4, ClO2 (b) HClO3, Cl2O 22. What is the temperature (K) at which RMS velocity of
(c) HClO3, ClO6 (d) HClO4, Cl2O7 SO2 molecules is half that of Ne molecules at 200 K?
20. In a set of reactions p-nitrotoluene yielded a product E.
23. An element with atomic mass 24 g/mol forms an fcc lattice.
CH3
The edge length of lattice is 400 pm and the observed
Br
¾¾¾
2 ® B ¾¾¾¾Sn/HCl
®C density is 2.45 g/cm3. What will be the percentage
FeBr3
occupancy of lattice points by this element?
NO2 NaNO CuBr 24. The volume of the vessel in which the reaction
¾¾¾¾
2 ® D ¾¾¾® E
HCl HBr 2NO + O2 ® 2NO2 is occurring is diminished to 1/4th of its
The product E would be: initial volume. The new rate of reaction becomes n times.
CH3 CH3 What is the value of n?
Br Br Br 25. The structure of Salvarsan (an antibiotic) resembles to
(a) (b) the structure of azodye. Hoe many arsenic atoms are
present in the molecule of this antibiotic?
Br Br

CH3 CH2 Br
Br
(c) (d)
Br
Br

You might also like